Lernmodul Mathematik

Website: TH Wildau: Zentrale E-Learning Plattform
Kurs: SOS Mathematik
Buch: Lernmodul Mathematik
Gedruckt von: Gast
Datum: Donnerstag, 25. April 2024, 16:38

Inhaltsverzeichnis

1. Zum Einstieg

Warum dieser Kurs?

Die Erfahrungen der letzten Jahre haben gezeigt, dass die Mathematik den Studienanfängerinnen und Studienanfängern leider zunehmend Schwierigkeiten bereitet. Die Gründe dafür sind vielfältig. Nur zum Teil fehlt es am grundlegenden mathematischen Verständnis. Ein weiteres Phänomen ist vielmehr, dass einfach zu wenig Übung / Routine / Erfahrung mit sogenannten elementaren Rechentechniken besteht. Wer darüber nachdenken muss, ob - und wenn ja wie - der Bruch \frac{-1+2a-a^2}{a-1} gekürzt, d. h. vereinfacht werden kann ("Warum ist das dasselbe wie 1-a???"), dem kann es schon passieren, bei den weiteren Ausführungen den Anschluss zu verlieren und schließlich nach und nach den Gesamtzusammenhang nicht mehr zu überblicken. Doch das muss nicht sein, deswegen sind Sie ja in diesem Kursraum ...

Sehen Sie diesen Kurs als eine Trainingseinheit, die Sie aufwärmt und oftmals schon in Vergessenheit geratene Gehirnzellen aktiviert. Dieses Training greift nicht dem anstehenden Wettkampf, sprich Studium, vorweg, sondern setzt deutlich davor an. Es hat auch nicht das Ziel, Sie alle zu "Mathematik-Weltmeistern" zu machen. Es geht vielmehr darum, eine gute Basis zu schaffen für alle Themen, die später darauf aufbauen. Nutzen Sie die Gelegenheit zur Übung und Wiederholung, scheuen Sie sich nicht, zu fragen, und setzen Sie sich (alleine oder in Gruppen) mit den Aufgaben ernsthaft auseinander - denn, wenn sich die ersten Erfolge einstellen, dann macht auch Mathematik Spaß!

 

Was ist vorab wichtig zu wissen?

  • Auf den Kapitel-Startseiten erfahren Sie jeweils, welche Lernziele Sie mit diesem Kapitel erreichen sollen.
  • Zusätzlich finden Sie auf (fast) allen Kapitel-Oberseiten eine Übersicht über typische Fehler zu den Inhalten des entsprechenden Kapitels. Bekanntermaßen sind die Möglichkeiten, in der Mathematik etwas falsch zu machen, ja sehr vielfältig. Um herauszufinden, welche Fehler am häufigsten auftreten, wurden daher Erstsemester-Klausuren untersucht.
    Selbst wenn Sie sich mit einem Thema schon ganz gut auskennen, lohnt es sich, bei den typischen Fehlern mal hineinzuschauen. Einige lassen sich leicht vermeiden, wenn man sie sich bewusst gemacht hat. Und man muss Fehler, die jemand anders schon gemacht hat, ja nicht unbedingt wiederholen …
  • Alle Kapitel (einzige Ausnahme: "Grundlagen") sind in drei Seiten eingeteilt:
    • Die erste Seite enthält Aufgaben.
    • Die zweite Seite umfasst Erklärungen und Beispiele.
    • Auf der dritten Seite finden Sie die Lösungen der Aufgaben.

  • Hier ein Vorschlag aus unserer Erfahrung, wie Sie Ihr Mathe-Training effektiv gestalten können:
    1. Versuchen Sie zunächst die Aufgaben zu lösen, ohne sich die Lösungen anzuschauen. Im Sport lernt man ja auch nicht, indem man der Nationalmannschaft im Fernsehen zuschaut - sondern indem man ganz viel selbst auf dem Sportplatz oder in der Turnhalle aktiv ist.
    2. Bei Schwierigkeiten mit den Aufgaben haben Sie die Möglichkeit, sich mithilfe der Hinweise und Erläuterungen auf der zweiten Seite ("Erklärungen") Lösungsstrategien zu erarbeiten. Dort werden natürlich auch die zum Thema gehörenden Fachbegriffe erläutert.
    3. Erst wenn auch das nicht hilft, sollten die Lösungen angeschaut werden. Selbstverständlich dienen die Lösungen auch zur Kontrolle Ihrer Ergebnisse.

  • Die Lösungswege sind in den meisten Fällen sehr ausführlich aufgeschrieben, ausführlicher, als es normalerweise notwendig ist. Das soll es Ihnen erleichtern, die Lösungswege nachzuvollziehen und mit Ihren Ergebnissen zu vergleichen. Umgekehrt bedeutet dies auch: Fehlen bei Ihnen einige Schritte, ist Ihr Lösungsweg nicht automatisch falsch. Gerade einfache Additionen, Multiplikationen etc. oder auch Umsortierungen der Variablen müssen nicht unbedingt hingeschrieben werden, wenn man geübt genug ist, den Lösungsweg auch so zu überblicken. Häufig ist der dargestellte Lösungsweg auch nicht der einzige Weg, der zum richtigen Ergebnis führt, z. B. können manchmal Schritte vertauscht werden. Wie im Sport führen auch hier manchmal mehrere Wege zum Trainings- bzw. Lernziel.

  • Dieser Kurs kann der Reihe nach und vollständig bearbeitet werden. Vermutlich ist es für Ihr Training aber sinnvoller, wenn Sie sich gezielt die Kapitel heraussuchen, die Sie gerade benötigen.
  • Die Navigation in diesem Kurs kann entweder über die Auswahl einzelner Kapitel im Inhaltsverzeichnis auf der rechten Seite erfolgen oder durch Nutzung der Vor- und Zurückpfeile zu Beginn jeder Seite. Im letzten Kapitel dieses Lernmoduls befindet sich ein Stichwortverzeichnis, über das Sie einzelne Begriffe, die im Lernmodul behandelt werden, direkt anwählen können.
  • Wenn Sie die Materialien auf einem Smartphone nutzen, ist es sinnvoll, das Handy für eine größere Darstellung der Bilder und Tabellen quer zu halten.

 

Worauf sollten Sie achten?

Bitte beachten Sie Folgendes beim Bearbeiten der Übungsaufgaben:

  • Es handelt sich eher um rein mathematische Aufgaben und weniger um Aufgaben mit einem wirklichen Anwendungsbezug. Reale Anwendungsaufgaben sind meist komplizierter als das, was wir in der Studienvorbereitung behandeln (können). Solche Aufgaben warten dann im Studium auf Sie.
  • Gleichzeitig werden wir mit den Aufgaben und Erklärungen nicht in die mathematische Tiefe vordringen. Das Ziel dieses Lernmoduls ist, die Anforderungen, die im Studium auf Sie warten, möglichst verständlich vorzubereiten und zu begleiten. Dazu werden Konzepte erläutert, Zusammenhänge aufgezeigt und Rechenwege diskutiert. Das geht natürlich nicht ohne eine gewisse mathematische Basis und formale Genauigkeit. Der Fokus der folgenden Kapitel liegt aber immer auf Anschaulichkeit und Klarheit.
  • Die Aufgaben sind zum großen Teil so gestaltet, dass sie ohne die Hilfe eines Taschenrechners zu lösen sind. Meistens benötigen Sie rechnerisch nicht (viel) mehr als Addition und Subtraktion im Zahlenraum bis 100 und das kleine Einmaleins. Gerade in diesem Trainingsbereich empfehlen wir es sehr, den Taschenrechner mal beiseitezulegen - aus folgenden Gründen:
    • Es ist sehr wichtig, Größenordnungen abschätzen bzw. überschlagen zu können, um Ergebnisse vom Taschenrechner zu plausibilisieren. Es ist ja nicht soooo unwahrscheinlich, dass man sich mal vertippt oder das Dezimalkomma vergisst ...
    • Spätestens wenn Variablen ins Spiel kommen, helfen "normale" wissenschaftliche Taschenrechner nur noch wenig weiter. "Moderne" wissenschaftliche Taschenrechner können zwar mit Variablen umgehen, aber das hilft in der Klausur meist nicht, da eigentlich immer ein nachvollziehbarer Lösungsweg gefordert ist.
    • Es gibt Anhaltspunkte dafür, dass man mathematische Konzepte (wie beispielsweise "Wurzel ziehen") besser versteht, wenn man sie nicht von Anfang an mit einer Taste auf dem Taschenrechner verbindet.
    • Und zuletzt: Aus mathematisch-formaler Sicht ist es auch gar nicht nötig, Ergebnisse als Dezimalzahl darzustellen: \frac {1}{3} bzw. \sqrt{2} u. Ä. sind exakter als Dezimalzahlen - was in der Mathematik grundsätzlich begrüßt wird.

  • Nicht alle Aufgaben, die Ihnen in den nächsten Kapiteln begegnen, sind lösbar. Die Erkenntnis, dass eine Aufgabe keine Lösung hat und welche Gründe das hat, ist ja auch nicht unwichtig oder uninteressant - eher im Gegenteil ...
  • Ganz wichtig: Die Aufgaben beziehen sich immer auf alle Themen, die zuvor behandelt wurden, nicht nur auf das Thema des entsprechenden Kapitels.
  • Teilweise hilft es, ein wenig "rückwärts" oder "um die Ecke" zu denken ...

 

Grundsätzliche Tipps - sowohl für die Studienvorbereitung als auch für das Studium:

  • Lesen Sie sich die Aufgaben gründlich durch!
  • Stellen Sie fest, was gegeben und was gesucht ist!
  • Manche Aufgaben lassen sich eher mit etwas Nachdenken und weniger mit viel Rechnen lösen!
  • Überprüfen Sie Ihre Lösungen kritisch!

 

Wir wünschen Ihnen viel Erfolg mit der Mathematik!

 

Zum Abschluss des Einstiegs

Wenn Sie Fehler in den Kapiteln finden, die sich trotz aller Sorgfalt nie ganz ausschließen lassen, bin ich über eine kurze Rückmeldung an xenia.jeremias@th-wildau.de dankbar!

 

Autorin dieses Lernmoduls: Dr. Xenia Valeska Jeremias, bis 14.11.2011 Mitarbeiterin am FB Holztechnik der Hochschule für nachhaltige Entwicklung Eberswalde - seit 15.11.2011 Mitarbeiterin im Zentrum für Studium und Lehre der Technischen Hochschule Wildau

Danksagung: Ich danke Prof. Dr. Johannes Creutziger von der HNE Eberswalde, meinen Kolleginnen/Kollegen Roger Faulhaber, Frederik Freckmann, Johanna Gröpler, Jacqueline Pudör, Christian Rabe und Birgit Sellmer sowie meinen studentischen Hilfskräften der TH Wildau für viele Korrekturen und hilfreiche Anmerkungen!

Dieses Lernmodul ist lizenziert unter CC BY-NC-SA 3.0.

2. Grundlagen - Lernziele und typische Fehler

Nach Durcharbeiten dieses Kapitels sollten Sie folgende Lernziele erreicht haben:

  • Sie können Mengen, Zahlenbereiche, Intervalle und n-Tupel unterscheiden und mathematisch korrekt notieren.
  • Sie können Elemente von gegebenen Mengen und Intervallen benennen und entscheiden, ob eine gegebene Zahl Element einer bestimmten Menge/eines bestimmten Intervalls ist.
  • Sie erinnern sich an die Grundrechenarten und können mit rationalen Zahlen rechnen.
  • Sie können Zahlen auf die angegebene Stelle runden.
  • Sie können proportionale und antiproportionale Verhältnisse unterscheiden.
  • Sie können in beiden Fällen den Dreisatz berechnen.
  • Sie können die grundlegenden Rechengesetze (Kommutativ-, Assoziativ- und Distributivgesetz) anwenden.
  • Sie wissen, dass Punktrechnung Vorrang vor Strichrechnung hat und wenden diese Regel in Rechnungen an.
  • Sie wissen grundsätzlich, wann und wie Klammern gesetzt werden müssen.
  • Sie kennen die Bestandteile eines kartesischen Koordinatensystems.
  • Sie können Punkte in ein kartesisches Koordinatensystem einzeichnen und aus einem kartesischen Koordinatensystem ablesen.


Typische Fehler
in diesem Kapitel sind:


Sonstige Fehler
aus allen Kapiteln sind:

  • Die Aufgabenstellung wird nicht ausreichend beachtet, z. B.
    • Teile der Aufgabenstellung werden nicht oder nicht zu Ende bearbeitet.
    • Es wird ein anderes Verfahren angewendet als gefordert.
    • Die Lösung wird nicht in der geforderten Form dargestellt.
    • Geforderte Lösungsmengen oder Antwortsätze werden nicht notiert.
    • Tipp: Lesen Sie sich die Aufgabenstellung gründlich durch und markieren Sie alle relevanten Anforderungen (ggf. farbig).
  • Der Rechenweg wird nicht nachvollziehbar aufgeschrieben, z. B.
    • Es wird nicht deutlich gemacht, welche Struktur der Lösungsweg hat.
    • Es wird nicht deutlich gemacht, welche Schritte warum in welcher Reihenfolge durchgeführt werden.
    • Es wird nicht deutlich gemacht, welche Rechengesetze/Rechenoperationen angewendet wurden.
    • Tipp: Achten Sie darauf, den Lösungsweg kleinschrittig und ggf. mit Zwischenüberschriften aufzuschreiben, sodass Sie selbst den Überblick behalten und diejenigen, die die Aufgabe korrigieren, auch ...
  • Es werden zu viele Rechenschritte auf einmal durchgeführt, was schnell dazu führt, dass man durcheinanderkommt - und am Ende keiner der Rechenschritte richtig ist. Nur wenn die Rechenschritte einzeln aufgeführt sind, kann es trotz Folgefehlern noch (Teil-)Punkte geben.
  • Es wird nach Abschluss der Rechnung nicht überprüft, ob
    • das berechnet wurde, was gefordert war.
    • die Ergebnisse plausibel sind.
    • die Einheit des Ergebnisses zum geforderten Wert passt.
    • die Ergebnisse zur Skizze passen.
    • alle in der Aufgabenstellung gegebenen Angaben in der Rechnung verwendet wurden.
  • Zahlenwerte und/oder Variablen gehen im Laufe der Rechnung verloren oder ändern sich ohne ersichtlichen Grund ...


Für Online-Selbsttests zu diesem Thema und weitere Informationen zur Mathematikunterstützung an der TH Wildau nutzen Sie bitte den Moodle-Kursraum "SOS Mathematik - Brückenkurs".


Weitere Hinweise zum Lösen einer mathematischen Übungsaufgabe, zum Lesen eines mathematischen Lehrbuchtextes sowie zum Mathematisch korrekten Schreiben finden Sie in diesen Studientipps. Dort werden auch die gerade beschriebenen Fehler in den Blick genommen.

Dieses Kapitel enthält die folgenden Themen:

 

2.1 Grundlagen - Mengen u. a.

In den folgenden Abschnitten wird es um einige Grundlagenbegriffe gehen, die später in verschiedenen Zusammenhängen wichtig werden. Das Kapitel hat nicht den Anspruch, ein mathematisch umfassendes Bild von Mengen etc. aufzuzeigen, sondern vielmehr, Ihnen eine Idee zu vermitteln, was Ihre Mathematikdozenten und -dozentinnen meinen, wenn sie beispielsweise etwas in geschweifte Klammern schreiben oder von n-Tupel reden.

 

Vorab

Ist eine Zahl x größer als 0, geschrieben x>0, nennt man sie positiv.
Ist eine Zahl x größer oder gleich 0, geschrieben x \geq 0, nennt man sie nichtnegativ.
Ist eine Zahl x kleiner als 0, geschrieben x < 0, nennt man sie negativ.
Den Begriff nichtpositiv für Zahlen, die x kleiner oder gleich 0 sind, geschrieben x \leq 0, verwendet man nicht oder nur sehr selten - aus welchen Gründen auch immer ...

Wichtig: 0 ist weder positiv noch negativ.

Und noch ein wichtiges Zeichen: \neq bedeutet "ungleich". Beispielsweise ist 3\neq 14, also "3 ungleich 14".

 

Mengen

Grundlegendes zu Mengen

Definition: Eine Menge ist eine Zusammenfassung von wohlunterschiedenen Objekten zu einem Ganzen. Es muss entscheidbar sein, ob ein Element zu der Menge gehört oder nicht. Die Reihenfolge der Elemente ist hingegen nicht von Belang.
Die etwas "unmathematische" Vorstellung: Man hat einen Beutel, in den man Objekte hineinlegt. Entscheidend ist dann die Frage: Ist das Objekt im Beutel enthalten oder nicht? Diese Vorstellung passt auch deswegen, weil Objekte in einem Beutel durcheinander fallen, also keine festgelegte Reihenfolge haben.

Zur Notation: Mengen werden üblicherweise mit großen lateinischen Buchstaben bezeichnet, besonders M wird (naheliegenderweise) gerne verwendet. Hat man mehrere Mengen, die M heißen sollen, kann man für die Unterscheidung einen so genannten Index, also eine kleine, tiefgestellte Zahl, zum Durchnummerieren verwenden (Plural von Index ist Indizes, gesprochen: "Indizees"). Für die Angabe der Elemente verwendet man geschweifte Klammern, z. B.

  • M_1=\{1; 2; 3\} bedeutet: Die Menge, die die Elemente 1, 2 und 3 (und sonst nichts) enthält.
  • M_2=\{1; 3; 5;...\} bedeutet: Die Menge aller positiven, ungeraden Zahlen. Die drei Punkte \ldots am Ende verwendet man, da es unendlich viele positive, ungerade Zahlen gibt und man eben nicht alle aufschreiben kann.  
  • M_3=\left\{\dfrac{n}{m} \; \vert \; n,m \; \text{ungerade}\right\} bedeutet: Die Menge aller Brüche (das ist der Teil vor dem \vert ) mit der Eigenschaft, dass Zähler und Nenner ungerade Zahlen sind (das ist der Teil nach dem \vert ). Z. B. sind \frac{1} {5} und \frac{9}{3}=3 in dieser Menge enthalten.

Es gibt verschiedene Aspekte, in denen sich Mengen unterscheiden können:

  • Sie können entweder über eine Aufzählung von Elementen (wie bei Mengen M_1 und M_2) oder über erklärende Eigenschaften (wie bei Menge M_3) definiert werden.
  • Mengen können endlich viele Elemente (siehe Menge M_1) oder unendlich viele Elemente (siehe Mengen M_2 und M_3) enthalten.


Vielleicht ist es überraschend: Eine ganz wichtige Menge ist die leere Menge, die keine Elemente enthält. Dafür schreibt man das Symbol \emptyset.

Und noch ein paar Symbole, die in diesem Zusammenhang wichtig sind:
Möchte man aussagen, dass ein einzelner Wert Teil einer Menge ist oder sein soll, verwendet man das Symbol \in. Z. B. meint x \in M_1 (gesprochen: "x ist Element von M 1"), dass x Teil der Menge M_1 ist oder sein soll. x könnte also 1, 2 oder 3 sein.

Ein Symbol für das Gegenteil gibt es natürlich auch: Möchte man aussagen, dass ein einzelner Wert nicht Teil einer Menge ist oder sein soll, verwendet man \not \in. Z. B. meint  0{,}5 \not\in M_2 (gesprochen: "0{,}5 ist nicht Element von M 2"), dass 0{,}5 nicht Teil der Menge M_2 ist. 0{,}5 ist eben keine positive, ungerade Zahl.


Bemerkung 1:
Um Missverständnissen vorzubeugen, ist es manchmal sinnvoll, Zahlen in einer Menge mit Semikolons zu trennen. Sonst könnte M_1=\{1, 2, 3\} sowohl M_1=\{1{,}2; 3\} als auch M_1=\{1; 2{,}3\} oder M_1=\{1; 2; 3\} bedeuten - und solche Mehrdeutigkeiten sind bei der Verständigung z. B. über Lösungswege sehr hinderlich und auch grundsätzlich in der Mathematik äußerst unbeliebt ...
Bemerkung 2: Üblich ist, die Elemente der Größe nach zu sortieren. Wenn Variablen enthalten sind, werden diese alphabetisch sortiert. Dies hat keine mathematischen Hintergründe (Die Reihenfolge spielt ja hier keine Rolle ...), sorgt aber dafür, dass man den Überblick behält - und das ist ja nie verkehrt!

 

Spezielle Mengen

Nun noch ein paar Begriffe, die wir später brauchen werden:
Die Schnittmenge M_S zweier gegebener Mengen A und B umfasst alle Elemente, die sowohl in A als auch in B enthalten sind. Anders formuliert: Alle Elemente der Schnittmenge müssen in A und in B liegen. Mengen, deren Schnittmenge leer ist, nennt man disjunkt. Dann haben die Mengen keine gemeinsamen Elemente.
Beispiel: Nehmen wir M_1 und M_2 von oben. Nur die Elemente 1 und 3 sind in beiden Mengen enthalten, also besteht daraus ihre Schnittmenge. Mathematisch schreibt man das: M_S=M_1\cap M_2=\{1;3\}

Die Vereinigungsmenge M_V zweier gegebener Mengen A und B umfasst alle Elemente, die in A oder in B enthalten sind. Anders formuliert: Die Vereinigungsmenge besteht aus allen Elementen, die in einer der beiden Mengen liegen.
Beispiel: Wir schauen wieder M_1 und M_2 von oben an. Um die Vereinigungsmenge zu bestimmen, nehmen wir erst mal alle Elemente, die in M_2 enthalten sind (das sind schließlich mehr). Hinzukommt die 2 aus der Menge M_1 . Um die 1 und die 3 aus M_1 brauchen wir uns nicht mehr zu kümmern, weil sie ja sowieso schon in M_2 enthalten sind. Mathematisch schreibt man das: M_V=M_1\cup M_2=\{1; 2; 3; 5; 7; 9; \dots \}

Eine Menge A heißt Teilmenge der Menge B, wenn alle Elemente von A auch in B enthalten sind.
Beispiel: M_4=\{3; 5; 9\} ist eine Teilmenge von M_2=\{1; 3; 5;...\} , weil 3, 5 und 9 positive, ungerade Zahlen sind. Mathematisch schreibt man hier: M_4 \subseteq M_2 . Um genau zu sein, handelt es sich sogar um eine echte Teilmenge, da in M_4 nicht alle Elemente aus M_2 enthalten sind, z. B. ist die 1 ein Element von M_2, aber nicht von M_4 . Auch hierfür gibt es (natürlich) eine mathematische Schreibweise: M_4\subset M_2 . Wenn man einfach von einer Teilmenge (ohne "echt") spricht, können die Mengen also auch gleich sein. Das deutet man bei \subseteq durch den Strich unter dem Bogen an, der an ein Gleichheitszeichen erinnern soll.

 

Zahlenbereiche

Einige Mengen sind in der Mathematik so wichtig, dass sie eigene Symbole bekommen haben. Diese Symbole haben meistens irgendwo einen Doppelstrich. Beispiele für solch wichtige Mengen sind die verschiedenen Zahlenbereiche:

\mathbb{N}: Menge der natürlichen Zahlen, also 0, 1, 2, ...

\mathbb{Z}: Menge der ganzen Zahlen, also ..., -3, -2, -1, 0, 1, 2, 3, ...

\mathbb{Q}: Menge der rationalen Zahlen, also alle Zahlen, die sich als Bruch und damit als endliche oder periodische Dezimalzahl darstellen lassen, mathematisch formuliert: \mathbb{Q}=\left\{\dfrac{n}{m} \; \vert \; n, m \in \mathbb{Z}\textrm{; } m \neq 0 \right\}, z. B. -4, 0, 0{,}\overline{1}, 0{,}3,  \frac13,  \frac22, 2.000, ...

\mathbb{R}: Menge der reellen Zahlen. Zusätzlich zu den rationalen Zahlen sind hier alle unendlichen, nichtperiodischen Dezimalzahlen enthalten, z. B. 0{,}1010010001..., \sqrt{2}, \sqrt [3]{5}, \pi, ...
Diese "zusätzlichen" Zahlen werden irrationale Zahlen genannt.

Diese Zahlenbereiche sind so gestaltet, dass sie jeweils echte Teilmengen voneinander sind: Die natürlichen Zahlen sind eine Teilmenge der ganzen Zahlen, die ganzen Zahlen eine Teilmenge der rationalen Zahlen und die rationalen Zahlen eine Teilmenge der reellen Zahlen. Warum ist das so? Da in den reellen Zahlen alle Brüche (und noch viel mehr Zahlen) enthalten sind, sind natürlich auch die periodischen und endlichen dabei, sprich die rationalen Zahlen. Bastelt man in den rationalen Zahlen einen Bruch mit dem Nenner 1, also z. B. \frac{4}{1}=4 , landet man bei einer ganzen Zahl. Jede nichtnegative, ganze Zahl ist gleichzeitig eine natürliche Zahl.
Wer möchte, kann das mathematisch so aufschreiben: \mathbb{N}\subset\mathbb{Z}\subset\mathbb{Q}\subset\mathbb{R}

Benennung von Variablen

Für den besseren Überblick werden Variablen

  • aus der Menge der natürlichen oder ganzen Zahlen meist n oder m,
  • aus der Menge der rationalen Zahlen meist p oder q sowie
  • aus der Menge der reellen Zahlen meist x, y oder z

genannt. Es ist natürlich nicht verpflichtend, Variablen so zu benennen - manchmal geht es auch gar nicht ... Diese Namenskonventionen haben sich nur eingebürgert, weil es damit einfacher ist, den Überblick zu behalten, welche Variable aus welchem Zahlenbereich stammt.

Zur Notation: Im Zusammenhang mit den Zahlenbereichen werden häufig weitere Symbole und Schreibweisen verwendet, u. a.

  • Ein hinter dem Zahlenbereichssymbol hochgestelltes + bedeutet, dass nur der positive Teil dieses Zahlenbereichs gemeint ist (0 nicht eingeschlossen), z. B. meint \mathbb{R}^+ (gesprochen: "R plus") die Menge aller reellen Zahlen, die größer als 0 sind.
  • Ein hinter dem Zahlenbereichssymbol hochgestelltes - bedeutet, dass nur der negative Teil dieses Zahlenbereichs gemeint ist (0 nicht eingeschlossen), z. B. meint \mathbb{R}^- (gesprochen: "R minus") die Menge aller reellen Zahlen, die kleiner als 0 sind.
  • Eine hinter dem Zahlenbereichssymbol tiefgestellte 0 bedeutet, dass die 0 in den Zahlenbereich eingeschlossen wird, z. B. meint \mathbb{R}^+_0 (gesprochen: "R null plus") die Menge aller reellen Zahlen, die größer oder gleich 0 sind. Das ist natürlich nur dann nötig, wenn die 0 ansonsten nicht in dem Zahlenbereich enthalten wäre.
  • Möchte man einzelne Zahlen oder Intervalle aus einem Zahlenbereich ausschließen, verwendet man \setminus , z. B. meint \mathbb{R} \setminus_{ \{0\} } (gesprochen: "R ohne null") die Menge der reellen Zahlen ohne die Zahl 0.

Bemerkung: Manchmal werden auch andere (ähnliche) Symbole verwendet. Es sollte dann zu Beginn des Textes erklärt sein, welches Symbol für welchen Zusammenhang verwendet wird.

 

Intervalle

Definition: Ein Intervall ist die Menge aller reellen Zahlen, die zwischen zwei gegebenen Zahlen a und b liegen, wobei a sein muss. Diese Bedingung a bedeutet dabei nur, dass der untere Rand kleiner sein muss als der obere. Anders gesagt: Ein Intervall ist eine Menge von reellen Zahlen ohne Lücken.
Man unterscheidet abgeschlossene, halboffene und offene Intervalle: Bei offenen Intervallen sind die Randwerte nicht im Intervall enthalten. Abgeschlossene Intervalle umfassen auch die Randwerte. Halboffene Intervalle beinhalten einen der beiden Randwerte.

Zur Notation: Leider ist die Notation hier nicht ganz eindeutig: Man verwendet für die Darstellung von Intervallen eckige und teilweise auch runde Klammern. Nach innen gerichtete eckige Klammern schließen den Randwert in das Intervall ein. Nach außen gerichtete eckige Klammern und innen gerichtete runde Klammern schließen den Randwert aus. Da +\infty (also "plus unendlich") und -\infty (also "minus unendlich") keine (reellen) Zahlen sind, muss das Intervall hier immer offen bzw. halboffen sein. Hier ein paar Beispiele:

  • Das abgeschlossene Intervall \lbrack -1;2 \rbrack ist die Menge aller reellen Zahlen, die größer oder gleich -1 und kleiner oder gleich 2 sind.
    Wenn man die Mengenschreibweise von oben wiederholen möchte, kann man dafür auch \{x\in\mathbb{R} \, \vert \, -1 \leq x \leq 2\} schreiben.

 

  • Das halboffene Intervall [-1;2[ bzw. [-1;2) ist die Menge aller reellen Zahlen, die größer oder gleich -1 und kleiner 2 sind. Die 2 selber ist in dem Intervall nicht enthalten. Wichtig: Das bedeutet nicht, dass das Intervall bei 1 endet! Zwischen 1 und 2 liegen ja noch ganz viele weitere reelle Zahlen (z. B. 1{,}000001; \sqrt{2}; 1{,}5; \frac{17}{9} ), die alle in diesem Intervall enthalten sind.
    In Mengenschreibweise: \{x\in\mathbb{R} \, \vert \, -1 \leq x < 2\}
  • Das halboffene Intervall ]-1;2] bzw. (-1;2] ist die Menge aller reellen Zahlen, die größer, aber nicht gleich -1 und kleiner oder gleich 2 sind.
    In Mengenschreibweise: \{x\in\mathbb{R} \, \vert \, -1 < x \leq 2\}
  • Das halboffene Intervall ]-\infty;2] bzw. (-\infty;2] ist die Menge aller reellen Zahlen, die größer -\infty und kleiner oder gleich 2 sind. Man benötigt diese Schreibweise z. B., wenn man mithilfe eines Intervalls alle Zahlen beschreiben möchte, die kleiner oder gleich 2 sind. Da das Intervall ja auch eine untere Grenze braucht, nicht nur eine obere, schreibt man dort -\infty.
    In Mengenschreibweise: \{x\in\mathbb{R} \, \vert \, -\infty < x \leq 2\}

 

  • Das offene Intervall ]-1;2[ bzw. (-1;2) ist die Menge aller reellen Zahlen, die größer, aber nicht gleich -1 und kleiner, aber nicht gleich 2 sind. In Mengenschreibweise: \{x\in\mathbb{R} \, \vert \, -1 < x < 2\}

 

  • So etwas wie [5;2] geht nicht, weil der untere Randwert kleiner sein muss als der obere.

Bemerkung: Auch bei Intervallen können das \in-und das \not \in-Zeichen verwendet werden, z. B. gelten 1{,}5 \in [1;2[ und 2 \not \in [1;2[

 

n-Tupel

Definition: Ein n-Tupel ist eine geordnete Liste von n Zahlen.
Geordnet bedeutet, dass (anders als bei Mengen) die Reihenfolge, in der die Zahlen notiert sind, wichtig ist. (1;2) bedeutet also etwas Anderes als (2;1). Deswegen darf man hier natürlich auch nicht der Größe nach sortieren, wie es für Mengen empfohlen wird.
Statt 2-Tupel sagt man Paar und statt 3-Tupel Tripel.

Kartesisches Produkt von Mengen: Möchte man die Zahlenbereiche festlegen, aus denen die einzelnen Komponenten eines n-Tupels stammen sollen, so notiert man die entsprechenden Zahlenbereichssymbole mit dem Symbol \times dazwischen. Soll beispielsweise die erste Komponente eines Zahlenpaars ein Element der reellen Zahlen und die zweite Komponente ein Element der positiven reellen Zahlen sein, schreibt man \mathbb{R} \times \mathbb{R}^+.

Zur Notation: Man verwendet für die Darstellung von n-Tupeln runde Klammern, z. B.

  • (1{,}5;-4{,}3) ist ein Zahlenpaar, dessen erste Komponente 1{,}5 und dessen zweite Komponente -4{,}3 ist. So könnte z. B. ein Punkt in einem 2-dimensionalen Koordinatensystem beschrieben werden.
  • (7;7) ist ein Zahlenpaar, dessen erste Komponente 7 und dessen zweite Komponente auch 7 ist. So etwas könnte bei Mengen nicht passieren.
  • \left(\frac{1}{2};\frac{1}{3};\frac{1}{4}\right) ist ein Zahlentripel, dessen erste Komponente \frac{1} {2} , dessen zweite Komponente \frac{1} {3} und dessen dritte Komponente \frac{1} {4} ist. Hierbei könnte es sich um einen Punkt in einem 3-dimensionalen Koordinatensystem handeln.
  • (-12;0;5;28;-35) ist ein 5-Tupel, dessen erste Komponente -12, dessen zweite Komponente 0, dessen dritte Komponente 5, dessen vierte Komponente 28 und dessen fünfte Komponente -35 ist. Solche Angaben werden später bei Vektoren wichtig.

Dieses Kapitel enthält die folgenden Themen:

 

2.2 Grundlagen - Grundrechenarten

Wie der Name schon sagt, sind die Grundrechenarten die Grundlage aller weiteren Rechenoperationen. Daher lohnt es sich, einen Blick drauf zu werfen ... 

 

Bezeichnungen 

Die folgenden Bezeichnungen helfen, wenn über Rechnungen und Aufgaben gesprochen wird. Beispielsweise ist die Aussage "Einer der Summanden ist 3." eindeutig und deutlich weniger umständlich als "Eine der Zahlen, die vor oder hinter dem Pluszeichen steht, ist 3." Ebenso ist es bei "der Quotient aus x und y" im Vergleich zu "das Ergebnis, das ich erhalte, wenn ich x durch y teile".
Die Bezeichnungen Minuend, Subtrahend, Dividend und Divisor werden weniger häufig verwendet. Man sollte aber zumindest wissen, zu welcher Rechenart sie gehören:

Addition: \text{Summand} + \text{Summand} = \text{Summe} Das Adjektiv zu "Addition" ist "additiv".
Addition und Subtraktion werden auch Strichrechnung genannt.
Subtraktion: \text{Minuend} - \text{Subtrahend} = \text{Differenz}  
Multiplikation: \text{Faktor} \cdot \text{Faktor} = \text{Produkt} Das Adjektiv zu "Multiplikation" ist "multiplikativ".
Multiplikation und Division werden auch Punktrechnung genannt.
Division: \text{Dividend} : \text{Divisor} = \text{Quotient} Durch 0 darf man nicht teilen!


Dass man durch 0 nicht teilen darf, ist hinlänglich bekannt. Warum ist das so? Die Division beantwortet die Frage "Wie oft muss man den Divisor vom Dividend abziehen, damit das Ergebnis 0 ist?" Beispiel: 10 : 5 Wir rechnen 10-5-5 = 0. Die Antwort ist also 2. Hätten wir 10 : 0, kämen wir nie zu einer Antwort.
Ein praxisnäheres Beispiel: Wie verteilt man 10 Objekte auf 5 Plätze? Die Antwort: Auf jeden Platz kommen 2 Objekte. Die Frage "Wie verteilt man 10 Objekte auf 0 Plätze?" lässt sich hingegen nicht sinnvoll beantworten.

Wichtig: Sobald Variablen ins Spiel kommen, sieht man manchmal nicht mehr so leicht, ob ein Divisor 0 ist. Dann müssen spezielle Überlegungen angestellt werden, um diese Fälle auszuschließen.

 

Manchmal darf man auch ein bisschen faul sein bei der Notation: Solange Missverständnisse ausgeschlossen sind, darf der "Malpunkt" weggelassen werden:

  • Beispielsweise meinen a(b+c) und a \cdot (b+c) dasselbe. Bei 5 \, t und 5 \cdot t ist das ebenso. Logischerweise muss bei Aufgaben wie 4 \cdot 7 der "Malpunkt" immer hingeschrieben werden ...
  • Auch kann der Faktor 1 bei der Multiplikation weggelassen werden, z. B. ist x = 1x = 1 \cdot x.

Es ist in beiden Fällen natürlich nie falsch, den "Malpunkt" einfach mit hinzuschreiben.

 

Weitere Begriffe

Definition: Die Gegenzahl bzw. das Negative einer Zahl a ist -a . Es gilt: a+(-a)=0.
Bemerkung: -a muss nicht kleiner 0 sein. Im Gegenteil: Für alle negativen Zahlen ist die Gegenzahl positiv, z. B. ist 4 die Gegenzahl zu -4.

Definition: Der Betrag einer Zahl a , in einer Formel: \left| a\right|, ist ihr absoluter Wert. D. h. für positive Zahlen und 0 entspricht der Betrag einer Zahl a der Zahl selber, also \left| a \right| = a. Für negative Zahlen entspricht der Betrag einer Zahl a ihrer Gegenzahl, also \left| a \right| = -a. Wichtig ist der Betrag z. B. bei Abstandsberechnungen, weil es dabei ja nur auf die absolute Entfernung ankommt und nicht auf die Richtung, in der diese Entfernung durchlaufen wird. Der Betrag, so wie er hier definiert ist, liefert für jede Zahl ihren Abstand vom Nullpunkt.

Definition: Der Kehrwert zu einer Zahl a ist \dfrac{1} {a}. Man sagt auch a und \dfrac{1} {a} sind reziprok zueinander. Es gilt: a \cdot \dfrac{1}{a}=1.
Bemerkung 1: Da durch 0 nicht geteilt werden darf, muss hierbei a \neq 0 gelten.
Bemerkung 2: \frac {1}{a} muss nicht kleiner 1 sein, z. B. ist 2 der Kehrwert zu \frac{1} {2}.

 

Rechnen mit rationalen Zahlen

Es ist sehr wichtig, die folgenden Rechenregeln zu kennen, auch wenn in vielen Fällen natürlich der Taschenrechner weiterhilft. Sobald nämlich Variablen in den Rechnungen auftauchen, stoßen Taschenrechner sehr schnell an ihre Grenzen ...

 

Rechenregeln für die Addition rationaler Zahlen

Haben die beiden Summanden das gleiche Vorzeichen, werden die Beträge addiert. Die Summe bekommt das gemeinsame Vorzeichen.
Haben die beiden Summanden unterschiedliche Vorzeichen, zieht man den betragsmäßig kleineren Summanden vom betragsmäßig größeren ab. Die Summe bekommt das Vorzeichen des Summanden, der den größeren Betrag hat.

Die Subtraktion entspricht der Addition der Gegenzahl.

Das bedeutet konkret:

\begin{array}{rcrcccc} 20+7 &=& 27 &=& +20+(+7) &=& +20-(-7) \cr \cr 20-7 &=& 13 &=& +20+(-7) &=& +20-(+7) \cr \cr -20+7 &=& -13 &=& -20+(+7) &=& -20-(-7) \cr \cr -20-7 &=& -27 &=& -20+(-7) &=& -20-(+7) \end{array}

 

Rechenregeln für die Multiplikation rationaler Zahlen

Haben die beiden Faktoren das gleiche Vorzeichen, werden die Beträge multipliziert. Das Produkt ist positiv.
Haben die beiden Faktoren unterschiedliche Vorzeichen werden die Beträge multipliziert. Das Produkt ist negativ.

Kurze Merkregeln:

  • "minus mal minus ist plus" bzw. "plus mal plus ist plus"
  • "plus mal minus ist minus" bzw. "minus mal plus ist minus"

Die Division entspricht der Multiplikation mit dem Kehrwert.

Das bedeutet konkret:

\begin{array}{rcccrcccl} 3 \cdot 9 &=& +3 \cdot (+9) &=& 27 &=& +3 : \left(+\frac{1}{9}\right) &=& 3 : \frac{1}{9} \cr \cr3 \cdot (-9) &=& +3 \cdot (-9) &=& -27 &=& +3 : \left(-\frac{1}{9}\right) &=& 3 : \left(-\frac{1}{9}\right) \cr \cr -3 \cdot 9 &=& -3 \cdot (+9) &=& -27 &=& -3 : \left(+\frac{1}{9}\right) &=& -3 : \frac{1}{9} \cr \cr & & -3 \cdot (-9) &=& 27 &=& -3 : \left(-\frac{1}{9}\right) \end{array}

Bemerkung 1: "Überzählige" Pluszeichen (also Pluszeichen, die nur eine Vorzeichen- und keine Rechenfunktion haben) müssen nicht hingeschrieben werden.
Bemerkung 2: Immer, wenn ein Rechen- und ein Vorzeichen aufeinandertreffen, werden Klammern gesetzt.


Um die Regel “minus mal plus ist minus” ein bisschen plausibler zu machen, schauen wir uns Addition und Subtraktion auf dem Zahlenstrahl an:
Addiert man eine positive Zahl, beispielsweise 2, lässt sich dies auf dem Zahlenstrahl als 2 Schritte nach rechts veranschaulichen. Addiert man die gleiche Zahl mehrfach, wiederholt man diese Schritte. Das kann man als Multiplikation auffassen. Beispiel: 2+2+2 = 6 ergibt das Gleiche wie 3\cdot 2 = 6.

Veranschaulichung der Addition am Zahlenstrahl

Subtrahiert man eine positive Zahl, beispielsweise 2, lässt sich dies auf dem Zahlenstrahl als 2 Schritte nach links veranschaulichen. Subtrahiert man die gleiche Zahl mehrfach, wiederholt man ebenfalls diese Schritte. Auch das kann man als Multiplikation auffassen. Beispiel: -2-2-2 = -6 ergibt das Gleiche wie 3\cdot (-2) = -6. Man sieht also, dass die Multiplikation einer positiven mit einer negativen Zahl zu einem negativen Ergebnis führt.

Veranschaulichung der Subtraktion am Zahlenstrahl

 

Wichtig sind auch die Quadrat- und Kubikzahlen:

Quadratzahlen   Kubikzahlen
1\cdot 1 = -1\cdot (-1) = 1   1\cdot 1\cdot 1 = 1
2\cdot 2 = -2\cdot (-2) = 4   2\cdot 2\cdot 2 = 8
3\cdot 3 = -3\cdot (-3) = 9   3\cdot 3\cdot 3 = 27
4\cdot 4 = -4\cdot (-4) = 16   4\cdot 4\cdot 4 = 64
5\cdot 5 = -5\cdot (-5) = 25   5\cdot 5\cdot 5 = 125
6\cdot 6 = -6\cdot (-6) = 36   6\cdot 6\cdot 6 = 216
7\cdot 7 = -7\cdot (-7) = 49   7\cdot 7\cdot 7 = 343
8\cdot 8 = -8\cdot (-8) = 64   8\cdot 8\cdot 8 = 512
9\cdot 9 = -9\cdot (-9) = 81   9\cdot 9\cdot 9 = 729
10\cdot 10 = -10\cdot (-10) = 100   10\cdot 10\cdot 10 = 1.000
11\cdot 11 = -11\cdot (-11) = 121        
12\cdot 12 = -12\cdot (-12) = 144   -1\cdot (-1)\cdot (-1) = -1
13\cdot 13 = -13\cdot (-13) = 169   -2\cdot (-2)\cdot (-2) = -8
14\cdot 14 = -14\cdot (-14) = 196   -3\cdot (-3)\cdot (-3) = -27
15\cdot 15 = -15\cdot (-15) = 225   -4\cdot (-4)\cdot (-4) = -64
16\cdot 16 = -16\cdot (-16) = 256   -5\cdot (-5)\cdot (-5) = -125
17\cdot 17 = -17\cdot (-17) = 289   -6\cdot (-6)\cdot (-6) = -216
18\cdot 18 = -18\cdot (-18) = 324   -7\cdot (-7)\cdot (-7) = -343
19\cdot 19 = -19\cdot (-19) = 361   -8\cdot (-8)\cdot (-8) = -512
20\cdot 20 = -20\cdot (-20) = 400   -9\cdot (-9)\cdot (-9) = -729
21\cdot 21 = -21\cdot (-21) = 441   -10\cdot (-10)\cdot (-10) = -1.000
22\cdot 22 = -22\cdot (-22) = 484        
23\cdot 23 = -23\cdot (-23) = 529        
24\cdot 24 = -24\cdot (-24) = 576        
25\cdot 25 = -25\cdot (-25) = 625        

Bemerkung: In vielen Fällen wird ein Punkt gesetzt, um die einzelnen Tausender voneinander abzugrenzen. Manchmal wird stattdessen auch eine kleine Lücke in der Zahl gelassen. Verpflichtend ist beides nicht. Hauptsache, große Zahlen bleiben übersichtlich ...

 

Teilbarkeitsregeln

Vorab ein paar Begriffe:
Eine Zahl, die ohne Rest durch 2 teilbar ist, nennt man gerade Zahl. Bleibt beim Teilen durch 2 ein Rest, nennt man die Zahl ungerade.
Die Quersumme einer Zahl berechnet man, indem man einfach alle Ziffern addiert, z. B. ist die Quersumme von 123: 1+2+3=6

Die folgenden Regeln sind insofern bemerkenswert, weil sie Aussagen über die Teilbarkeit kompletter Zahlen ermöglichen - und dafür nur Teilinformationen heranziehen. Z. B. reicht es für die Aussage "123.456 ist durch 2 teilbar." aus, die letzte Ziffer, nämlich die 6, zu betrachten. Alle anderen Ziffern müssen gar nicht angeschaut werden. Sie sind nicht relevant.
Diese Regeln werden sowohl bei der Bruchrechnung als auch bei Potenzen sehr weiterhelfen.

Eine ganze Zahl ist
ohne Rest teilbar durch ...
wenn ...
2 ihre letzte Ziffer gerade ist.
3 ihre Quersumme durch 3 teilbar ist.
4 ihre letzten beiden Ziffern durch 4 teilbar sind.
5 ihre letzte Ziffer 5 oder 0 ist.
6 die Zahl durch 2 und durch 3 teilbar ist.
8 ihre letzten 3 Ziffern durch 8 teilbar sind.
9 ihre Quersumme durch 9 teilbar ist.
10 ihre letzte Ziffer 0 ist.
12 die Zahl durch 3 und durch 4 teilbar ist.
...  

 

Eine Zahl, die nur durch 1 und sich selbst teilbar ist, nennt man Primzahl. Jede Zahl ist durch 1 und sich selbst teilbar - das Besondere an Primzahlen ist, dass sie keine weiteren Teiler haben.

 

Runden

Gerundet wird u. a., um die Anzahl der Nachkommastellen zu reduzieren oder wenn nur die Größenordnung einer Zahl, nicht aber der exakte Wert wichtig ist. Fürs Runden gibt es folgende Regeln:

  • Abrunden: Ist die erste wegzulassende Dezimalstelle kleiner als 5, wird abgerundet.
  • Aufrunden: Ist die erste wegzulassende Dezimalstelle größer oder gleich 5, wird aufgerundet.

Nach dem Runden ist es wichtig, das Ungefährzeichen \approx anstelle des Gleichheitszeichens = zu verwenden, weil durch das Runden ja Genauigkeit verloren geht.

Ein paar Beispiele:
Beim Runden auf eine Nachkommastelle ist die zweite Nachkommastelle ausschlaggebend. Alle anderen Stellen werden ignoriert.
\begin{array}{rcl}1{,}44 &\approx& 1{,}4 \cr 2{,}48 &\approx& 2{,}5 \cr 5{,}12693 &\approx& 5{,}1 \end{array}

Beim Runden auf drei Nachkommastellen ist die vierte Nachkommastelle ausschlaggebend. Alle anderen Stellen werden ignoriert.
\begin{array}{ccrcl} & & 10{,}12348 &\approx& 10{,}123 \cr & & 8{,}8767 &\approx& 8{,}877 \cr 129{,} \overline3 &=& 129{,}3333 \ldots &\approx& 129{,}333 \end{array}

Es kann auch auf ganze 100.000 gerundet werden. Dann ist die 10.000er Stelle ausschlaggebend. Alle anderen Stellen werden ignoriert.
\begin{array}{rcl}872.146 &\approx& 900.000 \cr 927.549 &\approx& 900.000 \cr 590.190{,}12 &\approx& 600.000 \end{array}

Natürlich könnte man auch auf andere Stellen runden.


Wichtig: Runden Sie bei komplexeren Aufgaben nicht zu früh, sondern rechnen Sie so lange wie möglich mit den exakten Werten, weil sich sonst die Rundungsfehler sehr schnell zu problematischen Größenordnungen anhäufen. Ein (zugegebenermaßen leicht übertriebenes) Beispiel:
1.200\cdot 0{,}003 = 3{,}6
Rundet man die 0{,}003 korrekt auf zwei Stellen nach dem Komma, erhält man 0. Aus unserer Rechnung würde dann 1.200\cdot 0 = 0
Dass dies kein geschicktes Vorgehen ist, wird spätestens dann deutlich, wenn man sich überlegt, was in dieser Rechnung mit anderen Faktoren passieren würde: Es ist nun völlig egal, ob man 1.200 mit der "gerundeten 0{,}003" multipliziert oder -10.819 oder 6{,}841841 - das Ergebnis ist immer 0.

Dieses Kapitel enthält die folgenden Themen:

 

2.3 Grundlagen - Rechengesetze

Die Gesetze, die in diesem Kapitel besprochen werden, sind ein wenig wie die Regeln im Fußball: Im Sport wird mit den Regeln der zulässige Umgang mit dem Ball und den Mitspielenden festgelegt - in der Mathematik der zulässige Umgang mit Zahlen, Variablen etc. In beiden Fällen entscheidet die Einhaltung der Regeln darüber, ob man ein zulässiges Ergebnis erhält ...
Viele dieser Rechenregeln klingen eher unspektakulär - teilweise sicher auch, weil sie uns so in Fleisch und Blut übergegangen sind, dass wir gar nicht immer merken, wenn wir sie anwenden. Dieses Kapitel soll sie noch mal ins Bewusstsein rufen. Gleichzeitig macht es deutlich, dass diese Rechengesetze nicht selbstverständlich sind. Wenn im Verlauf der Mathematikausbildung weitere mathematische Objekte wie Vektoren und Matrizen hinzukommen, muss immer wieder die Frage gestellt werden, ob die Gesetze, die wir hier als "normal" ansehen, dafür auch gelten - und nicht überall ist das dann auch tatsächlich der Fall.

 

Das vielleicht wichtigste Rechengesetz zuerst: Punktrechnung geht vor Strichrechnung! Das heißt: Eine Kombination aus Summe/Differenz und Produkt/Quotient lässt sich nicht einfach von links nach rechts zusammenfassen.
Nur Klammern können diese Reihenfolge ändern.

 

Klammern sind also in vielen Fällen unerlässlich, weil sie und zwar nur sie die "normale" Rangfolge der Rechenoperationen ändern können. Ein paar Beispiele:

3+4\cdot 5 = 3+20 = 23   Die Punktrechnung muss zuerst gerechnet werden, auch wenn sie "weiter rechts" steht.
(3+4)\cdot 5 = 7\cdot 5 = 35   Die Klammern "erzwingen", dass die Summe zuerst berechnet wird.

7-6\cdot 10 = 7-60 = -53   Genauso hier: Erst multiplizieren, dann subtrahieren.
(7-6)\cdot 10 = 1\cdot 10 = 10   Bei Klammern: Erst die Klammern, dann alles andere.

11+7:7 = 11+1 = 12   Auch hier muss die Punktrechnung, in diesem Fall Division, zuerst gerechnet werden.
(11+7):7 = 18:7 = 2{,}571428...   Berechnet man zuerst die Summe, weil sie in Klammern steht, ist das Ergebnis deutlich anders.


Sie sehen, dass es tatsächlich einen Unterschied macht, in welcher Reihenfolge man die Rechenoperationen anwendet. Wichtig ist also, nicht einfach "drauf los" zu rechnen, sondern sich die Struktur der Aufgaben erst in Ruhe anzuschauen. Wenn man weiß, was auf einen zukommt, ist es viel einfacher, den Überblick zu behalten und die Rangfolge nicht durcheinander zu bringen.
Das ist nicht nur bei solch einfachen Rechnungen so, wo man den Unterschied recht schnell sieht, sondern erst recht dann, wenn die Rechnungen komplexer werden bzw. Variablen ins Spiel kommen. Anders gesagt: Beachtet man die Rangfolge der Rechenoperationen nicht, passieren quasi zwangsläufig Fehler. Manchmal sieht man die Fehler früher, manchmal später und manchmal fallen sie gar nicht auf ...
Das bedeutet zum Beispiel auch, dass Sie zwingend daran denken müssen, Klammern zu setzen, wenn im Laufe einer Rechnung mit einer Summe oder Differenz multipliziert werden muss. Dass Sie mit einer Summe oder Differenz multiplizieren müssen, wird häufig bei vielen verschiedenen Themen vorkommen, z. B. beim Lösen einer Bruchgleichung, wo mit dem Nenner multipliziert werden muss, oder beim Ableiten von verketteten Funktionen, wo mit der inneren Ableitung multipliziert werden muss. Es ist daher extrem wichtig, dass Sie sich jetzt schon mit diesen Rechenregeln vertraut machen - denn niemand wird Ihnen bei solchen Aufgaben im Studium sagen, dass Sie an die Klammern denken sollen …

 

Kommutativgesetz

Das Kommutativgesetz wird auch Vertauschungsgesetz genannt.

Es gilt für alle reellen Zahlen a,b \in \mathbb{R}:

a+b = b+a
a \cdot b = b \cdot a

 

Im Gegensatz zur Addition und Multiplikation sind die Subtraktion, die Division und die Potenzierung im Bereich der reellen Zahlen nicht kommutativ.

Hinweis: Bei allen folgenden Beispielen müssen die Gleichungen von außen nach innen gelesen werden.
Beispiele für kommutative Rechenoperationen

Addition: 5+12 = 17 = 12 + 5
Multiplikation: 4 \cdot 9 = 36 = 9 \cdot 4


Beispiele für nicht kommutative Rechenoperationen

Subtraktion: 14-8 = 6 \quad \neq \quad -6 = 8-14
Division: 2 : 1 = 2 \neq 0{,}5 = 1 : 2
Potenzierung: 3^2 = 9 \neq 8 = 2^3

 

Das Kommutativgesetz ist nicht in allen Situationen so selbstverständlich, wie es manchmal scheint. Gerade im Alltag gibt es viele Situationen, in denen Handlungen oder Vorgehensweisen nicht vertauschbar sind. Denken Sie beispielsweise daran, was passieren würde, wenn Sie morgens erst die Jacke und dann das Unterhemd anziehen oder erst die Marmelade und dann die Butter aufs Brot streichen ...

 

Assoziativgesetz

Das Assoziativgesetz wird auch Verknüpfungs- oder Klammergesetz genannt.

Es gilt für alle reellen Zahlen a, b \in \mathbb{R}:

(a+b)+c = a+(b+c)
(a \cdot b) \cdot c = a \cdot (b \cdot c)

 

Im Gegensatz zur Addition und Multiplikation sind die Subtraktion, die Division und die Potenzierung im Bereich der reellen Zahlen nicht assoziativ.

Hinweis: Auch hier muss wieder von außen nach innen gelesen werden.
Beispiele für assoziative Rechenoperationen

Addition: (1+2)+3 = 3+3 = 6 = 1+5 = 1+(2+3)
Multiplikation: (2 \cdot 3) \cdot 4 = 6 \cdot 4 = 24 = 2 \cdot 12 = 2 \cdot (3 \cdot 4)


Beispiele für nicht assoziative Rechenoperationen

Subtraktion: (1-2)-3 = -1-3 = -4 \neq 2 = 1-(-1) = 1-(2-3)
Division: (8:4):2 = 2:2 = 1 \neq 4 = 8:2 = 8:(4:2)
Potenzierung: \left(2^2 \right)^3 = 4^3 = 64 \neq 256 = 2^8 = 2^{\left(2^3\right)}

 

Das Assoziativgesetz, das ja besagt, dass es in bestimmten Situationen egal ist, wie man bei gleichartigen Rechenoperationen die Klammern setzt, gilt glücklicherweise für sehr viele Rechenoperationen. Wichtig: Kommen verschiedene Rechenoperationen zusammen, ist es meist überhaupt nicht egal, wie die Klammern gesetzt werden!
Auch in der Sprache ist das grundsätzlich anders: Beim Wort "Wollhosenträger" kann man nicht ohne Weiteres entscheiden, ob es sich um einen Hosenträger aus Wolle oder um den Träger einer Wollhose handelt ...

 

Das Assoziativgesetz ist der Grund, warum -(a \cdot b) = -1 \cdot (a \cdot b) = -1 \cdot a \cdot b = -ab gilt.

 

Distributivgesetz

Das Distributivgesetz stellt eine Verknüpfung zwischen verschiedenen Rechenoperationen her, z. B. zwischen Multiplikation und Addition bzw. Subtraktion.

Es gilt für alle reellen Zahlen a,b,c \in \mathbb{R}:

a \cdot \left( b \pm c \right) = a \cdot b \pm a \cdot c
\left( a \pm b \right) \cdot c = a \cdot c \pm b \cdot c
\left( a \pm b \right) : c = a : c \pm b : c

Erklärung: Das Zeichen \pm (gesprochen: "plusminus") ist eine abkürzende Schreibweise und bedeutet, dass die Gleichung sowohl gilt, wenn an dieser Stelle ein + steht als auch, wenn dort ein - steht. Man liest in der Gleichung entweder bei allen Doppelzeichen das Obenstehende oder bei allen Doppelzeichen das Untenstehende.

 

Hinweis: Lesen Sie bitte wieder von außen nach innen.
Beispiele für distributive Rechenoperationen

4 \cdot (10+7) = 4 \cdot 17 = 68 = 40+28 = 4 \cdot 10+4 \cdot7
(20-2) \cdot 9 = 18 \cdot 9 = 162 = 180-18 = 20 \cdot 9-2 \cdot 9
(20+6):2 = 26:2 = 13 = 10+3 = 20:2+6:2


Beispiel für nicht distributive Rechenoperationen

20:(2+2) = 20:4 = 5 \neq 20 = 10+10 = 20:2+20:2

 

Eine direkte Folge des Distributivgesetzes ist das sogenannte Ausmultiplizieren (auch "Klammern auflösen" genannt): (a+b) \cdot (c+d)=ac+ad+bc+bd.
Beim Ausmultiplizieren muss man also jeden Summanden der einen Summe mit jedem Summanden der anderen Summe multiplizieren. Beispiel:
\begin{array}{rcl}(5-14)\cdot(-10+1) &=& 5\cdot(-10) + 5\cdot 1+(-14)\cdot(-10)+(-14)\cdot 1 \cr &=& -50+5+140-14 \cr &=& 81 \end{array}
Alternativ hätte man hier natürlich auch einfach (5-14)\cdot(-10+1) = -9\cdot(-9) = 81 rechnen können ... Sobald Variablen ins Spiel kommen, geht es aber nicht mehr ohne die Formel oben.

Wendet man die Rechenvorschrift von rechts nach links an, heißt der Vorgang Ausklammern oder Faktorisieren. Der Begriff "Faktorisieren" macht deutlich, dass dadurch ein Produkt entsteht.
Beim Ausklammern muss man zuerst alle Summanden so weit wie möglich in Faktoren zerlegen. Der größte gemeinsame Faktor kann dann vor die Klammer gezogen werden. Den "Rest" jedes Summanden schreibt man in die Klammer. Alternativ kann man jeden Summanden durch den größten gemeinsamen Faktor dividieren, um zu ermitteln, was in der Klammer stehen bleibt. Beispiel:
\begin{array}{rcl} -60-\dfrac{5}{2}+35 &=& -5\cdot 12+(-5)\cdot\dfrac{1}{2}-5\cdot (-7) \cr &=& -5\left(12+\dfrac{1}{2}-7\right) \end{array}
Wenn man jede Seite zusammenrechnet, erhält man -\dfrac{55}{2} = -5\cdot\dfrac{11}{2}. Passt!

Ausmultiplizieren und Ausklammen sind Gegenoperationen. Das bedeutet, dass sie sich gegenseitig aufheben. Wenn Sie also unsicher sind, ob Sie richtig ausgeklammert haben, können Sie die Probe durchführen, indem Sie die Klammer wieder ausmultiplizieren.

 

Das Distributivgesetz ist der Grund, warum ein Minuszeichen vor einer Klammer alle Vorzeichen in der Klammer ändert: -(-a+b) = -1 \cdot (-a+b) = -1 \cdot (-a) + (-1) \cdot (+b) = a-b

Bitte beachten Sie den Unterschied zum roten Kasten beim Assoziativgesetz!

Bei -x versteckt sich diese Rechenregel ein bisschen, aber -x meint ja -1\cdot x und ist damit auch eine Multiplikation. Wenn x irgendeine Summe ist, z. B. x = -4+17y, müssen also unbedingt Klammern gesetzt werden: -x = -(-4+17y) = -1\cdot(-4+17y) = -1\cdot(-4)+(-1)\cdot(+17y) = 4-17y

 

Klammern sind wichtig!

Last but not least: Ein paar grundsätzliche Dinge zu Klammern:

  • Da Klammern die einzige Möglichkeit sind, Einfluss auf die Rechenreihenfolge zu nehmen (siehe oben), sollte man sich immer sorgfältig überlegen, ob welche gebraucht werden. Das gilt vor allem dann, wenn verschiedene Rechenoperationen aufeinandertreffen. Dabei gilt: Einmal Klammern zu viel ist fast immer besser als einmal Klammern zu wenig! Wenn Sie sich nicht sicher sind, schreiben Sie also lieber Klammern hin.
    Auch der Taschenrechner berechnet exakt das, was Sie eingeben, und achtet dabei peinlich genau auf jede einzelne Klammer …

  • Wenn ein Term ansonsten zu unübersichtlich werden würde, können Sie verschiedene Arten von Klammern nutzen, z. B. runde, eckige, geschweifte, spitze.
    Achtung: Bei Mengen, Intervallen und n-Tupel haben verschiedene Arten von Klammern unterschiedliche Bedeutungen! Hier darf also nicht "einfach so" eine andere Klammerart genutzt werden.

  • Verschachtelte Klammern werden von innen nach außen aufgelöst! Ein Beispiel:
      4\cdot\left(\right.- \left(20\right. - \left(7+8\right) \left.\right) + 3 \cdot \left(2-6\right) \cdot \left(\frac{1}{2}+1\right) \left.+1\right)
    = 4\cdot\left(\right.- \left(20\right. - 15 \left.\right) + 3 \cdot \left(2-6\right) \cdot \left(\frac{1}{2}+1\right) \left.+1\right)
    = 4\cdot\left(\right.- 5 + 3 \cdot \left(-4\right) \cdot \frac{3}{2} \left.+1\right)
    = 4\cdot\left(\right.- 5 - 18 \left.+1\right)
    = 4\cdot\left(\right. -22 \left.\right)
    = -88
    Achten Sie bitte darauf, dass in der dritten Zeile natürlich "Punktrechnung geht vor Strichrechnung" gilt! Das Produkt 3\cdot\left(-4\right)\cdot\frac{3}{2} muss daher zuerst berechnet werden.

  • Ein Bruchstrich wirkt wie eine Klammer. Dafür wird es im Kapitel Bruchrechnung noch ein paar Beispiele geben.

  • Und natürlich sollte es immer so viele öffnende Klammern wie schließende geben ...

Dieses Kapitel enthält die folgenden Themen:

 

2.4 Grundlagen - Dreisatz

Der Dreisatz, auch Verhältnisgleichung oder Schlussrechnung genannt, ist ein üblicherweise dreischrittiges Verfahren, mit dem aus drei gegebenen Größen eine vierte berechnet werden kann, wenn die Größen in einem bestimmten Verhältnis zueinanderstehen. Bereits Adam Ries beschrieb dieses Vorgehen im 16. Jahrhundert in seinen Rechenbüchlein.

 

Proportional und Antiproportional

Zunächst müssen wir zwei wichtige Formen von Verhältnissen klären.

Zwei Größen heißen proportional zueinander, wenn sie sich im gleichen Verhältnis ändern. Das heißt: Verdoppelt sich die eine Größe, verdoppelt sich auch die andere. Verdreifacht sich die eine Größe, verdreifacht sich auch die andere. Wird die eine Größe durch 5 dividiert, wird auch die andere Größe durch 5 dividiert. Und so weiter …
Merksätze: „Je mehr, desto mehr.“ oder „Je weniger, desto weniger.“

Bei der grafischen Darstellung solcher Zuordnungen ergibt sich eine Ursprungsgerade:

 

Zwei Größen heißen antiproportional (auch indirekt oder umgekehrt proportional) zueinander, wenn sie sich im umgekehrten Verhältnis ändern. Das heißt: Verdoppelt sich die eine Größe, halbiert sich die andere. Wird die eine Größe mit 3 multipliziert, wird die andere durch 3 dividiert. Wird die eine Größe durch 5 dividiert, wird die andere mit 5 multipliziert. Und so weiter …
Merksätze: „Je mehr, desto weniger.“ oder „Je weniger, desto mehr.“

Bei der grafischen Darstellung solcher Zuordnungen ergibt sich eine Hyperbel:

Berechnung

Ist ein Paar zusammengehörender Werte bekannt, kann über den Dreisatz ein zweites Paar berechnet werden, wenn dort nur ein Wert gegeben ist. Für die konkrete Berechnung schauen wir uns zwei Beispiele an.

Möchte man Eierkuchen für 3 Personen backen, benötigt man 420\,g Mehl. Wie viel Mehl benötigt man für 8 Personen?

 

Eine Tippgemeinschaft bestehend aus 5 Personen hat im Lotto gewonnen. Jede/r von ihnen erhielt 55.800 \;\text{EUR}. Wie viel Geld hätte jede/r gewonnen, wenn die Tippgemeinschaft aus 6 Personen bestanden hätte?

 

Als erstes muss entschieden werden, ob eine proportionale oder eine antiproportionale Zuordnung vorliegt.

 

Für doppelt so viele Personen werden doppelt so viele Eierkuchen, sprich doppelt so viel Mehl, benötigt. Es handelt sich also um eine proportionale Zuordnung.

 

Besteht die Tippgemeinschaft aus doppelt so vielen Personen, ist der Gewinnanteil für jede Person nur halb so groß. Es handelt sich also um eine antiproportionale Zuordnung.

 

 

1. Schritt: Zunächst müssen die gegebenen Werte in eine Art Gleichung geschrieben werden. Praktischerweise sollte dabei die Größe, deren Wert gesucht ist, auf der rechten Seite stehen.
Bitte achten Sie darauf, dass zwischen den Werten kein Gleichheitszeichen, sondern ein "entspricht"-Zeichen \begin{array}{rcl}\widehat{=}\end{array} stehen muss. 3 Personen sind sicher nicht das Gleiche wie 420\,g Mehl ...

 
1. Schritt Dreisatz proportional
 
1. Schritt Dreisatz antiproportional
 

 

2. Schritt: Bei beiden Varianten wird als Zwischenschritt die Entspricht-Gleichung so umgeformt, dass auf der linken Seite 1 "Einheit" steht, was auch immer diese Einheit ist. Hier wird also berechnet, wieviel Mehl für "1 Person" nötig ist bzw. wieviel Geld "1 Person" bekommen würde. Dazu wird der Wert auf der linken Seite entsprechend multipliziert oder dividiert.

Achtung 1: Beim Dreisatz wird nur multipliziert und dividiert, nie addiert oder subtrahiert!
Achtung 2: Abhängig davon, ob es sich um eine proportionale oder antiproportionale Zuordnung handelt, unterscheiden sich die Vorgehensweisen! Bei einer proportionalen Zuordnung wird auf der rechten Seite exakt die gleiche Rechenoperation durchgeführt wie links. Bei antiproportionalen Zuordnungen ist die Gegenoperation nötig, sprich: Wird auf der linken Seite multipliziert, muss rechts dividiert werden und umgekehrt.

 
2. Schritt Dreisatz proportional
 
2. Schritt Dreisatz antiproportional
 

 

3. Schritt: Wir formen weiter um, sodass sich in der dritten Zeile links der aus der Aufgabenstellung gegebene Wert ergibt. Auf der rechten Seite steht dann das Ergebnis.

Achtung: Ebenso wie oben unterscheiden sich hier die Vorgehensweisen für proportionale und antiproportionale Zuordnungen.

 
3. Schritt Dreisatz proportional

Ergebnis: Möchte man Eierkuchen für 8 Personen backen, benötigt man 1.120\,g Mehl.

 
3. Schritt Dreisatz antiproportional

Ergebnis: Würde die Tippgemeinschaft aus 6 Personen bestehen, bekäme jede/r nur 46.500\;\text{EUR}.

 

Eine Erkenntnis zum Abschluss: Die Einheiten verändern sich bei Dreisatzrechnungen nicht. Das führt insbesondere dazu, dass untereinander immer gleiche Einheiten stehen.

 

Worauf man achten muss

Es gibt natürlich auch Fragestellungen, bei denen die Größen weder proportional noch antiproportional zueinander sind.

Ein Beispiel: Zwei Musiker spielen das Lied „Happy Birthday“ in 25 Sekunden. Wie lange brauchen vier Musiker? Natürlich hat die Anzahl der Musizierenden keinen Einfluss auf die Dauer – die Aufgabe klingt also nur nach Dreisatz … Man muss sich also immer vor der Dreisatzrechnung davon überzeugen, dass die Größen tatsächlich voneinander abhängig sind.

Auch das Eierkuchenbeispiel von oben ist durchaus problematisch: Wir sind stillschweigend davon ausgegangen, dass die Portionsgrößen alle gleich sind, sprich dass alle gleich viel essen. Dies muss aber nicht so sein: Stellen Sie sich vor, es kommen zwei kleine Kinder und sechs ausgehungerte Jugendliche zum Essen … Dann werden die Portionsgrößen sehr unterschiedlich sein. Nur zu prüfen, ob es sich um eine "Je mehr, desto mehr"- oder um eine "Je mehr, desto weniger"-Zuordnung handelt, ist also zu wenig. Es muss sichergestellt sein, dass jede Einheit gleich groß ist, gleich viel kostet etc. Insbesondere wenn viele Faktoren eine Größe beeinflussen, ist das häufig schwierig festzustellen.

Dieses Kapitel enthält die folgenden Themen:

 

2.5 Grundlagen - Koordinatensystem

Ein Koordinatensystem ist ein geometrisches Schema, welches benutzt wird, um Punkte, Funktionsgraphen etc. eindeutig zu positionieren. In diesem Lernmodul werden nur Koordinatensysteme, bei denen die Achsen im rechten Winkel zueinanderstehen, verwendet. Man nennt sie auch kartesische Koordinatensysteme - nach dem französischen Philosophen und Mathematiker René Descartes, der sich als einer der ersten intensiv mit ihnen beschäftigt hat und damit für ihre Verbreitung gesorgt hat.

 

Das zweidimensionale kartesische Koordinatensystem

Für zwei Dimensionen (sprich: für die Ebene) braucht man naheliegenderweise zwei Achsen. Die horizontale Achse nennen wir in einem solchen Fall Abszisse, die vertikale Achse Ordinate. Heißen die Variablen x und y, sagt man statt Abszisse auch x-Achse und statt Ordinate auch y-Achse; bei anderen Variablenbezeichnungen entsprechend. Die Achsen sollten immer beschriftet werden. Wenn es sich anbietet, kann dazu der inhaltliche Zusammenhang einschließlich der entsprechenden Einheit verwendet werden, z. B. "Zeit in Stunden". Um die Darstellung im Koordinatensystem nicht zu verzerren, ist es meist hilfreich, auf beiden Achsen die gleiche Skaleneinteilung zu verwenden. Um das Koordinatensystem vollständig zu machen, bekommen die Achsen an ihrem positiven Ende einen kleinen Pfeil, der andeutet, dass auch größere Zahlenwerte betrachtet werden könnten.
Jeder Punkt in diesem Koordinatensystem hat eine eindeutige "Adresse", die aus zwei Koordinaten besteht: einer x-Koordinate und einer y-Koordinate. Man schreibt dafür P \; (x \mid y). Wichtig dabei ist die Reihenfolge: Der erste Wert bezieht sich immer auf die x-Achse und gibt an, wie weit links oder rechts sich der Punkt befindet. Der zweite Wert bezieht sich immer auf die y-Achse und gibt an, wie weit oben oder unten sich der Punkt befindet. Formal gesehen ist ein Punkt also ein 2-Tupel oder Paar. Der Punkt (0 \mid 0) heißt Koordinatenursprung oder kurz Ursprung.
Punkte werden klassischerweise mit großen lateinischen Buchstaben, am liebsten mit A, B, C und D oder P, Q und R, bezeichnet. Auch hier können Indizes verwendet werden. Für den Koordinatenursprung hat sich der Buchstabe O, vom lateinischen Wort origo für Ursprung, eingebürgert.

Schauen wir uns ein paar Punkte im Koordinatensystem (siehe Grafik rechts) an:

P1 \; (1 \mid 2)
P2 \; (-3 \mid 8)
P3 \; (-6 \mid -4{,}5)
P4 \; (9 \mid -9)

 

Das obere rechte Viertel des Koordinatensystems heißt 1. Quadrant. Hier sind sowohl x- als auch y-Werte positiv.
Das obere linke Viertel des Koordinatensystems heißt 2. Quadrant. Hier sind die x-Werte negativ und die y-Werte positiv.
Das untere linke Viertel des Koordinatensystems heißt 3. Quadrant. Hier sind sowohl x- als auch y-Werte negativ.
Das untere rechte Viertel des Koordinatensystems heißt 4. Quadrant. Hier sind die x-Werte positiv und die y-Werte negativ.

Bitte wundern Sie sich nicht über die Reihenfolge der Nummerierung. Drehungen entgegengesetzt des Uhrzeigersinns sind in der Mathematik die Regel und werden auch als "mathematisch positiv" bezeichnet. Warum man oben rechts angefangen hat zu zählen, ist vermutlich klar ;-).

 

Das dreidimensionale kartesische Koordinatensystem

rechte Hand mit Achsbeschriftungen
3-dimensionales Koordinatensystem

Das Ganze gibt es (natürlich) auch mit drei Achsen, um mathematische Objekte im Raum beschreiben zu können. Wieder stehen (zumindest in diesem Lernmodul) diese Achsen senkrecht zueinander und schneiden sich im Punkt O \; (0\mid 0 \mid 0). Sie können sich das wie die Ecke eines "normalen" Zimmers (ohne Dachschräge und so) vorstellen. In jeder Kante (Wand-Wand, Boden-Wand, Boden-andere Wand) liegt dann eine Achse.

Wenn wir bei den Bezeichnungen von oben bleiben, kommt zur x- und y-Achse nun die z-Achse hinzu. Gelegentlich werden sie auch x_1-, x_2- und x_3-Achse genannt oder die Bezeichnungen an den inhaltlichen Zusammenhang angepasst, aber das ändert logischerweise nichts an den Grundsätzen. Üblicherweise nutzt man die z-Achse, um die Höhe zu beschreiben, und die x- und y-Achse für die Grundfläche, wobei die x-Achse nach vorne und die y-Achse zur Seite zeigt. Dadurch entsteht ein so genanntes Rechtssystem oder auch rechtshändiges Koordinatensystem. D. h., wenn man den Daumen der rechten Hand in Richtung der x-Achse zeigen lässt, zeigen der Zeigefinger in Richtung der y-Achse und der Mittelfinger in Richtung der z-Achse.

Wichtig zu beachten ist, dass eine zweidimensionale Darstellung eines dreidimensionalen Objektes immer eine Verzerrung bewirkt. Im klassischen Schrägbild zeichnet man die x-Achse in einem Winkel von 135° zu den beiden anderen Achsen ein (schräg nach vorne), wobei die Einheiten dieser Achse um den Faktor \frac{1}{2} \sqrt{2} verkürzt werden. Meist beschränkt man sich auf die positiven Abschnitte der Achsen, weil das Gesamtbild sonst zu unübersichtlich würde.


Auch hier hat jeder Punkt anhand der Koordinaten eine "Adresse". Diese muss natürlich aus drei Komponenten bestehen - für jede Achsenrichtung eine. Um den Punkt P \; (x\mid y \mid z) in das Koordinatensystem einzutragen, verfährt man folgendermaßen:

  1. x Einheiten parallel zur x-Achse abtragen (also nach vorne oder - wenn x negativ ist - nach hinten)
  2. von dort y Einheiten parallel zur y-Achse abtragen (also nach rechts oder - wenn y negativ ist - nach links)
  3. von dort z Einheiten parallel zur z-Achse abtragen (also nach oben oder - wenn z negativ ist - nach unten)


Ein Beispiel: In dem Koordinatensystem ist der Punkt P \; (4\mid 5 \mid 1) inklusive Hilfslinien eingezeichnet.

In dieser Grafik sieht man allerdings auch, welches Problem bei Punkten in einem solchen dreidimensionalen Koordinatensystem auftritt: Sie lassen sich zwar problemlos einzeichnen. Das Ablesen eines gegebenen Punktes liefert jedoch keine eindeutigen Koordinaten. Der eingezeichnete Punkt könnte beispielsweise auch die Koordinaten (2\mid 4 \mid 0) oder (5\mid 5{,}5 \mid 1{,}5) haben. Allein aus der Darstellung kann man den Unterschied nicht erkennen. Zur Veranschaulichung von Funktionsgebirgen (das sind Graphen von Funktionen mit mehreren Variablen) und Vektoren lohnt sich das Zeichnen solcher Koordinatensysteme trotzdem.

3. Bruchrechnung - Lernziele und typische Fehler

Nach Durcharbeiten dieses Kapitels sollten Sie folgende Lernziele erreicht haben:

  • Sie erinnern sich an die Begriffe der Bruchrechnung.
  • Sie wissen, was das kleinste gemeinsame Vielfache zweier natürlicher Zahlen ist und können dieses berechnen.
  • Sie wissen, was der größte gemeinsame Teiler zweier natürlicher Zahlen ist und können diesen berechnen.
  • Sie verstehen Visualisierungen von Bruchteilen.
  • Sie können Brüche erweitern und kürzen.
  • Sie wissen, was ein Hauptnenner ist und wozu man ihn braucht.
  • Sie können zwei Brüche gleichnamig machen.
  • Sie können Brüche addieren, subtrahieren, multiplizieren und dividieren.
  • Sie können Brüche in gemischte Zahlen umwandeln und umgekehrt.
  • Sie wissen, warum gemischte Zahlen mit Vorsicht zu behandeln sind.
  • Sie können Brüche in Dezimalzahlen umrechnen und umgekehrt.


Typischer Fehler
in diesem Kapitel ist:

  • Brüche werden vor dem Addieren und Subtrahieren nicht gleichnamig gemacht. Erklärung


Für Online-Selbsttests zu diesem Thema und weitere Informationen zur Mathematikunterstützung an der TH Wildau nutzen Sie bitte den Moodle-Kursraum "SOS Mathematik - Brückenkurs".

Übersicht:

 

3.1 Bruchrechnung - Aufgaben

Alle diese Aufgaben sollten Sie ohne Taschenrechner berechnen. Sinn der Übung ist ja nicht, dass Sie Ihren Taschenrechner bedienen lernen, sondern dass Sie den Umgang mit Brüchen trainieren. Spätestens in Kapitel 5, in dem Variablen ins Spiel kommen, hilft Ihnen der Taschenrechner ohnehin nur noch eingeschränkt weiter ... Wenn man dann den Umgang mit Brüchen nie geübt hat, gehen auch Ableitungen und Integrale leicht schief - selbst wenn man die wesentlich komplizierteren Ableitungs- und Integrationsregeln eigentlich kann.

 

1. Aufgabe

Erweitern Sie die folgenden Brüche mit der jeweils angegebenen Zahl!

1) \dfrac {4}{5} mit 3

  11) \dfrac {29}{30} mit 4

2) \dfrac{1}{10} mit 12

  12) \dfrac{654}{125} mit 3

3) \dfrac{7}{12} mit 5

  13) \dfrac{70}{93} mit 80

4) \dfrac{583}{15} mit 10

  14) \dfrac{12}{35} mit 6

5) \dfrac {2}{3} mit 21

  15) \dfrac {576}{688} mit 1.000

6) \dfrac{123}{456} mit 100

  16) \dfrac{334}{777} mit 2

7) \dfrac{5}{6} mit 4

  17) \dfrac{6}{7} mit 13

8) \dfrac{3}{8} mit 11

  18) \dfrac{70}{3} mit 5

9) \dfrac{11}{7} mit 9

  19) \dfrac{5}{8} mit 9

10) \dfrac{211}{30} mit 25

  20) \dfrac{130}{621} mit 20

 

2. Aufgabe

Erweitern Sie die folgenden Brüche so, dass sie gleichnamig werden.

1) \dfrac{10}{13} und \dfrac {2}{3}

  11) \dfrac {1}{2} und \dfrac{1}{20} und \dfrac{3}{10}

2) \dfrac {8}{9} und \dfrac{4}{15}

  12) \dfrac {1}{3} und \dfrac{7}{20} und \dfrac{2}{11}

3) \dfrac {1}{8} und \dfrac {5}{6}

  13) \dfrac {7}{4} und \dfrac{1}{20} und \dfrac{132}{5}

4) \dfrac{99}{12} und \dfrac{5}{18}

  14) \dfrac {3}{70} und \dfrac{201}{30} und \dfrac{1}{14}

5) \dfrac{11}{20} und \dfrac{2}{19}

  15) \dfrac {5}{21} und \dfrac {1}{12} und \dfrac{4} {3} und \dfrac{2}{21}

6) \dfrac{20}{7} und \dfrac{7}{50}

  16) \dfrac {3}{4} und \dfrac {1}{8} und \dfrac{13} {16} und \dfrac{5}{24}

7) \dfrac{17}{3} und \dfrac{10}{9}

  17) \dfrac {3}{5} und \dfrac {20}{7} und \dfrac{13} {2} und \dfrac{54}{70}

8) \dfrac{3}{11} und \dfrac{1}{6}

  18) \dfrac {19}{2} und \dfrac {13}{6} und \dfrac{37} {120} und \dfrac{91}{180}

9) \dfrac{32}{27} und \dfrac{3}{2}

  19) \dfrac {1}{7} und \dfrac{8} {9} und \dfrac{33} {14} und \dfrac{10} {63} und \dfrac{25} {24}

10) \dfrac{3}{100} und \dfrac{7}{4}

  20) \dfrac {1}{2} und \dfrac{1} {3} und \dfrac{1} {4} und \dfrac{1} {5} und \dfrac{1} {6} und \dfrac{1} {15} und \dfrac{1} {36}

 

3. Aufgabe

Kürzen Sie die folgenden Brüche so weit wie möglich!

1) \dfrac{10}{18}

  11) \dfrac{121}{22}

2) \dfrac{51}{17}

  12) \dfrac{91}{70}

3) \dfrac{38}{171}

  13) \dfrac{90}{810}

4) \dfrac{30}{205}

  14) \dfrac{96}{48}

5) \dfrac{38}{4}

  15) \dfrac{131}{3}

6) \dfrac{124.000}{987.000}

  16) \dfrac{2.500}{5.000}

7) \dfrac{21}{49}

  17) \dfrac{110}{1.320}

8) \dfrac{39}{169}

  18) \dfrac{42}{7}

9) \dfrac{28}{42}

  19) \dfrac{69}{6}

10) \dfrac{23}{30}

 
20) \dfrac{120}{18}

 

4. Aufgabe

Wandeln Sie die folgenden unechten Brüche in gemischte Zahlen um und umgekehrt. Kürzen Sie die entstehenden Brüche, wenn möglich.

1) \dfrac{67}{6}

  11) 2\dfrac{1}{9}

2) \dfrac{25}{23}

  12) 1\dfrac{13}{14}

3) \dfrac{94}{24}

  13) 8\dfrac{4}{7}

4) \dfrac{119}{17}

  14) 10\dfrac{14}{15}

5) \dfrac{235}{50}

  15) 5\dfrac{20}{33}

6) \dfrac{95}{3}

  16) 12\dfrac{3}{4}

7) \dfrac{123}{11}

  17) 3\dfrac{14}{27}

8) \dfrac{155}{12}

  18) 19\dfrac{1}{8}

9) \dfrac{41}{7}

  19) 63\dfrac{7}{9}

10) \dfrac{80}{9}

  20) 32\dfrac{11}{13}

 

5. Aufgabe

Wandeln Sie folgende Brüche in Dezimalzahlen um (wenn nötig, gerundet auf 4 Stellen nach dem Komma) und umgekehrt.

1) \dfrac{7}{12}

  11) 0{,}24

2) \dfrac{18}{5}

  12) 2{,}\overline{6}

3) \dfrac{61}{650}

  13) 0{,}67

4) \dfrac{18}{18}

  14) 6{,}25

5) \dfrac{23}{16}

  15) 0{,}\overline{83}

6) \dfrac{2}{99}

  16) 3{,}008

7) \dfrac{120}{10}

  17) 0{,}5058

8) \dfrac{63}{85}

  18) 5

9) \dfrac{16}{25}

  19) 6{,}625

10) \dfrac{1}{130}

  20) 0{,}025

 

6. Aufgabe

Berechnen Sie folgende Aufgaben.

1) \dfrac{3}{4} \, + \, \dfrac{3}{2}

 

11)  34 \cdot \dfrac{1}{2}

2) 1\dfrac{5}{6} \, + \, 2\dfrac{7}{8}

  12) 2\dfrac{1}{4} \, \cdot \, \dfrac{2} {7}

3) \dfrac{3}{2} \, + \, 12

  13)  \dfrac{3}{8} \, : \, \dfrac{5}{4}

4)  \dfrac{9}{11} + \dfrac{3}{4}

  14)  \dfrac{5}{6} \, : \, \dfrac{25}{12}

5)  \dfrac{1}{7} \, - \, \dfrac{3}{5}

  15)  \dfrac{8}{9} : \dfrac{4}{27}

6)  11 - \dfrac{13}{3}

  16)  4\dfrac{2}{5} \, : \, 3\dfrac{1}{10}

7)  4\dfrac{2}{9} \, - \, 1\dfrac{1}{3}

  17)  4-\dfrac{2}{3} \, \cdot \, \dfrac{5}{8}

8) -\dfrac{20}{7}-\dfrac{7}{10}

  18)  \dfrac{1}{2} \, + \, 5 \, : \, \dfrac{10}{13} \, + \, 2\dfrac{1}{8}

9)  \dfrac{42}{5} \cdot \dfrac{10}{63}

  19)  \dfrac{13}{7} \, : \, \left( -\dfrac{26}{21} \right) \, \cdot \, \dfrac{8}{27}

10)  \dfrac{7}{9} \cdot 6

  20)  -\dfrac{3}{2} \, + \, \dfrac{15}{4} \, \cdot \, \left( -\dfrac{16}{5} \right) \, - \, \dfrac{9}{6} \, : \, \left(-3\right)

 

7. Aufgabe

1) \genfrac{}{}{1pt}{0}{\frac{3}{2}}{\frac{7}{3}}

  11) \genfrac{}{}{1pt}{0}{-\frac{21}{8}}{-\frac{12}{5}}

2) \genfrac{}{}{1pt}{0}{\frac{-5}{-3}}{-\frac{25}{4}}

  12) \genfrac{}{}{1pt}{0}{-34}{\frac{-43}{-12}}
3) \genfrac{}{}{1pt}{0}{-15}{\frac{11}{-30}}

  13) \genfrac{}{}{1pt}{0}{\frac{-5}{72}}{\frac{-40}{11}}
4) \genfrac{}{}{1pt}{0}{\frac{8}{12}}{5}

  14) \genfrac{}{}{1pt}{0}{\frac{5}{4}}{-\frac{21}{52}}
5) \genfrac{}{}{1pt}{0}{\frac{38}{-3}}{\frac{19}{4}}

  15) \genfrac{}{}{1pt}{0}{\frac{19}{15}}{\frac{-7}{102}}
6) \genfrac{}{}{1pt}{0}{\frac{27}{16}}{\frac{3}{11}}

  16) \genfrac{}{}{1pt}{0}{\frac{-33}{16}}{-\frac{99}{14}}
7) \genfrac{}{}{1pt}{0}{113}{\frac{17}{2}}   17) -\genfrac{}{}{1pt}{0}{-\frac{121}{17}}{\frac{77}{170}}
8) \genfrac{}{}{1pt}{0}{\frac{-42}{5}}{102}   18) \genfrac{}{}{1pt}{0}{\frac{142}{27}}{\frac{71}{72}}
9) \genfrac{}{}{1pt}{0}{-\frac{1}{9}}{\frac{1}{81}}    19) \genfrac{}{}{1pt}{0}{\frac{2}{15}}{\frac{-46}{-65}}
10) \genfrac{}{}{1pt}{0}{\frac{14}{23}}{-6}   20) \genfrac{}{}{1pt}{0}{-\frac{25}{256}}{-\frac{225}{32}}

 

8. Aufgabe

Berechnen Sie so schnell wie möglich und ohne Hilfsmittel!
Wie viel ist die Hälfte von zwei Drittel von drei Viertel von vier Fünftel von fünf Sechstel von sechs Siebtel von sieben Achtel von acht Neuntel von neun Zehntel von 10?

 

9. Aufgabe

Gesucht ist für jedes Sternchen eine Ziffer (also 0, 1, 2, ... 9) oder - falls sich keine Ziffer finden lässt - eine möglichst kleine natürliche Zahl, sodass die Rechnungen stimmen. Verschiedene Sternchen innerhalb einer Aufgabe können dabei durchaus verschiedene Ziffern / Zahlen bedeuten. Begründen Sie Ihre Ergebnisse!

Bemerkung: Diese Aufgaben sind ein bisschen tricky, führen aber gleichzeitig sehr schön in mathematische Denkweisen und Argumentationen ein - und benötigen dabei nicht mehr als die Grundrechenarten und Bruchrechnung.

1) \dfrac{*}{11}-\dfrac{28}{*} \, = \, 0

2) \dfrac{*}{3}\cdot \dfrac{*}{8} \, = \, \dfrac{35}{*}

3) \dfrac{5}{*}+\dfrac{*}{5} \, = \, 4\dfrac{9}{10}

4) \dfrac{1}{5}-\dfrac{*}{9} \, = \, -\dfrac{1}{*}

5) \dfrac{*}{4}+\dfrac{13}{*} \, = \, \dfrac{0}{*}

6) \dfrac{5}{*1} : \dfrac{2}{*} \, = \, \dfrac{5}{2*}

Dieses Kapitel enthält die folgenden Themen:

 

3.2 Bruchrechnung - Erklärungen

Bei den Zahlenbereichen hatten wir schon gesehen, dass es nicht nur ganze Zahlen gibt, sondern auch gebrochene. In diesem Kapitel schauen wir uns diese Zahlen und den Umgang mit ihnen etwas genauer an.

 

Was ist ein Bruch?

Bei einem Bruch \dfrac {p}{q} heißt p Zähler und q Nenner.
Der Bruchstrich steht dabei für eine Division. Auch wenn für p und q grundsätzlich beliebige reelle Zahlen eingesetzt werden dürfen, ist es üblich, in Brüchen ganze Zahlen zu verwenden, also p,q \in \mathbb{Z}. Immer gilt, dass der Nenner q \neq 0 sein muss, da durch 0 nicht geteilt werden darf!


Aufgrund der Rechenregeln für die Division gilt:

  • \dfrac{p}{p}=1 für alle Zahlen p \in \mathbb{R}\backslash_{ \{0\} }

  • \dfrac{-3}{4}=\dfrac{3}{-4}=-\dfrac{3}{4}

  • \dfrac{3}{4}=\dfrac{+3}{+4}=\dfrac{-3}{-4}=+\dfrac{3}{4}

Zur Schreibweise: Es ist egal, ob man 4 \cdot \dfrac{3}{10} oder \dfrac{3}{10} \cdot 4 oder \dfrac{4 \cdot 3}{10} schreibt. Allerdings ist bei allen drei Schreibweisen der "Malpunkt" zwingend erforderlich, da 4 \dfrac{3}{10} als 4 + \dfrac{3}{10} verstanden wird. Bei \dfrac{4 \cdot 3}{10} erklärt es sich eigentlich von selbst, warum der "Malpunkt" hier nicht einfach weggelassen werden darf ...

Bemerkung: Man kann Bruchstriche auch schräg schreiben. Das spart Platz und ist manchmal übersichtlicher.

 

Welche Arten von Brüchen gibt es?

Man unterscheidet folgende Arten von Brüchen:

Echte und unechte Brüche

Bei echten Brüchen ist der Betrag des Zählers kleiner als der Betrag des Nenners, d. h. der Betrag des gesamten Bruches ist kleiner als 1, z. B. \dfrac {3}{8}=0{,}375 oder -\dfrac {4}{5}=-0{,}8.

Bei unechten Brüchen ist der Betrag des Zählers größer als der Betrag des Nenners, d. h. der Betrag des gesamten Bruches ist größer als 1, z. B. \dfrac{11}{8}=1{,}375 oder -\dfrac{17}{5}=-3{,}4.

 

Gemischte Zahlen

Gemischte Zahlen bestehen aus einer ganzen Zahl und einem echten Bruch, z. B. \dfrac {13}{6}=\dfrac{12}{6}+\dfrac{1}{6}=2+ \dfrac{1}{6} = 2 \dfrac{1}{6} oder -\dfrac{13}{6}=-\left(2+\dfrac{1}{6}\right).

Ganz wichtig: Bitte beachten Sie, dass die ganze Zahl und der Bruch addiert werden, auch wenn das Pluszeichen weggelassen wird! Normalerweise werden in der Mathematik ausschließlich Malzeichen nicht geschrieben, wenn die Formel o. Ä. trotz des Weglassens eindeutig bleibt. Dies hier ist die große Ausnahme. Da das in vielen Fällen zu Verwirrung führt, sollte diese Schreibweise nur verwendet werden, wenn es dafür wichtige Gründe gibt! Und davon gibt es nicht sehr viele ...
Im Übrigen werden gemischte Zahlen eigentlich gar nicht benötigt. Echte und unechte Brüche reichen vollkommen aus, um alle Brüche abzubilden. Als Alternative gibt es auch noch die Dezimalzahlen. Statt einen unechten Bruch in eine gemischte Zahl umzuwandeln, ist es üblicherweise besser, ihn einfach so stehen zu lassen.

 

Gleichnamige und ungleichnamige Brüche

Brüche, die den gleichen Nenner haben, heißen gleichnamig. Der entsprechende Nenner heißt Hauptnenner der Brüche. Z. B. sind die Brüche \dfrac{1} {5} und \dfrac{4} {5} gleichnamig. Ihr Hauptnenner ist 5.

Brüche, die nicht den gleichen Nenner haben, heißen ungleichnamig, z. B. sind die Brüche \dfrac{2} {3} und \dfrac{2} {7} ungleichnamig.

 

Das "kleinste gemeinsame Vielfache" und der "größte gemeinsame Teiler"

Für das Erweitern und Kürzen, worum es ein Stück weiter unten gehen wird, sind die Konzepte vom kleinsten gemeinsamen Vielfachen (kgV) und vom größten gemeinsamen Teiler (ggT) zweier natürlicher Zahlen a und b (also a, b \in \mathbb {N}) nützlich:

Definition: Das kleinste gemeinsame Vielfache der Zahlen a und b ist die kleinste natürliche Zahl, die sowohl ein Vielfaches von a als auch ein Vielfaches von b ist, z. B. ist das kgV von 3 und 5 gleich 15.

Die Bestimmung des kgV hilft u. a., wenn zwei Brüche gleichnamig gemacht werden müssen, z. B. \dfrac{1}{6} + \dfrac{1}{8} = \dfrac{1 \cdot 4}{6 \cdot 4} + \dfrac{1 \cdot 3}{8 \cdot 3} = \dfrac{4}{24} + \dfrac{3}{24}. Natürlich wäre auch 48=6 \cdot 8 ein Hauptnenner von \dfrac {1}{6} und \dfrac {1}{8}. Allerdings wären dann die Zähler und Nenner jeweils doppelt so groß und üblicherweise rechnet es sich mit kleineren Zahlen leichter.

 

Definition: Der größte gemeinsame Teiler der Zahlen a und b ist die größte natürliche Zahl, durch die sich sowohl a als auch b ohne Rest teilen lässt, z. B. ist der ggT von 7 und 21 gleich 7.

Zahlen, deren ggT gleich 1 ist, heißen teilerfremd.

 

Rechenregeln für Brüche

Erweitern und Kürzen

Zwei Brüche werden erweitert, indem man Zähler und Nenner mit der gleichen Zahl multipliziert. Der Wert des Bruches ändert sich dabei nicht, z. B. \dfrac{3}{8}=\dfrac{3 \cdot 2}{8 \cdot 2}=\dfrac{6}{16}.
Veranschaulichung erweitern


Zwei Brüche werden gekürzt, indem man Zähler und Nenner durch die gleiche Zahl dividiert. Der Wert des Bruches ändert sich dabei nicht, z. B. \dfrac{4}{12}=\dfrac{4 : 4} {12 : 4}=\dfrac{1}{3}.
Wenn ein Bruch gekürzt werden soll, hilft die Bestimmung des ggT, z. B. ist 4 der ggT von 4 und 12.
Veranschaulichung kürzen


Es versteht sich (hoffentlich) von selbst, dass 0 keine geeignete Zahl zum Erweitern oder Kürzen ist, weil man ja nun mal durch 0 nicht teilen darf ...

Achtung: Aus Summen darf man nicht kürzen! Summen gehören ja schließlich zur Strichrechnung und das Kürzen zur Punktrechnung.

Noch ein paar Worte zum Kürzen aus Summen:
Auch wenn man es nicht auf den ersten Blick sieht, handelt es sich hierbei um eine Kombination von Addition/Subtraktion und Division, über die wir uns im Kapitel Rechengesetze schon Gedanken gemacht hatten. Wenn im Zähler oder Nenner eine Summe/Differenz steht, entsteht genau die Situation, in der wir auf die Rangfolge der Rechenoperationen achten müssen: Der Bruchstrich steht ja für eine Division und wirkt somit wie eine Klammer. Dazu kommt, dass auch das Kürzen eine Art von Dividieren ist. All das verträgt sich nicht mit der Strichrechnung ...
Käme man bei \frac{4+11}{4} auf die Idee, die 4 im Zähler mit der 4 im Nenner zu „kürzen“, erhielte man als Ergebnis 11.
Richtig ist aber \frac{4+11}{4} = \frac{15}{4} = 3{,}75

 

Addition und Subtraktion

Veranschaulichung addieren + subtrahieren

Zwei gleichnamige Brüche werden addiert, indem man die Zähler addiert und den Nenner beibehält, z. B. \dfrac{4}{10}+\dfrac{3}{10}=\dfrac{4+3}{10}=\dfrac{7}{10}.

Zwei ungleichnamige Brüche werden addiert, indem man sie gleichnamig macht (z. B. durch Erweitern) und dann addiert, z. B. \dfrac{1}{2}+\dfrac{1}{3}= \dfrac{1 \cdot 3}{2 \cdot 3}+\dfrac{1 \cdot 2}{3 \cdot 2}=\dfrac{3}{6}+\dfrac{2}{6}=\dfrac{3+2}{6}=\dfrac{5}{6}.


Zwei gleichnamige Brüche werden subtrahiert, indem man die Zähler subtrahiert und den Nenner beibehält, z. B. \dfrac{7}{10}-\dfrac{3}{10}=\dfrac{7-3}{10}=\dfrac{4}{10}.

Zwei ungleichnamige Brüche werden subtrahiert, indem man sie gleichnamig macht (z. B. durch Erweitern) und dann subtrahiert, z. B. \dfrac{4} {5}-\dfrac{1}{15}=\dfrac{4 \cdot 3}{5 \cdot 3}-\dfrac{1}{15}=\dfrac{12}{15}-\dfrac{1}{15}=\dfrac{12-1}{15}= \dfrac{11}{15}.

 

Ganz wichtig: Es gibt keine Rechenregel, die besagt, dass die Nenner irgendwie addiert bzw. subtrahiert werden müssen. Die Addition und Subtraktion von Brüchen funktioniert wirklich nur auf dem Weg, der hier vorgestellt wurde. Das gilt auch, wenn die Brüche Variablen enthalten, wie das in späteren Kapiteln der Fall sein wird. Dann mag es manchmal etwas umständlich sein, die Brüche gleichnamig zu machen - es muss aber sein!
Was passiert, wenn man die Brüche nicht gleichnamig macht, sehen sie an folgendem Vergleich:
Richtig ist: \frac{1}{2} +\frac{3}{4} = \frac{2}{4} + \frac{3}{4} = \frac{5}{4} > 1
Addiert man - fälschlicherweise - Zähler und Nenner von \frac{1}{2} und \frac{3}{4} separat, erhält man \frac{4}{6} < 1. Da kann also was nicht stimmen…

 

Multiplikation und Division

Zwei Brüche werden multipliziert, indem man Zähler mit Zähler und Nenner mit Nenner multipliziert, z. B. \dfrac{1}{4} \cdot \dfrac{3}{7}=\dfrac{1 \cdot 3}{4 \cdot 7}=\dfrac{3} {28}.

Zwei Brüche werden dividiert, indem man mit dem Kehrwert des zweiten Bruches multipliziert, z. B. \dfrac{1}{6} : \dfrac{2}{11}=\dfrac{1}{6} \cdot \dfrac{11}{2}=\dfrac{1 \cdot 11}{6 \cdot 2}=\dfrac{11}{12}.

Bemerkung zur Multiplikation und Division: Nützlich ist, immer vor dem Multiplizieren zu überprüfen, ob die Brüche gegeneinander gekürzt werden können, da dadurch die Zahlen kleiner werden. Je früher in einer Rechnung gekürzt wird, desto handlicher bleibt die Aufgabe.
Beispiel: Multipliziert man bei \dfrac{33}{14} \cdot \dfrac{280}{15} einfach die beiden Zähler und die beiden Nenner, erhält man \dfrac{9{.}240}{210}, wo nicht offensichtlich ist, durch welche Zahl gekürzt werden kann. Kürzt man vor dem Multiplizieren, sieht die Rechnung so aus: \dfrac{33}{14} \cdot \dfrac{280}{15} = \dfrac{11}{1} \cdot \dfrac{20}{5} = 11 \cdot 4 = 44
Vor dem Multiplizieren wurden hier die 33 (Zähler vom ersten Faktor) mit der 15 (Nenner vom zweiten Faktor) durch 3 sowie die 14 (Nenner vom ersten Faktor) mit der 280 (Zähler vom zweiten Faktor) durch 14 gekürzt.
Damit geht die gesamte Rechnung problemlos im Kopf ...


Bemerkung allgemein:
Ist das Ergebnis einer Bruchrechnungsaufgabe ein Bruch, sollte dieser so weit wie möglich gekürzt werden. Abhängig von der Aufgabenstellung (z. B. wenn die Größenordnung von Bedeutung ist) kann es sinnvoll sein, das Ergebnis als Dezimalzahl oder in Ausnahmefällen als gemischte Zahl darzustellen. In anderen Situationen, z. B. beim Multiplizieren oder beim Abschätzen von Wurzeln, eignen sich Brüche wesentlich besser.

 

Brüche und Klammern

Im vorherigen Kapitel hieß es schon, dass ein Bruchstrich wie eine Klammer wirkt. Eine Klammer muss immer dann gesetzt werden, wenn der Bruchstrich durch : ersetzt wird bzw. wenn mehrere Brüche auf einem Bruchstrich zusammengefasst werden! Hier nun die versprochenen Beispiele:

  • \dfrac{x+10}{3x-17}=(x+10) : (3x-17)
    Diese Schreibweise ist z. B. für die Polynomdivision wichtig.

  • \dfrac{5}{4} \cdot \dfrac{x+10}{3x-17} = \dfrac{5\cdot (x+10)}{4\cdot (3x-17)} = \dfrac{5x+50}{12x-68}
    Hier müssen im zweiten Bruch Klammern gesetzt werden, weil die Regel für die Multiplikation von Brüchen ja lautet "Zähler mal Zähler" und "Nenner mal Nenner". Ohne Klammern hätte man 5\cdot x+10 und 4\cdot 3x -17 , würde also nur einen Teil vom zweiten Zähler/Nenner mit dem Zähler/Nenner vom ersten Bruch multiplizieren.

  • \dfrac{5-8x}{7x+4} \cdot \dfrac{x+10}{3x-17} = \dfrac{(5-8x) \cdot (x+10)}{(7x+4) \cdot (3x-17)} = \dfrac{5x+50-8x\cdot x-80x}{21x\cdot x-119x+12x-68}

Hier ist es ebenso. Zusätzlich gelten natürlich auch bei Brüchen die "ganz normalen" Rechengesetze, wie Distributiv-, Kommutativ- und Assoziativgesetz. Anders gesagt: Ausmultiplizieren funktioniert im Zähler und Nenner genauso wie ohne Bruch drumherum ...


Zum Abschluss noch ein Beispiel, nur mit Zahlen, damit leichter zu sehen ist, warum das mit den Klammern auch wirklich wichtig ist:

Betrachten wir die Aufgabe \dfrac{2}{3}\cdot\dfrac{1+1}{4+5} = \dfrac{2\cdot (1+1)}{3\cdot (4+5)} = \dfrac{2\cdot 2}{3\cdot 9} = \dfrac{4}{27} = 0{,}\overline{148}   Das ist offensichtlich nicht das Gleiche!
Ohne Klammern stünde dort     \dfrac{2\cdot 1+1}{3\cdot 4+5} = \dfrac{2+1}{12+5} = \dfrac{3}{17} \approx 0{,}17647  

 

Dezimalzahlen

Auch bei den Dezimalzahlen, manchmal "Kommazahlen" genannt, unterscheidet man verschiedene "Sorten":

  • Endliche Dezimalzahlen: eine Dezimalzahl mit endlich vielen Nachkommastellen, z. B. 1{,}25 oder 10 {,}123456789987654321

  • Unendliche Dezimalzahlen: eine Dezimalzahl mit unendlich vielen Nachkommastellen
    Hier unterscheidet man weiter:
    • Periodische Dezimalzahlen: Bei periodischen Dezimalzahlen wiederholt sich innerhalb der unendlichen Folge von Nachkommastellen ein bestimmtes Muster immer wieder, z. B. 0{,}33333... oder 0{,}142857\,142857... oder 0{,} 756565656.... Dies macht man kenntlich, indem man nur das erste Auftreten dieser Periode notiert - mit einem horizontalen Strich darüber, z. B. 0{,}33333...=0{,}\overline{3} oder 0{,}142857142857...=0{,}\overline {142857} oder 0{,}756565656...=0{,}7\overline{56} . Dann benötigt man keine Pünktchen mehr dahinter. Sie sollen ja nur andeuten, dass die Zahl unendlich weitergeht und nicht nach den angegeben Nachkommastellen aufhört. Der (mathematische) Unterschied zwischen 0{,}33333 und 0{,}33333... ist zwar nicht groß, aber er ist da ...
    • Nicht periodische Dezimalzahlen: Nicht periodische Dezimalzahlen haben kein solches Muster in ihrer unendlichen Folge von Nachkommastellen, z. B. 0{,}1010010001... oder \sqrt{2}=1{,}414213562... oder \pi=3{,} 141592653... (die beiden letzten Beispiele werden Sie in den Kapiteln Potenzen, Wurzeln, Logarithmen bzw. Geometrie kennen lernen). Wichtig ist auch hier, dass die drei kleinen Pünktchen am Ende nicht vergessen werden. Benötigt man nur eine bestimmte Anzahl von Nachkommastellen (was ja meistens der Fall ist), muss entsprechend gerundet werden.



Umrechnung von Brüchen in Dezimalzahlen: Dezimalzahlen erhält man, indem man den Zähler eines Bruches durch seinen Nenner teilt. Hier ein paar Beispiele:

\dfrac{8}{10} = 8:10 = 0{,}8        
\dfrac{2}{7} = 2:7 = 0{,}285714\;285714\;285714\;285714... = 0{,}\overline{285714}   Da sich die Folge der Nachkommastellen unendlich wiederholt, erhalten wir eine periodische Dezimalzahl.
\dfrac{5}{4} = 5:4 = 1{,}25        

Wie Sie sehen, entstehen auf diese Art und Weise endliche oder periodische Dezimalzahlen, aber keine nicht periodischen. 


Umrechnung von Dezimalzahlen in Brüche: Natürlich kann man endliche oder periodische Dezimalzahlen auch in Brüche "rück-umformen" ... Bei endlichen Dezimalzahlen nimmt man dafür die Nachkommastelle(n) der Dezimalzahl als Zähler und ergänzt im Nenner eine 10, wenn der Zähler eine Stelle hat, eine 100, wenn der Zähler zwei Stellen hat, eine 1.000, wenn der Zähler drei Stellen hat, und so weiter ... Bei periodischen Dezimalzahlen funktioniert es genauso, nur dass im Nenner eine 9, eine 99, eine 999 und so weiter ... stehen muss. Anschließend kann man häufig noch kürzen.
Auch hier ein paar Beispiele:

0{,}4 = \dfrac{4}{10} = \dfrac{2\cdot 2}{2\cdot 5} = \dfrac{2}{5}    
0{,}\overline{567} = \dfrac{567}{999} = \dfrac{27\cdot 21}{27\cdot 37} = \dfrac{21}{37}   Bitte beachten Sie, dass 0{,}\overline{567} periodisch ist und deshalb nicht 1.000, sondern 999 im Nenner stehen muss.
5{,}81 = 5+\dfrac{81}{100} = \dfrac{500}{100}+\dfrac{81}{100} = \dfrac{581}{100}    

Übersicht:

 

3.3 Bruchrechnung - Lösungen

1. Aufgabe

1) \dfrac{4}{5}=\dfrac{4 \cdot 3}{5 \cdot 3}=\dfrac{12}{15}

  11) \dfrac{29}{30}=\dfrac{29 \cdot 4}{30 \cdot 4}=\dfrac{116}{120}

2) \dfrac{1}{10}=\dfrac{1 \cdot 12}{10 \cdot 12}=\dfrac{12}{120}

  12) \dfrac{654}{125}=\dfrac{654 \cdot 3}{125 \cdot 3}=\dfrac{1.962}{375}

3) \dfrac{7}{12}=\dfrac{7 \cdot 5}{12 \cdot 5}=\dfrac{35}{60}

  13) \dfrac{70}{93}=\dfrac{70 \cdot 80}{93 \cdot 80}=\dfrac{5.600}{7.440}

4) \dfrac{583}{15}=\dfrac{583 \cdot 10}{15 \cdot 10}=\dfrac{5.830}{150}

  14) \dfrac{12}{35}=\dfrac{12 \cdot 6}{35 \cdot 6}=\dfrac{72}{210}

5) \dfrac{2}{3}=\dfrac{2 \cdot 21}{3 \cdot 21}=\dfrac{42}{63}

  15) \dfrac{576}{688}=\dfrac{576 \cdot 1.000}{688 \cdot 1.000}=\dfrac{576.000}{688.000}

6) \dfrac{123}{456}=\dfrac{123 \cdot 100}{456 \cdot 100}=\dfrac{12.300}{45.600}

  16) \dfrac{334}{777}=\dfrac{334 \cdot 2}{777 \cdot 2}=\dfrac{668}{1.554}

7) \dfrac{5}{6}=\dfrac{5 \cdot 4}{6 \cdot 4}=\dfrac{20}{24}

  17) \dfrac{6}{7}=\dfrac{6\cdot 13}{7 \cdot 13}=\dfrac{78}{91}

8) \dfrac{3}{8}=\dfrac{3 \cdot 11}{8 \cdot 11}=\dfrac{33}{88}

  18) \dfrac{70}{3}=\dfrac{70\cdot 5}{3\cdot 5}=\dfrac{350}{15}

9) \dfrac{11}{7}=\dfrac{11 \cdot 9}{7 \cdot 9}=\dfrac{99}{63}

  19) \dfrac{5}{8}=\dfrac{5\cdot 9}{8\cdot 9}=\dfrac{45}{72}

10) \dfrac{211}{30}=\dfrac{211 \cdot 25}{30 \cdot 25}=\dfrac{5275}{750}

  20) \dfrac{130}{621}=\dfrac{130\cdot 20}{621\cdot 20}=\dfrac{2.600}{12.420}

 

2. Aufgabe

1) \dfrac{10}{13}=\dfrac{10 \cdot 3}{13 \cdot 3}=\dfrac{30}{39}    und   \dfrac{2}{3}=\dfrac{2 \cdot 13}{3 \cdot 13}= \dfrac{26}{39}


2) \dfrac{8}{9}=\dfrac{8 \cdot 5}{9 \cdot 5}=\dfrac{40}{45}   und   \dfrac{4}{15}= \dfrac{4 \cdot 3}{15 \cdot 3}=\dfrac{12}{45}


3) \dfrac{1}{8}=\dfrac{1 \cdot 3}{8 \cdot 3}=\dfrac{3} {24}   und   \dfrac{5}{6}=\dfrac{5 \cdot 4}{6 \cdot 4}=\dfrac{20}{24}


4) \dfrac{99}{12}=\dfrac{99 \cdot 3}{12 \cdot 3}=\dfrac{297}{36}   und   \dfrac{5}{18}=\dfrac{5 \cdot 2}{18 \cdot 2}=\dfrac{10}{36}


5) \dfrac{11}{20}=\dfrac{11 \cdot 19}{20 \cdot 19}=\dfrac{209}{380}   und   \dfrac{2}{19}=\dfrac{2 \cdot 20}{19 \cdot 20}=\dfrac{40}{380}


6) \dfrac{20}{7}=\dfrac{20 \cdot 50}{7 \cdot 50}=\dfrac{1000}{350}    und   \dfrac{7}{50}=\dfrac{7 \cdot 7}{50 \cdot 7}= \dfrac{49}{350}


7) \dfrac {17}{3}=\dfrac{17 \cdot 3}{3 \cdot 3}=\dfrac{51}{9}   und   \dfrac{10}{9}


8) \dfrac {3}{11}=\dfrac{3 \cdot 6}{11 \cdot 6}=\dfrac{18}{66}   und   \dfrac {1}{6}=\dfrac{1 \cdot 11}{6 \cdot 11}=\dfrac{11}{66}


9)  \dfrac {32}{27}=\dfrac{32 \cdot 2}{27 \cdot 2}=\dfrac{64}{54}   und   \dfrac {3}{2}=\dfrac{3 \cdot 27}{2 \cdot 27}=\dfrac{81}{54} 


10) \dfrac{3}{100}   und   \dfrac{7}{4}=\dfrac{7 \cdot 25}{4 \cdot 25}=\dfrac{175}{100}


11) \dfrac{1}{2}=\dfrac{1 \cdot 10}{2 \cdot 10}=\dfrac{10}{20}   und   \dfrac{1} {20} und \dfrac{3}{10}=\dfrac{3 \cdot 2}{10 \cdot 2}=\dfrac{6}{20}


12) \dfrac {1}{3}=\dfrac{1 \cdot 220}{3 \cdot 220}=\dfrac{220}{660}   und   \dfrac{7}{20}=\dfrac{7 \cdot 33}{20 \cdot 33}=\dfrac{231}{660} und \dfrac {2}{11}=\dfrac{2 \cdot 60}{11 \cdot 60}=\dfrac{120}{660}


13) \dfrac {7}{4}=\dfrac{7 \cdot 5}{4 \cdot 5}=\dfrac{35}{20}   und   \dfrac{1}{20} und \dfrac {132}{5}=\dfrac{132 \cdot 4}{5 \cdot 4}=\dfrac{528}{20}


14) \dfrac {3}{70}=\dfrac{3 \cdot 6}{70 \cdot 6}=\dfrac{18}{420}   und   \dfrac{201}{30}=\dfrac{201 \cdot 14}{30 \cdot 14}=\dfrac{2814}{420} und \dfrac {1}{14}=\dfrac{1 \cdot 30}{14 \cdot 30}=\dfrac{30}{420}


15) \dfrac {5}{21}=\dfrac{5 \cdot 4}{21 \cdot 4}=\dfrac{20}{84}   und   \dfrac {1}{12}=\dfrac{1 \cdot 7}{12 \cdot 7}=\dfrac{7}{84}   und   \dfrac {4}{3}=\dfrac{4 \cdot 28}{3 \cdot 28}=\dfrac{112}{84}   und   \dfrac {2}{21}=\dfrac{2 \cdot 4}{21 \cdot 4}=\dfrac{8}{84}


16) \dfrac {3}{4}=\dfrac{3 \cdot 12}{4 \cdot 12}=\dfrac{36}{48}   und   \dfrac {1}{8}=\dfrac{1 \cdot 6}{8 \cdot 6}=\dfrac{6}{48}   und   \dfrac {13}{16}=\dfrac{13 \cdot 3}{16 \cdot 3}=\dfrac{39}{48}   und   \dfrac {5}{24}=\dfrac{5 \cdot 2}{24 \cdot 2}=\dfrac{10}{48}


17) \dfrac {3}{5}=\dfrac{3 \cdot 14}{5 \cdot 14}=\dfrac{42}{70}   und   \dfrac {20}{7}=\dfrac{20 \cdot 10}{7 \cdot 10}=\dfrac{200}{70}   und   \dfrac {13}{2}=\dfrac{13 \cdot 35}{2 \cdot 35}=\dfrac{455}{70}   und   \dfrac {54}{70}


18) \dfrac {19}{2}=\dfrac{19 \cdot 180}{2 \cdot 180}=\dfrac{3420}{360}   und   \dfrac {13}{6}=\dfrac{13 \cdot 60}{6 \cdot 60}=\dfrac{780}{360}   und   \dfrac {37}{120}=\dfrac{37 \cdot 3}{120 \cdot 3}=\dfrac{111}{360}   und   \dfrac {91}{180}=\dfrac{91 \cdot 2}{180 \cdot 2}=\dfrac{182}{360}


19) \dfrac {1}{7}=\dfrac{1 \cdot 72}{7 \cdot 72}=\dfrac{72}{504}   und   \dfrac{8}{9}=\dfrac{8 \cdot 56}{9 \cdot 56}=\dfrac{448}{504}   und   \dfrac{33} {14}=\dfrac{33 \cdot 36}{14 \cdot 36}=\dfrac{1188}{504}   und   \dfrac{10}{63}=\dfrac{10 \cdot 8}{63 \cdot 8}=\dfrac{80}{504}   und   \dfrac {25}{24}=\dfrac{25 \cdot 21}{24 \cdot 21}=\dfrac{525}{504}


20) \dfrac {1}{2}=\dfrac{1 \cdot 90}{2 \cdot 90}=\dfrac{90}{180}   und   \dfrac{1} {3}=\dfrac{1 \cdot 60}{3 \cdot 60}=\dfrac{60}{180}   und   \dfrac{1} {4}=\dfrac{1 \cdot 45}{4 \cdot 45}=\dfrac{45}{180}   und   \dfrac{1} {5}=\dfrac{1 \cdot 36}{5 \cdot 36}=\dfrac{36}{180}   und   \dfrac {1} {6}=\dfrac{1 \cdot 30}{6 \cdot 30}=\dfrac{30}{180}   und   \dfrac{1} {15}=\dfrac{1 \cdot 12}{15 \cdot 12}=\dfrac{12} {180}   und   \dfrac{1} {36}=\dfrac{1 \cdot 5}{36 \cdot 5}=\dfrac{5}{180}

 

3. Aufgabe

1) \dfrac{10}{18}=\dfrac{10 : 2}{18 : 2}=\dfrac{5}{9}

  11) \dfrac{121}{22}=\dfrac{121 : 11}{22 : 11}=\dfrac{11}{2}

2) \dfrac{51}{17}=\dfrac{51 : 17}{17 : 17}=3

  12) \dfrac{91}{70}=\dfrac{91 : 7}{70 : 7}=\dfrac{13}{10}

3) \dfrac{38}{171}=\dfrac{38 : 19}{171 : 19}=\dfrac{2}{9}

  13) \dfrac{90}{810}=\dfrac{90 : 90}{810 : 90}=\dfrac{1}{9}

4) \dfrac{30}{205}=\dfrac{30 : 5}{205 : 5}=\dfrac{6}{41}

  14) \dfrac{96}{48}=\dfrac{96 : 12}{48 : 12}=\dfrac{8}{4} = 2

5) \dfrac{38}{4}= \dfrac{38 : 2}{4 : 2}=\dfrac{19}{2}

  15) \dfrac{131}{3}=\dfrac{131}{3}

6) \dfrac{124.000}{987.000}= \dfrac{124.000 : 1.000}{987.000 : 1.000}=\dfrac{124}{987}

  16) \dfrac{2.500}{5.000}=\dfrac{2.500 : 100}{5.000 : 100}=\dfrac{25 : 25}{50 : 25}=\dfrac{1}{2}

7) \dfrac{21}{49}= \dfrac{21 : 7}{49 : 7}=\dfrac{3}{7}

  17) \dfrac{110}{1.320}=\dfrac{110 : 110}{1.320 : 110}=\dfrac{1}{12}

8) \dfrac{39}{169}= \dfrac{39 : 13}{169 : 13}=\dfrac{3}{13}

  18) \dfrac{42}{7}=\dfrac{42 : 7}{7 : 7}=\dfrac{6}{1}=6

9) \dfrac{28}{42}= \dfrac{28 : 14}{42 : 14}=\dfrac{2}{3}

  19) \dfrac{69}{6}=\dfrac{69 : 3}{6 : 3}=\dfrac{23}{2}

10) \dfrac{23}{30}

Bemerkung: 23 und 30 enthalten keine gemeinsamen Teiler. Also kann man diesen Bruch nicht kürzen.
  20) \dfrac{120}{18}=\dfrac{120 : 6}{18 : 6}=\dfrac{20}{3}

 

4. Aufgabe

1) \dfrac{67}{6}=\dfrac{66}{6}+\dfrac{1}{6}=11\dfrac{1}{6}

  11) 2\dfrac{1}{9}=\dfrac{2\cdot 9}{9}+\dfrac{1}{9}=\dfrac{18}{9}+\dfrac{1}{9}=\dfrac{19}{9}

2) \dfrac{25}{23}=\dfrac{23}{23}+\dfrac{2} {23}=1\dfrac{2}{23}

  12) 1\dfrac{13}{14}=\dfrac{1\cdot 14} {14}+\dfrac{13}{14}=\dfrac{14} {14}+\dfrac{13}{14}=\dfrac{27}{14}

3) \dfrac{94}{24}=\dfrac{72}{24}+\dfrac{22}{24}=3\dfrac{22}{24}=3\dfrac{11}{12}

  13) 8\dfrac{4}{7}=\dfrac{8\cdot 7}{7}+\dfrac{4}{7}=\dfrac{56}{7}+\dfrac{4}{7}=\dfrac{60}{7}

4) \dfrac{119}{17}=7

  14) 10\dfrac{14}{15}=\dfrac{10\cdot 15}{15}+\dfrac{14}{15}=\dfrac{150}{15}+\dfrac{14}{15}=\dfrac{164}{15}

5) \dfrac{235}{50}=\dfrac{200}{50}+\dfrac{35}{50}=4\dfrac{35}{50}=4\dfrac{7}{10}

  15) 5\dfrac{20}{33}=\dfrac {5\cdot 33}{33}+\dfrac{20}{33}=\dfrac {165}{33}+\dfrac{20}{33}=\dfrac{185}{33}

6) \dfrac{95}{3}=\dfrac{93}{3}+\dfrac{2}{3}=31\dfrac{2}{3}

  16) 12\dfrac{3}{4}=\dfrac{12 \cdot 4}{4}+\dfrac{3}{4}=\dfrac{48}{4}+\dfrac{3}{4}=\dfrac{51}{4}

7) \dfrac{123}{11}=\dfrac{121}{11}+\dfrac{2}{11}=11\dfrac{2}{11}

  17) 3\dfrac{14}{27}=\dfrac{3 \cdot 27}{27}+\dfrac{14}{27}=\dfrac{81}{27}+\dfrac{14}{27}=\dfrac{95}{27}

8) \dfrac{155}{12}=\dfrac{144}{12}+\dfrac{11}{12}=12\dfrac{11}{12}

  18) 19\dfrac{1}{8}=\dfrac{19 \cdot 8}{8}+\dfrac{1}{8}=\dfrac{152}{8}+\dfrac{1}{8}=\dfrac{153}{8}

9) \dfrac{41}{7}=\dfrac{35}{7}+\dfrac{6}{7}=5\dfrac{6}{7}

  19) 63\dfrac{7}{9}=\dfrac{63 \cdot 9}{9}+\dfrac{7}{9}=\dfrac{567}{9}+\dfrac{7}{9}=\dfrac{574}{9}

10) \dfrac{80}{9}=\dfrac{72}{9}+\dfrac{8}{9}=8\dfrac{8}{9}

  20) 32\dfrac{11}{13}=\dfrac{32 \cdot 13}{13}+\dfrac{11}{13}=\dfrac{416}{13}+\dfrac{11}{13}=\dfrac{427}{13}

 

5. Aufgabe

1) \dfrac{7}{12}=7:12\approx0{,}5833   11) 0{,}24=\dfrac{24}{100}=\dfrac{6}{25}

2) \dfrac{18}{5}=18:5=3{,}6

  12) 2{,}\overline{6}=2+\dfrac{6}{9}=\dfrac{2\cdot9}{9}+\dfrac{6}{9}=\dfrac{24}{9}=\dfrac{8}{3}

3) \dfrac{61}{650}=61:650\approx0{,}0938

  13) 0{,}67=\dfrac{67}{100}

4) \dfrac{18}{18}=18:18=1

  14) 6{,}25=\dfrac{25}{4}

5) \dfrac{23}{16}=23:16=1{,}4375

  15) 0{,}\overline{83}=\dfrac{83}{99}

6) \dfrac{2}{99}=2:99\approx0{,}0202

  16) 3{,}008=\dfrac{376}{125}

7) \dfrac{120}{10}=120:10=12

  17) 0{,}5058=\dfrac{2529}{5000}

8) \dfrac{63}{85}=63:85\approx0{,}7412

  18) 5=\dfrac{5}{1}

9) \dfrac{16}{25}=16:25=0{,}64

  19) 6{,}625=\dfrac{53}{8}

10) \dfrac{1}{130}=1:130\approx0{,}0077

  20) 0{,}025=\dfrac{1}{40}

 

6. Aufgabe

Wichtig: Bei allen Multiplikations- und Divisionsaufgaben, in denen gemischte Zahlen enthalten sind, muss die gemischte Zahl vor dem Multiplizieren bzw. Dividieren in einen unechten Bruch umgewandelt werden, sonst kommen anschließend Punkt- und Strichrechnung durcheinander ...
Zur Wiederholung: Dies ist die einzige Stelle in der Mathematik, bei der nicht ein Malzeichen, sondern ein Pluszeichen weggelassen wird. Steht in einer Aufgabe z. B. die gemischte Zahl 1\dfrac{1}{2}, ist damit die Summe 1+\dfrac{1} {2} gemeint. Diese Schreibweise verkompliziert Rechnungen also eher bzw. macht sie fehleranfälliger. Sie wird in diesem Lernmodul auch nur eingeführt, weil z. B. einige Taschenrechner sie verwenden und es deswegen nötig ist zu verstehen, wie sie gelesen werden muss.


1)  \dfrac{3}{4} \, + \, \dfrac{3}{2} \, = \, \dfrac{3}{4} \, + \, \dfrac{3 \cdot 2}{2 \cdot 2} \, = \, \dfrac{3}{4} \, + \, \dfrac{6}{4} \, = \, \dfrac{9}{4} \, = \, 2\dfrac{1}{4}  


2) { 1\dfrac{5}{6}+2\dfrac{7}{8}=\dfrac{6}{6}+\dfrac{5}{6}+\dfrac{16}{8}+\dfrac{7}{8}=\dfrac{11}{6}+\dfrac{23}{8}=\dfrac{11 \cdot 4}{6 \cdot 4}+\dfrac{23 \cdot 3}{8 \cdot 3}=\dfrac{44}{24}+\dfrac{69}{24}=\dfrac{113}{24}=4\dfrac{17}{24} } 


3)  \dfrac{3}{2} \, + \, 12 \, = \, \dfrac{3}{2} \, + \, \dfrac{24}{2} \, = \, \dfrac{27}{2}\, = \, 13 \dfrac{1} {2}


4) \dfrac{9}{11} \, + \, \dfrac{3}{4} \, = \, \dfrac{9\cdot 4}{11\cdot 4} \, + \, \dfrac{3\cdot 11}{4\cdot 11} \, = \, \dfrac{36}{44} \, + \, \dfrac{33}{44} \, = \, \dfrac{69}{44} \, = \, 1\dfrac{25}{44}


5)  \dfrac{1}{7} \, - \, \dfrac{3}{5} \, = \, \dfrac{1 \cdot 5}{7 \cdot 5} \, - \, \dfrac{3 \cdot 7}{5 \cdot 7} \, = \, \dfrac{5}{35} \, - \, \dfrac{21}{35} \, = \, - \dfrac{16}{35}


6) 11 \, - \, \dfrac{13}{3} \, = \, \dfrac{33}{3} \, - \, \dfrac{13}{3} \, = \, \dfrac{20}{3} \, = \, 6\dfrac{2}{3}


7) {4 \dfrac{2}{9} \, - \, 1 \dfrac{1}{3} \, = \, \dfrac{36}{9} \, + \, \dfrac{2}{9} \, - \, \left( \dfrac{3}{3} \, + \, \dfrac{1}{3} \right) \, = \, \dfrac{38}{9} \, - \, \dfrac{4}{3} \, = \, \dfrac{38}{9} \, - \, \dfrac{4 \cdot 3}{3 \cdot 3} \, = \, \dfrac{38}{9} \, - \, \dfrac{12}{9} \, = \, \dfrac{26}{9} \, = \, 2 \dfrac{8}{9}}


8)  -\dfrac{20}{7} \, - \, \dfrac{7}{10} \, = \, -\dfrac{20\cdot 10}{7\cdot 10} \, -\, \dfrac{7\cdot 7}{10\cdot 7} \, =\, -\dfrac{200}{70} \, - \, \dfrac{49}{70} \, = \, -\dfrac{249}{70} \, = \, -3\dfrac{39}{70}


9) \dfrac{42}{5} \, \cdot \, \dfrac{10}{63} \, = \, \dfrac{2}{1} \, \cdot \, \dfrac{2}{3} \, = \, \dfrac{4}{3} \, = \, 1\dfrac{1}{3}

Bemerkung: Vor dem Multiplizieren wurden hier die 42 (Zähler vom ersten Faktor) mit der 63 (Nenner vom zweiten Faktor) durch 21 sowie die 5 (Nenner vom ersten Faktor) mit der 10 (Zähler vom zweiten Faktor) durch 5 gekürzt. Das Kürzen kann natürlich auch schrittweise (z. B. erst durch 7 und dann durch 3 - kleines Einmaleins!) erfolgen, wenn man nicht sofort sieht, dass die 21 sowohl in der 42 als auch in der 63 enthalten ist.


10)  \dfrac{7}{9} \, \cdot \, 6 \, = \, \dfrac{7}{9} \, \cdot \, \dfrac{6}{1} \, = \, \dfrac{7}{3} \, \cdot \, \dfrac{2}{1} \, = \, \dfrac{14}{3} \, = \, 4 \dfrac{2}{3}

Bemerkung: Vor dem Multiplizieren wurden hier die 9 (Nenner vom ersten Faktor) mit der 6 (Zähler vom zweiten Faktor) durch 3 gekürzt.


11) 34 \, \cdot \, \dfrac{1}{2} \, = \, \dfrac{34}{2} \, = \, 17


12) 2\dfrac{1}{4} \, \cdot \, \dfrac{2}{7} \, = \, \dfrac{9}{4} \, \cdot \, \dfrac{2}{7} \, = \, \dfrac{9}{2} \, \cdot \, \dfrac{1}{7} \, = \, \dfrac{9}{14}

Bemerkung: Vor dem Multiplizieren wurden hier die 4 (Nenner vom ersten Faktor) mit der 2 (Zähler vom zweiten Faktor) durch 2 gekürzt.


13)  \dfrac{3}{8} \, : \, \dfrac{5}{4} \, = \, \dfrac{3}{8} \, \cdot \, \dfrac{4}{5} \, = \, \dfrac{3}{2} \, \cdot \, \dfrac{1}{5} \, = \, \dfrac{3}{10}

Bemerkung: Vor dem Multiplizieren wurden hier die 8 (Nenner vom ersten Faktor) mit der 4 (Zähler vom zweiten Faktor) durch 4 gekürzt.


14)  \dfrac{5}{6} \, : \, \dfrac{25}{12} \, = \, \dfrac{5}{6} \, \cdot \, \dfrac{12}{25} \, = \, \dfrac{1}{1} \, \cdot \dfrac{2}{5} \, = \, \dfrac {2}{5}

Bemerkung: Vor dem Multiplizieren wurden hier die 5 (Zähler vom ersten Faktor) mit der 25 (Nenner vom zweiten Faktor) durch 5 sowie die 6 (Nenner vom ersten Faktor) mit der 12 (Zähler vom zweiten Faktor) durch 6 gekürzt.


15) \dfrac{8}{9} : \dfrac{4}{27} = \dfrac{8}{9} \cdot \dfrac{27}{4} = 6

Bemerkung: Vor dem Multiplizieren wurden hier die 8 (Zähler vom ersten Faktor) mit der 4 (Nenner vom zweiten Faktor) durch 4 sowie die 9 (Nenner vom ersten Faktor) mit der 27 (Zähler vom zweiten Faktor) durch 9 gekürzt.


16) {4\dfrac{2}{5} \, : \, 3\dfrac{1}{10} \, = \, \left(\dfrac{20}{5}\, +\, \dfrac{2}{5}\right) \, : \, \left(\dfrac{30}{10}\, +\, \dfrac{1}{10}\right) \, =\, \dfrac{22}{5} \, : \, \dfrac{31}{10} \, = \, \dfrac{22}{5} \, \cdot \, \dfrac{10}{31} \, = \, \dfrac{44}{31} = 1\dfrac {13}{31}}

Bemerkung: Vor dem Multiplizieren wurden hier die 5 (Nenner vom ersten Faktor) mit der 10 (Zähler vom zweiten Faktor) durch 5 gekürzt.


17) 4 \, - \, \dfrac{2}{3} \, \cdot \, \dfrac{5}{8} \, = \, 4 \, - \, \dfrac{1}{3} \, \cdot \, \dfrac{5}{4} \, = \,\dfrac{16}{4} \, - \, \dfrac{5}{12} \, = \,\dfrac{16}{4} \, - \, \dfrac{5}{12}\, = \, \dfrac{48}{12} \, - \, \dfrac{5}{12} \, = \, \dfrac{43}{12} \, = \, 3\dfrac{7}{12}

Bemerkung: Vor dem Multiplizieren wurden hier die 2 (Zähler vom ersten Faktor) mit der 8 (Nenner vom zweiten Faktor) durch 2 gekürzt.


18) { \dfrac{1}{2} \, + \, 5 \, : \, \dfrac{10}{13} \, + \, 2\dfrac{1}{8} \, = \, \dfrac{1}{2} \, + \, 5 \, \cdot \, \dfrac{13}{10} \, + \, \dfrac{16}{8} \, + \, \dfrac{1}{8} \, = \, \dfrac{1}{2} \, + \, \dfrac{13}{2} \, + \, \dfrac{17}{8} \, = \, \dfrac{4}{8} \, + \, \dfrac{52}{8} \, + \, \dfrac{17}{8} \, = \, \dfrac{73}{8} \, = \, 9\dfrac{1}{8} }

Bemerkung:
Vor dem Multiplizieren wurden hier die 5 (erster Faktor) mit der 10 (Nenner vom zweiten Faktor) durch 5 gekürzt.


19) {\dfrac{13}{7} \, : \, \left(-\dfrac{26}{21}\right) \, \cdot \, \dfrac{8}{27} \, = \, \dfrac{13}{7} \, \cdot \, \left(-\dfrac{21}{26}\right) \, \cdot \, \dfrac{8}{27} \, = \, \dfrac{1}{1} \, \cdot \, \left(-\dfrac{3}{2}\right) \, \cdot \, \dfrac{8}{27} \, = \, 1 \, \cdot \, \left(-\dfrac{1}{1}\right) \, \cdot \, \dfrac{4}{9} \, = \, -\dfrac{4}{9} }

Bemerkung:
Vor dem Multiplizieren wurden hier der Übersichtlichkeit wegen in zwei Schritten gekürzt:
1. Es wurden die 13 (Zähler vom ersten Faktor) mit der 26 (Nenner vom zweiten Faktor) durch 13 sowie die 7 (Nenner vom ersten Faktor) mit der 21 (Zähler vom zweiten Faktor) durch 7 gekürzt.
2. Es wurden die 3 (Zähler vom gekürzten zweiten Faktor) mit der 27 (Nenner vom dritten Faktor) durch 3 sowie die 2 (Nenner vom gekürzten zweiten Faktor) mit der 8 (Zähler vom dritten Faktor) durch 2 gekürzt.


20) {-\dfrac{3}{2} \, + \, \dfrac{15}{4} \, \cdot \, \left(-\dfrac{16}{5}\right) \, - \, \dfrac{9}{6} \, : \, \left(-3\right) \, = \, -\dfrac{3}{2} \, + \, \dfrac{15}{4} \, \cdot \, \left(-\dfrac{16}{5}\right) \, - \, \dfrac{9}{6} \, \cdot \, \left(-\dfrac{1}{3}\right) \, = \, -\dfrac{3}{2} \, + \, \dfrac{3}{1} \, \cdot \, \left(-\dfrac{4}{1}\right) -\dfrac{3}{6}\, \cdot \, \left(-\dfrac{1}{1}\right) \, = \, -\dfrac{3}{2} \,- \, 12 \, + \, \dfrac{1}{2} \, = \, -\dfrac{26}{2} \, = \, -13 }

Bemerkung:
Vor dem Multiplizieren wurden hier in beiden Produkten gekürzt:
1. Produkt: Es wurden die 15 (Zähler vom ersten Faktor) mit der 5 (Nenner vom zweiten Faktor) durch 5 sowie die 4 (Nenner vom ersten Faktor) mit der 16 (Zähler vom zweiten Faktor) durch 4 gekürzt.
2. Produkt: Es wurden die 9 (Zähler vom ersten Faktor) mit der 3 (Nenner vom zweiten Faktor) durch 3 gekürzt.

 

7. Aufgabe

1) \genfrac{}{}{1pt}{0}{\frac{3}{2}}{\frac{7}{3}} = \dfrac{3}{2} : \dfrac{7}{3} = \dfrac{3}{2} \cdot \dfrac{3}{7} = \dfrac{9}{14} 


2) \genfrac{}{}{1pt}{0} {\frac{-5}{-3}}{-\frac{25}{4}} = \dfrac{-5}{-3} : \left( -\dfrac{25}{4} \right) = \dfrac{5}{3} \cdot \left( -\dfrac{4}{25} \right) = \dfrac{1}{3} \cdot \left( -\dfrac{4}{5} \right) = - \dfrac {4}{15}

Bemerkung: Vor dem Multiplizieren wurden hier die 5 (Zähler vom ersten Faktor) mit der 25 (Nenner vom zweiten Faktor) durch 5 gekürzt.


3) \genfrac{}{}{1pt}{0}{-15}{\frac{11}{-30}} = -15 : \left(\dfrac{11}{-30} \right) = -15 \cdot \left(-\dfrac{30}{11} \right) = \dfrac{450}{11}


4) \genfrac{}{}{1pt}{0}{\frac{8}{12}}{5} = \dfrac{8}{12} : \dfrac{5}{1} = \dfrac{8}{12} \cdot \dfrac{1}{5} = \dfrac{2}{15}


5) \genfrac{}{}{1pt}{0}{\frac{38}{-3}}{\frac{19}{4}} = \dfrac{38}{-3} : \dfrac{19}{4} = -\dfrac{38}{3} \cdot \dfrac{4}{19} = -\dfrac{2}{3} \cdot \dfrac{4}{1} = -\dfrac{8}{3}

Bemerkung: Vor dem Multiplizieren wurden hier die 38 (Zähler vom ersten Faktor) mit der 19 (Nenner vom zweiten Faktor) durch 19 gekürzt.


6) \genfrac{}{}{1pt}{0}{\frac{27}{16}}{\frac{3}{11}} = \dfrac{27}{16} : \dfrac{3}{11} = \dfrac{27}{16} \cdot \dfrac{11}{3} = \dfrac{9}{16} \cdot \dfrac{11}{1} = \dfrac{99}{16}

Bemerkung: Vor dem Multiplizieren wurden hier die 27 (Zähler vom ersten Faktor) mit der 3 (Nenner vom zweiten Faktor) durch 3 gekürzt.


7) \genfrac{}{}{1pt}{0}{113}{\frac{17}{2}} = 113 : \dfrac{17}{2} = 113 \cdot \dfrac{2}{17} = \dfrac{226}{17}


8) \genfrac{}{}{1pt}{0}{\frac{-42}{5}}{102} = \dfrac{-42}{5} : \dfrac{102}{1} = -\dfrac{42}{5} \cdot \dfrac{1}{102} = -\dfrac{7}{5} \cdot \dfrac{1}{17} = -\dfrac{7}{85}

Bemerkung: Vor dem Multiplizieren wurden hier die 42 (Zähler vom ersten Faktor) mit der 102 (Nenner vom zweiten Faktor) durch 6 gekürzt.


9) \genfrac{}{}{1pt}{0}{-\frac{1}{9}}{\frac{1}{81}} = -\dfrac{1}{9} : \dfrac{1}{81} = -\dfrac{1}{9} \cdot \dfrac{81}{1} = -\dfrac{1}{1} \cdot \dfrac{9}{1} = -9

Bemerkung: Vor dem Multiplizieren wurden hier die 9 (Nenner vom ersten Faktor) mit der 81 (Zähler vom zweiten Faktor) durch 9 gekürzt.


10) \genfrac{}{}{1pt}{0}{\frac{14}{23}}{-6} = \dfrac{14}{23} : \left(-\dfrac{6}{1} \right) = \dfrac{14}{23} \cdot \left(-\dfrac{1}{6} \right) = \dfrac{7}{23} \cdot \left( -\dfrac{1}{3} \right) = -\dfrac{7}{69}

Bemerkung: Vor dem Multiplizieren wurden hier die 14 (Zähler vom ersten Faktor) mit der 6 (Nenner vom zweiten Faktor) durch 2 gekürzt.


11) \genfrac{}{}{1pt}{0}{-\frac{21}{8}}{-\frac{12}{5}} = -\dfrac{21}{8} : \left(-\dfrac{12}{5} \right) = -\dfrac{21}{8} \cdot \left(-\dfrac{5}{12} \right) = -\dfrac{7}{8} \cdot \left( -\dfrac{5}{4} \right) = \dfrac{35}{32}

Bemerkung: Vor dem Multiplizieren wurden hier die 21 (Zähler vom ersten Faktor) mit der 12 (Nenner vom zweiten Faktor) durch 3 gekürzt.


12) \genfrac{}{}{1pt}{0}{-34}{\frac{-43}{-12}} = -34 : \left(\dfrac{-43}{-12}\right) = -34 \cdot \dfrac{12}{43} = -\dfrac{408}{43}

13) \genfrac{}{}{1pt}{0}{\frac{-5}{72}}{\frac{-40}{11}} = \dfrac{-5}{72} : \dfrac{-40}{11} = -\dfrac{5}{72} \cdot \left(-\dfrac{11}{40}\right) = -\dfrac{1}{72} \cdot \left(-\dfrac{11}{8}\right) = \dfrac{11}{576}

Bemerkung: Vor dem Multiplizieren wurden hier die 5 (Zähler vom ersten Faktor) mit der 40 (Nenner vom zweiten Faktor) durch 5 gekürzt.


14) \genfrac{}{}{1pt}{0}{\frac{5}{4}}{-\frac{21}{52}} = \dfrac{5}{4} : \left(-\dfrac{21}{52}\right) = \dfrac{5}{4} \cdot \left(-\dfrac{52}{21}\right) = \dfrac{5}{1} \cdot \left( -\dfrac{13}{21} \right) = -\dfrac{65}{21}

Bemerkung: Vor dem Multiplizieren wurden hier die 4 (Nenner vom ersten Faktor) mit der 52 (Zähler vom zweiten Faktor) durch 4 gekürzt.


15) \genfrac{}{}{1pt}{0}{\frac{19}{15}}{\frac{-7}{102}} = \dfrac{19}{15} : \dfrac{-7}{102} = \dfrac{19}{15} \cdot \left(-\dfrac{102}{7}\right) = \dfrac{19}{5} \cdot \left(-\dfrac{34}{7}\right) = -\dfrac{646}{35}

Bemerkung: Vor dem Multiplizieren wurden hier die 15 (Nenner vom ersten Faktor) mit der 102 (Zähler vom zweiten Faktor) durch 3 gekürzt.


16) \genfrac{}{}{1pt}{0}{\frac{-33}{16}}{-\frac{99}{14}} = -\dfrac{33}{16} : \left(-\dfrac{99}{14}\right) = -\dfrac{33}{16} \cdot \left(-\dfrac{14}{99}\right) = -\dfrac{1}{8} \cdot \left(-\dfrac{7}{3}\right) = \dfrac{7}{24}

Bemerkung: Vor dem Multiplizieren wurden hier die 33 (Zähler vom ersten Faktor) mit der 99 (Nenner vom zweiten Faktor) durch 33 sowie die 16 (Nenner vom ersten Faktor) mit der 14 (Zähler vom zweiten Faktor) durch 2 gekürzt.


17) -\genfrac{}{}{1pt}{0}{-\frac{121}{17}}{\frac{77}{170}} = -\left(-\dfrac{121}{17} : \dfrac{77}{170}\right) = -\left(-\dfrac{121}{17} \cdot \dfrac{170}{77}\right) = -\left(-\dfrac{11}{1} \cdot \dfrac{10}{7}\right) = \dfrac{110}{7}

Bemerkung: Vor dem Multiplizieren wurden hier die 121 (Zähler vom ersten Faktor) mit der 77 (Nenner vom zweiten Faktor) durch 11 sowie die 17 (Nenner vom ersten Faktor) mit der 170 (Zähler vom zweiten Faktor) durch 17 gekürzt.


18) \genfrac{}{}{1pt}{0}{\frac{142}{27}}{\frac{71}{72}} = \dfrac{142}{27} : \dfrac{71}{72} = \dfrac{142}{27} \cdot \dfrac{72}{71} = \dfrac{2}{3} \cdot \dfrac{8}{1} = \dfrac{16}{3}

Bemerkung: Vor dem Multiplizieren wurden hier die 142 (Zähler vom ersten Faktor) mit der 71 (Nenner vom zweiten Faktor) durch 71 sowie die 27 (Nenner vom ersten Faktor) mit der 72 (Zähler vom zweiten Faktor) durch 9 gekürzt.


19) \genfrac{}{}{1pt}{0}{\frac{2}{15}}{\frac{-46}{-65}} = \dfrac{2}{15} : \dfrac{-46}{-65} = \dfrac{2}{15} \cdot \dfrac{65}{46} = \dfrac{1}{3} \cdot \dfrac{13}{23} = \dfrac{13}{69}

Bemerkung: Vor dem Multiplizieren wurden hier die 2 (Zähler vom ersten Faktor) mit der 46 (Nenner vom zweiten Faktor) durch 2 sowie die 15 (Nenner vom ersten Faktor) mit der 65 (Zähler vom zweiten Faktor) durch 5 gekürzt.


20) \genfrac{}{}{1pt}{0}{-\frac{25}{256}}{-\frac{225}{32}} = -\dfrac{25}{256} : \left(-\dfrac{225}{32}\right) = -\dfrac{25}{256} \cdot \left(-\dfrac{32}{225}\right) = -\dfrac{1}{8} \cdot \left(-\dfrac{1}{9}\right) = \dfrac{1}{72}

Bemerkung: Vor dem Multiplizieren wurden hier die 25 (Zähler vom ersten Faktor) mit der 225 (Nenner vom zweiten Faktor) durch 25 sowie die 256 (Nenner vom ersten Faktor) mit der 32 (Zähler vom zweiten Faktor) durch 32 gekürzt.

 

8. Aufgabe

Schreibt man die Rechnung mit Brüchen auf, kommt man mit viel Kürzen, dafür wenig Rechnen, zum Ergebnis: 
\dfrac{1}{2}\cdot\dfrac{2}{3}\cdot\dfrac{3}{4}\cdot\dfrac{4}{5}\cdot\dfrac{5}{6}\cdot\dfrac{6}{7}\cdot\dfrac{7}{8}\cdot\dfrac{8}{9}\cdot\dfrac{9}{10}\cdot 10 = 1
An dieser Aufgabe wird auch offensichtlich, warum Brüche häufig einen Vorteil gegenüber Dezimalzahlen haben. Bei 
0{,}5\cdot 0{,}666\ldots\cdot 0{,}75\cdot 0{,}8\cdot 0{,}833\ldots\cdot 0{,}857\ldots\cdot 0{,}875\cdot 0{,}888\ldots\cdot 0{,}9\cdot 10 = 1
wäre die Logik der Rechnung nämlich nicht so deutlich wie oben. Mal abgesehen davon, dass Rundungsfehler meist dazu führen werden, dass die Lösung nicht exakt 1 ist.

 

9. Aufgabe

Bemerkung 1: Grundsätzlich sind diese Aufgaben nicht eindeutig zu lösen. Neben der Anforderung, dass die Rechnungen stimmen sollen, ist daher die Vorgabe, dass möglichst kleine Ziffern gefunden werden sollen, entscheidend. Ein bisschen "Rumprobieren" wird aber auch so nötig sein.

Bemerkung 2: Auch wenn z. B. im Zähler "nur" ein Sternchen steht, muss dieses beim Erweitern mit der entsprechenden Zahl multipliziert werden, weil erweitern nun mal heißt "Zähler und Nenner mit der gleichen Zahl multiplizieren". Das gilt auch für Sternchen!


1)
Überlegung: Damit eine Differenz 0 ergibt, müssen Minuend und Subtrahend gleich sein. Die naheliegendste Lösung ist also \dfrac{28}{11}-\dfrac{28}{11} \, = \, 0
 
Bemerkung: Dies ist aber nicht die einzige Lösung, z. B. stimmt auch \dfrac{14}{11}-\dfrac{28}{22} \, = \, 0. Und es kann noch nicht mal entschieden werden, welche Zahlenkombination die kleinere ist ...


2)
1. Überlegung: Da beim Multiplizieren von Brüchen jeweils die Zähler und die Nenner multipliziert werden, können sie separat betrachtet werden.
 
2. Überlegung: Im Nenner muss einfach nur multipliziert werden: 3 \cdot 8=24, also ist \dfrac{*}{3}\cdot \dfrac{*}{8} \, = \, \dfrac{35}{24}
 
3. Überlegung: Die 35 im Zähler des Bruches auf der rechten Seite kann im Bereich der natürlichen Zahlen nur auf genau eine Weise in Faktoren zerlegt werden, nämlich 35=5 \cdot 7. Es kann allerdings nicht entschieden werden, welcher Faktor an erster und welcher Faktor an zweiter Stelle in der ursprünglichen Rechnung steht. Es gibt daher zwei Lösungen.
 
Lösung: \dfrac{5}{3}\cdot \dfrac{7}{8} \, = \, \dfrac{35}{24} oder \dfrac{7}{3}\cdot \dfrac{5}{8} \, = \, \dfrac{35}{24}


3)
1. Überlegung: Einfacher wird die Rechnung, wenn die gemischte Zahl auf der rechten Seite in einen unechten Bruch umgewandelt wird. Es ergibt sich: \dfrac{5}{*}+\dfrac{*}{5} \, = \, \dfrac{49}{10}
 
2. Überlegung: Die kleinste Ziffer, die für den Nenner des ersten Bruches infrage kommt, ist 1. 0 scheidet ja als Kandidat für einen Nenner aus. Er ergibt sich: \dfrac{5}{1}+\dfrac{*}{5} \, = \, \dfrac {49}{10}
 
3. Überlegung: Das lässt sich umformen zu \dfrac{*}{5} \, = \, \dfrac {49}{10}-5 oder \dfrac{*}{5} \, = \, -\dfrac{1}{10}. Dies ist (mathematisch gesprochen) die Frage, welche Zahl ergibt -\dfrac{1}{10}, wenn man sie durch 5 teilt. Eigentlich muss man an dieser Stelle gar nicht weiterrechnen, weil nur eine negative Zahl hier für ein richtiges Ergebnis sorgen kann: Eine positive Zahl geteilt durch eine andere positive Zahl (und die 5 ist ja hier als positive Zahl schon vorgegeben) wäre ja wieder positiv und nicht -\dfrac{1}{10}. Damit kann man die Vorgabe, dass für die Sternchen Ziffern eingesetzt werden sollen, nie erfüllen. Für diejenigen, die das genaue Ergebnis in diesem Fall wissen möchten: Für das Sternchen müsste -\dfrac{1}{2} eingesetzt werden, damit die Rechnung stimmt, was auch deswegen nicht funktioniert, weil es nicht ganzzahlig ist.

4. Überlegung: Probieren wir es also mit der nächstgrößeren Ziffer 2. Wir bekommen: \dfrac{5}{2}+\dfrac{*}{5} \, = \, \dfrac{49}{10} oder (wenn wir alles auf den Hauptnenner bringen) \dfrac{25}{10}+\dfrac{2\cdot *}{10} \, = \, \dfrac{49}{10}

5. Überlegung: Da die Nenner nun gleich sind, können wir sie für den folgenden Schritt ignorieren. Es ergibt sich: 25+2\cdot *=49. Daraus folgt: 2 \cdot *=24 und damit *=12

Lösung: \dfrac{5}{2}+\dfrac{12}{5} \, = \, 4\dfrac{9}{10}


4)
1. Überlegung: Das Sternchen auf der rechten Seite muss für 45 stehen, da dies der Hauptnenner von 5 und 9 ist. Es ergibt sich: \dfrac{1}{5}-\dfrac{*}{9} \, = \, -\dfrac{1}{45} oder, wenn man gleich alles auf den Hauptnenner bringt, \dfrac{9}{45}-\dfrac{5\cdot *}{45} \, = \, -\dfrac{1}{45}
 
2. Überlegung: Da die Nenner nun gleich sind, können wir sie wieder ignorieren. Bleibt die Frage: 9-5\cdot *=-1, die uns zu der Erkenntnis führt, dass 5\cdot * offensichtlich gleich 10 sein muss. Wenn 5 \cdot *=10 ist, ist *=2
 
Lösung: \dfrac{9}{45}-\dfrac{10}{45} \, = \, -\dfrac{1}{45} oder \dfrac{1}{5}-\dfrac{2}{9} \, = \, -\dfrac{1}{45}


5)
1. Überlegung: Auf der rechten Seite hat das Sternchen im Nenner keine Auswirkungen auf die Rechnung, egal durch welche Zahl die 0 im Zähler geteilt wird, das Ergebnis ist immer 0; mit Ausnahme der 0 selbst, denn \frac{0}{0} ist nicht definiert. Da eine möglichst kleine Ziffer gefordert war, können wir die 1 für dieses Sternchen nehmen - oder es einfach weglassen.
 
2. Überlegung: Damit das Ergebnis insgesamt 0 ist, müsste auf der linken Seite eine der Zahlen negativ sein, denn die Summe von zwei positiven Zahlen ist wieder positiv. Negative Zahlen haben wir aber wegen der Einschränkung, dass Ziffern gesucht sind, nicht zur Verfügung. Insofern lassen sich hier keine passenden Zahlen finden.


6)
1. Überlegung: Man teilt durch einen Bruch, indem man mit dem Kehrwert multipliziert. Das bedeutet hier: \dfrac{5}{*1} \cdot \dfrac{*}{2} \, = \, \dfrac{5}{2*}
 
2. Überlegung: Schauen wir uns die Rechnung im Zähler an: 5\cdot *=5. Also muss dieses Sternchen gleich 1 sein. Es ergibt sich: \dfrac{5}{*1} \cdot \dfrac{1}{2} \, = \, \dfrac{5}{2*} bzw. in der Originalrechnung: \dfrac{5}{*1} : \dfrac{2}{1} \, = \, \dfrac{5}{2*}
 
3. Überlegung: Für den Nenner folgt daraus: *1\cdot 2=2*. Für das rechte Sternchen berechnet man 1 \cdot 2 = 2; für das linke umgekehrt. Das Ergebnis ist also 11\cdot 2=22
 
Lösung: \dfrac{5}{11} : \dfrac{2}{1} \, = \, \dfrac{5}{22} bzw. \dfrac{5}{11} : 2 \, = \, \dfrac{5}{22}, was man natürlich auch dafür schreiben kann

4. Prozentrechnung - Lernziele und typische Fehler

Nach Durcharbeiten dieses Kapitels sollten Sie folgende Lernziele erreicht haben:

  • Sie erinnern sich an die Begriffe der Prozentrechnung.
  • Sie können die Formel für die Prozentrechnung nennen und damit Prozentsatz, Prozentwert und Grundwert berechnen.
  • Sie können entscheiden, ob in einer Aufgabe Prozentsatz, Prozentwert oder Grundwert gesucht ist.
  • Sie können entsprechende Textaufgaben lösen und die Lösungswege mathematisch vernünftig aufschreiben.


Typischer Fehler
in diesem Kapitel ist:

  • Erhöhte oder verminderte Grundwerte werden nicht beachtet. Erklärung


Für Online-Selbsttests zu diesem Thema und weitere Informationen zur Mathematikunterstützung an der TH Wildau nutzen Sie bitte den Moodle-Kursraum "SOS Mathematik - Brückenkurs".

Übersicht:

 

4.1 Prozentrechnung - Aufgaben

1. Aufgabe

Wie viel Prozent sind ...

1) 3 von 24?
 
2) 23 von 46?
 
3) 96{,}32 von 112?
 
4) 12 von 50?
 
5) 45 von 120?
 
6) In einem Affengehege leben 12 Männchen und 15 Weibchen. Wie groß ist der prozentuale Anteil der Weibchen?
 
7) Eine Sportmannschaft besteht aus 21 Spielerinnen, davon sind 5 Spielerinnen vor der Saison neu hinzugekommen. Gesucht ist der prozentuale Anteil der Neuzugänge.
 
8) Ein Autohersteller hat es geschafft, den Benzinverbrauch von durchschnittlich 5{,}6\,\text{l} pro 100\,\text{km} auf 4{,}9\,\text{l} pro 100\,\text{km} zu reduzieren. Wie viel Kraftstoff wird durchschnittlich eingespart?

9) Der Milchpreis ist von 1{,}20 \; \text{EUR} auf 1{,}30 \;\text{EUR} pro Liter gestiegen. Wie hoch ist der Preisanstieg in Prozent?

10) In einer Box befinden sich 50 Kugeln, von denen 15 schwarz und der Rest blau sind. Sie ziehen nun 4 blaue Kugeln aus der Box. Wie hoch ist der Anteil an schwarzen Kugeln in der Box?

 

2. Aufgabe

Diese Prozente sollte man im Kopf wissen.

1) \dfrac{3}{4}   7) \dfrac {1}{8}

2) \dfrac{2}{3}   8) \dfrac {1}{10}

3) \dfrac{1}{2}   9) \dfrac {1}{20}

4) \dfrac {1}{3}

  10) \dfrac {1}{25}
5) \dfrac {1}{4}   11) \dfrac {1}{100}

6) \dfrac {1}{5}   12) \dfrac {1}{1.000}

 

3. Aufgabe

Wie viel sind ...

1) 12 \,\% von 384?
 
2) 67{,}1 \, \% von 1.250?
 
3) 45 \, \% von 692?
 
4) 73 \, \% von 14.600?
 
5) 52{,}6 \, \% von 725?
 
6) In einem Lager sind 72{,}3\,\% von 1.200\,\text{l} Saft ausgelaufen. Wie viel Saft ist verloren gegangen?
 
7) Es sind 95\,\% von 980 Zügen pünktlich. Wie viele sind das?
 
8) Im Laufe der letzten Jahre ist eine Waschmaschine um 12\,\% teurer geworden. Nun kostet sie 357\,\text{EUR}. Wie hoch ist der Preisanstieg bezogen auf den aktuellen Preis?

9) Ein Geschäft besitzt 500\;\text{kg} Reis. Davon verkauft es an einem Morgen 10 \,\% und nachmittags nochmal 15\, \% der restlichen Menge. Wie viele Kilogramm Reis liegen anschließend noch im Lager?

10) Ein rechteckiges Stück Land ist 15\;\text{m} lang und 12\;\text{m} breit. Der Grundstücksbesitzer möchte diese Fläche als Kleingarten bewirtschaften und plant 20 \,\% des Grundstücks für den Bau einer Gartenhütte ein. Wie groß kann die Grundfläche der Hütte werden?

 

4. Aufgabe

Wie groß ist der Ausgangswert?

1) 81 \, \% entsprechen 162
 
2) 99 \, \% entsprechen 12{,}672
 
3) 50{,}5 \, \% entsprechen 30{,}3
 
4) 22{,}2 \, \% entsprechen 19{,}425
 
5) 36{,}7 \, \% entsprechen 183{,}5
 
6) Nach einer Grillparty sind noch 2 Kisten Getränke übrig. Das sind 40\,\% der ursprünglich gekauften. Wie viele Kisten wurden eingekauft?
 
7) Der Preis eines Sessels wurde um 32\,\% reduziert, sodass er jetzt noch 150\,\text{EUR} kostet. Wie teuer war er ursprünglich?
 
8) Eine Wasserpumpe kostet 223\,\text{EUR}, nachdem ihr Preis auf 75\,\% des Originalpreises herabgesetzt wurde. Wie hoch war der Originalpreis?

9) Ein Lebensmittelgeschäft erzielte nach dem Ausverkauf aller vorrätigen Waren Einnahmen in Höhe von 16.800\;\text{EUR}. Diese Einnahmen betragen nach internen Kalkulationen 20 \,\% mehr, als für diese Waren ausgegeben wurde. Wie teuer waren die Waren beim Ankauf?

10) Eine Farm hat nach Geldengpässen 330 Hühner verkauft. Die restlichen Hühner machen nur noch 40 \,\% der ursprünglichen Hühnerpopulation aus. Wie viele Hühner lebten einst auf der Farm?

 

5. Aufgabe

Ein paar gemischte Aufgaben ...

1) Jemand hat 65\,\% seiner Schulden in Höhe von insgesamt 450\,\text{EUR} bereits zurückgezahlt. Wie viel ist noch offen?
 
2) Bei einer Radtour sind ein paar Freunde bereits 60\,\text{km} geradelt. 20\,\% ihrer Strecke haben sie noch vor sich. Wie lang ist die Strecke insgesamt?
 
3) Der Preis einer Hose wurde im Schlussverkauf auf 85 \, \% des Originalpreises reduziert, sodass die Hose jetzt noch 59{,}50\,\text{EUR} kostet. Wie viel kostete die Hose ursprünglich?
 
4) Der Preis eines Pullovers, der vor dem Schlussverkauf 65{,}99\,\text{EUR} kostete, soll um 25 \, \% gesenkt werden. Wie teuer ist der Pullover jetzt?
 
5) Ein T-Shirt, das vormals 12\,\text{EUR} kostete, kostet nun noch 9\,\text{EUR}. Um wie viel Prozent ist der Preis reduziert worden?
 
6) Eine Familie macht eine Wanderung von 14.600\,\text{m} Länge, von der sie 56\,\% bereits geschafft haben. Wie viele \text{km} müssen sie noch laufen?
 
7) In den neuen Bundesländern leben etwa 12{,}5\,\text{Mio.} Menschen. Wie viele Personen leben dort prozentual gesehen - bezogen auf Gesamtdeutschland (81{,}2\,\text{Mio.} Einwohner)?
 
8) Wenn in einer Schulklasse 3 von 28 Kindern mit der linken Hand schreiben, wie groß ist dann der prozentuale Anteil der Linkshänder/-innen in dieser Klasse?
 
9) Für ein Auto muss 1\,\% des Kaufpreises, also 190\,\text{EUR} angezahlt werden. Wie teuer ist es?

10) Meerwasser enthält ca. 4 \,\% Salz. Wie viele Gramm Süßwasser müssen zu 400\;\text{g} Meerwasser gegeben werden, um einen Salzanteil von nur noch 2{,}5 \,\% zu erhalten?

 

6. Aufgabe

Die Marketing-Abteilung der Hochschule hat für eine Veranstaltung 400 kleine Schokotäfelchen bereitgestellt. 99\,\% davon enthalten Nüsse.

Wie viele Tafeln müssen gegessen werden, damit der Anteil von Schokolade mit Nüssen auf 98\,\% sinkt?

Dieses Kapitel enthält die folgenden Themen:

 

4.2 Prozentrechnung - Erklärungen

Die Prozentrechnung ist nützlich, wenn Größenverhältnisse verglichen werden sollen. Sie baut auf der Bruchrechnung auf.

 

Formel

Es gilt: p =\dfrac{W}{G} \cdot 100\,\% mit

  • p: Prozentsatz in \%
  • W: Prozentwert
  • G: Grundwert

Diese Formel kann natürlich umgestellt werden. Man erhält dann G= \dfrac{W}{p\,\%} \cdot 100 bzw. W = \dfrac{p\,\% \cdot G}{100}

Bemerkung: Achten Sie immer genau darauf, was in der Aufgabe gegeben ist! Besonders Prozent- und Grundwert werden häufig verwechselt.

 

Berechnung

Eigentlich ist nicht viel zu tun ... Man setzt jeweils die gegebenen Werte in die passende Formel ein und rechnet aus.
Hier ein paar Beispiele:

1) Gegeben sind W=74 und G=512
Gesucht ist p in \%

p=\dfrac{74}{512}\cdot 100\,\% \approx 14{,}45\,\%


2) Gegeben sind p=83\,\% und W=125
Gesucht ist G

G=\dfrac{125}{83\,\%}\cdot 100\,\% \approx 150{,}60


3) Gegeben sind p=16\,\% und G=49
Gesucht ist W

W=\dfrac{16\,\%\cdot 49}{100\,\%} = 7{,}84


Alternative: Alle Prozentaufgaben können auch mit dem Dreisatz berechnet werden. Das macht weder vom Aufwand noch vom Ergebnis her einen Unterschied. Sie können daher einfach ausprobieren, welche Variante Sie bevorzugen.
Wir betrachten dafür noch einmal die Beispiele von oben:

1) Gegeben sind W=74 und G=512
Gesucht ist p in \%

\begin{array}{rclcr}512 &\widehat{=}& 100\,\% \cr \cr 1 &\widehat{=}& \dfrac{100\,\%}{512} \cr \cr 74 &\widehat{=}& \dfrac{74 \cdot 100\,\%}{512} &\approx& 14{,}45\,\% \end{array}


2) Gegeben sind p=83\,\% und W=125
Gesucht ist G

\begin{array}{rclcr}83\,\% &\widehat{=}& 125 \cr \cr 1\,\% &\widehat{=}& \dfrac{125}{83} &\approx& 1{,}51 \cr \cr 100\,\% &\widehat{=}& \dfrac{100\cdot 125}{83} &\approx& 150{,}60 \end{array}


3) Gegeben sind p=16\,\% und G=49
Gesucht ist W

\begin{array}{rclcr}100\,\% &\widehat{=}& 49 \cr \cr 1\,\% &\widehat{=}& \dfrac{49}{100} &=& 0{,}49 \cr \cr 16\,\% &\widehat{=}& \dfrac{16\cdot 49}{100} &=& 7{,}84 \end{array}

 

Naja, ein bisschen mehr ist natürlich doch zu tun … Die bislang besprochenen Aufgaben in diesem Kapitel sind nicht besonders realistisch. Sowohl im Studium als auch in der Arbeitswelt wird Ihnen niemand sagen "Berechnen Sie hier doch mal den Prozentsatz!", sondern es wird sich eine Fragestellung ergeben, bei der Sie selbst auf die Idee kommen müssen, dass der Prozentsatz gesucht ist. Man muss sich also bei jeder Aufgabe genau klarmachen, was gegeben und was gesucht ist. Das ist besonders tricky bei Aufgaben, in denen der Grundwert gesucht ist.
Ein Beispiel:
Der Bambusbecher für Coffee to go kostet in der Mensa 6\;\text{EUR}. Wie viel kostet er ohne Mehrwertsteuer (19\,\%)?
Klar ist, dass p=19\,\%, da p immer der Wert mit dem \% ist. Aber wie ist das mit den 6\;\text{EUR}? In diesem Wert sind Grundwert und Mehrwertsteuer enthalten, die 6\;\text{EUR} entsprechen also 100\,\% + 19\,\% = 119\,\%. Wir möchten den Grundwert ohne die Mehrwertsteuer berechnen. Damit haben wir den Ansatz:
\begin{array}{rclcr}119\,\% &\widehat{=}& 6\;\text{EUR} \cr \cr 1\,\% &\widehat{=}& \dfrac{6\;\text{EUR}}{119} \cr \cr 100\,\% &\widehat{=}& \dfrac{100\cdot 6\;\text{EUR}}{119} &\approx& 5{,}04\;\text{EUR} \end{array}

Übersicht:

 

4.3 Prozentrechnung - Lösungen

Eine Bemerkung vorab: Der Abwechslung halber und um Ihnen auch Beispiele für diesen Rechenweg zu zeigen, wurden alle Textaufgaben mit dem Dreisatz gelöst. Mit der Prozentformel geht es natürlich auch. Umgekehrt können natürlich auch die anderen Aufgaben mithilfe der Dreisatzrechnung gelöst werden.

Und noch eine Bemerkung: Bei Textaufgaben ist es üblich, einen Antwortsatz zu schreiben.

 

1. Aufgabe

Gegeben sind hier jeweils W und G.
Gesucht ist p in \%.

Die Prozentformel wird also in der Form p \, \% = \dfrac{W}{G} \cdot 100 \, \% angewendet.

1) 3 \text{ von } 24 = \dfrac{3}{24} \cdot 100 \, \% = 12{,}5 \, \%


2) 23 \text{ von }46 = \dfrac{23}{46} \cdot 100 \, \% = 50 \, \%


3) 96{,}32 \text{ von } 112 = \dfrac{96{,}32}{112} \cdot 100 \, \% = 86 \, \%


4) 12 \text{ von } 50 = \dfrac{12}{50} \cdot 100 \, \% = 24 \, \%


5) 45 \text{ von } 120 = \dfrac{45}{120} \cdot 100 \, \% = 37{,}5 \, \%


6)
\begin{array}{rclcr}27\;\text{Affen}&\widehat{=}& 100\,\% \cr \cr 1\;\text{Affe}&\widehat{=}& \dfrac{100\,\%}{27} \cr \cr 15\;\text{Affen}&\widehat{=}& \dfrac{15\cdot 100\,\%}{27} &\approx& 55{,}56\,\% \end{array}

Etwa 55{,}56\, \% der Affen in diesem Gehege sind weiblich.


7)
\begin{array}{rclcr}21\;\text{Spielerinnen}&\widehat{=}& 100\,\% \cr \cr 1\;\text{Spieler}&\widehat{=}& \dfrac{100\,\%}{21} \cr \cr 5\;\text{Spielerinnen}&\widehat{=}& \dfrac{5\cdot 100\,\%}{21} &\approx& 23{,}81\,\% \end{array}

Etwa 23{,}81\, \% der Spielerinnen sind neu im Team.


8)
Zunächst müssen wir die Ersparnis berechnen: 5{,}6\,\text{l}-4{,}9\,\text{l}=0{,}7\,\text{l}

\begin{array}{rclcr}5{,}6\,\text{l}&\widehat{=}& 100\,\% \cr \cr 1\,\text{l}&\widehat{=}& \dfrac{100\,\%}{5{,}6} \cr \cr 0{,}7\,\text{l}&\widehat{=}& \dfrac{0{,}7\cdot 100\,\%}{5{,}6} &=& 12{,}5\,\% \end{array}

Es werden 12{,}5\,\% Benzin eingespart.


9)
Zunächst muss die absolute Preisänderung berechnet werden: 1{,}30\;\text{EUR} - 1{,}20\;\text{EUR} = 0{,}10\;\text{EUR}

\begin{array}{rclcl} 1{,}20\;\text{EUR} &\widehat{=}& 100\,\% \cr\cr 1 \;\text{EUR} &\widehat{=}& \dfrac{100\,\%}{1{,}2} \cr\cr 0{,}1 \;\text{EUR} &\widehat{=}& \dfrac{0{,}1 \cdot 100\, \%}{1{,}2} &\approx& 8{,}33\% \end{array}

Der Milchpreis ist um rund 8{,}33 \,\% gestiegen.


10)
Nachdem 4 Kugeln gezogen wurden, befinden sich noch 46 Kugeln in der Box. Davon sind immer noch 15 schwarz und der Rest blau.

\begin{array}{rclcl} 45 \;\text{Kugeln} &\widehat{=}& 100\,\% \cr\cr 1 \;\text{Kugel} &\widehat{=}& \dfrac{100\,\%}{46} \cr\cr 15 \; \text{Kugeln} &\widehat{=}& \dfrac{15 \cdot 100\,\%}{46} &\approx& 32{,}61\, \% \end{array}

Rund 32{,}61 \% der in der Box verbliebenen Kugeln sind schwarz.

 

2. Aufgabe

1) \dfrac {3}{4} = 0{,}75 = 75 \, \%   7) \dfrac {1}{8} = 0{,}125 = 12{,}5 \, \%

2) \dfrac {2}{3} \approx 0{,}66667 \approx 66{,}67 \, \%   8) \dfrac{1}{10} = 0{,}1 = 10 \, \%

3) \dfrac {1}{2} = 0{,}5 =50 \, \%   9) \dfrac{1}{20} = 0{,}05 = 5 \, \%

4) \dfrac {1}{3} \approx 0{,}33333 \approx 33{,}33 \, \%

  10) \dfrac{1}{25} = 0{,}04 = 4 \, \%
5) \dfrac {1}{4} = 0{,}25 = 25 \, \%   11) \dfrac{1}{100} = 0{,}01 = 1 \, \%

6) \dfrac {1}{5} = 0{,}2 = 20 \, \%   12) \dfrac{1}{1.000} = 0{,}001 = 1 \, \permil


Bemerkung: Bitte achten Sie darauf, dass bei den Aufgaben 7) und 9) gerundet wurde und deshalb dort das Ungefährzeichen \approx stehen muss.

  

3. Aufgabe

Gegeben sind hier jeweils p in \% und G.
Gesucht ist W.

Die Prozentformel wird also in der Form W = \dfrac{p \cdot G}{100} angewendet.

1) 12 \, \% \text{ von } 384 = \dfrac{12 \cdot 384}{100} = 46{,}08


2) 67{,}1 \, \% \text{ von } 1.250 = \dfrac{67{,}1 \cdot 1.250}{100} = 838{,}75


3) 45 \, \% \text{ von } 692 = \dfrac{45 \cdot 692}{100} = 311{,}4


4) 73 \, \% \text{ von } 14.600 = \dfrac{73 \cdot 14.600}{100} = 10.658


5) 52{,}6 \, \% \text{ von } 725 = \dfrac{52{,}6 \cdot 725}{100} = 381{,}35


6)
\begin{array}{rclcr}100\,\% &\widehat{=}& 1.200\,\text{l} \cr \cr 1\,\% &\widehat{=}& \dfrac{1.200\,\text{l}}{100} \cr \cr 72{,}3\,\% &\widehat{=}& \dfrac{72{,}3\cdot 1.200\,\text{l}}{100} &=& 867{,}6\,\text{l} \end{array}

Es sind 867{,}6\,\text{l} Saft verloren gegangen.

 
7)
\begin{array}{rclcr}100\,\% &\widehat{=}& 980\;\text{Züge} \cr \cr 1\,\% &\widehat{=}& \dfrac{980\;\text{Züge}}{100} \cr \cr 95\,\% &\widehat{=}& \dfrac{95\cdot 980\;\text{Züge}}{100} &=& 931\;\text{Züge} \end{array}

Es waren 931 Züge pünktlich.

 
8)
\begin{array}{rclcr}100\,\% &\widehat{=}& 357\;\text{EUR} \cr \cr 1\,\% &\widehat{=}& \dfrac{357\;\text{EUR}}{100} \cr \cr 12\,\% &\widehat{=}& \dfrac{12\cdot 357\;\text{EUR}}{100} &=& 42{,}84\;\text{EUR} \end{array}

Der Preis ist um 42{,}84\;\text{EUR} angestiegen.


9)
1. Schritt:
Reismenge nach dem ersten Verkauf: 100\, \% - 10 \,\% = 90 \,\%

\begin{array}{rclcl} 100\,\% &\widehat{=}& 500 \,\text{kg} \cr\cr 1 \,\% &\widehat{=}& \dfrac{500}{100} \;\text{kg} \cr\cr 90 \,\% &\widehat{=}& \dfrac{90 \cdot 500 \;\text{kg}}{100} &=& 450 \;\text{kg} \end{array}

2. Schritt:
Die nach dem Morgen verbliebenen 450\;\text{kg} stellen nun 100 \,\% der Restmenge dar.
Reismenge nach dem zweiten Verkauf: 100 \,\% - 15 \,\% = 85 \,\%

\begin{array}{rclcl} 100\;\% &\widehat{=}& 450 \;\text{kg} \cr\cr 1 \,\% &\widehat{=}& \dfrac{450}{100} \;\text{kg} \cr\cr 85 \;\% &\widehat{=}& \dfrac{85 \cdot 450 \;\text{kg}}{100} &=& 382{,}5 \;\text{kg} \end{array}

Nach beiden Verkäufen befinden sich noch 382{,}5\;\text{kg} Reis im Lager.


10)
Zunächst muss die Grundstücksfläche insgesamt berechnet werden: 15 \;\text{m} \cdot 12 \;\text{m} = 180 \;\text{m}^2

Nun kann der Anteil der Fläche für die Hütte berechnet werden:
\begin{array}{rclcl} 100\,\% &\widehat{=}& 180 \;\text{m}^2 \cr\cr 20 \,\% &\widehat{=}& \dfrac{180\;\text{m}^2}{5} &=& 36 \;\text{m}^2 \end{array}

Die Hütte kann eine Grundfläche von 36\;\text{m}^2 haben.

 

4. Aufgabe

Gegeben sind hier jeweils p in \% und W.
Gesucht ist G.

Die Prozentformel wird also in der Form G = \dfrac{W}{p} \cdot 100 angewendet.
 
1) 81 \, \% \text{ entsprechen } 162 \Rightarrow G = \dfrac{162}{81} \cdot 100 = 200


2) 99 \, \% \text{ entsprechen } 12{,}672 \Rightarrow G = \dfrac{12{,}672}{99} \cdot 100 = 12{,}8


3) 50{,}5 \, \% \text{ entsprechen } 30{,}3 \Rightarrow G = \dfrac{30{,}3}{50{,}5} \cdot 100 = 60


4) 22{,}2 \, \% \text{ entsprechen } 19{,}425 \Rightarrow G = \dfrac{19{,}425}{22{,}2} \cdot 100 = 87{,}5


5) 36{,}7 \, \% \text{ entsprechen } 183{,}5 \Rightarrow G = \dfrac{183{,}5}{36{,}7} \cdot 100 = 500


6)
\begin{array}{rclcr}40\,\% &\widehat{=}& 2\;\text{Kisten} \cr \cr 1\,\% &\widehat{=}& \dfrac{2\;\text{Kisten}}{40} \cr \cr 100\,\% &\widehat{=}& \dfrac{100\cdot 2\;\text{Kisten}}{40} &=& 5\;\text{Kisten} \end{array}

Es wurden 5 Kisten eingekauft.


7)
Hier sind der noch zu zahlende Preis und der Prozentsatz der Reduzierung gegeben. Zunächst muss daher ausgerechnet werden, wieviel Prozent dem noch zu zahlenden Preis entsprechen: 100\,\%-32\,\%=68\,\%

\begin{array}{rclcr}68\,\% &\widehat{=}& 150\;\text{EUR} \cr \cr 1\,\% &\widehat{=}& \dfrac{150\;\text{EUR}}{68} \cr \cr 100\,\% &\widehat{=}& \dfrac{100\cdot 150\;\text{EUR}}{68} &\approx& 220{,}59\;\text{EUR} \end{array}

Der Sessel kostete ursprünglich 220{,}59\,\text{EUR}.

 
8)
\begin{array}{rclcr}75\,\% &\widehat{=}& 223\;\text{EUR} \cr \cr 1\,\% &\widehat{=}& \dfrac{223\;\text{EUR}}{75} \cr \cr 100\,\% &\widehat{=}& \dfrac{100\cdot 223\;\text{EUR}}{75} &\approx& 297{,}33\;\text{EUR} \end{array}

Die Pumpe hat im Original 297{,}33\;\text{EUR} gekostet.


9)
\begin{array}{rclcl} 120\,\% &\widehat{=}& 16.800 \;\text{EUR} \cr\cr 1 \,\% &\widehat{=}& \dfrac{16.800}{120} \;\text{EUR} \cr\cr 100 \,\% &\widehat{=}& \dfrac{100 \cdot 16.800\;\text{EUR}}{120} &=& 14.000 \;\text{EUR} \end{array}

Das Geschäft gab 14.000\;\text{EUR} für den Kauf der Waren aus.


10) 
Zunächst muss der Anteil der verkauften Hühner berechnet werden: 100\,\% - 40\, \% = 60\, \%

\begin{array}{rclcl} 60 \,\% &\widehat{=}& 330 \;\text{Hühner} \cr\cr 1 \;\% &\widehat{=}& \dfrac{330}{60} \;\text{Hühner} \cr\cr 100 \,\% &\widehat{=}& \dfrac{100 \cdot 330\;\text{Hühner}}{60} &=& 550 \;\text{Hühner} \end{array}

Die Farm besaß vor dem Verkauf 550 Hühner.

 

5. Aufgabe

Eine Bemerkung vorab: Machen Sie sich zu Beginn jeder Aufgabe genau klar, was gegeben und was gesucht ist - das ist ja hier das Entscheidende!

Noch eine Bemerkung: Hier wurden manche Aufgaben mit dem Dreisatz und manche mit der Prozentformel berechnet. Es geht natürlich jede Variante bei jeder Aufgabe.


1)
Gegeben sind p in \% und G.
Gesucht ist W.
Gefragt ist, wieviel Geld noch zurückgezahlt werden muss. Die 65\,\% beziehen sich allerdings auf den Betrag, der bereits zurückgezahlt wurde. Daher muss dies vorher noch umgerechnet werden: 100\,\%-65\,\%=35\,\%

\begin{array}{rclcr}100\,\% &\widehat{=}& 450\,\text{EUR}\cr \cr 1\,\% &\widehat{=}& \dfrac{450\,\text{EUR}}{100} \cr \cr 35\,\% &\widehat{=}& \dfrac{35\cdot 450\,\text{EUR}}{100} &=& 157{,}50\,\text{EUR} \end{array}

Es müssen noch 157{,}50\,\text{EUR} zurückgezahlt werden.


2)
Gegeben sind p in \% und W.
Gesucht ist G.
Da sich die Streckenangabe auf den bereits zurücklegten Teil der Tour bezieht und die Prozentangabe auf den noch vor ihnen liegenden, muss der Prozentsatz erst umgerechnet werden: 100\,\%-20\,\%=80\,\%

\begin{array}{rclcr}80\,\% &\widehat{=}& 60\,\text{km} \cr \cr 1\,\% &\widehat{=}& \dfrac{60\,\text{km}}{80} \cr \cr 100\,\% &\widehat{=}& \dfrac{100\cdot 60\,\text{km}}{80} &=& 75\,\text{km} \end{array}

Die Strecke ist insgesamt 75\,\text{km} lang.


3)
Gegeben sind p in \% und W.
Gesucht ist G.

G = \dfrac{W}{p} \cdot 100 = \dfrac{59{,}5}{85} \cdot 100 = 70

Die Hose kostete vor dem Ausverkauf 70\,\text{EUR}


4)
Gegeben sind p in \% und G.
Gesucht ist W.

W = \dfrac{p \cdot G}{100} = \dfrac{25 \cdot 65{,}99}{100} = 16{,}4975

Dies ist aber noch nicht der neue Preis, sondern der Betrag, um den der Preis des Pullovers reduziert wurde. Den neuen Preis erhält man durch 65{,}99 \text{ EUR }-16{,}4975 \text{ EUR } = 49{,}4925 \text{ EUR } \approx 49{,}49 \text{ EUR } .

Alternative: W = \dfrac{p \cdot G}{100} = \dfrac{75 \cdot 65{,}99}{100} = 49{,}4925

 
5)
Gegeben sind W und G.
Gesucht ist p in \%.

p \, \% = \dfrac{W}{G} \cdot 100 \, \% = \dfrac{9}{12} \cdot 100 \, \% = 75 \, \%

Auch dies ist noch nicht die Antwort auf die Frage, um wie viel Prozent der Preis reduziert worden ist. Da der neue Preis laut Rechnung 75 \, \% des alten Preises entspricht, beträgt der Preisnachlass also 25 \, \%.

 
6)
Gegeben sind p in \% und G.
Gesucht ist W.
Da sich die Prozentangabe auf den bereits zurücklegten Teil der Tour bezieht, die Frage aber auf den noch vor ihnen liegenden, muss der Prozentsatz erst umgerechnet werden: 100\,\%-56\,\%=44\,\%

\begin{array}{rclcr}100\,\% &\widehat{=}& 14.600\,\text{m} \cr \cr 1\,\% &\widehat{=}& \dfrac{14.600\,\text{m}}{100} \cr \cr 44\,\% &\widehat{=}& \dfrac{44\cdot 14.600\,\text{m}}{100} &=& 6.424\,\text{m} \end{array}

Sie müssen noch 6{,}424\,\text{km} laufen.

Bemerkung: Bitte achten Sie auf die unterschiedlichen Einheiten (einmal Meter und einmal Kilometer)!

 
7)
Gegeben sind W und G.
Gesucht ist p in \%.

\begin{array}{rclcr}81{,}2\;\text{Mio. Menschen}&\widehat{=}& 100\,\% \cr \cr 1\;\text{Mio. Menschen}&\widehat{=}& \dfrac{100\,\%}{81{,}2\;\text{Mio.}} \cr \cr 12{,}5\;\text{Mio. Menschen}&\widehat{=}& \dfrac{12{,}5\;\text{Mio.}\cdot 100\,\%}{81{,}2\;\text{Mio.}} &\approx& 15{,}39\,\% \end{array}

Etwa 15{,}39\, \% der Menschen in Deutschland leben in den neuen Bundesländern.


8)
Gegeben sind W und G.
Gesucht ist p in \%.

\begin{array}{rclcr}28\;\text{Kinder}&\widehat{=}& 100\,\% \cr \cr 1\;\text{Kind}&\widehat{=}& \dfrac{100\,\%}{28} \cr \cr 3\;\text{Kinder}&\widehat{=}& \dfrac{3\cdot 100\,\%}{28} &\approx& 10{,}71\,\% \end{array}

Der Anteil der Linkshänder/-innen in der Klasse beträgt etwa 10{,}71\, \%.


9)
Gegeben sind p in \% und W.
Gesucht ist G.

\begin{array}{rclcr}1\,\% &\widehat{=}& 190\;\text{EUR} \cr \cr 100\,\% &\widehat{=}& 100\cdot 190\;\text{EUR} &=& 19.000\;\text{EUR} \end{array}

Das Auto kostet 19.000\;\text{EUR}.

Bemerkung: Da in dieser Aufgabe netterweise bereits angegeben ist, wie der Prozentwert für 1\,\% lautet, vereinfacht sich die Rechnung.


10)
Zunächst berechnet man die Menge des tatsächlich vorhandenen Salzes im Meerwasser.
\begin{array}{rclcl} 100 \,\% &\widehat{=}& 400 \;\text{g} \cr\cr 1 \,\% &\widehat{=}& \dfrac{400}{100} \;\text{g} \cr\cr 4 \,\% &\widehat{=}& \dfrac{4 \cdot 400\;\text{g}}{100} &=& 16 \;\text{g} \end{array}

Diese Menge Salz soll nun nicht 4 \,\%, sondern nur 2{,}5 \,\% ausmachen.
\begin{array}{rclcl} 2{,}5 \,\% &\widehat{=}& 16 \;\text{g} \cr\cr 1 \,\% &\widehat{=}& \dfrac{16}{2{,}5} \;\text{g} \cr\cr 100 \,\% &\widehat{=}& \dfrac{16 \cdot 100\;\text{g}}{2{,}5} &=& 640 \;\text{g} \end{array}

Um einen Salzanteil von 2{,}5 \,\% in der Lösung zu erhalten, müssen 640\; \text{g} - 400 \;\text{g} = 240 \;\text{g} Süßwasser zum Meerwasser gegeben werden.

 

6. Aufgabe

Berechnen wir zunächst die Prozentwerte aus den gegebenen Größen:

Schokolade mit Nüssen: \begin{array}{rccr}99\,\% &\text{von } 400 & = & 396 \end{array}
Schokolade ohne Nüsse: \begin{array}{rccr}1\,\%&\text{von } 400 & = & 4 \end{array}


Der Trick ist nun, von den Schokotafeln ohne Nüsse auszugehen, weil wir hiervon sowohl den Prozentsatz (Bei 98\,\% Schokotafeln mit Nüssen müssen es wohl 2\,\% Schokotafeln ohne Nüsse sein ...) als auch den Prozentwert (Da wir nur Schokolade mit Nüssen essen sollen, bleibt es bei 4 Tafeln ohne Nüsse ...) kennen:
\begin{array}{rcrl}2\,\% & \widehat{=} & 4 & \text{Tafeln} \cr\cr 98\,\% & \widehat{=} & 196 & \text{Tafeln}\end{array}

Da wir ursprünglich 396 Tafeln Schokolade mit Nüssen hatten, müssen also 200 Tafeln gegessen werden, damit nur noch 196 Tafeln übrig sind. Ganz schön viele ...

5. Lineare Gleichungen - Lernziele und typische Fehler

Nach Durcharbeiten dieses Kapitels sollten Sie folgende Lernziele erreicht haben:

  • Sie kennen die Vokabeln "Konstante", "Variable", "Koeffizient", "Term" und "Gleichung" und können sie sicher verwenden.
  • Sie können Variablen addieren und subtrahieren.
  • Sie können Zahlenwerte für Variablen einsetzen und die entsprechenden Werte berechnen.
  • Sie können zu einer linearen Gleichung den passenden Definitionsbereich bestimmen.
  • Sie kennen die allgemeine Form einer linearen Gleichung.
  • Sie wissen, dass es lineare Gleichungen mit keiner/einer/unendlich vielen Lösung(en) gibt und können anhand des Rechenweges feststellen, welcher Fall vorliegt.
  • Sie können lineare Gleichungen lösen.
  • Sie kennen die Bedeutung des Definitionsbereichs und wissen, warum er beim Lösen von Gleichungen wichtig ist.
  • Sie können die Lösungsmenge mathematisch korrekt notieren.
  • Sie können mithilfe der Probe überprüfen, ob die gefundene Lösung tatsächlich richtig ist.
  • Sie können entsprechende Textaufgaben lösen und die Lösungswege mathematisch vernünftig aufschreiben.
  • Sie können lineare Gleichungen von anderen Gleichungsarten unterscheiden.


Typische Fehler
in diesem Kapitel sind:

  • Terme werden mit Gleichungen verwechselt. Erklärung
  • Beim Multiplizieren der Gleichung mit einer Zahl werden nicht alle Bestandteile der Gleichung multipliziert. Erklärung


Für Online-Selbsttests zu diesem Thema und weitere Informationen zur Mathematikunterstützung an der TH Wildau nutzen Sie bitte den Moodle-Kursraum "SOS Mathematik - Brückenkurs".

Übersicht:

 

5.1 Lineare Gleichungen - Aufgaben

1. Aufgabe

Fassen Sie so weit wie möglich zusammen!

1) -3(-4x \cdot x+2x)

  11) \dfrac{2}{3a-9} \, + \, \dfrac{b}{-3+a}

2) 4x \cdot x-3x \cdot y+11x \cdot x+16x-40y+3y \cdot x

  12) \dfrac{4x}{5} \, \cdot \, \dfrac{2}{3x}

3) 60a \cdot a \cdot a \cdot b \cdot c \cdot c+10a \cdot b \cdot b \cdot b \cdot c-30a \cdot b \cdot c \cdot c \cdot c

  13) \dfrac{2a}{5} \, : \, \dfrac{a}{10}

4) 6-(8x-4y)+2(10y-7)+12-18x

  14) \dfrac{8}{z}+\left(\dfrac{z}{2}-12\right) \left(1+\dfrac{4}{z}\right)

5) -(2a+d)(-2d+a)

  15) \dfrac{n-2}{n\cdot n+5n} \, : \, \dfrac{3n-6}{10n}

6) \dfrac{4}{5}t-\dfrac{t}{10}+\dfrac{7}{15}t

 

16) 6yz-\{12+ 7[yz +16-3x(15+8y)]\}+13x

7) \dfrac{3x}{8}-\dfrac{5}{12}x-\dfrac{4}{x}

  17) \{8x[2+(13-5x)\cdot9x]-5\}\cdot4

8) \dfrac{2a}{b}+\dfrac{a}{m}+\dfrac{a}{b}

  18) \dfrac{1}{2}[6(12+4ab)\cdot3c-b(10a+20c)]+19

9) \dfrac{1}{x+1}-\dfrac{1}{x-1}

  19)  (x-15)[-u(2x+14ux-7)+16u-24xu]

10) \dfrac{y+2}{y-2}-\dfrac{2y-2}{y+4}

  20)  \dfrac{2}{3}\left[21+(11+18y)(-x-3)-2x\{9xy-5x+12\}-27x\right]

 

2. Aufgabe

Berechnen Sie jeweils die gesuchte Variable!

1) Gesucht: a mit a=2b+c, b=3+c und c=1
 
2) Gesucht: s mit 2s= -4r-t+12, r=3t-16 und t=24
 
3) Gesucht: x mit x=a-10-4z, 2a=18z+2 und z=-5
 
4) Gesucht: b mit 12b=3l+ \dfrac{1}{3}z+11, 2l=2z-14 und z=-9
 
5) Gesucht: u mit \dfrac{1}{5}u=100-25v-8w, 2v=16-8w und -3w=27

 

3. Aufgabe

Sind die folgenden Gleichungen linear? Hinweis: Es ist nicht nach der Lösung der Gleichungen gefragt.

1) \dfrac{1}{7}x+8= -12 \cdot \dfrac{x}{6}

  6) 1+x \cdot\sqrt{4}=0
2)  \cos(x+2)=(21x-1)(-9+18x)

  7) 7(x-5) \cdot (3-2x)=-29
3) 17-4x+8x^2=11   8) -5^3 \cdot x + 16=56
4) \dfrac{1}{x}+4=-3(2x-10)

  9) \dfrac {x}{\sqrt{64}}=-14x+36
5) 4(3y-44)=(-y+6)\cdot17   10) -15= \dfrac{7}{x}+24

 

4. Aufgabe

Lösen Sie folgende Gleichungen und machen Sie - wenn möglich - die Probe! Soweit nichts anderes angegeben ist, soll \mathbb{D} = \mathbb{R} sein.

1) 2x - 4 \, = \, -4x - 1   11) 13m+m+30-23m-45+17+12m = -31

2) 3\, (2x-1) = -5\, (17+7x)

  12)  \dfrac{x+3}{2} = \dfrac{x+3}{4}

3) 6(4x-8) = (-12x+24) \cdot (-2)

  13)  5 \left(x+\dfrac{18}{25}\right) = 3\left(x-\dfrac{4}{5}\right) mit \mathbb{D} = \mathbb{R}^+

4) 3x \, (-4x-10)=(2-2x) \cdot (6x+15)

  14)  0{,}2a-0{,}3-0{,}5a+3{,}1a+0{,}7a = 1{,}9a+5-0{,}4a+2{,}7

5) 4 \, (4a-1)=8 \, \left(\dfrac{1}{2}+2a\right)

  15)  \dfrac{-12x+24}{6} = 2\left(3x-38\right)

6) -3 \, (6y+2)=12y-5 mit \mathbb{D} = \mathbb{N}

  16)  \dfrac{3x}{5}-\dfrac{4}{5}x+2x = x+4

7)  2(3x-1) = 3(2+5x)+1

  17)  0 = 7+3x+4-5x+4x-8+x+30

8)  \dfrac{1}{3} (-3y+6) = \dfrac{3}{2} (6y-2)

  18) -9+3\left(-8p+3\right) = -2\left(10p-2\right)+4\left(-1-p\right)

9)  6 \left(2x-4\right)-7 = 9-8\left(\dfrac{1}{4}x+5\right)

  19) 3\left(5b+1\right)-3b = 21-2\left(4+\dfrac{5}{2}b\right)

10)  4 \left(\dfrac{x}{2}-\dfrac{1}{3}\right) + 5 \left(\dfrac{x}{5}-\dfrac{1}{3}\right) = 0

  20) 30z-26\left(\dfrac{2z}{13}-1\right)+5z = -6+2\left(25z+16\right)-19z

 

5. Aufgabe

Pythagoras auf die Frage, wie viele Schüler er habe: „Die Hälfte studiert Mathematik, ein Viertel Physik, ein Siebtel lernt das Schweigen und der Rest sind 3 kleine Knaben.“ Wie viele Schüler hat Pythagoras?

Dieses Kapitel enthält die folgenden Themen:

 

5.2 Lineare Gleichungen - Erklärungen

Gleichungen sind etwas sehr Wichtiges in der Mathematik. Daher schauen wir uns in diesem Kapitel die grundlegenden Prinzipien und Vorgehensweisen anhand des einfachsten Gleichungstyps an. In späteren Kapiteln werden zwar die Gleichungen immer komplexer - an diesen Prinzipien und Vorgehensweisen ändert sich aber wenig. Insofern lohnt es sich, hier ganz genau hinzuschauen und mitzudenken.

 

Was ist ...

... eine Konstante?
Ein Platzhalter für einen festen Zahlenwert

... eine Variable?
Ein Platzhalter für einen veränderlichen Zahlenwert

... ein Koeffizient?
Die Zahl in einem Produkt aus Zahl und Variable

... ein Term?
Eine mathematisch sinnvolle Kombination aus Zahlen, Konstanten, Variablen, Klammern und Rechenoperationen

... eine Gleichung?
Etwas in der Art: "ein Term = ein (anderer) Term"

Um diese Begriffe etwas greifbarer zu machen ...: Terme sind so etwas wie die "Wörter" der Mathematik, aus denen dann komplexe "Texte", z. B. Gleichungen, zusammengesetzt werden. Ebenso wie Texte die Basis z. B. für literarische Untersuchungen sind, benötigt die Mathematik Gleichungen für Berechnungen und Analysen aller Art.

Bitte achten Sie auf den (wichtigen!) Unterschied zwischen Term und Gleichung: Simpel gesagt, besteht eine Gleichung aus zwei Termen, die durch ein Gleichheitszeichen verbunden sind. Sie steht für die Aussage, dass irgendein Term gleich einem anderen Term oder einem bestimmten Zahlenwert ist oder sein soll. Das sagt ja bereits der Name … Meist gehört zu Gleichungen die Aufgabe, eine Lösung zu finden, also den Zahlenwert, für den die Terme tatsächlich gleich sind (dazu unten mehr).
Terme hingegen enthalten zunächst mal kein Gleichheitszeichen. Sie haben auch keine zugehörige Aufgabe, weil ihr Einsatzgebiet sehr vielfältig ist. Ein paar Beispiele, wo Terme mit ganz unterschiedlichen "typischen Aufgaben" auftauchen (auch wenn diese teilweise auf spätere Kapitel vorgreifen):

  • Gerade wurde schon die Kombination von Termen zu Gleichungen mit der Aufgabe, Lösungen zu finden, erwähnt.
  • Bei Funktionen werden Terme für verschiedene Ziele eingesetzt, z. B. Funktionswerte berechnen, Nullstellen bestimmen oder die Funktion ableiten.
  • In dem Term \pi r^2 erkennen Sie vielleicht die Flächenformel für einen Kreis: Bei solchen Termen geht es meist darum, aus einem gegebenen Radius r die Fläche A_{Kreis} zu berechnen - wieder ein ganz anderer Anwendung von Termen.

Sie sehen an diesen Beispielen, dass der sinnvolle Umgang mit einem Term stark vom Kontext abhängt, in dem der Term steht. Es gibt ganz viele verschiedene Ansätze und Lösungsstrategien, die man ordentlich unterscheiden muss. Man darf daher z. B. nicht einfach =0 hinter einen Term schreiben, ihn damit als Gleichung verstehen und eine Lösung berechnen. Vielleicht war der Einsatzbereich ja ein ganz anderer.

 

Rechnen mit Variablen

So, nun sind die wichtigsten Begrifflichkeiten geklärt und wir können uns um das eigentliche Rechnen mit Variablen kümmern, ohne welches lineare Gleichungen nicht zu lösen sind.

Gleich zu Beginn eine gute Nachricht: Da Variablen ja nichts Anderes als Platzhalter für Zahlen sind, rechnet man mit Variablen im Großen und Ganzen genauso wie mit Zahlen. Man fasst einfach gleichartige Objekte zusammen, indem man sie quasi "zählt". Ein paar Beispiele:

2+2+2+2+2 = 5\cdot 2 = 10    
x+x+x+x+x = 5\cdot x       Nur ausrechnen kann man es halt nicht, solange man nicht weiß, für welche Zahl x in diesem Fall steht.
y+z+z+z+y+z+y = 3\cdot y+4\cdot z       Hier passt die - vielleicht schon etwas überstrapazierte - Analogie von den Äpfeln und Birnen: Man kann zu 3 Äpfeln 4 weitere Äpfel hinzuzählen, aber nicht 4 Birnen ... Gleichermaßen kann man 3y+4z nicht weiter zusammenfassen.

 

3\cdot 4+3\cdot 4+3\cdot 4+3\cdot 4+3\cdot 4+3\cdot 4 = 6\cdot 3\cdot 4 = 72
7\cdot y+7\cdot y+7\cdot y+7\cdot y+7\cdot y+7\cdot y = 6\cdot 7\cdot y = 42\cdot y

 

Es gelten grundsätzlich - auch wenn Variablen ins Spiel kommen - die "ganz normalen" Rechenregeln, z. B. aus den Kapiteln Rechengesetze und Bruchrechnung, also Klammern auflösen, Brüche erweitern etc. Es gelten aber auch die gleichen Einschränkungen: Beispielsweise darf man auch bei Brüchen mit Variablen nicht aus Summen kürzen. Erst wenn neue Rechenkonzepte, wie Potenzen, Wurzeln und Logarithmen, hinzukommen, werden neue Rechenregeln (natürlich mit neuen Einschränkungen ...) benötigt. Der Oberbegriff für die ganzen Verfahren, mit denen man einen Term in eine einfachere, kompaktere Form überführt, ist, "einen Term vereinfachen".
Wenn man einen Term vereinfacht, steht zwischen den verschiedenen Schritten natürlich ein Gleichheitszeichen - eine Umformung, die dazu führt, dass die Terme nicht mehr gleich sind, wäre ja nicht zielführend.
Ein Beispiel:
\begin{array}{rclcl}-90b-15\left(\dfrac{1}{5}a-(10-2b)(a-3)\right)+30ab &=& -90b-15\left(\dfrac{1}{5}a-(10a-30-2ab+6b)\right)+30ab &\vert& \text{innere Klammern mit dem Distributivgesetz aufgelöst} \cr&=& -90b-15\left(\dfrac{1}{5}a-10a+30+2ab-6b\right)+30ab &\vert& \text{Minusklammer aufgelöst, ebenfalls Distributivgesetz} \cr&=& -90b-3a+150a-450-30ab+90b+30ab &\vert& \text{ausmultipliziert} \cr&=& -3a+150a-30ab+30ab+90b-90b-450 &\vert& \text{"sortiert", also das Kommutativgesetz angewendet} \cr&=& 147a-450 &\vert& \text{zusammengefasst}\end{array}
Wir haben nun eine einfachere, kürzere Variante des Ausgangsterms. Z. B. erkennt man, dass b in diesem Term eigentlich gar keine Rolle spielt.
Wenn man Zahlen für die Variablen einsetzt und dann ausrechnet, erhält man in jeder Zeile das gleiche Ergebnis. Nehmen wir beispielsweise a=-2 und b=9:

Ausgangsterm: -90\cdot 9-15\left(\dfrac{1}{5}\cdot (-2)-(10-2\cdot 9)(-2-3)\right)+30\cdot (-2) \cdot 9 = -744
1. Umformung: -90\cdot 9-15\left(\dfrac{1}{5}\cdot (-2)-(10\cdot (-2)-30-2\cdot (-2)\cdot 9+6\cdot 9)\right)+30\cdot (-2) \cdot 9 = -744
2. Umformung: -90\cdot 9-15\left(\dfrac{1}{5}\cdot (-2)-10\cdot (-2)+30+2\cdot (-2)\cdot 9-6\cdot 9)\right)+30\cdot (-2) \cdot 9 = -744
3. Umformung: -90\cdot 9-3\cdot (-2)+150\cdot (-2)-450-30\cdot (-2)\cdot 9+90\cdot 9+30\cdot (-2) \cdot 9 = -744
4. Umformung: -3\cdot (-2)+150\cdot (-2) -30\cdot (-2)\cdot 9+30\cdot (-2) \cdot 9+90\cdot 9-90\cdot 9-450 = -744
5. Umformung: 147\cdot(-2)-450 = -744

Wenn Sie den Rechenaufwand in den einzelnen Zeilen vergleichen, wird vermutlich schnell klar, warum Termvereinfachungen keine so schlechte Idee sind … Machen Sie sich aber bitte klar, dass wir hier keinen Wert für die Variablen ausgerechnet haben (wie das beim Lösen einer Gleichung der Fall wäre). Stattdessen haben wir nur Beispielwerte eingesetzt, um zu zeigen, dass die verschiedenen Umformungsschritte tatsächlich den gleichen Wert liefern.

Damit haben wir auch gleich das Vorgehen geklärt, wenn man ausrechnen möchte (oder muss), welchen Wert ein Term annimmt, wenn man für die Variablen bestimmte Zahlenwerte einsetzt. Das wird z. B. weiter unten bei der Probe wichtig. Schauen wir uns jetzt noch an, wie man Terme in andere Terme einsetzt:

Gesucht ist x, welches folgendermaßen berechnet werden soll: x=3y-10z, dabei ist y=-2z-4 und z=23
\begin{array}{cclll} x &=& 3y-10z & \vert & \text{Setze y ein} \cr x &=& 3\cdot(-2z-4)-10z & \vert & \text{Setze z ein} \cr x &=& 3\cdot(-2\cdot 23-4)-10\cdot 23 \cr x &=& 3 \cdot (-50)-230 \cr x &=& -380 \end{array}

Am sinnvollsten ist es hier, Schritt für Schritt vorzugehen, also erst den komplizierteren Term (hier y) einzusetzen, sich alles hinzuschreiben und dann den anderen Term einzusetzen. Das Ausrechnen ist dann der letzte Schritt.
Ganz wichtig: Klammern nicht vergessen! Der y-Term besteht aus einer Differenz, die dann im Gesamtterm multipliziert werden muss. Also treffen hier Punkt- und Strichrechnung aufeinander und bekanntermaßen geht Punktrechnung vor Strichrechnung.

Der Wert, den ein Term beim Einsetzen von Zahlen, annimmt, ändert sich durch Termvereinfachungen nicht. Schauen wir uns nochmal das Beispiel von oben in einer leicht veränderten Form an: Gesucht ist x, welches folgendermaßen berechnet werden soll: x=y+y+y-z-z-z-z-z-z-z-z-z-z, dabei ist y=-2z-4 und z=23.
Es ist (hoffentlich) nicht so schwer zu erkennen, dass x=y+y+y-z-z-z-z-z-z-z-z-z-z=3y-10z. Dann gehen wir genauso vor wie oben:
\begin{array}{cclll} x &=& y+y+y-z-z-z-z-z-z-z-z-z-z & \vert & \text{Setze y ein} \cr x &=& -2z-4+(-2z-4)+(-2z-4)-z-z-z-z-z-z-z-z-z-z & \vert & \text{Setze z ein} \cr x &=& -2\cdot 23-4+(-2\cdot 23-4)+(-2\cdot 23-4)-23-23-23-23-23-23-23-23-23-23 \cr x &=& -50+(-50)+(-50)-23-23-23-23-23-23-23-23-23-23 \cr x &=& -380 \end{array}
In dieser Variante muss man sich zwar nicht so sehr um Klammern kümmern - dafür ist es deutlich schwieriger, den Überblick zu behalten.


Achtung:
Nicht jeder Buchstabe, der in der Mathematik auftaucht, ist eine Variable! Denken Sie beispielsweise an \pi.
Ebenso ist nicht alles, was einen Rechenoperator enthält, variabel: \sqrt{23+2}=\sqrt{25}=5

 

Lineare Gleichungen

Definition: Eine lineare Gleichung ist eine spezielle Form einer Gleichung, in der ausschließlich folgende Terme auftreten:

  • Produkte aus einer Variablen und Konstanten/Zahlen
  • Summen aus diesen Produkten und (anderen) Konstanten/Zahlen

 

Beispiele für lineare Gleichungen

  • 3x+4=\dfrac{x}{3}-12
    Diese Gleichung ist linear, da die einzige Variable x nur mit Zahlenwerten multipliziert wird. \frac{x} {3} ist dabei nichts Anderes als \frac{1}{3} \cdot x . Dann werden noch Zahlen addiert. Das ist alles erlaubt.

  • -2(9t+1)-9 (12+3t)=0
    Bei dieser Gleichung ist durch die Klammern nicht sofort zu erkennen, ob sie linear ist. Beim Ausmultiplizieren werden die Terme, die die Variable t enthalten, aber nur mit weiteren Zahlen multipliziert, sodass auch diese Gleichung linear ist. Würden Terme dieser Art -2t(9t+1)=-18t\cdot t-2t vorhanden sein, wäre das nicht der Fall.

  • \sqrt{16} \cdot x=8
    Diese Gleichung ist tatsächlich linear, denn \sqrt{16} (also "Wurzel aus 16") sieht zwar kompliziert aus, ist aber einfach nur eine Zahl, nämlich 4.

  • 3x+9y-18z=20
    Auch diese Gleichung ist linear, da die Variablen nur mit Zahlen multipliziert werden (es ist ja in der Definition nicht gesagt, dass es nur eine Variable geben darf). Anschließend werden diese Produkte addiert. Damit sind in dieser Gleichung nur erlaubte Terme enthalten. Nicht erlaubt wären beispielsweise die Terme 3x \cdot y oder -18z \cdot z , da hier Variablen miteinander multipliziert werden.

Beispiele für nicht lineare Gleichungen

  • \dfrac{1}{x}-12=0
    Hier wird durch eine Variable dividiert. Das ist in linearen Gleichungen nicht erlaubt.

  • 3z^2-7=90
    In linearen Gleichungen dürfen Variablen nur mit Zahlen, nicht aber mit Variablen multipliziert werden. z^2 ist ja eine abkürzende Schreibweise für z \cdot z.

  • \sqrt{16\cdot x}=8
    Im Unterschied zu oben steht hier auch die Variable unter der Wurzel. Das verletzt die Bedingungen für lineare Gleichungen - Wurzeln sind ja etwas Anderes als Produkte bzw. Summen ...

  • \sin(x)-3x=4x-10
    Eine lineare Gleichung darf keine trigonometrischen Funktionen, wie den Sinus, angewendet auf eine Variable enthalten.

 

Lösen einer Gleichung

Grundsätzlich bedeutet "Gleichung lösen", für die Variable alle Werte zu finden, die beim Einsetzen in die Gleichung beide Seiten der Gleichung gleich groß werden lassen.
Ein Beispiel: Offensichtlich ist x=4 eine Lösung der Gleichung 3\cdot x=12 , denn 3\cdot 4=12 . Setzt man x=4 in die Gleichung ein, werden also beide Seiten gleich groß. In diesem Fall steht dann auf beiden Seiten 12. Alle Werte meint dabei, dass eine Gleichung durchaus mehrere Lösungen haben kann. Um diese Werte zu finden, wird die Gleichung durch Umformungen in eine Form gebracht, in der man diese Werte ablesen kann.
Dabei ist eines wichtig: Die Lösungen der Gleichung dürfen sich während der Umformungen nicht ändern, sonst wären die am Ende abgelesenen Zahlenwerte ja keine Lösungen der ursprünglichen Gleichung! Das heißt: Es gibt Umformungen, die beim Lösen einer Gleichung weiterhelfen - bei anderen muss man vorsichtig sein. Die Umformungen, die einem weiterhelfen, weil sie die Lösungsmenge der Gleichung nicht ändern, nennt man Äquivalenzumformungen. Dazu gehören die folgenden Umformungen:

  1. auf beiden Seiten der Gleichung die gleiche reelle Zahl, egal welche, addieren
  2. auf beiden Seiten der Gleichung das gleiche Vielfache einer Variablen addieren
  3. beide Seiten der Gleichung mit der gleichen reellen Zahl, ausgenommen der 0, multiplizieren
  4. beide Seiten der Gleichung mit dem gleichen Term ungleich 0 multiplizieren
  5. Terme vereinfachen, wie Klammern auflösen, Variablen zusammenfassen und Zahlen ausrechnen
  6. die beiden Seiten der Gleichung vertauschen


Bemerkung: Bitte beachten Sie, dass in Addieren und Multiplizieren (Äquivalenzumformungen 1. bis 4.) auch Subtrahieren und Dividieren enthalten sind. Statt 5 zu subtrahieren, kann man ja -5 addieren. Statt durch 10 zu dividieren, kann man ja mit \frac{1}{10} multiplizieren.


Ganz wichtig:
Wird beim Lösen einer Gleichung etwas addiert oder multipliziert, muss dies immer auf beiden Seiten der Gleichung gemacht werden. Sonst ändert sich die Lösungsmenge und die Mühe, einen Wert auszurechnen, war umsonst.
Hier ein Beispiel, indem korrekterweise alle Summanden in der Gleichung mit 2 multipliziert werden:
\begin{array}{rclcl} \dfrac{1}{2}x+13 &=& 9 &\vert& \cdot 2 \cr x+26 &=& 18 &\vert&-26 \cr x &=& -8 \end{array}

Überprüfung:
\begin{array}{rcl} \dfrac{1}{2}\cdot (-8)+13 &=& 9 \cr-4+13 &=& 9 \cr9 &=& 9 \end{array}
Setzt man den berechneten Wert in die Gleichung ein, ergibt sich auf beiden Seiten die gleiche Zahl. Der berechnete Wert ist also die Lösung der linearen Gleichung.

Würde man fälschlicherweise nur die beiden Summanden auf der linken Seite mit 2 multiplizieren, ergäbe sich folgende Rechnung:
\begin{array}{rclcl} x+26 &=& 9 &\vert& -26 \cr x &=& -17 \end{array}

Überprüfung:
\begin{array}{rcl} \dfrac{1}{2}\cdot (-17)+13 &\neq& 9 \cr-8{,}5+13 &\neq& 9 \cr-4{,}5 &\neq& 9 \end{array}

Würde man fälschlicherweise nur den ersten Summanden auf der linken Seite mit 2 multiplizieren, ergäbe sich folgende Rechnung:
\begin{array}{rclcl} x+13 &=& 9 &\vert& -13 \cr x &=& -4 \end{array}

Überprüfung:
\begin{array}{rcl} \dfrac{1}{2}\cdot (-4)+13 &\neq& 9 \cr-2+13 &\neq& 9 \cr11 &\neq& 9 \end{array}
Man sieht in diesen beiden Fällen, dass man 1. für x einen anderen Wert berechnet als oben und 2. die Überprüfung nicht aufgeht. Die Werte, die sich ergeben, wenn man nur einige Summanden mit 2 multipliziert, können also nicht Lösung der Gleichung sein.

Berücksichtigen Sie daher bei solchen Umformungen immer alle Terme auf beiden Seiten der Gleichung!

Etwas ganz Ähnliches wird Ihnen bei weiteren Gleichungstypen in späteren Kapiteln wieder begegnen. Es ist also wichtig, dass Sie sich hier schon damit vertraut machen.



Vielleicht haben Sie sich gefragt, warum die Multiplikation mit 0 keine Äquivalenzumformung ist. Nehmen wir die (sehr einfache) Beispielgleichung:
8=7
Dies ist offensichtlich falsch. Wenn wir nun beide Seiten mit 0 multiplizieren, passiert das Folgende:
8\cdot 0=7 \cdot 0 vereinfacht also 0=0
Das wiederum ist richtig ... Wenn eine Umformung aus einer falschen Aussage eine richtige macht, ändert sich offensichtlich die Lösungsmenge. Damit erfüllt die Multiplikation mit 0 nicht das Kriterium für eine Äquivalenzumformung. Die Erkenntnis, die man aus der umgeformten Gleichung ziehen kann, hat nämlich nichts mehr mit der ursprünglichen Gleichung zu tun. Die Umformung hilft uns auf dem Weg zur Lösung nicht weiter.


Die gerade besprochenen Äquivalenzumformungen sind grundsätzlich bei allen Gleichungen zulässig; allerdings reichen sie bei nicht linearen Gleichungen (wie sie in späteren Kapiteln behandelt werden) nicht immer aus. Dann kommen weitere Umformungsmöglichkeiten hinzu.

 

Hier ein ungefährer Plan zum Lösen einer Gleichung:

  1. Klammern auflösen
  2. auf beiden Seiten der Gleichung zusammenfassen (alle Zahlenwerte zusammenrechnen, alle Variablen zusammenrechnen)
  3. "sortieren": Mithilfe der Addition alle Terme, die die Variable enthalten, auf die eine Seite der Gleichung bringen; alles ohne Variable auf die andere Seite
    Achtung: Die Variable muss nicht x heißen!
  4. durch den Koeffizienten der Variable dividieren

Ein 5. Schritt wird weiter unten noch hinzukommen.


Ein Beispiel:

\begin{array}{rclcl}-\left(4x-9\right)-2\left(-\dfrac{x}{2}+1\right)+7 &=& -11x-6+3x &\vert & \text{Klammern auflösen} \cr -4x+9+x-2+7 &=& -11x-6+3x &\vert & \text{zusammenfassen} \cr -3x+14 &=& -8x-6 &\vert & \text{"sortieren": } +3x+6 \cr 14+6 &=& -8x+3x &\vert & \text{zusammenfassen}\cr 20 &=& -5x &\vert & :\left(-5\right) \cr -4 &=& x \end{array}

Das Entscheidende an diesen Gleichungen ist, dass sie alle die gleiche Lösung haben. Anders gesagt: Löst eine Zahl eine dieser Gleichungen, löst sie alle! Sie können das direkt nachrechnen: Wenn Sie die Lösung der letzten Gleichung x=-4 in die anderen einsetzen, muss das Ergebnis der linken Seite immer gleich dem Ergebnis der rechten Seite sein.

Zur Notation: Es hat sich eingebürgert, am Ende einer Gleichungszeile hinter einem senkrechten Strich anzugeben, welche Rechenoperationen / Umformungen auf die Gleichung in diesem Schritt angewendet werden. Das muss man nicht machen, ist aber gerade am Anfang meist ziemlich hilfreich.

Bemerkung 1: Je nach Art der Gleichung kann man manchmal einen oder mehrere Schritte überspringen.
Bemerkung 2: Es ist egal, ob die Variable auf der rechten oder der linken Seite der Gleichung "gesammelt" wird, da die beiden Seiten einer Gleichung ja getauscht werden dürfen. x=-4 meint das Gleiche wie -4=x

 

Definitionsbereich und Lösungsmenge

Das war (leider) noch nicht ganz alles, was man beim Lösen linearer (und auch anderer) Gleichungen beachten muss ...

Zu jeder Gleichung gehört die Angabe eines Definitionsbereichs (Formelzeichen: \mathbb{D}). Dieser gibt an, welche Zahlen grundsätzlich als Lösung infrage kommen. Wenn nichts weiter angegeben ist und keine inhaltlichen bzw. formalen Gründe dagegensprechen, kann man davon ausgehen, dass der Definitionsbereich die Menge der reellen Zahlen ist, also \mathbb{D}=\mathbb{R}.

Stehen Aufgaben in einem inhaltlichen Zusammenhang, sind häufig nur Lösungen aus einem bestimmten Zahlenbereich sinnvoll. Ein Beispiel: Als Nervennahrung, um die Matheklausur zu bestehen, benötigt ein Student der TH Wildau 5 Schokoriegel. In der Mensa zahlt er dafür 4{,}50\;\text{EUR}. Wie teuer ist ein Schokoriegel?
Sei s der Preis eines Schokoriegels. Dann ist die folgende Gleichung zu lösen: 5\cdot s=4{,}50
Es dürfte einleuchtend sein, dass die Lösung dieser Gleichung positiv sein muss - ok, 0 ist als Lösung auch möglich, wenn auch (leider) unwahrscheinlich ... Mathematisch notiert man: \mathbb{D}=\mathbb{R}^+_0. Wenn man noch genauer sein möchte, kann man argumentieren, dass irrationale Zahlen, wie \sqrt{2} und \pi, ebenfalls nicht als Preise infrage kommen (Wie sollte man die unendlich vielen Nachkommastellen bezahlen?), die positiven rationalen Zahlen reichen also als Definitionsbereich aus: \mathbb{D}=\mathbb{Q}^+_0.

Variieren wir das Beispiel ein bisschen: Eine Studentin der TH Wildau möchte 5{,}20 \;\text{EUR} für Gummibärchen ausgeben. Eine Packung kostet 1{,}30\;\text{EUR}. Wie viele Packungen kann sie kaufen?
Sei g die Anzahl der Gummibärchentüten. Dann ist die folgende Gleichung zu lösen: 1{,}30\cdot g=5{,}20
Hier gibt es noch mehr Einschränkungen: Weder negative noch gebrochene Zahlenwerte kommen als Lösung infrage. 0 ist hier wieder theoretisch als Lösung möglich. Umgekehrt formuliert, benötigen wir für die Lösung nichtnegative, ganze Zahlen. Oder - noch anders formuliert - gesucht ist eine natürliche Zahl: \mathbb{D}=\mathbb{N}.

Auch aus formalen Gründen kann es notwendig sein, den Definitionsbereich einzuschränken: Da man durch 0 ja nicht teilen darf, muss man dafür sorgen, dass der Nenner eines Bruches nie 0 wird. Auch hier ein Beispiel: Bei der Gleichung \frac{1}{x}=13 muss x=0 aus dem Definitionsbereich ausgeschlossen werden. Man schreibt: \mathbb{D}=\mathbb{R}\setminus_{\{0\}}.

Bei diesen Aufgaben sind die Beschränkungen des Definitionsbereichs alle relativ offensichtlich. Das liegt vor allem daran, dass die Beispiele recht einfach gewählt sind, um überhaupt erstmal das Prinzip deutlich zu machen. In späteren Kapiteln (insbesondere bei Wurzel- und Logarithmusgleichungen) werden Sie weitere Einschränkungen kennenlernen. So als Faustregel: Je komplexer die Gleichung wird, desto relevanter wird dabei auch der Definitionsbereich.


Die Menge aller Lösungen einer Gleichung heißt Lösungsmenge (Formelzeichen: \mathbb{L}).
Nach dem Lösen einer Gleichung muss immer noch geprüft werden, ob der berechnete Wert im Definitionsbereich liegt (vielleicht kommt er ja als Lösung gar nicht infrage ...). Daher ist die Zahl, die in der letzten Zeile steht, nicht automatisch eine Lösung der Gleichung! Nach dem Abgleich mit dem Definitionsbereich sollte die Lösung einer Gleichung dann immer in Form einer Lösungsmenge angegeben werden.
Zu den Beispielen von oben:
Schokoriegel: \mathbb{L}_S=\{0{,}90\}
Gummibärchen: \mathbb{L}_G=\{4\}

 

Der ungefähre Plan zum Lösen einer Gleichung lautet also:

  1. Klammern auflösen
  2. auf beiden Seiten der Gleichung zusammenfassen (alle Zahlenwerte zusammenrechnen, alle Variablen zusammenrechnen)
  3. "sortieren": Mithilfe der Addition alle Terme, die die Variable enthalten, auf die eine Seite der Gleichung bringen; alles ohne Variable auf die andere Seite.
    Achtung: Die Variable muss nicht x heißen!
  4. durch den Koeffizienten der Variable teilen
  5. prüfen, ob die als Lösung ermittelte Zahl im Definitionsbereich liegt

Lösungsmenge aufschreiben nicht vergessen!

 

Überprüfen der gefundenen Lösungen

Um zu prüfen, ob die gefundene Zahl tatsächlich eine Lösung der Gleichung ist, kann man die Probe machen, d. h. man setzt den gefundenen Wert in die Ausgangsgleichung ein und prüft, ob beide Seiten denselben Wert ergeben. Ist das nicht der Fall, sollte man noch mal nachrechnen ...

Betrachten wir noch einmal das Beispiel von oben: -\left(4x-9\right)-2\left(-\dfrac{x}{2}+1\right)+7 = -11x-6+3x mit der Lösung x=-4
Setzt man für jedes x in der Gleichung -4 ein, erhält man
\begin{array}{rcl}-\left(4\cdot (-4)-9\right)-2\left(-\dfrac{-4}{2}+1\right)+7 &=& -11\cdot (-4)-6+3\cdot (-4) \cr -\left(-16-9\right)-2\left(-(-2)+1\right)+7 &=& 44-6+(-12) \cr -(-25)-2\cdot 3+7 &=& 44-6-12 \cr 25-6+7 &=& 26 \cr 26 &=& 26 \end{array}

Stimmt! Wobei man vielleicht dazu sagen sollte, dass diese Probe hier sehr ausführlich aufgeschrieben ist ... Nicht jeder dieser Schritte muss wirklich dringend notiert werden.

Hier kann man auch nochmal gut sehen, warum es wirklich keine gute Idee ist, durch 0 zu dividieren: Wenn die Division durch 0 irgendein Ergebnis x ergäbe, also z. B. 10:0 = x, dann müsste es möglich sein, die Probe zu machen. Das wäre in diesem Fall 0\cdot x = 10. Das Produkt von 0 mit irgendeiner reellen Zahl ist aber immer 0. 0\cdot x kann also nie 10 werden - egal, für welche Zahl x steht.

 

Lösbarkeit linearer Gleichungen

Zum Abschluss noch die Frage: Ist jede lineare Gleichung lösbar? Anders formuliert: Lässt sich für jede lineare Gleichung (mit dem oben gezeigten Weg) eine Zahl finden, sodass beide Seiten gleich groß werden?

Schauen wir uns dafür beispielhaft die folgenden drei Gleichungen an:

-2x+1 = -2x+2-1
Diese lineare Gleichung ist mehrdeutig lösbar. D. h., es gibt mehr als eine Zahl, die diese Gleichung löst. Um genau zu sein, gibt es sogar unendlich viele Lösungen. Rechnet man nämlich die Zahlen auf der rechten Seite zusammen, erhält man -2x+1 = -2x+1 . Hier sieht man: Egal, welche Zahl man für x einsetzt, man erhält immer auf beiden Seiten das gleiche Ergebnis. Was sollte auch sonst passieren, wenn die beiden Seiten der Gleichung identisch sind? Jedes Element des Definitionsbereiches ist also Lösung der Gleichung.
Man notiert die Lösungsmenge: \mathbb{L} = \mathbb{D}. Konkret ist die Lösungsmenge dieser Aufgabe \mathbb{L} = \mathbb{R}; bei eingeschränkten Definitionsbereichen natürlich entsprechend diese Mengen.


3x-1 = 5
Diese lineare Gleichung ist eindeutig lösbar. D. h., es gibt genau eine Zahl, die diese Gleichung löst. Es ist also der gleiche Fall wie in dem Beispiel oben. Formt man sie um, indem man erst 1 addiert und anschließend durch 3 teilt, erhält man nämlich x=2 . Dies ist die einzige Lösung, denn keine andere Zahl ergibt 5, wenn man sie mit 3 multipliziert und anschließend 1 subtrahiert.
Man notiert die Lösungsmenge: \mathbb{L} = \{2\}


4x = 4x+1
Die lineare Gleichung ist nicht lösbar. D. h., es gibt keine Zahl, die diese Gleichung löst. Zieht man auf beiden Seiten 4x ab, erhält man 0 = 1 . Das ist ein Widerspruch. Anders gesagt: Es gibt keine Zahl, die genauso groß ist wie ihr Nachfolger.
Man notiert die Lösungsmenge: \mathbb{L} = \emptyset


Zusammenfassung: Lineare Gleichungen können nicht lösbar (d. h. es gibt keine Lösungen), eindeutig lösbar (d. h. es gibt genau eine Lösung) oder mehrdeutig lösbar mit unendlich vielen Lösungen sein. Andere Möglichkeiten gibt es nicht.

Übersicht:

 

5.3 Lineare Gleichungen - Lösungen

1. Aufgabe

Erste Bemerkung vorab: Malpunkte zwischen den Variablen dürfen bei allen Aufgaben auch weggelassen werden. Das ist eigentlich die übliche Schreibweise. In diesem Kapitel wurden sie nur hingeschrieben, um deutlich zu machen, wo genau hier multipliziert wird.

Zweite Bemerkung vorab: Statt x\cdot x dürfen Sie natürlich auch x^2 und statt x\cdot x\cdot x auch x^3 schreiben. Hier wurde das nur deswegen nicht gemacht, weil die Potenzschreibweise "offiziell" erst in Kapitel 8 eingeführt wird ...

 

1)
\begin{array}{rclcl} -3(-4x \cdot x+2x) &=& -3 \cdot (-4x \cdot x)-3 \cdot 2x &=& 12x \cdot x-6x \end{array}

Vorgehen: Klammern auflösen

Bemerkung: Auf das Minuszeichen vor der Klammer achten!

 
2)
\begin{array}{rcl} 4x \cdot x-3x \cdot y+11x \cdot x+16x-40y+3y \cdot x &=& 4x \cdot x+11x \cdot x-3x \cdot y+3x \cdot y+16x-40y \cr &=& 15x \cdot x+16x-40y \end{array}

Vorgehen: zusammenfassen
 
Bemerkung 1: Gleiche Produkte von Variablen dürfen addiert und subtrahiert werden. Dabei ändert sich nur der Koeffizient.
 
Bemerkung 2: Die Reihenfolge der Variablen in einem Produkt ist egal. Üblich ist, die Variablen in alphabetischer Reihenfolge aufzuschreiben, weil das die Übersicht erleichtert.

 
3)
\begin{array}{rcl} 60a \cdot a \cdot a \cdot b \cdot c \cdot c +10a \cdot b \cdot b \cdot b \cdot c-30a \cdot b \cdot c \cdot c \cdot c &=& 10a \cdot b \cdot c(6a \cdot a \cdot c+b \cdot b-3c \cdot c) \end{array}

Vorgehen: ausklammern

 
4)
\begin{array}{rcl} 6-(8x-4y)+2(10y-7)+12-18x &=& 6-8x+4y+20y-14+12-18x \cr &=& -26x+24y+4 \end{array}

Vorgehen: Klammern auflösen, anschließend zusammenfassen

 
5)
\begin{array}{rcl} -(2a+d)(-2d+a) &=& -(-4a \cdot d+2a \cdot a-2d \cdot d+a \cdot d) \cr &=& -(-3a \cdot d+2a \cdot a-2d \cdot d) \cr &=& -2a \cdot a+3a \cdot d+2d \cdot d \end{array}

Vorgehen: Klammern auflösen
 
Bemerkung: Auf das Minuszeichen vor den Klammern achten!

 
6)
\begin{array}{rcl} \dfrac{4}{5}t-\dfrac{t}{10}+\dfrac{7}{15}t &=& \dfrac{24}{30}t-\dfrac{3}{30}t+\dfrac{14}{30}t \cr \cr &=& \dfrac{24-3+14}{30}t \cr \cr &=& \dfrac{35}{30}t \cr \cr &=& \dfrac{7}{6}t \end{array}

Vorgehen: Brüche gleichnamig machen und addieren, anschließend kürzen
 
Bemerkung 1: Ob die Variable auf oder hinter dem Bruchstrich steht, ist egal.
 
Bemerkung 2: Da die Variablen alle nur mit Zahlenwerten (und nicht mit weiteren Variablen) multipliziert werden, dürfen diese Koeffizienten einfach addiert bzw. subtrahiert werden.

 
7)
\begin {array}{rcl} \dfrac{3x}{8}-\dfrac{5}{12}x-\dfrac{4}{x} &=& \dfrac{9x \cdot x}{24x}-\dfrac{10x \cdot x}{24x}-\dfrac {96}{24x} \cr \cr &=& \dfrac{9x \cdot x-10x \cdot x-96}{24x} \cr \cr &=& \dfrac{-x \cdot x-96}{24x} \end{array}

Vorgehen: Brüche gleichnamig machen und addieren
 
Bemerkung: Die Variable muss im Hauptnenner berücksichtigt werden, da sie beim dritten Bruch im Nenner steht.

 
8)
\begin{array}{rcl} \dfrac{2a}{b}+\dfrac{a}{m}+\dfrac{a}{b} &=& \dfrac{2a}{b}+\dfrac{a}{b}+ \dfrac{a}{m} \cr \cr &=& \dfrac{3a}{b}+\dfrac{a}{m} \cr \cr &=& \dfrac{3am}{bm}+\dfrac{ab}{bm} \cr \cr &=& \dfrac{3am+ab}{bm} \cr \cr &=& \dfrac{a(3m+b)}{bm} \end{array}

Vorgehen: Brüche gleichnamig machen und addieren, anschließend ausklammern
 
Bemerkung: Den ersten und den dritten Bruch kann man sofort addieren, weil sie bereits den gleichen Nenner haben.

 
9)
\begin{array} {rcl} \dfrac{1}{x+1}-\dfrac{1}{x-1} &=& \dfrac{1\cdot(x-1)}{(x+1)(x-1)}-\dfrac{1\cdot(x+1)}{(x-1)(x+1)} \cr \cr &=& \dfrac{(x-1)-(x+1)}{(x-1)(x+1)} \cr \cr &=& \dfrac{x-1-x-1}{(x-1)(x+1)} \cr \cr &=& \dfrac{- 2}{(x-1)(x+1)} \end{array}

Vorgehen: Brüche gleichnamig machen und subtrahieren

Bemerkung 1: Auf das Minuszeichen zwischen den Klammern im Zähler achten! Hier müssen unbedingt Klammern gesetzt werden, da der gesamte Zähler des zweiten Bruches subtrahiert werden muss.

Bemerkung 2: Man könnte im Nenner noch die Klammern auflösen.

 
10)
\begin{array}{rcl} \dfrac{y+2}{y-2}-\dfrac{2y- 2}{y+4} &=& \dfrac{(y+2)(y+4)}{(y-2)(y+4)}-\dfrac{(2y-2)(y-2)}{(y+4)(y-2)} \cr \cr &=& \dfrac{y \cdot y +6y+8}{(y-2)(y+4)}-\dfrac{2y \cdot y-6y+4}{(y-2)(y+4)} \cr \cr &=& \dfrac{y \cdot y+6y+8-(2y \cdot y-6y+4)}{(y- 2)(y+4)} \cr \cr &=& \dfrac{-y \cdot y+12y+4}{(y-2)(y+4)} \end{array}

Vorgehen: Brüche gleichnamig machen und subtrahieren
 
Bemerkung 1: Beim Zusammenfassen der Brüche müssen Klammern um den Zähler des zweiten Bruches gesetzt werden, da sich das Minuszeichen sonst nicht auf den gesamten Zähler auswirkt.
 
Bemerkung 2: Man könnte im Nenner noch die Klammern auflösen.

 
11)
\begin{array}{rcl} \dfrac{2}{3a-9} + \dfrac{b}{-3+a} &=& \dfrac{2}{3\left(a-3\right)} + \dfrac{b}{a-3} \cr \cr &=& \dfrac{2}{3\left(a-3\right)} + \dfrac{3b}{3\left(a-3\right)} \cr \cr &=& \dfrac{2+3b}{3\left(a-3\right)}\end{array}

Vorgehen: Brüche gleichnamig machen und addieren

Bemerkung: -3+a ist das Gleiche wie a-3

 
12)
\begin{array}{rclcl} \dfrac{4x}{5} \cdot \dfrac{2}{3x} &=& \dfrac{8x} {15x} &=& \dfrac{8}{15} \end{array}

Vorgehen: Brüche multiplizieren ("Zähler mal Zähler, Nenner mal Nenner"), anschließend kürzen
 
Bemerkung: x muss bei dieser Aufgabe ungleich 0 sein, da durch 0 nicht geteilt werden darf.

 
13)
\begin{array}{rclcl} \dfrac{2a}{5} : \dfrac{a}{10} &=& \dfrac{2a}{5} \cdot \dfrac{10}{a} &=& 4 \end{array}

Vorgehen: Brüche dividieren (mit dem Kehrwert des zweiten Bruches multiplizieren), anschließend kürzen

Bemerkung: a muss bei dieser Aufgabe ungleich 0 sein, da durch 0 nicht geteilt werden darf.

 
14)
\begin{array}{rcl} \dfrac{8}{z} + \left(\dfrac{z} {2}-12\right) \left(1+\dfrac{4}{z}\right) &=& \dfrac{8}{z} + \dfrac{z}{2} \cdot 1 + \dfrac{z}{2} \cdot \dfrac{4}{z} - 12 \cdot 1 -12 \cdot \dfrac{4}{z} \cr \cr &=& \dfrac{8}{z} + \dfrac{z}{2} + 2 - 12 - \dfrac{48}{z} \cr \cr &=& \dfrac{z}{2} - 10 - \dfrac{40}{z} \end{array}

Vorgehen: Klammer auflösen nach dem Distributivgesetz, anschließend zusammenfassen

 
15)
\begin{array}{rcl} \dfrac{n-2}{n\cdot n+5n} : \dfrac{3n-6}{10n} &=& \dfrac{n-2}{n\cdot n+5n} \cdot \dfrac{10n}{3n-6} \cr \cr &=& \dfrac{n-2}{n\left(n+5\right)} \cdot \dfrac{10n} {3\left(n-2\right)} \cr \cr &=& \dfrac{1}{n+5} \cdot \dfrac{10}{3} \cr \cr &=& \dfrac{10}{3\left(n +5\right)} \end{array}

Vorgehen: Brüche dividieren (mit dem Kehrwert des zweiten Bruches multiplizieren), anschließend kürzen und zusammenfassen

Bemerkung 1: Aus der Summe n+5 darf nicht gekürzt werden.

Bemerkung 2: Der Term n-2 als Ganzes darf gekürzt werden. Bitte darauf achten, dass nach dem Kürzen im Zähler des ersten Bruches eine 1 stehen bleibt.

 
16)
\begin{array}{rcl} 6yz-\{12+7[yz +16-3x(15+8y)]\}+13x &=&6yz-\{12+7[yz+16-3x\cdot 15+(-3x)\cdot 8y]\}+13x \cr\cr &=&6yz-\{12+7[yz+16-45x-24xy]\}+13x \cr\cr &=&6yz-\{12+7\cdot yz+7\cdot 16+7\cdot(-45x)+7\cdot (-24xy)\}+13x \cr\cr &=&6yz-\{12+7yz+112-315x-168xy\}+13x \cr\cr &=&6yz-12-7yz-112+315x+168xy+13x \cr\cr &=&-yz-124+328x+168xy \cr\cr &=&168xy-yz+328x-124 \end{array}

Vorgehen: Klammern nach dem Distributivgesetz von innen nach außen auflösen (erst die runden Klammern, dann die eckigen und zum Schluss die geschweiften) und anschließend zusammenfassen

 
17)
\begin{array}{rcl} \{8x[2+(13-5x)\cdot9x]-5\}\cdot4 &=&\{8x[2+13\cdot9x-5x\cdot9x]-5\}\cdot4 \cr\cr &=&\{8x[2+117x-45x\cdot x]-5\}\cdot4 \cr\cr &=&\{8x\cdot 2+8x\cdot 117x+8x\cdot (-45x\cdot x)-5\}\cdot4 \cr\cr &=&\{16x+936x\cdot x-360x\cdot x\cdot x-5\}\cdot4 \cr\cr &=&16x\cdot4+936x\cdot x\cdot4-360x\cdot x\cdot x\cdot4-5\cdot4 \cr\cr &=&64x+3744x\cdot x-1440x\cdot x\cdot x-20 \cr\cr &=&-1440x\cdot x\cdot x+3744x\cdot x+64x-20 \end{array}

Vorgehen: Klammern nach dem Distributivgesetz von innen nach außen auflösen (erst die runden Klammern, dann die eckigen und zum Schluss die geschweiften) und anschließend zusammenfassen

 
18)
\begin{array}{rcl} \dfrac{1}{2}[6(12+4ab)\cdot3c-b(10a+20c)]+19 &=&\dfrac{1}{2}[(6\cdot12+6\cdot4ab)\cdot3c-10ab-20bc]+19 \cr\cr &=&\dfrac{1}{2}[(72+24ab)\cdot3c-10ab-20bc]+19 \cr\cr &=&\dfrac{1}{2}[72\cdot3c+24ab\cdot3c-10ab-20bc]+19 \cr\cr &=&\dfrac{1}{2}[216c+72abc-10ab-20bc]+19 \cr\cr &=&\dfrac{1}{2}\cdot216c+\dfrac{1}{2}\cdot72abc+\dfrac{1}{2}\cdot(-10ab)+\dfrac{1}{2}\cdot(-20bc)+19 \cr\cr &=&36abc-5ab-10bc+108c+19 \end{array}

Vorgehen: Klammern nach dem Distributivgesetz von innen nach außen auflösen (erst die beiden runden Klammern und dann die eckigen) und anschließend zusammenfassen

 
19)
\begin{array}{rcl} (x-15)[-u(2x+14ux-7)+16u-24xu] &=&(x-15)[-u\cdot2x-u\cdot14ux-u\cdot(-7)+16u-24ux] \cr\cr &=&(x-15)[-2ux-14uux+7u+16u-24ux] \cr\cr &=&(x-15)[-14uux-26ux+23u] \cr\cr &=&x\cdot(-14uux)+x\cdot(-26ux)+x\cdot23u-15\cdot(-14uux)-15\cdot(-26ux)-15\cdot23u \cr\cr &=&-14uuxx-26uxx+23ux+210uux+390ux-345u \cr\cr &=&-14uuxx+210uux-26uxx+413ux-345u \end{array}

Vorgehen: erst innerhalb der eckigen Klammern "aufräumen" (runde Klammern nach dem Distributivgesetz auflösen, zusammenfassen, sortieren), dann runde Klammer von ganz vorne mit der eckigen Klammer mithilfe des Distributivgesetzes auflösen und anschließend zusammenfassen
 
Bemerkung: Innerhalb der einzelnen Produkte sollten die Variablen in alphabetischer Reihenfolge geschrieben werden. Dann sieht man nämlich beispielsweise besser, dass -2ux und -24ux zusammengefasst werden können. Bei den Produkten selbst werden die mit den "meisten" Variablen zuerst geschrieben.

 
20)
\begin{array}{rcl} \dfrac{2}{3}[21+(11+18y)(-x-3)-2x\{9xy-5x+12\}-27x] &=&\dfrac{2}{3}[21+11\cdot(-x)+11\cdot(-3)+18y\cdot(-x)+18y\cdot(-3)-2x\cdot9xy-2x\cdot(-5x)-2x\cdot12-27x] \cr\cr &=&\dfrac{2}{3}[21-11x-33-18xy-54y-18xxy+10xx-24x-27x] \cr\cr &=&\dfrac{2}{3}[-18xxy+10xx-18xy-62x-54y-12] \cr\cr &=&\dfrac{2}{3}\cdot(-18xxy)+\dfrac{2}{3}\cdot10xx+\dfrac{2}{3}\cdot(-18xy)+\dfrac{2}{3}\cdot(-62x)+\dfrac{2}{3}\cdot(-54y)+\dfrac{2}{3}\cdot(-12) \cr\cr &=&-12xxy+\dfrac{20}{3}xx-12xy-\dfrac{124}{3}x-36y-8 \end{array}

Vorgehen: Klammern nach dem Distributivgesetz von innen nach außen auflösen (erst die beiden runden sowie die geschweiften Klammern und dann die eckigen) und anschließend zusammenfassen

 

2. Aufgabe

Wichtig: Treffen Punkt- und Strichrechnung aufeinander, müssen Klammern gesetzt werden!

1)
\begin{array}{cclll} a &=& 2b+c & \vert & \text{Setze b ein} \cr a &=& 2(3+c)+c & \vert & \text{Setze c ein} \cr a &=& 2(3+1)+1 \cr a &=& 2 \cdot 4+1 \cr a &=& 9 \end{array}


2)
\begin{array}{cclll} 2s &=& -4r-t+12 & \vert & \text{Setze r ein} \cr 2s &=& -4(3t-16)-t+12 & \vert & \text {Setze t ein} \cr 2s &=& -4(3 \cdot 24-16)-24+12 \cr 2s &=& -4 \cdot 56-24+12 \cr 2s &=& -224 -24+12 \cr 2s &=& -236 & \vert & :2 \cr s &=& -118 \end{array}


3)
\begin {array}{cclll} 2a &=& 18z+2 & \vert & :2 \cr a &=& 9z+1 \cr \cr x &=& a-10-4z & \vert & \text{Setze a ein} \cr x &=& 9z+1-10-4z & \vert & \text{Setze z ein} \cr x &=& 9 \cdot (-5)+1-10-4 \cdot (-5) \cr x &=& -45+1-10+20 \cr x &=& -44-10+20 \cr x &=& -34 \end {array}

Bemerkung: Um x zu berechnen, muss in der entsprechenden Zeile a eingesetzt werden. Es ist aber kein Term für a gegeben, sondern nur einer für 2a. Daher muss diese Zeile zunächst umgeformt werden.

 
4)
\begin{array}{cclll} 2l &=& 2z-14 & \vert & :2 \cr l &=& z-7 \cr \cr 12b &=& 3l+ \dfrac{1}{3}z+11 & \vert & \text{Setze l ein} \cr 12b &=& 3(z-7)+\dfrac{1}{3}z+11 & \vert & \text{Setze z ein} \cr 12b &=& 3(-9-7)+ \dfrac{1}{3} \cdot (-9)+11 \cr 12b &=& 3 \cdot (-16)-3+11 \cr 12b &=& -48-3+11 \cr 12b &=& -40 & \vert & :12 \cr b &=& -\dfrac{40}{12} = - \dfrac{10} {3} \end{array}


5)
\begin{array}{cclll} 2v &=& 16-8w & \vert & :2 \cr v &=& 8-4w \cr \cr -3w &=& 27 & \vert &:(-3) \cr w &=& -9 \cr \cr \dfrac{1}{5}u &=& 100-25v-8w & \vert & \text{Setze v ein} \cr \dfrac{1}{5}u &=& 100-25(8-4w)-8w & \vert & \text{Setze w ein} \cr \dfrac{1}{5}u &=& 100-25(8-4 \cdot(-9))-8 \cdot (-9) \cr \dfrac{1}{5}u &=& 100-25(8+36)+72 \cr \dfrac{1}{5}u &=& 100-25 \cdot 44+72 \cr \dfrac{1}{5}u &=& 100-1100+72 \cr \dfrac{1} {5}u &=& -928 & \vert & \cdot 5 \cr u &=& -4640 \end{array}

 

3. Aufgabe

1) Es handelt sich um eine lineare Gleichung.

Bemerkung: Ob die Variable x auf oder hinter dem Bruchstrich steht, ist egal. Dies sind nur zwei verschiedene Schreibweisen, die das Gleiche meinen.

 
2) Es handelt sich nicht um eine lineare Gleichung, da auf der linken Seite der Kosinus auf die Variable angewendet wird. Außerdem entsteht beim Ausmultiplizieren der Klammern auf der rechten Seite x\cdot x = x^2 .


3) Es handelt sich nicht um eine lineare Gleichung, da die Variable x auch quadriert auftritt.


4) Es handelt sich nicht um eine lineare Gleichung, da die Variable x im Nenner eines Bruches auftritt.


5) Es handelt sich um eine lineare Gleichung.


6) Es handelt sich um eine lineare Gleichung.

Bemerkung: Durch das Wurzelzeichen sieht die Gleichung zwar auf den ersten Blick nicht linear aus. Da unter der Wurzel aber "nur" eine Zahl steht, kann die Wurzel einfach berechnet werden: \sqrt{4}=2 , also steht auf der linken Seite der Gleichung nichts anderes als 1+2x.


7) Es handelt sich nicht um eine lineare Gleichung, da nach dem Ausmultiplizieren der Klammern die Variable x auch quadriert auftritt.


8) Es handelt sich um eine lineare Gleichung.


9) Es handelt sich um eine lineare Gleichung.

Bemerkung: Die Wurzel im Nenner kann einfach berechnet werden, weil sie "nur" eine Zahl enthält. Also steht auf der linken Seite \dfrac{x}{8}. Das ist gleichbedeutend mit \dfrac{1}{8} \cdot x. Also, alles in Ordnung.


10) Es handelt sich nicht um eine lineare Gleichung, da die Variable x im Nenner eines Bruches auftritt.

 

4. Aufgabe

1)
\begin{array}{rclll} 2x - 4 \, &=& \, -4x - 1 & \vert & +4x+4 \cr 2x+4x &=& -1+4 \cr 6x &=& 3 & \vert & :6 \cr x &=& \dfrac{1}{2} \end{array}
 
Probe:
\begin{array}{rcl} 2 \cdot \dfrac{1}{2} - 4 \, &=& \, -4 \cdot \dfrac{1}{2} - 1 \cr 1-4 &=& -2-1 \cr -3 &=& -3 \end{array}

Für x = \dfrac{1}{2} ergibt sich eine wahre Aussage: \mathbb{L} = \left\{\dfrac{1}{2} \right\}


2)
\begin{array}{rclll} 3 (2x-1) &=& -5 (17+7x) \cr 6x-3 &=& -85-35x & \vert & -6x+85 \cr -3+85 &=& -35x-6x \cr 82 &=& -41x & \vert & :(-41) \cr -2 &=& x \end{array}

Probe:
\begin {array}{rcl} 3 \cdot (2 \cdot \left(-2\right)-1) &=& -5\cdot (17+7 \cdot \left(-2\right)) \cr 3 \cdot (-4-1) &=& -5 \cdot (17-14) \cr 3 \cdot (-5) &=& -5 \cdot 3 \cr -15 &=& -15 \end{array}

Für x = -2 ergibt sich eine wahre Aussage: \mathbb{L} = \left\{-2 \right\}


3)
\begin{array}{rclll} 6(4x-8) &=& (-12x+24) \cdot (-2) \cr 24x-48 &=& 24x-48 & \vert & -24x+48 \cr 24x-24x &=& - 48+48 \cr 0&=& 0 \cr \cr \mathbb{L} &=& \mathbb{R} \end{array}

Bemerkung: Unabhängig davon, welches Element des Definitionsbereichs in diese Gleichung eingesetzt wird, erhält man immer auf beiden Seiten dasselbe Ergebnis. 0=0 ist schließlich immer richtig. Jede reelle Zahl löst also diese Gleichung, d. h. die Lösungsmenge entspricht dem Definitionsbereich.


4)
\begin{array}{rclll} 3x \, (-4x-10) &=& (2-2x) \cdot (6x+15) \cr -12x^2-30x &=& 12x+30-12x^2-30x & \vert & +12x^2+30x-12x \cr -12x^2+12x^2-30x+30x-12x &=& 30 \cr -12x &=& 30 & \vert & :(-12)\cr x &=& -\dfrac{30}{12} = -\dfrac{5}{2} \end{array}

Probe:
\begin{array}{rcl} 3 \cdot \left(-\dfrac{5}{2}\right) \cdot \, \left(-4 \cdot \left(-\dfrac{5}{2}\right)-10\right) &=& \left(2-2 \, \cdot \left(-\dfrac{5}{2}\right)\right) \cdot \left(6 \cdot \left(-\dfrac{5}{2}\right)+15\right) \cr -\dfrac{15}{2} \cdot (10-10) &=& (2+5) \cdot (-15+15) \cr 0 &=& 0 \end{array}

Für x = -\dfrac{5}{2} ergibt sich eine wahre Aussage: \mathbb{L} = \left\{-\dfrac{5}{2} \right\}

5)
\begin{array}{rclll} 4 \, (4a-1) &=& 8 \, \left(\dfrac{1}{2}+2a\right) \cr 16a-4 &=& 4+16a & \vert & - 16a+4 \cr 16a-16a &=& 4+4 \cr 0 &=& 8 \cr \cr \mathbb{L} &=& \emptyset \end{array}

Bemerkung: Beim Umformen der Gleichung entsteht ein Widerspruch: 0=8 kann nie stimmen. Daher ist diese Gleichung nicht lösbar: \mathbb{L} = \emptyset

 
6)
\begin{array}{rclll} -3 \, (6y+2) &=& 12y-5 \cr -18y-6 &=& 12y-5 & \vert & -12y+6 \cr -18y-12y &=& -5+6 \cr -30y &=& 1 & \vert & :(-30) \cr y &=& -\dfrac{1}{30} \not\in\mathbb{D} \cr \cr \mathbb{L} &=& \emptyset \end{array}

Bemerkung: Als Definitionsbereich ist hier die Menge der natürlichen Zahlen (\mathbb{D} = \mathbb{N}) gegeben. -\frac{1}{30} ist aber nun mal keine natürliche Zahl (-\frac{1}{30} \not\in\mathbb{N}). Also kann für diese Gleichung keine Zahl gefunden werden, die beide Seiten gleich groß werden lässt und im Definitionsbereich liegt.


7)
\begin{array}{rclll} 2 \, (3x-1) &=& 3(2+5x)+1 \cr 6x-2 &=& 6+15x+1 \cr 6x-2 &=& 15x+7 & \vert & -15x+2 \cr 6x-15x &=& 7+2 \cr -9x &=& 9 & \vert & :(-9) \cr x &=& -1 \end{array}

Probe:
\begin{array}{rclll} 2\cdot (3\cdot (-1)-1) &=& 3\cdot (2+5\cdot (-1))+1 \cr 2\cdot (-4) &=& 3\cdot(-3)+1 \cr -8 &=& -8 \end{array}

Für x = -1 ergibt sich eine wahre Aussage: \mathbb{L} = \left\{-1 \right\}


8)
\begin{array}{rclll} \dfrac{1}{3} (-3y+6) &=& \dfrac{3}{2} (6y-2) \cr -y+2 &=& 9y-3 & \vert & -9y-2 \cr -y-9y &=& -3-2 \cr -10y &=& -5 & \vert & :(-10) \cr y &=& \dfrac{1}{2} \end{array}

Probe:
\begin{array}{rclll} \dfrac{1}{3}\cdot\left(-3\cdot \dfrac{1}{2}+6\right) &=& \dfrac{3}{2}\cdot\left(6\cdot\dfrac{1}{2}-2\right) \cr\cr \dfrac{1}{3}\cdot\dfrac{9}{2} &=& \dfrac{3}{2}\cdot 1 \cr\cr \dfrac{3}{2} &=& \dfrac{3}{2} \end{array}

Für x = \dfrac{1}{2} ergibt sich eine wahre Aussage: \mathbb{L} = \left\{\dfrac{1}{2} \right\}


9)
\begin{array}{rclll} 6 \, (2x-4)-7 &=& 9-8\left(\dfrac{1}{4}x+5\right) \cr 12x-24-7 &=& 9-2x-40 \cr 12x-31 &=& -2x-31 & \vert & +2x+31 \cr 12x+2x &=& -31+31 \cr 14x &=& 0 & \vert & :14 \cr x &=& 0 \end{array}

Probe:
\begin{array}{rclll} 6\cdot (2\cdot 0 -4)-7 &=& 9-8\cdot \left(\dfrac{1}{4}\cdot0+5\right) \cr 6\cdot(-4)-7 &=& 9-8\cdot 5 \cr -24-7 &=& 9-40 \cr\cr -31 &=& -31 \end{array}

Für x = 0 ergibt sich eine wahre Aussage: \mathbb{L} = \left\{0 \right\}


10)
\begin{array}{rclll} 4 \left(\dfrac{x}{2}-\dfrac{1}{3}\right)+5\left(\dfrac{x}{5}-\dfrac{1}{3}\right) &=& 0 \cr 2x-\dfrac{4}{3}+x-\dfrac{5}{3} &=& 0 \cr 3x-3 &=& 0 & \vert & +3 \cr 3x &=& 3 & \vert & :3 \cr x &=& 1 \end{array}

Probe:
\begin{array}{rclll} 4\cdot \left(\dfrac{1}{2}-\dfrac{1}{3}\right)+5\left(\dfrac{1}{5}-\dfrac{1}{3}\right) &=& 0 \cr\cr 4\cdot\dfrac{1}{6}+5\cdot\left(-\dfrac{2}{15}\right) &=& 0 \cr\cr \dfrac{2}{3}-\dfrac{2}{3} &=& 0 \cr\cr 0 &=& 0 \end{array}

Für x = 1 ergibt sich eine wahre Aussage: \mathbb{L} = \left\{1 \right\}


11)
\begin{array}{rclll} 13m+m+30-23m-45+17+12m &=& -31 \cr 3m+2 &=& -31 & \vert & -2 \cr 3m &=& -31-2 \cr 3m &=& -33 & \vert & :3 \cr m &=& -11 \end{array}

Probe:
\begin{array}{rclll} 13\cdot(-11)+(-11)+30-23\cdot(-11)-45+17+12\cdot(-11) &=& -31 \cr -143-11+30+253-45+17-132 &=& -31 \cr -31 &=& -31 \end{array}

Für x = -11 ergibt sich eine wahre Aussage: \mathbb{L} = \left\{-11 \right\}


12)
\begin{array}{rclll} \dfrac{x+3}{2} &=& \dfrac{x+3}{4}&\vert&\cdot4 \cr 2(x+3)&=&x+3 \cr 2x+6&=&x+3&\vert&-6-x \cr 2x-x&=&3-6 \cr x&=&-3\end{array}

Probe:
\begin{array}{rclll} \dfrac{-3+3}{2} &=& \dfrac{-3+3}{4} \cr\cr \dfrac{0}{2} &=& \dfrac{0}{4} \cr\cr 0 &=& 0 \end{array}

Für x = -3 ergibt sich eine wahre Aussage: \mathbb{L} = \left\{-3 \right\}


13)
\begin{array}{rclll}5\left(x+\dfrac{18}{25}\right) &=& 3\left(x-\dfrac{4}{5}\right) \cr 5x+\dfrac{18}{5} &=& 3x-\dfrac{12}{5} & \vert & -\dfrac{18}{5}-3x \cr 5x-3x &=& -\dfrac{12}{5}-\dfrac{18}{5} \cr 2x &=& -6 & \vert & : 2 \cr x &=& -3 \not\in\mathbb{D} \cr\cr \mathbb{L}&=&\emptyset \end{array}

Bemerkung: Als Definitionsbereich ist hier die Menge der positiven reellen Zahlen (\mathbb{D} = \mathbb{R}^+) gegeben. -3 ist nun mal negativ. Also kann für diese Gleichung keine Zahl gefunden werden, die beide Seiten gleich groß werden lässt und im Definitionsbereich liegt.


14)
\begin{array}{rclll}0{,}2a-0{,}3-0{,}5a+3{,}1a+0{,}7a &=& 1{,}9a+5-0{,}4a+2{,}7 \cr 3{,}5a-0{,}3 &=& 1{,}5a+7{,}7 & \vert & +0{,}3-1{,}5a \cr 3{,}5a-1{,}5a &=& 7{,}7+0{,}3 \cr 2a &=& 8 & \vert & : 2 \cr a &=& 4 \end{array}

Probe:
\begin{array}{rclll} 0{,}2\cdot 4-0{,}3-0{,}5\cdot 4+3{,}1\cdot 4+0{,}7\cdot 4 &=& 1{,}9\cdot 4+5-0{,}4\cdot 4+0{,}5+2{,}2 \cr 0{,}8-0{,}3-2+12{,}4+2{,}8 &=& 7{,}6+5-1{,}6+0{,}5+2{,}2 \cr 13{,}7 &=& 13{,}7 \end{array}

Für x = 4 ergibt sich eine wahre Aussage: \mathbb{L} = \left\{4 \right\}


15)
\begin{array}{rclll}\dfrac{-12x+24}{6} &=& 2(3x-38) \cr \dfrac{1}{6}(-12x+24) &=& 2(3x-38) \cr -2x+4 &=& 6x-76 & \vert & -4-6x \cr -2x-6x &=& -76-4 \cr -8x &=& -80 &\vert & : \left(-8\right) \cr x &=& 10 \end{array}

Probe:
\begin{array}{rclll} \dfrac{-12\cdot 10+24}{6} &=& 2\cdot(3\cdot 10-38) \cr \dfrac{-96}{6} &=& 2\cdot (-8) \cr -16 &=& -16 \end{array}

Für x = 10 ergibt sich eine wahre Aussage: \mathbb{L} = \left\{10 \right\}


16)
\begin{array}{rclll}\dfrac{3x}{5}-\dfrac{4}{5}x+2x &=& x+4 \cr\cr \dfrac{3}{5}x-\dfrac{4}{5}x+\dfrac{10}{5}x &=& x+4 \cr\cr \dfrac{9}{5}x &=& x+4 & \vert & -x \cr\cr \dfrac{9}{5}x-\dfrac{5}{5}x &=& 4 \cr\cr \dfrac{4}{5}x &=& 4 &\vert & \cdot \dfrac{5}{4} \cr\cr x &=& 5 \end{array}

Probe:
\begin{array}{rclll} \dfrac{3\cdot 5}{5}-\dfrac{4}{5}\cdot 5 +2\cdot 5 &=& 5+4 \cr 3-4+10 &=& 9 \cr 9 &=& 9 \end{array}

Für x = 5 ergibt sich eine wahre Aussage: \mathbb{L} = \left\{5 \right\}


17)
\begin{array}{rclll}0&=&7+3x+4-5x+4x-8+x+30 \cr 0&=& 33+3x &\vert & -3x \cr -3x&=&33 &\vert & :\left(-3\right) \cr x &=& -11 \end{array}

Probe:
\begin{array}{rclll} 0 &=& 7+3\cdot(-11)+4-5\cdot(-11)+4\cdot(-11)-8+(-11)+30 \cr 0 &=& 7-33+4+55-44-8-11+30 \cr 0 &=& 0 \end{array}

Für x = -11 ergibt sich eine wahre Aussage: \mathbb{L} = \left\{-11 \right\}


18)
\begin{array}{rclll}-9+3\left(-8p+3\right)&=&-2\left(10p-2\right)+4\left(-1-p\right)\cr-9-24p+9&=&-20p+4-4-4p\cr-24p&=&-24p &\vert& +24p \cr 0&=&0 \cr\cr\mathbb{L}&=&\mathbb{R}\end{array}

Bemerkung: Unabhängig davon, welches Element des Definitionsbereichs in diese Gleichung eingesetzt wird, erhält man immer auf beiden Seiten dasselbe Ergebnis. 0=0 ist schließlich immer richtig. Jede reelle Zahl löst also diese Gleichung, d. h. die Lösungsmenge entspricht dem Definitionsbereich.


19)
\begin{array}{rclll}3\left(5b+1\right)-3b&=&21-2\left(4+\dfrac{5}{2}b\right)\cr\cr15b+3-3b&=&21-8-5b \cr 12b+3&=&-5b+13&\vert&+5b-3 \cr 17b&=&10&\vert&:17 \cr b&=&\dfrac{10}{17}\end{array}

Probe:
\begin{array}{rclll} 3\left(5\cdot\dfrac{10}{17}+1\right)-3\cdot\dfrac{10}{17}&=&21-2\left(4+\dfrac{5}{2}\cdot\dfrac{10}{17}\right) \cr\cr 3\left(\dfrac{50}{17}+1\right)-\dfrac{30}{17}&=&21-2\left(4+\dfrac{25}{17}\right) \cr\cr 3\cdot\dfrac{67}{17}-\dfrac{30}{17}&=&21-2\cdot\dfrac{93}{17} \cr\cr \dfrac{201}{17}-\dfrac{30}{17}&=&21-\dfrac{186}{17} \cr\cr \dfrac{171}{17}&=&\dfrac{171}{17} \end{array}

Für x = \dfrac{10}{17} ergibt sich eine wahre Aussage: \mathbb{L} = \left\{\dfrac{10}{17} \right\}


20)
\begin{array}{rclll} 30z-26\left(\dfrac{2z}{13}-1\right)+5z&=&-6+2\left(25z+16\right)-19z \cr\cr 30z-4z+26+5z&=&-6+50z+32-19z \cr 31z+26&=&31z+26&\vert&-26 \cr 31z&=&31z &\vert& -31z \cr 0&=&0 \cr\cr \mathbb{L}&=&\mathbb{R} \end{array}

Bemerkung: Unabhängig davon, welches Element des Definitionsbereichs in diese Gleichung eingesetzt wird, erhält man immer auf beiden Seiten dasselbe Ergebnis. 0=0 ist schließlich immer richtig. Jede reelle Zahl löst also diese Gleichung, d. h. die Lösungsmenge entspricht dem Definitionsbereich.

 

5. Aufgabe

Sei x die Anzahl der Schüler des Pythagoras.
Dann "übersetzt" man:

  • "die Hälfte studiert Mathematik" mit \dfrac {x}{2}
  • "ein Viertel studiert Physik" mit \dfrac {x}{4}
  • "ein Siebtel lernt das Schweigen" mit \dfrac {x}{7}

Zusammen ergibt sich also:
\begin{array}{rclcl}x &=& \dfrac{x}{2}+\dfrac{x}{4}+\dfrac{x}{7}+3 & \vert& \text{erweitern} \cr x &=& \dfrac{14x}{28}+\dfrac{7x}{28}+\dfrac{4x}{28}+3 &\vert& \cdot 28 \cr 28x &=& 14x+7x+4x+84 \cr 28x &=& 25x+84 &\vert& -25x \cr 3x &=& 84 &\vert& \cdot \dfrac{1}{3} \cr x &=& 28 \end{array}

Pythagoras hat also 28 Schüler.

 

Bemerkung zu Textaufgaben allgemein:

Der erste Schritt beim Lösen einer Textaufgabe ist immer, sich die benötigten Variablen zu definieren. Anders formuliert: Es muss die Frage "Was ist hier eigentlich gesucht?" bzw. "Was soll hier eigentlich berechnet werden?" beantwortet werden. Das ist zum einen wichtig, da es einfacher wird, die Logik der Aufgabe zu durchschauen, wenn man das Ziel kennt. Zum anderen kann nur dann eine sinnvolle Gleichung formuliert werden, wenn klar ist, was die Variablen genau bezeichnen. Welche Bezeichnung Sie dabei wählen, ist nicht wichtig. Es hat eine gewisse Tradition, die unbekannte Größe x zu nennen. Wenn sich aus inhaltlichen Gründen eine andere Bezeichnung anbietet (hier könnte man z. B. die Anzahl der Schüler auch gut s nennen), spricht üblicherweise nichts dagegen, diese Bezeichnung zu verwenden. Im Gegenteil: Solche Benennungen können helfen, eine möglicherweise komplexere Aufgabe samt Lösungsweg übersichtlich zu halten.
Eine Schritt-für-Schritt-"Übersetzung" der in der Aufgabenstellung gegebenen Sachverhalte in mathematische Formeln kann ebenfalls helfen, die Gleichung aufzustellen (und bringt evtl. in der Klausur noch dringend benötigte Teilpunkte ...).
Den Antwortsatz nicht vergessen!

6. Lineare Funktionen - Lernziele und typische Fehler

Nach Durcharbeiten dieses Kapitels sollten Sie folgende Lernziele erreicht haben:

  • Sie kennen die Definition "Funktion" und können entscheiden, ob eine Kurve in einem Koordinatensystem eine Funktion darstellt oder nicht.
  • Sie können die Begriffe "Definitionsbereich", "Wertebereich" und "Funktionsgleichung" an einem Beispiel erläutern.
  • Sie können Funktionswerte berechnen und diese in einer Wertetabelle übersichtlich darstellen.
  • Sie können zu einer linearen Funktion den passenden Definitionsbereich bestimmen.
  • Sie wissen, wie der Graph einer linearen Funktion typischerweise aussieht und können ihn in ein kartesisches Koordinatensystem zeichnen.
  • Sie kennen die Funktionsgleichung einer linearen Funktion (allgemeine Geradengleichung).
  • Sie wissen, welchen Einfluss die Parameter in der Funktionsgleichung (Steigung und y-Achsenabschnitt) auf den Verlauf des Graphen haben und können diese Informationen für die Interpretation von Zusammenhängen nutzen.
  • Sie können die Funktionsgleichung einer linearen Funktion aus gegebenen Informationen (z. B. aus zwei Punkten) berechnen.
  • Sie können aus dem gegebenen Graphen einer linearen Funktion die Funktionsgleichung ermitteln.
  • Sie können die Lagebeziehung von zwei linearen Funktionen ermitteln.
  • Sie kennen den Zusammenhang zwischen linearen Gleichungen und linearen Funktionen.
  • Sie können lineare Funktionen von anderen Funktionstypen unterscheiden (grafisch und anhand der Funktionsgleichung).


In diesem Kapitel sind bislang keine typischen Fehler aufgefallen.


Für Online-Selbsttests zu diesem Thema und weitere Informationen zur Mathematikunterstützung an der TH Wildau nutzen Sie bitte den Moodle-Kursraum "SOS Mathematik - Brückenkurs".

Übersicht:

 

6.1 Lineare Funktionen - Aufgaben

1. Aufgabe

Bestimmen Sie jeweils die lineare Funktion, die durch die beiden angegebenen Punkte verläuft, und zeichnen Sie die Funktionsgraphen in ein Koordinatensystem!

1) A \left(2 \mid 12 \right) und B \left(-1 \mid 3 \right)

  11) A \left(5 \mid 3 \right) und B \left(7 \mid 9 \right)

2) A \left(-1 \mid -1 \right) und B \left(\dfrac {3}{2} \mid 4 \right)

  12) A \left(-2 \mid 3 \right) und B \left(7 \mid 2 \right)

3) P \left(0 \mid 5 \right) und Q \left(10 \mid 0 \right)

  13) A \left(0 \mid 22 \right) und B \left(11 \mid 0 \right)

4) P \left(\dfrac12 \mid \dfrac16 \right) und Q \left(\dfrac13 \mid \dfrac13 \right)

  14) Q \left( \dfrac{19}{32} \mid \dfrac{8}{3} \right) und P \left( - \dfrac{7}{5} \mid \dfrac{8}{3} \right)

5) A \left(2 \mid -3 \right) und B \left(3 \mid 8 \right)   15) R \left( -7 \mid -55 \right) und S \left( 1 \mid 9 \right)

6) R \left (22 \mid -4 \right) und S \left (-14 \mid -4 \right)

  16) P \left( 0 \mid 0 \right) und Q \left( -36 \mid -\dfrac{24}{7} \right)

7) K \left(\dfrac{1}{2} \mid \dfrac{1}{4}\right) und L \left(-2 \mid -\dfrac {7}{2} \right)

  17) A \left( \dfrac{3}{2} \mid \dfrac{1}{4} \right) und B \left( -5 \mid -\dfrac{7}{2} \right)

8) A \left (3 \mid \dfrac{7}{8} \right) und B \left (-1 \mid -\dfrac18 \right )

  18) P_1 \left( \dfrac{15}{8} \mid \dfrac{3}{4} \right) und P_2 \left( - \dfrac{5}{2} \mid - \dfrac{1}{2} \right)
9) P \left (\dfrac14 \mid -8 \right) und Q \left (-1 \mid 7 \right)

  19) C \left( 17 \mid -\dfrac{17}{2} \right) und D \left( -19 \mid \dfrac{53}{4} \right)

10) G \left (-3 \mid \dfrac {11}{6} \right) und H \left(0 \mid -\dfrac16 \right)

  20) P_1 \left( -25 \mid 3 \right) und P_2 \left( 100 \mid -2 \right)

 

2. Aufgabe

Stellen Sie jeweils fest, ob der angegebene Punkt P auf der Geraden f(x) liegt!

1) P\left(2\mid 24\right) und f(x)=15x-6

  6) P\left(0{,}1\mid -9\right) und f(x)=-296x

2) P\left(8\mid 9\right) und f(x)=0{,}4x+6

  7) P\left(0\mid 0\right) und f(x)=288x

3) P\left(25\mid -560\right) und f(x)=-63x+15

  8) P\left(14\mid -4959\right) und f(x)=-347x-101

4) P\left(14\mid 12{,}25\right) und f(x)=\dfrac{7}{8}x

  9) P\left(7\mid 88\right) und f(x)=11x+24

5) P\left(-6\mid -21\right) und f(x)=4x+3

  10) P\left(22\mid -1\right) und f(x)=\dfrac{12}{11}x-\dfrac{1}{9}

 

3. Aufgabe

Beantworten Sie zu den linearen Funktionen jeweils die folgenden Fragen:

  • Wie ist das Steigungsverhalten der Geraden? Steigt/Fällt sie oder ist sie konstant?
  • Schneidet die Gerade die y-Achse bei einer positiven/negativen Zahl oder bei 0?

Und das Alles: Ohne zu rechnen und zu zeichnen! Es geht nämlich bei dieser Aufgabe darum, den Funktionsterm zu verstehen und daraus Informationen über den Funktionsgraphen abzulesen, um z. B. Rechnungen plausibilisieren zu können.

Hinweis: Wer (noch) nicht weiß, was Potenzausdrücke, wie (-754)^{39}, bedeuten, kann die Aufgaben 8) - 10) zurückstellen und erst das Kapitel Potenzen, Wurzeln, Logarithmen bearbeiten.

1)  f(x)=-1.234{,}5678   6) f(x)=-\dfrac{20}{-99}x
2) f(x)=\dfrac{83}{9}x-\dfrac{9}{83}   7) f(x)=2a-23 mit a\;\in\;\mathbb{R}^-
3) f(b)=\dfrac{64-8b}{64}   8) f(z)=(-754)^{39}z
4) f(a)=23a-2   9) v(x)=(-1)^{56}x+(-1)^{55}
5) g(x)=24\cdot(1-x)   10) u(x)=(-3)^3x+(-2)^2

 

4. Aufgabe

Bestimmen Sie jeweils die Schnittpunkte der Geraden g(x) mit den Koordinatenachsen!

1) g(x)=24x-6

  6) g(x)=\dfrac{5}{4}x+41

2) g(x)=\dfrac{4}{3}x-2

  7) g(x)=\dfrac{6}{7}x-10

3) g(x)=-6x+8

  8) g(x)=-12x

4) g(x)=x+48

  9) g(x)=-\dfrac{3}{2}x-2

5) g(x)=15x-11

  10) g(x)=-23

 

5. Aufgabe

1)
Gegeben sei die Funktion f(x)=3x+5 mit x\in \mathbb{R}.
Gesucht ist jeweils die fehlende Koordinate des Punktes P(x\mid y),
a) wenn x=2{,}5
b) wenn y=26

  6)
Gegeben sei die Funktion f(x)=-13x mit x\in \mathbb{R}
Gesucht ist jeweils die fehlende Koordinate des Punktes P(x\mid y),
a) wenn x=5
b) wenn y=-104

2)
Gegeben sei die Funktion f(x)=-x-\dfrac{1}{2} mit x\in \mathbb{R}.
Gesucht ist jeweils die fehlende Koordinate des Punktes P(x\mid y),
a) wenn x=31
b) wenn y=7{,}5

  7)
Gegeben sei die Funktion f(x)=-3x+\dfrac{5}{4} mit x\in \mathbb{R}.
Gesucht ist jeweils die fehlende Koordinate des Punktes P(x\mid y),
a) wenn x=\dfrac{3}{4}
b) wenn y=-16

3)
Gegeben sei die Funktion f(x)=-4{,}5x+37 mit x\in \mathbb{R}.
Gesucht ist jeweils die fehlende Koordinate des Punktes P(x\mid y),
a) wenn x=2
b) wenn y=130

  8)
Gegeben sei die Funktion f(x)=\dfrac{1}{20}x+20 mit x\in \mathbb{R}.
Gesucht ist jeweils die fehlende Koordinate des Punktes P(x\mid y),
a) wenn x=18
b) wenn y=20{,}45

4)
Gegeben sei die Funktion f(x)=\dfrac{3}{2}x-24 mit x\in \mathbb{R}.
Gesucht ist jeweils die fehlende Koordinate des Punktes P(x\mid y),
a) wenn x=-5
b) wenn y=-20

  9)
Gegeben sei die Funktion f(x)=-\dfrac{2}{5}x+1 mit x\in \mathbb{R}.
Gesucht ist jeweils die fehlende Koordinate des Punktes P(x\mid y),
a) wenn x=11
b) wenn y=0

5)
Gegeben sei die Funktion f(x)=\dfrac{2}{3}x-\dfrac{5}{4} mit x\in \mathbb{R}.
Gesucht ist jeweils die fehlende Koordinate des Punktes P(x\mid y),
a) wenn x=2
b) wenn y=\dfrac{19}{12}

  10)
Gegeben sei die Funktion f(x)=-12x+\dfrac{11}{5} mit x\in \mathbb{R}
Gesucht ist jeweils die fehlende Koordinate des Punktes P(x\mid y),
a) wenn x=-\dfrac{2}{5}
b) wenn y=4{,}2

 

6. Aufgabe

Bestimmen Sie alle Lagebeziehungen der folgenden Geraden! Wenn sich die Geraden schneiden, berechnen Sie auch den Schnittpunkt!

f_1(x)=3x-6

f_2(x)=-7x-6

f_3(x)=\dfrac{27}{9}x+1

f_4(x)=-\dfrac{21}{3}x-\dfrac{24}{4}

 

7. Aufgabe

Die Grafik zeigt verschiedene Funktionsgraphen. Ermitteln Sie für jeden Graphen die Geradengleichung!

5 Geraden im Koordinatensystem

 

8. Aufgabe

Zwei gleich lange Kerzen werden gleichzeitig angezündet. Die erste Kerze hat eine Brenndauer von acht Stunden, die zweite eine von vier Stunden. Nach welcher Zeit ist die erste Kerze doppelt so lang wie die zweite?

 

9. Aufgabe

Für Profis: Welche Lagebeziehungen sind bei zwei Geraden im Raum möglich?

Dieses Kapitel enthält die folgenden Themen:

 

6.2 Lineare Funktionen - Erklärungen

Nachdem im vorherigen Kapitel Gleichungen als etwas sehr Grundlegendes in der Mathematik vorgestellt wurden, soll es nun um Funktionen gehen. Funktionen sind das zweite wirklich grundlegende Konzept in der Mathematik. Bevor wir uns - wie der Name des Kapitels verspricht - um lineare Funktionen kümmern, wird es daher um Funktionen an sich gehen.

 

Funktionen allgemein

Definition: Eine Funktion ist eine Zuordnung, die jedem Element einer Menge, Definitionsbereich (Formelzeichen: \mathbb{D}) genannt, genau ein Element einer anderen Menge, Wertebereich (Formelzeichen: \mathbb{W}) genannt, zuordnet. Das Element des Definitionsbereichs nennt man üblicherweise x-Wert, unabhängige Variable oder Argument der Funktion, das Element des Wertebereichs y-Wert oder abhängige Variable.

Beispiel: Eine Vorschrift, die jedem Monat in einem normalen Jahr (gemeint ist ein Jahr, welches kein Schaltjahr ist) die Zahl seiner Tage zuordnet, ist eine Funktion. Im blauen Schaubild sieht man, dass von jedem Element im Definitionsbereich (also von jedem Monat) genau ein Pfeil ausgeht. Was im Wertebereich passiert, ist dabei egal: Bei den einzelnen Elementen dürfen mehrere Pfeile ankommen oder auch gar keine.
Drehen wir die Vorschrift um, sodass nun den Monatslängen jeweils die betreffenden Monate zugeordnet werden. Im orangefarbenen Schaubild sieht man, dass von zwei Elementen des Definitionsbereichs ("30 Tage" und "31 Tage") mehrere Pfeile ausgehen und von einem anderen Element "29 Tage" keiner. Dies ist laut Funktionsdefinition nicht erlaubt, sodass diese Zuordnung keine Funktion ist. Dass bei den Elementen im Wertebereich jeweils nur genau ein Pfeil ankommt, kann die Situation auch nicht retten ...

Schaubild Funktion     Schaubild keine Funktion

 

In diesem Lernmodul werden nur Funktionen betrachtet, deren Definitions- und Wertebereiche aus Teilmengen der reellen Zahlen bestehen und die sich im weitesten Sinne "vernünftig" verhalten. Es gibt Funktionen, bei denen einige der hier besprochenen Eigenschaften so nicht gelten - aber darum können Sie sich, wenn nötig, später kümmern ...

 

Meistens nennt man Funktionen (naheliegenderweise) f. Hinter das f schreibt man in Klammern die Variable, meist x. Das ergibt dann also f(x) (gesprochen: "f von x"). Auf der anderen Seite vom Gleichheitszeichen folgt dann der Funktionsterm, z. B. so etwas wie -3x+12.
Das erste, was man mit Funktionen machen kann, ist das Berechnen von Funktionswerten (statt Funktionswert sagt man auch y-Wert): Ist eine Funktion f(x) mit der Variable x gegeben, dann bedeutet "Funktionswerte berechnen", dass man für die Variable konkrete Zahlenwerte einsetzt und das dann ausrechnet. 

Ein Beispiel
\begin{array}{lclcr} f(x) &=& -3x+12 \cr \cr f(-4) &=& -3 \cdot (-4)+12 &=& 24 \cr f(0) &=& -3 \cdot 0+12 &=& 12 \cr f(1) &=& -3 \cdot 1+12 &=& 9 \cr & \dots \end{array}

Bemerkung 1: Bitte beachten Sie die Schreibweise: Im ersten Schritt wird die Variable auf beiden Seiten der Funktionsgleichung durch den Zahlenwert ersetzt. Erst danach wird ausgerechnet.
Bemerkung 2: Dass für dieses Beispiel die x-Werte -4, 0 und 1 ausgewählt wurden, hat keinen wichtigen Hintergrund. Es geht ja nur darum, dass Prinzip des "Funktionswerte-Ausrechnen" zu verdeutlichen. Dafür hätte man auch ganz andere Zahlenwerte nehmen können.

Fasst man die oben ausgerechneten und noch einige weitere Werte zusammen und ordnet sie nach der Größe der x-Werte, erhält man folgende Wertetabelle:

x   -5 -4 -3 -2 -1 0 1 2 3 4 5
f(x)   27 24 21 18 15 12 9 6 3 0 -3

Bemerkung: Bei der Wahl der x-Werte für die Wertetabelle muss man auf die Besonderheiten der Funktion achten. Hätte die Funktion beispielsweise eine Nullstelle bei \frac{1} {2} , würden wir das bei dieser Wertetabelle nur schlecht bemerken. Zumindest für den Anfang sind die ganzen Zahlen von -5 bis 5 aber schon recht gute Kandidaten. Sollte dies nicht reichen, kann man ja immer noch mehr Werte berechnen ...

 

Der Funktionsgraph

Definition: Ein Funktionsgraph ist die grafische Darstellung einer Funktion in einem Koordinatensystem.
Für eine solche grafische Darstellung zeichnet man zunächst die Punkte aus der Wertetabelle in der Form P (x \mid y) in das Koordinatensystem und verbindet diese anschließend - soweit möglich - mit einer Linie. Das bedeutet, dass die Elemente des Definitionsbereiches bezogen auf die x-Achse und die Elemente des Wertebereiches bezogen auf die y-Achse eingezeichnet werden.

 

Beispiele für Funktionsgraphen

vier Funktionen

 

Beispiele für Kurven in einem Koordinatensystem, die keine Funktionen darstellen

Bei allen folgenden Beispielen wird nicht jedem x-Wert eindeutig ein y-Wert zugeordnet. Hier gibt es immer auch x-Werte, zu denen mehrere y-Werte gefunden werden, und/oder x-Werte, die keinen y-Wert zugeordnet bekommen. Z. B. findet man im ersten Bild zu x=0 einen positiven und einen negativen y-Wert. Auf der anderen Seite findet man zu x=10 (und zu vielen anderen x-Werten) gar keinen y-Wert. Daher handelt es sich bei den Zuordnungen nicht um Funktionen und bei ihren grafischen Darstellungen nicht um Funktionsgraphen.

vier Kurven, die keine Funktionen darstellen

 

Lineare Funktionen

Was man allgemein wissen sollte

Lineare Funktionen werden durch die allgemeine Geradengleichung beschrieben. Sie lautet: f(x) = m x + n, wobei m und n reelle Zahlen sind (m, n \in \mathbb{R}). m bezeichnet die Steigung und n den y-Achsenabschnitt. Das bedeutet:

  1. An m kann man ablesen, um wie viele Einheiten sich der y-Wert ändert, wenn x um eine Einheit größer wird.
  2. n gibt die y-Koordinate des Punktes an, an dem der Graph der linearen Funktion die y-Achse des Koordinatensystems schneidet.

Der Graph einer linearen Funktion ist eine Gerade.
Bemerkung 1: Nicht jede Gerade in einem Koordinatensystem ist Graph einer linearen Funktion. Senkrechte Geraden verletzen die Funktionsdefinition, da bei ihnen zu einem x-Werte unendlich viele y-Werte gefunden werden können (siehe Bild oben).
Bemerkung 2: Zwei Punkte reichen aus, um eine Gerade eindeutig zu bestimmen. Anders formuliert: Durch zwei Punkte verläuft genau eine Gerade. Das wusste schon der griechische Mathematiker Euklid vor knapp 2500 Jahren!

Wissenswertes zur Steigung:
Ist die Steigung m negativ, fällt die Gerade.
Ist die Steigung m=0, ist die Gerade konstant.
Ist die Steigung m positiv, steigt die Gerade.

Wissenswertes zum y-Achsenabschnitt:
Ist der y-Achsenabschnitt n negativ, schneidet die Gerade die y-Achse unterhalb der x-Achse.
Ist der y-Achsenabschnitt n=0, verläuft die Gerade durch den Koordinatenursprung. Eine solche Gerade nennt man Ursprungsgerade.
Ist der y-Achsenabschnitt n positiv, schneidet die Gerade die y-Achse oberhalb der x-Achse.


Hier ein paar Beispiele, die uns im restlichen Kapitel noch etwas begleiten werden:
f_1(x) ist eine fallende Ursprungsgerade.
f_2(x) ist eine steigende Gerade.
f_3(x) ist eine konstante Gerade.

3 Geraden in einem Koordinatensystem

 

Klassischerweise möchte (oder muss) man mit linearen Funktionen rechnen. Das heißt vor allem, dass man die Steigung und den y-Achsenabschnitt aus gegebenen Punkten bestimmen muss.

 

Ermittlung der Steigung

Sind die Punkte P_1 \left(x_1 \mid y_1 \right) und P_2 \left(x_2 \mid y_2 \right) gegeben, lässt sich die Steigung nach folgender Formel berechnen: m=\dfrac{y_2-y_1}{x_2-x_1}=\dfrac{\Delta y}{\Delta x} (gesprochen: "delta y geteilt durch delta x"). Anders formuliert: Die Differenz der y-Werte geteilt durch die Differenz der x-Werte ergibt die Steigung.
Falls keine Punkte gegeben sind, kann man aus der Zeichnung zwei Punkte des Graphen ablesen. Dabei muss man allerdings mit der Einschränkung leben, dass man auch knapp daneben liegen kann. Ob eine Funktion durch den Punkt P (-4 \mid 7) oder durch P (-3{,}98 \mid 7{,}0000123) verläuft, lässt sich nämlich nur schwer erkennen...


Zu unseren Beispielen von oben

Die Funktion f_1(x) verläuft durch die Punkte P_1 (-2 \mid 8) und P_2 (-1 \mid 4). Man berechnet:
  m = \dfrac{8-4}{-2-(-1)} = \dfrac{4}{-1} = -4  
Die Funktion f_2(x) verläuft durch die Punkte Q_1(2 \mid 1) und Q_2(5 \mid 7). Man berechnet:
  m = \dfrac{1-7}{2-5} = \dfrac{-6}{-3} = 2  
Die Funktion f_3(x) verläuft durch die Punkte R_1(1 \mid 5{,}7) und R_2(3{,}9 \mid 5{,}7). Man berechnet:
  m = \dfrac{5{,}7-5{,}7}{3{,}9-1} = \dfrac{0}{2{,}9} = 0  

Bemerkung 1: Mit diesen Rechnungen haben wir bestätigt, was wir oben schon über die Steigung der Geraden ausgesagt hatten.
Bemerkung 2: \Delta ist der griechische Buchstabe "delta", der in der Mathematik gerne für Differenzen verwendet wird.


Um die Steigung zu veranschaulichen, nutzt man Steigungsdreiecke. In der nächsten Grafik sieht man, dass bei f_3(x) kein Steigungsdreieck eingezeichnet werden kann, denn egal, wie groß man \Delta x wählt, \Delta y ist immer 0. Daraus ergibt sich dann auch rechnerisch, dass bei konstanten Geraden die Steigung 0 ist, was ja anschaulich offensichtlich ist.

3 Geraden im Koordinatensystem mit Steigungsdreiecken

 

Wichtig: Die Größe und die Lage des Steigungsdreiecks haben keinen Einfluss auf die Größe der Steigung, da nur das Verhältnis aus den Längen der beiden Dreiecksseiten berechnet wird. Um das zu verdeutlichen, wurden in der nächsten Grafik beispielhaft für f_2(x) verschiedene Steigungsdreiecke eingezeichnet und daraus die Steigung berechnet:

Mittleres Steigungsdreieck mit Q_1(2 \mid 1) und Q_2(5 \mid 7): m = \dfrac{1-7}{2-5} = \dfrac{-6}{-3} = 2
Kleines Steigungsdreieck mit Q_3(3 \mid 3) und Q_4(4 \mid 5): m = \dfrac{5-3}{4-3} = \dfrac {2}{1} = 2
Großes Steigungsdreieck mit Q_5(-3 \mid -9) und Q_6(6 \mid 9): m = \dfrac{-9-9}{-3-6} = \dfrac{-18}{-9} = 2

 

eine Gerade mit mehreren Steigungsdreiecken


Ebenfalls keinen Einfluss auf die Steigung hat die Reihenfolge, in der man die Punkte in die Steigungsformel einsetzt. Ein Beispiel: Für das mittlere Steigungsdreieck könnte man auch m=\dfrac{7-1}{5-2}=\dfrac{6}{3}=2 rechnen. Das Ergebnis bleibt (glücklicherweise) gleich.

 

Ermittlung des y-Achsenabschnitts

Wie der y-Achsenabschnitt n ermittelt werden kann, hängt von der Aufgabenstellung ab:

1. mögliche Aufgabenstellung:
Soll die Geradengleichung f(x)=mx+n aus zwei gegebenen Punkten ermittelt werden, benötigt man zunächst die Steigung m (haben wir ja gerade schon berechnet). Um den y-Achsenabschnitt zu berechnen, setzt man dann m und die Koordinaten eines gegebenen Punktes in die Gleichung ein. f(x) entspricht dabei der y-Koordinate. So erhält man eine lineare Gleichung, die nach n aufgelöst werden kann.
Welchen Punkt man für diese Rechnung verwendet, ist egal, da die Gerade ja ohnehin durch beide Punkte verlaufen soll. Der zweite Punkt kann dann zur Überprüfung des Ergebnisses genutzt werden: Man setzt seine Koordinaten in die ermittelte Geradengleichung ein und schaut, ob beide Seiten gleich groß werden. Das nennt man Punktprobe.

Bei unseren Beispielen berechnet man also

Für f_1(x): 8 = -4 \cdot (-2)+n man errechnet n = 0 also ist f_1(x) = -4 \cdot x+0
Für f_2(x): 1 = 2 \cdot 2 +n man errechnet n = -3 also ist f_2(x) = 2\cdot x-3
Für f_3(x): 5{,}7 = 0 \cdot 1 +n man errechnet n = 5{,}7 also ist f_3(x) = 0 \cdot x+5{,}7


Punktproben

Für f_1(x): 4 = -4 \cdot (-1)+0 man errechnet 4 = 4  
  Das ist eine wahre Aussage. Die berechnete Geradengleichung für f_1(x) ist also richtig.
Für f_2(x): 7 = 2 \cdot 5 -3 man errechnet 7 = 7  
  Das ist eine wahre Aussage. Die berechnete Geradengleichung für f_2(x) ist also richtig.
Für f_3(x): 5{,}7 = 0 \cdot 3{,}9+5{,}7 man errechnet 5{,}7 = 5{,}7  
  Das ist eine wahre Aussage. Die berechnete Geradengleichung für f_3(x) ist also richtig.


Damit sind die Geradengleichungen vollständig bestimmt. Sie lauten:
\begin{array}{lcl} f_1(x) &=& - 4x \cr f_2(x) &=& 2x-3 \cr f_3(x) &=& 5{,}7 \end{array}
FERTIG!


2. mögliche Aufgabenstellung:
Soll die Geradengleichung f(x)=mx+n aus einem gegebenen Graph ermittelt werden, stellt man einfach fest, wo der Graph die y-Achse schneidet. Der y-Wert dieses Punktes ist der y-Achsenabschnitt.

 

Lagebeziehungen zweier Geraden in der Ebene

Stellen Sie sich zwei Geraden in einer Ebene vor, z. B. in einem zweidimensionalen Koordinatensystem. Vor Ihrem geistigen Auge sollte nun (mindestens) eine der folgenden Grafiken entstehen:

Lagebeziehungen zweier Geraden in der Ebene

Es gibt also verschiedene Möglichkeiten, wie zwei Geraden in der Ebene zueinander liegen können. Man nennt dies die Lagebeziehung der beiden Geraden:

  1. Die beiden Geraden können parallel zueinander sein. In diesem Fall haben die Geraden keine gemeinsamen Punkte. In der Geradengleichung erkennt man dies daran, dass die Geraden die gleiche Steigung aber einen unterschiedlichen y-Achsenabschnitt haben. Im Beispiel oben handelt es sich um die Geraden f_1(x)=3x+3 und f_2(x)=3x-6.
  2. Die beiden Geraden können aufeinander liegen. Sie haben in diesem Fall unendlich viele gemeinsame Punkte. Anders formuliert: Sie sind identisch. Damit sind natürlich auch die Geradengleichungen gleich - auch wenn man das nicht immer auf den ersten Blick sieht. Manchmal muss man erst ein bisschen umformen. Im mittleren Bild oben sind die Geraden f_3(x)=0{,}5x+5 und f_4(x)=\frac{1}{2}x+\frac{20}{4} eingezeichnet.
  3. Die beiden Geraden können einen Schnittpunkt haben. Dann haben sie exakt einen Punkt gemeinsam. Feststellen kann man diesen Fall anhand unterschiedlicher Steigungen in der Geradengleichung. Der y-Achsenabschnitt spielt keine Rolle. In diesem Beispiel schneiden sich die Geraden f_5(x)=4x-8 und f_6(x)=-2x+8 in einem Punkt, den wir unten gleich berechnen.

Mehr Möglichkeiten gibt es nicht! Wenn zwei Geraden mehr als einen gemeinsamen Punkt (und weniger als unendlich) hätten, wären sie krumm ...

Wenn zwei Geraden einen Schnittpunkt haben, möchte man üblicherweise wissen, wo dieser liegt. Man muss den Schnittpunkt also berechnen können: Da ein Schnittpunkt - man schreibt dafür üblicherweise S mit den Koordinaten x_s und y_s - ein gemeinsamer Punkt der beiden Funktionen ist, müssen die zu x_s gehörenden Funktionswerte beider Geraden übereinstimmen. In unserem Beispiel müssen also f_5(x_s) und f_6(x_s) gleich groß sein. Man setzt daher die beiden Funktionsterme gleich. Löst man die entstehende lineare Gleichung, erhält man den x-Wert des Schnittpunktes.

In unserem Beispiel
x-Wert berechnen:
\begin{array}{rclcl} f_5 (x_s) &=& f_6(x_s) \cr \cr 4x_s-8 &=& -2x_s+8 &\vert& +8+2x_s \cr \cr 6x_s &=& 16 & \vert& : (6) \cr \cr x_s &=& \dfrac{16}{6}=\dfrac{8}{3} \end{array}

y-Wert berechnen:
\begin{array} {rcl} f_5(x_s) &=& f_5\left(\dfrac{8}{3}\right) \cr \cr &=& 4 \cdot \dfrac{8}{3}-8 \cr \cr &=& \dfrac{32}{3}-\dfrac{24}{3} \cr\cr &=& \dfrac{8}{3} \end {array}
ODER
\begin{array}{rcl} f_6(x_s) &=& f_6\left(\dfrac{8}{3}\right) \cr \cr &=& -2 \cdot \dfrac {8}{3} +8\cr \cr &=&-\dfrac{16}{3}+\dfrac{24}{3} \cr\cr &=& \dfrac{8}{3} \end{array}

Die zweite Rechnung für den y-Wert dient gleichzeitig als Probe. Der Punkt S\left(\dfrac{8}{3} \mid \dfrac{8}{3}\right) liegt tatsächlich auf beiden Geraden und ist somit der (einzige) Schnittpunkt von f_5(x) und f_6(x).

Übersicht:

 

6.3 Lineare Funktionen - Lösungen

1. Aufgabe

1)
m = \dfrac{12-3}{2-(-1)} = \dfrac{9}{3} = 3
Einsetzen von m = 3 und A \left(2 \mid 12 \right) in die allgemeine Geradengleichung liefert: 12 = 3 \cdot 2 + n und damit n = 6
Also ist: f_1(x) = 3x + 6


2)
m = \genfrac{}{}{1 pt}{0} {-1-4}{-1-\frac{3}{2}} = \genfrac{}{}{1 pt}{0} {-5}{-\frac{5}{2}} = 2
Einsetzen von m = 2 und A \left(-1 \mid -1 \right) in die allgemeine Geradengleichung liefert: -1 = 2 \cdot (-1) + n und damit n = 1
Also ist: f_2(x) = 2x + 1


3)
m = \dfrac{5-0}{0-10} = \dfrac{5}{-10} = -\dfrac{1}{2}
Einsetzen von m = -\dfrac{1}{2} und P \left(0 \mid 5 \right) in die allgemeine Geradengleichung liefert: 5 = -\dfrac{1}{2} \cdot 0 + n und damit n = 5
Also ist: f_3(x) = - \dfrac{1}{2}x + 5


4)
m = \genfrac{}{}{1 pt}{0} {\frac{1}{6}-\frac{1}{3}}{\frac{1}{2}-\frac{1}{3}} = \genfrac{}{}{1 pt}{0} {-\frac{1}{6}}{\frac{1}{6}} = -1
Einsetzen von m = -1 und Q \left( \dfrac{1}{3} \mid \dfrac{1}{3} \right) in die allgemeine Geradengleichung liefert:  \dfrac{1}{3} = -1 \cdot \dfrac{1}{3} + n und damit n = \dfrac{2}{3}
Also ist: f_4(x) = -x + \dfrac{2}{3}


5)
m = \dfrac{8-(-3)}{3-2} = 11
Einsetzen von m = 11 und B \left(3 \mid 8 \right) in die allgemeine Geradengleichung liefert: 8 = 11 \cdot 3 + n und damit n = -25
Also ist: f_5(x) = 11x - 25

die Geraden f1(x) bis f5(x) im Koordinatensystem

Diese Skaleneinteilung ist für f_5(x) nicht ideal, weil ihr y-Achsenabschnitt außerhalb des gezeigten Bereichs liegt. Daher noch eine Grafik mit entsprechend angepassten Skalen:

die Geraden f1(x) bis f5(x) in einem Koordinatensystem mit angepassten Achsen

Diese Skaleneinteilung ist für die anderen Geraden nicht so geschickt, weil die Details aufgrund des großen Maßstabs verloren gehen. Daher haben beide Grafiken ihre Berechtigung.

6)
m = \dfrac{-4-(-4)}{22-(-14)} = \dfrac{0}{36} = 0 
Einsetzen von m=0 und R \left (22 \mid -4\right) in die allgemeine Geradengleichung liefert: -4 = 0 \cdot 22 + n und damit n= -4
Also ist: f_6(x) = -4


7)
m = \genfrac{}{}{1 pt}{0} {\frac{1}{4}- \left(-\frac{7}{2}\right)}{\frac{1}{2}-(-2)} = \genfrac{}{}{1 pt}{0} {\frac{15}{4}}{\frac{5}{2}} = \dfrac{3}{2}
Einsetzen von m=\dfrac{3}{2} und L \left(-2 \mid -\dfrac {7}{2} \right) in die allgemeine Geradengleichung liefert: -\dfrac{7}{2} = \dfrac{3}{2} \cdot (-2) + n und damit n= -\dfrac{1}{2}
Also ist: f_7(x) = \dfrac{3}{2}x -\dfrac{1}{2}


8)
m = \genfrac{}{}{1 pt}{0} {\frac{7}{8}- \left(-\frac{1}{8} \right)}{3-(-1)} = \genfrac{}{}{1 pt}{0} {\frac{8}{8}}{4} = \dfrac{1}{4}
Einsetzen von m=\dfrac14 und B \left (-1 \mid -\dfrac18 \right ) in die allgemeine Geradengleichung liefert: -\dfrac18 = \dfrac14 \cdot (-1) + n und damit n= \dfrac18
Also ist: f_8(x) = \dfrac{1}{4}x +\dfrac{1}{8}


9)
m = \genfrac{}{}{1 pt}{0} {-8-7}{\frac14 - (-1)} = \genfrac{}{}{1 pt}{0} {-15}{\frac{5}{4}} = -12
Einsetzen von m=-12 und P \left (\dfrac14 \mid -8 \right) in die allgemeine Geradengleichung liefert: -8= -12 \cdot \dfrac14 + n und damit n = -5
Also ist: f_9(x) = -12x -5


10)
m = \genfrac{}{}{1 pt}{0} {\frac{11}{6}-\left(-\frac16\right)}{-3-0}= \genfrac{}{}{1 pt}{0} {\frac{12}{6}}{-3} = -\dfrac{2}{3}
Einsetzen von m = -\dfrac{2}{3} und H \left(0 \mid -\dfrac16 \right) in die allgemeine Geradengleichung liefert: -\dfrac16 = -\dfrac{2}{3} \cdot 0 +n und damit n = -\dfrac16
Also ist: f_{10}(x) = -\dfrac{2}{3}x - \dfrac16

die Geraden f6(x) bis f10(x) im Koordinatensystem

11)
m = \dfrac{9-3}{7-5} = 3
Einsetzen von m=3 und A \left( 5 \mid 3 \right) in die allgemeine Geradengleichung liefert: 3 = 3 \cdot 5 + n und damit n = -12
Also ist: f_{11}(x) = 3x - 12

12)
m = \dfrac{2-3}{7+2} = - \dfrac{1}{9} \approx 0{,}11
Einsetzen von m = -\dfrac{1}{9} und B \left( 7 \mid 2 \right) in die allgemeine Geradengleichung liefert: 2 = -\dfrac{1}{9} \cdot 7 + n und damit n = \dfrac{25}{9} \approx 2{,}78
Also ist: f_{12} (x) = -\dfrac{1}{9} x + \dfrac{25}{9}


13)
m = \dfrac{0-22}{11-0} = -2
Einsetzen von m = -2 und B \left( 11 \mid 0 \right) in die allgemeine Geradengleichung liefert: 0 = -2 \cdot 11 + n und damit n = 22
Also ist: f_{13} (x) = -2x +22


14)
m = \genfrac{}{}{1 pt}{0} {\frac{8}{3} - \frac{8}{3}}{- \frac{7}{5} - \frac{19}{32}} = 0
Einsetzen von m = 0 und P \left( -\dfrac{7}{5} \mid \dfrac{8}{3} \right) in die allgemeine Geradengleichung liefert:  \dfrac{8}{3} = 0 \cdot \left( - \dfrac{7}{5} \right) + n und damit n = \dfrac{8}{3}
Also ist: f_{14} (x) = \dfrac{8}{3} \approx 2{,}67


15)
m = \dfrac{9 + 55}{1+7} = 8
Einsetzen von m = 8 und S \left( 1 \mid 9 \right) in die allgemeine Geradengleichung liefert: 9 = 8 \cdot 1 + n und damit n = 1
Also ist: f_{15} (x) = 8x +1

die Geraden f11(x) bis f15(x) im Koordinatensystem

Diese Skaleneinteilung ist für f_{11}(x) und f_{13}(x) nicht ideal, weil ihr y-Achsenabschnitt außerhalb des gezeigten Bereichs liegt. Daher noch eine Grafik mit entsprechend angepassten Skalen:

die Geraden f11(x) bis f15(x) in einem Koordinatensystem mit angepassten Achsen


16)
m = \genfrac{}{}{1 pt}{0} {- \frac{24}{7} - 0}{-36-0} = \dfrac{2}{21} \approx 0{,}10
Einsetzen von m = \dfrac{2}{21} und P \left( 0 \mid 0 \right) in die allgemeine Geradengleichung liefert: 0 = \dfrac{2}{21} \cdot 0 +n und damit n = 0
Also ist: f_{16} (x) = \dfrac{2}{21} x


17)
m = \genfrac{}{}{1 pt}{0} {- \frac{7}{2}- \frac{1}{4}}{-5 - \frac{3}{2}} = \dfrac{15}{26} \approx 0{,}58
Einsetzen von m = \dfrac{15}{26} und A \left( \dfrac{3}{2} \mid \dfrac{1}{4} \right) in die allgemeine Geradengleichung liefert:  \dfrac{1}{4} = \dfrac{15}{26} \cdot \dfrac{3}{2} + n und damit n = - \dfrac{8}{13} \approx -0{,}62
Also ist: f_{17} (x) = \dfrac{15}{26} x - \dfrac{8}{13}


18)
m = \genfrac{}{}{1 pt}{0} {- \frac{1}{2} - \frac{3}{4}}{- \frac{5}{2} - \frac{15}{8}} = \dfrac{2}{7} \approx 0{,}29
Einsetzen von m = \dfrac{2}{7} und P_1 \left( \dfrac{15}{8} \mid \dfrac{3}{4} \right) in die allgemeine Geradengleichung liefert: \dfrac{3}{4} = \dfrac{2}{7} \cdot \dfrac{15}{8} + n und damit n = \dfrac{3}{14} \approx 0{,}21
Also ist: f_{18} (x) = \dfrac{2}{7} x + \dfrac{3}{14}


19)
m = \genfrac{}{}{1 pt}{0} {\frac{53}{4} + \frac{17}{2}}{-19-17} = - \dfrac{29}{48} \approx -0{,}60
Einsetzen von m = - \dfrac{29}{48} und D \left( -19 \mid \dfrac{53}{4} \right) in die allgemeine Geradengleichung liefert:  \dfrac{53}{4} = - \dfrac{29}{48} \cdot \left( -19 \right) + n und damit  n = \dfrac{85}{48} \approx 1{,}77
Also ist: f_{19} (x) = - \dfrac{29}{48} x + \dfrac{85}{48}


20)
m = \dfrac{-2-3}{100+25} = - \dfrac{1}{25} = -0{,}04
Einsetzen von m = - \dfrac{1}{25} und P_1 \left( -25 \mid 3 \right) in die allgemeine Geradengleichung liefert: 3 = - \dfrac{1}{25} \cdot (-25) + n und damit n = 2
Also ist: f_{20} (x) = - \dfrac{1}{25} x + 2

die Geraden f16(x) bis f20(x) im Koordinatensystem

Diesmal ist die Skaleneinteilung nicht ideal, weil in der Nähe des Koordinatenursprungs "zu viel passiert" und die Grafik daher in diesem wichtigen Bereich nicht aufschlussreich ist. Daher noch eine Grafik mit entsprechend angepassten Skalen:

die Geraden f16(x) bis f20(x) in einem Koordinatensystem mit angepassten Achsen

 

2. Aufgabe

Um festzustellen, ob ein Punkt auf einem Graphen liegt, muss der Funktionswert von f(x) für den gegebenen x-Wert ausgerechnet werden und dieser mit dem gegebenen y-Wert verglichen werden.

1) f(2)=15\cdot 2-6=24
Der ausgerechnete Funktionswert stimmt mit dem gegebenen Wert überein. Der Punkt P liegt also auf der Geraden f(x).


2) f(8)=0{,}4\cdot 8+6=9{,}2
Der ausgerechnete Funktionswert stimmt nicht mit dem gegebenen Wert überein. Der Punkt P liegt also nicht auf der Geraden f(x).


3) f(25)=-63\cdot 25+15=-1560
Der ausgerechnete Funktionswert stimmt nicht mit dem gegebenen Wert überein. Der Punkt P liegt also nicht auf der Geraden f(x).


4) f(14)=\dfrac{7}{8}\cdot 14=12{,}25
Der ausgerechnete Funktionswert stimmt mit dem gegebenen Wert überein. Der Punkt P liegt also auf der Geraden f(x)


5) f(-6)=4\cdot \left(-6\right)+3=-21
Der ausgerechnete Funktionswert stimmt mit dem gegebenen Wert überein. Der Punkt P liegt also auf der Geraden f(x).


6) f(0{,}1)=-296\cdot 0{,}1=-29{,}6
Der ausgerechnete Funktionswert stimmt nicht mit dem gegebenen Wert überein. Der Punkt P liegt also nicht auf der Geraden f(x).


7) f(0)=288\cdot 0=0
Der ausgerechnete Funktionswert stimmt mit dem gegebenen Wert überein. Der Punkt P liegt also auf der Geraden f(x).


8) f(14)=-347\cdot 14-101=-4.959
Der ausgerechnete Funktionswert stimmt mit dem gegebenen Wert überein. Der Punkt P liegt also auf der Geraden f(x).


9) f(7)=11\cdot 7+24=101
Der ausgerechnete Funktionswert stimmt nicht mit dem gegebenen Wert überein. Der Punkt P liegt also nicht auf der Geraden f(x).


10) f(22)=\dfrac{12}{11}\cdot 22-\dfrac{1}{9}\approx23{,}89
Der ausgerechnete Funktionswert stimmt nicht mit dem gegebenen Wert überein. Der Punkt P liegt also nicht auf der Geraden f(x).

 

3. Aufgabe

1) Der Graph von f(x) ist konstant, da m=0, und schneidet die y-Achse bei einer negativen Zahl, da n < 0.


2) Der Graph von f(x) steigt, da m>0, und schneidet die y-Achse bei einer negativen Zahl, da n < 0.


3) Der Graph von f(b) fällt, da m < 0, und schneidet die y-Achse bei einer positiven Zahl, da n > 0.


4) Der Graph von f(a) steigt, da m>0, und schneidet die y-Achse bei einer negativen Zahl, da n < 0.


5) Der Graph von g(x) fällt, da m < 0, und schneidet die y-Achse bei einer positiven Zahl, da n > 0.


6) Der Graph von f(x) steigt, da m>0, und schneidet die y-Achse bei 0.


7) Der Graph von f(x) ist konstant, da m=0, und schneidet die y-Achse bei einer negativen Zahl, da n < 0.

Bemerkung: Da die Funktion f(x) heißt, wissen wir, dass die Variable x ist und nicht a, wie man denken könnte. a\in\mathbb{R}^- ist eine negative Konstante.


8) Der Graph von f(z) fällt, da m < 0, und schneidet die y-Achse bei 0.

Bemerkung: Man muss (-754)^{39} nicht ausrechnen, um die Frage nach dem Schnittpunkt mit der y-Achse beantworten zu können. Es reicht festzustellen, dass hier eine negative Basis mit einem ungeraden Exponenten potenziert wird. Da "minus mal minus gleich plus" ist, hebt sich eine gerade Anzahl an Minuszeichen auf. Ein Minuszeichen bleibt quasi übrig. Das Ergebnis ist also negativ.


9) Der Graph von v(x) steigt, da m>0, und schneidet die -Achse bei einer negativen Zahl, da n < 0.


10) Der Graph von u(x) fällt, da m < 0, und schneidet die y-Achse bei einer positiven Zahl, da n>0.

 

4. Aufgabe

Schnittpunkt mit der x-Achse S_x: Beim Schnittpunkt eines Graphen mit der x-Achse muss die y-Koordinate 0 sein. Wäre die y-Koordinate ungleich 0, würde der Punkt ober- oder unterhalb der x-Achse liegen. Der Funktionsterm, der ja für die y-Koordinate steht, muss also nullgesetzt werden. Die Lösung der entstehenden lineare Gleichung ist die gesuchte Koordinate.

Schnittpunkt mit der y-Achse S_y: Bei linearen Funktionen ist es einfach, den Schnittpunkt des Graphen mit der y-Achse zu bestimmen. Der y-Achsenabschnitt ist ja definiert als Schnittpunkt des Graphen mit der y-Achse, sodass die gesuchte Koordinate direkt abgelesen werden kann. Die x-Koordinate dieses Schnittpunkts ist 0. Bei komplizierteren Funktionen müsste x=0 in den Funktionsterm eingesetzt und das Ergebnis ausgerechnet werden.


1)
Schnittpunkt mit der x-Achse:
\begin{array}{rclcl} 24x-6 &=& 0 &\vert & +6 \cr 24x &=& 6 &\vert& : 24 \cr x &=& \dfrac{1}{4} \end{array}

Der Schnittpunkt mit der x-Achse ist also  S_x \left(\dfrac{1}{4} \mid 0\right) .

Der Schnittpunkt mit der y-Achse ist  S_y \left(0\mid -6\right) .


2)
Schnittpunkt mit der x-Achse:
\begin{array}{rclcl} \dfrac{4}{3}x-2 &=& 0 &\vert& +2 \cr \dfrac{4}{3}x &=& 2 &\vert& \cdot \dfrac{3}{4} \cr x &=& \dfrac{6}{4} \cr\cr x &=& \dfrac{3}{2} \end{array}

Der Schnittpunkt mit der x-Achse ist also  S_x \left(\dfrac{3}{2} \mid 0\right) .

Der Schnittpunkt mit der y-Achse ist  S_y \left(0\mid -2\right) .


3)
Schnittpunkt mit der x-Achse:
\begin{array}{rclcl} -6x+8 &=& 0 &\vert& -8 \cr -6x &=& -8 &\vert& :(-6) \cr x &=& \dfrac{8}{6} \cr\cr x &=& \dfrac{4}{3} \end{array}

Der Schnittpunkt mit der x-Achse ist also  S_x \left(\dfrac{4}{3} \mid 0\right) .

Der Schnittpunkt mit der y-Achse ist  S_y \left(0\mid 8\right) .


4)
Schnittpunkt mit der x-Achse:
\begin{array}{rclcl} x+48 &=& 0 &\vert& -48 \cr x &=& -48 \end{array}

Der Schnittpunkt mit der x-Achse ist also  S_x \left(-48 \mid 0\right) .

Der Schnittpunkt mit der y-Achse ist  S_y \left(0\mid 48\right) .


5)
Schnittpunkt mit der x-Achse:
\begin{array}{rclcl} 15x-11 &=& 0 &\vert& +11 \cr 15x &=& 11 &\vert& :15 \cr x &=& \dfrac{11}{15} \end{array}

Der Schnittpunkt mit der x-Achse ist also  S_x \left(\dfrac{11}{15} \mid 0\right) .

Der Schnittpunkt mit der y-Achse ist  S_y \left(0\mid -11\right) .


6)
Schnittpunkt mit der x-Achse: 
\begin{array}{rclcl} \dfrac{5}{4}x+41 &=& 0 &\vert& -41 \cr \dfrac{5}{4}x &=& -41 &\vert& \cdot \dfrac{4}{5} \cr x &=& -\dfrac{164}{5} \end{array}

Der Schnittpunkt mit der x-Achse ist also  S_x \left(-\dfrac{164}{5} \mid 0\right) .

Der Schnittpunkt mit der y-Achse ist  S_y \left(0\mid 41\right) .


7)
Schnittpunkt mit der x-Achse: 
\begin{array}{rclcl} \dfrac{6}{7} x-10 &=& 0 &\vert&+10 \cr \dfrac{6}{7}x &=& 10 &\vert& \cdot \dfrac{7}{6} \cr x &=& \dfrac{70}{6} \cr\cr x &=& \dfrac{35}{3} \end{array}

Der Schnittpunkt mit der x-Achse ist also  S_x \left(\dfrac{35}{3} \mid 0\right) .

Der Schnittpunkt mit der y-Achse ist  S_y \left(0\mid -10\right) .


8)
Schnittpunkt mit der x-Achse:
\begin{array}{rclcl} -12x &=& 0 &\vert& :(-12) \cr x &=& 0 \end{array}

Der Schnittpunkt mit der x-Achse ist also  S_x \left(0 \mid 0\right) .

Der Schnittpunkt mit der y-Achse ist  S_y \left(0\mid 0\right) .


9)
Schnittpunkt mit der x-Achse:
\begin{array}{rclcl} -\dfrac{3}{2}x-2 &=& 0 &\vert& +2 \cr -\dfrac{3}{2}x &=& 2 &\vert& \cdot \left(-\dfrac{2}{3}\right) \cr x &=& -\dfrac{4}{3} \end{array}

Der Schnittpunkt mit der x-Achse ist also  S_x \left(-\dfrac{4}{3} \mid 0\right) .

Der Schnittpunkt mit der y-Achse ist  S_y \left(0\mid -2\right) .


10)
Schnittpunkt mit der x-Achse: 
\begin{array}{rclcl} -23 &=& 0 \end{array}

Diese Gleichung hat keine Lösung. S_x existiert also nicht. Grafisch erkennt man dies daran, dass die Gerade horizontal verläuft. Daher schneidet sie die x-Achse nicht.

Der Schnittpunkt mit der y-Achse ist  S_y \left(0\mid -23\right) .

 

5. Aufgabe

1)
a)
\begin{array}{rclll} f(2{,}5) &=& 3 \cdot 2{,}5+5 &=& 12{,}5 \quad \rightarrow \quad P_1(2{,}5 \mid 12{,}5) \end{array}

b)
\begin{array}{rclll} 26 &=& 3x+5 &\vert & -5 \cr21 &=& 3x &\vert & :3 \cr7 &=& x & & \rightarrow \quad P_2(7 \mid 26) \end{array}


2)
a)
\begin{array}{rclll} f(31) &=& -31-\dfrac{1}{2} &=& -31{,}5 \quad \rightarrow \quad P_1(31 \mid -31{,}5) \end{array}

b)
\begin{array}{rclll} 7{,}5 &=& -x-\dfrac{1}{2} & \vert & +\dfrac{1}{2} \cr8 &=& -x & \vert & \cdot (-1) \cr-8 &=& x && \rightarrow \quad P_2(-8 \mid 7{,}5) \end{array}


3)
a)
\begin{array}{rclll} f(2) &=& -4{,}5 \cdot 2+37 &=& 28 \quad \rightarrow \quad P_1(2 \mid 28) \end{array}

b)
\begin{array}{rclll} 130 &=& -4{,}5x+37 &\vert & -37 \cr93 &=& -4{,}5x &\vert & :(-4{,}5) \cr-\dfrac{62}{3} &=& x && \rightarrow \quad P_2\left(-\dfrac{62}{3} \mid 130\right) \end{array}


4)
a)
\begin{array}{rclll} f(-5) &=& \dfrac{3}{2} \cdot (-5)-24 &=& -31{,}5 \quad \rightarrow \quad P_1(-5 \mid -31{,}5) \end{array}

b)
\begin{array}{rclll} -20 &=& \dfrac{3}{2}x-24 &\vert & +24 \cr4 &=& \dfrac{3}{2}x &\vert & : \dfrac{3}{2} \cr\dfrac{8}{3} &=& x && \rightarrow \quad P_2\left(\dfrac{8}{3} \mid -20\right) \end{array}


5)
a)
\begin{array}{rclll} f(2) &=& \dfrac{2}{3} \cdot 2-\dfrac{5}{4} &=& \dfrac{1}{12} \quad \rightarrow \quad P_1\left(2 \mid \dfrac{1}{12}\right) \end{array}

b)
\begin{array}{rclll} \dfrac{19}{12} &=& \dfrac{2}{3}x-\dfrac{5}{4} &\vert & +\dfrac{5}{4} \cr\dfrac{17}{6} &=& \dfrac{2}{3}x &\vert & : \dfrac{2}{3} \cr\dfrac{17}{4} &=& x && \rightarrow \quad P_2\left(\dfrac{17}{4} \mid \dfrac{19}{12}\right) \end{array}


6)
a)
\begin{array}{rclll} f(5) &=& -13 \cdot 5 &=& -65 \quad \rightarrow \quad P_1(5 \mid -65) \end{array}

b)
\begin{array}{rclll} -104 &=& -13x &\vert & :(-13) \cr8 &=& x && \rightarrow \quad P_2(8 \mid -104) \end{array}


7)
a)
\begin{array}{rclll} f\left(\dfrac{3}{4}\right) &=& -3 \cdot \dfrac{3}{4}+\dfrac{5}{4} &=& -1 \quad \rightarrow \quad P_1\left(\dfrac{3}{4} \mid -1\right) \end{array}

b)
\begin{array}{rclll} -16 &=& -3x+\dfrac{5}{4} &\vert & -\dfrac{5}{4} \cr-\dfrac{69}{4} &=& -3x &\vert & :(-3) \cr\dfrac{23}{4} &=& x && \rightarrow \quad P_2\left(\dfrac{23}{4} \mid -16\right) \end{array}


8)
a)
\begin{array}{rclll} f(18) &=& \dfrac{1}{20} \cdot 18+20 &=& 20{,}9 \quad \rightarrow \quad P_1(18 \mid 20{,}9) \end{array}

b)
\begin{array}{rclll} 20{,}45 &=& \dfrac{1}{20}x+20 &\vert & -20 \cr0{,}45 &=& \dfrac{1}{20}x &\vert & \cdot 20 \cr9 &=& x && \rightarrow \quad P_2(9 \mid 20{,}45) \end{array}


9)
a)
\begin{array}{rclll} f(11) &=& -\dfrac{2}{5} \cdot 11+1 &=& -3{,}4 \quad \rightarrow \quad P_1(11 \mid -3{,}4) \end{array}

b)
\begin{array}{rclll} 0 &=& -\dfrac{2}{5}x+1 &\vert & -1 \cr-1 &=& -\dfrac{2}{5}x &\vert & : \left(-\dfrac{2}{5}\right) \cr\dfrac{5}{2} &=& x && \rightarrow \quad P_2\left(\dfrac{5}{2} \mid 0\right) \end{array}


10)
a)
\begin{array}{rclll} f\left(-\dfrac{2}{5}\right) &=& -12 \cdot -\dfrac{2}{5}+\dfrac{11}{5} &=& 7 \quad \rightarrow \quad P_1\left(-\dfrac{2}{5} \mid 7\right) \end{array}

b)
\begin{array}{rclll} 4{,}2 &=& -12x+\dfrac{11}{5} &\vert & -\dfrac{11}{5} \cr2 &=& -12x &\vert & :(-12) \cr-\dfrac{1}{6} &=& x && \rightarrow \quad P_2\left(-\dfrac{1}{6} \mid 4{,}2\right) \end{array}

 

6. Aufgabe

Lagebeziehung von f_1(x) und f_2(x)
Wir prüfen zuerst, ob die Funktionen f_1(x) und f_2(x) die gleiche Steigung haben: m_1=3 und m_2=-7. Da die Steigungen offensichtlich unterschiedlich sind, können die Geraden weder parallel noch identisch sein. Sie müssen also einen Schnittpunkt, nennen wir ihn S(x_s \mid y_s), haben.

x-Wert vom Schnittpunkt berechnen:
\begin{array}{rclcl} f_1(x_s) &=& f_2(x_s) \cr 3x_s-6 &=& -7x_s-6 &\vert & +6 \cr 3x_s &=& -7x_s &\vert & +7x_s \cr 10x_s &=& 0 &\vert & :10 \cr x_s &=& 0\end{array}

y-Wert vom Schnittpunkt berechnen:
\begin{array}{rcl} f_1(x_s) &=& f_1(0) \cr &=& 3 \cdot 0 -6 \cr &=& -6 \end{array}

Die Koordinaten des Schnittpunkts sind S (0 \mid -6). Zur Überprüfung kann man den y-Wert mit f_2(x) noch einmal berechnen (und dabei hoffentlich das gleiche Ergebnis erhalten) oder eine Punktprobe durchführen.

Bemerkung: Wer in diesem Fall direkt sieht, dass beide Geraden den gleichen y-Achsenabschnitt haben, darf den Schnittpunkt auch einfach ablesen ... Mehr als diesen einen kann es hier ja nicht geben.


Lagebeziehung von f_1(x) und f_3(x)
Wir prüfen zuerst, ob f_1(x) und f_3(x) die gleiche Steigung haben: m_1=3 und m_3=\frac{27}{9}=3. Da die Steigungen gleich sind, können die Geraden parallel oder identisch sein. Einen einzelnen Schnittpunkt können sie nicht haben.

Zu prüfen ist jetzt, ob die Geraden parallel oder identisch sind. Dazu vergleichen wir die Schnittpunkte mit der y-Achse: n_1=-6 und n_3=1. Die beiden Geraden schneiden die y-Achse an unterschiedlichen Punkten. Sie sind somit nicht identisch, sondern verlaufen parallel.


Lagebeziehung von
f_1(x) und f_4(x)
Wir prüfen zuerst, ob f_1(x) und f_4(x) die gleiche Steigung haben: m_1=3 und m_4=-\frac{21}{3}=-7. Da die Steigungen offensichtlich unterschiedlich sind, können die Geraden weder parallel noch identisch sein. Sie müssen also einen Schnittpunkt, nennen wir ihn S(x_s \mid y_s), haben.

An den Geradengleichungen kann man ablesen, dass n_1=-6 und n_4=-6 ist. Beide Geraden schneiden die y-Achse bei -6. Damit hat der Schnittpunkt der Funktionen f_1(x) und f_4(x) die Koordinaten S (0 \mid -6).


Lagebeziehung von
f_2(x) und f_3(x)
Zunächst prüfen wir wieder, ob die Funktionen f_2(x) und f_3(x) die gleiche Steigung haben: m_2=-7 und m_3=\frac{27}{9}=3. Da die Steigungen offensichtlich unterschiedlich sind, können die Geraden weder parallel noch identisch sein. Sie müssen also einen Schnittpunkt S(x_s \mid y_s) haben.

x-Wert vom Schnittpunkt berechnen:
\begin{array}{rclcl} f_2(x_s) &=& f_3(x_s) \cr -7x_s-6 &=& \dfrac{27}{9}x_s+1 \cr -7x_s-6 &=& 3x_s+1 &\vert & +6 \cr -7x_s &=& 3x_s+7 &\vert & -3x_s \cr -10x_s &=& 7 &\vert & :(-10) \cr x_s &=& -\dfrac{7}{10}\end{array}

y-Wert vom Schnittpunkt berechnen:
\begin{array}{rcl} f_2(x_s) &=& f_2\left(-\dfrac{7}{10}\right) \cr &=& -7 \cdot \left(-\dfrac{7}{10}\right) -6 \cr &=& -\dfrac{11}{10} \end{array}

Die Koordinaten des Schnittpunkts sind S \left(-\dfrac{7}{10} \mid -\dfrac{11}{10}\right).


Lagebeziehung von f_2(x) und f_4(x)
Wir prüfen zuerst, ob f_2(x) und f_4(x) die gleiche Steigung haben: m_2=-7 und m_4=-\frac{21}{3}=-7. Da die Steigungen gleich sind, können die Geraden parallel oder identisch sein. Einen einzelnen Schnittpunkt können sie nicht haben.

Um zu prüfen, ob die Geraden parallel oder identisch sind, lesen wir den jeweiligen Schnittpunkt mit der y-Achse ab: n_2=-6 und n_4=-\frac{24}{4}=-6. Die beiden Geraden schneiden die y-Achse im selben Punkt. Sie sind also identisch.


Lagebeziehung von f_3(x) und f_4(x)
Wir prüfen wieder zuerst, ob f_3(x) und f_4(x) die gleiche Steigung haben: m_3=\frac{27}{9}=3 und m_4=-\frac{21}{3}=-7. Anhand der unterschiedlichen Steigungen ist zu erkennen, dass die Geraden weder parallel noch identisch sein können. Sie werden also einen Schnittpunkt S(x_s \mid y_s) haben.

x-Wert vom Schnittpunkt berechnen:
\begin{array}{rclcl} f_3(x_s) &=& f_4(x_s) \cr\dfrac{27}{9}x_s+1 &=& -\dfrac{21}{3}x_s-\dfrac{24}{4} \cr 3x_s+1 &=& -7x_s-6 &\vert & -1 \cr 3x_s &=& -7x_s-7 &\vert & +7x_s \cr 10x_s &=& -7 &\vert & :10 \cr x_s &=& -\dfrac{7}{10} \end{array}

y-Wert vom Schnittpunkt berechnen:
\begin{array}{rcl} f_3(x_s) &=& f_3\left(-\dfrac{7}{10}\right) \cr &=&\dfrac{27}{9} \cdot \left(-\dfrac{7}{10} \right)+1 \cr &=&3\cdot\left(-\dfrac{7}{10} \right)+\dfrac{10}{10}\cr &=&-\dfrac{11}{10}\end{array}

Die Koordinaten des Schnittpunkts sind S \left(-\dfrac{7}{10} \mid -\dfrac{11}{10}\right).

Bemerkung: Wer die Idee hatte, die Brüche in der Funktion vor dem Berechnen zu kürzen, ist vermutlich schneller zu diesem Ergebnis gekommen: f_4(x)=-\dfrac{21}{3}x-\dfrac{24}{4}=-7x-6. Diese Funktion kennen wir schon als f_2(x) ...

 

7. Aufgabe

f_1(x) = -4x+0 = -4x
Begründung:
Der Schnittpunkt mit der y-Achse liegt direkt im Nullpunkt, also bei 0.
Die Änderung des y-Wertes im Verhältnis zum x-Wert ist hier \dfrac{-4}{1}. Die Steigung ist also -4.
 
f_2(x) = 5x+2
Begründung:
Der Schnittpunkt mit der y-Achse liegt bei 2.
Die Änderung des y-Wertes im Verhältnis zum x-Wert ist hier \dfrac{5}{1}. Die Steigung ist also 5.
 
f_3(x) = \dfrac{1}{4}x-3
Begründung:
Der Schnittpunkt mit der y-Achse liegt bei -3.
Die Änderung des y-Wertes im Verhältnis zum x-Wert ist hier \dfrac{1}{4}. Die Steigung ist also \dfrac{1}{4}.
 
f_4(x) = 0x+ 7 = 7
Begründung:
Der Schnittpunkt mit der y-Achse liegt bei 7.
Die Änderung des y-Wertes im Verhältnis zum x-Wert ist hier \dfrac{0}{0}. Die Steigung ist also 0.
 
f_5(x) = -2x-6
Begründung:
Der Schnittpunkt mit der y-Achse liegt -6.
Die Änderung des y-Wertes im Verhältnis zum x-Wert ist hier \dfrac{-2}{1}. Die Steigung ist also -2.

 

8. Aufgabe

Sei L die Länge der Kerzen zum Zeitpunkt 0. Die Zeit ist dabei die unabhängige Variable und wird auf der x-Achse eingetragen. Die Länge der Kerze, die ja von der (Brenn-)Zeit abhängt, ist die abhängige Variable und wird auf der y-Achse eingetragen. Mathematisch kann man die erste Aussage daher als Punkt P(0 \mid L) notieren.

Für die erste Kerze kennen wir also den Punkt P_1(0 \mid L). Außerdem wissen wir, dass sie nach 8 Stunden die Länge 0 hat, was wir als Punkt Q_1(8 \mid 0) notieren können.

Für die zweite Kerze kennen wir ebenfalls den Punkt P_2(0 \mid L) und außerdem Q_2(4 \mid 0), weil sie nach 4 Stunden die Länge 0 hat.

Zeichnet man für jede Kerze die beiden Punkte in ein Koordinatensystem und verbindet sie, ergibt sich folgende Grafik (Bitte beachten Sie, dass die Längen-Achse keine Einheiten hat - diese ist ja in der Aufgabe nicht gegeben!):

Koordinatensystem mit Graphen zu dieser Aufgabe

Daraus kann man nun lineare Funktionen  f(x) = mx + n berechnen:

f_1(x):
Steigung: m=\dfrac{\Delta y}{\Delta x} = \dfrac{0-L}{8-0} = -\dfrac{L}{8}

y-Achsenabschnitt: n = L (Das sieht man z. B. in der Grafik.)

\Rightarrow f_1(x) = -\dfrac{L}{8}x+L =\left(-\dfrac{1}{8}x+1\right)L

f_2(x):

Steigung: m=\dfrac{\Delta y}{\Delta x} = \dfrac{0-L}{4-0} = -\dfrac{L}{4}

y-Achsenabschnitt: n = L (Kann man ebenfalls in der Grafik ablesen.)

\Rightarrow f_2(x) = -\dfrac{L}{4}x+L =\left(-\dfrac{1}{4}x+1\right)L


Gesucht ist nun der Zeitpunkt x_0, an dem die erste Kerze doppelt so lang ist wie die zweite. Anders gesagt: "2 mal die Länge der zweiten Kerze" ist genauso groß wie die Länge der ersten Kerze.

Mathematisch schreibt man:

\begin{array}{rclll} 2\left(-\dfrac{1}{4}x_0+1\right)L &=& \left(-\dfrac{1}{8}x_0+1\right)L &\vert& :L \cr\cr 2\left(-\dfrac{1}{4}x_0+1\right) &=& -\dfrac{1}{8}x_0+1 \cr\cr -\dfrac{1}{2}x_0+2 &=& -\dfrac{1}{8}x_0+1 &\vert& +\dfrac{1}{8}x_0-2 \cr\cr-\dfrac{3}{8}x_0 &=& -1 &\vert& :\left(-\dfrac{3}{8}\right) \cr\cr x_0 &=& \dfrac{8}{3}\end{array}

Nach \frac{8}{3} Stunden ist also die erste Kerze doppelt so lang ist wie die zweite. Das könnte man grundsätzlich so stehen lassen, meist würde man aber eher umrechnen und sagen: Nach 2 Stunden und 40 Minuten ist die erste Kerze doppelt so lang wie die zweite.

 

9. Aufgabe

Im dreidimensionalen Raum gibt es zunächst mal die gleichen Lagebeziehungen wie im zweidimensionalen: Die Geraden können parallel zueinander sein, sie können aufeinander liegen und sie können einen Schnittpunkt haben. Da eine Ebene ja Teil des dreidimensionalen Raumes ist, kann sich an diesen Möglichkeiten gar nichts ändern.

Durch die zusätzliche Dimension wird aber eine weitere Lagebeziehung möglich: Hier ist es möglich, dass zwei nicht parallele Geraden keine gemeinsamen Punkte haben. Dies wird als windschief bezeichnet.
Ein (leicht idealisiertes ...) Beispiel aus dem Alltag: Stellen Sie sich zwei schnurgerade Straßen vor. Die eine verbindet Berlin und Köln miteinander, die andere Hamburg und München. Verliefen beide auf dem Erdboden (also annähernd zweidimensional) gäbe es irgendwo in der Mitte Deutschlands eine Kreuzung; mathematisch gesehen wäre dies ein Schnittpunkt. Im Dreidimensionalen kann man sich vorstellen, dass die eine Straße auf einer Brücke in 10 \,\text{m} Höhe verläuft. Nun "schneiden" sich die Straßen nicht mehr; sie verlaufen aneinander vorbei. Da sie auch nicht parallel sind (sie verlaufen ja offensichtlich in unterschiedlichen Richtungen), sind sie also windschief.

7. Lineare Gleichungssysteme - Lernziele und typische Fehler

Nach Durcharbeiten dieses Kapitels sollten Sie folgende Lernziele erreicht haben:

  • Sie können zu einem linearen Gleichungssystem den passenden Definitionsbereich bestimmen.
  • Sie wissen, dass es lineare Gleichungssysteme mit keiner/einer/unendlich vielen Lösung(en) gibt und können anhand des Rechenweges feststellen, welcher Fall vorliegt.
  • Sie können lineare Gleichungssysteme mit zwei Gleichungen und zwei Unbekannten lösen (Additions-, Gleichsetzungs- bzw. Einsetzungsverfahren sowie grafisch).
  • Sie können die Lösungsmenge mathematisch korrekt notieren.
  • Sie können mithilfe der Probe überprüfen, ob die gefundene Lösung tatsächlich richtig ist.
  • Sie können entsprechende Textaufgaben lösen und die Lösungswege mathematisch vernünftig aufschreiben.
  • Für Profis: Sie können lineare Gleichungssysteme mit drei Gleichungen und drei Unbekannten lösen.


Typischer Fehler in diesem Kapitel ist:

  • Hier passieren häufig Flüchtigkeitsfehler. Erklärung


Für Online-Selbsttests zu diesem Thema und weitere Informationen zur Mathematikunterstützung an der TH Wildau nutzen Sie bitte den Moodle-Kursraum "SOS Mathematik - Brückenkurs".

Übersicht:

 

7.1 Lineare Gleichungssysteme - Aufgaben

1. Aufgabe

Lösen Sie die folgenden linearen Gleichungssysteme! Für alle Aufgaben ist \mathbb{D} = \mathbb{R} \times \mathbb{R}.

1)
\begin{array}{rcr} 3x+4y &=& -6 \cr -5x-2y &=& -4 \end{array}
  6)
\begin{array}{rcr} 3x-4y &=& -24 \cr 3x+3y &=& 18\end{array}

2)
\begin{array}{rcr} 2y+7z &=& 8 \cr -16y-56z &=& 2 \end{array}
  7)
\begin{array}{rcr} 52a+156y &=& -104 \cr -4a-12y &=& 8 \end{array}

3)
\begin{array}{rcr} \dfrac{1}{4}x+\dfrac{3}{8}y &=& -\dfrac{5}{8} \cr \cr -\dfrac{1}{4}x-\dfrac{2}{3}y &=& \dfrac{1}{3} \end{array}

  8)
\begin{array}{rcl} p &=& 24q-9 \cr \cr \dfrac{1}{3}p+3 &=& 8q \end{array}

4)
\begin{array}{rcr} 3x+2y &=& 9 \cr 7x+y &=& 32 \end{array}
  9)
\begin{array}{rcr} 6a-8b &=& 2 \cr -3a+12b &=& 1\end{array}

5)
\begin{array}{rcr} 5y-\dfrac{1}{2}z &=& 10 \cr \cr 5y-\dfrac{1}{2}z &=& -14 \end{array}
  10)
\begin{array}{rcr} \dfrac{1}{16}s+t &=& -12 \cr \cr \dfrac{1}{16}s -\dfrac{3}{2}t &=& 23\end{array}

 

2. Aufgabe

Wie viele Hühner und Schweine besitzt Herr Müller, wenn die Tiere zusammen 37 Köpfe und 90 Beine haben? Bestimmen Sie die Lösung rechnerisch und zeichnerisch!

 

3. Aufgabe

Ein Supermarkt bietet eine Großpackung Schokoladentäfelchen an, die zwei unterschiedliche Sorten enthält. Von der billigen Sorte kosten 20 Täfelchen 1{,}20 EUR; von der teuren kosten 10 Täfelchen 1{,}40 EUR. Die Großpackung kostet 11{,}20 EUR.
a) Wie viele Schokoladentafeln der billigen und der teuren Sorte könnte die Großpackung enthalten? Geben Sie verschiedene Möglichkeiten an!
b) Angenommen, die Großpackung enthält genau 100 Tafeln, wie viele davon gehören zur teuren Sorte?

 

4. Aufgabe

Lösen Sie die folgenden linearen Gleichungssysteme! Für alle Aufgaben ist \mathbb{D} = \mathbb{R} \times \mathbb{R} \times \mathbb{R}.

1)
\begin{array}{rcrcrcrl} 8x &+& 9y &-& 5z &=& -9 \cr -6x &-& 3y &+& 5z &=& -27 \cr \frac{1}{2}x &+& y &-& 2z &=& -10 \end{array}

  6)
 \begin{array}{rcrcrcrl} 4x &+& 2y &-& z &=& 1 \cr -8x &-& 6y &+& 8z &=& -13 \cr -4x &+& 10y &-& 5z &=& 9 \end{array}

2)
\begin{array}{rcrcrcrl} 7x &+& 5y &-& 10z &=& 12 \cr -28x &-& 35y &+& 16z &=& -23 \cr -21x &-& 30y &+& 6z &=& 9 \end{array}

  7)
 \begin{array}{rcrcrcrl} 7x &-& 5y &+& 2z &=& 16 \cr 14x &-& 7y & & &=& 14 \cr -3x & & &-& z &=& -3 \end{array}

3)
\begin{array}{rcrcrcrl} -\dfrac{3}{2}x_1 &+& \dfrac{1}{7}x_2 &+& x_3 &=& 6 \cr \cr \dfrac{5}{6}x_1 &+& \dfrac{1}{3}x_2 &-& \dfrac{1}{3}x_3 &=& 2 \cr \cr & & -\dfrac{4}{3}x_2 &-& \dfrac{5}{3}x_3 &=& -3 \end{array}

  8)
\begin{array}{rcrcrcrl} 2r &+& 2s &+& 3t &=& 7 \cr &-& 4s &-& 4t &=& 5 \cr 4r &+& 8s &+& 10t &=& 6 \end{array}

4)
\begin{array}{rcrcrcrl} -18x &-& 4y &-& 26z &=& 4 \cr 9x &+& 8y &+& 13z &=& -5 \cr 9x &-& 12y &+& 7z &=& 11 \end{array}

  9)
\begin{array}{rcrcrcrl} 4x &+& 5y &-& 8z &=& 21 \cr -2x&+& 3y &-& 6z &=& -17 \cr 2x &+& 8y &-& 14z &=& 4 \end{array}

5)
\begin{array}{rcrcrcrl} 11a &-& 8b &+& c &=& 37 \cr -a &-& 2b &+& 7c &=& -35 \cr 12a &+& 9b &-& 6c &=& 12 \end{array}

  10)
\begin{array}{rcrcrcrl} 3x_1 &-& 6x_2 &+& 9x_3 &=& 21 \cr -x_1 &+& 2x_2 &+& 4x_3 &=& 21 \cr -x_1 & & &+& 5x_3 &=& 21 \end{array}

Dieses Kapitel enthält die folgenden Themen:

 

7.2 Lineare Gleichungssysteme - Erklärungen

Lineare Gleichungssysteme allgemein

Ein lineares Gleichungssystem ist ein System, das aus mehreren linearen Gleichungen besteht, die gemeinsam gelöst werden sollen. Das bedeutet, dass Zahlen gesucht werden, die gleichzeitig Lösung von allen Gleichungen des Systems sind. Grundsätzlich ist die Zahl der Gleichungen und Variablen nicht beschränkt. Wir beschäftigen uns hier allerdings vor allem mit linearen Gleichungssystemen, die zwei lineare Gleichungen und zwei Variablen enthalten. Das reicht fürs Erste ...
Ein lineares Gleichungssystem kann beispielsweise folgendermaßen aussehen:
\begin{array}{crcl} \text{I} & 2x+6y &=& 14 \cr \text{II} & x+6y &=& 13 \end{array}
Manchmal schreibt man Gleichungssysteme auch in der sogenannten Matrixschreibweise. Dabei lässt man die Variablen und die Gleichheitszeichen weg. Dafür muss man dann mit der Anordnung der Zahlenwerte sehr sorgsam sein:
\left(\begin{matrix} 2 & 6 & \vert & 14 \cr 1 & 6 & \vert & 13 \end{matrix}\right)


Der Definitionsbereich: Auch bei Gleichungssystemen muss angegeben werden, aus welchen Zahlenbereichen die Variablen stammen. Da es zwei Variablen gibt, benötigt der Definitionsbereich \mathbb{D} auch zwei Mengen - für jede Variable eine. Notiert wird das mit dem Zeichen \times zwischen den Mengen.
Für unser Beispiel ist der Definitionsbereich: \mathbb{D} = \mathbb{R} \times \mathbb{R} (gesprochen: "D gleich R kreuz R"). Diese Schreibweise gibt an, dass zwei Zahlen gesucht sind und diese Zahlen beide den reellen Zahlen entstammen sollen. Natürlich kann es auch Fälle geben, bei denen die Mengen unterschiedlich sind.

Auch die Lösung (so vorhanden) besteht in diesem Fall aus zwei Zahlen, wieder für jede Variable eine. Genauer gesagt, besteht die Lösung aus einem geordneten Zahlenpaar (oder auch 2-Tupel), das folgendermaßen notiert wird: (x; y). Geordnet bedeutet dabei, dass diese beiden Werte nicht vertauscht werden dürfen.
Unser Beispiel-Gleichungssystem hat die Lösung \left(1; 2\right). Das werden wir unten noch ausrechen. Im Moment begnügen wir uns damit, das Ergebnis zu überprüfen. Sprich: Wir machen die Probe, indem wir für x die 1 und für y die 2 einsetzen:
\begin{array}{crcl} \text{I} & 2\cdot 1+6\cdot 2 &=& 14 \cr \text{II} & 1+6\cdot 2 &=& 13 \cr\cr \text{I} & 2+12 &=& 14 \cr \text{II} & 1+12 &=& 13 \cr\cr \text{I} & 14 &=& 14 \cr \text{II} & 13 &=& 13 \end{array}
STIMMT! Jetzt noch überprüfen, ob die Lösung im Definitionsbereich liegt: 1\in\mathbb{R} und 2\in\mathbb{R}. Beides in Ordnung! Dann müssen wir nur noch die Lösungsmenge aufschreiben und sind fertig. Die Lösungsmenge notiert man hier mit den runden Klammern des Zahlenpaars, um deutlich zu machen, dass die Anordnung wichtig ist, also \mathbb{L} = \{(1; 2)\} oder in allgemeiner Form \mathbb{L} = \{(x; y)\}.

Schauen wir noch eben, was passiert, wenn man die Werte vertauscht, also mit \left(2; 1\right) rechnet:
\begin{array}{crcl} \text{I} & 2\cdot 2+6\cdot 1 &=& 14 \cr \text{II} & 2+6\cdot 1 &=& 13 \cr\cr \text{I} & 4+6 &=& 14 \cr \text{II} & 2+6 &=& 13 \cr\cr \text{I} & 10 &=& 14 \cr \text{II} & 8 &=& 13 \end{array}
WIDERSPRUCH! Wie man sieht, löst das vertauschte Zahlenpaar \left(2; 1\right) unser Beispiel-Gleichungssystem nicht. Womit gezeigt wäre, dass man die Anordnung tatsächlich nicht ignorieren sollte ...


Bei linearen Gleichungssystemen mit mehr als zwei Variablen und Gleichungen werden diese Notationen entsprechend erweitert.

 

Lösbarkeit von linearen Gleichungssystemen

Lineare Gleichungssysteme können keine, eine oder unendlich viele Lösungen haben, d. h. sie können nicht, eindeutig oder mehrdeutig (und dann mit unendlich vielen Lösungen) lösbar sein. Da lineare Gleichungssysteme nun mal aus linearen Gleichungen bestehen, benötigen wir hier keine neuen Überlegungen, was die Lösbarkeit angeht. Wir können stattdessen ausnutzen, dass wir uns diese Gedanken bereits im Kapitel Lineare Gleichungen gemacht haben.
Die beste Voraussetzung für eine eindeutige Lösung ist, ein lineares Gleichungssystem mit genauso vielen Gleichungen wie Variablen.

 

Lösen linearer Gleichungssysteme

Wichtige Verfahren zum Lösen linearer Gleichungssysteme sind: das Additionsverfahren, das Gleichsetzungsverfahren und das Einsetzungsverfahren. Häufig ist auch eine grafische Lösung möglich. Grundsätzlich liefern alle Verfahren dasselbe Ergebnis, wenn man sie auf dasselbe lineare Gleichungssystem anwendet. Nur der Lösungsweg kann u. U. etwas länger und umständlicher werden, wenn man nicht den geschicktesten Weg wählt.

Primäres Ziel der Verfahren ist es, das lineare Gleichungssystem so umzuformen, dass in einer der Zeilen nur noch eine Variable vorhanden ist, da lineare Gleichungen mit einer Variablen (wie wir ja schon gesehen haben) problemlos gelöst werden können. Neben den Verfahren, die gleich besprochen werden, benötigt man dazu die Strategien zum Rechnen mit Variablen.

Für alle drei genannten Verfahren gilt: Die Lösungsmenge des Gleichungssystems darf sich während der Anwendung nicht ändern, sonst wären die Zahlen, die man berechnet, ja nicht Lösung des ursprünglichen Gleichungssystems. Es müssen also Äquivalenzumformungen verwendet werden. Zu den Äquivalenzumformungen, die für lineare Gleichungen grundsätzlich gelten, kommen hier noch weitere hinzu und zwar:

  • die Addition von einer Zeile des linearen Gleichungssystems zu einer anderen Zeile
  • das Vertauschen von zwei Zeilen des linearen Gleichungssystems

In den Beispielrechnungen, die gleich folgen werden, wird vor allem die Addition verschiedener Zeilen eine wichtige Rolle spielen.

Je komplexer das Gleichungssystem wird, desto größer ist die Gefahr von Flüchtigkeitsfehlern. Die Berechnung der Lösung an sich ist nicht übermäßig kompliziert (man benötigt nicht viel mehr als die vier Grundrechenarten ...), aber durch die Vielzahl an Umformungen und Berechnungsschritten verliert man leicht mal den Überblick - und wenn dann Fehler passieren, wird der Rest der Rechnung häufig sehr unschön ...
Es mag banal klingen: Achten Sie daher unbedingt darauf,

Und wenn die Zwischenergebnisse zu unhandlich werden: Sicherheitshalber den ersten Teil der Rechnung nochmal überprüfen ...

Zur Notation: Für einen besseren Überblick ist u. a. folgende Notation nützlich: Am Ende einer Gleichung hinter dem senkrechten Strich wird (wie immer) angegeben, welche Rechenoperation auf diese Zeile des Gleichungssystems angewendet wird. Die Nummerierung der Gleichungen (z. B. \text{I} und \text{II}) ist dafür da, dass man sie gut auseinanderhalten kann. Gleichzeitig deutet man damit an, welche Gleichungen wie zusammengerechnet werden (z. B. \text{I+II}).

 

Das Additionsverfahren

Idee: Die einzelnen Gleichungen werden mit entsprechenden Zahlen so multipliziert, dass die Koeffizienten vor einer der Variablen Gegenzahlen sind. Dann können die Gleichungen addiert werden mit der Folge, dass diese Variable den Koeffizienten 0 hat, also wegfällt. Es bleibt eine Gleichung mit einer Variablen übrig, die dann gelöst werden kann. Der so erhaltene Wert für die Variable wird in eine der Ausgangsgleichungen eingesetzt, damit die andere Variable berechnet werden kann.

Beispiel:

\begin{array}{crclllcc} \text{I} & 2x+6y &=& 14 \cr \text{II} & x+6y &=& 13 & \vert & \cdot (-2) \cr \cr \text{I} & 2x+6y &=& 14 \cr \text{II} & -2x-12y &=& -26 \cr \cr \text{I} & 2x+6y &=& 14 \cr \text{I+II} & 0x-6y &=& -12 & \vert & :(-6) & \Rightarrow & y = 2 \cr \cr y=2 \text{ in I} & 2x+6 \cdot 2 &=& 14 \cr & 2x+12 &=& 14 & \vert &-12 \cr & 2x &=& 2 & \vert & :2 & \Rightarrow & x = 1 \cr \cr & \mathbb{L} &=& \{\left(1; 2\right)\} \end{array}

 

Das Gleichsetzungsverfahren

Idee: Hat eine Variable in beiden Gleichungen denselben Koeffizienten, kann man diese Tatsache ausnutzen, indem man diese gleichen Summanden in ihren Gleichungen isoliert. Sprich: Man formt die Gleichungen so um, dass die gleichen Summanden jeweils alleine z. B. auf den linken Seiten der Gleichungen stehen. Man folgert: Sind die linken Seiten der beiden Gleichungen gleich, müssen es auch die rechten sein. Man kann diese also gleichsetzen. Auf den beiden rechten Seiten ist aber nur noch eine Variable enthalten, sodass man eine lineare Gleichung mit einer Variablen erhält. Diese löst man und setzt den erhaltenen Zahlenwert in einer der Ausgangsgleichungen ein, um die andere Variable zu berechnen.

Beispiel:

\begin{array}{crclllcc} \text{I} & 2x+6y &=& 14 & \vert & -2x \cr \text{II} & x+6y &=& 13 & \vert & -x \cr \cr \text{I} & 6y &=& 14-2x \cr \text{II} & 6y &=& 13-x \cr \cr \text{I = II} & 14-2x &=& 13-x & \vert & +x -14 \cr & -2x+x &=& 13-14 & \cr & -x &=& -1 & \vert & \cdot (-1) & \Rightarrow & x = 1 \cr \cr x=1 \text{ in I} & 6y &=& 14-2 \cdot 1 \cr & 6y &=& 12 & \vert & :6 & \Rightarrow & y = 2 \cr \cr & \mathbb{L} &=& \{\left(1; 2\right)\} \end{array}

 

Das Einsetzungsverfahren

Idee: Dieses Verfahren bietet sich an, wenn eine Variable in einer Gleichung den Koeffizienten 1 hat. (Hinweis: Den Koeffizienten 1 notiert man häufig nicht!) Man formt die Gleichung dann so um, dass die Variable mit dem Koeffizienten 1 alleine auf einer Seite der Gleichung steht. Dann hat man eine Möglichkeit gefunden, diese Variable mithilfe der anderen auszudrücken. Diese Umschreibung der Variablen kann man in die andere Gleichung einsetzen (Hinweis: Häufig sind hier Klammern nötig!) und erhält eine Gleichung, die nur noch eine Variable enthält. Wenn man diese gelöst hat, setzt man den gefundenen Wert in eine der ursprünglichen Gleichungen ein und kann so auch für die zweite Variable einen Wert berechnen.

Beispiel:

\begin{array}{crclllcc} \text{I} & 2x+6y &=& 14 \cr \text{II} & x+6y &=& 13 & \vert & -6y \cr \cr \text{I} & 2x+6y &=& 14 \cr \text{II} & x &=& 13-6y \cr \cr \text{II in I} & 2(13-6y)+6y &=& 14 \cr & 26-12y+6y &=& 14 & \vert & -26 \cr & -6y &=& -12 & \vert & :(-6) & \Rightarrow & y = 2 \cr \cr y=2 \text{ in II} & x &=& 13-6 \cdot 2 \cr & x &=& 13-12 & \Rightarrow & x = 1 \cr \cr & \mathbb{L} &=& \{\left(1; 2\right)\} \end{array}

 

Grafische Lösung

Idee: Bei der grafischen Lösung eines linearen Gleichungssystems fasst man die einzelnen linearen Gleichungen als lineare Funktionen auf. Evtl. muss man sie dafür ein wenig umformen, sodass sie in der Form y=\dots vorliegen. Anschließend kann man sie in ein Koordinatensystem eintragen und den Schnittpunkt bestimmen. Dieser ist die gesuchte Lösung.

Beispiel:

\begin{array}{crclllcc} \text{I} & 2x+6y &=& 14 & \vert & -2x \cr \text{II} & x+6y &=& 13& \vert & -x \cr \cr \text{I} & 6y &=& -2x+14 & \vert & : 6 \cr \text{II} & 6y &=& -x+13 & \vert & : 6 \cr \cr \cr \text{I} & y &=& -\dfrac{1}{3}x+\dfrac{7}{3} \cr \cr \text{II} & y &=& -\dfrac{1}{6}x+\dfrac{13}{6} \end{array}

In einem Koordinatensystem sehen diese beiden Funktionen dann so aus:

Grafische Lösung der Beispielaufgabe

Hier lässt sich die Lösung \left(1; 2\right) problemlos ablesen. Hätten wir ein Gleichungssystem mit der Lösung \left(3{,}4556; -17{,}98441\right) (soll vorkommen ...), würden wir mit dem grafischen Lösungsverfahren nicht glücklich werden ...

 

Wie es weitergeht

Natürlich gibt es auch lineare Gleichungssysteme mit mehr als zwei Gleichungen und Variablen. Diese sind (natürlich) aufwändiger zu lösen. Die gute Nachricht: Am Prinzip ändert sich nichts. Wenn Sie also verstanden haben, wie man lineare Gleichungssysteme mit zwei Gleichungen und zwei Variablen löst, wissen Sie auch, wie man lineare Gleichungssysteme mit zehn Gleichungen und zehn Variablen löst. Dass man das meist nicht mehr mit Stift und Papier machen möchte, ist eine andere Sache ...
Hier ein Beispiel eines linearen Gleichungssystems mit drei Gleichungen und drei Variablen:

Hinweis: Bitte beachten Sie, dass die Rechenoperationen hier nicht mehr hinter die Gleichungen geschrieben, sondern in die Nummerierung integriert wurden. Das hat den Grund, dass beim ersten Schritt Gleichung \text{I} einmal mit 3 und einmal mit -6 multipliziert werden muss, damit bei der Addition der Gleichungszeilen das entsprechende Element in den Zeilen \text{II} bzw. \text{III} wegfällt. Das alles hinter die Gleichung zu schreiben, wäre zu viel und unübersichtlich. Man schreibt es daher vor die neu entstandene Gleichungszeile. Ein kleiner Strich hinter einer Zeilennummer (z. B. \text{II'}) deutet an, dass es sich zwar noch um Gleichung \text{II} handelt, mit dieser Gleichung aber schon Berechnungen vorgenommen wurden. Es ist also nicht mehr die Originalgleichung.

Für den Definitionsbereich gilt \mathbb{D} = \mathbb{R} \times \mathbb{R} \times \mathbb{R}

\begin{array}{crcrcrcrl} \text{I} & x &+& y &+& 4z &=& -3 \cr \text{II} & -3x &+& 9y &-& z &=& -13 \cr \text{III} & 6x &-& 2y &+& 10z &=& 10 \cr \cr \text{I} & x &+& y &+& 4z &=& -3 \cr \text{3 I+II} & & & 12y &+& 11z &=& -22 \cr \text{-6 I+III} & & & -8y &-& 14z &=& 28 \cr \cr \text{I} & x &+& y &+& 4z &=& -3 \cr \text{II'} & & & 12y &+& 11z &=& -22 \cr \text{2 II'+3 III'} & & & & & -20z &=& 40 & \Rightarrow z = -2 \cr \cr z=-2 \text{ in II'} & & & 12y &+& 11 \cdot (-2) &=& -22 & \Rightarrow y = 0 \cr z=-2 \text{ und } y=0 \text{ in I} & x &+& 0 &+& 4 \cdot (-2) &=& -3 & \Rightarrow x = 5 \end{array}

Die Lösungsmenge lautet also \mathbb{L} = \{(5; 0; -2)\}

Sie haben vielleicht bemerkt, dass der Lösungsweg eine Erweiterung des oben beschriebenen Additionsverfahrens ist. Bei linearen Gleichungssystemen mit mehr als zwei Gleichungen und Variablen entscheidet man sich meist dafür (und nicht für das Gleichsetzungs- oder Einsetzungsverfahren), weil das Gleichungssystem damit überschaubarer bleibt - aber auch weil dieser Rechenweg die Grundlage für das wohl bekannteste Lösungsverfahren in diesem Zusammenhang ist: das gaußsche Eliminationsverfahren, benannt nach Carl Friedrich Gauß, welches auch deswegen von so großer Bedeutung ist, weil es sich gut in Computerprogrammen abbilden lässt. Damit ist dann auch gesichert, dass wir lineare Gleichungssysteme mit zehn Gleichungen und zehn Variablen tatsächlich nicht mehr mit Stift und Papier lösen müssen.

Übersicht:

 

7.3 Lineare Gleichungssysteme - Lösungen

1. Aufgabe

1) Lösung nach dem Additionsverfahren
\begin{array}{crclllcc} \text{I} & 3x+4y &=& -6 & \vert \cdot 5 \cr \text{II} & -5x-2y &=& -4 & \vert \cdot 3 \cr \cr \text{I} & 15x+20y &=& -30 \cr \text{II} & -15x-6y &=& -12 \cr \cr \text{I} & 15x+20y &=& -30 \cr \text{I+II} & 0x +14y &=& -42 & \Rightarrow y = -3 \cr \cr y=-3 \text{ in I} & 3x+4 \cdot (-3) &=& -6 &\vert +12 \cr & 3x &=& 6 & \Rightarrow x = 2 \cr \cr & \mathbb{L} &=& \{\left(2; -3\right)\} \end{array}


2) Lösung nach dem Additionsverfahren
\begin{array}{crclllcc} \text{I} & 2y+7z &=& 8 & \vert \cdot 8 \cr \text{II} & -16y-56z &=& 2 \cr \cr \text{I} & 16y+56z &=& 64 \cr \text{II} & -16y-56z &=& 2 \cr \cr \text{I} & 16y+56z &=& 64 \cr \text{I+II} & 0y+0z &=& 66 \cr \cr & \mathbb{L} &=& \emptyset \end{array}

Bemerkung: Es gibt keine Zahlen y und z, die jeweils mit 0 multipliziert und dann addiert, 66 ergeben. Das Produkt aus einer beliebigen reellen Zahl und 0 ist immer 0, die Summe von 0 und 0 ebenfalls. Daher ist diese Gleichung nicht lösbar.


3) Lösung nach dem Additionsverfahren
\begin{array}{crclllcc} \text{I} & \dfrac{1}{4}x+\dfrac{3}{8}y &=& -\dfrac{5}{8} \cr \cr \text{II} & -\dfrac{1}{4}x-\dfrac{2}{3}y &=& \dfrac{1}{3} \cr \cr \cr \text{I} & \dfrac{1}{4}x+\dfrac{3}{8}y &=& -\dfrac{5}{8} \cr \cr \text{I+II} & -\dfrac{7}{24}y &=& -\dfrac{7}{24} & \Rightarrow y = 1 \cr \cr \cr y=1 \text{ in I} & \dfrac{1}{4}x+\dfrac{3}{8} \cdot 1 &=& -\dfrac{5}{8} &\vert -\dfrac{3}{8} \cr \cr & \dfrac{1}{4}x &=& -1 & \Rightarrow x = -4 \cr \cr \cr & \mathbb{L} &=& \{\left(-4; 1\right)\} \end{array}


4) Lösung nach dem Einsetzungsverfahren
\begin{array}{crclllcc} \text{I} & 3x+2y &=& 9 \cr \text{II} & 7x+y &=& 32 & \vert -7x \cr \cr \text{I} & 3x+2y &=& 9 \cr \text{II} & y &=& 32-7x \cr \cr \text{II in I} & 3x+2 \cdot (32-7x) &=& 9 \cr & 3x+64-14x &=& 9 &\vert -64 \cr & -11x &=& -55 & \Rightarrow x = 5 \cr \cr x=5 \text{ in II} & y &=& 32-7 \cdot 5 & \Rightarrow y = -3 \cr \cr & \mathbb{L} &=& \{\left(5; -3 \right) \} \end{array}


5) Lösung nach dem Gleichsetzungsverfahren
\begin{array}{crclllcc} \text{I} & 5y-\dfrac{1}{2}z &=& 10 &\vert& -5y \cr \cr \text{II} & 5y-\dfrac{1}{2}z &=& -14 &\vert& -5y \cr \cr \cr \text{I} & -\dfrac{1}{2}z &=& 10-5y \cr \cr \text{II} & -\dfrac{1}{2}z &=& -14-5y \cr \cr \cr \text{I=II} & 10-5y &=& -14-5y &\vert& +5y \cr & 10 &=& -14 \cr \cr \cr & \mathbb{L} &=& \emptyset \end{array}

Bemerkung: 10=-14 ist ein Widerspruch. Daher ist dieses Gleichungssystem nicht lösbar. Das hätte man allerdings auch ohne Rechnung erkennen können, da auf den linken Seiten der beiden Gleichungen jeweils der gleiche Term stand, aber unterschiedliche Ergebnisse vorgegeben waren. Für solche Gleichungssysteme kann es keine Lösung geben.


6) Lösung nach dem Gleichsetzungsverfahren
\begin{array}{crclllcc} \text{I} & 3x-4y &=& -24 & \vert +4y \cr \text{II} & 3x+3y &=& 18 & \vert -3y \cr \cr \text{I} & 3x &=& -24+4y \cr \text{II} & 3x &=& 18-3y \cr \cr \text{I = II} & -24+4y &=& 18-3y &\vert +24+3y \cr & 4y+3y &=& 18+24 \cr & 7y &=& 42 & \Rightarrow y = 6 \cr \cr y=6 \text{ in II} & 3x &=& 18-3 \cdot 6 \cr & 3x &=& 0 & \Rightarrow x = 0 \cr \cr & \mathbb{L} &=& \{\left(0; 6\right)\} \end{array}


7) Lösung nach dem Additionsverfahren
\begin{array}{crclllcc} \text{I} & 52a+156y &=& -104 & \vert : 13 \cr \text{II} & -4a-12y &=& 8 \cr \cr \text{I} & 4a+12y &=& -8 \cr \text{II} & -4a-12y &=& 8 \cr \cr \text{I} & 4a+12y &=& -8 \cr \text{I+II} & 0a+0y &=& 0 \end{array}

Die Gleichung I+II ergibt für alle reellen Zahlen a und y eine wahre Aussage. Die Lösung dieses linearen Gleichungssystems hängt also nur von Gleichung I ab. Da diese lineare Gleichung allerdings zwei Variablen hat, bekommen wir keine eindeutige Lösung. Wir müssen so tun, als wäre eine Variable fix. Wir nehmen hier mal hier mal an, dass y als reelle Zahl festgelegt wäre (mit a würde es aber genauso funktionieren). y wird damit zu einem sogenannten Parameter. Dann können wir Gleichung I so umformen, dass a alleine auf einer Seite steht:

\begin{array}{crclllcc} \text{I} & 4a+12y &=& -8 & \vert & -12y \cr & 4a &=& -8-12y & \vert & :4 \cr & a &=& -2-3y \cr \cr & \mathbb{L} &=& \{\left(-2-3y; y\right) \text{ mit }y \in \mathbb{R}\} \end{array}

Wir bekommen also nur dann eine Lösung des Gleichungssystems, wenn a=-2-3y ist. Da wir y ja selbst bestimmen dürfen, finden wir von diesen Zahlenpaaren dann aber unendlich viele, z. B.

y=-2 \;\;\Rightarrow\;\; a = -2-3 \cdot (-2) = 4
Probe:
\begin{matrix} \text{I} & 52 \cdot 4+156 \cdot (-2) &=& 208-312 &=& -104 \cr \text{II} & -4 \cdot 4-12 \cdot (-2) &=& -16+24 &=& 8 \end{matrix}
Für a=4 und y=-2 ergeben beide Gleichungen wahre Aussagen.

y=0 \;\;\Rightarrow\;\; a = -2-3 \cdot 0 = -2
Probe:
\begin{array}{crclcc} \text{I} & 52 \cdot (-2)+156 \cdot 0 &=& -104+0 &=& -104 \cr \text{II} & -4 \cdot (-2)-12 \cdot 0 &=& 8-0 &=& 8 \end{array}
Für a=-2 und y=0 ergeben beide Gleichungen wahre Aussagen.

y=5 \;\;\Rightarrow\;\; a = -2-3 \cdot 5 = -17
Probe:
\begin{array}{crclcc} \text{I} & 52 \cdot (-17)+156 \cdot 5 &=& -884+780 &=& -104 \cr \text{II} & -4 \cdot (-17)-12 \cdot 5 &=& 68-60 &=& 8 \end{array}
Für a=-17 und y=5 ergeben beide Gleichungen wahre Aussagen.

Und viele weitere mehr ...


8) Lösung nach dem Einsetzungsverfahren
\begin{array}{crclllcc} \text{I} & p &=& 24q-9 \cr \text{II} & \dfrac{1}{3}p+3 &=& 8q \cr \cr\text{I in II} & \dfrac{1}{3} \left(24q-9\right)+3 &=& 8q \cr & 8q-3+3 &=& 8q&\vert - 8q \cr & 0 &=& 0q \end{array}

Auch dieses Gleichungssystem ist offensichtlich mehrdeutig lösbar. Hier müssen wir aber im Gegensatz zu Aufgabe 7) gar nicht weiter umformen, da Gleichung I - netterweise - schon in der richtigen Form vorliegt.
Die Lösungsmenge lautet also \mathbb{L} = \{\left(24q-9; q \right) \text{ mit }q \in \mathbb{R}\}


9) Lösung nach dem Additionsverfahren
\begin{array}{crclllcc} \text{I} & 6a-8b &=& 2 \cr \text{II} & -3a+12b &=& 1 & \vert \cdot 2 \cr \cr \text{I} & 6a-8b &=& 2 \cr \text{II} & -6a+24b &=& 2 \cr \cr \text{I} & 6a-8b &=& 2 \cr \text{I+II} & 16b &=& 4 & \Rightarrow b = \dfrac{1}{4} \cr \cr b=\frac{1}{4} \text{ in I} & 6a-8 \cdot \dfrac{1}{4} &=& 2 \cr & 6a -2 &=& 2 &\vert +2 \cr & 6a &=& 4 & \Rightarrow a = \dfrac{2}{3} \cr \cr & \mathbb{L} &=& \left\{\left(\dfrac{2}{3}; \dfrac{1}{4}\right)\right\} \end{array}


10) Lösung nach dem Gleichsetzungsverfahren
\begin{array}{crclllcc}\text{I} & \dfrac{1}{16}s+t &=& -12 &\vert -t \cr \cr \text{II} & \dfrac{1}{16}s-\dfrac{3}{2}t &=& 23 &\vert +\dfrac{3}{2}t \cr \cr \cr \text{I} & \dfrac{1}{16}s&=& -12-t \cr \cr \text{II} & \dfrac{1}{16}s &=& 23+\dfrac{3}{2}t \cr \cr \cr \text{I = II} & -12-t &=& 23+\dfrac{3}{2}t &\vert -\dfrac{3}{2}t+12 \cr \cr & -t-\dfrac{3}{2}t &=& 23+12 \cr \cr & -\dfrac{5}{2}t &=& 35 & \Rightarrow t=-14 \cr \cr \cr t=-14 \text{ in I} & \dfrac{1}{16}s-14 &=& -12 &\vert +14 \cr \cr & \dfrac{1}{16}s &=& 2 & \Rightarrow s = 32 \cr \cr \cr & \mathbb{L} &=& \{\left(32; -14\right)\} \end{array}

 

2. Aufgabe

Sei h die Anzahl der Hühner und s die Anzahl der Schweine (siehe Bemerkung zu Textaufgaben).

Dann gilt:
I h+s=37
Denn jedes der Tiere hat genau einen Kopf.
II 2h+4s=90
Denn jedes Huhn hat 2 Beine, also haben h Hühner zusammen 2h Beine. Schweine haben 4 Beine, also haben s Schweine 4s Beine. Zusammen haben alle Tiere folglich 2h+4s Beine.

Rechnerische Lösung (Additionsverfahren):
Zu lösen ist also das folgende lineare Gleichungssystem:
\begin{array}{crclllcc} \text{I} & h+s &=& 37 & \vert \cdot (-2) \cr \text{II} & 2h+4s &=& 90 \cr \cr \text{I} & -2h-2s &=& -74 \cr \text{II} & 2h+4s &=& 90 \cr \cr \text{I} & -2h-2s &=& -74 \cr \text{I+II} & 2s &=& 16 & \Rightarrow s = 8 \cr \cr s = 8 \text{ in I} & h+8 &=& 37 & \Rightarrow h = 29 \end{array}

Herr Müller hat 29 Hühner und 8 Schweine.


Zeichnerische Lösung:
Man formt die Gleichungen I und II von oben so um, dass sie der allgemeinen Geradengleichung entsprechen, nämlich:
I h+s=37 \quad \Rightarrow \quad f_1(h)=s=37-h

II 2h+4s=90 \quad \Rightarrow \quad f_2(h)=s=\dfrac{45}{2}-\dfrac{1}{2}h

Diese beiden Geraden können dann in ein Koordinatensystem eingetragen und der Schnittpunkt abgelesen werden.

die entsprechenden zwei Geraden im Koordinatensystem

Der Schnittpunkt der beiden Geraden liegt bei S(29 \mid 8). h, also die Anzahl der Hühner, ist 29 und s, also die Anzahl der Schweine, ist 8. Jedes andere Ergebnis hätte uns stutzig machen sollen ...

Da in diesem Zusammenhang offensichtlicherweise nur ganzzahlige Lösungen sinnvoll sind, ist die folgende Grafik, in der keine Geraden, sondern einzelne Punkte an den entsprechenden Stellen eingezeichnet wurden, besser:

2 Gerade im Koordinatensystem mit Punkte gezeichnet

 

3. Aufgabe

Sei b die Anzahl der billigen Schokoladentäfelchen und t die Anzahl der teuren Schokoladentäfelchen (siehe Bemerkung zu Textaufgaben).
Eine billige Schokolade kostet laut Aufgabenstellung 0{,}06 EUR, eine teure 0{,}14 EUR.

a)
Es gilt: 0{,}06b + 0{,}14t = 11{,}20
Der Preis für eine billigen Schokolade mal die Anzahl der billigen Schokoladentafeln plus den Preis für eine teure Schokolade mal die Anzahl der teuren Tafeln ergibt den Gesamtpreis.

Mögliche Ergebnisse (b; t) sind also: (0; 80), (7; 77), (14; 74), (21; 71),…, (161; 11), (168; 8), (175; 5), (182; 2)
Denn 0{,}06 \cdot 0+0{,}14 \cdot 80 = 11{,}20 usw. Das findet man "einfach" durch Einsetzen und Ausprobieren heraus.

Bitte achten Sie darauf, dass hier nur ganzzahlige Angaben infrage kommen!
 
b)
Zur oberen Gleichung kommt dazu: b+t = 100
Denn die Gesamtzahl der Schokoladentäfelchen soll 100 betragen.

Es ergibt sich das folgende lineare Gleichungssystem, das mit dem Einsetzungsverfahren gelöst wird:
\begin{array}{crclcl} \text{I} & 0{,}06b+0{,}14t &=& 11{,}20 \cr \text{II} & b+t &=& 100 &\vert& -t \cr \cr \text{I} & 0{,}06b+0{,}14t &=& 11{,}20 \cr \text{II} & b &=& 100-t \cr \cr \text{II in I} & 0{,}06 \cdot (100-t)+0{,}14t &=&11{,}20 \cr & 6-0{,}06t+0{,}14t &=& 11{,}20 &\vert& -6 \cr & 0{,}08t &=& 5{,}20 &\vert& : 0{,}08 \cr & t &=& 65 \end{array}

Es sind 65 teure Schokoladen in der Großpackung enthalten.

Bemerkung: Da nur nach der Anzahl der teuren Schokoladentäfelchen gefragt war, kann die Bearbeitung der Aufgabe hier abgebrochen werden. Hätte das Gleichungssystem vollständig gelöst werden sollen, müsste auch für b ein Wert ermittelt werden.

 

4. Aufgabe

1)
\begin{array}{crcrcrcrl} \text{I} & 8x &+& 9y &-& 5z &=& -9 \cr \text{II} & -6x &-& 3y &+& 5z &=& -27 \cr \text{III} & \frac{1}{2}x &+& y &-& 2z &=& -10 \cr \cr \text{I} & 8x &+& 9y &-& 5z &=& -9 \cr \text{3 I+4 II} & & & 15y &+& 5z &=& -135 \cr \text{I-16 III} & & & -7y &+& 27z &=& 151 \cr \cr \text{I} & 8x &+& 9y &-& 5z &=& -9 \cr \text{II'} & & & 15y &+& 5z &=& -135 \cr \text{7 II'+15 III'} & & & & & 440z &=& 1320 & \Rightarrow z = 3 \cr \cr z = 3 \text{ in II'} & & & 15y &+& 5 \cdot 3 &=& -135 & \Rightarrow y = -10 \cr z = 3 \text{ und } y = -10 \text{ in I} & 8x &+& 9 \cdot \left(-10\right) &-& 5 \cdot 3 &=& -9 & \Rightarrow x = 12 \end{array}

Die Lösungsmenge lautet also \mathbb{L} = \{\left(12; -10; 3\right)\}


2)
\begin{array}{crcrcrcrl} \text{I} & 7x &+& 5y &-& 10z &=& 12 \cr \text{II} & -28x &-& 35y &+& 16z &=& -23 \cr \text{III} & -21x &-& 30y &+& 6z &=& 9 \cr \cr \text{I} & 7x &+& 5y &-& 10z &=& 12 \cr \text{4 I+II} & & & -15y &-& 24z &=& 25 \cr \text{3I+III} & & & -15y &-& 24z &=& 45 \cr \cr \text{I} & 7x &+& 5y &-& 10z &=& 12 \cr \text{II'} & & & -15y &-& 24z &=& 25 \cr \text{II'-III'} & & & & & 0 &=& -20 \end{array}

Die Lösungsmenge lautet also \mathbb{L} = \emptyset


3)
\begin{array}{crcrcrcrl} \text{I} & -\dfrac{3}{2}a &+& \dfrac{1}{7}b &+& c &=& 6 \cr \cr \text{II} & \dfrac{5}{6}a &+& \dfrac{1}{3}b &-& \dfrac{1}{3}c &=& 2 \cr \cr \text{III} & & & -\dfrac{4}{3}b &-& \dfrac{5}{3}c &=& -3 \cr \cr \cr \text{14 I} & -21a &+& 2b &+& 14c &=& 84 \cr \text{6 II} & 5a &+& 2b &-& 2c &=& 12 \cr \text{3 III} & & & -4b &-& 5c &=& -9 \cr \cr \text{I} & -21a &+& 2b &+& 14c &=& 84 \cr \text{5 I+21 II} & & & 52b &+& 28c &=& 672 \cr \text{III} & & & -4b &-& 5c &=& -9 \cr \cr \text{I} & -21a &+& 2b &+& 14c &=& 84 \cr \text{II'} & & & 52b &+& 28c &=& 672 \cr \text{II'+13 III'} & & & & & -37c &=& 555 & \Rightarrow c = -15 \cr \cr c = -15 \text{ in II'} & & & 52b &+& 28 \cdot \left(-15\right) &=& 672 & \Rightarrow b = 21 \cr c = -15 \text{ und } b = 21 \text{ in I} & -21a &+& 2 \cdot 21 &+& 14 \cdot \left(-15\right) &=& 84 & \Rightarrow a = -12 \end{array}

Die Lösungsmenge lautet also \mathbb{L} = \{\left(-12; 21; -15\right)\}

Bemerkung: Die ersten Umformungen (Multiplikation jeweils mit dem Hauptnenner) wurden durchgeführt, damit in der Folgerechnung keine Brüche mehr auftauchen. Allerdings werden dadurch die Zahlenwerte natürlich größer ...


4)
\begin{array}{crcrcrcrl} \text{I} & -18x &-& 4y &-& 26z &=& 4 \cr \text{II} & 9x &+& 8y &+& 13z &=& -5 \cr \text{III} & 9x &-& 12y &+& 7z &=& 11 \cr \cr \text{I} & -18x &-& 4y &-& 26z &=& 4 \cr \text{I+2 II} & & & 12y & & &=& -6 & \Rightarrow y = -\dfrac{1}{2} \cr \text{I+2 III} & & & -28y &-& 12z &=& 26 \cr \cr \cr y =-\frac{1}{2} \text{ in III'} & & & -28\cdot \left(-\dfrac{1}{2}\right) &-& 12z &=& 26 & \Rightarrow z = -1 \cr \cr y =-\frac{1}{2} \text{ und }z = -1 \text{ in I} & -18x &-& 4 \cdot \left(-\dfrac{1}{2}\right) &-& 26 \cdot \left(-1\right) &=& 4 & \Rightarrow x = \dfrac{4}{3} \end{array}

Die Lösungsmenge lautet also \mathbb{L} = \left\{\left(\dfrac{4}{3}; -\dfrac{1}{2}; -1\right)\right\}


5)
\begin{array}{crcrcrcrl} \text{I} & 11a &-& 8b &+& c &=& 37 \cr \text{II} & -a &-& 2b &+& 7c &=& -35 \cr \text{III} & 12a &+& 9b &-& 6c &=& 12 \cr \cr \text{II} & -a &-& 2b &+& 7c &=& -35 \cr \text{I} & 11a &-& 8b &+& c &=& 37 \cr \text{III} & 12a &+& 9b &-& 6c &=& 12 \cr \cr \text{II} & -a &-& 2b &+& 7c &=& -35 \cr \text{11 II+I} & & & -30b &+& 78c &=& -348 \cr \text{12 II+III} & & & -15b &+& 78c &=& -408 \cr \cr \text{II} & -a &-& 2b &+& 7c &=& -35 \cr \text{I'} & & & -30b &+& 78c &=& -348\cr \text{I'-2 III'} & & & & & -78c &=& 468 & \Rightarrow c = -6 \cr \cr c = -6 \text{ in I'} & & & -30b &+& 78\cdot \left(-6\right) &=& -348 & \Rightarrow b = -4 \cr c = -6 \text{ und } b = -3 \text{ in II} & -a &-& 2\cdot \left(-4\right) &+& 7\cdot \left(-6\right) &=& -35 & \Rightarrow a = 1 \end{array}

Die Lösungsmenge lautet also \mathbb{L} = \{\left(1; -4; -6\right)\}

Bemerkung: Im ersten Schritt wurden nur die ersten beiden Gleichungen vertauscht (Sie sehen das an der Nummerierung der Gleichungen.), da die weiteren Umformungen mit dem Koeffizienten -1 wie in Gleichung \text{II} einfacher durchzuführen sind.


6)
\begin{array}{crcrcrcrl} \text{I} & 4x &+& 2y &-& z &=& 1 \cr \text{II} & -8x &-& 6y &+& 8z &=& -13 \cr \text{III} & -4x &+& 10y &-& 5z &=& 9 \cr \cr \text{I} & 4x &+& 2y &-& z &=& 1 \cr \text{2I+II} & & & -2y &+& 6z &=& -11 \cr \text{I+III} & & & 12y &-& 6z &=& 10 \cr \cr \text{I} & 4x &+& 2y &-& z &=& 1 \cr \text{II'} & & & -2y &+& 6z &=& -11 \cr \text{II'+III'} & & & 10y & & &=& -1 & \Rightarrow y = -\dfrac{1}{10} \cr \cr \cr y = -\frac{1}{10} \text{ in II'} & & & -2\cdot \left( -\dfrac{1}{10} \right) &+& 6z &=& -1 & \Rightarrow z = -\dfrac{28}{15} \cr \cr y = -\frac{1}{10} \text{ und } z = -\frac{28}{15} \text{ in I} & 4x &+& 2\cdot \left( -\dfrac{1}{10} \right) &-& \left(-\dfrac{28}{15}\right) &=& 1 & \Rightarrow x = -\dfrac{1}{6} \end{array}

Die Lösungsmenge lautet also \mathbb{L} = \left\{\left(-\dfrac{1}{6}; -\dfrac{1}{10}; -\dfrac{28}{15}\right)\right\}


7)
\begin{array}{crcrcrcrl} \text{I} & 7x &-& 5y &+& 2z &=& 16 \cr \text{II} & 14x &-& 7y & & &=& 14 \cr \text{III} & -3x & & &-& z &=& -3 \cr \cr \text{I} & 7x &-& 5y &+& 2z &=& 16 \cr \text{II} & 14x &-& 7y & & &=& 14 \cr \text{I+2III} & x &-& 5y & & &=& 10 \cr \cr \text{I} & 7x &-& 5y &+& 2z &=& 16 \cr \text{II} & x &-& 5y & & &=& 10 \cr \text{II-14III'} & & & 63y & & &=& -126 & \Rightarrow y = -2 \cr \cr y = -2 \text{ in II} & & & x &-& 5\cdot \left(-2\right) &=& 10 & \Rightarrow x = 0 \cr x = 0 \text{ und } y = -2 \text{ in I} & 7\cdot0 &-& 5\cdot \left(-2\right) &+& 2z &=& 16 & \Rightarrow z = 3 \end{array}

Die Lösungsmenge lautet also \mathbb{L} = \{\left(0; -2; 3\right)\}


8)
\begin{array}{crcrcrcrl} \text{I} & 2r &+& 2s &+& 3t &=& 7 \cr \text{II} & & & -4s &-& 4t &=& 5 \cr \text{III} & 4r &+& 8s &+& 10t &=& 6 \cr \cr \text{I} & 2r &+& 2s &+& 3t &=& 7 \cr \text{II} & & & -4s &-& 4t &=& 5 \cr \text{2I-III} & & & -4s &-& 4t &=& 8 \cr \cr \text{I} & 2r &+& 2s &+& 3t &=& 7 \cr \text{II} & & & 4s &+& 4t &=& 5 \cr \text{II-III'} & & & & & 0 &=& -3 \end{array}

Die Lösungsmenge lautet also \mathbb{L} = \emptyset


9)
\begin{array}{crcrcrcrl} \text{I} & 4x &+& 5y &-& 8z &=& 21 \cr \text{II} & -2x &+& 3y &-& 6z &=& -17 \cr \text{III} & 2x &+& 8y &-& 14z &=& 4 \cr \cr \text{I} & 4x &+& 5y &-& 8z &=& 21 \cr \text{I+2II} & & & 11y &-& 20z &=& -13 \cr \text{II+III} & & & 11y &-& 20z &=& -13 \cr \cr \text{I} & 4x &+& 5y &-& 8z &=& 21 \cr \text{II'} & & & 11y &-& 20z &=& -13 \cr \text{II'-III'} & & & 0y &+& 0z &=& 0 & \end{array}

Die Gleichung II'+III' ergibt für alle reellen Zahlen y und z eine wahre Aussage. Die Lösung dieses linearen Gleichungssystems hängt also nur von den Gleichungen I und II' ab. Da diese beiden linearen Gleichungen allerdings drei Variablen haben, bekommen wir keine eindeutige Lösung. Wir müssen so tun, als wäre eine Variable aus der Gleichung II' (das ist die, die nur noch zwei Variablen enthält) fix. Wir nehmen hier mal hier mal an, dass z als reelle Zahl festgelegt wäre (mit y würde es aber genauso funktionieren). z wird damit zu einem sogenannten Parameter. Dann können wir Gleichung II' so umformen, dass y alleine auf einer Seite steht:

\begin{array}{crcrcrcl} \text{II'} & 11y &-& 20z &=& -13 & \vert & +20z \cr & 11y & & &=& -13+20z & \vert & :11 \cr\cr&&& y &=& -\dfrac{13}{11}+\dfrac{20}{11}z \end{array}

Damit wissen wir, wie sich y berechnet lässt, vorausgesetzt wir haben einen Wert für z. Bleibt die Frage, was mit x ist. Dafür nehmen wir die dritte Gleichung dieses Gleichungssystems, also Gleichung I, und setzten y = -\frac{13}{11}+\frac{20}{11}z und z ein:

\begin{array}{rcrcrcrcrl} \text{I} & 4x &+& 5y &-& 8z &=& 21 \cr \cr & 4x &+& 5\left(-\dfrac{13}{11}+\dfrac{20}{11}z\right) &-& 8z &=& 21 \cr \cr &4x &-& \dfrac{65}{11}+\dfrac{100}{11}z &-& 8z &=& 21 & \vert &+\dfrac{65}{11} \cr \cr & 4x & & &+& \dfrac{12}{11}z &=& 21+\dfrac{65}{11} & \vert & -\dfrac{12}{11}z \cr \cr & 4x& & & & &=& \dfrac{296}{11}-\dfrac{12}{11}z & \vert & :4 \cr \cr & & & & & x &=& \dfrac{74}{11}-\dfrac{3}{11}z & \end{array}

Die Lösungsmenge lautet also \mathbb{L} = \left\{\left(\dfrac{74}{11}-\dfrac{3}{11}z; -\dfrac{13}{11}+\dfrac{20}{11}z; z\right) \text{ mit } z \in \mathbb{R} \right\}


10)
\begin{array}{crcrcrcrl} \text{I} & 3x_1 &-& 6x_2 &+& 9x_3 &=& 21 \cr \text{II} & -x_1 &+& 2x_2 &+& 4x_3 &=& 21 \cr \text{III} & -x_1 & & &+& 5x_3 &=& 21 \cr \cr \text{I} & 3x_1 &-& 6x_2 &+& 9x_3 &=& 21 \cr \text{I + 3II} & & & & & 21x_3 &=& 84 &\Rightarrow x_3=4 \cr \text{III} & -x_1 & & &+& 5x_3 &=& 21 \cr\cr x_3=4 \text{ in III} & -x_1 & & &+& 5\cdot 1 &=& 21 &\Rightarrow x_1=-1 \cr x_1=-1 \text{ und } x_3=4 \text{ in I} & 3\cdot (-1) &-& 6 \cdot x_2 &+& 9\cdot 4 &=& 21 &\Rightarrow x_2=2 \end{array}

Die Lösungsmenge lautet also \mathbb{L} = \{\left(-1; 2; 4\right)\}

8. Potenzen, Wurzeln, Logarithmen - Lernziele und typische Fehler

Nach Durcharbeiten dieses Kapitels sollten Sie folgende Lernziele erreicht haben:

  • Sie kennen die grundlegende Definition einer Potenz, inkl. der Begrifflichkeiten.
  • Sie wissen, dass Potenzrechnung Vorrang vor Punkt- und Strichrechnung hat und wenden diese Regel in Rechnungen an.
  • Sie können die grundlegenden Gesetze der Potenzrechnung anwenden.
  • Sie können die binomischen Formeln "in beiden Richtungen" anwenden.
  • Sie kennen den Zusammenhang zwischen Potenzen, Wurzeln und Logarithmen (Umkehroperationen).
  • Sie können mit Wurzeln und Logarithmen rechnen.
  • Sie können für Wurzel- und Logarithmusterme passende Definitionsbereiche bestimmen.
  • Sie können komplexere Terme (auch Doppelbrüche) vereinfachen.
  • Sie können Terme aus (einfachen) Textaufgaben ableiten.
  • Sie können entsprechende Textaufgaben lösen und die Lösungswege mathematisch korrekt aufschreiben.


Typische Fehler in diesem Kapitel sind:

  • Die Rangfolge der Rechenoperationen wird nicht beachtet, z. B.
    • Es wird aus Summen gekürzt. Erklärung
    • Das Addieren/Subtrahieren von Potenzen ändern den Exponenten. Erklärung
    • Beim Potenzieren von Termen in Klammern werden Teile des Terms nicht mit potenziert. Meist betrifft dies den Koeffizienten. Erklärung
  • Die binomischen Formeln, vor allem in der Anwendung "von rechts nach links", werden nicht so gut verinnerlicht, dass sie im Kontext angewendet werden können. Erklärung 1 - Erklärung 2 - Erklärung 3
  • Die Potenzgesetze werden nicht korrekt angewendet. Erklärung 1 - Erklärung 2
  • Wurzelterme werden nicht korrekt von Wurzel- in Potenzschreibweise (und umgekehrt) umgeformt. Erklärung
  • Es werden Wurzeln aus Summen gezogen. Erklärung
  • Die Logarithmengesetze werden nicht korrekt angewendet. Erklärung 1 - Erklärung 2


Für Online-Selbsttests zu diesem Thema und weitere Informationen zur Mathematikunterstützung an der TH Wildau nutzen Sie bitte den Moodle-Kursraum "SOS Mathematik - Brückenkurs".

Übersicht:

 

8.1 Potenzen, Wurzeln, Logarithmen - Aufgaben

Eine Bemerkung vorneweg: Bitte trainieren Sie die Aufgaben in diesem Kapitel sehr sorgfältig! Sie sind - zugegebenermaßen - nicht besonders spannend, aber sie sind die Grundlage für vieles, was in den nächsten Kapiteln auf Sie zukommen wird. Die Erfahrung hat gezeigt, dass diejenigen, die diese Aufgaben nur zur Hälfte bewältigen, bei späteren Kapiteln vielleicht noch 10\,\% schaffen ...

 

1. Aufgabe

Schreiben Sie die folgenden Zahlen als Potenzen mit möglichst kleiner Basis b (für 1) bis 8): b\in \mathbb{N}, für 9) bis 10): b\in \mathbb{Q})!

1) 400

  6) 1.331

2) 100.000

  7) 196

3) 256

  8) 98

4) 121

  9) \dfrac{1}{64}

5) 81

  10) \dfrac{81}{625}

 

2. Aufgabe

Fassen Sie zusammen!

1) 2\sqrt{x} + 3\sqrt{x}


2) 2\sqrt{x} - 3\sqrt{x}


3) 2\sqrt{x} \cdot 3\sqrt{x}


4) 2\sqrt{x} : 3\sqrt{x}

 

3. Aufgabe

Lösen Sie die Klammerterme auf! Verwenden Sie dabei, soweit es geht, die binomischen Formeln.

1) \left(12x+7y\right)^2   11) -\dfrac{1}{5}\left(-10s+11t\right)\left(10s-11t\right)

2) 10\left(6k+8l\right)^2

  12) -\left(8n-q\right)\left(q+8n\right)

3) \left(\dfrac{1}{2}x-z\right)^2   13) 3\left(x-2\right)\left(x+2\right)\left(x+3\right)

4) \left(-4a+7b\right)\left(-7a+4b\right)   14) -15\left(a+b\right)^2+30ba

5) -\left(4m-9p\right)^2   15) \left(\dfrac{1}{4}x-1\right)^2 \left(8x+16\right)

6) \left(12a-b\right)\left(12a+b\right)   16) \left(x+y\right)^2-\left(x-y\right)^2-4xy

7) \dfrac{3}{4}\left(10y-\dfrac{8}{9}z\right)\left(-\dfrac{8z}{9}+10y \right)   17) \left(3x+5y\right)^2+4\left(3x-5y\right)^2

8) 46\left(\dfrac{5}{6}x-\dfrac{5}{3}z\right)\left(\dfrac{6}{5}x-\dfrac{3} {5}z\right)   18) \dfrac{\left(9a-5b\right)^2}{\left(81a^2-90ab+25b^2\right)^2}

9) \left(p+5q\right)\left(5q+p\right)   19) \left(\dfrac{7h-9n}{7h+9n}\right)^2

10) \left(-x+3z\right)\left(-x-3z\right)   20) 5\cdot\dfrac{64s^2-256t^2}{\left(8s-16t\right)^2}

 

4. Aufgabe

Fassen Sie - wenn möglich - mithilfe der binomischen Formeln zusammen!

1) 100s^2-100st+25t^2

  11) \dfrac{a^2-12a+36}{2a-12}

2) \dfrac{49}{4}v^2+7vw+w^2

  12) \dfrac{b^2-9a^2}{b^2+6ba+9a^2}

3) 169x^2+49y^2

  13) \dfrac{x^2-\frac{2}{3}x+\frac{1}{9}}{x^2+\frac{26}{3}x-3}

4) x^2-64z^2

  14) \dfrac{10s+120}{s^2-144} : \dfrac{s^2+24s+144}{\left(s-12\right)\left (s+12\right)}

5) 36u^2+168u+196x^2

  15) \dfrac{4x^2-y^2}{4x^2-4xy+y^2}

6) 25a^2+60ab+100b^2

  16) \dfrac{144y^2-169z^2}{144y^2+312zy+169z^2}

7) \dfrac{5}{2}wy+\dfrac{5}{8}y^2+\dfrac{5}{2}w^2

  17) 361a^2+36b^2+121c^2+144d^2+228ab-264cd

8) -72a^2+36ac-\dfrac{9}{2}c^2

  18) \dfrac{289p^2+680pq+400}{289p^2-400q^2}

9) \dfrac{81}{2}a^2+36ad-4d^2

  19) \dfrac{1}{14x+15z} : \dfrac{14x-15z}{196x^2-225z^2}

10) 80-405x^2

  20) \dfrac{-48m}{36m^2+16}

 

5. Aufgabe

1) \genfrac{}{}{1pt}{0}{\frac{1}{2}}{x}

  11) \genfrac{}{}{1pt}{0}{-44x^9y^5}{\frac{-22x}{19y^2}}

2) \genfrac{}{}{1pt}{0}{q^5}{\frac{7}{q^2}}

  12) \genfrac{}{}{1pt}{0}{-\frac{63x^2 y^3}{4z}}{\frac{7x^3 \cdot 3z^{23}}{z}}
3) \genfrac{}{}{1pt}{0}{\frac{a}{-13}}{\frac{b}{12}}

  13) \genfrac{}{}{1pt}{0}{\frac{24x^{10}}{5x^2y}}{\frac{-14x^2y^7}{3x^{10}}}
4) \genfrac{}{}{1pt}{0}{\frac{-3d}{-g^3}}{-4c}

  14) \genfrac{}{}{1pt}{0}{\frac{55qrt}{12x^2}}{-\frac{121x^5}{9q^2}}
5) \genfrac{}{}{1pt}{0}{-45x}{\frac{5y}{9x}}

  15) \genfrac{}{}{1pt}{0}{\frac{144x^2 y^{17}}{-35z^3}}{288x^{11}yz^{12}}
6) \genfrac{}{}{1pt}{0}{\frac{29a}{-60b}}{\frac{-c}{4b^2}}

  16) \genfrac{}{}{1pt}{0}{33b^{22}q^7}{\frac{99b^{19}q^{11}}{m^2}}
7) \genfrac{}{}{1pt}{0}{\frac{14c}{-5}}{\frac{-18c}{25a}}   17) \genfrac{}{}{1pt}{0}{11xyz^2}{\frac{7r^2 x}{-5rz^2}}

8)\genfrac{}{}{1pt}{0}{-\frac{31}{2r^2}}{\frac{t}{10}}

  18) \genfrac{}{}{1pt}{0}{-\frac{8y^{13}}{7x}}{\frac{24y^2 z}{49rt^{11}}}
9) \genfrac{}{}{1pt}{0}{-4d^2}{\frac{32}{135d^4}}   19) \genfrac{}{}{1pt}{0}{\frac{x^4}{y^2z}}{\frac{z^4}{xy}}
10) \genfrac{}{}{1pt}{0}{\frac{-71k}{-4lm}}{-401k}   20) \genfrac{}{}{1pt}{0}{\frac{14a^2 w}{9az^{17}}}{\frac{28wz}{123a}}

 

6. Aufgabe

Vereinfachen Sie die folgenden Terme so weit wie möglich! Überlegen Sie sich jeweils, ob und wenn ja, welche Variablenwerte ausgeschlossen werden müssen!
Bei Aufgabe 16) soll nach der Vereinfachung keine Wurzeln im Nenner stehen.
Bei den Aufgaben 17) - 30) vereinfachen Sie bitte so, dass im Ergebnis keine negativen und gebrochenen Exponenten mehr enthalten sind.

1) x^2 \cdot x^3

  16) \dfrac{1}{\sqrt{x}}

  31) \log(c \cdot d)-\log(d)

2) m^5 \cdot m^n \cdot m^{-3}  

17) \left(y^{\frac{1}{3}}\right)^2

  32) \log_a(16)-\log_a(2)

3) \dfrac{b^6}{b^4}

  18) 13\dfrac{\sqrt{4m-1}}{\left(4m-1\right)^{-0{,}5}}

  33) \dfrac{\ln\left(u^5\right)}{5}

4) y : y^2

  19) \sqrt[3]{x} \cdot \sqrt{x}

  34) \log_b(3x)+\log_b(x)-(2\log_b(x)-\log_b(y))

5) \left(x^3\right)^4

  20) a^{\frac{1}{2}} \cdot \left(a^3\right)^2 : a

  35) \ln\left(e^{x^2}\right)

6) \left(a^2\right)^b

  21) \dfrac{ \sqrt[5]{b^3}}{b}

  36) 3\log\left(\sqrt[3]{z}\right)

7) 2^n \cdot 3^n

  22) \left( \sqrt[4]{z^2}\right)^3

  37) \log_a(2x)-\log_a(4)+\log_a\left(\dfrac{2}{x}\right)

8) \dfrac{a^5}{(2a)^5}

  23) \dfrac{ \sqrt{x^3}}{x \cdot \sqrt{x}}

  38) \log_2\left(4^x\right)

9) 16^m : 4^m

  24) \left( \dfrac{1}{x^2} \cdot x^{-2} \right)^2 \cdot \sqrt[2]{x}

  39) \log_{12}\left(\dfrac{c-d^2}{(c-d)^3}\right)

10) \dfrac{r^2 s^{-3} r^{-4} s^{-1} t t^6}{t^{-5} r^4 s^6 s^{-2} r t^{10} r^{-3} s^{-7}}

  25) \sqrt[3]{x^2d} \cdot d^{-\frac{4}{3}}

  40) \log_b\left(10y\right)-\log_b\left(5y^2\right)+\log_c\left(\dfrac{y}{100}\right)+\log_b\left(2^{-1}\right)

11) (4p^3-6q^5)^3

 

26) \left(x^{1{,}25} : y^{-0{,}625}\right)^{-\frac{4}{5}}

  41) 10\lg\left(\sqrt{f^2+w^2}\right)

12) \dfrac{y^4-16}{2y^4-16y^2+32}

  27) \sqrt{121a^8+166a^4b^2+64b^4}

  42) \sqrt{\log_2(512)}

13) 5e^{-x+1}-2xe^{-x+1}

  28) \dfrac{4y\sqrt{x^3}}{16xy^2}

  43) \ln\left(-\sqrt{18x^4}\right)

14) \genfrac{}{}{1pt}{0}{\frac{x+7}{2}}{\frac{8}{y-1}}

  29) \sqrt{x^{-2}}

  44) \log\left(1+x^3\right) \cdot [\log\left(\sqrt{x}\right)+1]

15) \genfrac{}{}{1pt}{0}{\frac{a}{b}-\frac{b}{a}}{\frac{1}{b}+\frac{1}{a}}\; : \; \left(a-b\right)

  30) \sqrt[5]{\dfrac{\sqrt[4]{\left(a^4b^2\right)^{10}}}{\sqrt{a^{10}b^5}}}

  45) \log_5\left(\genfrac{}{}{1pt}{0}{\frac{375}{2a^6}}{\frac{3}{10x^2}}\right)

 

7. Aufgabe

Aus dem Papyrus Rhind, einem ägyptischen Rechenbuch aus dem 16. Jahrhundert vor Christus:
7 Leute besitzen je 7 Katzen. Jede Katze vertilgt 7 Mäuse. Jede Maus hätte 7 Ähren Getreide gefressen. Aus jeder Ähre können 7 Maß Gerste wachsen. Wie viele Maß Gerste sind also den Katzen zu verdanken?
[zitiert nach: Lambacher-Schweizer (1974): Algebra 2. Stuttgart (S. 236).]

 

8. Aufgabe

1) Wie viele Nachkommastellen hat die Dezimaldarstellung der Zahl 2^{-n} mit n\in\mathbb{N}^+


2) Stellen Sie fest, ob die Zahl 3.141.592.653 eine Quadratzahl ist - OHNE Taschenrechner, Computer, Handy und Co zu benutzen!


3) Multiplizieren Sie \left(1-\dfrac{1}{4}\right)\left(1-\dfrac{1}{9}\right)\left(1-\dfrac{1}{16}\right)\left(1-\dfrac{1}{25}\right)\dots\left(1-\dfrac{1}{100}\right) - OHNE Taschenrechner, Computer, Handy und Co zu benutzen!

Dieses Kapitel enthält die folgenden Themen:

 

8.2 Potenzen, Wurzeln, Logarithmen - Erklärungen

Wir haben die Grundlagen noch nicht ganz geschafft ... In diesem Kapitel geht es noch einmal um etwas ganz Wichtiges, worauf eigentlich alle weiteren Themen aufbauen: nämlich Terme, die sich aus Potenzen, Wurzeln und Logarithmen zusammensetzen.
Warum ist dieses Kapitel so wichtig? Kommen wir noch mal auf das Fußballbeispiel aus dem Kapitel Rechengesetze zurück: Wir trainieren in diesem Kapitel den zulässigen Umgang mit bestimmten, sehr wichtigen Arten von Termen. Das ist vergleichbar mit einer Fußballmannschaft, die Dribbeln und Standards (wie Elfmeter, Ecken, Einwürfe) einübt und trainiert, wie man der gegnerischen Mannschaft den Ball abnimmt, ohne zu foulen etc. Diese Dinge werden alle trainiert, bevor man das erste 90-Minuten-Match bestreitet, weil man darin die grundlegenden Fußballtechniken sicher beherrschen muss. Fußballspielerinnen und -spielern wird es schwerfallen, am Spielverlauf sinnvoll teilzunehmen, wenn sie sich noch auf das Dribbeln konzentrieren müssen …
Genauso ist es hier: Wir üben den zulässigen Umgang mit Potenzen, Wurzeln und Logarithmen, um diese anschließend in komplexeren Zusammenhängen, wie Gleichungen und Funktionen, sicher anwenden zu können. Sonst ist es kaum möglich, darauf aufbauende Aufgaben zu bewältigen. Wenn Sie beim Lösen einer Exponentialgleichung oder beim Berechnen eines Integrals noch überlegen müssen, wie e^{2x}\cdot e^{x+1} zusammengefasst werden kann, wird es Ihnen schwerfallen, den Gesamtlösungsweg im Blick zu behalten und die berechnete Lösung am Ende zu plausibilisieren.

 

Potenzen - die grundlegende Definition

Grundlegende Definition einer Potenz: a^n= \underbrace{a \cdot a \cdot \ldots \cdot a}_n für a \in \mathbb{R} und n \in \mathbb{N} (a^n gesprochen: "a hoch n")
Die Potenz a^n ist also erst mal nur eine abkürzende Schreibweise für ein Produkt mit vielen gleichen Faktoren. Das hat den Vorteil, dass man die Faktoren nicht mehr zählen muss ... a heißt dabei Basis (vom griechischen Wort basis für Grundlage) und n Exponent (vom lateinischen Wort exponere, welches herausstellen bedeutet).
Ist der Exponent 2, sagt man auch Quadrieren.

Beispiele:

3^2 = 3\cdot 3 = 9
10^5 = 10\cdot 10 \cdot 10 \cdot 10 \cdot 10 = 100.000
27^1 = 27    
5{,}1^5 = 5{,}1\cdot 5{,}1 \cdot 5{,}1 \cdot 5{,}1 \cdot 5{,}1 = 3.450{,}25251
(-6)^4 = (-6)\cdot (-6)\cdot (-6)\cdot (-6) = 1.296
(-100)^3 = (-100)\cdot (-100)\cdot (-100) = -1.000.000

 

Schauen wir uns die Definition noch etwas genauer an: Dort heißt es a^n für a \in \mathbb{R} und n \in \mathbb{N}. Das bedeutet, dass die Basis irgendeine reelle und der Exponent eine natürliche Zahl sein muss (zumindest im Moment, weiter unten erweitern wir dies). Ist der Exponent also 2, 5, 10 oder vielleicht auch 4.923, ergibt sich ein Produkt mit entsprechend vielen Faktoren, wobei diese Faktoren immer reelle Zahlen sind. Ein bisschen aus der Reihe tanzen die Exponenten 1 und 0 (zumindest erkennt man dort das Produkt nicht mehr): 

  • Wir hatten gerade schon ein entsprechendes Beispiel: a^1=a, egal für welche reelle Zahl a steht. "Unmathematisch" formuliert: Man kann den Exponenten in diesem Fall einfach weglassen.

  • Etwas überraschend ist vielleicht diese Festlegung: a^0=1 für a\neq 0. Anders gesagt: Wenn man eine reelle Zahl ungleich 0 mit 0 potenziert, ist das Ergebnis immer 1 - völlig egal, welche reelle Zahl man potenziert.

  • Bleibt die Frage, was mit 0^0 ist: 0^0 ist nicht definiert. Das heißt, hier gibt es keine Vereinfachung und kein Ergebnis.

 

Ganz wichtig: Im Allgemeinen ist a \cdot a = a^2 \neq 2a = a+a.

"Im Allgemeinen" bedeutet dabei, dass es einzelne Zahlenwerte gibt, für die diese Gleichungen gelten, aber für die (überwältigende) Mehrheit der Werte ist a^2 eben nicht gleich 2a. Hierfür ein paar Beispiele:

4\cdot 4 = 4^2 \neq 2\cdot 4 = 4+4   berechnet 16\neq 8
11\cdot 11 = 11^2 \neq 2\cdot 11 = 11+11   berechnet 121\neq 22
-85\cdot (-85) = (-85)^2 \neq 2\cdot (-85) = -85+(-85)   berechnet 7.225\neq -170

 

Und noch ein paar Erkenntnisse zu Potenzen, die später mal helfen können, z. B. wenn man ein Ergebnis auf Plausibilität prüfen möchte:

  • Ist der Exponent n eine gerade Zahl, ist die gesamte Potenz immer größer oder gleich 0, mathematisch formuliert: a^n \geq 0

  • Ist die Basis einer Potenz ungleich 0 (mathematisch aufgeschrieben: a\neq 0), ist auch die gesamte Potenz immer ungleich 0 (auch hier die mathematische Formulierung a^n\neq 0).

 

Vielleicht noch wichtiger: Potenzrechnung geht vor Punktrechnung geht vor Strichrechnung!
Auch hier können ausschließlich Klammern eine andere Reihenfolge der Rechenoperationen festlegen.

Wir hatten im Kapitel Rechengesetze schon mal besprochen, dass die unterschiedlichen Rechenoperationen eine bestimmte Rangfolge erfordern, man also nicht "einfach so" drauf losrechnen darf. Die Regel "Punkt- vor Strichrechnung" gilt hier natürlich immer noch und wird um eine "Ebene", nämlich um die Potenzrechnung, erweitert. Viele typische Fehler aus diesem Kapitel gehen auf dieses Problem zurück. Hier noch ein paar Beispiele dazu:

3+4\cdot 5^6 = 3+4\cdot 15.625 = 3+62.500 = 62.503
(3+4)\cdot 5^6 = 7\cdot 5^6 = 7\cdot 15.625 = 109.375
3+(4\cdot 5)^6 = 3+20^6 = 3+64.000.000 = 64.000.003
(3+4\cdot 5)^6 = (3+20)^6 = 23^6 = 148.035.889

Je nachdem, wie die Klammen gesetzt wurden, kommen also sehr unterschiedliche Werte heraus. Der letzte ist fast das 2.400-Fache vom ersten! Um Fehler zu vermeiden, lohnt sich also auch hier, sehr sorgfältig zu sein.

 

Binomische Formeln

Weil eben Potenz- vor Punkt- vor Strichrechnung geht, braucht man spezielle Überlegungen, wenn die verschiedenen Rechenoperationen aufeinandertreffen. Ein bekanntes Beispiel sind die binomischen Formeln beim Zusammentreffen von Addition/Subtraktion und Quadrieren/Multiplizieren:

1. binomische Formel:  (a+b)^2 = a^2+2ab+b^2
2. binomische Formel: (a-b)^2 = a^2-2ab+b^2
3. binomische Formel: (a+b)(a-b) = a^2-b^2


Beispiele:

1. binomische Formel: (1+p)^2 = 1^2+2\cdot 1\cdot p+p^2 = 1+2p+p^2
2. binomische Formel: (a-4)^2 = a^2-2\cdot 4 \cdot a+4^2 = a^2-8a+16
3. binomische Formel: (z+13)(z-13) = z^2-13^2 = z^2-169


Sie bieten eine Art Abkürzung für das Auflösen von speziellen Klammertermen, die sonst mithilfe des Distributivgesetzes ausmultipliziert werden müssten, z. B. (a+b)^2 = (a+b)(a+b)=a^2+ab+ba+b^2= a^2+2ab+b^2. Das geht natürlich auch - ist aber länger und damit meist fehleranfälliger.

Ganz wichtig: Egal, welchen Weg man wählt: Das Ergebnis von (a+b)^2 ist nicht a^2+b^2.

Hier ein paar Rechenbeispiele, um das zu verdeutlichen:

(1+8)^2 = 9^2 = 81 aber 1^2+8^2 = 1+64 = 65
(13-6)^2 = 7^2 = 49 aber 13^2-6^2 = 169-36 = 133


Anders gesagt: Bitte vergessen Sie bei der 1. und 2. binomischen Formel nicht den mittleren Term 2ab bzw. -2ab. Damit ändern sich die Rechenbeispiele von oben zu:

(1+8)^2 = 9^2 = 81 und 1^2+2\cdot 1\cdot 8+8^2 = 1+16+64 = 81   Passt!
(13-6)^2 = 7^2 = 49 und 13^2-2\cdot 13\cdot 6+6^2 = 169-156+36 = 49  


Das gilt natürlich genauso, wenn Variablen in den Termen enthalten sind - man sieht mögliche Fehler dann nur nicht mehr so schnell.

 

Binomische Formeln "rückwärts": Häufig ist es nützlich, die binomischen Formeln "von rechts nach links", d. h. zum Faktorisieren, anzuwenden. Wenn man irgendwo kürzen möchte, braucht man ja ein Produkt, welches man auf diesem Weg herstellen kann.

Beispiele:

1. binomische Formel: m^2+18m+81 = m^2+2\cdot 9 \cdot m+9^2 = (m+9)^2
2. binomische Formel: 100-20b+b^2 = 10^2-2\cdot 10\cdot b+b^2 = (10+b)^2
3. binomische Formel: 144-k^2 = 12^2-k^2 = (12-k)(12+k)


Hierbei muss man ein bisschen aufpassen, vor allem auf die Struktur des mittleren Terms bei der 1. und 2. binomischen Formel: Die beiden quadratischen Terme am Anfang und Ende sind meist problemlos zu erkennen. Aber diese Umformungen funktionieren wirklich nur, wenn der mittlere Term die Struktur 2ab bzw. -2ab hat. Da (m+9)^2 = m^2+18m+81, würde sich beispielsweise m^2+10m+81 nicht mithilfe der binomischen Formel zusammenfassen lassen und auch mit keiner anderen Formel.
Die 3. binomische Formel funktioniert auch nur dann, wenn einer der quadratischen Terme positiv und einer negativ ist.
Weiter unten wird es noch ein paar komplexere Beispiele geben, nämlich von binomischen Formeln im Zusammenspiel mit Potenzgesetzen.

 

Von Potenzen zu Wurzeln und Logarithmen

Wir nehmen eine Potenz a^n, die irgendein Ergebnis x liefert:

Zusammenhänge:Ergebnis gesucht: Rechnen mit PotenzenBasis gesucht: Rechnen mit WurzelnExponent gesucht: Rechnen mit Logarithmen


Das Rechnen mit Potenzen gibt Antwort auf die Frage: Welches Ergebnis erhalte ich, wenn ich die Zahl a n-mal mit sich selbst multipliziere?

Das Rechnen mit Wurzeln gibt Antwort auf die Frage: Welche Zahl ergibt x, wenn ich sie n-mal mit sich selbst multipliziere?
Formal geschrieben: a=\sqrt[n]{x}

Das Rechnen mit Logarithmen gibt Antwort auf die Frage: Wie oft muss ich die Zahl a mit sich selbst multiplizieren, damit das Ergebnis x ist?
Formal geschrieben: n=\log_a {x}

Diese Fragen zeigen es schon: Potenzieren, Radizieren (also das Rechnen mit Wurzeln) und Logarithmieren gehören eng zusammen und heben sich - vorausgesetzt die Basen bzw. die Exponenten sind gleich - gegenseitig auf. Mathematisch sagt man: Sie sind Umkehroperationen

In den folgenden Abschnitten wird es nun darum gehen, wie man mit Potenzen, Wurzeln und Logarithmen umgeht und welche Rechengesetze in den einzelnen Fällen gelten. Natürlich werden auch einige Beispiele nicht fehlen.

 

Rechnen mit Potenzen

Beim Rechnen mit Potenzen benötigt man in erster Linie die fünf Potenzgesetze. Wie alle Rechengesetze können sie sowohl "von links nach rechts" als auch "von rechts nach links" gelesen werden - je nachdem, was sich für die entsprechende Aufgabe anbietet.

1. Potenzgesetz: a^n \cdot a^m = a^{n+m}
Merksatz: Potenzen mit gleicher Basis werden multipliziert, indem man die Exponenten addiert.
Beispiel: 5^2 \cdot 5^6 = 5\cdot 5\;\cdot\; 5\cdot 5\cdot 5\cdot 5\cdot 5\cdot 5 = 5^{2+6} = 5^8


2. Potenzgesetz: a^n : a^m = \dfrac{a^n}{a^m} = a^{n-m}
Merksatz: Potenzen mit gleicher Basis werden dividiert, indem man die Exponenten subtrahiert.
Beispiel: 10^8 : 10^6 = \frac{10^8}{10^6} = 10^{8-6} = 10^2


3. Potenzgesetz: \left(a^n\right)^m = a^{n \cdot m}
Merksatz: Potenzen werden potenziert, indem man die Exponenten multipliziert.
Beispiel: \left(3^4\right)^5 = 3^{4 \cdot 5} = 3^{20}


4. Potenzgesetz: a^n \cdot b^n = \left(a \cdot b \right)^n
Merksatz: Potenzen mit gleichen Exponenten werden multipliziert, indem man die Basen multipliziert.
Beispiel: 8^2 \cdot 9^2 = \left(8 \cdot 9 \right)^2 =72^2


5. Potenzgesetz: a^n : b^n = \dfrac{a^n} {b^n} = \left(\dfrac{a}{b}\right)^n
Merksatz: Potenzen mit gleichen Exponenten werden dividiert, indem man die Basen dividiert.
Beispiel: 14^3 : 7^3 = \frac{14^3} {7^3} = \left(\frac{14}{7}\right)^3 = 2^3 

 

Worauf man unbedingt achten muss:
Das 4. Potenzgesetz "von rechts nach links" gelesen besagt \left(a \cdot b \right)^n = a^n \cdot b^n. Das bedeutet: Steht eine Basis (bestehend aus mehreren Faktoren) in Klammern, müssen alle Bestandteile der Basis potenziert werden! Und umgekehrt: Steht diese Basis nicht in Klammern, darf nur der Faktor potenziert werden, der den Exponenten trägt. Ein Beispiel:

\left(3\cdot 4\right)^2 = 3^2\cdot 4^2 = 9 \cdot 16 = 144


  3\cdot 4^2

=

3 \cdot 16

=

48

\left(9\cdot y\right)^2 = 9^2\cdot y^2 = 81 \cdot y^2    
    9\cdot y^2 = 9y^2    


Ob a^n \cdot b^n oder a \cdot b^n gerechnet wird, macht also offensichtlich einen Unterschied. Die Klammern sind hier - mal wieder - sehr wichtig!

Aus dem gleichen Grund ist auch (-4)^2 nicht das Gleiche wie -4^2. Um genau zu sein:

(-4)^2 = -4\cdot (-4) = 16
-4^2 = -4\cdot 4 = -16


Im ersten Fall wird das Minuszeichen mit quadriert, im zweiten nicht. Auch wenn das auf den ersten Blick eher unpraktisch erscheint, haben beide Varianten ihre Berechtigung. Daher muss man sie (durch den Einsatz von Klammern) sorgfältig unterscheiden.

 

Ganz wichtig: Es gibt keine Potenzgesetze für die Addition/Subtraktion von Potenzen!
Das bedeutet, dass Addition und Subtraktion nichts am Exponenten einer Potenz ändern können. Diese Tatsache wird (hoffentlich) einleuchtend, wenn man sich überlegt, dass Potenzen "Abkürzungen" für Produkte mit vielen gleichen Faktoren sind. Produkte gehören zur Punktrechnung und gehen damit vor jede Addition/Subtraktion!
Die Addition/Subtraktion von Potenzen ist also nur möglich, wenn alle Basen und alle Exponenten in der Summe/Differenz gleich sind - und sie kann nur etwas am Koeffizienten der Potenz ändern (siehe Rechnen mit Variablen).

Ein paar Beispiele:

    13x^2-20x^2 = -7x^2  
Hier ist alles ok, weil die Basen und Exponenten gleich sind.


a^2bc^4-10a^2c^4b+28abc^4a = 1a^2bc^4-10a^2bc^4+28a^2bc^4 = 19a^2bc^4  
Anwendung des Kommutativgesetzes und Zusammenfassen innerhalb der Summanden geht auch klar.


\dfrac{1}{2}kn^2-\dfrac{3}{4}kn^2+\dfrac{7}{10}kn^2w-kn^2 = \dfrac{2}{4}kn^2-\dfrac{3}{4}kn^2-\dfrac{4}{4}kn^2+\dfrac{7}{10}kn^2w = -\dfrac{5}{4}kn^2-\dfrac{7}{10}kn^2w  
Der Summand kn^2w kann nicht mit den anderen zusammengefasst werden, weil die Variable w nur hier enthalten ist. Mit den Brüchen rechnet man wie sonst auch.


-17p+5p^5-30p^2+p-p^5+101p^2-29p^2 = -17p+p-30p^2+101p^2-29p^2+5p^5-p^5 = -16p+42p^2+4p^5  
Auch wenn die Basen gleich sind, dürfen nicht alle Summanden zusammengefasst werden, weil auch die Exponenten übereinstimmen müssen.


 

Kommen wir jetzt noch mal zurück zu den binomischen Formeln: Bislang waren die Beispiele alle von der Struktur her recht einfach. Interessanter wird es, wenn Zahlen und Variablen gemeinsam in binomischen Formeln auftreten. Schauen wir uns dazu ein paar Beispiele an:

\left(3x+7y^2\right)^2 = (3x)^2+2\cdot(3x)\cdot\left(7y^2\right)+ \left(7y^2\right)^2 = 3^2x^2+2\cdot 3\cdot 7\cdot xy^2+7^2\left(y^2\right)^2 = 9x^2+42xy^2+49y^4  
Hier benötigt man zusätzlich zur 1. binomischen Formel das 4. Potenzgesetz für (3x)^2 = 9x^2 und das 3. und 4. Potenzgesetz für \left(7y^2\right)^2 = 49y^4.


\left(\dfrac{1}{2}+20d\right)\left(\dfrac{1}{2}-20d\right) = \left(\dfrac{1}{2}\right)^2-(20d)^2 = \dfrac{1^2}{2^2}-20^2d^2 = \dfrac{1}{4}-400d^2  
Hier benötigt man zusätzlich zur 3. binomischen Formel das 5. Potenzgesetz für \left(\dfrac{1}{2}\right)^2 = \dfrac{1}{4} und das 4. Potenzgesetz für (20d)^2 = 400d^2.


\dfrac{64a^2}{b^6}-\dfrac{16a}{b^3}+1 = \left(\dfrac{8a}{b^3}\right)^2-2\cdot\dfrac{8a}{b^3}\cdot 1+1^2 = \left(\dfrac{8a}{b^3}-1\right)^2      
Das Ganze geht natürlich auch "rückwärts": Hier benötigt man zusätzlich zur 2. binomischen Formel das 3., 4. und 5. Potenzgesetz für \left(\dfrac{8a}{b^3}\right)^2= \dfrac{64a^2}{b^6}.


Sie sehen, dass diese Rechnungen auch mit so wenig Regeln (Wurzeln und Logarithmen kommen ja erst ...) schon ganz schön komplex werden können. Umso wichtiger ist es, diese Regeln gut zu verinnerlichen und einzuüben. Sie werden sie brauchen!

 

Nun gehen wir einen Schritt weiter:
Ganz oben wurde erklärt, dass Potenzen a^n für a \in \mathbb{R} und n \in \mathbb{N} definiert sind - und auch schon in Aussicht gestellt, dass es für diese Definitionsbereiche noch Erweiterungen geben wird. Darum wollen wir uns jetzt kümmern.
Um eine der Erweiterungen herzuleiten, schauen wir uns ein Beispiel an: Nehmen wir \frac{5^2}{5^4}=5^{2-4}=5^{-2}. Hier wurde das 2. Potenzgesetz angewendet. Schreiben wir stattdessen die Potenz aus und kürzen ein bisschen, erhalten wir \frac{5^2}{5^4}=\frac{5\cdot 5}{5\cdot 5\cdot 5\cdot 5}=\frac{1}{5\cdot 5}=\frac{1}{5^2}. Offensichtlich ist 5^{-2} = \frac{1}{5^2} ist. Allgemein aufgeschrieben:
a^{-n} = \dfrac{1}{a^n} mit a \in \mathbb{R}\backslash_{\{0\}} und n \in \mathbb{Z}
Potenzen können also auch negative Exponenten haben. Anders gesagt: Die Exponenten können aus dem Zahlenbereich der ganzen Zahlen \mathbb{Z} stammen. a^{-n} bedeutet dabei nicht, dass der Potenzausdruck negativ wird (Dazu müsste das Minuszeichen vor der Potenz stehen!), sondern dass der Kehrwert von a^n gebildet wird. Da die Potenz damit in den Nenner gelangt, darf die Basis natürlich nicht 0 sein.
Wichtig: Das 4. Potenzgesetz gilt natürlich auch, wenn der Exponent negativ ist. Ein paar Beispiele:

\left(10\cdot 5\right)^{-2} = 10^{-2} \cdot 5^{-2} = \dfrac{1}{10^2\cdot 5^2} = \dfrac{1}{2.500}
10\cdot 5^{-2} = 10^1\cdot 5^{-2} = \dfrac{10}{5^2} = \dfrac{10}{25}
\left(z\cdot 14\right)^{-2} = z^{-2} \cdot 14^{-2} = \dfrac{1}{z^2 \cdot 14^2} = \dfrac{1}{196z^2}
z\cdot 14^{-2} = z^1\cdot 14^{-2} = \dfrac{z}{14^2} = \dfrac{z}{196}

 

Die folgende Erweiterung bringt uns nun direkt zum nächsten Abschnitt dieses Kapitels.

 

Rechnen mit Wurzeln

Potenzen können nicht nur negative, sondern auch gebrochene Exponenten haben:
\displaystyle{ a^{\frac{1}{n}} = \sqrt[n]{a}} (gesprochen: "n-te Wurzel aus a") mit a \in \mathbb{R}^+_0, wenn n \in \mathbb{N}^+ gerade, oder a \in \mathbb{R}, wenn n \in \mathbb{N}^+ ungerade
n heißt Wurzelexponent und ist eine natürliche Zahl (mit Ausnahme von 0). a heißt Radikand und ist eine (nichtnegative) reelle Zahl.

Diese Potenzschreibweise einer Wurzel mag einem auf den ersten Blick merkwürdig oder unpraktisch vorkommen - sie hat aber große Vorteile! Der größte: Durch sie gelten auch für Wurzeln die oben aufgeführten Potenzgesetze. Man benötigt also für das Rechnen mit Wurzeln keine zusätzlichen Regeln (auch wenn sie in vielen Formelsammlungen aufgeführt sind ...). Außerdem wird der Zusammenhang zwischen Potenzen und Wurzeln sehr gut deutlich. Und nicht zuletzt: Beim Ableiten und Integrieren kommt man ohne diese Schreibweise quasi nicht aus. Es ist also günstig, sich jetzt schon mal daran zu "gewöhnen".

Bemerkung: Solange keine Missverständnisse auftreten können, spricht man von "Wurzel" statt von "2. Wurzel" bzw. "Quadratwurzel" und schreibt \sqrt{x} statt \sqrt[2] {x}.Trotzdem geht die 2 natürlich nicht verloren ...


Beispiele:

\sqrt{x} = \sqrt[2]{x} = x^\frac{1}{2}    
\sqrt{t^3} = \sqrt[2]{t^3} = t^{3\cdot\frac{1}{2}} = t^\frac{3}{2}
    \sqrt[3]{q^3} = q^{3\cdot\frac{1}{3}} = q^\frac{3}{3}=q^1=q
    w^\frac{1}{5} = \sqrt[5]{w}    
z^\frac{2}{9} = z^{2\cdot\frac{1}{9}} = \sqrt[9]{z^2}    

 
Ein paar allgemeinere Erkenntnisse zu Wurzeln:
Wurzeln verhalten sich unterschiedlich, je nachdem, ob der Wurzelexponent gerade oder ungerade ist:

  • Bei geraden Wurzelexponenten:
    • Da es keine reelle Zahl gibt, die mit sich selbst multipliziert ein negatives Ergebnis, wie -4, ergibt, können aus negativen Zahlen keine Quadratwurzeln gezogen werden. Da "minus mal minus gleich plus" ist, gilt diese Erkenntnis für alle Wurzeln mit geraden Wurzelexponenten. Klassischerweise formuliert man: Aus negativen reellen Zahlen können keine Wurzeln mit geraden Wurzelexponenten gezogen werden!
    • Solche Wurzeln sind immer größer oder gleich 0, mathematisch formuliert: \sqrt[n]{x} \geq 0.
      Das hat eine wichtige Folge: Da beim Quadrieren möglicherweise negative Vorzeichen "verloren" gehen, erhält man beim Wurzelziehen aus quadratischen Termen den Betrag des Radikanden: \sqrt{x^2} = \vert x \vert (analog für höhere Exponenten und Wurzelexponenten). Wir wissen ja nicht, ob das x vor dem Quadrieren positiv oder negativ gewesen ist ...

  • Bei ungeraden Wurzelexponenten:
    • Multipliziert man eine negative reelle Zahl so mit sich selbst, dass man eine ungerade Anzahl von Faktoren hat, ist das Ergebnis negativ (Es bleibt ja quasi ein Minuszeichen "übrig" ...), z. B. -2\cdot (-2)\cdot (-2) = (-2)^3=-8. Daher können Wurzeln mit ungeraden Wurzelexponenten aus negativen Zahlen gezogen werden.
      Aus mathematischen Gründen formuliert man: Ist n ungerade und a negativ, gilt: \sqrt [n]{a}= -\sqrt[n]{\vert a \vert}.
    • Solche Wurzeln können dann auch negative Ergebnisse haben. Zum Beispiel ist \sqrt[3]{-8}=-\sqrt[3]{\vert-8\vert}=-2.


Die nächste Frage ist nun natürlich: Wie rechnet man mit Wurzeln eigentlich? Dazu schauen wir uns die Potenzgesetze noch mal an:

1. Potenzgesetz: a^n \cdot a^m = a^{n+m}
Beispiel: \sqrt[5]{(2z)^3}\cdot\sqrt[5]{(2z)^2} = (2z)^\frac{3}{5}\cdot (2z)^\frac{2}{5} = (2z)^{\frac{3}{5}+\frac{2}{5}} = (2z)^1 = 2z mit z\in \mathbb{R}^+_0
Wichtig: Klammern um 2z nicht vergessen!


2. Potenzgesetz: a^n : a^m = \frac{a^n}{a^m} = a^{n-m}
Beispiel: \dfrac{\sqrt[3]{a}}{\sqrt{a^9}} = \dfrac{a^\frac{1}{3}}{a^\frac{9}{2}} = a^{\frac{1}{3}-\frac{9}{2}} = a^{-\frac{25}{6}} = \dfrac{1}{a^\frac{25}{6}} = \dfrac{1}{\sqrt[6]{a^{25}}} mit a\in \mathbb{R}^+
Wichtig: Bei der Nenner-Potenz die 2 im Exponenten nicht vergessen! Es handelt sich ja um eine 2. Wurzel, auch wenn der Wurzelexponent nicht hingeschrieben wurde.


3. Potenzgesetz: \left(a^n\right)^m = a^{n \cdot m}
Beispiel 1: \left(\sqrt{s^3}\right)^4 = \left(s^\frac{3}{2}\right)^4 = s^{\frac{3}{2}\cdot 4} = s^\frac{12}{2} = s^6 mit s\in \mathbb{R}^+_0
Wichtig: Auch hier die 2 im Exponenten nicht vergessen!

Beispiel 2: x^\frac{2}{5} = x^{2\cdot\frac{1}{5}} = \left(x^2\right)^\frac{1}{5} = \sqrt[5]{x^2} mit x\in \mathbb{R}
Bemerkung: Diese Umformung kann man auch von rechts nach links anwenden.


4. Potenzgesetz: \left(a \cdot b \right)^n = a^n \cdot b^n
Beispiel: \sqrt{4\cdot y} = \sqrt{4}\cdot\sqrt{y} = 2\cdot\sqrt{y} mit y\in \mathbb{R}^+_0
Bemerkung: Diese Vorgehensweise nennt man auch "teilweises Wurzelziehen". Damit kann man Wurzelterme häufig noch ein gutes Stück vereinfachen.


5. Potenzgesetz: \left(\frac{a}{b}\right)^n = a^n : b^n = \frac{a^n} {b^n}
Beispiel: \sqrt[3]{\dfrac{p^3}{8}} =\dfrac{\sqrt[3]{p^3}}{\sqrt[3]{8}} = \dfrac{p^{3\cdot\frac{1}{3}}}{2} = \dfrac{p^1}{2} = \dfrac{p}{2} mit p\in \mathbb{R}
Bemerkung: Werte wie \sqrt[3]{8} dürfen natürlich einfach ausgerechnet werden.

 

Erkenntnisse aus diesen Beispielen:

  • Natürlich können Potenzen auch Exponenten haben, die negativ und gebrochen sind. Dann werden einfach beide Regeln nacheinander angewendet (siehe Beispiel zum 2. Potenzgesetz).

  • Radizieren (das "offizielle" Wort für Wurzelziehen) und Potenzieren mit dem gleichen Exponenten heben einander auf (siehe Beispiel zum 5. Potenzgesetz). Achten Sie aber darauf, dass bei geraden Wurzelexponenten ggf. ein Betrag nötig wird, da solche Wurzelterme ja immer nichtnegativ sein müssen.

  • Da es nur diese fünf Potenzgesetze gibt, lassen sich auch Wurzeln oder Vielfache von Wurzeln ausschließlich dann addieren und subtrahieren, wenn sie gleiche Wurzelexponenten und gleiche Radikanden haben! Siehe oben ...

 

Ganz wichtig: Aus Summen können keine Wurzeln gezogen werden!
Anders formuliert: Wurzeln aus Summen lassen sich im Allgemeinen nicht vereinfachen, insbesondere ist \sqrt{a +b} \neq \sqrt{a}+\sqrt{b} für fast alle a und b!

Wie oben, ein paar Beispiele hierfür:

\sqrt{16 + 9} = \sqrt{25} \neq \sqrt{16}+\sqrt{9} = 4+3   berechnet 5\neq 7
\sqrt{64 + 100} = \sqrt{164} \neq \sqrt{64}+\sqrt{100} = 8+10   berechnet 12{,}8062...\neq 18


Bitte beachten Sie, dass sich in der zweiten Zeile einmal eine irrationale Zahl, d. h. eine unendliche, nichtperiodische Dezimalzahl, und einmal eine natürliche Zahl als Ergebnis ergibt. Die Ergebnisse können also schon aus formalen Gründen nicht gleich sein! Allgemein gilt: Betrachtet man nur natürliche Zahlen im Radikanden, sind Wurzeln aus Quadratzahlen, wie 1, 4, 9 etc., wieder natürliche Zahlen; Wurzeln aus Nicht-Quadratzahlen, wie 2, 3, 5 etc., sind irrationale Zahlen.

 

Rechnen mit Logarithmen

Kümmern wir uns um die letzte Rechenart in diesem Zusammenhang: den Logarithmus \log_a(x). Wobei es eigentlich nicht den einen Logarithmus gibt, sondern viele, da ja nicht festgelegt ist, welche Zahl a ist. Für den Umgang mit Logarithmen macht das aber keinen Unterschied, nur fürs Ausrechnen am Ende ...

Der Logarithmus \log_a(x) (gesprochen: "Logarithmus zur Basis a von x") ist definiert für a \in \mathbb{R}^+\backslash_{\{1\}} und x \in \mathbb{R}^+.
a heißt Basis und x Argument des Logarithmus (oder kurz: Argument). Teilweise sagt man statt Argument auch Numerus.
Wichtig: Da das Argument eines Logarithmus positiv sein muss, sind Ausdrücke wie \log_a(0) oder \log_a\left(-x^2\right) unmöglich.

Für alle, die es genauer wissen möchten, wird im nächsten Abschnitt erklärt, warum diese Einschränkungen für die Basis und das Argument gelten. Alle anderen dürfen diesen Abschnitt überspringen und gleich mit den Hinweisen zur Schreibweise weitermachen ...

  • Potenzen mit negativen Basen sind im Bereich der reellen Zahlen nur für ganzzahlige Exponenten definiert. Das heißt: Ein Term wie (-2)^{\frac{1}{2}}=\sqrt{-2} liefert kein Ergebnis. Genau dieses Ergebnis bräuchte der Logarithmus aber als Argument. Also geht das Ganze nur mit positiven Basen.

  • Ein Potenzausdruck mit der Basis 1 wäre eine ziemlich langweilige Angelegenheit, denn 1^n = 1, egal welche reelle Zahl n ist. Betrachtet man umgekehrt den Logarithmus, also die Frage, wie oft die Basis 1 mit sich selbst multipliziert werden muss, damit das Ergebnis 1 ist, gibt es darauf keine eindeutige Antwort. Da es in der Mathematik wenig Fans von uneindeutigen Antworten gibt (sie sind auch meist nicht sinnvoll), schließt man 1 als Basis aus.

  • Wenn die Basis positiv ist, kann das Ergebnis des Potenzausdrucks nie negativ oder 0 werden, denn für negative Exponenten gilt ja a^{-n} = \frac{1}{a^n} und das ist in diesem Fall immer größer als 0. Da der Logarithmus genau dieses Ergebnis als Argument verwendet, kommen also nur positive Argumente infrage.


Noch ein paar Hinweise zur Schreibweise:

  • Teilweise werden (aus Faulheit ...) beim Logarithmus die Klammern weggelassen, beispielsweise \log_a(x) = \log_a x. Das ist aber nicht immer möglich: Bei \log_ay+9 könnte sowohl \log_a(y)+9 als auch \log_a (y+9) gemeint sein. Damit Missverständnisse ausgeschlossen sind, müssen hier unbedingt Klammern gesetzt werden. Sie haben also die nächste Stelle kennengelernt, wo man sehr, sehr gut auf die Klammern achtgeben sollte!

  • Mit der Basis ist es ähnlich: Man schreibt beispielsweise \log(x) und meint \log_a(x). Das ist dann ok, wenn die Umformung allgemein für alle Basen gilt oder wenn klar ist, um welche Basis es sich handelt. Spätestens, wenn es um das Ausrechnen von Zahlenwerten geht, muss man die Basis aber nennen. Die Antworten auf die folgenden Fragen sind ja offensichtlich nicht gleich:
    • Wie oft muss ich die Zahl 2 mit sich selbst multiplizieren, damit das Ergebnis 10 ist? Antwort: ca. \log_2(10) = 3{,}32, denn 2^{3{,}32} \approx 10
    • Wie oft muss ich die Zahl 10 mit sich selbst multiplizieren, damit das Ergebnis 10 ist? Antwort: \log_{10}(10) = 1, denn 10^1 = 10

 

Leider gibt es für Logarithmen keine so praktische Umschreibung in Potenzschreibweise wie bei den Wurzeln (alle möglichen Potenzen sind ja schon "verbraucht" ...), sodass man hier wirklich zusätzliche Rechengesetze benötigt. Darum geht es nun im folgenden Abschnitt.

Logarithmengesetze

Für das Rechnen mit Logarithmen gelten die folgenden Logarithmengesetze. Wie auch die Potenzgesetze können sie sowohl "von links nach rechts" als auch "von rechts nach links" gelesen werden - je nachdem, was sich für die entsprechende Aufgabe anbietet.

1. Logarithmengesetz: \log_a(x \cdot y) = \log_a(x) + \log_a(y)
Merksatz: Der Logarithmus eines Produkts ist gleich der Summe der Logarithmen der einzelnen Faktoren.
Beispiel: \log_{10}(10x) = \log_{10}(10)+\log_{10}(x) = 1+\log_{10}(x)


2. Logarithmengesetz: \log_a\left(\dfrac{x}{y}\right) = \log_a(x) - \log_a(y)
Merksatz: Der Logarithmus eines Quotienten ist gleich der Differenz aus dem Logarithmus des Dividenden und dem Logarithmus des Divisors.
Beispiel: {\log_2\left(\frac{a}{0{,}5}\right) = \log_2(a)-\log_2(0{,}5) = \log_2(a)-\log_2\left(\frac{1}{2}\right) = \log_2(a)-\log_2\left(2^{-1}\right) = \log_2(a)-(-1) = \log_2(a)+1}


3. Logarithmengesetz: \log_a\left(x^k\right) = k \cdot \log_a(x)
Merksatz: Der Logarithmus einer Potenz ist gleich dem Produkt aus dem Exponenten und dem Logarithmus der Basis.
Beispiel 1: \log_{100}\left(m^{17}\right) = 17\cdot\log_{100}(m)
Beispiel 2: \log_5\left(25^{x+1}\right) = (x+1) \cdot \log_5(25) = (x+1)\cdot 2 = 2x+2
Bemerkung: Achten Sie im Beispiel 2 - ganz wichtig - darauf, dass Klammern um den Exponenten gesetzt werden müssen!

Vielleicht kommen Ihnen die Logarithmengesetze fremd vor ... Der Logarithmus war aber aufgrund genau dieser Gesetze lange Zeit eine sehr, sehr nützliche und angesehene Rechenart. Wenn Sie sich die drei Gesetze anschauen, sehen Sie nämlich, dass jeweils die Rechenart einfacher wird:

  1. Logarithmengesetz: Aus einem Produkt wird eine Summe.
  2. Logarithmengesetz: Aus einem Quotienten wird eine Differenz.
  3. Logarithmengesetz: Aus einer Potenz wird ein Produkt.

Bevor es Taschenrechner und Computer gab, war dies eine immense Erleichterung!


Um das Ganze etwas plausibler zu machen:
Nach dem 1. Logarithmengesetz ist \log_a(x)+ \log_a(x) = \log_a(x\cdot x) = \log_a\left(x^2\right). Alternativ können Sie natürlich auch rechnen \log_a(x)+ \log_a(x) = 2\log_a(x) = \log_a\left(x^2\right). Hier kommen - zusätzlich zu den Logarithmengesetzen - die "guten alten" Rechenregeln aus Kapitel 5.2 zum Tragen.
Ähnlich ist nach dem 2. Logarithmengesetz \log_a(x)-\log_a(x) = \log_a\left(\dfrac{x}{x}\right) = \log_a(1) = 0. Alternativ können Sie natürlich auch argumentieren, dass das Ergebnis immer 0 ist, wenn man zwei gleiche Werte voneinander subtrahiert.

 

Ganz wichtig: Die drei genannten Logarithmengesetze sind die einzigen Gesetze für das Rechnen mit Logarithmen!
Das bedeutet:
1. Steht im Argument ein Produkt, ein Quotient oder eine Potenz, kann man dies nur genau so zusammenfassen, wie hier gezeigt wurde. Klammern inklusive …
2. Steht im Argument eine andere Rechenoperation, z. B. eine Summe oder eine Differenz, kann man den Term nicht weiter umformen.


Ebenfalls ganz wichtig: Alle anderen Rechenregeln, die Sie bislang kennengelernt haben (z. B. Punkt vor Strich, Ausklammern, Bruchrechenregeln, binomische Formeln), gelten auch dann, wenn Logarithmen in einer Rechnung auftauchen!


Ein Beispiel mit Zahlen:
\begin{array}{rclcl} \log_{10}(1.000\cdot 100) &=& \log_{10}(1.000)+\log_{10}(100) &\vert& \text{1. Logarithmengesetz} \cr&=& \log_{10}(10^3)+\log_{10}(10^2) \cr&=& 3+2 \cr&=& 5 \end{array}
Alternativ kann man rechnen:
\begin{array}{rclcl} \log_{10}(1.000\cdot 100) &=& \log_{10}(100.000) \cr&=& \log_{10}(10^5) \cr&=& 5 \end{array}

An dieser Rechnung sehen Sie auch, dass \log_{10}(1.000\cdot 100) = 5 nicht das Gleiche ist wie \log_{10}(1.000) \cdot \log_{10}(100) = 3\cdot 2 = 6. Das 1. Logarithmengesetz besagt ja aber auch, dass nicht multipliziert, sondern addiert werden muss ...


Außer in solchen Spezialfällen kann man Logarithmen - ähnlich wie Wurzeln - im Kopf häufig nur abschätzen. Sie dürfen, wenn es um das Ausrechnen konkreter Werte geht, also ruhig Ihren Taschenrechner fragen …


Da die Logarithmengesetze vermutlich vielen nicht vertraut sind, sollten wir uns einige Fälle, die ähnlich aussehen, aber sorgfältig unterschieden werden müssen, genauer anschauen:

\log_a(x)+\log_a(y)+\log_a(z) = \log_a(x\cdot y\cdot z)  
\log_a(x)\cdot\log_a(y)\cdot\log_a(z) = ?  
Im ersten Fall werden mehrere Logarithmen addiert, im zweiten Fall werden sie multipliziert. Einmal kann man die Logarithmen zusammenfassen, einmal nicht. Es hat ja bislang immer einen Unterschied gemacht, ob man addiert oder multipliziert. Insofern wäre es schon überraschend, wenn das ausgerechnet bei Logarithmen anders wäre ...


\log_a(x)+\log_a(y)+\log_a(z) = \log_a(x\cdot y\cdot z)  
\log_a(x+y+z) = ?  
Im ersten Fall werden mehrere Logarithmen addiert, im zweiten Fall wird im Argument eines Logarithmus addiert. Das hat Auswirkungen auf die Rechenreihenfolge: Im ersten Fall werden erst die Logarithmen berechnet und dann addiert; im zweiten Fall wird erst addiert und dann der Logarithmus berechnet.


\log_a(x)\cdot\log_a(y)\cdot\log_a(z) = ?  
\log_a(x\cdot y\cdot z) = \log_a(x)+\log_a(y)+\log_a(z)  
Im ersten Fall werden mehrere Logarithmen multipliziert, im zweiten Fall wird im Argument eines Logarithmus multipliziert. Hier gibt es nur für den zweiten Fall eine Umformungsregel, die dem 1. Logarithmengesetz entspricht (von "rechts nach links" geschrieben).


\log_a(x+y+z) = ?  
\log_a(x\cdot y\cdot z) = \log_a(x)+\log_a(y)+\log_a(z)  
Im ersten Fall wird im Argument des Logarithmus addiert, im zweiten Fall wird im Argument des Logarithmus multipliziert. Im zweiten Fall kann wieder das 1. Logarithmengesetz angewendet werden.


 

Die gleichen Fälle müssen unterschieden werden, wenn nicht addiert und multipliziert, sondern subtrahiert und dividiert wird. Ein kleiner "Stolperstein" kommt hinzu: -\log_a(x)-\log_a(y)-\log_a(z) = \log_a\left(\dfrac{1}{x\cdot y\cdot z}\right). Alle Argumente müssen (wegen des Minuszeichens vor den Logarithmen) in den Nenner. Der Zähler kann aber natürlich nicht leer bleiben. Dass es eine 1 im Zähler sein muss, hat den gleichen Grund, wie beim "normalen" Bilden eines Kehrwertes. Der Kehrwert von 5 ist ja beispielsweise \frac{1}{5}.

Achtung: Keine dieser Rechenoperationen funktioniert, wenn die Basen unterschiedlich sind. Z. B. ist \log_a(x)+\log_b(y)+\log_a(z) = \log_a(xz)+\log_b(y). Potenzen mit unterschiedlichen Basen kann man ja auch nicht zusammenfassen ...


Dann muss man noch etwas aufpassen, wenn Logarithmen mit anderen Rechenoperationen kombiniert werden, was ja meist der Fall ist:

  • \log_a\left(5x^2\right) \neq 2\log_a(5x), sondern \log_a\left(5x^2\right) = \log_a(5)+2\log_a(x)
    Das Quadrat bezieht sich nur auf die Variable x und nicht auf den Koeffizienten 5. Wenn man 2\log_a(5x) "zurückumformt", ergibt sind 2\log_a(5x) = \log_a\left((5x)^2\right) = \log_a\left(25x^2\right).

  • 8\log_a(x\cdot y) \neq 8\log_a(x)+\log_a(y), sondern 8\log_a(x\cdot y) = 8\left(\log_a(x)+\log_a(y)\right) = 8\log_a(x)+8\log_a(y)
    Der Koeffizient 8 bezieht sich auf den gesamten Logarithmus und damit auf beide Argumente. Das muss auch dann so bleiben, wenn es nach dem Umformen zwei Logarithmenterme mit jeweils einem Argument gibt. Ebenso müssen Klammern gesetzt werden, wenn statt dem 1. das 2. Logarithmengesetz angewendet wird.

In beiden Fällen hat das "Aufpassen-Müssen" eigentlich weniger mit Logarithmen zu tun, sondern vielmehr mit der Rechenregel Potenz- geht vor Punkt- geht vor Strichrechnung. Sie werden es schon gemerkt haben, dass die Mathematik ziemlich pingelig ist, was das angeht ...

 

Erkenntnisse aus den Beispielen und darüber hinaus:

  • Wichtig: Der Logarithmus selbst (also das "Ergebnis") kann trotz der Einschränkungen für Basis und Argument durchaus negativ oder nicht ganzzahlig sein (siehe Beispiel zum 2. Logarithmengesetz). Der Logarithmus ist ja die Frage nach dem Exponenten, und dass es negative oder gebrochene Exponenten geben kann, haben wir in diesem Kapitel an verschiedenen Stellen gesehen.

  • Logarithmieren und Potenzieren mit der gleichen Basis heben einander auf. Sie "beantworten" ja genau die gegenteiligen Fragen: \log_a\left(a^x\right) = x (siehe Beispiel zum 1. Logarithmengesetz). Ein Spezialfall davon ist: \log_a\left(a\right) = \log_a\left(a^1\right) =1

  • Umgekehrt funktioniert es genauso: a^{\log_a(x)} = x

  • Egal, welche Basis wir betrachten: \log_a(1) = 0
    Denn es ist ja a^0=1 für alle Basen a, die wir hier betrachten.

 

Noch ein letztes Rechengesetz zum Logarithmus, die sogenannte Basistransformation: \log_b(x) = \dfrac{\log_a(x)}{\log_a(b)}
Der Bruch auf der rechten Seite sieht zwar kompliziert aus ... Der Nenner \log_a(b) ist aber einfach nur eine Zahl. Insofern wird das Ganze auch nicht komplizierter, als es ohnehin schon war. Diese Umrechnung wird u. a. gebraucht, wenn

  • b eine ungewöhnliche Basis für den Logarithmus ist.
  • in einer Rechnung mehrere Logarithmen mit unterschiedlichen Basen auftreten.

Dieses Gesetz ermöglicht es, eine beliebige andere Zahl a (mit den genannten Einschränkungen) zur Basis des Logarithmus zu machen. Man "transformiert" also die Basis. Dadurch kann man z. B.

  • eine Basis wählen, mit der der Taschenrechner umgehen kann. Die meisten Taschenrechner "kennen" nämlich nur die Basen e und 10.
  • alle Basen in einer Rechnung vereinheitlichen, damit man die Logarithmen dann mithilfe der Logarithmengesetze zusammenfassen kann (siehe oben).

 

Namenskonventionen

Einige Logarithmen, die man besonders häufig braucht, haben spezielle Namen bekommen:

  • Zweierlogarithmus oder logarithmus dualis: \log_2(x) = \text{ld}(x)
    Der Zweierlogarithmus wird u. a. in der Informatik für Rechnungen im Binärsystem verwendet.
  • Natürlicher Logarithmus oder logarithmus naturalis: \log_e(x) = \ln(x)
    Der natürliche Logarithmus hängt eng mit Wachstumsprozessen und der Zinsrechnung zusammen.
  • Zehnerlogarithmus oder dekadischer Logarithmus: \log_{10}(x) = \lg(x)
    Der Zehnerlogarithmus spielt beispielsweise bei der Berechnung des pH-Werts und der Bestimmung der Erdbebenstärke auf der Richterskala eine Rolle.


Ein paar Worte noch zum natürlichen Logarithmus: Er ist vermutlich der am häufigsten verwendete Logarithmus. Das liegt wohl daran, dass er eng mit der e-Funktion zusammenhängt, die wiederum beim Ableiten/Integrieren sehr angenehme Eigenschaften hat. Seine Basis ist die nach dem Schweizer Mathematiker Leonhard Euler benannte eulersche Zahl e = 2{,}718281828459..., wie \pi eine irrationale Zahl. Man erhält sie z. B. aus dieser unendlichen Rechnung:
{\large e = 1 \,+ \,\frac{1}{1} \,+ \,\frac{1}{1 \cdot 2} \,+ \,\frac{1}{1 \cdot 2 \cdot 3} \,+ \,\frac{1}{1 \cdot 2 \cdot 3 \cdot 4} \,+ \,\frac{1}{1 \cdot 2 \cdot 3 \cdot 4 \cdot 5} \,+ \,\frac{1}{1 \cdot 2 \cdot 3 \cdot 4 \cdot 5 \cdot 6} \,+ \,\frac{1}{1 \cdot 2 \cdot 3 \cdot 4 \cdot 5 \cdot 6 \cdot 7} \,+ \,\dots}

Wichtig: So verlockend es auch sein mag: \ln\left(e^x+2\right), \ln\left (e^{2x}-e^x\right) und ähnliche Terme lassen sich nicht vereinfachen, da im Argument eine Summe bzw. eine Differenz steht.

 

Termumformungen

Wir hatten "Terme" im Kapitel Lineare Gleichungen definiert als "eine mathematisch sinnvolle Kombination aus Zahlen, Konstanten, Variablen, Klammern und Rechenoperationen". Durch die Konzepte dieses Kapitels sind viele neue Möglichkeiten, Terme zu definieren, hinzugekommen, die alle an der einen oder anderen Stelle gebraucht werden. Hier zum Abschluss noch ein paar Überlegungen für den Anwendungsfall:

Schreibkonventionen

In Termen ist es üblich:

  • Zahlen an den Anfang zu schreiben
  • Variablen (auch in Produkten) alphabetisch zu ordnen
  • bei Potenzen die Variablen mit dem höchsten Exponenten zuerst zu schreiben

Diese Punkte haben keine mathematische Bedeutung, helfen aber dabei, den Überblick zu behalten - und das ist ja immer gut ...

 

Was darf man mit Termen machen?

Man darf:

  • Zahlen einsetzen und den sich ergebenden Wert ausrechnen
  • einen Term in andere einsetzen
  • Terme vergleichen
  • Terme vereinfachen


Wichtig: Man muss immer darauf achten, nur mit Werten zu hantieren, für die der Term definiert ist. Für Wurzeln und Logarithmen wurden die Definitionsbereiche oben auf dieser Seite beschrieben. Zusätzlich muss man noch aufpassen, dass nicht durch 0 dividiert wird.

Noch eine Art Vokabel: Man sagt "ein Term nimmt einen speziellen Wert an", wenn das Einsetzen von irgendeiner Zahl zu diesem speziellen Wert führt, z. B. nimmt der Term \frac{x^2}{x-4} für x=1 den Wert -\frac{1}{3} an. Für x=4 ist der Term \frac{x^2}{x-4} nicht definiert, weil bei diesem Wert der Nenner 0 würde.

 

Verfahren zur Termvereinfachung

Terme zu vereinfachen, ist vermutlich der häufigste Anwendungsfall - und wahrscheinlich auch einer der schwierigsten ... Es gibt nämlich zum Vereinfachen von Termen viele verschiedene Verfahren und Rechengesetze, wie ja vermutlich in diesem Kapitel deutlich wurde. Daneben können folgende Gesetz und Regeln helfen:

Schauen wir uns die beiden Einschränkungen beim Kürzen noch etwas genauer an:

zu 1.
Im Kapitel Bruchrechnung hatten wir schon mal betrachtet, was passiert, wenn man aus Summen kürzt: Das Ergebnis ist dann falsch! Genauso verhält es sich, wenn die Brüche Variablen enthalten - man sieht die Fehler dann nur nicht so schnell. Beispiel: Schauen wir uns den Bruch \dfrac{5+x}{x} an. Wenn man hier x einfach herausstreichen würde, bleibt als Ergebnis nur die 5. Warum das nicht stimmen kann, sieht man, wenn man für das x verschiedene Zahlenwerte einsetzt: Dann erhält man nämlich auch verschiedene Ergebnisse, beispielsweise

x=1: \dfrac{5+1}{1} = \dfrac{6}{5} = 1{,}2
x=0{,}03: \dfrac{5+0{,}03}{0{,}03} = \dfrac{503}{3} \approx 167{,}67
x=3.018: \dfrac{5+3.018}{3.018} = \dfrac{3.023}{3.018} \approx 1{,}00166

Die Unterschiede sind offensichtlich, was (hoffentlich) klar macht, dass man wirklich nicht aus Summer kürzen darf …

Etwas anderes ist es, wenn der Bruch \dfrac{5\cdot x}{x} lautet. Dann darf das x gekürzt werden, weil ja hier nur Punktrechnung aufeinandertrifft. Auch hierzu ein paar Beispiele:

x=1: \dfrac{5\cdot 1}{1} = \dfrac{5}{5} = 5
x=0{,}03: \dfrac{5\cdot 0{,}03}{0{,}03} = \dfrac{0{,}15}{0{,}03} = 5
x=3.018: \dfrac{5\cdot 3.018}{3.018} = \dfrac{15.090}{3.018} = 5

Ganz wichtig: Bei diesen Rechnungen scheint der Bruch zu "verschwinden", aber es ist ja 5 = \dfrac{5}{1}. Der Bruch versteckt sich also nur etwas ... Bei \dfrac{x}{3x} = \dfrac{1}{3} darf man auf keinen Fall die 1 im Zähler vergessen! Ob man \dfrac{1}{3} Kuchen hat oder 3 ist ja ein deutlicher Unterschied …

zu 2.
Haben Potenzen im Zähler und im Nenner unterschiedliche Basen, darf nicht gekürzt werden, auch nicht im Exponenten! Dazu ein Beispiel: \dfrac{x^4}{y^2} ist im Allgemeinen nicht das gleiche wie \dfrac{x^2}{y}, denn \dfrac{x^4}{y^2} = \dfrac{x\cdot x\cdot x\cdot x}{y\cdot y} \; \neq \; \dfrac{x\cdot x}{y} = \dfrac{x^2}{y}. Das wird spätestens dann deutlich, wenn statt der Variablen Zahlenwerte im Bruch stehen: \dfrac{5^4}{3^2} = \dfrac{5 \cdot 5\cdot 5\cdot 5}{3\cdot 3} \; \neq \; \dfrac{5\cdot 5}{3} = \dfrac{5^2}{3}. Wahrscheinlich würde hier auch niemand auf die Idee kommen zu kürzen, da Zähler und Nenner ja offensichtlich nicht die gleichen Faktoren enthalten. Sind Variablen in einem Bruch enthalten, wissen wir nicht, ob sie für Zahlenwerte mit gleichen Faktoren stehen oder nicht. Daher darf in diesem Fall nicht gekürzt werden.
Das gilt natürlich ebenso, wenn die Terme in Zähler und Nenner komplexer werden, z. B. \dfrac{3x^2}{(x-5)^2} = \dfrac{3\cdot x \cdot x}{(x-5)\cdot(x-5)} = \dfrac{3\cdot x \cdot x}{25-10x+x^2}. Und das ist eben nicht das gleiche wie \dfrac{3x}{x-5}, was man erhielte, wenn man bei \dfrac{3x^2}{(x-5)^2} die Quadrate "wegkürzen" würde.
Insofern: Man darf aus Exponenten kürzen - aber nur dann, wenn die Basen gleich sind, weil die Basis den Faktor bestimmt und der Exponent "nur" festlegt, wie häufig dieser Faktor mit sich selbst multipliziert wird.


Schwierig ist das Vereinfachen von Termen vor allem deswegen, weil nicht allgemein gesagt werden kann, welche Form des Terms die geschickteste ist. Das hängt stark von der gegebenen Aufgabe ab, z. B. ist es zum Kürzen notwendig, dass die Terme faktorisiert vorliegen. Umgekehrt erleichtert es das Ableiten, wenn ein Term möglichst wenig Produkte enthält. Um zu erkennen, welche Umformungen möglich und sinnvoll sind, ist (leider) viel Übung nötig ...

 

Zum Abschluss

Warum sind Terme, die sich aus Potenzen, Wurzeln und Logarithmen zusammensetzen, so wichtig? Das liegt daran, dass sie uns allgemeingültige Beschreibungen von Zusammenhängen und damit umfassende, absolute Problemlösungen ermöglichen. Das ist nicht zuletzt eine Voraussetzung für den Computereinsatz zum Lösen mathematischer Probleme: Wir müssen den Computern ja die Vorgehensweisen für den Lösungsprozess "mitteilen" - und das geschieht über Terme. Etwas naheliegender ist der Gebrauch von Termen beim Lösen von Gleichungen. Auch hier muss man wissen, nach welchen Regeln - mathematisch korrekt - mit den Termen umgegangen werden muss, sonst würde man nicht die richtige Lösung ermitteln können. Ein letzter Grund, warum es sich lohnt, gewisse Fähigkeiten im Umgang mit Termen zu erwerben: Wer Terme sicher umformen kann, muss sich von bestimmten Formeln (z. B. von der Prozentformel) nur eine Form merken - und kann sich die restlichen Formen, wenn sie benötigt werden, herleiten.

Übersicht:

 

8.3 Potenzen, Wurzeln, Logarithmen - Lösungen

1. Aufgabe

Hinweis 1: Über die Teilbarkeitsregeln können Basen ausgeschlossen werden. Z. B. ist 121 nicht durch 2, 3, 4, 5, 6, 7, 8, 9 und 10 teilbar. Erst die 11 kommt überhaupt als Basis infrage.
Hinweis 2: Das Erkennen von Quadratzahlen ist in vielen Fällen, so auch hier, sehr nützlich.
Hinweis 3: Aufgrund des 5. Potenzgesetzes können Sie bei 9) und 10) Zähler und Nenner getrennt betrachten.

1) 400=20^2

  6) 1.331=11^3
2) 100.000=10^5

  7) 196=14^2

3) 256=2^8

  8) 98=98^1

4) 121=11^2

  9) \dfrac{1}{64}=\left(\dfrac{1}{2}\right)^6

5) 81=3^4

  10) \dfrac{81}{625}=\left(\dfrac{3}{5}\right)^4

 

2. Aufgabe

Bemerkung zu 1) und 2): Auch bei Wurzeln wirken Addition und Subtraktion nur auf die Koeffizienten nicht auf Wurzelexponenten und Radikanden. Erläuterungen siehe auch hier.

1)
2\sqrt{x} + 3\sqrt{x} = 5\sqrt{x}


2)
2\sqrt{x} - 3\sqrt{x} = -1\sqrt{x} = -\sqrt{x}


3)
2\sqrt{x} \cdot 3\sqrt{x} = 6x


4)
2\sqrt{x} : 3\sqrt{x} = 2\sqrt{x} : 3 \cdot \sqrt{x} = \dfrac{2}{3} \sqrt{x} \cdot \sqrt{x} = \dfrac{2}{3} x

Bemerkung: Das Ergebnis mag vielleicht verwundern ... Aber eine Rechenoperation bezieht sich immer nur auf die nächste Zahl bzw. die nächste Variable - solange keine Klammern gesetzt wurden. 2\sqrt{x} : 3\sqrt{x} ist also nicht das Gleiche wie 2\sqrt{x} : \left(3\sqrt{x}\right) = \dfrac{2\sqrt{x}}{3\sqrt{x}} = \dfrac{2}{3}. Hier ist es - mal wieder - sehr wichtig, die Variante mit Klammern sorgfältig von der Variante ohne Klammern zu unterscheiden.

 

3. Aufgabe

Eine Bemerkung vorweg: Bitte denken Sie immer an die richtige Reihenfolge der Rechenoperationen: Potenz- vor Punkt- vor Strichrechnung!


1)
\left(12x+7y\right)^2 = 12^2x^2+2\cdot 12x\cdot 7y+7^2y^2 = 144x^2+168xy+49y^2

Vorgehen: 1. binomische Formel


2)
{10\left(6k+8l\right)^2 = 10\left(6^2k^2+2\cdot 6k\cdot 8l+8^2l^2\right) = 10\left(36k^2+96kl+64l^2\right) = 360k^2+960kl+640l^2}

Vorgehen:
1. binomische Formel


3)
\left(\dfrac{1}{2}x-z\right)^2 = \left(\dfrac{1}{2}\right)^2x^2-2\cdot \dfrac{1}{2}x\cdot z+z^2 = \dfrac{1}{4}x^2-xz+z^2

Vorgehen:
2. binomische Formel


4)
{\left(-4a+7b\right)\left(-7a+4b\right) = -4a\cdot\left(-7a\right)-4a\cdot 4b+7b\cdot\left(-7a\right)+7b\cdot 4b = 28a^2-16ab-49ab+28b^2 = 28a^2-65ab+28b^2}

Vorgehen:
Distributivgesetz

Bemerkung: Hier greift keine der binomischen Formeln. Daher müssen die Klammern hier auf herkömmliche Art und Weise aufgelöst werden.

 
5)
{-\left(4m-9p\right)^2 = -\left(4^2m^2-2\cdot 4m\cdot 9p+9^2p^2\right) = -\left(16m^2-72mp+81p^2\right) = -16m^2+72mp-81p^2}

Vorgehen:
2. binomische Formel


6)
\left(12a-b\right)\left(12a+b\right) = 12^2a^2-b^2 = 144a^2-b^2

Vorgehen:
3. binomische Formel

 
7)
{\dfrac{3}{4}\left(10y-\dfrac{8}{9}z\right)\left(-\dfrac{8z}{9}+10y\right) = \dfrac{3}{4}\left(10^2y^2-2\cdot 10y\cdot\dfrac{8}{9}z+\left(\dfrac{8}{9}\right)^2z^2\right) = \dfrac{3}{4}\left(100y^2-\dfrac{160}{9}yz+\dfrac{64}{81}z^2\right) = 75y^2-\dfrac{40}{3}yz+\dfrac{16}{27}z^2}

Vorgehen:
2. binomische Formel
 
Bemerkung: \left(10y-\dfrac{8}{9}z\right)\left(-\dfrac{8z}{9}+10y\right) sieht vielleicht ein bisschen ungewöhnlich aus für eine 2. binomische Formel, ist aber nichts anderes als \left(10y-\dfrac{8}{9}z\right)^2. Die Summanden innerhalb der Klammern dürfen ja wegen des Kommutativgesetzes vertauscht werden.


8)
{46\left(\dfrac{5}{6}x-\dfrac{5}{3}z\right)\left(\dfrac{6}{5}x-\dfrac{3}{5}z\right) = 46\left(\dfrac{5}{6}x\cdot\dfrac{6}{5}x+\dfrac{5}{6}x\cdot\left(-\dfrac{3}{5}z\right)-\dfrac{5}{3}z\cdot\dfrac{6}{5}x-\dfrac{5}{3}z\cdot\left(-\dfrac{3}{5}z\right)\right) = 46\left(x^2-\dfrac{1}{2}xz-2xz+z^2\right) = 46x^2-115xz+46z^2}

Vorgehen:
Distributivgesetz
 
Bemerkung: Hier greift keine der binomischen Formeln. Daher müssen die Klammern hier auf herkömmliche Art und Weise aufgelöst werden.


9)
\left(p+5q\right)\left(5q+p\right) = p^2+2\cdot p\cdot 5q+5^2q^2 = p^2+10pq+25q^2

Vorgehen:
1. binomische Formel
 
Bemerkung: \left(p+5q\right)\left(5q+p\right) sieht vielleicht ein bisschen ungewöhnlich aus für eine 1. binomische Formel, ist aber nichts anderes als \left(p+5q\right)^2. Die Summanden innerhalb der Klammern dürfen ja wegen des Kommutativgesetzes vertauscht werden.


10)
\left(-x+3z\right)\left(-x-3z\right) = x^2-3^2z^2 = x^2-9z^2

Vorgehen:
3. binomische Formel

 
11)
{-\dfrac{1}{5}\left(-10s+11t\right)\left(10s-11t\right) = -\dfrac{1}{5}\left(-10s\cdot 10s-10s\cdot\left(-11t\right)+11t\cdot 10s+11t\cdot\left(-11t\right)\right) = -\dfrac{1}{5}\left(-100s^2+110st+110st-121t^2\right) = 20s^2-44st+\dfrac{121}{5}t^2}

Vorgehen:
Distributivgesetz
 
Bemerkung: Hier greift keine der binomischen Formeln. Daher müssen die Klammern hier auf herkömmliche Art und Weise aufgelöst werden.

 
12)
-\left(8n-q\right)\left(q+8n\right) = -\left(8^2n^2-q^2\right) = -64n^2+q^2

Vorgehen:
3. binomische Formel
 
Bemerkung: \left(8n-q\right)\left(q+8n\right) sieht vielleicht ein bisschen ungewöhnlich aus für eine 3. binomische Formel, ist aber nichts anderes als \left(8n-q\right)\left(8n+q\right). Die Summanden innerhalb der Klammern dürfen ja wegen des Kommutativgesetzes vertauscht werden.


13)
{3\left(x-2\right)\left(x+2\right)\left(x+3\right) = 3\left(x^2-4\right)\left(x+3\right) = 3\left(x^3+3x^2-4x-12\right) = 3x^3+9x^2-12x-36}

Vorgehen:
3. binomische Formel und Distributivgesetz


14)
{-15\left(a+b\right)^2+30ba = -15\left(a^2+2ab+b^2\right)+30ab = -15a^2-30ab-15b^2+30ab = -15a^2-15b^2}

Vorgehen:
1. binomische Formel


15)
{\left(\dfrac{1}{4}x-1\right)^2 \left(8x+16\right) = \left(\dfrac{1}{16}x^2-\dfrac{1}{2}x+1\right)\left(8x+16\right) = \dfrac{1}{2}x^3-4x^2+8x+x^2-8x+16 = \dfrac{1}{2}x^3-3x^2+16}

Vorgehen:
2. binomische Formel und Distributivgesetz


16)
\left(x+y\right)^2-\left(x-y\right)^2-4xy = x^2+2xy+y^2-\left(x^2-2xy+y^2\right)-4xy = 0

Vorgehen:
1. und 2. binomische Formel


17)
{\left(3x+5y\right)^2+4\left(3x-5y\right)^2 = 9x^2+30xy+25y^2+4\left(9x^2-30xy+25y^2\right) = 9x^2+30xy+25y^2+36x^2-120xy+100y^2 = 45x^2-90xy+125y^2}

Vorgehen:
1. und 2. binomische Formel


18)
\dfrac{\left(9a-5b\right)^2}{\left(81a^2-90ab+25b^2\right)^2} = \dfrac{\left(9a-5b\right)^2}{\left(\left(9a-5b\right)^2\right)^2} = \dfrac{\left(9a-5b\right)^2}{\left(9a-5b\right)^4} = \dfrac{1}{\left(9a-5b\right)^2}

Vorgehen:
2. binomische Formel, dann kürzen


19)
\left(\dfrac{7h-9n}{7h+9n}\right)^2 = \dfrac{\left(7h-9n\right)^2}{\left(7h+9n\right)^2} = \dfrac{49h^2-126hn+81n^2}{49h^2+126hn+81n^2}

Vorgehen:
5. Potenzgesetz, 1. und 2. binomische Formel
 
Bemerkung: Hier kann man nicht kürzen!


20)
{5\cdot\dfrac{64s^2-256t^2}{\left(8s-16t\right)^2} = \dfrac{5\left(8s-16t\right)\left(8s+16t\right)}{\left(8s-16t\right)^2} = \dfrac{5\left(8s+16t\right)}{8s-16t} = \dfrac{40s+80t}{8s-16t} = \dfrac{8\left(5s+10t\right)}{8\left(s-2t\right)} = \dfrac{5s+10t}{s-2t}}

Vorgehen:
3. binomische Formel, dann kürzen und noch mal kürzen

 

4. Aufgabe

1)
100s^2-100st+25t^2 = 10^2s^2-2\cdot 10\cdot 5st+5^2t^2 = (10s-5t)^2

Vorgehen: 2. binomische Formel

 
2)
\dfrac{49}{4}v^2+7vw+w^2 = \left(\dfrac{7}{2}\right)^2v^2+2\cdot\dfrac{7}{2}\cdot 1vw+1^2w^2 = \left(\dfrac{7}{2}v+w\right)^2

Vorgehen: 1. binomische Formel

 
3)
169x^2+49y^2

Bemerkung: Dieser Term lässt sich nicht mithilfe der binomischen Formeln vereinfachen. Um die 3. binomische Formel anwenden zu können, müsste in der Mitte ein Minuszeichen stehen.

 
4)
x^2-64z^2 = x^2-8^2z^2 = \left(x-8z\right)\left(x+8z\right)

Vorgehen: 3. binomische Formel


5)
36u^2+168u+196x^2

Bemerkung: Dieser Term lässt sich nicht mithilfe der binomischen Formeln vereinfachen. Um die 1. binomische Formel anwenden zu können, müsste der Term in der Mitte die Variablen u und x enthalten.


6)
25a^2+60ab+100b^2

Bemerkung: Dieser Term lässt sich nicht mithilfe der binomischen Formeln vereinfachen. Um die 1. binomische Formel anwenden zu können, müsste der Term in der Mitte 2\cdot 5\cdot 10ab=100ab lauten.

 
7)
{\dfrac{5}{2}wy+\dfrac{5}{8}y^2+\dfrac{5}{2}w^2 = \dfrac{5}{8}\left(4w^2+4wy+y^2\right) = \dfrac{5}{8}\left(2^2w^2+2\cdot 2\cdot 1wy+1^2y^2\right) = \dfrac{5}{8}\left(2w+y\right)^2}

Vorgehen: 1. binomische Formel


8)
{-72a^2+36ac-\dfrac{9}{2}c^2 = -\dfrac{1}{2}\left(144a^2-72ac+9c^2\right) = -\dfrac{1}{2}\left(12^2a^2-2\cdot 12\cdot 3ac+3^2c^2\right) = -\dfrac{1}{2}\left(12a-3c\right)^2}

Vorgehen: 2. binomische Formel


9)
\dfrac{81}{2}a^2+36ad-4d^2

Bemerkung: Dieser Term lässt sich nicht mithilfe der binomischen Formeln vereinfachen. Die Vorzeichen passen nicht.


10)
80-405x^2 = 5\left(16-81x^2\right) = 5\left(4^2-9^2x^2\right) = 5\left(4+9x\right)\left(4-9x\right)
 
Vorgehen: 3. binomische Formel


11)
\dfrac{a^2-12a+36}{2a-12} = \dfrac{\left(a-6\right)^2}{2\left(a-6\right)} = \dfrac{a-6}{2}
 
Vorgehen: im Zähler 2. binomische Formel, im Nenner ausklammern, anschließend kürzen

 
12)
\dfrac{b^2-9a^2}{b^2+6ba+9a^2} = \dfrac{\left(b-3a\right)\left(b+3a\right)}{\left(b+3a\right)^2} = \dfrac{b-3a}{b+3a}
 
Vorgehen: im Zähler 3. binomische Formel, im Nenner 1. binomische Formel, anschließend kürzen

 
13)
\dfrac{x^2-\frac{2}{3}x+\frac{1}{9}}{x^2+\frac{26}{3}x-3} = \dfrac{\left(x-\frac{1}{3}\right)^2}{\left(x-\frac{1}{3}\right)\left(x+9\right)} = \dfrac{x-\frac{1}{3}}{x+9}

Vorgehen: im Zähler 2. binomische Formel, im Nenner ausklammern, anschließend kürzen

 
14)
{\dfrac{10s+120}{s^2-144} : \dfrac{s^2+24s+144}{\left(s-12\right)\left(s+12\right)} = \dfrac{10\left(s+12\right)}{\left(s+12\right)\left(s-12\right)} \cdot \dfrac{\left(s-12\right)\left(s+12\right)}{s^2+24s+144} = \dfrac{10\left(s+12\right)}{\left(s+12\right)\left(s-12\right)} \cdot \dfrac{\left(s-12\right)\left(s+12\right)}{\left(s+12\right)^2} = \dfrac{10}{s+12}}

Vorgehen: Multiplikation mit dem Kehrwert des zweiten Bruchs, dann im Zähler des ersten Bruchs ausklammern, im Nenner des ersten Bruchs 3. binomische Formel, im Nenner des zweites Bruchs 1. binomische Formel, anschließend kürzen

 
15)
\dfrac{4x^2-y^2}{4x^2-4xy+y^2} = \dfrac{\left(2x+y\right)\left(2x-y\right)}{\left(2x-y\right)^2} = \dfrac{2x+y}{2x-y}
 
Vorgehen: im Zähler 3. binomische Formel, im Nenner 2. binomische Formel, anschließend kürzen


16)
\dfrac{144y^2-169z^2}{144y^2+312zy+169z^2} = \dfrac{\left(12y-13z\right)\left(12y+13z\right)}{\left(12y+13z\right)^2} = \dfrac{12y-13z}{12y+13z}
 
Vorgehen: Im Zähler 3. binomische Formel, im Nenner 1. binomische Formel, dann kürzen


17)
{361a^2+36b^2+121c^2+144d^2+228ab-264cd = 361a^2+228ab+36b^2+121c^2-264cd+144d^2 = \left(19a+6b\right)^2+\left(11c-12d\right)^2}

Vorgehen: 1. und 2. binomische Formel

Bemerkung: Ein bisschen sortieren hilft ...


18)
\dfrac{289p^2+680pq+400}{289p^2-400q^2} = \dfrac{289p^2+680pq+400}{\left(17p+20q\right)\left(17p-20q\right)}
 
Vorgehen: im Nenner 3. binomische Formel

Bemerkung: Der Zähler kann nicht mithilfe der 1. binomischen Formel zusammengefasst werden, weil im letzten Term keine Variable steht.

 
19)
{\dfrac{1}{14x+15z} : \dfrac{14x-15z}{196x^2-225z^2} = \dfrac{1}{14x+15z} \cdot \dfrac{196x^2-225z^2}{14x-15z} = \dfrac{196x^2-225z^2}{\left(14x+15z\right)\left(14x-15z\right)} = \dfrac{\left(14x+15z\right)\left(14x-15z\right)}{\left(14x+15z\right)\left(14x-15z\right)} = 1}
 
Vorgehen: Multiplikation mit dem Kehrwert des zweiten Bruchs, im Zähler 3. binomische Formel, dann kürzen

 
20)
\dfrac{-48m}{36m^2+16}

Bemerkung: Dieser Term lässt sich nicht mithilfe der binomischen Formeln vereinfachen. Das ginge nur, wenn alle Bestandteile entweder im Zähler oder im Nenner stünden.

 

5. Aufgabe

1)
\genfrac{}{}{1pt}{0}{\frac{1}{2}}{x} = \dfrac{1}{2} : x = \dfrac{1}{2} \cdot \dfrac{1}{x} = \dfrac{1}{2x}

x \in \mathbb{R}\setminus_{\{0\}}


2)
\genfrac{}{}{1pt}{0}{q^5}{\frac{7}{q^2}} = q^5 :\dfrac{7}{q^2} = q^5 \cdot \dfrac{q^2}{7} = \dfrac{q^7}{7}

q \in \mathbb{R}


3)
\genfrac{}{}{1pt}{0}{\frac{a}{-13}}{\frac{b}{12}} = \dfrac{a}{-13} :\dfrac{b}{12} = -\dfrac{a}{13} \cdot \dfrac{12}{b} = -\dfrac{12a}{13b}

a \in \mathbb{R} und  b \in \mathbb{R}\setminus_{\{0\}}

4)
\genfrac{}{}{1pt}{0}{\frac{-3d}{-g^3}}{-4c} = \dfrac{-3d}{-g^3} : (-4c) = \dfrac{3d}{g^3} \cdot \left(-\dfrac{1}{4c} \right) = -\dfrac{3d}{4cg^3}

c,g \in \mathbb{R}\setminus_{\{0\}} und d \in \mathbb{R}


5)
\begin{array}{rclll} \genfrac{}{}{1pt}{0}{-45x}{\frac{5y}{9x}} &=& -45x : \dfrac{5y}{9x} \cr\cr &=& -45x \cdot \dfrac{9x}{5y} \cr\cr &=& -9x \cdot \dfrac{9x}{y} \cr\cr &=& -\dfrac{81x^2}{y} \end{array}

x,y \in \mathbb{R}\setminus_{\{0\}}

Bemerkung: Vor dem Multiplizieren wurden hier die 45x (erster Faktor) mit der 5y (Nenner vom zweiten Faktor) durch 5 gekürzt.


6)
\begin{array}{rclll}\dfrac{\frac{29a}{-60b}}{\frac{-c}{4b^2}} &=& \dfrac{29a}{-60b} : \dfrac{-c}{4b^2} \cr \cr &=& -\dfrac{29a}{60b} \cdot \left(-\dfrac{4b^2}{c}\right) \cr\cr &=& -\dfrac{29a}{15} \cdot \left( -\dfrac{b}{c}\right) \cr\cr &=& \dfrac{29ab}{15c} \end{array}

a \in \mathbb{R} und b,c \in \mathbb{R}\setminus_{\{0\}}

Bemerkung: Vor dem Multiplizieren wurden hier die 60b (Nenner vom ersten Faktor) mit der 4b^2 (Zähler vom zweiten Faktor) durch 4b gekürzt.


7)
\begin{array}{rclll} \genfrac{}{}{1pt}{0}{\frac{14c}{-5}}{\frac{-18c}{25a}} &=& \dfrac{14c}{-5} : \dfrac{-18c}{25a} \cr\cr &=& -\dfrac{14c}{5} \cdot \left(-\dfrac{25a}{18c}\right) \cr\cr &=& -\dfrac{7}{1} \cdot \left(-\dfrac{5a}{9}\right) \cr\cr &=& \dfrac{35a}{9} \end{array}

a,c \in \mathbb{R}\setminus_{\{0\}}

Bemerkung: Vor dem Multiplizieren wurden hier die 14c (Zähler vom ersten Faktor) mit der 18c (Nenner vom zweiten Faktor) durch 2c sowie die 5 (Nenner vom ersten Faktor) mit der 25a (Zähler vom zweiten Faktor) durch 5 gekürzt.


8)
\begin{array}{rclll} \genfrac{}{}{1pt}{0}{-\frac{31}{2r^2}}{\frac{t}{10}} &=& -\dfrac{31}{2r^2} : \dfrac{t}{10} \cr\cr &=& -\dfrac{31}{2r^2} \cdot \dfrac{10}{t} \cr\cr &=& -\dfrac{31}{r^2} \cdot \dfrac{5}{t} \cr\cr &=& -\dfrac{155}{r^2t}\end{array}

r,t \in \mathbb{R}\setminus_{\{0\}}

Bemerkung: Vor dem Multiplizieren wurden hier die 2r^2 (Nenner vom ersten Faktor) mit der 10 (Zähler vom zweiten Faktor) durch 2 gekürzt.


9)
\begin{array}{rclll} \genfrac{}{}{1pt}{0}{-4d^2}{\frac{32}{135d^4}} &=& -4d^2 : \dfrac{32}{135d^4} \cr\cr &=& -4d^2 \cdot \dfrac{135d^4}{32} \cr\cr &=& -d^2 \cdot \dfrac{135d^4}{8} \cr\cr &=& -\dfrac{135d^6}{8}\end{array}

d \in \mathbb{R}

Bemerkung: Vor dem Multiplizieren wurden hier die -4d^2 (erster Faktor) mit der 32 (Nenner vom zweiten Faktor) durch 4 gekürzt.


10)
\begin{array}{rcl} \genfrac{}{}{1pt}{0}{\frac{-71k}{-4lm}}{-401k} &=& \dfrac{-71k}{-4lm} : (-401k) \\\\&=& \dfrac{71k}{4lm} \cdot \left(-\dfrac{1}{401k} \right) \\\\&=& \dfrac{71}{4lm} \cdot \left(-\dfrac{1}{401} \right) \\\\&=& -\dfrac{71}{1.604lm} \end{array}

k,l,m \in \mathbb{R}\setminus_{\{0\}}

Bemerkung: Vor dem Multiplizieren wurden hier die 71k (Zähler vom ersten Faktor) mit der 401k (Nenner vom zweiten Faktor) durch k gekürzt.


11)
\begin{array}{rcl} \genfrac{}{}{1pt}{0}{-44x^9y^5}{\frac{-22x}{19y^2}} &=& -44x^9y^5 : \dfrac{-22x}{19y^2} \\\\ &=& -44x^9y^5 \cdot \left(-\dfrac{19y^2}{22x}\right) \\\\&=& -2x^8y^5 \cdot \left(\dfrac{-19y^2}{1}\right) \\\\&=& 38x^8y^7 \end{array}

x,y \in \mathbb{R}\setminus_{\{0\}}

Bemerkung: Vor dem Multiplizieren wurden hier die -44x^9y^5 (erster Faktor) mit der 22x (Nenner vom zweiten Faktor) durch 22x gekürzt.


12)
\begin{array}{rcl} \genfrac{}{}{1pt}{0}{-\frac{63x^2 y^3}{4z}}{\frac{7x^3 \cdot 3z^{23}}{z}} &=& -\dfrac{63x^2 y^3}{4z} : \dfrac{7x^3 \cdot 3z^{23}}{z} \\\\ &=& -\dfrac{63x^2 y^3}{4z} \cdot \dfrac{z}{21x^3 z^{23}} \\\\ &=& -\dfrac{3 y^3}{4} \cdot \dfrac{1}{x z^{23}} \\\\&=& -\dfrac{3y^3}{4xz^{23}} \end{array}

x,z \in \mathbb{R}\setminus_{\{0\}} und y\in\mathbb{R}

Bemerkung: Vor dem Multiplizieren wurden hier die 63 x^2 y^3 (Zähler vom ersten Faktor) mit der 21x^3 z^{23} (Nenner vom zweiten Faktor) durch 21x^2 sowie die 4z (Nenner vom ersten Faktor) mit der z (Zähler vom zweiten Faktor) durch z gekürzt.


13)
\begin{array}{rcl} \genfrac{}{}{1pt}{0}{\frac{24x^{10}}{5x^2y}}{\frac{-14x^2y^7}{3x^{10}}} &=& \dfrac{24x^{10}}{5x^2y} : \dfrac{-14x^2y^7}{3x^{10}} \\\\ &=& \dfrac{24x^{10}}{5x^2y} \cdot \left(-\dfrac{3x^{10}}{14x^2y^7}\right) \\\\ &=& \dfrac{12x^8}{5y} \cdot \left(-\dfrac{3x^8}{7y^7}\right) \\\\ &=& -\dfrac{36x^{16}}{35y^8} \end{array}

x,y \in \mathbb{R}\setminus_{\{0\}}

Bemerkung: Vor dem Multiplizieren wurden hier die 24x^{10} (Zähler vom ersten Faktor) mit der 14x^2y^7 (Nenner vom zweiten Faktor) durch 2x^2 sowie die 5x^2 y (Nenner vom ersten Faktor) mit der 3x^{10} (Zähler vom zweiten Faktor) durch x^2 gekürzt.


14)
\begin{array}{rcl} \genfrac{}{}{1pt}{0}{\frac{55qrt}{12x^2}}{-\frac{121x^5}{9q^2}} &=& \dfrac{55qrt}{12x^2} : \left(-\dfrac{121x^5}{9q^2}\right) \\\\ &=& \dfrac{55qrt}{12x^2} \cdot \left(-\dfrac{9q^2}{121x^5}\right) \\\\ &=& \dfrac{5qrt}{4x^2} \cdot \left(-\dfrac{3q^2}{11x^5}\right) \\\\ &=& -\dfrac{15q^3 rt}{44x^7} \end{array}

q,x \in \mathbb{R}\setminus_{\{0\}} und r,t \in \mathbb{R}

Bemerkung: Vor dem Multiplizieren wurden hier die 55qrt (Zähler vom ersten Faktor) mit der 121x^5 (Nenner vom zweiten Faktor) durch 11 sowie die 12x^2 (Nenner vom ersten Faktor) mit der 9q^2 (Zähler vom zweiten Faktor) durch 3 gekürzt. 


15)
\begin{array}{rcl} \genfrac{}{}{1pt}{0}{\frac{144x^2 y^{17}}{-35z^3}}{288x^{11}yz^{12}} &=& \dfrac{144x^2 y^{17}}{-35z^3} : \left(288x^{11}yz^{12} \right) \\\\&=& -\dfrac{144x^2 y^{17}}{35z^3} \cdot \dfrac{1}{288x^{11}yz^{12}} \\\\&=& -\dfrac{y^{16}}{35z^3} \cdot \dfrac{1}{2x^9z^{12}} \\\\&=& -\dfrac{y^{16}}{70x^9z^{15}} \end{array}

x,y,z \in \mathbb{R}\setminus_{\{0\}}

Bemerkung: Vor dem Multiplizieren wurden hier die 144x^2 y^{17} (Zähler vom ersten Faktor) mit der 288x^{11}yz^{12} (Nenner vom zweiten Faktor) durch 144x^2 y gekürzt. 


16)
\begin{array}{rcl}\genfrac{}{}{1pt}{0}{33b^{22}q^7}{\frac{99b^{19}q^{11}}{m^2}} &=& 33b^{22}q^7 : \dfrac{99b^{19}q^{11}}{m^2} \\\\&=& 33b^{22}q^7 \cdot \dfrac{m^2}{99b^{19}q^{11}} \\\\&=& b^3 \cdot \dfrac{m^2}{3q^4} \\\\&=& \dfrac{b^3m^2}{3q^4} \end{array}

b,m,q \in \mathbb{R}\setminus_{\{0\}}

Bemerkung: Vor dem Multiplizieren wurden hier die 33b^{22}q^7 (erster Faktor) mit der 99b^{19}q^{11} (Nenner vom zweiten Faktor) durch 33b^{19} q^7 gekürzt. 


17)
\begin{array}{rcl} \genfrac{}{}{1pt}{0}{11xyz^2}{\frac{7r^2 x}{-5rz^2}} &=& 11xyz^2 : \dfrac{7r^2 x}{-5rz^2} \\\\&=& 11xyz^2 \cdot \left(-\dfrac{5rz^2}{7r^2 x}\right) \\\\&=& 11yz^2 \cdot \left(-\dfrac{5z^2}{7r}\right) \\\\&=& -\dfrac{11yz^4}{7r} \end{array}

r,x,z \in \mathbb{R}\setminus_{\{0\}}

Bemerkung: Vor dem Multiplizieren wurden hier die 11xyz^2 (erster Faktor) mit der 7r^2 x (Nenner vom zweiten Faktor) durch x sowie der zweite Bruch durch r gekürzt. 


18)
\begin{array}{rcl} \genfrac{}{}{1pt}{0}{-\frac{8y^{13}}{7x}}{\frac{24y^2 z}{49rt^{11}}} &=& -\dfrac{8y^{13}}{7x} : \dfrac{24y^2 z}{49rt^{11}} \\\\ &=& -\dfrac{8y^{13}}{7x} \cdot \dfrac{49rt^{11}}{24y^2 z} \\\\&=& -\dfrac{y^{11}}{x} \cdot \dfrac{7rt^{11}}{3z} \\\\&=& -\dfrac{7rt^{11}y^{11}}{3xz} \end{array}

r,t,x,y,z \in \mathbb{R}\setminus_{\{0\}}

Bemerkung: Vor dem Multiplizieren wurden hier die 8y^{13} (Zähler vom ersten Faktor) mit der 24y^2 z (Nenner vom zweiten Faktor) durch 8y^2 sowie die 7x (Nenner vom ersten Faktor) mit der 49rt^{11} (Zähler vom zweiten Faktor) durch 7 gekürzt. 


19)
\begin{array}{rcl} \genfrac{}{}{1pt}{0}{\frac{23x^{66}}{3y^4}}{20x^{66}y^4} &=& \dfrac{23x^{66}}{3y^4} : \left(20x^{66}y^4\right) \\\\&=& \dfrac{23x^{66}}{3y^4} \cdot \dfrac{1}{20x^{66}y^4} \\\\&=& \dfrac{23}{3y^4} \cdot \dfrac{1}{20y^4} \\\\&=& \dfrac{23}{60y^8} \end{array}

x,y \in \mathbb{R}\setminus_{\{0\}}

Bemerkung: Vor dem Multiplizieren wurden hier die 23x^{66} (Zähler vom ersten Faktor) mit der 20x^{66}y^4 (Nenner vom zweiten Faktor) durch x^{66} gekürzt. 


20)
\begin{array}{rcl}\genfrac{}{}{1pt}{0}{\frac{14a^2 w}{9az^{17}}}{\frac{28wz}{123a}} &=& \dfrac{14a^2 w}{9az^{17}} : \dfrac{28wz}{123a} \\\\&=& \dfrac{14a^2 w}{9az^{17}} \cdot \dfrac{123a}{28wz} \\\\&=& \dfrac{a^2}{3z^{17}} \cdot \dfrac{41}{2z} \\\\ &=& \dfrac{41a^2}{6z^{18}} \end{array}

a,w,z \in \mathbb{R}\setminus_{\{0\}}

Bemerkung: Vor dem Multiplizieren wurden hier die 14a^2 w (Zähler vom ersten Faktor) mit der 28wz (Nenner vom zweiten Faktor) durch 14w sowie die 9az^{17} (Nenner vom ersten Faktor) mit der 123a (Zähler vom zweiten Faktor) durch 3a gekürzt. 

 

6. Aufgabe

Variablenwerte müssen ausgeschlossen werden, wenn

  • der Nenner für diesen Wert / diese Werte 0 wird,
  • der Radikand für diesen Wert / diese Werte negativ wird,
  • das Logarithmusargument für diesen Wert / diese Werte negativ oder 0 wird bzw.
  • die Basis eines Logarithmus' für diesen Wert / diese Werte negativ, 0 oder 1 ist.


Ganz wichtig: Egal, wie komplex die folgenden Aufgaben werden, es gelten immer die fundamentalen Rechenregeln, wie "Potenz vor Punkt vor Strich", Klammergesetze, die Gesetze der Bruchrechnung etc.


1)
Für alle x\in \mathbb{R} gilt:
\begin{array}{rcccl} x^2 \cdot x^3 &=& x^{2+3} &=& x^5\end{array}

Vorgehen: 1. Potenzgesetz


2)
Für alle m \in \mathbb{R}\backslash_{\{0\}} und n \in \mathbb{Z} oder für alle m\in \mathbb{R}^+ und n\in \mathbb{Q} gilt:
\begin{array}{rcccl} m^5 \cdot m^n \cdot m^{-3} &=& m^{5+n-3} &=& m^{2+n}\end{array}

Vorgehen: 1. Potenzgesetz


3)
Für alle b \in \mathbb{R}\backslash_{\{0\}} gilt:
\begin{array}{rcccl} \dfrac{b^6}{b^4} &=& b^{6-4} &=& b^2\end{array}

Vorgehen: 2. Potenzgesetz


4)
Für alle y \in \mathbb{R}\backslash_{\{0\}} gilt:
\begin{array}{rcccccccl}y : y^2 &=& y^1 : y^2 &=& y^{1-2} &=& y^{-1} &=& \dfrac{1}{y}\end{array}

Vorgehen: 2. Potenzgesetz, Festlegung: x^{-n}=\dfrac{1}{x^n}

 
5)
Für alle x\in \mathbb{R} gilt:
\begin{array}{rcccl} \left(x^3\right)^4 &=& x^{3 \cdot 4} &=& x^{12}\end{array}

Vorgehen: 3. Potenzgesetz

 
6)
Für alle a\in \mathbb{R} und b\in \mathbb{Z} oder für alle a\in \mathbb{R}^+_0 und b\in \mathbb{Q} gilt:
\begin{array}{rcl}\left(a^2\right)^b &=& a^{2 \cdot b}\end{array}

Vorgehen: 3. Potenzgesetz

Bemerkung: Ist b negativ oder 0, muss zusätzlich a\neq 0 gelten.

 
7)
Für alle n\in \mathbb{Q} gilt:
\begin{array}{rcccl}2^n \cdot 3^n &=& (2 \cdot 3)^n &=& 6^n \end{array}

Vorgehen: 4. Potenzgesetz

 
8)
Für alle a \in \mathbb{R}\backslash_{\{0\}} gilt:
\begin{array}{rcccccl}\dfrac{a^5}{(2a)^5} &=& \left( \dfrac{a}{2a}\right)^5 &=& \left(\dfrac{1}{2}\right)^5 &=& \dfrac{1}{32}\end{array}

Vorgehen: 5. Potenzgesetz

 
9)
Für alle m\in \mathbb{Q} gilt:
\begin{array}{rcccccl}16^m : 4^m &=& \dfrac{16^m}{4^m} &=& \left(\dfrac{16}{4}\right)^m &=& 4^m \end{array}

Vorgehen: 5. Potenzgesetz

 
10)
Für alle r,s,t \in \mathbb{R}\backslash_{\{0\}} gilt:
\begin{array}{ccl} \dfrac{r^2 s^{-3} r^{-4} s^{-1} t t^6}{t^{-5} r^4 s^6 s^{-2} r t^{10} r^{-3} s^{-7}} &=& \dfrac{r^{2-4} s^{-3-1} t^{1+6}}{r^{4+1-3} s^{6-2-7} t^{-5+10}} \cr\cr &=& \dfrac{r^{-2} s^{-4} t^7}{r^2 s^{-3} t^5} \cr\cr &=& r^{-2-2} s^{-4-(-3)} t^{7-5} \cr\cr &=& r^{-4} s^{-1} t^2 \cr\cr &=& \dfrac{t^2}{r^4 s^1} \end{array}

Vorgehen: Kommutativgesetz, 1. und 2. Potenzgesetz

 
11)
Für alle p,q \in \mathbb{R} gilt:
\begin{array}{ccl} (4p^3-6q^5)^3 &=& \left(4p^3-6q^5 \right)^2 \left(4p^3-6q^5 \right) \cr\cr &=& \lbrack \left(4p^3 \right)^2 + 2 \cdot 4p^3 \left(-6q^5 \right)+\left(-6q^5 \right)^2 \rbrack \left(4p^3-6q^5 \right) \cr\cr &=& \lbrack 16p^6-48p^3q^5+36q^{10}\rbrack \left(4p^3-6q^5 \right) \cr\cr &=& 16p^6 \cdot 4p^3 + 16p^6 \cdot \left(-6q^5 \right) - 48 p^3q^5 \cdot 4p^3 - 48 p^3q^5 \cdot \left(-6q^5 \right) + 36 q^{10} \cdot 4p^3 + 36 q^{10} \cdot \left(-6q^5 \right) \cr\cr &=& 64p^9-96p^6q^5-192p^6q^5+288p^3q^{10}+144p^3q^{10}-216q^{15} \cr\cr &=& 64p^9-288p^6q^5+432p^3q^{10}-216q^{15} \end{array}

Vorgehen: 2. binomische Formel, ausmultiplizieren, 3. und 4. Potenzgesetz
 
Bemerkung 1: Werden in einer Rechnung viele Klammern ineinander verschachtelt benötigt, kann man zur besseren Übersicht auch eckige oder geschweifte Klammern verwenden.
 
Bemerkung 2: Achten Sie darauf, dass z. B. bei \left(-6q^5\right)^2 Klammern gesetzt werden müssen. Auch das Minuszeichen gehört mit in die Klammer!
 
12)
Für alle y \in \mathbb{R}\backslash_{\{-2;2\}} gilt:
\begin{array}{rcl}\dfrac{y^4-16}{2y^4-16y^2+32} &=& \dfrac{ \left(y^2-4 \right) \left(y^2+4 \right)}{2\left(y^4-8y^2+16\right)} \\\\ &=& \dfrac{ \left(y^2-4 \right) \left(y^2+4 \right)}{2\left(y^2-4 \right)^2} \\\\ &=& \dfrac{y^2+4}{2\left(y^2-4\right)}\end{array}

Vorgehen: binomische Formeln "rückwärts", kürzen

Bemerkung: Sie erkennen die 3. binomische Formel im Zähler daran, dass zwei Terme, die jeweils für sich quadratisch sind, voneinander subtrahiert werden. Im Nenner gibt es zwei quadratische Terme mit jeweils positivem Vorzeichen sowie einen gemischten Term. Solch eine Konstellation ist ein guter Kandidat für die Anwendung der 1. oder 2. binomischen Formel. Es kann aber sein, dass sie sich nicht in dieser Weise umformen lässt, wenn nämlich der gemischte Term nicht zu den beiden quadratischen passt.

 
13)
Für alle x\in \mathbb{R} gilt:
\begin{array}{rcl} 5e^{-x+1}-2xe^{-x+1} &=& (5-2x)e^{-x+1}\end{array}

Vorgehen: ausklammern

 
14)
Für alle x \in \mathbb{R} und y \in \mathbb{R}\backslash_{\{1\}} gilt:
\begin{array}{ccl} \genfrac{}{}{1pt}{}{\dfrac{x+7}{2}}{\dfrac{8}{y-1}} &=& \dfrac{x+7}{2} : \dfrac{8}{y-1} \cr\cr &=& \dfrac{x+7}{2} \cdot \dfrac{y-1}{8} \cr\cr &=& \dfrac{\left(x+7\right)\left(y-1\right)}{2\cdot 8} \cr\cr &=& \dfrac{xy-x+7y-7}{16} \end{array}

Vorgehen: mit dem Kehrwert des Nennerbruches multiplizieren

Bemerkung 1: Dies ist grundsätzlich ein geschicktes Vorgehen für das Auflösen von Doppelbrüchen.

Bemerkung 2: Achten Sie darauf, beim Multiplizieren der Brüche die "versteckten" Klammern zu setzen!


15)
Für alle a \in \mathbb{R}\setminus_{\{0\}} und b \in \mathbb{R}\setminus_{\{0\}} mit a\neq b gilt:
\begin{array}{rcl} \genfrac{}{}{1pt}{0}{\frac{a}{b}-\frac{b}{a}}{\frac{1}{b}+\frac{1}{a}} \; : \; \left(a-b\right) &=& \genfrac{}{}{1pt}{0}{\frac{a^2}{ab}-\frac{b^2}{ab}}{\frac{a}{ab}+\frac{b}{ab}} \; : \; \left(a-b\right) \cr\cr &=& \genfrac{}{}{1pt}{0}{\frac{a^2-b^2}{ab}}{\frac{a+b}{ab}} \; : \; \left(a-b\right) \cr\cr &=& \dfrac{a^2-b^2}{ab} \; : \; \dfrac{a+b}{ab} \; : \; \left(a-b\right) \cr\cr &=& \dfrac{a^2-b^2}{ab} \; \cdot \; \dfrac{ab}{a+b}\cdot \dfrac{1}{a-b} \cr\cr &=& \dfrac{a^2-b^2}{1} \; \cdot \; \dfrac{1}{a+b}\cdot \dfrac{1}{a-b} \cr\cr&=& \dfrac{a^2-b^2}{(a+b) \cdot (a-b)} \cr\cr&=& \dfrac{(a+b)(a-b)}{(a+b)(a-b)} \cr\cr&=& 1\end{array}

Vorgehen: Brüche im Zähler und im Nenner gleichnamig machen und addieren/subtrahieren, dann mit dem Kehrwert des Nennerbruches multiplizieren, 3. binomische Formel, kürzen


16)
Für alle x \in \mathbb{R}^+ gilt:
\begin{array}{ccccl} \dfrac{1}{\sqrt{x}} &=& \dfrac{1 \cdot \sqrt{x}}{\sqrt{x} \cdot \sqrt{x}} &=& \dfrac{\sqrt{x}}{x} \end{array}

Vorgehen: erweitern mit \sqrt{x}
 
Bemerkung: Mit einer Wurzel im Nenner rechnet es sich meist nicht sehr gut. Daher erweitert man solche Brüche so, dass der Nenner rational wird. Eine ganz ähnliche Vorgehensweise wird bei der Division komplexer Zahlen, z. B. in der Elektrotechnik, benötigt.

 
17)
Für alle y \in \mathbb{R}^+_0 gilt:
\begin{array}{ccl} \left(y^{\frac{1}{3}}\right)^2 &=& y^{\frac{1}{3} \cdot 2} \cr &=& y^{\frac{2}{3}} \cr &=& \sqrt [3]{y^2} \end{array}

Vorgehen: 3. Potenzgesetz, Potenz in Wurzelschreibweise 

 
18)
Für alle m \in \mathbb{R}^+_0 gilt:
\begin{array}{ccl} 13\dfrac{\sqrt{4m-1}}{\left(4m-1\right)^{-0{,}5}} &=& 13\left(4m-1\right)^{\frac{1}{2}}\left(4m-1\right)^{-\left(-\frac{1}{2}\right)} \cr\cr &=& 13 \left(4m-1\right)^{\frac{1}{2}+\frac{1}{2}} \cr\cr &=& 13\left(4m-1\right)^1 \cr\cr &=& 52m-13 \end{array}

Vorgehen: Wurzel in Potenzschreibweise, Festlegung: x^{-n}=\dfrac{1}{x^n}, 1. Potenzgesetz

 
19)
Für alle x \in \mathbb{R}^+_0 gilt:
\begin{array}{ccl} \sqrt[3]{x} \cdot \sqrt{x} &=& x^{\frac{1}{3}} \cdot x^{\frac{1}{2}} \cr &=& x^{\frac{1}{3} + \frac{1}{2}} \cr &=& x^{\frac{5}{6}} \cr &=& \sqrt [6]{x^5} \end{array}

Vorgehen: Wurzeln in Potenzschreibweise, 1. Potenzgesetz

 
20)
Für alle a \in \mathbb{R}^+ gilt:
\begin{array}{ccl} a^{\frac{1}{2}} \cdot \left(a^3\right)^2 : a &=& a^{\frac{1}{2}} \cdot a^{3 \cdot 2} \cdot a^{-1} \cr &=& a^{\frac{1}{2}} \cdot a^6 \cdot a^{-1} \cr &=& a^{\frac{1}{2}+6-1} \cr &=& a^{\frac{11}{2}} \cr &=& \sqrt{a^{11}} \cr &=& \sqrt{a^{10} \cdot a} \cr &=& a^5 \cdot \sqrt{a} \end{array}

Vorgehen: 3. und 1. Potenzgesetz, Festlegung: x^{-n}=\dfrac{1}{x^n}, Potenz in Wurzelschreibweise
 
Bemerkung: Die Umformung vom vorletzten zum letzten Term nennt man "teilweises Wurzelziehen".

 
21)
Für alle b\in \mathbb{R}^+ gilt:
\begin{array}{ccl} \dfrac{ \sqrt[5]{b^3}}{b} &=& b^{\frac{3}{5}} \cdot b^{-1} \cr &=& b^{\frac{3}{5}-1} \cr &=& b^{-\frac{2}{5}} \cr &=& \dfrac{1}{\sqrt [5]{b^2}} \end{array}

Vorgehen: Wurzel in Potenzschreibweise, Festlegung: x^{-n}=\dfrac{1}{x^n}, 1. Potenzgesetz, Potenz wieder in Wurzelschreibweise

 
22)
Für alle z\in \mathbb{R} gilt:
\begin{array}{ccl} \left( \sqrt[4]{z^2}\right)^3 &=& \left( z^{\frac{2}{4}} \right)^3 \cr &=& \vert z \vert^{\frac{1}{2} \cdot 3} \cr &=& \vert z \vert^{\frac{3}{2}} \cr &=& \sqrt{\vert z \vert^3} \end{array}

Vorgehen: Wurzel in Potenzschreibweise und wieder zurück, zwischendurch 3. Potenzgesetz
 
Bemerkung: Bitte beachten Sie: \sqrt[4]{z^2}=\sqrt{\vert z \vert}, da es sonst Schwierigkeiten bei negativen Zahlen gibt. Bei \sqrt[4]{z^2} und \sqrt{\vert z \vert} dürfen sie eingesetzt werden - bei \sqrt{z} nicht.

 
23)
Für alle x \in \mathbb{R}^+ gilt:
\begin{array}{ccl} \dfrac{ \sqrt{x^3}}{x \cdot \sqrt{x}} &=& x^{\frac{3}{2}} \cdot x^{-1} \cdot x^{-\frac{1}{2}} \cr &=& x^{\frac{3}{2}-1-\frac{1}{2}} \cr &=& x^0 \cr &=& 1 \end{array}

Vorgehen: Wurzel in Potenzschreibweise, Festlegung: x^{-n}=\dfrac{1}{x^n}, 1. Potenzgesetz, Festlegung: x^0=1

 
24)
Für alle x \in \mathbb{R}^+ gilt:
\begin{array}{ccl} \left( \dfrac{1}{x^2} \cdot x^{-2} \right)^2 \cdot \sqrt[2]{x} &=& \left( x^{-2} \cdot x^{-2} \right)^2 \cdot x^{\frac{1}{2}} \cr &=& \left( x^{-2-2} \right)^2 \cdot x^{\frac{1}{2}} \cr &=& x^{-4 \cdot 2} \cdot x^{\frac{1}{2}} \cr &=& x^{-8+\frac{1}{2}} \cr &=& x^{-\frac{15}{2}} \cr\cr &=& \dfrac{1}{\sqrt{x^{15}}} \cr\cr &=& \dfrac{1}{x^7 \cdot \sqrt{x}} \cr\cr &=& \dfrac{\sqrt{x}}{x^8} \end{array}

Vorgehen: Festlegung: x^{-n}=\dfrac{1}{x^n}, Wurzel in Potenzschreibweise, 1., 3. und 4. Potenzgesetz
 
Bemerkung: Auch hier wurde so umgeformt, dass der Nenner am Ende rational ist (siehe Aufgabe 16).

 
25)
Für alle d \in \mathbb{R}\setminus_{\{0\}} und x\in \mathbb{R} gilt:
\begin{array}{ccl} \sqrt[3]{x^2d} \cdot d^{-\frac{4}{3}} &=& \sqrt[3]{x^2} \cdot \sqrt[3]{d} \cdot d^{-\frac{4}{3}} \cr\cr &=& \sqrt[3]{x^2} \cdot d^\frac{1}{3} \cdot d^{-\frac{4}{3}} \cr\cr &=& \sqrt[3]{x^2} \cdot d^{\frac{1}{3}-\frac{4}{3}} \cr\cr &=& \sqrt[3]{x^2} \cdot d^{-1} \cr\cr &=& \dfrac{\sqrt[3]{x^2}}{d} \end{array}

Vorgehen: 4. Potenzgesetz, Wurzel in Potenzschreibweise, 1. Potenzgesetz, Festlegung: x^{-n}=\dfrac{1}{x^n}

 
26)
Für alle x,y \in \mathbb{R}^+ gilt:
\begin{array}{ccl} \left(x^{1{,}25} : y^{-0{,}625}\right)^{-\frac{4}{5}} &=& \left(x^{1{,}25} : \dfrac{1}{y^{0{,}625}}\right)^{-\frac{4}{5}} \cr\cr &=& \left( x^{1{,}25} \cdot y^{0{,}625} \right)^{-\frac{4}{5}} \cr\cr &=& x^{1{,}25 \cdot \left(-\frac{4}{5}\right)}\cdot y^{0{,}625\cdot\left(-\frac{4}{5}\right)} \cr \cr &=& x^{\frac{5}{4} \cdot \left(-\frac{4}{5}\right)}\cdot y^{\frac{5}{8}\cdot\left(-\frac{4}{5}\right)} \cr \cr &=& x^{-1} \cdot y^{-\frac{1}{2}} \cr\cr &=& \dfrac{1}{x^1 \cdot y^{\frac{1}{2}}} \cr\cr &=& \dfrac{1}{x\cdot \sqrt{y}} \cr\cr &=& \dfrac{\sqrt{y}}{x\cdot y} \end{array}


Vorgehen: Festlegung: x^{-n}=\dfrac{1}{x^n}, mit dem Kehrwert des 2. Bruches multiplizieren, 3. und 4. Potenzgesetz, Potenz in Wurzelschreibweise
 
Bemerkung: Auch hier wurde so umgeformt, dass der Nenner am Ende rational ist (siehe Aufgabe 16).

 
27)
Für alle a,b \in \mathbb{R} gilt:
\sqrt{121a^8+166a^4b^2+64b^4}

Bemerkung: Der Term unter der Wurzel lässt sich nicht in ein Produkt umformen, da der mittlere Term nicht dem entspricht, was für die Anwendung der 1. binomischen Formel nötig wäre (siehe Aufgabe 12). Stünde unter der Wurzel 121a^8+176a^4b^2+64b^4= (11a^4+8b^2)^2, dann könnte man die 1. binomische Formel anwenden. Da das hier nicht der Fall ist und aus Summen keine Wurzeln gezogen werden können, kann man nicht weiter vereinfachen.

 
28)
Für alle x \in \mathbb{R}^+ und y \in \mathbb{R}\backslash_{\{0\}} gilt:
\begin{array}{ccl} \dfrac{4y \sqrt{x^3}}{16xy^2} &=& \dfrac{4x^{\frac{3}{2}}y}{16xy^2} \cr\cr &=& \dfrac{1}{4} x^{\frac{3}{2}-1}y^{1-2} \cr\cr &=& \dfrac{1}{4} x^{\frac{1}{2}}y^{-1} \cr\cr &=& \dfrac{\sqrt{x}}{4y} \end{array}

Vorgehen: Wurzel in Potenzschreibweise, Festlegung: x^{-n}=\dfrac{1}{x^n}, 1. und 2. Potenzgesetz

 
29)
Für alle x \in \mathbb{R}\backslash_{\{0\}} gilt:
\begin{array}{ccl} \sqrt{x^{-2}} &=& \sqrt{\dfrac{1}{x^2}} \cr\cr &=& \dfrac{\sqrt{1}}{\sqrt{x^2}} \cr\cr &=& \dfrac{1}{\vert x \vert} \end{array}

Vorgehen: Festlegung: x^{-n}=\dfrac{1}{x^n}, 4. Potenzgesetz
 
Bemerkung 1: Ein negativer Exponent erzeugt einen Bruch. Das Ergebnis muss nicht zwangsläufig auch negativ sein. Im Gegenteil: Ist der Exponent eine gerade Zahl, kann das Ergebnis nie negativ werden. Deshalb kann hier problemlos die Wurzel gezogen werden.
 
Bemerkung 2: Bitte beachten Sie, dass \sqrt{x^2} = \vert x \vert

 
30)
Für alle a, b\in \mathbb{R}^+ gilt:
\begin{array}{ccl} \sqrt[5]{\dfrac{\sqrt[4]{\left(a^4b^2\right)^{10}}}{\sqrt{a^{10}b^5}}} &=& \sqrt[5]{\dfrac{\sqrt[4]{a^{40}b^{20}}}{\sqrt{a^{10}b^5}}} \cr \cr &=& \sqrt[5]{\dfrac{a^{40\cdot \frac{1}{4}}b^{20 \cdot \frac{1}{4}}}{a^{10 \cdot \frac{1}{2}}b^{5 \cdot \frac{1}{2}}}} \cr \cr &=& \sqrt[5]{\dfrac{a^{10}b^5}{a^5b^\frac{5}{2}}} \cr \cr &=& \dfrac{a^{10 \cdot \frac{1}{5}}b^{5 \cdot \frac{1}{5}}}{a^{5 \cdot \frac{1}{5}}b^{\frac{5}{2} \cdot \frac{1}{5}}} \cr \cr &=& \dfrac{a^2b}{ab^\frac{1}{2}} \cr \cr &=& a^2ba^{-1}b^{-\frac{1}{2}} \cr &=& a^{2-1}b^{1-\frac{1}{2}} \cr &=& ab^\frac{1}{2} \cr &=& a\sqrt{b} \end{array}

Vorgehen: 3. Potenzgesetz, Wurzeln in Potenzschreibweise, Festlegung: x^{-n}=\dfrac{1}{x^n}, 1. Potenzgesetz

 
31)
Für alle c,d \in \mathbb{R}^+ gilt:
\begin{array}{rcccl} \log(c \cdot d)-\log(d) &=& \log(c) + \log(d) - \log(d) &=& \log(c) \end{array}

Vorgehen: 1. Logarithmengesetz

 
32)
Für alle a\in\mathbb{R}^+\backslash_{\{1\}} gilt:
\begin{array}{ccl} \log_a(16)-\log_a(2) &=& \log_a\left(\dfrac{16}{2}\right) \cr &=& \log_a(8) \end{array}

Vorgehen: 2. Logarithmengesetz

 
33)
Für alle u\in \mathbb{R}^+ gilt:
\begin{array}{ccl} \dfrac{\ln\left(u^5\right)}{5} &=& \dfrac{1}{5}\ln\left(u^5\right) \cr\cr &=& \ln \left(\left(u^5\right)^\frac{1}{5}\right) \cr\cr &=& \ln\left(\sqrt[5]{u^5}\right) \cr\cr &=& \ln(u) \end{array}

Vorgehen: 3. Logarithmengesetz, 3. Potenzgesetz, Potenz in Wurzelschreibweise

 
34)
Für alle b\in\mathbb{R}^+\backslash_{\{1\}} und x,y \in \mathbb{R}^+ gilt:
\begin{array}{ccl} \log_b(3x)+\log_b(x)-\left(2\log_b(x)-\log_b(y)\right) &=& \log_b(3x)+\log_b(x)-2\log_b(x)+\log_b(y) \cr\cr &=& \log_b(3x)+\log_b(x)-\log_b\left(x^2\right)+\log_b(y) \cr\cr &=& \log_b\left(\dfrac{3x \cdot x \cdot y}{x^2}\right) \cr\cr &=& \log_b\left(3y\right) \end{array}

Vorgehen: 1., 2. und 3. Logarithmengesetz
 
Bemerkung: Achten Sie auf das Minuszeichen vor der Klammer!

 
35)
Für alle x\in \mathbb{R} gilt:
\begin{array}{rcl}\ln\left(e^{x^2}\right) &=& x^2 \end{array}

Vorgehen: \ln(x)=\log_e(x) und e^x sind Gegenoperationen. Sie heben sich in ihrer Wirkung also auf.

 
36)
Für alle z\in \mathbb{R}^+ gilt:
\begin{array}{rcccl} 3\log\left(\sqrt[3]{z}\right) &=& \log\left(\left(\sqrt[3]{z}\right)^3\right) &=& \log(z) \end{array}

Vorgehen: 3. Logarithmengesetz, 3. Potenzgesetz

 
37)
Für alle a\in\mathbb{R}^+\backslash_{\{1\}} und x\in \mathbb{R}^+ gilt:
\begin{array}{ccl} \log_a(2x)-\log_a(4)+\log_a\left(\dfrac{2}{x}\right) &=& \log_a\left(\dfrac{2x\cdot 2}{4 \cdot x}\right) \cr &=& \log_a(1) \cr &=& 0 \end{array}

Vorgehen: 1. und 2. Logarithmengesetz

 
38)
Für alle x\in \mathbb{R} gilt:
\begin{array}{ccl} \log_2\left(4^x\right) &=& \log_2\left(2^2\right)^x \cr &=& \log_2\left(2^{2x}\right) \cr &=& 2x \end{array}

Vorgehen: 4=2^2, 3. Potenzgesetz, der Logarithmus zur Basis 2 ist die Gegenoperation zu 2^x 

 
39)
Für alle c,d\in \mathbb{R}^+ mit d gilt:
\begin{array}{rcccl} \log_{12}\left(\dfrac{c-d^2}{(c-d)^3}\right) &=& \log_{12}\left(c-d^2\right)-\log_{12}\left((c-d)^3\right) &=& \log_{12}\left(c-d^2\right)-3\log_{12}\left(c-d\right) \end{array}

Vorgehen: 2. und 3. Logarithmengesetz
 
Bemerkung: Es gibt kein Gesetz für die Vereinfachung vom Logarithmus einer Summe. Deswegen können der Zähler- und den Nennerterm nicht weiter vereinfacht werden.

 
40)
Für alle b,c\in\mathbb{R}^+\backslash_{\{1\}} und y\in \mathbb{R}^+ gilt:
\begin{array}{ccl} \log_b(10y)-\log_b(5y^2)+\log_c\left(\dfrac{y}{100}\right)+\log_b(2^{-1}) &=& \log_b\left(\dfrac{10y}{5y^2 \cdot 2}\right)+\log_c\left(\dfrac{y}{100}\right) \cr\cr &=& \log_b\left(\dfrac{1}{y}\right)+\log_c\left(\dfrac{y}{100}\right) \cr\cr &=& -\log_b(y)+\log_c(y)-\log_c(100) \end{array}

Vorgehen: 1. und 2. Logarithmengesetz

Bemerkung: Achten Sie auf die unterschiedlichen Basen!

 
41)
Für alle f,w \in \mathbb{R} mit f und w nicht gleichzeitig 0 gilt:
\begin{array}{ccl} 10 \lg \left( \sqrt{f^2+w^2} \right) &=& 10 \lg \left( \left( f^2+w^2 \right)^\frac{1}{2} \right) \cr\cr &=& 10 \cdot \dfrac{1}{2} \cdot \lg \left( f^2+w^2 \right) \cr\cr &=& 5 \lg \left( f^2+w^2 \right) \end{array}

Vorgehen: Wurzel in Potenzschreibweise, 3. Logarithmengesetz
 
Bemerkung 1: Es gibt kein Gesetz für die Vereinfachung vom Logarithmus einer Summe. Deswegen kann der Term nicht weiter vereinfacht werden
 
Bemerkung 2: "f und w nicht gleichzeitig 0" bedeutet, dass alle Zahlenkombinationen für f und w eingesetzt werden dürfen, außer f=w=0. Dann würde sich nämlich 10 \lg \left( \sqrt{0^2+0^2} \right)=10 \lg \left(0\right) ergeben und das ist nicht definiert.

 
42)
\begin{array}{rcccl} \sqrt{\text{ld}(512)} &=& \sqrt{9} &=& 3 \end{array}

Vorgehen: 512=2^9, d. h. \text{ld}(512)=\log_2(512)=9

 
43)
\ln\left(-\sqrt{18x^4}\right)

Bemerkung: Dieser Term ist nicht definiert, da das Ergebnis einer Wurzel immer positiv oder 0 ist. Also ist -\sqrt{18x^4} immer negativ oder 0. Der Logarithmus ist aber nur für positive Zahlen definiert.

 
44)
Für alle x \in \mathbb{R}^+ gilt:
\begin{array}{ccl} \log\left(1+x^3\right) \cdot \left[ \log\left(\sqrt{x}\right)+1 \right] &=& \log\left(1+x^3\right) \cdot \left[\log\left(x^{\frac{1}{2}} \right)+1 \right] \cr\cr &=& \log\left(1+x^3\right) \cdot \left[ \dfrac{1}{2} \log\left(x\right)+1 \right] \cr\cr &=& \dfrac{1}{2}\log\left(1+x^3\right) \cdot \log\left(x\right)+ \log\left(1+x^3\right) \end{array}

Vorgehen: 3. Logarithmengesetz, ausmultiplizieren

Bemerkung: Es gibt kein Gesetz für die Vereinfachung eines Produkts aus mehreren Logarithmustermen. Hier können nur die allgemeinen Rechenregeln angewendet werden.

 
45)
Für alle a,x \in \mathbb{R}^+ gilt:
\begin{array}{ccl} \log_5\left(\genfrac{}{}{1pt}{}{\dfrac{375}{2a^6}}{\dfrac{3}{10x^2}}\right) &=& \log_5\left(\dfrac{375}{2a^6} : \dfrac{3}{10x^2}\right) \cr\cr &=& \log_5\left(\dfrac{375}{2a^6} \cdot \dfrac{10x^2}{3}\right) \cr\cr &=& \log_5\left(\dfrac{375 \cdot 10}{2 \cdot 3} \cdot \dfrac{x^2}{a^6}\right) \cr\cr &=& \log_5\left(625 \cdot \dfrac{x^2}{a^6}\right) \cr\cr &=& \log_5\left(625\right) + \log_5\left( \left(\dfrac{x}{a^3}\right)^2 \right) \cr\cr &=& 4+2\log_5\left(\dfrac{x}{a^3}\right) \cr &=& 4+2(\log_5(x)-\log_5\left(a^3\right)) \cr &=& 4+2(\log_5(x)-3\log_5(a)) \cr &=& 4+2\log_5(x)-6\log_5(a) \end{array}

Vorgehen: Doppelbruch auflösen durch Multiplikation mit dem Kehrwert des Nennerbruches, 625=5^4, 1., 2. und 3. Logarithmengesetz

 

7. Aufgabe

Den Katzen sind 7 \cdot 7 \cdot 7 \cdot 7 \cdot 7 = 7^5 = 16.807 Maß Gerste zu verdanken.

Funfact am Rande: Der Titel des Papyrus Rhind lautet "Genaues Rechnen. Einführung in die Kenntnisse aller existierenden Gegenstände und aller dunklen Geheimnisse".

 

8. Aufgabe

1)
Schauen wir uns zunächst ein paar Zahlen konkret an:
2^{-1}=\dfrac{1}{2}=0{,}5
2^{-2}=\dfrac{1}{4}=0{,}25
2^{-3}=\dfrac{1}{8}=0{,}125
2^{-4}=\dfrac{1}{16}=0{,}0625
Es sieht also so aus, als hätte 2^{-n} mit n\in\mathbb{N}^+ jeweils n Stellen nach dem Komma. In der Mathematik ist man aber immer auf der Suche nach einer allgemeingültigen Antwort, sodass ein paar Beispiele nicht ausreichen. Wir müssen also ein bisschen rechnen und argumentieren ...
Dazu schreiben wir 2^{-n} = \left(\dfrac{1}{2}\right)^n = \left(\dfrac{5}{10}\right)^n = 5^n \cdot \dfrac{1}{10^n} und betrachten dann die beiden Faktoren separat:
1. Faktor: 5^n mit n\in\mathbb{N}^+ ist eine natürliche Zahl, z. B. 25, 125, und hat als solche keine Nachkommastellen. Die letzte Ziffer von 5^n ist immer eine 5, egal welche Zahl n ist, da die Zahl 5 ja nur mit sich selbst multipliziert wird.
2. Faktor: \dfrac{1}{10^n} liefert uns für die verschiedenen Werte von n die Zahlenfolge \dfrac{1}{10} = 0{,}1, \dfrac{1}{10^2} = 0{,}01, \dfrac{1}{10^3} = 0{,}001, ...
Nun fassen wir die Erkenntnisse über die beiden Faktoren wieder zusammen: Bei der Multiplikation einer natürlichen Zahl mit 0{,}1, 0{,}01, 0{,}001, ... hat das Produkt so viele Nachkommastellen wie der zweite Faktor. Es sei denn, die natürliche Zahl würde auf 0 enden, da die 0 als letzte Nachkommastelle wegfallen würde. Dieser Fall kann hier aber nicht eintreten, weil wir oben festgestellt haben, dass 5^n immer auf 5 endet. Unsere Vermutung, dass 2^{-n} mit n\in\mathbb{N}^+ jeweils n Nachkommastellen hat, ist also richtig.


2)
Wie Sie sich vermutlich denken können, gibt es hierbei einen Trick ... Wir müssen uns nämlich nur die letzte Stelle der Zahl anschauen.
Überlegen wir uns zunächst, welche Endziffern Quadratzahlen haben können:

  • Die letzte Stelle von 1^2 ist 1.
  • Die letzte Stelle von 2^2 ist 4.
  • Die letzte Stelle von 3^2 ist 9.
  • Die letzte Stelle von 4^2 ist 6.
  • Die letzte Stelle von 5^2 ist 5.
  • Die letzte Stelle von 6^2 ist 6.
  • Die letzte Stelle von 7^2 ist 9.
  • Die letzte Stelle von 8^2 ist 4.
  • Die letzte Stelle von 9^2 ist 1.
  • Die letzte Stelle von 10^2 ist 0.

  • Die letzte Stelle von 11^2 ist 1.
  • Die letzte Stelle von 12^2 ist 4.
  • Die letzte Stelle von 13^2 ist 9.
  • Die letzte Stelle von 14^2 ist 6.
  • Die letzte Stelle von 15^2 ist 5.
  • Die letzte Stelle von 16^2 ist 6.
  • Die letzte Stelle von 17^2 ist 9.
  • Die letzte Stelle von 18^2 ist 4.
  • Die letzte Stelle von 19^2 ist 1.
  • Die letzte Stelle von 20^2 ist 0.

Und so geht es auch weiter ... Eine Quadratzahl endet also immer mit 0, 1, 4, 5, 6 oder 9.
Da "unsere" Zahl 3.141.592.653 an der letzten Stelle eine 3 hat, kann sie also keine Quadratzahl sein.

Wichtig: Umgekehrt funktioniert die Argumentation nicht! Beispielsweise ist die 44 keine Quadratzahl, obwohl sie eine 4 an letzter Stelle hat.


3)
Zunächst stellen wir fest, dass in jeder Klammer zwei Quadratzahlen subtrahiert werden. Daher formen wir jede Klammer mithilfe der dritten binomischen Formel in zwei Klammer um. Dadurch wird der Term zwar länger, aber letztendlich einfacher:
\begin{array}{rcl}\left(1-\dfrac{1}{4}\right)\left(1-\dfrac{1}{9}\right)\left(1-\dfrac{1}{16}\right)\left(1-\dfrac{1}{25}\right)\dots\left(1-\dfrac{1}{100}\right) &=& \left(1-\dfrac{1}{2}\right)\left(1+\dfrac{1}{2}\right)\left(1-\dfrac{1}{3}\right)\left(1+\dfrac{1}{3}\right)\left(1-\dfrac{1}{4}\right)\left(1+\dfrac{1}{4}\right)\left(1-\dfrac{1}{5}\right)\left(1+\dfrac{1}{5}\right)\dots\left(1-\dfrac{1}{10}\right)\left(1+\dfrac{1}{10}\right) \cr\cr&=& \dfrac{1}{2}\cdot\;\dfrac{3}{2}\cdot\dfrac{2}{3}\;\cdot\;\dfrac{4}{3}\cdot\dfrac{3}{4}\;\cdot\;\dfrac{5}{4}\cdot\dfrac{4}{5}\;\cdot\;\dfrac{6}{5}\cdot\;\dots\;\cdot\dfrac{9}{10}\cdot\;\dfrac{11}{10} \cr\cr&=& \dfrac{1}{2}\cdot 1\cdot 1\cdot 1\cdot\;\dots\;\cdot\dfrac{11}{10} \cr\cr&=& \dfrac{11}{20}\end{array}
In der zweiten Reihe der Rechnung haben sich lauter Paare von Brüchen gebildet, deren Produkt 1 ist, z. B. der zweite und der dritte Bruch. Es bleiben nur der erste und der letzte Bruch "übrig", die wir dann noch multiplizieren müssen.

9. Quadratische Gleichungen - Lernziele und typische Fehler

Nach Durcharbeiten dieses Kapitels sollten Sie folgende Lernziele erreicht haben:

  • Sie können zu einer quadratischen Gleichung den passenden Definitionsbereich bestimmen.
  • Sie kennen die allgemeine Form einer quadratischen Gleichung.
  • Sie wissen, dass es quadratische Gleichungen mit keiner/einer/zwei/unendlich vielen Lösung(en) gibt und können anhand des Rechenweges feststellen, welcher Fall vorliegt.
  • Sie können zu einer vorliegenden quadratischen Gleichung die geschickteste Lösungsmethode auswählen und die Gleichungen damit lösen.
  • Sie können die Lösungsmenge mathematisch korrekt notieren.
  • Sie können mithilfe der Probe überprüfen, ob die gefundene Lösung tatsächlich richtig ist.
  • Sie können entsprechende Textaufgaben lösen und die Lösungswege mathematisch vernünftig aufschreiben.
  • Sie können quadratische Gleichungen von anderen Gleichungsarten unterscheiden.


Typische Fehler in diesem Kapitel sind:

  • Es wird nicht der geschickteste Lösungsweg verwendet. Das ist zwar kein Fehler, aber eben ungeschickt ... Erklärung
  • Es wird die p-q-Formel angewendet, obwohl die Gleichung nicht in Normalform vorliegt. Erklärung


Für Online-Selbsttests zu diesem Thema und weitere Informationen zur Mathematikunterstützung an der TH Wildau nutzen Sie bitte den Moodle-Kursraum "SOS Mathematik - Brückenkurs".

Übersicht:

 

9.1 Quadratische Gleichungen - Aufgaben

1. Aufgabe

Lösen Sie folgende Gleichungen, wobei \mathbb{D}=\mathbb{R} ist (siehe Zahlenbereiche)!

1) x^2-49 = 0

  11) x^2+x = 6\sqrt{10}x

2) 5 x^2 - 80 = 0

  12) 357-2x^2 = -35

3) 2x^2+8 = 0

  13) 24-x^2 = 2x

4) -3x^2+363 = 0

  14) -2x^2+6x-5 = 3

5) -2x^2 = -20x

  15) 5x^2-38x+225 = 4x^2-8x

6) x^2+9x = 0

  16) 9x^2+18x = 81

7) x^2+x-12 = 0

  17) 0 = -301z+128z^2-95z^2+217z-33z^2+84z

8) x^2+2x+2 = 0

  18) 809-9x^2 = -10x^2-215

9) 8x^2-3 = -2x

  19) 16y^2+61y+169+97y = -201-24y^2+5-122y-125

10) \dfrac{1}{2}x^2+12x = -72

  20) -\dfrac{5}{4}x^2-\dfrac{1}{9}+\dfrac{7}{18}+\dfrac{x^2}{2} = \dfrac{5}{18}-\dfrac{9}{8}x^2+\dfrac{3x^2}{8}

 

2. Aufgabe

Formen Sie die folgenden Gleichungen in die Normalform oder die allgemeine Form quadratischer Gleichungen um und lösen Sie sie, wobei \mathbb{D}=\mathbb{R} ist (siehe Zahlenbereiche)!

1) 50\left(1-\dfrac{1}{5}x\right) = (x-5)^2

  11) (-7x+7)(7x-7)-31 = 21x^2+60

2) \dfrac{2x^2-3}{3} = \dfrac{1}{5} (x^2-8)+\dfrac{1}{3}

  12) 5(-3x^2+11x-115)-2x^2 = 101-((4x-1)^2+5x-1)

3) -\dfrac{x}{16}(-32x+64)-x = 2x^2-5x

  13) (x+33)^2 = -8(-47+x)^2

4) 5x^2-\left(\sqrt{175}-15\right)x = \dfrac{15}{2}\sqrt{7}

  14) (6+8y)^2 = 16(4y+3)\left(y+\dfrac{3}{4}\right)

5) 1+x^2 = 3x+36x\left(x-\dfrac{1}{8}\right)+\dfrac{3}{2}\left(x+\dfrac{2}{3}\right)-35x^2

  15) 12\left(\left(\dfrac{p}{2}-\dfrac{1}{2}\right)^2-\dfrac{5}{4}p^2\right)+12p^2 = 9

6) -12t(t-5)-47t = 13t

  16) -76(13+x)+87x^2 = 43x(-1+2x)

7) (x+2)^2 = 4(1-x)

  17) x\left(\dfrac{31}{20}x-\dfrac{7}{40}\right)-x^2 = 0

8) (x+9)(x-9) = \dfrac{1}{2}(x+18)+\dfrac{11x}{2}

  18) -\dfrac{12}{27}x^2 = \dfrac{1}{9}\left(\dfrac{1}{3}\left(15x^2-\dfrac{2}{3}\right)-\dfrac{7}{2}x\right)

9) (8+3x)^2 = -2(19-12x-5x^2)-10

  19) \dfrac{1}{2}(14x+x)x = -\dfrac{-21x^2+x}{6}+\dfrac{x^2+30}{3}+812{,}5

10) 4x^2-69 = -5+2x(8+2x)

  20) 80+(11x-6)(11x+6)+4x^2 = 6\left(\dfrac{100}{3}x-6\right)

 

3. Aufgabe

Verlängert man bei einem Quadrat zwei gegenüberliegende Seiten um je 8\,cm und verkürzt die beiden anderen Seiten um je 6\,cm, so entsteht ein Rechteck, dessen Flächeninhalt um 4\,cm^2 kleiner ist als der Flächeninhalt des Quadrats. Wie groß sind die Flächeninhalte von Quadrat und Rechteck?

Dieses Kapitel enthält die folgenden Themen:

 

9.2 Quadratische Gleichungen - Erklärungen

Nachdem wir uns schon mit linearen Gleichungen beschäftigt haben und im letzten Kapitel geklärt haben, was quadratische Terme sind, kommen wir nun zum nächsten Gleichungstyp, der diese beiden Konzepte kombiniert. Es wird damit etwas komplexer als bei linearen Gleichungen - aber auch etwas spannender ...

 

Allgemeines

Zunächst (natürlich) die Frage, wie eine quadratische Gleichung aussieht: Eine quadratische Gleichung ist eine Gleichung in folgender Form:
ax^2+bx+c =0 mit a,b,c \in \mathbb{R} und a\neq 0
Zum x-Term in der Gleichung, den wir von den linearen Gleichungen schon kennen, kommt also ein x^2-Term hinzu. Beispiele für quadratische Gleichungen sind also: -5x^2+9x-10=0 oder 14t^2+19=0 oder -z^2-37z=0 oder ...
Die Koeffizienten a, b und c aus der allgemeinen Formel sind dabei irgendwelche reellen Zahlen - mit einer Ausnahme: a darf nicht 0 sein. Warum ist das so? Schauen wir uns an, was passiert, wenn a doch 0 wäre: Dann würde die Gleichung zu 0\cdot x^2+bx+c =0, also bx+c = 0 werden. Damit wäre sie linear und wir bräuchten kein neues Kapitel ...

 

Verschiedene Lösungswege

Wie bei linearen Gleichungen sollen auch beim Lösen quadratischer Gleichungen für die Variable alle Werte gefunden werden, die beim Einsetzen beide Seiten gleich groß werden lassen. Viele Überlegungen, die fürs Lösen quadratischer Gleichungen nötig sind, haben wir daher schon erledigt. Netterweise gibt es für den Rest einige unkomplizierte Lösungsformeln ...

Eine Vorbemerkung: Im Folgenden werden verschiedene Möglichkeiten, eine quadratische Gleichung zu lösen, vorgestellt. Grundsätzlich führen alle Wege zum richtigen Ergebnis. Je nach Form, in der die quadratische Gleichung vorliegt, kann aber der eine oder der andere Weg geschickter sein, weil er kürzer und damit weniger fehleranfällig ist. Es lohnt sich also durchaus, die Gleichung erstmal genau anzuschauen, um ihren Aufbau zu erkennen, und sich danach für einen Lösungsweg zu entscheiden. Das hat den zusätzlichen Vorteil, dass Sie üben, die Struktur einer Gleichung zu erfassen, was Ihnen später, wenn andere Gleichungstypen dazugekommen sind, sehr nützlich sein wird. In Anwendungsaufgaben wird nämlich niemand sagen: "Das ist eine quadratische Gleichung. Bitte lösen Sie diese!" Sie müssen dort selbst erkennen, welche Art Gleichung vorliegt und wie diese am besten gelöst werden kann.

 

Die p-q-Formel

Ein Klassiker in diesem Zusammenhang ist die p-q-Formel: Liegt eine quadratische Gleichung in Normalform vor, also in der Form x^2+px+q=0 mit p, q \in \mathbb{R}, lässt sie sich am einfachsten mithilfe der p-q-Formel lösen. "Normalform" bedeutet, dass der Koeffizient von dem x^2-Term 1 ist und dass auf der "rechten Seite" der Gleichung eine 0 steht.

Die p-q-Formel lautet: x_{1,2}=-\dfrac{p}{2} \pm \sqrt{\left(\dfrac{p}{2}\right)^2-q}

Eigentlich sind dies zwei Formeln in einer: x_1=-\dfrac{p}{2} + \sqrt{\left(\dfrac{p}{2}\right)^2-q} und x_2=-\dfrac{p}{2} - \sqrt{\left(\dfrac{p}{2}\right)^2-q}. Da sich die beiden Formeln aber nur im Rechenzeichen vor der Wurzel unterscheiden, schreibt man sie üblicherweise in einem mit dem Zeichen "plusminus". Es würde sich auch nicht lohnen, alles doppelt aufzuschreiben: Wenn man eine quadratische Gleichung mithilfe der p-q-Formel löst, sind die Rechnungen nämlich - bis auf den letzten Schritt - komplett gleich. Sie sehen das im folgenden Beispiel: Bis zur Leerzeile ist jeder Rechenschritt mit einem \pm geschrieben. Erst danach gibt es eine Zeile mit + und eine Zeile mit -. Bitte achten Sie darauf, dass dann auf der linken Seite der Gleichung nicht mehr x_{1,2} steht, sondern x_1 und x_2. Jede Lösung hat sozusagen ihren eigenen "Namen".


Ein Beispiel:

\begin{array}{rclcc} x^2-3x-4 &=& 0 \cr x_{1,2} &=& -\dfrac{-3}{2} \pm \sqrt{\left(\dfrac{-3}{2} \right)^2-\left(-4\right)} \cr\cr x_{1,2} &=& \dfrac{3}{2} \pm \sqrt{\dfrac{9}{4}+4} \cr\cr x_{1,2} &=& \dfrac{3}{2} \pm \sqrt{\dfrac{9+16}{4}} \cr\cr x_{1,2} &=& \dfrac{3}{2} \pm \sqrt{\dfrac{25}{4}} \cr\cr x_{1,2} &=& \dfrac{3}{2} \pm \dfrac{5}{2} \cr\cr\cr x_1 &=& \dfrac{3}{2}+\dfrac{5}{2} = \dfrac{8}{2} = 4 \cr\cr x_2 &=& \dfrac{3}{2} -\dfrac{5}{2} = -\dfrac{2}{2} = -1 \cr \cr \mathbb{L} &=& \{-1;4\} \end{array}

Bemerkung: Achten Sie darauf, dass p und q Vorzeichen haben! In diesem Beispiel ist p=-3 und q=-4.


Jetzt müssen wir noch kurz überlegen, wie man vorgeht, wenn die quadratische Gleichung nicht in Normalform vorliegt:
Nehmen wir als Beispiel: 5x^2-15x=20. Hier sind beide "Bedingungen" für die Normalform verletzt. Wir müssen die Gleichung also entsprechend umformen:
\begin{array}{rclcc}5x^2-15x &=& 20 &\vert & -20 \cr 5x^2-15x-20 &=& 0 &\vert & :5 \cr x^2-3x-4 &=& 0 \cr ...\end{array}
Den Rest der Rechnung kennen wir schon.
Über diese Äquivalenzumformungen, die Sie schon von den linearen Gleichungen kennen, kann man bei jeder quadratischen Gleichung die Normalform herstellen. Alternativ belässt man die quadratische Gleichung in der Form 5x^2-15x-20 = 0 und wendet die a-b-c-Formel an.

 

Die a-b-c-Formel

Eine quadratische Gleichung in allgemeiner Form, also in der Form ax^2+bx+c = 0 mit a, b, c \in \mathbb{R} und a \neq 0, lässt sich mithilfe der a-b-c-Formel lösen.

Die a-b-c-Formel lautet: x_{1,2}=\dfrac{-b \pm \sqrt{b^2-4ac}}{2a}


Auch hier ein Beispiel:
\begin{array}{rclcc} 5x^2-15x-20 &=& 0 \cr x_{1,2} &=& \dfrac{-(-15) \pm \sqrt{\left(-15\right)^2-4\cdot 5\cdot \left(-20\right)}}{2\cdot 5} \cr\cr x_{1,2} &=& \dfrac{15 \pm \sqrt{225+400}}{10} \cr\cr x_{1,2} &=& \dfrac{15 \pm \sqrt{625}}{10} \cr\cr x_{1,2} &=& \dfrac{15\pm 25}{10} \cr\cr\cr x_1 &=& \dfrac{15+25}{10} = \dfrac{40}{10} = 4 \cr\cr x_2 &=& \dfrac{15-25}{10} = -\dfrac{10}{10} = -1 \cr\cr \mathbb{L} &=& \{-1;4\} \end{array}

Bemerkung: Auch hier die Vorzeichen von a, b und c nicht vergessen!


Wenn Sie lieber nicht mit der a-b-c-Formel rechnen möchten, können Sie die Gleichung in Normalform bringen, sodass Sie mit der p-q-Formel rechnen können. Das ist auch gar nicht kompliziert: Man muss die Gleichung nur durch a teilen. In unserem Beispiel wäre das x^2-\dfrac{15}{5}x-\dfrac{20}{5} = 0. Diese Gleichung kennen wir schon aus dem Beispiel von oben. Umgekehrt kann man natürlich auch die a-b-c-Formel anwenden, wenn die quadratische Gleichung im Normalform vorliegt: Dann ist a=1.
Letztendlich ist es also egal, für welchen der beiden Wege Sie sich entscheiden.

 

Der Satz vom Nullprodukt

Liegt eine quadratische Gleichung faktorisiert vor, d. h. in der Form (ax+b)(cx+d)=0, oder lässt sich leicht entsprechend umformen, hilft der

Satz vom Nullprodukt: Ein Produkt reeller Zahlen ist genau dann 0, wenn einer der Faktoren 0 ist.

Anders formuliert: Multipliziert man eine reelle Zahl mit 0, ist das Ergebnis immer 0. "Einer der Faktoren" meint dabei, dass auch beide Faktoren 0 sein dürfen. Nur \infty dürfen die Faktoren nicht sein - dann hätten wir ja aber auch den Bereich der reellen Zahlen verlassen.


Für unsere quadratische Gleichung bedeutet dies: Man kann die Terme in den Klammern separat betrachten. Denn, wenn wir wissen, für welchen x-Wert eine der Klammern 0 ergibt, ist es völlig egal, welchen Wert der andere Klammerterm annimmt - das Produkt der beiden ist immer 0. Es sind also letztendlich nur zwei lineare Gleichungen zu lösen.


Ein Beispiel:
\begin{array}{rrclll} & (x+1)(x-4) &=& 0 \cr\cr \text{Faktor 1:} & x_1+1 &=& 0 &\vert& -1 \cr & x_1 &=& -1 \cr\cr\text{Faktor 2:} & x_2-4 &=& 0 &\vert& +4 \cr& x_2 &=& 4 \cr\cr & \mathbb{L} &=& \{-1;4\}\end{array}


Auf den ersten Blick mag es wenig sinnvoll erscheinen, aus einer Gleichung zwei zu machen - die zwei entstehenden Gleichungen sind aber deutlich einfacher als die Ausgangsgleichung. Man kann die Lösungen quasi ablesen ... Was wäre die Alternative? Man könnte die beiden Klammern in der Gleichung ausmultiplizieren und die entstehende Gleichung soweit vereinfachen, dass man dann mit der p-q- oder der a-b-c-Formel weiterkommt. Für unser Beispiel bedeutet das:
\begin{array}{rcl} (x+1)(x-4) &=& 0 \cr x^2 - 4x + 1x - 4 &=& 0 \cr x^2 - 3x - 4 &=& 0 \cr ... \end{array}
Diese Gleichung kennen wir schon - es ist das oberste Beispiel in diesem Kapitel. Vergleichen Sie selbst, welcher Weg schneller und einfacher zum Ergebnis führt ...


Dieser Weg über den Satz vom Nullprodukt funktioniert auch wunderbar für Gleichungen der Form (ax+b)^2 = (ax+b)(ax+b) = 0 oder x^2+px = x(x+p) = 0.
Ganz wichtig: Den Satz vom Nullprodukt kann man natürlich nur anwenden, wenn das Ergebnis des Produkts wirklich 0 sein soll! Bei allen anderen Zahlen müssen wir z. B. zur p-q- oder a-b-c-Formel greifen.

 

Umformen

Quadratische Gleichungen in der Form x^2+q = 0 oder ax^2+c=0, also quadratische Gleichungen ohne x-Term, kann man so umformen, dass auf einer Seite der Gleichung nur noch x^2 steht (mathematisch gesprochen: "man isoliert x^2"). Anschließend kommt man durch Wurzelziehen zur Lösung.
Dabei aufpassen: Auch wenn Quadratwurzeln aus reellen Zahlen immer nichtnegativ sind, entstehen beim Wurzelziehen aus einer positiven Zahl zwei Lösungen. Daher steht vor der Wurzel das inzwischen schon bekannte Zeichen \pm.


Und noch ein letztes Beispiel:
\begin{array}{rclcl} 2x^2 &=& 98 &\vert & :2 \cr x^2 &=& 49 &\vert &\pm\sqrt{} \cr x_{1,2} &=& \pm\sqrt{49} \cr\cr x_1 &=& 7 \cr x_2 &=& -7 \cr\cr \mathbb{L} &=& \{-7;7\} \end{array}

 

So, nun haben wir alle üblichen Lösungswege für quadratische Gleichungen angeschaut. Wie in der Einleitung schon gesagt, ist es wichtig, sich damit gut vertraut zu machen und so viele quadratische Gleichungen zu lösen, bis man das im Schlaf kann. In den Kapiteln 13, 14, 15, 16, 17 und 18 werden Sie weitere Gleichungsarten kennenlernen, deren Lösungsalgorithmen eigentlich alle in der einen oder anderen Weise auf den Lösungsalgorithmen für lineare und quadratische Gleichungen aufbauen.

 

Lösbarkeit quadratischer Gleichungen

Zum Abschluss dieses Kapitels kommt man - analog zu linearen Gleichungen - auf die Frage: Ist jede quadratische Gleichung im Bereich der reellen Zahlen lösbar?

Für die Antwort schauen wir uns beispielhaft drei Fälle an:

1. Fall:
\begin{array}{rclcl} x^2-4 &=& 0 &\vert& +4 \cr x^2 &=& 4 &\vert& \pm\sqrt{} \cr x &=& \pm 2 \cr\cr \mathbb{L} &=& \{-2;2\} \end{array}
Diese quadratische Gleichung ist lösbar und hat zwei Lösungen.

2. Fall:
\begin{array}{rclcl} (x-1)^2 &=& 0 &\vert& \pm\sqrt{} \cr x-1 &=& 0 &\vert& +1 \cr x &=& 1 \cr\cr \mathbb{L} &=& \{1\} \end{array}
Diese quadratische Gleichung ist lösbar und hat eine Lösung.

3. Fall:
\begin{array}{rclcl} x^2+10 &=& 0 & \vert &-10 \cr x^2 &=& -10 & \vert & \pm\sqrt{} \cr x &=& \pm \sqrt{-10}\cr\cr \mathbb{L} &=& \emptyset \end{array}
Die quadratische Gleichung ist nicht lösbar. D. h., es gibt keine Zahl, die diese Gleichung löst, da aus negativen Zahlen in \mathbb{R} keine Quadratwurzeln gezogen werden können.

Zusammenfassung: Quadratische Gleichung können im Bereich der reellen Zahlen keine, eine oder zwei Lösungen haben. Im nächsten Kapitel, wo es um quadratische Funktionen und ihre Graphen geht, wird dies noch etwas plastischer werden.

Übersicht:

 

9.3 Quadratische Gleichungen - Lösungen

1. Aufgabe

1)
Hier ist die Anwendung der p-q- oder a-b-c-Formel nicht nötig. Umformen reicht aus.
\begin{array}{rclcl} x^2-49 &=& 0 & \vert & +49 \cr x^2 &=& 49 & \vert & \pm \sqrt{} \cr x &=& \pm \sqrt{49} \cr\cr x_1 &=& 7 \cr x_2 &=& -7 \end{array}

Probe: Auch hier kann selbstverständlich eine Probe durchgeführt werden.
Für x_1:
\begin{array}{rcl} 7^2-49 &=& 0 \cr 49-49 &=& 0 \end{array}

Für x_2:
\begin{array}{rcl} (-7)^2-49 &=& 0 \cr 49-49 &=& 0 \end{array}

Für x_1 = 7 sowie x_2 = -7 ergeben sich wahre Aussagen: \mathbb{L} = \left\{-7; 7 \right\}

 
2)
Hier ist die Anwendung der p-q- oder a-b-c-Formel nicht nötig. Umformen reicht aus.
\begin{array}{rclcl} 5 x^2 - 80 &=& 0 & \vert & +80 \cr 5x^2 &=& 80 & \vert & :5 \cr x^2 &=& 16 & \vert & \pm \sqrt{} \cr x &=& \pm \sqrt{16} \cr\cr x_1 &=& 4 \cr x_2 &=& -4 \cr \cr \mathbb{L} &=& \{-4; 4\} \end{array}

Probe: ...

 
3)
Hier ist die Anwendung der p-q- oder a-b-c-Formel nicht nötig. Umformen reicht aus.
\begin{array}{rclcl} 2x^2+8 &=& 0 & \vert & -8 \cr 2x^2 &=& -8 & \vert & :2 \cr x^2 &=& -4 &\vert& \pm\sqrt{} \cr x &=& \pm\sqrt{-4} \end{array}

Da aus negativen reellen Zahlen keine Wurzeln mit geraden Wurzelexponenten gezogen werden können, hat diese Gleichung keine Lösung: \mathbb{L} = \emptyset

 
4)
Hier ist die Anwendung der p-q- oder a-b-c-Formel nicht nötig. Umformen reicht aus.
\begin{array}{rclcl} -3x^2+363 &=& 0 & \vert & -363 \cr -3x^2 &=& -363 & \vert & :(-3) \cr x^2 &=& 121 & \vert & \pm \sqrt{} \cr\cr x_1 &=& 11 \cr x_2 &=& -11 \cr \cr \mathbb{L} &=& \{-11;11\} \end{array}

Probe: ...


5)
Hier ist die Anwendung der p-q- oder a-b-c-Formel nicht nötig. Ausklammern und der Satz vom Nullprodukt reichen aus.
\begin{array}{crclcl} & -2x^2 &=& -20x & \vert & +20x \cr & -2x^2+20x &=& 0 \cr & x(-2x+20) &=& 0 \cr \text{Faktor 1:} & x_1 &=& 0 \cr\cr \text{Faktor 2:} & -2x_2+20 &=& 0 \cr & x_2 &=& 10 \cr \cr & \mathbb{L} &=& \{0;10\} \end{array}

Probe: ...

 
6)
Hier ist die Anwendung der p-q- oder a-b-c-Formel nicht nötig. Ausklammern und der Satz vom Nullprodukt reichen aus.
\begin{array}{crclcl} & x^2+9x &=& 0 \cr & x(x+9) &=& 0 \cr \text{Faktor 1:} & x_1 &=& 0 \cr\cr \text{Faktor 2:} & x_2+9 &=& 0 \cr & x_2 &=& -9 \cr\cr & \mathbb{L} &=& \{-9;0\} \end{array}

Probe: ...

 
7)
Hier wird die p-q-Formel angewendet.
\begin{array}{rclcl} x^2+x-12 &=& 0 \cr x_{1,2} &=& -\dfrac{1}{2} \pm \sqrt{\left(\dfrac{1}{2} \right)^2-\left(-12\right)} \cr\cr x_{1,2} &=& -\dfrac{1}{2} \pm \sqrt{\dfrac{1}{4}+12} \cr\cr x_{1,2} &=& -\dfrac{1}{2} \pm \sqrt{\dfrac{1+48}{4}} \cr\cr x_{1,2} &=& -\dfrac{1}{2} \pm \sqrt{\dfrac{49}{4}} \cr\cr x_{1,2} &=& -\dfrac{1}{2} \pm \dfrac{7}{2} \cr\cr\cr x_1 &=& -\dfrac{1}{2}+\dfrac{7}{2} = 3 \cr\cr x_2 &=& -\dfrac{1}{2} -\dfrac{7}{2} = -4 \cr \cr \mathbb{L} &=& \{-4;3\} \end{array}

Probe: ...

 
8)
Hier wird die p-q-Formel angewendet.
\begin{array}{rclcl} x^2+2x+2 &=& 0 \cr x_{1,2} &=& -\dfrac{2}{2} \pm \sqrt{\left(\dfrac{2}{2}\right)^2-2} \cr x_{1,2} &=& -1 \pm \sqrt{1-2} \cr x_{1,2} &=& -1 \pm \sqrt{-1} \end{array}

Da aus negativen reellen Zahlen keine Wurzeln mit geraden Wurzelexponenten gezogen werden können, hat diese Gleichung keine Lösung: \mathbb{L} = \emptyset

 
9)
Hier wird die a-b-c-Formel angewendet.
\begin{array}{rclcc} 8x^2-3 &=& -2x & \vert & +2x \cr 8x^2+2x-3 &=& 0 \cr x_{1,2} &=& \dfrac{-2 \pm \sqrt{2^2-4 \cdot 8 \cdot (-3)}}{2 \cdot 8} \cr \cr x_{1,2} &=& \dfrac{-2 \pm \sqrt{4- (-96)}}{16} \cr \cr x_{1,2} &=& \dfrac{-2 \pm \sqrt{100}}{16} \cr \cr x_{1,2} &=& \dfrac{-2 \pm 10}{16} \cr \cr \cr x_1 &=& \dfrac{8}{16} = \dfrac{1}{2} \cr \cr x_2 &=& \dfrac{-12}{16} = -\dfrac{3}{4} \cr \cr \mathbb{L} &=& \left\{-\dfrac{3}{4}; \dfrac{1}{2}\right\} \end{array}

Probe: ...


10)
Hier wird die p-q-Formel angewendet.
\begin{array}{rclcl} \dfrac{1}{2}x^2+12x &=& -72 & \vert & \cdot 2 \cr x^2+24x &=& -144 & \vert & +144 \cr x^2+24x+144 &=& 0 \cr x_{1,2} &=& -\dfrac{24}{2} \pm \sqrt{\left(\dfrac{24}{2}\right)^2-144} \cr x_{1,2} &=& -12 \pm \sqrt{12^2-144} \cr x_{1,2} &=& -12 \pm \sqrt{0} \cr x_{1,2} &=& -12 \cr \cr \mathbb{L} &=& \{-12\}\end{array}

Probe: ...


11)
Hier ist die Anwendung der p-q- oder a-b-c-Formel nicht nötig. Ausklammern und der Satz vom Nullprodukt reichen aus.
\begin{array}{crclcl} & x^2+x &=& 6\sqrt{10}x &\vert& -6\sqrt{10}x \cr & x^2-6\sqrt{10}x+x &=& 0 \cr & x(x-6\sqrt{10}+1) &=& 0 \cr \text{Faktor 1:} & x_1 &=& 0 \cr\cr \text{Faktor 2:} & x_2-6\sqrt{10}+1 &=& 0 \cr & x_2 &=& 6\sqrt{10}-1 \cr\cr & \mathbb{L} &=& \left\{0; 6\sqrt{10}-1\right\} \end{array}

Bemerkung: Beim Ausklammern die 1 in der Klammer nicht vergessen!

Probe: ...


12)
Hier ist die Anwendung der p-q- oder a-b-c-Formel nicht nötig. Umformen reicht aus.
\begin{array}{rclcl} 357-2x^2 &=& -35 & \vert & -357 \cr -2x^2 &=& -392 & \vert & :(-2) \cr x^2 &=& 196 & \vert & \pm \sqrt{} \cr x &=& \pm \sqrt{196} \cr\cr x_1 &=& 14 \cr x_2 &=& -14 \cr \cr \mathbb{L} &=& \{-14; 14\} \end{array}

Probe: ...


13)
Hier wird die p-q-Formel angewendet.
\begin{array}{rclcl} 24-x^2 &=& 2x & \vert & +x^2-24 \cr 0 &=& x^2+2x-24 \cr x_{1,2} &=& -1\pm\sqrt{1+24} \cr\cr x_1 &=& 4 \cr x_2 &=& -6 \cr \cr \mathbb{L} &=& \{-6;4\} \end{array}

Probe: ...


14)
Hier wird die p-q-Formel angewendet.
\begin{array}{rclcl} -2x^2+6x-5 &=& 3 & \vert & +2x^2-6x+5 \cr 0 &=& 2x^2-6x+8 & \vert & :2 \cr 0 &=& x^2-3x+4 \cr x_{1,2} &=& \dfrac{3}{2}\pm\sqrt{\left(\dfrac{3}{2}\right)^2-4} \cr x_{1,2} &=& \dfrac{3}{2}\pm\sqrt{-\dfrac{7}{4}} \end{array}

Da aus negativen reellen Zahlen keine Wurzeln mit geraden Wurzelexponenten gezogen werden können, hat diese Gleichung keine Lösung: \mathbb{L} = \emptyset


15)
Hier wird die p-q-Formel angewendet.
\begin{array}{rclcl} 5x^2-38x+225 &=& 4x^2-8x & \vert & -4x^2+8x \cr x^2-30x+225 &=& 0 \cr x_{1,2} &=& -\dfrac{-30}{2}\pm\sqrt{\left(\dfrac{-30}{2}\right)^2-225} \cr x_{1,2} &=& 15\pm\sqrt{0} \cr x_{1,2} &=& 15 \cr \cr \mathbb{L} &=& \{15\} \end{array}

Probe: ...


16)
Hier wird die p-q-Formel angewendet.
\begin{array}{rclcl} 9x^2+18x &=& 81 & \vert & :9 \cr x^2+2x &=& 9 & \vert & -9 \cr x^2+2x-9 &=& 0 \cr x_{1,2} &=& -1\pm\sqrt{1^2+9} \cr\cr x_1 &=& -1+\sqrt{10} \cr x_2 &=& -1-\sqrt{10} \cr \cr \mathbb{L} &=& \{-1+\sqrt{10};-1-\sqrt{10}\} \end{array}

Probe: ...


17)
Hier ist die Anwendung der p-q- oder a-b-c-Formel nicht nötig. Umformen reicht aus.
\begin{array}{rclcl} 0 &=& -301z+128z^2-95z^2+217z-33z^2+84z \cr 0 &=& 128z^2-95z^2-33z^2-301z+217z+84z \cr 0 &=& z^2(128-95-33)+z(-301+217+84) \cr 0 &=& 0z^2+0z \cr 0 &=& 0 \cr \cr \mathbb{L} &=& \mathbb{R} \end{array}

Bemerkung: Unabhängig davon, welches Element des Definitionsbereichs in diese Gleichung eingesetzt wird, erhält man immer auf beiden Seiten dasselbe Ergebnis. 0=0 ist schließlich immer richtig. Jede reelle Zahl löst also diese Gleichung, d. h. die Lösungsmenge entspricht dem Definitionsbereich.


18)
Hier ist die Anwendung der p-q- oder a-b-c-Formel nicht nötig. Umformen reicht aus.
\begin{array}{rclcl} 809-9x^2 &=& -10x^2-215 & \vert & +10x^2-809 \cr x^2 &=& -1.024 &\vert& \pm\sqrt{} \cr x &=& \pm\sqrt{-1.024} \end{array}

Da aus negativen reellen Zahlen keine Wurzeln mit geraden Wurzelexponenten gezogen werden können, hat diese Gleichung keine Lösung: \mathbb{L} = \emptyset

19)
Hier wird die p-q-Formel angewendet.
\begin{array}{rclcl} 16y^2+61y+169+97y &=& -201-24y^2+5-122y-125 \cr 16y^2+158y+169 &=& -24y^2-122y-321 & \vert & +24y^2+122y+321 \cr 40y^2+280y+490 &=& 0 &\vert& : 40 \cr y^2+7y+\dfrac{49}{4} &=& 0\cr y_{1,2} &=& -\dfrac{7}{2}\pm\sqrt{\left(\dfrac{7}{2}\right)^2-\dfrac{49}{4}} \cr y_{1,2} &=& -\dfrac{7}{2}\pm\sqrt{0} \cr y_{1,2} &=& -\dfrac{7}{2} \cr \cr \mathbb{L} &=& \left\{-\dfrac{7}{2}\right\} \end{array}

Probe: ...


20)
Hier ist die Anwendung der p-q- oder a-b-c-Formel nicht nötig. Umformen reicht aus.
\begin{array}{rclcl} -\dfrac{5}{4}x^2-\dfrac{1}{9}+\dfrac{7}{18}+\dfrac{x^2}{2} &=& \dfrac{5}{18}-\dfrac{9}{8}x^2+\dfrac{3x^2}{8} \cr -\dfrac{3}{4}x^2\dfrac{5}{18} &=& -\dfrac{6}{8}x^2+\dfrac{5}{18} & \vert & -\dfrac{5}{18} \cr -\dfrac{3}{4}x^2 &=& -\dfrac{6}{8}x^2 & \vert & \cdot 4 \cr -3x^2 &=& -3x^2 & \vert & +3x^2 \cr 0 &=& 0 \cr \cr \mathbb{L} &=& \mathbb{R} \end{array}

Bemerkung: Unabhängig davon, welches Element des Definitionsbereichs in diese Gleichung eingesetzt wird, erhält man immer auf beiden Seiten dasselbe Ergebnis. 0=0 ist schließlich immer richtig. Jede reelle Zahl löst also diese Gleichung, d. h. die Lösungsmenge entspricht dem Definitionsbereich.

 

2. Aufgabe

Bei dieser Aufgabe wurden die Lösungswege nicht mehr ganz so ausführlich aufgeschrieben wie oben. Sie kennen sich ja inzwischen aus ... Wenn Sie mit einem anderen Lösungsweg zum gleichen Ergebnis kommen, ist das natürlich auch völlig in Ordnung.

1)
Hier ist die Anwendung der p-q- oder a-b-c-Formel nicht nötig. Umformen reicht aus.
\begin{array}{rclcl} 50\left(1-\dfrac{1}{5}x\right) &=& (x-5)^2 \cr 50-10x &=& x^2-10x+25 &\vert& +10x-25 \cr 25 &=& x^2 &\vert& \pm \sqrt{} \cr x &=& \pm \sqrt{25} \cr\cr x_1 &=& 5 \cr x_2 &=& -5 \cr \cr \mathbb{L} &=& \{-5; 5\} \end{array}

 
2)
Hier ist die Anwendung der p-q- oder a-b-c-Formel nicht nötig. Umformen reicht aus.
\begin{array}{rclcl} \dfrac{2x^2-3}{3} &=& \dfrac{1}{5} (x^2-8)+\dfrac{1}{3} &\vert& \cdot 3 \cdot 5 \cr\cr 5(2x^2-3) &=& 3(x^2-8)+5 \cr 10x^2-15 &=& 3x^2-24+5 &\vert& -3x^2+15 \cr 7x^2 &=& -4 &\vert& :7 \cr x^2 &=& -\dfrac{4}{7} &\vert& \pm\sqrt{} \cr\cr x_{1,2} &=& \pm\sqrt{-\dfrac{4}{7}} \end{array}

Da aus negativen reellen Zahlen keine Wurzeln mit geraden Wurzelexponenten gezogen werden können, hat diese Gleichung keine Lösung: \mathbb{L} = \emptyset

3)
Hier ist die Anwendung der p-q- oder a-b-c-Formel nicht nötig. Umformen reicht aus.
\begin{array}{rclcl} -\dfrac{x}{16}(-32x+64)-x &=& 2x^2-5x \cr 2x^2-4x-x &=& 2x^2-5x \cr 2x^2-5x &=& 2x^2-5x & \vert & -2x^2+5x \cr 0 &=& 0 \cr \cr \mathbb{L} &=& \mathbb{R} \end{array}

Bemerkung: Unabhängig davon, welches Element des Definitionsbereichs in diese Gleichung eingesetzt wird, erhält man immer auf beiden Seiten dasselbe Ergebnis. 0=0 ist schließlich immer richtig. Jede reelle Zahl löst also diese Gleichung, d. h. die Lösungsmenge entspricht dem Definitionsbereich.


4)
Hier wird die p-q-Formel angewendet.
\begin{array}{rclcl} 5x^2-\left(\sqrt{175}-15\right)x &=& \dfrac{15}{2}\sqrt{7} &\vert& -\dfrac{15}{2}\sqrt{7} \cr\cr 5x^2-\left(\sqrt{25\cdot 7}-15\right)x-\dfrac{15}{2}\sqrt{7} &=& 0 &\vert& :5 \cr\cr x^2-\dfrac{1}{5}\left(5\left(\sqrt{7}-3\right)\right)x-\dfrac{3}{2}\sqrt{7} &=& 0 \cr\cr x^2-\left(\sqrt{7}-3\right)x-\dfrac{3}{2}\sqrt{7} &=& 0 \cr\cr x_{1,2} &=& \dfrac{\sqrt{7}-3}{2} \pm \sqrt{\left(\dfrac{\sqrt{7}-3}{2}\right)^2 -\left(-\dfrac{3}{2}\sqrt{7}\right)} \cr\cr &=& \dfrac{\sqrt{7}-3}{2} \pm \sqrt{\dfrac{7-6\sqrt{7}+9}{4} +\dfrac{6}{4}\sqrt{7}} \cr\cr &=& \dfrac{\sqrt{7}-3}{2} \pm \sqrt{\dfrac{16}{4}} \cr\cr &=& \dfrac{\sqrt{7}-3}{2} \pm \sqrt{4} \cr\cr &=& \dfrac{\sqrt{7}-3}{2} \pm 2 \cr\cr\cr x_1 &=& \dfrac{\sqrt{7}}{2}-\dfrac{3}{2}+2 = \dfrac{\sqrt{7}}{2}+\dfrac{1}{2} \cr\cr x_2 &=& \dfrac{\sqrt{7}}{2}-\dfrac{3}{2}-2 = \dfrac{\sqrt{7}}{2}-\dfrac{7}{2} \cr \cr \mathbb{L} &=& \left\{\dfrac{\sqrt{7}}{2}-\dfrac{7}{2}; \dfrac{\sqrt{7}}{2}+\dfrac{1}{2} \right\} \end{array}

Bemerkung 1: Achten Sie beim zweiten Umformungsschritt darauf, wirklich alle Summanden der Gleichung mit 5 zu multiplizieren.
Bemerkung 2: Die p-q-Formel funktioniert natürlich auch, wenn p und q so "unhandlich" sind wie hier ...


5)
Hier ist die Anwendung der p-q- oder a-b-c-Formel nicht nötig. Umformen reicht aus.
\begin{array}{rclcl} 1+x^2 &=& 3x+36x\left(x-\dfrac{1}{8}\right)+\dfrac{3}{2}\left(x+\dfrac{2}{3}\right)-35x^2 \cr 1+x^2 &=& 3x+36x^2-\dfrac{9}{2}x+\dfrac{3}{2}x+1-35x^2 \cr 1+x^2 &=& x^2+1 & \vert & -x^2-1 \cr 0 &=& 0 \cr \cr \mathbb{L} &=& \mathbb{R} \end{array}

Bemerkung: Unabhängig davon, welches Element des Definitionsbereichs in diese Gleichung eingesetzt wird, erhält man immer auf beiden Seiten dasselbe Ergebnis. 0=0 ist schließlich immer richtig. Jede reelle Zahl löst also diese Gleichung, d. h. die Lösungsmenge entspricht dem Definitionsbereich.


6)
Hier ist die Anwendung der p-q- oder a-b-c-Formel nicht nötig. Umformen reicht aus.
\begin{array}{rclcl} -12t(t-5)-47t &=& 13t \cr -12t^2+60t-47t &=& 13t \cr -12t^2+13t &=& 13t & \vert & -13t \cr -12t^2 &=& 0 & \vert & :(-12) \cr t^2 &=& 0 & \vert & \pm \sqrt{} \cr t &=& 0 \cr \cr \mathbb{L} &=& \{0\} \end{array}

7)
Hier ist die Anwendung der p-q- oder a-b-c-Formel nicht nötig. Umformen, ausklammern und der Satz vom Nullprodukt reichen aus.
\begin{array}{crclcl} & (x+2)^2 &=& 4(1-x) \cr & x^2+4x+4 &=& 4-4x & \vert & -4+4x \cr & x^2+8x &=& 0 \cr & x(x+8) &=& 0 \cr \text{Faktor 1:} & x_1 &=& 0 \cr\cr \text{Faktor 2:} & x+8 &=& 0 &\vert& -8 \cr & x_2 &=& -8 \cr\cr & \mathbb{L} &=& \{-8; 0\} \end{array}


8)
Hier wird die a-b-c-Formel angewendet.
\begin{array}{rclcl} (x+9)(x-9) &=& \dfrac{1}{2}(x+18)+\dfrac{11x}{2} \cr\cr x^2-81 &=& \dfrac{1}{2}(x+18)+\dfrac{11x}{2} & \vert & \cdot 2 \cr\cr 2x^2-162 &=& x+18+11x \cr\cr 2x^2-162 &=& 12x+18 & \vert & -12x-18 \cr\cr 2x^2-12x-180 &=& 0 \cr\cr x_{1,2} &=& \dfrac{12 \pm \sqrt{144+1440}}{4} \cr\cr x_1 &=& \dfrac{12}{4}+\dfrac{12\sqrt{11}}{4} = 3+3\sqrt{11} \cr\cr x_2 &=& \dfrac{12}{4}-\dfrac{12\sqrt{11}}{4} = 3-3\sqrt{11} \cr \cr \mathbb{L} &=& \{3-3\sqrt{11}; 3+3\sqrt{11}\} \end{array}

Bemerkung: Achten Sie beim zweiten Umformungsschritt darauf, wirklich alle Summanden der Gleichung mit 2 zu multiplizieren.


9)
Hier wird die p-q-Formel angewendet.
\begin{array}{rclcl} (8+3x)^2 &=& -2(19-12x-5x^2)-10 \cr 64+48x+9x^2 &=& -2(19-12x-5x^2)-10 \cr 64+48x+9x^2 &=& -38+24x+10x^2-10 & \vert & -10x^2 -24x+48 \cr -x^2+24x+112 &=& 0 & \vert & \cdot (-1) \cr x^2-24x-112 &=& 0 \cr x_{1,2} &=& 12 \pm \sqrt{144+112} \cr\cr x_1 &=& 12+16 = 28 \cr x_2 &=& 12-16 = -4 \cr\cr \mathbb{L} &=& \{-4; 28\} \end{array}


10)
Hier ist die Anwendung der p-q- oder a-b-c-Formel nicht nötig. Umformen reicht aus.
\begin{array}{rclcl} 4x^2-69 &=& -5+2x(8+2x) \cr 4x^2-69 &=& -5+16x+4x^2 & \vert & -4x^2+5 \cr -64 &=& 16x & \vert & :16 \cr x &=& -4 \cr\cr \mathbb{L} &=& \{-4\} \end{array}


11)
Hier wird die p-q-Formel angewendet.
\begin{array}{rclcl} (-7x+7)(7x-7)-31 &=& 21x^2+60 \cr -49x^2+49x+49x-49-31 &=& 21x^2+60 & \vert & -21x^2-60 \cr -70x^2+98x-140 &=& 0 & \vert & :(-70) \cr x^2-\dfrac{7}{5}x+2 &=& 0 \cr x_{1,2} &=& \dfrac{7}{10} \pm \sqrt{\dfrac{49}{100}-2} \cr x_{1,2} &=& \dfrac{7}{10} \pm \sqrt{-\dfrac{151}{100}} \end{array}

Da aus negativen reellen Zahlen keine Wurzeln mit geraden Wurzelexponenten gezogen werden können, hat diese Gleichung keine Lösung: \mathbb{L} = \emptyset

Bemerkung: Achten Sie beim dritten Umformungsschritt darauf, wirklich alle Summanden der Gleichung durch 70 zu dividieren.


12)
Hier wird die p-q-Formel angewendet.
\begin{array}{rclcl} 5(-3x^2+11x-115)-2x^2 &=& 101-((4x-1)^2+5x-1) \cr -15x^2+55x-575-2x^2 &=& 101-(16x^2-8x+1+5x-1) \cr -17x^2+55x-575 &=& 101-(16x^2-3x) \cr -17x^2+55x-575 &=& 101-16x^2+3x & \vert & +16x^2-3x-101 \cr -x^2+52x-676 &=& 0 & \vert & \cdot (-1) \cr x^2-52x+676 &=& 0 \cr x_{1,2} &=& 26 \pm \sqrt{676-676} \cr x_{1,2} &=& 26 \cr\cr \mathbb{L} &=& \{26\} \end{array}


13)
Hier wird die p-q-Formel angewendet.
\begin{array}{rclcl} (x+33)^2 &=& -8(-47+x)^2 \cr x^2+66x+1.089 &=& -8(x^2-94x+2.209) \cr x^2+66x+1.089 &=& -8x^2+752x-17.672 & \vert & +8x^2-752x+17.672 \cr 9x^2-686x+18.761 &=& 0 & \vert & :9 \cr x^2-\dfrac{686}{9}x+\dfrac{18.761}{9} &=& 0 \cr x_{1,2} &=& \dfrac{343}{9} \pm \sqrt{\dfrac{117.649}{81}-\dfrac{18.761}{9}} \cr x_{1,2} &=& \dfrac{343}{9} \pm \sqrt{-\dfrac{51.200}{81}} \end{array}

Da aus negativen reellen Zahlen keine Wurzeln mit geraden Wurzelexponenten gezogen werden können, hat diese Gleichung keine Lösung: \mathbb{L} = \emptyset

Bemerkung: Achten Sie beim vierten Umformungsschritt darauf, wirklich alle Summanden der Gleichung durch 9 zu dividieren.


14)
Hier ist die Anwendung der p-q- oder a-b-c-Formel nicht nötig. Umformen reicht aus.
\begin{array}{rclcl} (6+8y)^2 &=& 16(4y+3)\left(y+\dfrac{3}{4}\right) \cr\cr 36+96y+64y^2 &=& 16(4y+3)\left(y+\dfrac{3}{4}\right) & \vert & :4 \cr\cr 9+24y+16y^2 &=& 16 \left(y+\dfrac{3}{4}\right)\left(y+\dfrac{3}{4}\right) \cr\cr 9+24y+16y^2 &=& 16\left(y^2+\dfrac{3}{2}y+\dfrac{9}{16}\right) \cr\cr 9+24y+16y^2 &=& 16y^2+24y+9 &\vert& -16y^2-24y-9 \cr\cr 0 &=& 0 \cr \cr \mathbb{L} &=& \mathbb{R} \end{array}

Bemerkung 1: Unabhängig davon, welches Element des Definitionsbereichs in diese Gleichung eingesetzt wird, erhält man immer auf beiden Seiten dasselbe Ergebnis. 0=0 ist schließlich immer richtig. Jede reelle Zahl löst also diese Gleichung, d. h. die Lösungsmenge entspricht dem Definitionsbereich.

Bemerkung 2: Achten Sie beim zweiten Umformungsschritt darauf, wirklich alle Summanden der Gleichung durch 4 zu dividieren. Da auf der rechten Seite nur ein Summand steht (es wird ja multipliziert ...), muss dort natürlich nur einmal durch 4 geteilt werden.


15)
Hier ist die Anwendung der p-q- oder a-b-c-Formel nicht nötig. Umformen reicht aus.
\begin{array}{rclcl} 12\left(\left(\dfrac{p}{2}-\dfrac{1}{2}\right)^2-\dfrac{5}{4}p^2\right)+12p^2 &=& 9 \cr\cr 12\left(\dfrac{1}{4}p^2-\dfrac{1}{2}p+\dfrac{1}{4}-\dfrac{5}{4}p^2\right)+12p^2 &=& 9 \cr\cr 12\left(-p^2-\dfrac{1}{2}p+\dfrac{1}{4}\right)+12p^2 &=& 9 \cr\cr -12p^2-6p+3+12p^2 &=& 9 & \vert & -3 \cr -6p &=& 6 & \vert & :(-6) \cr p &=& -1 \cr\cr \mathbb{L} &=& \{-1\} \end{array}


16)
Hier wird die p-q-Formel angewendet.
\begin{array}{rclcl} -76(13+x)+87x^2 &=& 43x(-1+2x) \cr -988-76x+87x^2 &=& -43x+86x^2 & \vert & -86x^2+43x \cr x^2-33x-988 &=& 0 \cr x_{1,2} &=& \dfrac{33}{2} \pm \sqrt{\dfrac{1.089}{4}+988} \cr\cr x_1 &=& \dfrac{33}{2}+\dfrac{71}{2} = 52 \cr\cr x_2 &=& \dfrac{33}{2}-\dfrac{71}{2} = -19 \cr \cr \mathbb{L} &=& \{-19; 52\} \end{array}


17)
Hier ist die Anwendung der p-q- oder a-b-c-Formel nicht nötig. Umformen und der Satz vom Nullprodukt reichen aus.
\begin{array}{crclcl} & x\left(\dfrac{31}{20}x-\dfrac{7}{40}\right)-x^2 &=& 0 \cr\cr & \dfrac{31}{20}x^2-\dfrac{7}{40}x-x^2 &=& 0 & \vert & \cdot 40 \cr\cr & 62x^2-7x-40x^2 &=& 0 \cr & 22x^2-7x &=& 0 \cr & x(22x-7) &=& 0 \cr \text{Faktor 1:} & x_1 &=& 0 \cr\cr \text{Faktor 2:} & 22x-7 &=& 0 &\vert& +7 \cr & 22x &=& 7 &\vert& :22 \cr & x_2 &=& \dfrac{7}{22} \cr\cr & \mathbb{L} &=& \left\{0;\dfrac{7}{22}\right\} \end{array}

Bemerkung: Achten Sie beim zweiten Umformungsschritt darauf, wirklich alle Summanden der Gleichung mit 40 zu multiplizieren.


18)
Hier wird die p-q-Formel angewendet.
\begin{array}{rclcl} -\dfrac{12}{27}x^2 &=& \dfrac{1}{9}\left(\dfrac{1}{3}\left(15x^2-\dfrac{2}{3}\right)-\dfrac{7}{2}x\right) \cr\cr -\dfrac{12}{27}x^2 &=& \dfrac{1}{27}\left(15x^2-\dfrac{2}{3}\right)-\dfrac{7}{18}x \cr\cr -\dfrac{12}{27}x^2 &=& \dfrac{15}{27}x^2-\dfrac{2}{81}-\dfrac{7}{18}x &\vert& +\dfrac{12}{27}x^2 \cr\cr 0 &=& x^2-\dfrac{7}{18}x-\dfrac{2}{81} \cr\cr x_{1,2} &=& \dfrac{7}{36} \pm \sqrt{\dfrac{49}{1.296}+\dfrac{2}{81}} \cr\cr x_1 &=& \dfrac{7}{36}+\dfrac{1}{4} = \dfrac{4}{9} \cr\cr x_2 &=& \dfrac{7}{36}-\dfrac{1}{4} = -\dfrac{1}{18} \cr \cr \mathbb{L} &=& \left\{-\dfrac{1}{18};\dfrac{4}{9}\right\} \end{array}


19)
Hier wird die p-q-Formel angewendet.
\begin{array}{rclcl} \dfrac{1}{2}(14x+x)x &=& -\dfrac{-21x^2+x}{6}+\dfrac{x^2+30}{3}+812{,}5 & \vert & \cdot 6 \cr\cr 3x\cdot(15x) &=& 21x^2-x+2x^2+60+4.875 \cr\cr 45x^2 &=& 23x^2-x+4.935 & \vert & -45x^2 \cr\cr 0 &=& -22x^2-x+4.935 & \vert & :(-22) \cr\cr 0 &=& x^2+\dfrac{1}{22}x-\dfrac{4.935}{22} \cr\cr x_{1,2} &=& -\dfrac{1}{44} \pm \sqrt{\dfrac{1}{1.936}+\dfrac{4.935}{22}} \cr\cr x_1 &=& \dfrac{329}{22} \cr\cr x_2 &=& -15 \cr \cr \mathbb{L} &=& \left\{-15;\dfrac{329}{22}\right\} \end{array}

Bemerkung: Achten Sie beim ersten Umformungsschritt darauf, wirklich alle Summanden der Gleichung mit 6 zu multiplizieren. Da auf der linken Seite nur ein Summand steht (es wird ja multipliziert ...), muss dort natürlich nur einmal durch 6 geteilt werden. Beim vierten Umformungsschritt müssen dann alle Summanden der Gleichung durch -22 dividiert werden.


20)
Hier wird die p-q-Formel angewendet.
\begin{array}{rclcl} 80+(11x-6)(11x+6)+4x^2 &=& 6\left(\dfrac{100}{3}x-6\right) \cr 80+121x^2-36+4x^2 &=& 200x-36 & \vert & +36 \cr 80+125x^2 &=& 200x & \vert & -125x^2-80 \cr 0 &=& -125x^2+200x-80 & \vert & :(-125) \cr 0 &=& x^2-\dfrac{8}{5}x+\dfrac{16}{25} \cr\cr x_{1,2} &=& \dfrac{4}{5} \pm \sqrt{\dfrac{16}{25}-\dfrac{16}{25}} \cr\cr x_{1,2} &=& \dfrac{4}{5} \cr\cr \mathbb{L} &=& \left\{\dfrac{4}{5}\right\}\end{array}

Bemerkung: Achten Sie beim vierten Umformungsschritt darauf, wirklich alle Summanden der Gleichung durch -125 zu dividieren.

 

3. Aufgabe

Sei x die Länge der Quadratseiten (siehe Bemerkung zu Textaufgaben).
Dann ergeben sich folgende Terme:

  • der Flächeninhalt des Quadrats: x^2
  • der Flächeninhalt des Quadrats vermindert um 4\, cm^2: x^2-4
  • die um 8\, cm verlängerte Seite: x+8
  • die um 6\, cm verkürzte Seite: x-6
  • der Flächeninhalt des entstehenden Rechtecks: (x+8)(x-6)

Zu lösen ist also die folgende Gleichung:
\begin{array}{rclcl} (x+8)(x-6) &=& x^2-4 \cr x^2+8x-6x-48 &=& x^2-4 \cr x^2+2x-48 &=& x^2-4 &\vert& -x^2 \cr 2x-48 &=& -4 &\vert& +48 \cr 2x &=& 44 &\vert& \cdot \dfrac{1}{2} \cr x &=& 22 \end{array}

Das Quadrat hatte also eine Seitenlänge von 22\, cm und damit einen Flächeninhalt von 484\, cm^2
Beim Rechteck sind die Seiten 16 \, cm und 30 \, cm lang. Sein Flächeninhalt beträgt 480\, cm^2

Bemerkung: Beim Lösen von Textaufgaben dürfen in der Mathematik (im Gegensatz zur Physik) Einheiten weggelassen werden. Erst im Antwortsatz gehören auch die Einheiten wieder zur Lösung.

10. Quadratische Funktionen - Lernziele und typische Fehler

Nach Durcharbeiten dieses Kapitels sollten Sie folgende Lernziele erreicht haben:

  • Sie können zu einer quadratischen Funktion den passenden Definitionsbereich bestimmen.
  • Sie wissen, wie der Graph einer quadratischen Funktion typischerweise aussieht, und können ihn in ein kartesisches Koordinatensystem zeichnen.
  • Sie kennen die allgemeine Funktionsgleichung einer quadratischen Funktion.
  • Sie wissen, welchen Einfluss die Parameter in der Funktionsgleichung auf den Verlauf des Graphen haben und können diese Informationen für die Interpretation von Zusammenhängen nutzen.
  • Sie können eine Parabel verschieben, strecken und stauchen.
  • Sie können aus dem gegebenen Graphen einer quadratischen Funktion die Funktionsgleichung ermitteln.
  • Sie können den Scheitelpunkt einer Parabel (grafisch und rechnerisch) ermitteln.
  • Sie können den Schnittpunkt von zwei quadratischen Funktionen bzw. von einer linearen mit einer quadratischen Funktion berechnen.
  • Für Profis: Sie können die Funktionsgleichung einer quadratischen Funktion aus gegebenen Informationen (z. B. aus drei Punkten) berechnen.
  • Sie kennen den Zusammenhang zwischen quadratischen Gleichungen und quadratischen Funktionen.
  • Sie können quadratische Funktionen von anderen Funktionstypen unterscheiden (grafisch und anhand der Funktionsgleichung).


Typischer Fehler in diesem Kapitel ist:

  • Verschiebungen der Parabel nach links und rechts werden verwechselt. Erklärung


Für Online-Selbsttests zu diesem Thema und weitere Informationen zur Mathematikunterstützung an der TH Wildau nutzen Sie bitte den Moodle-Kursraum "SOS Mathematik - Brückenkurs".

 

Dieses Kapitel wird gerade überarbeitet.

Übersicht:

 

10.1 Quadratische Funktionen - Aufgaben

1. Aufgabe

a)

Zeichnen Sie eine Normalparabel, d. h. die Funktion f(x)=x^2, in ein Koordinatensystem!

Skizzieren Sie - ausgehend von diesem Graphen - die Graphen zu folgenden Funktionen! Gemeint ist diese Aufgabe so, dass Sie überlegen, wie die Funktionen sich von der Normalparabel unterscheiden und daraus ableiten, wie der Graph aussehen muss. Eine ausführliche Wertetabelle ist nicht nötig.

1) f_1(x) = x^2+4   5) f_5(x) = 4 x^2

2) f_2(x) = x^2-4   6) f_6(x) = -4x^2

3) f_3(x) = (x+4)^2   7) f_7(x) = \dfrac{1}{4}x^2

4) f_4(x) = (x-4)^2

  8) f_8(x) = -\dfrac{1}{4}x^2

 

b)

Skizzieren Sie auch die folgenden Graphen in einem Koordinatensystem - auch hier nicht über eine Wertetabelle, sondern über Überlegungen zum Scheitelpunkt und zur Streckung/Stauchung.

1) f_1(x) = \dfrac{1}{2}x^2-12   6) f_6(x) = x^2-8

2) f_2(x) = -2(x-3)^2   7) f_7(x) = -\dfrac{7}{13}(x-9)^2

3) f_3(x) = (x-5)^2+5

  8) f_8(x) = -(x+6)^2+4
4) f_4(x) = \dfrac{10}{3} \left(x+10 \right)^2+2

  9) f_9(x) = -x^2+2
5) f_5(x) = \left(x+\dfrac{1}{2}\right)^2-4

  10) f_{10}(x) = (x-3{,}5)^2

 

2. Aufgabe

Beantworten Sie zu den quadratischen Funktionen jeweils die folgenden Fragen:

  • Wie viele Nullstellen hat die Funktion?
  • Ist die Parabel nach oben oder nach unten geöffnet?
  • Ist die Parabel symmetrisch zur y-Achse oder zu einer Parallelen der y-Achse?
  • Ist der Funktionsgraph steiler als, weniger steil als oder genauso steil wie eine Normalparabel?

Und das Alles: Ohne zu rechnen! Es geht nämlich bei dieser Aufgabe darum, den Funktionsterm zu verstehen und daraus Informationen über den Funktionsgraphen abzulesen, um z. B. Rechnungen plausibilisieren zu können.

1) g(x)=-x^2+6

  11) s(t)=-\dfrac{2}{3}t^2-\dfrac{2}{3}

2) h(x)=26x^2-13x+5

  12) f(x)=(x+2)^2

3) f(x)=-5x^2-2x

  13) u(x)=17(518+x)(x-96)

4) f(x)=30(x+5)(x-10)

  14) v(x)=\left(-\dfrac{1}{2}x-2\right)^2

5) f(z)=-\dfrac{1}{25}z^2

  15) f(x)=-\dfrac{2}{3}x^2-\dfrac{2}{3}x-\dfrac{2}{3}

6) f(x)=-\dfrac{1}{2}x^2+\dfrac{1}{4}x-\dfrac{1}{3}

 

16) y(x)=5x^2+50x-500

7) f(x)=\dfrac{1}{2}x^2+2

  17) z(x)=-22x^2-12x+2

8) f(x)=x^2+2x-3

  18) f(x)=-(25x-201)^2

9) f(x)=\dfrac{1}{3}x^2-\dfrac{1}{9}x

  19) f(x)=x^2+4\pi x+\pi

10) f(x)=-x^2+\dfrac{4}{5}x-1

  20) f(x)=-\sqrt{2}x^2-\sqrt{112}

 

3. Aufgabe

Bestimmen Sie zu den eingezeichneten Graphen jeweils die Funktionsgleichung! Es sind quadratische Funktionen gesucht.

 

4. Aufgabe

1)
Gegeben sei die Funktion f(x)=7x^2-14x+49 mit x\in \mathbb{R}.
Gesucht ist jeweils die fehlende Koordinate des Punktes P(x\mid y),
a) wenn x=7
b) wenn y=105

  6)
Gegeben sei die Funktion f(x)=2(x-1)^2-\dfrac{4}{3} mit x\in \mathbb{R}
Gesucht ist jeweils die fehlende Koordinate des Punktes P(x\mid y),
a) wenn x=17
b) wenn y=-\dfrac{4}{3}

2)
Gegeben sei die Funktion f(x)=-x^2-24x+2 mit x\in \mathbb{R}
Gesucht ist jeweils die fehlende Koordinate des Punktes P(x\mid y),
a) wenn x=-11{,}5
b) wenn y=25

  7)
Gegeben sei die Funktion f(a)=(-3+a)^2-6 mit a\in \mathbb{R}
Gesucht ist jeweils die fehlende Koordinate des Punktes P(a\mid y),
a) wenn a=\dfrac{15}{4}
b) wenn y=-43

3)
Gegeben sei die Funktion f(x)=32x^2-x mit x\in \mathbb{R}
Gesucht ist jeweils die fehlende Koordinate des Punktes P(x\mid y),
a) wenn x=8
b) wenn y=-4

  8)
Gegeben sei die Funktion f(x)=(4x+12)^2+8 mit x\in \mathbb{R}
Gesucht ist jeweils die fehlende Koordinate des Punktes P(x\mid y)
a) wenn x=5
b) wenn y=8

4)
Gegeben sei die Funktion f(x)=-3x^2+7x mit x\in \mathbb{R}
Gesucht ist jeweils die fehlende Koordinate des Punktes P(x\mid y),
a) wenn x=6
b) wenn y=4

  9)
Gegeben sei die Funktion f(x)=\dfrac{1}{10}(x-7)^2 mit x\in \mathbb{R}
Gesucht ist jeweils die fehlende Koordinate des Punktes P(x\mid y),
a) wenn x=18
b) wenn y=0

5)
Gegeben sei die Funktion g(x)=6x^2-17 mit x\in \mathbb{R}
Gesucht ist jeweils die fehlende Koordinate des Punktes P(x\mid y),
a) wenn x=-\dfrac{5}{6}
b) wenn y=0

  10)
Gegeben sei die Funktion f(x)=100x^2-50x+25 mit x\in \mathbb{R}
Gesucht ist jeweils die fehlende Koordinate des Punktes P(x\mid y),
a) wenn x=\dfrac{1}{2}
b) wenn y=775

 

5. Aufgabe

Bestimmen Sie jeweils den Scheitelpunkt der Parabel! Anders formuliert: Formen Sie die Funktionsgleichungen so um: f(x) = a(x-d)^2+e

1) f(x) = x^2-6x+16   6) f(x)= -9x^2 - 90x - 225
2) f(x) = 2x^2-4x+2

  7) f(x)= 4x^2 -4x +3
3) f(x) = -13x^2-8   8) f(x) = 3x^2-4x-\dfrac{14}{3}
4) f(x) = \frac{1}{4} x^2-6x+32

  9) f(x) = \dfrac12 x^2 +4x -7
5) f(x) = -11x^2-44x-34   10) f(x) = -x^2 +26x -99

 

6. Aufgabe

Stellen Sie sich eine Gerade und eine Parabel vor. Wie können diese beiden Graphen zueinander liegen und wie viele Schnittpunkte kann es zwischen ihnen geben?

Stellen Sie sich zwei Parabeln vor. Wie können diese beiden Graphen zueinander liegen und wie viele Schnittpunkte kann es zwischen ihnen geben?

 

7. Aufgabe

Berechnen Sie jeweils den Schnittpunkt bzw. die Schnittpunkte der folgenden zwei Funktionen!

1) f(x) = 2x^2-4x+2
und g(x) = \dfrac{1}{2}x+1
  6) f(x) = -5x^{2} + x + 2
und g(x) = -x^{2} + x + 1

2) f(x) = -9x^2+8x-1
und g(x) = -10x^2-\dfrac{6}{5}x+7
  7) f(x) = -x + 4
und g(x) = -x^2 + 4{,}5x + 4
3) f(x) = \dfrac{3}{2}x^2+5x+15
und g(x) = x^2-x-20

  8) f(x) = 1{,}5x^2 + 3x
und g(x) = 1{,}5x^2 + 10x + 14
4) f(x) = 4x^2-11
und g(x) = 12x-11

  9) f(x) = 3x + 7
und g(x) = -x^2 + 10x + 1
5) f(x) = -3x^2-100x+300
und g(x) = -15x^2+20x
  10) f(x) = \dfrac12 x^2-13x-15
und g(x) = -8x-15

 

8. Aufgabe

Bestimmen Sie jeweils die Schnittpunkte der Funktion mit den Koordinatenachsen!

1)
f(x) = \dfrac{17}{3}\cdot (x^2-4x-221)

  6)
f(z) = -3z^2+22z-42

2)
f(a) = -\dfrac{1}{4}a^2+a

  7)
q(t) = \left(\dfrac{t+17}{\sqrt{2}}\right)^2

3)
h(x) = \left(x+\sqrt{2}\right)^2

  8)
f(x) = x^2+4x-5

4)
f(x) = -\dfrac{3}{4}x^2+2

  9)
f(x) = \dfrac{a-1}{3}x^2-a
5)
f(x) = 3x^2-6x-1

  10)
f(x) = 3x^2+(6-6r)x-12r

 

9. Aufgabe

Gegeben sei die Funktion f(x)=2x^2+ax-1. Für welches a gibt es
a) genau eine Nullstelle?
b) keine Nullstelle?

 

10. Aufgabe

Für Profis: Bestimmen Sie jeweils die quadratische Funktion, die durch die angegebenen Punkte verläuft!

1) P_1(0 \mid -16), P_2(1 \mid -10) und P_3(3\mid 14)

2) P_1\left(2 \mid 1\right), P_2\left(1 \mid 2\right) und P_3\left(0 \mid 1\right)

3) P_1\left(0 \mid 0\right), P_2\left(\dfrac{1}{3} \mid -\dfrac{1}{3}\right) und P_3\left(\dfrac{2}{3} \mid 0\right)

4) P_1\left(4 \mid 3\right), P_2\left(2 \mid 0\right) und P_3\left(-5 \mid 5{,}25\right)

5) P_1 \left(-1 \mid 2 \right), P_2 \left(0 \mid 4 \right) und P_3 \left(2 \mid 12 \right)

Dieses Kapitel enthält die folgenden Themen:

 

10.2 Quadratische Funktionen - Erklärungen

So, wie es zu den linearen Gleichungen lineare Funktionen gibt, gibt es auch quadratische Funktionen passend zu den quadratischen Gleichungen. Um diese wird es im Folgenden gehen. Es wird damit ein bisschen komplexer als bei linearen Funktionen - dafür auch etwas spannender.

 

Was man allgemein wissen sollte

Die allgemeine Gleichung einer quadratischen Funktion lautet: f(x) = ax^2+bx+c mit a,b,c \in \mathbb{R} (siehe Zahlenbereiche) oder auch f(x) = a(x-d)^2+e mit a,d,e \in \mathbb{R}. In beiden Fälle muss a\neq 0 sein, weil sonst der quadratische Term der Funktionsgleichung (oder sogar noch mehr) wegfällt. Das "Problem" kennen wir schon von den quadratischen Gleichungen.

Der Graph einer quadratischen Funktion ist eine Parabel. Schauen wir uns als Erstes den Graphen von f(x) = x^2, der einfachsten quadratischen Funktion, an. Dazu erstellen wir eine Wertetabelle mit einigen Funktionswerten:

x   -5 -4 -3 -2 -1 0 1 2 3 4 5
f(x)   25 16 9 4 1 0 1 4 9 16 25


Im Koordinatensystem sieht die Parabel dann so aus:

die Normalparabel

Es fällt auf, dass die Parabel für negative x-Werte fällt und für positive x-Werte steigt. Daher hat sie bei x=0 einen tiefsten Punkt. Dieser Verlauf ist typisch für Parabeln. Die Alternative zu diesem Verlauf ist, dass die Parabel für negative x-Werte steigt und für positive x-Werte fällt und damit einen höchsten Punkt hat (Beispiele dafür kommen etwas später im Laufe dieses Kapitels ...). Der tiefste bzw. höchste Punkt einer Parabel wird Scheitelpunkt genannt. Damit haben wir auch schon - bis auf Verschiebungen, um die wir uns weiter unten kümmern werden - alle möglichen Verläufe von Parabeln besprochen. Der Gegensatz zu linearen Funktionen, die entweder nur steigen oder nur fallen (wenn sie nicht konstant sind) und daher auch keinen höchsten oder tiefsten Punkt haben können, ist vermutlich offensichtlich.

 

In den nächsten Abschnitten schauen wir uns jeweils anhand von Beispielen an, was man mit einer Parabel im Koordinatensystem alles machen kann. In der allgemeinen Gleichung einer quadratischen Funktion tauchen mehrere Konstanten auf, die jeweils einen bestimmten Einfluss auf den Verlauf der Parabel haben. Diese Einflüsse gibt es nicht bloß bei Parabeln sondern bei jedem Funktionsgraphen, sodass wir hier einen Grundstein für die Betrachtung von ganz verschiedenen Funktionstypen legen.

 

Strecken und Stauchen einer Parabel

Schauen wir als erstes, wie die reelle Zahl a\neq 0 aus der Parabelgleichung (f(x) = ax^2+bx+c oder f(x) = a(x-d)^2+e) den Verlauf des Graphen beeinflusst: Da f(x)=x^2 die Kurzform für f(x)=1\cdot x^2 ist (Sie wissen ja, eine 1 wird gerne mal weggelassen ...), hatten wir oben schon ein Beispiel dafür gesehen, dass Parabeln mit positivem a, also a \in \mathbb{R}^+, nach oben geöffnet sind.

Ist a dabei größer als 1, also a>1, dann wird die Parabel steiler. Mathematisch sagt man: Sie wird gestreckt. Beispielhaft schauen wir uns die Wertetabelle von f_1(x)=2x^2 an. Sie sehen, dass die Funktionswerte alle deutlicher größer sind als bei f(x):

x   -5 -4 -3 -2 -1 0 1 2 3 4 5
f_1(x)   50 32 18 8 2 0 2 8 18 32 50

Der Graph von f_1(x) ist im folgenden Koordinatensystem als dünne Linie eingezeichnet.

Ist a zwar positiv, aber kleiner als 1, also 0 < a < 1, dann wird die Parabel bauchiger. Mathematisch sagt man: Sie wird gestaucht. Das Beispiel hier ist f_2(x)=\dfrac{1}{10}x^2:

x   -5 -4 -3 -2 -1 0 1 2 3 4 5
f_2(x)   2{,}5 1{,}6 0{,}9 0{,}4 0{,}1 0 0{,}1 0{,}4 0{,}9 1{,}6 2{,}5

Der Graph von f_2(x) ist im folgenden Koordinatensystem als Linie mit Sternchen eingezeichnet. Als Vergleich ist der Graph von f(x)=x^2 als dicke Linie miteingezeichnet.

gestreckte und gestauchte Parabeln

Sie sehen, dass die drei Parabeln sehr ähnlich verlaufen: Alle sind nach oben geöffnet. Alle haben den Scheitelpunkt im Punkt (0\mid 0). Alle haben die typische Parabelform. Nur in der Streckung/Stauchung unterscheiden sich die drei Parabeln. In Abhängigkeit von a ändert sich also das Steigungsverhalten der Parabel.


Nun kann a auch negativ sein, also a\in \mathbb{R}^-. Dann ist die Parabel nach unten geöffnet. Wir betrachten als Beispiel die einfachste quadratische Funktion mit einem negativen a, nämlich g(x)=-x^2=-1\cdot x^2:

x   -5 -4 -3 -2 -1 0 1 2 3 4 5
g(x)   -25 -16 -9 -4 -1 0 -1 -4 -9 -16 -25

Der Graph von g(x) ist im folgenden Koordinatensystem als dicke Linie eingezeichnet.

Ist a dabei kleiner als -1, also a < -1, dann wird die Parabel wieder gestreckt, allerdings "kopfüber". Beispielhaft schauen wir uns die Wertetabelle von g_1(x)=-2x^2 an:

x   -5 -4 -3 -2 -1 0 1 2 3 4 5
g_1(x)   -50 -32 -18 -8 -2 0 -2 -8 -18 -32 -50

Der Graph von g_1(x) ist im folgenden Koordinatensystem als dünne Linie eingezeichnet.

Ist a zwar negativ, aber größer als -1, also -1 < a < 0, dann wird die Parabel "kopfüber" gestaucht. Sie sehen das an der folgenden Wertetabelle für g_2(x)=-\dfrac{1}{10}x^2:

x   -5 -4 -3 -2 -1 0 1 2 3 4 5
g_2(x)   -2{,}5 -1{,}6 -0{,}9 -0{,}4 -0{,}1 0 -0{,}1 -0{,}4 -0{,}9 -1{,}6 -2{,}5

Der Graph von g_2(x) ist im folgenden Koordinatensystem als Linie mit Sternchen eingezeichnet. Als Vergleich ist der Graph von g(x)=-x^2 als dicke Linie miteingezeichnet.

gestreckte und gestauchte Parabeln, diesmal nach unten geöffnet

Die Erkenntnisse hier sind ziemlich analog zu oben ... Die drei Parabeln verlaufen sehr ähnlich: Diesmal sind alle nach unten geöffnet. Es bleibt dabei, dass alle den Scheitelpunkt im Punkt (0\mid 0) und die typische Parabelform haben. Nur in der Streckung/Stauchung gibt es wieder Unterschiede im Verlauf der drei Parabeln.
Wenn Sie diese Grafik mit der darüber vergleichen, sehen Sie, dass f(x)=x^2 und g(x)=-x^2 spiegelbildlich zueinander verlaufen. Gleiches gilt für f_1(x) und g_1(x) sowie für f_2(x) und g_2(x). Was soll auch anderes passieren, wenn die Funktionswerte bis auf das Minuszeichen alle gleich sind?

Eine wichtige Vokabel noch in diesem Zusammenhang: Ist a=1 oder a=-1, also bei f(x)=x^2 und g(x)=-x^2, wird die Parabel auch Normalparabel genannt.

 

Nun wäre es ein bisschen langweilig, wenn wir zwar verschieden gestreckte/gestauchte Parabeln erzeugen könnten, der Scheitelpunkt der Parabel aber immer im Koordinatenursprung liegen müsste ... Die übrigen Konstanten aus der Parabelgleichung machen es möglich, die Parabel im Koordinatensystem in alle Richtungen zu verschieben.

 

Verschieben einer Parabel

Auch hier schauen wir uns verschiedene Beispiele als Wertetabelle und als Graph an. Ausgangspunkt ist jeweils die sogenannte Scheitelpunktform einer quadratischen Gleichung f(x) = a(x-d)^2+e mit a,d,e \in \mathbb{R} und a\neq 0. Um a hatten wir uns im vorherigen Abschnitt schon gekümmert. Nun interessieren uns d und e.
In den folgenden beiden Grafiken ist der Graph von f(x)=x^2 als dicke Linie zum Vergleich miteingezeichnet.

Wir nehmen eine positive reelle Zahl e, also e \in \mathbb{R}^+.
Wahrscheinlich können Sie sich vorstellen, was bei f_1(x)=x^2+e passiert, wenn also zu den Funktionswerten von f(x)=x^2 jeweils e addiert wird ... Richtig: Alle Funktionswerte werden um e größer; der Graph wird also um e Einheiten nach oben verschoben. Besonders gut sieht man das am Scheitelpunkt. Als Beispiel haben wir hier f_1(x)=x^2+5:

x   -5 -4 -3 -2 -1 0 1 2 3 4 5
f_1(x)   30 21 14 9 6 5 6 9 14 21 30

Der Graph von f_1(x) ist im folgenden Koordinatensystem als dünne Linie eingezeichnet.

Umgekehrt ist der Graph der Funktion f(x) = x^2-e gegenüber dem Graphen von f(x)=x^2 um e Einheiten nach unten verschoben. Wir schauen uns hier f_2(x)=x^2-2 an:

x   -5 -4 -3 -2 -1 0 1 2 3 4 5
f_2(x)   23 14 7 2 -1 -2 -1 2 7 14 23

Der Graph von f_2(x) ist im folgenden Koordinatensystem als Linie mit Sternchen eingezeichnet.

nach oben und nach unten verschobene Parabeln

In beiden Fällen bleibt der Scheitelpunkt bei x=0, aber sein Funktionswert ändert sich: Der Scheitelpunkt von f_1(x) liegt im Punkt (0\mid 5). Bei f_2(x) liegt der Scheitelpunkt im Punkt (0\mid -2). Da bei beiden Funktionen a=1 ist, sind sie nicht gestreckt/gestaucht, sondern haben exakt die gleiche Form wie die mit einer dicken Linie eingezeichnete Normalparabel.

 

Zu guter Letzt: d sei eine positive reelle Zahl, also d \in \mathbb{R}^+. Natürlich geht es jetzt um die Verschiebung nach rechts und links - allerdings funktioniert diese nicht ganz so, wie man es vielleicht erwartet:
Die Funktion f(x) = (x-d)^2 ist gegenüber der Normalparabel um d Einheiten nach rechts verschoben. Ein negatives Vorzeichen in der Klammer führt also zu einer Verschiebung in positive x-Richtung. Schauen wir uns beispielhaft g_1(x)=(x-1)^2 an:

x   -5 -4 -3 -2 -1 0 1 2 3 4 5
g_1(x)   36 25 16 9 4 1 0 1 4 9 16

Der Graph von g_1(x) ist im folgenden Koordinatensystem als dünne Linie eingezeichnet.

Die Funktion f(x) = (x+d)^2 ist gegenüber der Normalparabel um d Einheiten nach links verschoben. Ein positives Vorzeichen in der Klammer führt also zu einer Verschiebung in negative x-Richtung. Auch hier ein Beispiel: g_2(x)=(x+6)^2

x   -5 -4 -3 -2 -1 0 1 2 3 4 5
g_2(x)   1 4 9 16 25 36 49 64 81 100 121

Der Graph von g_2(x) ist im folgenden Koordinatensystem als Linie mit Sternchen eingezeichnet.

nach links und rechts verschobene Parabeln

Sie sehen, dass der Scheitelpunkt in die "verkehrte" Richtung rutscht: Steht in der Klammer x-1, ist die x-Koordinate des Scheitelpunkts +1; bei x+6 in der Klammer ist die x-Koordinate des Scheitelpunkts -6.
Woran das liegt, schauen wir uns am Beispiel von g_1(x) an. Genauer, wir betrachten den Scheitelpunkt von g_1(x)=(x-1)^2: Es handelt sich um eine nach oben geöffnete Parabel, da der Koeffizient von x^2 positiv ist. Damit ist der Scheitelpunkt der niedrigste Punkt der Parabel, also der Punkt mit der kleinsten y-Koordinate (Das sehen Sie auch in der Grafik ...). Da Quadrate nie negativ werden können, ist 0 der kleinste Wert, den der Funktionsterm (x-1)^2 annehmen kann. Wir müssen also die Frage beantworten, bei welchem x-Wert der Funktionsterm 0 wird, um zu sehen, wohin der Scheitelpunkt rutscht: Das passiert, wenn x=1 ist, denn g_1(1)=(1-1)^2=0^2=0. Der Scheitelpunkt von g_1(x) liegt also im Punkt (1\mid 0). Der negative Wert d in der Parabelgleichung führt also dazu, dass der Scheitelpunkt in den positiven x-Bereich rutscht.
Umgekehrt führt ein positiver Wert d in der Parabelgleichung dazu, dass der Scheitelpunkt im negativen x-Bereich liegt, weil (x+6)^2 (also der Funktionsterm von g_2(x)) dann 0 wird, wenn x=-6 ist.
Fazit: Lassen Sie sich von der auf den ersten Blick unlogischen Verschiebungsrichtung nicht irritieren! Am besten gelingt dies, wenn Sie sich am Scheitelpunkt orientieren.

Kleine Randbemerkung: Wie man in der Grafik sieht, ist die Wertetabelle von g_2(x) nicht sehr geschickt, weil die x-Werte so gewählt sind, dass der Scheitelpunkt in den berechneten Werten gar nicht enthalten ist. Aus den Werten geht nur der aufsteigende Teil der Parabel hervor, was einen falschen Eindruck vom Verlauf der Funktion erwecken kann. Auch fürs Erstellen von Wertetabellen ist es also gut, aus dem Funktionsterm eine Vorstellung vom Verlauf des Graphen entwickeln zu können. Dann kann man die x-Werte so auswählen, dass sie dem Verlauf des Graphen gerecht werden. In diesem Fall wären x-Werte zwischen -11 und -1 besser gewesen.

 

Zusammenfassung:

Wir sind von der Scheitelpunktform einer Parabel f(x) = a(x-d)^2+e mit a,d,e \in \mathbb{R} und a\neq 0 ausgegangen und sind über die Beispiele zu folgenden Erkenntnissen gekommen:

  • a bewirkt eine Streckung/Stauchung der Parabel.
  • d führt zu einer Verschiebung der Parabel nach rechts/links.
  • e verschiebt die Parabel nach oben/unten.

Selbstverständlich kann eine Parabel gleichzeitig gestreckt/gestaucht und nach rechts/links und nach oben/unten verschoben sein, z. B. f(x) = \dfrac{1}{2}\left(x-\dfrac{3}{2}\right)^2-\dfrac{25}{8}.
Sie ist

  • um den Faktor \dfrac{1}{2} gestaucht.
  • um \dfrac{3}{2}=1{,}5 Einheiten nach rechts verschoben.
  • um \dfrac{25}{8}=3{,}125 Einheiten nach unten verschoben.

Graphisch sieht diese Funktion so aus:

verschobene, gestauchte Parabel

Sowohl aus der Funktionsgleichung als auch aus dem Graphen können Sie ablesen, dass der Scheitelpunkt bei S\left(\dfrac{3}{2} \mid -\dfrac{25}{8}\right) liegt (Achtung: kein Minuszeichen bei der x-Koordinate!). Daher heißt diese Form der Funktionsgleichung Scheitelpunktform. Natürlich kann man den Funktionsterm auch ausmultiplizieren:
\begin{array}{rcl} f(x) &=& \dfrac{1}{2}\left(x-\dfrac{3}{2}\right)^2-\dfrac{25}{8} \\ \\ &=& \dfrac{1}{2}\left(x^2-3x+\dfrac{9}{4}\right)-\dfrac{25}{8} \\ \\ &=& \dfrac{1}{2}x^2-\dfrac{3}{2}x+\dfrac{9}{8}-\dfrac{25}{8} \\ \\ &=& \dfrac{1}{2}x^2-\dfrac{3}{2}x+2\end{array}
Das nennt man die allgemeine Form der Funktionsgleichung. Wenn Sie die Funktion z. B. ableiten möchten (Wir klären in einem späteren Kapitel, was das ist ...), ist die ausmultiplizierte Form der Funktionsgleichung sehr hilfreich. Allerdings ist in ausmultiplizierter Form, also bei f(x)=\dfrac{1}{2}x^2-\dfrac{3}{2}x+2, nicht mehr zu erkennen, wo der Scheitelpunkt liegt. Dafür ist - wie der Name schon sagt - die Scheitelpunktform nützlicher. Beide Formen haben also ihre Berechtigung und man muss schauen, in welchem Kontext die quadratische Funktion steht, um zu entscheiden, welche Form benötigt wird.
Unabhängig davon: Achten Sie beim Ausmultiplizieren unbedingt auf die richtige Reihenfolge der Rechenoperationen - Sie wissen ja: Klammern vor Potenz- vor Punkt- vor Strichrechnung! Und die 2. binomische Formel möchte beim Auflösen von \left(x-\dfrac{3}{2}\right)^2 auch berücksichtigt werden ...

 

Quadratische Ergänzung

Gerade habe wir uns angeschaut, wie man in wenigen Schritten von der Scheitelpunktform zur allgemeinen Gleichung einer quadratischen Funktion kommt. Der umgekehrte Weg ist (leider) komplizierter: Das Verfahren dazu heißt quadratische Ergänzung und beruht auf der Anwendung der 1. oder 2. binomischen Formel von rechts nach links. Wir schauen uns das genaue Vorgehen an folgendem Beispiel an:
\begin{array}{rclcc} f(x) &=& 2x^2-4x+6 & \vert & \text{ausklammern der 2 vor dem } x^2 \cr f(x) &=& 2(x^2-2x+3) \end{array}

Jetzt kommt man auf die Idee, den Term innerhalb der Klammer mithilfe der 2. binomischen Formel zusammenzufassen: x^2-2x+1 = (x-1)^2. Die ersten beiden Summanden passen dafür ja schon gut. Ganz so einfach funktioniert es dann aber doch nicht, da das Quadrieren mit der binomischen Formel eine 1 als letzten Summanden liefert und nicht eine 3. Wir können die 3 in unserer Funktionsgleichung aber so "zerlegen", dass die benötigte 1 entsteht und dann noch ein Rest, der uns erst mal nicht weiter kümmert. In diesem Fall ist das 2, denn 3 = 1+2. Salopp formuliert: Die 1 ist ein bisschen wie "Wünsch Dir was" und die 2 der Korrekturterm, damit sich der Term nicht ändert. Das fügt man in die obere Rechnung ein und erhält:
\begin{array}{rclcc} f(x) &=& 2x^2-4x+6 & \vert & \text{ausklammern der 2 vor dem } x^2 \cr f(x) &=& 2(x^2-2x+3) \cr f(x) &=& 2(x^2-2x+1+2) \end{array}

Nun hat man alle drei Summanden zusammen, um die 2. binomische Formel von rechts nach links anwenden zu können. Die letzte 2 in der Funktionsgleichung bleibt dabei einfach stehen:
\begin{array}{rclcl} f(x) &=& 2x^2-4x+6 & \vert & \text{ausklammern der 2 vor dem } x^2 \cr f(x) &=& 2(x^2-2x+3) \cr f(x) &=& 2(x^2-2x+1+2) \cr f(x) &=& 2((x-1)^2+2) &\vert & \text{ausmultiplizieren der äußeren Klammer} \cr f(x) &=& 2(x-1)^2+4 \end{array}

Jetzt sind wir fertig und können den Scheitelpunkt wie gewohnt ablesen: Er liegt bei S(1\mid 4).

 

Weitere interessante Punkte einer Parabel

Nun haben wir schon viel zum Verlauf einer Parabel zusammengetragen. Neben dem Scheitelpunkt gibt es noch weitere interessante Punkte, die über den Verlauf einer Parabel Aufschluss geben. Diese Punkte wollen wir uns zum Abschluss des Kapitels noch kurz anschauen.

Betrachten wir nochmal die Funktion f(x)=\dfrac{1}{2}\left(x-\dfrac{3}{2}\right)^2-\dfrac{25}{4} von oben, deren Scheitelpunkt bei S\left(\dfrac{3}{2}\mid-\dfrac{25}{4}\right) liegt. Wie wir gesehen hatten, schneidet diese Parabel die x-Achse. Solche Schnittpunkte nennt man Nullstellen (Mehr dazu wird es im nächsten Kapitel geben ...). Uns interessiert für den Moment, wie man sie bei quadratischen Funktionen ermittelt: Da der Funktionsgraph bei einer Nullstelle die x-Achse schneidet, muss der zugehörige Funktionswert 0 sein. Wir müssen also die x-Werte der Funktion berechnen, für die das zutrifft. Man setzt nun 0 an der Stelle von f(x) in die Funktionsgleichung ein. Das nennt man "die Funktion nullsetzen". Jetzt müssen wir nur noch die entstandene quadratische Gleichung lösen:
\begin{array}{rclcl} \dfrac{1}{2}\left(x-\dfrac{3}{2}\right)^2-\dfrac{25}{4} &=& 0 \cr & ...& \cr \dfrac{1}{2}x^2-\dfrac{3}{2}x+2 &=& 0 &\vert& \cdot 2 \cr x^2-3x-4 &=& 0 &\vert& \text{p-q-Formel} \cr x_{1,2} &=& -\dfrac{-3}{2} \pm \sqrt{\left(\dfrac{-3}{2} \right)^2-\left(-4\right)} \cr\cr x_{1,2} &=& \dfrac{3}{2} \pm \sqrt{\dfrac{9}{4}+4} \cr\cr x_{1,2} &=& \dfrac{3}{2} \pm \sqrt{\dfrac{25}{4}} \cr\cr x_1 &=& \dfrac{3}{2}+\dfrac{5}{2} = \dfrac{8}{2} = 4 \cr\cr x_2 &=& \dfrac{3}{2} -\dfrac{5}{2} = -\dfrac{2}{2} = -1 \end{array}

Zur Probe setzen wir beide Stellen in f(x) ein und berechnen die Funktionswerte.
Für x_1:
\begin{array}{rcl} f(4) &=& \dfrac{1}{2}\left(4-\dfrac{3}{2}\right)^2-\dfrac{25}{8} \cr&=& \dfrac{1}{2}\left(\dfrac{5}{2}\right)^2-\dfrac{25}{8} \cr&=& \dfrac{1}{2}\cdot\dfrac{25}{4}-\dfrac{25}{8} \cr&=& 0 \end{array}

Für x_2:
\begin{array}{rcl}f(-1) &=& \dfrac{1}{2}\left(-1-\dfrac{3}{2}\right)^2-\dfrac{25}{8} \cr &=& \dfrac{1}{2}\left(-\dfrac{5}{2}\right)^2-\dfrac{25}{8} \cr&=& \dfrac{1}{2}\cdot\dfrac{25}{4}-\dfrac{25}{4} \cr&=& 0 \end{array}

Ergebnis: Beide Funktionswerte sind 0. f(x) hat also bei x_1=4 und x_2=-1 Nullstellen.

Nutzen wir gleich noch ein paar Erkenntnisse von den quadratischen Gleichungen und übertragen sie auf Funktionen: Es gibt quadratische Gleichungen, die keine, eine oder zwei Lösungen haben. Das bedeutet für die entsprechenden Funktionen, dass sie keine, eine oder zwei Nullstellen haben. Für alle drei Fälle haben Sie weiter oben in diesem Kapitel schon Beispiele gesehen.

 

Wie Sie vielleicht gemerkt haben, geht der Wechsel von quadratischen Funktionen zu quadratischen Gleichungen - und auch umgekehrt - manchmal schnell (Die letzte Rechnung könnte Ihnen zumindest teilweise bekannt vorgekommen sein ...). Dieser Zusammenhang zwischen Gleichungen und Funktionen wird Ihnen (auch in diesem Lernmodul) immer wieder begegnen. Das ist eine ziemlich nützliche Eigenschaft, die es ermöglicht, bei vielen Aufgaben von zwei Seiten zu argumentieren: rechnerisch und graphisch. Stimmen dann die rechnerischen und die graphischen Ergebnisse nicht überein, weiß man, dass ein Fehler im Lösungsweg enthalten ist - und kann sich auf die Suche machen, um den Fehler zu finden und die Lösung zu korrigieren.

Übersicht:

 

10.3 Quadratische Funktionen - Lösungen

1. Aufgabe

Der Übersichtlichkeit wegen sind die Graphen von Aufgabe 1a) in zwei Koordinatensysteme aufgeteilt. Bitte achten Sie bei den Grafiken auf die unterschiedliche Skalierung der Achsen!

a)

5 Parabeln im Koordinatensystem

 

5 Parabeln im Koordinatensystem

 

b)

1)
Der Graph von f_1(x) ist nach oben geöffnet und gestaucht, da der Koeffizient von x^2 ein positiver, echter Bruch ist. Zusätzlich wird von den Funktionswerten jeweils 12 subtrahiert, sodass der Scheitelpunkt bei S(0 \mid -12) liegt.


2)
Der Graph von f_2(x) ist nach unten geöffnet und gestreckt, da der Koeffizient von x^2 kleiner als -1 ist. Die Funktionsgleichung liegt in Scheitelpunktform vor, sodass man den Scheitelpunkt ablesen kann: S(3 \mid 0).


3)
Der Graph von f_3(x) ist nach oben geöffnet und weder gestreckt noch gestaucht (Normalparabel), da der Koeffizient von x^2 exakt 1 ist. Die Funktionsgleichung liegt in Scheitelpunktform vor, sodass man den Scheitelpunkt ablesen kann: S(5 \mid 5).


4)
Der Graph von f_4(x) ist nach oben geöffnet und gestreckt, da der Koeffizient von x^2 ein positiver, unechter Bruch ist. Die Funktionsgleichung liegt in Scheitelpunktform vor, sodass man den Scheitelpunkt ablesen kann: S\left(-10 \mid 2\right).


5)
Der Graph von f_5(x) ist nach oben geöffnet und weder gestreckt noch gestaucht (Normalparabel), da der Koeffizient von x^2 exakt 1 ist. Die Funktionsgleichung liegt in Scheitelpunktform vor, sodass man den Scheitelpunkt ablesen kann: S\left(-\dfrac{1}{2} \mid -4\right).

5 Parabeln im Koordinatensystem

 

6)
Der Graph von f_6(x) ist nach oben geöffnet und weder gestreckt noch gestaucht (Normalparabel), da der Koeffizient von x^2 exakt 1 ist. Zusätzlich wird von den Funktionswerten jeweils 8 subtrahiert, sodass der Scheitelpunkt bei S(0 \mid -8) liegt.


7)
Der Graph von f_7(x) ist nach unten geöffnet und gestaucht, da der Koeffizient von x^2 ein negativer, echter Bruch ist. Die Funktionsgleichung liegt in Scheitelpunktform vor, sodass man den Scheitelpunkt ablesen kann: S\left(9 \mid 0\right).


8)
Der Graph von f_8(x) ist nach unten geöffnet und weder gestreckt noch gestaucht (Normalparabel), da der Koeffizient von x^2 exakt -1 ist. Die Funktionsgleichung liegt in Scheitelpunktform vor, sodass man den Scheitelpunkt ablesen kann: S\left(-6 \mid 4\right).


9)
Der Graph von f_9(x) ist nach unten geöffnet und weder gestreckt noch gestaucht (Normalparabel), da der Koeffizient von x^2 exakt -1 ist. Zusätzlich wird zu den Funktionswerten jeweils 2 addiert, sodass der Scheitelpunkt bei S(0 \mid 2) liegt.


10)
Der Graph von f_{10}(x) ist nach oben geöffnet und weder gestreckt noch gestaucht (Normalparabel), da der Koeffizient von x^2 exakt 1 ist. Die Funktionsgleichung liegt in Scheitelpunktform vor, sodass man den Scheitelpunkt ablesen kann: S\left(3{,}5 \mid 0\right).

5 Parabeln im Koordinatensystem

 

2. Aufgabe

Vorüberlegungen zur Lösung:
Um zu erkennen, wie viele Nullstellen eine Funktion hat, setzen wir den Funktionsterm null und schauen uns die entstehende quadratische Gleichung an. Wichtig: Wir müssen die Gleichung (diesmal) nicht lösen, da ja nur gefragt ist, wie viele Nullstellen die Funktion hat und nicht, wo sie liegen. Dazu folgende Überlegungen:

  • Liegt die quadratische Gleichung in faktorisierter Form vor oder kann leicht in diese Form gebracht werden, liefert jeder Faktor eine Lösung. Die Funktion hat also zwei Nullstellen.
  • Bei quadratischen Gleichungen, die durch Umformen gelöst werden können, gibt es zwei Möglichkeiten (Beispielrechnungen siehe hier: 1. Fall und 3. Fall):
    • Haben der quadratische und der absolute Term verschiedene Vorzeichen, hat die Gleichung zwei Lösungen und die Funktion somit zwei Nullstellen.
    • Hat der quadratische Term das gleiche Vorzeichen wie der absolute, ist die Gleichung im Bereich der reellen Zahlen nicht lösbar. Die Funktion hat somit keine Nullstellen.
  • Ist in der Gleichung nur ein quadratischer Term enthalten, gibt es eine Nullstelle. 
  • Auch wenn der Funktionsterm in Form einer binomischen Formel vorliegt, hat die Funktion eine Nullstelle (Beispielrechnung siehe hier: 2. Fall).
  • Liegen Funktionsterme in Normal- oder allgemeiner Form vor, kommen wir mit diesen Überlegungen nicht weiter. Dann müssten wir doch rechnen ...

Parabeln dieser Art sind alle achsensymmetrisch (Genauer schauen wir uns dieses Thema im nächsten Kapitel an ...). Die Frage, die sich hier dahinter verbirgt, ist eigentlich "nur", ob die Parabel in x-Richtung verschoben ist.
Ist sie nicht in x-Richtung verschoben, liegt also ihr Scheitelpunkt bei x=0, ist die Parabel symmetrisch zur y-Achse. 
In allen anderen Fällen (egal, in welche Richtung und wie viel sie verschoben ist) ist die Parabel symmetrisch zu einer Parallelen der y-Achse. Das ist auch dann der Fall, wenn der Funktionsterm faktorisiert vorliegt und durch Ausmultiplizieren in allgemeine Form gebracht werden kann. Nur wenn die Nullstellen symmetrisch zur y-Achse liegen, also z. B. bei f(x)=(x-4)(x+4), ergibt sich keine Verschiebung in x-Richtung.


Nun die Lösungen der einzelnen Aufgaben:

1)
Da die entstandene Gleichung durch Umformen gelöst werden kann und die beiden Terme unterschiedliche Vorzeichen haben, hat die Funktion zwei Nullstellen.
Da der Koeffizient von x^2 kleiner als 0 ist, ist diese Parabel nach unten geöffnet.
Da der Funktionsterm in der Form f(x)=x^2+e vorliegt, ist der Graph nur nach oben verschoben. Damit ist er achsensymmetrisch zur y-Achse.
Da der Koeffizient von x^2 einen Betrag von 1 hat, ist diese Parabel genauso steil wie eine Normalparabel.


2)
Da die entstandene Gleichung in allgemeiner Form vorliegt, kann die Anzahl der Nullstellen ohne Rechnung nicht bestimmt werden.
Da der Koeffizient von x^2 größer als 0 ist, ist diese Parabel nach oben geöffnet.
Da der Funktionsterm in allgemeiner Form vorliegt, ist der Graph auch in x-Richtung verschoben. Damit ist er achsensymmetrisch zu einer Parallelen der y-Achse.
Da der Koeffizient von x^2 einen Betrag größer als 1 hat, ist diese Parabel steiler als eine Normalparabel.


3)
Da die entstandene Gleichung durch Ausklammern faktorisiert werden kann, hat die Funktion zwei Nullstellen.
Da der Koeffizient von x^2 kleiner als 0 ist, ist diese Parabel nach unten geöffnet.
Da der Funktionsterm in allgemeiner Form vorliegt, ist der Graph auch in x-Richtung verschoben. Damit ist er achsensymmetrisch zu einer Parallelen der y-Achse.
Da der Koeffizient von x^2 einen Betrag größer als 1 hat, ist diese Parabel steiler als eine Normalparabel.


4)
Da die entstandene Gleichung faktorisiert vorliegt, hat die Funktion zwei Nullstellen.
Da der Koeffizient von x^2 größer als 0 ist, ist diese Parabel nach oben geöffnet.
Da der Funktionsterm durch Ausmultiplizieren in allgemeine Form gebracht werden kann, ist der Graph auch in x-Richtung verschoben. Damit ist er achsensymmetrisch zu einer Parallelen der y-Achse.
Da der Koeffizient von x^2 einen Betrag größer als 1 hat, ist diese Parabel steiler als eine Normalparabel.


5)
Da die entstandene Gleichung nur einen quadratischen Term enthält, hat die Funktion eine Nullstelle.
Da der Koeffizient von x^2 kleiner als 0 ist, ist diese Parabel nach unten geöffnet.
Da der Funktionsterm in der Form f(x)=ax^2 vorliegt, ist der Graph nur gestreckt/gestaucht. Damit ist er achsensymmetrisch zur y-Achse.
Da der Koeffizient von x^2 einen Betrag kleiner als 1 hat, ist diese Parabel weniger steiler als eine Normalparabel.


6)
Da die entstandene Gleichung in allgemeiner Form vorliegt, kann die Anzahl der Nullstellen ohne Rechnung nicht bestimmt werden.
Da der Koeffizient von x^2 kleiner als 0 ist, ist diese Parabel nach unten geöffnet.
Da der Funktionsterm in allgemeiner Form vorliegt, ist der Graph auch in x-Richtung verschoben. Damit ist er achsensymmetrisch zu einer Parallelen der y-Achse.
Da der Koeffizient von x^2 einen Betrag kleiner als 1 hat, ist diese Parabel weniger steiler als eine Normalparabel.


7)
Da die entstandene Gleichung durch Umformen gelöst werden kann und die beiden Terme das gleiche Vorzeichen haben, hat die Funktion keine Nullstellen.
Da der Koeffizient von x^2 größer als 0 ist, ist diese Parabel nach oben geöffnet.
Da der Funktionsterm in der Form f(x)=ax^2+e vorliegt, ist der Graph nur gestreckt/gestaucht und nach oben verschoben. Damit ist er achsensymmetrisch zur y-Achse.
Da der Koeffizient von x^2 einen Betrag kleiner als 1 hat, ist diese Parabel weniger steiler als eine Normalparabel.


8)
Da die entstandene Gleichung in Normalform vorliegt, kann die Anzahl der Nullstellen ohne Rechnung nicht bestimmt werden.
Da der Koeffizient von x^2 größer als 0 ist, ist diese Parabel nach oben geöffnet.
Da der Funktionsterm in Normalform vorliegt, ist der Graph auch in x-Richtung verschoben. Damit ist er achsensymmetrisch zu einer Parallelen der y-Achse.
Da der Koeffizient von x^2 einen Betrag von 1 hat, ist diese Parabel genauso steil wie eine Normalparabel.


9)
Da die entstandene Gleichung durch Ausklammern faktorisiert werden kann, hat die Funktion zwei Nullstellen.
Da der Koeffizient von x^2 größer als 0 ist, ist diese Parabel nach oben geöffnet.
Da der Funktionsterm in allgemeiner Form vorliegt, ist der Graph auch in x-Richtung verschoben. Damit ist er achsensymmetrisch zu einer Parallelen der y-Achse.
Da der Koeffizient von x^2 einen Betrag kleiner als 1 hat, ist diese Parabel weniger steiler als eine Normalparabel.


10)
Da die entstandene Gleichung in allgemeiner Form vorliegt, kann die Anzahl der Nullstellen ohne Rechnung nicht bestimmt werden.
Da der Koeffizient von x^2 kleiner als 0 ist, ist diese Parabel nach unten geöffnet.
Da der Funktionsterm in allgemeiner Form vorliegt, ist der Graph auch in x-Richtung verschoben. Damit ist er achsensymmetrisch zu einer Parallelen der y-Achse.
Da der Koeffizient von x^2 einen Betrag von 1 hat, ist diese Parabel genauso steil wie eine Normalparabel.


11)
Da die entstandene Gleichung durch Umformen gelöst werden kann und die beiden Terme das gleiche Vorzeichen haben, hat die Funktion keine Nullstellen.
Da der Koeffizient von x^2 kleiner als 0 ist, ist diese Parabel nach unten geöffnet.
Da der Funktionsterm in der Form f(x)=ax^2-e vorliegt, ist der Graph nur gestreckt/gestaucht und nach unten verschoben. Damit ist er achsensymmetrisch zur y-Achse.
Da der Koeffizient von x^2 einen Betrag kleiner als 1 hat, ist diese Parabel weniger steiler als eine Normalparabel.


12)
Da die entstandene Gleichung in Form einer binomischen Formel vorliegt, hat die Funktion eine Nullstelle.
Da der Koeffizient von x^2 größer als 0 ist, ist diese Parabel nach oben geöffnet.
Da der Funktionsterm durch Ausmultiplizieren in allgemeine Form gebracht werden kann, ist der Graph auch in x-Richtung verschoben. Damit ist er achsensymmetrisch zu einer Parallelen der y-Achse.
Da der Koeffizient von x^2 einen Betrag von 1 hat, ist diese Parabel genauso steil wie eine Normalparabel.


13)
Da die entstandene Gleichung faktorisiert vorliegt, hat die Funktion zwei Nullstellen.
Da der Koeffizient von x^2 größer als 0 ist, ist diese Parabel nach oben geöffnet.
Da der Funktionsterm durch Ausmultiplizieren in allgemeine Form gebracht werden kann, ist der Graph auch in x-Richtung verschoben. Damit ist er achsensymmetrisch zu einer Parallelen der y-Achse.
Da der Koeffizient von x^2 einen Betrag größer als 1 hat, ist diese Parabel steiler als eine Normalparabel.


14)
Da die entstandene Gleichung in Form einer binomischen Formel vorliegt, hat die Funktion eine Nullstelle.
Da der Koeffizient von x^2 größer als 0 ist, ist diese Parabel nach oben geöffnet.
Da der Funktionsterm durch Ausmultiplizieren in allgemeine Form gebracht werden kann, ist der Graph auch in x-Richtung verschoben. Damit ist er achsensymmetrisch zu einer Parallelen der y-Achse.
Da der Koeffizient von x^2 einen Betrag kleiner als 1 hat, ist diese Parabel weniger steiler als eine Normalparabel.


15)
Da die entstandene Gleichung in allgemeiner Form vorliegt, kann die Anzahl der Nullstellen ohne Rechnung nicht bestimmt werden.
Da der Koeffizient von x^2 kleiner als 0 ist, ist diese Parabel nach unten geöffnet.
Da der Funktionsterm in allgemeiner Form vorliegt, ist der Graph auch in x-Richtung verschoben. Damit ist er achsensymmetrisch zu einer Parallelen der y-Achse.
Da der Koeffizient von x^2 einen Betrag kleiner als 1 hat, ist diese Parabel weniger steiler als eine Normalparabel.


16)
Da die entstandene Gleichung in allgemeiner Form vorliegt, kann die Anzahl der Nullstellen ohne Rechnung nicht bestimmt werden.
Da der Koeffizient von x^2 größer als 0 ist, ist diese Parabel nach oben geöffnet. Da der Funktionsterm in allgemeiner Form vorliegt, ist der Graph auch in x-Richtung verschoben. Damit ist er achsensymmetrisch zu einer Parallelen der y-Achse.
Da der Koeffizient von x^2 einen Betrag größer als 1 hat, ist diese Parabel steiler als eine Normalparabel.


17)
Da die entstandene Gleichung in allgemeiner Form vorliegt, kann die Anzahl der Nullstellen ohne Rechnung nicht bestimmt werden.
Da der Koeffizient von x^2 kleiner als 0 ist, ist diese Parabel nach unten geöffnet.
Da der Funktionsterm in allgemeiner Form vorliegt, ist der Graph auch in x-Richtung verschoben. Damit ist er achsensymmetrisch zu einer Parallelen der y-Achse.
Da der Koeffizient von x^2 einen Betrag größer als 1 hat, ist diese Parabel steiler als eine Normalparabel.


18)
Da die entstandene Gleichung in Form einer binomischen Formel vorliegt, hat die Funktion eine Nullstelle.
Da der Koeffizient von x^2 kleiner als 0 ist, ist diese Parabel nach unten geöffnet.
Da der Funktionsterm durch Ausmultiplizieren in allgemeine Form gebracht werden kann, ist der Graph auch in x-Richtung verschoben. Damit ist er achsensymmetrisch zu einer Parallelen der y-Achse.
Da der Koeffizient von x^2 einen Betrag größer als 1 hat, ist diese Parabel steiler als eine Normalparabel.


19)
Da die entstandene Gleichung in Normalform vorliegt, kann die Anzahl der Nullstellen ohne Rechnung nicht bestimmt werden.
Da der Koeffizient von x^2 größer als 0 ist, ist diese Parabel nach oben geöffnet.
Da der Funktionsterm in allgemeiner Form vorliegt, ist der Graph auch in x-Richtung verschoben. Damit ist er achsensymmetrisch zu einer Parallelen der y-Achse.
Da der Koeffizient von x^2 einen Betrag von 1 hat, ist diese Parabel genauso steil wie eine Normalparabel.


20)
Da die entstandene Gleichung durch Umformen gelöst werden kann und die beiden Terme das gleiche Vorzeichen haben, hat die Funktion keine Nullstellen.
Da der Koeffizient von x^2 kleiner als 0 ist, ist diese Parabel nach unten geöffnet.
Da der Funktionsterm in der Form f(x)=ax^2-e vorliegt, ist der Graph nur gestreckt/gestaucht und nach unten verschoben. Damit ist er achsensymmetrisch zur y-Achse.
Da der Koeffizient von x^2 einen Betrag größer als 1 hat, ist diese Parabel steiler als eine Normalparabel.

 

3. Aufgabe

Vorgehen:

  1. Ablesen der Koordinaten des Scheitelpunkts aus der Grafik und einsetzen der Koordinaten in die Scheitelpunktform der Parabel f(x)=a(x-d)^2+e, um d und e zu bestimmen
  2. Feststellen, ob die Parabel nach oben oder nach unten geöffnet ist, um zu bestimmen, ob a positiv oder negativ ist
  3. Ermitteln des Streckungs-/Stauchungsfaktors a
  4. Wenn man möchte: Ausmultiplizieren des Terms

Zur Vorbereitung ist es hilfreich, Aufgabe 1 und Aufgabe 6 bei den linearen Funktionen bearbeitet zu haben.


f_1(x) = 2x^2
Begründung:
Die Funktion hat ihren Scheitel im Punkt \left(0\mid 0\right), ist also weder nach links/rechts noch nach oben/unten verschoben.
Sie ist um den Faktor 2 gestreckt, weil sie nicht (wie eine Normalparabel) durch den Punkt \left(1\mid 1\right) , sondern durch den Punkt \left(2\mid 1\right) verläuft.

f_2(x) = (x-4)^2
Begründung:
Die Funktion hat ihren Scheitel im Punkt \left(4\mid 0\right) , ist also um 4 Einheiten nach rechts, aber nicht nach oben/unten verschoben.
Es handelt sich um eine Normalparabel, weil sie durch den Punkt \left(5\mid 1\right) verläuft.

f_3(x) = x^2+\dfrac{3}{2}
Begründung:
Die Funktion hat ihren Scheitel im Punkt \left(0\mid \frac{3}{2}\right), ist also nicht nach links/rechts, aber um 1{,}5 Einheiten nach oben verschoben.
Es handelt sich um eine Normalparabel, weil sie durch den Punkt \left(1\mid \frac{5}{2}\right) verläuft.

f_4(x) = (x+1)^2-3
Begründung:
Die Funktion hat ihren Scheitel im Punkt \left(-1\mid -3\right), ist also um 1 Einheit nach links und um 3 Einheiten nach unten verschoben.
Es handelt sich um eine Normalparabel, weil sie durch den Punkt \left(0\mid -2\right) verläuft.

f_5(x) = -\dfrac{1}{4}(x-2)^2+1
Begründung:
Die Funktion hat ihren Scheitel im Punkt \left(2 \mid 1\right), ist also um 2 Einheiten nach rechts und um 1 Einheit nach oben verschoben.
Sie ist um den Faktor \frac{1} {4} gestaucht, weil sie nicht (wie eine Normalparabel) durch den Punkt \left(4\mid -3\right), sondern durch den Punkt \left(4\mid 0\right) verläuft.
Das Minuszeichen nicht vergessen; schließlich ist die Parabel nach unten geöffnet.

 

4. Aufgabe

1)
a)
\begin{array}{rclll} f(7) &=& 7 \cdot 7^2-14 \cdot 7+49 &=& 294 \quad \rightarrow \quad P_1(7 \mid 294) \end{array}

b)
\begin{array}{rclcll} 105 &=& 7x^2-14x+49 & \vert & -105 \cr 0 &=& 7x^2-14x-56 & \vert & :7 \cr 0 &=& x^2-2x-8 &\vert& \text{p-q-Formel} \cr x_{1,2} &=& 1 \pm \sqrt{(-1)^2+8} \cr x_{1,2} &=& 1 \pm \sqrt{9} \cr\cr x_1 &=& 1+3 = 4 & \rightarrow \quad P_2(4 \mid 105) \cr x_2 &=& 1-3 = -2 & \rightarrow \quad P_3(-2 \mid 105) \end{array}


2)
a)
\begin{array}{rclll} f(-11{,}5) &=& -(-11{,}5)^2-24 \cdot (-11{,}5)+2 &=& 145{,}75 \quad \rightarrow \quad P_1(-11{,}5 \mid 145{,}75) \end{array}

b)
\begin{array}{rclcll} 25 &=& -x^2-24x+2 &\vert & -25 \cr 0 &=& -x^2-24x-23 &\vert & \cdot(-1) \cr 0 &=& x^2+24x+23 &\vert& \text{p-q-Formel} \cr x_{1,2} &=& -12 \pm \sqrt{12^2-23} \cr x_{1,2} &=& -12 \pm \sqrt{121} \cr\cr x_1 &=& -12+11 = -1 & \rightarrow \quad P_2(-1 \mid 25) \cr x_2 &=& -12-11 = - 23 & \rightarrow \quad P_3(-23 \mid 25) \end{array}


3)
a)
\begin{array}{rclll} f(8) &=& 32 \cdot 8^2-8 &=& 2040 \quad \rightarrow \quad P_1(8 \mid 2.040) \end{array}

b)
\begin{array}{rclcl} -4 &=& 32x^2-x &\vert & +4 \cr 0 &=& 32x^2-x+4 &\vert & :32 \cr 0 &=& x^2-\dfrac{1}{32}x+\dfrac{1}{8} &\vert& \text{p-q-Formel} \cr x_{1,2} &=& \dfrac{1}{64} \pm \sqrt{\left(-\dfrac{1}{64}\right)^2-\dfrac{1}{8}} \cr x_{1,2} &=& \dfrac{1}{64} \pm \sqrt{-\dfrac{511}{4.096}}\end{array}

Da aus negativen reellen Zahlen keine Wurzeln mit geraden Wurzelexponenten gezogen werden können, hat diese Gleichung keine Lösung: Das bedeutet, dass die Funktion nirgends den Funktionswert -4 annimmt.


4)
a)
\begin{array}{rclll} f(6) &=& -3 \cdot 6^2+7 \cdot 6 &=& -66 \quad \rightarrow \quad P_1(6 \mid -66) \end{array}

b)
\begin{array}{rclcll} 4 &=& -3x^2+7x &\vert & -4 \cr 0 &=& -3x^2+7x-4 &\vert & :(-3) \cr 0 &=& x^2-\dfrac{7}{3}x+\dfrac{4}{3} &\vert& \text{p-q-Formel} \cr x_{1,2} &=& \dfrac{7}{6} \pm \sqrt{\left(-\dfrac{7}{6}\right)^2-\dfrac{4}{3}} \cr x_{1,2} &=& \dfrac{7}{6} \pm \sqrt{\dfrac{1}{36}} \cr\cr x_1 &=& \dfrac{7}{6}+\dfrac{1}{6} = \dfrac{4}{3} & \rightarrow \quad P_2\left(\dfrac{4}{3} \mid 4\right) \cr x_2 &=& \dfrac{7}{6}-\dfrac{1}{6} = 1 & \rightarrow \quad P_3(1 \mid 4) \end{array}


5)
a)
\begin{array}{rclll} g\left(-\dfrac{5}{6}\right) &=& 6 \cdot \left(-\dfrac{5}{6}\right)^2-17 &=& -\dfrac{77}{6} \quad \rightarrow \quad P_1\left(-\dfrac{5}{6} \mid -\dfrac{77}{6}\right) \end{array}

b)
\begin{array}{rclcl} 0 &=& 6x^2-17 &\vert & :6 \cr 0 &=& x^2-\dfrac{17}{6} &\vert & -x^2 \cr -x^2 &=& -\dfrac{17}{6} &\vert & :(-1) \cr x^2 &=& \dfrac{17}{6} &\vert & \pm\sqrt{} \cr\cr x_1 &=& -\sqrt{\dfrac{17}{6}} & \rightarrow \quad P_2\left(-\sqrt{\dfrac{17}{6}} \mid 0\right) \cr x_2 &=& \sqrt{\dfrac{17}{6}} & \rightarrow \quad P_3\left(\sqrt{\dfrac{17}{6}} \mid 0\right) \end{array}


6)
a)
\begin{array}{rclll} f(17) &=& 2(17-1)^2-\dfrac{4}{3} &=& \dfrac{1.532}{3} \quad \rightarrow \quad P_1\left(17 \mid \dfrac{1.532}{3}\right) \end{array}

b)
\begin{array}{rclcll} -\dfrac{4}{3} &=& 2(x-1)^2-\dfrac{4}{3} &\vert & +\dfrac{4}{3} \cr 0 &=& 2(x-1)^2 &\vert & :2 \cr 0 &=& (x-1)^2 &\vert & \sqrt{} \cr 0 &=& x-1 &\vert& +1 \cr 1 &=& x & & \rightarrow \quad P_2\left(1 \mid -\dfrac{4}{3}\right) \end{array}


7)
a)
\begin{array}{rclll} f\left(\dfrac{15}{4}\right) &=& \left(-3+\dfrac{15}{4}\right)^2-6 &=& -\dfrac{87}{16} \quad \rightarrow \quad P_1\left(\dfrac{15}{4} \mid -\dfrac{87}{16}\right) \end{array}

b)
\begin{array}{rclcl} -43 &=& (-3+a)^2-6 &\vert & +43 \cr 0 &=& (3-a)^2+37 \cr 0 &=& a^2-6a+46 &\vert& \text{p-q-Formel} \cr a_{1,2} &=& 3 \pm \sqrt{(-3)^2-46} \cr a_{1,2} &=& 3 \pm \sqrt{-37}\end{array}

Da aus negativen reellen Zahlen keine Wurzeln mit geraden Wurzelexponenten gezogen werden können, hat diese Gleichung keine Lösung: Das bedeutet, dass die Funktion nirgends den Funktionswert -43 annimmt.


8)
a)
\begin{array}{rclll} f(5) &=& (4\cdot 5+12)^2+8 &=& 1.032 \quad \rightarrow \quad P_1(5 \mid 1.032) \end{array}

b)
\begin{array}{rclll} 8 &=& (4x+12)^2+8 &\vert & -8 \cr 0 &=& (4x+12)^2 &\vert & \sqrt{} \cr 0 &=& 4x+12 &\vert & -4x \cr -4x &=& 12 &\vert & :(-4) \cr x &=& -3 & & \rightarrow \quad P_2(-3 \mid 8) \end{array}


9)
a)
\begin{array}{rclll} f(18) &=& \dfrac{1}{10}(18-7)^2 &=& \dfrac{121}{10} \quad \rightarrow \quad P_1\left(18 \mid \dfrac{121}{10}\right) \end{array}

b)
\begin{array}{rclll} 0 &=& \dfrac{1}{10}(x-7)^2 &\vert & \cdot 10 \cr 0 &=& (x-7)^2 &\vert& \sqrt{} \cr 0 &=& x-7 &\vert & +7 \cr 7 &=& x & & \rightarrow \quad P_2(7 \mid 0) \end{array}


10)
a)
\begin{array}{rclll} f(0{,}5) &=& 100 \cdot 0{,}5^2-50 \cdot 0{,}5+25 &=& 25 \quad \rightarrow \quad P_1(0{,}5 \mid 25) \end{array}

b)
\begin{array}{rclcll} 775 &=& 100x^2-50x+25 &\vert & -775 \cr 0 &=& 100x^2-50x-750 &\vert & :100 \cr 0 &=& x^2-\dfrac{1}{2}x-\dfrac{15}{2} &\vert& \text{p-q-Formel} \cr x_{1,2} &=& \dfrac{1}{4} \pm \sqrt{\left(-\dfrac{1}{4}\right)^2+\dfrac{15}{2}} \cr x_{1,2} &=& \dfrac{1}{4} \pm \sqrt{\dfrac{121}{16}} \cr\cr x_1 &=& \dfrac{1}{4}+\dfrac{11}{4} = 3 & \rightarrow \quad P_2(3 \mid 775) \cr x_2 &=& \dfrac{1}{4}-\dfrac{11}{4} = -\dfrac{5}{2} & \rightarrow \quad P_3\left(-\dfrac{5}{2} \mid 775\right) \end{array}

 

5. Aufgabe

1)
\begin{array}{rcl} f(x) &=& x^2-6x+16 \cr &=& x^2-6x+9+7 \cr &=& x^2-2 \cdot 3x+3^2+7 \cr &=& (x-3)^2+7 \end{array}

Der Scheitelpunkt liegt bei S(3\mid 7).


2)
\begin{array}{rcl} f(x) &=& 2x^2-4x+2 \cr &=& 2(x^2-2x+1) \cr &=& 2(x^2-2 \cdot 1x+1^2) \cr &=& 2(x-1)^2 \end{array}

Der Scheitelpunkt liegt bei S(1\mid 0).


3)
\begin{array}{rcl} f(x) &=& -13x^2-8 \cr &=& -13(x-0)^2-8 \end{array}

Der Scheitelpunkt liegt bei S(0 \mid -8). Die Parabel ist nach unten geöffnet.


4)
\begin{array}{rcl} f(x) &=& \dfrac{1}{4}x^2-6x+32 \cr \cr &=& \dfrac{1}{4}(x^2-24x+128) \cr \cr &=& \dfrac{1}{4}(x^2-24x+144-16) \cr \cr &=& \dfrac{1}{4}(x^2-2 \cdot 12x+12^2-16) \cr \cr &=& \dfrac{1}{4}((x-12)^2 -16) \cr \cr &=& \dfrac{1}{4}(x-12)^2-4 \end{array}

Der Scheitelpunkt liegt bei S(12 \mid -4).


5)
\begin{array}{rcl} f(x) &=& -11x^2-44x-34 \cr &=& -11 \left(x^2+4x+\dfrac{34}{11} \right) \cr &=& -11 \left(x^2+2 \cdot 2x +4- \dfrac{10}{11} \right) \cr &=& -11 \left(x^2+2 \cdot 2x +2^2- \dfrac{10}{11} \right) \cr &=& -11 \left((x+2)^2- \dfrac{10}{11} \right) \cr &=& -11(x+2)^2+10 \end{array}

Der Scheitelpunkt liegt bei S(-2 \mid 10).

6)
\begin{array}{rcl} f(x) &=& -9x^2 -90x -225 \cr &=& -9(x^2+10x+25) \cr &=& -9(x^2+2 \cdot 5x+ 5^2) \cr &=& -9(x+5)^2 \end{array}

Der Scheitelpunkt liegt bei S(-5 \mid 0).


7)
\begin{array}{rcl} f(x) &=& 4x^2 -4x+3 \cr\cr &=& 4 \left(x^2- 1x+ \dfrac{3}{4} \right) \cr\cr &=& 4 \left(x^2 -1x+ \dfrac14 +\dfrac12 \right) \cr\cr &=& 4 \left(x^2 - 2 \cdot \dfrac12 x + \left(\dfrac12 \right)^2 + \dfrac12 \right) \cr\cr &=& 4 \left(\left(x- \dfrac12 \right)^2 +\dfrac{1}{2}\right) \cr\cr &=& 4 \left(x- \dfrac12 \right)^2 +2 \end{array}

Der Scheitelpunkt liegt bei S \left(\dfrac12 \mid 2 \right).


8)
\begin{array}{rcl} f(x) &=& 3x^2 -4x - \dfrac{14}{3} \cr\cr &=& 3 \left(x^2 - \dfrac{4}{3} x- \dfrac{14}{9} \right) \cr\cr &=& 3 \left(x^2 - 2 \cdot \dfrac{2}{3}x + \left(\dfrac{2}{3} \right)^2 - \left(\dfrac{2}{3} \right)^2 - \dfrac{14}{9} \right) \cr\cr &=& 3 \left(x^2-2\cdot \dfrac{2}{3}x+ \left(\dfrac{2}{3} \right)^2 -2 \right) \cr\cr &=& 3 \left(x-\dfrac{2}{3} \right)^2 -6 \cr\cr &=& 3 \left(\left(x-\dfrac{2}{3} \right)^2 -2\right) \end{array}

Der Scheitelpunkt liegt bei S \left(\dfrac{2}{3} \mid -6 \right).


9)
\begin{array}{rcl} f(x) &=& \dfrac12 x^2 +4x-7 \cr\cr &=& \dfrac12 \left(x^2 +8x-14 \right) \cr\cr &=& \dfrac12 (x^2 +2 \cdot 4x +4^2 -4^2 -14) \cr\cr &=& \dfrac12 (x^2 +2 \cdot 4x+4^2-30) \cr\cr &=& \dfrac12 (x+4)^2 -15 \cr\cr &=& \dfrac12\left( (x+4)^2 -30\right) \end{array}

Der Scheitelpunkt liegt bei S(-4 \mid -15).


10)
\begin{array}{rcl} f(x) &=& -x^2+26x-99 \cr &=& -(x^2-26x+99) \cr &=& -(x^2-2 \cdot 13x +13^2 -13^2 +99) \cr &=& -(x^2 -2 \cdot 13x +13^2 -70) \cr &=& -(x-13)^2 +70 \cr &=& -\left((x-13)^2 -70\right) \end{array}

Der Scheitelpunkt liegt bei S(13 \mid 70).

 

6. Aufgabe

Gerade und Parabel:
1. Möglichkeit: Die Gerade und die Parabel laufen aneinander vorbei. Es gibt also keinen Schnittpunkt, z. B. in der unteren Grafik g(x) und f_1(x).

2. Möglichkeit: Die Gerade und die Parabel berühren sich. Es gibt also einen Schnittpunkt, z. B. in der unteren Grafik g(x) und f_2(x).

3. Möglichkeit: Die Gerade und die Parabel schneiden sich in zwei Punkten, z. B. in der unteren Grafik g(x) und f_3(x).

3 Parabeln und 1 Gerade im Koordinatensystem


Zwei Parabeln:
1. Möglichkeit: Die beiden Parabeln laufen aneinander vorbei. Es gibt also keinen Schnittpunkt, z. B. in der unteren Grafik g(x) und f_1(x).

2. Möglichkeit: Die beiden Parabeln berühren sich. Es gibt also einen Schnittpunkt, z. B. in der unteren Grafik g(x) und f_2(x).

3. Möglichkeit: Die beiden Parabeln schneiden sich in zwei Punkten, z. B. in der unteren Grafik g(x) und f_3(x).

4 Parabeln im Koordinatensystem
 

7. Aufgabe

Vorgehen: Man setzt die beiden Funktionsterme gleich, da die Funktionswerte beider Funktionen gleich sein müssen, damit ein Schnittpunkt vorliegt. Es ergibt sich eine quadratische Gleichung, die (nach etwas Umformen) z. B. mithilfe der p-q-Formel gelöst werden kann.
Anschließend setzt man den ermittelten x-Wert in eine der beiden Funktionsgleichung ein, um den zugehörigen y-Wert zu ermitteln. Es ist egal, in welche Funktionsgleichung der x-Wert eingesetzt wird, da der Punkt ja bei beiden Funktionen identisch sein muss, sonst würden sie sich dort ja nicht schneiden ... Üblicherweise nimmt man daher die einfachere Funktion. Die andere Funktion kann zur Probe genutzt werden.

1)
x-Werte ausrechnen:
\begin{array}{rclcl} 2x^2-4x+2 &=& \dfrac{1}{2}x+1 & \vert & -\dfrac{1}{2}x -1 \cr 2x^2-\dfrac{9}{2}x+1 &=& 0 & \vert & :2 \cr x^2-\dfrac{9}{4}x+\dfrac{1}{2} &=& 0 &\vert& \text{p-q-Formel} \cr x_{1,2} &=& \dfrac{9}{8} \pm \sqrt{\left(-\dfrac{9}{8}\right)^2-\dfrac{32}{64}} \cr x_{1,2} &=& \dfrac{9}{8} \pm \sqrt{\dfrac{49}{64}} \cr \cr x_1 &=& \dfrac{9}{8}+\dfrac{7}{8} = 2 \cr x_2 &=& \dfrac{9}{8}-\dfrac{7}{8} = \dfrac{1}{4} \end{array}

y-Werte ausrechnen:
\begin{array}{rcl}g\left(x_1\right) &=& g\left(2\right) \cr &=& \dfrac{1}{2}\cdot 2+1 \cr &=& 2 \cr\cr g\left(x_2\right) &=& g\left(\dfrac{1}{4}\right) \cr &=& \dfrac{1}{2}\cdot \dfrac{1}{4}+1 \cr &=& \dfrac{9}{8}\end{array}

Die Schnittpunkte von f und g liegen also bei S_1 \left( 2 \mid 2 \right) und S_2 \left( \dfrac{1}{4} \mid \dfrac{9}{8} \right).


2)
x-Werte ausrechnen:
\begin{array}{rclcl} -9x^2+8x-1 &=& -10x^2-\dfrac{6}{5}x+7 & \vert & +10x^2+\dfrac{6}{5}x-7 \cr x^2+\dfrac{46}{5}x-8 &=& 0 &\vert& \text{p-q-Formel} \cr x_{1,2} &=& -\dfrac{46}{10} \pm \sqrt{\left(\dfrac{46}{10}\right)^2-(-8)} \cr x_{1,2} &=& -\dfrac{46}{10} \pm \sqrt{\dfrac{2.916}{100}} \cr \cr x_1 &=& -\dfrac{46}{10} + \dfrac{54}{10} = \dfrac{4}{5} \cr x_2 &=& -\dfrac{46}{10} - \dfrac{54}{10} = -10 \end{array}

y-Werte ausrechnen:
\begin{array}{rcl}g\left(x_1\right) &=& g\left(\dfrac{4}{5}\right) \cr&=& -9\cdot\left(\dfrac{4}{5}\right)^2+8\cdot\dfrac{4}{5}-1\cr&=& -\dfrac{9}{25} \cr\cr g\left(x_2\right) &=& g\left(-10\right) \cr&=& -9\cdot\left(-10\right)^2+8\cdot\left(-10\right)-1\cr&=& -981\end{array}

Die Schnittpunkte von f und g liegen also bei S_1 \left( -10 \mid -981 \right) und S_2 \left( \dfrac{4}{5} \mid -\dfrac{9}{25} \right).


3)
x-Werte ausrechnen:
\begin{array}{rclcl} \dfrac{3}{2}x^2+5x+15 &=& x^2-x-20 & \vert & -x^2+x+20 \cr \dfrac{1}{2}x^2+6x+35 &=& 0 & \vert & \cdot 2 \cr x^2+12x+70 &=& 0 &\vert& \text{p-q-Formel} \cr x_{1,2} &=& -\dfrac{12}{2} \pm \sqrt{\left(\dfrac{12}{2}\right)^2-70} \cr x_{1,2} &=& -6 \pm \sqrt{-34} \end{array}

Da aus negativen reellen Zahlen keine Wurzeln mit geraden Wurzelexponenten gezogen werden können, hat diese Gleichung keine Lösung: f und g haben also keine Schnittpunkte.


4)
x-Werte ausrechnen:
\begin{array}{crclcl} & 4x^2-11 &=& 12x-11 & \vert & -12x+11 \cr & 4x^2-12x &=& 0 \cr & 4x(x-3) &=& 0 & \vert & :4 \cr & x(x-3) &=& 0 & \vert & \text{Satz vom Nullprodukt} \cr \text{Faktor 1:} & x_1 &=& 0 \cr\cr \text{Faktor 2:} & x_2-3 &=& 0 & \vert & +3 \cr & x_2 &=& 3 \end{array}

y-Werte ausrechnen:
\begin{array}{rcl}g\left(x_1\right) &=& g\left(0\right) \cr&=& 12\cdot 0-11 \cr&=& -11 \cr\cr g\left(x_2\right) &=& g\left(\dfrac{1}{2}\right) \cr&=& 12\cdot 3-11 \cr&=& 25\end{array}

Die Schnittpunkte von f und g liegen also bei S_1 \left( 0 \mid -11 \right) und S_2 \left( 3 \mid 25 \right).


5)
x-Werte ausrechnen:
\begin{array}{rclcl} -3x^2-100x+300 &=& -15x^2+20x &\vert & 15x^2-20x \cr 12x^2-120x+300 &=& 0 & \vert & :12 \cr x^2-10x+25 &=& 0 &\vert& \text{p-q-Formel} \cr x_{1,2} &=& \dfrac{10}{2} \pm \sqrt{\left(-\dfrac{10}{2}\right)^2-25} \cr x_{1,2} &=& 5 \pm \sqrt{0} \cr \cr x_1 &=& 5 \end{array}

y-Werte ausrechnen:
\begin{array}{rcl}g\left(x_1\right) &=& g\left(5\right) \cr&=& -15\cdot 5^2+20 \cdot 5 \cr&=& -275\end{array}

Der Schnittpunkt von f und g liegt also bei S_1 \left( 5 \mid -275 \right) .


6)
x-Werte ausrechnen:
\begin{array}{rclcll}-5x^{2} + x + 2 &=& -x^{2} + x + 1 &\vert & +5x^{2} -x -2 \cr 0 &=& 4x^2 - 1 &\vert & +1 \cr 1 &=& 4x^2 &\vert & :4 \cr \dfrac{1}{4} &=& x^2 &\vert & \pm\sqrt{} \cr x_1 &=& \dfrac{1}{2} \cr x_2 &=& -\dfrac{1}{2} \end{array}

y-Werte ausrechnen:
\begin{array}{rcl}g\left(x_1\right) &=& g\left(\dfrac{1}{2}\right) \cr&=& -1\cdot\left(\dfrac{1}{2}\right)^2 + 1\cdot\dfrac{1}{2} + 1 \cr &=& \dfrac{5}{4} \cr\cr g\left(x_2\right) &=& g\left(-\dfrac{1}{2}\right) \cr&=& -1\cdot\left(-\dfrac{1}{2}\right)^2 + 1\cdot\left(-\dfrac{1}{2}\right) + 1 \cr &=& \dfrac{1}{4} \end{array}

Die Schnittpunkte von f und g liegen also bei S_1 \left(\dfrac{1}{2} \mid \dfrac{5}{4}\right) und S_2 \left(-\dfrac{1}{2} \mid \dfrac{1}{4}\right).


7)
x-Werte ausrechnen:
\begin{array}{crclcll}& -x + 4 &=& -x^2 + 4{,}5x + 4 &\vert & +x^2-4{,}5x-4 \cr & x^2-5{,}5x &=& 0 \cr & x\left(x-5{,}5\right) &=& 0 &\vert& \text{Satz vom Nullprodukt} \cr \text{Faktor 1:} & x_1 &=& 0 \cr\cr \text{Faktor 2:} & x_2-5{,}5 &=& 0 &\vert& +5{,}5 \cr & x_2 &=& 5{,}5 \end{array}

y-Werte ausrechnen:
\begin{array}{rcl}f\left(x_1\right) &=& f\left(0\right) \cr&=& -0 + 4 \cr &=& 4 \cr\cr f\left(x_2\right) &=& f\left(5{,}5\right) \cr&=& -5{,}5+ 4 \cr &=& -1{,}5 \end{array}

Die Schnittpunkte von f und g liegen also bei S_1 \left(0 \mid 4\right) und S_2 \left(5{,}5 \mid -1{,}5\right).


8)
x-Werte ausrechnen:
\begin{array}{rclcll}1{,}5x^2 + 3x &=&1{,}5x^2 + 10x + 14 &\vert & -1{,}5x^2 -3x \cr0 &=& 7x + 14 &\vert & -14 \cr-14 &=& 7x &\vert & :7 \cr x_1 &=& -2 \end{array}

y-Wert ausrechnen:
\begin{array}{rcl}f\left(x_1\right) &=& f\left(-2\right) \cr &=& 1{,}5\cdot\left(-2\right)^2 + 3\cdot\left(-2\right) \cr &=& 0 \end{array}

Der Schnittpunkt von f und g liegt also bei S_1 \left(-2 \mid 0\right).


9)
x-Werte ausrechnen:
\begin{array}{rclcll}3x + 7 &=& -x^2 + 10x + 1 &\vert & +x^2-10x-1 \cr x^2-7x+6 &=& 0 &\vert& \text{p-q-Formel} \cr x_{1,2} &=& \dfrac{7}{2} \pm \sqrt{\left(-\dfrac{7}{2}\right)^2- 6} \cr x_{1,2} &=& \dfrac{7}{2} \pm \sqrt{\dfrac{25}{4}} \cr\cr x_1 &=& \dfrac{7}{2} + \dfrac{5}{2} = 6 \cr x_2 &=& \dfrac{7}{2} - \dfrac{5}{2} = 1 \end{array}

y-Werte ausrechnen:
\begin{array}{rcl}f\left(x_1\right) &=& f\left(6\right) \cr &=& 3\cdot 6 + 7 \cr &=& 25 \cr\cr f\left(x_2\right) &=& f\left(1\right) \cr &=& 3\cdot 1 + 7 \cr &=& 10 \end{array}

Die Schnittpunkte von f und g liegen also bei S_1 \left(1 \mid 10\right) und S_2 \left(6 \mid 25\right).


10)
x-Werte ausrechnen:
\begin{array}{crclcll} & \dfrac12 x^2-13x-15 &=& -8x-15 & \vert & (+8x+15) \cr & \dfrac12 x^2 -5x &=& 0 & \vert & \cdot 2 \cr & x^2-10x &=& 0 & \vert & \text{Satz vom Nullprodukt} \cr & x(x-10) &=& 0 \cr \text{Faktor 1:} & x_1 &=& 0 \cr\cr \text{Faktor 2:} & x_2-10 &=& 0 &\vert& +10 \cr & x_2 &=& 10 \end{array}

y-Werte ausrechnen:
\begin{array}{rcl} g(x_1) &=& g(0) \cr &=& -8 \cdot 0 -15 \cr &=& -15\cr\cr g(x_2) &=& g(10) \cr &=& -8 \cdot 10 -15 \cr &=& -95 \end{array}

Die Schnittpunkte von f und g liegen also bei S_1 \left( 0 \mid -15 \right) und S_2 \left( 10 \mid -95 \right).

 

8. Aufgabe

1)
Schnittpunkt mit der x-Achse:
\begin{array}{rclcl} 0 &=& \dfrac{17}{3}\cdot (x^2-4x-221) &\vert& :\dfrac{17}{3} \cr\cr 0 &=& x^2-4x-221 &\vert& \text{p-q-Formel} \cr x_{1,2} &=& 2 \pm \sqrt{\left(-2\right)^2+221} \cr x_{1,2} &=& 2 \pm \sqrt{225} \cr\cr x_1 &=& 2+15 = 17 \cr x_2 &=& 2-15 = -13 \end{array}

Die Schnittpunkte mit der x-Achse sind also S_{x1}\left(17 \mid 0\right) und S_{x2}\left(-13 \mid 0\right).

Schnittpunkt mit der y-Achse:
\begin{array}{rclcll}f(0) &=& \dfrac{17}{3}\cdot\left(0^2-4\cdot 0-221\right)\\f(0) &=& -\dfrac{3.757}{3} \approx -1.252{,}33\end{array}

Der Schnittpunkt mit der y-Achse ist also S_{y}\left(0 \mid -\dfrac{3.757}{3}\right).


2)
Schnittpunkt mit der a-Achse:
\begin{array}{crclcl} & 0 &=& -\dfrac{1}{4}a^2+a \cr & 0 &=& a \left(-\dfrac{1}{4}a+1\right) & \vert & \text{Satz vom Nullprodukt} \cr\cr \text{Faktor 1:} & a_1 &=& 0 \cr\cr \text{Faktor 2:} & 0 &=& -\dfrac{1}{4}a_2+1 &\vert& -1 \cr & -1 &=& -\dfrac{1}{4}a_2 &\vert& : \left(-\dfrac{1}{4}\right) \cr & a_2 &=& 4 \end{array}

Die Schnittpunkte mit der a-Achse sind also S_{a1}\left(0 \mid 0\right) und S_{a2}\left(4 \mid 0\right).

Schnittpunkt mit der y-Achse:
\begin{array}{rclcll}f(0) &=& -\dfrac{1}{4}\cdot 0^2+0\\f(0) &=& 0\end{array}

Der Schnittpunkt mit der y-Achse ist also S_{y}\left(0 \mid 0\right).


3)
Schnittpunkt mit der x-Achse:
\begin{array}{rclcl} 0 &=& \left(x+\sqrt{2}\right)^2 &\vert& \sqrt{} \cr 0 &=& x+\sqrt{2} &\vert& -\sqrt{2} \cr x &=& -\sqrt{2} \end{array}

Der Schnittpunkt mit der x-Achse ist also S_{x}\left(-\sqrt{2} \mid 0\right).

Bemerkung: Da auf der linken Seite 0 steht, dürfen wir hier - ausnahmsweise - beim Wurzelziehen auf das \pm verzichten. 0 ist ja weder positiv noch negativ.

Schnittpunkt mit der y-Achse:
\begin{array}{rclcll}f(0) &=& \left(0+\sqrt{2}\right)^2\\f(0) &=& 2\end{array}

Der Schnittpunkt mit der y-Achse ist also S_{y}\left(0\;\vert\; 2\right).


4)
Schnittpunkt mit der x-Achse:
\begin{array}{rclcl} 0 &=& -\dfrac{3}{4}x^2+2 &\vert& -2 \cr\cr -2 &=& -\dfrac{3}{4}x^2 &\vert& : \left(-\dfrac{3}{4}\right) \cr\cr x^2 &=& \dfrac{8}{3} &\vert& \pm\sqrt{} \cr\cr x &=& \pm \sqrt{\dfrac{8}{3}} \end{array}

Die Schnittpunkte mit der x-Achse sind also S_{x1}\left(\sqrt{\dfrac{8}{3}} \mid 0\right) und S_{x2}\left(-\sqrt{\dfrac{8}{3}} \mid 0\right).

Schnittpunkt mit der y-Achse:
\begin{array}{rclcll}f(0) &=& -\dfrac{3}{4}\cdot 0^2+2\\f(0) &=& 2\end{array}

Der Schnittpunkt mit der y-Achse ist also S_{y}\left(0 \mid 2\right).


5)
Schnittpunkt mit der x-Achse:
\begin{array}{rclcl} 0 &=& 3x^2-6x-1 &\vert& :3 \cr 0 &=& x^2-2x-\dfrac{1}{3} &\vert& \text{p-q-Formel} \cr\cr x_{1,2} &=& 1\pm \sqrt{(-1)^2+\dfrac{1}{3}} \cr\cr x_1 &=& 1+\sqrt{\dfrac{4}{3}} \approx 2{,}15 \cr\cr x_2 &=& 1-\sqrt{\dfrac{4}{3}} \approx -0{,}15 \end{array}

Die Schnittpunkte mit der x-Achse sind also S_{x1}\left(1+\sqrt{\dfrac{4}{3}} \mid 0\right) und S_{x2}\left(1-\sqrt{\dfrac{4}{3}} \mid 0\right).

Schnittpunkt mit der y-Achse:
\begin{array}{rclcll}f(0) &=& 3\cdot 0^2-6\cdot 0 -1 \\f(0) &=& -1\end{array}

Der Schnittpunkt mit der y-Achse ist also S_{y}\left(0 \mid -1\right).


6)
Schnittpunkt mit der z-Achse:
\begin{array}{rclcl} 0 &=& -3z^2+22z-42 &\vert& :(-3) \cr 0 &=& z^2-\dfrac{22}{3}z+14 &\vert& \text{p-q-Formel} \cr\cr z_{1,2} &=& \dfrac{11}{3} \pm \sqrt{\left(-\dfrac{11}{3}\right)^2-14} \cr z_{1,2} &=& \dfrac{11}{3}\pm\sqrt{-\dfrac{5}{9}} \end{array}

Da aus negativen reellen Zahlen keine Wurzeln mit geraden Wurzelexponenten gezogen werden können, hat diese Funktion keine Schnittpunkte mit der z-Achse.

Schnittpunkt mit der y-Achse:
\begin{array}{rclcll}f(0) &=& 3\cdot 0^2+22\cdot 0-42 \\f(0) &=& -42\end{array}

Der Schnittpunkt mit der y-Achse ist also S_{y}\left(0 \mid -42\right).


7)
Schnittpunkt mit der t-Achse:
\begin{array}{rclcl} 0 &=& \left(\dfrac{t+17}{\sqrt{2}}\right)^2 &\vert& \sqrt{} \cr\cr 0 &=& \dfrac{t+17}{\sqrt{2}} &\vert& \cdot \sqrt{2} \cr 0 &=& t+17 &\vert& -17 \cr t &=& -17 \end{array}

Der Schnittpunkt mit der t-Achse ist also S_{t}\left(-17\;\vert\; 0\right).

Bemerkung: Da auf der linken Seite 0 steht, dürfen wir hier - ausnahmsweise - beim Wurzelziehen auf das \pm verzichten.

Schnittpunkt mit der y-Achse:
\begin{array}{rclcll}f(0) &=& \left(\dfrac{0+17}{\sqrt{2}}\right)^2\\f(0) &=& \dfrac{289}{2} = 144{,}5\end{array}

Der Schnittpunkt mit der y-Achse ist also S_{y}\left(0\;\vert\; \dfrac{289}{2}\right).


8)
Schnittpunkt mit der x-Achse:
\begin{array}{rclcl} 0 &=& x^2+4x-5 &\vert& \text{p-q-Formel} \cr x_{1,2} &=& -2 \pm \sqrt{2^2+5} \cr &=& -2 \pm \sqrt{9} \cr\cr x_1 &=& -2+3 = 1 \cr x_2 &=& -2-3 = -5 \end{array}

Die Schnittpunkte mit der x-Achse sind also S_{x1}\left(1 \mid 0\right) und S_{x2}\left(-5 \mid 0\right).

Schnittpunkt mit der y-Achse:
\begin{array}{rclcll}f(0) &=& 0^2+4\cdot 0-5\\f(0) &=& -5\end{array}

Der Schnittpunkt mit der y-Achse ist also S_{y}\left(0 \mid -5\right).


9)
Schnittpunkt mit der x-Achse:
\begin{array}{rclcl} 0 &=& \dfrac{a-1}{3}x^2-a &\vert& -\dfrac{a-1}{3}x^2 \cr\cr -\dfrac{a-1}{3}x^2 &=& -a &\vert& \cdot (-3) \cr\cr (a-1)x^2 &=& 3a &\vert& :(a-1) \cr x^2 &=& \dfrac{3a}{a-1} \cr\cr x_{1,2} &=& \pm \sqrt{\dfrac{3a}{a-1}} \end{array}

Die Schnittpunkte mit der x-Achse sind also S_{x1}\left(-\sqrt{\dfrac{3a}{a-1}} \mid 0\right) und S_{x2}\left(\sqrt{\dfrac{3a}{a-1}} \mid 0\right).

Schnittpunkt mit der y-Achse:
\begin{array}{rclcll}f(0) &=& \dfrac{a-1}{3}\cdot 0^2-a\\f(0) &=& -a\end{array}

Der Schnittpunkt mit der y-Achse ist also S_{y}\left(0 \mid -a\right).

Bemerkung: Wie immer, wenn ein Parameter in einer Aufgabe enthalten ist, wie hier das a, sollte man kurz überlegen, ob es Werte von a gibt, bei denen sich die Funktion anders verhält als sonst, bzw. Werte, die a nicht annehmen darf, wenn die Rechnung funktionieren soll. Schaut man sich den Funktionsterm von f(x) an, stellt man fest, dass die Funktion bei a=1 nicht mehr quadratisch ist: Für a=1 würde der Funktionsterm nämlich zu f(x) = \dfrac{1-1}{3}x^2-1 = 0x^2-1 = -1. Da erübrigt sich die Suche nach den Schnittpunkten mit der x-Achse ...
Problematisch wird a=1 auch dort, wo durch a-1 geteilt werden muss, denn 1-1 ist ja nun mal 0.
Bei allen anderen Werten passiert das nicht: Die Funktion bleibt quadratisch und teilen dürfen wir auch. Halten wir also fest: a\in\mathbb{R}\setminus_{\{1\}}


10)
Schnittpunkt mit der x-Achse:
\begin{array}{rclcl} 0 &=& 3x^2+(6-6r)x-12r \cr 0 &=& 3x^2+6(1-r)x-12r &\vert& :3 \cr 0 &=& x^2+2(1-r)x-4r &\vert& \text{p-q-Formel} \cr x_{1,2} &=& -\dfrac{2(1-r)}{2} \pm \sqrt{(1-r)^2+4r} \cr x_{1,2} &=& -(1-r) \pm \sqrt{1-2r+r^2+4r} \cr x_{1,2} &=& -1+r \pm \sqrt{r^2+2r+1} \cr x_{1,2} &=& -1+r \pm \sqrt{(r+1)^2} \cr\cr x_1 &=& -1+r+r+1 = 2r \cr x_2 &=& -1+r-r-1 = -2 \end{array}

Die Schnittpunkte mit der x-Achse sind also S_{x1}\left(2r \mid 0\right) und S_{x2}\left(-2 \mid 0\right).

Schnittpunkt mit der y-Achse:
\begin{array}{rclcll}f(0) &=& 3\cdot 0^2+\left(6-6r\right)\cdot 0-12r\\f(0) &=& -12r\end{array}

Der Schnittpunkt mit der y-Achse ist also S_{y}\left(0 \mid -12r\right).

Bemerkung: Auch hier noch eine kurze Überlegung zum Parameter r: Für r gibt es keine Einschränkungen - aus zwei Gründen.
1. r hat nur Auswirkungen auf den linearen und konstanten Teil des Funktionsterms. Das ist problemlos, da sowohl der Koeffizient des linearen Terms als auch der konstante Term jeden Wert (einschließlich 0) annehmen dürfen.
2. Es wird nirgends durch r geteilt oder die Wurzel aus r gezogen. Dort, wo r unter der Wurzel steht, lässt sich der Radikand zu (r+1)^2 umformen, was immer nichtnegativ ist. Hier können also auch keine Probleme entstehen.
Zusammengefasst: r\in\mathbb{R}

 

9. Aufgabe

Wir beginnen einfach mal so, wie man immer beginnt, wenn man Nullstellen berechnen möchte:
\begin{array}{rclcll}0 &=& 2x^2+ax-1 &\vert& :2 \cr0 &=& x^2+\dfrac{a}{2}x-\dfrac12 &\vert& \text{p-q-Formel} \cr x_{1,2} &=& -\dfrac{a}{4}\pm\sqrt{\left(\dfrac{a}{4}\right)^2+\dfrac12} \cr x_{1,2} &=& -\dfrac{a}{4}\pm\sqrt{\dfrac{a^2}{16}+\dfrac{8}{16}} \cr x_{1,2} &=& -\dfrac{a}{4}\pm\sqrt{\dfrac{a^2+8}{16}} \cr x_{1,2} &=& \dfrac{-a\pm\sqrt{a^2+8}}{4}\end{array}

a)
Die Funktion hat genau eine Nullstelle, wenn der Wurzelterm in der letzten Zeile der Rechnung oben 0 ist:
\begin{array}{crclcl} \sqrt{a^2+8} &=& 0 \cr a^2+8 &=& 0 \cr a^2 &=& -8\end{array}

Da Quadrate niemals negativ sein können, gibt es keinen Wert für a, bei dem der Wurzelterm 0 ist. Die Funktion f(x) kann also nicht genau eine Nullstelle haben.

b)
Die Parabel schneidet die y-Achse im Punkt (0\mid -1) (kann man einfach ausrechnen, indem man x=0 in die Funktionsgleichung einsetzt) und ist nach oben geöffnet (der Koeffizient vor x^2 ist ja positiv). Daher besitzt sie immer genau zwei Nullstellen. Es gibt deswegen keinen Wert für a, für den die Funktion f(x) keine Nullstelle hat.

Anmerkung: Wer gleich so argumentiert wie bei b), kann sich die Rechnung bei a) sparen ...

 

10. Aufgabe

Vorbemerkung: Für quadratische Funktionen gilt immer f(x)=ax^2+bx+c mit a,b,c \in\mathbb{R}. Normalerweise kennen wir a, b und c schon - diesmal müssen wir sie bestimmen. Was wir stattdessen kennen, sind die Funktionswerte von f(x) für die Punkte P_1, P_2 und P_3. Damit können wir starten.


1)
Wir beginnen damit, die gegebenen Punkte in die allgemeine Funktionsgleichung einzusetzen:
\begin{array}{lcrcrcrcr} f(0) &=& a\cdot 0^2 &+& b\cdot 0 &+& c &=& -16 \cr f(1) &=& a\cdot 1^2 &+& b\cdot 1 &+& c &=& -10 \cr f(3) &=& a\cdot 3^2 &+& b\cdot 3 &+& c &=& 14 \end{array}

Etwas vereinfacht und zusammengefasst erhalten wir ein lineares Gleichungssystem mit den Variablen a, b und c:
\begin{array}{crcrcrcr} \text{I} & & & & & c &=& -16 \cr \text{II} & a &+& b &+& c &=& -10 \cr \text{III} & 9a &+& 3b &+& c &=& 14 \end{array}

In Zeile I müssen wir gar nicht rechnen, sondern sehen sofort c=-16. Das setzen wir im nächsten Schritt in die anderen Zeilen ein, damit das lineare Gleichungssystem handlicher wird, nämlich:
\begin{array}{crcrcrcrcl} \text{II} & a &+& b &-& 16 &=& -10 &\vert& +16 \cr \text{III} & 9a &+& 3b &-& 16 &=& 14 &\vert& +16 \cr \cr \text{II} & a &+& b & & &=& 6 \cr \text{III} & 9a &+& 3b & & &=& 30 \end{array}

Nun ist ein lineares Gleichungssystem mit zwei Gleichungen und zwei Unbekannten entstanden, welches z. B. mithilfe des Einsetzungsverfahrens gelöst werden kann:
\begin{array}{crcrcl} \text{II} & a+b &=& 6 &\vert& -b \cr \text{III} & 9a+3b &=& 30 \cr \cr \text{II} & a &=& 6-b \cr \text{III} & 9a+3b &=& 30 \cr \cr \text{II in III} & 9(6-b)+3b &=& 30 \cr & 54-9b+3b &=& 30 \cr & 54-6b &=& 30 &\vert& -54 \cr & -6b &=& -24 &\vert& :\left(-6\right) \cr & b &=& 4 \cr \cr \text{b in II} & a &=& 6-4 \cr & a &=& 2 \end{array}

Das Gleichungssystem hat also die Lösung a=2, b=4 und c=-16. Um die gesuchte quadratische Funktion zu bestimmen, setzen wir diese Werte nun in die allgemeine Funktionsgleichung ein: f(x)=2x^2+4x-16

Wer möchte, kann zur Probe die gegebenen Punkte in die gefundene Funktionsgleichung einsetzen:
\begin{array}{rclcl}f(0) &=& 2\cdot 0^2+4\cdot 0-16 &=& -16 \cr f(1) &=& 2\cdot 1^2+4\cdot 1-16 &=& -10 \cr f(3) &=& 2\cdot 3^2+4\cdot 3-16 &=& 14 \end{array}

Alles richtig!


2)
Wir beginnen damit, die gegebenen Punkte in die allgemeine Funktionsgleichung einzusetzen:
\begin{array}{lcrcrcrcr} f(2) &=& a\cdot 2^2 &+& b\cdot 2 &+& c &=& 1 \cr f(1) &=& a\cdot 1^2 &+& b\cdot 1 &+& c &=& 2 \cr f(0) &=& a\cdot 0^2 &+& b\cdot 0 &+& c &=& 1 \end{array}

Etwas vereinfacht und zusammengefasst erhalten wir ein lineares Gleichungssystem mit den Variablen a, b und c:
\begin{array}{crcrcrcr} \text{I} & 4a &+& 2b &+& c &=& 1 \cr \text{II} & a &+& b &+& c &=& 2 \cr \text{III} & & & & & c &=& 1 \end{array}

In Zeile III müssen wir gar nicht rechnen, sondern sehen sofort c=1. Das setzen wir im nächsten Schritt in die anderen Zeilen ein, damit das lineare Gleichungssystem handlicher wird, nämlich:
\begin{array}{crcrcrcrcl} \text{I} & 4a &+& 2b &+& 1 &=& 1 &\vert& -1 \cr \text{II} & a &+& b &+& 1 &=& 2 &\vert& -1 \cr \cr \text{I} & 4a &+& 2b & & &=& 0 \cr \text{II} & a &+& b & & &=& 1 \end{array}

Nun ist ein lineares Gleichungssystem mit zwei Gleichungen und zwei Unbekannten entstanden, welches z. B. mithilfe des Einsetzungsverfahrens gelöst werden kann:
\begin{array}{crcrcl} \text{I} & 4a+2b &=& 0 \cr \text{II} & a+b &=& 1 &\vert& -b \cr \cr \text{I} & 4a+2b &=& 0 \cr \text{II} & a &=& 1-b \cr \cr \text{II in I} & 4(1-b)+2b &=& 0 \cr & 4-4b+2b &=& 0 \cr & 4-2b &=& 0 &\vert& -4 \cr & -2b &=& -4 &\vert& :\left(-2\right) \cr & b &=& 2 \cr \cr \text{b in II} & a &=& 1-2 \cr & a &=& -1 \end{array}

Das Gleichungssystem hat also die Lösung a=-1, b=2 und c=1. Um die gesuchte quadratische Funktion zu bestimmen, setzen wir diese Werte nun in die allgemeine Funktionsgleichung ein: f(x)=-x^2+2x+1

Die Probe:
\begin{array}{rclcl}f(2) &=& -2^2+2\cdot 2+1 &=& 1 \cr f(1) &=& -1^2+2\cdot 1+1 &=& 2 \cr f(0) &=& -0^2+2\cdot 0+1 &=& 1 \end{array}

Alles richtig!


3)
Wir beginnen damit, die gegebenen Punkte in die allgemeine Funktionsgleichung einzusetzen:
\begin{array}{lcrcrcrcr} f(0) &=& a\cdot 0^2 &+& b\cdot 0 &+& c &=& 0 \cr f\left(\dfrac{1}{3}\right) &=& a\cdot \left(\dfrac{1}{3}\right)^2 &+& b\cdot \dfrac{1}{3} &+& c &=& -\dfrac{1}{3} \cr f\left(\dfrac{2}{3}\right) &=& a\cdot \left(\dfrac{2}{3}\right)^2 &+& b\cdot \dfrac{2}{3} &+& c &=& 0 \end{array}

Etwas vereinfacht und zusammengefasst erhalten wir ein lineares Gleichungssystem mit den Variablen a, b und c:
\begin{array}{crcrcrcr} \text{I} & & & & & c &=& 0 \cr \text{II} & \dfrac{1}{9}a &+& \dfrac{1}{3}b &+& c &=& -\dfrac{1}{3} \cr \text{III} & \dfrac{4}{9}a &+& \dfrac{2}{3}b &+& c &=& 0 \end{array}

In Zeile I müssen wir gar nicht rechnen, sondern sehen sofort c=0. Das setzen wir im nächsten Schritt in die anderen Zeilen ein, damit das lineare Gleichungssystem handlicher wird, nämlich:
\begin{array}{crcrcrcrcl} \text{II} & \dfrac{1}{9}a &+& \dfrac{1}{3}b &+& 0 &=& -\dfrac{1}{3} \cr \text{III} & \dfrac{4}{9}a &+& \dfrac{2}{3}b &+& 0 &=& 0 \cr \cr \text{II} & \dfrac{1}{9}a &+& \dfrac{1}{3}b & & &=& -\dfrac{1}{3} \cr \text{III} & \dfrac{4}{9}a &+& \dfrac{2}{3}b & & &=& 0 \end{array}

Nun ist ein lineares Gleichungssystem mit zwei Gleichungen und zwei Unbekannten entstanden, welches z. B. mithilfe des Additionsverfahrens gelöst werden kann:
\begin{array}{crclllcc} \text{II} & \dfrac{1}{9}a+\dfrac{1}{3}b &=& -\dfrac{1}{3} &\vert& \cdot (-2)\cr \text{III} & \dfrac{4}{9}a+\dfrac{2}{3}b &=& 0 \cr\cr \text{II} & -\dfrac{2}{9}a-\dfrac{2}{3}b &=& \dfrac{2}{3} \cr \text{III} & \dfrac{4}{9}a+\dfrac{2}{3}b &=& 0 \cr\cr \text{II+III} & \dfrac{2}{9}a-0b &=& \dfrac{2}{3} \cr & \dfrac{2}{9}a &=& \dfrac{2}{3} &\vert& : \dfrac{2}{9} \cr & a &=& 3 \cr\cr \text{a in III} & \dfrac{4}{9}\cdot 3+\dfrac{2}{3}b &=& 0 &\vert& -\dfrac{4}{3} \cr & \dfrac{2}{3}b &=& -\dfrac{4}{3} &\vert& :\dfrac{2}{3} \cr & b &=& -2 \end{array}

Das Gleichungssystem hat also die Lösung a=3, b=-2 und c=0. Um die gesuchte quadratische Funktion zu bestimmen, setzen wir diese Werte nun in die allgemeine Funktionsgleichung ein: f(x)=3x^2 - 2x

Die Probe:
\begin{array}{rclcl}f(0) &=& 3\cdot 0^2 - 2\cdot 0 &=& 0 \cr f\left(\dfrac{1}{3}\right) &=& 3\cdot\left(\dfrac{1}{3}\right)^2 - 2\cdot\dfrac{1}{3} &=& -\dfrac{1}{3} \cr f\left(\dfrac{2}{3}\right) &=& 3\cdot\left(\dfrac{2}{3}\right)^2 - 2\cdot\dfrac{2}{3} &=& 0 \end{array}

Alles richtig!


4)
Wir beginnen damit, die gegebenen Punkte in die allgemeine Funktionsgleichung einzusetzen:
\begin{array}{lcrcrcrcr} f(4) &=& a\cdot 4^2 &+& b\cdot 4 &+& c &=& 3 \cr f(2) &=& a\cdot 2^2 &+& b\cdot 2 &+& c &=& 0 \cr f(-5) &=& a\cdot \left(-5\right)^2 &+& b\cdot (-5) &+& c &=& 5{,}25 \end{array}

Etwas vereinfacht und zusammengefasst erhalten wir ein lineares Gleichungssystem mit den Variablen a, b und c:
\begin{array}{crcrcrcr} \text{I} & 16a &+& 4b &+& c &=& 3 \cr \text{II} & 4a &+& 2b &+& c &=& 0 \cr \text{III} & 25a &-& 5b &+& c &=& 5{,}25\end{array}

Hier tut uns leider keine der Gleichungen den Gefallen, die Lösung sofort bereit zu halten. Wir müssen das Gleichungssystem (drei Gleichungen mit drei Unbekannten) also klassisch mit dem Additionsverfahren lösen:
\begin{array}{crcrcrcrl} \text{I} & 16a &+& 4b &+& c &=& 3 \cr \text{II} & 4a &+& 2b &+& c &=& 0 \cr \text{III} & 25a &-& 5b &+& c &=& 5{,}25 \cr \cr \text{I} & 16a &+& 4b &+& c &=& 3 \cr \text{I-II} & 12a &+& 2b & & &=& 3 \cr \text{I-III} & -9a &+& 9b & & &=& -2{,}25 \cr \cr \text{I} & 16a &+& 4b &+& c &=& 3 \cr \text{II'} & 12a &+& 2b & & &=& 3 \cr \text{3 II'+4 III'} & & & 42b & & &=& 0 & \Rightarrow b = 0 \cr \cr b = 0 \text{ in II'} & 12a &+& 2\cdot 0 & & &=& 3 & \Rightarrow a = \dfrac{1}{4} \cr b = 0 \text{ und } a = \dfrac{1}{4} \text{ in II} & 4\cdot\dfrac{1}{4} &+& 2 \cdot 0 &+& c &=& 0 & \Rightarrow c = -1 \end{array}

Das Gleichungssystem hat also die Lösung a=\dfrac{1}{4}, b=0 und c=-1. Um die gesuchte quadratische Funktion zu bestimmen, setzen wir diese Werte nun in die allgemeine Funktionsgleichung ein: f(x)=\dfrac{1}{4}x^2 - 1

Die Probe:
\begin{array}{rclcl}f(4) &=& \dfrac{1}{4} \cdot 4^2 - 1 &=& 3 \cr f(2) &=& \dfrac{1}{4} \cdot 2^2 - 1 &=& 0 \cr f(-5) &=& \dfrac{1}{4} \cdot (-5)^2 - 1 &=& 5{,}25 \end{array}

Alles richtig!


5)
Wir beginnen damit, die angegebenen Punkte in die allgemeine Funktionsgleichung einzusetzen:
\begin{array}{lcrcrcrcr} f(-1) &=& a \cdot (-1)^2 &+& b \cdot (-1) &+& c &=& 2 \cr f(0) &=& a \cdot 0^2 &+& b \cdot 0 &+& c &=& 4 \cr f(2) &=& a \cdot 2^2 &+& b \cdot 2 &+& c &=& 12 \end{array}

Etwas vereinfacht und zusammengefasst erhalten wir ein lineares Gleichungssystem mit den Variablen a, b und c:
\begin{array}{crcrcrcr} \text{I} & a &-& b &+& c &=& 2 \cr \text{II} & & & & & c &=& 4 \cr \text{III} & 4a &+& 2b &+& c &=& 12 \end{array}

In Zeile \text{II} müssen wir gar nicht rechnen, sondern sehen sofort c=4. Das setzen wir im nächsten Schritt in die anderen beiden Zeilen ein, damit das lineare Gleichungssystem handlicher wird, nämlich:
\begin{array}{crcrcrcrcl} \text{I} & a &-& b &+& 4 &=& 2 &\vert& -4 \cr \text{III} & 4a &+& 2b &+& 4 &=& 12 &\vert& -4 \cr\cr \text{I} & a &-& b & & &=& -2 \cr \text{III} & 4a &+& 2b & & &=& 8 \end{array}

Nun ist ein lineares Gleichungssystem mit zwei Gleichungen und zwei Unbekannten entstanden, welches z. B. mithilfe des Einsetzungsverfahrens gelöst werden kann:
\begin{array}{crcrcl} \text{I} & a - b &=& -2 &\vert& +b \cr \text{III} & 4a + 2b &=& 8 \cr\cr \text{I} & a &=& -2 + b \cr \text{III} & 4a + 2b &=& 8 \cr \cr \text{I in III} & 4\left(-2 +b \right) +2b &=& 8 \cr & -8 + 4b + 2b &=& 8 &\vert& +8 \cr & 6b &=& 16 &\vert& :6 \cr & b &=& \dfrac{8}{3} \cr\cr \text{b in I} & a &=& -2 + \dfrac83 \cr & a &=& \dfrac{2}{3} \end{array}

Das Gleichungssystem hat also die Lösung a=\dfrac23, b=\dfrac83 und c=4. Um die gesuchte quadratische Gleichung zu bestimmen, setzen wir diese Werte nun in die allgemeine Funktionsgleichung ein: f(x)=\dfrac23 x^2 + \dfrac83 x + 4

Die Probe:
\begin{array}{rclcl} f(-1) &=& \dfrac23 \cdot (-1)^2 + \dfrac83 \cdot (-1) +4 &=& 2 \cr\cr f(0) &=& \dfrac23 \cdot 0^2 + \dfrac83 \cdot 0 + 4 &=& 4 \cr\cr f(2) &=& \dfrac23 \cdot 2^2 + \dfrac83 \cdot 2 + 4 &=& 12 \end{array}

Alles richtig!

11. Ungleichungen - Lernziele und typische Fehler

Nach Durcharbeiten dieses Kapitels sollten Sie folgende Lernziele erreicht haben:

  • Sie kennen die Unterschiede zwischen den verschiedenen Vergleichs-/Relationszeichen.
  • Sie können Ungleichungen erkennen.
  • Sie können zu einer Ungleichung den passenden Definitionsbereich bestimmen.
  • Sie wissen, wie sich die Äquivalenzumformungen bei Ungleichungen von denen bei Gleichungen unterscheiden und beachten dies in Rechnungen.
  • Sie können lineare, quadratische und Betragsungleichungen lösen, ggf. mithilfe einer Fallunterscheidung.
  • Sie können die Lösungsmenge interpretieren und mathematisch korrekt notieren.


Typische Fehler in diesem Kapitel sind:

  • Das Relationszeichen wird bei der Multiplikation mit einer negativen Zahl nicht "umgedreht". Erklärung
  • Eine notwendige Fallunterscheidung wird nicht durchgeführt. Erklärung


Für weitere Informationen zur Mathematikunterstützung an der TH Wildau nutzen Sie bitte den Moodle-Kursraum "SOS Mathematik - Brückenkurs".

Übersicht:

 

11.1 Ungleichungen - Aufgaben

1. Aufgabe

Lösen Sie folgende Ungleichungen! Geben Sie jeweils den Definitionsbereich an!

1) 2x+5 \leq x-25

  11) x^2+6x+8 \geq 0

2) 6 \leq \dfrac{x}{2}+5

  12) \dfrac{1}{x} \geq5
3) 14(-x+2)\leq (4+2x)\cdot 7

  13) \dfrac{1}{x+3} < -5

4) 8\left(\dfrac{1}{2}-\dfrac{3}{4}x\right) -3 > 5-4x

  14) \dfrac{20}{-y+13} \leq 15
5) \dfrac{3-2x}{6} < \dfrac{2-x}{2}-1

  15) \dfrac{5x-7}{2x+3} < 4

6) (2x-11)\cdot 10 \geq -67+4x-8(4-2x)

  16) 2-\dfrac{6(x+3)}{4+2x} > 0

7) (2x+3)^2-13 \geq (x-5)(x+5)+3x(x-3)

  17) \dfrac{1}{x+5}-\dfrac{1}{x-5}>-1

8) \dfrac{t^2}{2}-3t>56

  18) \dfrac{3x}{3+x}+\dfrac{2x}{3-x}>1

9) 2x^2>-8+7x   19) \dfrac{x-7}{x-4} - \dfrac{2x-5}{x-2} \leq 0

10) 16x^2-64 < -48x-120   20) \dfrac{x+1}{x^2-x-2} < 0

 

2. Aufgabe

Skizzieren Sie die Lösungsmenge der folgenden Ungleichungen jeweils in einem Koordinatensystem!

1) x+y\geq3   6) 6x + 9y \leq 18
2) 2x+5y\leq10   7) 5y + 3x -1 < 2y
3) -4x-7y>-1

  8) \dfrac{y}{3} \geq x + \dfrac{14}{9}
4) 9x-3y\leq24

  9) 40x + 60 - 80y < 100
5) -5x+10y < 35

  10) x + 2y \leq -2x -y

 

3. Aufgabe

Behauptung: Alle Zahlen sind kleiner als 0.
Beweis: Sei x irgendeine positive Zahl. Es gilt logischerweise für alle x, dass x-1 kleiner als x ist. Das kann man mathematisch als Ungleichung schreiben. Wenn man mit dieser ein bisschen rechnet, erhält man:

\begin{array}{rclcl} x-1 & < & x &\vert&\cdot (-x) \cr -x^2+x & < & -x^2 &\vert& +x^2 \cr x & < & 0 \end{array}

Wo steckt der Fehler?

Dieses Kapitel enthält die folgenden Themen:

 

11.2 Ungleichungen - Erklärungen

In Kapitel 5 hatten wir Gleichungen eingeführt als "ein Term = ein (anderer) Term". Ganz ähnlich funktioniert es für Ungleichungen. Allerdings gibt es hier vier Varianten:

  • "ein Term  < ein (anderer) Term"
  • "ein Term \leq ein (anderer) Term"
  • "ein Term > ein (anderer) Term"
  • "ein Term \geq ein (anderer) Term"


Die Vergleichszeichen oder auch Relationszeichen heißen:

  • Kleinerzeichen:  < (gesprochen: "kleiner als", manchmal auch "echt kleiner als")
  • Kleinergleichzeichen: \leq (gesprochen: "kleiner gleich")
  • Größerzeichen: > (gesprochen: "größer als", manchmal auch "echt größer als")
  • Größergleichzeichen: \geq (gesprochen: "größer gleich")

Merke: Die Vergleichszeichen sind immer zur größeren Seite geöffnet.

Auch "ein Term \neq ein (anderer) Term" (mit dem Ungleichheitszeichen \neq) ist natürlich eine Ungleichung. Allerdings brauchen wir sie nicht gesondert zu behandeln, weil man ja "nur" die zugehörige Gleichung lösen muss. Alles, was dabei nicht Lösung ist, ist dann Lösung der Ungleichung.


Wie bei Gleichungen ist es auch bei Ungleichungen meist die Aufgabe, diejenigen Werte der Variablen zu ermitteln, für die die Ungleichung wahr wird. Anders formuliert: Man möchte die Lösungen der Ungleichungen finden. Grundsätzlich behandelt man Ungleichungen genauso wie Gleichungen, nämlich mit Äquivalenzumformungen. Beispielsweise müssen (natürlich!) auch bei Ungleichungen alle Umformungen auf beiden Seiten durchgeführt werden, damit sich die Lösungsmenge nicht ändert. Ein paar Unterschiede gibt es allerdings schon zu beachten. Die grundlegendsten Unterschiede sind:

  • Vertauscht man die beiden Seiten einer Ungleichung, muss auch das Vergleichszeichen "umgedreht" werden.
    Beispiel: Aus 4>2 wird naheliegenderweise 2 < 4.

  • Bei Multiplikation mit negativen Zahlen muss das Vergleichszeichen ebenfalls "umgedreht" werden. Dies schließt die Division mit ein.
    Beispiel: Aus 4>2 wird -4 < -2.

 

Diese Regeln gelten natürlich auch für die anderen Vergleichszeichen und wenn Variablen etc. in der Ungleichung enthalten sind. Bei Ungleichungen mit Variablen ist allerdings meist nicht mehr ganz so offensichtlich wie hier, wie man vorgehen muss ... In vielen Fällen kommt man ohne eine sogenannte Fallunterscheidung nicht aus. Fallunterscheidungen sind grundsätzlich nichts Geheimnisvolles oder Schlimmes. Sie sind dazu da, verschiedene Fälle, in denen unterschiedlich weiter gerechnet oder argumentiert werden muss, voneinander zu trennen. Vergisst man das, entstehen üblicherweise Fehler, weil man zu viele Dinge über einen Kamm schert. Multipliziert man beispielsweise eine Ungleichung mit der Variablen x\in\mathbb{R}, kann man nicht wissen, ob das Vergleichszeichen "umgedreht" werden muss oder nicht, weil man ja nicht weiß, ob x positiv oder negativ ist. Es ist ja das Wesen von Variablen, dass wir eben im Vorhinein nicht wissen, für welchen Zahlenwert sie stehen. Es könnte sogar sein, dass die Multiplikation als Äquivalenzumformung überhaupt nicht zulässig ist, nämlich wenn x=0 gilt.
Beispiel: Multipliziert man die Ungleichung 4>2 mit x, können wir nicht entscheiden, ob die neue Ungleichung 4x>2x lauten muss oder 4x < 2x. Bei x>0 wäre die erste Ungleichung richtig, bei x < 0 die zweite. Dass die Multiplikation mit x=0 mal wieder keine gute Idee ist, sieht man daran, dass damit aus der richtigen Aussage 4>2 die falsche Aussage 0>0 werden würde.

Durch die Fallunterscheidung müssen wir zwar statt einer Ungleichung mehrere lösen (abhängig von der Anzahl der zu unterscheidenden Fälle), allerdings sind diese Ungleichungen meist einfacher als die ursprüngliche Ungleichung. Da wir eine bestimmte Bedingung als gegeben annehmen, müssen wir uns um diese ja nicht mehr kümmern.
Am Ende der Rechnung ist dann wichtig, die berechneten Werte oder Intervalle mit dieser Bedingung abzugleichen. Falls sie sich widersprechen, sind die berechneten Werte oder Intervalle nämlich keine Lösungen. Alles, was wir berechnen, steht ja unter der Annahme, dass die Bedingung gilt. 
Beispiel: Rechnen wir in einem Fall unter der Bedingung, dass x>0 ist, kann x=-5 keine Lösung sein. Auch wenn unsere Rechnung, die zu x=-5 geführt hat, komplett korrekt ist, dürfen wir diesen Wert nicht in die Lösungsmenge aufnehmen.

Zusammengefasst kann man sagen: Ohne eine saubere Fallunterscheidung "verliert" man entweder Lösungen oder man hat am Ende zu viele - beides nicht gut ... 


Zwei Dinge müssen bei allen Fallunterscheidungen beachtet werden:

  1. Es muss jeder Wert aus dem Definitionsbereich in der Fallunterscheidung beachtet werden.
    Beispiel: x\geq\frac{1}{2} und x < \frac{1}{2} ist eine gute Fallunterscheidung, weil jede reelle Zahl entweder größer bzw. gleich \frac{1} {2} oder kleiner als \frac{1} {2} ist. Hätten wir stattdessen x>\frac{1}{2} und x < \frac{1}{2} gewählt, wäre keine Aussage über x=\frac{1}{2} möglich gewesen. Umgekehrt wäre die Zahl \frac{1} {2} bei x\geq\frac{1}{2} und x\leq\frac{1}{2} doppelt berücksichtigt gewesen, was zu Widersprüchen führen kann.

  2. Nach der Fallunterscheidung sind jeweils alle Rechenoperationen und Argumentationen, die mit dieser Voraussetzung arbeiten, erlaubt. Beispiel: Wird bei einer Fallunterscheidung angenommen, dass x\neq 0 ist, darf anschließend bedenkenlos durch x geteilt werden, weil eine Division durch 0 ja gar nicht vorkommen kann.


Ein paar Worte zur Lösungsmenge einer Ungleichung: Auch hier zeigt sich ein Unterschied zu Gleichungen. Während "normal lösbare" Gleichungen, wie sie in diesem Lernmodul betrachtet wurden, im Normalfall eine endliche Anzahl von Lösungen haben (z. B. quadratische Gleichungen mit einer oder zwei Lösungen), umfasst die typische Lösungsmenge bei Ungleichungen unendlich viele Zahlen. Das liegt daran, dass in Ungleichungen Größenvergleiche formuliert werden, also alle Zahlen gesucht werden, die kleiner (oder größer) als etwas Anderes sind und davon gibt es meistens eine ganze Menge ... Betrachtet man Ungleichungen im Bereich der reellen Zahlen ist die Lösungsmenge meist ein Intervall. In vielen Fällen ist es eine sehr sinnvolle Idee, die Lösungsmenge auf dem Zahlenstrahl zu veranschaulichen.

 

Lösungsstrategien für verschiedene Arten von Ungleichungen

Im Folgenden werden anhand von Beispielen Lösungsstrategien für verschiedene Arten von Ungleichungen eingeführt. Wenn Sie Schwierigkeiten haben, die Ungleichungsarten zu erkennen und auseinanderzuhalten, schauen Sie bitte in den entsprechenden Gleichungskapiteln nach. Quadratische Ungleichungen sind aus den gleichen Gründen "quadratisch" wie quadratische Gleichungen ...

 

Lineare Ungleichungen

Zunächst also lineare Ungleichungen:

Beispiel 1:
Bei einer Matheklausur gibt es pro richtige Aufgabe 5 Punkte. Vorab konnten 8 Bonuspunkte erarbeitet werden. Wie viele Aufgaben müssen mindestens richtig gelöst werden, wenn zum Bestehen 50 Punkte nötig sind?
Anmerkung: Wir gehen der Einfachheit halber davon aus, dass es keine Teilpunkte für halbrichtig gelöste Aufgaben gibt.

Lösung:
Sei x die Anzahl der Aufgaben in der Klausur.
Zu lösen ist die Ungleichung 8+5x \geq 50 , wobei für x nur natürliche Zahlen größer 0 eingesetzt werden dürfen, weil ja keine Teilpunkte vergeben werden und wir davon ausgehen können, dass die Klausur aus mindestens einer Aufgabe besteht.

\begin{array}{rclcl} \mathbb{D} &=& \mathbb{N^+} \cr \cr 8+5x & \geq & 50 & \vert & -8 \cr\cr 5x & \geq & 42 & \vert & :5 \cr\cr x & \geq & \dfrac{42}{5} = 8{,}4 \cr \cr \mathbb{L} &=& \{9; 10; 11; \dots\} \end{array}

Bemerkung: Sie sehen, dass der Lösungsweg quasi identisch zum Lösungsweg der entsprechenden linearen Gleichung ist. Wichtig ist noch, die 8{,}4 auf 9 aufzurunden, weil ja nur natürliche Zahlen als Lösungen zugelassen waren. Es muss trotz der 4 in der Nachkommastelle aufgerundet werden, weil ja nach "mindestens" gefragt war.


Abwandlung von Beispiel 1:
Wie ändert sich die Lösung, wenn Sie erfahren, dass die Klausur 12 Aufgaben umfasst?

Lösung:
Die zu lösende Gleichung inklusive Lösungsweg bleibt gleich, weil sich an der Punkteverteilung ja nichts geändert hat. Was sich ändert, ist der Definitionsbereich: Es kommen nun nicht mehr alle natürlichen Zahlen größer 0 als Lösungen infrage, sondern nur noch die natürlichen Zahlen zwischen 1 und 12 (jeweils eingeschlossen). Formal geschrieben lautet der Definitionsbereich also: \mathbb{D}=\{1; 2; 3; \dots; 10; 11; 12\}
Das hat (natürlich) Auswirkungen auf die Lösungsmenge: Berechnet haben wir, dass Zahlen größer oder gleich 9 die Ungleichung erfüllen. Davon gibt es im Definitionsbereich aber gar nicht so viele, nämlich nur 9, 10, 11 und 12. Da nur Zahlen, die im Definitionsbereich liegen, überhaupt als Lösungen infrage kommen, lautet die Lösungsmenge also: \mathbb{L} = \{9; 10; 11; 12\}


Beispiel 2:
Zu lösen ist -5y+27>-3\left(\dfrac{2y}{3}-7\right) mit \mathbb{D}=\mathbb{R}

Lösung:
\begin{array}{rclcl} -5y+27 &>& -3\left(\dfrac{2y}{3}-7\right) \cr\cr -5y+27 &>& -2y+21 & \vert & -27+2y \cr\cr -3y &>& -6 & \vert & :\left(-3\right) \cr\cr y & < & 2 \cr \cr \mathbb{L} &=& \rbrack-\infty;2\lbrack \end{array}

Bemerkung 1: Ganz wichtig: Bei der Umformung zur letzten Zeile muss das Vergleichszeichen "umgedreht" werden, weil mit einer negativen Zahl multipliziert wurde!
Bemerkung 2: Wir haben berechnet, dass alle Zahlen aus dem Definitionsbereich, die kleiner sind als 2, die Ungleichung lösen. Die Lösung ist also ein Teilbereich der reellen Zahlen, was man am besten als Intervall aufschreibt. Nach unten gibt es dabei keine Grenze. Mathematisch schreibt man dafür -\infty. Die 2 ist nicht Teil der Lösungsmenge, weil in der letzten Zeile der Rechnung ein Kleiner- und kein Kleinergleichzeichen steht. Die Lösungsmenge ist also das offene Intervall \rbrack-\infty;2\lbrack. Wichtig: Das bedeutet nicht, dass das Lösungsintervall nur aus ganzen Zahlen besteht und der größte Wert im Lösungsintervall die 1 ist. Ganz im Gegenteil: Zwischen 1 und 2 liegen noch etliche - um genau zu sein, unendlich viele - reelle Zahlen, die selbstverständlich ebenfalls Teil der Lösungsmenge sind, z. B. 1{,}5.

 

Quadratische Ungleichungen

Es gibt verschiedene Wege, quadratische Ungleichungen zu lösen. Wir beschränken uns hier auf einen, der relativ anschaulich ist und sich leicht auf weitere Arten von Ungleichungen übertragen lässt.

Beispiel 1:
Zu lösen ist \dfrac{1}{8}\left(2x^2-8\right) < x+2 mit \mathbb{D}=\mathbb{R}

Lösung:
Wir betrachten die linke und die rechte Seite der Ungleichung jeweils als Funktion, also f_L(x)=\dfrac{1}{8}\left(2x^2-8\right) und f_R(x)=x+2 und zeichnen die zugehörigen Graphen in ein Koordinatensystem (links). Gleichzeitig kann man die Ungleichung so umformen, dass auf der rechten Seite nur noch 0 steht: \dfrac{1}{4}x^2-x-3 < 0 und ebenfalls den Graphen zur dazu passenden Funktion f(x)= \dfrac{1}{4}x^2-x-3 zeichnen (Koordinatensystem rechts). Das sieht dann so aus:

Grafische Lösung der Ungleichung

Man sieht in der linken Grafik, dass f_L(x) zwischen -2 und 6 kleiner als f_R(x) ist. Die Lösungsmenge lautet daher \mathbb{L}=\;\rbrack-2;6\lbrack. Alternativ suchen wir in der rechten Grafik den Bereich, in dem f(x) kleiner als 0 ist.
Die Lösung der Ungleichung lässt sich also direkt aus der Grafik ablesen - mit der Einschränkung, dass das natürlich nur funktioniert, wenn die Schnittpunkte bei "glatten" Zahlenwerten liegen (werden wir gleich bestätigen). Lägen die Schnittpunkte z. B. bei -1{,}9897 und 6{,}001203, hätten wir mit dieser Methode keine Chance ... Trotzdem lohnen sich Zeichnungen in vielen Fällen, vor allem auch bei komplizierteren Ungleichungen, weil man dadurch zumindest eine Idee bekommt, wie die Lösung aussehen muss.


Rechnerisch nähern wir uns der Lösung, indem wir die zugehörige quadratische Gleichung betrachten, am geschicktesten direkt von der bereits umgeformten Ungleichung: \frac{1}{4}x^2-x-3=0 . Darüber lassen sich nämlich die Stellen, an denen f(x) Nullstellen hat, was ja den Rändern des Lösungsintervalls entspricht, exakt ermitteln. Wie man oben in der rechten Grafik sieht, ist die Funktion zwischen zwei Nullstellen entweder immer positiv oder immer negativ. D. h., wir müssen dann nur noch untersuchen, welcher Fall in welchem Teilintervall eintritt (Kleine Einschränkung: Das funktioniert nicht, wenn die Funktion Sprünge, Löcher etc. haben sollte, aber solche Funktionen betrachten wir hier nicht.). Also:

\begin{array}{rclcl} \dfrac{1}{4}x^2-x-3 &=& 0 & \vert & \cdot 4 \cr\cr x^2-4x-12 &=& 0 \cr x_{1,2} &=& 2\pm\sqrt{4+12} \cr x_{1,2} &=& 2\pm\sqrt{16} \cr \cr x_1 &=& 2+4 \; = \; 6 \cr x_2 &=& 2-4 \; = \; -2 \end{array}

Diese Werte liefern uns drei Intervalle:
Von -\infty bis zur kleineren Nullstelle, hier x_2: I_1=\;\rbrack-\infty;-2\lbrack
Zwischen den Nullstellen: I_2=\;\rbrack-2;6\lbrack
Von der größeren Nullstelle, hier x_1, bis +\infty : I_3=\;\rbrack6;\infty\lbrack

Wichtig: Die ermittelten Nullstellen, im vorliegenden Fall -2 und 6, werden nicht in die offenen Intervalle eingeschlossen. Bei Ungleichungen, die ein Kleiner- oder Größerzeichen enthalten, können die Nullstellen niemals eine Lösung der Ungleichung sein. Handelt es sich um Ungleichungen, die ein Kleinergleich- oder Größergleichzeichen enthalten, stellen die Nullstellen immer eine Lösung der Ungleichung dar. Bei den Nullstellen sollte man also genau hinschauen.

Nun müssen wir noch überprüfen, wo f(x) größer als 0 und wo f(x) kleiner als 0 ist, denn das war ja die eigentliche Frage. (Letztendlich kann man das natürlich auch aus den Grafiken oben ablesen, aber hier soll ein vollständiger Rechenweg gezeigt werden.) Das machen wir, indem wir einzelne Werte aus den drei Intervallen in den ursprünglichen Term einsetzen. Welche Zahlen wir dabei nehmen, ist egal, denn innerhalb der Intervalle gibt es ja entweder nur positive oder nur negative Funktionswerte. Wir dürfen uns also jeweils eine Zahl aussuchen, mit der es sich besonders schön weiterrechnet:
Für I_1 nehmen wir -4: f(-4)=\frac{1}{4}\cdot 16+4-3=5>0 Also kommt I_1 nicht als Lösung infrage.
Für I_2 nehmen wir 0: f(0)=\frac{1}{4}\cdot 0-0-3=-3 < 0 Hier passt es: I_2 kommt als Lösung infrage.
Für I_3 nehmen wir 8: f(8)=\frac{1}{4}\cdot 64-8-3=5>0 Also kommt I_3 auch nicht als Lösung infrage.

Bemerkung: Warum wurden gerade diese Zahlen für die Überprüfung der Intervalle gewählt?
0 ist eigentlich immer eine gute Option, weil damit alle Produkte wegfallen. -4 und 8 haben den Vorteil, dass sie die betragsmäßig kleinsten Zahlen aus den beiden Intervallen sind, die quadriert durch 4 teilbar sind. Anders gesagt: Für diese Zahlen liefert \frac{1}{4}x^2 eine ganze Zahl und keinen Bruch. Man könnte für I_3 natürlich auch mit \frac{6{,}12^2}{4}=\frac{37{,}4544}{4} oder mit \frac{7^2}{4}=\frac{49}{4} weiter rechnen, wäre vollkommen korrekt - aber wenn die Alternative \frac{8^2}{4}=16 ist ...

In die Lösungsmenge einer Ungleichung gehen grundsätzlich alle Intervalle ein, in denen die Überprüfung eine wahre Aussage ergibt. Da unsere Beispielungleichung ein Kleinerzeichen enthält, sind die Nullstellen nicht Teil der Lösung. Wir erhalten also als Lösungsmenge \mathbb{L}=\;\rbrack-2;6\lbrack.

Bemerkung: Die Dreiteilung des möglichen Lösungsbereiches ist recht typisch für quadratische Ungleichungen. Wenn Sie sich an den klassischen Verlauf von Parabeln mit zwei Nullstellen erinnern, wissen Sie auch warum. Die Lösung besteht dann entweder aus dem mittleren Intervall oder den beiden äußeren.
Allerdings ist es auch möglich, dass die Parabel die x-Achse gar nicht schneidet oder lediglich in einem Punkt berührt. Dann sieht der Lösungsbereich entsprechend anders aus. Ein Beispiel dafür folgt nun:


Beispiel 2:
Schauen wir uns die Ungleichung -25x^2+130x\geq 169 mit \mathbb{D}=\mathbb{R} an:
\begin{array}{rclcll}-25x^2+130x-169 &\geq & 0 &\vert & :\left(-25\right)\\x^2-\dfrac{26}{5}x+\dfrac{169}{25} &\leq & 0 \end{array}

Wir betrachten nun die zugehörige quadratische Gleichung:
\begin{array}{rclcl}x^2-\dfrac{26}{5}x+\dfrac{169}{25} &=& 0 &\vert& \text{p-q-Formel} \\ x_{1,2} &=& \dfrac{13}{5}\pm\sqrt{\left(-\dfrac{13}{5}\right)^2-\dfrac{169}{25}} \\\\x_{1,2} &=& \dfrac{13}{5}\pm\sqrt{0} \\\\x_{1,2} &=& \dfrac{13}{5}\end{array}

In diesem Fall bekommen wir nur zwei Intervalle:
Von -\infty bis zur Nullstelle: I_1=\;\left\rbrack-\infty;\dfrac{13}{5}\right\lbrack
Von der Nullstelle bis +\infty : I_2=\;\left\rbrack \dfrac{13}{5};\infty\right\lbrack

Nun müssen wir überprüfen, wo die linke Seite der Ungleichung kleiner oder gleich 0 ist; denn das war ja die eigentliche Frage:
Für I_1 nehmen wir 0: 0^2-\dfrac{26}{5} \cdot 0+\dfrac{169}{25} = \dfrac{169}{25} > 0 Also kommt I_1 nicht als Lösung infrage.
Für I_2 nehmen wir 5: 5^2-\dfrac{26}{5} \cdot 5+\dfrac{169}{25} = \dfrac{144}{25} > 0 Auch I_2 kommt als Lösung nicht infrage.

Die Intervallbetrachtung liefert uns also keine Lösungen der Ungleichung. Aber: Da die ursprüngliche Ungleichung ein Größergleichzeichen enthält, ist die oben berechnete Lösung der quadratischen Gleichung Teil der Lösungsmenge. Diese Ungleichung wird also von einem einzigen Wert erfüllt: \mathbb{L} = \left\{\dfrac{13}{5}\right\}

Grafische Interpretation: Betrachtet man die zugehörige quadratische Funktion f(x) = x^2-\dfrac{26}{5}x+\dfrac{169}{25}, hat die Parabel nur eine Nullstelle, nämlich bei x_N = \dfrac{13}{5}. Sie berührt die x-Achse also nur. Da der Koeffizient von x^2 positiv ist, ist die Parabel nach oben geöffnet. Deshalb verläuft sie links und rechts vom Berührpunkt oberhalb der Achse. Für die Lösung der Ungleichung x^2-\dfrac{26}{5}x+\dfrac{169}{25} \leq 0, also als Antwort auf die Frage, wo die Werte der Parabel auf oder unterhalb der x-Achse liegen, kommt also nur die Nullstelle infrage.

Beispiel 3:
Abschließend wollen wir das Beispiel 8y^2-32 \leq 0 mit \mathbb{D} = \mathbb{R} betrachten:
\begin{array}{rclcl} 8y^2-32 &\leq& 0 &\vert& +32 \\ 8y^2 &\leq& 32 &\vert& :8 \\ y^2 &\leq& 4 \end{array}
Jetzt könnte man auf die Idee kommen, auf beiden Seiten die Wurzel zu ziehen mit dem Ergebnis y \leq 2. Das ist nicht ganz verkehrt, aber leider auch nicht ganz richtig ... y \leq 2 entspricht ja dem Intervall ]-\infty; 2] und in diesem Intervall sind viele Zahlen enthalten, die unsere Ungleichung y^2 \leq 4 nicht erfüllen, z. B. y_1=-5 mit y_1^2 = 25. Wir müssen das Intervall also auch nach unten beschränken. Die kleinste Zahl, die die Ungleichung erfüllt, ist -2, also die Gegenzahl zu dem schon gefundenen Wert 2. Das führt uns zu dem Lösungsintervall [-2;2]. Die Lösung ist also nicht y \leq 2, sondern -2 \leq y \leq 2, was das gleiche ist wie \vert y\vert \leq 2. In Worten: Der Betrag von y muss kleiner oder gleich 2 sein.

Die vollständige Lösung der Beispielungleichung lautet also
\begin{array}{rclcl} 8y^2-32 &\leq& 0 &\vert& +32 \\ 8y^2 &\leq& 32 &\vert& :8 \\ y^2 &\leq& 4 &\vert& \sqrt{} \\ \vert y \vert &\leq& 2 \\\\\mathbb{L} &=& [-2;2]\end{array}

Bemerkung 1: Die Argumentation funktioniert genauso, wenn in der Ungleichung  < statt \leq steht, nur dass wir dann statt des abgeschlossenen Intervalls [-2;2] das offene Intervall ]-2;2[ erhalten, welches -2 < y < 2 entspricht.
Bemerkung 2: Bei Ungleichungen mit \geq oder > muss ebenfalls beim Wurzelziehen der Betrag gesetzt werden. Ohne den Betrag würden wir in diesem Fall nämlich viele Lösungen verlieren. Beispiel: Würde man aus y^2 \geq 4 folgern, dass y \geq 2 ist, dann fehlten alle negativen Lösungen der Ungleichungen, wie y_2=-10 mit y_2^2=100, was ja offensichtlich größer gleich 4 ist. Also ist hier die richtige Lösung \vert y\vert \geq 2.
Bemerkung 3: Vielleicht haben Sie bei dieser Aufgabe an quadratische Gleichungen, wie y^2 = 4, gedacht und daran, dass hier die Lösung y=\pm 2 ist ... Da man diese Lösung auch mit einem Betrag schreiben kann, nämlich als \vert y\vert =2, sind sich Gleichungen und Ungleichungen auch hier wieder sehr ähnlich.

 

Bruchungleichungen

Schauen wir uns zum Abschluss noch eine Ungleichung an, bei der eine Fallunterscheidung vorgenommen werden muss. Leider müssen wir hierfür ein bisschen vorgreifen, da der grundsätzliche Lösungsweg für Bruch(un)gleichungen erst im Kapitel 14 erläutert wird ... Wir beschränken uns daher hier auf einen einfachen Fall.

Beispiel:
Zu lösen ist \dfrac{12-x}{4x-8} \leq 1

Bestimmung des Definitionsbereichs:
Da nicht durch 0 geteilt werden darf, muss der Nenner 4x-8 ungleich 0 sein. Daher müssen wir zunächst mithilfe einer Ungleichung ausrechnen, für welche Variablenwerte diese Bedingung erfüllt wird, und den Definitionsbereich entsprechend "zuschneiden".
\begin{array}{rclcll}4x-8 &\neq & 0 &\vert & +8 \\4x &\neq & 8 &\vert & :4\\x &\neq & 2\end{array}

Aus der Betrachtung der Lösung können wir ableiten, dass wir alle Werte aus \mathbb{R} mit Ausnahme der 2 verwenden dürfen. Der Definitionsbereich ist also \mathbb{D} = \mathbb{R}\setminus_{\left\{2\right\}}.

Lösung der Ungleichung:
\begin{array}{rclcll}\dfrac{12-x}{4x-8} &\leq & 1 &\vert & \cdot\left(4x-8\right)\end{array}
Wir multiplizieren die Ungleichung mit dem Nenner (4x-8), um den Bruch auflösen. Da wir dabei mit der Variablen multiplizieren, ist eine Fallunterscheidung nötig. Wir wissen ja nicht, ob der Term, mit dem wir die Ungleichung multiplizieren, positiv oder negativ ist.

Fall 1: Wir nehmen an, dass 4x-8>0 ist, also x>2. Dann bleibt das Vergleichszeichen, wie es ist:
\begin{array}{rclcll}\dfrac{12-x}{4x-8} &\leq & 1 &\vert & \cdot\left(4x-8\right) \\12-x &\leq & 4x-8 &\vert & +8+x \\20 &\leq & 5x &\vert & :5 \\4 &\leq & x \\x &\geq& 4\end{array}

Als Ergebnis unserer Rechnung erhalten wir die Bedingung x \geq 4 (grüner Pfeil in der Grafik unten), die wir mit der Eintrittsbedingung unserer Fallunterscheidung x>2 (blauer Pfeil) abgleichen müssen. Beiden Bedingungen gemeinsam ist nur der Zahlenbereich x\geq 4, auch wenn x>2 noch weitere Bereiche umfasst, z. B. die Zahl 3, die zwar größer als 2, aber nicht größer gleich 4 ist. Die Teillösungsmenge für diesen Fall der Fallunterscheidung ist also \mathbb{L}_1 = \left[4;\infty\right[.
Zahlenstrahl mit Bedingungen

Fall 2: Wir nehmen an, dass 4x-8 < 0 ist, also x < 2. Nun muss das Vergleichszeichen "umgedreht" werden, weil wir mit einem negativen Faktor multiplizieren:
\begin{array}{rclcll}\dfrac{12-x}{4x-8} &\leq & 1 &\vert & \cdot\left(4x-8\right)\\12-x &\geq & 4x-8 &\vert & +8+x\\20 &\geq & 5x &\vert & :5\\4 &\geq & x \\x &\leq& 4\end{array}

In diesem Fall bekommen wir als Ergebnis die Bedingung x \leq 4 (grüner Pfeil), die wir wieder mit der Eintrittsbedingung x < 2 (blauer Pfeil) abgleichen. Beide Bedingungen sind im Zahlenbereich x < 2 erfüllt. Daraus ergibt sich die Teillösungsmenge \mathbb{L}_2 = \left]-\infty ; 2\right[.
Zahlenstrahl mit Bedingungen

Ergebnis: Für die Gesamtlösungsmenge müssen wir noch die Teillösungsmengen vereinigen: \mathbb{L} = \mathbb{L}_1 \cup \mathbb{L}_2 = \left]-\infty ; 2\right[ \cup \left[4;\infty\right[
Lösungsmenge gesamt

Bemerkung 1: Die Gesamtlösungsmenge lässt sich leider nicht einfacher aufschreiben ...

Bemerkung 2: Oben wurde erläutert, dass bei einer Fallunterscheidung immer alle Werte des Definitionsbereichs berücksichtigt werden müssen. In diesem Beispiel sieht es vielleicht auf den ersten Blick so aus, als hätten wir 4x-8=0 bei der Fallunterscheidung "vergessen". Allerdings steht 4x-8 in dieser Ungleichung im Nenner und dieser darf ja von vorneherein nicht 0 sein. Daher hatten wir den Fall 4x-8=0 über die Bestimmung des Definitionsbereichs schon ausgeschlossen.

Übersicht:

 

11.3 Ungleichungen - Lösungen

1. Aufgabe

Eine Bemerkung vorab: Der Definitionsbereich der Aufgaben 1) bis 11) umfasst jeweils die gesamten reellen Zahlen, weil es sich hier ausschließlich um lineare und quadratische Ungleichungen handelt. Die darin enthaltenen Potenzen kann man uneingeschränkt für jede reelle Zahl bilden. Erst ab Aufgabe 12) gibt es für den Definitionsbereich Einschränkungen.

1)
Definitionsbereich: \mathbb{D} = \mathbb{R}

Lösung der Ungleichung:
\begin{array}{rclcl}2x + 5 & \leq & x-25 & \vert & -5 -x \\x & \leq & -30\end{array}

Ergebnis: \mathbb{L} = \left]-\infty;-30\right]


2)
Definitionsbereich: \mathbb{D} = \mathbb{R}

Lösung der Ungleichung:
\begin{array}{rclcl}6 & \leq & \dfrac{x}{2}+5 & \vert & \cdot 2 \\12 & \leq & x + 10 & \vert & -10 \\2 & \leq & x \\x & \geq & 2\end{array}

Ergebnis: \mathbb{L} = \left[2;\infty\right[


3)
Definitionsbereich: \mathbb{D} = \mathbb{R}

Lösung der Ungleichung:
\begin{array}{rclcl}14(-x+2) & \leq & (4+2x) \cdot 7 \\-14x+28 & \leq & 28+14x & \vert & -28-14x \\-28x & \leq & 0 & \vert & :\left(-28\right) \\x & \geq & 0\end{array}

Ergebnis: \mathbb{L} = \left[0;\infty\right[ = \mathbb{R}^+_0


4)
Definitionsbereich: \mathbb{D} = \mathbb{R}

Lösung der Ungleichung:
\begin{array}{rclcl}8\left(\dfrac{1}{2}-\dfrac{3}{4}x\right) -3 & > & 5-4x \\4-6x-3 & > & 5-4x \\1-6x & > & 5-4x & \vert & +4x-1 \\-2x &>& 4 & \vert & :\left(-2\right) \\x & < &-2\end{array}

Ergebnis: \mathbb{L} = \rbrack -\infty;-2\lbrack


5)
Definitionsbereich: \mathbb{D} = \mathbb{R}

Lösung der Ungleichung:
\begin{array}{rclcl}\dfrac{3-2x}{6} & < & \dfrac{2-x}{2}-1 & \vert & \cdot 6 \\\\3-2x & < & 6\left(\dfrac{2-x}{2}-1\right) \\3-2x & < & 3\left(2-x\right)-6 \\3-2x & < & 6-3x-6 \\3-2x & < & -3x & \vert & +3x-3 \\x & < & -3\end{array}

Ergebnis: \mathbb{L} = \rbrack-\infty;-3\lbrack

Bemerkung: In dieser Ungleichung sind zwar Brüche enthalten, da in den Nennern aber nur natürliche Zahlen stehen, ist das unproblematisch. Es reicht, die ursprüngliche Ungleichung mit 6 zu multiplizieren, damit beide Nenner "verschwinden".


6)
Definitionsbereich: \mathbb{D} = \mathbb{R}

Lösung der Ungleichung:
\begin{array}{rclcl}(2x-11)\cdot 10 & \geq & -67+4x-8(4-2x) \\20x-110 & \geq & -67+4x-32+16x \\20x-110 & \geq & -99+20x & \vert & -20x \\-110 & \geq & -99\end{array}

Da -110 nun mal nicht größer oder gleich -99 ist, hat diese Ungleichung keine Lösung.

Ergebnis: \mathbb{L} = \emptyset


7)
Definitionsbereich: \mathbb{D} = \mathbb{R}

Lösung der Ungleichung:
\begin{array}{rclcl}(2x+3)^2-13 & \geq & (x-5)(x+5)+3x(x-3) \\4x^2+12x+9-13 & \geq & x^2-25+3x^2-9x \\4x^2+12x-4 & \geq & 4x^2-9x-25 & \vert & -4x^2 +9x +4 \\21x & \geq & -21 & \vert & :21 \\x & \geq & -1\end{array}

Ergebnis: \mathbb{L} = \lbrack -1;\infty \lbrack

Bemerkung: Diese Ungleichung sieht zwar am Anfang quadratisch aus. Im Laufe der Rechnung heben sich die quadratischen Terme aber gegenseitig auf. Es bleibt eine lineare Ungleichung übrig, die "normal" zu Ende gerechnet werden kann.


8)
Definitionsbereich: \mathbb{D} = \mathbb{R}

Lösung der Ungleichung:
\begin{array}{rclcl}\dfrac{t^2}{2}-3t &>& 56 & \vert & \cdot 2 \\t^2-6t &>& 112 & \vert & -112 \\t^2-6t-112 &>& 0\end{array}

Um diese quadratische Ungleichung zu lösen, wird im ersten Schritt die zugehörige quadratische Gleichung gelöst:
\begin{array}{rclcl}t^2-6t-112 & = & 0 &\vert& \text{p-q-Formel}\\t_{1,2} &=& 3\pm\sqrt{9+112} \\\\t_1 &=& 3+11 = 14 \\t_2 &=& 3-11 = -8\end{array}

Diese Werte liefern uns drei Intervalle:
Von -\infty bis t_2: I_1=\;\rbrack-\infty;-8\lbrack
Zwischen t_2 und t_1: I_2=\;\rbrack-8;14\lbrack
Von t_1 bis +\infty : I_3=\;\rbrack14;\infty\lbrack

Prüfen wir nun, für welche Intervalle die Ungleichung erfüllt ist:

Für I_1 wählen wir -10:
\begin{array}{rclcl}\dfrac{(-10)^2}{2}-3\cdot(-10) & > & 56 \\50+30 & > & 56 \\80 & > & 56\end{array}
Es handelt sich um eine wahre Aussage. Also ist I_1 Teil der Lösungsmenge.

Für I_2 wählen wir 0:
\begin{array}{rclcl}\dfrac{0^2}{2}-3\cdot 0 & > & 56 \\0+0 & > & 56 \\0 & > & 56\end{array}
Es handelt sich um eine falsche Aussage. Also ist I_2 nicht Teil der Lösungsmenge.

Für I_3 wählen wir 20:
\begin{array}{rclcl}\dfrac{20^2}{2}-3\cdot 20 & > & 56 \\200-60 & > & 56 \\140 & > & 56\end{array}
Es handelt sich um eine wahre Aussage. Also ist I_3 Teil der Lösungsmenge.

Ergebnis: Zur Bestimmung der Gesamtlösung müssen wir noch die Intervalle, die als Lösung infrage kommen, vereinigen. In diesem Fall sind das I_1 und I_3: \mathbb{L}=I_1 \cup I_3=\;\rbrack-\infty;-8\lbrack \; \cup \;\rbrack14;\infty\lbrack


9)
Definitionsbereich: \mathbb{D} = \mathbb{R}

Lösung der Ungleichung:
\begin{array}{rclcl}2x^2 &>& -8+7x & \vert & -7x+8 \\2x^2-7x+8 & > & 0 & \vert & :2 \\x^2-\dfrac{7}{2}x+4 & > & 0\end{array}

Um diese quadratische Ungleichung zu lösen, wird im ersten Schritt die zugehörige quadratische Gleichung gelöst:
\begin{array}{rclcl}x^2-\dfrac{7}{2}x+4 &=& 0 &\vert& \text{p-q-Formel}\\x_{1,2} & = & \dfrac{7}{4} \pm\sqrt{\dfrac{49}{16}-4} \\& = & \dfrac{7}{4} \pm\sqrt{-\dfrac{15}{16}}\end{array}

Wir stellen also fest, dass die quadratische Gleichung im Bereich der reellen Zahlen keine Lösungen hat. x^2-\frac{7}{2}x+4 ist also immer ungleich 0 . Auf die ursprüngliche Ungleichung bezogen bedeutet das, dass es keine Werte für x gibt, an denen der Term 2x^2 genau so groß wie der Term -8+7x ist. Das wiederum bedeutet: Im gesamten Definitionsbereich ist 2x^2 entweder immer größer oder immer kleiner als -8+7x. Die Möglichkeit, dass 2x^2 nur an einigen Stellen größer und an anderen Stellen kleiner ist, besteht nicht, weil dafür zwischenzeitlich Gleichheit zwischen beiden Termen herrschen müsste, was wir gerade ausgeschlossen haben.

Deswegen ergibt sich für die weitere Prüfung auch nur ein Intervall, das dem Definitionsbereich \mathbb{D} = \mathbb{R} entspricht: I=\;\rbrack-\infty;+\infty\lbrack

Prüfen wir nun, ob für das Intervall I die Ungleichung erfüllt ist. Hierzu wählen wir den Wert 0:
\begin{array}{rclcl}2 \cdot 0^2 & > & -8 + 7 \cdot 0 \\0 & > & -8\end{array}
Es handelt sich um eine wahre Aussage. Also ist I Teil der Lösungsmenge. Da wir nur ein Intervall zu betrachten hatten und demzufolge keine weiteren Intervalle zur Lösungsmenge hinzukommen können, ist I gleich der Lösungsmenge. Da I gleich dem Definitionsbereich ist, entspricht auch die Lösungsmenge dem Definitionsbereich. Die Ungleichung ist also für alle Werte x\in\mathbb{D} korrekt.

Ergebnis: \mathbb{L}=\mathbb{R}

Bemerkung: Man kann sich das Ganze auch grafisch veranschaulichen. Wenn Sie den Graphen der Funktion f(x)=x^2-\frac{7}{2}x+4 zeichnen, sehen Sie, dass die Parabel so weit nach oben verschoben ist, dass sie keine Nullstellen hat. Das entspricht der Tatsache, dass x^2-\frac{7}{2}x+4 immer größer als 0 ist. Wenn Sie sich die Graphen von f_L(x) =2x^2 und f_R(x)=-8+7x ansehen, werden Sie analog dazu feststellen, dass f_L(x) immer oberhalb von f_R(x) liegt, sodass es keine gemeinsamen Punkte gibt.


10)
Definitionsbereich: \mathbb{D} = \mathbb{R}

Lösung der Ungleichung:
\begin{array}{rclcl}16x^2-64 & < & -48x-120 &\vert& +120 \cr 16x^2+56 & < & -48x &\vert& +48x \cr 16x^2+48x+56 & < & 0 &\vert& :16 \cr x^2+3x+\dfrac{7}{2} & < & 0\end{array}

Um diese quadratische Ungleichung zu lösen, wird im ersten Schritt die zugehörige quadratische Gleichung gelöst:
\begin{array}{rclcl} x^2+3x+\dfrac{7}{2} &=& 0 &\vert& \text{p-q-Formel} \cr\cr x_{1,2} &=& -\dfrac{3}{2}\pm \sqrt{\dfrac{9}{4}-\dfrac{7}{2}} \cr\cr x_{1,2} &=& -\dfrac{3}{2} \pm \sqrt{-\dfrac{5}{4}} \cr\cr \end{array}

Da die quadratische Gleichung keine reelle Lösung hat, lässt sich für die weitere Prüfung nur ein Intervall I ableiten, das dem Definitionsbereich \mathbb{D} = \mathbb{R} entspricht: I=\;\rbrack-\infty;+\infty\lbrack

Prüfen wir nun, ob für das Intervall I die Ungleichung erfüllt ist. Hierzu wählen wir den Wert 0:
\begin{array}{rcl}16\cdot0^2 - 64 & < & -48\cdot 0 - 120 \cr-64 & < & -120\end{array}
Es handelt sich um eine falsche Aussage. Also ist I nicht Teil der Lösungsmenge. Da keine weiteren Intervalle existieren, die Teil der Lösungsmenge sein könnten, gibt es offensichtlich keinen einzigen Wert für x, für den die Ungleichung lösbar ist. Somit entspricht die Lösungsmenge der leeren Menge.

Ergebnis: \mathbb{L}= \emptyset


11)
Definitionsbereich: \mathbb{D} = \mathbb{R}

Lösung der Ungleichung:
\begin{array}{rcl}x^2+6x+8 &\geq & 0 \end{array}

Um diese quadratische Ungleichung zu lösen, wird im ersten Schritt die zugehörige quadratische Gleichung gelöst:
\begin{array}{rclcl} x^2+6x+8 &=& 0 &\vert& \text{p-q-Formel} \cr x_{1,2} &=& -3\pm\sqrt{9-8} \cr\cr x_1 &=& -3+1 = -2 \cr x_2 &=& -3-1 = -4 \end{array}

Diese Werte liefern uns drei Intervalle:
Von -\infty bis x_2: I_1=\;\rbrack-\infty;-4\lbrack
Zwischen x_2 und x_1: I_2=\;\rbrack-4;-2\lbrack
Von x_1 bis +\infty : I_3=\;\rbrack-2;\infty\lbrack

Prüfen wir nun, für welche Intervalle die Ungleichung erfüllt ist:

Für I_1 wählen wir -5:
\begin{array}{rclcl}(-5)^2+6 \cdot (-5)+8 &\geq& 0 \cr 3 &\geq& 0\end{array}
Es handelt sich um eine wahre Aussage. Also ist I_1 Teil der Lösungsmenge.

Für I_2 wählen wir -3:
\begin{array}{rclcl}(-3)^2+6 \cdot (-3)+8 &\geq& 0 \cr -1 &\geq& 0 \cr\cr\end{array}
Es handelt sich um eine falsche Aussage. Also ist I_2 nicht Teil der Lösungsmenge.

Für I_3 wählen wir 0:
\begin{array}{rclcl}0^2+6 \cdot 0+8 &\geq& 0 \cr 8 &\geq& 0 \cr\cr\end{array}
Es handelt sich um eine wahre Aussage. Also ist I_3 Teil der Lösungsmenge.

Da die Ungleichung ein Größergleichzeichen enthält, sind auch die oben berechneten Nullstellen der quadratischen Gleichung Teil der Lösungsmenge.

Ergebnis: \mathbb{L}=I_1 \cup \{-4\} \cup \{-2\} \cup I_3 =\;\rbrack-\infty;-4\rbrack \; \cup \;\lbrack-2;\infty\lbrack

 

Bemerkung für die folgenden Aufgaben: Bei den folgenden Ungleichungen müssen wir uns den Definitionsbereich \mathbb{D} genauer anschauen, weil nun Variablen im Nenner stehen und bekanntermaßen durch 0 nicht geteilt werden darf. Wir müssen also prüfen, für welche Werte der Variable der Nenner nicht 0 wird. Für die Überprüfung setzen wir jeweils den Nennerterm ungleich 0, lösen die zugehörige Ungleichung und leiten aus dem Ergebnis den Definitionsbereich ab.


12)
Bestimmung des Definitionsbereichs:
\begin{array}{rcl}x &\neq & 0\end{array}

Der Definitionsbereich ist also \mathbb{D} = \mathbb{R}\setminus_{\{0\}}.

Lösung der Ungleichung:
\dfrac{1}{x} \geq 5

Bei dieser Ungleichung ist eine Fallunterscheidung notwendig, da wir mit x multiplizieren müssen. 

Fall 1: Wir nehmen an, dass x>0 ist:
\begin{array}{rclcl}\dfrac{1}{x} & \geq & 5 & \vert & \cdot x \\1 & \geq & 5x &\vert& :5 \\\dfrac{1}{5} & \geq & x\end{array}
Der Abgleich der Bedingung x > 0 mit dem Ergebnis x \leq \dfrac{1}{5} führt zu folgender Teillösungsmenge: \mathbb{L}_1 = \left] 0;\dfrac{1}{5}\right]

Fall 2: Wir nehmen an, dass x < 0 ist:
\begin{array}{rclcl}\dfrac{1}{x} & \geq & 5 &\vert & \cdot x \\1 & \leq & 5x & \vert & : 5 \\\dfrac{1}{5} & \leq & x\end{array}
Der Abgleich der Bedingung x < 0 mit dem Ergebnis x \geq \dfrac{1}{5} führt zu folgender Teillösungsmenge: \mathbb{L}_2=\emptyset

Ergebnis: Die Gesamtlösungsmenge ist die Vereinigung der Teillösungsmengen: \mathbb{L} = \mathbb{L}_1 \cup \mathbb{L}_2 = \left] 0;\dfrac{1}{5} \right] \cup \emptyset = \left] 0;\dfrac{1}{5} \right]

Bemerkung: Den Fall x=0 müssen wir nicht betrachten, weil er schon durch den Definitionsbereich ausgeschlossen ist. Durch 0 darf man ja nun mal nicht teilen ...


13)
Bestimmung des Definitionsbereichs:
\begin{array}{rclll} x+3 &\neq& 0 &\vert& -3 \\x &\neq& -3\end{array}

Der Definitionsbereich ist also \mathbb{D} = \mathbb{R}\setminus_{\{-3\}}.

Lösung der Ungleichung:
\dfrac{1}{x+3} < -5

Bei dieser Ungleichung ist eine Fallunterscheidung notwendig, da wir mit \left(x+3\right) multiplizieren müssen.

Fall 1: Wir nehmen an, dass x+3>0, also x>-3, ist:
\begin{array}{rclcll}\dfrac{1}{x+3} & < & -5 &\vert & \cdot\left(x+3\right)\\1 & < & -5\left(x+3\right)\\1 & < & -5x-15 &\vert & +15\\16 & < & -5x &\vert & :\left(-5\right)\\-\dfrac{16}{5} &>& x\end{array}
Der Abgleich der Bedingung x > -3 mit dem Ergebnis x < -\dfrac{16}{5} führt zu folgender Teillösungsmenge: \mathbb{L}_1 = \emptyset

Fall 2: Wir nehmen an, dass x+3 < 0, also x < -3, ist:
\begin{array}{rclcll} \dfrac{1}{x+3} & < & -5 &\vert & \cdot\left(x+3\right)\\1 &>& -5\left(x+3\right)\\1 &>& -5x-15 &\vert & +15\\16 &>& -5x &\vert & :\left(-5\right)\\-\dfrac{16}{5} & < & x \end{array}
Der Abgleich der Bedingung x < -3 mit dem Ergebnis x > -\dfrac{16}{5} führt zu folgender Teillösungsmenge: \mathbb{L}_2 = \left]-\dfrac{16}{5};-3\right[

Ergebnis: \mathbb{L}=\mathbb{L}_1 \cup \mathbb{L}_2 = \emptyset \cup \left]-\dfrac{16}{5};-3\right[ = \left]-\dfrac{16}{5};-3\right[


14)
Bestimmung des Definitionsbereichs:
\begin{array}{rclll} -y+13 &\neq& 0 &\vert& +y \\13 &\neq& y\end{array}

Der Definitionsbereich ist also \mathbb{D} = \mathbb{R}\setminus_{\left\{13\right\}}.

Lösung der Ungleichung:
\dfrac{20}{-y+13} \leq 15

Bei dieser Ungleichung ist eine Fallunterscheidung notwendig, da wir mit \left(-y+13\right) multiplizieren müssen.

Fall 1: Wir nehmen an, dass -y + 13 > 0, also y < 13, ist:
\begin{array}{rclcll}\dfrac{20}{-y+13} &\leq & 15 &\vert & \cdot\left(-y+13\right)\\20 &\leq & 15\left(-y+13\right)\\20 &\leq & -15y+195 &\vert & -195\\-175 &\leq & -15y &\vert & :\left(-15\right)\\\dfrac{35}{3} &\geq & y\end{array}
Der Abgleich der Bedingung y < 13 mit dem Ergebnis y\leq \dfrac{35}{3} führt zu folgender Teillösungsmenge: \mathbb{L}_1 = \left]-\infty;\dfrac{35}{3}\right]

Fall 2: Wir nehmen an, dass -y + 13 < 0, also y > 13, ist:
\begin{array}{rclcll}\dfrac{20}{-y+13} &\leq & 15 &\vert & \cdot\left(-y+13\right)\\20 &\geq & 15\left(-y+13\right)\\20 &\geq & -15y+195 &\vert & -195\\-175 &\geq & -15y &\vert & :\left(-15\right)\\\dfrac{35}{3} &\leq & y\end{array}
Der Abgleich der Bedingung y > 13 mit dem Ergebnis y\geq \dfrac{35}{3} führt zu folgender Teillösungsmenge: \mathbb{L}_2 = \left]13;\infty\right[

Ergebnis: \mathbb{L}=\mathbb{L}_1 \cup \mathbb{L}_2=\left]-\infty;\dfrac{35}{3}\right] \cup \left]13;\infty\right[


15)
Bestimmung des Definitionsbereichs:
\begin{array}{rclll} 2x+3 &\neq& 0 &\vert& -3 \\2x &\neq& -3 &\vert& :2 \\x &\neq& -\dfrac{3}{2}\end{array}

Der Definitionsbereich ist also \mathbb{D} = \mathbb{R}\setminus_{\left\{-\frac{3}{2}\right\}}.

Lösung der Ungleichung:
\dfrac{5x-7}{2x+3} < 4

Bei dieser Ungleichung ist eine Fallunterscheidung notwendig, da wir mit \left(2x+3\right) multiplizieren müssen.

Fall 1: Wir nehmen an, dass 2x + 3 > 0, also x > -\dfrac{3}{2}, ist:
\begin{array}{rclcll}\dfrac{5x-7}{2x+3} & < & 4 &\vert & \cdot\left(2x+3\right)\\5x-7 & < & 4\left(2x+3\right)\\5x-7 & < & 8x+12 &\vert & -12-5x\\-19 & < & 3x &\vert & :3\\-\dfrac{19}{3} & < & x\end{array}
Der Abgleich der Bedingung x > -\dfrac{3}{2} mit dem Ergebnis x > -\dfrac{19}{3} führt zu folgender Teillösungsmenge: \mathbb{L}_1 = \left]-\dfrac{3}{2};\infty\right[

Fall 2: Wir nehmen an, dass 2x + 3 < 0, also x < -\dfrac{3}{2}, ist:
\begin{array}{rclcll}\dfrac{5x-7}{2x+3} & < & 4 &\vert & \cdot\left(2x+3\right)\\5x-7 &> & 4\left(2x+3\right)\\5x-7 &>& 8x+12 &\vert & -12-5x\\-19 &>& 3x &\vert & :3\\-\dfrac{19}{3} &> & x\end{array}
Der Abgleich der Bedingung x < -\dfrac{3}{2} mit dem Ergebnis x < -\dfrac{19}{3} führt zu folgender Teillösungsmenge: \mathbb{L}_2 = \left]-\infty;-\dfrac{19}{3}\right[

Ergebnis: \mathbb{L}=\mathbb{L}_1 \cup \mathbb{L}_2=\left]-\infty;-\dfrac{19}{3}\right[ \cup \left]-\dfrac{3}{2};\infty\right[


16)
Bestimmung des Definitionsbereichs:
\begin{array}{rclll}4+2x &\neq& 0 &\vert& -4 \\2x &\neq& -4 &\vert& :2\\x &\neq& -2\end{array}

Der Definitionsbereich ist also \mathbb{D} = \mathbb{R}\setminus_{\left\{-2\right\}}.

Lösung der Ungleichung:
\begin{array}{rclcll}2-\dfrac{6(x+3)}{4+2x} &>& 0 &\vert & +\dfrac{6(x+3)}{4+2x}\\2 &>& \dfrac{6(x+3)}{4+2x} \end{array}

Bei dieser Ungleichung ist eine Fallunterscheidung notwendig, da wir mit (4+2x) multiplizieren müssen.

Fall 1: Wir nehmen an, dass 4+2x > 0, also x > -2, ist:
\begin{array}{rclcll}2 & > & \dfrac{6(x+3)}{4+2x} & \vert & \cdot(4+2x)\\2(4+2x) & > & 6(x+3) \\8+4x & > & 6x+18 &\vert & -4x -18\\-10 & > & 2x &\vert & :2\\-5 & > & x\end{array}
Der Abgleich der Bedingung x > -2 mit dem Ergebnis x < -5 führt zu folgender Teillösungsmenge: \mathbb{L}_1 = \emptyset

Fall 2: Wir nehmen an, dass 4+2x < 0, also x < -2, ist:
\begin{array}{rclcll}2 & > & \dfrac{6(x+3)}{4+2x} & \vert & \cdot(4+2x)\\2(4+2x) & < & 6(x+3) \\8+4x & < & 6x+18 &\vert & -4x -18\\-10 & < & 2x &\vert & :2\\-5 & < & x\end{array}
Der Abgleich der Bedingung x < -2 mit dem Ergebnis x > -5 führt zu folgender Teillösungsmenge: \mathbb{L}_2 = \left]-5;-2\right[

Ergebnis: \mathbb{L}=\mathbb{L}_1 \cup \mathbb{L}_2=\emptyset \cup \left]-5;-2\right[ = \left]-5;-2\right[


17)
Bestimmung des Definitionsbereichs:
1. linker Nenner:
\begin{array}{rclll}x+5 &\neq& 0 &\vert& -5 \\x &\neq& -5\end{array}

2. linker Nenner:
\begin{array}{rclll}x-5 &\neq& 0 &\vert& +5 \\x &\neq& 5\end{array}

Der Definitionsbereich ist also \mathbb{D} = \mathbb{R}\setminus_{\left\{-5; 5\right\}}.

Lösung der Ungleichung:
\dfrac{1}{x+5}-\dfrac{1}{x-5}>-1

Bei dieser Ungleichung ist eine Fallunterscheidung notwendig, da wir mit (x+5) multiplizieren müssen.

Fall 1: Wir nehmen an, dass x + 5 > 0, also x > -5, ist:
\begin{array}{rclcl}\dfrac{1}{x+5}-\dfrac{1}{x-5} & > & -1 &\vert & \cdot\left(x+5\right) \cr\cr 1-\dfrac{x+5}{x-5} & > & -(x+5)\end{array}

Hier ist eine weitere Fallunterscheidung notwendig, da wir auch noch mit (x-5) multiplizieren müssen.

Fall 1.1: Wir nehmen an, dass x - 5 > 0, also x > 5, ist:
\begin{array}{rclcl}1-\dfrac{x+5}{x-5} & > & -(x+5) & \vert & \cdot \left(x-5\right) \\x-5 - (x+5) & > & -(x+5)(x-5) \\x-5-x-5 & > & -\left(x^2 -25\right) \\-10 & > & -x^2 + 25 & \vert & -25 \\-35 & > & -x^2 & \vert & +x^2 +35 \\x^2 & > & 35 &\vert & \sqrt{} \\\vert x\vert &>& \sqrt{35} \approx 5{,}92\end{array}
Es ergeben sich also zwei mögliche Lösungsintervalle: x < -\sqrt{35} oder x > \sqrt{35}.

Der Abgleich der Bedingungen x > -5 und x > 5 mit dem Ergebnis x < -\sqrt{35} oder x > \sqrt{35} führt zu folgender Teillösungsmenge: \mathbb{L}_{1.1} = \left]\sqrt{35};\infty\right[

Fall 1.2: Wir nehmen an, dass x - 5 < 0, also x < 5, ist:
\begin{array}{rclcl}1-\dfrac{x+5}{x-5} & > & -(x+5) & \vert & \cdot \left(x-5\right) \\x-5 - (x+5) & < & -(x+5)(x-5) \\x-5-x-5 & < & -\left(x^2 -25\right) \\-10 & < & -x^2 + 25 & \vert & -25 \\-35 & < & -x^2 & \vert & +x^2 +35 \\x^2 & < & 35 &\vert & \sqrt{} \\\vert x\vert & < & \sqrt{35} \approx 5{,}92\end{array}
Es ergibt sich das mögliche Lösungsintervall: -\sqrt{35} < x < \sqrt{35}.

Der Abgleich der Bedingungen x > -5 und x < 5 mit dem Ergebnis -\sqrt{35} < x < \sqrt{35} führt zu folgender Teillösungsmenge: \mathbb{L}_{1.2} = \left]-5;5\right[

Fall 2: Wir nehmen an, dass x + 5 < 0, also x < -5, ist:
\begin{array}{rclcl}\dfrac{1}{x+5}-\dfrac{1}{x-5} & > & -1 &\vert & \cdot\left(x+5\right) \cr\cr 1-\dfrac{x+5}{x-5} & < & -(x+5)\end{array}

Hier ist eine weitere Fallunterscheidung notwendig, da wir auch noch mit (x-5) multiplizieren müssen.

Fall 2.1: Wir nehmen an, dass x - 5 > 0, also x > 5, ist:
Da sich schon die Bedingungen von Fall 2 und Fall 2.1, nämlich x < -5 und x > 5, widersprechen, müssen wir diesen Fall nicht weiter bearbeiten: \mathbb{L}_{2.1} = \emptyset

Fall 2.2: Wir nehmen an, dass x - 5 < 0, also x < 5, ist:

\begin{array}{rclcl}1-\dfrac{x+5}{x-5} & < & -(x+5) & \vert & \cdot \left(x-5\right) \\x-5 - (x+5) & > & -(x+5)(x-5) \\x-5-x-5 & > & -\left(x^2 -25\right) \\-10 & > & -x^2 + 25 & \vert & -25 \\-35 & > & -x^2 & \vert & +x^2 +35 \\x^2 & > & 35 &\vert & \sqrt{} \\\vert x\vert &>& \sqrt{35} \approx 5{,}92\end{array}
Es ergeben sich also zwei mögliche Lösungsintervalle: x < -\sqrt{35} oder x > \sqrt{35}.

Der Abgleich der Bedingungen x < -5 und x < 5 mit dem Ergebnis x < -\sqrt{35} oder x > \sqrt{35} führt zu folgender Teillösungsmenge: \mathbb{L}_{2.2} = \left]-\infty;-\sqrt{35}\right[

Ergebnis:
\begin{array}{rcl} \mathbb{L} &=& \mathbb{L}_{1.1} \cup \mathbb{L}_{1.2} \cup \mathbb{L}_{2.1} \cup \mathbb{L}_{2.2} \cr &=& \left]-\infty;-\sqrt{35}\right[ \cup \left]-5;5\right[ \cup \emptyset \cup \left]\sqrt{35};\infty\right[ \cr &=& \left]-\infty;-\sqrt{35}\right[ \cup \left]-5;5\right[ \cup \left]\sqrt{35};\infty\right[ \end{array}


18)
Bestimmung des Definitionsbereichs:
1. linker Nenner:
\begin{array}{rclll}3+x &\neq& 0 &\vert& -3 \\x &\neq& -3\end{array}

2. linker Nenner:
\begin{array}{rclll}3-x &\neq& 0 &\vert& +x \\3 &\neq& x\end{array}

Der Definitionsbereich ist also \mathbb{D} = \mathbb{R}\setminus_{\left\{-3; 3\right\}}.

Lösung der Ungleichung:
\dfrac{3x}{3+x}+\dfrac{2x}{3-x}>1

Bei dieser Ungleichung ist eine Fallunterscheidung notwendig, da wir mit (3+x) multiplizieren müssen.

Fall 1: Wir nehmen an, dass 3 + x > 0, also x > -3, ist:
\begin{array}{rclcl}\dfrac{3x}{3+x}+\dfrac{2x}{3-x} & > & 1 &\vert & \cdot\left(3+x\right) \cr\cr 3x+\dfrac{2x(3+x)}{3-x} & > & 3+x\end{array}

Hier ist eine weitere Fallunterscheidung notwendig, da wir auch noch mit (3-x) multiplizieren müssen.

Fall 1.1: Wir nehmen an, dass 3 - x > 0, also x < 3, ist:
\begin{array}{rclcl}3x+\dfrac{2x(3+x)}{3-x} & > & 3+x & \vert & \cdot \left(3-x\right) \\3x(3-x) + 2x(3+x) & > & (3+x)(3-x) \\9x-3x^2+6x+2x^2 & > & 9 - x^2 \\15x - x^2 & > & 9 - x^2 & \vert & +x^2 \\15x & > & 9 & \vert & :15 \\x & > & \dfrac{3}{5}\end{array}
Der Abgleich der Bedingungen x > -3 und x < 3 mit dem Ergebnis x > \dfrac{3}{5} führt zu folgender Teillösungsmenge: \mathbb{L}_{1.1} = \left]\dfrac{3}{5};3\right[

Fall 1.2: Wir nehmen an, dass 3 - x < 0, also x > 3, ist:
\begin{array}{rclcl}3x+\dfrac{2x(3+x)}{3-x} & > & 3+x & \vert & \cdot \left(3-x\right) \\3x(3-x) + 2x(3+x) & < & (3+x)(3-x) \\9x-3x^2+6x+2x^2 & < & 9 - x^2 \\15x - x^2 & < & 9 - x^2 & \vert & +x^2 \\15x & < & 9 & \vert & :15 \\x & < & \dfrac{3}{5}\end{array}
Der Abgleich der Bedingungen x > -3 und x > 3 mit dem Ergebnis x < \dfrac{3}{5} führt zu folgender Teillösungsmenge: \mathbb{L}_{1.2} = \emptyset

Fall 2: Wir nehmen an, dass 3 + x < 0, also x < -3, ist:
\begin{array}{rclcl}\dfrac{3x}{3+x}+\dfrac{2x}{3-x} & > & 1 &\vert & \cdot\left(3+x\right) \cr\cr 3x+\dfrac{2x(3+x)}{3-x} & < & 3+x\end{array}

Hier ist eine weitere Fallunterscheidung notwendig, da wir auch noch mit (3-x) multiplizieren müssen.

Fall 2.1: Wir nehmen an, dass 3 - x > 0, also x < 3, ist:
\begin{array}{rclcl}3x+\dfrac{2x(3+x)}{3-x} & < & 3+x & \vert & \cdot \left(3-x\right) \\3x(3-x) + 2x(3+x) & < & (3+x)(3-x) \\9x-3x^2+6x+2x^2 & < & 9 - x^2 \\15x - x^2 & < & 9 - x^2 & \vert & +x^2 \\15x & < & 9 & \vert & :15 \\x & < & \dfrac{3}{5}\end{array}
Der Abgleich der Bedingungen x < -3 und x < 3 mit dem Ergebnis x < \dfrac{3}{5} führt zu folgender Teillösungsmenge: \mathbb{L}_{2.1} = \left]-\infty;-3\right[

Fall 2.2: Wir nehmen an, dass 3 - x < 0, also x > 3, ist:
Da sich schon die Bedingungen von Fall 2 und Fall 2.2, nämlich x < -3 und x > 3, widersprechen, müssen wir diesen Fall nicht weiter bearbeiten: \mathbb{L}_{2.2} = \emptyset

Ergebnis:
\begin{array}{rcl} \mathbb{L} &=& \mathbb{L}_{1.1} \cup \mathbb{L}_{1.2} \cup \mathbb{L}_{2.1} \cup \mathbb{L}_{2.2} \cr &=& \left]\dfrac{3}{5};3\right[ \cup \emptyset \cup \left]-\infty;-3\right[\ \cup \emptyset \cr &=& \left]-\infty;-3\right[\ \cup \left]\dfrac{3}{5};3\right[ \end{array}


19)
Bestimmung des Definitionsbereichs:
1. linker Nenner:
\begin{array}{rclll}x-4 &\neq& 0 &\vert& +4 \\x &\neq& 4 \end{array}

2. linker Nenner:
\begin{array}{rclll}x-2 &\neq& 0 &\vert& +2 \\x &\neq& 2\end{array}

Der Definitionsbereich ist also \mathbb{D} = \mathbb{R}\setminus_{\left\{2; 4\right\}}.

Lösung der Ungleichung:
Diese Ungleichung lösen wir auf einem anderen Weg als die Aufgaben 17) und 18), nämlich indem wir die Brüche zuerst subtrahieren, um dann mit dem Hauptnenner zu multiplizieren. Das erspart uns - wie wir sehen werden - eine doppelte Fallunterscheidung. Allerdings ist die Rechnung, bis wir zur Fallunterscheidung kommen, länger, weil (natürlich!) die ganz normalen Regeln für das Rechnen mit Brüchen, Klammern etc. berücksichtigt werden müssen ... Natürlich kann auch bei dieser Aufgabe mit beiden Nennern einzeln multipliziert werden. Beide Wege führen zur selben Lösung.

\begin{array}{rclcll}\dfrac{x-7}{x-4} - \dfrac{2x-5}{x-2} & \leq & 0 \\\\\dfrac{(x-7)(x-2)}{(x-4)(x-2)} - \dfrac{(2x-5)(x-4)}{(x-2)(x-4)} & \leq & 0 \\\\\dfrac{x^2-2x-7x+14}{(x-2)(x-4)} - \dfrac{2x^2-8x-5x+20}{(x-2)(x-4)} & \leq & 0 \\\\\dfrac{x^2-9x+14}{(x-2)(x-4)} - \dfrac{2x^2-13x+20}{(x-2)(x-4)} & \leq & 0 \\\\\dfrac{x^2-9x+14-\left(2x^2-13x+20\right)}{(x-2)(x-4)} & \leq & 0 \\\\\dfrac{x^2-9x+14-2x^2+13x-20}{(x-2)(x-4)} & \leq & 0 \\\\\dfrac{-x^2+4x-6}{(x-2)(x-4)} & \leq & 0\end{array}

Bei dieser Ungleichung ist eine Fallunterscheidung notwendig, da wir mit (x-2)(x-4) multiplizieren müssen.

Fall 1: Wir nehmen an, dass (x-2)(x-4) > 0 ist. Das ist genau dann der Fall, wenn beide Faktoren positiv oder beide Faktoren negativ sind. Somit muss x < 2 oder x > 4 sein:
\begin{array}{rclcll}\dfrac{-x^2+4x-6}{(x-2)(x-4)} & \leq & 0 & \vert & \cdot (x-2)(x-4) \\-x^2 + 4x - 6 & \leq & 0 & \vert & \cdot (-1) \\x^2 - 4x + 6 & \geq & 0 \\\end{array}

Um diese quadratische Ungleichung zu lösen, wird im ersten Schritt die zugehörige quadratische Gleichung gelöst:
\begin{array}{rclcl} x^2 - 4x + 6 &=& 0 &\vert& \text{p-q-Formel} \cr x_{1,2} &=& 2\pm\sqrt{4-6} \cr x_{1,2} &=& 2\pm\sqrt{-2} \cr\cr \end{array}

Da die quadratische Gleichung keine reelle Lösung hat, lässt sich für die weitere Prüfung nur ein Intervall I ableiten, das dem Definitionsbereich \mathbb{D} = \mathbb{R}\setminus_{\left\{2; 4\right\}} entspricht.

Prüfen wir nun, ob für das Intervall I die Ungleichung erfüllt ist. Hierzu wählen wir den Wert 0:
\begin{array}{rclcll}0^2 - 4 \cdot 0 + 6 & \geq & 0 \\6 & \geq & 0\end{array}
Es handelt sich um eine wahre Aussage. Die Ungleichung ist also für alle Werte x\in\mathbb{D} korrekt. Nun müssen wir dieses Ergebnis noch mit der Bedingung von Fall 1, also x < 2 oder x > 4, abgleichen. Daraus ergibt sich folgende Teillösungsmenge: \mathbb{L}_1=\left]-\infty;2\right[\ \cup \left]4;\infty\right[

Fall 2: Wir nehmen an, dass (x-2)(x-4) < 0 ist. Das ist genau dann der Fall, wenn die Faktoren ein unterschiedliches Vorzeichen haben. Das ist für 2 < x < 4 der Fall:
\begin{array}{rclcll}\dfrac{-x^2+4x-6}{(x-2)(x-4)} & \leq & 0 & \vert & \cdot (x-2)(x-4) \\-x^2 + 4x - 6 & \geq & 0 & \vert & \cdot (-1) \\x^2 - 4x + 6 & \leq & 0 \\\end{array}

Wir wissen bereits aus der Betrachtung von Fall 1, dass die zugehörige quadratische Gleichung keine Lösung hat, daher können wir uns das erneute Lösen der quadratischen Gleichung sparen. Außerdem wissen wir auch aus Fall 1, dass x^2 - 4x + 6 immer positiv ist. Somit kann die Ungleichung x^2 - 4x + 6 \leq 0 niemals erfüllt werden. Das führt zu folgender Teillösungsmenge: \mathbb{L}_2=\emptyset

Ergebnis: \mathbb{L}=\mathbb{L}_{1} \cup \mathbb{L}_{2}=\left]-\infty;2\right[\ \cup \left]4;\infty\right[ \cup \emptyset = \left]-\infty;2\right[\ \cup \left]4;\infty\right[


20)
Bestimmung des Definitionsbereichs:
\begin{array}{rclcl}x^2-x-2&=&0 \\x_{1,2}&=&\dfrac{1}{2}\pm\sqrt{\dfrac{1}{4}+2} \\\\x_1 &=& \dfrac{1}{2}+\dfrac{3}{2} = 2 \\x_2 &=& \dfrac{1}{2}-\dfrac{3}{2} = -1\end{array}

Der Definitionsbereich ist also \mathbb{D} = \mathbb{R}\setminus_{\left\{-1;2\right\}}.

Lösung der Ungleichung:
\dfrac{x+1}{x^2-x-2} < 0

Bei dieser Ungleichung ist eine Fallunterscheidung notwendig, da wir mit \left(x^2-x-2\right) multiplizieren müssen.

Fall 1: Wir nehmen an, dass x^2-x-2 > 0, also x < -1 oder x > 2, ist:
\begin{array}{rclcl}\dfrac{x+1}{x^2-x-2} & < & 0 &\vert & \cdot\left(x^2-x-2\right)\\x+1 & < & 0 &\vert & -1 \\x & < & -1\end{array}
Der Abgleich der Bedingung x < -1 oder x > 2 mit dem Ergebnis x < -1 führt zu folgender Teillösungsmenge: \mathbb{L}_1 = \left]-\infty;-1\right[

Dann noch die Frage: Wie kommt man bei der Bedingung des Falls von x^2-x-2 > 0 auf x < -1 oder x > 2?
Interpretiert man den Nennerterm der Ungleichung als quadratische Funktion f(x)=x^2-x-2, so haben wir bei der Bestimmung des Definitionsbereichs deren Nullstellen ermittelt. Da es sich um eine nach oben geöffnete Parabel handelt, müssen die Funktionswerte links von der ersten Nullstelle und rechts von der zweiten Nullstelle positiv sein.

Fall 2: Wir nehmen an, dass x^2-x-2 < 0, also x > -1 und x < 2, ist:

\begin{array}{rclcl}\dfrac{x+1}{x^2-x-2} & < & 0 &\vert & \cdot\left(x^2-x-2\right)\\x+1 & > & 0 &\vert & -1 \\x & > & -1\end{array}
Der Abgleich der Bedingung x > -1 und x < 2 mit dem Ergebnis x > -1 führt zu folgender Teillösungsmenge: \mathbb{L}_2 = \left]-1;2\right[

Ergebnis: \mathbb{L}=\mathbb{L}_1 \cup \mathbb{L}_2=\left]-\infty;-1\right[ \cup \left]-1;2\right[

 

2. Aufgabe

Allgemeiner Lösungsweg:
Um die Lösungsmenge in ein Koordinatensystem zeichnen zu können, benötigen wir die Ungleichung zunächst in der Form y>..., y\ge..., y < ... oder y\le.... Daraus ergibt sich die "Trennlinie" zwischen Lösungsmenge und Nicht-Lösungsmenge. Anschließend sind noch folgende Fragen zu klären:

  1. Liegt die Lösungsmenge ober- oder unterhalb der "Trennlinie"?
    Bei > und \ge liegt die Lösungsmenge oberhalb, bei  < und \le unterhalb der "Trennlinie".
  2. Ist die "Trennlinie" teil der Lösungsmenge oder nicht?
    Bei \ge und \le ist die "Trennlinie" teil der Lösungsmenge, bei > und  < nicht.

 

1)
\begin{array}{rclcl} x+y & \geq & 3 \cr y & \geq & -x+3 \end{array}

Lösungsmenge der Ungleichung im Koordinatensystem


2)
\begin{array}{rclcl} 2x+5y & \leq & 10 & \vert & -2x \cr 5y & \leq & 10-2x & \vert & :5 \cr y & \leq & -\dfrac{2}{5}x+2 \end{array}

Lösungsmenge der Ungleichung im Koordinatensystem


3)
\begin{array}{rclcl} -4x-7y & > & -1 & \vert & +4x \cr -7y & > & -1+4x & \vert & :(-7) \cr y & < & -\dfrac{4}{7}x+\dfrac{1}{7} \end{array}

Lösungsmenge der Ungleichung im Koordinatensystem


4)
\begin{array}{rclcl} 9x-3y & \leq & 24 & \vert & -9x \cr -3y & \leq & 24-9x & \vert & :(-3) \cr y & \geq & 3x-8 \end{array}

Lösungsmenge der Ungleichung im Koordinatensystem


5)
\begin{array}{rclcl} -5x+10y & < & 35 & \vert & +5x \cr 10y & < & 35+5x & \vert & :10 \cr y & < & \dfrac{1}{2}x+\dfrac{7}{2} \end{array}

Lösungsmenge der Ungleichung im Koordinatensystem


6)
\begin{array}{rclcl}6x + 9y & \leq & 18 &\vert& -6x \cr 9y & \leq & -6x +18 &\vert& :9 \cr y & \leq & - \dfrac23 x +2\end{array}

Lösungsmenge der Ungleichung im Koordinatensystem


7)
\begin{array}{rclcl}5y + 3x - 1 & < & 2y &\vert& -2y \cr 3y + 3x - 1 & < & 0 &\vert& -3x+1 \cr 3y & < & - 3x+1 &\vert& :3 \cr y & < & -x +\dfrac13\end{array}

Lösungsmenge der Ungleichung im Koordinatensystem


8)
\begin{array}{rclcl}\dfrac{y}{3} & \geq & x + \dfrac{14}{9} &\vert& \cdot 3 \cr y & \geq & 3x + \dfrac{14}{3}\end{array}

Lösungsmenge der Ungleichung im Koordinatensystem


9)
\begin{array}{rclcl}40x + 60 - 80y & < & 100 &\vert& -40x-60 \cr -80y & < & -40x+40 &\vert& :(-80) \cr y &>& \dfrac12 x - \dfrac12\end{array}

Lösungsmenge der Ungleichung im Koordinatensystem


10)
\begin{array}{rclcl}x + 2y & \leq & -2x - y &\vert& +y-x \cr 3y & \leq & -3x &\vert& :3 \cr y & \leq & -x\end{array}

Lösungsmenge der Ungleichung im Koordinatensystem

 

3. Aufgabe

Im "Beweis" wurde die Fallunterscheidung vergessen. Bei der Multiplikation mit -x kann keine Aussage darüber getroffen werden, ob mit einer positiven oder negativen Zahl multipliziert wird - und das hat bei Ungleichungen ja Auswirkungen auf die Richtung des Vergleichszeichen. Richtig müsste die Rechnung lauten:

1. Fall: Wir nehmen an, dass x>0 ist. Dann ist -x < 0:

\begin{array}{rclcl} x-1 & < & x &\vert&\cdot (-x) \cr -x^2+x &>& -x^2 &\vert& -x^2 \cr x &>& 0 \end{array}

Das ist nun wirklich nicht spektakulär, denn genau dies hatten wir zu Beginn der Rechnung angenommen.

Bemerkung: Die Multiplikation mit -x ist hier ohne Einschränkungen möglich, weil -x < 0. Eine Multiplikation mit 0 kann also nicht passieren.

2. Fall: Wir nehmen an, dass x=0 ist. Diesen Fall müssen wir extra betrachten, weil wir jetzt mit x nicht multiplizieren dürfen:

\begin{array}{rclcl} x-1 & < & x &\vert& -x \cr -1 & < & 0 \end{array}

Geht auch klar.

3. Fall: Wir nehmen an, dass x < 0 ist. Dann ist -x>0:

\begin{array}{rclcl} x-1 & < & x &\vert&\cdot (-x) \cr -x^2+x & < & -x^2 &\vert& -x^2 \cr x & < & 0 \end{array}

Bemerkung: Die Multiplikation mit -x ist hier ohne Einschränkungen möglich, weil -x > 0. Eine Multiplikation mit 0 kann also nicht passieren.

Das ist zwar die gleiche Rechnung, wie im "Beweis" auf der Aufgabenseite; mit der Zusatzinformation, dass wir hier von vorneherein nur Zahlen kleiner als 0 betrachten, erscheint die Aussage aber in einem ganz anderen Licht ...

Zusammenfassend stellen wir fest: Unterscheidet man alle Fälle ordentlich, ergeben sich drei recht belanglose Aussagen. Komische Ergebnisse treten nicht auf.

Wesentlich geschickter: Man lässt das Multiplizieren mit -x sein und subtrahiert x stattdessen lieber. Dann steht da nämlich ganz schnell -1 < 0 , was sicherlich niemand infrage stellen wird. Mehr Ergebnis bekommen wir hier nicht, was auch nicht so überraschend ist, denn die Aussage, mit der wir in den "Beweis" gestartet waren, war ja nicht sehr gehaltvoll ...

12. Eigenschaften und "besondere Punkte" von Funktionen - Lernziele und typische Fehler

Nach Durcharbeiten dieses Kapitels sollten Sie folgende Lernziele erreicht haben:

  • Sie wissen, was mit dem Randverhalten einer Funktion gemeint ist, und können es bei einer gegebenen Funktion beschreiben.
  • Sie können rechnerisch bestimmen, ob eine Funktion achsen- oder punktsymmetrisch ist und diese Information für das Skizzieren des Funktionsgraphen nutzen.
  • Sie kennen die Begriffe "Nullstelle", "Hochpunkt" ("Maximum"), "Tiefpunkt" ("Minimum"), "Wendepunkt", "Sattelpunkt" und "Polstelle" und können diese in Funktionsgraphen erkennen.
  • Sie können Nullstellen berechnen.


Typischer Fehler in diesem Kapitel ist:

  • Funktionen können nicht korrekt skizziert werden. Erklärung


Für Online-Selbsttests zu diesem Thema und weitere Informationen zur Mathematikunterstützung an der TH Wildau nutzen Sie bitte den Moodle-Kursraum "SOS Mathematik - Brückenkurs".

Übersicht:

 

12.1 Eigenschaften und "besondere Punkte" von Funktionen - Aufgaben

Die Aufgaben in diesem Kapitel beschränken sich (größtenteils) auf lineare und quadratische Funktionen, auch wenn man natürlich all diese Konzepte auch für andere Funktionstypen untersuchen kann. Die anderen Funktionstypen müssen Sie ja aber erst kennenlernen, worum wir uns in den folgenden Kapitel kümmern werden.

 

1. Aufgabe

Prüfen Sie die folgenden Funktionen auf Achsensymmetrie zur y-Achse und Punktsymmetrie zum Koordinatenursprung!

1) f(x)=5x^5-3x^3+x

  11) f(x)=-93x^2-72x+11

2) f(x)=6x^2-9x+3

  12) g(x)=\dfrac{1}{3}x^4+12

3) f(x)=13x

  13) f(z)=100x

4) f(x)=10x^8+10x^2+7

  14) f(x)=17x^6-4x^3+46

5) f(x)=12x-6   15) c(r)=-4r^8+83

6) f(x)=x^4 -5x^2+4

  16) g(x)=\dfrac{3}{41}x^{24}

7) f(x)=-x^6+7x^2-25

  17) f(z)=-7z^3 + 12z

8) g(y)=\dfrac{5}{2} y^3+ \dfrac{38}{9} y

  18) f(x)=\dfrac{7}{18}x^6-33x^3-47
9) f(x)=-14x^4+52x^3-13x+1

  19) h(y)=101y^{34}+9y^{12}

10) f(x)=3x^3-33x^2- \dfrac{5}{2} x+2

  20) d(x)=51x^4-\dfrac{63}{5}x^3+223

 

2. Aufgabe

Finden Sie - wenn vorhanden - die Nullstellen der folgenden Funktionen unter Berücksichtigung des Definitionsbereichs!

1) f(x)=x^3-2x^2
mit \mathbb{D}=\mathbb{R}

2) f(x)=7x+4
mit \mathbb{D}=\mathbb{R}^+

3) f(x)=\dfrac{1}{2}x^2-\dfrac{2}{3}x+\dfrac{2}{9}
mit \mathbb{D}=\mathbb{R}

4) f(x)=x^3-\dfrac{7}{12}x^2-\dfrac{5}{24}x
mit \mathbb{D}=\mathbb{R}^+_0

5) f(x)=11x^2+110x+286
mit \mathbb{D}=\mathbb{R}^+_0

 

3. Aufgabe

Finden Sie - wenn vorhanden - die Extrempunkte der folgenden Funktionen unter Berücksichtigung des Definitionsbereichs! Geben Sie zudem an, ob es sich um Hochpunkte oder Tiefpunkte handelt!
Diese Aufgabe ist so gedacht, dass Sie sie mithilfe von Überlegungen und Argumenten lösen - nicht mithilfe der Differenzialrechnung, die wir ja noch nicht besprochen haben.

1) f(x)=-\dfrac{7}{10}x+6
mit \mathbb{D}=\mathbb{R}

  6) f(x)=-13x -2
mit \mathbb{D}=[-1;\infty [
2) f(x)=x^2+4x+8
mit \mathbb{D}=\mathbb{R}_0^-

  7) f(x)=x^2 - 6x + 9
mit \mathbb{D}=\mathbb{R}

3) f(x)=\dfrac{4}{9}x-2
mit \mathbb{D}=[1; \infty[

  8) f(x)=33x + \dfrac{27}{5}
mit \mathbb{D}=\mathbb{R}
4) f(x)=8
mit \mathbb{D}=\mathbb{R}^+

  9) f(x)=42x - 15
mit \mathbb{D}=]0 ; 1[
5) f(x)=-2x^2+4x-2
mit \mathbb{D}=[-2; 0]

  10) f(x)=-8x^2 + 10
mit \mathbb{D}= \left[-\sqrt{2} ; \sqrt{2}\right]

 

4. Aufgabe

Betrachten Sie die folgenden Graphen und stellen Sie fest, ob und wenn ja, wo die Funktionen einen Wende- oder Sattelpunkt haben!

5 Graphen, die eventuell einen Wende- oder Sattelpunkt haben

weitere 5 Graphen, die eventuell einen Wende- oder Sattelpunkt haben

Dieses Kapitel enthält die folgenden Themen:

 

12.2 Eigenschaften und "besondere Punkte" von Funktionen - Erklärungen

Das Wissen über die Eigenschaften und "speziellen Punkte" von Funktionen/Graphen, um das es in diesem Kapitel geht, ist sehr nützlich und hilft häufig viel mehr als eine ausführliche Wertetabelle. Dazu zwei Beispiele:

  • Bei einer Funktion, die achsensymmetrisch zur y-Achse ist, reicht es aus, die Funktionswerte für positive x-Werte (und 0) zu berechnen - die entsprechenden Funktionswerte für negative x-Werte erhält man automatisch. Das erspart eine Menge Arbeit ...
  • Eine Nullstelle bei x=2 wird man vermutlich gut mithilfe einer Wertetabelle finden. Für eine Nullstelle, die bei x=-\sqrt{7} liegt, wird das schon schwieriger ... Vermutlich wird man sie häufig einfach übersehen.

Um solche Erkenntnisse nutzen zu können bzw. Fehler zu vermeiden, ist es enorm wichtig, ein Bild der Funktionsgraphen im Kopf zu haben und ungefähr zu wissen, wie sich ein bestimmter Funktionstyp "verhält". Schauen Sie sich dieses und die nächsten Kapitel daher genau an, um eine Vorstellung von typischen Eigenschaften der verschiedenen Funktionen zu entwickeln. Es soll dabei deutlich werden, wie Funktionsgleichung und Graph zusammenhängen. Vertieft wird dieser Eindruck dann in Ihrem Studium, auch anhand fachbezogener Aufgaben, die auf diesen grundlegenden Aufgaben aufbauen.


Zur Veranschaulichung, warum Funktionsbetrachtungen wichtig sind, hier ein nur leicht abgewandeltes Beispiel einer Klausuraufgabe: "Skizzieren Sie die Funktionen f(x)=-x^2-12 und g(x)=-\frac{3}{2}x-15 und berechnen Sie den Flächeninhalt der eingeschlossenen Fläche!" Wenn Ihre Skizze keine eingeschlossene Fläche enthält, weil Sie einen der Graphen falsch gezeichnet haben, wird die restliche Aufgabe ganz schön schwierig - selbst wenn Sie wissen, wie man die Funktionen integriert!

 

Skizzieren eines Funktionsgraphen

Bevor es richtig losgeht, noch ein paar Worte zum Skizzieren eines Funktionsgraphen:
Sehr wichtig ist der Unterschied zwischen "Skizze" und "Zeichnung": Wenn in einer Aufgabe nach einer Skizze gefragt ist (oder sich ergibt, dass das sinnvoll ist), müssen Sie keine exakte Zeichnung anfertigen, sondern eben eine Skizze, aus der zwar die wesentlichen Eigenschaften des Graphen hervorgehen, die aber ansonsten ein bisschen ungenau sein darf. Für eine ordentliche Zeichnung muss vorab eigentlich eine detaillierte Kurvendiskussion durchgeführt werden. Um die beiden Funktionen von oben "nur" zu skizzieren, überlegen wir uns:
Zu f(x)=-x^2-12:

Zu g(x)=-\frac{3}{2}x-15:

  • Hier haben wir eine lineare Funktion, also ist ihr Funktionsgraph eine Gerade.
  • Die Gerade fällt, da der Koeffizient vor dem x negativ ist.
  • Der Graph schneidet die y-Achse bei -15, also knapp unterhalb des Schnittpunktes der y-Achse mit dem Graphen von f(x).
  • Wer möchte, kann noch den Schnittpunkt mit der x-Achse ausrechnen. Das ist ja bei linearen Funktionen schnell und unkompliziert möglich.

Eine Skizze könnte also folgendermaßen aussehen:

Beispiel einer Skizze
Fertig! Wenn man so an die Fragestellung herangeht, sind Wertetabellen nicht nötig. Vorteil dieser Herangehensweise ist, dass man das "große Ganze" in den Blick nimmt, nämlich den Verlauf des Graphen, und nicht nur einzelne Punkte. Das ist für das Verständnis der Aufgabe meist viel besser. Das "große Ganze" in den Blick zu nehmen ist natürlich auch bei echten Zeichnungen sinnvoll.

Zusätzlich zu der Unterscheidung zwischen Skizze und Zeichnung, sollte man (wie immer) den Aufgabentext gründlich lesen: Wenn in der Aufgabe beispielsweise nur vom ersten Quadranten die Rede ist, reicht natürlich auch eine Skizze von genau diesem Bereich. Man muss sich ja nicht mehr Mühe machen, als nötig ...
Auch die Frage "Zu welchem Zweck erstelle ich die Skizze?" ist hilfreich. Soll die Skizze beispielsweise den Gesamtverlauf der Funktion zeigen oder eher den Bereich rund um den Scheitelpunkt fokussieren?

Zusammengefasst: Erst nachdenken, dann skizzieren oder zeichnen! ;-)

 

Eigenschaften von Funktionen

Randverhalten

Definition: Das Randverhalten beschreibt den Verlauf der Funktion an den Rändern des Definitionsbereichs.
Da beim Randverhalten (wie der Name schon sagt) nur das Verhalten an den Rändern des Definitionsbereichs interessiert, wird hier nichts über den Verlauf "dazwischen" ausgesagt.


Schauen wir uns als Beispiel die Funktion f(x)=\frac{1}{x-2} an. Ihr Definitionsbereich ist \mathbb{D}=\mathbb{R}\setminus_{\{2\}}, denn für x=2 würde der Nenner 0 werden, was ja nicht sein darf. Wenn man den Graphen von f(x) betrachtet, sieht man:

  • 1. Pfeil: Läuft x gegen - \infty (anders gesagt: werden die x-Werte immer kleiner), geht f(x) gegen 0.
  • 2. Pfeil: Läuft x von links in Richtung 2, geht f(x) gegen - \infty.
  • 3. Pfeil: Läuft x von rechts in Richtung 2, geht f(x) gegen +\infty.
  • 4. Pfeil: Läuft x gegen +\infty, geht f(x) gegen 0.

Randverhalten am Beispiel einer Hyperbel

Bemerkung 1: Wie Sie sehen, kann ein Definitionsbereich mehr als zwei Ränder haben.
Bemerkung 2: Um das Randverhalten "mathematisch ordentlich" zu beschreiben, müsste man Grenzwerte betrachten und berechnen, was in diesem Lernmodul nicht gemacht wird.

Die Betrachtung des Randverhaltens ist nützlich, um sich ein Bild vom groben Verlauf des Funktionsgraphen zu machen, und kann dabei helfen, die berechneten "besonderen" Punkte zu plausibilisieren. Wenn Sie beispielsweise an eine Parabel denken, deren Randverhalten "von +\infty nach +\infty" ist, muss der Extrempunkt (siehe unten) ein Tiefpunkt sein. 

 

Symmetrie

Viele Objekte (mathematische wie nicht mathematische) haben Symmetrie- oder Spiegelachsen. Das ist nichts Ungewöhnliches. Hier geht es nun um die mathematische Beschreibung des Symmetrieverhaltens bei Funktionen.

Definition: Eine Funktion f(x) ist achsensymmetrisch zur y-Achse, wenn gilt f(-x)=f(x).

Definition: Eine Funktion f(x) ist punktsymmetrisch zum Koordinatenursprung, wenn gilt -f(-x)=f(x).


In der folgenden Grafik sind die Funktionen f_1(x)=\frac{6}{5}x^2-8 und f_2(x)=4x zu sehen. f_1(x) ist achsensymmetrisch zur y-Achse und f_2(x) punktsymmetrisch zum Koordinatenursprung.

1 achsensymmetrische und 1 punktsymmetrische Funktion


Das kann man natürlich auch mithilfe der in den Definitionen angegebenen Formeln nachweisen:
Achsensymmetrie bei f_1(x):
\begin{array}{rcl}f_1(-x) &=& \dfrac{6}{5}(-x)^2-8 \\\\&=& \dfrac{6}{5}x^2-8 \\\\&=& f_1(x)\end{array}

Punktsymmetrie bei f_2(x):
\begin{array}{rcl}-f_2(-x) &=& -(4(-x)) \\&=& 4x \\&=& f_2(x)\end{array}


Eine Funktion kann auch zu anderen Geraden außer der y-Achse bzw. zu anderen Punkten außer dem Koordinatenursprung symmetrisch sein. Beispielsweise ist die Funktion f(x)=x^3+9x^2+27x+28=(x+3)^3+1 punktsymmetrisch zum Punkt (-3\mid 1) (siehe folgende Grafik).

Funktion. die punktsymmetrisch zu (-3/1) ist

 

"Besondere Punkte" einer Funktion

An die "besonderen Punkte", um die es in den kommenden Abschnitten gehen wird, erinnern sich vermutlich die meisten ...

Schnittpunkte mit den Achsen und Extrempunkte

Definition: Eine Funktion f(x) hat an einer Stelle x_0 eine Nullstelle, wenn f(x_0)=0 ist.
Anders formuliert: Eine Funktion f(x) hat an einer Stelle x_0 eine Nullstelle, wenn die Funktion an der Stelle x_0 die x-Achse schneidet oder berührt.

Definition: Der y-Achsenabschnitt einer Funktion f(x) beschreibt den Schnittpunkt des Funktionsgraphen mit der y-Achse, also den Funktionswert von f(x) bei x=0.

Definition: Eine Funktion f(x) hat an einer Stelle x_0 einen Tiefpunkt, wenn der zu x_0 gehörende Funktionswert f(x_0) in der unmittelbaren Umgebung von x_0 der kleinste Funktionswert ist.

Definition: Eine Funktion f(x) hat an einer Stelle x_0 einen Hochpunkt, wenn der zu x_0 gehörende Funktionswert f(x_0) in der unmittelbaren Umgebung von x_0 der größte Funktionswert ist.

Definition: Eine Funktion f(x) hat an einer Stelle x_0 einen Extrempunkt, wenn die Funktion in x_0 entweder einen Tiefpunkt oder einen Hochpunkt hat.

Wenn man ganz genau sein möchte (und das möchte man in der Mathematik meistens ...), bezeichnet der Begriff "Extremstelle" den x-Wert eines Extrempunktes und der Begriff "Extremum" oder "Extremwert" den zugehörigen y-Wert. Ein Extrempunkt besteht aus dem x-Wert und dem zugehörigen y-Wert.

Selbstverständlich werden hier nur Stellen x_0 betrachtet, die im Definitionsbereich liegen!


In der folgenden Grafik sind die Funktionen f_1(x)=-2x+8, f_2(x)=2x^2-9, f_3(x)=-\frac{1}{4}x^2-2x-5 und f_4(x)=4\sin(x) zu sehen (Mehr zur Funktion f_4(x) erfahren Sie im Kapitel Trigonometrie). Die Nullstellen sind mit braunen, die Schnittpunkte mit der y-Achse mit orangen, die Hochpunkte mit dunkelblauen und die Tiefpunkte mit hellblauen Kreisen markiert.

Graphen mit markieren Achsenschnittpunkten und Extrempunkten

Zur Berechnung von Extremstellen wird in vielen Fällen die Differenzialrechnung benötigt.

Achtung: Wie in der folgenden Grafik zu sehen ist, hängt es auch vom Definitionsbereich ab, ob eine Funktion an einer Stelle einen Extrempunkt hat. Die Grafik zeigt die gleichen Funktionen wie zuvor, allerdings mit \mathbb{D}=\mathbb{R}_0^+ statt \mathbb{D}=\mathbb{R}. Alle Extrempunkte im negativen x-Bereich sind weggefallen. Dafür finden sich an der Stelle x=0, also am unteren Rand des Definitionsbereiches, nun vier Extrempunkte statt wie zuvor nur einer. Solche Extrempunkte heißen - naheliegenderweise - "Randextremwerte".

4 Funktionen mit markierten Nullstellen, Minima und Maxima im nichtnegativen Bereich

 

Um die Nullstellen einer Funktion zu finden, muss der Funktionsterm nullgesetzt und die entstehende Gleichung nach der Variablen aufgelöst werden. Anschließend muss überprüft werden, ob der ermittelte Wert im Definitionsbereich liegt.
Was bedeutet "eine Funktion nullsetzen"?
Setzt man z. B. die Funktion f(x)=x^2-2x+5 null, erhält man die Gleichung 0=x^2-2x+5.

 

Wende- und Sattelpunkte

Definition: Ein Wendepunkt ist ein Punkt, an dem ein Funktionsgraph sein Krümmungsverhalten ändert - von einer Rechtskrümmung ("Rechtskurve") zu einer Linkskrümmung ("Linkskurve") oder umgekehrt.

Definition: Ein Sattelpunkt ist ein Wendepunkt, an dem die Steigung der Funktion 0 ist, d. h. der Graph der Funktion ist an dieser Stelle horizontal.


In der folgenden Grafik sind die Funktionen f_1(x)=\frac{1}{2}x^3+\frac{7}{2}x^2+6x+5 und f_2(x)=\frac{1}{20}x^3-\frac{3}{20}x^2+\frac{3}{20}x-\frac{5}{2} zu sehen. f_1(x) hat einen "normalen" Wendepunkt, f_2(x) einen Sattelpunkt. Beide sind mit orangefarbenen Kreisen markiert.

2 Funktionen mit markiertem Wende- bzw. Sattelpunkt

Wende- und Sattelpunkte lassen sich ohne Differenzialrechnung nicht exakt ermitteln.

 

Polstellen

Definition: Eine Polstelle ist eine Definitionslücke einer Funktion, in deren Umgebung die Funktionswerte beliebig groß oder beliebig klein werden. Etwas salopp formuliert, ist eine Definitionslücke ein "Loch" im Definitionsbereich.


In der folgenden Grafik ist die Funktion f(x)=\frac{1}{x-2} zu sehen. Wir hatten oben schon überlegt, dass ihr Definitionsbereich \mathbb{D}=\mathbb{R}\backslash_{\{2\}} ist. Direkt links neben der Definitionslücke sind die Funktionswerte beliebig klein, während sie direkt rechts daneben beliebig groß sind.

1 Funktion mit Polstelle

 

So, nun haben wir uns angeschaut, was bei Funktionen alles los sein kann, sozusagen die "Basics". In den folgenden Kapiteln werden wir uns verschiedene Funktionstypen detaillierter vornehmen und beleuchten, wie die Eigenschaften und "besondere Punkte" bei ihnen ausgeprägt sind. Im Kapitel 18 dieses Lernmoduls, wenn Sie alle Funktionstypen kennengelernt haben, wird es dann eine übergreifende Aufgabe zu allen Funktionstypen geben.

Übersicht:

 

12.3 Eigenschaften und "besondere Punkte" von Funktionen - Lösungen

1. Aufgabe

1)
\begin{array}{rcl} f(-x) &=& 5(-x)^5-3(-x)^3+(-x) \cr &=& -5x^5+3x^3-x \cr &\neq& f(x) \end{array}
Die Funktion ist nicht achsensymmetrisch zur y-Achse.

\begin{array}{rcl} -f(-x) &=& -(5(-x)^5-3(-x)^3+(-x)) \cr &=& -\left(-5x^5+3x^3-x\right) \cr &=& 5x^5-3x^3+x \cr &=& f(x) \end{array}
Die Funktion ist punktsymmetrisch zum Koordinatenursprung.


2)
\begin{array}{rcl} f(-x) &=& 6(-x)^2-9(-x)+3 \cr &=& 6x^2+9x+3 \cr &\neq& f(x) \end{array}
Die Funktion ist nicht achsensymmetrisch zur y-Achse.

\begin{array}{rcl} -f(-x) &=& -(6(-x)^2-9(-x)+3) \cr &=& -\left(6x^2+9x+3\right) \cr &=& -6x^2-9x-3 \cr &\neq& f(x) \end{array}
Die Funktion ist nicht punktsymmetrisch zum Koordinatenursprung.


3)
\begin{array}{rcl} f(-x) &=& 13(-x) \cr &=& -13x \cr &\neq& f(x) \end{array}
Die Funktion ist nicht achsensymmetrisch zur y-Achse.

\begin{array}{rcl} -f(-x) &=& -(13(-x)) \cr &=& 13x \cr &=& f(x) \end{array}
Die Funktion ist punktsymmetrisch zum Koordinatenursprung.


4)
\begin{array}{rcl} f(-x) &=& 10(-x)^8+10(-x)^2+7 \cr &=& 10x^8+10x^2+7 \cr &=& f(x) \end{array}
Die Funktion ist achsensymmetrisch zur y-Achse.

\begin{array}{rcl} -f(-x) &=& -\left(10(-x)^8+10(-x)^2+7\right) \cr &=& -\left(10x^8+10x^2+7\right) \cr &=& -10x^8-10x^2-7 \cr &\neq& f(x) \end{array}
Die Funktion ist nicht punktsymmetrisch zum Koordinatenursprung.


5)
\begin{array}{rcl} f(-x) &=& 12(-x)-6 \cr &=& -12x-6 \cr &\neq& f(x) \end{array}
Die Funktion ist nicht achsensymmetrisch zur y-Achse.

\begin{array}{rcl} -f(-x) &=& -\left(12(-x)-6\right) \cr &=& -\left(-12x-6\right) \cr &=& 12x+6 \cr &\neq& f(x) \end{array}
Die Funktion ist nicht punktsymmetrisch zum Koordinatenursprung.


6)
\begin{array}{rcl} f(-x) &=& (-x)^4-5(-x)^2 +4 \cr &=& x^4-5x^2+4 \cr &=& f(x) \end{array}
Die Funktion ist achsensymmetrisch zur y-Achse.

\begin{array}{rcl} -f(-x) &=& -\left((-x)^4-5(-x)^2+4\right) \cr &=& -\left(x^4-5x^2+4\right) \cr &=& -x^4+5x^2-4 \cr & \neq & f(x) \end{array}
Die Funktion ist nicht punktsymmetrisch zum Koordinatenursprung.


7)
\begin{array}{rcl} f(-x) &=& -(-x)^6+7(-x)^2-25 \cr &=& -x^6 + 7x^2-25 \cr &=& f(x) \end{array}
Die Funktion ist achsensymmetrisch zur y-Achse.

\begin{array}{rcl} -f(-x) &=& -\left(-(-x)^6+7(-x)^2-25\right) \cr &=& -\left(-x^6 + 7x^2-25\right) \cr &=& x^6-7x^2+25 \cr & \neq & f(x) \end{array}
Die Funktion ist nicht punktsymmetrisch zum Koordinatenursprung.


8)
\begin{array}{rcl} f(-x) &=& \dfrac{5}{2} (-y)^3 + \dfrac{38}{9} (-y) \cr \cr &=& - \dfrac{5}{2} y^3 - \dfrac{38}{9} y \cr \cr & \neq & f(x) \end{array}
Die Funktion ist nicht achsensymmetrisch zur y-Achse.

\begin{array}{rcl} -f(-x) &=& - \left( \dfrac{5}{2} (-y)^3 + \dfrac{38}{9} (-y) \right) \cr \cr &=& -\left( - \dfrac{5}{2} y^3 - \dfrac{38}{9} y \right) \cr \cr &=& \dfrac{5}{2} y^3 + \dfrac{38}{9} y \cr \cr &=& f(x) \end{array}
Die Funktion ist punktsymmetrisch zum Koordinatenursprung.


9)
\begin{array}{rcl} f(-x) &=& -14 (-x)^4 + 52 (-x)^3 -13 (-x) +1 \cr &=& -14x^4 - 52x^3 +13x +1 \cr & \neq & f(x) \end{array}
Die Funktion ist nicht achsensymmetrisch zur y-Achse.

\begin{array}{rcl} -f(-x) &=& -\left( - 14 (-x)^4 +52 (-x)^3 -13 (-x) +1\right) \cr &=& -\left( -14x^4 -52 x^3 +13x +1\right) \cr &=& 14 x^4 +52 x^3 -13x -1 \cr & \neq & f(x) \end{array}
Die Funktion ist nicht punktsymmetrisch zum Koordinatenursprung.


10)
\begin{array}{rcl} f(-x) &=& 3 (-x)^3 - 33 (-x)^2 - \dfrac{5}{2} (-x) +2 \cr \cr &=& -3x^3 - 33x^2 + \dfrac{5}{2}x +2 \cr \cr & \neq & f(x) \end{array}
Die Funktion ist nicht achsensymmetrisch zur y-Achse.

\begin{array}{rcl} -f(-x) &=& -\left(3(-x)^3 - 33 (-x)^2 - \dfrac{5}{2} (-x) +2\right) \cr \cr &=& -\left(-3x^3 - 33x^2 + \dfrac{5}{2}x +2\right) \cr \cr &=& 3x^3 + 33x^2 - \dfrac{5}{2}x -2 \cr \cr & \neq & f(x) \end{array}
Die Funktion ist nicht punktsymmetrisch zum Koordinatenursprung.


11)
\begin{array}{rcl} f(-x) &=& -93 (-x)^2 - 72(-x) +11 \cr &=& -93 x^2 +72x +11 \cr & \neq & f(x) \end{array}
Die Funktion ist nicht achsensymmetrisch zur y-Achse.

\begin{array}{rcl} -f(-x) &=& -\left(-93 (-x)^2 - 72(-x) +11\right) \cr &=& -\left(-93 x^2 + 72x +11\right) \cr &=& 93 x^2 - 72x - 11 \cr & \neq & f(x) \end{array}
Die Funktion ist nicht punktsymmetrisch zum Koordinatenursprung.


12)
\begin{array}{rcl} f(-x) &=& \dfrac{1}{3} (-x)^4 +12 \cr \cr &=& \dfrac{1}{3} x^4 +12 \cr \cr &=& f(x) \end{array}
Die Funktion ist achsensymmetrisch zur y-Achse.

\begin{array}{rcl} -f(-x) &=& - \left( \dfrac{1}{3} (-x)^4 +12 \right) \cr \cr &=& - \left( \dfrac{1}{3} x^4 +12 \right) \cr \cr &=& - \dfrac{1}{3} x^4 -12 \cr \cr & \neq & f(x) \end{array}
Die Funktion ist nicht punktsymmetrisch zum Koordinatenursprung.


13)
\begin{array}{rcl} f(-z) &=& 100x \cr &=& f(z) \end{array}
Die Funktion ist achsensymmetrisch zur y-Achse.

\begin{array}{rcl} -f(-z) &=& -(100x) \cr &=& -100x \cr & \neq & f(z) \end{array}
Die Funktion ist nicht punktsymmetrisch zum Koordinatenursprung.

Bemerkung: Bitte achten Sie darauf, dass die Variable hier z heißt und nicht x.


14)
\begin{array}{rcl} f(-x) &=& 17 (-x)^6 - 4 (-x)^3 + 46 \cr &=& 17 x^6 + 4x^3 + 46 \cr & \neq & f(x) \end{array}
Die Funktion ist nicht achsensymmetrisch zur y-Achse.

\begin{array}{rcl} -f(-x) &=& -\left(17 (-x)^6 - 4(-x)^3 + 46\right) \cr &=& -\left(17 x^6 + 4x^3 +46\right) \cr &=& -17 x^6 - 4x^3 -46 \cr & \neq & f(x) \end{array}
Die Funktion ist nicht punktsymmetrisch zum Koordinatenursprung.


15)
\begin{array}{rcl} c(-r) &=& -4(-r)^8 +83 \cr &=& -4r^8 +83 \cr &=& c(r) \end{array}
Die Funktion ist achsensymmetrisch zur y-Achse.

\begin{array}{rcl} -c(-r) &=& -\left(-4(-r)^8 +83\right) \cr &=& -\left(-4r^8 +83\right) \cr &=& 4r^8 - 83 \cr & \neq & c(r) \end{array}
Die Funktion ist nicht punktsymmetrisch zum Koordinatenursprung.


16)
\begin{array}{rcl} g(-x) &=& \dfrac{3}{41}(-x)^{24} \cr \cr &=& \dfrac{3}{41} x^{24} \cr \cr &=& g(x) \end{array}
Die Funktion ist achsensymmetrisch zur y-Achse.

\begin{array}{rcl} -g(-x) &=& - \left( \dfrac{3}{41} (-x)^{24} \right) \cr \cr &=& - \left( \dfrac{3}{41} x^{24} \right) \cr \cr &=& - \dfrac{3}{41} x^{24} \cr \cr & \neq & g(x) \end{array}
Die Funktion ist nicht punktsymmetrisch zum Koordinatenursprung.


17)
\begin{array}{rcl} f(-z) &=& -7 (-z)^3 + 12 (-z) \cr &=& 7z^3 -12z \cr & \neq & f(z) \end{array}
Die Funktion ist nicht achsensymmetrisch zur y-Achse.

\begin{array}{rcl} -f(-z) &=& -\left(-7 (-z)^3 + 12 (-z)\right) \cr &=& -\left(7z^3 - 12z\right) \cr &=& -7z^3 +12z \cr &=& f(z) \end{array}
Die Funktion ist punktsymmetrisch zum Koordinatenursprung.


18)
\begin{array}{rcl} f(-x) &=& \dfrac{7}{18} (-x)^6 - 33 (-x)^3 -47 \cr \cr &=& \dfrac{7}{18} x^6 + 33x^3 -47 \cr \cr & \neq & f(x) \end{array}
Die Funktion ist nicht achsensymmetrisch zur y-Achse.

\begin{array}{rcl} -f(-x) &=& -\left( \dfrac{7}{18} (-x)^6 -33 (-x)^3 -47 \right) \cr\cr &=& - \left( \dfrac{7}{18} x^6 +33 x^3 -47 \right) \cr \cr &=& -\dfrac{7}{18} x^6 -33x^3+47 \cr \cr & \neq & f(x) \end{array}
Die Funktion ist nicht punktsymmetrisch zum Koordinatenursprung.


19)
\begin{array}{rcl} h(-y) &=& 101(-y)^{34} + 9(-y)^{12} \cr &=& 101y^{34} + 9y^{12} \cr &=& h(y) \end{array}
Die Funktion ist achsensymmetrisch zur y-Achse.

\begin{array}{rcl} -h(-y) &=& -\left(101 (-y)^{34} +9(-y)^{12}\right) \cr &=& -\left(101y^{34} + 9y^{12}\right) \cr &=& -101y^{34} -9y^{12} \cr & \neq & h(y) \end{array}
Die Funktion ist nicht punktsymmetrisch zum Koordinatenursprung.


20)
\begin{array}{rcl} d(-x) &=& 51 (-x)^4 - \dfrac{63}{5} (-x)^3 +223 \cr \cr &=& 51x^4 + \dfrac{36}{5} x^3 +223 \cr \cr & \neq & d(x) \end{array}
Die Funktion ist nicht achsensymmetrisch zur y-Achse.

\begin{array}{rcl} -d(-x) &=& - \left( 51 (-x)^4 - \dfrac{63}{5} (-x)^3 + 223 \right) \cr \cr &=& - \left( 51x^4 + \dfrac{63}{5} x^3 +223 \right) \cr \cr &=& -51x^4 - \dfrac{63}{5} x^3 -223 \cr \cr & \neq & d(x) \end{array}
Die Funktion ist nicht punktsymmetrisch zum Koordinatenursprung.

Zusatzfrage: Es gibt eine Funktion, die sowohl achsensymmetrisch zur y-Achse als auch punktsymmetrisch zum Koordinatenursprung ist, nämlich f(x)=0.
Ok, keine besonders spannende Funktion ...

 

2. Aufgabe

1)
\begin{array}{crclcl} & 0 &=& x^3-2x^2 &\vert& \text{ausklammern} \cr & 0 &=& x^2(x-2) &\vert& \text{Satz vom Nullprodukt} \cr \text{Faktor 1:} & x_1 &=& 0 \cr\cr \text{Faktor 2:} & x-2 &=& 0 \cr & x_2 &=& 2 \end{array}

Beide Werte liegen im Definitionsbereich, daher sind x_1 und x_2 tatsächlich Nullstellen der Funktion.

 
2)
\begin{array}{rclcl} 0 &=& 7x+4 &\vert& -4 \cr -4 &=& 7x &\vert& :7 \cr -\dfrac{4}{7} &=& x \end{array}

Dies ist keine Nullstelle der Funktion, denn -\dfrac{4}{7} \not \in \mathbb{D}.

 
3)
\begin{array}{rclcl} \large 0 &=& \dfrac{1}{2}x^2-\dfrac{2}{3}x+\dfrac{2}{9} &\vert & \cdot 2 \cr 0 &=& x^2-\dfrac{4}{3}x+\dfrac{4}{9} &\vert & \text{p-q-Formel} \cr x_{1,2} &=& -\left(-\dfrac{2}{3}\right) \pm \sqrt{\left(-\dfrac{2}{3}\right)^2-\dfrac{4}{9}} \cr\cr &=& \dfrac{2}{3} \pm \sqrt{\dfrac{4}{9}-\dfrac{4}{9}} \cr\cr &=& \dfrac{2}{3}-0 \cr\cr &=& \dfrac{2}{3} \end{array}

Dieser Wert liegt im Definitionsbereich, daher ist x tatsächlich Nullstelle der Funktion.

 
4)
\begin{array}{crclcl} & 0 &=& x^3-\dfrac{7}{12}x^2-\dfrac{5}{24}x &\vert& \text{ausklammern} \cr & 0 &=& x \left(x^2-\dfrac{7}{12}x-\dfrac{5}{24} \right) &\vert& \text{Satz vom Nullprodukt} \cr \text{Faktor 1:} & x_1 &=& 0 \cr\cr \text{Faktor 2:} & x^2-\dfrac{7}{12}x-\dfrac{5}{24} &=& 0 &\vert& \text{p-q-Formel} \cr\cr& x_{2,3} &=& -\left(-\dfrac{7}{24}\right) \pm \sqrt{\left(-\dfrac{7}{24}\right)^2+\dfrac{5}{24}} \cr\cr & &=& \dfrac{7}{24} \pm \sqrt{\dfrac{49}{576}+\dfrac{120}{576}} \cr\cr & &=& \dfrac{7}{24} \pm \sqrt{\dfrac{169}{576}} \cr\cr & &=& \dfrac{7}{24} \pm \dfrac{13}{24} \cr \cr & x_2 &=& \dfrac{7}{24} + \dfrac{13}{24} = \dfrac{20}{24} = \dfrac{5}{6} \cr\cr & x_3 &=& \dfrac{7}{24} - \dfrac{13}{24} = -\dfrac{6}{24} = -\dfrac{1}{4} \end{array}

Von diesen Werten liegen nur x_1 und x_2 im Definitionsbereich, daher sind auch nur an diesen Stellen tatsächlich Nullstellen der Funktion.

 
5)
\begin{array}{rclcl} 0 &=& 11x^2+110x+286 &\vert& :11 \cr &=& x^2+10x+26 &\vert& \text{p-q-Formel} \cr x_{1,2} &=& -5 \pm \sqrt{5^2-26} \cr &=& -5 \pm \sqrt{25-26} \cr &=& -5 \pm \sqrt{-1} \end{array}

Da aus negativen reellen Zahlen keine Wurzeln mit geraden Wurzelexponenten gezogen werden können, hat diese Funktion keine Nullstellen.

 

3. Aufgabe

1)
Dies ist eine lineare Funktion. Wenn man mal von konstanten Funktionen absieht, können lineare Funktionen entweder ausschließlich ansteigend oder ausschließlich fallend sein. Mischformen gibt es nicht. Extrempunkte können also allenfalls an den Rändern des Definitionsbereichs entstehen. Sind solche Funktionen, wie bei dieser Aufgabe, für die gesamten reellen Zahlen definiert, haben sie keine Extrempunkte.

Bemerkung: -\infty und +\infty sind keine Zahlen, daher können dort auch keine Extrempunkte vorliegen.


2)
Quadratische Funktionen haben im Scheitelpunkt entweder einen Tiefpunkt (wenn die Parabel nach oben geöffnet ist) oder einen Hochpunkt (wenn die Parabel nach unten geöffnet ist). Den Scheitelpunkt kann man z. B. mithilfe der quadratischen Ergänzung bestimmen:
\begin{array}{rcl} f(x) &=& x^2+4x+8 \cr &=& x^2+2 \cdot 2x+4+4 \cr &=& (x+2)^2+4 \end{array}
Da -2 \in \mathbb{D} ist, liegt der Scheitelpunkt der Funktion also bei (-2\mid 4). Da die Parabel nach oben geöffnet ist (vor dem x^2-Term steht eine positive Zahl, nämlich 1), kann im Scheitelpunkt nur ein Tiefpunkt liegen.

Interessant ist noch der obere Rand des Definitionsbereichs, nämlich die Stelle x=0. Da es sich um eine nach oben geöffnete Parabel handelt und der Tiefpunkt bei (-2\mid 4) liegt, ist die Funktion bei x=0 ansteigend. Bei (0\mid 8) liegt also ein Hochpunkt.


3)
Dies ist wieder eine lineare Funktion. Im Gegensatz zu 1) ist sie aber nur für einen Teilbereich der reellen Zahlen definiert, nämlich für [1;\infty[. In diesem Fall liegt am unteren Ende des Definitionsbereichs ein Extrempunkt vor. Da die Steigung hier positiv ist, muss der Wert am unteren Ende des Definitionsbereichs der kleinste Funktionswert von allen sein. Es liegt also ein Tiefpunkt bei \left(1\mid -\dfrac{14}{9}\right) vor.


4)
Diese Funktion ist konstant. Daher kann sie keinen größten oder kleinsten Wert haben.


5)
f(x) lässt sich durch Ausklammern von -2 und mithilfe der 2. binomischen Formel auf folgende Form bringen: f(x)=-2(x-1)^2. Daran kann man 1. ablesen, dass die Funktion nach unten geöffnet ist (vor dem x^2-Term steht eine negative Zahl), und 2., dass die Funktion im Punkt (1 \mid 0) ihren Scheitelpunkt hat. Es gilt aber: 1 \not \in \mathbb{D}. Daher liegt hier kein Extrempunkt der Funktion vor.

Es müssen nun noch die Ränder des Definitionsbereichs betrachtet werden: Bei (-2\mid -18) liegt ein Tiefpunkt und bei (0\mid -2) ein Hochpunkt der Funktion, da f(x) als nach unten geöffnete Parabel mit einem Scheitelpunkt bei (1 \mid 0) bis x=1 ansteigend ist.


6)
Dies ist eine fallende lineare Funktion, die nur für einen Teilbereich der reellen Zahlen definiert ist, nämlich für [-1;\infty [. Ein Extrempunkt kann daher nur am unteren Rand des Definitionsbereiches auftreten. Der dazu gehörige Funktionswert kann dann einfach berechnet werden: f(-1) = 13 \cdot (-1) - 2 = -15. Da die Funktion fällt, ist es ein Hochpunkt. 


7)
Hier liegt eine quadratische Funktion vor, die nach oben geöffnet ist (der Koeffizient vor x^2 ist ja positiv). Es ist also ein Tiefpunkt zu erwarten. Der Scheitelpunkt wird mit Hilfe der quadratischen Ergänzung bestimmt:
\begin{array}{rclcl} f(x) &=& x^2 - 6x + 9 \cr &=& x^2 - 2 \cdot 3 + 9 \cr &=& (x-3)^2 \end{array}
Der Scheitelpunkt liegt also bei (3 \mid 0).

Da die Funktion in dieser Aufgabe für die gesamten reellen Zahlen definiert ist, können an den Rändern des Definitionsbereiches keine weiteren Extrempunkte existieren. 


8)
Hierbei handelt es sich wieder um eine lineare Funktion. Da der Definitionsbereich alle reellen Zahlen umfasst, kann in diesem Fall nirgends ein Extrempunkt auftreten.


9)
Hier haben wir es wieder mit einer steigenden linearen Funktion mit einem eingeschränkten Definitionsbereich zu tun. Da der Definitionsbereich aber auf ein offenes Intervall eingeschränkt ist, gibt es keine Randwerte, an denen Extrempunkte auftreten können.


10)
Hierbei handelt es sich um eine nach unten geöffnete quadratische Funktion, die um den Faktor -8 gestreckt und um 10 Einheiten nach oben verschoben ist. Da keine Verschiebung nach links/recht erfolgt, muss der Scheitelpunkt also bei x = 0 liegen. Es handelt sich um einen Hochpunkt, da die Parabel nach unten geöffnet ist. Deshalb treten auch an den Grenzen des Definitionsbereiches jeweils Tiefpunkte auf. Die Funktionswerte lassen sich wie folgt berechnen:
\begin{array}{rclcl} f \left(- \sqrt{2} \right) &=& -8\cdot\left(- \sqrt{2} \right)^2 +10 = -6 \cr \cr f(0) &=& -8\cdot 0^2 +10 = 10 \cr \cr f \left( \sqrt{2} \right) &=& -8\cdot\left( \sqrt{2} \right)^2 +10 = -6 \end{array}
Insgesamt hat die Funktion innerhalb der Grenzen des Definitionsbereiches also einen Tiefpunkt bei (- \sqrt{2} \mid -6), einen Hochpunkt bei (0 \mid 10) und einen weiteren Tiefpunkt bei ( \sqrt{2} \mid -6).

 

4. Aufgabe

In den ersten Grafik gibt es folgende Wende- und Sattelpunkte zu entdecken:

drei Funktionen ohne Wendepunkt, eine Funktion mit zwei Wendepunkten, eine Funktion mit Sattelpunkt

Nicht gefragt waren die konkreten Koordinaten, bei denen die Wende- und Sattelpunkte liegen. Diese können nämlich nur mithilfe von Ableitungen ermittelt werden, was in diesem Lernmodul noch nicht thematisiert wurde. Der Vollständigkeit halber hier noch die Werte:

  • f_1(x) hat zwei Wendepunkte, die bei W_1 \left(-2{,}45 \mid -6{,}2\right) und W_2 \left(2{,}45 \mid -6{,}2\right) liegen.
  • f_5(x) hat einen Sattelpunkt bei S(-5\mid3).
  • Die anderen Funktionen in der ersten Grafik haben keine Wende- oder Sattelpunkte.

In der zweiten Grafik finden sich folgende Wende- und Sattelpunkte:

drei Funktionen ohne Wendepunkt, eine Funktion mit einem Wendepunkt, eine Funktion mit mehreren Wendepunkten

Auch hier noch die konkreten Werte der Wendepunkte:

  • f_1(x) hat unendlich viele Wendepunkte bei W_n \left(n\pi \mid 6\right) mit n\in\mathbb{Z}.
  • f_3(x) hat einen Wendepunkt bei W \left(-5\mid -0{,}74\right).
  • Die anderen Funktionen in dieser Grafik haben keine Wende- oder Sattelpunkte.

13. Polynomgleichungen und -funktionen - Lernziele und typische Fehler

Nach Durcharbeiten dieses Kapitels sollten Sie folgende Lernziele erreicht haben:

  • Sie können zu einer Polynomgleichung den passenden Definitionsbereich bestimmen.
  • Sie kennen die allgemeine Form einer Polynomgleichung.
  • Sie wissen, wie viele Lösungen Polynomgleichungen haben können, und können anhand des Rechenweges feststellen, welcher Fall vorliegt.
  • Sie können (einfache) Polynomgleichungen lösen.
  • Sie können die Lösungsmenge mathematisch korrekt notieren.
  • Sie können mithilfe der Probe überprüfen, ob die gefundene Lösung tatsächlich richtig ist.
  • Sie können zu einer Polynomfunktion den passenden Definitionsbereich bestimmen.
  • Sie wissen, wie der Graph einer Polynomfunktion typischerweise aussieht, und können ihn in ein kartesisches Koordinatensystem zeichnen.
  • Sie kennen die allgemeine Funktionsgleichung einer Polynomfunktion.
  • Sie können den Grad eines Polynoms angeben.
  • Sie können gerade und ungerade Polynome unterscheiden.
  • Sie kennen Eigenschaften von Polynomen (Anzahl der Nullstellen, Symmetrie, Randverhalten, Extremstellen, typische Punkte) und können diese nutzen, um Beziehungen zwischen Funktionsterm und Funktionsgraphen herzustellen.
  • Sie kennen den Zusammenhang zwischen Polynomgleichungen und Polynomfunktionen.
  • Sie können Polynomgleichungen von anderen Gleichungsarten unterscheiden.
  • Sie können Polynomfunktionen von anderen Funktionstypen unterscheiden (grafisch und anhand der Funktionsgleichung).


Typischer Fehler in diesem Kapitel ist:

  • Beim Multiplizieren der Gleichung mit einem Term werden nicht alle Bestandteile der Gleichung multipliziert. Erklärung


Für Online-Selbsttests zu diesem Thema und weitere Informationen zur Mathematikunterstützung an der TH Wildau nutzen Sie bitte den Moodle-Kursraum "SOS Mathematik - Brückenkurs".


Zwei Bemerkungen:

  • Ab diesem Kapitel werden bei den Musterlösungen weniger Zwischenschritte angegeben als zuvor. Nach den vorangegangenen Kapiteln haben Sie ja ein gewisses Trainingslevel erreicht, sodass es (hoffentlich!) nicht mehr nötig ist, jedes Detail in den Rechnungen zu erwähnen. Beispielsweise wird ab jetzt statt -(-5) sofort +5 geschrieben oder bei \frac{1}{2}+\frac{1}{4} = \frac{2}{4}+\frac{1}{4} = \frac{3}{4} der mittlere Umformungsschritt weggelassen. Selbstverständlich sind alle für die Lösung essenziellen Schritte weiterhin in den Rechenwegen enthalten und kommentiert. Hintergrund dafür ist, dass das Weglassen von Details den Blick mehr auf das "große Ganze" richtet und - ganz praktisch - eine gute Vorbereitung auf Musterlösungen im Studium und in Lehrbüchern ist, die häufig sehr viel knapper gehalten sind.
    Wenn Sie Schwierigkeiten haben, die kürzeren Rechnungen zu verstehen, schauen Sie bitte in die ersten zwölf Kapitel: Insbesondere die Infos zu quadratischen Gleichungen, zum Ausklammern und dem Satz vom Nullprodukt können helfen.
  • Bei den Ausführungen in den folgenden Kapiteln fehlen größtenteils die mathematischen Begründungen, da diese über den Umfang des Vorkurses hinausgehen. Sie werden sich ggf. im Studium mit diesen auseinandersetzen.

Übersicht:

 

13.1 Polynomgleichungen und -funktionen - Aufgaben

1. Aufgabe

Bestimmen Sie jeweils den Grad des Polynoms!

1) p(x)=3x^2-2x+1

  6) p(x)=0x^7-4x^6+17x^5-23x^4+109x^3+77x^2-x+48
2) p(x)=17x^4-x+\dfrac{2}{11}

  7) p(x)=33x^7-\dfrac{2}{5} x^9+\dfrac{11}{30} x^{11}+100

3) p(x)=-180x^{21}-0{,}1x^{19}-13x^{13}+228

  8) p(x)=-22x^{19}+x^{16}-\dfrac{22}{9} x^{13}-20

4) p(x)=87

  9) p(x)=-x^{31}-x^{22}+\dfrac{41}{3} x^2-22

5) p(x)=-\dfrac{1}{6}x^{16}+\dfrac{2}{7}x^7-34x^5+15x^2-9   10) p(x)=-1+\dfrac{2}{30} x^2+\dfrac{4}{30} x^9-20x^{18}

 

2. Aufgabe

Lösen Sie die folgenden Polynomgleichungen!
Für einige Aufgaben finden Sie Lösungshinweise am Ende der Seite.

1) 0=x^3-5x^2-6x   11) -12x\left(x^2-19\right) = 24\left(-x^2+10\right)
2) \dfrac{1}{3}x^4+12=0   12) -\dfrac{1}{81}z^3\left(9z\left(-9z^4+2\right)\right) = \dfrac{2}{27}
3) \dfrac{x^3}{8} = x^2+2x-16   13) 2(x+5)^2-(x-4)(x+5) = (x-4)^2
4) 2x^3-x^2+6x=3x^2   14) 14x^2\left(x^2-2\right)+28x^3 = 14x
5) x^{10}+768=56x^5   15) \dfrac{9}{4}\left(\dfrac{4x}{3}\right)^4-4x\left(\dfrac{4x}{3}\right)^3 = -192
6) y^3+y^2+4 = -4y   16) \left(x^2-1\right)^2=x^3+1
7) \dfrac{1}{2}x^3+\dfrac{13}{2}x^2-2x = 26   17) \left(5x^2-\sqrt{5}x\right)\left(x^3+x^2-4x+2\right) = 0
8) 11x^3+5x^5+2x=0   18) 12x^2(x-1) = -\sqrt{144p} \cdot x (x-1)
mit p\in\mathbb{R}_0^+ konstant
9) x^4+5x^3-x^3\ln(5)-x^25\ln(5) = 0   19) 0 = \left(-\pi-k^2+x^2\right)\left(x^2-\pi^2-kx^2+k^2\pi^2\right)
mit k\in\mathbb{R} konstant
10) 8\left(\dfrac{3}{8}x^4-6\right) = 3\left(x^2-4\right)\left(x^2+4\right)   20) 3x^{2n}=2x^{2n-1}+x^{2n-1} mit n \in \mathbb{N}^+

 

3. Aufgabe

Welche Eigenschaften haben die folgenden Polynomfunktionen (alle mit \mathbb{D}=\mathbb{R})?
Es geht bei dieser Aufgabe darum, den Funktionsterm zu verstehen und daraus Informationen über den Funktionsgraphen abzulesen - und das Alles: Ohne zu rechnen! 
Skizzieren Sie die Graphen anhand Ihrer Überlegungen zu den Eigenschaften!

1) f(x)=-x^3+5x^2-x+5

2) f(x)=10x^4-3x^3+8x^2

3) f(x)=15x^4+21x^2-9

4) f(x)=2x^5+x^3-4x

5) f(x)=-x^6-x^4+4x^2+7

 

4. Aufgabe

Für einige Aufgaben finden Sie Lösungshinweise am Ende der Seite.

1)
Gegeben sei die Funktion f(x)=3x^4-4x^3 mit x\in \mathbb{R}.
Gesucht ist jeweils die fehlende Koordinate des Punktes P(x\mid y),
a) wenn x=1{,}5
b) wenn y=0

  6)
Gegeben sei die Funktion f(x)=3\left(\dfrac{4}{3}x^3-2x^2+1\right) mit x\in \mathbb{R}.
Gesucht ist jeweils die fehlende Koordinate des Punktes P(x\mid y),
a) wenn x=-\dfrac{3}{2}
b) wenn y=3

2)
Gegeben sei die Funktion f(x)=2x^4-x^3+3x^2+13 mit x\in \mathbb{R}.
Gesucht ist jeweils die fehlende Koordinate des Punktes P(x\mid y),
a) wenn x=-\dfrac{1}{3}
b) wenn y=13

  7)
Gegeben sei die Funktion f(x)=\dfrac{1}{2}\left(x^3-12x^2+5x\right)+80 mit x\in \mathbb{R}
Gesucht ist jeweils die fehlende Koordinate des Punktes P(x\mid y),
a) wenn x=-32
b) wenn y=5

3)
Gegeben sei die Funktion f(x)=4x^3+7x^2-2x-5 mit x\in \mathbb{R}
Gesucht ist jeweils die fehlende Koordinate des Punktes P(x\mid y),
a) wenn x=-11
b) wenn y=0

  8) 
Gegeben sei die Funktion f(x)=5x^{13}-7x^9-7 mit x\in \mathbb{R}.
Gesucht ist jeweils die fehlende Koordinate des Punktes P(x\mid y),
a) wenn x=-1
b) wenn y=-7

4)
Gegeben sei die Funktion f(x)=7x^6-4x^3+5 mit x\in \mathbb{R}.
Gesucht ist jeweils die fehlende Koordinate des Punktes P(x\mid y),
a) wenn x=0{,}1
b) \wenn y=485

  9) 
Gegeben sei die Funktion f(x)=3x^3-10x^2+7x-26 mit x\in \mathbb{R}
Gesucht ist jeweils die fehlende Koordinate des Punktes P(x\mid y),
a) wenn x=12
b) wenn y=-14

5)
Gegeben sei die Funktion f(x)=x^{16}-x^8-256 mit x\in \mathbb{R}.
Gesucht ist jeweils die fehlende Koordinate des Punktes P(x\mid y),
a) wenn x=0
b) wenn y=-256

  10)
Gegeben sei die Funktion f(x)=x^4-2x^2+1 mit x\in \mathbb{R}.
Gesucht ist jeweils die fehlende Koordinate des Punktes P(x\mid y),
a) wenn x=5
b) wenn y=9

 

5. Aufgabe

Bestimmen Sie von folgenden Funktionen die Nullstellen!
Für einige Aufgaben finden Sie Lösungshinweise am Ende der Seite.

1) f(a)=8a^5+6a^3-20a

  11) h(x)=(6x^4+17)(9x^4+3x^2-2)

2) f(x)=3x^3+6x^2-3x^2-6x

  12) f(x)=(2x^3+8x^2-6x)(9x^3-10x)

3) f(x)=x^3+4x^2-51x-54

  13) f(x)=(x^3-x^2-50x-48)

4) f(x)=4x^3+8x^2-x-2

  14) f(x)= \dfrac45 x^5 +16x^3

5) f(x)=2x^3+12x^2-48x-128

  15) f(x)=x^4-2x^3-7x^2+8x+12
6) f(x)=\dfrac{3}{4}x^4+2x^2-4   16) g(x)=2x^6+ \sqrt{2} x^3-15
7) f(y)=\left(y-3\right)\cdot\left(y^2-\dfrac{2}{3}y-3\right)   17) f(x) = (2x^3+8x^2-6x)(9x^3-30)(4x^4-12x^2+2)
8) f(x)=x^4-17x^2+16   18) f(x)= x^3-ax^2-x^2+ax-2x+2a
9) f(x)=\dfrac{1}{4}x^3+x^2+x   19) -x^3 +3x^2 +33x-35
10) f(x)=x^3+5x^2-2x-24   20) x^4 -3x^2-88

 

Lösungshinweise

Aufgabe 2.3: x_0=4 ist eine Lösung der Gleichung.
Aufgabe 2.7: x_0=-13 ist eine Lösung der Gleichung.
Aufgabe 2.11: x_0=1 ist eine Lösung der Gleichung.
Aufgabe 2.14: x_0=1 ist eine Lösung der Gleichung.
Aufgabe 2.17: x_0=1 ist eine Lösung der Gleichung.
Aufgabe 4.3: x_0=-1 ist eine Lösung der Gleichung.
Aufgabe 4.7: x_0=5 ist eine Lösung der Gleichung.
Aufgabe 4.9: x_0=3 ist eine Lösung der Gleichung.
Aufgabe 5.3: x_0=-1 ist eine Lösung der Gleichung.
Aufgabe 5.4: x_0=-2 ist eine Lösung der Gleichung.
Aufgabe 5.5: x_0=4 ist eine Lösung der Gleichung.
Aufgabe 5.10: x_0=2 ist eine Lösung der Gleichung.
Aufgabe 5.13: x_0=-1 ist eine Lösung der Gleichung.
Aufgabe 5.15: x_0=-2 und x_1=2 sind Lösungen der Gleichung.
Aufgabe 5.18: x_0=a ist eine Lösung der Gleichung.
Aufgabe 5.19: x_0=1 ist eine Lösung der Gleichung.

Dieses Kapitel enthält die folgenden Themen:

 

13.2 Polynomgleichungen und -funktionen - Erklärungen

In den Kapiteln 9 und 10 hatten wir uns quadratische Gleichungen und quadratische Funktionen angeschaut. Im Folgenden werden wir diese mathematischen Objekte verallgemeinern. Der Gedanke, dass in Gleichungen und Funktionen auch Potenzen mit einem Exponenten größer als 2 auftauchen können, liegt ja nicht allzu fern. Beispielsweise werden solche Gleichungen und Funktionen gebraucht, wenn Volumina betroffen sind. Man sieht ja schon an Einheiten wie m^3, dass man mit Quadraten offensichtlich nicht auskommt. In einem ingenieurwissenschaftlichen Studium werden Sie zudem später lernen, dass es sehr hilfreich sein kann, komplizierte Funktionen durch Polynome anzunähern (Stichwort: Taylorpolynom).
Sprachlich ist man hier häufig etwas ungenau und sagt einfach nur "Polynom" - egal, ob man eine Polynomgleichung oder eine Polynomfunktion meint. Was gemeint ist und was man tun soll, wird dann aus dem Zusammenhang bzw. der Aufgabenstellung klar.

 

Definition

Definition einer Polynomgleichung: Polynomgleichungen sind Gleichungen, bei denen Potenzterme mit beliebigen natürlichen Exponenten, ggf. multipliziert mit einem Koeffizientenaddiert werden.
Definition einer Polynomfunktion: Polynomfunktionen sind Funktionen, bei denen Potenzterme mit beliebigen natürlichen Exponenten, ggf. multipliziert mit einem Koeffizienten, addiert werden.
Allgemeine Darstellung einer Polynomgleichung: 0 = a_nx^n+a_{n-1}x^{n-1}+a_{n-2}x^{n-2}+ \ldots + a_2x^2+a_1x^1+a_0x^0 mit a_n, a_{n-1}, \,\ldots\,, a_1, a_0 \in \mathbb{R}, a_n \neq 0 und n \in \mathbb{N}
Allgemeine Darstellung einer Polynomfunktion: f(x) = a_nx^n+a_{n-1}x^{n-1}+a_{n-2}x^{n-2}+ \ldots + a_2x^2+a_1x^1+a_0x^0 mit a_n, a_{n-1}, \,\ldots\,, a_1, a_0 \in \mathbb{R}, a_n \neq 0 und n \in \mathbb{N}

Die Gleichungen/Funktionen, die wir in vorherigen Kapiteln angeschaut haben, sehen ziemlich ähnlich aus, nur mit weniger Bestandteilen. Wenn Sie sich z. B. das "Ende" der allgemeinen Polynomgleichung ansehen, bleibt genau eine quadratische Gleichung übrig: 0 = a_2x^2+a_1x^1+a_0x^0 = a_2x^2+a_1x+a_0

Die reellen Zahlen a_n, a_{n-1}, \,\ldots\,, a_1, a_0 heißen Koeffizienten des Polynoms. Da wir es hier mit reellen Zahlen zu tun haben, sind natürlich auch Brüche als Koeffizienten möglich. Wichtig: Da es sich bei den Koeffizienten um Zahlen handelt, ist klar, dass Variablen darin nicht vorkommen dürfen. Insbesondere dürfen Variablen nicht im Nenner eines Bruches stehen.

Die natürliche Zahl n heißt Grad des Polynoms. Der Grad des Polynoms entspricht also dem höchsten Exponenten, der in dem Polynom vorkommt. Wichtig: Der Koeffizient dieses höchsten Potenzterms darf nicht 0 sein: a_n\neq 0. Wäre a_n=0, würde diese Potenz ja wegfallen. Das hatten wir bei quadratischen Gleichungen ja auch schon diskutiert.

Definitionsbereich: Aus mathematischer Sicht gibt es keine Einschränkungen für den Definitionsbereich von Polynomen. Er umfasst also die gesamten reellen Zahlen, solange keine inhaltlichen Einschränkungen vorliegen (z. B. sollten die Maße eines Zylinders, für den das Volumen berechnet werden soll, positiv sein).

 

Polynomgleichungen

Starten wir mit ein paar Beispielen:

  • 0 = 4x^3-5x^2-6x+7
    Dies ist ein Polynom vom Grad 3 mit den Koeffizienten 4, -5, -6 und 7.

  • 0x^8+x^7+4{,}15x^4+\frac{7}{18}x^2-x+199 = 0 
    Dies ist ein Polynom vom Grad 7 mit den Koeffizienten 1, 4{,}15, \frac{7}{18}, -1 und 199. Der Grad ist 7, da der Koeffizient vom x^8-Term 0 ist.

  • 0 = \frac{2}{x}+15x^3-9x^2-\frac{3}{11}
    Dies ist kein Polynom, weil die Variable bei \frac{2}{x} im Nenner steht.

 

Lösungswege

Es gibt verschiedene Lösungsverfahren für Polynomgleichungen, die wir uns im Folgenden an verschiedenen Beispielen genauer anschauen werden. Bei manchen Polynomgleichungen benötigt man keine neuen mathematischen Verfahren. Zum Lösen genügen Methoden, die wir bereits in vorherigen Kapiteln kennengelernt haben. Wir müssen sie nur etwas kombinieren.
Leider gibt es bei allgemeinen Polynomgleichungen höheren Grades - anders als bei linearen und quadratischen Gleichungen - nur in Ausnahmefällen elementare Lösungsverfahren. Ein elementares Lösungsverfahren bei quadratischen Gleichungen ist z. B. die p-q-Formel, mit der man ohne große Komplikationen die Lösungen erhält. Das Lösen von Polynomgleichungen höheren Grades verläuft nur dann ähnlich unkompliziert, wenn die Gleichungen eine bestimmte Struktur aufweisen. Andernfalls kann das Lösen von Polynomen höheren Grades ziemlich aufwendig oder sogar unmöglich sein, sodass Lösungen nur näherungsweise bestimmt werden können. Solche Näherungsverfahren nennt man auch numerische Verfahren. Wenn nötig, werden Sie diese im Studium kennenlernen.

 

Umformen

Manche Polynomgleichungen kann man so umformen, dass auf einer Seite der Gleichung nur noch x^n steht (mathematisch gesprochen: "man isoliert x^n"). Anschließend kommt man durch Wurzelziehen zur Lösung.

Struktur der Gleichung: Die Gleichung enthält nur einen Potenzterm, d. h. sie lässt sich in die Form a_nx^n+a_0=0 bringen.

Bei Wurzeln mit geraden Wurzelexponenten aufpassen

  • Auch wenn solche Wurzeln im Bereich der reellen Zahlen immer nichtnegativ sind, entstehen beim Lösen der Gleichung zwei Lösungen, eine positive und eine negative.
  • Steht unter einer solchen Wurzel eine negative Zahl, ist das Wurzelziehen nicht möglich.

Bei Wurzel mit ungeradem Wurzelexponenten kann beides nicht passieren. Nur beim Wurzelziehen aus negativen Radikanden muss man auf den Betrag aufpassen.


Ein Beispiel:
\begin{array}{rclcl} 120 &=& \dfrac{2}{9}x^6-42 &\vert& -\dfrac{2}{9}x^6-120 \cr -\dfrac{2}{9}x^6 &=& -162 &\vert& :\left(-\dfrac{2}{9}\right) \cr x^6 &=& 729 &\vert& \pm\sqrt[6]{} \cr x_{1,2} &=& \pm\sqrt[6]{729} \cr\cr x_1 &=& 3 \cr x_2 &=& -3 \cr\cr \mathbb{L} &=& \{-3;3\} \end{array}

Und noch ein Beispiel:
\begin{array}{rclcl} 3.125x^5+1 &=& 0 &\vert & -1 \cr\cr 3.125x^5 &=& -1 &\vert & :3.125 \cr\cr x^5 &=& -\dfrac{1}{3.125} &\vert& \sqrt[5]{} \cr\cr x_1 &=& \sqrt[5]{-\dfrac{1}{3.125}} \cr\cr x_1 &=& -\sqrt[5]{\left\vert -\dfrac{1}{3.125}\right\vert} \cr\cr x_1 &=& -\dfrac{1}{5} \cr\cr \mathbb{L} &=& \left\{-\dfrac{1}{5}\right\} \end{array}

Letztendlich funktioniert das genauso, wie wir das bei quadratischen Gleichungen - Lösungsweg: Umformen schon gesehen hatte, nur dass der Wurzelexponent ein anderer ist.

 

Ausklammern

Eine weitere Lösungsmöglichkeit ist das Ausklammern, das als erster Schritt immer eine gute Option ist, wenn die Struktur der Gleichung sich dafür eignet. Dadurch reduziert sich nämlich der Grad des Polynoms.

Struktur der Gleichung: Alle Summanden der Gleichung enthalten einen gemeinsamen Faktor, in dem auch die Variable bzw. Potenzen der Variablen enthalten sind.

Beispiel:
\begin{array}{clrclcl}& & \mathbb{D} &=& \mathbb{R} \cr\cr\text{1. Zeile:} & & \dfrac{4}{3}x^4-20x^2 &=& \dfrac{8}{3}x^3+12x^2 &\vert& :\dfrac{4}{3} \cr \text{2. Zeile:} & & x^4-15x^2 &=& 2x^3+9x^2 &\vert& -2x^3-9x^2 \cr \text{3. Zeile:} & & x^4-2x^3-24x^2 &=& 0 &\vert& \text{ausklammern} \cr \text{4. Zeile:} & & x^2\cdot\left(x^2-2x-24\right) &=& 0 &\vert& \text{Satz vom Nullprodukt} \cr \text{5. Zeile:} & \text{Faktor 1:} & x_1^2 &=& 0 &\vert& \pm\sqrt{} \cr\text{6. Zeile:} & & x_1 &=& 0 \cr\cr\text{7. Zeile:} & \text{Faktor 2:} & x^2-2x-24 &=& 0 &\vert& \text{p-q-Formel} \cr& & x_{2,3} &=& 1\pm\sqrt{(-1)^2-(-24)} \cr& & &=& 1\pm\sqrt{25} \cr \cr& & x_2 &=& 1+5 = 6 \cr & & x_3 &=& 1-5 = -4 \cr\cr & & \mathbb{L} &=& \{-4; 0; 6\}\end{array}

Zur 1. Zeile: Es rechnet sich einfacher, wenn vor dem x^4 nur eine 1 steht. Daher wird durch den Koeffizienten von x^4, also durch \frac{4}{3}, dividiert. Wichtig: Alle Terme der Gleichung müssen durch \frac{4}{3} dividiert werden! Andernfalls werden höchstwahrscheinlich genau die Probleme auftreten, die bereits im Kapitel lineare Gleichungen erläutert wurden.

Zur 2. Zeile: Als Nächstes sollen alle Terme auf der linken Seite der Gleichung gesammelt werden. Dazu wird -2x^3-9x^2 gerechnet. Tipp: Für den Überblick ist es gut, die Terme so zu sortieren, dass der Term mit dem höchsten Exponenten immer zuerst steht und dann die übrigen Terme mit absteigendem Exponenten folgen.

Zur 3. Zeile: Es fällt auf, dass alle Summanden der Gleichung den Faktor x^2 enthalten. Das x^2 kann also ausgeklammert werden.

Zur 4. Zeile: Nun haben wir die Gleichung in ein Produkt zweier Faktoren mit dem Ergebnis 0 umgeformt, sodass uns der Satz vom Nullprodukt weiterhilft.

Ab der 5. Zeile: Nun betrachten wir die beiden Faktoren einzeln: Beim ersten Faktor (5. Zeile) ergibt sich die Lösung quasi von selbst. Beim zweiten Faktor (7. Zeile) müssen wir noch etwas weiterrechnen. Allerdings handelt es sich bei x^2-2x-24 = 0 um eine einfache quadratische Gleichung, die mit der p-q-Formel gelöst werden kann.

 

Substitution

Bei der Lösung einer Gleichung durch Substitution werden komplizierte Gleichungen vereinfacht und damit in eine Form gebracht, in der wir sie lösen können. Der Begriff "Substitution" leitet sich vom lateinischen Wort "substituere" ab, welches "ersetzen" bedeutet - genau das tun wir bei einer mathematischen Substitution: Wir ersetzen (vorübergehend) kompliziertere Terme durch einfachere. Substituiert man bei Polynomen, ist es das Ziel, dass die Exponenten kleiner werden (z. B. 2 und 1), sodass man idealerweise am Ende die p-q- oder die a-b-c-Formel anwenden kann.
Im Anschluss an das folgende Beispiel 1 werden die einzelnen Schritte erläutert.

Struktur der Gleichung: In einer Gleichung sind nur Terme enthalten, die "Varianten" eines Ausgangsterms sind.

Beispiel 1:
Struktur der Gleichung: In der Gleichung x^4+x^2-72 = 0 sind alle Terme Varianten von x^2. Wir ersetzen dieses x^2 durch u. Aus x^4=\left(x^2\right)^2 wird dann u^2.
Falls Sie sich gefragt haben, was mit der -72 ist: Auch diese ist - wenn man es mathematisch ganz genau nimmt - eine Variante von x^2, nämlich \left(x^2\right)^0, und passt damit ins Muster.

\begin{array}{rclcl}\mathbb{D} &=& \mathbb{R} \cr\cr x^4+x^2-72 &=& 0 \cr\left(x^2\right)^2+x^2-72 &=& 0\end{array}

Substitution: u = x^2
\begin{array}{rclcl}u^2+u-72 &=& 0 &\vert& \text{p-q-Formel} \cr u_{1,2} &=& -\dfrac{1}{2}\pm\sqrt{\left(\dfrac{1}{2}\right)^2-(-72)} \cr &=& -\dfrac{1}{2}\pm\sqrt{\dfrac{289}{4}} \cr \cr u_1 &=& -\dfrac{1}{2}+\dfrac{17}{2} = 8 \cr u_2 &=& -\dfrac{1}{2}-\dfrac{17}{2} = -9\end{array}

Rücksubstitution:
\begin{array}{rclcl} u_1 = x^2 &=& 8 &\vert& \pm\sqrt{} \cr x_{1,2} &=& \pm\sqrt{8} \cr\cr u_2 = x^2 &=& -9 &\vert& \pm\sqrt{} \cr x_{3,4} &=& \pm\sqrt{-9}\end{array}

Da aus negativen reellen Zahlen keine Wurzeln mit geraden Wurzelexponenten gezogen werden können, liefert die Rücksubstitution von u_2 keine weiteren Lösungen: \mathbb{L} = \left\{-\sqrt{8};\sqrt{8}\right\}


Erläuterung der einzelnen Schritte:

  1. Schritt - Substitution: Beim Substituieren wird der Ausgangsterm durch eine neue Variable ersetzt, meist u oder z. Dadurch wird die Gleichung deutlich handlicher.
  2. Schritt - Lösen der vereinfachten Gleichung: Je nach Gestalt der so entstandenen Gleichung kommen verschiedenste Verfahren zum Lösen der Gleichung infrage. In Beispiel oben entsteht eine "normale" quadratische Gleichung, die mithilfe der p-q-Formel gelöst werden kann.
  3. Schritt - Rücksubstitution: Zum Abschluss muss die Substitution rückgängig gemacht werden, da ja Lösungen für x und nicht für u oder z gesucht waren. 


Natürlich muss der Ausgangsterm, der substituiert wird, nicht immer x^2 sein. Daher nun ein zweites Beispiel ... In späteren Kapiteln bei anderen Gleichungstypen werden Sie sehen, dass man noch ganz andere Terme substituieren kann. Daher ist es wichtig, sich hier schon mal mit diesem Vorgehen vertraut zu machen.

Beispiel 2:
Struktur der Gleichung: In der Gleichung z^8 -97z^4+1.296 = 0 sind alle Terme Varianten von z^4. Wir ersetzen dieses z^4 durch u Aus z^8=\left(z^4\right)^2 wird somit u^2 und aus z^0=\left(z^4\right)^0 wird u^0=1. Achtung: z wäre hier kein guter Name für die substituierte Variable, weil ja die Variable der Ausgangsgleichung schon so heißt.

\begin{array}{rclcl} \mathbb{D} &=& \mathbb{R} \cr\cr z^8 -97z^4+1.296z^0 &=& 0 \cr \left(z^4\right)^2-97z^4+1.296\left(z^4\right)^0 &=& 0\end{array}

Substitution: u = z^4
\begin{array}{rclcl}u^2-97u+1.296 &=& 0 &\vert& \text{p-q-Formel} \cr u_{1,2} &=& \dfrac{97}{2}\pm\sqrt{\left(-\dfrac{97}{2}\right)^2-1.296} \cr &=& \dfrac{97}{2}\pm\sqrt{\dfrac{4.225}{4}} \cr \cr u_1 &=& \dfrac{97}{2}+\dfrac{65}{2} = 81 \cr u_2 &=& \dfrac{97}{2}-\dfrac{65}{2} = 16 \cr\end{array}

Rücksubstitution:
\begin{array}{rclcl} u_1 = z^4 &=& 81 &\vert& \pm\sqrt[4]{} \cr z_1 &=& \sqrt[4]{81} = 3 \cr z_2 &=& -\sqrt[4]{81} =-3 \cr\cr u_2 = z^4 &=& 16 &\vert& \pm\sqrt[4]{} \cr z_3 &=& \sqrt[4]{16} = 2 \cr z_4 &=& -\sqrt[4]{16} = -2\end{array}

\mathbb{L} = \left\{-3; -2; 2; 3 \right\}

 

Polynomdivision

Ein ziemlich wichtiges Verfahren beim Lösen von Polynomgleichungen ist die Polynomdivision. Dabei wird - wie der Name schon vermuten lässt - ein Polynom durch ein anderes dividiert. Vorteil dieses Verfahrens ist, dass der Grad des Ausgangspolynoms kleiner wird. Man sagt, "das Polynom wird reduziert". Wenn man Glück hat, ist irgendwann der Punkt erreicht, an dem für den restlichen Lösungsweg Verfahren wie die p-q-Formel weiterhelfen. Abhängig vom Grad der Polynome muss man dafür ggf. mehrfach dividieren. Nachteile dieses Verfahrens sind, dass man eine Lösung schon kennen muss, damit man anfangen kann, die übrigen zu berechnen, und nicht immer erhält man bei Anwendung der Polynomdivision am Ende alle Lösungen ...


Bevor wir uns intensiver mit der Polynomdivision beschäftigen, gehen wir nochmal einen Schritt zurück - so etwa in unsere Grundschulzeit - und erinnern uns an das schriftliche Dividieren von zwei natürlichen Zahlen. Im Großen und Ganzen funktioniert das Dividieren von zwei Polynomen nämlich genauso.

Ein Beispiel für die schriftliche Division:
 Berechnet werden soll 7.236:6. Dafür müssen wir drei Schritte so oft wiederholen, wie Stellen im Dividenden vorhanden sind.

  • 1. Stelle:
    1. Wir prüfen zunächst, wie oft der Divisor, also die 6, in die erste Stelle vom Dividenden, in diesem Fall in die 7, passt. Das ist 1-mal der Fall. Die 1 notieren wir hinter dem Gleichheitszeichen.
    2. Wir multiplizieren "rückwärts", also die gerade gefundene 1 mit dem Divisor von oben: 1\cdot 6=6 und notieren das Ergebnis unter der ersten Stelle vom Dividenden, also unter der 7.
    3. Wir subtrahieren die 6 von der 7, schreiben das Ergebnis unter die Rechnung und ergänzen dort die nächste Stelle vom Dividenden, in diesem Fall die 2.
      \begin{array}{lcl} 7.236:6 &=& 1 \cr\underline{6} \cr12 \end{array}

  • 2. Stelle:
    1. Wir prüfen, wie oft der Divisor in die unterste Zeile unserer Rechnung, in diesem Fall in die 12, passt. Das ist 2-mal der Fall. Die 2 notieren wir hinter dem Gleichheitszeichen und der 1.
    2. Wir multiplizieren "rückwärts": 2\cdot 6=12 und notieren das Ergebnis unter der Rechnung, also unter der 12.
    3. Wir subtrahieren die 12 von der 12, schreiben das Ergebnis unter die Rechnung und ergänzen dort die nächste Stelle, in diesem Fall die 3.
      \begin{array}{lcl} 7.236:6 &=& 12 \cr\underline{6} \cr12 \cr \underline{12} \cr \;\,03 \end{array}

  • 3. Stelle:
    1. Wir prüfen wieder, wie oft der Divisor in die unterste Zeile unserer Rechnung, in diesem Fall in die 03, passt. Das ist gar nicht der Fall. Wir notieren also eine 0 hinter dem Gleichheitszeichen und der 12.
    2. Wir multiplizieren "rückwärts": 0\cdot 6=0 und notieren das Ergebnis unter der Rechnung, also unter der 03.
    3. Wir subtrahieren die 0 von der 3, schreiben das Ergebnis unter die Rechnung und ergänzen dort die nächste Stelle, in diesem Fall die 6.
      \begin{array}{lcl} 7.236:6 &=& 120 \cr\underline{6} \cr12 \cr \underline{12} \cr \;\,03 \cr\;\,\underline{00} \cr \;\,\;\,36 \end{array}

  • 4. Stelle:
    1. Wir prüfen noch einmal, wie oft der Divisor in die unterste Zeile unserer Rechnung, in diesem Fall in die 36, passt. Das ist 6-mal der Fall. Die 6 notieren wir hinter dem Gleichheitszeichen und der 120.
    2. Wir multiplizieren "rückwärts": 6\cdot 6=36 und notieren das Ergebnis unter der Rechnung, also unter der 36.
    3. Wir subtrahieren die 36 von der 36 und schreiben das Ergebnis unter die Rechnung.
      \begin{array}{lcl} 7.236:6 &=& 1.206 \cr\underline{6} \cr12 \cr\underline{12} \cr \;\,03 \cr\;\,\underline{00} \cr \;\,\;\,36 \cr \;\,\;\,\underline{36} \cr \;\,\;\,\;\,0\end{array}


Probe: Um unsere Rechnung zu überprüfen, können wir das Ergebnis mit dem Divisor multiplizieren: 1.206\cdot 6 = 7.236. Passt!

Geschafft! Hoffentlich erinnern Sie sich daran ... Dann ist es nämlich bedeutend einfacher, die Polynomdivision zu durchschauen.


Kommen wir nun konkret dazu, wie die Polynomdivision funktioniert. Die drei sich wiederholenden Schritte sind dabei die gleichen wie oben, nur dass noch ein paar Variablen im Spiel sind. Auch hier nehmen wir uns ein Beispiel vor, nämlich die Polynomgleichung x^3-7x^2-28x+160 = 0, bei der die weiter oben besprochenen Lösungsverfahren nicht weiterhelfen. Um die Polynomdivision durchführen zu können, benötigen wir die Information, dass x=4 eine der Lösungen ist. Das sinnvollste Vorgehen, wie man das herausfindet, schauen wir uns weiter unten an.
Durch die Polynomdivision wird das ursprüngliche Polynom in ein Produkt zweier Faktoren zerlegt. Wichtig dabei ist, dass auf der anderen Seite der Gleichung 0 steht. Denn nur so können wir uns im Anschluss den Satz vom Nullprodukt zunutze machen. Für die gegebene Lösung x = +4 muss mindestens einer der beiden Faktoren 0 werden, damit das gesamte Produkt 0 wird. Der Faktor (x-4) erfüllt genau diese Anforderung. (Wäre die gegebene Lösung x = -4, bekämen wir den Faktor (x+4)). Letztendlich taucht die Lösung im Faktor immer mit umgekehrtem Vorzeichen auf. Ziel der Polynomdivision ist es, noch den zweiten Faktor zu bestimmen. Hierfür dividieren wir das Polynom durch unseren Faktor. Damit kann es losgehen: \left(x^3-7x^2-28x+160\right)\;:\;\left(x-4\right)

  • 1. Schritt:
    1. Wir prüfen, wie oft das x vom Divisor in den ersten Term des Dividenden, in diesem Fall in x^3, passt. Das ist x^2-mal der Fall. x^2 notieren wir hinter dem Gleichheitszeichen.
    2. Wir multiplizieren "rückwärts": x^2 \cdot (x-4) = x^3-4x^2 und notieren das Ergebnis unter dem Dividenden, also unter x^3-7x^2.
    3. Wir subtrahieren x^3-4x^2 von x^3-7x^2. Daher steht in unserer Rechnung in der zweiten Zeile -\left(x^3-4x^2\right) = -x^3+4x^2. Das Ergebnis schreiben wir unter die Rechnung und ergänzen dort den nächsten Term des Dividenden, in diesem Fall -28x. Beim Subtrahieren darauf achten, dass natürlich nur Potenzen mit gleichem Exponenten zusammengerechnet werden dürfen (Potenz- vor Punkt- vor Strichrechnung!).
      Durchführung Polynomdivision

Zur Notation: Ganz wichtig: In der ersten Zeile der Rechnung müssen um Dividend und Divisor auf alle Fälle Klammern gesetzt werden! Sonst würden in diesem Beispiel nur 160 und x durcheinander geteilt werden - Punktrechnung geht ja vor Strichrechnung!

Bemerkung: Den Satz beim 1. Unterpunkt "Wir prüfen, wie oft das x vom Divisor in den ersten Term des Dividenden, in diesem Fall in x^3, passt." könnte man auch so formulieren: "Wir prüfen, mit welchem Faktor man das x vom Divisor multiplizieren muss, um den ersten Term des Dividenden, in diesem Fall in x^3, zu erhalten."

  • 2. Schritt:
    1. Wir prüfen nun, wie oft das x vom Divisor in den ersten Term der untersten Zeile, in diesem Fall in -3x^2-28x, passt. Das ist -3x-mal der Fall. -3x notieren wir hinter dem Gleichheitszeichen und dem x^2.
    2. Wir multiplizieren "rückwärts": -3x \cdot (x-4) = -3x^2+12x und notieren das Ergebnis unter dem Dividenden, also unter -3x^2-28x.
    3. Wir subtrahieren -3x^2+12x von -3x^2-28x, schreiben das Ergebnis unter die Rechnung und ergänzen dort den nächsten Term des Dividenden, in diesem Fall 160.
      Durchführung Polynomdivision

  • 3. Schritt:
    1. Wir prüfen noch einmal, wie oft das x vom Divisor in den ersten Term der untersten Zeile, in diesem Fall in -40x, passt. Das ist -40-mal der Fall. -40 notieren wir hinter dem Gleichheitszeichen und dem x^2-3x.
    2. Wir multiplizieren "rückwärts": -40 \cdot (x-4) = -40x+160 und notieren das Ergebnis unter dem Dividenden, also unter -40x+160.
    3. Wir subtrahieren -40x+160 von -40x+160 und schreiben das Ergebnis unter die Rechnung.
      Durchführung Polynomdivision

Auch hier: Geschafft! 
Das reduzierte Polynom  x^2-3x-40 = 0 ist quadratisch, sodass wir mithilfe der p-q-Formel die restlichen Lösungen ermitteln können: \mathbb{L}=\{-5;4;8\}.


Bemerkung 1: Diese Polynomdivision ist ohne Rest aufgegangen, was die Bestätigung dafür ist, dass x=4 tatsächlich eine Lösung der Gleichung ist. Natürlich kann bei der Polynomdivision auch ein Rest bleiben. Dann war die angenommene Lösung doch keine (Dafür gibt es gleich das 1. Beispiel.) oder man hat sich verrechnet ...

Bemerkung 2: Hinter dem Gleichheitszeichen der letzten Rechnung steht das sogenannte "reduzierte Polynom", welches in diesem Fall quadratisch ist. Das muss aber nicht immer der Fall sein. Das reduzierte Polynom kann natürlich auch einen anderen Grad haben. Dazu schauen wir uns weiter unten noch das 2. Beispiel an.
Merke: Der Grad des reduzierten Polynoms entspricht immer dem Grad vom Ausgangspolynom minus dem Grad vom Divisor.


Wie bei der schriftlichen Division können wir zur Überprüfung unserer Rechnung eine Probe durchführen, indem wir das Ergebnis mit dem Divisor multiplizieren:
\begin{array}{rcl}\left(x^2-3x-40\right)\cdot(x-4) &=& x^3-4x^2-3x^2+12x-40x+160 \cr &=& x^2-7x^2-28x+160\end{array}
Stimmt also!


Zwei etwas andere Beispiele für die Polynomdivision: 
1. Beispiel: 
Wie bei der Division natürlicher Zahlen ist es natürlich auch möglich, dass bei einer Polynomdivision ein Rest bleibt:
Durchführung Polynomdivision
In diesem Fall ist x=4 keine Lösung der Ausgangsgleichung. Hinter dem Gleichheitszeichen steht auch nicht das reduzierte Polynom - da im letzten Summand durch die Variable geteilt wird, handelt es sich bei x^2-3x-47+\frac{-106}{x-4} nicht um ein Polynom, sondern um etwas gebrochen Rationales ...
Die Rechnung wird abgebrochen, wenn der Grad des Polynoms in der untersten Zeile kleiner ist als der Grad des Divisors. Diese Variante kommt vor allem bei gebrochen rationalen Funktionen vor, um diese integrieren zu können oder den globalen Verlauf zu erkennen.

2. Beispiel:
Polynomdivision ist auch mit einem quadratischen Divisor möglich:
Durchführung Polynomdivision
Der Ablauf der Polynomdivision bleibt so wie oben besprochen. Wir prüfen natürlich in diesem Fall, wie oft das x^2 vom Divisor in den ersten Term des Dividenden passt. Es muss dann darauf geachtet werden, dass sich der Grad der Terme beim Rückmultiplizieren jeweils um 2 unterscheidet. Daher bleiben beim Subtrahieren "Lücken", da ja nur Potenzen mit gleichem Exponenten zusammengerechnet werden dürfen (Potenz- vor Punkt- vor Strichrechnung!). Man muss hier also (noch) ein bisschen besser aufpassen ...

Zum Abschluss noch ein paar Worte, wie man die erste Lösung finden kann:

  • Zunächst kann in vielen Fällen die grafische Darstellung des Polynoms helfen.
  • Ansonsten bleibt einem hier (anders als sonst in der Mathematik) nicht viel mehr übrig als Raten und Probieren. Die besten Kandidaten zum Probieren sind die Teiler des konstanten Terms. Es gilt nämlich: Hat eine Polynomfunktion nur ganzzahlige Koeffizienten und besitzt ganzzahlige Nullstellen, dann ist eine der Nullstellen ein Teiler des konstanten Terms.
    Bei unserer Gleichung x^3-7x^2-28x+160 = 0 ist dies die 160 mit den Teilern 1, -1, 2, -2, 4, -4, 5, -5, 8, -8, 10, -10, 16, -16, 20, -20, 32, -32, 40, -40, 80, -80, 160 und -160.
    Für x=1: 1^3-7\cdot 1^2-28\cdot 1+160 = 126 \neq 0
    Für x=-1: (-1)^3-7\cdot (-1)^2-28\cdot (-1)+160 = 180 \neq 0
    Für x=2: 2^3-7\cdot 2^2-28\cdot 2+160 = 84 \neq 0
    Für x=-2: (-2)^3-7\cdot (-2)^2-28\cdot (-2)+160 = 180 \neq 0
    Für x=4: 4^3-7\cdot 4^2-28\cdot 4+160 = 0


Allerdings ist es nicht gesagt, dass man tatsächlich immer eine Lösung durch Probieren finden kann. Nehmen wir als Beispiel das Polynom 315x^3+16x^2-473x+210=0. Die Teiler des konstanten Terms 210 sind 1, -1, 2, -2, 3, -3, 5, -5, 6, -6, 7, -7, 10, -10, 14, -14, 15, -15, 21, -21, 30, -30, 35, -35, 42, -42, 70, -70, 105, -105, 210 und -210. Wenn wir uns den Graphen der zugehörigen Polynomfunktion anschauen, sehen wir, dass keiner dieser Teiler eine Lösung sein kann:

Graph von f(x)=315x^3+16x^2-473x+210

Die Lösungen sind -\dfrac{10}{7}, \dfrac{3}{5} und \dfrac{7}{9}. Das kann man durch einfaches Probieren nicht herausfinden. Insofern helfen uns die beiden oben genannten Strategien hier nicht weiter. Wir finden keine Lösung, mit der wir eine Polynomdivision durchführen könnten. In solchen Fällen bleibt die Situation also leider unklar: Findet man einfach nur keine weiteren Lösungen (Man kann ja mal etwas übersehen ...) oder gibt es tatsächlich keine? Auf diese Frage gibt es keine Antwort. Ganzzahlige Lösungen lassen sich verhältnismäßig leicht finden - aber gebrochene Lösungen (wie hier) oder irrationale (wie \sqrt{12}) sollen ja auch mal vorkommen ... Dann kommt man mit der Polynomdivision leider nicht weiter.

 

So, nun kennen Sie die wichtigsten Verfahren zum Lösen von Polynomgleichungen. Es gibt allerdings keine Garantie, dass man damit tatsächlich zur Lösung einer gegebenen Gleichung kommt. Was man in jedem Fall weiß: Die Anzahl der Lösungen einer Polynomgleichung ist im Bereich der reellen Zahlen maximal so groß wie der Grad des Polynoms. Anders formuliert: Polynome vom Grad n \geq 1 haben im Bereich der reellen Zahlen maximal n Lösungen. In etwas verallgemeinerter Form steckt diese Erkenntnis im sogenannten "Fundamentalsatz der Algebra" und geht auf die Doktorarbeit von Carl Friedrich Gauß zurück. Das liefert einem zumindest eine "Obergrenze", ab der man weiß, dass es keine weiteren Lösungen mehr geben kann.

 

Polynomfunktionen

Kommen wir nun zu den Polynomfunktionen. Statt Polynomfunktion sagt man manchmal auch ganz rationale Funktion. Das ist einfach nur ein anderer Begriff und meint genau das Gleiche. Wir werden uns in den folgenden Abschnitten anschauen, welche Eigenschaften und "besondere Punkte" sie haben. Entscheidend ist dabei meistens, ob der Grad der Polynomfunktion gerade oder ungerade ist, wobei wir im Folgenden nur Polynome mit einem Grad von mindestens 1 betrachten.
Um genau zu sein, ist auch die Funktion f(x)=a_0=a_0 x^0 eine Polynomfunktion geraden Grades. Allerdings ist dieser Sonderfall eines geraden Polynoms so einfach (man könnte auch sagen, er ist trivial), dass es sich nicht lohnt, hier viele Worte zu verlieren ... Alles Wissenswerte dazu können Sie im Kapitel über lineare Funktionen nachlesen.

 

Randverhalten

Bei Polynomen geraden Grades gibt es zwei Möglichkeiten für das Randverhalten: Entweder liefern sowohl sehr kleine x-Werte als auch sehr große x-Werte sehr große Funktionswerte (Anders formuliert: Der Funktionswert geht gegen +\infty.) oder sowohl sehr kleine x-Werte als auch sehr große x-Werte liefern sehr kleine Funktionswerte. "Sehr klein" bedeutet dabei, dass es sich um negative Zahlen mit einem sehr großen Betrag handelt, z. B. -1.000.000.000. Der Funktionswert geht also gegen -\infty.
Hintergrund ist, dass die höchste Potenz in einem Polynom den entscheidenden Einfluss auf den Gesamtverlauf des Graphen hat. Beispielsweise ist x^4 für sehr kleine und sehr große x-Werte deutlich größer als x^2 und überlagert damit den Effekt, den x^2 auf den Verlauf des Graphen nimmt. Außerdem sind Potenzen mit geradem Exponenten immer nichtnegativ, weil "minus mal minus gleich plus" ist. Das Vorzeichen vom Koeffizienten der höchsten Potenz "entscheidet" also, ob die Funktionswerte für sehr kleine und sehr große x-Werte positiv oder negativ sind.

Auch für Polynome ungeraden Grades gibt es zwei mögliche Verläufe: Entweder gehen die Funktionswerte für sehr kleine x-Werte gegen -\infty und die für sehr große x-Werte gegen +\infty. Oder umgekehrt. Wieder liegt es am Vorzeichen vom Koeffizienten der höchsten Potenz, welcher Verlauf vorliegt.

In der linken Grafik sind zwei Polynome geraden Grades abgebildet, in der rechten Grafik zwei Polynome ungeraden Grades. Schauen wir uns das Randverhalten dieser Polynome genauer an:

  • Das Polynom f_1(x)=\frac{1}{6}x^4-\frac{2}{3}x^3+x^2-\frac{2}{3}x+\frac{1}{6} hat einen geraden Grad und die höchste Potenz einen positiven Koeffizienten. Da x^4 nichtnegativ ist für alle x-Werte und der Koeffizient \frac{1}{6} am Vorzeichen nichts ändert, verläuft f_1(x) von +\infty nach +\infty.
  • Bei f_2(x)=-2x^2-8x+1 hat die höchste Potenz einen geraden Exponenten und ihr Koeffizient ist negativ. Der Graph ist nach unten geöffnet. Entsprechend gehen die Funktionswerte für sehr kleine und sehr große x-Werte gegen -\infty.
  • Das Polynom f_3(x)=x^3-9x^2+26x-23 hat einen ungeraden Grad und die höchste Potenz einen positiven Koeffizienten. x^3 ist negativ für negative x-Werte und positiv für positive x-Werte. Der Koeffizient 1 ändert daran nichts, also verläuft f_3(x) von -\infty nach +\infty.
  • Umgekehrt ist es bei f_4(x) =-x^5. Der Potenzterm x^5 ist zwar auch negativ für negative x-Werte und positiv für positive x-Werte. Durch die Multiplikation mit -1 verläuft der Graph von +\infty nach -\infty.

Randverhalten von Polynomfunktionen

 

Symmetrie

Polynome mit ausschließlich geraden Exponenten (auch "gerade Polynome" genannt) sind achsensymmetrisch zur y-Achse, da solche Polynome die Bedingung f(-x)=f(x) immer erfüllen.

Polynome mit ausschließlich ungeraden Exponenten (auch "ungerade Polynome" genannt) sind punktsymmetrisch zum Koordinatenursprung, da solche Polynome die Bedingung -f(-x)=f(x) immer erfüllen.

Bitte beachten Sie, dass ein konstanter Term im Polynom einen geraden Exponenten hat, da man dort ja ein x^0 ergänzen könnte, und 0 eine gerade Zahl ist.
Polynome mit beliebigen Exponenten sind weder achsensymmetrisch zur y-Achse noch punktsymmetrisch zum Koordinatenursprung, können aber zu anderen Geraden bzw. Punkten symmetrisch sein.

In der linken Grafik sind zwei Polynome geraden Grades abgebildet, in der rechten Grafik zwei Polynome ungeraden Grades. Sie sehen:

  • f_1(x)=x^4+x^2+2 ist als gerades Polynom achsensymmetrisch zur y-Achse.
  • Löst man bei f_2(x)=-0{,}1(x-4)^6+5 die Klammer auf, erhält man f_2(x)=-0{,}1x^6+2{,}4x^5-24x^4+128x^3-384x^2+614{,}4x-404{,}6, also ein Polynom mit geraden und ungeraden Exponenten. Der Graph von f_2(x) kann also nicht achsensymmetrisch zur y-Achse sein. Er ist aber achsensymmetrisch zur senkrechten Gerade durch x=4. Das überrascht nicht sehr, wenn wir uns die zusammengefasste Form anschauen: Die -4 in der Klammer bewirkt ja gerade eine Verschiebung des Graphen um 4 Einheiten nach rechts. Die +5 hinter der Klammer bewirkt eine Verschiebung des Graphen um 5 Einheiten nach oben (was aber auf die Achsensymmetrie keinen Einfluss hat).
  • f_3(x)=\frac{3}{10}x^3-\frac{27}{10}x ist als ungerades Polynom punktsymmetrisch zum Koordinatenursprung.
  • Achten Sie bei f_4(x)=-x^3-2x-3=-x^3-2x-3x^0 darauf, dass dies kein Polynom mit ausschließlich ungeraden Exponenten ist (auch wenn es auf den ersten Blick vielleicht so aussieht). Entsprechend ist sein Graph nicht punktsymmetrisch zum Ursprung. Er ist punktsymmetrisch zum Punkt (0\mid -3).

Symmetrie bei Polynomen

 

Nullstellen

Die Anzahl der Nullstellen eines Polynoms ist im Bereich der reellen Zahlen maximal so groß wie sein Grad. Anders formuliert: Polynome vom Grad n \geq 1 haben im Bereich der reellen Zahlen maximal n Nullstellen.
Polynome, die keinen konstanten Term enthalten, haben immer mindestens eine Nullstelle, nämlich (0\mid 0). Umgekehrt formuliert: Wenn alle Summanden im Polynom die Variable enthalten, ist der Funktionswert für x=0 immer 0, z. B. bei f(x)=-14x^6+92x^5-37x.

In der folgenden Grafik sind verschiedene Varianten des Polynoms geraden Grades f(x)=0{,}05x^4-1{,}25x^2 dargestellt. Wir wissen, dass maximal vier Nullstellen auftreten können, da es sich um ein Polynom vierten Grades handelt. Die konkrete Zahl der Nullstellen ist aber abhängig von der Konstanten, die zum Funktionsterm addiert wird:

  • Die oberste der abgebildeten Funktionen f_1(x)=0{,}05x^4-1{,}25x^2+8 hat keine Nullstellen.

  • Die zweitoberste der abgebildeten Funktionen f_2(x)=0{,}05x^4-1{,}25x^2+4 hat vier Nullstellen.

  • Die zweiunterste der abgebildeten Funktionen f_3(x)=0{,}05x^4-1{,}25x^2 hat drei Nullstellen.

  • Die unterste der abgebildeten Funktionen f_4(x)=0{,}05x^4-1{,}25x^2-4 hat zwei Nullstellen.

Nullstellen bei Polynomen geraden Grades


Polynome ungeraden Grades haben mindestens eine Nullstelle. Wenn Sie sich an das Randverhalten von Polynomen ungeraden Grades erinnern, kann das auch gar nicht anders sein: Ein Graph, der ohne Sprünge und Lücken von -\infty nach +\infty verläuft (oder umgekehrt), muss ja die x-Achse mindestens einmal schneiden.

In der folgenden Grafik sind verschiedene Varianten der Funktion f(x)=\frac{1}{27}x^3-x dargestellt. Da es sich um ein Polynom dritten Grades handelt, können maximal drei Nullstellen auftreten. Abhängig von der Konstanten, die addiert wird, ändert sich auch hier die Zahl der Nullstellen. Allerdings bewirkt keine Konstante, dass die Funktion vollständig über oder vollständig unter der x-Achse liegt:

  • Die oberste der abgebildeten Funktionen f_1(x)=\frac{1}{27}x^3-x+4 hat eine Nullstelle.

  • Die mittlere der abgebildeten Funktionen f_2(x)=\frac{1}{27}x^3-x+2 hat zwei Nullstellen.

  • Die unterste der abgebildeten Funktionen f_3(x)=\frac{1}{27}x^3-x hat drei Nullstellen.

Nullstellen bei Polynomen ungeraden Grades

 

Extrempunkte

Polynome vom Grad n\geq 2 haben im Bereich der reellen Zahlen maximal n-1 Extrempunkte. Ob es sich dabei um Hoch- oder um Tiefpunkte handelt, hängt vom konkreten Verlauf des Graphen ab.
Bei so vernünftig verlaufenden Funktionen wie Polynomen, die keine Sprünge, Lücken oder sonstige Auffälligkeiten haben, liegt zwischen zwei Hochpunkten immer ein Tiefpunkt und zwischen zwei Tiefpunkten immer ein Hochpunkt. Salopp formuliert: Wenn Sie auf einem Gipfel stehen, müssen Sie erstmal ins Tal, bevor Sie den nächsten Gipfel erklimmen können.
Auch zwischen zwei Nullstellen muss immer mindestens ein Extrempunkt liegen (Ausnahme ist das Polynom f(x)=0, was ja quasi nur aus Nullstellen besteht.). Umgekehrt gilt dies aber nicht: Ein Extrempunkt muss nicht immer zwischen zwei Nullstellen liegen. Eine Funktion, die beispielsweise so weit nach oben verschoben ist, dass sie die x-Achse nicht schneidet, darf ja trotzdem Extrempunkte haben.

Polynome geraden Grades haben immer mindestens ein Extrempunkt. Bei Polynomen ungeraden Grades weiß man nicht, ob sie Extrempunkte haben oder nicht. Beides ist möglich.

 

Wende- und Sattelpunkte

Polynome vom Grad n\geq 3 haben im Bereich der reellen Zahlen maximal n-2 Wende- bzw. Sattelpunkte.
Bei Polynomfunktionen, die ja - wie oben schon erwähnt - vernünftig verlaufen, muss sich zwischen zwei Extrempunkten immer mindestens ein Wendepunkt befinden. Allerdings müssen zwischen zwei Wendepunkten nicht zwingend auch Extrempunkte liegen.

Bei Polynomen geraden Grades kann man allgemein nichts über das Vorhandensein von Wende- und Sattelpunkten sagen: Sie können welche haben oder auch nicht. Polynome ungeraden Grades haben immer mindestens einen Wende- bzw. Sattelpunkt.

In der linken Grafik sind wieder zwei Polynome geraden Grades abgebildet, in der rechten Grafik zwei Polynome ungeraden Grades. Sie sehen:

  • Da das Polynom geraden Grades f_1(x)=-(x-3)^4-5 ein verschobenes, nach unten geöffnetes x^4-Polynom mit dem Hochpunkt (3\mid -5) ist, hat der Graph keinen Wende- oder Sattelpunkt, sondern nur einen Hochpunkt.
  • Bei f_2(x)=0{,}05x^4-1{,}25x^2+6, ebenfalls ein Polynom geraden Grades, passiert mehr: Es hat zwei Wendepunkt bei (-2{,}04\mid 1{,}66) und (2{,}04\mid 1{,}66), dazu vier Nullstellen und drei Extrempunkte.
  • Das Polynom ungeraden Grades f_3(x)=-1{,}4(x+6)^5 hat einen Sattelpunkt bei (-6\mid 0), der gleichzeitig Nullstelle ist. Hoch- und Tiefpunkte gibt es nicht.
  • Bei dem Polynom f_4(x)=\frac{1}{27}x^3-x-4 ist der Wendepunkt (0\mid -4) eingerahmt von zwei Extrempunkten. Eine Nullstelle gibt es auch noch.

Wende- und Sattelpunkte bei verschiedenen Polynomen

 

Polstellen

Da Polynome aus mathematischer Sicht für die gesamten reellen Zahlen definiert sind und somit keine Definitionslücken haben, können keine Polstellen auftreten. Auch Einschränkungen des Definitionsbereichs aufgrund inhaltlicher Rahmenbedingungen erzeugen keine Polstellen.

 

Spezielle Polynome

Einige häufig auftretende Polynome haben spezielle Bezeichnungen - unabhängig davon, ob es sich um eine Polynomgleichung oder eine Polynomfunktion handelt (a_4, a_3, a_2, a_1, a_0 \in \mathbb{R}):

  • Konstantes Polynom: p(x) = a_0

  • Lineares Polynom: 0 = a_1x+a_0 bzw. p(x) = a_1x+a_0 mit a_1\neq 0
    Statt "lineares Polynom" kann man auch "Polynom ersten Grades" sagen. Egal, wie man solche Polynome nennt, Sie kennen sie schon aus den Kapiteln lineare Gleichungen und lineare Funktionen.

  • Quadratisches Polynom: 0 = a_2x^2+a_1x+a_0 bzw. p(x) = a_2x^2+a_1x+a_0 mit a_2\neq 0
    Statt "quadratisches Polynom" kann man auch "Polynom zweiten Grades" sagen. Auch diese Polynome kennen Sie schon aus den Kapiteln quadratische Gleichungen und quadratische Funktionen.

  • Kubisches Polynom: 0 = a_3x^3+a_2x^2+a_1x+a_0 bzw. p(x) = a_3x^3+a_2x^2+a_1x+a_0 mit a_3\neq 0
    Statt "kubisches Polynom" sagt man häufig auch "Polynom dritten Grades".

  • Biquadratisches Polynom: 0 = a_4x^4+a_2x^2+a_0 bzw. p(x) = a_4x^4+a_2x^2+a_0 mit a_4 \neq 0, a_3=0, a_2\neq 0 und a_1=0
    Ein biquadratisches Polynom ist ein spezielles Polynom vierten Grades, bei dem die Koeffizienten a_3 und a_1 immer 0 sind. Es bleiben also nur die Potenzen x^4 und x^2 (ggf. mit Koeffizienten) und eventuell der konstante Term übrig.

Übersicht:

 

13.3 Polynomgleichungen und -funktionen - Lösungen

1. Aufgabe

1) Grad n=2
 
2) Grad n=4
 
3) Grad n=21
 
4) Grad n=0
 
5) Grad n=16
 
6) Grad n=6
 
Bemerkung: Der Grad ist hier 6 und nicht 7, weil x^7 den Koeffizienten 0 hat.

7) Grad n=11

8) Grad n=19

9) Grad n=31

10) Grad n=18

 

2. Aufgabe

1)
\begin{array}{crclcl} & \mathbb{D} &=& \mathbb{R} \cr \cr & x^3-5x^2-6x &=& 0 \cr & x\left(x^2-5x-6\right) &=& 0 & \vert & \text{Satz vom Nullprodukt} \cr \text{Faktor 1:} & x_1 &=& 0 \cr\cr \text{Faktor 2:} & x^2-5x-6 &=& 0 &\vert& \text{p-q-Formel} \cr & x_{2,3} &=& \dfrac{5}{2} \pm \sqrt{ \left(- \dfrac{5}{2} \right)^2+6} \cr\cr & &=& \dfrac{5}{2} \pm \sqrt{\dfrac{49}{4}} \cr \cr & x_2 &=& \dfrac{5}{2}+ \dfrac{7}{2} = 6 \cr\cr & x_3 &=& \dfrac{5}{2}- \dfrac{7}{2} = -1 \cr \cr & \mathbb{L} &=& \{-1;0;6\} \end{array}

 
2)
\begin{array}{rclcl} \mathbb{D} &=& \mathbb{R} \cr \cr \dfrac{1}{3}x^4+12 &=& 0 & \vert & -12 \cr \dfrac{1}{3}x^4 &=& -12 & \vert & \cdot 3 \cr x^4 &=& -36 & \vert &\pm\sqrt[4]{} \cr x_{1,2} &=& \pm\sqrt[4]{-36} \end{array}
 
Da aus negativen reellen Zahlen keine Wurzeln mit geraden Wurzelexponenten gezogen werden können, hat diese Gleichung keine Lösung: \mathbb{L} = \emptyset


3)
\begin{array}{crclcl} & \mathbb{D} &=& \mathbb{R} \cr\cr & \dfrac{x^3}{8} &=& x^2+2x-16 & \vert & \cdot 8 \cr\cr & x^3 &=& 8x^2+16x-128 & \vert & -x^3 \cr & 0 &=& -x^3+8x^2+16x-128 & \vert & \cdot (-1) \cr & 0 &=& x^3-8x^2-16x+128 \end{array}

Durch Probieren finden wir die Lösung x_1=4, denn
\begin{array}{rcl}0 &=& 4^3-8\cdot 4^2-16\cdot 4+128 \\ 0 &=& 0\end{array}

Polynomdivision mit x_1=4:
Durchführung Polynomdivision

Lösung des reduzierten Polynoms:
\begin{array}{rclcl} 0 &=& x^2-4x-32 &\vert& \text{p-q-Formel} \cr x_{1,2} &=& 2\pm\sqrt{(-2)^2+32} \cr x_{1,2} &=& 2\pm\sqrt{36} \cr\cr x_1 &=& 2+6=8 \cr x_2 &=& 2-6=-4 \cr\cr \mathbb{L} &=& \{-4;4;8\} \end{array}


4)
\begin{array}{crclcl} & \mathbb{D} &=& \mathbb{R} \cr \cr & 2x^3-x^2+6x &=& 3x^2 &\vert& -3x^2 \cr & 2x^3-4x^2+6x &=& 0 \cr & x\left(2x^2-4x+6\right) &=& 0 & \vert & \text{Satz vom Nullprodukt} \cr \text{Faktor 1:} & x_1 &=& 0 \cr\cr \text{Faktor 2:} & 2x^2-4x+6 &=& 0 & \vert & :2 \cr & x^2-2x+3 &=& 0 &\vert& \text{p-q-Formel} \cr & x_{2,3} &=& 1 \pm \sqrt{(-1)^2-3} \cr & &=& 1 \pm \sqrt{-2} \end{array}

Da aus negativen reellen Zahlen keine Wurzeln mit geraden Wurzelexponenten gezogen werden können, liefert der zweite Faktor keine weiteren Lösungen: \mathbb{L} = \{0\}


5)
\begin{array}{rclcl} \mathbb{D} &=& \mathbb{R} \cr \cr x^{10}-56x^5+768 &=& 0 \cr \left(x^5\right)^2-56x^5+768 &=& 0 \end{array}

Substitution: z=x^5
\begin{array}{rclcl} z^2-56z+768 &=& 0 &\vert& \text{p-q-Formel} \cr z_{1,2} &=& 28 \pm \sqrt{\left(-28\right)^2-768} \cr &=& 28 \pm \sqrt{16} \cr\cr z_1 &=& 28+4 = 32 \cr z_2 &=& 28-4 = 24 \end{array}

Rücksubstitution:
\begin{array}{rclcl} z_1 = x_1^5 &=& 32 &\vert& \sqrt[5]{} \cr x_1 &=& \sqrt[5]{32} \cr x_1 &=& 2 \cr\cr z_2 = x_2^5 &=& 24 &\vert& \sqrt[5]{} \cr x_2 &=& \sqrt[5]{24} \approx 1{,}89 \cr\cr \mathbb{L} &=& \left\{\sqrt[5]{24}; 2\right\}\end{array}


6)
\begin{array}{crclcl} & \mathbb{D} &=& \mathbb{R} \cr \cr & y^3+y^2+4 &=& -4y &\vert& +4y \cr & y^3+y^2+4y+4 &=& 0 \cr & \left(y^3+y^2\right)+(4y+4) &=& 0 \cr & y^2(y+1)+4(y+1) &=& 0 \cr & \left(y^2+4\right)(y+1) &=& 0 &\vert& \text{Satz vom Nullprodukt} \cr \text{Faktor 1:} & y^2+4 &=& 0 &\vert& -4 \cr & y^2 &=& -4 &\vert& \pm\sqrt{} \cr & y_{1,2} &=& \pm\sqrt{-4} \cr\cr \text{Faktor 2:} & y+1 &=& 0 &\vert& -1 \cr & y &=& -1 \end{array}

Da aus negativen reellen Zahlen keine Wurzeln mit geraden Wurzelexponenten gezogen werden können, liefert der erste Faktor keine weiteren Lösungen: \mathbb{L} = \left\{-1\right\}


7)
\begin{array}{crclcl} & \mathbb{D} &=& \mathbb{R} \cr\cr & \dfrac{1}{2}x^3+\dfrac{13}{2}x^2-2x &=& 26 & \vert & \cdot 2 \cr & x^3+13x^2-4x &=& 52 & \vert & -52 \cr & x^3+13x^2-4x-52 &=& 0 \end{array}

Durch Probieren finden wir die Lösung x_1=-13, denn
\begin{array}{rcl}(-13)^3+13\cdot(-13)^2-4\cdot (-13)-52 &=& 0 \\ 0 &=& 0\end{array}

Polynomdivision mit x_1 = -13:
Durchführung Polynomdivision

Lösung des reduzierten Polynoms:
\begin{array}{rclcl} 0 &=& x^2-4 &\vert& +4 \cr x^2 &=& 4 &\vert& \pm\sqrt{} \cr\cr x_1 &=& -2 \cr x_2 &=& 2 \cr\cr \mathbb{L} &=& \{-13;-2;2\} \end{array}


8)
\begin{array}{crclcl} & \mathbb{D} &=& \mathbb{R} \cr \cr & 11x^3+5x^5+2x &=& 0 \cr & x\left(5x^4+11x^2+2\right) &=& 0 &\vert& \text{Satz vom Nullprodukt} \cr \text{Faktor 1:} & x_1 &=& 0 \cr\cr \text{Faktor 2:} & 5x^4+11x^2+2 &=& 0 \cr & 5\left(x^2\right)^2+11x^2+2 &=& 0 \end{array}

Substitution: z=x^2
\begin{array}{rclcl}5z^2+11z+2 &=& 0 &\vert& \text{a-b-c-Formel} \\z_{1,2} &=& \dfrac{-11 \pm \sqrt{11^2-4 \cdot 5 \cdot 2}}{2\cdot 5} \\\\&=& \dfrac{-11 \pm \sqrt{81}}{10} \\\\z_{1} &=& \dfrac{-11 + 9}{10} = -\dfrac{1}{5}\\\\z_{2} &=& \dfrac{-11 - 9}{10} =-2\\\\\end{array}

Rücksubstitution: 
\begin{array}{rclcl} z_1 = x^2 &=& -\dfrac{1}{5} &\vert& \pm\sqrt{} \cr\cr x_{2,3} &=& \pm\sqrt{-\dfrac{1}{5}} \cr\cr z_2 = x^2 &=& -2 &\vert& \pm\sqrt{} \cr\cr x_{4,5} &=& \pm\sqrt{-2} \end{array} 

Da aus negativen reellen Zahlen keine Wurzeln mit geraden Wurzelexponenten gezogen werden können, liefert der zweite Faktor keine weiteren Lösungen: \mathbb{L} = \left\{0\right\}


9)
\begin{array}{crclcl} & \mathbb{D} &=& \mathbb{R} \cr\cr & 0 &=& x^4+5x^3-x^3\ln(5)-x^25\ln(5) \cr & 0 &=& x^2\left[x^2+\left(5-\ln(5)\right)x-5\ln(5)\right] &\vert& \text{Satz vom Nullprodukt} \cr \text{Faktor 1:} & x^2 &=& 0 &\vert& \pm\sqrt{} \cr& x_1 &=& 0 \cr\cr\text{Faktor 2:} & x^2+\left(5-\ln(5)\right)x-5\ln(5) &=& 0 &\vert& \text{p-q-Formel} \cr & x_{2,3} &=& -\dfrac{5-\ln(5)}{2}\pm\sqrt{\left(\dfrac{5-\ln(5)}{2}\right)^2+5\ln(5)} \cr & x_{2,3} &=& -\dfrac{5-\ln(5)}{2}\pm\sqrt{\dfrac{25-10\ln(5)+\left(\ln(5)\right)^2}{4}+\dfrac{20\ln(5)}{4}} \cr\cr & x_{2,3} &=& -\dfrac{5-\ln(5)}{2}\pm\sqrt{\dfrac{25+10\ln(5)+\left(\ln(5)\right)^2}{4}} \cr\cr & x_{2,3} &=& -\dfrac{5-\ln(5)}{2}\pm\sqrt{\left(\dfrac{5+\ln(5)}{2}\right)^2} \cr\cr & x_{2,3} &=& -\dfrac{5-\ln(5)}{2}\pm\dfrac{5+\ln(5)}{2} \cr\cr & x_2 &=& -\dfrac{5-\ln(5)}{2}+\dfrac{5+\ln(5)}{2} = \dfrac{-\left(5-\ln(5)\right)+\left(5+\ln(5)\right)}{2} = \dfrac{2\ln(5)}{2} = \ln(5) \cr\cr & x_3 &=& -\dfrac{5-\ln(5)}{2}-\dfrac{5+\ln(5)}{2} = \dfrac{-\left(5-\ln(5)\right)-\left(5+\ln(5)\right)}{2} = -5 \cr\cr & \mathbb{L} &=& \{-5; 0; \ln(5)\} \end{array}


10)
\begin{array}{rclcl} \mathbb{D} &=& \mathbb{R} \cr\cr 8\left(\dfrac{3}{8}x^4-6\right) &=& 3\left(x^2-4\right)\left(x^2+4\right) \cr 3x^4-48 &=& 3\left(x^4-16\right) \cr3x^4-48 &=& 3x^4-48 &\vert& -3x^4+48 \cr0 &=& 0 \cr\cr\mathbb{L} &=& \mathbb{R}\end{array}

Bemerkung: Unabhängig davon, welches Element des Definitionsbereichs in diese Gleichung eingesetzt wird, erhält man immer auf beiden Seiten dasselbe Ergebnis. 0=0 ist schließlich immer richtig. Jede reelle Zahl löst also diese Gleichung, d. h. die Lösungsmenge entspricht dem Definitionsbereich.


11)
\begin{array}{rclcl} \mathbb{D} &=& \mathbb{R} \cr \cr -12x\left(x^2-19\right) &=& 24\left(-x^2+10\right) \cr -12x^3+228x &=& -24x^2+240 &\vert& +24x^2-240 \cr -12x^3+24x^2+228x-240 &=& 0 \end{array}

Durch Probieren finden wir die Lösung x_1=1, denn
\begin{array}{rcl}-12\cdot 1^3+24\cdot 1^2+228\cdot 1-240 &=& 0 \\ 0 &=& 0\end{array}

Polynomdivision mit x_1=1:
Durchführung Polynomdivision

Lösung des reduzierten Polynoms:
\begin{array}{rclcl} 0 &=& -12x^2+12x+240 &\vert& \text{a-b-c-Formel} \cr x_{2,3} &=& \dfrac{-12\pm\sqrt{12^2-4\cdot(-12)\cdot 240}}{2\cdot(-12)} \cr\cr x_{2,3} &=& \dfrac{-12\pm\sqrt{11.664}}{-24} \cr\cr x_2 &=& \dfrac{-12+108}{-24} = -4 \cr\cr x_3 &=& \dfrac{-12-108}{-24} = 5 \cr\cr \mathbb{L} &=& \{-4;1;5\} \end{array}


12)
\begin{array}{rclcll}\mathbb{D} &=& \mathbb{R} \\\\-\dfrac{1}{81}z^3\left(9z\left(-9z^4+2\right)\right) &=& \dfrac{2}{27} \\\\-\dfrac{1}{81}z^3\left(-81z^5+18z\right) &=& \dfrac{2}{27} \\\\z^8-\dfrac{2}{9}z^4 &=& \dfrac{2}{27} &\vert & -\dfrac{2}{27} \\\\z^8-\dfrac{2}{9}z^4-\dfrac{2}{27} &=& 0 \\\end{array}

Substitution: u=z^4
\begin{array}{rclcl}u^2-\dfrac{2}{9}u-\dfrac{2}{27} &=& 0 &\vert& \text{p-q-Formel}\\\\u_{1,2} &=& \dfrac{1}{9}\pm\sqrt{\left(-\dfrac{1}{9}\right)^2+\dfrac{2}{27}}\\\\u_{1,2} &=& \dfrac{1}{9}\pm\sqrt{\dfrac{7}{81}}\\\\u_{1,2} &=& \dfrac{1}{9}\pm\dfrac{\sqrt{7}}{9}\\\\u_1 &=& \dfrac{1+\sqrt{7}}{9} \approx 0{,}41 \\\\u_2 &=& \dfrac{1-\sqrt{7}}{9} \approx -0{,}18\end{array}

Rücksubstitution:
\begin{array}{rclcl}u_1 = z^4 &=& \dfrac{1+\sqrt{7}}{9} &\vert & \pm\sqrt[4]{} \\\\ z_{1,2} &=& \pm\sqrt[4]{\dfrac{1+\sqrt{7}}{9}} \approx \pm 0{,}80 \\\\ u_2 = z^4 &=& \dfrac{1-\sqrt{7}}{9} &\vert & \pm\sqrt[4]{}\\\\ z_{3,4} &=& \pm\sqrt[4]{\dfrac{1-\sqrt{7}}{9}}\end{array}

Da aus negativen reellen Zahlen keine Wurzeln mit geraden Wurzelexponenten gezogen werden können, liefert die Rücksubstitution von u_2 keine weiteren Lösungen: \mathbb{L} = \left\{-\sqrt[4]{\dfrac{1+\sqrt{7}}{9}}; \sqrt[4]{\dfrac{1+\sqrt{7}}{9}}\right\}


13)
\begin{array}{rclcl} \mathbb{D} &=& \mathbb{R} \cr \cr 2(x+5)^2-(x-4)(x+5) &=& (x-4)^2 \cr 2x^2+20x+50-x^2-x+20 &=& x^2-8x+16 \cr x^2+19x+70 &=& x^2-8x+16 &\vert& -x^2+8x-70 \cr 27x &=& -54 &\vert& :27 \cr x &=& -2 \cr \cr \mathbb{L} &=& \{-2\} \end{array}


14)
\begin{array}{crclcl} & \mathbb{D} &=& \mathbb{R} \cr\cr & 14x^2\left(x^2-2\right)+28x^3 &=& 14x &\vert& -14x \cr & 14x^4+28x^3-28x^2-14x &=& 0 \cr & x\left(14x^3+28x^2-28x-14\right) &=& 0 & \vert & \text{Satz vom Nullprodukt} \cr \text{Faktor 1:} & x_1 &=& 0 \cr\cr \text{Faktor 2:} & 14x^3+28x^2-28x-14 &=& 0 \end{array}

Durch Probieren finden wir die Lösung x_2=1, denn
\begin{array}{rcl}14\cdot 1^3+28\cdot 1^2-28\cdot 1-14 &=& 0 \\ 0 &=& 0\end{array}

Polynomdivision mit x_2=1:
Durchführung Polynomdivision
Lösung des reduzierten Polynoms:
\begin{array}{rclcl} 0 &=& 14x^2+42x+14 &\vert& : 14 \cr 0 &=& x^2+3x+1 &\vert& \text{p-q-Formel} \cr x_{3,4} &=& -\dfrac{3}{2} \pm \sqrt{\left(\dfrac{3}{2}\right)^2-1} \cr\cr x_{3,4} &=& -\dfrac{3}{2} \pm \sqrt{\dfrac{5}{4}} \cr\cr x_3 &=& -\dfrac{3}{2}+\dfrac{\sqrt{5}}{2} = \dfrac{-3+\sqrt{5}}{2} \approx -0{,}38 \cr\cr x_4 &=& -\dfrac{3}{2}-\dfrac{\sqrt{5}}{2} = \dfrac{-3-\sqrt{5}}{2} \approx -2{,}62 \cr\cr \mathbb{L} &=& \left\{\dfrac{-3-\sqrt{5}}{2};\dfrac{-3+\sqrt{5}}{2};0;1\right\} \end{array}


15)
\begin{array}{rclcl} \mathbb{D} &=& \mathbb{R} \cr\cr \dfrac{9}{4}\left(\dfrac{4x}{3}\right)^4-4x\left(\dfrac{4x}{3}\right)^3 &=& -192 \cr\cr \dfrac{9}{4} \cdot \dfrac{256}{81}x^4-4x \cdot \dfrac{64x^3}{27} &=& -192 \cr\cr \dfrac{64}{9}x^4-\dfrac{256}{27}x^4 &=& -192 \cr\cr -\dfrac{64}{27}x^4 &=& -192 & \vert & :\left(-\dfrac{64}{27}\right) \cr\cr x^4 &=& 81 & \vert & \pm\sqrt[4]{} \cr x_{1,2} &=& \pm3 \cr\cr \mathbb{L} &=& \{-3;3\} \end{array}


16)
\begin{array}{crclcl} & \mathbb{D} &=& \mathbb{R} \cr \cr & \left(x^2-1\right)^2 &=& x^3+1 \cr & x^4-2x^2+1 &=& x^3+1 &\vert& -x^3-1 \cr & x^4-x^3-2x^2 &=& 0 \cr & x^2\left(x^2-x-2\right) &=& 0 &\vert& \text{Satz vom Nullprodukt} \cr \text{Faktor 1:} & x^2 &=& 0 &\vert& \pm\sqrt{} \cr & x_1 &=& 0 \cr\cr \text{Faktor 2:} & x^2-x-2 &=& 0 &\vert& \text{p-q-Formel} \cr & x_{2,3} &=& \dfrac{1}{2} \pm \sqrt{ \left(-\dfrac{1}{2} \right)^2+2} \cr\cr & &=& \dfrac{1}{2} \pm \sqrt{\dfrac{9}{4}} \cr\cr & x_2 &=& \dfrac{1}{2}+ \dfrac{3}{2} = 2 \cr\cr & x_3 &=& \dfrac{1}{2}- \dfrac{3}{2} = -1 \cr \cr & \mathbb{L} &=& \{-1;0;2\} \end{array}
 

17)
\begin{array}{lrclclcl} & \mathbb{D} &=& \mathbb{R} \cr\cr & \left(5x^2-\sqrt{5}x\right)\left(x^3+x^2-4x+2\right) &=& 0 &\vert& \text{Satz vom Nullprodukt} \cr \text{Faktor 1:} & 5x^2-\sqrt{5}x &=& 0 \cr & x\left(5x-\sqrt{5}\right) &=& 0 &\vert& \text{Satz vom Nullprodukt} \cr \text{Faktor 1.1:} & x_1 &=& 0 \cr\cr \text{Faktor 1.2:} & 5x-\sqrt{5} &=& 0 &\vert& +\sqrt{5} \cr & 5x &=& \sqrt{5} &\vert& :5 \cr & x_2 &=& \dfrac{\sqrt{5}}{5} \approx 0{,}45 \cr\cr \text{Faktor 2:} & x^3+x^2-4x+2 &=& 0 \end{array}

Durch Probieren finden wir die Lösung x_3=1, denn
\begin{array}{rcl}1^3+1^2-4\cdot 1+2 &=& 0 \\0 &=& 0\end{array}

Polynomdivision mit x_3 = 1:
Durchführung Polynomdivision

Lösung des reduzierten Polynoms:
\begin{array}{rclcl} 0 &=& x^2+2x-2 &\vert& \text{p-q-Formel} \cr x_{4,5} &=& -1\pm\sqrt{1^2+2} \cr\cr x_4 &=& -1+\sqrt{3} \approx 0{,}73 \cr\cr x_5 &=& -1-\sqrt{3} \approx -2{,}73 \cr\cr \mathbb{L} &=& \left\{-1-\sqrt{3}; 0; \dfrac{\sqrt{5}}{5}; -1+\sqrt{3}; 1\right\} \end{array}


18)
\begin{array}{crclcl} & \mathbb{D} &=& \mathbb{R} \cr\cr& 12x^2 \cdot \left(x-1 \right) &=& - \sqrt{144p} \cdot x \cdot \left(x-1 \right) &\vert& + \sqrt{144p} \cdot x \cdot \left(x-1 \right) \cr & 12x^2 \cdot \left(x-1 \right) + \sqrt{144p} \cdot x \cdot \left(x-1 \right) &=& 0 \cr & x \cdot \left(x-1 \right) \cdot \left(12x + \sqrt{144p} \right) &=& 0 &\vert& \text{Satz vom Nullprodukt} \cr \text{Faktor 1:} & x_{1} &=& 0 \cr \cr \text{Faktor 2:} & x_{2} -1 &=& 0 &\vert& +1 \cr & x_{2} &=& 1 \cr \cr \text{Faktor 3:} & 12x_{3} + \sqrt{144p} &=& 0 \cr & 12x_{3} &=& - \sqrt{144p} \cr & 12x_{3} &=& -12 \sqrt{p} &\vert& :12 \cr & x_{3} &=& - \sqrt{p} \cr \cr & \mathbb{L} &=& \left\{- \sqrt{p} ; 0 ; 1 \right\} \end{array}

Alternativer Lösungsweg:
\begin{array}{rclcll}\mathbb{D} &=& \mathbb{R} \\\\12x^2\left(x-1\right) &=& -\sqrt{144p}\cdot x\left(x-1\right)\end{array}

Man könnte als Alternative zu dem Lösungsweg oben auf die Idee kommen, die Gleichung durch (x-1) zu teilen. Schließlich ist (x-1) auf beiden Seiten der Gleichung als Faktor enthalten. Allerdings ist dann eine Fallunterscheidung nötig, da wir durch die Variable dividieren.

1. Fall: Wir nehmen an, dass x-1\neq 0 ist. Das bedeutet, dass x\neq 1 ist:
In diesem Fall dürfen wir durch (x-1) teilen. Eine Division durch 0 ist ja ausgeschlossen.
\begin{array}{lrclcll}& 12x^2\left(x-1\right) &=& -\sqrt{144p}\cdot x\cdot\left(x-1\right) &\vert & :\left(x-1\right) \\& 12x^2 &=& -\sqrt{144p}\cdot x &\vert & +\sqrt{144p}\cdot x \\& 12x^2+\sqrt{144p}\cdot x &=& 0 \\& x\left(12x+\sqrt{144p}\right) &=& 0 &\vert &\text{Satz vom Nullprodukt}\\\\\text{Faktor 1:} & x_1 &=& 0 \\\\\text{Faktor 2:} & 12x+\sqrt{144p} &=& 0 &\vert & -\sqrt{144p} \\& 12x &=& -\sqrt{144p} \\& 12x &=& -12\sqrt{p} &\vert & :12 \\& x_2 &=& -\sqrt{p}\end{array}

Nun müssen wir noch die beiden errechneten Werte mit der Eintrittsbedingung abgleichen: x_1 = 0 \neq 1 und x_2 = -\sqrt{p} \neq 1. Geht also beides in Ordnung.

2. Fall: Wir nehmen an, dass x-1=0, also x=1 ist:
In diesem Fall dürfen wir nicht durch (x-1) teilen, da wir dann durch 0 dividieren würden. Ersetzen wir in der Gleichung (x-1) durch 0, erhalten wir
\begin{array}{rclcll}12x^2\cdot 0 &=& -\sqrt{144p}\cdot x\cdot 0 \\0 &=& 0\end{array}

Dies ist eine wahre Aussage. Das heißt, x_3=1 erfüllt die Gleichung ebenfalls. Wir haben also eine weitere Lösung gefunden.

Lösung: Die Lösungsmenge besteht aus den Lösungen beider Fälle: \mathbb{L} = \left\{-\sqrt{p};0;1\right\}

Bemerkung: Sie sehen an diesem Beispiel, dass es keine gute Idee wäre, nur durch (x-1) zu dividieren, ohne die beiden entstehenden Fälle zu unterscheiden. Dann würden wir nämlich die Lösung x_3=1 "verlieren".


19)
\begin{array}{crclcl} & \mathbb{D} &=& \mathbb{R} \cr\cr & 0 &=& \left(-\pi-k^2+x^2\right)\left(x^2-\pi^2-kx^2+k^2\pi^2\right) &\vert& \text{Satz vom Nullprodukt} \cr \text{Faktor 1:} & -\pi-k^2+x^2 &=& 0 &\vert& +\pi+k^2 \cr & x^2 &=& \pi+k^2 &\vert& \pm\sqrt{} \cr & x_{1,2} &=& \pm\sqrt{\pi+k^2} \end{array}

Für welche Werte des Parameters k sind diese Werte Lösungen der Gleichung? Dazu müssen wir schauen, was im Radikanden passiert. Der darf ja nun mal nicht kleiner als 0 sein ... Hier haben wir keine Probleme: \pi ist positiv, k^2 ist nichtnegativ, beides wird addiert. Bleibt also positiv. x_{1,2} sind also ohne Einschränkung Lösungen der Gleichung.

\begin{array}{rrclcl}\text{Faktor 2:} & x^2-\pi^2-kx^2+k^2\pi^2 &=& 0 \cr & x^2-kx^2-\pi^2+k^2\pi^2 &=& 0 \cr & \left(1-k\right)x^2-(1-k^2)\pi^2 &=& 0 \\& \left(1-k\right)x^2-\left(1-k\right)\left(1+k\right)\pi^2 &=& 0 \\& (1-k)\left[x^2-(1+k)\pi^2\right] &=& 0\end{array}

Bevor wir an dieser Stelle durch (1-k) teilen, müssen wir eine Fallunterscheidung vornehmen, weil wir nicht wissen, welchen Wert (1-k) annimmt. (1-k) könnte ja 0 sein.

1. Fall: Wir nehmen an, dass 1-k\neq 0 ist. Anders formuliert: Wir nehmen an, dass k\neq 1 ist:
In diesem Fall dürfen wir durch (1-k) teilen. Eine Division durch 0 ist ja ausgeschlossen.
\begin{array}{rclcll}(1-k)\left[x^2-(1+k)\pi^2\right] &=& 0 &\vert& :(1-k) \\x^2-(1+k)\pi^2 &=& 0 &\vert& +(1+k)\pi^2 \\x^2 &=& \left(1+k\right)\pi^2 &\vert& \pm\sqrt{} \\x_{3,4} &=& \pm\sqrt{\left(1+k\right)\pi^2}\end{array}

Jetzt müssen wir noch prüfen, für welche Werte von k der Radikand nichtnegativ ist:
\begin{array}{rclcll}\left(1+k\right)\pi^2 &\geq & 0 &\vert & :\pi^2\\1+k &\geq & 0 &\vert & -1\\k &\geq & -1 \end{array}

Nun müssen wir noch die Eingangsbedingung der Fallunterscheidung mit der errechneten Bedingung abgleichen: Die Werte x_{3,4} sind dann Lösungen, wenn der Parameter k \geq -1, aber k\neq 1 ist.

2. Fall: Wir nehmen an, dass 1-k=0 ist. Also ist der Parameter k=1. Dann erhalten wir in der Gleichung:
\begin{array}{rclcll}0\cdot\left[x^2-(1+k)\pi^2\right] &=& 0 \\0 &=& 0\end{array}

In diesem Fall, also wenn k=1 ist, löst jede reelle Zahl diese Gleichung, d. h. die Lösungsmenge entspricht dem Definitionsbereich. 0=0 ist schließlich immer richtig.

Zusammenfassung der Lösung, abhängig vom Wert des Parameters k:
Für k\geq -1 und k\neq 1 ist \mathbb{L}=\left\{-\sqrt{\pi+k^2}\;;\;\sqrt{\pi+k^2}\;;\;-\sqrt{\pi^2\left(1+k\right)}\;;\;\sqrt{\pi^2\left(1+k\right)}\right\}.
Für k =1 ist \mathbb{L}=\mathbb{R}.
Für k < -1 ist \mathbb{L}=\left\{-\sqrt{\pi+k^2}\;;\;\sqrt{\pi+k^2}\right\}.

Bemerkung: Die Lösungen, die sich bei "Faktor 1" ergeben haben, nicht vergessen!


20)
\begin{array}{rclcl} \mathbb{D} &=& \mathbb{R} \cr \cr \cr 3x^{2n} &=& 2x^{2n-1}+x^{2n-1} \cr\cr 3x^{2n} &=& 3x^{2n-1} &\vert& : x^{2n-1} \cr\cr \dfrac{3x^{2n}}{x^{2n-1}} &=& \dfrac{3x^{2n-1}}{x^{2n-1}} \cr\cr 3x^{2n-(2n-1)} &=& 3x^{2n-1-(2n-1)} \cr\cr3x^1 &=& 3x^0 \cr 3x &=& 3 &\vert& :3 \cr x &=& 1 \end{array}

Das ist aber nicht die vollständige Lösung der Gleichung. Warum nicht? Wenn man durch x^{2n-1} dividiert, muss man natürlich sicher sein, dass der Term ungleich 0 ist, da man durch 0 ja nun mal nicht teilen darf. Darum müssen wir uns jetzt noch kümmern:
Eine Potenz wie x^{2n-1} kann grundsätzlich nur dann 0 sein, wenn die Basis 0 ist. Für x=0 ist auch x^{2n-1} = 0, egal welchen Wert n annimmt. Der einzige "Sonderfall" wäre, wenn sowohl Basis als auch Exponent 0 wären. Der Exponent 2n-1 ist 0, wenn n=0{,}5\not\in\mathbb{N}^+ ist. Das kann also nicht passieren. Trotzdem haben wir den Fall x=0 mit der Rechnung oben noch nicht abgedeckt und müssen jetzt noch schauen, was dabei passiert:
\begin{array}{rclcl} 3 \cdot 0^{2n} &=& 2 \cdot 0^{2n-1}+0^{2n-1} \cr3 \cdot 0 &=& 2 \cdot 0+0 \cr 0 &=& 0 \end{array}
Das ist eine wahre Aussage. 0 ist also auch Lösung der Gleichung.

Insgesamt ergibt sich folgende Lösungsmenge: \mathbb{L} = \{0;1\}

 

3. Aufgabe

Zu den Eigenschaften, die in diesen Zusammenhang interessieren, zählen: Achsen- bzw. Punktsymmetrie, das Randverhalten, die mögliche Anzahl von Nullstellen, Extrempunkten und Wende- bzw. Sattelpunkten etc.
Um einen Funktionsgraphen zu zeichen, kann zusätzlich eine Wertetabelle helfen. Diese sind in der Musterlösung nicht angegeben.

Bemerkung: Achten Sie jeweils auf die Skaleneinteilung der Koordinatensysteme! Diese sollte immer so gewählt werden, dass der charakteristische Teil des Graphen dargestellt wird. Dort, wo es sinnvoll ist, sind die Funktionen in den folgenden Aufgaben doppelt dargestellt - einmal in einem Koordinatensystem, in dem die Achsen jeweils den Zahlenbereich von -10 bis 10 umfassen (zur besseren Vergleichbarkeit), und einmal mit einer auf die Funktion angepassten Achseneinteilung.


1) f(x)=-x^3+5x^2-x+5
Symmetrie: Als Polynom, das sowohl gerade als auch ungerade Exponenten enthält, ist f(x) weder achsensymmetrisch zur y-Achse noch punktsymmetrisch zum Koordinatenursprung.

Randverhalten: Als Polynom ungeraden Grades, bei dem die höchste Potenz einen negativen Koeffizienten hat, sind die Funktionswerte bei sehr kleinen x-Werten sehr groß und bei sehr großen x-Werten sehr klein.

Nullstellen: Als Polynom ungeraden Grades hat f(x) mindestens eine Nullstelle. Maximal kann es drei Nullstellen geben.

Extrempunkte: f(x) kann maximal zwei Extrempunkte haben.

Wende- und Sattelpunkte: Als Polynom ungeraden Grades mit dem Grad 3 hat f(x) genau einen Wende- bzw. Sattelpunkt.

Sonstiges: Die Funktion schneidet die y-Achse im Punkt (0 \mid 5).

Graph vom Beispielpolynom

 

2) f(x)=10x^4-3x^3+8x^2
Symmetrie: Als Polynom, das sowohl gerade als auch ungerade Exponenten enthält, ist f(x) weder achsensymmetrisch zur y-Achse noch punktsymmetrisch zum Koordinatenursprung. Wenn man nur die Zeichnung betrachtet, könnte man denken, dass die Funktion achsensymmetrisch zur y-Achse ist. Man muss wirklich genau schauen (vor allem, wenn die Skaleneinteilung an den Achsen nicht optimal gewählt wurde), um zu erkennen, dass dem nicht so ist. Deswegen ist eine rechnerische oder argumentative Lösung in vielen Fällen besser als eine zeichnerische.

Randverhalten: Als Polynom geraden Grades, bei dem die höchste Potenz einen positiven Koeffizienten hat, sind die Funktionswerte sowohl bei sehr kleinen als auch bei sehr großen x-Werten sehr groß.

Nullstellen: Als Polynom ohne konstanten Term hat f(x) mindestens eine Nullstelle bei (0 \mid 0). Maximal kann es vier Nullstellen geben.

Extrempunkte: Als Polynom geraden Grades hat f(x) mindestens einen Extrempunkt. Maximal kann es drei Extrempunkte geben.

Wende- und Sattelpunkte: f(x) kann maximal zwei Wende- bzw. Sattelpunkte haben.

Sonstiges: Die Funktion schneidet die y-Achse im Punkt (0 \mid 0).

Graph vom Beispielpolynom

 

3) f(x)=15x^4+21x^2-9
Symmetrie: Als Polynom, das nur gerade Exponenten enthält, ist f(x) achsensymmetrisch zur y-Achse.

Randverhalten: Als Polynom geraden Grades, bei dem die höchste Potenz einen positiven Koeffizienten hat, sind die Funktionswerte sowohl bei sehr kleinen als auch bei sehr großen x-Werten sehr groß.

Nullstellen: f(x) kann maximal vier Nullstellen haben.

Extrempunkte: Als Polynom geraden Grades hat f(x) mindestens einen Extrempunkt. Maximal kann es drei Extrempunkte geben.

Wende- und Sattelpunkte: f(x) kann maximal zwei Wende- bzw. Sattelpunkte haben.

Sonstiges: Die Funktion schneidet die y-Achse im Punkt (0\mid -9). Deshalb und weil die Parabel nach oben geöffnet ist (der Koeffizient vor x^4 ist positiv), muss es mindestens zwei Nullstellen geben. Vom Punkt (0\mid -9) ausgehend schneidet der Graph die x-Achse nämlich einmal im positiven und einmal im negativen Bereich. Weiter kann man argumentieren, dass es nicht drei Nullstellen geben kann, weil die Funktion dann nicht achsensymmetrisch zur y-Achse wäre. Drei Nullstellen und Achsensymmetrie zur y-Achse gemeinsam sind nur dann möglich, wenn eine Nullstelle im Punkt (0 \mid 0) liegt. Der Punkt (0\mid 0) kann aber nicht Teil des Graphen sein, wenn der Punkt (0\mid -9) auf dem Graphen liegt.

Bemerkung: Wenn Sie sich die Argumentation zur Anzahl der Nullstellen veranschaulichen wollen, können viele kleine Skizzen helfen. Z. B. können Sie in einem Koordinatensystem den Punkt (0\mid -9) markieren und dann versuchen Graphen zu zeichnen, die zwei, drei oder vier Nullstellen haben und den übrigen Kriterien entsprechen, also achsensymmetrisch zur y-Achse bzw. nach oben geöffnet sind.

Graph vom Beispielpolynom

 

4) f(x)=2x^5+x^3-4x
Symmetrie: Als Polynom, das nur ungerade Exponenten enthält, ist f(x) punktsymmetrisch zum Koordinatenursprung.

Randverhalten: Als Polynom ungeraden Grades, bei dem die höchste Potenz einen positiven Koeffizienten hat, sind die Funktionswerte bei sehr kleinen x-Werten auch sehr klein und bei sehr großen x-Werten sehr groß.

Nullstellen: Als Polynom ohne konstanten Term hat f(x) mindestens eine Nullstelle bei (0 \mid 0). Maximal kann es fünf Nullstellen geben.

Extrempunkte: f(x) kann maximal vier Extrempunkte haben.

Wende- und Sattelpunkte: Als Polynom ungeraden Grades hat f(x) mindestens einen Wende- bzw. Sattelpunkt. Maximal kann es drei Wende- bzw. Sattelpunkte geben.

Sonstiges: Die Funktion schneidet die y-Achse im Punkt (0 \mid 0).

Graph vom Beispielpolynom

 

5) f(x)=-x^6-x^4+4x^2+7
Symmetrie: Als Polynom, das nur gerade Exponenten enthält, ist f(x) achsensymmetrisch zur y-Achse.

Randverhalten: Als Polynom geraden Grades, bei dem die höchste Potenz einen negativen Koeffizienten hat, sind die Funktionswerte sowohl bei sehr kleinen als auch bei sehr großen x-Werten sehr klein.

Nullstellen: f(x) kann maximal sechs Nullstellen haben.

Extrempunkte: Als Polynom geraden Grades hat f(x) mindestens einen Extrempunkt. Maximal kann es fünf Extrempunkte geben.

Wende- und Sattelpunkte: f(x) kann maximal vier Wende- bzw. Sattelpunkte haben.

Sonstiges: Die Funktion schneidet die y-Achse im Punkt (0 \mid 7). Deshalb und weil die Parabel nach unten geöffnet ist (der Koeffizient vor x^6 ist negativ), muss es folglich mindestens zwei Nullstellen geben. Weiter kann man argumentieren, dass es nicht drei oder fünf Nullstellen geben kann, weil die Funktion dann nicht achsensymmetrisch zur y-Achse wäre. Eine ungerade Anzahl Nullstellen und Achsensymmetrie zur y-Achse gemeinsam sind nur dann möglich, wenn eine Nullstelle im Punkt (0 \mid 0) liegt. Der Punkt (0 \mid 0) kann aber nicht Teil des Graphen sein, wenn der Punkt (0 \mid 7) auf dem Graph liegt.

Graph vom Beispielpolynom

 

4. Aufgabe

1)
a)
\begin{array}{rclll} f(1{,}5) &=& 3 \cdot 1{,}5^4-4 \cdot 1{,}5^3 &\approx& 1{,}69 \quad \rightarrow \quad P_1(1{,}5 \mid 1{,}69) \end{array}

b)
\begin{array}{crclll} & 0 &=& 3x^4-4x^3 \cr &0 &=& x^3(3x-4) & \vert & \text{Satz vom Nullprodukt} \cr \text{Faktor 1:} & x^3 &=& 0 &\vert& \sqrt[3]{} \cr & x_2 &=& 0 & & \rightarrow \quad P_2(0 \mid 0) \cr\cr \text{Faktor 2:} & 3x-4 &=& 0 &\vert& +4\cr & 3x &=& 4 &\vert& :3 \cr & x_3 &=& \dfrac{4}{3} & & \rightarrow \quad P_3\left(\dfrac{4}{3} \mid 0\right) \end{array}


2)
a)
\begin{array}{rclll} f\left(-\dfrac{1}{3}\right) &=& 2 \cdot \left(-\dfrac{1}{3}\right)^4-\left(-\dfrac{1}{3}\right)^3+3 \cdot \left(-\dfrac{1}{3}\right)^2+13 &=& \dfrac{1.085}{81} \approx 13{,}40 \quad \rightarrow \quad P_1\left(-\dfrac{1}{3} \mid \dfrac{1.085}{81} \right) \end{array}

b)
\begin{array}{crclll} & 13 &=& 2x^4-x^3+3x^2+13 &\vert & - 13 \cr & 0 &=& 2x^4-x^3+3x^2 \cr & 0 &=& x^2\left(2x^2-x+3\right) & \vert & \text{Satz vom Nullprodukt} \cr \text{Faktor 1:} & x^2 &=& 0 &\vert& \pm\sqrt{} \cr & x_2 &=& 0 & & \rightarrow \quad P_2(0 \mid 13) \cr\cr\text{Faktor 2:} & 2x^2-x+3 &=& 0 & \vert & :2 \cr & x^2-\dfrac{1}{2}x+\dfrac{3}{2} &=& 0 &\vert& \text{p-q-Formel} \cr & x_{3,4} &=& \dfrac{1}{4} \pm \sqrt{\left(-\dfrac{1}{4}\right)^2-\dfrac{3}{2}} \cr & x_{3,4} &=& \dfrac{1}{4} \pm \sqrt{-\dfrac{23}{16}} \end{array}

Da aus negativen reellen Zahlen keine Wurzeln mit geraden Wurzelexponenten gezogen werden können, liefert der zweite Faktor keine weiteren Punkte.

3)
a)
\begin{array}{rclll} f(-11) &=& 4 \cdot (-11)^3+7 \cdot (-11)^2-2 \cdot (-11)-5 &=& -4.460 \quad \rightarrow \quad P_1(-11 \mid -4.460) \end{array}

b)
\begin{array}{rclclll} 0 &=& 4x^3+7x^2-2x-5 \end{array}

Durch Probieren finden wir die Lösung x_2=-1, denn
\begin{array}{rcl}0 &=& 4\cdot (-1)^3+7\cdot (-1)^2-2\cdot (-1)-5 \\ 0 &=& 0 \\& \rightarrow & P_2(-1 \mid 0)\end{array}

Polynomdivision mit x_2=-1:
Durchführung Polynomdivision

Lösung des reduziertes Polynom mit der p-q-Formel:
\begin{array}{rclcl} 0 &=& 4x^2+3x-5 & \vert & :4 \cr 0 &=& x^2+\dfrac{3}{4}x-\dfrac{5}{4} \cr\cr x_{3,4} &=& -\dfrac{3}{8} \pm\sqrt{\left(\dfrac{3}{8}\right)^2+\dfrac{5}{4}} \cr\cr &=& -\dfrac{3}{8} \pm\sqrt{\dfrac{89}{64}} \cr\cr x_3 &=& -\dfrac{3+\sqrt{89}}{8} \approx 0{,}80 & & \rightarrow \quad P_3\left(-\dfrac{3+\sqrt{89}}{8} \mid 0\right) \cr\cr x_4 &=& -\dfrac{3-\sqrt{89}}{8} \approx -1{,}55 & & \rightarrow \quad P_4\left(-\dfrac{3-\sqrt{89}}{8} \mid 0\right) \end{array}

4)
a)
\begin{array}{rclll} f(0{,}1) &=& 7 \cdot (0{,}1)^6-4 \cdot (0{,}1)^3+5 &\approx& 5{,}00 \quad \rightarrow \quad P_1(0{,}1 \mid 5{,}00) \end{array}

b)
\begin{array}{rclcl} 485 &=& 7x^6-4x^3+5 & \vert & -485 \cr 0 &=& 7x^6-4x^3-480 & \vert & :7 \cr 0 &=& x^6-\dfrac{4}{7}x^3-\dfrac{480}{7} \cr 0 &=& \left(x^3\right)^2-\dfrac{4}{7}x^3-\dfrac{480}{7} \end{array}

Substitution: z=x^3
\begin{array}{rclcl} 0 &=& z^2-\dfrac{4}{7}z-\dfrac{480}{7} &\vert& \text{p-q-Formel} \cr\cr z_{1,2} &=& \dfrac{2}{7} \pm \sqrt{\left(-\dfrac{2}{7}\right)^2+\dfrac{480}{7}} \cr\cr z_{1,2} &=& \dfrac{2}{7} \pm \sqrt{\dfrac{3.364}{49}} \cr\cr z_1 &=& \dfrac{2}{7} + \dfrac{58}{7} = \dfrac{60}{7} \cr\cr z_2 &=& \dfrac{2}{7} - \dfrac{58}{7} = -8 \end{array}

Rücksubstitution:
\begin{array}{rclcl} z_1 = x^3 &=& \dfrac{60}{7} & \vert & \sqrt[3]{} \cr x_2 &=& \sqrt[3]{\dfrac{60}{7}} \approx 2{,}05 & & \rightarrow \quad P_2(2{,}05 \mid 485) \cr\cr z_2 = x^3 &=& -8 & \vert & \sqrt[3]{} \cr x_3 &=& \sqrt[3]{-8} \cr x_3 &=& -\sqrt[3]{\vert-8\vert} = -2 & & \rightarrow \quad P_3(-2 \mid 485) \end{array}

Infos zu -\sqrt[3]{\vert-8\vert} stehen unter "allgemeinere Erkenntnisse zu Wurzeln"


5)
a)
\begin{array}{rclll} f(0) &=& 0^{16}-0^8-256 &=& -256 \quad \rightarrow \quad P_1(0 \mid -256) \end{array}

b)
\begin{array}{crclll} & -256 &=& x^{16}-x^8-256 & \vert & +256 \cr & 0 &=& x^{16}-x^8 \cr & 0 &=& x^8\left(x^8-1\right) & \vert & \text{Satz vom Nullprodukt} \cr \text{Faktor 1:} & x^8 &=& 0 &\vert& \pm\sqrt[8]{}\cr & x_2 &=& 0 & & \rightarrow \quad P_2(0 \mid -256) \cr\cr \text{Faktor 2:} & x^8-1 &=& 0 & \vert & +1 \cr & x^8 &=& 1 & \vert & \pm\sqrt[8]{} \cr & x_3 &=& \sqrt[8]{1} = 1 & & \rightarrow \quad P_3(1 \mid -256) \cr & x_4 &=& -\sqrt[8]{1} =-1 & & \rightarrow \quad P_4(-1 \mid -256) \end{array}


6)
a)
\begin{array}{rclll} f\left(-\dfrac{3}{2}\right) &=& 3 \cdot \left(\dfrac{4}{3}\cdot\left(-\dfrac{3}{2}\right)^3-2\cdot\left(-\dfrac{3}{2}\right)^2+1\right) &=& -24 \quad \rightarrow \quad P_1\left(-\dfrac{3}{2} \mid -24\right) \end{array}

b)
\begin{array}{crclll} & 3 &=& 4x^3-6x^2+3 & \vert & -3 \cr & 0 &=& 4x^3-6x^2 \cr & 0 &=& x^2(4x-6) & \vert & \text{Satz vom Nullprodukt} \cr \text{Faktor 1:} & x^2 &=& 0 &\vert& \pm\sqrt{} \cr & x_2 &=& 0 & & \rightarrow \quad P_2(0 \mid 3) \cr\cr \text{Faktor 2:} & 4x-6 &=& 0 &\vert& +6 \cr & 4x &=& 6 &\vert& :4 \cr & x_3 &=& \dfrac{3}{2} & & \rightarrow \quad P_3\left(\dfrac{3}{2} \mid 3\right) \end{array}


7)
a)
\begin{array}{rclll} f(-32) &=& \dfrac{1}{2} \left(\left(-32\right)^3-12\cdot \left(-32\right)^2+5\cdot \left(-32\right) \right)+80 &=& -22.528 \quad \rightarrow \quad P_1(-32 \mid -22.528) \end{array}

b)
\begin{array}{rclclll} 5 &=& \dfrac{1}{2}\left(x^3-12x^2+5x\right)+80 & \vert & \cdot 2 \cr 10 &=& x^3-12x^2+5x+160 & \vert & -10 \cr 0 &=& x^3-12x^2+5x+150 \end{array}

Durch Probieren finden wir die Lösung x_2=5, denn
\begin{array}{rcl}0 &=& 5^3-12\cdot 5^2+5\cdot 5+150 \\ 0 &=& 0 \\& \rightarrow & P_2(5 \mid 5) \end{array}

Polynomdivision mit x_2=5:
Durchführung Polynomdivision

Lösung des reduziertes Polynom mit der p-q-Formel:
\begin{array}{rclcl} 0 &=& x^2-7x-30 \cr x_{3,4} &=& \dfrac{7}{2} \pm\sqrt{\left(-\dfrac{7}{2}\right)^2+30} \cr &=& \dfrac{7}{2} \pm\sqrt{\dfrac{169}{4}} \cr\cr x_3 &=& \dfrac{7}{2}+\dfrac{13}{2} = 10 & & \rightarrow \quad P_3(10 \mid 5) \cr x_4 &=& \dfrac{7}{2}-\dfrac{13}{2} = -3 & & \rightarrow \quad P_4(-3 \mid 5) \end{array}


8)
a)
\begin{array}{rclll} f(-1) &=& 5 \cdot (-1)^{13}-7 \cdot (-1)^9-7 &=& -5 \quad \rightarrow \quad P_1(-1 \mid -5) \end{array}

b)
\begin{array}{crclll} & -7 &=& 5x^{13}-7x^9-7 & \vert & +7 \cr & 0 &=& 5x^{13}-7x^9 \cr & 0 &=& x^9\left(5x^4-7\right) & \vert & \text{Satz vom Nullprodukt} \cr \text{Faktor 1:} & x^9 &=& 0 &\vert& \sqrt[9]{} \cr & x_2 &=& 0 & & \rightarrow \quad P_2(0 \mid -7) \cr\cr \text{Faktor 2:} & 5x^4-7 &=& 0 & \vert & +7 \cr & 5x^4 &=& 7 & \vert & :5 \cr & x^4 &=& \dfrac{7}{5} & \vert & \pm\sqrt[4]{} \cr & x_3 &=& \sqrt[4]{\dfrac{7}{5}} & & \rightarrow \quad P_3\left(\sqrt[4]{\dfrac{7}{5}} \mid -7\right) \cr & x_4 &=& -\sqrt[4]{\dfrac{7}{5}} & & \rightarrow \quad P_4\left(-\sqrt[4]{\dfrac{7}{5}} \mid -7\right) \end{array}


9)
a)
\begin{array}{rclll} f(12) &=& 3 \cdot 12^3-10 \cdot 12^2+7 \cdot 12-26 &=& 3.802 \quad \rightarrow \quad P_1(12 \mid 3.802) \end{array}

b)
\begin{array}{rclclll} -14 &=& 3x^3-10x^2+7x-26 & \vert & +14 \cr 0 &=& 3x^3-10x^2+7x-12 \end{array}

Durch Probieren finden wir die Lösung x_2=3, denn
\begin{array}{rcl}0 &=& 3\cdot 3^3-10\cdot 3^2+7\cdot 3-12 \\ 0 &=& 0 \\& \rightarrow & P_2(3 \mid -14)\end{array}

Polynomdivision mit x_2=3:
Durchführung Polynomdivision

Lösung des reduziertes Polynom mit der p-q-Formel:
\begin{array}{rclcl} 0 &=& 3x^2-x+4 & \vert & :3 \cr 0 &=& x^2-\dfrac{1}{3}x+\dfrac{4}{3} \cr\cr x_{3,4} &=& \dfrac{1}{6} \pm\sqrt{\left(-\dfrac{1}{6}\right)^2-\dfrac{4}{3}} \cr &=& \dfrac{1}{6} \pm\sqrt{-\dfrac{47}{36}} \end{array}

Da aus negativen reellen Zahlen keine Wurzeln mit geraden Wurzelexponenten gezogen werden können, hat diese Gleichung keine weiteren Lösungen. Die Funktion nimmt den Funktionswert -14 also nur bei x_2=3 an.

10)
a)
\begin{array}{rclll} f(5) &=& 5^4-2 \cdot 5^2+1 &=& 576 \quad \rightarrow \quad P_1(5 \mid 576) \end{array}

b)
\begin{array}{rclcl} 9 &=& x^4-2x^2+1 & \vert & -9 \cr 0 &=& x^4-2x^2-8 \cr 0 &=& \left(x^2\right)^2-2x^2-8 \end{array}

Substitution: z=x^2
\begin{array}{rclcl} 0 &=& z^2-2z-8 &\vert& \text{p-q-Formel} \cr z_{1,2} &=& 1 \pm \sqrt{1+8} \cr &=& 1 \pm \sqrt{9} \cr &=& 1 \pm 3 \cr\cr z_1 &=& 1+3 = 4 \cr z_2 &=& 1-3 = -2 \end{array}

Rücksubstitution:
\begin{array}{rclcl} z_1 = x^2 &=& 4 & \vert & \pm\sqrt{} \cr x_{2,3} &=& \pm \sqrt{4} \cr x_2 &=& 2 & & \rightarrow \quad P_2(2 \mid 9) \cr x_3 &=& -2 & & \rightarrow \quad P_3(-2 \mid 9) \cr\cr z_2 = x^2 &=& -2 & \vert & \pm\sqrt{} \cr x_{4,5} &=& \pm\sqrt{-2} \end{array}

Da aus negativen reellen Zahlen keine Wurzeln mit geraden Wurzelexponenten gezogen werden können, liefert die Rücksubstitution von z_2 keine weiteren Punkte.

 

5. Aufgabe

1)
\begin{array}{crclcl}& \mathbb{D} &=& \mathbb{R} \\\\& 8a^5+6a^3-20a &=& 0 \\& a\cdot\left(8a^4+6a^2-20\right) &=& 0 &\vert & \text{Satz vom Nullprodukt} \\\text{Faktor 1:} & a_1 &=& 0 \\\\\text{Faktor 2:} & 8a^4+6a^2-20 &=& 0 &\vert & :8 \\& a^4+\dfrac{3}{4}a^2-\dfrac{5}{2} &=& 0 \\& \left(a^2\right)^2+\dfrac{3}{4}a^2-\dfrac{5}{2} &=& 0\end{array}

Substitution: z=a^2
\begin{array}{rclcl}z^2+\dfrac{3}{4}z-\dfrac{5}{2} &=& 0 &\vert & \text{p-q-Formel} \\z_{1,2} &=& -\dfrac{3}{8}\pm\sqrt{\left(\dfrac{3}{8}\right)^2+\dfrac{5}{2}} \\\\z_{1,2} &=& -\dfrac{3}{8}\pm\sqrt{\dfrac{169}{64}} \\\\z_1 &=& -\dfrac{3}{8}+\dfrac{13}{8} = \dfrac{5}{4} = 1{,}25 \\\\z_2 &=& -\dfrac{3}{8}-\dfrac{13}{8} = -2 \end{array}

Rücksubstitution:
\begin{array}{rclcl}z_1 = a^2 &=& \dfrac{5}{4} &\vert& \pm\sqrt{} \\a_{2,3} &=& \pm\sqrt{\dfrac{5}{4}} \approx \pm 1{,}12 \\\\z_2 = a^2 &=& -2 &\vert& \pm\sqrt{} \\a_{4,5} &=& \pm\sqrt{-2}\end{array}

Da aus negativen reellen Zahlen keine Wurzeln mit geraden Wurzelexponenten gezogen werden können, liefert die Rücksubstitution von z_2 keine weiteren Lösungen: \mathbb{L} = \left\{-\dfrac{\sqrt{5}}{2}\;;\;0\;;\;\dfrac{\sqrt{5}}{2}\right\}


2)
\begin{array}{crclcl}& \mathbb{D} &=& \mathbb{R} \\\\& 3x^3+6x^2-3x^2-6x &=& 0 \\& 3x^3+3x^2-6x &=& 0 \\& x\cdot\left(3x^2+3x-6\right) &=& 0 &\vert & \text{Satz vom Nullprodukt}\\\text{Faktor 1:} & x_1 &=& 0 \\\\\text{Faktor 2:} & 3x^2+3x-6 &=& 0 &\vert & :3 \\& x^2+x-2 &=& 0 &\vert & \text{p-q-Formel} \\& x_{2,3} &=& -\dfrac{1}{2}\pm\sqrt{\left(\dfrac{1}{2}\right)^2+2} \\\\& x_{2,3} &=& -\dfrac{1}{2}\pm\sqrt{\dfrac{9}{4}} \\\\& x_2 &=& -\dfrac{1}{2}+\dfrac{3}{2} = 1 \\\\& x_3 &=& -\dfrac{1}{2}-\dfrac{3}{2} = -2 \\\\& \mathbb{L} &=& \left\{-2;0;1\right\}\end{array}


3)
\begin{array}{rcl}\mathbb{D} &=& \mathbb{R} \\\\x^3+4x^2-51x-54 &=& 0\end{array}

Durch Probieren finden wir die Lösung x_1 = -1, denn
\begin{array}{rcl}(-1)^3+4\cdot(-1)^2-51\cdot(-1)-54 &=& 0 \\0 &=& 0\end{array}

Polynomdivision mit x_1 = -1:

Polynomdivision

Lösung des reduzierten Polynoms:
\begin{array}{rclcl}x^2+3x-54 &=& 0 &\vert & \text{p-q-Formel}\\x_{2,3} &=& -\dfrac{3}{2}\pm\sqrt{\left(\dfrac{3}{2}\right)^2+54}\\\\x_{2,3} &=& -\dfrac{3}{2}\pm\sqrt{\dfrac{225}{4}}\\\\x_2 &=& -\dfrac{3}{2}+\dfrac{15}{2} = 6 \\\\x_3 &=& -\dfrac{3}{2}-\dfrac{15}{2} = -9 \\\\\mathbb{L} &=& \left\{-9;-1;6\right\}\end{array}


4)
\begin{array}{rcl}\mathbb{D} &=& \mathbb{R} \\\\4x^3+8x^2-x-2 &=& 0\end{array}

Durch Probieren finden wir die Lösung x_1 = -2, denn
\begin{array}{rcl}4\cdot(-2)^3+8\cdot(-2)^2-(-2)-2 &=& 0 \\0 &=& 0\end{array}

Polynomdivision mit x_1 = -2:

Polynomdivision

Lösung des reduzierten Polynoms:
\begin{array}{rclcl}4x^2-1 &=& 0 &\vert & +1 \\4x^2 &=& 1 &\vert & :4 \\x^2 &=& \dfrac{1}{4} &\vert & \pm\sqrt{} \\x &=& \pm\dfrac{1}{2} \\\\\mathbb{L} &=& \left\{-2;-\dfrac{1}{2};\dfrac{1}{2}\right\}\end{array}


5)
\begin{array}{rcl}\mathbb{D} &=& \mathbb{R} \\\\2x^3+12x^2-48x-128 &=& 0 \end{array}

Durch Probieren finden wir die Lösung x_1 = 4, denn
\begin{array}{rcl}2\cdot 4^3+12\cdot 4^2-48\cdot 4-128 &=& 0 \\0 &=& 0\end{array}

Polynomdivision mit x_1 = 4:

Polynomdivision

Lösung des reduzierten Polynoms:
\begin{array}{rclcl}2x^2+20x+32 &=& 0 &\vert & :2 \\x^2+10x+16 &=& 0 &\vert & \text{p-q-Formel} \\x_{2,3} &=& -5\pm\sqrt{5^2-16} \\x_{2,3} &=& -5\pm\sqrt{9} \\\\x_2 &=& -5+3 = -2 \\x_3 &=& -5-3 = -8 \\\\\mathbb{L} &=& \left\{-8;-2;4\right\}\end{array}


6)
\begin{array}{rclcll}\mathbb{D} &=& \mathbb{R} \\\\\dfrac{3}{4}x^4+2x^2-4 &=& 0 \\\dfrac{3}{4}\left(x^2\right)^2+2x^2-4 &=& 0 \end{array}

Substitution: z=x^2
\begin{array}{rclcl}\dfrac{3}{4}z^2+2z-4 &=& 0 &\vert & :\dfrac{3}{4} \\z^2+\dfrac{8}{3}z-\dfrac{16}{3} &=& 0 &\vert & \text{p-q-Formel} \\z_{1,2} &=& -\dfrac{4}{3}\pm\sqrt{\left(\dfrac{4}{3}\right)^2+\dfrac{16}{3}} \\\\z_{1,2} &=& -\dfrac{4}{3}\pm\sqrt{\dfrac{64}{9}} \\\\z_1 &=& -\dfrac{4}{3}+\dfrac{8}{3} = \dfrac{4}{3} \\z_2 &=& -\dfrac{4}{3}-\dfrac{8}{3} = -4 \end{array}

Rücksubstitution:
\begin{array}{rclcl}z_1 = x^2 &=& \dfrac{4}{3} &\vert& \pm\sqrt{} \\x_{1,2} &=& \pm\sqrt{\dfrac{4}{3}} \\ x_{1,2} &=& \pm\dfrac{2\cdot\sqrt{3}}{3} \approx \pm 1{,}15 \\\\z_2 = x^2 &=& -4 &\vert& \pm\sqrt{} \\x_{3,4} &=& \pm\sqrt{-4}\end{array}

Da aus negativen reellen Zahlen keine Wurzeln mit geraden Wurzelexponenten gezogen werden können, liefert die Rücksubstitution von z_2 keine weiteren Lösungen: \mathbb{L} = \left\{-\dfrac{2\cdot\sqrt{3}}{3}\;;\;\dfrac{2\cdot\sqrt{3}}{3}\right\}


7)
\begin{array}{lrclcll}& \mathbb{D} &=& \mathbb{R} \\\\& \left(y-3\right)\cdot\left(y^2-\dfrac{2}{3}y-3\right) &=& 0 &\vert & \text{Satz vom Nullprodukt}\\\text{Faktor 1:} & y-3 &=& 0 &\vert & +3 \\& y_1 &=& 3\\\\\text{Faktor 2:} & y^2-\dfrac{2}{3}y-3 &=& 0 &\vert & \text{p-q-Formel} \\& y_{2,3} &=& \dfrac{1}{3}\pm\sqrt{\left(-\dfrac{1}{3}\right)^2+3}\\\\& y_{2,3} &=& \dfrac{1}{3}\pm\sqrt{\dfrac{28}{9}}\\\\& y_2 &=& \dfrac{1+\sqrt{28}}{3} \approx 2{,}10 \\\\& y_3 &=& \dfrac{1-\sqrt{28}}{3} \approx -1{,}43 \\\\& \mathbb{L} &=& \left\{\dfrac{1-2\sqrt{7}}{3}\;;\;\dfrac{1+2\sqrt{7}}{3}\;;\; 3\right\}\end{array}


8)
\begin{array}{rcl}\mathbb{D} &=& \mathbb{R} \\\\x^4-17x^2+16 &=& 0 \\ \left(x^2\right)^2-17x^2+16 &=& 0 \end{array}

Substitution: u = x^2
\begin{array}{rclcl}u^2-17u+16 &=& 0 &\vert & \text{p-q-Formel} \\u_{1,2} &=& \dfrac{17}{2}\pm\sqrt{\left(-\dfrac{17}{2}\right)^2-16} \\\\u_{1,2} &=& \dfrac{17}{2}\pm\sqrt{\dfrac{225}{4}}\\\\u_1 &=& \dfrac{17}{2}+\dfrac{15}{2} = 16 \\u_2 &=& \dfrac{17}{2}-\dfrac{15}{2} = 1\end{array}

Rücksubstitution:
\begin{array}{rclcl}u_1 = x^2 &=& 16 &\vert& \pm\sqrt{} \\x_{1,2} &=& \pm 4 \\\\u_2 = x^2 &=& 1 &\vert& \pm\sqrt{} \\x_{3,4} &=& \pm 1 \\\\\mathbb{L} &=& \left\{-4;-1; 1; 4\right\}\end{array}


9)
\begin{array}{lrclcll}& \mathbb{D} &=& \mathbb{R} \\\\& \dfrac{1}{4}x^3+x^2+x &=& 0 \\& x\cdot\left(\dfrac{1}{4}x^2+x+1\right) &=& 0 &\vert & \text{Satz vom Nullprodukt} \\\text{Faktor 1:} & x_1 &=& 0 \\\\\text{Faktor 2:} & \dfrac{1}{4}x^2+x+1 &=& 0 &\vert & :\dfrac{1}{4} \\& x^2+4x+4 &=& 0 &\vert & \text{p-q-Formel} \\& x_{2,3} &=& -2\pm\sqrt{2^2-4} \\& x_{2,3} &=& -2\pm\sqrt{0} \\& x_{2,3} &=& -2 \\\\& \mathbb{L} &=& \left\{0;-2\right\}\end{array}


10)
\begin{array}{rclcl}\mathbb{D} &=& \mathbb{R} \\\\x^3+5x^2-2x-24 &=& 0 \end{array}

Durch Probieren finden wir die Lösung x_1 = 2, denn
\begin{array}{rcl}2^3+5\cdot 2^2-2\cdot 2-24 &=& 0 \\0 &=& 0\end{array}

Polynomdivision mit x_1 = 2:

Polynomdivision

Lösung des reduzierten Polynoms:
\begin{array}{rclcl}x^2+7x+12 &=& 0 &\vert &\text{p-q-Formel}\\x_{2,3} &=& -\dfrac{7}{2}\pm\sqrt{\left(\dfrac{7}{2}\right)^2-12} \\x_{2,3} &=& -\dfrac{7}{2}\pm\sqrt{\dfrac{1}{4}} \\\\x_2 &=& -\dfrac{7}{2}+\dfrac{1}{2} = -3 \\x_3 &=& -\dfrac{7}{2}-\dfrac{1}{2} = -4 \\\\\hspace{12em}\mathbb{L} &=& \left\{-4;-3; 2\right\}\end{array}


11)
\begin{array}{crclcl} & \mathbb{D} &=& \mathbb{R} \cr\cr & (6x^4+17)(9x^4+3x^2-2) &=& 0 &\vert& \text{Satz vom Nullprodukt} \cr \cr\text{Faktor 1:} & 6x^4+17 &=& 0 &\vert& -17 \cr & 6x^4 &=& -17 &\vert& :6 \cr\cr & x^4 &=& -\dfrac{17}{6} &\vert& \pm\sqrt[4]{} \cr\cr & x_{1,2} &=& \pm\sqrt[4]{-\dfrac{17}{6}} \cr\cr\text{Faktor 2:} & 9x^4+3x^2-2 &=& 0 \cr & 9(x^2)^2 +3x^2-2 &=& 0 \end{array}

Substitution: x^2=z
\begin{array}{rclcl} 0 &=& 9z^2+3z-2 &\vert& :9 \cr\cr 0 &=& z^2+\dfrac13 z-\dfrac29 &\vert& \text{p-q-Formel} \cr\cr z_{1,2}&=& -\dfrac16 \pm \sqrt{\left(\dfrac16\right)^2 + \dfrac29}\cr\cr z_{1,2} &=& -\dfrac16 \pm\sqrt{\dfrac{9}{36}} \cr\cr z_1 &=& -\dfrac16+\dfrac36 = \dfrac13 \cr z_2 &=& -\dfrac16-\dfrac36 = -\dfrac23\end{array}

Rücksubstitution:
\begin{array}{rclcl} z_1 = x^2 &=& \dfrac13 &\vert& \pm\sqrt{} \cr\cr x_{3,4} &=& \pm\sqrt{\dfrac13} \cr\cr z_2 = x^2 &=& -\dfrac23 &\vert& \pm\sqrt{} \cr\cr x_{5,6} &=& \pm\sqrt{-\dfrac23}\end{array}

Da aus negativen reellen Zahlen keine Wurzeln mit geraden Wurzelexponenten gezogen werden können, liefern der erste Faktor und die Rücksubstitution von z_2 keine weiteren Lösungen: \mathbb{L} = \left\{-\sqrt{\dfrac13}\; ; \; \sqrt{\dfrac13}\right\}


12)
\begin{array}{lrclcll} & \mathbb{D} &=& \mathbb{R} \cr\cr & (2x^3+8x^2-6x)(9x^3-10x) &=& 0 \cr\cr\text{Faktor 1:} & 2x^3+8x^2-6x &=& 0 \cr & x(2x^2+8x-6) &=& 0 &\vert& \text{Satz vom Nullprodukt} \cr\cr \text{Faktor 1.1:} & x_1 &=& 0 \cr\cr \text{Faktor 1.2:} & 2x^2+8x-6 &=& 0 &\vert& :2 \cr & x^2+4x-3 &=& 0 &\vert& \text{p-q-Formel} \cr & x_{2,3} &=& -\dfrac42\pm\sqrt{\left(\dfrac42\right)^2+3} \cr& x_{2,3} &=& -2\pm\sqrt{7} \cr\cr & x_2 &=& -2+\sqrt{7} \approx 0{,}65 \cr\cr & x_3 &=& -2-\sqrt{7} \approx -4{,}65 \cr\cr\text{Faktor 2:} & 9x^3-10x &=& 0 \cr & x(9x^2-10) &=& 0 &\vert& \text{Satz vom Nullprodukt} \cr\cr \text{Faktor 2.1:} & x_4 &=& 0 \cr\cr \text{Faktor 2.2:} & 9x^2-10 &=& 0 &\vert& +10 \cr & 9x^2 &=& 10 &\vert& :9 \cr & x^2 &=& \dfrac{10}{9} &\vert& \pm\sqrt{} \cr & x_{5,6} &=& \pm\sqrt{\dfrac{10}{9}} \approx \pm 1{,}05 \cr\cr & \mathbb{L} &=& \left\{-2-\sqrt{7};-\sqrt{\dfrac{10}{9}};0;-2+\sqrt{7};\sqrt{\dfrac{10}{9}}\right\}\end{array}


13)
\begin{array}{rcl}\mathbb{D} &=& \mathbb{R} \cr\cr x^3-x^2-50x-48 &=& 0\end{array}

Durch Probieren finden wir die Lösung x_1= -1, denn
\begin{array}{rcl}(-1)^3-(-1)^2 -50\cdot(-1)-48 &=& 0 \cr 0 &=& 0\end{array}

Polynomdivision mit x_1 = -1:
Polynomdivision
Lösung des reduzierten Polynoms:
\begin{array}{rclcl} 0 &=& x^2-2x-48 &\vert& \text{p-q-Formel} \cr x_{2,3} &=& 1\pm\sqrt{1^2+48} \cr x_{2,3} &=& 1\pm\sqrt{49} \cr\cr x_2 &=& 1+7 = 8 \cr x_3 &=& 1-7 = -6 \cr\cr\mathbb{L} &=& \left\{-6;-1;8\right\}\end{array}


14)
\begin{array}{lrclcll} & \mathbb{D} &=& \mathbb{R} \cr\cr & \dfrac45 x^5+16x^3 &=& 0 \cr& x^3\left(\dfrac45 x^2+16\right) &=& 0 &\vert& \text{Satz vom Nullprodukt} \cr\text{Faktor 1:} & x^3 &=& 0 &\vert& \sqrt[3]{} \cr& x_1 &=& 0 \cr\cr \text{Faktor 2:} & \dfrac45 x^2+16 &=& 0 &\vert& -16 \cr & \dfrac45 x^2 &=& -16 &\vert& : \dfrac45 \cr & x^2 &=& -20 &\vert& \pm\sqrt{} \cr & x_{2,3} &=& \pm\sqrt{-20}\end{array}

Da aus negativen reellen Zahlen keine Wurzeln mit geraden Wurzelexponenten gezogen werden können, liefert der zweite Faktor keine weiteren Lösungen: \mathbb{L} = \left\{0\right\}


15)
\begin{array}{rclcll} \mathbb{D} &=& \mathbb{R} \cr\cr x^4-2x^3-7x^2+8x+12 &=& 0\end{array}

Durch Probieren finden wir die Lösungen x_1=-2 und x_2=2, denn
\begin{array}{rcl}(-2)^4-2\cdot(-2)^3-7\cdot(-2)^2+8\cdot(-2)+12 &=& 0 \cr 0 &=& 0\end{array}
und
\begin{array}{rcl}2^4-2\cdot 2^3-7\cdot 2^2+8\cdot 2+12 &=& 0 \cr 0 &=& 0\end{array}

Polynomdivision mit x_1=2 und x_2=-2, also mit (x+2)\cdot(x-2)= \left(x^2-4\right):
Polynomdivision

Lösung des reduzierten Polynoms:
\begin{array}{rclcl} 0 &=& x^2-2x-3 &\vert& \text{p-q-Formel} \cr x_{3,4} &=& 1\pm\sqrt{(-1)^2+3} \cr x_{3,4} &=& 1\pm\sqrt{4} \cr\cr x_3 &=& 1+2 = 3 \cr x_4 &=& 1-2 = -1 \cr\cr\mathbb{L} &=& \left\{-2;-1;2;3\right\}\end{array}


16)
\begin{array}{rcl} \mathbb{D} &=& \mathbb{R} \cr\cr 2x^6+\sqrt{2}x^3-15 &=& 0 \cr\left(\sqrt{2}x^3\right)^2+\sqrt{2}x^3-15 &=& 0\end{array}

Substitution: z=\sqrt{2}x^3
\begin{array}{rclcl} z^2+z-15 &=& 0 &\vert& \text{p-q-Formel} \cr z_{1,2} &=& -\dfrac12\pm\sqrt{\left(\dfrac12\right)^2+15} \cr\cr z_{1,2} &=& -\dfrac12\pm\sqrt{\dfrac{61}{4}} \cr\cr z_1 &=& \dfrac{-1+\sqrt{61}}{2} \approx 3{,}41 \cr\cr z_2 &=& \dfrac{-1-\sqrt{61}}{2} \approx -4{,}41\end{array}

Rücksubstitution:
\begin{array}{rclcl} z_1 = \sqrt{2}x^3 &=& \dfrac{-1+\sqrt{61}}{2} &\vert& :\sqrt{2} \cr\cr x^3 &=& \dfrac{-1+\sqrt{61}}{2\sqrt{2}} &\vert& \sqrt[3]{} \cr\cr x &=& \sqrt[3]{\dfrac{-1+\sqrt{61}}{\left(\sqrt{2}\right)^3}} \cr\cr x &=& \dfrac{\sqrt[3]{-1+\sqrt{61}}}{\sqrt{2}} \approx 1{,}34 \cr\cr z_2 = \sqrt{2}x^3 &=& \dfrac{-1-\sqrt{61}}{2} &\vert& :\sqrt{2} \cr\cr x^3 &=& \dfrac{-1-\sqrt{61}}{2\sqrt{2}} &\vert& \sqrt[3]{} \cr\cr x &=& \sqrt[3]{\dfrac{-1-\sqrt{61}}{\left(\sqrt{2}\right)^3}} \cr\cr x &=& \dfrac{\sqrt[3]{-1-\sqrt{61}}}{\sqrt{2}} \approx -1{,}46 \cr\cr \mathbb{L} &=& \left\{\dfrac{\sqrt[3]{-1-\sqrt{61}}}{\sqrt{2}}\;;\;\dfrac{\sqrt[3]{-1+\sqrt{61}}}{\sqrt{2}}\right\}\end{array}


17)
\begin{array}{lrclcll} & \mathbb{D} &=& \mathbb{R} \cr\cr& (2x^3+8x^2-6x)(9x^3-30)(4x^4-12x^2+2) &=& 0 \cr\cr\text{Faktor 1:} & 2x^3+8x^2-6x &=& 0 \cr & x(2x^2+8x-6) &=& 0 &\vert& \text{Satz vom Nullprodukt} \cr\cr\text{Faktor 1.1:} & x_1 &=& 0 \cr\cr \text{Faktor 1.2:} & 2x^2+8x-6 &=& 0 &\vert& :2 \cr & x^2+4x-3 &=& 0 &\vert& \text{p-q-Formel} \cr& x_{2,3} &=& -2\pm\sqrt{2^2+3} \cr\cr & x_{2} &=& -2+\sqrt{7} \approx 0{,}65 \cr& x_{3} &=& -2-\sqrt{7} \approx -4{,}65 \cr\cr\text{Faktor 2:} & 9x^3-30 &=& 0 &\vert& +30 \cr & 9x^3 &=& 30 &\vert& :9 \cr & x^3 &=& \dfrac{10}{3} &\vert& \sqrt[3]{} \cr& x_4 &=& \sqrt[3]{\dfrac{10}{3}} \approx 1{,}49 \cr\cr\text{Faktor 3:} & 4x^4-12x^2+2 &=& 0 &\vert& :4 \cr & x^4-3x^2+\dfrac12 &=& 0\end{array}

Substitution: y=x^2
\begin{array}{rclcl} y^2-3y+\dfrac12 &=& 0 &\vert& \text{p-q-Formel} \cr y_{1,2} &=& \dfrac32\pm\sqrt{\left(-\dfrac32\right)^2 -\dfrac12} \cr y_{1,2} &=& \dfrac32\pm\sqrt{\dfrac74} \cr\cr y_1 &=& \dfrac{3+\sqrt{7}}{2} \approx 2{,}82 \cr y_2 &=& \dfrac{3-\sqrt{7}}{2} \approx 0{,}18\end{array}

Rücksubstitution:
\begin{array}{rclcl}y_1 = x^2 &=& \dfrac{3+\sqrt{7}}{2} &\vert& \pm\sqrt{} \cr x_{5,6} &=& \pm\sqrt{\dfrac{3+\sqrt{7}}{2}} \approx \pm1{,}68 \cr\cr y_2 = x^2 &=& \dfrac{3-\sqrt{7}}{2} &\vert& \pm\sqrt{} \cr x_{7,8} &=& \pm\sqrt{\dfrac{3-\sqrt{7}}{2}} \approx \pm0{,}42 \end{array}

\mathbb{L} = \left\{-2-\sqrt{7}\;;\;-\sqrt{\dfrac{3+\sqrt{7}}{2}}\;;\;-\sqrt{\dfrac{3-\sqrt{7}}{2}}\;;\;0\;;\;\sqrt{\dfrac{3-\sqrt{7}}{2}}\;;\;-2+\sqrt{7}\;;\;\sqrt[3]{\dfrac{10}{3}}\;;\;\sqrt{\dfrac{3+\sqrt{7}}{2}}\right\}


18)
\begin{array}{rcl} \mathbb{D} &=& \mathbb{R} \cr\cr x^3-ax^2-x^2+ax-2x+2a &=& 0\end{array}

Durch Probieren finden wir die Lösung x_1=a, denn
\begin{array}{rcl}a^3-a\cdot a^2-a^2+a\cdot a-2a+2a &=& 0 \cr 0 &=& 0\end{array}

Polynomdivision mit x_1=a:
Polynomdivision

Lösung des reduzierten Polynoms:
\begin{array}{rclcl} 0 &=& x^2-x-2 &\vert& \text{p-q-Formel} \cr x_{2,3} &=& \dfrac{1}{2}\pm \sqrt{\left(\dfrac{-1}{2}\right)^2+2} \cr x_{2,3} &=& \dfrac12\pm\sqrt{\dfrac94} \cr\cr x_2 &=& \dfrac12 +\dfrac32 = 2 \cr x_3 &=& \dfrac12 -\dfrac32 = -1 \cr\cr\mathbb{L} &=& \left\{a;-1;2\right\}\end{array}


19)
\begin{array}{rcl} \mathbb{D} &=& \mathbb{R} \cr\cr -x^3 +3x^2 +33x-35 &=& 0 \end{array}

Durch Probieren finden wir die Lösung x_1= 1, denn
\begin{array}{rcl} -1^3+ 3\cdot 1^2 +33 \cdot 1 -35 &=& 0 \cr 0 &=& 0 \end{array}

Polynomdivision mit x_1 = 1:
Polynomdivision

Lösung des reduzierten Polynoms:
\begin{array}{rclcl} 0 &=& -x^2 +2x +35 &\vert& \cdot(-1) \cr 0 &=& x^2 -2x -35 &\vert& \text{p-q-Formel} \cr x_{2,3} &=& 1 \pm \sqrt{1^2 +35} \cr x_{2,3} &=& 1\pm \sqrt{36} \cr x_{2,3} &=& 1\pm 6 \cr\cr x_2 &=& 1+6 = 7 \cr x_3 &=& 1-6 = -5 \cr\cr \mathbb{L} &=& \left\{-5;1;7\right\}\end{array}


20)
\begin{array}{rcl}\mathbb{D} &=& \mathbb{R} \cr\cr x^4 -3x^2-88 &=& 0 \cr (x^2)^2 -3x^2 -88 &=& 0 \end{array}

Substitution: z=x^2
\begin{array}{rclcl} 0 &=& z-3z-88 &\vert& \text{p-q-Formel} \cr z_{1,2} &=& \dfrac{3}{2}\pm\sqrt{\left(-\dfrac32\right)^2 +88} \cr z_{1,2} &=& \dfrac32 \pm \sqrt{\dfrac{361}{4}}\cr\cr z_1 &=& \dfrac32 +\dfrac{19}{2} = 11 \cr z_2 &=& \dfrac32 - \dfrac{19}{2} = -8\end{array}

Rücksubstitution:
\begin{array}{rclcl} z_1 = x^2 &=& 11 &\vert& \pm\sqrt{} \cr\cr x_{1,2} &=& \pm\sqrt{11} \approx 3{,}32 \cr\cr z_2 = x^2 &=& -8 &\vert& \pm\sqrt{} \cr\cr x_{3,4} &=& \pm\sqrt{-8} \end{array}

Da aus negativen reellen Zahlen keine Wurzeln mit geraden Wurzelexponenten gezogen werden können, liefert der zweite Faktor keine weiteren Lösungen: \mathbb{L} = \left\{-\sqrt{11}; \sqrt{11}\right\}

14. Bruchgleichungen und gebrochen rationale Funktionen - Lernziele und typische Fehler

Nach Durcharbeiten dieses Kapitels sollten Sie folgende Lernziele erreicht haben:

  • Sie können zu einer Bruchgleichung den passenden Definitionsbereich bestimmen.
  • Sie kennen die allgemeine Form einer Bruchgleichung.
  • Sie können Bruchgleichungen lösen.
  • Sie können die Lösungsmenge mathematisch korrekt notieren.
  • Sie können mithilfe der Probe überprüfen, ob die gefundene Lösung tatsächlich richtig ist.
  • Sie wissen, dass bei Bruchgleichungen Scheinlösungen auftreten können, wie man diese erkennt und wie man damit umgeht.
  • Sie können zu einer gebrochen rationalen Funktion den passenden Definitionsbereich bestimmen.
  • Sie wissen, wie der Graph einer gebrochen rationalen Funktion typischerweise aussieht, und können ihn in ein kartesisches Koordinatensystem zeichnen.
  • Sie kennen die allgemeine Funktionsgleichung einer gebrochen rationalen Funktion.
  • Sie kennen Eigenschaften von gebrochen rationalen Funktionen (Anzahl der Nullstellen, Symmetrie, Randverhalten, Extremstellen, Polstellen) und können diese nutzen, um Beziehungen zwischen Funktionsterm und Funktionsgraphen herzustellen.
  • Sie kennen den Zusammenhang zwischen Bruchgleichungen und gebrochen rationalen Funktionen.
  • Sie können Bruchgleichungen von anderen Gleichungsarten unterscheiden.
  • Sie können gebrochen rationale Funktionen von anderen Funktionstypen unterscheiden (grafisch und anhand der Funktionsgleichung).


Typische Fehler in diesem Kapitel sind:

  • Beim Multiplizieren der Gleichung mit einem der Nenner werden nicht alle Bestandteile der Gleichung multipliziert. Erklärung
  • Wenn der Nenner eine Summe/Differenz enthält, werden beim Multiplizieren mit dem Nenner keine Klammern gesetzt. Erklärung
  • Es wird falsch gekürzt. Erklärung
  • Es wird nicht geprüft, ob ein berechneter Wert eine Scheinlösung ist. Erklärung


Für Online-Selbsttests zu diesem Thema und weitere Informationen zur Mathematikunterstützung an der TH Wildau nutzen Sie bitte den Moodle-Kursraum "SOS Mathematik - Brückenkurs".

Übersicht:

 

14.1 Bruchgleichungen und gebrochen rationale Funktionen - Aufgaben

1. Aufgabe

Lösen Sie folgende Bruchgleichungen! Geben Sie jeweils den Definitionsbereich an!

1) \dfrac{x^2+2x-30}{x-3}=2

  11) 2 = \dfrac{10}{x-2}-\dfrac{10}{x+2}

2) \dfrac{x}{8-x^2}=-\dfrac{1}{x^2-8}

  12) -\dfrac{2(y-2)^2}{y}-2+\dfrac{(3y-8)^2}{3y} = \dfrac{14}{3y}

3) \dfrac{t^2+16t-4}{t+2}-10 = t

  13) 6(6t+1)=\dfrac{9+6t}{t}

4) \dfrac{x+21}{2}-4+\dfrac{1}{x}=7

  14) 4 \cdot \dfrac{2x+1}{2x+2} = \dfrac{x+1}{4x+2}
5) \dfrac{21}{x^3-7x}-4x=\dfrac{6}{x^3-7x}

  15) \dfrac{x^2-2x+10}{x+1}=\dfrac{3x+16}{x+1}

6) \dfrac{1}{x^2-81}=\dfrac{x}{x-9}-1

  16) -x\left(-x+2+\dfrac{2}{x}\right)=\dfrac{x}{14+2x} : \dfrac{x}{196-4x^2}

7) 6+\dfrac{5x^3+9x}{x^3}=-\dfrac{1}{x}+11

  17) \left(\dfrac{1}{x}-3\right) \cdot 2x^2 = \dfrac{x^2+8x+16}{x+4}

8) \dfrac{3z+4}{27z^2+72z+48}+\dfrac{z-6}{6z}=\dfrac{1}{3z+4}

  18) \dfrac{16x^2-120x}{x} = 8x^2 \cdot \dfrac{-x-1}{x+1}

9) \dfrac{x^2-8}{3(x-4)}=\dfrac{x^3-4x(-0{,}25x+2)-8}{3x^2-9x-12}   19) \dfrac{10x^2+4x}{16x-36x^2} = \dfrac{3}{x}

10) \dfrac{4x}{-4x-72} = \dfrac{9}{2x+3}

  20) \dfrac{27x^3+6x^2}{3x} = -\dfrac{3x+6}{27(x+2)}

 

2. Aufgabe

1)
Gegeben sei die Funktion f(x)= \dfrac{2+x}{x-1} mit \mathbb{D} = \mathbb{R}\setminus_{\left\{1\right\}}.
Gesucht ist jeweils die fehlende Koordinate des Punktes P(x\mid y),
a) wenn x=\dfrac{5}{3}
b) wenn y=-\dfrac{5}{2}

  6)
Gegeben sei die Funktion f(x)= \dfrac{x^2-2}{x^2+4}-2 mit \mathbb{D} = \mathbb{R}.
Gesucht ist jeweils die fehlende Koordinate des Punktes P(x\mid y),
a) wenn x=0
b) wenn y=-\dfrac{23}{20}

2)
Gegeben sei die Funktion f(x)= \dfrac{1}{x^2-4} mit \mathbb{D} = \mathbb{R}\setminus_{\left\{-2;2\right\}}.
Gesucht ist jeweils die fehlende Koordinate des Punktes P(x\mid y),
a) wenn x=6
b) wenn y=\dfrac{1}{5}

  7)
Gegeben sei die Funktion f(x)= \dfrac{20x^2}{2x-100} mit \mathbb{D} = \mathbb{R}\setminus_{\left\{50\right\}}.
Gesucht ist jeweils die fehlende Koordinate des Punktes P(x\mid y),
a) wenn x=\dfrac{1}{10}
b) wenn y=-\dfrac{25}{19}

3)
Gegeben sei die Funktion f(x)= \dfrac{5x-11}{x^3-8} mit \mathbb{D} = \mathbb{R}\setminus_{\left\{2\right\}}.
Gesucht ist jeweils die fehlende Koordinate des Punktes P(x\mid y),
a) wenn x=-4
b) wenn y=0

  8)
Gegeben sei die Funktion f(x)= \dfrac{-5x^3+16x^2-24x+160}{-2x^2+3x} mit \mathbb{D} = \mathbb{R}\setminus_{\left\{0;\;\frac{3}{2}\right\}}.
Gesucht ist jeweils die fehlende Koordinate des Punktes P(x\mid y),
a) wenn x=-5
b) wenn y=-8

4)
Gegeben sei die Funktion f(x)= \dfrac{2x^2-2x}{x^3+2x^2-3x} mit \mathbb{D} = \mathbb{R}\setminus_{\left\{0;1\right\}}.
Gesucht ist jeweils die fehlende Koordinate des Punktes P(x\mid y),
a) wenn x=-21
b) wenn y=-\dfrac{1}{8}

  9)
Gegeben sei die Funktion f(x)= \dfrac{x}{(x-2)^2} mit \mathbb{D} = \mathbb{R}\setminus_{\left\{2\right\}}.
Gesucht ist jeweils die fehlende Koordinate des Punktes P(x\mid y),
a) wenn x=7
b) wenn y=\dfrac{7}{169}

5)
Gegeben sei die Funktion f(x)= \dfrac{-15+x}{x} mit \mathbb{D} = \mathbb{R}\setminus_{\left\{0\right\}}.
Gesucht ist jeweils die fehlende Koordinate des Punktes P(x\mid y),
a) wenn x=6
b) wenn y=1

  10)
Gegeben sei die Funktion f(x)= \dfrac{2}{x(x-5)}-1 mit \mathbb{D} = \mathbb{R}\setminus_{\left\{0;\;5\right\}}.
Gesucht ist jeweils die fehlende Koordinate des Punktes P(x\mid y),
a) wenn x=-14
b) wenn y=-\dfrac{24}{25}

 

3. Aufgabe

Bestimmen Sie von folgenden Funktionen den Definitionsbereich und die Nullstellen!

1) f(x) = \dfrac{x+3}{x-2}   11) g(s) = \dfrac{s^2+2}{s}

2) g(x) = \dfrac{x^2-x+6}{x^2-3x+8}   12) f(x) = \dfrac{3x+4}{x^4-5}

3) f(x) = \dfrac{2x^2-4x+4}{x^3-10x^2+27x}   13) f(p) = \dfrac{2}{p+5}+3
4) f(z) = \dfrac{z^2-4}{z^3+3z}

  14) g(x) = \dfrac{\frac{3}{2}x^3+\frac{4}{3}x^2+\frac{5}{4}x}{x^7+x^3}
5) f(x) = \dfrac{(x^2+x-2)^2}{x^4+2x^2-8}   15) f(x) = \dfrac{4x^2+2}{4x-1}+1
6) f(x) = \dfrac{x^2-x-12}{x^2-2x-15}   16) f(y) = \dfrac{y^2-5}{2}-\dfrac{2}{y^2}+3

7) g(x) = \dfrac{x^2+8x+16}{x+4}   17) g(x) = \dfrac{x^5-x^2}{x}+\dfrac{x^3-4x^4}{4}

8) f(x) = \dfrac{x+7-x^2}{x^4+1}   18) f(x) = \dfrac{4x}{4x^3}-\left(\dfrac{1}{x^2}-4\right)+\dfrac{x^6-16}{2}
9) f(x) = \dfrac{2x+6}{(x^2-3)^4}

  19) f(x) = \dfrac{x^3+5x}{x^2+2}+3
10) f(x) = \dfrac{2x^3-5x^2+4x}{(x-1)^2}   20) f(b) = \dfrac{1}{b^3+5b}+2b

Dieses Kapitel enthält die folgenden Themen:

 

14.2 Bruchgleichungen und gebrochen rationale Funktionen - Erklärungen

Dieses Kapitel zeigt einen weiteren Gleichungs- bzw. Funktionstyp, nämlich Gleichungen/Funktionen, in denen Polynome durcheinander dividiert werden. Ein Anwendungsfall für solche Gleichungen und Funktionen sind antiproportionale Zusammenhänge: Beispielsweise berechnet sich die Geschwindigkeit bekanntermaßen nach der Gleichung v=\dfrac{s}{t}. Wenn die Geschwindigkeit v in Abhängigkeit der Strecke s bei festgelegter Zeit t berechnet werden soll, steht die Variable im Zähler - das kennen wir schon. Soll hingegen die Geschwindigkeit v in Abhängigkeit der Zeit t bei festgelegter Strecke s berechnet werden soll, steht die Variable im Nenner - darum werden wir uns in diesem Kapitel kümmern. Auch in den Wirtschaftswissenschaften gibt es Zusammenhänge, bei denen die Variable im Nenner des Funktionsterms steht, z. B. sogenannte "regressive Kostenfunktionen". Dabei handelt es sich um Kostenfunktionen, bei denen die Kosten/Stückkosten bei steigender Menge der Bezugsgrößen abnehmen.
Warum die Gleichungen dabei "Bruchgleichungen" und die Funktionen "gebrochen rational" heißen, kann ich Ihnen leider auch nicht sagen ...

 

Definition

Definition einer Bruchgleichung: Bruchgleichungen sind Gleichungen, bei denen die Variable im Nenner eines Bruchs steht.
Definition einer gebrochen rationalen Funktion: Gebrochen rationale Funktionen sind Funktionen, bei denen die Variable im Nenner eines Bruchs steht.

Anders formuliert: Eine Bruchgleichung/gebrochen rationale Funktion ist ein Quotient zweier Polynome, wobei der Grad des Polynoms im Nenner mindestens 1 sein muss.
Das Polynom im Zähler nennt man Zählerpolynom. Den Grad des Zählerpolynoms nennt man Zählergrad.
Das Polynom im Nenner nennt man Nennerpolynom. Den Grad des Nennerpolynoms nennt man Nennergrad.

Natürlich darf eine Bruchgleichung/gebrochen rationale Funktion auch mehrere Brüche enthalten, bei denen Variablen im Nenner stehen. Auch Brüche, bei denen die Variable im Zähler steht, sind erlaubt. Allerdings: Enthält eine Gleichung/Funktion nur Brüche, deren Nenner ausschließlich Zahlen enthalten, benötigen wir keinen neuen Funktions- und Gleichungstyp. Dann kämen wir mit linearen, quadratischen und Polynomgleichungen aus.

Definitionsbereich: Bei Bruchgleichungen/gebrochen rationalen Funktionen ist es wichtig, sich Gedanken zum Definitionsbereich zu machen: Da wir durch 0 nicht teilen dürfen, darf der Nenner niemals den Wert 0 annehmen. Wir müssen also prüfen, für welche Variablenwerte das passiert, und den Definitionsbereich so "zuschneiden", dass nur passende Werte enthalten sind. Dafür müssen Ungleichungen gelöst werden.

Wichtig: Der Definitionsbereich gehört fest zu Gleichungen/Funktionen dazu.
Das Umformen des Bruchterms, z. B. durch Kürzen, kann daher problematisch sein. Ein Beispiel: Die Funktion f(x)=\dfrac{x(x-1)(x+2)}{3(x-1)(x-4)} ist nicht identisch mit der Funktion f^{*}(x)=\dfrac{x(x+2)}{3(x-4)}. Denn: Der Definitionsbereich von f(x) ist \mathbb{D}=\mathbb{R}\setminus_{\{1;4\}}; der Definitionsbereich von f^{*}(x) ist \mathbb{D}=\mathbb{R}\setminus_{\{4\}}. Diese Veränderung darf nicht einfach so "unter den Tisch fallen". Weitere Erläuterungen dazu gibt es ganz unten in diesem Kapitel bei der Beispielfunktion f_5(x).

 

Bruchgleichungen

Starten wir auch hier wieder mit einigen Beispielen:

  • \dfrac{12}{x} = -36 mit \mathbb{D}=\mathbb{R}\setminus_{\{0\}}

  • \dfrac{x}{(x+1)^3} = \dfrac{-2x}{-1+x} mit \mathbb{D}=\mathbb{R}\setminus_{\{-1; 1\}}

  • \dfrac{3x-1}{x^2-4}-\dfrac{x}{x-2} = \dfrac{-x}{x+2} mit \mathbb{D}=\mathbb{R}\setminus_{\{-2; 2\}}
    Da der Nenner des ersten Bruchs mithilfe der 3. binomischen Formel umgeformt werden kann zu x^2-4 = (x-2)(x+2), gibt es nur zwei Variablenwerte, für die einer der Nenner 0 wird.

 

Lösungsweg

Bei Bruchgleichungen ist es immer das erste Ziel, die Brüche aus der Gleichung zu entfernen. Dann entstehen nämlich häufig lineare oder quadratische Gleichungen, von denen wir schon wissen, wie man sie löst. Das ist ein ganz typisches Vorgehen, wenn in der Mathematik neue Probleme auftauchen: Man versucht, das Problem so zu bearbeiten, dass etwas Bekanntes entsteht, wo man schon Lösungswege kennt. Dann muss man sich für den Rest keine weiteren Gedanken machen.
Für Bruchgleichungen bedeutet das, dass wir die gesamte Gleichung mit den enthaltenen Nennern multiplizieren müssen, wenn die Nenner Variablen enthalten. Sie erinnern sich, was passiert, wenn man einen Bruch mit seinem Nenner multipliziert? Richtig: Die Nenner kürzen sich gegenseitig weg, wie bei \dfrac{12}{x}\cdot x = 12

Ganz wichtig: Auch bei Bruchgleichungen darf man natürlich nicht aus Summen und ebenso nicht aus Potenzen mit unterschiedlichen Basen kürzen!

Zum Abschluss des Lösungsweges muss noch zwingend überprüft werden, ob sich alle ermittelten Werte für die Variable im Definitionsbereich befinden. Werte, die nicht im Definitionsbereich liegen, stellen keine Lösungen für die ursprüngliche Gleichung dar. Man bezeichnet sie daher als Scheinlösungen. Was genau das ist, wird weiter unten erklärt.



Schauen wir uns nun einige Beispiele an:

Beispiel 1:
Betrachten wir die Gleichung \dfrac{12}{x}+3 = -\dfrac{x}{x+6}+\dfrac{11}{2}.

Bestimmung des Definitionsbereichs:
Die beiden Nenner x und x+6 müssen ungleich 0 sein. Daher müssen wir nun ausrechnen, für welche x-Werte diese Bedingung erfüllt wird:
\begin{array}{rclll}x &\neq & 0 \\\\ x+6 &\neq & 0 &\vert & -6 \\ x &\neq & -6\end{array}

Aus der Betrachtung der beiden Ergebnisse können wir ableiten, dass wir alle Werte aus \mathbb{R} mit Ausnahme von -6 und 0 verwenden dürfen. Der Definitionsbereich ist also \mathbb{D} = \mathbb{R}\setminus_{\{-6;0\}}.

Lösung der Gleichung:
\begin{array}{crclcl}\text{1. Zeile:} & \dfrac{12}{x}+3 &=& -\dfrac{x}{x+6}+\dfrac{11}{2} &\vert& \cdot x \cr\text{2. Zeile:} &12+3x &=& -\dfrac{x}{x+6}\cdot x +\dfrac{11}{2}\cdot x &\vert& \cdot (x+6) \cr \text{3. Zeile:} &12(x+6)+ 3x(x+6) &=& -x\cdot x+\dfrac{11}{2}\cdot x(x+6) &\vert& \cdot 2 \cr \text{4. Zeile:} & 24(x+6)+6x(x+6) &=& -2x^2+11x(x+6) \cr & 24x+144+6x^2+36x &=& -2x^2+11x^2+66x \cr & 6x^2+60x+144 &=& 9x^2+66x &\vert&-9x^2-66x \cr & -3x^2-6x+144 &=& 0 &\vert& :(-3) \cr & x^2+2x-48 &=& 0 &\vert& \text{p-q-Formel} \cr & x_{1,2} &=& -1 \pm\sqrt{(-1)^2+48} \cr & &=& -1\pm\sqrt{49} \cr\cr & x_1 &=& -1+7 = 6 \in \mathbb{D} \cr & x_2 &=& -1-7 = -8 \in \mathbb{D}\cr\cr & \mathbb{L} &=& \left\{-8;6\right\}\end{array}

Zur 1. Zeile: Um die Brüche in der Gleichung zu eliminieren, wird zunächst mit x multipliziert. Dadurch verschwindet der Bruch auf der linken Seite. Wichtig: Alle Terme der Gleichung müssen mit dem x multipliziert werden! Ebenso wie beim Multiplizieren einer Gleichung mit einer Zahl alle Bestandteile der Gleichung mit dieser Zahl multipliziert werden müssen, ist dies auch beim Multiplizieren mit einem Nenner notwendig. Andernfalls können die Probleme auftreten, die bereits im Kapitel lineare Gleichungen erläutert wurden.

Zur 2. Zeile: Analog zur 1. Zeile soll jetzt der Bruch auf der rechten Seite verschwinden. Daher wird mit dem Nenner des Bruches, also mit (x+6) multipliziert. Auch hier müssen alle Terme mit dem Faktor durchmultipliziert werden! Zusätzlich wichtig: Da hier mit einer Summe multipliziert wird, müssen Klammern darum gesetzt werden. Sonst kommen Punkt- und Strichrechnung durcheinander ...

Zur 3. Zeile: Der letzte Bruch der Gleichung ist \frac{11}{2}. Diesen Bruch könnte man stehen lassen. Da \frac{11}{2} ja "nur" eine Zahl ist, würde er beim restlichen Lösungsweg nicht weiter stören. Wenn man alle Terme mit 2 multipliziert, wie wir das hier gemacht haben, kann man komplett ohne Brüche weiterrechnen.

Ab der 4. Zeile: Nachdem nun alle Brüche entfernt wurden, können die Klammern ausmultipliziert und anschließend die Terme zusammengefasst werden - so wie man das bei jeder anderen Gleichung auch tun würde. Das Ergebnis ist eine quadratische Gleichung, welche mit der p-q-Formel oder der a-b-c-Formel gelöst werden kann.

Zum Abschluss: Beide berechneten Werte liegen im Definitionsbereich und sind also Lösungen der Gleichung.

 

In der Erklärung zur 2. Zeile von Beispiel 1 wurde betont, dass Klammern um den Nenner gesetzt werden müssen, da der Nenner aus einer Summe besteht, die mit einem anderen Term multipliziert wird und andernfalls die Rechenoperationen nicht mehr in der richtigen Reihenfolge ausgeführt werden. Schauen wir uns in einem nächsten Beispiel an, was passiert, wenn man die Klammern vergisst.

Beispiel 2: 
Betrachten wir die Gleichung 3x = \dfrac{15}{4+x}.

Bestimmung des Definitionsbereichs:
\begin{array}{rclll}4+x &\neq & 0 &\vert & -4\\x &\neq & -4\end{array}

Der Definitionsbereich ist also \mathbb{D} = \mathbb{R}\setminus_{\{-4\}}.

Zunächst die korrekte Rechnung mit Probe
\begin{array}{rclcl} 3x &=& \dfrac{15}{4+x} &\vert& \cdot(4+x) \cr 3x\cdot(4+x) &=& 15 &\vert& \text{ausmultiplizieren} \cr 12x+3x^2 &=& 15 &\vert& -12x-3x^3 \cr 0 &=& -3x^2-12x+15 &\vert& :(-3) \cr 0 &=& x^2+4x-5 &\vert& \text{p-q-Formel} \cr x_{1,2} &=& -\dfrac{4}{2}\pm\sqrt{\left(\dfrac{4}{2}\right)^2-(-5)} \cr x_{1,2} &=& -2\pm 3 \cr\cr x_1 &=& -2+3 = 1 \cr x_2 &=& -2-3 = -5 \end{array}

Probe:
Für x_1=1
\begin{array}{rcl} 3\cdot 1 &=& \dfrac{15}{4+1} \cr3 &=& \dfrac{15}{5} \cr3 &=& 3 \end{array}

Für x_2=-5
\begin{array}{rcl} 3\cdot (-5) &=& \dfrac{15}{4+(-5)} \cr-15 &=& \dfrac{15}{-1} \cr-15 &=& -15 \end{array}

Für x_1=1 und x_2=-5 ergeben sich wahre Aussagen: \mathbb{L} = \left\{-5;1\right\}

Würde man in der zweiten Zeile dieser Rechnung fälschlicherweise keine Klammern setzen, erhielte man die folgende Rechnung:
\begin{array}{rclcl} 3x\cdot 4+x &=& 15 \cr12x+x &=& 15 \cr13x &=& 15 &\vert & :13 \cr x &=& \dfrac{15}{13} \end{array}

Probe für x=\dfrac{15}{13}:
\begin{array}{rcl} 3\cdot\dfrac{15}{13} &=& \dfrac{15}{4+\frac{15}{13}} \cr\cr\dfrac{45}{13} &=& \dfrac{15}{\frac{67}{13}} \cr\cr\dfrac{45}{13} &=& \dfrac{195}{67} \end{array}

Man sieht, dass ohne die Klammern aus der quadratischen Gleichung mit zwei Lösungen eine lineare Gleichung mit nur einer Lösung wird. Setzt man die Lösung der linearen Gleichung in die Ausgangsgleichung ein, geht die Probe nicht auf. Das heißt, die fehlenden Klammern ändern die Rechnung so erheblich, dass der ausgerechnete Wert keine Lösung mehr ist. Daher: Die Klammern müssen also immer gesetzt werden, wenn der Nenner eine Summe/Differenz enthält!

 

Beispiel 3:
Dass man nicht einfach mit jedem Nenner durchmultiplizieren sollte, sieht man an der folgenden Bruchgleichung -\dfrac{1}{x+10}+\dfrac{6}{5} = \dfrac{25}{x^2+20x+100}.

Bestimmung des Definitionsbereichs:
\begin{array}{rclll}x+10 &\neq & 0 &\vert & -10\\x &\neq & -10\\\\x^2+20x+100 &\neq & 0 &\vert &\text{p-q-Formel}\\x_{1,2} &\neq & -10\pm\sqrt{10^2-100}\\x_{1,2} &\neq & -10\pm 0\\x_{1,2} &\neq & -10\end{array}

Der Definitionsbereich ist also \mathbb{D} = \mathbb{R}\setminus_{\{-10\}}.

1. Lösung der Gleichung: Zunächst der geschicktere Lösungsweg, bei dem man sich allerdings zuerst die Nenner genau anschauen muss. Dann stellt man nämlich fest, dass der Nenner auf der rechten Seite mithilfe der 1. binomischen Formel zusammengefasst werden kann und das Quadrat des ersten Nenners auf der linken Seite ist (der zweite Nenner links ist ja "nur" eine Zahl und damit eigentlich unerheblich). Man muss also die Gleichung nur mit (x+10)^2 multiplizieren, um beide Nenner "loszuwerden".
\begin{array}{crclcl}\text{1. Zeile:} & -\dfrac{1}{x+10}+\dfrac{6}{5} &=& \dfrac{25}{x^2+20x+100} \cr \text{2. Zeile:} & -\dfrac{1}{x+10}+\dfrac{6}{5} &=& \dfrac{25}{(x+10)^2} &\vert& \cdot (x+10)^2 \cr\text{3. Zeile:} & -\dfrac{1}{x+10}\cdot (x+10)^2 +\dfrac{6}{5}\cdot (x+10)^2 &=& 25 &\vert& \cdot 5 \cr \text{4. Zeile:} & -5(x+10)+6\left(x^2+20x+100\right) &=& 125 \cr & -5x-50 +6x^2+120x+600 &=& 125 \cr & 6x^2+115x+550 &=& 125 &\vert& -125 \cr & 6x^2+115x+425 &=& 0 \cr & & \dots \cr & x_1 &=& -\dfrac{85}{6} \in \mathbb{D} \cr & x_2 &=& -5 \in \mathbb{D} \cr\cr& \mathbb{L} &=& \left\{-\dfrac{85}{6}; -5\right\}& \end{array}

Zur 1. Zeile: Wie oben beschrieben, wird hier der Nenner des Bruchs auf der rechten Seite mithilfe der binomischen Formel zusammengefasst, quasi als Vorbereitung des eigentlichen Lösungsweges.

Zur 2. Zeile: Nun werden auch hier die Brüche eliminiert. Dazu wird die Gleichung mit dem Nenner des rechten Bruches durchmultipliziert.

Zur 3. Zeile: Jetzt sieht man, dass durch Umformung aus der 2. Zeile auch der erste Bruch auf der linken Seite verschwindet, da (x+10) und (x+10)^2 gekürzt werden können. Zudem wird nun mit 5 multipliziert, um auch den zweiten Bruch auf der linken Seite zu entfernen.

Ab der 4. Zeile: Nun haben wir die Bruchgleichung zu einer quadratischen umgeformt, die wir nach den üblichen Methoden lösen können. Vergessen Sie nicht zu prüfen, ob x_1 und x_2 im Definitionsbereich liegen!


2. Lösung der Gleichung: Natürlich kann man auch - ohne die Überlegungen zu den Nennern - einfach mit jedem Nenner multiplizieren. Dann passiert Folgendes:
\begin{array}{crclcl}\text{1. Zeile:} &-\dfrac{1}{x+10}+\dfrac{6}{5} &=& \dfrac{25}{x^2+20x+100} &\vert& \cdot (x+10) \cr \text{2. Zeile:} & -1+\dfrac{6}{5}\cdot (x+10) &=& \dfrac{25}{x^2+20x+100}\cdot (x+10) &\vert& \cdot 5 \cr \text{3. Zeile:} & -5+6(x+10) &=& \dfrac{125}{x^2+20x+100}\cdot (x+10) &\vert& \cdot \left(x^2+20x+100\right) \cr \text{4. Zeile:} & -5\left(x^2+20x+100\right)+6(x+10)\left(x^2+20x+100\right) &=& 125(x+10) \cr & -5x^2-100x-500+(6x+60)\left(x^2+20x+100\right) &=& 125x+1.250 \cr & -5x^2-100x-500+6x^3+120x^2+600x+60x^2+1.200x+6.000 &=& 125x+1.250 \cr & 6x^3+175x^2+1.700x+5.500 &=& 125x+1.250 &\vert& -125x-1.250 \cr & 6x^3+175x^2+1.575x+4.250 &=& 0 \cr & & \dots \cr & x_1 &=& -\dfrac{85}{6} \cr & x_2 &=& -10 \not\in\mathbb{D} \cr & x_3 &=& -5 \cr\cr& \mathbb{L} &=& \left\{-\dfrac{85}{6}; -5\right\}\end{array}

Zur 1. Zeile: Wir ersparen uns die Vorbereitung des Lösungsweges und multiplizieren sofort mit dem Nenner des linken Bruches.

Zur 2. Zeile: Nun wird die Gleichung mit 5 multipliziert, um den zweiten Bruch auf der linken Seite zu entfernen.

Zur 3. Zeile: Der letzte Bruch der Gleichung befindet sich auf der rechten Seite. Nun wird die Gleichung also mit dessen Nenner multipliziert.

Ab der 4. Zeile: Nun sind alle Brüche aus der Gleichung entfernt und die einzelnen Terme können ausmultipliziert und sortiert werden. Dabei erhält man hier eine kubisches Polynom, also eine Gleichung 3. Grades.

Wenn man die beiden entstehenden Gleichungen 6x^2+115x+425 = 0 und 6x^3+175x^2+1.575x+4.250 = 0 vergleicht, sieht man schnell, warum es sich lohnt, die Überlegungen zu den Nennern anzustellen. Während die erste relativ problemlos mit der a-b-c-Formel oder der p-q-Formel gelöst werden kann, bleibt bei der zweiten nur, eine Lösung zu erraten und dann eine Polynomdivision durchzuführen. Dazu kommt, dass sich eine dritte Lösung ergibt, die aber nicht im Definitionsbereich der Ausgangsgleichung liegt. Man kann zwar etwas schneller losrechnen, der Lösungsweg wird aber sehr viel komplizierter und damit auch fehleranfälliger.

 

Nach Abschluss der Rechnung: Prüfung auf Scheinlösungen! 
Leider sind nicht alle Gleichungsarten so gutmütig wie lineare und quadratische. Beispielsweise können bei Bruchgleichungen und bestimmten anderen Gleichungstypen sogenannte Scheinlösungen entstehen. Das bedeutet, dass man trotz völlig richtiger Rechnung Werte ausrechnet, die die Ausgangsgleichung doch nicht lösen. Ursache dafür ist, dass uns bestimmte Umformungen im Lösungsweg zwar weiterbringen, sie aber trotzdem die Lösungsmenge ändern können. Mathematisch sagt man: Sie sind keine Äquivalenzumformungen

Schauen wir uns das am Beispiel \dfrac{x^2+x-2}{x-1} = 0 an:

Bestimmung des Definitionsbereichs:
\begin{array}{rclll}x-1 &\neq & 0 &\vert & +1\\x &\neq & 1\end{array}

Der Definitionsbereich ist also \mathbb{D} = \mathbb{R}\setminus_{\{1\}}. Für x=1 würde man ja durch 0 dividieren, was bekanntermaßen nicht geht.

Lösung der Gleichung:
\begin{array}{rclcl} \dfrac{x^2+x-2}{x-1} &=& 0 &\vert& \cdot (x-1) \cr x^2+x-2 &=& 0 &\vert& \text{p-q-Formel} \cr & \dots \cr x_1 &=& 1 \cr x_2 &=& -2\end{array}

Multipliziert man die Ausgangsgleichung mit dem Nenner, erhält man die quadratische Gleichung x^2+x-2 = 0, die die Lösungen x_1=1 und x_2=-2 hat (Das können Sie selbst ausrechen ...).
Nun kommt das Problem: Die quadratische Gleichung "weiß" nicht, dass es mal einen Nenner gegeben hat ... Dieser Nenner würde 0 werden, wenn wir x_1=1 einsetzten. Daher hatten wir zu Beginn 1 aus dem Definitionsbereich ausgeschlossen. Der Wert 1 kommt als Lösung der Ausgangsgleichung gar nicht infrage. x_1=1\not\in\mathbb{D} ist also eine Scheinlösung der Bruchgleichung.
Nur x_2=-2\in\mathbb{D} ist wirklich eine Lösung der ursprünglichen Bruchgleichung. Der Definitionsbereich spielt also bei dieser Art von Gleichung eine ziemlich wichtige Rolle. Es muss immer, nachdem die Gleichung gelöst wurde, überprüft werden, ob die berechneten Werte im Definitionsbereich liegen. Dann kann man die Scheinlösungen aus der Lösungsmenge ausschließen. Da Scheinlösungen die ursprüngliche Gleichung ja nicht lösen, haben sie in der Lösungsmenge natürlich nichts zu suchen ... Die Lösungsmenge unserer Beispielgleichung ist demnach: \mathbb{L}=\{-2\}
Sie sehen also, dass das korrekte Rechnen nur ein Teil des richtigen Lösungsweges ist. Noch mal zurück zu den Ausgangsbedingungen und zum Definitionsbereich zu gehen und zu prüfen, ob damit alles in Ordnung ist, ist hier zwingend nötig.

 

Gebrochen rationale Funktionen

Wir schauen uns den Verlauf und die Eigenschaften von gebrochen rationalen Funktionen zunächst anhand der einfachsten Beispiele, die möglich sind, an. Dabei müssen wir zwei Fälle unterscheiden, weil der Verlauf der Graphen abhängig ist von den Exponenten im Nenner. Als dritter Fall kommen dann Beispiele für komplexere gebrochen rationale Funktionen hinzu.

 

1. Fall

Im Nenner des Funktionsterms steht nur ein Term und dieser hat einen ungeraden Exponenten, also Funktionen der folgenden Gestalt f(x)=\dfrac{1}{x^n}=x^{-n}, wobei n eine ungerade natürliche Zahl ist. Die Graphen nennt man Hyperbeln ungerader Ordnung. Hier einige Beispiele:

  • f_1(x)=\dfrac{1}{x} mit \mathbb{D}=\mathbb{R}\backslash_{\{0\}}

  • f_2(x)=\dfrac{1}{x^3} mit \mathbb{D}=\mathbb{R}\backslash_{\{0\}} 

Beispiele für gebrochen rationale Funktionen ungeraden Grades

Wertebereich: Der Wertebereich umfasst außer der 0 alle reellen Zahlen. 0 kann sich ja nicht als Ergebnis einer Division ergeben, wenn 1 durch eine andere reelle Zahl geteilt wird.

Randverhalten: An den Rändern des Definitionsbereichs nähern sich die Funktionswerte sowohl für sehr kleine als auch für sehr große x-Werte der 0 an.

Symmetrie: Als Funktionen mit ausschließlich ungeraden Exponenten sind ihre Graphen punktsymmetrisch zum Koordinatenursprung.

"Besondere Punkte": Die Hyperbeln haben keine Nullstellen, Extrempunkte und Wendepunkte. Es gibt aber eine Polstelle bei x=0, da die Funktion hier eine Definitionslücke hat. Unmittelbar links von der Definitionslücke werden die Funktionswerte sehr klein und unmittelbar rechts davon werden sie sehr groß.

Weitere Besonderheiten: Alle Funktionsgraphen in diesem Fall verlaufen durch die Punkte (-1 \mid -1) und (1 \mid 1). Das kann man einfach nachrechnen:

  • Da n ungerade ist, gilt: f(-1)=\dfrac{1}{(-1)^n}=\dfrac{1}{-1}=-1
  • Immer gilt: f(1)=\dfrac{1}{1^n}=\dfrac{1}{1}=1

  

2. Fall

Im Nenner des Funktionsterms steht nur ein Term und dieser hat einen geraden Exponenten, also Funktionen der folgenden Gestalt f(x)=\dfrac{1}{x^n}=x^{-n}, wobei n eine gerade natürliche Zahl, außer 0, ist. Die Graphen nennt man Hyperbeln gerader Ordnung. Hier einige Beispiele:

  • f_3(x)=\dfrac{1}{x^2} mit \mathbb{D}=\mathbb{R}\backslash_{\{0\}} 

  • f_4(x)=\dfrac{1}{x^4} mit \mathbb{D}=\mathbb{R}\backslash_{\{0\}}

Beispiele für gebrochen rationale Funktionen geraden Grades

Wertebereich: Der Wertebereich umfasst nur die positiven reellen Zahlen. Egal, welche Zahl für x eingesetzt wird: Da mit einer gerade Zahl potenziert wird, ist der Nenner immer positiv. Und wenn man zwei positive Zahlen durcheinander teilt, muss das Ergebnis ebenfalls positiv sein.

Randverhalten: Die Funktionswerte nähern sich sowohl für sehr kleine als auch für sehr große x-Werte der 0 an.

Symmetrie: Als Funktionen mit ausschließlich geraden Exponenten sind ihre Graphen achsensymmetrisch zur y-Achse.

"Besondere Punkte": Die Hyperbeln haben keine Nullstellen, Extrempunkte und Wendepunkte, aber eine Polstelle bei x=0.

Weitere Besonderheiten: Alle Funktionsgraphen verlaufen durch die Punkte (-1\mid 1) und (1 \mid 1). Auch hier kann man das nachrechnen:

  • Da n gerade ist, gilt: f(-1)=\dfrac{1}{(-1)^n}=\dfrac{1}{1}=1
  • Immer gilt: f(1)=\dfrac{1}{1^n}=\dfrac{1}{1}=1

 

3. Fall

Gebrochen rationale Funktionen können aber noch viel mehr. Die Definition erlaubt ja jede Art von Polynom in Zähler und Nenner. Sie werden in den folgenden Grafiken sehen, dass der Verlauf von komplexeren gebrochen rationalen Funktionsgraphen ziemlich unterschiedlich sein kann. Sie heißen dann auch nicht mehr Hyperbeln. Hier einige Beispiele für gebrochen rationale Funktionen allgemeiner Art:

  • f_5(x)=\dfrac{x(x-1)(x+2)}{3(x-1)(x-4)} mit \mathbb{D}=\mathbb{R}\backslash_{\{1; 4\}}

  • f_6(x)=\dfrac{20x^2-40}{\left(x^2+5\right)\left(x^2+1\right)} mit \mathbb{D}=\mathbb{R}

  • f_7(x)=\dfrac{1+6\left((x+3)^4+2\right)}{\left((x+3)^4+2\right)} mit \mathbb{D}=\mathbb{R}

  • f_8(x)=\dfrac{x-5}{(x-1)\left((x-1)^4-2\right)} mit \mathbb{D}=\mathbb{R}\backslash_{\left\{1-\sqrt[4]{2}; \;1; \;1+\sqrt[4]{2}\right\}}

Beispiele für beliebige gebrochen rationale Funktionen

Wertebereich: Der Wertebereich kann sämtliche reellen Zahlen oder nur Teilbereiche von ihnen umfassen. Beispielsweise ist der Wertebereich von f_7(x) nur das halboffene Intervall \left]6;\frac{13}{2}\right].

Randverhalten: Ob sich die Funktionswerte für sehr kleine und sehr große x-Werte der 0 oder einer anderen reellen Zahl annähern oder sehr groß bzw. sehr klein werden, hängt vom Verhältnis von Zähler- zu Nennergrad ab. Um einen Eindruck zu bekommen, wie die Funktion global verläuft, kann man eine Polynomdivision durchführen. Am Beispiel von f_5(x):
f_5(x)=\dfrac{x(x-1)(x+2)}{3(x-1)(x-4)} = \dfrac{1}{3}x+2+\dfrac{24}{3x-12}
Wir hatten oben schon gesehen, dass bei einer Polynomdivision ein Rest bleiben kann. Das passiert auch hier. Je größer der Betrag von x wird, desto mehr nähert sich der Wert des Rest-Bruches der 0 an. Für sehr große und sehr kleine x-Werte spielt er also keine Rolle, sodass die Funktion annähernd so verläuft wie die lineare Funktion g(x)=\frac{1}{3}x+2. Man nennt dies das asymptotische Verhalten. Zwischen den ganz kleinen und ganz großen x-Werten kann natürlich alles Mögliche passieren (Nullstellen, Extrempunkte, ...). Darüber sagt das asymptotische Verhalten nichts aus. f_5(x) hat beispielsweise bei x=4 eine Polstelle, was bei linearen Funktionen nicht vorkommen kann. 

"Besondere Punkte":
Eine gebrochen rationale Funktion in allgemeiner Form kann Nullstellen haben und zwar an den Nullstellen des Zählerpolynoms. Allerdings muss eine Nullstelle des Zählerpolynoms nicht zwangsläufig eine Nullstelle der gesamten Funktion bewirken (das hängt davon ab, ob das Nennerpolynom bei diesem x-Wert auch eine Nullstelle hat oder nicht). 

Ob eine gebrochen rationale Funktion Extrem- und Wendepunkte hat, kann man nicht im Allgemeinen sagen. 

Gebrochen rationale Funktionen können an den Nullstellen des Nennerpolynoms (also an den Definitionslücken) Polstellen haben oder auch nicht (das hängt davon ab, ob das Zählerpolynom an dieser Stelle auch eine Nullstelle hat).
Schauen wir uns f_5(x)=\dfrac{x(x-1)(x+2)}{3(x-1)(x-4)} mit \mathbb{D}=\mathbb{R}\backslash_{\{1; 4\}} nochmal etwas genauer an: Sowohl im Zähler als auch im Nenner steht der Faktor (x-1). Daher ist die 1 auch nicht Element des Definitionsbereichs, ebenso wie die 4, die durch den Faktor (x-4) ausgeschlossen wird. In der Grafik oben sehen wir aber, dass sich die Funktion bei x=1 und x=4 ganz unterschiedlich verhält: Bei x=4 hat sie eine Polstelle, während bei x=1 auf der ersten Blick gar nichts Auffälliges passiert. Was man auf den ersten Blick nicht sieht, was aber trotzdem da ist, ist eine Definitionslücke bei x=1, d. h. der Graph hat hier ein "Loch". Daher darf man auch nicht einfach den Faktor (x-1), der in Zähler und Nenner vorkommt, kürzen und den Definitionsbereich anpassen. Das wäre eine andere Funktion! Gleichwohl ist x=1 eine "besondere" Definitionslücke, da bei diesem Wert sowohl im Zähler als auch im Nenner eine Nullstelle auftritt. Man nennt dies eine hebbare Definitionslücke, weil man durch das Kürzen quasi die Definitionslücke "beheben" kann. Man kommt dann zu der neuen Funktion f_5^*(x)=\dfrac{x(x+2)}{3(x-4)} mit \mathbb{D}=\mathbb{R}\backslash_{\{4\}}.

Übersicht:

 

14.3 Bruchgleichungen und gebrochen rationale Funktionen - Lösungen

Eine Bemerkung zur Bestimmung des Definitionsbereichs: Auch bei unkomplizierten Brüchen wie \dfrac{1}{x} (Aufgabe 1.4) oder -\dfrac{2(y-2)^2}{y} (Aufgabe 1.12) muss im Definitionsbereich berücksichtigt werden, dass der Nenner nicht 0 werden darf. Möglich sind in diesem Fall alle reellen Zahlen außer der 0, also ist \mathbb{D}=\mathbb{R}\setminus_{\{0\}}. Da der Nenner hier so einfach ist, wurde dafür in den folgenden Aufgaben keine Rechnung aufgeschrieben.

 

1. Aufgabe

1)
Bestimmung des Definitionsbereichs:
\begin{array}{rclll} x-3 &\neq& 0 &\vert& +3 \\x &\neq& 3\end{array}

Der Definitionsbereich ist also \mathbb{D} = \mathbb{R}\setminus_{\left\{3\right\}}.

Lösung der Gleichung:
\begin{array}{rclll} \dfrac{x^2+2x-30}{x-3} &=& 2 & \vert & \cdot(x-3) \cr x^2+2x-30 &=& 2(x-3) \cr x^2+2x-30 &=& 2x-6 & \vert & -2x+30 \cr x^2 &=& 24 & \vert & \pm\sqrt{} \cr\cr x_1 &=& \sqrt{24} \;\in\;\mathbb{D} \cr x_2 &=& -\sqrt{24} \;\in\;\mathbb{D} \cr\cr\mathbb{L} &=& \left\{-\sqrt{24};\sqrt{24}\right\} \end{array}

Bemerkung: Die Multiplikation mit (x-3) ist hier ohne Einschränkungen möglich, weil x=3 \not\in \mathbb{D}. Eine Multiplikation mit 0 kann also nicht passieren.


2)
Bestimmung des Definitionsbereichs:
linker Nenner:
\begin{array}{rclll}8-x^2 &\neq & 0 & \vert & +x^2 \\x^2 &\neq & 8 & \vert &\pm\sqrt{} \\x &\neq & \pm\sqrt{8}\\\\\end{array}

rechter Nenner:
\begin{array}{rclll}x^2-8 &\neq & 0 & \vert & +8 \\x^2 &\neq & 8 & \vert &\pm\sqrt{} \\x &\neq & \pm\sqrt{8}\\\\\end{array}

Der Definitionsbereich ist also \mathbb{D} = \mathbb{R}\setminus_{\left\{-\sqrt{8};\;\sqrt{8}\right\}}.

Lösung der Gleichung:
\begin{array}{rclll} \dfrac{x}{8-x^2} &=& -\dfrac{1}{x^2-8} &\vert & \cdot \left(8-x^2\right) \cr x &=& -\dfrac{-(-8+x^2)}{x^2-8} \cr x &=& \dfrac{x^2-8}{x^2-8} \cr x &=&1 \;\in\;\mathbb{D} \cr\cr\mathbb{L} &=& \left\{1\right\} \end{array}

Bemerkung: Die Multiplikation mit \left(8-x^2\right) ist hier ohne Einschränkungen möglich, weil x=-\sqrt{8} \not\in \mathbb{D} und x=\sqrt{8} \not\in\mathbb{D}. Eine Multiplikation mit 0 kann also nicht passieren.


3)
Bestimmung des Definitionsbereichs:
\begin{array}{rclll} t+2 &\neq & 0 &\vert &-2\\t &\neq & -2\\\end{array}

Der Definitionsbereich ist also \mathbb{D} = \mathbb{R}\setminus_{\left\{-2\right\}}.

Lösung der Gleichung:
\begin{array}{rclll} \mathbb{D} &=& \mathbb{R}\setminus_{\left\{-2\right\}} \cr\cr \dfrac{t^2+16t-4}{t+2}-10 &=& t &\vert & \cdot(t+2) \cr t^2+16t-4-10(t+2) &=& t(t+2) \cr t^2+16t-4-10t-20 &=& t^2+2t \cr t^2+6t-24 &=& t^2+2t &\vert & -t^2-2t+24 \cr 4t &=& 24 &\vert & :4 \cr t &=& 6 \;\in\;\mathbb{D} \cr\cr \mathbb{L} &=& \left\{6\right\} \end{array}

Bemerkung: Die Multiplikation mit (t+2) ist hier ohne Einschränkungen möglich, weil t=-2 \not\in \mathbb{D}. Eine Multiplikation mit 0 kann also nicht passieren.


4)
Definitionsbereich: \mathbb{D} = \mathbb{R}\setminus_{\left\{0\right\}}

Lösung der Gleichung:
\begin{array}{rclll}\dfrac{x+21}{2}-4+\dfrac{1}{x} &=& 7 &\vert & -7 \cr\cr\dfrac{x}{2}+\dfrac{21}{2}-11+\dfrac{1}{x} &=& 0 &\vert & \cdot x \cr\cr\dfrac{1}{2}x^2-\dfrac{1}{2}x+1 &=& 0 &\vert& \cdot 2 \cr x^2-x+2 &=& 0 &\vert& \text{p-q-Formel} \cr x_{1,2} &=& \dfrac{1}{2}\pm\sqrt{\left(-\dfrac{1}{2}\right)^2-2} \cr x_{1,2} &=& \dfrac{1}{2}\pm\sqrt{-\dfrac{7}{4}} \end{array}

Da aus negativen reellen Zahlen keine Wurzeln mit geraden Wurzelexponenten gezogen werden können, hat diese Gleichung keine Lösung: \mathbb{L} = \emptyset

Bemerkung: Die Multiplikation mit x ist hier ohne Einschränkungen möglich, weil x=0 \not\in \mathbb{D}. Eine Multiplikation mit 0 kann also nicht passieren.


5)
Bestimmung des Definitionsbereichs:
\begin{array}{crclll} & x^3-7x &\neq & 0 \\& x\left(x^2-7\right) & \neq & 0\\\text{Faktor 1:} & x &\neq & 0\\\\\text{Faktor 2:} & x^2-7 & \neq & 0 &\vert & +7\\& x^2 &\neq & 7 &\vert &\pm\sqrt{}\\& x &\neq & \pm\sqrt{7}\\\end{array}

Der Definitionsbereich ist also \mathbb{D} = \mathbb{R}\setminus_{\left\{-\sqrt{7};\;0;\;\sqrt{7}\right\}}.

Lösung der Gleichung:
\begin{array}{rclcl} \dfrac{21}{x^3-7x}-4x &=& \dfrac{6}{x^3-7x} &\vert & \cdot \left(x^3-7x\right) \cr 21-4x(x^3-7x) &=& 6 &\vert & -6 \cr -4x^4+28x^2+15 &=& 0 \cr -4\left(x^2\right)^2+28x^2+15 &=& 0 \end{array}

Bemerkung: Die Multiplikation mit \left(x^3-7x\right) ist hier ohne Einschränkungen möglich, weil x=-\sqrt{7} \not\in \mathbb{D}, x=0 \not\in\mathbb{D} und x=\sqrt{7} \not\in\mathbb{D}. Eine Multiplikation mit 0 kann also nicht passieren.

Substitution: z=x^2
\begin{array}{rclcl} -4z^2+28z+15 &=& 0 &\vert & :(-4) \cr z^2-7z-\dfrac{15}{4} &=& 0 &\vert & \text{p-q-Formel} \cr z_{1,2} &=& \dfrac{7}{2}\pm\sqrt{\left(-\dfrac{7}{2}\right)^2+\dfrac{15}{4}} \cr z_{1,2} &=& \dfrac{7}{2}\pm\sqrt{16} \cr\cr z_1 &=& \dfrac{7}{2}+4 = \dfrac{15}{2} \cr\cr z_2 &=& \dfrac{7}{2}- 4 = -\dfrac{1}{2} \end{array}

Rücksubstitution:
\begin{array}{rclcl} z_1 = x^2 &=& \dfrac{15}{2} &\vert & \pm\sqrt{} \cr x_{1,2} &=& \pm\sqrt{\dfrac{15}{2}} \approx\pm 2{,}74\;\in\;\mathbb{D} \cr\cr z_2 = x^2 &=& -\dfrac{1}{2} &\vert & \pm\sqrt{} \cr x_{3,4} &=& \pm\sqrt{-\dfrac{1}{2}} \end{array}

Da aus negativen reellen Zahlen keine Wurzeln mit geraden Wurzelexponenten gezogen werden können, liefert die Rücksubstitution von z_2 keine weiteren Lösungen: \mathbb{L} = \left\{-\sqrt{\dfrac{15}{2}};\sqrt{\dfrac{15}{2}}\right\}

Bemerkung: Am Anfang der Rechnung beim Multiplizieren mit dem Nenner die Klammern nicht vergessen! Hier treffen ja Punkt- und Strichrechnung aufeinander ...


6)
Bestimmung des Definitionsbereichs:
linker Nenner:
\begin{array}{rclll}x^2-81 &\neq & 0 &\vert & +81\\x^2 &\neq & 81 &\vert & \pm\sqrt{}\\x &\neq & \pm 9\\\\\end{array}

rechter Nenner:
\begin{array}{rclll}x-9 &\neq & 0 &\vert & +9\\x &\neq & 9\\\end{array}

Der Definitionsbereich ist also \mathbb{D} = \mathbb{R}\setminus_{\left\{-9;\;9\right\}}.

Lösung der Gleichung:
\begin{array}{rclll}\dfrac{1}{x^2-81} &=& \dfrac{x}{x-9}-1 &\vert& -\dfrac{1}{x^2-81} \cr0 &=& -\dfrac{1}{x^2-81}+\dfrac{x}{x-9}-1 \cr0 &=& -\dfrac{1}{x^2-81}+\dfrac{x\cdot\left(x+9\right)}{\left(x-9\right)\left(x+9\right)}-1 \cr0 &=& \dfrac{-1+x^2+9x}{x^2-81}-1 &\vert& \cdot\left(x^2-81\right) \cr0 &=& -1+x^2+9x-1\cdot\left(x^2-81\right) \cr 0 &=& 9x+80 &\vert & -9x \cr -9x &=& 80 &\vert & :(-9) \cr x &=& -\dfrac{80}{9} \;\in\;\mathbb{D} \cr\cr \mathbb{L} &=& \left\{-\dfrac{80}{9}\right\} \end{array}

Bemerkung: Bitte beachten Sie, dass der Hauptnenner x^2-81=\left(x-9\right)\left(x+9\right) ist.

Bemerkung: Die Multiplikation mit \left(x^2-81\right) ist hier ohne Einschränkungen möglich, weil x=-9 \not\in \mathbb{D} und x=9 \not\in\mathbb{D}. Eine Multiplikation mit 0 kann also nicht passieren.


7)
Bestimmung des Definitionsbereichs:
\begin{array}{rclll} x^3 &\neq & 0 &\vert &\sqrt[3]{}\\x &\neq & 0\\\end{array}

Der Definitionsbereich ist also \mathbb{D} = \mathbb{R}\setminus_{\left\{0\right\}}.

Lösung der Gleichung:
\begin{array}{crclll}& 6+\dfrac{5x^3+9x}{x^3} &=& -\dfrac{1}{x}+11 &\vert & +\dfrac{1}{x}-11 \cr\cr & -5+\dfrac{5x^3+9x}{x^3}+\dfrac{1}{x} &=& 0 \cr\cr & -5+\dfrac{5x^3+9x}{x^3}+\dfrac{x^2}{x^3} &=& 0 \cr\cr & -5+\dfrac{5x^3+x^2+9x}{x^3} &=& 0 &\vert & \cdot x^3 \cr\cr & -5x^3+5x^3+x^2+9x &=& 0 \cr & x^2+9x &=& 0 \cr & x(x+9) &=& 0 & \vert & \text{Satz vom Nullprodukt} \cr \text{Faktor 1:} & x_1 &=& 0 \;\not\in\;\mathbb{D} \cr\cr \text{Faktor 2:} & x_2+9 &=& 0 &\vert & -9 \cr & x_2 &=& -9 \;\in\;\mathbb{D} \cr\cr & \mathbb{L} &=& \left\{-9\right\} \end{array}

Bemerkung: Bitte beachten Sie, dass der Hauptnenner x^3=x\cdot x^2 ist.

Bemerkung: Die Multiplikation mit x^3 ist hier ohne Einschränkungen möglich, weil x=0 \not\in \mathbb{D}. Eine Multiplikation mit 0 kann also nicht passieren.


8)
Bestimmung des Definitionsbereichs:
1. linker Nenner:
\begin{array}{rclll} 27z^2+72z+48 &\neq & 0 &\vert & :27\\\\z^2+\dfrac{8}{3}z+\dfrac{16}{9} &\neq & 0 &\vert & \text{ p-q-Formel}\\\\z_{1,2} &\neq & -\dfrac{4}{3}\pm\sqrt{\left(\dfrac{4}{3}\right)^2-\dfrac{16}{9}}\\\\z_{1,2} &\neq & -\dfrac{4}{3}\pm\sqrt{0}\\\\\\\end{array}

2. linker Nenner:
\begin{array}{rclll} 6z &\neq & 0 &\vert & :6\\z &\neq & 0\\\\\\\end{array}

rechter Nenner:
\begin{array}{rclll} 3z+4 &\neq & 0 &\vert & -4\\3z &\neq & -4 &\vert & :3\\z &\neq & -\dfrac{4}{3}\end{array}

Der Definitionsbereich ist also \mathbb{D} = \mathbb{R}\setminus_{\left\{-\frac{4}{3};\;0\right\}}.

Lösung der Gleichung:
\begin{array}{rclll}\dfrac{3z+4}{27z^2+72z+48}+\dfrac{z-6}{6z} &=& \dfrac{1}{3z+4} \cr\cr \dfrac{3z+4}{3\left(9z^2+24z+16\right)}+\dfrac{z-6}{6z} &=& \dfrac{1}{3z+4} &\vert & -\dfrac{1}{3z+4} \cr\cr \dfrac{1}{3(3z+4)}+\dfrac{z-6}{6z}-\dfrac{1}{3z+4} &=& 0 \cr\cr \dfrac{2z}{6z(3z+4)}+\dfrac{(z-6)(3z+4)}{6z(3z+4)}-\dfrac{6z}{6z(3z+4)} &=& 0 \cr\cr \dfrac{2z}{6z(3z+4)}+\dfrac{3z^2+4z-18z-24}{6z(3z+4)}-\dfrac{6z}{6z(3z+4)} &=& 0 \cr\cr\dfrac{2z+3z^2-14z-24-6z}{6z(3z+4)} &=& 0 &\vert & \cdot 6z(3z+4)\cr\cr 3z^2-18z-24 &=& 0 &\vert& :3\cr z^2-6z-8 &=& 0 &\vert& \text{p-q-Formel} \cr z_{1,2} &=& 3\pm \sqrt{(-3)^2-(-8)} \cr z_{1,2} &=& 3 \pm \sqrt{17} \cr\cr z_{1,2} &=& 3+\sqrt{17} \approx 7{,}12 \;\in\;\mathbb{D} \cr z_{1,2} &=& 3-\sqrt{17} \approx -1{,}12 \;\in\;\mathbb{D} \cr\cr\mathbb{L} &=& \left\{3-\sqrt{17};3+\sqrt{17}\right\} \end{array}

Bemerkung: Bitte beachten Sie, dass der Hauptnenner 6z(3z+4) ist.

Bemerkung: Die Multiplikation mit 6z(3z+4) ist hier ohne Einschränkungen möglich, weil z=-\dfrac43 \not\in \mathbb{D} und z=0 \not\in\mathbb{D}. Eine Multiplikation mit 0 kann also nicht passieren.


9)
Bestimmung des Definitionsbereichs:
linker Nenner:
\begin{array}{rclll} 3(x-4) &\neq & 0 &\vert & :3 \\x-4 &\neq & 0 &\vert & +4 \\x & \neq & 4\\\\\end{array}

rechter Nenner:
\begin{array}{rclll}3x^2-9x-12 &\neq & 0 & \vert & :3\\x^2-3x-4 &\neq & 0 &\vert &\textrm{p-q-Formel}\\x_{1,2} &\neq & \dfrac{3}{2}\pm\sqrt{\left(\dfrac{3}{2}\right)^2+4}\\\\x_{1,2} &\neq & \dfrac{3}{2}\pm\sqrt{\dfrac{25}{4}}\\\\x_1 &\neq & \dfrac{3}{2}+\dfrac{5}{2} = 4 \\x_2 &\neq & \dfrac{3}{2}-\dfrac{5}{2} = -1\end{array}

Der Definitionsbereich ist also \mathbb{D} = \mathbb{R}\setminus_{\left\{-1;\;4\right\}}.

Lösung der Gleichung:
\begin{array}{rclll}\dfrac{x^2-8}{3(x-4)} &=& \dfrac{x^3-4x(-0{,}25x+2)-8}{3x^2-9x-12} \cr\cr \dfrac{x^2-8}{3(x-4)} &=& \dfrac{x^3+x^2-8x-8}{3(x^2-3x-4)} &\vert & \cdot 3 \cr\cr \dfrac{x^2-8}{x-4} &=& \dfrac{x^3+x^2-8x-8}{x^2-3x-4} \cr\cr \dfrac{\left(x^2-8\right)\left(x^2-3x-4\right)}{(x-4)\left(x^2-3x-4\right)} &=& \dfrac{\left(x^3+x^2-8x-8\right)(x-4)}{\left(x^2-3x-4\right)(x-4)} &\vert & \cdot\left(x^2-3x-4\right)(x-4)\cr\cr x^4-3x^3-12x^2+24x+32 &=& x^4-3x^3-12x^2+24x+32 &\vert & -x^4+3x^3+12x^2-24x-32 \cr\cr 0 &=& 0\cr\cr \mathbb{L} &=& \mathbb{R}\setminus_{\left\{-1;\;4\right\}} \end{array}

Bemerkung: Unabhängig davon, welches Element des Definitionsbereichs in diese Gleichung eingesetzt wird, erhält man immer auf beiden Seiten dasselbe Ergebnis. 0=0 ist schließlich immer richtig. Jede reelle Zahl löst also diese Gleichung, d. h. die Lösungsmenge entspricht dem Definitionsbereich.

Bemerkung: Die Multiplikation mit \left(x^2-3x-4\right)(x-4) ist hier ohne Einschränkungen möglich, weil x=-1 \not\in \mathbb{D} und x=4 \not\in\mathbb{D}. Eine Multiplikation mit 0 kann also nicht passieren.


10)
Bestimmung des Definitionsbereichs:
linker Nenner:
\begin{array}{rclll} -4x-72 &\neq & 0 &\vert & +72\\-4x &\neq & 72 &\vert & :-4\\x &\neq & -18\\\\\end{array}

rechter Nenner:
\begin{array}{rclll} 2x+3 &\neq & 0 &\vert & -3\\2x &\neq & -3 &\vert & :2\\x &\neq & -\dfrac{3}{2}\end{array}

Der Definitionsbereich ist also \mathbb{D} = \mathbb{R}\setminus_{\left\{-18;\;-\frac{3}{2}\right\}}.

Lösung der Gleichung:
\begin{array}{rclll}\dfrac{4x}{-4x-72} &=& \dfrac{9}{2x+3}&\vert & \cdot\left(-4x-72\right)\left(2x+3\right) \cr\cr4x\cdot\left(2x+3\right) &=& 9\cdot\left(-4x-72\right) \cr8x^2+12x &=& -36x-648 &\vert &+36x+648 \cr8x^2+48x+648 &=& 0 &\vert & : 8 \cr x^2+6x+81 &=& 0 &\vert& \text{p-q-Formel} \cr x_{1,2} &=& -3\pm\sqrt{3^2-81} \cr x_{1,2} &=& -3\pm\sqrt{-72} \end{array}

Da aus negativen reellen Zahlen keine Wurzeln mit geraden Wurzelexponenten gezogen werden können, hat diese Gleichung keine Lösung: \mathbb{L} = \emptyset

Bemerkung: Die Multiplikation mit (-4x-72)(2x+3) ist hier ohne Einschränkungen möglich, weil x=-18 \not\in \mathbb{D} und x= -\dfrac32 \not\in\mathbb{D}. Eine Multiplikation mit 0 kann also nicht passieren.


11)
Bestimmung des Definitionsbereichs:
1. rechter Nenner:
\begin{array}{rclll} x-2 &\neq & 0 &\vert & +2\\x &\neq & 2\\\\\end{array}

2. rechter Nenner:
\begin{array}{rclll}x+2 &\neq & 0 &\vert & -2\\x &\neq & -2\end{array}

Der Definitionsbereich ist also \mathbb{D} = \mathbb{R}\backslash_{\{-2;2\}}.

Lösung der Gleichung:
\begin{array}{rclcl}2 &=& \dfrac{10}{x-2}-\dfrac{10}{x+2} &\vert& \cdot (x+2)(x-2) \cr\cr 2(x+2)(x-2) &=& 10(x+2) - 10(x-2) \cr 2(x^2-4) &=& 10x+20-10x+20 \cr 2x^2-8 &=& 40 &\vert& +8 \cr 2x^2 &=& 48 &\vert& :2 \cr x^2 &=& 24 &\vert& \pm \sqrt{} \cr\cr x_{1,2} &=& \pm \sqrt{24} \cr&=& \pm \sqrt{4 \cdot 6} \cr &=& \pm 2\sqrt{6} \approx \pm 4{,}90 \;\in\;\mathbb{D} \cr \cr \mathbb{L} &=& \left\{-2\sqrt{6}; 2\sqrt{6}\right\} \end{array}

Bemerkung: Die Multiplikation mit (x+2)(x-2) ist hier ohne Einschränkungen möglich, weil x=-2 \not\in \mathbb{D} und x=2 \not\in\mathbb{D}. Eine Multiplikation mit 0 kann also nicht passieren.


12)
Bestimmung des Definitionsbereichs:
\begin{array}{rclll} 3y &\neq & 0 &\vert &:3\\y &\neq & 0\end{array}

Der Definitionsbereich ist also \mathbb{D} = \mathbb{R}\backslash_{\{0\}}.

Lösung der Gleichung:
\begin{array}{rclcl} -\dfrac{2(y-2)^2}{y}-2+\dfrac{(3y-8)^2}{3y} &=& \dfrac{14}{3y} &\vert& \cdot 3y \cr\cr -6(y-2)^2-6y+(3y-8)^2 &=& 14 &\vert& -14\cr -6y^2+24y-24-6y+9y^2-48y+64-14 &=& 0 \cr 3y^2-30y+26 &=& 0 &\vert& : 3 \cr y^2-10y+\dfrac{26}{3} &=& 0 &\vert& \text{p-q-Formel} \cr y_{1,2} &=& 5 \pm \sqrt{(-5)^2-\dfrac{26}{3}} \cr\cr y_{1,2} &=& 5 \pm \sqrt{\dfrac{49}{3}} \cr \cr\cr y_1 &=& 5+\dfrac{7}{\sqrt{3}} = 5+\dfrac{7\sqrt{3}}{3} \approx 9{,}04 \;\in\;\mathbb{D} \cr y_2 &=& 5-\dfrac{7}{\sqrt{3}} = 5-\dfrac{7\sqrt{3}}{3} \approx 0{,}96 \;\in\;\mathbb{D} \cr \cr \mathbb{L} &=& \left\{5+\dfrac{7\sqrt{3}}{3}; 5-\dfrac{7\sqrt{3}}{3}\right\} \end{array}

Bemerkung: Da man im Nenner ungerne Wurzeln zu stehen hat, wurden y_1 und y_2 jeweils mit \sqrt{3} erweitert. Das nennt man "den Nenner rational machen".

Bemerkung: Die Multiplikation mit 3y ist hier ohne Einschränkungen möglich, weil y=0 \not\in \mathbb{D}. Eine Multiplikation mit 0 kann also nicht passieren.


13)
Definitionsbereich: \mathbb{D} = \mathbb{R}\backslash_{\{0\}}

Lösung der Gleichung:
\begin{array}{rclcl} 6(6t+1) &=& \dfrac{9+6t}{t} &\vert& \cdot t \cr\cr 6t(6t+1) &=& 9+6t \cr 36t^2+6t &=& 9+6t &\vert& -6t \cr 36t^2 &=& 9 &\vert& :36 \cr t^2 &=& \dfrac{1}{4} &\vert& \pm \sqrt{} \cr\cr t_{1,2} &=& \pm \dfrac{1}{2} \in\mathbb{D} \cr \cr \mathbb{L} &=& \left\{-\dfrac{1}{2}; \dfrac{1}{2}\right\} \end{array}

Bemerkung: Die Multiplikation mit t ist hier ohne Einschränkungen möglich, weil t=0 \not\in \mathbb{D}. Eine Multiplikation mit 0 kann also nicht passieren.


14)
Bestimmung des Definitionsbereichs:
linker Nenner:
\begin{array}{rclll} 2x+2 &\neq & 0 &\vert & -2\\2x &\neq & -2 &\vert & :2\\x &\neq & -1\\\\\end{array}

rechter Nenner:
\begin{array}{rclll} 4x+2 &\neq & 0 &\vert &-2\\4x &\neq & -2 &\vert & :4 \\x &\neq & -\dfrac{1}{2}\end{array}

Der Definitionsbereich ist also \mathbb{D} = \mathbb{R}\backslash_{\left\{-1; -\frac{1}{2}\right\}}.

Lösung der Gleichung:
\begin{array}{rclcl}4 \cdot \dfrac{2x+1}{2x+2} &=& \dfrac{x+1}{4x+2} &\vert& \cdot (4x+2)(2x+2) \cr\cr 4(2x+1)(4x+2) &=& (x+1)(2x+2) \cr 32x^2+32x+8 &=& 2x^2+4x+2 &\vert& -2x^2-4x-2 \cr 30x^2+28x+6 &=& 0 &\vert& :30 \cr x^2+\dfrac{14}{15}x+\dfrac{1}{5} &=& 0 &\vert& \text{p-q-Formel} \cr x_{1,2} &=& -\dfrac{7}{15} \pm \sqrt{\left(\dfrac{7}{15}\right)^2-\dfrac{1}{5}} \cr\cr &=& -\dfrac{7}{15} \pm \sqrt{\dfrac{4}{225}} \cr\cr x_1 &=& -\dfrac{7}{15} + \dfrac{2}{15} = -\dfrac{1}{3} \in\mathbb{D} \cr\cr x_2 &=& -\dfrac{7}{15} - \dfrac{2}{15} = -\dfrac{3}{5} \in\mathbb{D} \cr \cr \mathbb{L} &=& \left\{-\dfrac{3}{5}; -\dfrac{1}{3} \right\} \end{array}

Bemerkung: Die Multiplikation mit (4x+2)(2x+2) ist hier ohne Einschränkungen möglich, weil x=-1 \not\in \mathbb{D} und x= -\dfrac12 \not\in\mathbb{D}. Eine Multiplikation mit 0 kann also nicht passieren.


15)
Bestimmung des Definitionsbereichs:
\begin{array}{rclll}x+1 &\neq & 0 &\vert & -1\\x &\neq & -1\end{array}

Der Definitionsbereich ist also \mathbb{D} = \mathbb{R}\backslash_{\{-1\}}.

Lösung der Gleichung:
\begin{array}{rclcl} \dfrac{x^2-2x+10}{x+1} &=& \dfrac{3x+16}{x+1} &\vert& \cdot (x+1) \cr\cr x^2-2x+10 &=& 3x+16 &\vert& -3x-16 \cr x^2-5x-6 &=& 0 &\vert& \text{p-q-Formel} \cr x_{1,2} &=& \dfrac{5}{2} \pm \sqrt{\left(-\dfrac{5}{2}\right)^2+6} \cr\cr &=& \dfrac{5}{2} \pm \sqrt{\dfrac{49}{4}} \cr\cr x_1 &=& \dfrac{5}{2} + \dfrac{7}{2} = 6 \in\mathbb{D} \cr\cr x_2 &=& \dfrac{5}{2} - \dfrac{7}{2} = -1 \;\not\in\;\mathbb{D} \cr \cr \mathbb{L} &=& \{6\} \end{array}

Bemerkung: Die Multiplikation mit (x+1) ist hier ohne Einschränkungen möglich, weil x=-1 \not\in \mathbb{D}. Eine Multiplikation mit 0 kann also nicht passieren.


16)
Bestimmung des Definitionsbereichs:
1. rechter Nenner:
\begin{array}{rclll}14+2x &\neq & 0 &\vert & -14\\2x &\neq & -14 &\vert & :2\\x &\neq & -7\\\\\end{array}

2. rechter Nenner:
\begin{array}{rclll}196-4x^2 &\neq & 0 &\vert & +4x^2\\4x^2 &\neq & 196 &\vert & :4\\x^2 &\neq & 49 &\vert &\pm\sqrt{}\\x &\neq & \pm 7 \end{array}

Der Definitionsbereich ist also \mathbb{D} = \mathbb{R}\backslash_{\{-7; 0; 7\}}.

Lösung der Gleichung:
\begin{array}{rclcl} -x\left(-x+2+\dfrac{2}{x}\right) &=& \dfrac{x}{14+2x} : \dfrac{x}{196-4x^2} \cr\cr x^2-2x-2 &=& \dfrac{x}{14+2x} \cdot \dfrac{(14+2x)(14-2x)}{x} \cr\cr x^2-2x-2 &=& 14-2x &\vert& +2x+2 \cr x^2 &=& 16 &\vert& \pm \sqrt{} \cr\cr x_{1,2} &=& \pm 4 \;\in\;\mathbb{D} \cr \cr \mathbb{L} &=& \{-4;4\} \end{array}

Bemerkung: Auf der rechten Seite: Kehrwert bilden, 3. binomische Formel und kürzen!


17)
Bestimmung des Definitionsbereichs:
\begin{array}{rclll}x+4 &\neq& 0 &\vert& -4 \\x &\neq& -4\end{array}

Der Definitionsbereich ist also \mathbb{D} = \mathbb{R}\setminus_{\left\{-4;\;0\right\}}.

Lösung der Gleichung:
\begin{array}{rclcll} \left(\dfrac{1}{x}-3\right) \cdot 2x^2 &=& \dfrac{x^2+8x+16}{x+4} \cr 2x-6x^2 &=& \dfrac{(x+4)^2}{x+4} \cr 2x-6x^2 &=& x+4 & \vert & -2x+6x^2 \cr 0 &=& 6x^2-x+4 & \vert & :6 \cr 0 &=& x^2-\dfrac{1}{6}x+\dfrac{2}{3} &\vert& \text{p-q-Formel} \cr\cr x_{1,2} &=& \dfrac{1}{12} \pm \sqrt{\left(\dfrac{1}{12}\right)^2-\dfrac{2}{3}} \cr x_{1,2} &=& \dfrac{1}{12} \pm \sqrt{-\dfrac{95}{144}} \end{array}

Da aus negativen reellen Zahlen keine Wurzeln mit geraden Wurzelexponenten gezogen werden können, hat diese Gleichung keine Lösung: \mathbb{L} = \emptyset


18)
Bestimmung des Definitionsbereichs:
\begin{array}{rclll}x+1 &\neq & 0 &\vert & -1\\x &\neq & -1\end{array}

Der Definitionsbereich ist also \mathbb{D} = \mathbb{R}\setminus_{\left\{-1;\;0\right\}}.

Lösung der Gleichung:
\begin{array}{rclcll}\dfrac{16x^2-120x}{x} &=& 8x^2 \cdot \dfrac{-x-1}{x+1} \cr\cr 16x-120 &=& 8x^2 \cdot \dfrac{-1(x+1)}{x+1} \cr\cr 16x-120 &=& -8x^2 & \vert & -16x+120 \cr 0 &=& -8x^2-16x+120 & \vert & :(-8) \cr 0 &=& x^2+2x-15 &\vert& \text{p-q-Formel} \cr x_{1,2} &=& -1 \pm \sqrt{1^2+15} \cr x_{1,2} &=& -1 \pm \sqrt{16} \cr\cr x_1 &=& -1+4 = 3 \;\in\;\mathbb{D} \cr x_2 &=& -1-4 = -5 \;\in\;\mathbb{D} \cr\cr \mathbb{L} &=& \{-5;3\} \end{array}


19)
Bestimmung des Definitionsbereichs:
\begin{array}{crclll}& 16x-36x^2 &\neq & 0 \\& x\left(16-36x\right) &\neq & 0\\\text{Faktor 1:} & x &\neq & 0\\\\\text{Faktor 2:} & 16-36x &\neq & 0 &\vert & +36x\\& 36x &\neq & 16 &\vert & :36\\& x &\neq & \dfrac{4}{9}\end{array}

Der Definitionsbereich ist also \mathbb{D} = \mathbb{R}\setminus_{\left\{0;\;\frac{4}{9}\right\}}.

Lösung der Gleichung:
\begin{array}{rclcll}\dfrac{10x^2+4x}{16x-36x^2} &=& \dfrac{3}{x} \cr\cr \dfrac{2x(5x+2)}{2x(8-18x)} &=& \dfrac{3}{x} \cr\cr \dfrac{5x+2}{-18x+8} &=& \dfrac{3}{x} &\vert& \text{Kehrwert} \cr\cr \dfrac{-18x+8}{5x+2} &=& \dfrac{x}{3} & \vert & \cdot (5x+2)\cdot 3 \cr -54x+24 &=& 5x^2+2x & \vert & +54x-24 \cr 0 &=& 5x^2+56x-24 & \vert & :5 \cr 0 &=& x^2+\dfrac{56}{5}x-\dfrac{24}{5} &\vert& \text{p-q-Formel} \cr x_{1,2} &=& -\dfrac{28}{5} \pm \sqrt{\left(\dfrac{28}{5}\right)^2+\dfrac{24}{5}} \cr x_{1,2} &=& -\dfrac{28}{5} \pm \sqrt{\dfrac{904}{25}} \cr\cr x_1 &=& -\dfrac{28+\sqrt{904}}{5} \approx 0{,}41 \;\in\;\mathbb{D} \cr x_2 &=& -\dfrac{28-\sqrt{904}}{5} \approx -11{,}61 \;\in\;\mathbb{D}\end{array}

Da wir bei der Lösung der Gleichung den Kehrwert bilden, erhalten wir einen neuen Nenner, bei dem wir noch nicht ausgeschlossen haben, dass er den Wert 0 annimmt:
\begin{array}{rclll}5x+2 &\neq & 0 & \vert & -2\\5x &\neq & -2 & \vert & :5\\x &\neq & -\dfrac{2}{5}\end{array}

Das Bilden des Kehrwertes ist also nur dann erlaubt, wenn x\neq-\dfrac{2}{5} ist. Allerdings ist dieser Wert Teil des Definitionsbereichs und kommt damit grundsätzlich als Lösung infrage. Wir müssen also separat prüfen, ob x=-\dfrac{2}{5} eine Lösung der Gleichung ist, indem wir diesen Wert in die Ausgangsgleichung einsetzen:
\begin{array}{rclll}\dfrac{10\cdot\left(-\dfrac{2}{5}\right)^2+4\cdot\left(-\dfrac{2}{5}\right)}{16\cdot\left(-\dfrac{2}{5}\right)-36\cdot\left(-\dfrac{2}{5}\right)^2} &=& \dfrac{3}{-\frac{2}{5}} \\0 &=& -\dfrac{15}{2}\end{array}

Da dies ein Widerspruch ist, ist x=-\dfrac{2}{5} keine Lösung der Gleichung. Die Lösungsmenge ist also \mathbb{L} = \left\{\dfrac{-28-\sqrt{904}}{5};\dfrac{-28+\sqrt{904}}{5}\right\}.


20)
Bestimmung des Definitionsbereichs:
linker Nenner:
\begin{array}{rclll}3x &\neq & 0 & \vert & :3\\x &\neq & 0\end{array}

rechter Nenner:
\begin{array}{rclll}27(x+2) &\neq & 0 &\vert & :27\\x+2 &\neq & 0 &\vert & -2\\x &\neq & -2\end{array}

Der Definitionsbereich ist also \mathbb{D} = \mathbb{R}\setminus_{\left\{-2;\;0\right\}}.

Lösung der Gleichung:
\begin{array}{rclcll}\dfrac{27x^3+6x^2}{3x} &=& -\dfrac{3x+6}{27(x+2)} \cr\cr \dfrac{3x(9x^2+2x)}{3x} &=& -\dfrac{3(x+2)}{27(x+2)} \cr\cr 9x^2+2x &=& -\dfrac{1}{9} & \vert & +\dfrac{1}{9} \cr 9x^2+2x+\dfrac{1}{9} &=& 0 & \vert & :9 \cr x^2+\dfrac{2}{9}x+\dfrac{1}{81} &=& 0 &\vert& \text{p-q-Formel} \cr x_{1,2} &=& -\dfrac{1}{9} \pm \sqrt{\left(\dfrac{1}{9}\right)^2-\dfrac{1}{81}} \cr\cr x_{1,2} &=& -\dfrac{1}{9} \pm\sqrt{0} \cr\cr x_{1,2} &=& -\dfrac{1}{9} \;\in\;\mathbb{D} \cr\cr \mathbb{L} &=& \left\{-\dfrac{1}{9}\right\} \end{array}

 

2. Aufgabe

1)
a)
\begin{array}{rclll} f\left(\dfrac{5}{3}\right) &=& \dfrac{2+\frac{5}{3}}{\frac{5}{3}-1} &=& \dfrac{11}{2} \quad \rightarrow \quad P_1\left(\dfrac{5}{3} \mid \dfrac{11}{2}\right)\end{array}

b)
\begin{array}{rclcll} -\dfrac{5}{2} &=& \dfrac{2+x}{x-1} & \vert & \cdot (x-1) \cr -\dfrac{5}{2}(x-1) &=& 2+x \cr -\dfrac{5}{2}x+\dfrac{5}{2} &=& 2+x & \vert & -\dfrac{5}{2}-x \cr -\dfrac{7}{2}x &=& -\dfrac{1}{2} & \vert & :\left(-\dfrac{7}{2}\right) \cr x &=& \dfrac{1}{7} \in\mathbb{D} & & \rightarrow \quad P_2\left(\dfrac{1}{7} \mid -\dfrac{5}{2}\right)\end{array}

Bemerkung: Die Multiplikation mit (x-1) ist hier ohne Einschränkungen möglich, weil x=1 \not\in \mathbb{D}. Eine Multiplikation mit 0 kann also nicht passieren.


2)
a)
\begin{array}{rclll} f(6) &=& \dfrac{1}{6^2-4} &=& \dfrac{1}{32} \quad \rightarrow \quad P_1\left(6 \mid \dfrac{1}{32}\right) \end{array}

b)
\begin{array}{rclcll} \dfrac{1}{5} &=& \dfrac{1}{x^2-4} & \vert & \cdot 5\left(x^2-4\right) \cr x^2-4 &=& 5 & \vert & +4 \cr x^2 &=& 9 &\vert& \pm\sqrt{} \cr\cr x_1 &=& -3\in\mathbb{D} & & \rightarrow \quad P_2\left(-3 \mid \dfrac{1}{5}\right) \cr x_2 &=& 3\in\mathbb{D} & & \rightarrow \quad P_3\left(3 \mid \dfrac{1}{5}\right)\end{array}

Bemerkung: Die Multiplikation mit \left(x^2-4\right) ist hier ohne Einschränkungen möglich, weil x=-2 \not\in \mathbb{D} und x=2 \not\in \mathbb{D}. Eine Multiplikation mit 0 kann also nicht passieren.


3)
a)
\begin{array}{rclll} f(-4) &=& \dfrac{5 \cdot (-4)-11}{(-4)^3-8} &=& \dfrac{31}{72} \quad \rightarrow \quad P_1\left(-4 \mid \dfrac{31}{72}\right)\end{array}

b)
\begin{array}{rclcll} 0 &=& \dfrac{5x-11}{x^3-8} & \vert & \cdot \left(x^3-8\right) \cr 0 &=& 5x-11 & \vert & +11 \cr 5x &=& 11 & \vert & :5 \cr\cr x &=& \dfrac{11}{5}\in\mathbb{D} & & \rightarrow \quad P_2\left(\dfrac{11}{5} \mid 0\right)\end{array}

Bemerkung: Die Multiplikation mit \left(x^3-8\right) ist hier ohne Einschränkungen möglich, weil x=2 \not\in \mathbb{D}. Eine Multiplikation mit 0 kann also nicht passieren.


4)
a)
\begin{array}{rclll} f(-21) &=& \dfrac{2 \cdot (-21)^2-2 \cdot (-21)}{(-21)^3+2 \cdot (-21)^2-3 \cdot (-21)} &=& -\dfrac{1}{9} \quad \rightarrow \quad P_1\left(-21 \mid -\dfrac{1}{9}\right)\end{array}

b)
\begin{array}{lrclclll} &-\dfrac{1}{8} &=& \dfrac{2x^2-2x}{x^3+2x^2-3x} & \vert & \cdot \left(x^3+2x^2-3x\right) \cr &-\dfrac{1}{8}(x^3+2x^2-3x) &=& 2x^2-2x \cr &-\dfrac{1}{8}x^3-\dfrac{1}{4}x^2+\dfrac{3}{8}x &=& 2x^2-2x & \vert& +\dfrac{1}{8}x^3+\dfrac{1}{4}x^2-\dfrac{3}{8}x \cr &0 &=& \dfrac{1}{8}x^3+\dfrac{9}{4}x^2-\dfrac{19}{8}x \cr &0 &=& x\left(\dfrac{1}{8}x^2+\dfrac{9}{4}x-\dfrac{19}{8}\right) & \vert & \textrm{Satz vom Nullpunkt} \cr\cr\text{Faktor 1:} & x_1 &=& 0\not\in\; \mathbb{D} \cr\cr \text{Faktor 2:} & \dfrac{1}{8}x^2+\dfrac{9}{4}x-\dfrac{19}{8} &=& 0 &\vert& :\dfrac{1}{8} \cr & x^2+18x-19 &=& 0 &\vert& \text{p-q-Formel} \cr &x_{2,3} &=& -9 \pm \sqrt{9^2+19} \cr &&=& -9 \pm \sqrt{100} \cr\cr &x_2 &=& -9+10 = 1\not\in\; \mathbb{D} \cr &x_3 &=& -9-10 = -19\in\mathbb{D} & & \rightarrow \quad P_2\left(-19 \mid -\dfrac{1}{8}\right)\end{array}

Bemerkung: Die Multiplikation mit \left(x^3+2x^2-3x\right) ist hier ohne Einschränkungen möglich, weil x=0 \not\in \mathbb{D} und x=1 \not\in \mathbb{D}. Eine Multiplikation mit 0 kann also nicht passieren.


5)
a)
\begin{array}{rclll} f(6) &=& \dfrac{-15+6}{6} &=& -\dfrac{3}{2} \quad \rightarrow \quad P_1\left(6 \mid -\dfrac{3}{2}\right)\end{array}

b)
\begin{array}{rclcll} 1 &=& \dfrac{-15+x}{x} & \vert & \cdot x \cr x &=& -15+x & \vert & -x \cr 0 &=& -15 \end{array}

Da dies ein Widerspruch ist, hat diese Gleichung keine Lösung. Das bedeutet, dass die Funktion nirgends den Funktionswert 1 annimmt.

Bemerkung: Die Multiplikation mit x ist hier ohne Einschränkungen möglich, weil x=0 \not\in \mathbb{D}. Eine Multiplikation mit 0 kann also nicht passieren.


6)
a)
\begin{array}{rclll} f(0) &=& \dfrac{0^2-2}{0^2+4}-2 &=& -\dfrac{5}{2} \quad \rightarrow \quad P_1\left(0 \mid -\dfrac{5}{2}\right)\end{array}

b)
\begin{array}{rclcll} -\dfrac{23}{20} &=& \dfrac{x^2-2}{x^2+4}-2 & \vert & +2 \cr\cr \dfrac{17}{20} &=& \dfrac{x^2-2}{x^2+4} &\vert& \cdot \left(x^2+4\right) \cr\cr \dfrac{17}{20}\cdot \left(x^2+4\right) &=& x^2-2 \cr\cr\dfrac{17}{20}x^2+\dfrac{17}{5} &=& x^2-2 & \vert & -x^2-\dfrac{17}{5} \cr\cr -\dfrac{3}{20}x^2 &=& -\dfrac{27}{5} &\vert& :\left(-\dfrac{3}{20}\right) \cr x^2 &=& 36 &\vert& \pm\sqrt{} \cr\cr x_1 &=& 6\in\mathbb{D} & & \rightarrow \quad P_2\left(6 \mid -\dfrac{23}{20}\right) \cr x_2 &=& -6\in\mathbb{D} & & \rightarrow \quad P_3\left(-6 \mid -\dfrac{23}{20}\right)\end{array}

Bemerkung: Die Multiplikation mit \left(x^2+4\right) ist hier ohne Einschränkungen möglich, weil x^2+4 immer größer 0 ist. Eine Multiplikation mit 0 kann also nicht passieren.


7)
a)
\begin{array}{rclll} f\left(\dfrac{1}{10}\right) &=& \dfrac{20 \cdot \left(\frac{1}{10}\right)^2}{2 \cdot \frac{1}{10}-100} &=& -\dfrac{1}{499} \quad \rightarrow \quad P_1\left(\dfrac{1}{10} \mid -\dfrac{1}{499}\right)\end{array}

b)
\begin{array}{rclcll} -\dfrac{25}{19} &=& \dfrac{20x^2}{2x-100} & \vert & \cdot (2x-100) \cr\cr -\dfrac{25}{19}(2x-100) &=& 20x^2 \cr\cr -\dfrac{50}{19}x+\dfrac{2.500}{19} &=& 20x^2 &\vert& +\dfrac{50}{19}x-\dfrac{2.500}{19} \cr\cr 0 &=& 20x^2+\dfrac{50}{19}x-\dfrac{2.500}{19} & \vert & :20 \cr\cr 0 &=& x^2+\dfrac{5}{38}x-\dfrac{125}{19} &\vert& \text{p-q-Formel} \cr\cr x_{1,2} &=& -\dfrac{5}{76} \pm \sqrt{\left(\dfrac{5}{76}\right)^2+\dfrac{125}{19}} \cr &=& -\dfrac{5}{76} \pm \sqrt{\dfrac{38.025}{5.776}} \cr\cr x_1 &=& -\dfrac{5}{76} + \dfrac{195}{76} = \dfrac{5}{2}\in\mathbb{D} & & \rightarrow \quad P_2\left(\dfrac{5}{2} \mid -\dfrac{25}{19}\right) \cr x_2 &=& -\dfrac{5}{76} - \dfrac{195}{76} = -\dfrac{50}{19}\in\mathbb{D} & & \rightarrow \quad P_3\left(-\dfrac{50}{19} \mid -\dfrac{25}{19}\right)\end{array}

Bemerkung: Die Multiplikation mit (2x-100) ist hier ohne Einschränkungen möglich, weil x=50 \not\in \mathbb{D}. Eine Multiplikation mit 0 kann also nicht passieren.


8)
a)
\begin{array}{rclll} f(-5) &=& \dfrac{-5 \cdot (-5)^3+16 \cdot (-5)^2-24 \cdot (-5)+160}{-2 \cdot (-5)^2+3 \cdot (-5)} &=& -\dfrac{261}{13} \quad \rightarrow \quad P_1\left(-5 \mid -\dfrac{261}{13}\right)\end{array}

b)
\begin{array}{rclcll} -8 &=& \dfrac{-5x^3+16x^2-24x+160}{-2x^2+3x} & \vert & \cdot \left(-2x^2+3x\right) \cr\cr -8\cdot \left(-2x^2+3x\right) &=& -5x^3+16x^2-24x+160 \cr 16x^2-24x &=& -5x^3+16x^2-24x+160 & \vert & -16x^2+24x \cr 0 &=& -5x^3+160 & \vert & +5x^3 \cr 5x^3 &=& 160 & \vert & :5 \cr x^3 &=& 32 & \vert & \sqrt[3]{} \cr x &=& \sqrt[3]{32}\approx 3{,}17\in\mathbb{D} & & \rightarrow \quad P_2\left(\sqrt[3]{32} \mid -8\right)\end{array}

Bemerkung: Die Multiplikation mit \left(-2x^2+3x\right) ist hier ohne Einschränkungen möglich, weil x=0 \not\in \mathbb{D} und x= \dfrac32 \not\in \mathbb{D}. Eine Multiplikation mit 0 kann also nicht passieren.


9)
a)
\begin{array}{rclll} f(7) &=& \dfrac{7}{(7-2)^2} &=& \dfrac{7}{25} \quad \rightarrow \quad P_1\left(7 \mid \dfrac{7}{25}\right)\end{array}

b)
\begin{array}{rclcll} \dfrac{7}{169} &=& \dfrac{x}{(x-2)^2} \cr\cr \dfrac{7}{169} &=& \dfrac{x}{x^2-4x+4} & \vert & \cdot \left(x^2-4x+4\right) \cr\cr \dfrac{7}{169}\cdot \left(x^2-4x+4\right) &=& x \cr\cr\dfrac{7}{169}x^2-\dfrac{28}{169}x+\dfrac{28}{169} &=& x & \vert & -x \cr\cr \dfrac{7}{169}x^2-\dfrac{197}{169}x+\dfrac{28}{169} &=& 0 & \vert & :\dfrac{7}{169} \cr\cr x^2-\dfrac{197}{7}x+4 &=& 0 &\vert& \text{p-q-Formel} \cr x_{1,2} &=& \dfrac{197}{14} \pm \sqrt{\left(-\dfrac{197}{14}\right)^2-4} \cr &=& \dfrac{197}{14} \pm \sqrt{\dfrac{38.025}{196}} \cr\cr x_1 &=& \dfrac{197}{14} + \dfrac{195}{14} = 28\in\mathbb{D} & & \rightarrow \quad P_2\left(28 \mid \dfrac{7}{169}\right) \cr x_2 &=& \dfrac{197}{14} - \dfrac{195}{14} = \dfrac{1}{7}\in\mathbb{D} & & \rightarrow \quad P_3\left(\dfrac{1}{7} \mid \dfrac{7}{169}\right)\end{array}

Bemerkung: Die Multiplikation mit \left(x^2-4x+4\right) ist hier ohne Einschränkungen möglich, weil x=2 \not\in \mathbb{D}. Eine Multiplikation mit 0 kann also nicht passieren.


10)
a)
\begin{array}{rclll} f\left(-14\right) &=& \dfrac{2}{-14 \cdot (-14-5)}-1 &=& -\dfrac{132}{133} \quad \rightarrow \quad P_1\left(-14 \mid -\dfrac{132}{133}\right)\end{array}

b)
\begin{array}{rclcll} -\dfrac{24}{25} &=& \dfrac{2}{x(x-5)}-1 & \vert & +1 \cr \dfrac{1}{25} &=& \dfrac{2}{x(x-5)} \cr \dfrac{1}{25} &=& \dfrac{2}{x^2-5x} & \vert & \cdot \left(x^2-5x\right) \cr \dfrac{x^2-5x}{25} &=& 2 & \vert & \cdot 25 \cr x^2-5x &=& 50 & \vert & -50 \cr x^2-5x-50 &=& 0 &\vert& \text{p-q-Formel} \cr x_{1,2} &=& \dfrac{5}{2} \pm \sqrt{\left(-\dfrac{5}{2}\right)^2+50} \cr &=& \dfrac{5}{2} \pm \sqrt{\dfrac{225}{4}} \cr\cr x_1 &=& \dfrac{5}{2}+\dfrac{15}{2} = 10\in\mathbb{D} & & \rightarrow \quad P_2\left(10 \mid -\dfrac{24}{25}\right) \cr x_2 &=& \dfrac{5}{2}-\dfrac{15}{2} = -5\in\mathbb{D} & & \rightarrow \quad P_3\left(-5 \mid -\dfrac{24}{25}\right)\end{array}

Bemerkung: Die Multiplikation mit \left(x^2-5x\right) ist hier ohne Einschränkungen möglich, weil x=0 \not\in \mathbb{D} und x=5 \not\in \mathbb{D}. Eine Multiplikation mit 0 kann also nicht passieren.

 

3. Aufgabe

1)
Bestimmung des Definitionsbereichs:
\begin{array}{rclll}x-2 &\neq & 0 & \vert & +2\\x &\neq & 2\end{array}

Der Definitionsbereich ist also \mathbb{D} = \mathbb{R}\setminus_{\left\{2\right\}}.

Berechnung der Nullstellen:
\begin{array}{rclll} \dfrac{x+3}{x-2} &=& 0 &\vert& \cdot(x-2) \cr x+3 &=& 0 & \vert& -3 \cr x &=& -3 \in\mathbb{D} \end{array}

Ergebnis: Die Nullstelle von f(x) liegt bei x=-3.

Bemerkung: Die Multiplikation mit (x-2) ist hier ohne Einschränkungen möglich, weil x=2 \not\in \mathbb{D}. Eine Multiplikation mit 0 kann also nicht passieren.


2)
Bestimmung des Definitionsbereichs:
\begin{array}{rclll} x^2-3x+8 &\neq & 0 &\vert &\text{p-q-Formel}\\x_{1,2} &\neq & \dfrac{3}{2}\pm\sqrt{\left(\dfrac{3}{2}\right)^2-8}\\x_{1,2} &\neq & \dfrac{3}{2}\pm\sqrt{-\dfrac{23}{4}}\end{array}

Da aus negativen reellen Zahlen keine Wurzeln mit geraden Wurzelexponenten gezogen werden können, ist der Definitionsbereich \mathbb{D} = \mathbb{R}.

Berechnung der Nullstellen:
\begin{array}{rclll}\dfrac{x^2-x+6}{x^2-3x+8} &=& 0 &\vert& \cdot\left(x^2-3x+8\right) \cr x^2-x+6 &=& 0 & \vert & \text{p-q-Formel} \cr x_{1,2} &=& \dfrac{1}{2}\pm\sqrt{\left(\dfrac{1}{2}\right)^2-6} \cr x_{1,2} &=& \dfrac{1}{2}\pm\sqrt{-\dfrac{23}{4}}\end{array}

Ergebnis: Da aus negativen reellen Zahlen keine Wurzeln mit geraden Wurzelexponenten gezogen werden können, hat diese Funktion keine Nullstellen.

Bemerkung: Die Multiplikation mit \left(x^2-3x+8\right) ist hier ohne Einschränkungen möglich, weil \left(x^2-3x+8\right) immer größer als 0 ist. Eine Multiplikation mit 0 kann also nicht passieren.


3)
Bestimmung des Definitionsbereichs:
\begin{array}{crclll} & x^3-10x^2+27x &\neq & 0 \\& x \left(x^2-10x+27\right) &\neq & 0\\\text{Faktor 1:} & x &\neq & 0\\\\\text{Faktor 2:} & x^2-10x+27 &\neq & 0 &\vert & \text{p-q-Formel}\\& x_{2,3} &\neq & \dfrac{10}{2}\pm\sqrt{\left(\dfrac{10}{2}\right)^2-27}\\& x_{2,3} &\neq & 5\pm\sqrt{-2}\end{array}

Da der zweite Faktor keine weiteren Nullstellen des Nenners liefert, ist der Definitionsbereich also \mathbb{D} = \mathbb{R}\setminus_{\left\{0\right\}}.

Berechnung der Nullstellen:
\begin{array}{rclll} \dfrac{2x^2-4x+4}{x^3-10x^2+27} &=& 0 & \vert & \cdot\left(x^3-10x^2+27x\right) \cr\cr 2x^2-4x+4 &=& 0 & \vert & \text{a-b-c-Formel} \cr\cr x_{1,2} &=& \dfrac{4\pm\sqrt{16-32}}{4} \cr\cr x_{1,2} &=& \dfrac{4\pm\sqrt{-16}}{4} \end{array}


Ergebnis: Da aus negativen reellen Zahlen keine Wurzeln mit geraden Wurzelexponenten gezogen werden können, hat diese Funktion keine Nullstellen.

Bemerkung: Die Multiplikation mit \left(x^3-10x^2+27x\right) ist hier ohne Einschränkungen möglich, weil x=0 \not\in \mathbb{D}. Eine Multiplikation mit 0 kann also nicht passieren.


4)
Bestimmung des Definitionsbereichs:
\begin{array}{crclll} & z^3+3z &\neq & 0 \\& z\left(z^2+3\right) &\neq & 0\\\text{Faktor 1:} & z &\neq & 0\\\\\text{Faktor 2:} & z^2+3 &\neq & 0 &\vert & -3\\& z^2 &\neq & -3 &\vert & \pm\sqrt{}\\& z &\neq & \sqrt{-3}\end{array}

Da der zweite Faktor keine weiteren Nullstellen des Nenners liefert, ist der Definitionsbereich also \mathbb{D} = \mathbb{R}\setminus_{\left\{0\right\}}.

Berechnung der Nullstellen:
\begin{array}{rclll} \dfrac{z^2-4}{z^3+3z} &=& 0 &\vert& \cdot\left(z^3+3z\right) \cr z^2-4 &=& 0 &\vert& +4 \cr z^2 &=& 4 &\vert& \pm\sqrt{} \cr z^2 &=& \pm\sqrt{4} \cr\cr z_{1} &=& 2 \in\mathbb{D} \cr z_{2} &=& -2 \in\mathbb{D} \end{array}

Ergebnis: Die Nullstellen von f(z) liegen bei z_1=2 und z_2=-2.

Bemerkung: Die Multiplikation mit \left(z^3+3z\right) ist hier ohne Einschränkungen möglich, weil z=0 \not\in \mathbb{D}. Eine Multiplikation mit 0 kann also nicht passieren.


5)
Bestimmung des Definitionsbereichs:
\begin{array}{rclll}x^4+2x^2-8 &\neq & 0\\\left(x^2\right)^2+2x^2-8 &\neq& 0\end{array}

Substitution: u=x^2
\begin{array}{rclll}u^2+2u-8 &\neq & 0 &\vert & \text{p-q-Formel}\\u_{1,2} &\neq & -1\pm\sqrt{1^2+8}\\u_{1,2} &\neq & -1\pm\sqrt{9}\\\\u_1 &\neq & -1+3 = 2\\u_2 &\neq & -1-3 = -4\end{array}

Rücksubstitution:
\begin{array}{rclll}u_1 = x^2 &\neq& 2 &\vert& \pm\sqrt{} \\x_{1,2} &\neq& \pm\sqrt{2}\\\\u_2 = x &\neq& -4 &\vert& \pm\sqrt{}\\x_{3,4} &\neq& \pm\sqrt{-4}\end{array}

Da aus negativen reellen Zahlen keine Wurzeln mit geraden Wurzelexponenten gezogen werden können, liefert die Rücksubstitution von u_2 keine weiteren Nullstellen des Nenners. Der Definitionsbereich ist also \mathbb{D} = \mathbb{R}\setminus_{\left\{-\sqrt{2};\sqrt{2}\right\}}.

Berechnung der Nullstellen:
\begin{array}{rclll}\dfrac{(x^2+x-2)^2}{x^4+2x^2-8} &=& 0 &\vert& \cdot \left(x^4+2x^2-8\right)\cr (x^2+x-2)^2 &=& 0 &\vert& \pm\sqrt{} \cr x^2+x-2 &=& 0 &\vert& \text{a-b-c-Formel} \cr x_{1,2} &=& \dfrac{-1\pm\sqrt{1+8}}{2} \cr\cr x_{1} &=& 1 \in\mathbb{D} \cr x_{2} &=& -2 \in\mathbb{D} \end{array}

Ergebnis: Die Nullstellen von f(x) liegen bei x_1=1 und x_2=-2.

Bemerkung: Die Multiplikation mit \left(x^4+2x^2-8\right) ist hier ohne Einschränkungen möglich, weil x=-\sqrt{2} \not\in \mathbb{D} und x=\sqrt{2} \not\in \mathbb{D}. Eine Multiplikation mit 0 kann also nicht passieren.


6)
Bestimmung des Definitionsbereichs:
\begin{array}{rclll} x^2-2x-15 &\neq & 0 &\vert &\text{p-q-Formel}\\x_{1,2} &\neq & 1\pm\sqrt{1^2+15}\\x_{1,2} &\neq & 1\pm\sqrt{16}\\\\x_1 &\neq & 1+4 = 5\\x_2 &\neq & 1-4 = -3\end{array}

Der Definitionsbereich ist also \mathbb{D} = \mathbb{R}\setminus_{\{-3;5\}}.

Berechnung der Nullstellen:
\begin{array}{rclll}\dfrac{x^2-x-12}{x^2-2x-15} &=& 0 &\vert& \cdot\left(x^2-2x-15\right) \cr x^2-x-12 &=& 0 &\vert& \text{p-q-Formel} \cr x_{1,2} &=& \dfrac{1}{2}\pm\sqrt{\left(\dfrac{1}{2}\right)^2+12} \cr x_{1,2} &=& \dfrac{1}{2}\pm\sqrt{\dfrac{49}{4}} \cr\cr x_1 &=& \dfrac{1}{2}+\dfrac{7}{2} = 4 \in\mathbb{D} \cr x_2 &=& \dfrac{1}{2}-\dfrac{7}{2} = -3 \not\in\mathbb{D} \end{array}

Ergebnis: Die Nullstelle von f(x) liegt bei x=4.

Bemerkung: Die Multiplikation mit \left(x^2-2x-15\right) ist hier ohne Einschränkungen möglich, weil x=-3 \not\in \mathbb{D} und x=5 \not\in \mathbb{D}. Eine Multiplikation mit 0 kann also nicht passieren.


7)
Bestimmung des Definitionsbereichs:
\begin{array}{rclll} x+4 &\neq & 0 &\vert & -4\\x &\neq & -4\end{array}

Der Definitionsbereich ist also \mathbb{D} = \mathbb{R}\setminus_{\{-4\}}.

Berechnung der Nullstellen:
\begin{array}{rclll}\dfrac{x^2+8x+16}{x+4} &=& 0 \cr\dfrac{(x+4)^2}{x+4} &=& 0 \cr x+4 &=& 0 &\vert& -4 \cr x &=& -4 \not\in\mathbb{D} \end{array}

Ergebnis: Da x=-4 nicht im Definitionsbereich liegt, hat diese Funktion keine Nullstellen.


8)
Bestimmung des Definitionsbereichs:
\begin{array}{rclll} x^4+1 &\neq & 0 &\vert & -1\\x^4 &\neq & -1 &\vert & \pm\sqrt[4]{}\\x &\neq & \pm\sqrt[4]{-1}\end{array}

Da aus negativen reellen Zahlen keine Wurzeln mit geraden Wurzelexponenten gezogen werden können, ist der Definitionsbereich \mathbb{D} = \mathbb{R}.

Berechnung der Nullstellen:
\begin{array}{rclll}\dfrac{x+7-x^2}{x^4+1} &=& 0 &\vert& \cdot\left(x^4+1\right) \cr -x^2+x+7 &=& 0 &\vert& \text{a-b-c-Formel} \cr x_{1,2} &=& \dfrac{-1\pm\sqrt{1+28}}{-2} \cr\cr x_{1} &=& \dfrac{1+\sqrt{29}}{2} \approx 3{,}19 \in\mathbb{D} \cr x_{2} &=& \dfrac{1-\sqrt{29}}{2} \approx -2{,}19 \in\mathbb{D} \end{array}

Ergebnis: Die Nullstellen von f(x) liegen bei x_1=\dfrac{1+\sqrt{29}}{2} und x_2=\dfrac{1-\sqrt{29}}{2}.

Bemerkung: Die Multiplikation mit \left(x^4+1\right) ist hier ohne Einschränkungen möglich, weil \left(x^4+1\right) immer größer 0 ist. Eine Multiplikation mit 0 kann also nicht passieren.


9)
Bestimmung des Definitionsbereichs:
\begin{array}{rclll} \left(x^2-3\right)^4 &\neq & 0 &\vert & \pm\sqrt[4]{}\\x^2-3 &\neq & 0 &\vert & +3\\x^2 &\neq & 3 &\vert & \pm\sqrt{}\\x_{1,2} &\neq & \pm\sqrt{3}\end{array}

Der Definitionsbereich ist also \mathbb{D} = \mathbb{R}\setminus_{\left\{-\sqrt{3};\sqrt{3}\right\}}.

Berechnung der Nullstellen:
\begin{array}{rclll}\dfrac{2x+6}{\left(x^2-3\right)^4} &=& 0 &\vert& \cdot \left(x^2-3\right)^4 \cr 2x+6 &=& 0 &\vert& -6 \cr 2x &=& -6 &\vert& :2 \cr x &=& -3 \in\mathbb{D} \end{array}

Ergebnis: Die Nullstelle von f(x) liegt bei x=-3.

Bemerkung: Die Multiplikation mit \left(x^2-3\right)^4 ist hier ohne Einschränkungen möglich, weil x=-\sqrt{3} \not\in \mathbb{D} und x= \sqrt{3} \not\in \mathbb{D}. Eine Multiplikation mit 0 kann also nicht passieren.


10)
Bestimmung des Definitionsbereichs:
\begin{array}{rclll} \left(x-1\right)^2 &\neq & 0 &\vert & \pm\sqrt{}\\x-1 &\neq & 0 &\vert & +1\\x &\neq & 1\end{array}

Der Definitionsbereich ist also \mathbb{D} = \mathbb{R}\setminus_{\{1\}}.

Berechnung der Nullstellen:
\begin{array}{crclcl}& \mathbb{D} &=& \mathbb{R}\setminus_{\{1\}} \\ \\& \dfrac{2x^3-5x^2+4x}{(x-1)^2} &=& 0 &\vert& \cdot (x-1)^2 \cr & 2x^3-5x^2+4x &=& 0 \cr & x\left(2x^2-5x+4\right) &=& 0 &\vert& \text{Satz vom Nullprodukt} \cr \text{Faktor 1:} & x_{1} &=& 0 \in\mathbb{D} \cr\cr \text{Faktor 2:} & 2x^2-5x+4 &=& 0 &\vert& \text{a-b-c-Formel} \cr & x_{2,3} &=& \dfrac{5\pm\sqrt{25-32}}{4} \cr& x_{2,3} &=& \dfrac{5\pm\sqrt{-7}}{4} \end{array}

Ergebnis: Da aus negativen reellen Zahlen keine Wurzeln mit geraden Wurzelexponenten gezogen werden können, liefert der zweite Faktor keine weiteren Nullstellen. Die einzige Nullstelle von f(x) liegt bei x=0.

Bemerkung: Die Multiplikation mit (x-1)^2 ist hier ohne Einschränkungen möglich, weil x=1 \not\in \mathbb{D}. Eine Multiplikation mit 0 kann also nicht passieren.


11)
Definitionsbereich: \mathbb{D} = \mathbb{R}\setminus_{\{0\}}

Berechnung der Nullstellen:
\begin{array}{rclll}\dfrac{s^2+2}{s} &=& 0 &\vert& \cdot s \cr s^2+2 &=& 0 &\vert& -2 \cr s^2 &=& -2 &\vert& \pm\sqrt{} \cr\cr s_{1,2} &=& \pm\sqrt{-2} \end{array}

Ergebnis: Da aus negativen reellen Zahlen keine Wurzeln mit geraden Wurzelexponenten gezogen werden können, hat diese Funktion keine Nullstellen.

Bemerkung: Die Multiplikation mit s ist hier ohne Einschränkungen möglich, weil s=0 \not\in \mathbb{D}. Eine Multiplikation mit 0 kann also nicht passieren.


12)
Bestimmung des Definitionsbereichs:
\begin{array}{rclll} x^4-5 &\neq & 0 &\vert & +5\\x^4 &\neq & 5 &\vert &\pm\sqrt[4]{}\\x &\neq & \pm\sqrt[4]{5}\end{array}

Der Definitionsbereich ist also \mathbb{D} = \mathbb{R}\setminus_{\left\{-\sqrt[4]{5};\sqrt[4]{5}\right\}}.

Berechnung der Nullstellen:
\begin{array}{rclll} \dfrac{3x+4}{x^4 -5} &=& 0 & \vert & \cdot \left(x^4-5\right) \cr\cr 3x+4 &=& 0 & \vert & -4 \cr\cr 3x &=& -4 & \vert & :3 \cr\cr x &=& -\dfrac43 \in \mathbb{D} \cr\cr \end{array}

Ergebnis: Die Nullstelle von f(x) liegt bei x=-\dfrac{4}{3}.

Bemerkung: Die Multiplikation mit \left(x^4-5\right) ist hier ohne Einschränkungen möglich, weil x=-\sqrt[4]{5} \not\in \mathbb{D} und x= \sqrt[4]{5}. Eine Multiplikation mit 0 kann also nicht passieren.


13)
Bestimmung des Definitionsbereichs:
\begin{array}{rclll} p+5 &\neq& 0 &\vert& -5 \\p &\neq& -5\end{array}

Der Definitionsbereich ist also \mathbb{D} = \mathbb{R}\setminus_{\left\{-5\right\}}.

Berechnung der Nullstellen:
\begin{array}{rclll} \dfrac{2}{p+5}+3 &=& 0 &\vert & \cdot \left(p+5\right)\\2+3\left(p+5\right) &=& 0 \\2+3p+15 &=& 0 \\3p+17 &=& 0 &\vert & -17\\3p &=& -17 &\vert & :3\\p &=& -\dfrac{17}{3}\;\in\;\mathbb{D}\end{array}

Ergebnis: Die Nullstelle von f(p) liegt bei p=-\dfrac{17}{3}.

Bemerkung: Die Multiplikation mit (p+5) ist hier ohne Einschränkungen möglich, weil p=-5 \not\in \mathbb{D}. Eine Multiplikation mit 0 kann also nicht passieren.


14)
Bestimmung des Definitionsbereichs:
\begin{array}{crclll} & x^7+x^3 &\neq & 0 \\& x^3\left(x^4+1\right) &\neq & 0\\\text{Faktor 1:} & x^3 &\neq & 0 &\vert& \sqrt[3]{}\\& x &\neq & 0\\\\\text{Faktor 2:} & x^4+1 &\neq & 0 &\vert & -1 \\& x^4 &\neq & -1 &\vert& \pm\sqrt[4]{}\\& x &\neq & \pm\sqrt[4]{-1}\end{array}

Da der zweite Faktor keine weiteren Nullstellen des Nenners liefert, ist der Definitionsbereich also \mathbb{D} = \mathbb{R}\setminus_{\left\{0\right\}}.

Berechnung der Nullstellen:
\begin{array}{crclll} & \dfrac{\frac{3}{2}x^3+\frac{4}{3}x^2+\frac{5}{4}x}{x^7+x^3} &=& 0 &\vert & \cdot \left(x^7+x^3\right)\\\\& \dfrac{3}{2}x^3+\dfrac{4}{3}x^2+\dfrac{5}{4}x &=& 0 \\\\& x\left(\dfrac{3}{2}x^2+\dfrac{4}{3}x+\dfrac{5}{4}\right) &=& 0 &\vert & \text{Satz vom Nullpunkt}\\\text{Faktor 1:} & x &=& 0 \;\not\in\;\mathbb{D}\\\\\text{Faktor 2:} & \dfrac{3}{2}x^2+\dfrac{4}{3}x+\dfrac{5}{4} &=& 0 &\vert & :\dfrac{3}{2}\\\\& x^2+\dfrac{8}{9}x+\dfrac{10}{12} &=& 0 &\vert & \text{p-q-Formel}\\\\& x_{1,2} &=& -\dfrac{4}{9}\pm\sqrt{\left(\dfrac{4}{9}\right)^2-\dfrac{5}{6}}\\\\& x_{1,2} &=& -\dfrac{4}{9}\pm\sqrt{-\dfrac{309}{486}}\end{array}

Ergebnis: Da aus negativen reellen Zahlen keine Wurzeln mit geraden Wurzelexponenten gezogen werden können, hat diese Funktion keine Nullstellen.

Bemerkung: Die Multiplikation mit \left(x^7 +x^3\right) ist hier ohne Einschränkungen möglich, weil x=0 \not\in \mathbb{D}. Eine Multiplikation mit 0 kann also nicht passieren.


15)
Bestimmung des Definitionsbereichs:
\begin{array}{rclll} 4x-1 &\neq & 0 &\vert & +1\\4x &\neq & 1 &\vert & :4\\x &\neq & \dfrac{1}{4}\end{array}

Der Definitionsbereich ist also \mathbb{D} = \mathbb{R}\setminus_{\left\{\frac{1}{4}\right\}}.

Berechnung der Nullstellen:
\begin{array}{rclll}\dfrac{4x^2+2}{4x-1}+1 &=& 0 &\vert &\cdot \left(4x-1\right)\\4x^2+2+1\cdot(4x-1) &=& 0\\4x^2+4x+1 &=& 0 &\vert & :4\\x^2+x+\dfrac{1}{4} &=& 0 &\vert &\text{1. binomische Formel} \\\left(x+\dfrac{1}{2}\right)^2 &=& 0 &\vert &\sqrt{}\\x+\dfrac{1}{2} &=& 0 &\vert & -\dfrac{1}{2}\\x &=& -\dfrac{1}{2}\;\in\;\mathbb{D}\end{array}

Ergebnis: Die Nullstelle von f(x) liegt bei x=-\dfrac{1}{2}.

Bemerkung: Die Multiplikation mit (4x-1) ist hier ohne Einschränkungen möglich, weil x=\dfrac14 \not\in \mathbb{D}. Eine Multiplikation mit 0 kann also nicht passieren.


16)
Bestimmung des Definitionsbereichs:
\begin{array}{rclll} y^2 &\neq & 0 &\vert& \pm\sqrt{}\\y &\neq & 0\end{array}

Der Definitionsbereich ist also \mathbb{D} = \mathbb{R}\setminus_{\left\{0\right\}}.

Berechnung der Nullstellen:
\begin{array}{rclll} \dfrac{y^2-5}{2}-\dfrac{2}{y^2}+3 &=& 0 &\vert &\cdot 2y^2\\\\\dfrac{y^2-5}{2}\cdot 2y^2-\dfrac{2}{y^2}\cdot 2y^2+3\cdot 2y^2 &=& 0\\\\\left(y^2-5\right)\cdot y^2-4+6y^2 &=& 0\\y^4-5y^2-4+6y^2 &=& 0\\y^4+y^2-4 &=& 0 \\\left(y^2\right)^2+y^2-4 &=& 0\end{array}

Substitution: z=y^2
\begin{array}{rclll} z^2+z-4 &=& 0 &\vert& \text{p-q-Formel}\\z_{1,2} &=& -\dfrac{1}{2}\pm\sqrt{\left(\dfrac{1}{2}\right)^2+4}\\z_{1,2} &=& -\dfrac{1}{2}\pm\sqrt{\dfrac{17}{4}}\\z_1 &=& \dfrac{-1+\sqrt{17}}{2} \approx 1{,}56\\z_2 &=& \dfrac{-1-\sqrt{17}}{2} \approx -2{,}56\\\end{array}

Rücksubstitution:
\begin{array}{rclcl}z_1 = y^2 &=& \dfrac{-1+\sqrt{17}}{2} &\vert& \pm\sqrt{} \\y_{1,2} &=&\pm\sqrt{\dfrac{-1+\sqrt{17}}{2}} \approx\pm 1{,}25 \;\in\;\mathbb{D}\\\\z_2 = y^2 &=& \dfrac{-1-\sqrt{17}}{2} &\vert& \pm\sqrt{} \\y_{3,4} &=& \pm\sqrt{\dfrac{-1-\sqrt{17}}{2}}\end{array}

Ergebnis: Da aus negativen reellen Zahlen keine Wurzeln mit geraden Wurzelexponenten gezogen werden können, liefert die Rücksubstitution von z_2 keine weiteren Nullstellen. Die Nullstellen von f(y) liegen bei  y_1=-\sqrt{\dfrac{-1+\sqrt{17}}{2}} und y_2=\sqrt{\dfrac{-1+\sqrt{17}}{2}}.

Bemerkung: Die Multiplikation mit 2y^2 ist hier ohne Einschränkungen möglich, weil y=0 \not\in \mathbb{D}. Eine Multiplikation mit 0 kann also nicht passieren.


17)
Definitionsbereich: \mathbb{D} = \mathbb{R}\setminus_{\left\{0\right\}}

Berechnung der Nullstellen:
\begin{array}{crclll} & \dfrac{x^5-x^2}{x}+\dfrac{x^3-4x^4}{4} &=& 0 \\& \dfrac{x\left(x^4-x\right)}{x}+\dfrac{4\left(\frac{1}{4}x^3-x^4\right)}{4} &=& 0 \\& x^4-x+\dfrac{1}{4}x^3-x^4 &=& 0 \\& \dfrac{1}{4}x^3-x &=& 0 \\& x\left(\dfrac{1}{4}x^2-1\right) &=& 0 &\vert & \text{Satz vom Nullprodukt}\\\text{Faktor 1:} & x &=& 0 \;\not\in\;\mathbb{D}\\\\\text{Faktor 2:} & \dfrac{1}{4}x^2-1 &=& 0 &\vert & +1\\& \dfrac{1}{4}x^2 &=& 1 &\vert & :\dfrac{1}{4}\\& x^2 &=& 4 &\vert & \pm\sqrt{}\\& x &=& \pm\sqrt{4}\\\\& x_1 &=& 2\;\in\;\mathbb{D}\\& x_2 &=& -2\;\in\;\mathbb{D}\end{array}

Ergebnis: Die Nullstellen von g(x) liegen bei x_1=2 und bei x_2=-2


18)
Bestimmung des Definitionsbereichs:
1. linker Nenner:
\begin{array}{rclll} 4x^3 &\neq & 0 &\vert & :4\\x^3 &\neq & 0 &\vert & \sqrt[3]{}\\x &\neq & 0 \\\\\end{array}

2. linker Nenner:
\begin{array}{rclll} x^2 &\neq & 0 &\vert& \pm\sqrt{}\\x &\neq & 0\end{array}

Der Definitionsbereich ist also \mathbb{D} = \mathbb{R}\setminus_{\left\{0\right\}}.

Berechnung der Nullstellen:
\begin{array}{rclll} \dfrac{4x}{4x^3}-\left(\dfrac{1}{x^2}-4\right)+\dfrac{x^6-16}{2} &=& 0 \\\\\dfrac{1}{x^2}-\dfrac{1}{x^2}+4+\dfrac{x^6-16}{2} &=& 0 \\\\4+\dfrac{x^6-16}{2} &=& 0 &\vert & -4\\\\\dfrac{x^6-16}{2} &=& -4 &\vert & \cdot 2\\\\x^6-16 &=& -8 &\vert & +16\\x^6 &=& 8 &\vert & \pm\sqrt[6]{}\\x &=& \pm\sqrt[6]{8} \\x &=& \pm\left(2^3\right)^{\frac{1}{6}}\\x &=& \pm\left(2^3\right)^{\frac{1}{2}\cdot\frac{1}{3}}\\x &=& \pm2^{3\cdot\frac{1}{2}\cdot\frac{1}{3}}\\x &=& \pm2^{\frac{1}{2}}\\x &=& \pm\sqrt{2}\\\\x_1 &=& \sqrt{2}\;\in\;\mathbb{D}\\x_2 &=& -\sqrt{2}\;\in\;\mathbb{D}\end{array}

Ergebnis: Die Nullstellen von f(x) liegen bei x_1=\sqrt{2} und bei x_2=-\sqrt{2}.


19)
Bestimmung des Definitionsbereichs:
\begin{array}{rclll} x^2+2 &\neq & 0 &\vert & -2\\x^2 &\neq & -2 &\vert & \pm\sqrt{}\\x &\neq & \pm\sqrt{-2} \end{array}

Da aus negativen reellen Zahlen keine Wurzeln mit geraden Wurzelexponenten gezogen werden können, ist der Definitionsbereich also \mathbb{D} = \mathbb{R}.

Berechnung der Nullstellen:
\begin{array}{crclll} & \dfrac{x^3+5x}{x^2+2}+3 &=& 0 &\vert & \cdot \left(x^2+2\right)\\\\& x^3+5x+3\left(x^2+2\right) &=& 0 \\& x^3+3x^2+5x+6 &=& 0 \\& x^3+2x^2 \;+\; x^2+5x+6 &=& 0 \\& x^2(x+2) \;+\; (x+2)(x+3) &=& 0 \\& (x+2)\left(x^2+x+3\right) &=& 0 &\vert & \text{Satz vom Nullprodukt}\\\text{Faktor 1:} & x+2 &=& 0 &\vert& -2\\& x &=& -2 \;\in\;\mathbb{D}\\\\\text{Faktor 2:} & x^2+x+3 &=& 0 &\vert & \text{p-q-Formel}\\& x^2 &=& -\dfrac{1}{2} \pm\sqrt{\left(\dfrac{1}{2}\right)^2-3}\\\end{array}

Ergebnis: Da aus negativen reellen Zahlen keine Wurzeln mit geraden Wurzelexponenten gezogen werden können, liefert der zweite Faktor keine weiteren Nullstellen. Die einzige Nullstellen von f(x) liegt bei  x=-2.

Bemerkung: Die Multiplikation mit \left(x^2+2\right) ist hier ohne Einschränkungen möglich, weil \left(x^2+2\right) immer größer 0 ist. Eine Multiplikation mit 0 kann also nicht passieren.


20)
Bestimmung des Definitionsbereichs:
\begin{array}{crclll} & b^3+5b &\neq & 0 \\& b\left(b^2+5\right) &\neq & 0 &\vert & \text{Satz vom Nullprodukt}\\\text{Faktor 1:} & b &\neq & 0 \\\\\text{Faktor 2:} & b^2+5 &\neq & 0 &\vert & -5\\& b^2 &\neq & -5 &\vert & \pm\sqrt{}\\& b &\neq & \pm\sqrt{-5}\end{array}

Da der zweite Faktor keine weiteren Nullstellen des Nenners liefert, ist der Definitionsbereich also \mathbb{D} = \mathbb{R}\setminus_{\left\{0\right\}}.

Berechnung der Nullstellen:
\begin{array}{rclll} \dfrac{1}{b^3+5b}+2b &=& 0 &\vert & \cdot \left(b^3+5b\right)\\\\1+2b\left(b^3+5b\right) &=& 0\\2\left(b^2\right)^2+10b^2+1 &=& 0 \\\end{array}

Substitution: z=b^2
\begin{array}{rclll} 2z^2+10z+1 &=& 0 & \vert & :2\\z^2+5z+\dfrac{1}{2} &=& 0 &\vert & \text{p-q-Formel}\\z_{1,2} &=& -\dfrac{5}{2}\pm\sqrt{\left(\dfrac{5}{2}\right)^2-\dfrac{1}{2}}\\z_{1,2} &=& -\dfrac{5}{2}\pm\sqrt{\dfrac{23}{4}}\\\\z_1 &=& -\dfrac{5}{2}+\dfrac{\sqrt{23}}{2} \approx -0{,}10\\z_2 &=& -\dfrac{5}{2}-\dfrac{\sqrt{23}}{2} \approx -4{,}90\\\end{array}

Rücksubstitution:
\begin{array}{rclll} z_1 = b^2 &=& -\dfrac{5}{2}+\dfrac{\sqrt{23}}{2} &\vert& \pm\sqrt{} \\b_{1,2} &=& \pm\sqrt{-\dfrac{5}{2}+\dfrac{\sqrt{23}}{2}}\\\\z_2 = b^2 &=& -\dfrac{5}{2}-\dfrac{\sqrt{23}}{2} &\vert& \pm\sqrt{} \\b_{3,4} &=& \pm\sqrt{-\dfrac{5}{2}-\dfrac{\sqrt{23}}{2}}\end{array}

Ergebnis: Da aus negativen reellen Zahlen keine Wurzeln mit geraden Wurzelexponenten gezogen werden können, hat diese Funktion keine Nullstellen.

Bemerkung: Die Multiplikation mit \left(b^3+5b\right) ist hier ohne Einschränkungen möglich, weil b=0 \not\in \mathbb{D}. Eine Multiplikation mit 0 kann also nicht passieren.

15. Wurzelgleichungen und -funktionen - Lernziele und typische Fehler

Nach Durcharbeiten dieses Kapitels sollten Sie folgende Lernziele erreicht haben:

  • Sie können zu einer Wurzelgleichung den passenden Definitionsbereich bestimmen.
  • Sie kennen die allgemeine Form einer Wurzelgleichung.
  • Sie können Wurzelgleichungen lösen.
  • Sie können die Lösungsmenge mathematisch korrekt notieren.
  • Sie können mithilfe der Probe überprüfen, ob die gefundene Lösung tatsächlich richtig ist.
  • Sie wissen, dass bei Wurzelgleichungen Scheinlösungen auftreten können, wie man diese erkennt und wie man damit umgeht.
  • Sie können zu einer Wurzelfunktion den passenden Definitionsbereich bestimmen.
  • Sie wissen, wie der Graph einer Wurzelfunktion typischerweise aussieht, und können ihn in ein kartesisches Koordinatensystem zeichnen.
  • Sie kennen die allgemeine Funktionsgleichung einer Wurzelfunktion.
  • Sie kennen Eigenschaften von Wurzelfunktionen (Anzahl der Nullstellen, Randverhalten, typische Punkte) und können diese nutzen, um Beziehungen zwischen Funktionsterm und Funktionsgraphen herzustellen.
  • Sie kennen den Zusammenhang zwischen Wurzelgleichungen und Wurzelfunktionen.
  • Sie können Wurzelgleichungen von anderen Gleichungsarten unterscheiden.
  • Sie können Wurzelfunktionen von anderen Funktionstypen unterscheiden (grafisch und anhand der Funktionsgleichung).


Typische Fehler in diesem Kapitel sind:

  • Beim Potenzieren der Gleichung werden nicht alle Bestandteile der Gleichung potenziert. Erklärung
  • Der Wurzelterm wird nicht auf einer Seite der Gleichung isoliert. Erklärung
  • Beim Quadrieren einer Gleichung, die eine Summe von (Wurzel-)Termen auf einer Seite enthält, wird keine binomische Formel angewendet. Erklärung
  • Es wird nicht geprüft, ob ein berechneter Wert eine Scheinlösung ist. Erklärung


Für Online-Selbsttests zu diesem Thema und weitere Informationen zur Mathematikunterstützung an der TH Wildau nutzen Sie bitte den Moodle-Kursraum "SOS Mathematik - Brückenkurs".

Übersicht:

 

15.1 Wurzelgleichungen und -funktionen - Aufgaben

1. Aufgabe

Lösen Sie folgende Wurzelgleichungen! Geben Sie jeweils den Definitionsbereich an!

1) \sqrt{x^2+13} = 7

  11) x-5\sqrt{x}+6 = 0

2) 5\sqrt{-1+x}+80 = 5x+15

  12) \sqrt{23y^3+4y^2+7y} = 0

3) \sqrt{4x-17}\sqrt{17+4x} = \sqrt{16x^2-289}

  13) \sqrt[3]{-5+x} = \sqrt[6]{-15x+49}

4) 5 = \sqrt{3+k}-\sqrt{1-k}

  14) \sqrt{p^2+1+3\sqrt{p-\dfrac{1}{2}}} = p+1

5) \sqrt{x^4-x^2+9} = 3

  15) \sqrt{x}\sqrt{x+1}-1 = -\sqrt{x}\sqrt{x+2}

6) y\sqrt[3]{y^2+4} = 0

  16) 2+\sqrt[3]{x^3-4x^2+8x-14} = x

7) x+\sqrt[3]{6x^2+5x} = 0

  17) \sqrt[3]{z}+10 = \sqrt[3]{z+1.000}

8) \sqrt[4]{144x^2-192x+64} = \sqrt{-12x+8}

  18) \dfrac{\sqrt{4k+3}+1}{2-\sqrt{k+2}} = 3

9) \dfrac{\sqrt{10t-35}}{8} = -\dfrac{101}{\sqrt{11-t}}   19) \sqrt{t+\sqrt{2t+5}} = \sqrt{3t+1}

10) 22+\sqrt{x^4+2x^2} = 23

  20) 9x^2-42x+49 = 0

 

2. Aufgabe

1)
Gegeben sei die Funktion f(x)= \sqrt{14x+35} mit \mathbb{D} = \left\{ x\in\mathbb{R}\mid x \ge -\dfrac{5}{2} \right\} .
Gesucht ist jeweils die fehlende Koordinate des Punktes P(x\mid y),
a) wenn x=0
b) wenn y=0

  6)
Gegeben sei die Funktion f(x)= \dfrac{1}{32}\sqrt[3]{x^3-1} mit \mathbb{D} = \mathbb{R}.
Gesucht ist jeweils die fehlende Koordinate des Punktes P(x\mid y),
a) wenn x=1
b) wenn y=\dfrac{5}{16}

2)
Gegeben sei die Funktion f(x)= -\sqrt{x^3+28}+7 mit \mathbb{D} = \left\{ x\in\mathbb{R}\mid x \ge \sqrt[3]{-28} \right\} .
Gesucht ist jeweils die fehlende Koordinate des Punktes P(x\mid y),
a) wenn x=9
b) wenn y=8

  7)
Gegeben sei die Funktion f(x)= \dfrac{\sqrt{2x+14}}{6}-14 mit \mathbb{D} = \left\{ x\in\mathbb{R}\mid x \ge -7 \right\} .
Gesucht ist jeweils die fehlende Koordinate des Punktes P(x\mid y),
a) wenn x=11
b) wenn y=-1

3)
Gegeben sei die Funktion f(x)= 5-\sqrt[3]{x+10} mit \mathbb{D} = \mathbb{R}.
Gesucht ist jeweils die fehlende Koordinate des Punktes P(x\mid y),
a) wenn x=1.321
b) wenn y=515

  8)
Gegeben sei die Funktion f(x)= -\sqrt[4]{16-x^4}+17 mit \mathbb{D} = \left\{ x\in\mathbb{R}\mid -2 \le x \le 2 \right\}.
Gesucht ist jeweils die fehlende Koordinate des Punktes P(x\mid y),
a) wenn x=-2
b) wenn y=25

4)
Gegeben sei die Funktion f(x)= \sqrt[4]{x^2+4x}-1 mit \mathbb{D} = \left\{ x\in\mathbb{R}\mid x \le -4 \text{ oder } x \ge 0 \right\} .
Gesucht ist jeweils die fehlende Koordinate des Punktes P(x\mid y),
a) wenn x=-33
b) wenn y=1

  9)
Gegeben sei die Funktion f(x)= \dfrac{\sqrt{x^2-9}}{4}-3 mit \mathbb{D} = \left\{ x\in\mathbb{R}\mid x \le -3 \text{ oder } x \ge 3 \right\}.
Gesucht ist jeweils die fehlende Koordinate des Punktes P(x\mid y),
a) wenn x=-11
b) wenn y=27

5)
Gegeben sei die Funktion f(x)= \sqrt{4x^2-2x}-4 mit \mathbb{D} = \left\{ x\in\mathbb{R}\mid x \le 0 \text{ oder } x \ge \dfrac{1}{2} \right\} .
Gesucht ist jeweils die fehlende Koordinate des Punktes P(x\mid y),
a) wenn x=\dfrac{5}{2}
b) wenn y=-5

  10)
Gegeben sei die Funktion f(x)= \sqrt[5]{x}-10 mit \mathbb{D} = \mathbb{R}.
Gesucht ist jeweils die fehlende Koordinate des Punktes P(x\mid y),
a) wenn x=32
b) wenn y=-7

 

3. Aufgabe

Bestimmen Sie von folgenden Funktionen den Definitionsbereich und die Nullstellen!

1) f(x)=20\sqrt{2x-15}-15

  11) f(x)=\dfrac{\sqrt[3]{\left(x^2+8x+16\right)^3}}{\sqrt{17+x}}

2) g(x)=\sqrt{100-x^2}

  12) f(x)=\sqrt{4x+8}+\sqrt{12x-24}

3) f(t)=2\sqrt[3]{t^2-8}-1

  13) f(x)= -34x\cdot\sqrt[5]{\dfrac{11x}{32}}+51x

4) f(x)=\sqrt{x^3-x^2}

  14) f(x)=\sqrt[3]{x^3-37\cdot10^3}+40

5) f(x)=\dfrac{\sqrt{x+7}}{2}+77

  15) f(x)=\sqrt[2]{\sqrt[2]{49^{\frac{1}{2}}\cdot x}\cdot 19}

6) r(x)=\sqrt{\dfrac{x^4}{2}}-x

  16) f(x)=\dfrac{7}{8} \sqrt{-63x+9x^2}

7) f(x)=\dfrac{1}{\sqrt[4]{x^3}}-\sqrt[3]{x^2}

  17) f(x)= \sqrt{104x^2+36}-\sqrt{225x^2+11}

8) f(x)=3-\sqrt{x^3+3}

  18) f(x)= \dfrac{\sqrt{15x^3}}{ \sqrt{5}}-\left(\sqrt{5}\right)^3

9) g(x)=\sqrt[3]{x^2-9}+4

  19) f(x)=\dfrac{\sqrt{64}}{\sqrt{8x}}-128

10) f(x)=-2\sqrt{x+15}+7   20) f(x)=\dfrac{x}{\sqrt{3}}+\sqrt[3]{3x^2}\cdot \sqrt[3]{3^2}

Dieses Kapitel enthält die folgenden Themen:

 

15.2 Wurzelgleichungen und -funktionen - Erklärungen

Nachdem wir uns im Kapitel Potenzen, Wurzeln, Logarithmen angeschaut haben, wofür Wurzeln gut sind und wie sie funktionieren, ist es wohl nicht sehr überraschend, dass sie auch Teil von Gleichungen und Funktionen sein können. Benötigt werden Wurzeln in Gleichungen und Funktionen beispielsweise, wenn der Satz des Pythagoras irgendwo "mitspielt" (z. B. bei der Diagonalen eines Quadrates in Abhängigkeit der Seitenlänge), aber auch in der Statistik, wo Wurzeln in der Formel für die Standardabweichung enthalten sind.
Wer sich (noch) nicht mit den Rechenregeln für Wurzeln vertraut gemacht hat, sollte dies nun nachholen. Auch mit Potenzen und binomischen Formeln sollten Sie sich auskennen. Wir werden diese Rechenregeln und -gesetze im nun folgenden Kapitel brauchen.

 

Definition

Definition einer Wurzelgleichung: Wurzelgleichungen sind Gleichungen, bei denen die Variable im Radikanden von Wurzeln steht.
Definition einer Wurzelfunktion: Wurzelfunktionen sind Funktionen, bei denen die Variable im Radikanden von Wurzeln steht.

Es ist egal, ob es sich um eine zweite, dritte, vierte etc. Wurzel handelt. Natürlich müssen die Wurzelterme nicht alleine in der Gleichung/Funktion stehen, sondern dürfen mit allem, was wir bislang in diesem Lernmodul kennengelernt haben, z. B. mit Brüchen, kombiniert werden.

Definitionsbereich: Auch bei dieser Art von Gleichung/Funktion ist es wichtig, sich Gedanken zum Definitionsbereich zu machen: 

  1. Fall: Bei geraden Wurzelexponenten darf der Radikand im Bereich der reellen Zahlen nicht negativ sein. Wir müssen also prüfen, für welche Variablenwerte das passiert, und den Definitionsbereich so "zuschneiden", dass nur passende Werte enthalten sind. Dafür müssen Ungleichungen gelöst werden. 
  2. Fall: Bei ungeraden Wurzelexponenten dürfen auch negative Zahlen im Radikand stehen, denn es gilt ja: \sqrt [n]{a }= -\sqrt[n]{\vert a \vert} für ungerade n.

 

Wurzelgleichungen

Wie immer einige einleitende Beispiele:

  • \sqrt[4]{-x-10} = 100 mit \mathbb{D} = \left[-\infty;-10\right[

  • 5-\sqrt{x} = \dfrac{\sqrt{23x}}{2} mit \mathbb{D} = \mathbb{R}^+_0

  • \sqrt{\dfrac{144}{49}}x = 0 mit \mathbb{D} = \mathbb{R}
    Achtung: Diese Gleichung sieht nur aus wie eine Wurzelgleichung. Man kann sie leicht vereinfachen, indem man die Wurzel ausrechnet. Dann steht da nur noch eine lineare Gleichung, nämlich \dfrac{12}{7}x = 0.

 

Lösungsweg

Grundsätzliches Vorgehen bei Wurzelgleichungen ist, dass man zunächst versucht, die Wurzel allein auf eine Seite der Gleichung zu bringen. Mathematisch sagt man: Die Wurzel wird auf einer Seite der Gleichung isoliert. Potenziert man anschließend die Gleichung mit dem Wurzelexponenten, bleibt von der isolierten Wurzel nur der Radikand stehen. Je nachdem, woraus der Radikand besteht, bleibt danach in vielen Fällen nur eine lineare oder quadratische Gleichung übrig. Damit haben wir wieder ein neues Problem auf etwas zurückgeführt, von dem wir schon wissen, wie man damit umgeht.
Wichtig zu wissen ist noch, dass beim Lösen von Wurzelgleichungen die Probe zwingend zum Lösungsweg dazugehört. Durch den eingeschränkten Definitionsbereich kann es nämlich passieren, dass sogenannte Scheinlösungen entstehen. Dazu weiter unten mehr ... 


Schauen wir uns ein paar Beispiele an:

Beispiel 1:
Betrachten wir die Gleichung \sqrt[3]{x+1}+4 = 9.

Bestimmung des Definitionsbereichs:
Da Wurzeln aus negativen Zahlen dann erlaubt sind, wenn der Wurzelexponent ungerade ist, gibt es hier keine Einschränkungen: Der Definitionsbereich ist also \mathbb{D} = \mathbb{R}.

Lösung der Gleichung:
\begin{array}{crclcl}\text{1. Zeile:} & \sqrt[3]{x+1}+4 &=& 9 &\vert& -4 \cr \text{2. Zeile:} & \sqrt[3]{x+1} &=& 5 &\vert& ()^3 \cr \text{3. Zeile:} & x+1 &=& 125 &\vert& -1 \cr & x &=& 124 \;\in\;\mathbb{D}\end{array}

Probe:
\begin{array}{rcl}\sqrt[3]{124+1}+4 &=& 9 \cr \sqrt[3]{125}+4 &=& 9 \cr 5+4 &=& 9 \cr 9 &=& 9\end{array}

Für x=124 ergibt sich eine wahre Aussage: \mathbb{L}=\left\{124\right\}

Zur 1. Zeile: Wir subtrahieren 4, um den Wurzelterm auf der linken Seite der Gleichung zu isolieren.

Zur 2. Zeile: Die gesamte Gleichung wird mit 3 potenziert, um die Wurzel aufzuheben. Wichtig: Solche Umformungen müssen immer auf beiden Seiten der Gleichung durchgeführt werden und auf die gesamte Gleichungsseite angewendet werden. Wie wir schon bei linearen Gleichungen und anderen Gleichungstypen gesehen haben, führt es zu falschen Werten, wenn Umformungen nur auf einige Bestandteile der Gleichung angewendet werden.

Ab der 3. Zeile: Resultat ist eine lineare Gleichung, die man löst wie jede andere lineare Gleichung auch.

Zur Probe: Die Probe für den berechneten Wert geht auf. Somit ist x=124 Lösung der Gleichung.

Sie sehen, dass das Lösen von Wurzelgleichungen gar nicht so kompliziert sein muss. Durch das Potenzieren entsteht eine Gleichung, die sich mit bereits bekannten Verfahren lösen lässt. So etwas mag man in der Mathematik immer gerne ...

 

Bei den Lösungsstrategien für Wurzelgleichungen sind zwei Aspekte besonders wichtig:

  1. Der Wurzelterm muss auf einer Seite der Gleichung isoliert werden.
  2. Wenn vor dem Quadrieren auf einer Seite der Gleichung eine Summe/Differenz steht, muss die binomische Formel verwendet werden.

Schauen wir uns an, was passiert, wenn man das nicht tut ...

Beispiel 2: 
Betrachten wir die Gleichung \sqrt{4-x}+2 = 4.

Bestimmung des Definitionsbereichs:
Da der Wurzelexponent gerade ist (\sqrt{} = \sqrt[2]{}), muss der Radikand größer oder gleich 0 sein.
\begin{array}{rclll} 4-x &\geq & 0 &\vert & -4 \\-x &\geq& -4 &\vert& \cdot(-1) \\x &\leq & 4\end{array}

Der Definitionsbereich ist also \mathbb{D} = \left]-\infty;4\right].

Zunächst die korrekte Rechnung mit Probe:
\begin{array}{rclll}\sqrt{4-x}+2 &=& 4 & \vert &-2 \\\sqrt{4-x} &=& 2 &\vert & \left(\right)^2\\4-x &=& 4 &\vert & -4 \\-x &=& 0 &\vert& \cdot(-1) \\x &=& 0 \in\mathbb{D}\end{array}

Probe:
\begin{array}{rclll}\sqrt{4-0}+2 &=& 4 \\\sqrt{4}+2 &=& 4 \\2+2 &=& 4\end{array}

Für x=0 ergibt sich eine wahre Aussage: \mathbb{L} = \left\{0\right\}

Würde man den Wurzelterm nicht isolieren, wäre die Rechnung zwar grundsätzlich richtig, aber nicht zielführend:
\begin{array}{rclll}\sqrt{4-x}+2 &=& 4 &\vert & \left(\right)^2 \\\left(\sqrt{4-x}+2\right)^2 &=& 16 \\4-x+2\cdot\sqrt{4-x}\cdot 2+4 &=& 16 \\4\sqrt{4-x}-x+8 &=& 16 &\vert & \left(\right)^2 \\ \left(4\sqrt{4-x}-x+8\right)\left(4\sqrt{4-x}-x+8\right) &=& 256 \\16(4-x)-4x\sqrt{4-x}+4\cdot 8\sqrt{4-x}-4x\sqrt{4-x}+x^2-8x+8\cdot 4\sqrt{4-x}-8x+64 &=& 256 \\(-8x+64)\sqrt{4-x}+x^2-32x+128 &=& 256 \\&\dots&\end{array}

Das ist prinzipiell nicht falsch: Die Gleichung (-8x+64)\sqrt{4-x}+x^2-32x+128 = 256 hat die gleiche Lösung wie die ursprüngliche Gleichung \sqrt{4-x}+2 = 4. Dass es uns auf dem Weg zur Lösung nicht weiterbringt, wenn die Gleichung nach jeder Umformung komplizierter ist als vorher, ist aber wohl auch einleuchtend. Fazit: Damit die Gleichung einfacher wird und wir der Lösung näherkommen, muss der Wurzelterm auf einer Seite der Gleichung isoliert werden.

Schlimmer als das fehlende Isolieren des Wurzelterms ist aber, wenn man beim Quadrieren einer Summe keine binomische Formel anwendet. Dann wird das Ergebnis wirklich falsch. Schauen wir uns an unserer Beispielgleichung \sqrt{4-x}+2 = 4 an, was passiert, wenn man fälschlicherweise \left(\sqrt{4-x}+2\right)^2 zu 4-x+4 umformen würde:
\begin{array}{rclll}4-x+4 &=& 16 &\vert & -8 \\-x &=& 8 &\vert& \cdot(-1) \\x &=& -8\;\in\mathbb{D} \end{array}

Probe:
\begin{array}{rclll}\sqrt{4-\left(-8\right)}+2 &=& 4 \\\sqrt{12}+2 &=& 4 &\vert & -2 \\\sqrt{12} &=& 2\end{array}

Das ist eine falsche Aussage. Die Probe geht in diesem Fall also nicht auf. Das bedeutet x=-8 ist keine Lösung unserer Gleichung. Letztendlich überrascht das nicht, da wir ja gleich zu Beginn eine fehlerhafte Umformung durchgeführt haben. Wir erhalten also eine zusätzliche Bestätigung, dass die binomische Formel zu Beginn der Rechnung wirklich wichtig gewesen wäre. In diesem Beispiel hätte man das Problem natürlich umgehen können, indem man den Wurzelterm auf der linken Seite der Gleichung isoliert.

 

Beispiel 3:
Wenn mehrere Wurzelterme in einer Gleichung und/oder zusätzlich Terme ohne Wurzel enthalten sind, muss man die Schritte aus Beispiel 1 ggf. wiederholen. Um dies deutlich zu machen, betrachten wir die Gleichung 7-\sqrt{\dfrac{3}{2}x+81} = -2-\sqrt{2x}.

Bestimmung des Definitionsbereichs:
Da beide Wurzelexponenten gerade sind, müssen beide Radikanden größer oder gleich 0 sein.
linker Radikand:
\begin{array}{rclll}\dfrac{3}{2}x+81 &\geq & 0 &\vert & -81\\\dfrac{3}{2}x &\geq & -81 &\vert &:\dfrac{3}{2}\\ x &\geq & -54 \end{array}

rechter Radikand:
\begin{array}{rclll}2x &\geq & 0 & \vert &:2\\ x &\geq & 0\end{array}

Für die abschließende Ermittlung des Definitionsbereichs bekommen wir also zwei Bedingungen: x\geq-54 (für den linken Radikanden) und x\geq0 (für den rechten Radikanden). Da beide Radikanden Teil der Gleichung sind, müssen natürlich auch beide Bedingungen erfüllt sein. Dies müssen wir nun prüfen:

  • x-Werte kleiner als -54 erfüllen keine Bedingung.
  • x-Werte, die größer gleich -54, aber kleiner als 0 sind, erfüllen nur die erste Bedingung.
  • Nur x-Werte größer oder gleich 0 erfüllen beide Bedingungen.

Der Definitionsbereich ist also \mathbb{D} = \mathbb{R}_0^+.

Lösung der Gleichung:
\begin{array}{ccrclcl}\text{1. Zeile:} & & 7-\sqrt{\dfrac{3}{2}x+81} &=& -2-\sqrt{2x} &\vert& +2 \cr \text{2. Zeile:} & & 9-\sqrt{\dfrac{3}{2}x+81} &=& -\sqrt{2x} &\vert& ()^2 \cr \text{3. Zeile:} & & \left(9-\sqrt{\dfrac{3}{2}x+81}\right)^2 &=& \left(-\sqrt{2x}\right)^2 \cr\text{4. Zeile:} & & 81-18\sqrt{\dfrac{3}{2}x+81}+\dfrac{3}{2}x+81 &=& 2x \cr \text{5. Zeile:} & & \dfrac{3}{2}x+162-18\sqrt{\dfrac{3}{2}x+81} &=& 2x &\vert& -\dfrac{3}{2}x-162 \cr \text{6. Zeile:} & & -18\sqrt{\dfrac{3}{2}x+81} &=& \dfrac{1}{2}x-162 &\vert& :(-18) \cr \text{7. Zeile:} & & \sqrt{\dfrac{3}{2}x+81} &=& -\dfrac{1}{36}x+9 &\vert& ()^2 \cr \text{8. Zeile:} & & \dfrac{3}{2}x+81 &=& \dfrac{1}{1.296}x^2-\dfrac{1}{2}x+81 &\vert& -\dfrac{1}{1.296}x^2+\dfrac{1}{2}x-81 \cr & & -\dfrac{1}{1.296}x^2+2x &=& 0 &\vert& \text{ausklammern} \cr & & x\left(-\dfrac{1}{1.296}x+2\right) &=& 0 &\vert& \text{Satz vom Nullprodukt} \cr & \text{Faktor 1:} & x_1 &=& 0 \;\in\;\mathbb{D} \cr\cr& \text{Faktor 2:} & -\dfrac{1}{1.296}x+2 &=& 0 &\vert& -2 \cr & & -\dfrac{1}{1.296}x &=& -2 &\vert& :\left(-\dfrac{1}{1.296}\right) \cr & & x_2 &=& 2.592 \;\in\;\mathbb{D}\end{array}

Probe:
Für x_1:
\begin{array}{rcl}-\sqrt{\dfrac{3}{2}\cdot 0+81}+7 &=& -2-\sqrt{2\cdot 0} \cr -\sqrt{81}+7 &=& -2-\sqrt{0} \cr -9+7 &=& -2 \cr -2 &=& -2\end{array}

Für x_2:
\begin{array}{rcl}-\sqrt{\dfrac{3}{2}\cdot 2.592+81}+7 &=& -2-\sqrt{2\cdot 2.592} \cr -\sqrt{3.969}+7 &=& -2-\sqrt{5.184} \cr -63+7 &=& -2-72 \cr -56 &=& -74\end{array}

Nur für x_1=0 ergibt sich eine wahre Aussage: \mathbb{L} = \left\{0\right\}

Zur 1. Zeile: Zunächst fasst man die Gleichung so weit wie möglich zusammen. Je weniger einzelne Terme, desto einfacher ...

Zur 2. Zeile: Nun wird die Gleichung quadriert. Das hilft aber diesmal nur teilweise, weil nur eine der Wurzeln isoliert werden kann. Die 9 ist ja auch noch da und steht auf der linken Gleichungsseite gemeinsam mit einem Wurzelterm.

Zur 3. Zeile: Diese Zeile müsste man grundsätzlich nicht hinschreiben. Man sieht hier aber gut, worauf beim Quadrieren geachtet werden muss: In der Klammer auf der linken Seite steht nämlich eine Differenz. Das heißt, dieses Quadrat kann nur mithilfe der 2. binomischen Formel (oder mit dem Distributivgesetz) aufgelöst werden. Das ist wichtig! Wir hatten uns oben bei Beispiel 2 und im Kapitel Potenzen, Wurzeln, Logarithmen ja angeschaut, was passiert, wenn man z. B. den mittleren Term nicht berücksichtigt. Genau der gleiche Fehler passiert hier auch.
Das ist ebenfalls eine Erklärung, warum es sinnvoll ist, die Wurzel vor dem Quadrieren auf einer Seite der Gleichung zu isolieren, wenn es möglich ist. Wenn die Wurzel nämlich alleine auf einer Seite der Gleichung steht, können wir ganz beruhigt quadrieren, weil hier keine Kombination von Potenz-, Punkt- und Strichrechnung auftritt.

Zur 4. Zeile: Da der mittlere Term der binomischen Formel -2ab ist, in diesem Fall also -2\cdot 9\cdot\sqrt{\dfrac{3}{2}x+81}, bleibt eine Wurzel weiterhin bestehen. Gut ist, dass dies nun die einzige Wurzel ist.

Zur 5. und 6. Zeile: Die verbleibende Wurzel muss nun wieder durch die üblichen Umformungen isoliert werden.

Zur 7. Zeile: Nun befindet sich die Wurzel allein auf einer Seite, und es kann ohne Probleme quadriert werden. Auf der rechten Seite muss auf die binomische Formel geachtet werden. Alle Wurzeln sind nach dieser Umformung verschwunden.

Ab der 8. Zeile: Es bleibt eine quadratische Gleichung ohne konstanten Term übrig. Daher kann x ausgeklammert und anschließend der Satz vom Nullprodukt angewendet werden.

Zur Probe: Bei dieser Gleichung liefert nur x_1=0 eine wahre Aussage. Bei x_2=2.592 geht die Probe nicht auf (-56=-74 ist ja offensichtlich nicht richtig). Die einzige Lösung ist somit x_1=0.

 

Nach Abschluss der Rechnung: Prüfung auf Scheinlösungen! 
Dass im dritten Beispiel die Probe für x_2 nicht aufgeht, obwohl wir völlig richtig gerechnet haben, ist bei Wurzelgleichungen (leider) durchaus typisch. Beim Lösen von Wurzelgleichungen können nämlich Scheinlösungen entstehen. Bei Bruchgleichungen hatten wir ja auch schon gesehen, dass in der Rechnung Werte entstehen können, die die Gleichung nur scheinbar lösen.
Bei Wurzelgleichungen ist die Ursache dafür, dass Quadrieren keine Äquivalenzumformung ist: Quadrieren wir die Gleichung x=4, erhalten wir x^2=16. Das sieht erst mal unproblematisch aus, allerdings haben wir eine Lösung hinzubekommen: x=-4 löst die neue Gleichung ja auch ... Die Lösungsmengen der ursprünglichen und der neuen Gleichung sind also nicht identisch. Da man beim Lösen von Wurzelgleichungen kaum um das Quadrieren (oder allgemein das Potenzieren) herumkommt, besteht immer die Gefahr, dass solche Scheinlösungen entstehen. Um diese zu finden, muss bei Wurzelgleichungen also immer eine Probe durchgeführt werden. Bei Scheinlösungen geht die Probe nämlich nicht auf, sodass man Lösungen von Scheinlösungen unterscheiden und dann die Scheinlösungen aus der Lösungsmenge ausschließen kann.
Um noch ein bisschen präziser zu sein: Das Problem tritt dann auf, wenn der Wurzelexponent gerade ist und wir daher auf dem Lösungsweg mit einer geraden Zahl potenzieren müssen. Es schadet aber nicht, auch bei Wurzelgleichungen mit ungeraden Wurzelexponenten eine Probe durchzuführen. Lieber eine Probe zu viel, als eine zu wenig ... 

 

Wurzelfunktionen

Auch den Verlauf und die Eigenschaften von Wurzelfunktionen wollen wir uns anhand einiger Beispiele anschauen. Wie schon bei den gebrochen rationalen Funktionen unterscheiden wir hier zwei einfache Fälle und einen komplexeren Fall.

 

1. Fall

Der Radikand besteht nur aus der Variablen und der Wurzelexponent ist gerade. Wir betrachten also nun Funktionen dieser Form f(x)=\sqrt[n]{x} = a^{\frac{1}{n}}, wobei n eine gerade Zahl ist. Hier ist der Definitionsbereich \mathbb{D}=\mathbb{R}^+_0, da aus negativen Zahlen ja keine Wurzeln gezogen werden können, wenn der Wurzelexponent gerade ist. Hier zwei Beispiele:

  • f_1(x)=\sqrt{x}=\sqrt[2]{x}

  • f_2(x)=\sqrt[10]{x}

Beispiele für Wurzelfunktionen mit geradem Wurzelexponenten

Wertebereich: Der Wertebereich dieser Funktionen umfasst nur die nichtnegativen reellen Zahlen, da Wurzeln mit geraden Wurzelexponenten grundsätzlich so definiert sind, dass keine negativen Ergebnisse entstehen können. Das ist dann natürlich auch hier so.

Randverhalten: Am unteren Rand vom Definitionsbereich können wir den Funktionswert direkt ausrechnen, weil 0 ja Teil des Intervalls ist: f(0) = 0 bei allen Funktionen, die wir hier gerade betrachten. Für sehr große x-Werte werden die Funktionswerte immer größer. Wurzelfunktionen wachsen zwar langsamer als alle Polynomfunktionen, aber schneller als Logarithmusfunktionen.

Symmetrie: Aufgrund des einseitig eingeschränkten Definitionsbereichs können die Graphen von Wurzelfunktionen nicht symmetrisch sein.

"Besondere Punkte": Die hier betrachteten Funktionen haben eine Nullstelle bei x=0, aber keine Extrempunkte, Wendepunkte und Polstellen.

Weitere Besonderheiten: Diese einfachen Wurzelfunktionen verlaufen alle durch den Punkt (1 \mid 1). Das liegt daran, dass Wurzeln die Antwort auf die Frage "Welche Zahl ergibt 1, wenn man sie n-mal mit sich selbst multipliziert?" geben - und egal, wie oft man die 1 mit sich selbst multipliziert, das Ergebnis ist immer 1.

 

2. Fall

Der Radikand besteht nur aus der Variablen und der Wurzelexponent ist ungerade. Wir betrachten hier also Funktionen dieser Form f(x)=\sqrt[n]{x} = a^{\frac{1}{n}}, wobei n eine ungerade Zahl ist und zwar konkret folgende Beispiele:

  • f_3(x)=\sqrt[3]{x} mit \mathbb{D}=\mathbb{R}

  • f_4(x)=\sqrt[9]{x} mit \mathbb{D}=\mathbb{R}

Beispiele für Wurzelfunktionen mit ungeradem Wurzelexponenten

Bemerkung: In der Grafik sieht es ein bisschen so aus, als wäre der Graph bei x=0 unterbrochen. Das ist aber nicht so, sondern liegt nur an dem etwas unpräzisen Plot ...


Wertebereich: Der Wertebereich umfasst alle reellen Zahlen. 

Randverhalten: Für sehr kleine x-Werte werden die Funktionswerte immer kleiner; für sehr große x-Werte immer größer. Dass Wurzelfunktionen langsamer als Polynom- aber schneller als Logarithmusfunktionen wachsen, ist hier genauso.

Symmetrie: In diesem Fall sind die Graphen von Wurzelfunktionen punktsymmetrisch zum Koordinatenursprung.

"Besondere Punkte": Die nun betrachteten Funktionen haben ebenfalls eine Nullstelle bei x=0, aber keine Extrempunkte, Wendepunkte und Polstellen.

Weitere Besonderheiten: Im zweiten Fall verlaufen alle Funktionen durch die Punkte (1 \mid 1) und (-1 \mid -1). Für den Punkt (1\mid 1) hatten wir oben schon geklärt, warum das so ist. Für (-1 \mid -1) können wir nachrechnen: 
f(-1) = \sqrt[n]{-1} = -\sqrt[n]{\vert -1\vert} = -\sqrt[n]{1} = -1, da n ungerade ist

 

3. Fall

Was kann man sonst noch so mit Wurzelfunktionen machen? Natürlich geht mit Wurzelfunktionen noch viel mehr, abhängig davon was man in den Radikanden steckt und was sonst so im Funktionsterm passiert. Schauen wir uns dazu die folgenden Funktionen an:

  • f_5(x)=\sqrt[4]{x^2+4x} mit \mathbb{D}=\left\{ x\in\mathbb{R}\mid x \le -4 \text{ oder } x \ge 0 \right\}

  • f_6(x)=\dfrac{\sqrt{4x+3}+1}{2-\sqrt{x+2}} mit \mathbb{D} = \left\{k\in\mathbb{R} \left\vert k\geq -\dfrac{3}{4} \text{ und } k\neq 2 \right. \right\}

  • f_7(x)=-\sqrt[3]{5x^2+5} mit \mathbb{D}=\mathbb{R}

  • f_8(x)=\sqrt{8+x}-\sqrt{5-x}+3 mit \mathbb{D}=[-8;5]

Beispiele für beliebige Wurzelfunktionen

Wie Sie sehen, kann man bei Wurzelfunktionen, die über die beiden einfachen Fälle hinausgehen, keine allgemeinen Aussagen über den Verlauf und besondere Punkte der Graphen treffen.

Wertebereich: Der Wertebereich kann nur ein verhältnismäßig kleines Intervall umfassen (z. B. bei f_8(x): \left[3-\sqrt{13}\; ; \;3+\sqrt{13}\right]) oder (zumindest einseitig) unbeschränkt sein, wie bei f_5(x) und f_7(x). Bei f_6(x) hat der Wertebereich sogar eine "Lücke", ist aber nach oben und unten unbeschränkt.
Das ist ganz ähnlich zum Definitionsbereich.

Randverhalten: Mal werden die Funktionswerte an den Rändern des Definitionsbereichs beliebig groß oder beliebig klein. Mal können sie - wie bei f_8(x) - einfach ausgerechnet werden (kleinster Wert: 3-\sqrt{13}\approx -0{,}61, größter Wert: 3+\sqrt{13}\approx 6{,}61).

Symmetrie: Nachdem wir in den oberen Fällen gesehen haben, dass einfache Wurzelfunktionen nicht symmetrisch bzw. punktsymmetrisch sind, sind bei diesen Beispielen zwei achsensymmetrische Graphen dabei: f_5(x) ist achsensymmetrisch zu einer senkrechten Gerade durch x=-2 und f_7(x) ist achsensymmetrisch zur y-Achse.

"Besondere Punkte":
In der Grafik wird deutlich, dass nicht jede dieser komplexeren Funktionen eine Nullstelle hat (siehe z. B. f_6(x) und f_7(x)). Allerdings ist es schon so, dass mehr "besondere Punkte" auftreten können, je komplexer der Funktionsterm ist:

Beispielsweise hat f_7(x) einen Extrempunkt H \left(0 \mid -\sqrt[3]{5}\right). Um genau zu sein, ist es ein Hochpunkt.

Die Funktion f_8(x) hat einen Wendepunkt bei x=-\dfrac{3}{2}. Dort ändert sich die Krümmung von einer Rechtskurve zu einer Linkskurve.

f_6(x) hat eine Polstelle bei x=2, weil dort der Nenner eine Nullstelle hat, die nicht gleichzeitig Nullstelle des Zählers ist.

Übersicht:

 

15.3 Wurzelgleichungen und -funktionen - Lösungen

Erste Bemerkung zur Bestimmung des Definitionsbereichs: Auch bei unkomplizierten Wurzeln wie \sqrt{x} (Aufgabe 1.15) muss im Definitionsbereich berücksichtigt werden, dass der Radikand nicht negativ werden darf. Möglich sind in diesem Fall alle positiven reellen Zahlen und 0, also ist \mathbb{D} = \mathbb{R}_0^+. Da der Radikand hier so einfach ist, wurde dafür in den folgenden Aufgaben keine Rechnung aufgeschrieben.
Bei Wurzeln mit ungeraden Wurzelexponenten sind grundsätzlich alle reellen Zahlen zulässig. Insofern ist hierfür keine gesonderte Bestimmung des Definitionsbereichs nötig.

Zweite Bemerkung zur Bestimmung des Definitionsbereichs: Da der Radikand bei Wurzeln mit geradem Wurzelexponenten größer oder gleich 0 sein muss, müssen zur Bestimmung des Definitionsbereichs Ungleichungen gelöst werden. Wenn Sie damit Schwierigkeiten haben, schauen Sie bitte im entsprechenden Kapitel nach.

 

1. Aufgabe

1)
Bestimmung des Definitionsbereichs:
Da ein Quadrat im Bereich der reellen Zahlen immer nichtnegativ ist, ist der Radikand als Summe aus einem Quadrat und der Zahl 13 immer positiv. Der Definitionsbereich ist also \mathbb{D} = \mathbb{R}.

Lösung der Gleichung:
\begin{array}{rclcl} \sqrt{x^2+13} &=& 7 &\vert& ()^2 \cr x^2+13 &=& 49 &\vert& -13 \cr x^2 &=& 36 &\vert& \pm\sqrt{} \cr\cr x_1 &=& 6\;\in\;\mathbb{D} \cr x_2 &=& -6\;\in\;\mathbb{D} \end{array}

Probe:
für x_1:
\begin{array}{rcl} \sqrt{6^2+13} &=& 7 \cr \sqrt{49} &=& 7 \cr 7 &=& 7 \end{array}

Für x_2:
\begin{array}{rcl} \sqrt{(-6)^2+13} &=& 7 \cr \sqrt{49} &=& 7 \cr 7 &=& 7 \end{array}

Ergebnis: Für x_1 = 6 sowie x_2 = -6 ergeben sich wahre Aussagen: \mathbb{L} = \left\{-6; 6 \right\}


2)
Bestimmung des Definitionsbereichs:
\begin{array}{rclll} -1+x &\geq & 0 &\vert & +1\\x &\geq & 1\end{array}

Der Definitionsbereich ist also \mathbb{D} = \left[1;\infty\right[.

Lösung der Gleichung:
\begin{array}{rclcl} 5\sqrt{-1+x}+80 &=& 5x+15 &\vert& -80 \cr 5\sqrt{-1+x} &=& 5x-65 &\vert& :5 \cr \sqrt{-1+x} &=& x-13 &\vert& ()^2 \cr -1+x &=& (x-13)^2 \cr -1+x &=& x^2-26x+169 &\vert& +1-x \cr 0 &=& x^2-27x+170 &\vert& \text{p-q-Formel} \cr x_{1,2} &=& \dfrac{27}{2} \pm \sqrt{\left(-\dfrac{27}{2}\right)^2-170} \cr x_{1,2} &=& \dfrac{27}{2} \pm \sqrt{\dfrac{49}{4}} \cr\cr x_1 &=& \dfrac{27}{2} + \dfrac{7}{2} = 17\;\in\;\mathbb{D} \cr x_2 &=& \dfrac{27}{2} - \dfrac{7}{2} = 10\;\in\;\mathbb{D} \end{array}

Probe:
Für x_1:
\begin{array}{rcl} 5 \cdot \sqrt{-1+17}+80 &=& 5 \cdot 17+15 \cr 100 &=& 100 \end{array}

Für x_2:
\begin{array}{rcl} 5 \cdot \sqrt{-1+10}+80 &=& 5 \cdot 10+15 \cr 95 &=& 65 \end{array}

Ergebnis: Nur für x_1 = 17 ergibt sich eine wahre Aussage: \mathbb{L} = \left\{17 \right\}


3)
Bestimmung des Definitionsbereichs:
1. Radikand links:
\begin{array}{rclll} 4x-17 &\geq & 0 &\vert & +17\\4x &\geq & 17 &\vert & :4\\x &\geq & \dfrac{17}{4}\\\\\end{array}

2. Radikand links:
\begin{array}{rclll} 17+4x &\geq & 0 &\vert & -17\\4x &\geq & -17 &\vert & :4\\x &\geq & -\dfrac{17}{4}\\\\\end{array}

Radikand rechts:
\begin{array}{rclll} 16x^2-289 &\geq & 0 &\vert & +289\\16x^2 &\geq & 289 &\vert & :16\\x^2 &\geq & \dfrac{289}{16} &\vert & \sqrt{}\\\\\vert x\vert &\geq & \dfrac{17}{4}\end{array}

Für die abschließende Ermittlung des Definitionsbereichs muss nun überprüft werden, für welche x-Werte alle ermittelten Bedingungen erfüllt sind. Das ist nur der Fall für x\geq \dfrac{17}{4}. Der Definitionsbereich ist also \mathbb{D} = \left[\dfrac{17}{4};\infty\right[.

Lösung der Gleichung:
\begin{array}{rclcl} \sqrt{4x-17} \cdot \sqrt{17+4x} &=& \sqrt{16x^2-289} \cr \sqrt{(4x-17) \cdot (17+4x)} &=& \sqrt{16x^2-289} \cr \sqrt{16x^2-289} &=& \sqrt{16x^2-289} \cr 0 &=& 0 \cr\cr \mathbb{L} &=& \left[\dfrac{17}{4};\infty\right[ \end{array}

Bemerkung:
 Unabhängig davon, welches Element des Definitionsbereichs in diese Gleichung eingesetzt wird, erhält man immer auf beiden Seiten dasselbe Ergebnis. 0=0 ist schließlich immer richtig. Jede reelle Zahl löst also diese Gleichung, d. h. die Lösungsmenge entspricht dem Definitionsbereich.


4)
Bestimmung des Definitionsbereichs:
1. Radikand rechts:
\begin{array}{rclll} 3+k &\geq & 0 &\vert & -3\\k &\geq & -3\\\\\end{array}

2. Radikand rechts:
\begin{array}{rclll} 1-k &\geq & 0 &\vert & -1 \\-k &\geq & -1 &\vert& \cdot (-1) \\k &\leq& 1\end{array}

Für die abschließende Ermittlung des Definitionsbereichs muss nun überprüft werden, für welche x-Werte alle ermittelten Bedingungen erfüllt sind. Das ist nur der Fall für -3 \leq k \leq 1. Der Definitionsbereich ist also \mathbb{D} = \left[-3 ;1\right].

Lösung der Gleichung:
\begin{array}{rclcl} 5 &=& \sqrt{3+k}-\sqrt{1-k} &\vert& ()^2 \cr 25 &=& (3+k)-2\sqrt{3+k}\sqrt{1-k}+(1-k) \cr 25 &=& 4-2\sqrt{(3+k)(1-k)} &\vert& -4 \cr 21 &=& -2\sqrt{-k^2-2k+3} &\vert& ()^2 \cr 441 &=& 4(-k^2-2k+3) \cr 441 &=& -4k^2-8k+12 &\vert& -441 \cr 0 &=& -4k^2-8k-429 &\vert& :(-4) \cr 0 &=& k^2+2k+\dfrac{429}{4} &\vert& \text{p-q-Formel} \cr\cr k_{1,2} &=& -1 \pm \sqrt{1-\dfrac{429}{4}} \cr k_{1,2} &=& -1 \pm \sqrt{-\dfrac{425}{4}} \end{array}

Ergebnis: Da aus negativen reellen Zahlen keine Wurzeln mit geraden Wurzelexponenten gezogen werden können, hat diese Gleichung keine Lösung: \mathbb{L} = \emptyset


5)
Bestimmung des Definitionsbereichs:
x^4-x^2+9 \geq 0

Um diese biquadratische Ungleichung zu lösen, wird im ersten Schritt die zugehörige biquadratische Gleichung gelöst:
x^4-x^2+9 = 0

Substitution: z=x^2
\begin{array}{rclcl}z^2-z+9 &=& 0 &\vert & \text{p-q-Formel}\\z_{1,2} &=& \dfrac{1}{2}\pm\sqrt{\left(-\dfrac{1}{2}\right)^2-9}\\z_{1,2} &=& \dfrac{1}{2}\pm\sqrt{-\dfrac{35}{4}}\end{array}

Da aus negativen reellen Zahlen keine Wurzeln mit geraden Wurzelexponenten gezogen werden können, hat diese Gleichung keine Lösung. Somit hat auch die biquadratische Gleichung keine Lösung.
Jetzt muss noch geprüft werden, ob der Term nur positive oder nur negative Werte annimmt. Es ist sinnvoll, für die Prüfung einen möglichst einfachen Wert für x zu verwenden. Z. B. liefert x=0 als Ergebnis 9. Das bedeutet, dass x^4-x^2+9 für jeden x-Wert positiv ist.

Bemerkung: Nach der Feststellung, dass die Gleichung x^4-x^2+9=0 keine Lösungen hat, hätte man die Eigenschaft, dass der Term zwingend positiv ist, auch daran erkennen können, dass das zugehörige Polynom 4. Grades nach oben geöffnet ist.

Der Definitionsbereich ist also \mathbb{D} = \mathbb{R}.

Lösung der Gleichung:
\begin{array}{crclcl} & \sqrt{x^4-x^2+9} &=& 3 &\vert& ()^2 \cr & x^4-x^2+9 &=& 9 &\vert& -9 \cr & x^4-x^2 &=& 0 \cr & x^2 \cdot (x^2-1) &=& 0 & \vert & \text{Satz vom Nullprodukt} \cr \cr \text{Faktor 1:} & x_1 &=& 0\;\in\;\mathbb{D} \cr\cr \text{Faktor 2:} & x^2-1 &=& 0 & \vert & +1 \cr & x^2 &=& 1 &\vert& \pm \sqrt{} \cr\cr & x_2 &=& 1\;\in\;\mathbb{D} \cr & x_3 &=& -1\;\in\;\mathbb{D} \end{array}

Probe:
Für x_1:
\begin{array}{rcl} \sqrt{0^4-0^2+9} &=& 3 \cr 3 &=& 3 \end{array}

Für x_2:
\begin{array}{rcl} \sqrt{1^4-1^2+9} &=& 3 \cr 3 &=& 3 \end{array}

Für x_3:
\begin{array}{rcl} \sqrt{(-1)^4-(-1)^2+9} &=& 3 \cr 3 &=& 3 \end{array}

Ergebnis: Für x_1 = 0x_2 = 1 sowie x_3 = -1 ergeben sich wahre Aussagen: \mathbb{L} = \{-1;0;1\}


6)
Definitionsbereich: \mathbb{D} = \mathbb{R}

Lösung der Gleichung:
\begin{array}{crclcl} & y \cdot \sqrt[3]{y^2+4} &=& 0 & \vert & \text{Satz vom Nullprodukt} \cr \cr \text{Faktor 1:} & y_1 &=& 0\;\in\;\mathbb{D} \cr\cr \text{Faktor 2:} & \sqrt[3]{y^2+4} &=& 0 &\vert& ()^3 \cr & y^2+4 &=& 0 &\vert& -4 \cr & y^2 &=& -4 &\vert& \pm\sqrt{} \cr & y_{2,3} &=& \pm \sqrt{-4} \end{array}

Da aus negativen reellen Zahlen keine Wurzeln mit geraden Wurzelexponenten gezogen werden können, kommt nur 0 als Lösung der Gleichung infrage.

Probe:
\begin{array}{rcl} 0 \cdot \sqrt[3]{0^2+4} &=& 0 \cr 0 &=& 0 \end{array}

Ergebnis: Für y_1 = 0 ergibt sich eine wahre Aussage: \mathbb{L} = \{0\}


7)
Definitionsbereich: \mathbb{D} = \mathbb{R}

Lösung der Gleichung:
\begin{array}{crclcl} & x+\sqrt[3]{6x^2+5x} &=& 0 &\vert& -x \cr & \sqrt[3]{6x^2+5x} &=& -x &\vert& ()^3 \cr & 6x^2+5x &=& -x^3 &\vert& +x^3 \cr & x^3+6x^2+5x &=& 0 \cr & x \cdot (x^2+6x+5) &=& 0 &\vert& \text{Satz vom Nullprodukt} \cr \cr\text{Faktor 1:} & x_1 &=& 0\;\in\;\mathbb{D} \cr\cr \text{Faktor 2:} & x^2+6x+5 &=& 0 &\vert& \text{p-q-Formel} \cr & x_{2,3} &=& -3 \pm \sqrt{3^2-5} \cr & &=& -3 \pm \sqrt{4} \cr\cr & x_2 &=& -3+2 = -1\;\in\;\mathbb{D} \cr & x_3 &=& -3-2 = -5\;\in\;\mathbb{D} \end{array}

Probe:
Für x_1:
\begin{array}{rcl} 0+\sqrt[3]{6 \cdot 0^2+5 \cdot 0} &=& 0 \cr 0 &=& 0 \end{array}

Für x_2:
\begin{array}{rcl} -1+\sqrt[3]{6 \cdot (-1)^2+5 \cdot (-1)} &=& 0 \cr 0 &=& 0 \end{array}

Für x_3:
\begin{array}{rcl} -5+\sqrt[3]{6 \cdot (-5)^2+5 \cdot (-5)} &=& 0 \cr 0 &=& 0 \end{array}

Ergebnis: Für x_1 = 0x_2 = -1 sowie x_3 = -5 ergeben sich wahre Aussagen: \mathbb{L} = \{-5;-1;0\}


8)
Bestimmung des Definitionsbereichs:
Radikand links:
144x^2-192x+64 \geq 0

Um diese quadratische Ungleichung zu lösen, wird im ersten Schritt die zugehörige quadratische Gleichung gelöst:
\begin{array}{rclll} 144x^2-192x+64 &=& 0 &\vert & :144\\x^2-\dfrac{4}{3}x+\dfrac{4}{9} &=& 0 &\vert & \text{p-q-Formel}\\x_{1,2} &=& \dfrac{2}{3}\pm\sqrt{\left(-\dfrac{2}{3}\right)^2-\dfrac{4}{9}}\\x_{1,2} &=& \dfrac{2}{3}\pm0\\x_{1,2} &=& \dfrac{2}{3}\end{array}

Diese Gleichung hat also genau eine Lösung bei x = \dfrac{2}{3}. Dieser x-Wert ist auf jeden Fall Teil des Definitionsbereichs. Es ergeben sich die Intervalle \left]-\infty;\dfrac{2}{3}\right[ und \left]\dfrac{2}{3};\infty\right[.
Jetzt muss noch geprüft werden, in welchen Intervallen der Term positive bzw. negative Werte annimmt. Es ist sinnvoll, für die Prüfung möglichst einfache x-Werte zu verwenden. Z. B. liefert x=0 als Ergebnis 64 und für x=1 ergibt sich 16. Das bedeutet, dass 144x^2-192x+64 niemals negativ wird und somit der Definitionsbereich an dieser Stelle nicht eingeschränkt werden muss.

Bemerkung: Nach der Feststellung, dass die Gleichung 144x^2-192x+64=0 genau eine Lösung hat, hätte man die Eigenschaft, dass der Term zwingend nichtnegativ ist, auch daran erkennen können, dass die zugehörige Parabel nach oben geöffnet ist.

Radikand rechts:
\begin{array}{rclll}-12x+8 &\geq & 0 &\vert & -8\\-12x &\geq & -8 &\vert & :(-12) \\x &\leq & \dfrac{2}{3}\end{array}

Der Definitionsbereich ist also \mathbb{D} = \left]-\infty;\dfrac{2}{3}\right].

Lösung der Gleichung:
\begin{array}{rclcl} \sqrt[4]{144x^2-192x+64} &=& \sqrt{-12x+8} &\vert& ()^4 \cr 144x^2-192x+64 &=& (-12x+8)^2 \cr 144x^2-192x+64 &=& 144x^2-192x+64 \cr 0 &=& 0 \cr\cr \mathbb{L} &=& \left]-\infty;\dfrac{2}{3}\right] \end{array}

Bemerkung: Unabhängig davon, welches Element des Definitionsbereichs in diese Gleichung eingesetzt wird, erhält man immer auf beiden Seiten dasselbe Ergebnis. 0=0 ist schließlich immer richtig. Jede reelle Zahl löst also diese Gleichung, d. h. die Lösungsmenge entspricht dem Definitionsbereich.

9)
Bei dieser Aufgabe lohnt es sich, nicht einfach loszurechnen (Die Rechnung wird auch tatsächlich recht hässlich ...), sondern sich die Struktur der Gleichung zuvor etwas genauer anzuschauen: Auf der linken Seite steht ein nichtnegativer Term (Zur Erinnerung: Das Ergebnis der Wurzel ist immer 0 oder positiv. Wenn man dies durch eine positive Zahl, nämlich die 8, teilt, bleibt das so.). Auf der rechten Seite haben wir eine positive Zahl, die 101, und eine positive Wurzel, die ja im Nenner steht und daher nicht 0 sein darf. Das Minuszeichen vor dem Bruch sorgt dann dafür, dass alles negativ wird. Auf der rechten Seite steht also ein negativer Term. Damit können die beiden Seiten nie gleich sein. Diese Gleichung hat also keine Lösung: \mathbb{L} = \emptyset


10)
Bestimmung des Definitionsbereichs:
\begin{array}{crclll} & x^4+2x^2 &\geq & 0 \\& x^2 \cdot (x^2+2) &\geq & 0\end{array}

Das Produkt zweier Faktoren ist genau dann nichtnegativ, wenn entweder mindestens ein Faktor 0 ist oder beide Faktoren das gleiche Vorzeichen besitzen. Der erste Faktor ist ein Quadrat, das immer nichtnegativ ist. Für x=0 wird das Quadrat 0, so dass dieser Wert auf jeden Fall Teil des Definitionsbereichs ist.
Beim zweiten Faktor wird zu einem Quadrat die Zahl 2 addiert. Somit ist der zweite Faktor zwingend positiv, da das Quadrat mindestens den Wert 0 hat.
Das bedeutet, dass die Ungleichung x^2(x^2+2)\geq 0 für jeden x-Wert wahr ist. Der Definitionsbereich ist also \mathbb{D} = \mathbb{R}.

Lösung der Gleichung:
\begin{array}{rclcl} 22+\sqrt{x^4+2x^2} &=& 23 &\vert& -22 \cr \sqrt{x^4+2x^2} &=& 1 &\vert& ()^2 \cr x^4+2x^2 &=& 1 &\vert& -1\cr \left(x^2\right)^2+2x^2-1 &=& 0 \end{array}

Substitution:  u=x^2
\begin{array}{rclcl} u^2+2u-1 &=& 0 &\vert& \text{p-q-Formel} \cr u_{1,2} &=& -1 \pm \sqrt{1+1} \cr\cr u_1 &=& -1+\sqrt{2} \cr u_2 &=& -1-\sqrt{2}\end{array}

Rücksubstitution:
\begin{array}{rclcl} u_1 = x^2 &=& -1+\sqrt{2} &\vert& \pm\sqrt{} \cr x_1 &=& \sqrt{-1+\sqrt{2}}\approx 0{,}64\;\in\;\mathbb{D} \cr x_2 &=& -\sqrt{-1+\sqrt{2}}\approx -0{,}64\;\in\;\mathbb{D} \cr\cr u_2 = x^2 &=& -1-\sqrt{2} &\vert& \pm\sqrt{} \cr x_3 &=& \sqrt{-1-\sqrt{2}} \cr x_4 &=& -\sqrt{-1-\sqrt{2}} \end{array}

Da aus negativen reellen Zahlen keine Wurzeln mit geraden Wurzelexponenten gezogen werden können, kommen nur  x_1 = \sqrt{-1+\sqrt{2}} und  x_2 = -\sqrt{-1+\sqrt{2}} als Lösungen der Gleichung infrage.

Probe:
Für x_1:
\begin{array}{rcl} 22+\sqrt{(\sqrt{-1+\sqrt{2}})^4+2 \cdot (\sqrt{-1+\sqrt{2}})^2} &=& 23 \cr 23 &=& 23 \end{array}

Für x_2:
\begin{array}{rcl} 22+\sqrt{(-\sqrt{-1+\sqrt{2}})^4+2 \cdot (-\sqrt{-1+\sqrt{2}})^2} &=& 23 \cr 23 &=& 23 \end{array}

Ergebnis: Für x_1 = \sqrt{-1+\sqrt{2}} sowie x_2 = -\sqrt{-1+\sqrt{2}} ergeben sich wahre Aussagen: \mathbb{L} = \left\{-\sqrt{-1+\sqrt{2}};\sqrt{-1+\sqrt{2}}\right\}


11)
Definitionsbereich: \mathbb{D} = \mathbb{R}^+_0

Lösung der Gleichung:
\begin{array}{rclll}x-5\sqrt{x}+6 &=& 0 \\\\\left(\sqrt{x}\right)^2-5\sqrt{x}+6 &=& 0\end{array}

Substitution: a=\sqrt{x}
\begin{array}{rclll}a^2-5a+6 &=& 0 & \vert & \text{p-q-Formel}\\a_{1,2} &=& \dfrac{5}{2}\pm\sqrt{\left(-\dfrac{5}{2}\right)^2-6}\\a_{1,2} &=& \dfrac{5}{2}\pm\dfrac{1}{2}\\\\a_1 &=& \dfrac{5}{2}+\dfrac{1}{2} = 3\\a_2 &=& \dfrac{5}{2}-\dfrac{1}{2} = 2\end{array}

Rücksubstitution:
\begin{array}{rclcl}a_1 = \sqrt{x} &=& 3 &\vert& ()^2 \\x_1 &=& 9 \\\\a_1 = \sqrt{x} &=& 2 &\vert& ()^2 \\x_2 &=& 4 \end{array}

Probe:
Für x_1:
\begin{array}{rcl}9-5\sqrt{9}+6 &= & 0 \\0 &=& 0\end{array}

Für x_2:
\begin{array}{rcl}4-5\sqrt{4}+6 &=& 0 \cr 0 &=& 0\end{array}

Ergebnis: Für x_1 = 9 sowie x_2 = 4 ergeben sich wahre Aussagen: \mathbb{L} = \left\{4; 9 \right\}

Bemerkung: Man könnte diese Gleichung auch lösen, indem man die Wurzel auf einer Seite der Gleichung isoliert \sqrt{x} = \dfrac{x-6}{5} und dann quadriert. Dabei die 2. binomische Formel im Zähler nicht vergessen!


12)
Bestimmung des Definitionsbereichs:
\begin{array}{crclll} & 23y^3+4y^2+7y &\geq & 0 \\& y\cdot \left(23y^2+4y+7\right) &\geq & 0\end{array}

Das Produkt zweier Faktoren ist genau dann nichtnegativ, wenn entweder mindestens ein Faktor 0 ist oder beide Faktoren das gleiche Vorzeichen besitzen. Der erste Faktor nimmt im Intervall ]-\infty;0[ negative Werte, im Intervall ]0;\infty[ positive Werte und für y=0 den Wert 0 an. y=0 ist somit auf jeden Fall Teil des Definitionsbereichs.
Der zweite Faktor ist eine Parabel. Wir lösen wieder die zugehörige quadratische Gleichung:
\begin{array}{rclll}23y^2+4y+7 & = & 0 &\vert & :23\\y^2+\dfrac{4}{23}y+\dfrac{7}{23} & = & 0 &\vert & \text{p-q-Formel}\\y_{1,2} & = & \dfrac{2}{23}\pm\sqrt{\left(-\dfrac{2}{23}\right)^2-\dfrac{7}{23}}\\\\y_{1,2} & = & \dfrac{2}{23}\pm\sqrt{-\dfrac{157}{529}}\\\\\end{array}

Da aus negativen reellen Zahlen keine Wurzeln mit geraden Wurzelexponenten gezogen werden können, hat diese Gleichung keine Lösung.
Jetzt muss noch geprüft werden, ob der Term nur positive oder nur negative Werte annimmt. Es ist sinnvoll, für die Prüfung einen möglichst einfachen Wert für y zu verwenden. Z. B. liefert y=0 als Ergebnis 7. Das bedeutet, dass 23y^2+4y+7 für jeden y-Wert positiv ist.

Bemerkung: Nach der Feststellung, dass die Gleichung 23y^2+4y+7=0 keine Lösungen hat, hätte man die Eigenschaft, dass der Term zwingend positiv ist, auch daran erkennen können, dass die zugehörige Parabel nach oben geöffnet ist.

Da der zweite Faktor immer positiv ist, kann die Ungleichung y\cdot \left(23y^2+4y+7\right) \geq 0 nur dann wahr sein, wenn der erste Faktor 0 oder positiv ist. Das trifft im Intervall [0;\infty[ zu. Der Definitionsbereich ist also \mathbb{D} = \mathbb{R}^+_0.

Lösung der Gleichung:
\begin{array}{crclll} & \sqrt{23y^3+4y^2+7y} &=& 0\\& \sqrt{y\left(23y^2+4y+7\right)} &=& 0 \\& \sqrt{y} \cdot \sqrt{23y^2+4y+7} &=& 0 &\vert & \text{Satz vom Nullprodukt}\\\\\text{Faktor 1:} & \sqrt{y} &=& 0 &\vert& ()^2 \\& y &=& 0 \;\in\;\mathbb{D}\\\\\text{Faktor 2:} & \sqrt{23y^2+4y+7} &=& 0 &\vert & ()^2 \\& 23y^2+4y+7 &=& 0 &\vert & :23 \\& y^2+\dfrac{4}{23}y+\dfrac{7}{23} &=& 0 &\vert & \text{p-q-Formel}\\& y_{1,2} &=& -\dfrac{2}{23}\pm\sqrt{\left(-\dfrac{2}{23}\right)^2-\dfrac{7}{23}}\\& y_{1,2} &=& -\dfrac{2}{23}\pm\sqrt{-\dfrac{157}{529}}\end{array}

Da aus negativen reellen Zahlen keine Wurzeln mit geraden Wurzelexponenten gezogen werden können, kommt nur 0 als Lösung der Gleichung infrage.

Bemerkung: Wenn Sie die Gleichung erst quadrieren und dann den Satz vom Nullprodukt anwenden, ist das natürlich auch in Ordnung.

Probe:
\begin{array}{rcl}\sqrt{23\cdot 0^3+4\cdot 0^2+7\cdot 0} &=& 0 \\\sqrt{0} &=& 0 \\0 &=& 0\end{array}

Ergebnis: Für y_1 = 0 ergibt sich eine wahre Aussage: \mathbb{L} = \left\{0 \right\}


13)
Bestimmung des Definitionsbereichs:
\begin{array}{rclll} -15x+49 &\geq & 0 &\vert & -49\\-15x &\geq & -49 &\vert & :\left(-15\right)\\x &\leq & \dfrac{49}{15}\end{array}

Der Definitionsbereich ist also \mathbb{D} = \left]-\infty; \dfrac{49}{15}\right].

Lösung der Gleichung:
\begin{array}{rclll} \sqrt[3]{-5+x} &=& \sqrt[6]{-15x+49} &\vert & ()^6\\\left(-5+x\right)^2 &=& -15x+49 \\25-10x+x^2 &=& -15x+49 &\vert & +15x-49\\x^2+5x-24 &=& 0 &\vert & \text{p-q-Formel}\\x_{1,2} &=& -\dfrac{5}{2}\pm\sqrt{\left(\dfrac{5}{2}\right)^2+24}\\x_{1,2} &=& -\dfrac{5}{2}\pm\sqrt{\dfrac{121}{4}}\\\\x_1 &=& -\dfrac{5}{2}+\dfrac{11}{2} = 3\;\in\;\mathbb{D}\\x_2 &=& -\dfrac{5}{2}-\dfrac{11}{2} = -8\;\in\;\mathbb{D}\end{array}

Probe:
Für x_1:
\begin{array}{rcl} \sqrt[3]{-5+3} &=& \sqrt[6]{-15\cdot 3+49} \\\sqrt[3]{-2} &=& \sqrt[6]{4}\\\sqrt[3]{-2} &=& \sqrt[6]{2^2} \\(-2)^{\frac{1}{3}} &=& 2^{2\cdot\frac{1}{6}} \\(-2)^{\frac{1}{3}} &=& 2^{\frac{1}{3}} \\\end{array}

Für x_2:
\begin{array}{rcl} \sqrt[3]{-5-8} &=& \sqrt[6]{-15\cdot (-8)+49} \\\sqrt[3]{-13} &=& \sqrt[6]{169}\\\sqrt[3]{-13} &=& \sqrt[6]{13^2} \\(-13)^{\frac{1}{3}} &=& 13^{2\cdot\frac{1}{6}} \\(-13)^{\frac{1}{3}} &=& 13^{\frac{1}{3}} \\\end{array}

Ergebnis: Für x_1 = 3 sowie x_2 = -8 ergeben sich falsche Aussagen: \mathbb{L} = \emptyset


14)
Bestimmung des Definitionsbereichs:
innerer Radikand:
\begin{array}{rclll} p-\dfrac{1}{2} &\geq & 0 &\vert & +\dfrac{1}{2} \\p &\geq & \dfrac{1}{2}\end{array}

äußerer Radikand:
Der Term p^2+1+3\sqrt{p-\dfrac{1}{2}} ist zwingend positiv, da sowohl der quadratische Ausdruck p^2 als auch das Produkt 3\sqrt{p-\dfrac{1}{2}} jeweils mindestens den Wert 0 haben und durch die Addition von 1 der Term unter der Wurzel mindestens den Wert 1 hat. Somit bleibt es bei der Einschränkung des Definitionsbereichs, die sich aus der Betrachtung der inneren Wurzel ergibt.

Der Definitionsbereich ist also \mathbb{D} = \left[\dfrac{1}{2}; \infty\right[.

Lösung der Gleichung:
\begin{array}{rclll} \sqrt{p^2+1+3\sqrt{p-\dfrac{1}{2}}} &=& p+1 &\vert & ()^2 \\p^2+1+3\sqrt{p-\dfrac{1}{2}} &=& p^2+2p+1 &\vert & -p^2-1 \\3\sqrt{p-\dfrac{1}{2}} &=& 2p &\vert& :3 \\\sqrt{p-\dfrac{1}{2}} &=& \dfrac{2p}{3} &\vert & ()^2 \\p-\dfrac{1}{2} &=& \dfrac{4p^2}{9} &\vert & -\dfrac{4p^2}{9} \\-\dfrac{4}{9}p^2+p-\dfrac{1}{2} &=& 0 &\vert & :\left(-\dfrac{4}{9}\right) \\p^2-\dfrac{9}{4}+\dfrac{9}{8} &=& 0 &\vert & \text{p-q-Formel} \\p_{1,2} &=& \dfrac{9}{8}\pm\sqrt{\left(-\dfrac{9}{8}\right)^2-\dfrac{9}{8}} \\p_{1,2} &=& \dfrac{9}{8}\pm\sqrt{\dfrac{9}{64}} \\\\p_{1} &=& \dfrac{9}{8}+\dfrac{3}{8} = \dfrac{3}{2} \in\mathbb{D} \\\\p_{2} &=& \dfrac{9}{8}\pm\dfrac{3}{8} = \dfrac{3}{4} \in\mathbb{D}\end{array}

Bemerkung: Bitte beachten Sie bei der ersten Umformung, dass auf der rechten Gleichungsseite die 1. binomische Formel angewendet werden muss, da eine Summe quadriert wird.

Probe:
Für p_1:
\begin{array}{rcl} \sqrt{\left(\dfrac{3}{2}\right)^2+1+3\sqrt{\dfrac{3}{2}-\dfrac{1}{2}}} &=& \dfrac{3}{2}+1 \\\\\sqrt{\dfrac{9}{4}+1+3\sqrt{1}} &=& \dfrac{5}{2} \\\\\sqrt{\dfrac{25}{4}} &=& \dfrac{5}{2} \\\\\dfrac{5}{2} &=& \dfrac{5}{2}\end{array}

Für p_2:
\begin{array}{rcl} \sqrt{\left(\dfrac{3}{4}\right)^2+1+3\sqrt{\dfrac{3}{4}-\dfrac{1}{2}}} &=& \dfrac{3}{4}+1 \\\\\sqrt{\dfrac{9}{16}+1+3\sqrt{\dfrac{1}{4}}} &=& \dfrac{7}{4} \\\\\sqrt{\dfrac{9}{16}+1+\dfrac{3}{2}} &=& \dfrac{7}{4} \\\\\sqrt{\dfrac{49}{16}} &=& \dfrac{7}{4} \\\\\dfrac{7}{4} &=& \dfrac{7}{4}\end{array}

Ergebnis: Für p_1 = \dfrac{3}{2} sowie p_2 = \dfrac{3}{4} ergeben sich wahre Aussagen: \mathbb{L} = \left\{\dfrac{3}{4}; \dfrac{3}{2} \right\}


15)
Bestimmung des Definitionsbereichs:
1. Radikand links und 1. Radikand rechts:
\begin{array}{rclll} x &\geq & 0 \\\\\end{array}

2. Radikand links:
\begin{array}{rclll} x+1 &\geq & 0 &\vert & -1 \\x &\geq & -1 \\\\\end{array}

2. Radikand rechts:
\begin{array}{rclll} x+2 &\geq & 0 &\vert & -2\\x &\geq & -2 \\ \end{array}

Für die abschließende Ermittlung des Definitionsbereichs muss nun überprüft werden, für welche x-Werte alle ermittelten Bedingungen erfüllt sind. Das ist nur der Fall für x\geq 0. Der Definitionsbereich ist also \mathbb{D} = \mathbb{R}^+_0.

Lösung der Gleichung:
\begin{array}{rclll} \sqrt{x}\sqrt{x+1}-1 &=& -\sqrt{x}\sqrt{x+2} &\vert & +1+\sqrt{x}\sqrt{x+2}\\\\\sqrt{x}\sqrt{x+1}+\sqrt{x}\sqrt{x+2} &=& 1 \\\\\sqrt{x}\left(\sqrt{x+1}+\sqrt{x+2}\right) &=& 1 &\vert & ()^2\\\\x\left(x+1+2\sqrt{x+1}\sqrt{x+2}+x+2\right) &=& 1 \\\\2x^2+3x+2x\sqrt{x+1}\sqrt{x+2} &=& 1 &\vert & -2x^2-3x\\\\2x\sqrt{x+1}\sqrt{x+2} &=& -2x^2-3x+1 &\vert & ()^2\\\\4x^2\left(x+1\right)\left(x+2\right) &=& \left(-2x^2-3x+1\right)\left(-2x^2-3x+1\right)\\\\4x^2\left(x^2+3x+2\right) &=& 4x^4+6x^3-2x^2+6x^3+9x^2-3x-2x^2-3x+1 \\\\4x^4+12x^3+8x^2 &=& 4x^4+12x^3+5x^2-6x+1 &\vert& -4x^4-12x^3-5x^2+6x-1\\\\3x^2+6x-1 &=& 0 &\vert & :3\\x^2+2x-\dfrac{1}{3} &=& 0 &\vert & \text{p-q-Formel}\\x_{1,2} &=& -1\pm\sqrt{1+\dfrac{1}{3}}\\\\x_{1,2} &=& -1\pm\sqrt{\dfrac{4}{3}} \\\\ x_1 &=& -1+\sqrt{\dfrac{4}{3}} = -1+\dfrac{2\sqrt{3}}{3} \;\in\;\mathbb{D}\\x_2 &=& -1-\sqrt{\dfrac{4}{3}} = -1-\dfrac{2\sqrt{3}}{3} \;\not\in\;\mathbb{D}\end{array}

Bemerkung: Bitte beachten Sie bei der siebten Umformung, dass auf der rechten Gleichungsseite das Distributivgesetz angewendet werden muss, da eine Summe quadriert wird.

Probe:
\begin{array}{rcl} \sqrt{-1+\dfrac{2\sqrt{3}}{3}}\sqrt{-1+\dfrac{2\sqrt{3}}{3}+1}-1 &=& -\sqrt{-1+\dfrac{2\sqrt{3}}{3}}\sqrt{-1+\dfrac{2\sqrt{3}}{3}+2} \\\\\sqrt{-1+\dfrac{2\sqrt{3}}{3}}\sqrt{\dfrac{2\sqrt{3}}{3}}-1 &=& -\sqrt{-1+\dfrac{2\sqrt{3}}{3}}\sqrt{1+\dfrac{2\sqrt{3}}{3}} \\\\\sqrt{\left(-1+\dfrac{2\sqrt{3}}{3}\right)\dfrac{2\sqrt{3}}{3}}-1 &=& -\sqrt{\left(-1+\dfrac{2\sqrt{3}}{3}\right)\left(1+\dfrac{2\sqrt{3}}{3}\right)} \\\\\sqrt{-\dfrac{2\sqrt{3}}{3}+\dfrac{4\cdot 3}{9}}-1 &=& -\sqrt{-1-\dfrac{2\sqrt{3}}{3}+\dfrac{2\sqrt{3}}{3}+\dfrac{4\cdot 3}{9}} \\\\\sqrt{\dfrac{4-2\sqrt{3}}{3}}-1 &=& -\sqrt{-1+\dfrac{4}{3}} \\\\\dfrac{\sqrt{4-2\sqrt{3}}-\sqrt{3}}{\sqrt{3}} &=& -\sqrt{\dfrac{1}{3}} \\\\\dfrac{\sqrt{3-2\sqrt{3}+1}-\sqrt{3}}{\sqrt{3}} &=& -\dfrac{1}{\sqrt{3}} \\\\\dfrac{\sqrt{\left(\sqrt{3}-1\right)^2}-\sqrt{3}}{\sqrt{3}} &=& -\dfrac{1}{\sqrt{3}} \\\\\dfrac{\sqrt{3}-1-\sqrt{3}}{\sqrt{3}} &=& -\dfrac{1}{\sqrt{3}} \\\\\dfrac{-1}{\sqrt{3}} &=& \dfrac{-1}{\sqrt{3}}\end{array}

Ergebnis: Für x_1 = -1+\dfrac{2\sqrt{3}}{3} ergibt sich eine wahre Aussage: \mathbb{L} = \left\{-1+\dfrac{2\sqrt{3}}{3}\right\}

16)
Definitionsbereich: \mathbb{D} = \mathbb{R}

Lösung der Gleichung:
\begin{array}{rclll} 2+\sqrt[3]{x^3-4x^2+8x-14} &=& x &\vert & -2\\\sqrt[3]{x^3-4x^2+8x-14} &=& x-2 &\vert & ()^3\\x^3-4x^2+8x-14 &=& (x-2)(x-2)(x-2) \\x^3-4x^2+8x-14 &=& (x^2-4x+4)(x-2) \\x^3-4x^2+8x-14 &=& x^3-2x^2-4x^2+8x+4x-8 \\x^3-4x^2+8x-14 &=& x^3-6x^2+12x-8 &\vert & -x^3+6x^2-12x+8\\2x^2-4x-6 &=& 0 &\vert & :2\\x^2-2x-3 &=& 0 &\vert & \text{p-q-Formel}\\x_{1,2} &=& 1\pm\sqrt{1+3}\\x_{1,2} &=& 1\pm\sqrt{4}\\\\x_1 &=& 1+2 = 3 \;\in\;\mathbb{D}\\x_2 &=& 1-2 = -1\;\in\;\mathbb{D}\end{array}

Bemerkung: Wer für die Umformung (x-2)^3 (dritte Zeile) andere Möglichkeiten, wie das pascalsche Dreieck, kennt, darf diese natürlich auch nutzen. Hauptsache, die dritte Potenz dieser Summe wird korrekt berechnet.

Probe:
Für x_1:
\begin{array}{rcl} 2+\sqrt[3]{3^3-4\cdot 3^2+8\cdot 3-14} &=& 3\\2+\sqrt[3]{27-36+24-14} &=& 3\\2+\sqrt[3]{1} &=& 3\\2+1 &=& 3 \\3 &=& 3\end{array}

Für x_2:
\begin{array}{rcl} 2+\sqrt[3]{(-1)^3-4\cdot (-1)^2+8\cdot (-1)-14} &=& -1 \cr 2+\sqrt[3]{-1-4-8-14} &=& -1 \cr 2+\sqrt[3]{-27} &=& -1 \cr2-3 &=& -1\\-1 &=& -1\end{array}

Ergebnis: Für x_1 = 3 sowie x_2 = -1 ergeben sich wahre Aussagen: \mathbb{L} = \left\{-1; 3 \right\}


17)
Definitionsbereich: \mathbb{D} = \mathbb{R}

Lösung der Gleichung:
\begin{array}{crclll} & \sqrt[3]{z}+10 &=& \sqrt[3]{z+1.000} &\vert &-10-\sqrt[3]{z+1.000}\\& \sqrt[3]{z}-\sqrt[3]{z+1000} &=& -10 &\vert& ()^3\\& z-3\left(\sqrt[3]{z}\right)^2\cdot\sqrt[3]{z+1.000}+3\sqrt[3]{z}\cdot\left(\sqrt[3]{z+1.000}\right)^2-z-1.000 &=& -1.000 &\vert& +1.000\\& 3\sqrt[3]{z}\cdot\sqrt[3]{z+1.000}\left(\sqrt[3]{z+1.000}-\sqrt[3]{z}\right) &=& 0 &\vert & \text{Satz vom Nullprodukt}\\\\\text{Faktor 1:} & 3\sqrt[3]{z} &=& 0 &\vert & :3 \\ & \sqrt[3]{z} &=& 0 &\vert & ()^3\\& z_1 &=& 0\;\in\;\mathbb{D}\\\\\text{Faktor 2:} & \sqrt[3]{z+1.000} &=& 0 &\vert & ()^3\\& z+1.000 &=& 0 &\vert & -1.000\\& z_2 &=& -1.000\;\in\;\mathbb{D}\\\\\text{Faktor 3:s} & \sqrt[3]{z+1.000}-\sqrt[3]{z} &=& 0 &\vert& +\sqrt[3]{x}\\ & \sqrt[3]{x+1.000} &=& \sqrt[3]{x} &\vert& ()^3 \\ & x+1.000 &=& x &\vert& -x \\& 1.000 &=& 0\end{array}

Da der dritte Faktor auf einen Widerspruch hinausläuft, liefert er keine weiteren Lösungen.

Probe:
Für z_1:
\begin{array}{rcl} \sqrt[3]{0}+10 &=& \sqrt[3]{0+1.000} \\10 &=& 10\end{array}

Für z_2:
\begin{array}{rcl} \sqrt[3]{-1.000}+10 &=& \sqrt[3]{-1.000+1.000} \\0 &=& 0\end{array}

Ergebnis: Für z_1 = 0 sowie z_2 = -1.000 ergeben sich wahre Aussagen: \mathbb{L} = \left\{-1.000; 0 \right\}


18)
Bestimmung des Definitionsbereichs:
Radikand im Zähler:
\begin{array}{rclll} 4k+3 &\geq & 0 &\vert & -3 \\4k &\geq & -3 &\vert & :4 \\k &\geq & -\dfrac{3}{4}\\\\\end{array}

Radikand im Nenner:
\begin{array}{rclll} k+2 &\geq & 0 &\vert & -2\\k &\geq & -2 \\\\\end{array}

Nenner:
\begin{array}{rclll} 2-\sqrt{k+2} &\neq & 0 &\vert & -2 \\-\sqrt{k+2} &\neq & -2 &\vert & \left(\right)^2\\k+2 &\neq & 4 &\vert & -2 \\k &\neq & 2\end{array}

Korrekterweise müssen wir für die letzte Berechnung noch eine Probe machen, weil beim Quadrieren ja Scheinlösungen auftreten können: 2-\sqrt{2+2} = 2-2 = 0. Also alles in Ordnung!

Für die abschließende Ermittlung des Definitionsbereichs muss nun überprüft werden, für welche x-Werte alle ermittelten Bedingungen erfüllt sind. Das ist nur der Fall für k\geq -\dfrac{3}{4} unter Ausschluss der Zahl 2. Der Definitionsbereich ist also \mathbb{D} = \left\{k\in\mathbb{R} \mid k\geq -\dfrac{3}{4} \text{ und } k\neq 2 \right\}.

Lösung der Gleichung:
\begin{array}{rclll} \dfrac{\sqrt{4k+3}+1}{2-\sqrt{k+2}} &=& 3 &\vert & \cdot \left(2-\sqrt{k+2}\right) \\\\\sqrt{4k+3}+1 &=& 6-3\sqrt{k+2} &\vert & -1+3\sqrt{k+2} \\\sqrt{4k+3}+3\sqrt{k+2} &=& 5 &\vert & ()^2\\4k+3+6\sqrt{\left(4k+3\right)\left(k+2\right)}+9\left(k+2\right) &=& 25 \\6\sqrt{4k^2+11k+6}+13k+21 &=& 25 &\vert & -13k-21\\6\sqrt{4k^2+11k+6} &=& 4-13k &\vert & ()^2\\36\left(4k^2+11k+6\right) &=& 16-104k+169k^2 \\144k^2+396k+216 &=& 16-104k+169k^2 &\vert & -16+104k-169k^2\\-25k^2+500k+200 &=& 0 &\vert & :(-25)\\k^2-20k-8 &=& 0 &\vert & \text{p-q-Formel}\\k_{1,2} &=& 10\pm\sqrt{(-10)^2+8}\\k_{1,2} &=& 10\pm\sqrt{108}\\\\ k_1 &=& 10+6\sqrt{3}\;\in\;\mathbb{D}\\k_2 &=& 10-6\sqrt{3}\;\in\;\mathbb{D}\end{array}

Bemerkung 1: Die Multiplikation mit \left(2- \sqrt{k+2}\right) ist hier ohne Einschränkungen möglich, weil k=2 \not\in \mathbb{D}. Eine Multiplikation mit 0 kann also nicht passieren.

Bemerkung 2: Bitte beachten Sie bei der dritten Umformung, dass auf der linken Gleichungsseite die 1. binomische Formel angewendet werden muss, da eine Summe (mit Wurzeltermen als Summanden) quadriert wird, ebenso bei der sechsten Umformung auf der rechten Seite.

Probe:
Für k_1:
\begin{array}{rcl} \dfrac{\sqrt{4\left(10+6\sqrt{3}\right)+3}+1}{2-\sqrt{10+6\sqrt{3}+2}} &=& 3\\\\\dfrac{\sqrt{40+24\sqrt{3}+3}+1}{2-\sqrt{12+6\sqrt{3}}} &=& 3\\\\\dfrac{\sqrt{43+24\sqrt{3}}+1}{2-\sqrt{9+6\sqrt{3}+3}} &=& 3\\\\\dfrac{\sqrt{16+24\sqrt{3}+27}+1}{2-\sqrt{\left(3+\sqrt{3}\right)^2}} &=& 3\\\\\dfrac{\sqrt{16+24\sqrt{3}+9\left(\sqrt{3}\right)^2}+1}{2-3-\sqrt{3}} &=& 3\\\\\dfrac{\sqrt{\left(4+3\sqrt{3}\right)^2}+1}{-1-\sqrt{3}} &=& 3\\\\\dfrac{4+3\sqrt{3}+1}{-1-\sqrt{3}} &=& 3\\\\\dfrac{5+3\sqrt{3}}{-1-\sqrt{3}} &=& 3\\\\\dfrac{5+3\sqrt{3}}{-1-\sqrt{3}} \cdot \dfrac{-1+\sqrt{3}}{-1+\sqrt{3}} &=& 3\\\\\dfrac{-5-3\sqrt{3}+5\sqrt{3}+3\cdot 3}{-1-3} &=& 3 \\\\\dfrac{4+2\sqrt{3}}{-2} &=& 3 \\\\-2-\sqrt{3} &=& 3\end{array}

Für k_2:
\begin{array}{rcl} \dfrac{\sqrt{4\left(10-6\sqrt{3}\right)+3}+1}{2-\sqrt{10-6\sqrt{3}+2}} &=& 3\\\\\dfrac{\sqrt{40-24\sqrt{3}+3}+1}{2-\sqrt{12-6\sqrt{3}}} &=& 3\\\\\dfrac{\sqrt{43-24\sqrt{3}}+1}{2-\sqrt{9-6\sqrt{3}+3}} &=& 3\\\\\dfrac{\sqrt{16-24\sqrt{3}+27}+1}{2-\sqrt{\left(3-\sqrt{3}\right)^2}} &=& 3\\\\\dfrac{\sqrt{16-24\sqrt{3}+9\left(\sqrt{3}\right)^2}+1}{2-3+\sqrt{3}} &=& 3\\\\\dfrac{\sqrt{\left(4-3\sqrt{3}\right)^2}+1}{-1+\sqrt{3}} &=& 3\\\\\dfrac{-4+3\sqrt{3}+1}{-1+\sqrt{3}} &=& 3\\\\\dfrac{-3+3\sqrt{3}}{-1+\sqrt{3}} &=& 3\\\\3\cdot \left(\dfrac{-1+\sqrt{3}}{-1+\sqrt{3}}\right) &=& 3\\\\3 &=& 3 \end{array}

Ergebnis: Nur für k_2 = 10-6\sqrt{3} ergibt sich eine wahre Aussage: \mathbb{L} = \left\{10-6\sqrt{3} \right\}


19)
Bestimmung des Definitionsbereichs:
Radikand im Radikanden links:
\begin{array}{rclll}2t+5 &\geq & 0 &\vert & -5\\2t &\geq & -5 &\vert & :2\\t &\geq & -\dfrac{5}{2}\end{array}

gesamter Radikand links:
t+\sqrt{2t+5} \geq 0

Um diese quadratische Ungleichung zu lösen, wird im ersten Schritt die zugehörige quadratische Gleichung gelöst:
\begin{array}{rclll}t+\sqrt{2t+5} &=& 0 &\vert & -t \\\sqrt{2t+5} &=& -t &\vert & \left(\right)^2 \\2t+5 &=& t^2 &\vert & -2t-5 \\t^2-2t-5 &=& 0 &\vert & \text{p-q-Formel} \\t_{1,2} &=& 1\pm\sqrt{(-1)^2+5} \\t_1 &=& 1+\sqrt{6} \approx 3{,}45 \\t_2 &=& 1-\sqrt{6} \approx -1{,}45\end{array}

Korrekterweise müssen wir für die letzte Berechnung noch eine Probe machen, weil beim Quadrieren ja Scheinlösungen auftreten können:
Für t_1: 1+\sqrt{6}+\sqrt{2\left(1+\sqrt{6}\right)+5} = 2+2\sqrt{6} \neq 0. Bei t_1 handelt es sich also um eine Scheinlösung.
Für t_2: 1-\sqrt{6}+\sqrt{2\left(1-\sqrt{6}\right)+5} = 0. Bei t_2 ist also alles in Ordnung!

Diese Gleichung hat also genau eine Lösung bei t_2 = 1-\sqrt{6}. Dieser t-Wert ist auf jeden Fall Teil des Definitionsbereichs. Im Abgleich zwischen Eintrittsbedingung und Lösung ergeben sich die Intervalle \left]-\frac{5}{2};1-\sqrt{6}\right[ bzw. \left]1-\sqrt{6};\infty\right[
Jetzt muss noch geprüft werden, in welchen Intervallen der Term positive bzw. negative Werte annimmt. Es ist sinnvoll, für die Prüfung möglichst einfache Werte für t zu verwenden. Z. B. liefert t=-2 als Ergebnis -1 und für t=0 ergibt sich 64. Das bedeutet, dass t+\sqrt{2t+5} im zweiten Intervall positiv ist. Die ursprüngliche Ungleichung t+\sqrt{2t+5} \geq 0 ist also nur in diesem Intervall wahr.

Radikand rechts:
\begin{array}{rclll}3t+1 &\geq & 0 &\vert & -1\\3t &\geq & -1 &\vert & :3\\t &\geq & -\dfrac{1}{3}\end{array}

Für die abschließende Ermittlung des Definitionsbereichs muss nun überprüft werden, für welche x-Werte alle ermittelten Bedingungen erfüllt sind. Das ist nur der Fall für t\geq -\dfrac{1}{3}. Der Definitionsbereich ist also \mathbb{D} = \left[-\dfrac{1}{3};\infty\right[.

Lösung der Gleichung:
\begin{array}{rclll} \sqrt{t+\sqrt{2t+5}} &=& \sqrt{3t+1} &\vert & ()^2\\t+\sqrt{2t+5} &=& 3t+1 &\vert & -t \\\sqrt{2t+5} &=& 2t+1 &\vert & ()^2\\2t+5 &=& 4t^2+4t+1 &\vert & -4t^2-4t-1\\-4t^2-2t+4 &=& 0 &\vert & :(-4)\\t^2+\dfrac{1}{2}t-1 &=& 0 &\vert &\text{p-q-Formel}\\t_{1,2} &=& -\dfrac{1}{4}\pm\sqrt{\left(\dfrac{1}{4}\right)^2+1}\\t_{1,2} &=& -\dfrac{1}{4}\pm\sqrt{\dfrac{17}{16}}\\\\t_1 &=& \dfrac{-1+\sqrt{17}}{4} \approx 0{,}78 \;\in\;\mathbb{D}\\t_2 &=& \dfrac{-1-\sqrt{17}}{4} \approx -1{,}28 \;\not\in\;\mathbb{D}\end{array}

Bemerkung: Bitte beachten Sie bei der dritten Umformung, dass auf der rechten Gleichungsseite die 1. binomische Formel angewendet werden muss, da eine Summe quadriert wird.

Probe:
Für t_1:
\begin{array}{rcl}\sqrt{\dfrac{-1+\sqrt{17}}{4}+\sqrt{2\cdot\dfrac{-1+\sqrt{17}}{4}+5}} &=& \sqrt{3\cdot\dfrac{-1+\sqrt{17}}{4}+1}\\\\\sqrt{\dfrac{-1+\sqrt{17}}{4}+\sqrt{\dfrac{-2+2\sqrt{17}}{4}+\dfrac{20}{4}}} &=& \sqrt{\dfrac{-3+3\sqrt{17}}{4}+\dfrac{4}{4}}\\\\\sqrt{\dfrac{-1+\sqrt{17}}{4}+\sqrt{\dfrac{18+2\sqrt{17}}{4}}} &=& \sqrt{\dfrac{1+3\sqrt{17}}{4}}\\\\\sqrt{\dfrac{-1+\sqrt{17}}{4}+\dfrac{1}{2}\cdot\sqrt{18+2\sqrt{17}}} &=& \dfrac{1}{2}\cdot\sqrt{1+3\sqrt{17}}\\\\\sqrt{\dfrac{-1+\sqrt{17}}{4}+\dfrac{1}{2}\cdot\sqrt{1+2\sqrt{17}+17}} &=& \dfrac{1}{2}\cdot\sqrt{1+3\sqrt{17}}\\\\\sqrt{\dfrac{-1+\sqrt{17}}{4}+\dfrac{1}{2}\cdot\sqrt{\left(1+\sqrt{17}\right)^2}} &=& \dfrac{1}{2}\cdot\sqrt{1+3\sqrt{17}}\\\\\sqrt{\dfrac{-1+\sqrt{17}}{4}+\dfrac{1+\sqrt{17}}{2}} &=& \dfrac{1}{2}\cdot\sqrt{1+3\sqrt{17}}\\\\\sqrt{\dfrac{-1+\sqrt{17}}{4}+\dfrac{2+2\sqrt{17}}{4}} &=& \dfrac{1}{2}\cdot\sqrt{1+3\sqrt{17}}\\\\\sqrt{\dfrac{1+3\sqrt{17}}{4}} &=& \dfrac{1}{2}\cdot\sqrt{1+3\sqrt{17}}\\\\\dfrac{1}{2}\cdot\sqrt{1+3\sqrt{17}} &=& \dfrac{1}{2}\cdot\sqrt{1+3\sqrt{17}}\end{array}

Ergebnis: Für t_1 = \dfrac{-1+\sqrt{17}}{4} ergibt sich eine wahre Aussage: \mathbb{L} = \left\{\dfrac{-1+\sqrt{17}}{4} \right\}


20)
Bestimmung des Definitionsbereichs:
9x^2-42x+49 \geq 0

Um diese quadratische Ungleichung zu lösen, wird im ersten Schritt die zugehörige quadratische Gleichung gelöst:
\begin{array}{rclll} 9x^2-42x+49 &=& 0 &\vert & :9\\x^2-\dfrac{14}{3}x+\dfrac{49}{9} &=& 0 &\vert & \text{p-q-Formel}\\x_{1,2} &=& \dfrac{7}{3}\pm\sqrt{\left(-\dfrac{7}{3}\right)^2-\dfrac{49}{9}}\\x_{1,2} &=& \dfrac{7}{3}\pm 0\end{array}

Diese Gleichung hat also genau eine Lösung bei x = \dfrac{7}{3}. Dieser x-Wert ist auf jeden Fall Teil des Definitionsbereichs. Es ergeben sich die Intervalle \left]-\infty;\dfrac{7}{3}\right[ und \left]\dfrac{7}{3};\infty\right[.
Jetzt muss noch geprüft werden, in welchen Intervallen der Term positive bzw. negative Werte annimmt. Es ist sinnvoll, für die Prüfung möglichst einfache x-Werte zu verwenden. Z. B. liefert x=0 als Ergebnis 49 und für x=3 ergibt sich 4. Das bedeutet, dass 9x^2-42x+49 niemals negativ wird und somit der Definitionsbereich nicht eingeschränkt werden muss.

Bemerkung: Nach der Feststellung, dass die Gleichung 9x^2-42x+49=0 genau eine Lösung hat, hätte man die Eigenschaft, dass der Term zwingend nichtnegativ ist, auch daran erkennen können, dass die zugehörige Parabel nach oben geöffnet ist.

Der Definitionsbereich ist also \mathbb{D} = \mathbb{R}.

Lösung der Gleichung:
\begin{array}{crclll} & \sqrt{9x^2-42x+49}+3x &=& 7 \\\\& \sqrt{\left(3x-7\right)^2}+3x &=& 7 \\& \left\vert 3x-7\right\vert +3x &=& 7 \end{array}

Hier ist eine Fallunterscheidung nötig, da die Gleichung einen Betrag enthält.

Fall 1: Wir nehmen an, dass x \geq \dfrac{7}{3} ist.
\begin{array}{rclcl}\left(3x-7\right)+3x &=& 7 \\ 3x-7+3x &=& 7 &\vert & +7 \\ 6x &=& 14 &\vert & :6 \\ x &=& \dfrac{7}{3} \end{array}

Fall 2: Wir nehmen an, dass x < \dfrac{7}{3} ist.
\begin{array}{rclcl}-\left(3x-7\right)+3x &=& 7 \\ -3x+7+3x &=& 7 \\ 7 &=& 7\end{array}

Da im 2. Fall eine wahre Aussage entsteht, sind alle Werte, die wir laut Eintrittsbedingung betrachtet haben, auch Lösungen der Gleichung: x < \dfrac{7}{3}. Hinzu kommt die Lösung, die wir im 1. Fall berechnet haben: x = \dfrac{7}{3}.
Die Lösungsmenge ist also \mathbb{L} = \left]-\infty;\dfrac{7}{3}\right].

Eine Alternative? Nun kommt man ja vielleicht nicht sofort drauf, den Radikanden über die 2. binomische Formel zusammenzufassen, dann die Wurzel zu ziehen und schlussendlich mit dem Betrag weiterzurechnen. Schauen wir uns an, was beim "Standardweg" für das Auflösen einer Wurzelgleichung passieren würde:
\begin{array}{rclcll}\sqrt{9x^2-42x+49}+3x &=& 7 &\vert & -3x \\\sqrt{9x^2-42x+49} &=& -3x+7 &\vert & ()^2 \\9x^2-42x+49 &=& 9x^2-42x+49 &\vert& -9x^2+42x-49 \\0 &=& 0 \end{array}

Da \mathbb{D}=\mathbb{R} ist, könnte man an dieser Stelle meinen, dass auch \mathbb{L}=\mathbb{R} ist. Allerdings haben wir auf dem Lösungsweg quadriert und somit eine Nicht-Äquivalenzumformung durchgeführt. Das bedeutet, dass wir mit Scheinlösungen rechnen müssen. Also müssten wir für alle gefundenen Lösungen eine Probe durchführen, um mögliche Scheinlösungen zu finden und aus der Lösungsmenge auszuschließen. Dummerweise ist das bei unendlichen vielen Lösungen nicht möglich. Somit wird es uns nicht gelingen, alle eventuell vorhandenen Scheinlösungen auszusortieren. Wir können die Lösungsmenge auf diesem Weg daher nicht bestimmen.

 

2. Aufgabe

1)
a)
\begin{array}{rclll} f(0) &=& \sqrt{14\cdot 0+35} &=& \sqrt{35} \quad \rightarrow \quad P_1(0 \mid \sqrt{35}) \end{array}

b)
\begin{array}{rclll} 0 &=& \sqrt{14x+35} &\vert & ()^2 \cr 0 &=& 14x+35 &\vert & -35 \cr -35 &=& 14x &\vert & :14 \cr -\dfrac{35}{14} &=& x \cr x &=& -\dfrac{5}{2} \in\mathbb{D}\end{array}

Probe:
\begin{array}{rcl} f\left(-\dfrac{5}{2}\right) &=& \sqrt{14 \cdot \left(-\dfrac{5}{2}\right)+35} \cr &=& 0 \end{array}

Ergebnis: Für x = -\dfrac{5}{2} ergibt sich der richtige Funktionswert: P_2\left( -\dfrac{5}{2} \mid 0\right) ist also tatsächlich ein Punkt des Graphen.


2)
a)
\begin{array}{rclll} f(9) &=& -\sqrt{9^3+28}+7 &=& -\sqrt{757}+7 \quad \rightarrow \quad P_1(9 \mid -\sqrt{757}+7) \end{array}

b)
\begin{array}{rclll} 8 &=& -\sqrt{x^3+28}+7 &\vert & -7 \cr 1 &=& -\sqrt{x^3+28} &\vert & ()^2 \cr 1 &=& x^3+28 &\vert & -28\cr -27 &=& x^3 &\vert & \sqrt[3]{ } \cr x &=& -3 \in\mathbb{D}\end{array}

Probe:
\begin{array}{rcl}f(-3) &=& -\sqrt{(-3)^3+28}+7\\&=& -\sqrt{-27+28}+7\\&=& -\sqrt{1}+7 \\&=& 6 \\&\neq& 8\\\end{array}

Ergebnis: Für x = -3 ergibt sich eine falsche Aussage: -3 ist also nicht Lösung der Gleichung. Das bedeutet, dass die Funktion nirgends den Funktionswert 8 annimmt.


3)
a)
\begin{array}{rclll} f(1.321) &=& 5-\sqrt[3]{1.321+10} &=& -6 \quad \rightarrow \quad P_1(1.321 \mid -6) \end{array}

b)
\begin{array}{rclll} 515 &=& 5-\sqrt[3]{x+10} &\vert & -5 \cr 510 &=& -\sqrt[3]{x+10} &\vert & \cdot (-1) \cr -510 &=& \sqrt[3]{x+10} &\vert & ()^3 \cr -132.651.000 &=& x+10 &\vert & -10 \cr x &=& -132.651.010 \in\mathbb{D}\end{array}

Probe:
\begin{array}{rcl}f(-132.651.010) &=& 5-\sqrt[3]{-132.651.010+10}\\&=& 5-\sqrt[3]{-132.651.000}\\&=& 5-(-510)\\&=& 515\\\end{array}

Ergebnis: Für x = -132.651.010 ergibt sich der richtige Funktionswert: P_2\left(-132.651.010 \mid 515 \right) ist also tatsächlich ein Punkt des Graphen. 


4)
a)
\begin{array}{rclll} f(-33) &=& \sqrt[4]{(-33)^2+4}-1 &=& \sqrt[4]{957}-1 \quad \rightarrow \quad P_1(-33 \mid \sqrt[4]{957}-1) \end{array}

b)
\begin{array}{rclll}1 &=& \sqrt[4]{x^2+4x}-1 &\vert & +1\cr 2 &=& \sqrt[4]{x^2+4x} &\vert & ()^4\cr 16 &=& x^2+4x &\vert & -16 \cr 0 &=& x^2+4x-16 &\vert& \text{p-q-Formel}\cr x_{1,2} &=& -2 \pm \sqrt{4+16}\cr\cr x_{1} &=&-2+\sqrt{20}\approx 2{,}47 \in\mathbb{D}\cr x_{2} &=&-2-\sqrt{20}\approx -6{,}47 \in\mathbb{D}\end{array}

Probe:
Für x_1:
\begin{array}{rcl}f\left(-2+\sqrt{20}\right) &=& \sqrt[4]{\left(-2+\sqrt{20}\right)^2+4\left(-2+\sqrt{20}\right)}-1\\&=& \sqrt[4]{4+20-4\sqrt{20}-8+4\sqrt{20}}-1\\&=& \sqrt[4]{16}-1\\&=& 2-1\\&=& 1\\\end{array}

Für x_2:
\begin{array}{rcl}f\left(-2-\sqrt{20}\right) &=& \sqrt[4]{\left(-2-\sqrt{20}\right)^2+4\left(-2-\sqrt{20}\right)}-1\\&=& \sqrt[4]{4+20+4\sqrt{20}-8-4\sqrt{20}}-1\\&=& \sqrt[4]{16}-1\\&=& 2-1\\&=& 1\\\end{array}

Ergebnis: Für x_1 = -2+\sqrt{20} sowie x_2 = -2-\sqrt{20} ergeben sich die richtigen Funktionswerte: P_2\left(-2+\sqrt{20} \mid 1 \right) und P_3\left(-2-\sqrt{20} \mid 1 \right) sind also tatsächlich Punkte des Graphen.


5)
a)
\begin{array}{rclll} f\left(\dfrac{5}{2}\right) &=& \sqrt{4\cdot \left(\dfrac{5}{2}\right)^2-2\cdot \left(\dfrac{5}{2}\right)}-4 &=& \sqrt{20}-4 \quad \rightarrow \quad P_1\left( \dfrac{5}{2} \mid \sqrt{20}-4\right) \end{array}

b)
\begin{array}{rclll}-5 &=& \sqrt{4x^2-2x}-4 &\vert & +4\cr -1 &=& \sqrt{4x^2-2x}\end{array}

Wer an dieser Stelle bemerkt, dass die Gleichung nicht lösbar ist (Da Wurzeln mit geraden Wurzelexponenten ja immer nichtnegativ sind, kann -1 als Ergebnis nicht sein!), kann die Rechnung abbrechen. Alle anderen rechnen zwei Werte aus, die aber nur Scheinlösungen sind. Denken Sie also bei solchen Aufgaben unbedingt daran, die Probe zu machen!


6)
a)
\begin{array}{rclll} f\left(1\right) &=& \dfrac{1}{32}\sqrt[3]{1^3-1} &=&0 \quad \rightarrow \quad P_1\left( 1 \mid 0\right) \end{array}

b)
\begin{array}{rclll}\dfrac{5}{16} &=& \dfrac{1}{32}\sqrt[3]{x^3-1} &\vert & \cdot 32\cr 10 &=& \sqrt[3]{x^3-1} &\vert & ()^3\cr 1.000 &=& x^3-1 &\vert & +1\cr 1.001 &=& x^3 &\vert & \sqrt[3]{ }\cr x &=&\sqrt[3]{1.001}\approx 10{,}0033 \in\mathbb{D}\end{array}

Probe:
\begin{array}{rcl}f\left(\sqrt[3]{1.001}\right) &=& \dfrac{1}{32} \cdot \sqrt[3]{\left(\sqrt[3]{1.001}\right)^3-1}\\\\&=& \dfrac{1}{32} \cdot \sqrt[3]{1.001-1}\\\\&=& \dfrac{1}{32} \cdot \sqrt[3]{1.000}\\\\&=& \dfrac{1}{32} \cdot 10\\\\&=& \dfrac{5}{16}\\\end{array}

Ergebnis: Für x = \sqrt[3]{1.001} ergibt sich der richtige Funktionswert: P_2\left(\sqrt[3]{1.001} \mid \dfrac{5}{16} \right) ist also tatsächlich ein Punkt des Graphen.


7)
a)
\begin{array}{rclll} f\left(11\right) &=&\dfrac{\sqrt{2\cdot 11+14}}{6}-14 &=&-13 \quad \rightarrow \quad P_1\left( 11 \mid -13\right) \end{array}

b)
\begin{array}{rclll}-1 &=& \dfrac{\sqrt{2x+14}}{6}-14 &\vert & +14\cr 13 &=& \dfrac{\sqrt{2x+14}}{6} &\vert & \cdot 6\cr 78 &=& \sqrt{2x+14} &\vert & ()^2\cr 6.084 &=& 2x+14 &\vert & -14\cr 6.070 &=& 2x &\vert & :2\cr x &=& 3.035 \in\mathbb{D}\end{array}

Probe:
\begin{array}{rcl}f(3.035) &=& \dfrac{\sqrt{2 \cdot 3.035+14}}{6}-14\\\\&=& \dfrac{\sqrt{6.084}}{6}-14\\\\&=& \dfrac{78}{6}-14\\&=& 13-14\\&=&-1\\\end{array}

Ergebnis: Für x = 3.035 ergibt sich der richtige Funktionswert: P_2\left(3.035 \mid -1 \right) ist also tatsächlich ein Punkt des Graphen.


8)
a)
\begin{array}{rclll} f(-2) &=& -\sqrt[4]{16-(-2)^4}+17 &=& 17 \quad \rightarrow \quad P_1(-2 \mid 17)\end{array}

b)
\begin{array}{rclll} 25 &=& -\sqrt[4]{16-x^4}+17 &\vert & -17 \cr 8 &=& -\sqrt[4]{16-x^4} &\vert & ()^4 \cr 4096 &=& 16-x^4 &\vert & +x^4 -4.096 \cr x^4 &=& -4.080 &\vert & \pm\sqrt[4]{} \cr x &=& \pm\sqrt[4]{-4.080} \end{array}

Ergebnis: Da aus negativen reellen Zahlen keine Wurzeln mit geraden Wurzelexponenten gezogen werden können, hat diese Gleichung keine Lösung. Das bedeutet, dass die Funktion nirgends den Funktionswert 25 annimmt.


9)
a)
\begin{array}{rclll} f(-11) &=& \dfrac{\sqrt{(-11)^2-9}}{4}-3 &=& \sqrt{7}-3 \quad \rightarrow \quad P_1(-11 \mid \sqrt{7}-3) \end{array}

b)
\begin{array}{rclll} 27 &=& \dfrac{\sqrt{x^2-9}}{4}-3 &\vert& +3 \cr 30 &=& \dfrac{\sqrt{x^2-9}}{4} &\vert& \cdot 4 \cr 120 &=& \sqrt{x^2-9} &\vert& ()^2 \cr 14.400 &=& x^2-9 &\vert& +9 \cr 14.409 &=& x^2 &\vert &\pm\sqrt{} \cr\cr x_1 &=& \sqrt{14.409}\approx 120{,}04 \in\mathbb{D}\cr x_2 &=& -\sqrt{14.409}\approx -120{,}04\in\mathbb{D} \end{array}

Probe:
Für x_1:
\begin{array}{rcl}f\left(\sqrt{14.409}\right) &=& \dfrac{\sqrt{\left(\sqrt{14.409}\right)^2-9}}{4}-3\\\\&=& \dfrac{\sqrt{14.409-9}}{4}-3\\\\&=& \dfrac{\sqrt{14.400}}{4}-3\\\\&=& \dfrac{120}{4}-3\\\\&=& 30-3\\&=& 27\\\end{array}

Für x_2:
\begin{array}{rcl}f\left(\sqrt{-14.409}\right) &=& \dfrac{\sqrt{\left(\sqrt{-14.409}\right)^2-9}}{4}-3\\\\&=& \dfrac{\sqrt{14.409-9}}{4}-3\\\\&=& \dfrac{\sqrt{14.400}}{4}-3\\\\&=& \dfrac{120}{4}-3\\\\&=& 30-3\\&=& 27\\\end{array}

Ergebnis: Für x_1 = -\sqrt{14.409} sowie  x_2 = \sqrt{14.409} ergeben sich die richtigen Funktionswerte: P_2\left(-\sqrt{14.409} \mid 27\right) und P_3\left(\sqrt{14.409} \mid 27\right) sind also tatsächlich Punkte des Graphen.


10)
a)
\begin{array}{rclll} f(32) &=& \sqrt[5]{32}-10 &=& -8 \quad \rightarrow \quad P_1(32 \mid -8)\end{array}

b)
\begin{array}{rclll} -7 &=& \sqrt[5]{x}-10 &\vert& +10 \cr 3 &=& \sqrt[5]{x} &\vert& ()^5 \cr x &=& 243 \in\mathbb{D}\end{array}

Probe:
\begin{array}{rcl}f(243) &=& \sqrt[5]{243}-10\\&=& 3-10\\&=& -7\\\end{array}

Ergebnis: Für x = 243 ergibt sich der richtige Funktionswert: P_2\left(243 \mid -7\right) ist also tatsächlich ein Punkt des Graphen.

 

3. Aufgabe

1)
Bestimmung des Definitionsbereichs:
\begin{array}{rclll} 2x-15 & \geq & 0 &\vert& :2 \\x-\dfrac{15}{2} &\geq & 0 &\vert& +\dfrac{15}{2} \\x &\geq& \dfrac{15}{2} \\\end{array}

Der Definitionsbereich ist also \mathbb{D} = \left[\dfrac{15}{2};\infty \right[.

Berechnung der Nullstellen:
\begin{array}{rclcll}0 &=& 20\sqrt{2x-15}-15 &\vert& +15 \\15 &=& 20\sqrt{2x-15} &\vert& :20 \\\dfrac{3}{4} &=& \sqrt{2x-15} &\vert& ()^2 \\\\\left(\dfrac{3}{4}\right)^2 &=& 2x-15 &\vert& +15 \\\\\dfrac{249}{16} &=& 2x &\vert & :2 \\\\x &=& \dfrac{249}{32}\approx 7{,}78\;\in\;\mathbb{D}\\\end{array}

Probe:
\begin{array}{rcll}f\left(\dfrac{249}{32}\right) &=& 20\sqrt{2\cdot\dfrac{249}{32}-15}-15 \\\\&=& 20\sqrt{\dfrac{9}{16}}-15 \\&=& 20\cdot\dfrac{3}{4}-15 \\&=& 0 \\\end{array}

Ergebnis: Die Nullstelle von f(x) liegt also bei x=\dfrac{249}{32}.


2)
Bestimmung des Definitionsbereichs:
\begin{array}{rclll} 100-x^2 & \geq & 0 & \vert & +x^2\\ 100 & \geq & x^2 & \vert & \pm\sqrt{} \\ 10 &\geq& \vert x\vert \end{array}

Der Definitionsbereich ist also \mathbb{D} = \left[-10;10 \right].

Berechnung der Nullstellen:
\begin{array}{rclcl} 0 &=& \sqrt{100-x^2} &\vert& ()^2 \cr 0 &=& 100-x^2 &\vert& +x^2 \cr x^2 &=& 100 &\vert& \pm\sqrt{} \cr x_{1,2} &=& \pm \sqrt{100} \cr\cr x_1 &=& 10\;\in\;\mathbb{D} \cr x_2 &=& -10\;\in\;\mathbb{D} \end{array}

Probe:
Für x_1:
\begin{array}{rclll} g(10) &=& \sqrt{100-(10)^2} \cr &=& 0 \end{array}

Für x_2:
\begin{array}{rclll} g(-10) &=& \sqrt{100-(-10)^2} \cr &=& 0 \end{array}

Ergebnis: Die Nullstellen von g(x) liegen also bei x_1=10 und x_2=-10.


3)
Definitionsbereich: \mathbb{D} = \mathbb{R}

Berechnung der Nullstellen:
\begin{array}{rclcll}0 &=& 2\sqrt[3]{t^2-8}-1 &\vert& +1 \\1 &=& 2\sqrt[3]{t^2-8} &\vert& :2 \\ \\\dfrac{1}{2} &=& \sqrt[3]{t^2-8} &\vert& ()^3 \\ \\\left(\dfrac{1}{2}\right)^3 &=& t^2-8 \\ \\\dfrac{1}{8} &=& t^2-8 &\vert& +8 \\ \\\dfrac{65}{8} &=& t^2 &\vert& \pm\sqrt{} \\ \\t_{1,2} &=& \pm\sqrt{\dfrac{65}{8}} \\ \\t_1 &=& \sqrt{\dfrac{65}{8}} \approx 2{,}85 \;\in\;\mathbb{D} \\ \\t_2 &=& -\sqrt{\dfrac{65}{8}} \approx -2{,}85 \;\in\;\mathbb{D} \\\end{array}

Probe:
Für t_1:
\begin{array}{rcllll}f\left(\sqrt{\dfrac{65}{8}}\right) &=& 2\sqrt[3]{\left(\sqrt{\dfrac{65}{8}}\right)^2-8}-1 \\ \\&=& 2\sqrt[3]{\dfrac{1}{8}}-1 \\&=& 2\cdot\dfrac{1}{2}-1 \\&=& 0 \end{array}

Für t_2:
\begin{array}{rcllll}f\left(-\sqrt{\dfrac{65}{8}}\right) &=& 2\sqrt[3]{\left(-\sqrt{\dfrac{65}{8}}\right)^2-8}-1 \\ \\&=& 2\sqrt[3]{\dfrac{1}{8}}-1 \\&=& 0\end{array}

Ergebnis: Die Nullstellen von f(t) liegen also bei t_1=\sqrt{\dfrac{65}{8}} und bei t_2=-\sqrt{\dfrac{65}{8}}.


4)
Bestimmung des Definitionsbereichs:
\begin{array}{crclll} & x^3-x^2 & \geq & 0 \\& x^2(x-1) & \geq & 0\end{array}

Das Produkt zweier Faktoren ist genau dann nichtnegativ, wenn entweder mindestens ein Faktor 0 ist oder beide Faktoren das gleiche Vorzeichen besitzen. Der erste Faktor ist ein Quadrat, das immer nichtnegativ ist. Für x=0 wird das Quadrat 0, sodass dieser Wert auf jeden Fall Teil des Definitionsbereichs ist.
Nun stellt sich die Frage, wann der zweite Faktor ebenfalls positiv oder 0 wird. Dazu lösen wir folgende Ungleichung:
\begin{array}{rclll}x-1 &\geq & 0 &\vert & +1\\x &\geq & 1\end{array}

Das bedeutet, dass im Intervall [1;\infty [ beide Faktoren das gleiche Vorzeichen haben bzw. der zweite Faktor 0 ist.

Für die abschließende Ermittlung des Definitionsbereichs muss nun überprüft werden, für welche x-Werte alle ermittelten Bedingungen erfüllt sind. Das ist nur der Fall für x=0 oder für x\geq 1. Der Definitionsbereich ist also \mathbb{D} = \left\{x \in \mathbb{R} \mid x=0 \text{ oder } x \geq 1\right\}.

Berechnung der Nullstellen:
\begin{array}{crclcl}& 0 &=& \sqrt{x^3-x^2} &\vert& ()^2 \\& 0 &=& x^3-x^2 \\& 0 &=& x^2\cdot(x-1) &\vert& \text{Satz vom Nullprodukt} \\\\\text{Faktor 1:} & 0 &=& x^2 &\vert& \pm\sqrt{} \\& x_1 &=& 0\;\in\;\mathbb{D} \\ \\\text{Faktor 2:} & 0 &=& x-1 &\vert &+1 \\& x_2 &=& 1\;\in\;\mathbb{D} \\\end{array}

Probe:
Für x_1:
\begin{array}{rcll}f(0) &=& \sqrt{0^3-0^2}\\&=& \sqrt{0}\\&=& 0 \end{array}

Für x_2:
\begin{array}{rcll}f(1) &=& \sqrt{1^3-1^2}\\&=& \sqrt{0}\\&=& 0\end{array}

Ergebnis: Die Nullstellen von f(x) liegen also bei x_1=0 und bei x_2=1.


5)
Bestimmung des Definitionsbereichs:
\begin{array}{rclll} x+7 &\geq& 0 &\vert& -7\\x &\geq& -7\end{array}

Der Definitionsbereich ist also \mathbb{D} = \left[-7;\infty \right[.

Berechnung der Nullstellen:
\begin{array}{rclcll}0 &=& \dfrac{\sqrt{x+7}}{2}+77 &\vert& -\dfrac{\sqrt{x+7}}{2} \\-\dfrac{\sqrt{x+7}}{2} &=& 77 &\vert& \cdot (-2) \\\sqrt{x+7} &=& -154 &\vert& ()^2 \\x+7 &=& 23.716 &\vert& -7 \\x &=& 23.709\;\in\;\mathbb{D} \end{array}

Probe:
\begin{array}{rcll}f(23.709) &=& \dfrac{\sqrt{23.709+7}}{2}+77 \\&=& \dfrac{\sqrt{23.716}}{2}+77 \\&=& \dfrac{154}{2}+77 \\&=& 154 \\&\neq& 0\end{array}

Ergebnis: Es ergibt sich ein falscher Funktionswert. Die Funktion f(x) hat also keine Nullstellen.

Bemerkung: Wer in der Zeile \sqrt{x+7} = -154 genau hinschaut, merkt schon dort, dass wir keine Nullstellen erhalten: Auf der linken Seite steht ja eine Wurzel, die immer nichtnegativ ist und damit niemals -154 werden kann.


6)
Bestimmung des Definitionsbereichs:
\dfrac{x^4}{2} \geq 0

Da eine Potenz mit geradem Exponenten immer nichtnegativ ist, ist der Radikand als Quotient aus einer solcher Potenz und der Zahl 2 ebenfalls immer nichtnegativ. Der Definitionsbereich ist also \mathbb{D} = \mathbb{R}.

Berechnung der Nullstellen:
\begin{array}{crclcl}& 0 &=& \sqrt{\dfrac{x^4}{2}}-x &\vert& +x \\ \\& x &=& \sqrt{\dfrac{x^4}{2}} &\vert& ()^2 \\ \\& x^2 &=& \dfrac{x^4}{2} &\vert &\cdot 2 \\ \\& 2x^2 &=& x^4 &\vert& -2x^2 \\& 0 &=& x^4-2x^2 \\& 0 &=& x^2\cdot(x^2-2) &\vert& \text{Satz vom Nullprodukt} \\\\\text{Faktor 1:} & 0 &=& x^2 &\vert &\pm\sqrt{} \\& x_1 &=& 0\;\in\;\mathbb{D} \\ \\\text{Faktor 2:} & 0 &=& x^2-2 &\vert& +2 \\& 2 &=& x^2 &\vert &\pm\sqrt{} \\ \\& x_2 &=& \sqrt{2}\;\in\;\mathbb{D} \\& x_3 &=& -\sqrt{2}\;\in\;\mathbb{D}\end{array}

Probe:
Für x_1:
\begin{array}{rcll}r(0) &=& \sqrt{\dfrac{0^4}{2}}-0 \\&=& 0 \end{array}

Für x_2:
\begin{array}{rcll}r\left(\sqrt{2}\right) &=& \sqrt{\dfrac{\left(\sqrt{2}\right)^4}{2}}-\sqrt{2} \\&=& \sqrt{\dfrac{4}{2}}-\sqrt{2} \\&=& \sqrt{2}-\sqrt{2} \\&=& 0 \end{array}

Für x_3:
\begin{array}{rcll}r\left(-\sqrt{2}\right) &=& \sqrt{\dfrac{\left(-\sqrt{2}\right)^4}{2}}-\left(-\sqrt{2}\right) \\&=& \sqrt{2}+\sqrt{2} \\&=& 2\sqrt{2} \\&\neq& 0\end{array}

Ergebnis: Die Nullstellen von r(x) liegen also bei x_1=0 und bei x_2=\sqrt{2}.


7)
Bestimmung des Definitionsbereichs:
\begin{array}{rclll}x^3 &>& 0 &\vert & \sqrt[3]{}\\x &>& 0\end{array}

Der Definitionsbereich ist also \mathbb{D} = \mathbb{R}^+.

Bemerkung: Da x^3 nicht nur unter der Quadratwurzel steht, sondern die Wurzel auch im Nenner zu finden ist, muss bei dieser Aufgabe zum einen - wie üblich bei Wurzeln mit geraden Wurzelexponenten - ausgeschlossen werden, dass der Radikand negativ wird. Zum anderen darf der Radikand auch nicht 0 werden, da ansonsten der Nenner 0 wird. Deswegen wurde die Ungleichung nicht mit "größer gleich" sondern mit "echt größer" formuliert.

Berechnung der Nullstellen:
\begin{array}{rclcll}0 &=& \dfrac{1}{\sqrt[4]{x^3}}-\sqrt[3]{x^2} &\vert& +\sqrt[3]{x^2} \\ \\\sqrt[3]{x^2} &=& \dfrac{1}{\sqrt[4]{x^3}} \\ \\x^{\frac{2}{3}} &=& \dfrac{1}{x^{\frac{3}{4}}} &\vert& \cdot x^{\frac{3}{4}} \\x^{\frac{2}{3}}\cdot x^{\frac{3}{4}} &=& 1 \\x^{\frac{17}{12}} &=& 1 &\vert& ()^{\frac{12}{17}} \\x &=& 1\;\in\;\mathbb{D}\end{array}

Probe:
\begin{array}{rcll}f(1) &=& \dfrac{1}{\sqrt[4]{1^3}}-\sqrt[3]{1^2} \\ \\&=& \dfrac{1}{1}-1 \\ \\&=& 0\end{array}

Ergebnis: Die Nullstelle von f(x) liegt also bei x=1.

Bemerkung: Die Multiplikation mit x^{\frac{3}{4}} ist hier ohne Einschränkungen möglich, weil x=0 \not\in \mathbb{D}. Eine Multiplikation mit 0 kann also nicht passieren.


8)
Bestimmung des Definitionsbereichs:
\begin{array}{rclll} x^3+3 &\geq& 0 &\vert& -3 \\x^3 &\geq& -3 &\vert& \sqrt[3]{} \\x &\geq& \sqrt[3]{-3}\\x &\geq& -\sqrt[3]{\vert -3\vert}\end{array}

Der Definitionsbereich ist also \mathbb{D} = \left[-\sqrt[3]{3};\infty \right[.

Berechnung der Nullstellen:
\begin{array}{rclcll}0 &=& 3-\sqrt{x^3+3} &\vert& +\sqrt{x^3+3} \\\sqrt{x^3+3} &=& 3 &\vert& ()^2 \\x^3+3 &=& 9 &\vert& -3 \\x^3 &=& 6 &\vert& \sqrt[3]{} \\x &=& \sqrt[3]{6}\;\in\;\mathbb{D}\end{array}

Probe:
\begin{array}{rcll}f(\sqrt[3]{6}) &=& 3-\sqrt{\sqrt[3]{6}^3+3} \\&=& 3-\sqrt{9} \\&=& 0\end{array}

Ergebnis: Die Nullstelle von f(x) liegt also bei x=\sqrt[3]{6}.


9)
Definitionsbereich: \mathbb{D} = \mathbb{R}

Berechnung der Nullstellen:
\begin{array}{rclcll} 0 &=& \sqrt[3]{x^2-9}+4 &\vert& -4 \\ -4 &=& \sqrt[3]{x^2-9} &\vert& ()^3 \\ -64 &=& x^2-9 &\vert& +9 \\ -55 &=& x^2 &\vert& \pm\sqrt{} \\ \pm\sqrt{-55} &=& x \\ \end{array}

Ergebnis: Da aus negativen reellen Zahlen keine Wurzeln mit geraden Wurzelexponenten gezogen werden können, hat diese Funktion keine Nullstellen.


10)
Bestimmung des Definitionsbereichs:
\begin{array}{rclll} x+15 &\geq& 0 &\vert& -15\\x &\geq& -15\end{array}

Der Definitionsbereich ist also \mathbb{D} = \left[-15;\infty \right[.

Berechnung der Nullstellen:
\begin{array}{rclcll}0 &=& -2\sqrt{x+15}+7 &\vert& +2\sqrt{x+15} \\2\sqrt{x+15} &=& 7 &\vert& :2 \\ \\\sqrt{x+15} &=& \dfrac{7}{2} &\vert& ()^2 \\ \\x+15 &=& \dfrac{49}{4} &\vert& -15 \\ \\x &=& -\dfrac{11}{4}\;\in\;\mathbb{D}\end{array}

Probe:
\begin{array}{rcll}f\left(-\dfrac{11}{4}\right) &=& -2\cdot\sqrt{-\dfrac{11}{4}+15}+7 \\ \\&=& -2\cdot\sqrt{\dfrac{49}{4}}+7 \\ \\&=& -2\cdot\dfrac{7}{2}+7 \\ \\&=& 0\end{array}

Ergebnis: Die Nullstelle von f(x) liegt also bei x=-\dfrac{11}{4}.


11)
Bestimmung des Definitionsbereichs:
\begin{array}{rclll} 17+x &>& 0 &\vert& -17 \\x &>& -17\end{array}

Der Definitionsbereich ist also \mathbb{D}=\left]-17;\infty \right[.

Bemerkung: Da 17+x nicht nur unter der Quadratwurzel steht, sondern die Wurzel auch im Nenner zu finden ist, muss bei dieser Aufgabe zum einen - wie üblich bei Wurzeln mit geraden Wurzelexponenten - ausgeschlossen werden, dass der Radikand negativ wird. Zum anderen darf der Radikand auch nicht 0 werden, da ansonsten der Nenner 0 wird. Deswegen wurde die Ungleichung nicht mit "größer gleich" sondern mit "echt größer" formuliert.

Berechnung der Nullstellen:
\begin{array}{rclcl}0 &=& \dfrac{\sqrt[3]{\left(x^2+8x+16\right)^3}}{\sqrt{17+x}} \\\\0 &=& \dfrac{x^2+8x+16}{\sqrt{17+x}} \\\\0 &=& \dfrac{\left(x+4\right)^2}{\sqrt{17+x}} &\vert& \cdot \sqrt{17+x} \\\\0 &=& \left(x+4\right)^2 &\vert& \pm\sqrt{} \\ 0 &=& x+4 &\vert& -4 \\ x &=& -4 \in \mathbb{D} \end{array}

Probe:
\begin{array}{rcl}f(-4) &=& \dfrac{\sqrt[3]{\left((-4)^2+8\cdot(-4)+16\right)^3}}{\sqrt{17-4}} \\\\&=& \dfrac{\sqrt[3]{\left(16-32+16\right)^3}}{\sqrt{13}} \\\\&=& \dfrac{\sqrt[3]{0^3}}{\sqrt{13}}\\\\&=& 0\end{array}

Ergebnis: Die Nullstelle von f(x) liegt also bei x=-4.

Bemerkung: Die Multiplikation mit \sqrt{17+x} ist hier ohne Einschränkungen möglich, weil x=-17 \not\in \mathbb{D}. Eine Multiplikation mit 0 kann also nicht passieren.


12)
Bestimmung des Definitionsbereichs:
1. Radikand links:
\begin{array}{rclll} 4x+8 &\geq& 0 &\vert& -8 \\4x &\geq& -8 &\vert& :4 \\x &\geq& -2 \end{array}

2. Radikand links:
\begin{array}{rclll}12x-24 &\geq& 0 &\vert& +24 \\12x &\geq& 24 &\vert& :12 \\x &\geq& 2 \\\end{array}

Für die abschließende Ermittlung des Definitionsbereichs muss nun überprüft werden, für welche x-Werte alle ermittelten Bedingungen erfüllt sind. Das ist nur der Fall für x\geq 2. Der Definitionsbereich ist also \mathbb{D}=\left[2;\infty \right[.

Berechnung der Nullstellen:
\begin{array}{rclcl}\sqrt{4x+8}+\sqrt{12x-24} &=& 0 &\vert& -\sqrt{12x-24} \\\sqrt{4x+8} &=& -\sqrt{12x-24} &\vert& ()^2 \\4x+8 &=& 12x-24 &\vert& -8 -12x \\-8x &=& -32 &\vert& :(-8) \\x &=& 4 \in \mathbb{D} \\\end{array}

Probe:
\begin{array}{rcl}f(4) &=& \sqrt{4\cdot 4+8}+\sqrt{12\cdot 4-24} \\&=& \sqrt{24}+\sqrt{24} \\&\neq& 0\end{array}

Ergebnis: Es ergibt sich ein falscher Funktionswert. Die Funktion f(x) hat also keine Nullstellen.


13)
Definitionsbereich: \mathbb{D}=\mathbb{R}

Berechnung der Nullstellen:
\begin{array}{lrclcl}& 0 &=& -34x\cdot\sqrt[5]{\dfrac{11x}{32}}+51x \\& 0 &=& x\cdot\left(-34\cdot\sqrt[5]{\dfrac{11x}{32}}+51\right) &\vert& \text{Satz vom Nullprodukt} \\\\\text{Faktor 1:} & x_1 &=& 0 \in \mathbb{D}\\\\\\\text{Faktor 2:} & 0 &=& -34\cdot\sqrt[5]{\dfrac{11x}{32}}+51 &\vert& -51 \\\\& -51 &=& -34\cdot\sqrt[5]{\dfrac{11x}{32}} &\vert& :(-34) \\\\& \dfrac{3}{2} &=& \sqrt[5]{\dfrac{11x}{32}} &\vert& ()^5 \\\\& \dfrac{243}{32} &=& \dfrac{11x}{32} &\vert& :\dfrac{11}{32} \\\\& x_2 &=& \dfrac{243}{11} \in \mathbb{D}\end{array}

Probe:
Für x_1:
\begin{array}{rcl}f(0) &=& -34\cdot 0 \cdot\sqrt[5]{\dfrac{11\cdot 0}{32}}+51\cdot 0 \\&=& 0 \cdot\sqrt[5]{0}+0 \\&=& 0 \\\end{array}

Für x_2:
\begin{array}{rcl}f\left(\dfrac{243}{11}\right)&=& -34\cdot\dfrac{243}{11}\cdot\sqrt[5]{\genfrac{}{}{1pt}{0}{11\cdot\frac{243}{11}}{32}}+51\cdot\dfrac{243}{11}\\\\&=& -\dfrac{8.262}{11}\cdot\dfrac{3}{2}+\dfrac{12.393}{11}\\\\&=& -\dfrac{12.393}{11}+\dfrac{12.393}{11} \\&=& 0\end{array}

Ergebnis: Die Nullstellen von f(x) liegen also bei x_1=0 und x_2=\dfrac{243}{11}.


14)
Definitionsbereich: \mathbb{D} = \mathbb{R}

Berechnung der Nullstellen:
\begin{array}{rclcl}0 &=& \sqrt[3]{x^3-37\cdot10^3}+40 &\vert& -40 \\-40 &=& \sqrt[3]{x^3-37.000} &\vert& ()^3 \\ -64.000 &=& x^3-37.000 &\vert& +37.000 \\-27.000 &=& x^3 &\vert& \sqrt[3]{} \\-30 &=& x \in \mathbb{D} \\ \end{array}

Probe:
\begin{array}{rcl}f(-30) &=& \sqrt[3]{(-30)^3-37\cdot10^3}+40 \\&=& \sqrt[3]{-64.000}+40 \\&=& -40+40 \\&=& 0 \\\end{array}

Ergebnis: Die Nullstelle von f(x) liegt also bei x=-30.


15)
Bestimmung des Definitionsbereichs:
innere Radikand:
\begin{array}{rclcl}49^{\frac{1}{2}}\cdot x &\geq & 0 &\vert & :49^{\frac{1}{2}} \\x &\geq & 0\end{array}

äußerer Radikand:
Der Radikand der äußeren Quadratwurzel \sqrt[2]{49^{\frac{1}{2}}\cdot x}\cdot 19 muss nicht geprüft werden, da das Produkt zwingend nichtnegativ sein muss. Es wird ja der nichtnegative Wert der inneren Quadratwurzel mit einer positiven Zahl multipliziert.

Der Definitionsbereich ist also \mathbb{D} = \mathbb{R}^+_0.

Berechnung der Nullstellen:
\begin{array}{rclcl}0 &=& \sqrt[2]{\sqrt[2]{49^{\frac{1}{2}}\cdot x}\cdot 19}\\0 &=& \sqrt[4]{7x}\cdot\sqrt{19} &\vert& :\sqrt{19}\\0 &=& \sqrt[4]{7x} &\vert& ()^4\\ 0 &=& 7x &\vert& :7\\0 &=& x \in \mathbb{D} \end{array}

Probe:
\begin{array}{rcl}f(0) &=& \sqrt[2]{\sqrt[2]{49^{\frac{1}{2}}\cdot 0}\cdot 19}\\&=& 0\end{array}

Ergebnis: Die Nullstelle von f(x) liegt also bei x=0.


16)
Bestimmung des Definitionsbereichs:
\begin{array}{rclll}-63x+9x^2 &\geq& 0 &\vert& :9 \\-7x+x^2 &\geq& 0 &\vert& +7x \\x^2 &\geq& 7x\end{array}

Hier ist eine Fallunterscheidung nötig, da wir durch die Variable dividieren.

Fall 1: Wir nehmen an, dass x>0 ist.
\begin{array}{rcllll}x^2&\geq& 7x &\vert& :x \\x &\geq& 7\end{array}

Im Abgleich zwischen Eintrittsbedingung und Lösung des 1. Falls ergibt sich folgendes Zwischenergebnis: x \geq 7

Fall 2: Wir nehmen an, dass x < 0 ist.
\begin{array}{rcllll}x^2&\geq& 7x &\vert& :x \\x &\leq& 7\end{array}

Im Abgleich zwischen Eintrittsbedingung und Lösung des 2. Falls ergibt sich folgendes Zwischenergebnis: x < 0

Fall 3: Wir nehmen an, dass x=0 ist.
\begin{array}{rcllll}x^2&\geq& 7x &\vert& \text{0 einsetzen} \\0 &\geq& 0\end{array}

Im Abgleich zwischen Eintrittsbedingung und Lösung des 3. Falls ergibt sich folgendes Zwischenergebnis: x=0

Die Fallunterscheidung liefert uns x \leq 0 oder x \geq 7. Der Definitionsbereich ist also \mathbb{D}=\left\{x\in\mathbb{R}\mid x\leq 0 \text{ oder } x \geq 7 \right\}.

Berechnung der Nullstellen:
\begin{array}{lrclcl}&0 &=& \dfrac{7}{8} \sqrt{-63x+9x^2} &\vert& : \dfrac{7}{8}\\&0 &=& \sqrt{-63x+9x^2} &\vert& ()^2\\&0 &=& -63x+9x^2 \\&0 &=& x \cdot (63+9x) &\vert& \text{Satz vom Nullprodukt}\\\\\text{Faktor 1: } & x_1 &=& 0 \in \mathbb{D}\\\\\text{Faktor 2: } & -63+9x &=& 0 &\vert& +63\\& 9x &=& 63 &\vert& :9\\& x_2 &=& 7 \in \mathbb{D}\\\end{array}

Probe:
Fürx_1:
\begin{array}{rcl}f(0) &=& \dfrac{7}{8} \sqrt{-63\cdot 0+9\cdot 0^2} \\\\&=& \dfrac{7}{8} \sqrt{-63\cdot 0+9\cdot 0^2} \\\\&=& \dfrac{7}{8} \sqrt{0} \\\\&=& 0\end{array}

Fürx_2:
\begin{array}{rcl}f(7) &=& \dfrac{7}{8} \sqrt{-63\cdot 7+9\cdot 7^2} \\\\&=& \dfrac{7}{8} \sqrt{-441+441} \\\\&=& \dfrac{7}{8} \sqrt{0} \\\\&=& 0\end{array}

Ergebnis: Die Nullstellen von f(x) liegen also bei x_1=0 und x_2=7.


17)
Bestimmung des Definitionsbereichs:
1. Radikand rechts:
104x^2+36 \geq 0
Da ein Quadrat im Bereich der reellen Zahlen immer nichtnegativ ist, ist der Radikand als Summe aus einem Quadrat mit positivem Koeffizienten und der Zahl 36 immer positiv. Somit folgt hieraus keine Einschränkung für den Definitionsbereich.

2. Radikand rechts:
224x^2+11 \geq 0
Für den zweiten Radikanden gilt die gleiche Argumentation wie für den ersten.

Der Definitionsbereich ist also \mathbb{D} = \mathbb{R}.

Berechnung der Nullstellen:
\begin{array}{rclcl}0 &=& \sqrt{104x^2+36}-\sqrt{225x^2+11} &\vert& +\sqrt{225x^2+11}\\\sqrt{225x^2+11} &=& \sqrt{104x^2+36} &\vert& ()^2\\225x^2+11 &=& 104x^2+36 &\vert& -11 -104x^2\\121x^2 &=& 25 &\vert& :121\\\\x^2 &=& \dfrac{25}{121} &\vert& \pm\sqrt{}\\\\x_{1} &=& \dfrac{5}{11} \in \mathbb{D} \\x_{2} &=& -\dfrac{5}{11} \in \mathbb{D}\end{array}

Probe:
Für x_{1}:
\begin{array}{rcl}f\left(\dfrac{5}{11}\right) &=& \sqrt{104\cdot\left(\dfrac{5}{11}\right)^2+36}-\sqrt{225\cdot\left(\dfrac{5}{11}\right)^2+11}\\&=& \sqrt{\dfrac{6.956}{121}}-\sqrt{\dfrac{6.956}{121}}\\&=& 0\end{array}

Für x_{2}:
\begin{array}{rcl}f\left(-\dfrac{5}{11}\right) &=& \sqrt{104\cdot\left(-\dfrac{5}{11}\right)^2+36}-\sqrt{225\cdot\left(-\dfrac{5}{11}\right)^2+11}\\&=& \sqrt{\dfrac{6.956}{121}}-\sqrt{\dfrac{6.956}{121}}\\&=& 0\end{array}

Ergebnis: Die Nullstellen von f(x) liegen also bei x_{1}=\dfrac{5}{11} und x_{2}=-\dfrac{5}{11}.


18)
Bestimmung des Definitionsbereichs:
\begin{array}{rclcl}15x^3 &\geq& 0 &\vert& :15 \\x^3 &\geq& 0 &\vert& \sqrt[3]{} \\x &\geq& 0 \end{array}

Der Definitionsbereich ist also \mathbb{D}=\mathbb{R}^+_0.

Berechnung der Nullstellen:
\begin{array}{rclcl}0 &=& \dfrac{\sqrt{15x^3}}{\sqrt{5}}-\left(\sqrt{5}\right)^3 &\vert& +\left(\sqrt{5}\right)^3 \\\\\left(\sqrt{5}\right)^3 &=& \dfrac{\sqrt{15x^3}}{\sqrt{5}} &\vert& \cdot\sqrt{5} \\\\\left(\sqrt{5}\right)^4 &=& \sqrt{15x^3} \\\\25 &=& \sqrt{15x^3} &\vert& ()^2 \\\\625 &=& 15x^3 &\vert& :15 \\\\\dfrac{125}{3} &=& x^3 &\vert& \sqrt[3]{} \\\\\sqrt[3]{\dfrac{125}{3}} &=& x \in \mathbb{D}\end{array}

Probe:
\begin{array}{rcl}f\left(\sqrt[3]{\dfrac{125}{3}}\right) &=& \genfrac{}{}{1pt}{0}{\sqrt{15\cdot\left(\sqrt[3]{\frac{125}{3}}\right)^3}}{\sqrt{5}}-\left(\sqrt{5}\right)^3\\\\&=& \genfrac{}{}{1pt}{0}{\sqrt{15\cdot\frac{125}{3}}}{\sqrt{5}}-\left(\sqrt{5}\right)^3 \\\\&=& \dfrac{\sqrt{625}}{\sqrt{5}}-\left(\sqrt{5}\right)^3 \\\\&=& \dfrac{\sqrt{5^4}}{\sqrt{5}}-\left(\sqrt{5}\right)^3 \\\\&=& \sqrt{5^3}-\left(\sqrt{5}\right)^3 \\&=& 0\end{array}

Ergebnis: Die Nullstelle von f(x) liegt also bei x=\sqrt[3]{\dfrac{125}{3}}.


19)
Bestimmung des Definitionsbereichs:
\begin{array}{rclcl}8x &>& 0 &\vert & :8 \\x &>& 0\end{array}

Der Definitionsbereich ist also \mathbb{D} = \mathbb{R}^+.

Bemerkung: Da 8x nicht nur unter der Quadratwurzel steht, sondern die Wurzel auch im Nenner zu finden ist, muss bei dieser Aufgabe zum einen - wie üblich bei Wurzeln mit geraden Wurzelexponenten - ausgeschlossen werden, dass der Radikand negativ wird. Zum anderen darf der Radikand auch nicht 0 werden, da ansonsten der Nenner 0 wird. Deswegen wurde die Ungleichung nicht mit "größer gleich" sondern mit "echt größer" formuliert.

Berechnung der Nullstellen:
\begin{array}{rclcl}0 &=& \dfrac{\sqrt{64}}{\sqrt{8x}}-128 &\vert& +128 \\\\128 &=& \dfrac{8}{\sqrt{8x}} &\vert& : 8 \\\\16 &=& \dfrac{1}{\sqrt{8x}} &\vert& \text{Kehrwert} \\\\\dfrac{1}{16} &=& \sqrt{8x} &\vert& ()^2 \\\\\dfrac{1}{256} &=& 8x &\vert& :8 \\\\\dfrac{1}{2.048} &=& x \in \mathbb{D}\end{array}

Probe:
\begin{array}{rcl}f\left(\dfrac{1}{2.048}\right) &=& \genfrac{}{}{1pt}{0}{\sqrt{64}}{\sqrt{8\cdot\frac{1}{2.048}}}-128\\\\&=& \dfrac{8}{\frac{1}{16}}-128\\\\&=& 128-128\\&=& 0\end{array}

Ergebnis: Die Nullstelle von f(x) liegt also bei x=\dfrac{1}{2.048}.


20)
Definitionsbereich: \mathbb{D} = \mathbb{R}

Berechnung der Nullstellen:
\begin{array}{lrclcl}& 0 &=& \dfrac{x}{\sqrt{3}}+\sqrt[3]{3x^2}\cdot \sqrt[3]{3^2} &\vert& -\dfrac{x}{\sqrt{3}} \\\\& -\dfrac{x}{\sqrt{3}} &=& \sqrt[3]{3x^2\cdot3^2} \\\\& -\dfrac{x}{\sqrt{3}} &=& \sqrt[3]{27x^2} &\vert& ()^3 \\\\& -\dfrac{x^3}{\left(\sqrt{3}\right)^3} &=& 27x^2 &\vert& +\dfrac{x^3}{\left(\sqrt{3}\right)^3} \\\\& 0 &=& 27x^2+\dfrac{x^3}{\left(\sqrt{3}\right)^3} \\\\& 0 &=& x^2\cdot\left(27+\dfrac{x}{\left(\sqrt{3}\right)^3}\right) &\vert& \text{Satz vom Nullprodukt} \\\\\text{Faktor 1:} & x^2 &=& 0 &\vert& \pm\sqrt{} \\\\& x_1 &=& 0 \in \mathbb{D} \\\\\\\text{Faktor 2:} & 0 &=& 27+\dfrac{x}{\left(\sqrt{3}\right)^3} &\vert& -\dfrac{x}{\left(\sqrt{3}\right)^3} \\\\& -\dfrac{x}{\left(\sqrt{3}\right)^3} &=& 3^3 &\vert& \cdot \left(-\left(\sqrt{3}\right)^3\right) \\\\& x &=& 3^3\cdot\left(-\left(\sqrt{3}\right)^3\right) \\& x &=& -(\sqrt{3})^{2 \cdot 3}\cdot (\sqrt{3})^3 \\& x &=& -(\sqrt{3})^9 \\\\& x_2 &=& -\sqrt{3^9} \approx -140{,}30 \in \mathbb{D}\end{array}

Probe:
Für x_1
\begin{array}{rcl}f(0) &=& \dfrac{0}{\sqrt{3}}+\sqrt[3]{3\cdot0^2}\cdot \sqrt[3]{3^2} \\&=& 0+0 \\&=& 0\end{array}

Für x_2
\begin{array}{rcl}f\left(-\sqrt{3^9}\right) &=& \dfrac{-\sqrt{3^9}}{\sqrt{3}}+\sqrt[3]{3\cdot\left(-\left(\sqrt{3^9}\right)\right)^2}\cdot \sqrt[3]{3^2} \\&=& -81+81 \\&=& 0\end{array}

Ergebnis: Die Nullstellen von f(x) liegen also bei x_1=0 undx_2=-\sqrt{3^9} .

16. Exponentialgleichungen und -funktionen - Lernziele und typische Fehler

Nach Durcharbeiten dieses Kapitels sollten Sie folgende Lernziele erreicht haben:

  • Sie können zu einer Exponentialgleichung den passenden Definitionsbereich bestimmen.
  • Sie kennen die allgemeine Form einer Exponentialgleichung.
  • Sie können Exponentialgleichungen lösen.
  • Sie können die Lösungsmenge mathematisch korrekt notieren.
  • Sie können mithilfe der Probe überprüfen, ob die gefundene Lösung tatsächlich richtig ist.
  • Sie können zu einer Exponentialfunktion den passenden Definitionsbereich bestimmen.
  • Sie wissen, wie der Graph einer Exponentialfunktion typischerweise aussieht, und können ihn in ein kartesisches Koordinatensystem zeichnen.
  • Sie kennen die allgemeine Funktionsgleichung einer Exponentialfunktion.
  • Sie kennen Eigenschaften von Exponentialfunktionen (Randverhalten, typische Punkte) und können diese nutzen, um Beziehungen zwischen Funktionsterm und Funktionsgraphen herzustellen.
  • Sie kennen den Zusammenhang zwischen Exponentialgleichungen und Exponentialfunktionen.
  • Sie können Exponentialgleichungen von anderen Gleichungsarten unterscheiden.
  • Sie können Exponentialfunktionen von anderen Funktionstypen unterscheiden (grafisch und anhand der Funktionsgleichung).


Typische Fehler in diesem Kapitel sind:

  • Beim Logarithmieren der Gleichung werden nicht alle Bestandteile der Gleichung logarithmiert. Erklärung
  • Das dritte Logarithmengesetz wird nicht richtig angewendet, insbesondere wenn im Exponenten eine Summe steht. Erklärung


Für Online-Selbsttests zu diesem Thema und weitere Informationen zur Mathematikunterstützung an der TH Wildau nutzen Sie bitte den Moodle-Kursraum "SOS Mathematik - Brückenkurs".

Übersicht:

 

16.1 Exponentialgleichungen und -funktionen - Aufgaben

1. Aufgabe

Lösen Sie folgende Exponentialgleichungen! Geben Sie jeweils den Definitionsbereich an!

1) 26^x = 15

  11) -33+5^{3x-7}=12

2) 2^{5+x}=8^x

  12) 7 \cdot e^{15x^2-30x}=21

3) 27^x+4=0

  13) 5^{2t}-16 \cdot 5^t=0

4) e^{2x+3}=2 \cdot e^x

  14) 4 \cdot e^{25x+18}-1=51

5) 10^x=100^{x+1}

  15) e^x \cdot e^{2x}-e^{18}=0

6) 11^{x+2}=8

  16) 2^{5z^2} \cdot 2^{95z-45} \cdot 2^{-55}=1

7) 1{,}06^{x-1}=2{,}08^x

  17) \dfrac{1.000^{x+1}}{10^{5x}}-12=43

8) 17^s=e

  18) 2 \cdot 10^{2x} - 16 \cdot 10^x+30=0

9) 66^y=1

  19) -9 \cdot e^{4x}+45 \cdot e^{2x}=0

10) 48^{\frac{1}{18}x-39}=48

  20) 3\left(\dfrac{1}{3} x^2-112\right) e^{2x+6} = -\dfrac{109x}{e^{-2x-6}}

 

2. Aufgabe

1000 EUR werden zu einem Zinssatz von 2{,}5 \;\% p.a. angelegt. Nach wie vielen Jahren ist das Kapital auf 5000 EUR angewachsen?
Hinweis: Sie können diese Aufgaben rechnerisch oder grafisch lösen.

 

3. Aufgabe

Ermitteln Sie, ausgehend von der Funktion f(x)=e^x, die Graphen der folgenden Funktionen!

1) f_1(x)=e^x+2   7) f_7(x)=e^{x+2}

2) f_2(x)=e^x-2   8) f_8(x)=e^{x-2}

3) f_3(x)=2 e^x   9) f_9(x)=\left(2e\right)^x
4) f_4(x)= -2e^x

  10) f_{10}(x)=\left(-2e\right)^x
5) f_5(x)=e^{2x}   11) f_{11}(x)=\dfrac{1}{e^{2x}}
6) f_6(x)=e^{-2x}   12) f_{12}(x)=\dfrac{1}{e^{-2x}}

Was fällt Ihnen auf, wenn Sie die Graphen von f_5(x) und f_6(x) miteinander vergleichen?
Fällt Ihnen an f_{11}(x) und f_{12}(x) etwas auf?

 

4. Aufgabe

1)
Gegeben sei die Funktion f(x)= \dfrac{5^{x-1}}{10} mit \mathbb{D} = \mathbb{R}.
Gesucht ist jeweils die fehlende Koordinate des Punktes P(x\mid y),
a) wenn x=0
b) wenn y=0{,}2

  6)
Gegeben sei die Funktion f(x)= -13 \cdot e^{22x} mit \mathbb{D} = \mathbb{R}.
Gesucht ist jeweils die fehlende Koordinate des Punktes P(x\mid y),
a) wenn x=\dfrac{1}{11}
b) wenn y=-13

2)
Gegeben sei die Funktion f(a)= 2^{5a-3} mit \mathbb{D} = \mathbb{R}.
Gesucht ist jeweils die fehlende Koordinate des Punktes P(a\mid y),
a) wenn a=-32
b) wenn y=8

  7)
Gegeben sei die Funktion f(x)= 10^{x^2-4}-1 mit \mathbb{D} = \mathbb{R}.
Gesucht ist jeweils die fehlende Koordinate des Punktes P(x\mid y),
a) wenn x=-9
b) wenn y=9

3)
Gegeben sei die Funktion f(x)= \dfrac{1}{2} \cdot 3{,}5^{2x} mit \mathbb{D} = \mathbb{R}
Gesucht ist jeweils die fehlende Koordinate des Punktes P(x\mid y),
a) wenn x=-14
b) wenn y=\dfrac{49}{8}

  8)
Gegeben sei die Funktion f(x)= 27 \cdot e^{x^3-x^2} mit \mathbb{D} = \mathbb{R}
Gesucht ist jeweils die fehlende Koordinate des Punktes P(x\mid y),
a) wenn x=-0{,}5
b) wenn y=27

4)
Gegeben sei die Funktion f(x)= \dfrac{100}{3^{x-5}} mit \mathbb{D} = \mathbb{R}.
Gesucht ist jeweils die fehlende Koordinate des Punktes P(x\mid y),
a) wenn x=0{,}1
b) wenn y=8.100

  9)
Gegeben sei die Funktion g(y)= 2^{\frac{1}{2}y^2-1}+1 mit \mathbb{D} = \mathbb{R}.
Gesucht ist jeweils die fehlende Koordinate des Punktes P(x\mid y),
a) wenn x=\dfrac{11}{10}
b) wenn y=1

5)
Gegeben sei die Funktion f(x)= e^{7x-15} mit \mathbb{D} = \mathbb{R}.
Gesucht ist jeweils die fehlende Koordinate des Punktes P(x\mid y),
a) wenn x=10
b) wenn y=-15

  10)
Gegeben sei die Funktion f(x)= 1+\dfrac{2.000}{4^{x+1}} mit \mathbb{D} = \mathbb{R}.
Gesucht ist jeweils die fehlende Koordinate des Punktes P(x\mid y),
a) wenn x=17
b) wenn y=32.001

 

5. Aufgabe

Bestimmen Sie von folgenden Funktionen die Nullstellen!

1) f(x)=x e^x

  11) f(z) = (-5z^2+35)\cdot e^{z}

2) f(x)=6e^x+xe^x+2e^x

  12) f(a) = (a^2x-3a)\cdot e^{2-ax}

3) f(x)=123(x+13)10^{2x+5}

  13) g(y) = \left(\dfrac{5}{3}y^2-\dfrac{7}{4}y+8\right)\cdot e^{-\frac{1}{2}y^2+2}

4) f(x)=2x\left(2x^2e^{x^3+1}-9exe^{x^3}-5e^{x^3+1}\right)

  14) f(x)=\dfrac{e^{x^2}\cdot e^{-2}}{e^x}-1

5) f(v) = (v^2+2v-2)\cdot e^{-6v}

  15) g(x)=\dfrac{8}{2^{x^2}}-2^{-2x}

6) f(x)=3^x-2

  16) f(a)=a\cdot4^a-2\dfrac{1}{4^{-a}}+a^2\cdot 4^a

7) f(y)=\dfrac{e^y}{e^{2y+1}}-{e^y}^2

  17) g(t)=t\cdot e^{t-1}+e^t-e^{t-1}

8) f(x)=10^x+100^x-10

  18) f(x)= \dfrac{3}{\left(x+1\right)^2}\cdot\left(e^{\frac{4}{5}x+3}-3\right)

9) f(x) = -12\cdot 5^{3x}\cdot 5^{-x+1}+60


  19) f(y) = 3e^{-\frac{2}{3}y+5}\cdot\left(y^3+3y^2-4y\right)

10) f(x) = -2\cdot 3^x+9^x-2

  20) f(x) = \dfrac{e^{13x}-64}{201e^{x+52}}

Dieses Kapitel enthält die folgenden Themen:

 

16.2 Exponentialgleichungen und -funktionen - Erklärungen

Im Kapitel Polynomgleichungen und -funktionen haben wir uns Gleichungen und Funktionen angeschaut, bei denen die Variable in der Basis eines Potenzterms steht. Möglich ist aber natürlich auch, dass die Variable im Exponenten steht. Diese sogenannten Exponentialgleichungen bzw. -funktionen spielen in ganz unterschiedlichen Kontexten eine wichtige Rolle. Charakteristisch für diesen Gleichungs- und Funktionstyp sind Vorgänge, bei denen sich in gleich großen Intervallen der Wert um den gleichen Faktor ändert. Beispiele dafür sind:

  • Wachstumsvorgänge, z. B. bei Bakterien oder der Zinsentwicklung: Dabei ist die Basis größer als 1. Man sagt in diesem Fall auch "exponentielles Wachstum".
  • Zerfallsprozesse, z. B. bei radioaktiven Materialien: Hierbei tritt eine Basis zwischen 0 und 1 auf.

Auch in der Wahrscheinlichkeitsrechnung und bei Differentialgleichungen trifft man auf Exponentialgleichungen bzw. Exponentialfunktionen.
Wie schon im vorherigen Kapitel gilt auch hier: Wer sich (noch) nicht mit den Rechenregeln für Potenzen und Co. vertraut gemacht hat, sollte dies nun nachholen. Wir werden diese Rechenregeln und -gesetze im nun folgenden Kapitel brauchen.

 

Definition

Definition einer Exponentialgleichung: Exponentialgleichungen sind Gleichungen, bei denen die Variable als Exponent einer konstanten Basis a mit a\in\mathbb{R}^+\setminus_{\{1\}} auftritt.
Definition einer Exponentialfunktion: Exponentialfunktionen sind Funktionen, bei denen die Variable als Exponent einer konstanten Basis a mit a\in\mathbb{R}^+\setminus_{\{1\}} auftritt.

Definitionsbereich: Der Definitionsbereich von Exponentialgleichungen und Exponentialfunktionen umfasst die gesamten reellen Zahlen.

Exkurs: Schauen wir uns an, warum für die Basis nur positive reelle Zahlen mit Ausnahme der 1 zugelassen sind:
1 als Basis einer Exponentialgleichung oder -funktion wäre eine ziemlich langweilige Angelegenheit, denn 1^x = 1, egal für welche reelle Zahl die Variable x steht. Wenn man die Konstante 1 mal braucht, kann man sie ja einfach so hinschreiben, ohne Potenz ...
Ähnlich ist es mit der Basis 0: 0^x = 0 für alle reellen Zahlen x mit Ausnahme von x=0. 0^0 ist nämlich nicht definiert, sodass wir in diesem Fall den Definitionsbereich einschränken müssten. Diese Mühe lohnt sich nicht, wenn mit der Funktion ansonsten sowieso nichts passiert ...
Negative Basen würden uns insofern Schwierigkeiten machen, da gebrochene Exponenten (die laut Definitionsbereich zulässig sind) ja zu Wurzeltermen führen. Ein solcher Wurzelterm hätte einen negativen Radikanden, was in Kombination mit einem geraden Wurzelexponenten im Bereich der reellen Zahlen nicht berechnet werden kann. Man hätte bei negativen Basen also den Fall, dass einige Kombinationen von Basis und Exponent funktionieren ((-3)^2 = 9) und andere nicht ((-3)^{\frac{1}{2}} = \sqrt[2]{-3}). Da das nicht sinnvoll ist, schließt man negative Basen bei Exponentialgleichungen oder -funktionen grundsätzlich aus.

Ganz wichtig: Bitte beachten Sie den Unterschied zwischen Polynomen und Exponentialgleichungen/-funktionen: Während wir uns in diesem Kapitel um Gleichungen und Funktionen kümmern, bei denen die unabhängige Variable im Exponenten steht, ist es bei Polynomen die Basis, die variabel ist.

 

Exponentialgleichungen

Auch in diesem Kapitel einige Beispiele zum Start:

  • e^{8x+2} = 119
    Die eulersche Zahl e ist eine typische Basis für Exponentialgleichungen.

  • 10.240 = 10\cdot 2^{x-3}

  • 10^{y^2-10} \cdot 100^{y} = 10^{y-7}

 

Lösungsweg

Einige Exponentialgleichungen sind nur auf den ersten Blick wirklich schwierig. Häufig lassen sie sich nämlich mit wenigen Umformungen auf einfache Polynomgleichungen zurückführen. Logarithmieren als eine Umkehroperation zum Potenzieren hilft auf dem Weg zur Lösung oft weiter. Grundsätzlich ist es immer eine gute Idee, die Bestandteile der Gleichung zunächst so weit wie möglich zusammenzufassen. Dabei helfen u. a. die Potenz- und Logarithmengesetze.
Zur Lösbarkeit von Exponentialgleichungen kann man leider nicht so präzise Aussagen machen, weil dies ganz stark von der Struktur der Gleichung abhängt. Wie auch bei Polynomgleichungen ist es möglich, dass Exponentialgleichungen nur numerisch lösbar sind. Dies passiert vor allem dann, wenn eine Gleichung Exponentialterme und zugleich Polynome enthält, z. B. e^{2x} \cdot e^6 = x+12. Um die Exponentialterme auf der linken Seite aufzulösen, müsste man die Gleichung logarithmieren - dann würden aber auf der rechten Seite Logarithmusterme entstehen, sodass man exponenzieren müsste. Das würde aber direkt wieder zur Ausgangsgleichung führen. Daher bleiben in solchen Fällen bloß numerische Verfahren, um eine Lösung zu finden. Dieses Thema werden wir in diesem Lernmodul aber nicht behandeln. Das wird im Studium thematisiert werden, wenn Sie es benötigen.

Zur Schreibweise: Mit \ln() hinter dem senkrechten Strich am Ende einer Zeile ist gemeint, dass der natürliche Logarithmus auf beiden Seiten der Gleichung angewendet wird; analog für Logarithmen zu anderen Basen.


Schauen wir uns - wie schon in den vorangegangenen Kapiteln - einige Beispiele an:

Beispiel 1:
\begin{array}{crclcl} & \mathbb{D} &=& \mathbb{R} \cr \cr \text{1. Zeile:} & 1{,}1^{x+1} &=& 1{,}2^x &\vert& \ln() \cr \text{2. Zeile:} & \ln\left(1{,}1^{x+1}\right) &=& \ln\left(1{,}2^x\right) \cr \text{3. Zeile:} & (x+1)\ln(1{,}1) &=& x\ln(1{,}2) \cr & x\ln(1{,}1)+\ln(1{,}1) &=& x\ln(1{,}2) &\vert&\ -\ln(1{,}1)-x\ln(1{,}2) \cr & x\ln(1{,}1)-x\ln(1{,}2) &=& -\ln(1{,}1) \cr & x\left(\ln(1{,}1)-\ln(1{,}2)\right) &=& -\ln(1{,}1) &\vert& : \left(\ln(1{,}1)-\ln(1{,}2)\right) \cr & x &=& -\dfrac{\ln(1{,}1)}{\ln(1{,}1)-\ln(1{,}2)} \cr \cr & \mathbb{L} &=& \left\{-\dfrac{\ln(1{,}1)}{\ln(1{,}1)-\ln(1{,}2)}\right\} \end{array}

Zur 1. Zeile: Die Gleichung wird logarithmiert. Das ist bei vielen Exponentialgleichungen, die nur Exponentialterme enthalten, eine gute Idee. Ob man dabei lieber mit dem natürlichen Logarithmus oder einem Logarithmus zu einer anderen Basis rechnet, hat keinen Einfluss auf das Ergebnis. Wichtig ist hingegen, dass auf beiden Seiten der Gleichung logarithmiert wird - so wie es bei linearen Gleichungen wichtig ist, beim Multiplizieren mit einem Faktoren immer beide Seiten der Gleichung zu berücksichtigen.

Zur 2. Zeile: In diesem Schritt wird das 3. Logarithmengesetz angewendet. Achten Sie dabei auf der linken Seite unbedingt darauf, den Exponenten in Klammern zu setzen, weil er ja aus einer Summe besteht, die als Ganzes multipliziert werden soll.

Ab der 3. Zeile: Auch wenn es vielleicht auf den ersten Blick überrascht, ist die Gleichung nun linear. \ln(1{,}1)\approx 0{,}0953 und \ln(1{,}2)\approx 0{,}1823 sind nämlich einfach nur Zahlen, wenn sie auch erstmal nicht so aussehen ...
Der restliche Lösungsweg verläuft so wie bei anderen linearen Gleichungen auch: Klammer auf der linken Seite der Gleichung ausmultiplizieren - Terme sortieren - auf der linken Seite x ausklammern - durch den Koeffizienten von x teilen - fertig!

Bemerkung: -\frac{\ln(1{,}1)}{\ln(1{,}1)-\ln(1{,}2)} \approx 1{,}095
Warum ist das Ergebnis positiv, wo doch ein Minuszeichen vor dem Bruch steht? Ohne genaue Zahlenwerte für die Logarithmen zu ermitteln, kann man sich Folgendes überlegen: Der Logarithmus ist eine Funktion, bei der größere Argumente zu größeren Funktionswerten führen (Mathematisch sagt man: Die Funktion ist streng monoton steigend.). Da \ln(1)=0 ist, muss \ln(1{,}1) etwas größer als 0 sein. Außerdem muss \ln(1{,}2) etwas größer als \ln(1{,}1) sein (die beiden genauen Werte stehen weiter oben - aber ganz genau braucht man die für diese Überlegungen gar nicht). Daraus folgt, dass im Nenner des Bruches eine negative Zahl steht. Da der Zähler positiv ist, ist der gesamte Bruch also negativ. Zusammen mit dem Minuszeichen davor ergibt sich also ein positiver Wert.
Man kann natürlich auch den Taschenrechner fragen ...

 

Beispiel 2:
\begin{array}{ccrclcl}& & \mathbb{D} &=& \mathbb{R} \\\\\text{1. Zeile:} & & \dfrac{2}{e^{-x+1}} \left(x^2-x\right) &=& 40e^{x-1} \\\\\text{2. Zeile:} & & 2\left(x^2-x\right)e^{-(-x+1)} &=& 40e^{x-1} \\\\\text{3. Zeile:} & & \left(2x^2-2x\right)e^{x-1} &=& 40e^{x-1} & \vert & -40e^{x-1} \\\\\text{4. Zeile:} & & \left(2x^2-2x\right)e^{x-1}-40e^{x-1} &=& 0 \\\\\text{5. Zeile:} & & e^{x-1}\left(2x^2-2x-40\right) &=& 0 &\vert& \text{Satz vom Nullprodukt}\\\\\text{6. Zeile:} & \text{Faktor 1:} & e^{x-1} &=& 0 \\\\\\\text{7. Zeile:} & \text{Faktor 2:} & 2x^2-2x-40 &=& 0 & \vert& :2 \\\\& & x^2-x -20 &=& 0 & \vert& \text{p-q-Formel} \\\\& & x_{1,2}&=&\dfrac{1}{2}\pm\sqrt{\left(-\dfrac{1}{2}\right)^2+20}\\\\& & x_{1,2}&=&\dfrac{1}{2}\pm\sqrt{\dfrac{81}{4}}\\\\& & x_{1}&=&\dfrac{1}{2}+\dfrac{9}{2} = 5 \\\\ & & x_{2}&=&\dfrac{1}{2}-\dfrac{9}{2} = -4 \\\\\\& & \mathbb{L} &=& \{-4;5\}\end{array}

Zur 1. Zeile: Zunächst vereinfachen wir die Gleichung, weil der Term e^{-x+1} im Nenner ungünstig ist. Um den Term in den Zähler zu ziehen, müssen wir den Exponenten mit -1 multiplizieren. Dies hatten wir uns im Kapitel Potenzen, Wurzeln, Logarithmen angeschaut. Angenehme Folge: Wir können nun ohne Bruch weiterrechnen.

Zur 2. Zeile: Nun wird die Minusklammer im Exponenten aufgelöst und man erhält e^{x-1}. Außerdem werden die 2 und die Klammer vor dem Exponentialterm zusammengefasst (Distributivgesetz).

Zur 3. Zeile: Als nächstes subtrahiert man auf beiden Seiten der Gleichung 40e^{x-1}, damit auf der rechten Seite 0 steht.

Zur 4. Zeile: Da der Faktor e^{x+1} in beiden Summanden vorhanden ist, kann dieser ausgeklammert werden. Somit entsteht ein Produkt aus zwei Faktoren.

Zur 5. Zeile: Bei einem Produkt reeller Zahlen, das 0 ergeben muss, kann der Satz vom Nullprodukt angewendet werden. Es werden im Folgenden also die beiden Faktoren einzelnen betrachtet.

Zur 6. Zeile: Der erste Faktor wird betrachtet: Dieser Faktor liefert keine Lösung der Gleichung, da der Term e^{x-1} nie 0 werden kann. 
Warum kann e^{x-1} nicht 0 werden? Laut der Definition einer Potenz handelt es sich hierbei um ein Produkt, bei dem x-1-mal der Faktor e multipliziert wird. Ein Produkt kann aber nur dann 0 werden, wenn einer der Faktoren 0 ist. Da bei uns der Faktor e ist, kann das Produkt also nie 0 werden. Das gilt im Übrigen auch für die meisten anderen Potenzterme, da eine Potenz mit der Basis 0 eine ziemlich langweilige Angelegenheit wäre. Daher gibt es ja die oben diskutierten Einschränkungen für die Basis. 

Ab der 7. Zeile: Der zweite Faktor ergibt eine quadratische Gleichung: Wird auf beiden Seiten durch 2 dividiert, liegt sie in Normalform vor. Das ist die Voraussetzungen zur Anwendung der p-q-Formel. Dann kann die Gleichung wie gewohnt gelöst werden.

Bemerkung: Wenn Sie auf die Idee kommen, gleich die erste Zeile mit dem Faktor e^{-(x-1)} zu multiplizieren, ist das auch völlig in Ordnung, da dieser Faktor nicht 0 sein kann (siehe Kommentar zur 6. Zeile). Auf der linken Seite kürzen sich dann dieser Faktor und der Nenner weg. Auf der rechten Seite ergibt sich e^{x-1}\cdot e^{-(x-1)}= e^{x-1-x+1}=e^0=1. Damit sind die Exponentialterme "verschwunden" und man kann gleich in Zeile 7 weiterrechnen.

 

Beispiel 3:
\begin{array}{crclcl}&\mathbb{D} &=& \mathbb{R} \\\\\text{1. Zeile:} & 100^t-3 &=& -\dfrac{11}{2}\cdot 10^t & \vert & + \dfrac{11}{2}\cdot 10^t \\\text{2. Zeile:} & 10^{2t}+ \dfrac{11}{2}\cdot 10^t-3 &=& 0 \\\text{3. Zeile:} & \left(10^t\right)^2+ \dfrac{11}{2}\cdot 10^t-3 &=& 0\end{array}

Substitution: u=10^t
\begin{array}{crclcl}\text{4. Zeile:} & u^{2}+ \dfrac{11}{2} u-3 &=& 0 & \vert & \text{p-q-Formel} \\\text{5. Zeile:} & u_{1,2} &=& -\dfrac{11}{4}\pm\sqrt{\left(\dfrac{11}{4}\right)^2+3} \\\\\text{6. Zeile:} & u_{1,2} &=& -\dfrac{11}{4}\pm\sqrt{\dfrac{169}{16}} \\\\\text{7. Zeile:} & u_1 &=& -\dfrac{11}{4}+\dfrac{13}{4} = \dfrac{1}{2} \\\text{8. Zeile:} & u_2 &=& -\dfrac{11}{4}-\dfrac{13}{4} = -6\end{array}

Rücksubstitution:
\begin{array}{rrclcl}\text{9. Zeile:} & u_1 = 10^t &=& -6 &\vert& \lg() \\\\\text{10. Zeile:} & t_1 &=& \lg\left(-6\right) \\\\\text{11. Zeile:} & u_2 = 10^t &=& \dfrac{1}{2} &\vert& \lg() \\\text{12. Zeile:} & t_2 &=& \lg\left(\dfrac{1}{2}\right) \\\\& \mathbb{L} &=& \left\{\lg\left(\dfrac{1}{2}\right)\right\}\end{array}

Zur 1. Zeile: In dieser Exponentialgleichung sind nur Terme enthalten, die "Varianten" des Terms 10^t sind. Daher hat die Gleichung die benötigte Struktur, um sie mithilfe der Substitution lösen zu können. Dafür formen wir 100^t zu 10^{2t} um, denn 100 ist ja nichts anderes als 10^2. Des Weiteren addieren wir auf beiden Seiten \dfrac{11}{2}\cdot 10^t.

Zur 2. Zeile: Nun wenden wir das 3. Potenzgesetz auf 10^{2t} an und formen den Term zu \left(10^t\right)^2 um.

Zur 3. Zeile: Um jetzt eine quadratische Gleichung zu erhalten, wird 10^t durch u substituiert.

Zur 4. bis 8. Zeile: Die substituierte Gleichung wird nun mit der p-q-Formel gelöst.

Zur 9. Zeile: Nun werden die beiden zuvor ausgerechneten Zwischenlösungen rücksubstituiert: Es wird also u_1 genommen und mit dem Ursprungsterm, in diesem Fall 10^t, gleichgesetzt. Um die Gleichung zu lösen, wird der dekadische Logarithmus benötigt.

Zur 10. Zeile: Dieser Schritt bringt uns im Normalfall zu unserer ersten Lösung t_1. Hier ist es allerdings so, dass -6 nicht im Definitionsbereich des dekadischen Logarithmus liegt und daher keine Lösung berechnet werden kann.

Zur 11. Zeile: Das gleiche Vorgehen nun für u_2: Nach dem Aufstellen der Gleichung zur Rücksubstitution wird wieder der dekadische Logarithmus benötigt.

Zur 12. Zeile: Hier erhalten wir direkt die Lösung für t_2, da \dfrac{1}{2} als positive Zahl ein erlaubtes Argument für den Logarithmus ist.

Bemerkung: Substitution ist ein recht häufiges Lösungsverfahren bei Exponentialgleichungen. Man erreicht dadurch, dass zwischenzeitlich keine Exponentialterme in der Gleichung mehr enthalten sind und man es (in diesem Fall) "ganz gemütlich" mit einer quadratischen Gleichung zu tun hat. Es kann natürlich passieren, dass z. B. eine kubische oder eine biquadratische Gleichung entsteht, aber auch dafür haben wir schon Lösungswege kennengelernt.

 

Exponentialfunktionen

Für Exponentialfunktionen gibt es natürlich ebenfalls typische Beispiele, die wir anhand verschiedener Fälle betrachten wollen. Anders als bei den gebrochen rationalen und Wurzelfunktionen spielt es hier bei der Unterscheidung der Fälle keine Rolle, ob die Zahlenwerte im Funktionsterm gerade bzw. ungerade sind, sondern ob die (immer positive) Basis größer oder kleiner als 1 sind.

Eine Bemerkung vorab: Der Begriff "Exponentialfunktion" hat mehrere Bedeutungen: Manchmal wird er als Oberbegriff für alle Funktionen, bei denen die Variable im Exponenten steht, verwendet. Teilweise meint man damit konkret die Funktion f(x)=e^x, wobei e\approx 2{,}7182 die eulersche Zahl ist. Die Basis e hat nämlich angenehmere Eigenschaften als andere Basen, wie wir beispielsweise bei Ableitungen sehen werden. Wirklich schlimm ist diese Mehrdeutigkeit allerdings nicht: Mit der Basistransformation können wir (wenn nötig) jede andere Exponentialfunktion so umformen, dass sie e als Basis hat.

 

1. Fall

Wir beginnen mit Funktionsbeispielen, die eine Basis größer als 1 haben. Das passt insofern, dass die typischen Basen für Exponentialfunktionen 2, e und 10 sind. Schauen wir uns diesen Funktionen nun grafisch an: 

  • f_1(x)=e^x mit \mathbb{D}=\mathbb{R}

  • f_2(x)=2^x mit \mathbb{D}=\mathbb{R}

  • f_3(x) =10^x mit \mathbb{D}=\mathbb{R}

Beispiele für Exponentialfunktionen mit Basis größer 1

Wertebereich: Der Wertebereich dieser Exponentialfunktionen ist \mathbb{D}=\mathbb{R}^+. Wir hatten uns schon im Kapitel Potenzen, Wurzeln, Logarithmen überlegt, dass Potenzen mit positiver Basis immer positiv sein müssen. Da für Exponentialfunktionen laut Definition oben nur positive Basen (mit Ausnahme der 1) infrage kommen, können die Funktionswerte folglich nie negativ oder 0 werden.

Randverhalten: Für sehr kleine x-Werte nähert sich der Graph der 0 an. Für sehr große x-Werte werden auch die Funktionswerte sehr groß. Exponentielles Wachstum ist dabei immer schneller als polynomiales Wachstum.

Symmetrie: Exponentialfunktionen sind nicht symmetrisch.

"Besondere Punkte": Exponentialfunktionen haben keine Nullstellen, Extrempunkte, Wende- und Polstellen.

Weitere Besonderheiten: 
Alle dieser einfachen Exponentialfunktionen verlaufen durch den Punkt (0 \mid 1), denn x^0 = 1 für alle x\in\mathbb{R}^+\setminus_{\{1\}}.

Je größer die Basis im Funktionsterm ist, desto steiler verläuft der Graph im positiven x-Bereich und desto flacher verläuft der Graph im negativen x-Bereich.

 

2. Fall

Als Beispiele für Exponentialfunktionen mit einer Basis kleiner als 1 betrachten wir die Kehrwerte der Funktionen aus dem 1. Fall:

  • f_4(x)=\dfrac{1}{e^x}=\left(\dfrac{1}{e}\right)^x mit \mathbb{D}=\mathbb{R}

  • f_5(x)=\dfrac{1}{2^x}=\left(\dfrac{1}{2}\right)^x mit \mathbb{D}=\mathbb{R}

  • f_6(x) =\dfrac{1}{10^x}=\left(\dfrac{1}{10}\right)^x mit \mathbb{D}=\mathbb{R}

Bespiele für Exponentialfunktionen mit Basis kleiner 1

Wertebereich: Da die Basen bei diesen Funktionen zwar gebrochen, aber trotzdem positiv sind (sonst würden wir ja gegen die Definition der Exponentialfunktion verstoßen), bleibt es dabei: Der Wertebereich ist \mathbb{D}=\mathbb{R}^+.

Randverhalten: Bei Basen zwischen 0 und 1 verläuft die Funktion umgekehrt: Für sehr kleine x-Werte werden die Funktionswerte sehr groß und für sehr große x-Werte nähert sich der Graph der 0 an.

Symmetrie: Auch in diesem Fall sind die Exponentialfunktionen nicht symmetrisch.

"Besondere Punkte": Exponentialfunktionen haben keine Nullstellen, Extrempunkte, Wende- und Polstellen.

Weitere Besonderheiten:
Wieder verlaufen alle Graphen durch den Punkt (0 \mid 1).

Betrachtet man die Funktionen aus dem 1. und 2. Fall gemeinsam, fällt auf: Sind die Basen reziprok zueinander, sind die Graphen achsensymmetrisch zur y-Achse. Beispielsweise ist f_1(x) mit der Basis e spiegelbildlich zu f_4(x) mit der Basis \frac{1}{e} usw. Daraus folgt: Je größer die Basis, desto steiler der Graph - allerdings diesmal im negativen x-Bereich. Im positiven x-Bereich verlaufen Graphen von Funktionen mit großer Basis flacher.

 

3. Fall

Auch hier natürlich noch ein paar spannendere Beispielfunktionen:

  • f_7(x)=\dfrac{e^{(x+6)^2}}{15} mit \mathbb{D}=\mathbb{R}

  • f_8(x)=5^{2x}-4\cdot 5^x mit \mathbb{D}=\mathbb{R}

  • f_9(x)=e^{-x^2} mit \mathbb{D}=\mathbb{R}

  • f_{10}(x)=e^x-e^{-x} mit \mathbb{D}=\mathbb{R}

Beispiele für beliebige Exponentialfunktionen

Wertebereich: Der Wertebereich bei allgemeinen Exponentialfunktionen kann alle reellen Zahlen umfassen, siehe f_{10}(x), oder einseitig unbeschränkt sein, was dem Normalfall bei Exponentialfunktionen entspricht. Bei f_9(x) entspricht der Wertebereich dem halboffenen Intervall ]0; 1] und ist damit beidseitig beschränkt.

Randverhalten: Die Funktionen können sich an einer Seite des Definitionsbereichs einer festen Zahl annähern (z. B. f_8(x)), wie das für Exponentialfunktionen typisch ist. Bei f_7(x) werden - aufgrund des quadratischen Exponenten - die Funktionswerte an beiden Rändern des Definitionsbereichs unbeschränkt groß. Die Kombination zweier Exponentialterme, von denen einer einen positiven und einer einen negativen Exponenten hat, führt dazu, dass die Funktionswerte bei f_{10}(x) für große x-Werte sehr groß und für kleine x-Werte sehr klein werden.

Symmetrie: f_7(x) ist achsensymmetrisch zu einer senkrechten Gerade durch x=-6. f_9(x) ist achsensymmetrisch zur y-Achse. f_{10}(x) ist punktsymmetrisch zum Koordinatenursprung. Die letzte Funktion folgt eher dem typischen Verlauf und ist daher nicht symmetrisch.

Weitere Besonderheiten:
Was Sie bei diesen Funktionen (mit Ausnahme von f_9(x), wo der Exponentialterm im Nenner steht) sehen, ist ein ziemlich rasantes Wachstum. Das ist charakteristisch für Exponentialfunktionen.
Bitte beachten Sie, dass f_9(x)=e^{-x^2} = \dfrac{1}{e^{x^2}} ist. Das ist nicht das gleiche wie \dfrac{e}{x^2}. Bei der alternativen Schreibweise für Potenzen mit negativen Exponenten muss die gesamte Potenz (dann mit positivem Exponenten) im Nenner stehen, nicht nur Teile davon. Basis und Exponent einer Potenz "verwandeln" sich nicht einfach so in Zähler und Nenner eines Bruches!

Übersicht:

 

16.3 Exponentialgleichungen und -funktionen - Lösungen

Eine Bemerkung vorab: Bei vielen Aufgaben könnte man auch einen Logarithmus zu einer anderen Basis nehmen. Es gilt nämlich z. B.: \dfrac{\ln(15)}{\ln(26)} = \dfrac{\lg(15)}{\lg(26)} \approx 0{,}83

 

1. Aufgabe

1)
\begin{array}{rclcl} \mathbb{D} &=& \mathbb{R} \cr\cr 26^x &=& 15 &\vert& \ln() \cr \ln(26^x) &=& \ln(15) &\vert& \text{3. Logarithmengesetz} \cr x \cdot \ln(26) &=& \ln(15) &\vert& :\ln(26) \cr x &=& \dfrac{\ln(15)}{\ln(26)} \approx 0{,}83 \cr\cr \mathbb{L} &=& \left\{\dfrac{\ln(15)}{\ln(26)}\right\} \end{array}

Bemerkung: Die Darstellung \dfrac{\ln(15)}{\ln(26)} des Ergebnisses ist die mathematisch beste, weil sie exakt ist. Wer das Ergebnis lieber als Dezimalzahl hat, sollte erst ganz am Ende umformen und runden, da die Rundungsfehler sonst zu groß werden.
 

2)
\begin{array}{rclcl} \mathbb{D} &=& \mathbb{R} \\\\ 2^{5+x} &=& 8^x \\2^{5+x} &=& \left(2^3\right)^x \\2^{5+x} &=& 2^{3x} &\vert& \text{ld}() \\5+x &=& 3x &\vert& -x \\5 &=& 2x &\vert& :2 \\\dfrac{5}{2} &=& x \\\\\mathbb{L} &=& \left\{\dfrac{5}{2}\right\}\end{array}
 

3)
\begin{array}{rclcl}\mathbb{D} &=& \mathbb{R} \cr\cr 27^x+4 &=& 0 &\vert& -4 \cr 27^x &=& -4 \end{array}

Da Potenzen niemals negativ oder 0 sein können, hat diese Gleichung keine Lösung: \mathbb{L} = \emptyset

4)
\begin{array}{rclcl} \mathbb{D} &=& \mathbb{R} \cr \cr e^{2x+3} &=& 2 \cdot e^x &\vert& \ln() \cr \ln\left(e^{2x+3}\right) &=& \ln\left(2 \cdot e^x\right) &\vert& \text{1. Logarithmengesetz} \cr 2x+3 &=& \ln\left(2\right) + \ln\left(e^x\right) \cr 2x+3 &=& \ln(2)+x &\vert& -x -3 \cr x &=& \ln(2)-3 \approx -2{,}31 \cr \cr \mathbb{L} &=& \left\{\ln(2)-3\right\} \end{array}
 

5)
\begin{array}{rclcl} \mathbb{D} &=& \mathbb{R} \cr \cr 10^x &=& 100^{x+1} \cr 10^x &=& \left(10^2\right)^{x+1} &\vert& \text{5. Potenzgesetz} \cr 10^x &=& 10^{2(x+1)} &\vert& \lg() \cr \lg\left(10^x\right) &=& \lg\left(10^{2(x+1)}\right) \cr x &=& 2(x+1) \cr x &=& 2x+2 &\vert& -2x \cr -x &=& 2 &\vert& \cdot (-1) \cr x &=& -2 \cr \cr \mathbb{L} &=& \{-2\} \end{array}

6)
\begin{array}{rclcl} \mathbb{D} &=& \mathbb{R} \cr \cr 11^{x+2} &=& 8 &\vert& \ln() \cr \ln\left(11^{x+2}\right) &=& \ln(8) &\vert& \text{3. Logarithmengesetz} \cr (x+2)\ln(11) &=& \ln(8) &\vert& : \ln(11) \cr x+2 &=& \dfrac{\ln(8)}{\ln(11)} &\vert& -2 \cr x &=& \dfrac{\ln(8)}{\ln(11)}-2 \approx -1{,}13 \cr \cr \mathbb{L} &=& \left\{ \dfrac{\ln(8)}{\ln(11)} - 2\right\} \end{array}
 

7)
\begin{array}{rclcl} \mathbb{D} &=& \mathbb{R} \cr \cr 1{,}06^{x-1} &=& 2{,}08^x &\vert& \ln() \cr \ln\left(1{,}06^{x-1}\right) &=& \ln\left(2{,}08^x\right) &\vert& \text{3. Logarithmengesetz} \cr (x-1)\ln(1{,}06) &=& x\ln(2{,}08) \cr x\ln(1{,}06)-\ln(1{,}06) &=& x\ln(2{,}08) &\vert& +\ln(1{,}06)-x\ln(2{,}08) \cr x\ln(1{,}06)-x\ln(2{,}08) &=& \ln(1{,}06) \cr x\left(\ln(1{,}06)-\ln(2{,}08)\right) &=& \ln(1{,}06) &\vert& : \left(\ln(1{,}06)-\ln(2{,}08)\right) \cr x &=& \dfrac{\ln(1{,}06)}{\ln(1{,}06)-\ln(2{,}08)} \approx -0{,}09 \cr \cr \mathbb{L} &=& \left\{\dfrac{\ln(1{,}06)}{\ln(1{,}06)-\ln(2{,}08)}\right\} \end{array}

Bemerkung: Auch wenn es vielleicht nicht so aussieht: Ab der dritten Zeile ist die Ursprungsgleichung auf eine lineare Gleichung zurückgeführt. Es wird ab diesem Zeitpunkt keine Umformung oder Rechenoperation benötigt, die nicht schon im Kapitel 5 eingeführt worden ist. Die Gleichung lässt sich also ziemlich unkompliziert lösen - ok, die Koeffizienten sind etwas unhandlich ...
 

8)
\begin{array}{rclcl} \mathbb{D} &=& \mathbb{R} \cr \cr 17^s &=& e &\vert& \ln() \cr \ln(17^s) &=& \ln(e) &\vert& \text{3. Logarithmengesetz} \cr s \cdot \ln(17) &=& 1 &\vert& : \ln(17) \cr s &=& \dfrac{1}{\ln(17)} \approx 0{,}35 \cr \cr \mathbb{L} &=& \left\{\dfrac{1}{\ln(17)}\right\} \end{array}
 

9)
\begin{array}{rclcl}\mathbb{D} &=& \mathbb{R} \cr \cr 66^y &=& 1 & \vert & \ln() \cr \ln(66^y) &=& \ln(1) &\vert& \text{3. Logarithmengesetz} \cr y \cdot \ln(66) &=& 0 & \vert & :\ln(66) \cr y &=& 0 \cr \cr \mathbb{L} &=& \left\{0\right\}\end{array}
 

10)
\begin{array}{rclcl}\mathbb{D} &=& \mathbb{R} \cr \cr 48^{\frac{1}{18}x-39} &=& 48 &\vert& \log_{48}() \cr \dfrac{1}{18}x-39 &=& 1 &\vert& +39 \cr \dfrac{1}{18}x &=& 40 &\vert& : \dfrac{1}{18} \cr x &=& 720 \cr \cr \mathbb{L} &=& \left\{720\right\}\end{array}
 

11)
\begin{array}{rclcl}\mathbb{D} &=& \mathbb{R} \cr \cr -33+5^{3x-7} &=& 12 &\vert& +33 \cr 5^{3x-7} &=& 45 &\vert& \ln() \cr \ln(5^{3x-7}) &=& \ln(45) &\vert& \text{3. Logarithmengesetz} \cr \left(3x-7 \right)\ln(5) &=& \ln(5 \cdot 9) &\vert& \text{1. Logarithmengesetz} \cr 3x \cdot \ln(5) - 7\ln(5) &=& \ln(5) + \ln(9) &\vert& +7\ln(5) \cr 3x\ln(5) &=& 8\ln(5) + \ln(3^2) &\vert& : 3\ln(5) \cr x &=& \dfrac{8\ln(5) + 2\ln(3)}{3\ln(5)} \approx 3{,}12 \cr \cr \mathbb{L} &=& \left\{\dfrac{8\ln(5) + 2\ln(3)}{3\ln(5)}\right\}\end{array}

12)
\begin{array}{rclcl}\mathbb{D} &=& \mathbb{R} \cr \cr 7 \cdot e^{15x^2-30x} &=& 21 &\vert& :7 \cr e^{15x^2-30x} &=& 3 &\vert& \ln() \cr 15x^2-30x &=& \ln(3) &\vert& :15 \cr x^2-2x &=& \dfrac{\ln(3)}{15} &\vert& - \dfrac{\ln(3)}{15} \cr x^2-2x-\dfrac{\ln(3)}{15} &=& 0 &\vert&\text{p-q-Formel} \cr x_{1,2} &=& 1\pm\sqrt{1+\dfrac{\ln(3)}{15}} \cr\cr x_1 &=& 1+\sqrt{1+\dfrac{\ln(3)}{15}} \approx 2{,}04 \cr x_2 &=& 1-\sqrt{1+\dfrac{\ln(3)}{15}} \approx -0{,}04 \cr \cr \mathbb{L} &=& \left\{1+\sqrt{1+\dfrac{\ln(3)}{15}};\,1-\sqrt{1+\dfrac{\ln(3)}{15}}\right\}\end{array}
 

13)
\begin{array}{rclcl} \mathbb{D} &=& \mathbb{R} \cr\cr 5^{2t}-16\cdot 5^t &=& 0 &\vert& \text{3. Potenzgesetz} \cr \left(5^t\right)^2-16\cdot 5^t &=& 0 \end{array}

Substitutionu=5^t
\begin{array}{crclcl} & u^2-16u &=& 0 \cr & u\left(u-16 \right) &=& 0 \cr \text{Faktor 1: } & u_1 &=& 0 \cr\cr \text{Faktor 2: } & u -16 &=& 0 \cr & u_2 &=& 16\end{array}

Rücksubstitution:
\begin{array}{rclcl} u_1 = 5^{t_1} &=& 0 \cr\cr u_2 = 5^{t_2} &=& 16 &\vert&\ln() \cr \ln(5^t_2) &=& \ln(16) &\vert& \text{3. Logarithmengesetz} \cr t_2 \cdot \ln(5) &=& \ln(16) &\vert&:\ln(5) \cr t_2 &=& \dfrac{\ln(16)}{\ln(5)} \approx1{,}72 \end{array}

Da Potenzen niemals negativ oder 0 sein können, liefert die Rücksubstitution von u_1 keine weiteren Lösungen: \mathbb{L} = \left\{\dfrac{\ln(16)}{\ln(5)}\right\}


14)
\begin{array}{rclcl} \mathbb{D} &=& \mathbb{R} \cr\cr 4 \cdot e^{25x+18}-1 &=& 51 &\vert& +1 \cr 4 \cdot e^{25x+18} &=& 52 &\vert& :4 \cr e^{25x+18} &=& 13 &\vert& \ln() \cr \ln(e^{25x+18}) &=& \ln(13) \cr 25x+18 &=& \ln(13) &\vert& -18 \cr 25x &=& \ln(13)-18 &\vert& :25 \cr x &=& \dfrac{\ln(13)-18}{25} \approx -0{,}62 \cr\cr \mathbb{L}&=&\left\{\dfrac{\ln(13)-18}{25}\right\} \end{array}
 

15)
\begin{array}{rclcl} \mathbb{D} &=& \mathbb{R} \cr\cr e^x \cdot e^{2x} -e^{18} &=& 0 &\vert& \text{1. Potenzgesetz} \cr e^{x+2x} -e^{18} &=& 0 &\vert& +e^{18} \cr e^{3x} &=& e^{18} &\vert& \ln() \cr 3x &=& 18 &\vert& :3 \cr x &=& 6 \cr\cr \mathbb{L}&=&\left\{6\right\} \end{array}

16)
\begin{array}{rclcl} \mathbb{D} &=& \mathbb{R} \cr\cr 2^{5z^2} \cdot 2^{95z-45} \cdot 2^{-55} &=& 1 &\vert& \text{1. Potenzgesetz} \cr 2^{5z^2+(95z-45)-55} &=& 1 &\vert& \text{ld()} \cr 5z^2 + 95z -100 &=& 0 &\vert& :5 \cr z^2+19z-20 &=& 0 &\vert&\text{p-q-Formel} \cr z_{1,2} &=& -\dfrac{19}{2} \pm \sqrt{\left(\dfrac{19}{2}\right)+20} \cr &=& -\dfrac{19}{2} \pm \sqrt{\dfrac{441}{4}} \cr\cr z_1 &=& -\dfrac{19}{2} + \dfrac{21}{2} = 1 \cr z_2 &=& -\dfrac{19}{2} - \dfrac{21}{2} =-20 \cr\cr \mathbb{L}&=&\left\{-20; 1\right\} \end{array}


17)
\begin{array}{rclcl} \mathbb{D} &=& \mathbb{R} \cr\cr \dfrac{1.000^{x+1}}{10^{5x}}-12 &=& 43 &\vert& \text{5. Potenzgesetz} \cr\cr \dfrac{\left(10^3\right)^{x+1}}{10^{5x}}-12 &=& 43 &\vert& \text{2. Potenzgesetz} \cr\cr 10^{3x+3-5x}-12 &=& 43 &\vert& +12 \cr 10^{-2x+3} &=& 55 &\vert& \lg() \cr -2x+3 &=& \lg(55) &\vert& -3 \cr -2x &=& \lg(55)-3 &\vert& :(-2) \cr x &=& \dfrac{\lg(55)-3}{-2} \approx 0{,}63 \cr\cr \mathbb{L}&=&\left\{\dfrac{\lg(55)-3}{-2}\right\} \end{array}


18)
\begin{array}{rclcl} \mathbb{D} &=& \mathbb{R} \cr\cr 2 \cdot 10^{2x}-16 \cdot 10^x+30 &=& 0 &\vert& \text{3. Potenzgesetz} \cr 2 \cdot \left(10^x \right)^2 -16 \cdot 10^x+30 &=& 0 \end{array}

Substitution: u=10^x
\begin{array}{rclcl} 2u^2 -16u+30 &=& 0 &\vert& :2 \cr u^2-8u+15 &=& 0 &\vert& \text{p-q-Formel} \cr u_{1,2} &=& 4\pm \sqrt{16-15} \cr u_{1,2} &=& 4\pm \sqrt{1}\cr\cr u_1 &=& 4+1 = 5 \cr u_2 &=& 4-1 = 3\end{array}

Rücksubstitution:
\begin{array}{rclcl} u_1 = 10^{x_1} &=& 5 &\vert& \lg() \cr x_1 &=& \lg(5) \approx 0{,}70 \cr\cr u_2 = 10^{x_2} &=& 3 &\vert& \lg() \cr x_2 &=& \lg(3) \approx 0{,}48 \cr\cr\mathbb{L}&=&\left\{\lg(3); \lg(5) \right\}\end{array}


19)
\begin{array}{rclcl} \mathbb{D} &=& \mathbb{R} \cr\cr -9 \cdot e^{4x}+45 \cdot e^2x &=& 0 &\vert& \text{3. Potenzgesetz} \cr -9 \cdot \left(e^{2x} \right)^2+45 \cdot e^{2x} &=& 0 \end{array}

Substitution: u=e^{2x}
\begin{array}{crclcl} & -9u^2+45u &=& 0 \cr & 9u(-u+5) &=& 0 \cr \text{Faktor 1:} & u_1 &=& 0 \cr\cr \text{Faktor 2:} & -u+5 &=& 0 \cr & u_2 &=& 5\end{array}

Rücksubstitution:
\begin{array}{rclcl} u_1 = e^{2x_1} &=& 0 \cr\cr u_2 = e^{2x_2} &=& 5 &\vert& \ln() \cr 2x_2 &=& \ln(5) &\vert& :2 \cr x_2 &=& \dfrac{\ln(5)}{2} \approx 0{,}80 \end{array}

Da Potenzen niemals negativ oder 0 sein können, liefert die Rücksubstitution von u_1 keine weiteren Lösungen: \mathbb{L} = \left\{\dfrac{\ln(5)}{2}\right\}


20)
\begin{array}{crclcl}& \mathbb{D} &=& \mathbb{R} \\\\& 3\left(\dfrac{1}{3} x^2-112\right) e^{2x+6} &=& -\dfrac{109x}{e^{-2x-6}} \\\\& \left(x^2-336\right) e^{2x+6} &=& -109x\cdot e^{-(-2x-6)} &\vert& +109xe^{2x+6}\\\\& \left(x^2-336\right) e^{2x+6} +109xe^{2x+6} &=& 0\\\\& \left(x^2+109x-336\right) e^{2x+6} &=& 0 &\vert& \text{Satz vom Nullprodukt} \\\\\text{Faktor 1:} & x^2+109x-336 &=& 0 &\vert & \text{p-q-Formel} \\\\& x_{1,2} &=& -\dfrac{109}{2}\pm \sqrt{\left(\dfrac{109}{2}\right)^2+336} \\\\& x_{1,2} &=& -\dfrac{109}{2}\pm \sqrt{\dfrac{13.225}{4}} \\\\& x_{1} &=& -\dfrac{109}{2}+\dfrac{115}{2} = 3 \\\\& x_{2} &=& -\dfrac{109}{2}-\dfrac{115}{2} = -112 \\\\\\\text{Faktor 2:} & e^{2x+6} &=& 0\end{array}

Da Potenzen niemals negativ oder 0 sein können, liefert der zweite Faktor keine weiteren Lösungen: \mathbb{L} = \{-112;3\}

 

2. Aufgabe

Sei x die Anzahl der Jahre.
Dann lautet die Gleichung, die zu lösen ist: 1.000 \cdot 1{,}025^x=5.000

\begin{array}{rclcl} 1.000 \cdot 1{,}025^x &=& 5.000 &\vert& :1.000 \cr 1{,}025^x &=& 5 &\vert& \ln() \cr \ln\left(1{,}025^x\right) &=& \ln(5) &\vert& \text{3. Logarithmengesetz} \cr x\cdot \ln(1{,}025) &=& \ln(5) &\vert& : \ln(1{,}025) \cr x &=& \dfrac{\ln(5)}{\ln(1{,}025)} \approx 65{,}18 \end{array}

Nach ungefähr 65{,}18 Jahren beträgt das Kapital insgesamt 5.000 EUR.

 

3. Aufgabe

Der Übersicht wegen sind immer nur zwei Graphen in ein Koordinatensystem eingezeichnet. Zusätzlich zu f_1(x) bis f_{12}(x) wurde jeweils f(x)=e^x (fett gedruckt) mit abgebildet, um einen besseren Vergleich zu haben.

Graphen für f1(x) und f2(x)


Graphen für f3(x) und f4(x)


Graphen für f5(x) und f6(x)

f_5(x) und f_6(x) sind achsensymmetrisch zur y-Achse. Dies gilt allgemein für f(x)=a^x und f(x)=\left(\frac{1}{a}\right)^x=a^{-x}.


Graphen für f7(x) und f8(x)


Graph für f9(x)

Was ist mit f_{10}(x)?
f_{10}(x)=(-2e)^x hat eine negative Basis, und Potenzen mit negativen Basen sind im Bereich der reellen Zahlen nur für ganzzahlige Exponenten definiert, weil sonst Wurzeln aus negativen Zahlen gezogen werden müssen, z. B. ist ja (-2e)^\frac{1}{2}=\sqrt{-2e} nicht definiert. Da Funktionen üblicherweise für die reellen Zahlen (oder Teilbereiche davon) definiert sind, gäbe es hier also für beliebig viele x-Werte keinen Funktionswert. Daher beschreibt f_{10}(x) keine Funktion.

Graphen für f11(x) und f12(x)

Die Graphen von f_{11}(x) und f_{12}(x) sollten Ihnen bekannt vorkommen, denn sie sind bereits im dritten Koordinatensystem von oben mit den Funktionsgraphen von f_5(x) und f_6(x) zu sehen gewesen. Ist es überraschend, dass zwei unterschiedlich aussehende Funktionsterme den gleichen Graphen haben? Nein, denn nach dem Potenzgesetzen gilt:

f_5(x)=e^{2x}=\dfrac{1}{e^{-2x}}=f_{12}(x)
und
f_6(x)=e^{-2x}=\dfrac{1}{e^{2x}}=f_{11}(x)

 

4. Aufgabe

1)
a)
\begin{array}{rclll} f(0) &=& \dfrac{5^{0-1}}{10} &=& \dfrac{1}{50} \quad \rightarrow \quad P_1\left(0 \mid \dfrac{1}{50}\right) \end{array}

b)
\begin{array}{rclll} 0{,}2 &=& \dfrac{5^{x-1}}{10} &\vert & \cdot 10 \cr 2 &=& 5^{x-1} &\vert & \ln{()} \cr \ln{(2)} &=& \ln{\left(5^{x-1}\right)}\cr \ln{(2)} &=& (x-1) \cdot \ln{(5)} &\vert & :\ln{(5)}\cr \dfrac{\ln{(2)}}{\ln{(5)}} &=& x-1 &\vert & +1 \cr \dfrac{\ln{(2)}}{\ln{(5)}}+1 &=& x \cr x &\approx& 1{,}43 \in\mathbb{D} & & \rightarrow \quad P_2(1{,}43 \mid 0{,}2) \end{array}


2)
a)
\begin{array}{rclll} f(-32) &=& f(a)= 2^{5 \cdot (-32)-3} &\approx& 8{,}55 \cdot 10^{-50} \quad \rightarrow \quad P_1(-32 \mid 8{,}55\cdot 10^{-50} ) \end{array}

b)
\begin{array}{rclll}8 &=& 2^{5a-3} &\vert & \log_2()\cr 3 &=& 5a-3 &\vert & +3\cr 6 &=& 5a &\vert & :5\cr a &=& \dfrac{6}{5} \in\mathbb{D} & & \rightarrow \quad P_2\left(\dfrac{6}{5} \mid 8\right)\end{array}


3)
a)
\begin{array}{rclll} f(-14) &=& \dfrac{1}{2} \cdot 3{,}5^{2 \cdot (-14)} &\approx& 2{,}92\cdot 10^{-16} \quad \rightarrow \quad P_1(-14 \mid 2{,}92\cdot 10^{-16}) \end{array}

b)
\begin{array}{rclll}\dfrac{49}{8} &=& \dfrac{1}{2} \cdot \left(\dfrac{7}{2}\right)^{2x} & \vert & \cdot 2\cr \dfrac{49}{4} &=& \left(\dfrac{7}{2}\right)^{2x} & \vert & \ln{()}\cr \ln{\left(\dfrac{49}{4}\right)} &=& \ln{\left(\left(\dfrac{7}{2}\right)^{2x}\right)}\cr \ln{\left(\left(\dfrac{7}{2}\right)^2\right)} &=& 2x \cdot \ln{\left(\dfrac{7}{2}\right)} \cr 2\ln{\left(\dfrac{7}{2}\right)} &=& 2x \cdot \ln{\left(\dfrac{7}{2}\right)} & \vert & : 2\ln \left(\dfrac{7}{2}\right)\cr x &=& 1 \in\mathbb{D} & & \rightarrow \quad P_2\left(1 \mid 6{,}125\right)\end{array}


4)
a)
\begin{array}{rclll} f(0{,}1) &=& \dfrac{100}{3^{0{,}1-5}} &\approx& 21.771{,}79 \quad \rightarrow \quad P_1(0{,}1 \mid 21.771{,}79) \end{array}

b)
\begin{array}{rclll} 8.100 &=& \dfrac{100}{3^{x-5}} &\vert & \text{Kehrwert bilden}\cr\cr \dfrac{1}{8.100} &=& \dfrac{3^{x-5}}{100} &\vert & \cdot 100\cr \dfrac{1}{81} &=& 3^{x-5}\cr 3^{-4} &=& 3^{x-5} &\vert & \log_3()\cr -4 &=& x-5 &\vert & +5\cr x &=& 1\in\mathbb{D} & & \rightarrow \quad P_2(1 \mid 8.100) \end{array}


5)
a)
\begin{array}{rclll} f(10) &=& e^{7 \cdot 10-15} &\approx& 5{,}83\cdot 10^{34} \quad \rightarrow \quad P_1(10 \mid 5{,}83\cdot 10^{34}) \end{array}

b)
\begin{array}{rclll} -15 &=& e^{7 \cdot x-15} \cr -15 &=& \dfrac{e^{7x}}{e^{15}} &\vert & \cdot e^{15}\cr -15\cdot e^{15} &=& e^{7x} &\vert & \ln() \cr \ln(-15\cdot e^{15}) &=& 7x\end{array}

Da der Logarithmus einer negativen Zahl nicht definiert ist, hat diese Gleichung keine Lösung: Das bedeutet, dass die Funktion nirgends den Funktionswert -15 annimmt.


6)
a)
\begin{array}{rclll} f\left(\dfrac{1}{11}\right) &=& -13 \cdot e^{22 \cdot \frac{1}{11}} &\approx & -96{,}06 \quad \rightarrow \quad P_1\left(\dfrac{1}{11} \mid -96{,}06 \right) \end{array}

b)
\begin{array}{rclll} -13 &=& -13 \cdot e^{22x} &\vert & : \left(-13\right)\cr 1 &=& e^{22x} &\vert & \ln()\cr 0 &=& 22x &\vert & : 22\cr x &=& 0 \in\mathbb{D} & & \rightarrow \quad P_2\left(0 \mid -13\right) \end{array}


7)
a)
\begin{array}{rclll} f(-9) &=& 10^{(-9)^2-4}-1 &=& 10^{77}-1 \quad \rightarrow \quad P_1(-9 \mid 10^{77}-1) \end{array}

b)
\begin{array}{rclll} 9 &=& 10^{x^2-4}-1 &\vert & +1 \cr 10 &=& 10^{x^2-4} &\vert & \log_{10}()\cr 1 &=& x^2-4 &\vert & +4\cr 5 &=& x^2 &\vert & \pm\sqrt{ }\cr\cr x_2 &=& \sqrt{5} \in\mathbb{D} & & \rightarrow \quad P_2\left(\sqrt{5} \mid 9\right) \cr x_3 &=& -\sqrt{5} \in\mathbb{D} & & \rightarrow \quad P_3\left(-\sqrt{5} \mid 9\right)\end{array}


8)
a)
\begin{array}{rclll} f(-0{,}5) &=& 27 \cdot e^{(-0{,}5)^3-(-0{,}5)^2} &\approx& 18{,}56 \quad \rightarrow \quad P_1(-0{,}5 \mid 18{,}56) \end{array}

b)
\begin{array}{crclll} & 27 &=& 27 \cdot e^{x^3-x^2} &\vert & : 27 \cr &1 &=& e^{x^3-x^2} &\vert & \ln{()} \cr & 0 &=& x^3-x^2 \cr & 0 &=& x^2(x-1) & \vert & \text{Satz vom Nullprodukt} \cr \text{Faktor 1:} & x^2 &=& 0 &\vert& \pm\sqrt{} \cr & x_1 &=& 0 \in\mathbb{D} & & \rightarrow \quad P_2(0 \mid 27) \cr\cr \text{Faktor 2:} & x-1 &=& 0 &\vert & +1 \cr & x_2 &=& 1 \in\mathbb{D} & & \rightarrow \quad P_3(1 \mid 27) \end{array}


9)
a)
\begin{array}{rclll} g\left(\dfrac{11}{10}\right) &=& 2^{\frac{1}{2} \cdot \left(\frac{11}{10}\right)^2-1}+1 &\approx & 1{,}76 \quad \rightarrow \quad P_1\left(\dfrac{11}{10} \mid 1{,}76 \right) \end{array}

b)
\begin{array}{rclll} 1 &=& 2^{\frac{1}{2} y^2-1}+1 &\vert & -1 \cr 0 &=& 2^{\frac{1}{2} y^2-1}\cr 0 &=& \dfrac{2^{\frac{1}{2} y^2}}{2} &\vert & \cdot 2\cr 0 &=& 2^{\frac{1}{2} y^2} &\vert & \log_{2}{( )}\cr \log_{2}{(0)} &=& \dfrac{1}{2} y^2\end{array}

Da der Logarithmus von 0 nicht definiert ist, hat diese Gleichung keine Lösung: Das bedeutet, dass die Funktion nirgends den Funktionswert 1 annimmt.


10)
a)
\begin{array}{rclll} f(17) &=& 1+\dfrac{2.000}{4^{17+1}} &\approx & 1{,}000000029 \quad \rightarrow \quad P_1(17 \mid 1{,}000000029) \end{array}

b)

\begin{array}{rclll} 32.001 &=& 1+\dfrac{2.000}{4^{x+1}} &\vert & -1 \cr\cr 32.000 &=& \dfrac{2.000}{4^{x+1}} &\vert & \text{Kehrwert bilden}\cr\cr \dfrac{1}{32.000} &=& \dfrac{4^{x+1}}{2.000} &\vert & \cdot 2.000\cr \dfrac{1}{16} &=& 4^{x+1} \cr 4^{-2} &=& 4^{x+1} &\vert & \log_{4}()\cr -2 &=& x+1 &\vert & -1\cr x &=& -3 \in\mathbb{D} & & \rightarrow \quad P_2(-3 \mid 32.001) \end{array}

 

5. Aufgabe

1)
\begin{array}{crclcl} & \mathbb{D} &=& \mathbb{R} \cr\cr & 0 &=& x e^x &\vert& \text{Satz vom Nullprodukt} \cr \text{Faktor 1:} & 0 &=& x \cr\cr \text{Faktor 2:} & 0 &=& e^x \end{array}

Da Potenzen niemals negativ oder 0 sein können, liefert der zweite Faktor keine weiteren Nullstellen. Die einzige Nullstelle von f(x) liegt bei x=0.


2)
\begin{array}{crclcl} & \mathbb{D} &=& \mathbb{R} \cr\cr & 0 &=& 6e^x+xe^x+2e^x \cr & 0 &=& (x+8)e^x &\vert& \text{Satz vom Nullprodukt} \cr \text{Faktor 1:} & 0 &=& x+8 &\vert& -8 \cr & -8 &=& x \cr\cr \text{Faktor 2:} & 0 &=& e^x \end{array}

Da Potenzen niemals negativ oder 0 sein können, liefert der zweite Faktor keine weiteren Nullstellen. Die einzige Nullstelle von f(x) liegt bei x=-8.


3)
\begin{array}{crclcl} & \mathbb{D} &=& \mathbb{R} \cr\cr & 0 &=& 123(x+13)10^{2x+5} &\vert& \text{Satz vom Nullprodukt} \cr \text{Faktor 1:} & 0 &=& 123(x+13) &\vert& :123 \cr & 0 &=& x+13 &\vert& -13 \cr & -13 &=& x \cr\cr \text{Faktor 2:} & 0 &=& 10^{2x+5} \end{array}

Da Potenzen niemals negativ oder 0 sein können, liefert der zweite Faktor keine weiteren Nullstellen. Die einzige Nullstelle von f(x) liegt bei x=-13.


4)
\begin{array}{lrclcl} & \mathbb{D} &=& \mathbb{R} \cr\cr & 0 &=& 2x\left(2x^2e^{x^3+1}-9exe^{x^3}-5e^{x^3+1}\right) \cr & 0 &=& 4x^3e^{x^3+1}-18x^2e^{x^3+1}-10xe^{x^3+1} \cr & 0 &=& e^{x^3+1}\left(4x^3-18x^2-10x\right) &\vert& \text{Satz vom Nullprodukt} \cr \text{Faktor 1:} & 0 &=& e^{x^3+1} \cr\cr \text{Faktor 2:} & 0 &=& 4x^3-18x^2-10x \cr & 0 &=& x\left(4x^2-18x-10\right) &\vert& \text{Satz vom Nullprodukt} \cr \text{Faktor 2.1:} & 0 &=& x_1 \cr\cr \text{Faktor 2.2:} & 0 &=& 4x^2-18x-10 &\vert& :4 \cr & 0 &=& x^2-\dfrac{9}{2}x-\dfrac{5}{2} &\vert& \text{p-q-Formel} \cr\cr & x_{2,3} &=& \dfrac{9}{4} \pm \sqrt{\left(-\dfrac{9}{4}\right)^2+\dfrac{5}{2}} \cr & &=& \dfrac{9}{4} \pm \sqrt{\dfrac{121}{16}} \cr\cr & x_2 &=& \dfrac{9}{4} + \dfrac{11}{4} = 5 \cr\cr & x_3 &=& \dfrac{9}{4} - \dfrac{11}{4} = -\dfrac{1}{2} \end{array}

Da Potenzen niemals negativ oder 0 sein können, liefert der erste Faktor keine weiteren Nullstellen. Die Nullstellen von f(x) liegen bei x_1=0, x_2=5 und x_3=-\dfrac{1}{2}.


5)
\begin{array}{lrclcl} & \mathbb{D} &=& \mathbb{R} \cr\cr & 0 &=& (v^2+2v-2) \cdot e^{-6v} &\vert& \text{Satz vom Nullprodukt} \cr \text{Faktor 1:} & 0 &=& e^{-6v} \cr\cr \text{Faktor 2:} & 0 &=& v^2+2v-2 &\vert& \text{p-q-Formel} \cr & v_{1,2} &=& -1 \pm \sqrt{1^2+2} \cr\cr & v_1 &=& -1+\sqrt{3} \approx 0{,}73 \cr & v_2 &=& -1-\sqrt{3} \approx -2{,}73 \end{array}

Da Potenzen niemals negativ oder 0 sein können, liefert der erste Faktor keine weiteren Nullstellen. Die Nullstellen von f(v) liegen bei v_1=-1+\sqrt{3} und v_2=-1-\sqrt{3}.


6)
\begin{array}{rclcll}\mathbb{D} &=& \mathbb{R} \cr\cr 0 &=& 3^x-2 &\vert& -3^x \\-3^x &=& -2 &\vert&\cdot (-1) \\3^x &=& 2 &\vert& \log_{3}() \\x &=& \log_{3}(2)\end{array}

Die Nullstelle von f(x) liegt bei x=\log_{3}(2).


7)
\begin{array}{rclcll}\mathbb{D} &=& \mathbb{R}\setminus_{\{-\frac{1}{2}\}} \\ \\0 &=& \dfrac{e^y}{e^{2y+1}}-{e^y}^2 &\vert & +{e^y}^2 \\ \\{e^y}^2 &=& e^{y-(2y+1)} \\{e^y}^2 &=& e^{-y-1} &\vert& \ln() \\\ln\left({e^y}^2\right) &=& \ln\left(e^{-y-1}\right) \\y^2 &=& -y-1 &\vert& +y+1 \\y^2+y+1 &=& 0 &\vert &\text{p-q-Formel} \\y_{1,2} &=& -\dfrac{1}{2}\pm\sqrt{\left(\dfrac{1}{2}\right)^2-1} \\&=& -\dfrac{1}{2}\pm\sqrt{-\dfrac{3}{4}} \\\end{array}

Da aus negativen reellen Zahlen keine Wurzeln mit geraden Wurzelexponenten gezogen werden können, hat diese Funktion keine Nullstellen.


8)
\begin{array}{rclcll}\mathbb{D} &=& \mathbb{R} \cr\cr 0 &=& 100^x+10^x-10 \\0 &=& (10^2)^x+10^x-10 \\0 &=& (10^x)^2+10^x-10 \\\end{array}

Substitution: u=10^x
\begin{array}{rclcll}u^2+u-10 &=& 0 &\vert& \text{p-q-Formel} \\u_{1,2} &=& -\dfrac{1}{2}\pm\sqrt{\left(\dfrac{1}{2}\right)^2+10} \\ \\&=& -\dfrac{1}{2}\pm\sqrt{\dfrac{41}{4}} \\ \\u_1 &=& -\dfrac{1}{2}+\dfrac{\sqrt{41}}{2} \approx 2{,}70 \\ \\u_2 &=& -\dfrac{1}{2}-\dfrac{\sqrt{41}}{2} \approx -3{,}70 \\ \\\end{array}

Rücksubstitution:
\begin{array}{rclcll}u_1 =10^{x_1} &=& -\dfrac{1}{2}+\dfrac{\sqrt{41}}{2} &\vert& \lg() \\ \\x_1 &=& \lg \left(-\dfrac{1}{2}+\dfrac{\sqrt{41}}{2}\right)\approx 0{,}4316 \\ \\u_2 =10^{x_2} &=& -\dfrac{1}{2}-\dfrac{\sqrt{41}}{2}\end{array}

Da Potenzen niemals negativ oder 0 sein können, liefert die Rücksubstitution von u_2 keine weiteren Nullstellen. Die einzige Nullstelle von f(x) liegt bei x=\lg\left(-\dfrac{1}{2}+\dfrac{\sqrt{41}}{2}\right).


9)
\begin{array}{rclcll}\mathbb{D} &=& \mathbb{R} \cr\cr 0 &=& -12\cdot 5^{3x}\cdot 5^{-x+1}+60 &\vert& -60 \\-60 &=& -12\cdot 5^{3x}\cdot 5^{-x+1} \\-60 &=& -12\cdot 5^{2x+1} &\vert& :(-12) \\5 &=& 5^{2x+1} &\vert& \log_{5}() \\\log_{5} \left(5\right) &=& \log_{5} \left(5^{2x+1}\right) \\1 &=& 2x+1 &\vert& -1\\0 &=& 2x &\vert& :2\\x &=& 0\end{array} 

Die Nullstelle von f(x) liegt bei x=0.


10)
\begin{array}{rclcll}\mathbb{D} &=& \mathbb{R} \cr\cr 0 &=& -2\cdot 3^x+9^x-2 \\0 &=& (3^2)^x-2\cdot 3^x-2 \\0 &=& (3^x)^2-2\cdot 3^x-2\end{array}

Substitution: u = 3^x
\begin{array}{rclcll}u^2-2u-2 &=& 0 &\vert& \text{p-q-Formel} \\u_{1,2} &=& 1\pm\sqrt{(-1)^2+2}\\&=& 1\pm\sqrt{3} \\ \\u_1 &=& 1+\sqrt{3} \approx 2{,}73 \\u_2 &=& 1-\sqrt{3} \approx -0{,}73\end{array}

Rücksubstitution:
\begin{array}{rclcll}u_1 = 3^x &=& 1+\sqrt{3} &\vert& \log_3() \\x_1 &=& \log_{3} (1+\sqrt{3})\approx 0{,}9148 \\ \\u_2 = 3^x &=& 1-\sqrt{3} \\\end{array}

Da Potenzen niemals negativ oder 0 sein können, liefert die Rücksubstitution von u_2 keine weiteren Nullstellen. Die einzige Nullstelle von f(x) liegt bei x_1=\log_{3}\left(1+\sqrt{3}\right).


11)
\begin{array}{lrclcl} & \mathbb{D} &=& \mathbb{R} \cr\cr & 0 &=& (-5z^2+35) \cdot e^{z} &\vert& \text{Satz vom Nullprodukt} \cr \text{Faktor 1:} & 0 &=& e^{z} \cr\cr \text{Faktor 2:} & 0 &=& -5z^2+35 &\vert& -35 \cr & -35 &=& -5z^2 &\vert& :(-5) \cr & 7 &=& z^2 \cr\cr & z_1 &=& \sqrt{7} \cr & z_2 &=& -\sqrt{7} \end{array}

Da Potenzen niemals negativ oder 0 sein können, liefert der erste Faktor keine weiteren Nullstellen. Die Nullstellen von f(z) liegen bei z_1=\sqrt{7} und z_2=-\sqrt{7}.


12)
\begin{array}{lrclcl} & \mathbb{D} &=& \mathbb{R} \cr\cr & 0 &=& (a^2x-3a)\cdot e^{2-ax} &\vert& \text{Satz vom Nullprodukt} \cr \text{Faktor 1:} & 0 &=& e^{2-ax} \cr\cr \text{Faktor 2:} & 0 &=& a^2x-3a \cr & 0 &=& a(ax-3) &\vert& \text{Satz vom Nullprodukt} \cr \text{Faktor 2.1:} & 0 &=& a_1 \cr\cr \text{Faktor 2.2:} & 0 &=& ax-3 &\vert& +3 \cr & 3 &=& ax &\vert& :x \cr & a_2 &=& \dfrac{3}{x} \end{array}

Da Potenzen niemals negativ oder 0 sein können, liefert der erste Faktor keine weiteren Nullstellen. Die Nullstelle von f(a) liegen bei a_1=0 und a_2=\dfrac{3}{x}.

Bemerkung: Bitte achten Sie darauf, dass die Variable hier a und nicht x ist! Das erkennt man daran, dass die Funktion f(a) heißt ...
Bei der Lösung a_2 wird deutlich, dass der Parameter x\neq 0 sein muss, da wir ja durch den Parameter teilen. Das wird auch dann plausibel, wenn man sich anschaut, wie der Funktionsterm für x=0 aussieht, nämlich f(a) = -3a\cdot e^{2}. Diese Funktion ist linear, da e^2 \approx 7{,}39 eine Konstante ist, und lineare Funktionen können nun mal nicht mehr als eine Nullstelle haben.


13)
\begin{array}{lrclcl} & \mathbb{D} &=& \mathbb{R} \cr\cr & 0 &=& \left(\dfrac{5}{3}y^2-\dfrac{7}{4}y+8\right) \cdot e^{-\frac{1}{2}y^2+2} &\vert& \text{Satz vom Nullprodukt} \cr \text{Faktor 1:} & 0 &=& e^{-\frac{1}{2}y^2+2} \cr\cr \text{Faktor 2:} & 0 &=& \dfrac{5}{3}y^2-\dfrac{7}{4}y+8 &\vert& \cdot \dfrac{3}{5} \cr\cr & 0 &=& y^2-\dfrac{21}{20}y+\dfrac{24}{5} &\vert& \text{p-q-Formel} \cr\cr & y_{1,2} &=& \dfrac{21}{40} \pm \sqrt{\left(-\dfrac{21}{40}\right)^2-\dfrac{24}{5}} \cr & y_{1,2} &=& \dfrac{21}{40} \pm \sqrt{-\dfrac{7.239}{1.600}} \end{array}

Da Potenzen niemals negativ oder 0 sein können und aus negativen reellen Zahlen keine Wurzeln mit geraden Wurzelexponenten gezogen werden können, hat g(y) keine Nullstellen.


14)
\begin{array}{rclcll}\mathbb{D} &=& \mathbb{R} \cr\cr0 &=& \dfrac{e^{x^2}\cdot e^{-2}}{e^x}-1 &\vert& +1 \\1 &=& \dfrac{e^{x^2}\cdot e^{-2}}{e^x} \\1 &=& e^{x^2-2}\cdot e^{-x} \\1 &=& e^{x^2-2-x} &\vert& \ln()\\\ln (1) &=& \ln \left(e^{x^2-2-x}\right) \\0 &=& x^2-x-2 &\vert&\text{p-q-Formel} \\x_{1,2} &=& \dfrac{1}{2}\pm\sqrt{\left(-\dfrac{1}{2}\right)^2+2} \\\\&=& \dfrac{1}{2}\pm\sqrt{\dfrac{9}{4}} \\\\x_1 &=& \dfrac{1}{2}+\dfrac{3}{2} = 2 \\x_2 &=& \dfrac{1}{2}-\dfrac{3}{2} = -1\end{array}

Die Nullstellen von f(x) liegen bei x_1=2 und bei x_2=-1.


15)
\begin{array}{rclcll}\mathbb{D} &=& \mathbb{R} \\ \\0 &=& \dfrac{8}{2^{x^2}}-2^{-2x} &\vert& +2^{-2x} \\ \\2^{-2x} &=& \dfrac{8}{2^{x^2}} &\vert& \cdot 2^{x^2} \\2^{-2x}\cdot 2^{x^2} &=& 8 \\2^{x^2-2x} &=& 2^3 &\vert& \log_2() \\x^2-2x &=& 3 &\vert& -3 \\x^2-2x-3 &=& 0 &\vert& \text{p-q-Formel} \\x_{1,2} &=& 1\pm\sqrt{(-1)^2+3} \\&=& 1\pm\sqrt{4} \\ \\x_1 &=& 1-2=-1 \\x_2 &=& 1+2=3\end{array}

Die Nullstellen von f(x) liegen bei x_1=-1 und bei x_2=3.

Bemerkung: Die Multiplikation mit 2^{x^2} ist hier ohne Einschränkungen möglich, weil 2^{x^2} immer größer als 0 ist. Eine Multiplikation mit 0 kann also nicht passieren.


16)
\begin{array}{rclll}\mathbb{D} &=& \mathbb{R} \cr\cr a\cdot4^a-2\dfrac{1}{4^{-a}}+a^2\cdot 4^a &=& 0 \\a^2\cdot 4^a+a\cdot4^a-2\cdot 4^a &=& 0 &\vert & :4^a\\a^2+a-2 &=& 0 &\vert & \text{p-q-Formel}\\a_{1,2} &=& -\dfrac{1}{2}\pm\sqrt{\left(\dfrac{1}{2}\right)^2+2}\\a_{1,2} &=& -\dfrac{1}{2}\pm\sqrt{\dfrac{9}{4}}\\\\a_1 &=& -\dfrac{1}{2}+\dfrac{3}{2} = 1\\a_2 &=& -\dfrac{1}{2}-\dfrac{3}{2} = -2\end{array}

Die Nullstellen von f(a) liegen bei a_1=1 und bei a_2=-2.

Bemerkung: Die Division durch 4^{a} ist hier ohne Einschränkungen möglich, weil 4^{a} immer größer als 0 ist. Eine Division durch 0 kann also nicht passieren.


17) 
\begin{array}{rclll}\mathbb{D} &=& \mathbb{R} \cr\cr t\cdot e^{t-1}+e^t-e^{t-1} &=& 0 \\e^{t-1}\left(t+e^1-1\right) &=& 0 &\vert & :e^{t-1} \\t+e-1 &=& 0 &\vert& -e+1\\t &=& 1-e\approx -1{,}7183\end{array}

Die Nullstelle von g(t) liegt bei t=1-e.

Bemerkung: Die Division durch e^{t-1} ist hier ohne Einschränkungen möglich, weil e^{t-1} immer größer als 0 ist. Eine Division durch 0 kann also nicht passieren.


18)
\begin{array}{crclll}& \mathbb{D} &=& \mathbb{R}\setminus_{\{-1\}} \\\\& \dfrac{3}{\left(x+1\right)^2}\cdot\left(e^{\frac{4}{5}x+3}-3\right) &=& 0 \\\\\text{Faktor 1:} & \dfrac{3}{\left(x+1\right)^2} &=& 0 &\vert& \cdot \left(x+1\right)^2 \\& 3 &=& 0 \\\\\text{Faktor 2:} & e^{\frac{4}{5}x+3}-3 &=& 0 &\vert & +3 \\& e^{\frac{4}{5}x+3} &=& 3 &\vert & \ln() \\& \dfrac{4}{5}x+3 &=& \ln(3) &\vert & -3\\& \dfrac{4}{5}x &=& \ln(3) -3 &\vert & :\dfrac{4}{5}\\& x &=& \dfrac{5}{4}\left(\ln(3) -3\right) \approx -2{,}38\end{array}

Da der erste Faktor zu einem Widerspruch führt, liefert er keine weiteren Nullstellen. Die einzige Nullstelle von f(x) liegt bei x=\dfrac{5}{4}\left(\ln(3) -3\right).

Bemerkung: Die Multiplikation mit (x+1)^2 ist hier ohne Einschränkungen möglich, weil x=-1 \not\in \mathbb{D}. Eine Multiplikation mit 0 kann also nicht passieren.


19) 
\begin{array}{lrclll}& \mathbb{D} &=& \mathbb{R} \\\\& 3e^{-\frac{2}{3}y+5}\cdot\left(y^3+3y^2-4y\right) &=& 0 \\\\\text{Faktor 1:} & 3e^{-\frac{2}{3}y+5} &= & 0 &\vert& :3 \\& e^{-\frac{2}{3}y+5} &= & 0 \\\\\text{Faktor 2:} & y^3+3y^2-4y &=& 0 \\& y\left(y^2+3y-4\right) &=& 0 \\\text{Faktor 2.1:} & y_1 &=& 0 \\\\\text{Faktor 2.2:} & y^2+3y-4 &=& 0 &\vert& \text{p-q-Formel} \\& y_{2,3} &=& -\dfrac{3}{2}\pm\sqrt{\left(\dfrac{3}{2}\right)^2+4}\\& y_{2,3} &=& -\dfrac{3}{2}\pm\sqrt{\dfrac{25}{4}}\\\\& y_2 &=& -\dfrac{3}{2}+\dfrac{5}{2} = 1\\& y_3 &=& -\dfrac{3}{2}-\dfrac{5}{2} = -4\end{array}

Da Potenzen niemals negativ oder 0 sein können, liefert der erste Faktor keine weiteren Nullstellen. Die Nullstellen von f(y) liegen bei y_1=0, y_2=1 und bei y_3=-4.


20)
\begin{array}{rclll}\mathbb{D} &=& \mathbb{R} \\\\\dfrac{e^{13x}-64}{201e^{x+52}} &=& 0 &\vert& \cdot 201e^{x+52} \\\\e^{13x}-64 &=& 0 &\vert & +64\\e^{13x} &=& 64 &\vert & \ln() \\13x &=& \ln(64) &\vert &:13 \\x &=& \dfrac{\ln(64)}{13} \approx 0{,}32\end{array}

Die Nullstelle von f(x) liegt bei x=\dfrac{\ln(64)}{13}.

Bemerkung: Die Multiplikation mit 201e^{x+52} ist hier ohne Einschränkungen möglich, weil 201e^{x+52} immer größer als 0 ist. Eine Multiplikation mit 0 kann also nicht passieren.

17. Logarithmusgleichungen und -funktionen - Lernziele und typische Fehler

Nach Durcharbeiten dieses Kapitels sollten Sie folgende Lernziele erreicht haben:

  • Sie können zu einer Logarithmusgleichung den passenden Definitionsbereich bestimmen.
  • Sie kennen die allgemeine Form einer Logarithmusgleichung.
  • Sie können Logarithmusgleichungen lösen.
  • Sie können die Lösungsmenge mathematisch korrekt notieren.
  • Sie können mithilfe der Probe überprüfen, ob die gefundene Lösung tatsächlich richtig ist.
  • Sie wissen, dass bei Logarithmusgleichungen Scheinlösungen auftreten können, wie man diese erkennt und wie man damit umgeht.
  • Sie können zu einer Logarithmusfunktion den passenden Definitionsbereich bestimmen.
  • Sie wissen, wie der Graph einer Logarithmusfunktion typischerweise aussieht, und können ihn in ein kartesisches Koordinatensystem zeichnen.
  • Sie kennen die allgemeine Funktionsgleichung einer Logarithmusfunktion.
  • Sie kennen Eigenschaften von Logarithmusfunktionen (Anzahl der Nullstellen, Randverhalten, typische Punkte) und können dies nutzen, um Beziehungen zwischen Funktionsterm und Funktionsgraphen herzustellen.
  • Sie kennen den Zusammenhang zwischen Logarithmusgleichungen und Logarithmusfunktionen.
  • Sie können Logarithmusgleichungen von anderen Gleichungsarten unterscheiden.
  • Sie können Logarithmusfunktionen von anderen Funktionstypen unterscheiden (grafisch und anhand der Funktionsgleichung).
  • Sie können den Zusammenhang zwischen Exponential- und Logarithmusfunktion beschreiben.


Typische Fehler in diesem Kapitel sind:

  • Beim Exponenzieren der Gleichung werden nicht alle Bestandteile der Gleichung exponenziert. Erklärung
  • Es wird nicht geprüft, ob ein berechneter Wert eine Scheinlösung ist. Erklärung


Für Online-Selbsttests zu diesem Thema und weitere Informationen zur Mathematikunterstützung an der TH Wildau nutzen Sie bitte den Moodle-Kursraum "SOS Mathematik - Brückenkurs".

Übersicht:

 

17.1 Logarithmusgleichungen und -funktionen - Aufgaben 

1. Aufgabe

Lösen Sie folgende Logarithmusgleichungen! Geben Sie jeweils den Definitionsbereich an!

1) \lg(x-1)+\lg(x+2)=1

  11) 2\ln(x-1)+2\ln(3) = 2\ln(1)+3\ln(2)

2) x \cdot \ln(x)=2x

  12) 8\ln(ex+2) = 8

3) 5 \lg(x)=3 \lg(12)+\lg(32)-3(\lg(4)+\lg(3))

  13) \ln\left(\sqrt{-s+13}\right)+\ln\left(\sqrt[3]{-s+13}\right)-1 = 0

4) \log_3(x-75)-3=\log_3(x+3)

  14) \ln\left(4x^{\frac{7}{8}}\right)+\ln\left(3x^{-\frac{1}{8}}\right)=\dfrac{7}{4}\ln\left(\sqrt[7]{16}x\right)

5) \ln(x-2)+\ln(x+3)=\ln(x^2+x-6)

  15) \dfrac{\log_4(x-3)}{\log_4(x)}=-1

6) \log_2(16^x)+\log_5(25^{x^2})=16

  16) \log_{27}\left(\sqrt[3]{9x^2}\right)=\log_{27}\left(\dfrac{1}{x}\right)

7) \ln(\sqrt{5x+2})+\ln((5x+2)^2)=1

  17) \log_{4}(-3x+15)=\ln(4)

8) \dfrac{1}{2}\log_5\left(\dfrac{x}{4}+1\right) = \dfrac{1}{4}

  18) \ln\left(\dfrac{1}{5}e^7 \cdot 2x^2\right)=2\ln\left(6x\right)

9) \log_2(3)-\lg(100) = \log_2(2x-1)

  19) x \cdot \log_{10}(4x)=6x\cdot\log_{10}\left(\sqrt{10}\right)

10) \lg\left(x^2+6x+9\right)-\log_3(81) = \lg(x+3)   20) \log_{\pi}(37x) =14\log_{\pi}(e)

 

2. Aufgabe

Ermitteln Sie, ausgehend von der Funktion f(x)= \ln(x), die Graphen der folgenden Funktionen!

1) f_1(x)=\ln(x)+3    5) f_5(x)=3\ln(x)

2) f_2(x)=\ln(x)-3   6) f_6(x)= -3\ln(x)

3) f_3(x)=\ln(x+3)   7) f_7(x)=\ln(3x)
4) f_4(x)=\ln(x-3)

  8) f_8(x)=\ln(-3x)

 

3. Aufgabe

1)
Gegeben sei die Funktion f(x)= \dfrac{1}{4} \ln(9x-10) mit \mathbb{D} = \left\{x\in \mathbb{R} \mid x >\dfrac{10}{9}\right\}.
Gesucht ist jeweils die fehlende Koordinate des Punktes P(x\mid y),
a) wenn x=\dfrac{11}{9}
b) wenn y=0

  6)
Gegeben sei die Funktion f(x)= -\log_{2} \left( \dfrac{1}{16}x^2-\dfrac{1}{2}x \right) mit \mathbb{D} = \left\{x\in\mathbb{R} \mid x < 0 \text{ oder } x>8 \right\}.
Gesucht ist jeweils die fehlende Koordinate des Punktes P(x\mid y),
a) wenn x=9
b) wenn y=4

2)
Gegeben sei die Funktion f(x)= -\dfrac{\ln(x^2-8)+1}{20} mit \mathbb{D} = \left\{x\in\mathbb{R}\mid x < -\sqrt{8} \text{ oder } x>\sqrt{8}\right\}.
Gesucht ist jeweils die fehlende Koordinate des Punktes P(x\mid y),
a) wenn x=8
b) wenn y=-\dfrac{1}{20}

  7)
Gegeben sei die Funktion f(x)= \ln \left( \dfrac{-x+5}{12}\right) mit \mathbb{D} = \left\{x\in \mathbb{R} \mid x < 5 \right\}.
Gesucht ist jeweils die fehlende Koordinate des Punktes P(x\mid y),
a) wenn x=3
b) wenn y=\ln(7)

3)
Gegeben sei die Funktion f(x)= \log_8(x^3)+15 mit \mathbb{D} = \mathbb{R^+}.
Gesucht ist jeweils die fehlende Koordinate des Punktes P(x\mid y),
a) wenn x=10
b) wenn y=2

  8)
Gegeben sei die Funktion f(x)= \log_{10} (x)-3 mit \mathbb{D} = \mathbb{R}^+.
Gesucht ist jeweils die fehlende Koordinate des Punktes P(x\mid y),
a) wenn x=1.000
b) wenn y=2

4)
Gegeben sei die Funktion f(x)= \log_5(x^2+2x) mit \mathbb{D}=\left\{x\in \mathbb{R} \mid x < -2 \text{ oder } x>0 \right\}.
Gesucht ist jeweils die fehlende Koordinate des Punktes P(x\mid y),
a) wenn x=-5
b) wenn y=23

  9)
Gegeben sei die Funktion f(x)= 10 \ln (x^2-4x+4)-10 mit \mathbb{D} =\mathbb{R}\setminus_{\{2\}}.
Gesucht ist jeweils die fehlende Koordinate des Punktes P(x\mid y),
a) wenn x=-\dfrac{2}{5}
b) wenn y=30

5)
Gegeben sei die Funktion f(x)= -2 \log_{10}\left(x^4-\dfrac{8}{5} \right) mit \mathbb{D} =\left\{x\in\mathbb{R} \mid x < -\sqrt[4]{\dfrac{8}{5}} \text{ oder } x>\sqrt[4]{\dfrac{8}{5}}\right\}.
Gesucht ist jeweils die fehlende Koordinate des Punktes P(x\mid y),
a) wenn x=-6
b) wenn y=0

  10)
Gegeben sei die Funktion f(x)= \dfrac{2}{3} \log_2 \left(\dfrac{2}{x}\right)+1 mit \mathbb{D} = \mathbb{R}^+.
Gesucht ist jeweils die fehlende Koordinate des Punktes P(x\mid y),
a) wenn x=12
b) wenn y=-27

 

4. Aufgabe

Bestimmen Sie von folgenden Funktionen den Definitionsbereich und die Nullstellen!

1) f_1(x)=\ln\left((x-81)^2\right)

  11) f(x)=2\log_{10}(5x)+3\log_{10}(x)+8

2) f_2(x)=x\ln(x)+x\ln\left(x^2\right)+x\ln(27)

  12) f(x)=\ln(x-2)\cdot \dfrac{2x}{x^2-1}

3) f_3(x)=\log_7\left(6x^2+12x-17\right)

  13) f(z)=\ln\left(\dfrac{3z}{44z+11}\right)-\ln\left(-\dfrac{9}{22z}\right)

4) f_4(x)=\left(x^3-8\right)\lg(x+2)

  14) f(x)= 5\ln(-7+13x)+5\ln\left(\dfrac{x^2}{13x-7}\right)-10

5) f(x)=\left(x^2-4\right)\ln\left(x^2+x-6\right)

  15) f(z)= \log_3\left(5z^2\right)-\log_3(10z)-3

6) g(x)=3x\log_8\left(2\right)+x\log_8\left(2x\right)+2x\log_8\left(10x\right)

  16) f(x)= \dfrac{\log_4(-x+47)}{\log_4(e) \cdot \ln(-x+47)}\cdot \left(-14x^2+56x+98\right)

7) f(w)=(-100w^4-225)\cdot \log_7\left(w^2+17\right)

  17) f(x)= \log_{76} \left(12x^2+18x-11\right)

8) f(x)=\left(x^2-2\right)\cdot\log_{11}\left(x^2+2x+2\right)

  18) f(x)= \log_{28}\left(-\left(5x^2+1\right)\left(x^2+20\right)\right)

9) f(x)=\dfrac{\ln(6x)}{\sqrt{9}}+2

  19) f(x)= \dfrac{5}{4}\log_{10}\left(16e^x+1\right)\cdot \ln\left(x^3+9\right)

10) f(x)=\dfrac{\log_{12}\left(\frac{5}{4}x\right)\cdot\ln(7)}{\ln(2x)}   20) g(k)= \log_{314}(-k+9)+\log_{314}(4k)

Dieses Kapitel enthält die folgenden Themen:

 

17.2 Logarithmusgleichungen und -funktionen - Erklärungen

Zu guter Letzt bleiben aus dem Kapitel Potenzen, Wurzeln, Logarithmen noch die Logarithmen. Auch sie dürfen natürlich Bestandteil von Gleichungen und Funktionen sein. Logarithmische Zusammenhänge tauchen in verschiedenen Kontexten auf: Beispielsweise sind die Richterskala, auf der die Stärke von Erdbeben eingeordnet wird, und die Dezibel-Skala zum Abbilden von Lautstärken ebenso logarithmisch wie der pH-Wert. Für uns werden Anwendungen in der Zinsrechnung am relevantesten sein.
Wie der Name schon sagt: Wer sich (noch) nicht mit den Rechenregeln für Logarithmen vertraut gemacht hat, sollte dies nun nachholen. Ebenso hilft ein systematisches und umfassendes Verständnis der Potenzgesetze. Wir werden diese Rechenregeln und -gesetze im nun folgenden Kapitel brauchen.

 

Definition

Definition einer Logarithmusgleichung: Logarithmusgleichungen sind Gleichungen, bei denen die Variable im Argument eines Logarithmus steht.
Definition einer Logarithmusfunktion: Logarithmusfunktionen sind Funktionen, bei denen die Variable im Argument eines Logarithmus steht.
Anders formuliert: Eine Logarithmusfunktion ist eine Funktion, die jeder Zahl ihren Logarithmus zu einer festen Basis a\in\mathbb{R}^+\setminus{_{\{1\}}} zuordnet.

Exkurs zur Basis: 1 ist nie Basis einer Logarithmusgleichung oder -funktion. Der Logarithmus beantwortet ja die Frage, wie oft die Basis mit sich selbst multipliziert werden muss, damit das Ergebnis ein bestimmter Wert ist. Bei der Basis 1 gibt es darauf keine vernünftige Antwort, da 1^x = 1, egal welchen Wert x hat. Auf die Frage, wie oft 1 mit sich selbst multipliziert werden muss, damit das Ergebnis 1 ist, gibt es unendlich viele Lösungen. Auf die Frage, wie oft 1 mit sich selbst multipliziert werden muss, damit das Ergebnis z. B. -1 oder 2 ist, gibt es keine Lösungen. Das ist keine sinnvolle Grundlage für eine Gleichung/Funktion.
Aus quasi dem gleichen Grund ist 0 als Basis einer Logarithmusgleichung/-funktion nicht zugelassen.

Definitionsbereich: Auch bei dieser Art von Gleichung ist es wichtig, sich Gedanken zum Definitionsbereich zu machen, da das Argument ja niemals 0 oder negativ sein darf. Wir müssen also prüfen, für welche Variablenwerte das passiert, und den Definitionsbereich so "zuschneiden", dass nur passende Werte enthalten sind. Dafür müssen wir wieder Ungleichungen lösen.

 

Logarithmusgleichungen

Schauen wir uns einige Beispiele für Logarithmusgleichungen an:

  • \ln\left(\dfrac{1}{5}x-25\right) = 97 mit \mathbb{D} = \left]125;\infty\right[

  • -34\log_6\left(a^2-12a+53\right) = 0 mit \mathbb{D} = \mathbb{R}

  • \ln(e) = -\ln(17)x mit \mathbb{D} = \mathbb{R}
    Achtung: Diese Gleichung sieht nur aus wie eine Logarithmusgleichung, da in den Argumenten nur Zahlen stehen: \ln(e) = 1 (Das ist wichtig zu wissen!) und -\ln(17) \approx -2{,}83 (Das kann aber auch einfach als -\ln(17) stehen bleiben.). Da die Variable x außerhalb des Arguments steht und ansonsten nur Zahlen in der Gleichung enthalten sind, haben wir es hier also nur mit der linearen Gleichung 1 = -\ln(17)\cdot x zu tun. 

 

Lösungsweg

Wie bei allen Gleichungstypen, die wir bislang in diesem Lernmodul besprochen haben, ist es auch bei Logarithmusgleichungen das Ziel, die Gleichung so umzuformen, dass etwas entsteht, womit wir schon umgehen können, am liebsten also eine Polynomgleichung. Das klappt auch recht häufig durch Exponenzieren, sodass etliche Logarithmusgleichungen nur auf den ersten Blick wirklich schwierig sind. Grundsätzlich ist es eine gute Idee, die Bestandteile der Gleichung zunächst so weit wie möglich zusammenzufassen. Dabei helfen u. a. die Potenz- und Logarithmengesetze.
Zum Abschluss des Lösungsweges muss - wie schon bei den Bruchgleichungen - noch zwingend überprüft werden, ob sich alle ermittelten Werte für die Variable im Definitionsbereich befinden. Werte, die nicht im Definitionsbereich liegen, stellen keine Lösungen für die ursprüngliche Gleichung dar. Man bezeichnet sie daher als Scheinlösungen. Wie es dazu kommt, wird weiter unten erklärt.
Wie bei Exponentialgleichungen gibt es auch Logarithmusgleichungen, für die sich keine exakten Lösungen finden lassen, sodass man sie nur näherungsweise (also numerisch) lösen kann.

Zur Schreibweise: Mit e^{*} hinter dem senkrechten Strich am Ende einer Zeile ist gemeint, dass beide Seiten der Gleichung die Zahl e als Basis bekommen, also "e hoch" die entsprechenden Terme gerechnet wird; das gilt analog für andere Basen.


Beispiel 1:
Betrachten wir die Gleichung \ln(2x) = \ln(x+1).

Bestimmung des Definitionsbereichs:
Logarithmen sind nur für positive Argumente definiert. Daher müssen wir nun für beide in der Gleichung enthaltenen Logarithmen bestimmen, für welche x-Werte das der Fall ist.
linkes Argument:
\begin{array}{rclll}2x &>& 0 &\vert & :2\\x &>& 0 \end{array}

rechtes Argument:
\begin{array}{rclll}x+1 &>& 0 &\vert & -1 \\x &>& -1\end{array}

Für die abschließende Ermittlung des Definitionsbereichs bekommen wir also zwei Bedingungen: x>0 (für das linke Argument) und x>-1 (für das rechte Argument). Da beide Argumente Teil der Gleichung sind, müssen natürlich auch beide Bedingungen erfüllt sein. Dies müssen wir nun prüfen:

  • x-Werte kleiner oder gleich -1 erfüllen keine Bedingung.
  • x-Werte, die größer als -1, aber kleiner gleich 0 sind, erfüllen nur die zweite Bedingung.
    Sie können zur Veranschaulichung einen Wert zwischen -1 und 0 probehalber in den linken Logarithmus einsetzen: Nehmen wir z. B. -0{,}5. Das Argument von \ln\left(2\cdot(-0{,}5)\right) = \ln(-1) liegt nicht im zulässigen Bereich. Genau das wollen wir mit der Eingrenzung des Definitionsbereichs ja ausschließen.
  • Nur x-Werte größer als 0 erfüllen beide Bedingungen.

Der Definitionsbereich ist also \mathbb{D} = ]0;\infty[ = \mathbb{R}^+.

Lösung der Gleichung:
\begin{array}{crclcl} \text{1. Zeile:} & \ln(2x) &=& \ln(x+1) &\vert& e^{*} \cr\cr \text{2. Zeile:} & {e^{\ln(2x)}} &=& e^{\ln(x+1)} \cr\cr \text{3. Zeile:} & 2x &=& x+1 &\vert& -x \cr\cr& x &=& 1 \in\mathbb{D} \cr\cr\cr& \mathbb{L} &=& {1}\end{array}

Zur 1. und 2. Zeile: Die Gleichung wird exponenziert, weil Logarithmieren und Exponenzieren Umkehroperationen sind und sich somit gegenseitig aufheben, wenn die Basen übereinstimmen. Wichtig ist hier (wie bei allen anderen Gleichungstypen), dass solche Umformungen auf beiden Seiten der Gleichung durchgeführt werden.

Ab der 3. Zeile: Nach der ersten Umformung bleibt eine ziemlich simple lineare Gleichung.

Zum Abschluss: Der berechnete Wert liegt im Definitionsbereich und ist also Lösung der Gleichung.

 

Beispiel 2:
Betrachten wir die Gleichung \ln(e^x) = \ln(2)+\ln(7)-\ln\left(\dfrac{1}{2}\right)+2\ln\left(2^{-1}\right) .

Bestimmung des Definitionsbereichs: 
\begin{array}{rcl}e^x &>& 0\end{array}

Wir wissen bereits aus dem Kapitel Potenzen, Wurzeln, Logarithmen, dass e^x niemals 0 oder kleiner sein kann. Insofern ist diese Ungleichung für alle Werte von x erfüllt. Der Definitionsbereich ist also \mathbb{D} = \mathbb{R}.

Lösung der Gleichung:
\begin{array}{crclcl}\text{1. Zeile:} & \ln(e^x) &=& \ln(2)+\ln(7)-\ln\left(\dfrac{1}{2}\right)+2\ln\left(2^{-1}\right) \cr\cr \text{2. Zeile:} & x &=& \ln(2)+\ln(7)-\ln\left(\dfrac{1}{2}\right)+2\ln\left(2^{-1}\right) \cr\cr \text{3. Zeile:} & x &=& \ln\left(\frac{2\cdot 7}{\frac{1}{2}}\right)+\ln\left(2^{-2}\right) \cr\cr\text{4. Zeile:} & x &=& \ln\left(2\cdot 7\cdot 2\cdot 2^{-2}\right) \cr\cr & x &=& \ln(7) \in\mathbb{D} \cr\cr\cr& \mathbb{L} &=& \left\{\ln(7)\right\}\end{array}

Zur 1. Zeile: Da \ln(x) und e^x Umkehroperationen sind, vereinfacht sich die linke Seite der Gleichung sofort zu x.

Zur 2. Zeile: Auf der rechten Seite steht gar keine Variable, sodass nur zusammengefasst werden muss. Genutzt werden für die ersten drei Logarithmen das 1. und 2. Logarithmengesetz und für den letzten Logarithmus das 3. Logarithmengesetz.

Zur 3. Zeile: Es wird der Doppelbruch im ersten Logarithmus aufgelöst (Sie wissen ja, multiplizieren mit dem Kehrwert ...) und das 1. Logarithmengesetz erneut angewendet.

Ab der 4. Zeile: Um den Ausdruck auf der rechten Seite weiter zu vereinfachen, werden nun im Argument des Logarithmus die Potenzgesetze angewendet.

 

Nach Abschluss der Rechnung: Prüfung auf Scheinlösungen!
Wir hatten schon beim Lösen von Bruch- und Wurzelgleichungen gesehen, dass nicht jeder Wert, der berechnet wird, auch tatsächlich Lösung der Ausgangsgleichung sein muss, dass also sogenannte Scheinlösungen auftreten können. Bei Logarithmusgleichung kann das ebenfalls passieren. Schauen wir uns die Gleichung \ln\left(x\right) = -\ln(4x) als Beispiel an:

Bestimmung des Definitionsbereichs:
linkes Argument:
\begin{array}{rclcl}x &>& 0 \end{array}

rechtes Argument:
\begin{array}{rclcl}4x &>& 0 &\vert& :4 \\x &>& 0\end{array}

Für die abschließende Ermittlung des Definitionsbereichs bekommen wir wieder zwei Bedingungen: Diesmal sind die Bedingungen gleich, nämlich x>0. Der Definitionsbereich ist also \mathbb{D} = \mathbb{R}^+.

Lösung der Gleichung:
\begin{array}{rclcl}\ln(x) &=& -\ln(4x) &\vert& +\ln(4x) \cr \ln(x)+\ln(4x) &=& 0 &\vert& \text{1. Logarithmengesetz} \cr \ln\left(4x^2\right) &=& 0 &\vert& e^{*} \cr 4x^2 &=& 1 \cr& ... & \cr x_1 &=& \dfrac{1}{2} \cr x_2 &=& -\dfrac{1}{2}\end{array}

x_1=\frac{1}{2} und x_2=-\frac{1}{2} lösen zwar beide die quadratische Gleichung 4x^2=1, aber nur x_1=\frac{1}{2} löst auch die ursprüngliche Gleichung \ln(x)=-\ln(4x), weil x_2=-\frac{1}{2} außerhalb des zu Beginn bestimmten Definitionsbereichs \mathbb{D}=\mathbb{R}^+ liegt. Setzen wir x_2=-\frac{1}{2} in die Ausgangsgleichung ein, bekommen wir folglich ein Problem:
\ln\left(-\dfrac{1}{2}\right) = -\ln\left(4\cdot\left(-\dfrac{1}{2}\right)\right)
Das geht nicht, weil der Logarithmus für negative Werte nicht definiert ist! Ursache dafür ist, dass die quadratische Gleichung einen größeren Definitionsbereich hat als die Logarithmusgleichung (bei quadratischen Gleichungen umfasst der Definitionsbereich ja die gesamten reellen Zahlen). Beim Lösen der quadratischen Gleichung erhalten wir einen Wert, der nicht im ursprünglichen Definitionsbereich liegt und müssen diesen aus der Lösungsmenge unserer Beispielgleichung ausschließen. Es ist demnach: \mathbb{L}=\left\{\dfrac{1}{2}\right\}
Nach dem Lösen einer Logarithmusgleichung muss also immer geprüft werden, ob der berechnete Wert im Definitionsbereich liegt. Andersfalls handelt es sich um eine Scheinlösung.

 

Logarithmusfunktionen

Bei den Logarithmusfunktionen betrachten wir - anders als in den Kapiteln zuvor - nur zwei Fälle: einen mit den klassischen Logarithmusfunktionen (die alle eine Basis größer als 1 haben) und einen mit komplexeren. Logarithmusfunktionen mit Basen zwischen 0 und 1 kommen in Anwendungsfälle nicht sehr häufig vor, sodass wir uns hier nicht um sie kümmern müssen.

 

1. Fall

In der folgenden Grafik sind als Beispiele die Funktionen

  • f_1(x) = \ln(x)= \log_e(x) mit \mathbb{D} = \mathbb{R}^+

  • f_2(x) = \text{ld}(x) = \log_2(x) mit \mathbb{D} = \mathbb{R}^+

  • f_3(x) = \lg(x) = \log_{10}(x) mit \mathbb{D} = \mathbb{R}^+

dargestellt. Die typischen Basen e (natürlicher Logarithmus), 2 (Zweierlogarithmus) und 10 (Zehnerlogarithmus) haben wir damit abgedeckt.

Beispiele für Logarithmusfunktionen

Wertebereich: Der Wertebereich bei klassischen Logarithmusfunktionen ist \mathbb{D}=\mathbb{R}.
Da Logarithmusfunktionen eng mit Exponentialfunktionen zusammenhängen (um genau zu sein: sie sind Umkehrfunktionen), "vertauschen" die beiden Funktionstypen Definitions- und Wertebereich: Bei den Exponentialfunktionen ist \mathbb{D}=\mathbb{R} und \mathbb{W}=\mathbb{R}^+ und bei Logarithmusfunktionen \mathbb{D}=\mathbb{R}^+ und \mathbb{W}=\mathbb{R}.

Randverhalten: An den Rändern des Definitionsbereichs passiert Folgendes: x-Werte nahe der 0 haben sehr kleine Funktionswerte. Sehr große x-Werte haben auch sehr große Funktionswerte. Logarithmusfunktionen wachsen aber trotzdem langsamer als jedes Polynom.

Symmetrie: Die hier betrachteten Logarithmusfunktionen können - aufgrund des einseitig eingeschränkten Definitionsbereichs - nicht symmetrisch sein.

"Besondere Punkte": Einfache Logarithmusfunktionen haben eine Nullstelle bei x=1, keine Extrempunkte, Wende- und Polstellen.

Weitere Besonderheiten: Alle diese Logarithmusfunktionen verlaufen durch den Punkt (1 \mid 0), haben also eine gemeinsame Nullstelle.

 

2. Fall

Nun wird es (noch) interessanter, was die Funktionsverläufe angeht:

  • f_4(x) = \lg\left(x^2+x+6\right) mit \mathbb{D}=\mathbb{R}

  • f_5(x) = \ln\left(\sqrt[6]{(-x+13)^5}\right)-7 mit \mathbb{D}=]-\infty; 13[

  • f_6(x)=3\cdot\text{ld}\left(\dfrac{x+5}{12}\right)+25

  • f_7(x) = -4\ln\left(x^2-4x+4\right)+10 mit \mathbb{D}=\mathbb{R}\setminus_{\{2\}}

Beispiele für beliebige Logarithmusfunktionen

Wie bei den Funktionstypen, die wir in den Kapiteln zuvor betrachtet haben, kann man bei Logarithmusfunktionen, die über den einfachen Fall hinausgehen, keine allgemeinen Aussagen über den Verlauf und besondere Punkte der Graphen treffen. Schauen wir uns einen kleinen Ausschnitt von dem, was so möglich ist, an:

Wertebereich: Der Wertebereich kann (zumindest einseitig) beschränkt sein, wie bei f_4(x) mit \mathbb{W}=\left[\lg\left(\dfrac{23}{4}\right); \infty\right[. Bei den anderen hier abgebildeten Funktionen umfasst der Wertebereich alle reellen Zahlen, auch wenn dies bei f_5(x) vielleicht auf den ersten Blick nicht so aussieht ...

Symmetrie: Im Gegensatz zum einfachen Fall sind bei diesen Beispielen zwei achsensymmetrische Graphen dabei: f_4(x) ist achsensymmetrisch zu einer senkrechten Gerade durch x=-\dfrac{1}{2} und f_7(x) ist achsensymmetrisch zu einer senkrechten Gerade durch x=2. In beiden Fällen liegt dies an dem quadratischen Term im Argument.

"Besondere Punkte":
In der Grafik wird deutlich, dass nicht jede dieser komplexeren Funktionen eine Nullstelle hat (siehe z. B. f_4(x)). f_5(x) hat eine Nullstelle, die allerdings außerhalb des abgebildeten Bereichs liegt, nämlich bei x_N=13-e^{\frac{42}{5}} \approx -4.434{,}07, was nochmal deutlich macht, dass die Funktionswerte von f_5(x) im negativen Bereich immer größer werden, auch wenn der Graph hier nahezu konstant aussieht. Aufgrund des quadratischen Terms im Argument hat f_7(x) sogar zwei Nullstellen.
Da die Funktionsterme komplexer sind als im 1. Fall, können hier weitere "besondere Punkte" auftreten, die oben ausgeschlossen waren:

Beispielsweise hat f_4(x) einen Extrempunkt T \left(-\dfrac{1}{2} \mid \lg\left(\dfrac{23}{4}\right)\right). Wie man sehen kann, ist es ein Tiefpunkt.

Zudem hat die Funktion f_4(x) zwei Wendepunkte bei x_{W1}=\dfrac{1}{2}\left(-1-\sqrt{23}\right) \approx -2{,}90 und x_{W2}=\dfrac{1}{2}\left(-1+\sqrt{23}\right) \approx 1{,}90. Dort ändert sich jeweils das Krümmungsverhalten.

f_7(x) hat eine Polstelle bei x=2, weil dort ihr Argument 0 wird und Logarithmen in diesem Fall nicht definiert sind.

Abschließend fällt auf, dass alle hier betrachteten Beispielgraphen einen Schnittpunkt mit der y-Achse haben, was bei den einfachen Logarithmusfunktionen aus dem 1. Fall nicht auftritt. f_6(x) schneidet die y-Achse bei 3\cdot\text{ld}\left(\dfrac{5}{12}\right)+25\approx 21{,}21, also außerhalb des im Koordinatensystem abgebildeten Bereichs.

Und noch eine Besonderheit: f_6(x) sieht in dem hier betrachteten Bereich nahezu senkrecht aus, was ja der Funktionsdefinition widersprechen würde. Natürlich ist der Graph von f_6(x) nicht senkrecht, sondern ganz leicht "schief". Das würde man sehen, wenn man in das Koordinatensystem "hineinzoomen" würde.

Übersicht:

 

17.3 Logarithmusgleichungen und -funktionen - Lösungen

Erste Bemerkung zur Bestimmung des Definitionsbereichs: Auch bei unkomplizierten Logarithmen wie \ln(x) (Aufgabe 1.2) muss im Definitionsbereich berücksichtigt werden, dass das Argument nicht negativ oder 0 werden darf. Möglich sind in diesem Fall alle positiven reellen Zahlen, also ist \mathbb{D} = \mathbb{R}^+. Da im Fall von \ln(x) das Argument so einfach ist, wurde dafür in den folgenden Aufgaben keine Rechnung aufgeschrieben.
Um Logarithmen wie \ln(27) (Aufgabe 1.3) oder \log_{\pi}(e) (Aufgabe 1.20) müssen wir uns bei der Bestimmung des Definitionsbereichs nicht kümmern, weil \ln(27)\approx 3{,}30 bzw. \log_{\pi}(e)\approx 0{,}87 einfach Zahlen sind.

Zweite Bemerkung zur Bestimmung des Definitionsbereichs: Da das Argument größer als 0 sein muss, müssen zur Bestimmung des Definitionsbereichs Ungleichungen gelöst werden. Wenn Sie damit Schwierigkeiten haben, schauen Sie bitte im entsprechenden Kapitel nach.

 

1. Aufgabe

1)
Bestimmung des Definitionsbereichs:
1. Argument links:
\begin{array}{rclll}x-1 &>& 0 &\vert & +1\\x &>& 1 \\\\\end{array}

2. Argument links:
\begin{array}{rclll}x+2 &>& 0 &\vert & -2\\x &>& -2\\\end{array}

Für die abschließende Ermittlung des Definitionsbereichs muss nun überprüft werden, für welche x-Werte alle ermittelten Bedingungen erfüllt sind. Das ist nur der Fall für x > 1. Der Definitionsbereich ist also \mathbb{D} = ]1;\infty[.

Lösung der Gleichung:
\begin{array}{rclcl} \lg(x-1)+\lg(x+2) &=& 1 &\vert& \text{1. Logarithmengesetz} \cr \lg\left((x-1)(x+2)\right) &=& 1 &\vert& 10^{*} \cr 10^{\lg\left((x-1)(x+2)\right)} &=& 10^1 \cr (x-1)(x+2) &=& 10 \cr x^2+x-2 &=& 10 &\vert& -10 \cr x^2+x-12 &=& 0 &\vert& \text{p-q-Formel} \cr x_{1,2} &=& -\dfrac{1}{2} \pm \sqrt{\left(\dfrac{1}{2}\right)^2+12} \cr &=& -\dfrac{1}{2} \pm \sqrt{\dfrac{49}{4}} \cr\cr x_1 &=& -\dfrac{1}{2}+\dfrac{7}{2} = 3 \;\in\;\mathbb{D} \cr\cr x_2 &=& -\dfrac{1}{2}-\dfrac{7}{2} = -4 \;\not\in\; \mathbb{D} \\\\\mathbb{L} &=& \{3\}\end{array}


2)
Definitionsbereich: \mathbb{D} = \mathbb{R}^+

Lösung der Gleichung:
\begin{array}{rclcl} x \cdot \ln(x) &=& 2x &\vert& :x \cr \ln(x) &=& 2 &\vert& e^{*} \cr x &=& e^2 \approx 7{,}39 \;\in\;\mathbb{D} \\\\ \mathbb{L} &=& \left\{e^2\right\} \end{array}

Bemerkung: Die Division durch x ist hier ohne Einschränkungen möglich, weil x=0 \not\in \mathbb{D}. Eine Division durch 0 kann also nicht passieren.


3)
Definitionsbereich: \mathbb{D} = \mathbb{R}^+

Lösung der Gleichung:
\begin{array}{rclcl} 5\lg(x) &=& 3\lg(12)+\lg(32)-3(\lg(4)+\lg(3)) \cr\cr 5\lg(x) &=& 3\lg(12)+\lg(32)-3\lg(4)-3\lg(3) &\vert& \text{3. Logarithmengesetz} \cr\cr \lg(x^5) &=& \lg(12^3)+\lg(32)-\lg(4^3)-\lg(3^3) &\vert& \text{1. Logarithmengesetz} \cr\cr \lg(x^5) &=& \lg(12^3 \cdot 32)-\lg(4^3)-\lg(3^3) &\vert& \text{2. Logarithmengesetz} \cr\cr \lg(x^5) &=& \lg\left(\dfrac{12^3 \cdot 32}{4^3}\right)-\lg(3^3) &\vert& \text{2. Logarithmengesetz} \cr\cr \lg(x^5) &=& \lg\left(\dfrac{12^3 \cdot 32}{4^3 \cdot 3^3}\right) &\vert& \text{4. Potenzgesetz} \cr\cr \lg(x^5) &=& \lg\left(\dfrac{12^3 \cdot 32}{(4 \cdot 3)^3}\right) \cr\cr \lg(x^5) &=& \lg\left(\dfrac{12^3 \cdot 32}{12^3}\right) \cr\cr \lg(x^5) &=& \lg(32) &\vert& 10^{*} \cr\cr 10^{\lg(x^5)} &=& 10^{\lg(32)} \cr\cr x^5 &=& 32 &\vert& \sqrt[5]{} \cr\cr x &=& 2 \;\in\;\mathbb{D} \\\\ \mathbb{L} &=& \{2\} \end{array}


4)
Bestimmung des Definitionsbereichs:
Argument links:
\begin{array}{rclll}x-75 &>& 0 &\vert & +75\\x &>& 75\\\\\end{array}

Argument rechts:
\begin{array}{rclll}x+3 &>& 0 &\vert & -3\\x &>& -3\end{array}

Für die abschließende Ermittlung des Definitionsbereichs muss nun überprüft werden, für welche x-Werte alle ermittelten Bedingungen erfüllt sind. Das ist nur der Fall für x > 75. Der Definitionsbereich ist also \mathbb{D} = ]75;\infty[.

Lösung der Gleichung:
\begin{array}{rclcl} \log_3(x-75)-3 &=& \log_3(x+3) &\vert& -\log_3(x+3)+3 \cr\cr \log_3(x-75)-\log_3(x+3) &=& 3 &\vert& \text{2. Logarithmengesetz} \cr\cr \log_3\left(\dfrac{x-75}{x+3}\right) &=& 3 &\vert& 3^{*} \cr\cr 3^{\log_3\left(\dfrac{x-75}{x+3}\right)} &=& 3^3 \cr\cr \dfrac{x-75}{x+3} &=& 27 &\vert& \cdot (x+3)\cr x-75 &=& 27(x+3) \cr x-75 &=& 27x+81 &\vert& +75-27x \cr -26x &=& 156 &\vert& :\left(-26\right) \cr x &=& -6 \; \not \in \; \mathbb{D} \cr\cr & \mathbb{L} = \emptyset \end{array}

Bemerkung: Die Multiplikation mit (x+3) ist hier problemlos möglich, weil x=-3 \not\in \mathbb{D}. Eine Multiplikation mit 0 kann also nicht passieren.


5)
Bestimmung des Definitionsbereichs:
1. Argument links:
\begin{array}{rclll}x-2 &>& 0 &\vert & +2\\x &>& 2\\\\\end{array}

2. Argument links:
\begin{array}{rclll}x+3 &>& 0 &\vert & -3\\x &>& -3\\\\\end{array}

1. Argument rechts:
\begin{array}{rclll}x^2+x-6 &>& 0\end{array}

Um diese quadratische Ungleichung zu lösen, wird im ersten Schritt die zugehörige quadratische Gleichung gelöst:
\begin{array}{rclll}x^2+x-6 &=& 0 &\vert & \text{p-q-Formel}\\x_{1,2} &=& -\dfrac{1}{2}\pm\sqrt{\left(\dfrac{1}{2}\right)^2+6}\\x_{1,2} &=& -\dfrac{1}{2}\pm\sqrt{\dfrac{1}{4}+\dfrac{24}{4}}\\x_{1,2} &=& -\dfrac{1}{2}\pm\dfrac{5}{2}\\\\x_1 &=& 2\\x_2 &=& -3\end{array}

Diese Gleichung hat also zwei Lösungen bei x_1=2 und x_2=-3. Diese x-Werte sind auf jeden Fall nicht Teil des Definitionsbereichs. Es ergeben sich die Intervalle ]-\infty;-3[, ]-3;2[ und ]2;\infty[.
Jetzt muss noch geprüft werden, in welchen Intervallen der Term positive bzw. negative Werte annimmt. Es ist sinnvoll, für die Prüfung möglichst einfache x-Werte zu verwenden, Z. B. liefert x=-4 als Ergebnis 6, für x=0 ergibt sich -6 und x=3 führt zu 6. Das bedeutet, dass x^2+x-6 im ersten und dritten Intervall positiv ist. Die ursprüngliche Ungleichung x^2+x-6 > 0 ist also nur in diesen beiden Intervallen wahr.

Für die abschließende Ermittlung des Definitionsbereichs muss nun überprüft werden, für welche x-Werte alle ermittelten Bedingungen erfüllt sind. Das ist nur der Fall für x > 2. Der Definitionsbereich ist also \mathbb{D} = ]2;\infty[.

Lösung der Gleichung:
\begin{array}{rclcl} \ln(x-2)+\ln(x+3) &=& \ln(x^2+x-6) &\vert& \text{1. Logarithmengesetz} \cr \ln\left((x-2)(x+3)\right) &=& \ln(x^2+x-6) \cr \ln\left(x^2+x-6\right) &=& \ln\left(x^2+x-6\right) &\vert& e^{*} \cr e^{\ln\left(x^2+x-6\right)} &=& e^{\ln\left(x^2+x-6\right)} \cr x^2+x-6 &=& x^2+x-6 &\vert& -x^2-x+6 \cr 0 &=& 0 \cr\cr \mathbb{L} &=& ]2;\infty[ \end{array}

Bemerkung: Unabhängig davon, welches Element des Definitionsbereichs in diese Gleichung eingesetzt wird, erhält man immer auf beiden Seiten dasselbe Ergebnis. 0=0 ist schließlich immer richtig. Jede reelle Zahl löst also diese Gleichung, d. h. die Lösungsmenge entspricht dem Definitionsbereich.


6)
Bestimmung des Definitionsbereichs:
1. Argument links:
\begin{array}{rclll}16^x &>& 0 \end{array}

2. Argument links:
\begin{array}{rclll}25^{x^2} &>& 0\end{array}

Da Potenzen mit einer positiven Basis, wie 16 und 25, ausschließlich positive Werte annehmen, sind die gegebenen Ungleichungen für alle x-Werte wahr. Der Definitionsbereich ist also \mathbb{D} = \mathbb{R}.

Lösung der Gleichung:
\begin{array}{rclcl} \log_2\left(16^x\right)+\log_5\left(25^{x^2}\right) &=& 16 &\vert& \text{3. Logarithmengesetz} \cr x\cdot \log_2(16)+x^2\cdot \log_5(25) &=& 16 &\vert& \text{Logarithmen berechnen} \cr 4x+2x^2 &=& 16 &\vert& -16 \cr 2x^2+4x-16 &=& 0 &\vert& :2 \cr x^2+2x-8 &=& 0 &\vert& \text{p-q-Formel} \cr x_{1,2} &=& -1 \pm \sqrt{1+8} \cr &=& -1 \pm \sqrt{9} \cr \cr x_1 &=& -1+3 = 2 \;\in\;\mathbb{D} \cr x_2 &=& -1-3 = -4 \;\in\;\mathbb{D} \\\\ \mathbb{L} &=& \{-4;2\} \end{array}


7)
Bestimmung des Definitionsbereichs:
1. Argument links:
Da das Argument des ersten Logarithmus \sqrt{5x+2} eine Quadratwurzel ist und solche Wurzeln ausschließlich nichtnegative Ergebnisse liefern, muss nur vermieden werden, dass die Wurzel den Wert 0 annimmt:
\begin{array}{rclcl} \sqrt{5x+2} &\neq& 0 &\vert& ()^2 \\ 5x+2 &\neq& 0 &\vert& -2 \\ 5x &\neq& -2 &\vert& :5 \\ x &\neq& -\dfrac{2}{5}\end{array}

Radikand im 1. Argument links:
\begin{array}{rclll} 5x+2 &\geq& 0 &\vert& -2\\5x &\geq& -2 &\vert& :5 \\x &\geq& -\dfrac{2}{5}\end{array}

2. Argument links:
Da das Argument des zweiten Logarithmus (5x+2)^2 ein quadratischer Ausdruck ist und Quadrate im Bereich der reellen Zahlen immer nichtnegativ sind, muss nur ausgeschlossen werden, dass das Quadrat den Wert 0 annimmt:
\begin{array}{rclll}\left(5x+2\right)^2 &\neq& 0 &\vert & \pm\sqrt{}\\5x+2 &\neq& 0\\x &\neq& -\dfrac{2}{5}\end{array}

Für die abschließende Ermittlung des Definitionsbereichs muss nun überprüft werden, für welche x-Werte alle ermittelten Bedingungen erfüllt sind. Das ist nur der Fall für x > -\dfrac{2}{5}. Der Definitionsbereich ist also \mathbb{D} = \left]-\dfrac{2}{5};\infty\right[.

Lösung der Gleichung:
\begin{array}{rclcl} \ln\left(\sqrt{5x+2}\right)+\ln\left((5x+2)^2\right) &=& 1 \cr \ln\left((5x+2)^{\frac{1}{2}}\right)+\ln\left((5x+2)^2\right) &=& 1 &\vert& \text{3. Logarithmengesetz} \cr \dfrac{1}{2}\ln(5x+2)+2\ln(5x+2) &=& 1 \cr \dfrac{5}{2}\ln(5x+2) &=& 1 &\vert& :\dfrac{5}{2} \cr \ln(5x+2) &=& \dfrac{2}{5} &\vert& e^{*} \cr e^{\ln(5x+2)} &=& e^{\frac{2}{5}} \cr 5x+2 &=& e^{\frac{2}{5}} &\vert& -2 \cr 5x &=& e^{\frac{2}{5}}-2 &\vert& : 5 \cr x &=& \dfrac{e^{\frac{2}{5}}-2}{5} \approx -0{,}10 \;\in\;\mathbb{D} \\\\ \mathbb{L} &=& \left\{\dfrac{e^{\frac{2}{5}}-2}{5}\right\} \end{array}


8)
Bestimmung des Definitionsbereichs:
\begin{array}{rclll} \dfrac{x}{4}+1 &>& 0 &\vert & -1\\\dfrac{1}{4}x &>& -1 &\vert & : \dfrac{1}{4}\\x &>& -4\end{array}

Der Definitionsbereich ist also \mathbb{D} = \left]-4;\infty\right[.

Lösung der Gleichung:
\begin{array}{rclcll}\dfrac{1}{2}\log_5\left(\dfrac{x}{4}+1\right) &=& \dfrac{1}{4} &\vert & :\dfrac{1}{2} \\\\\log_5\left(\dfrac{x}{4}+1\right) &=& \dfrac{1}{2} &\vert& 5^{*} \\\\5^{\log_5\left(\dfrac{x}{4}+1\right)} &=& 5^{\frac{1}{2}} \\\\\dfrac{x}{4}+1 &=& \sqrt{5} &\vert& -1 \\\dfrac{x}{4} &=& \sqrt{5}-1 &\vert& \cdot 4 \\x &=& 4\sqrt{5}-4 \approx 4{,}94 \;\in\;\mathbb{D} \\\\\mathbb{L} &=& \left\{4\sqrt{5}-4\right\}\end{array}


9)
Bestimmung des Definitionsbereichs:
\begin{array}{rclll} 2x-1 &> & 0 &\vert & +1\\2x &> & 1 &\vert & :2\\x &> & \dfrac{1}{2}\end{array}

Der Definitionsbereich ist also \mathbb{D} = \left ]\dfrac{1}{2};\infty\right[.

Lösung der Gleichung:
\begin{array}{rclclll}\log_2(3)-\lg(100) &=& \log_2(2x-1) &\vert& -\log_2(2x-1) +\lg(100) \\\log_2(3)-\log_2(2x-1) &=& \lg\left(10^2\right) &\vert& \text{2. Logarithmengesetz} \\\log_2\left(\dfrac{3}{2x-1}\right) &=& 2 &\vert& 2^{*} \\2^{\log_2\left(\dfrac{3}{2x-1}\right)} &=& 2^2 \\\dfrac{3}{2x-1} &=& 4 &\vert& \cdot (2x-1) \\3 &=& 8x-4 &\vert& +4 \\7 &=& 8x &\vert& :8 \\x &=& \dfrac{7}{8} \;\in\;\mathbb{D} \\ \\\mathbb{L} &=& \left\{\dfrac{7}{8}\right\}\end{array}

Bemerkung: Die Multiplikation mit (2x-1) ist hier ohne Einschränkungen möglich, weil x=\dfrac12 \not\in \mathbb{D}. Eine Multiplikation mit 0 kann also nicht passieren.


10)
Bestimmung des Definitionsbereichs:
1. Argument links:
\begin{array}{rclll}x^2+6x+9 &> & 0 \\(x+3)^2 &>& 0\end{array}

Da ein Quadrat im Bereich der reellen Zahlen immer nichtnegativ ist, muss im vorliegenden Fall nur vermieden werden, dass der quadratische Ausdruck 0 wird:
\begin{array}{rclcl}(x+3)^2 &\neq& 0 &\vert& \pm\sqrt{} \\ x+3 &\neq & 0 &\vert & -3\\x &\neq & -3\\\\\end{array}

Argument rechts:
\begin{array}{rclll}x+3 &> & 0 &\vert & -3\\x &> & -3\end{array}

Für die abschließende Ermittlung des Definitionsbereichs muss nun überprüft werden, für welche x-Werte alle ermittelten Bedingungen erfüllt sind. Das ist nur der Fall für x > -3. Der Definitionsbereich ist also \mathbb{D} = ]-3;\infty[.

Lösung der Gleichung:
\begin{array}{rclclll}\lg\left(x^2+6x+9\right)-\log_3(81) &=& \lg(x+3) &\vert& +\log_3(81) -\lg(x+3) \\\lg\left((x+3)^2\right)-\lg(x+3) &=& \log_3\left(3^4\right) &\vert& \text{2. Logarithmengesetz} \\\lg\left(\dfrac{(x+3)^2}{x+3}\right) &=& 4 \\\lg(x+3) &=& 4 &\vert& 10^{*} \\10^{\lg(x+3)} &=& 10^4 \\x+3 &=& 10.000 &\vert& -3 \\x &=& 9.997 \;\in\;\mathbb{D} \\ \\\mathbb{L} &=& \{9.997\}\end{array}


11)
Bestimmung des Definitionsbereichs:
\begin{array}{rclll} x-1 &>& 0 &\vert & +1\\x &>& 1\end{array}

Der Definitionsbereich ist also \mathbb{D} = ]1;\infty[.

Lösung der Gleichung:
\begin{array}{rclclll}2\ln(x-1)+2\ln(3) &=& 2\ln(1)+3\ln(2) &\vert& \text{3. Logarithmengesetz} \\\ln\left((x-1)^2\right)+\ln\left(3^2\right) &=& 0+\ln\left(2^3\right) &\vert& -\ln\left(3^2\right) \\\ln\left((x-1)^2\right) &=& \ln\left(2^3\right)-\ln\left(3^2\right) &\vert& \text{2. Logarithmengesetz}\\\ln\left((x-1)^2\right) &=& \ln\left(\dfrac{2^3}{3^2}\right) &\vert& e^{*} \\(x-1)^2 &=& \dfrac{8}{9} &\vert& \pm\sqrt{} \\x-1 &=& \pm\sqrt{\dfrac{8}{9}} &\vert&+1 \\\\x_1 &=& 1+\dfrac{2\sqrt{2}}{3} \approx 1{,}94 \;\in\;\mathbb{D} \\ x_2 &=& 1-\dfrac{2\sqrt{2}}{3} \approx 0{,}06 \;\not\in\;\mathbb{D} \\ \\\mathbb{L} &=& \left\{1+\dfrac{2\sqrt{2}}{3}\right\}\end{array}


12)
Bestimmung des Definitionsbereichs:
\begin{array}{rclll}ex+2 &>& 0 &\vert& -2\\ex &>& -2 &\vert & :e\\x &>& -\dfrac{2}{e}\end{array}

Der Definitionsbereich ist also \mathbb{D} = \left]-\dfrac{2}{e};\infty\right[.

Lösung der Gleichung:
\begin{array}{rclcll}8\ln(ex+2) &=& 8 &\vert &:8\\\ln(ex+2) &=& 1 &\vert &e^{*}\\ex+2 &=& e &\vert & -2 \\ex &=& e-2 &\vert & :e \\x &=& 1-\dfrac{2}{e} \approx 0{,}26 \;\in\;\mathbb{D} \\ \\\mathbb{L} &=& \left\{1-\dfrac{2}{e}\right\}\end{array}


13)
Bestimmung des Definitionsbereichs:
1. Argument links:
Da das Argument des ersten Logarithmus \sqrt{-s+13} eine Quadratwurzel ist und solche Wurzeln ausschließlich nichtnegative Ergebnisse liefern, muss nur vermieden werden, dass die Wurzel den Wert 0 annimmt:
\begin{array}{rclll}\sqrt{-s+13} &\neq& 0 &\vert& ()^2 \\ -s+13 &\neq& 0 &\vert & -13 \\ -s &\neq& -13 &\vert& \cdot (-1) \\ s & \neq & 13\end{array}

Radikand im 1. Argument links:
\begin{array}{rclll}-s+13 &\geq& 0 &\vert & -13\\ -s &\geq& -13 &\vert& \cdot (-1) \\ s &\leq& 13\end{array}

2. Argument links:
Da das Argument des zweiten Logarithmus \sqrt[3]{-s+13} eine 3. Wurzel ist, könnte das Ergebnis der Wurzel auch negativ oder 0 sein. Um das auszuschließen, muss der Radikand -s+13 einen positiven Wert haben. Die zugehörige Ungleichung -s+13 > 0 wurde aber schon bei der Betrachtung des Radikanden im 1. Argument auf der linken Gleichungsseite gelöst.

Für die abschließende Ermittlung des Definitionsbereichs muss nun überprüft werden, für welche s-Werte alle ermittelten Bedingungen erfüllt sind. Das ist nur der Fall für s < 13. Der Definitionsbereich ist also \mathbb{D} = ]-\infty; 13[.

Lösung der Gleichung:
\begin{array}{rclcll}\ln\left(\sqrt{-s+13}\right)+\ln\left(\sqrt[3]{-s+13}\right)-1 &=& 0 &\vert& +1 \\ \\\ln\left(\sqrt{-s+13}\right)+\ln\left(\sqrt[3]{-s+13}\right) &=& 1 \\ \\\ln\left((-s+13)^{\frac{1}{2}}\right)+\ln\left((-s+13)^{\frac{1}{3}}\right) &=& 1 \\ \\\ln\left((-s+13)^{\frac{1}{2}}\cdot (-s+13)^{\frac{1}{3}}\right) &=& 1 \\ \\\ln\left((-s+13)^{\frac{5}{6}}\right) &=& 1 &\vert& e^{*} \\(-s+13)^{\frac{5}{6}} &=& e &\vert& ()^6 \\(-s+13)^5 &=& e^6 &\vert& ()^{\frac{1}{5}} \\-s+13 &=& e^{\frac{6}{5}} &\vert& -13\\-s &=& e^{\frac{6}{5}}-13 &\vert & \cdot (-1) \\s &=& -e^{\frac{6}{5}}+13 \approx 9{,}68 \;\in\;\mathbb{D}\end{array}

Da diese Logarithmusgleichung Wurzeln enthält, muss eine Probe durchgeführt werden:
\begin{array}{rclll}\ln\left(\sqrt{-(-e^{\frac{6}{5}}+13)+13}\right)+\ln\left(\sqrt[3]{-(-e^{\frac{6}{5}}+13)+13}\right)-1 &=& 0 \\\ln\left(\sqrt{e^{\frac{6}{5}}-13+13}\right)+\ln\left(\sqrt[3]{e^{\frac{6}{5}}-13+13}\right)-1 &=& 0 \\\ln\left(\sqrt{e^{\frac{6}{5}}}\right)+\ln\left(\sqrt[3]{e^{\frac{6}{5}}}\right)-1 &=& 0 \\\ln\left(e^{\frac{6}{10}}\right)+\ln\left(e^{\frac{6}{15}}\right)-1 &=& 0 \\\frac{6}{10}\ln\left(e\right)+\frac{6}{15}\ln\left(e\right)-1 &=& 0 \\\frac{6}{10}\cdot 1+\frac{6}{15}\cdot 1-1 &=& 0 \\\frac{18}{30}+\frac{12}{30}-\frac{30}{30} &=& 0 \\0 &=& 0\end{array}

Es ergibt sich eine wahre Aussage: \mathbb{L} = \left\{-e^{\frac{6}{5}}+13\right\}


14)
Bestimmung des Definitionsbereichs:
1. Argument links:
\begin{array}{rclcl}4x^{\frac{7}{8}} &>& 0 &\vert & :4 \\\sqrt[8]{x^7} &>& 0 &\vert & ()^8 \\x^7 &>& 0 &\vert & \sqrt[7]{}\\x &>& 0 \\\\\end{array}

2. Argument links:
\begin{array}{rclcl}3x^{-\frac{1}{8}} &>& 0 &\vert & :3 \\\dfrac{1}{\sqrt[8]{x}} &>& 0 &\vert & ()^8 \\\dfrac{1}{x} &>& 0 &\vert & \cdot x^2\\x &>& 0 \\\\\end{array}

Bemerkung: Durch die Multiplikation mit x^2 kann eine Fallunterscheidung bei der Ungleichung vermieden werden, da ein Quadrat im Bereich der reellen Zahlen immer nichtnegativ ist. Somit ist eine Umkehrung des Vergleichszeichens ausgeschlossen.

Argument rechts:
\begin{array}{rclcl}\sqrt[7]{16}x &>& 0 &\vert & :\sqrt[7]{16} \\x &>& 0\end{array}

Für die abschließende Ermittlung des Definitionsbereichs muss nun überprüft werden, für welche x-Werte alle ermittelten Bedingungen erfüllt sind. Das ist der Fall für x > 0. Der Definitionsbereich ist also \mathbb{D}=\mathbb{R}^+.

Lösung der Gleichung:
\begin{array}{rclcl}\ln\left(4x^{\frac{7}{8}}\right)+\ln\left(3x^{-\frac{1}{8}}\right) &=& \dfrac{7}{4}\ln\left(\sqrt[7]{16}x\right) &\vert& \text{1. Logarithmengesetz} \\\\\ln\left(4x^{\frac{7}{8}}\cdot 3x^{-\frac{1}{8}}\right) &=& \dfrac{7}{4}\ln\left(\sqrt[7]{16}x\right) &\vert& \text{3. Logarithmengesetz} \\\ln\left(12x^{\frac{3}{4}}\right) &=& \ln\left(\left(\sqrt[7]{16}x\right)^{\frac{7}{4}}\right) \\\ln\left(12x^{\frac{3}{4}}\right) &=& \ln\left(\left(\sqrt[7]{16}\right)^{\frac{7}{4}}x^{\frac{7}{4}}\right) \\\ln\left(12x^{\frac{3}{4}}\right) &=& \ln\left(2x^{\frac{7}{4}}\right) &\vert& e^* \\12x^{\frac{3}{4}} &=& 2x^{\frac{7}{4}} &\vert& :2 \\6x^{\frac{3}{4}} &=& x^{\frac{7}{4}} &\vert& :x^{\frac{3}{4}} \\6 &=& x^{\frac{7}{4}-\frac{3}{4}} \\6 &=& x^{1} \\6 &=& x \in \mathbb{D} \\\\\mathbb{L} &=& \{6\}\end{array}

Bemerkung: Die Division durch x^{\frac{3}{4}} ist hier ohne Einschränkungen möglich, weil x=0 \not\in \mathbb{D}. Eine Division durch 0 kann also nicht passieren.


15)
Bestimmung des Definitionsbereichs:
Argument:
\begin{array}{rclcl}x-3 &>& 0 &\vert& +3 \\x &>& 3\\\\\end{array}

Nenner:
\begin{array}{rclcl}\log_4(x) &\neq& 0 &\vert& 4^* \\x &\neq& 1\end{array}

Für die abschließende Ermittlung des Definitionsbereichs muss nun überprüft werden, für welche x-Werte alle ermittelten Bedingungen erfüllt sind. Das ist nur der Fall für x > 3. Der Definitionsbereich ist also \mathbb{D}=\left]3;\infty \right[.

Lösung der Gleichung:
\begin{array}{rclcl}\dfrac{\log_4(x-3)}{\log_4(x)} &=& -1 &\vert& \cdot \log_4(x) \\\log_4(x-3) &=& -\log_4(x) \\\log_4(x-3) &=& \log_4\left(x^{-1}\right) &\vert& 4^* \\x-3 &=& x^{-1} &\vert& \cdot x \\x^2-3x &=& 1 &\vert& -1 \\x^2-3x-1 &=& 0 &\vert& \text{p-q-Formel} \\x_{1,2} &=& \dfrac{3}{2} \pm \sqrt{\left(-\dfrac{3}{2}\right)^2+1} \\x_{1,2} &=& \dfrac{3}{2} \pm \sqrt{\dfrac{13}{4}} \\\\x_1 &=& \dfrac{3}{2}+\sqrt{\dfrac{13}{4}} \approx 3{,}30 \in \mathbb{D} \\x_2 &=& \dfrac{3}{2}-\sqrt{\dfrac{13}{4}} \approx -0{,}30 \not \in \mathbb{D} \\\\\mathbb{L} &=& \left\{\dfrac{3}{2}+\sqrt{\dfrac{13}{4}}\right\}\end{array}

Bemerkung: Die Multiplikationen mit \log_4(x) und x sind hier ohne Einschränkungen möglich, weil x=1 \not\in \mathbb{D} und x=0 \not\in \mathbb{D}. Eine Multiplikation mit 0 kann also nicht passieren.


16)
Bestimmung des Definitionsbereichs:
Argument links:
\begin{array}{rclcl}\sqrt[3]{9x^2} &>& 0 &\vert & ()^3 \\9x^2 &>& 0 &\vert & :9 \\x^2 &>& 0\end{array}

Da ein Quadrat im Bereich der reellen Zahlen immer nichtnegativ ist, muss im vorliegenden Fall nur vermieden werden, dass der quadratische Ausdruck 0 wird:
\begin{array}{rclcl}x^2 &\neq& 0 &\vert &\pm\sqrt{}\\x &\neq& 0 \\\\\end{array}

Argument rechts:
Da für x schon bei der Betrachtung des linken Arguments ausgeschlossen wurden, dass die Variable den Wert 0 annimmt, wurde damit auch gleichzeitig sichergestellt, dass im rechten Argument keine Division durch 0 auftritt.

\begin{array}{rclcl}\dfrac{1}{x} &>& 0&\vert &\cdot x^2 \\x &>& 0\end{array}

Bemerkung: Durch die Multiplikation mit x^2 kann eine Fallunterscheidung bei der Ungleichung vermieden werden, da ein Quadrat im Bereich der reellen Zahlen immer nichtnegativ ist. Somit ist eine Umkehrung des Vergleichszeichens ausgeschlossen.

Für die abschließende Ermittlung des Definitionsbereichs muss nun überprüft werden, für welche x-Werte alle ermittelten Bedingungen erfüllt sind. Das ist nur der Fall für x > 0. Der Definitionsbereich ist also \mathbb{D}=\mathbb{R}^+.

Lösung der Gleichung:
\begin{array}{rclcl}\log_{27}\left(\sqrt[3]{9x^2}\right) &=& \log_{27}\left(\dfrac{1}{x}\right) &\vert& 27^* \\\sqrt[3]{9x^2} &=& \dfrac{1}{x} &\vert& ( )^3 \\\\9x^2 &=& \dfrac{1}{x^3} &\vert& \cdot x^3 \\\\9x^5 &=& 1 &\vert& :9 \\\\x^5 &=& \dfrac{1}{9} &\vert& \sqrt[5]{} \\x &=& \sqrt[5]{\dfrac{1}{9}} \approx 0{,}66 \in \mathbb{D}\end{array}

Bemerkung: Die Multiplikation mit x^3 ist hier ohne Einschränkungen möglich, weil x=0 \not\in \mathbb{D}. Eine Multiplikation mit 0 kann also nicht passieren.

Auch hier muss eine Probe durchgeführt werden, weil in der Gleichung eine Wurzel enthalten ist:
\begin{array}{rclcll}\log_{27}\left(\sqrt[3]{9\left(\sqrt[5]{\dfrac{1}{9}}\right)^2}\right) &=& \log_{27}\left(\dfrac{1}{\sqrt[5]{\dfrac{1}{9}}}\right)\\\log_{27}\left(\sqrt[3]{9\left(\dfrac{1}{\sqrt[5]{9}}\right)^2}\right) &=& \log_{27}\left(\sqrt[5]{9}\right)\\\log_{27}\left(\sqrt[3]{9\dfrac{1}{\left(\sqrt[5]{9}\right)^2}}\right) &=& \log_{27}\left(9^{\frac{1}{5}}\right) \\\log_{27}\left(\sqrt[3]{\dfrac{9}{9^{\frac{2}{5}}}}\right) &=& \dfrac{1}{5}\log_{27}9 \\\log_{27}\left(\sqrt[3]{9^{\frac{3}{5}}}\right) &=& \dfrac{1}{5}\log_{27}9\\\log_{27}\left(9^{\frac{3}{5}\cdot\frac{1}{3}}\right) &=& \dfrac{1}{5}\log_{27}9\\\log_{27}\left(9^{\frac{1}{5}}\right) &=& \dfrac{1}{5}\log_{27}9\\\dfrac{1}{5}\log_{27}9 &=& \dfrac{1}{5}\log_{27}9\end{array}

Es ergibt sich eine wahre Aussage: \mathbb{L} = \left\{\sqrt[5]{\dfrac{1}{9}}\right\}


17)
Bestimmung des Definitionsbereichs:
\begin{array}{rclcl}-3x+15 & > & 0 & \vert& -15 \\-3x & > & -15 &\vert& :(-3) \\x & < & 5\end{array}

Der Definitionsbereich ist also \mathbb{D}=\left]-\infty;5 \right[.

Lösung der Gleichung:
\begin{array}{rclcl}\log_{4}(-3x+15) &=& \ln(4) &\vert& 4^* \\\\-3x+15 &=& 4^{\ln(4)} &\vert& -15 \\-3x &=& 4^{\ln(4)} -15 &\vert& :(-3) \\x &=& -\dfrac{4^{\ln(4)}-15}{3} \approx 2{,}72 \in\mathbb{D} \\\\\mathbb{L} &=& \left\{-\dfrac{4^{\ln(4)}-15}{3}\right\}\end{array}


18)
Bestimmung des Definitionsbereichs:
Argument links:
\begin{array}{rclcl}\dfrac{1}{5}e^7\cdot 2x^2 &>& 0 &\vert & :\left(\dfrac{1}{5}e^7\cdot 2\right) \\x^2 &>& 0\end{array}

Da ein Quadrat im Bereich der reellen Zahlen immer nichtnegativ ist, muss im vorliegenden Fall nur vermieden werden, dass der quadratische Ausdruck 0 wird:
\begin{array}{rclcl}x^2 &\neq& 0 &\vert &\pm\sqrt{}\\x &\neq& 0 \\\\\end{array}

Argument rechts:
\begin{array}{rclcl}6x &>& 0 &\vert & :6 \\x &>& 0\end{array}

Für die abschließende Ermittlung des Definitionsbereichs muss nun überprüft werden, für welche x-Werte alle ermittelten Bedingungen erfüllt sind. Das ist nur der Fall für x > 0. Der Definitionsbereich ist also \mathbb{D}=\mathbb{R}^+.

Lösung der Gleichung:
\begin{array}{rclcl}\ln\left(\dfrac{1}{5}e^7 \cdot 2x^2\right) &=& 2\ln\left(6x\right) &\vert & \text{3. Logarithmengesetz} \\\ln\left(\dfrac{2}{5}e^7x^2\right) &=& \ln\left(36x^2\right) &\vert & e^* \\\dfrac{2}{5}e^7x^2 &=& 36x^2 &\vert & :x^2 \\\dfrac{2}{5}e^7 &=& 36 \\\end{array}

Da dies ein Widerspruch ist, hat diese Gleichung keine Lösung: \mathbb{L}=\emptyset

Bemerkung: Die Division durch x^2 ist hier ohne Einschränkungen möglich, weil x=0 \not\in \mathbb{D}. Eine Division durch 0 kann also nicht passieren.


19)
Bestimmung des Definitionsbereichs:
\begin{array}{rclcl}4x &>& 0 &\vert & :4 \\x &>& 0\end{array}

Der Definitionsbereich ist also \mathbb{D}=\mathbb{R}^+.

Lösung der Gleichung:
\begin{array}{rclcl}x \cdot \log_{10}(4x) &=& 6x\cdot\log_{10}(\sqrt{10}) \\x \cdot \log_{10}(4x) &=& 6x\cdot \log_{10}\left(10^{\frac{1}{2}}\right) \\x \cdot \log_{10}(4x) &=& 6x\cdot\dfrac{1}{2}&\vert& :x \\\log_{10}(4x) &=& 3 &\vert& 10^* \\4x &=& 10^3 &\vert& :4 \\x &=& 250 \in \mathbb{D} \\\\\mathbb{L} &=& \left\{250\right\}\end{array}

Bemerkung: Die Division durch x ist hier ohne Einschränkungen möglich, weil x=0 \not\in \mathbb{D}. Eine Division durch 0 kann also nicht passieren.


20)
Bestimmung des Definitionsbereichs:
\begin{array}{rclcl}37x &>& 0 &\vert & :37 \\x &>& 0\end{array}

Der Definitionsbereich ist also \mathbb{D}=\mathbb{R}^+.

Lösung der Gleichung:
\begin{array}{rclcl}\log_{\pi}(37x) &=& 14\log_{\pi}(e) &\vert& \text{3. Logarithmengesetz}\\\log_{\pi}(37x) &=& \log_{\pi}(e^{14}) &\vert& \pi^*\\37x &=& e^{14} &\vert& :37\\x &=& \dfrac{e^{14}}{37} \approx 32.502{,}82 \in \mathbb{D} \\\\\mathbb{L} &=& \left\{\dfrac{e^{14}}{37}\right\}\end{array}

 

2. Aufgabe

Wie bei den Exponentialfunktionen wurden hier immer nur zwei Graphen in ein Koordinatensystem eingezeichnet und als Orientierung die Funktion f(x)= \ln(x) (fett gedruckt) mit abgebildet.

Logarithmusfunktionen

Logarithmusfunktionen

Logarithmusfunktionen

Logarithmusfunktionen

 

3. Aufgabe

1)
a)
\begin{array}{rclll} f\left(\dfrac{11}{9} \right) &=&\dfrac{1}{4} \ln\left(9\cdot \dfrac{11}{9}-10\right) &=& \dfrac{1}{4} \ln(1) = 0 \quad \rightarrow \quad P_1\left( \dfrac{11}{9} \mid 0 \right)\end{array}

b)
\begin{array}{rclll}0 &=& \dfrac{1}{4} \ln(9x-10) &\vert & \cdot 4\cr 0 &=& \ln(9x-10) &\vert & e^{*}\cr 1 &=& 9x-10 &\vert & +10\cr 11 &=& 9x &\vert & :9\cr x &=& \dfrac{11}{9} \approx 1{,}22 \;\in\;\mathbb{D} & & \rightarrow \quad P_2 \left( \dfrac{11}{9} \mid 0 \right)\end{array}

Bemerkung: Schaut man sich die Lösung zu Teil a) genau an, sieht man, dass es sich bei P_1 um eine Nullstelle handelt. Da es sich beim Argument des Logarithmus um einen linearen Term handelt, hätte man sich die Rechnung für Teil b) sparen können, da es nur einen Wert für x geben kann, der dazu führt, dass das Argument 1 und somit der Logarithmus 0 wird. Die Funktion kann also nur eine Nullstelle haben ...


2)
a)
\begin{array}{rclll} f\left(8 \right) &=&-\dfrac{\ln(8^2-8)+1}{20}&=& -\dfrac{\ln(56)+1}{20} \quad \rightarrow \quad P_1\left( 8 \mid -\dfrac{\ln(56)+1}{20} \right)\end{array}

b)
\begin{array}{rclll}-\dfrac{1}{20} &=& -\dfrac{\ln(x^2-8)+1}{20} &\vert & \cdot (-20)\cr 1 &=& \ln(x^2-8)+1 &\vert & -1\cr 0 &=& \ln(x^2-8) &\vert & e^{*}\cr 1 &=& x^2-8 &\vert & +8\cr 9 &=& x^2 &\vert & \pm\sqrt{}\cr\cr x_1 &=&3\;\in\;\mathbb{D} & & \rightarrow \quad P_2 \left( 3 \mid -\dfrac{1}{20} \right)\cr\cr x_2 &=&-3\;\in\;\mathbb{D} & & \rightarrow \quad P_3 \left( -3 \mid -\dfrac{1}{20} \right)\end{array}


3)
a)
\begin{array}{rclll} f\left(10\right) &=& \log_8(10^3)+15 &=& 3\cdot \log_8(10)+15 \quad \rightarrow \quad P_1\left(10 \mid 3\cdot \log_8(10)+15 \right)\end{array}

b)
\begin{array}{rclll}
2 &=& \log_8(x^3)+15 &\vert & -15\cr -13 &=& \log_8(x^3) &\vert & 8^{*}\cr 8^{-13} &=& x^3 &\vert & \sqrt[3]{}\cr x &=& \dfrac{1}{8.192} \approx 0{,}00012 \;\in\;\mathbb{D} & & \rightarrow \quad P_2 \left( \dfrac{1}{8.192} \mid 2 \right)\end{array}


4)
a)
\begin{array}{rclll} f\left(-5\right) &=& \log_5((-5)^2+2\cdot(-5)) &=& \log_5(15) \quad \rightarrow \quad P_1\left( -5\mid \log_5(15) \right)\end{array}

b)
\begin{array}{rclll}23 &=& \log_5(x^2+2x) &\vert & 5^{*}\cr 5^{23} &=& x^2+2x &\vert & - 5^{23}\cr 0 &=& x^2+2x-5^{23} &\vert& \text{p-q-Formel}\cr x_{1,2} &=& -1\pm \sqrt{1+5^{23}}\cr\cr x_1 &=& -1+ \sqrt{1+5^{23}} \approx 109.183.005{,}71 \;\in\;\mathbb{D} & & \rightarrow \quad P_2 \left( -1+ \sqrt{1+5^{23}} \mid 23 \right)\cr x_2 &=& -1- \sqrt{1+5^{23}} \approx -109.183.007{,}71 \;\in\;\mathbb{D} & & \rightarrow \quad P_3 \left( -1- \sqrt{1+5^{23}} \mid 23 \right)\end{array}

5)
a)
\begin{array}{rclll} f\left(-6\right) &=& -2 \log_{10}\left((-6)^4-\dfrac{8}{5} \right)&=& -2 \log_{10}\left(\dfrac{6472}{5} \right) \quad \rightarrow \quad P_1\left(-6 \mid -2 \log_{10}\left(\dfrac{6.472}{5} \right) \right)\end{array}

b)
\begin{array}{rclll}0&=& -2 \log_{10}\left(x^4-\dfrac{8}{5} \right) &\vert & : (-2)\cr 0&=& \log_{10}\left(x^4-\dfrac{8}{5} \right) &\vert & 10^{*}\cr 1&=& x^4-\dfrac{8}{5} &\vert &+\dfrac{8}{5}\cr \dfrac{13}{5}&=& x^4 &\vert &\pm\sqrt[4]{}\cr\cr x_1 &=& \sqrt[4]{\dfrac{13}{5}}\approx 1{,}27\;\in\;\mathbb{D} & & \rightarrow \quad P_2 \left( \sqrt[4]{\dfrac{13}{5}} \mid 0 \right)\cr\cr x_2 &=& -\sqrt[4]{\dfrac{13}{5}}\approx -1{,}27\;\in\;\mathbb{D} & & \rightarrow \quad P_3 \left( -\sqrt[4]{\dfrac{13}{5}} \mid 0 \right)\end{array}

6)
a)
\begin{array}{rclclll} f\left(9\right) &=&-\log_{2} \left( \dfrac{1}{16}\cdot 9^2-\dfrac{1}{2}\cdot 9 \right) &=& -\log_{2} \left(\left(\dfrac{3}{4}\right)^2\right) &=&-2\log_{2} \left( \dfrac{3}{4}\right) \quad \rightarrow \quad P_1\left( 9 \mid -2\log_{2} \left( \dfrac{3}{4}\right) \right)\end{array}

b)
\begin{array}{rclll}4 &=&-\log_{2} \left( \dfrac{1}{16}x^2-\dfrac{1}{2}x \right) &\vert & \cdot (-1)\cr -4 &=&\log_{2} \left( \dfrac{1}{16}x^2-\dfrac{1}{2}x \right) &\vert & 2^{*}\cr\cr \dfrac{1}{16} &=&\dfrac{1}{16}x^2-\dfrac{1}{2}x &\vert & -\dfrac{1}{16}\cr\cr 0 &=&\dfrac{1}{16}x^2-\dfrac{1}{2}x-\dfrac{1}{16} &\vert & : \dfrac{1}{16}\cr 0 &=& x^2-8x-1 &\vert& \text{p-q-Formel}\cr x_{1,2} &=& 4 \pm \sqrt{16+1}\cr\cr x_1 &=& 4 +\sqrt{17}\approx 8{,}12\;\in\;\mathbb{D} & & \rightarrow \quad P_2 \left( 4+ \sqrt{17} \mid 4 \right)\cr x_2 &=& 4 - \sqrt{17}\approx -0{,}12\;\in\;\mathbb{D} & & \rightarrow \quad P_3 \left( 4- \sqrt{17} \mid 4 \right)\end{array}


7)
a)
\begin{array}{rclllll} f\left(3\right) &=& \ln \left( \dfrac{-3+5}{12}\right) &=& \ln \left(6^{-1}\right) &=& -\ln \left(6\right) \quad \rightarrow \quad P_1\left(3 \mid -\ln \left(6\right)\right)\end{array}

b)
\begin{array}{rclll}\ln(7) &=& \ln \left( \dfrac{-x+5}{12}\right) &\vert & e^{*}\cr 7 &=& \dfrac{-x+5}{12} &\vert & \cdot 12\cr 84 &=& -x+5 &\vert & -5\cr 79 &=& -x &\vert & \cdot (-1)\cr -79 &=& x\;\in\;\mathbb{D} & & \rightarrow \quad P_2 \left( -79 \mid \ln\left(7\right) \right)\end{array}


8)
a)
\begin{array}{rclll} f\left(1.000\right) &=& \log_{10} (1.000)-3 &=& 0 \quad \rightarrow \quad P_1\left(1.000 \mid 0 \right)\end{array}

b)
\begin{array}{rclll}2 &=& \log_{10} (x)-3 &\vert &+3\cr 5 &=& \log_{10} (x) &\vert & 10^{*}\cr 10^5 &=& x\cr x &=& 100.000\;\in\;\mathbb{D} & & \rightarrow \quad P_2 \left(100.000 \mid 2 \right)\end{array}


9)
a)
\begin{array}{rclllll} f\left( -\dfrac{2}{5} \right) &=& 10 \ln \left(\left(-\dfrac{2}{5}\right)^2-4\cdot \left(-\dfrac{2}{5}\right)+4\right)-10 &=& 10\ln \left(\left(\dfrac{12}{5}\right)^2\right)-10 &=& 20\ln \left( \dfrac{12}{5}\right)-10 \quad \rightarrow \quad P_1\left(-\dfrac{2}{5} \mid 20 \ln \left(\dfrac{12}{5}\right)-10\right)\end{array}

b)
\begin{array}{rclll}30 &=& 10 \ln (x^2-4x+4)-10 &\vert & +10\cr 40 &=& 10 \ln (x^2-4x+4) &\vert & : 10\cr 4 &=& \ln (x^2-4x+4) &\vert & e^{*}\cr e^{4} &=& x^2-4x+4 &\vert & -e^{4}\cr 0 &=& x^2-4x+4-e^{4} &\vert& \text{p-q-Formel}\cr x_{1,2} &=& 2 \pm \sqrt{4-4+e^{4}}\cr x_{1,2} &=& 2 \pm \sqrt{e^{4}}\cr\cr x_1 &=& 2+e^{2}\approx 9{,}40 \;\in\;\mathbb{D} & & \rightarrow \quad P_2 \left(2+e^{2} \mid 30 \right)\cr x_2 &=& 2-e^{2}\approx -5{,}40 \;\in\;\mathbb{D} & & \rightarrow \quad P_3 \left( 2-e^{2} \mid 30 \right)\end{array}


10)
a)
\begin{array}{rclll} f\left(12\right) &=& \dfrac{2}{3} \log_2 \left(\dfrac{2}{12}\right)+1 &=&\dfrac{3+2\log_{2}\left(\frac{1}{6}\right)}{3} \quad \rightarrow \quad P_1\left( 12 \mid \dfrac{3+2\log_{2}\left(\frac{1}{6}\right)}{3}\right)\end{array}

b)
\begin{array}{rclll}-27 &=&\dfrac{2}{3} \log_2 \left(\dfrac{2}{x}\right)+1 &\vert &-1\cr -28 &=&\dfrac{2}{3} \log_2 \left(\dfrac{2}{x}\right) &\vert & : \dfrac{2}{3}\cr -42 &=& \log_2 \left(\dfrac{2}{x}\right) &\vert& \text{2. Logarithmengesetz}\cr -42 &=& \log_2(2) - \log_2(x)\cr -42 &=& 1-\log_2(x) &\vert &-1\cr -43 &=& -\log_2(x) &\vert &\cdot (-1)\cr 43 &=& \log_2(x) &\vert &2^{*}\cr x &=& 2^{43} = 8.796.093.022.208 \;\in\;\mathbb{D} & & \rightarrow \quad P_2 \left( 2^{43} \mid -27 \right)\end{array}

 

4. Aufgabe

1)
Bestimmung des Definitionsbereichs:
Da das Argument des Logarithmus (x-81)^2 ein quadratischer Term ist und Quadrate im Bereich der reellen Zahlen immer nichtnegativ sind, muss nur vermieden werden, dass das Quadrat den Wert 0 annimmt:
\begin{array}{rclcl}(x-81)^2 &\neq& 0 &\vert& \pm\sqrt{} \\ x-81 &\neq & 0 & \vert & +81\\x &\neq & 81\end{array}

Der Definitionsbereich ist also \mathbb{D} = \mathbb{R}\setminus_{\{81\}}.

Berechnung der Nullstellen:
\begin{array}{rclcl}0 &=& \ln\left((x-81)^2\right) &\vert& e^{*} \cr 1 &=& (x-81)^2 &\vert & \pm\sqrt{} \cr \pm 1 &=& x-81 &\vert & +81 \cr \cr x_1 &=& 1+81 = 82\;\in\;\mathbb{D} \cr x_2 &=& -1+81 = 80\;\in\;\mathbb{D} \end{array}

Ergebnis: Die Nullstellen von f_1(x) liegen bei x_1=82 und x_2=80.


2)
Bestimmung des Definitionsbereichs:
1. Argument rechts:
x > 0

2. Argument rechts:
Da das Argument des zweiten Logarithmus x^2 ein quadratischer Term ist und Quadrate im Bereich der reellen Zahlen immer nichtnegativ sind, muss nur vermieden werden, dass das Quadrat den Wert 0 annimmt:
\begin{array}{rclcl} x^2 &\neq& 0 &\vert& \pm\sqrt{} \\ x &\neq& 0\end{array}

Für die abschließende Ermittlung des Definitionsbereichs muss nun überprüft werden, für welche x-Werte alle ermittelten Bedingungen erfüllt sind. Das ist nur der Fall für x > 0. Der Definitionsbereich ist also \mathbb{D} = \mathbb{R}^+.

Berechnung der Nullstellen:
\begin{array}{crclcl} & 0 &=& x\ln(x)+x\ln\left(x^2\right)+x\ln(27) \cr & 0 &=& x\left(\ln(x)+\ln\left(x^2\right)+\ln(27)\right) &\vert& \text{1. Logarithmengesetz} \cr & 0 &=& x\cdot\ln(27x^3) &\vert& \text{Satz vom Nullprodukt} \cr \cr \text{Faktor 1:} & 0 &=& x_1 \; \not \in \; \mathbb{D} \cr\cr \text{Faktor 2:} & 0 &=& \ln(27 x^3) &\vert& e^{*} \cr & 1 &=& 27x^3 &\vert& : 27 \cr & \dfrac{1}{27} &=& x^3 &\vert& \sqrt[3]{} \cr\cr & \dfrac{1}{3} &=& x_2\;\in\;\mathbb{D} \end{array}

Ergebnis: Die einzige Nullstelle von f_2(x) liegt bei x_2=\dfrac{1}{3}.


3)
Bestimmung des Definitionsbereichs:
6x^2+12x-17 > 0

Um diese quadratische Ungleichung zu lösen, wird im ersten Schritt die zugehörige quadratische Gleichung gelöst:
\begin{array}{rclcl}6x^2+12x-17 &=& 0 &\vert & :6 \\x^2+2-\dfrac{17}{6} &=& 0 &\vert & \text{p-q-Formel} \\x_{1,2} &=& -1\pm\sqrt{1^2+\dfrac{17}{6}}\\x_{1,2} &=& -1\pm\sqrt{\dfrac{23}{6}} \\\\x_1 &=& -1+\sqrt{\dfrac{23}{6}} \approx 0{,}96\\x_2 &=& -1-\sqrt{\dfrac{23}{6}} \approx -2{,}96\end{array}

Diese Gleichung hat also zwei Lösungen bei x_1=-1+\sqrt{\dfrac{23}{6}} und x_2=-1-\sqrt{\dfrac{23}{6}}. Diese x-Werte sind auf jeden Fall nicht Teil des Definitionsbereichs. Es ergeben sich die Intervalle \left]-\infty;-1-\sqrt{\dfrac{23}{6}}\right[, \left]-1-\sqrt{\dfrac{23}{6}};-1+\sqrt{\dfrac{23}{6}}\right[ und \left]-1+\sqrt{\dfrac{23}{6}};\infty\right[.
Jetzt muss noch geprüft werden, in welchen Intervallen der Term positive bzw. negative Werte annimmt. Es ist sinnvoll, für die Prüfung möglichst einfache x-Werte zu verwenden. Z. B. liefert x=-3 als Ergebnis 1, für x=0 ergibt sich -17 und x=1 führt zu 1. Das bedeutet, dass 6x^2+12x-17 im ersten und dritten Intervall positiv ist. Die ursprüngliche Ungleichung 6x^2+12x-17 > 0 ist also nur in diesen beiden Intervallen wahr.

Der Definitionsbereich ist also \mathbb{D} = \left\{x\in \mathbb{R} \mid x < -1-\sqrt{\dfrac{23}{6}} \text{ oder } x>-1+\sqrt{\dfrac{23}{6}} \right\}.

Berechnung der Nullstellen:
\begin{array}{rclcl}0 &=& \log_7\left(6x^2+12x-17\right) &\vert& 7^* \\1 &=& 6x^2+12x-17 &\vert& -1 \\0 &=& 6x^2+12x-18 &\vert& :6 \\0 &=& x^2+2x-3 &\vert& \text{p-q-Formel} \\x_{1,2} &=& -1\pm\sqrt{1+3} \\&=& -1 \pm \sqrt{4} \\\\x_1 &=& -1+2 = 1\;\in\;\mathbb{D} \\x_2 &=& -1-2 = -3\;\in\;\mathbb{D}\end{array}

Ergebnis: Die Nullstellen von f_3(x) liegen bei x_1=1 und x_2=-3.


4)
Bestimmung des Definitionsbereichs:
\begin{array}{rclcl}x+2 &>& 0 &\vert & -2 \\x &>& -2\end{array}

Der Definitionsbereich ist also \mathbb{D} = \left]-2;\infty \right[.

Berechnung der Nullstellen:
\begin{array}{crclcl} & 0 &=& \left(x^3-8\right)\cdot\lg(x+2) &\vert& \text{Satz vom Nullprodukt} \cr \cr \text{Faktor 1:} & 0 &=& x^3-8 &\vert& +8 \cr & 8 &=& x^3 &\vert& \sqrt[3]{} \cr & 2 &=& x_1\;\in\;\mathbb{D} \cr\cr \text{Faktor 2:} & 0 &=& \lg(x+2) &\vert& 10^{*} \cr & 1 &=& x+2 &\vert& -2 \cr & -1 &=& x_2\;\in\;\mathbb{D} \end{array}

Ergebnis: Die Nullstellen von f_4(x) liegen bei x_1=2 und x_2=-1.

5)
Bestimmung des Definitionsbereichs:
x^2+x-6 > 0

Um diese quadratische Ungleichung zu lösen, wird im ersten Schritt die zugehörige quadratische Gleichung gelöst:
\begin{array}{rclll} x^2+x-6 &=& 0 &\vert &\text{p-q-Formel}\\x_{1,2} &=& -\dfrac{1}{2}\pm\sqrt{\left(\dfrac{1}{2}\right)^2+6}\\x_{1,2} &=& -\dfrac{1}{2}\pm\sqrt{\dfrac{25}{4}}\\\\x_1 &=& -\dfrac{1}{2}\pm\dfrac{5}{2} = 2\\x_2 &=& -\dfrac{1}{2}\pm\dfrac{5}{2} = -3\end{array}
Diese Gleichung hat also zwei Lösungen bei x_1=2 und x_2=-3. Diese x-Werte sind auf jeden Fall nicht Teil des Definitionsbereichs. Es ergeben sich die Intervalle ]-\infty;-3[, ]-3;2[ und ]2;\infty[.
Jetzt muss noch geprüft werden, in welchen Intervallen der Term positive bzw. negative Werte annimmt. Es ist sinnvoll, für die Prüfung möglichst einfache x-Werte zu verwenden, Z. B. liefert x=-4 als Ergebnis 6, für x=0 ergibt sich -6 und x=3 führt zu 6. Das bedeutet, dass x^2+x-6 im ersten und dritten Intervall positiv ist. Die ursprüngliche Ungleichung x^2+x-6 > 0 ist also nur in diesen beiden Intervallen wahr.

Der Definitionsbereich ist also \mathbb{D}=\{x\in \mathbb{R} \mid x < -3 \text{ oder } x>2 \}.

Berechnung der Nullstellen:
\begin{array}{crclcll}& 0 &=& \left(x^2-4\right)\cdot\ln\left(x^2+x-6\right) & \vert & \text{Satz vom Nullprodukt}\\\\\text{Faktor 1:} & x^2-4 &=& 0 &\vert &+4 \\& x^2 &=& 4 &\vert &\pm\sqrt{} \\& x &=& \pm\sqrt{4} \\\\& x_1 &=& 2 \not\in\mathbb{D} \\& x_2 &=& -2 \not\in\mathbb{D} \\\\\text{Faktor 2:} & \ln\left(x^2+x-6\right) &=& 0 &\vert& e^{*} \\& x^2+x-6 &=& 1&\vert& -1 \\& x^2+x-7 &=& 0 &\vert& \text{p-q-Formel} \\& x_{1,2} &=& -\dfrac{1}{2}\pm\sqrt{\dfrac{1}{4}+7} \\\\&& =& -\dfrac{1}{2}\pm\sqrt{\dfrac{29}{4}} \\\\& x_1 &=& -\dfrac{1}{2}+\dfrac{\sqrt{29}}{2} \approx 2{,}19 \;\in\;\mathbb{D} \\\\& x_2 &=& -\dfrac{1}{2}-\dfrac{\sqrt{29}}{2} \approx -3{,}19 \;\in\;\mathbb{D}\end{array}

Ergebnis: Die Nullstellen von f(x) liegen bei x_1 = -\dfrac{1}{2}+\dfrac{\sqrt{29}}{2} und x_2 = -\dfrac{1}{2}-\dfrac{\sqrt{29}}{2}.


6)
Bestimmung des Definitionsbereichs:
2. Argument links:
\begin{array}{rclll}2x &> & 0 &\vert & :2\\x &>& 0\\\\\end{array}

3. Argument links:
\begin{array}{rclll}10x &>& 0 &\vert & :10\\x &>& 0\end{array}

Für die abschließende Ermittlung des Definitionsbereichs muss nun überprüft werden, für welche x-Werte alle ermittelten Bedingungen erfüllt sind. Das ist der Fall für x > 0. Der Definitionsbereich ist also \mathbb{D} = \mathbb{R}^+.

Berechnung der Nullstellen:
\begin{array}{lrclcll}& 0 &=& 3x\log_8\left(2\right)+x\log_8\left(2x\right)+2x\log_8\left(10x\right) \\\\& 0 &=& x\left[3\log_8\left(2\right)+\log_8\left(2x\right)+2\log_8\left(10x\right)\right] &\vert& \text{3. Logarithmengesetz} \\\\& 0 &=& x\left[\log_8\left(2^3\right)+\log_8\left(2x\right)+\log_8\left((10x)^2\right)\right] \\\\& 0 &=& x\left[\log_8(8)+\log_8(2x)+\log_8\left(100x^2\right)\right] &\vert& \text{2. Logarithmengesetz}\\\\& 0 &=& x\cdot \log_8\left(8\cdot 2x\cdot 100x^2\right) \\& 0 &=& x\cdot \log_8\left(1.600x^3\right) & \vert & \text{Satz vom Nullprodukt}\\\\\text{Faktor 1:} & x_1 &=& 0 \not\in\mathbb{D} \\\\\text{Faktor 2:} & \log_8\left(1.600x^3\right) &=& 0 &\vert& 8^{*}\\& 1.600x^3 &=& 1 &\vert& :1.600 \\& x^3 &=& \dfrac{1}{1.600} &\vert &\sqrt[3]{} \\\\& x_2 &=& \sqrt[3]{\dfrac{1}{1.600}} \approx 0{,}09 \;\in\;\mathbb{D}\end{array}

Ergebnis: Die Nullstelle von g(x) liegt bei x = \sqrt[3]{\dfrac{1}{1.600}}.


7)
Bestimmung des Definitionsbereichs:
w^2+17 > 0

Da ein Quadrat im Bereich der reellen Zahlen immer nichtnegativ ist, ist das Argument als Summe aus einem Quadrat und der Zahl 17 immer positiv. Der Definitionsbereich ist also \mathbb{D} = \mathbb{R}.

Berechnung der Nullstellen:
\begin{array}{lrclcll}& 0 &=& (-100w^4-225)\cdot \log_7\left(w^2+17\right) & \vert & \text{Satz vom Nullprodukt}\\\\\text{Faktor 1:} & -100w^4-225 &=& 0 &\vert &+225 \\& -100w^4 &=& 225 &\vert& :(-100) \\& w^4 &=& -\dfrac{9}{4} &\vert& \pm\sqrt[4]{} \\& w_{1,2} &=& \pm\sqrt[4]{-\dfrac{9}{4}} \\\\\text{Faktor 2:} & \log_7(w^2+17) &=& 0 &\vert& 7^{*}\\& w^2+17 &=& 1 &\vert& -17 \\& w^2 &=& -16 &\vert& \pm\sqrt{} \\& w_{3,4} &=& \pm\sqrt{-16}\end{array}

Ergebnis: Da im Bereich der reellen Zahlen aus negativen Zahlen keine Wurzeln mit geraden Wurzelexponenten gezogen werden können, hat diese Funktion keine Nullstellen.


8)
Bestimmung des Definitionsbereichs:
x^2+2x+2>0

Um diese quadratische Ungleichung zu lösen, wird im ersten Schritt die zugehörige quadratische Gleichung gelöst:
\begin{array}{rclll} x^2+2x+2 &=& 0 &\vert &\text{p-q-Formel}\\x_{1,2} &=& -1\pm\sqrt{1^2-2}\\\\x_{1,2} &=& -1\pm\sqrt{-1} \end{array}

Da aus negativen reellen Zahlen keine Wurzeln mit geraden Wurzelexponenten gezogen werden können, hat diese Gleichung keine Lösung.
Jetzt muss noch geprüft werden, ob der Term nur positive oder nur negative Werte annimmt. Es ist sinnvoll, für die Prüfung einen möglichst einfachen x-Wert zu verwenden. Z. B. liefert x=0 als Ergebnis 2. Das bedeutet, dass x^2+2x+2 für jeden x-Wert positiv ist.

Der Definitionsbereich ist also \mathbb{D} = \mathbb{R}.

Berechnung der Nullstellen:
\begin{array}{lrclcll}& 0 &=& \left(x^2-2\right)\cdot\log_{11}(x^2+2x+2) & \vert & \text{Satz vom Nullprodukt} \\\\\text{Faktor 1:} & x^2-2 &=& 0 &\vert& +2 \\& x^2 &=& 2 &\vert& \pm\sqrt{} \\\\& x_1 &=& \sqrt{2} \;\in\;\mathbb{D} \\& x_2 &=& -\sqrt{2} \;\in\;\mathbb{D} \\\\\text{Faktor 2:} & \log_{11}\left(x^2+2x+2\right) &=& 0 &\vert& 11^{*}\\& x^2+2x+2 &=& 1 &\vert& -1 \\& x^2+2x+1 &=& 0 \\& (x+1)^2 &=& 0 &\vert& \pm\sqrt{} \\& x+1 &=& 0 &\vert& -1 \\& x_3 &=& -1 \;\in\;\mathbb{D}\end{array}

Ergebnis: Die Nullstellen von f(x) liegen bei x_1 = \sqrt{2}, x_2 = -\sqrt{2} und x_3=-1.


9)
Bestimmung des Definitionsbereichs:
\begin{array}{rclcl} 6x &>& 0 &\vert & :6 \\x &>& 0 \end{array}

Der Definitionsbereich ist also \mathbb{D} = \mathbb{R}^+.

Berechnung der Nullstellen:
\begin{array}{rclcl}0 &=& \dfrac{\ln(6x)}{\sqrt{9}}+2 & \vert & -2 \\\\-2 &=& \dfrac{\ln(6x)}{3} & \vert & \cdot 3 \\\\-6 &=& \ln(6x) & \vert & e^*\\\\e^{-6} &=& 6x & \vert & :6 \\\\x &=& \dfrac{1}{6} e^{-6} \approx 0{,}0004 \in \mathbb{D}\end{array}

Ergebnis: Die Nullstelle von f(x) liegt bei x=\dfrac{1}{6} e^{-6}.


10)
Bestimmung des Definitionsbereichs:
1. Argument im Zähler:
\begin{array}{rclcl} \dfrac{5}{4}x &>& 0 &\vert & :\dfrac{5}{4} \\x &>& 0 \\\\\end{array}

Argument in Nenner:
\begin{array}{rclcl} 2x &>& 0 &\vert & :2 \\x &>& 0\end{array}

Nenner:
\begin{array}{rclcl} \ln(2x) &\neq& 0 &\vert & e^* \\2x &\neq& 1 &\vert& :2 \\x &\neq& \dfrac{1}{2}\end{array}

Für die abschließende Ermittlung des Definitionsbereichs muss nun überprüft werden, für welche x-Werte alle ermittelten Bedingungen erfüllt sind. Das ist der Fall für x > 0 unter Ausschluss der Zahl \dfrac{1}{2}. Der Definitionsbereich ist also \mathbb{D} = \mathbb{R}^+\setminus_{\{\frac{1}{2}\}}.

Berechnung der Nullstellen:
\begin{array}{rclcl}0 &=& \dfrac{\log_{12}\left(\dfrac{5}{4}x\right)\cdot\ln(7)}{\ln(2x)} &\vert & \cdot\ln(2x) \\\\0 &=&\log_{12}\left(\dfrac{5}{4}x\right)\cdot\ln(7) &\vert& :\ln(7) \\\\0 &=& \log_{12}\left(\dfrac{5}{4}x\right) &\vert & 12^* \\\\1 &=& \dfrac{5}{4}x &\vert & : \dfrac{5}{4} \\\\x &=& \dfrac{4}{5} \in \mathbb{D}\end{array}

Ergebnis: Die Nullstelle von f(x) liegt bei x=\dfrac{4}{5}.

Bemerkung: Die Multiplikation mit \ln(2x) ist hier ohne Einschränkungen möglich, weil x=\dfrac{1}{2} \not\in \mathbb{D}. Eine Multiplikation mit 0 kann also nicht passieren.


11)
Bestimmung des Definitionsbereichs:
\begin{array}{rclcl} 5x &>& 0 &\vert & :5 \\x &>& 0 \end{array}

Der Definitionsbereich ist also \mathbb{D} = \mathbb{R}^+.

Berechnung der Nullstellen:
\begin{array}{rclcl}0 &=& 2\log_{10}(5x)+3\log_{10}(x)+8 &\vert& \text{3. Logarithmengesetz} \\\\0 &=& \log_{10}(25x^2)+\log_{10}(x^3)+8 &\vert& 10^* \\\\1 &=& 10^{\log_{10}(25x^2)+\log_{10}(x^3)+8} \\\\1 &=& 10^{\log_{10}\left(25x^2\right)} \cdot 10^{\log_{10}(x^3)}\cdot 10^8 \\1 &=& 25x^2 \cdot x^3 \cdot 10^8 \\1 &=& 25 \cdot 10^8x^5 &\vert& :(25\cdot 10^8) \\\dfrac{1}{25\cdot 10^8} &=& x^5 &\vert& \sqrt[5]{} \\x &=& \sqrt[5]{\dfrac{1}{25\cdot 10^8}} \approx 0{,}01 \in \mathbb{D} \end{array}

Ergebnis: Die Nullstelle von f(x) liegt bei x=\sqrt[5]{\dfrac{1}{25\cdot 10^8}}.


12)
Bestimmung des Definitionsbereichs:
Argument:
\begin{array}{rclcl} x-2 &>& 0 &\vert& +2 \\x &>& 2 \\\\\end{array}

Nenner:
\begin{array}{rclcl} x^2-1 &\neq& 0 &\vert& +1 \\x^2 &\neq& 1 &\vert& \pm\sqrt{} \\x &\neq& \pm 1\end{array}

Für die abschließende Ermittlung des Definitionsbereichs muss nun überprüft werden, für welche x-Werte alle ermittelten Bedingungen erfüllt sind. Das ist nur der Fall für x > 2. Der Definitionsbereich ist also \mathbb{D}=\left]2;\infty \right[.

Berechnung der Nullstellen:
\begin{array}{lrclcl}& 0 &=& \ln(x-2)\cdot \dfrac{2x}{x^2-1} &\vert& \text{Satz vom Nullprodukt} \\\text{Faktor 1:} & 0 &=& \ln(x-2) &\vert& e^* \\& 1 &=& x-2 &\vert& +2 \\& 3 &=& x_1 \in \mathbb{D} \\\\\text{Faktor 2:} & 0 &=& \dfrac{2x}{x^2-1} &\vert& \cdot \left(x^2-1\right) \\& 0 &=& 2x &\vert& :2 \\& 0 &=& x_2 \not \in \mathbb{D}\end{array}

Ergebnis: Die Nullstelle von f(x) liegt bei x_1=3.

Bemerkung: Die Multiplikation mit (x^2-1) ist hier problemlos möglich, weil x=-1 \not\in \mathbb{D} und x=1 \not\in \mathbb{D}. Eine Multiplikation mit 0 kann also nicht passieren.


13)
Bestimmung des Definitionsbereichs:
Nenner des 1. Arguments rechts:
\begin{array}{rclcl} 44z+11 &\neq& 0 &\vert & -11 \\44z &\neq& -11 &\vert & :44 \\z &\neq& -\dfrac{1}{4}\end{array}

1. Argument rechts:
\begin{array}{rcl}\dfrac{3z}{44z+11} &>& 0\end{array}

Hier ist eine Fallunterscheidung nötig, da wir mit der Variablen multiplizieren müssen.

Fall 1: Wir nehmen an, dass 44z+11>0 ist. Dann muss z>-\dfrac{1}{4} sein.
\begin{array}{rclcl}\dfrac{3z}{44z+11} &>& 0 &\vert & \cdot (44z+11) \\3z &>& 0 &\vert& :3\\z &>& 0\end{array}

Im Abgleich zwischen Eintrittsbedingung und Lösung des 1. Falls ergibt sich folgendes Zwischenergebnis: z>0

Fall 2: Wir nehmen an, dass 44z+11 < 0 ist. Dann muss z < -\dfrac{1}{4} sein.
\begin{array}{rclcl}\dfrac{3z}{44z+11} & > & 0 &\vert & \cdot (44z+11) \\3z & < & 0 &\vert& :3\\z & < & 0\end{array}

Im Abgleich zwischen Eintrittsbedingung und Lösung des 2. Falls ergibt sich folgendes Zwischenergebnis: z < -\dfrac{1}{4}

Die Fallunterscheidung liefert uns also z < -\dfrac{1}{4} oder z > 0.

Nenner des 2. Arguments rechts:
\begin{array}{rclcl}22z & \neq & 0 &\vert& :22 \\z & \neq & 0 \\\\\end{array}

2. Argument rechts:
\begin{array}{rclcl}-\dfrac{9}{22z} & > & 0 &\vert& : \left(-\dfrac{9}{22}\right) \\\dfrac{1}{z} & < & 0 &\vert& \cdot z^2\\z & < & 0\end{array}

Für die abschließende Ermittlung des Definitionsbereichs muss nun überprüft werden, für welche x-Werte alle ermittelten Bedingungen erfüllt sind. Das ist nur der Fall für z < -\dfrac{1}{4}. Der Definitionsbereich ist also \mathbb{D} = \left]-\infty;-\dfrac{1}{4}\right[.

Berechnung der Nullstellen:
\begin{array}{rclcl}0 &=& \ln\left(\dfrac{3z}{44z+11}\right)-\ln\left(-\dfrac{9}{22z}\right) &\vert& \text{2. Logarithmengesetz} \\0 &=& \ln\left(\dfrac{3z}{44z+11} : \left(-\dfrac{9}{22z}\right)\right) \\0 &=& \ln\left(\dfrac{3z}{11(4z+1)}\cdot \left(-\dfrac{22z}{9}\right)\right) \\0 &=& \ln\left(-\dfrac{2z^2}{12z+3}\right) &\vert& e^*\\1 &=& -\dfrac{2z^2}{12z+3} &\vert& \cdot (12z+3) \\12z+3 &=& -2z^2 &\vert& -12z-3 \\0 &=& -2z^2-12z-3 &\vert& :(-2) \\0 &=& z^2+6z+\dfrac{3}{2} &\vert& \text{p-q-Formel} \\\\z_{1,2} &=& -3 \pm \sqrt{3^2-\dfrac{3}{2}} \\z_{1,2} &=& -3 \pm \sqrt{\dfrac{15}{2}} \\\\z_{1} &=& -3+\sqrt{\dfrac{15}{2}} \approx -0{,}26 \in \mathbb{D} \\\\z_{2} &=& -3-\sqrt{\dfrac{15}{2}} \approx -5{,}74 \in \mathbb{D} \\\end{array}

Ergebnis: Die Nullstelle von f(z) liegt bei z_1=-3+\sqrt{\dfrac{15}{2}} und z_2=-3-\sqrt{\dfrac{15}{2}}.

Bemerkung: Die Multiplikation mit (12z+3) ist hier ohne Einschränkungen möglich, weil x=-\dfrac14 \not\in \mathbb{D}. Eine Multiplikation mit 0 kann also nicht passieren.


14)
Bestimmung des Definitionsbereichs:
1. Argument links:
\begin{array}{rclcl} -7+13x &>& 0 &\vert & +7 \\13x &>& 7 &\vert & :13 \\x &>& \dfrac{7}{13} \\\\\end{array}

2. Argument links:
Ein Bruch ist genau dann positiv, wenn Zähler und Nenner das gleiche Vorzeichen besitzen. Das beinhaltet, dass weder Zähler noch Nenner 0 sein dürfen.
In diesem Funktionsterm steht im Zähler ein Quadrat. Da ein Quadrat im Bereich der reellen Zahlen immer nichtnegativ ist, muss nur vermieden werden, dass das Quadrat den Wert 0 annimmt:
\begin{array}{rclcl}x^2 &\neq& 0 &\vert& \pm\sqrt{} \\x &\neq& 0\end{array}

Damit der gesamte Bruch einen positiven Wert annimmt, muss also auch der Nenner positiv sein. Die zugehörige Ungleichung -7+13x > 0 wurde aber schon bei der Betrachtung des 1. Arguments auf der linken Gleichungsseite gelöst.

Für die abschließende Ermittlung des Definitionsbereichs muss nun überprüft werden, für welche x-Werte alle ermittelten Bedingungen erfüllt sind. Das ist nur der Fall für x > \dfrac{7}{13}. Der Definitionsbereich ist also \mathbb{D} = \left]\dfrac{7}{13} ; \infty\right[.

Berechnung der Nullstellen:
\begin{array}{rclcl}5\ln(-7+13x)+5\ln\left(\dfrac{x^2}{13x-7}\right)-10 &=& 0 &\vert& +10 \\\\5\ln(-7+13x)+5\ln\left(\dfrac{x^2}{13x-7}\right) &=& 10 &\vert& :5 \\\\\ln(-7+13x)+\ln\left(\dfrac{x^2}{13x-7}\right) &=& 2 &\vert& \text{1. Logarithmengesetz} \\\\\ln\left(\left(-7+13x\right)\cdot \dfrac{x^2}{13x-7} \right) &=& 2 \\\\\ln\left(x^2\right) &=& 2 &\vert& \text{3. Logarithmengesetz} \\\\2\ln\left(x\right) &=& 2 &\vert& :2 \\\ln\left(x\right) &=& 1 &\vert& e^* \\x &=& e \in\mathbb{D}\end{array}

Ergebnis: Die Nullstelle von f(x) liegt bei x=e.


15)
Bestimmung des Definitionsbereichs:
1. Argument rechts:
Da das Argument des ersten Logarithmus ein Produkt aus einer positiven Zahl und einem Quadrat ist und solche Terme ausschließlich nichtnegative Ergebnisse liefern, muss nur vermieden werden, dass der Term den Wert 0 annimmt:
\begin{array}{rclcl} 5z^2 &\neq& 0 &\vert& :5 \\ z^2 &\neq& 0 &\vert & \pm\sqrt{} \\ z &\neq& 0 \\\\\end{array}

2. Argument rechts:
\begin{array}{rclcl} 10z &>& 0 &\vert & :10 \\z &>& 0 \end{array}

Für die abschließende Ermittlung des Definitionsbereichs muss nun überprüft werden, für welche x-Werte alle ermittelten Bedingungen erfüllt sind. Das ist nur der Fall für z > 0. Der Definitionsbereich ist also \mathbb{D} = \mathbb{R}^+.

Berechnung der Nullstellen:
\begin{array}{rclcl}0 &=& \log_3(5z^2)-\log_3(10z)-3 & \vert & +3 \\3 &=& \log_3(5z^2)-\log_3(10z) & \vert & \text{2. Logarithmengesetz} \\\\3 &=& \log_3\left(\dfrac{5z^2}{10z}\right) \\\\3 &=& \log_3\left(\dfrac{z}{2}\right) & \vert &\ 3^* \\27 &=& \dfrac{z}{2} & \vert &\ \cdot 2 \\x &=& 54 \in \mathbb{D}\end{array}

Ergebnis: Die Nullstelle von f(z) liegt bei z=54.


16)
Bestimmung des Definitionsbereichs:
Argument im Zähler:
\begin{array}{rclcl} -x+47 & > & 0 &\vert& -47 \\-x & > & -47 & \vert& \cdot (-1) \\x & < & 47 \\\end{array}

Nenner:
\begin{array}{rclcl}\ln(-x+47) &\neq & 0 &\vert& e^* \\-x+47 &\neq & 1 &\vert& -47 \\-x &\neq & -46 &\vert& \cdot(-1)\\x &\neq& 46\end{array}

Argument im Nenner:
Die zugehörige Ungleichung -x+47 > 0 wurde schon bei der Betrachtung des Arguments im Zähler gelöst.

Für die abschließende Ermittlung des Definitionsbereichs muss nun überprüft werden, für welche x-Werte alle ermittelten Bedingungen erfüllt sind. Das ist nur der Fall für x < 47 unter Ausschluss der Zahl 46. Der Definitionsbereich ist also \mathbb{D} = \left\{x\in\mathbb{R} \mid x < 47 \text{ und } x\neq 46 \right\}.

Berechnung der Nullstellen:
\begin{array}{rclcl}0 &=& \dfrac{\log_4(-x+47)}{\log_4(e) \cdot \ln(-x+47)}\cdot \left(-14x^2+56x+98\right) &\vert & \text{Basistransformation} \\\\0 &=& \dfrac{\log_4(-x+47)}{\log_4(e) \cdot \frac{\log_4(-x+47)}{\log_4(e)}}\cdot \left(-14x^2+56x+98\right) \\\\0 &=& \dfrac{\log_4(-x+47)}{1 \cdot \log_4(-x+47)}\cdot \left(-14x^2+56x+98\right) \\\\0 &=& 1 \cdot \left(-14x^2+56x+98\right) \\\\0 &=& -14x^2+56x+98 &\vert & :(-14) \\0 &=& x^2-4x-7 &\vert & \text{p-q-Formel} \\x_{1,2} &=& 2 \pm \sqrt{4+7} \\ x_{1,2} &=& 2 \pm \sqrt{11} \\\\x_{1} &=& 2 + \sqrt{11} \approx 5{,}32 \in \mathbb{D} \\x_{2} &=& 2 - \sqrt{11} \approx -1{,}32 \in \mathbb{D}\end{array}

Ergebnis: Die Nullstellen von f(x) liegen bei x_{1} = 2 + \sqrt{11} und x_{2} = 2 - \sqrt{11}.


17)
Bestimmung des Definitionsbereichs:
12x^2+18x-11 > 0

Um diese quadratische Ungleichung zu lösen, wird im ersten Schritt die zugehörige quadratische Gleichung gelöst:
\begin{array}{rclcl} 12x^2+18x-11 &=& 0 &\vert& :12 \\x^2+\dfrac{3}{2}x-\dfrac{11}{12} &=& 0 &\vert& \text{p-q-Formel} \\x_{1,2} &=& -\dfrac{3}{4} \pm \sqrt{\dfrac{9}{16}+\dfrac{11}{12}} \\x_{1,2} &=& -\dfrac{3}{4} \pm \sqrt{\dfrac{71}{48}} \\\\x_{1} &=& -\dfrac{3}{4} + \sqrt{\dfrac{71}{48}} \approx 0{,}47 \\x_{2} &=& -\dfrac{3}{4} - \sqrt{\dfrac{71}{48}} \approx -1{,}97\end{array}
Diese Gleichung hat also zwei Lösungen bei x_1=-\dfrac{3}{4} + \sqrt{\dfrac{71}{48}} und x_2=-\dfrac{3}{4} - \sqrt{\dfrac{71}{48}}. Diese x-Werte sind auf jeden Fall nicht Teil des Definitionsbereichs. Es ergeben sich die Intervalle \left]-\infty;-\dfrac{3}{4} - \sqrt{\dfrac{71}{48}}\right[, \left]-\dfrac{3}{4} - \sqrt{\dfrac{71}{48}};-\dfrac{3}{4} + \sqrt{\dfrac{71}{48}}\right[ und \left]-\dfrac{3}{4} + \sqrt{\dfrac{71}{48}};\infty\right[.
Jetzt muss noch geprüft werden, in welchen Intervallen der Term positive bzw. negative Werte annimmt. Es ist sinnvoll, für die Prüfung möglichst einfache x-Werte zu verwenden. Z. B. liefert x=-2 als Ergebnis 1, für x=0 ergibt sich -11 und x=1 führt zu 19. Das bedeutet, dass der Term 12x^2+18x-11 im ersten und dritten Intervall positiv ist. Die ursprüngliche Ungleichung 12x^2+18x-11 > 0 ist also nur in diesen beiden Intervallen wahr.

Der Definitionsbereich ist also  \mathbb{D}=\left\{ x \in \mathbb{R}\mid x < -\dfrac{3}{4} - \sqrt{\dfrac{71}{48}} \text{ oder } x > -\dfrac{3}{4} + \sqrt{\dfrac{71}{48}} \right\}

Berechnung der Nullstellen:
\begin{array}{rclcl}\log_{76} \left(12x^2+18x-11\right) &=& 0 &\vert& 76^* \\12x^2+18x-11 &=& 1 &\vert& -1 \\12x^2+18x-12 &=& 0 &\vert& :12 \\x^2+\dfrac{3}{2}x-1 &=& 0 &\vert& \text{p-q-Formel} \\x_{1,2} &=& - \dfrac{3}{4} \pm \sqrt{\dfrac{9}{16}+1} \\x_{1,2} &=& - \dfrac{3}{4} \pm \sqrt{\dfrac{25}{16}} \\\\x_1 &=& - \dfrac{3}{4}+\dfrac{5}{4} = \dfrac{1}{2} \in \mathbb{D} \\x_2 &=& - \dfrac{3}{4}-\dfrac{5}{4} = -2 \in \mathbb{D}\end{array}

Ergebnis: Die Nullstellen von f(x) liegen bei x_1=\dfrac{1}{2} und x_2=-2.


18)
Bestimmung des Definitionsbereichs:
\begin{array}{rclcl} -\left(5x^2+1\right)\cdot\left(x^2+20\right) & > & 0 \\-5x^4-101x^2-20 & > & 0 &\vert & :\left(-5\right) \\x^4+\dfrac{101}{5}x^2+4 & < & 0 \end{array}

Da Potenzen mit geraden Exponenten im Bereich der reellen Zahlen immer nichtnegativ sind, ist das Argument als Summe aus solchen Potenzen und der Zahl 4 immer positiv. Somit kann die gegebene Ungleichung niemals wahr werden. Der Definitionsbereich ist also \mathbb{D} =\emptyset.

Da diese Funktion für keine Werte definiert ist, erübrigt sich die Suche nach Nullstellen.

19)
Bestimmung des Definitionsbereichs:
1. Argument rechts:
\begin{array}{rclcl} 16e^x+1 &>& 0 &\vert& -1 \\ 16e^x &>& -1 &\vert& :16 \\e^x &>& -\dfrac{1}{16}\end{array}

Da Potenzen mit einer positiven Basis, wie e, ausschließlich positive Werte annehmen, ist die gegebene Ungleichung für alle x-Werte wahr.

2. Argument rechts:
\begin{array}{rclcl}x^3+9 &>& 0 &\vert& -9 \\x^3 &>& -9 &\vert& \sqrt[3]{} \\x &>& \sqrt[3]{-9}\end{array}

Der Definitionsbereich ist also \mathbb{D}=\left]\sqrt[3]{-9};\infty \right[.

Berechnung der Nullstellen:
\begin{array}{lrclcl}& 0 &=& \dfrac{5}{4}\log_{10}\left(16e^x+1\right)\cdot \ln\left(x^3+9\right) &\vert& :\dfrac{5}{4} \\& 0 &=& \log_{10}\left(16e^x+1\right)\cdot \ln\left(x^3+9\right) &\vert& \text{Satz vom Nullprodukt} \\\\\text{Faktor 1:} & 0 &=& \log_{10}\left(16e^x+1\right) &\vert& 10^* \\& 1 &=& 16e^x+1 &\vert& -1 \\& 0 &=& 16e^x &\vert& :16 \\& 0 &=& e^x &\vert& \ln() \\& \ln(0) &=& x_1 \\\\\text{Faktor 2:} & 0 &=& \ln(x^3+9) &\vert& e^* \\& 1 &=& x^3+9 &\vert& -9 \\& -8 &=& x^3 &\vert& \sqrt[3]{} \\& x_2 &=& -2 \in \mathbb{D} \end{array}

Ergebnis: Da der Logarithmus von 0 nicht definiert ist, liegt die einzige Nullstelle von f(x) bei x_2=-2.


20)
Bestimmung des Definitionsbereichs:
1. Argument rechts:
\begin{array}{rclcl} -k+9 & > & 0 &\vert & -9 \\-k & > & -9 &\vert & \cdot(-1)\\k & < & 9 \\\\\end{array}

2. Argument rechts:
\begin{array}{rclcl} 4k &>& 0 &\vert & :4 \\k &>& 0 \end{array}

Für die abschließende Ermittlung des Definitionsbereichs muss nun überprüft werden, für welche x-Werte alle ermittelten Bedingungen erfüllt sind. Das ist nur der Fall für 0 < k < 9. Der Definitionsbereich ist also \mathbb{D} = \left]0 ; 9\right[.

Berechnung der Nullstellen:
\begin{array}{rclcl}0 &=& \log_{314}(-k+9)+\log_{314}(4k) & \vert & \text{1. Logarithmengesetz} \\0 &=& \log_{314}\left((-k+9)\cdot 4k\right) \\0 &=& \log_{314}\left(-4k^2+36k\right) & \vert & 314^* \\1 &=& -4k^2+36k & \vert & -1 \\0 &=& -4k^2+36k-1 & \vert & :(-4) \\0 &=& k^2-9k+\frac{1}{4} & \vert & \text{p-q-Formel} \\k_{1,2} &=& \dfrac{9}{2} \pm \sqrt{\dfrac{81}{4}-\dfrac{1}{4}} \\&=& \dfrac{9}{2} \pm \sqrt{20} = \dfrac{9}{2}\pm\sqrt{4\cdot 5} \\\\k_1 &=& \dfrac{9}{2} + 2\sqrt{5} \approx 8{,}97 \in \mathbb{D} \\k_2 &=& \dfrac{9}{2} - 2\sqrt{5} \approx 0{,}03 \in \mathbb{D}\end{array}

Ergebnis: Die Nullstellen vong(k) liegen bei k_1 = \dfrac{9}{2} + 2\sqrt{5} und k_2 = \dfrac{9}{2} - 2\sqrt{5}.

18. Betragsgleichungen und -funktionen - Lernziele und typische Fehler

Nach Durcharbeiten dieses Kapitels sollten Sie folgende Lernziele erreicht haben:

  • Sie können zu einer Betragsgleichung den passenden Definitionsbereich bestimmen.
  • Sie kennen die allgemeine Form einer Betragsgleichung.
  • Sie können Betragsgleichungen lösen, ggf. mithilfe einer Fallunterscheidung.
  • Sie können die Lösungsmenge mathematisch korrekt notieren.
  • Sie können mithilfe der Probe überprüfen, ob die gefundene Lösung tatsächlich richtig ist.
  • Sie wissen, dass bei Betragsgleichungen Scheinlösungen auftreten können, wie man diese erkennt und wie man damit umgeht.
  • Sie können zu einer Betragsfunktion den passenden Definitionsbereich bestimmen.
  • Sie wissen, wie der Graph einer Betragsfunktion typischerweise aussieht, und können ihn in ein kartesisches Koordinatensystem zeichnen.
  • Sie kennen die allgemeine Funktionsgleichung einer Betragsfunktion.
  • Sie kennen Eigenschaften von Betragsfunktionen ("Knick", typische Punkte) und können dies nutzen, um Beziehungen zwischen Funktionsterm und Funktionsgraphen herzustellen.
  • Sie kennen den Zusammenhang zwischen Betragsgleichungen und Betragsfunktionen.
  • Sie können Betragsgleichungen von anderen Gleichungsarten unterscheiden.
  • Sie können Betragsfunktionen von anderen Funktionstypen unterscheiden (grafisch und anhand der Funktionsgleichung).


Typische Fehler in diesem Kapitel sind:

  • Eine notwendige Fallunterscheidung wird nicht durchgeführt. Erklärung
  • Es wird nicht geprüft, ob ein berechneter Wert eine Scheinlösung ist. Erklärung


Für Online-Selbsttests zu diesem Thema und weitere Informationen zur Mathematikunterstützung an der TH Wildau nutzen Sie bitte den Moodle-Kursraum "SOS Mathematik - Brückenkurs".

 

Dieses Kapitel wird gerade überarbeitet.

Übersicht:

 

18.1 Betragsgleichungen und -funktionen - Aufgaben

1. Aufgabe

Wie verlaufen die Graphen der folgenden Funktionen? Welche Eigenschaften und "speziellen Punkte" haben sie?
Geben Sie zudem die Definitionsbereiche an!

Hinweis: Der erste Teil der Aufgabe ist so gemeint, dass aus dem Funktionsterm Erkenntnisse über der Verlauf des Graphen abgeleitet werden sollen. Die konkreten Funktionswerte und Verläufe der Graphen interessieren hier erst im zweiten Schritt.

1) f(x) = x^\frac{1}{2}+4   11) f(x) = \dfrac{1}{\vert x\vert+1}

2) f(x) = \sqrt{256} \cdot \dfrac{1}{x}   12) f(x) = -2-\sqrt{\dfrac{1}{2}x}


3) f(x) = \vert-17\vert x   13) 
4) f(x) = \dfrac{2}{x-8}

  14) 
5) f(x) = \left\vert\dfrac{1}{x^4}\right\vert   15) 
6) f(x) = \sqrt{\vert x\vert}   16)
7) f(x) = -\left\vert\dfrac{2}{3}x\right\vert+6   17)
8) f(x) = \dfrac{\sqrt[3]{8}}{\sqrt{x}}   18)
9) f(x) = \left\vert \dfrac{1}{x+4}+5 \right\vert   19)
10) f(x) = \left\vert \sqrt{\vert x\vert}-2\right\vert   20)

 

2. Aufgabe

1)
Gegeben sei die Funktion f(x)= \left\vert x-1 \right\vert +2 mit x\in \mathbb{R}.
Gesucht ist jeweils die fehlende Koordinate des Punktes P(x\mid y),
a) wenn x=0
b) wenn y=6

  6)
Gegeben sei die Funktion f(x)=\left\vert -2x^2-20x+78 \right\vert mit x\in \mathbb{R}.
Gesucht ist jeweils die fehlende Koordinate des Punktes P(x\mid y),
a) wenn x=\dfrac{1}{3}
b) wenn y=0
2)
Gegeben sei die Funktion f(x)= -\left\vert x-13 \right\vert +\dfrac{x}{2} mit x\in \mathbb{R}.
Gesucht ist jeweils die fehlende Koordinate des Punktes P(x\mid y),
a) wenn x=-24
b) wenn y=2

  7)
Gegeben sei die Funktion f(x)= - 4\left\vert (6x+5)(-26x-13)\right\vert-10 mit x\in \mathbb{R}.
Gesucht ist jeweils die fehlende Koordinate des Punktes P(x\mid y),
a) wenn x=0
b) wenn y=-10
3)
Gegeben sei die Funktion f(z)= 2 \cdot \left\vert z+4 \right\vert mit z\in \mathbb{R}.
Gesucht ist jeweils die fehlende Koordinate des Punktes P(z\mid y),
a) wenn z=103
b) wenn y=20

  8)
Gegeben sei die Funktion f(x)= \left\vert x\right\vert \cdot x +4x-1 mit x\in \mathbb{R}.
Gesucht ist jeweils die fehlende Koordinate des Punktes P(x\mid y),
a) wenn x=-21
b) wenn y=116
4)
Gegeben sei die Funktion f(x)=\left\vert \dfrac{7}{2}x^2-2 \right\vert \cdot 3-\dfrac{9}{2}x^2 mit x\in \mathbb{R}.
Gesucht ist jeweils die fehlende Koordinate des Punktes P(x\mid y),
a) wenn x=-\dfrac{4}{3}
b) wenn y=-18

  9)
Gegeben sei die Funktion s(t)= t \cdot \left\vert 3t\right\vert -23 mit t\in \mathbb{R}.
Gesucht ist jeweils die fehlende Koordinate des Punktes P(t\mid y),
a) wenn t=8
b) wenn y=256
5)
Gegeben sei die Funktion f(x)= 12+ \left\vert x^3 \right\vert-150 mit x\in \mathbb{R}.
Gesucht ist jeweils die fehlende Koordinate des Punktes P(x\mid y),
a) wenn x=-60
b) wenn y=-111

  10)
Gegeben sei die Funktion f(x)= -7\left\vert x^2-x \right\vert-6 mit x\in \mathbb{R}.
Gesucht ist jeweils die fehlende Koordinate des Punktes P(x\mid y),
a) wenn x = \dfrac{1}{8}
b) wenn y= -6

 

3. Aufgabe

Was fällt Ihnen auf, wenn Sie die folgenden zwei Graphen vergleichen? Wie erklären Sie sich die Unterschiede?

zwei Funktionen zum Vergleich

Dieses Kapitel enthält die folgenden Themen:

 

18.2 Betragsgleichungen und -funktionen - Erklärungen

 

Betragsungleichungen

Zum Lösungen von Ungleichungen, die Terme mit Beträgen enthalten, braucht man eigentlich immer Fallunterscheidungen, die wir uns im Kapitel Ungleichungen schon mal angeschaut hatten, denn der Betrag ist ja unterschiedlich definiert, je nachdem, ob der Term im Betrag positiv bzw. 0 oder negativ ist. Bei Zahlen lässt sich das ja ohne Weiteres entscheiden, sodass der Betrag dann leicht berechnen werden kann. Durch die Variablen ist das hier nicht ganz so einfach.

Beispielaufgabe:
Schauen wir uns \left| 2x-1\right|\geq 1+3x an.

Lösung:
Will man diese Ungleichung lösen, muss man den Betrag auflösen. Dabei hilft die Definition des Betrages:
\left|a\right|=a, wenn a\geq 0 und
\left|a\right|=-a, wenn a < 0

Da wir nicht wissen können, ob 2x-1 negativ oder nichtnegativ ist, schauen wir uns einfach beide Fälle separat an. Das nennt man eine Fallunterscheidung.

1. Fall: Wir nehmen an 2x-1\geq 0 oder alternativ x\geq\frac{1}{2} (was das gleiche ist, nur ein bisschen umgeformt). Jetzt greift nämlich die erste Zeile der Betragsdefinition: Da der Term im Betrag in diesem Fall positiv ist, dürfen die Betragsstriche weggelassen werden. Dadurch vereinfacht sich die Ungleichung zu

\begin{array}{rclcl} 2x-1 & \geq & 1+3x & \vert & -3x+1 \cr -x & \geq & 2 & \vert & \cdot (-1) \cr x & \leq & -2 \end{array}

Das sieht nach einem schönen Ergebnis aus - ist es aber leider nicht. Wir hatten ja oben bei der Fallunterscheidung angenommen, dass x\geq\frac{1}{2}. Das bedeutet, dass nur solche Zahlen überhaupt als Lösungen infrage kommen. Die Frage, die nun noch beantwortet werden muss, ist also: Gibt es Zahlen für die beide Bedingungen gelten? Oder konkret für dieses Beispiel: Gibt es Zahlen, die kleiner oder gleich -2 und größer oder gleich \frac{1} {2} sind? Eindeutige Antwort: Solche Zahlen kann es nicht geben! Sie müssten ja gleichzeitig positiv und negativ sein. Also gibt es im 1. Fall keine Lösungen: \mathbb{L}_1=\emptyset

2. Fall: Wir nehmen an 2x-1< 0 oder alternativ x < \frac{1}{2} (auch hier nur ein bisschen umgeformt). Hier müssen wir die zweite Zeile der Betragsdefinition anwenden: Wir schreiben ein Minuszeichen vor den Term und dürfen dafür die Betragsstriche weglassen. Wichtig ist, an die Klammern zu denken, weil sich das Minuszeichen ja auf den gesamten Term auswirken soll. Dadurch vereinfacht sich die Ungleichung zu

\begin{array}{rclcl} -(2x-1) & \geq & 1+3x \cr -2x+1 & \geq & 1+3x & \vert & -3x-1 \cr -5x & \geq & 0 & \vert & :\left(-5\right) \cr x & \leq & 0 \end{array}

Auch nun müssen wir natürlich das Rechenergebnis mit der Bedingung, die sich aus der Fallunterscheidung ergeben hat, abgleichen, um eine Aussage über Lösungen treffen zu können. Beide Bedingungen müssen erfüllt sein. Frage: Gibt es Zahlen, die sowohl kleiner als \frac{1} {2} als auch kleiner oder gleich 0 sind? Hier haben wir Glück, denn alle Zahlen, die kleiner oder gleich 0 sind, sind natürlich auch kleiner als \frac{1} {2}. Die Lösungsmenge lautet hier: \mathbb{L}_2=\;\rbrack-\infty;0\rbrack.

Letzter Schritt: Um zur Lösungsmenge der ursprünglichen Ungleichung zu kommen, müssen die beiden Teillösungsmengen zusammengefügt werden. Das tut man mathematisch, indem man die Vereinigungsmenge der beiden Mengen bildet:
\mathbb{L}=\mathbb{L}_1\cup\mathbb{L}_2=\emptyset\;\cup\;\rbrack-\infty;0\rbrack=\;\rbrack-\infty;0\rbrack

Bemerkung 1: Das Bilden der Vereinigungsmenge ist bei dieser Aufgabe ein bisschen witzlos, weil \mathbb{L}_1 ja leer ist. Es können zu \mathbb{L}_2 also gar keine Elemente hinzukommen. Trotzdem gehört dieser Schritt zum vollständigen Lösungsweg dazu. Meist hat man es ja auch mit Teillösungsmengen zu tun, bei denen sich das vereinigen lohnt ...
Bemerkung 2: Hier sieht man wieder, dass eine saubere Fallunterscheidung dazu führt, dass die Aufgabe danach einfacher ist als vorher. Nach der Fallunterscheidung haben wir keinen Betrag mehr in der Ungleichung. Es bleiben zwei kleine lineare Ungleichungen übrig, die in wenigen Schritten gelöst werden können. 

 

Betragsfunktionen

Definition: Eine Betragsfunktion ist eine Funktion, bei der auf einen Term, der die Variable enthält, ein Betrag angewendet wird.

In der folgenden Grafik sind die Funktionen

  • f_1(x) = \left| x \right| mit \mathbb{D}=\mathbb{R}

  • f_2(x) = -\left| 2(x+3)^2-0{,}1(x+3)-6 \right| mit \mathbb{D}=\mathbb{R}

  • f_3(x) = 0{,}25 \left| (x+4)^3 \right|-3 mit \mathbb{D}=\mathbb{R}

  • f_4(x) = \left| 12^{x-5}\right| mit \mathbb{D}=\mathbb{R}

dargestellt. Da grundsätzlich jede Funktion, die wir in den vorherigen Kapiteln kennengelernt haben, Bestandteil einer Betragsfunktion sein kann, gibt es hier keine wirklich typischen Beispiele. Aus diesem Grund ist es hier - im Gegensatz zu den bisher besprochenen Funktionstypen - nicht möglich, allgemeine Aussagen über die Eigenschaften und "besondere Punkte" vom Betragsfunktionen zu treffen. Sie sind einfach zu vielfältig ...

Beispiele für Betragsfunktionen

Weitere Besonderheiten: Typisch für eine Betragsfunktion ist ein "Knick" im Funktionsgraphen, der sich ergibt, wenn die Ursprungsfunktion (also die Funktion ohne den Betrag) positive und negative Funktionswerte hatte. Auf diesen "Knick" muss z. B. bei der Differenzialrechnung besonders geachtet werden. Aber wie Sie in der rechten Grafik sehen muss auch nicht jede Betragsfunktion einen "Knick" haben. Das liegt daran, dass f^*_3(x)=0{,}25 (x+4)^3 -3 (also die Funktion ohne den Betrag) im Punkt (-4\mid -3) einen Sattelpunkt hat.

Übersicht:

 

18.3 Betragsgleichungen und -funktionen - Lösungen

Eine Bemerkung vorab: Verschachtelte Funktionen betrachtet man am besten von innen nach außen.

 

1. Aufgabe

1)
\begin{array}{rcl} f_1(x) &=& x^\frac{1}{2}+4 \cr \mathbb{D} &=& \mathbb{R}_0^+ \end{array}

Bei f_1(x) handelt es sich um eine um 4 nach oben verschobene Wurzelfunktion.
f_1(x) hat keine Nullstellen, weil \sqrt{x}\geq 0 und deswegen \sqrt{x}+4\geq 4 für alle x\in\mathbb{D}. Bei x=0 hat f_1(x) ein Minimum, denn \sqrt{0} ist kleiner als \sqrt{x} für alle anderen x\in\mathbb{D}.
Für den Wertebereich gilt: \mathbb{W}=[4; \infty[


2)
\begin{array}{rcl} f_2(x) &=& \sqrt{256} \cdot \dfrac{1}{x} \cr \mathbb{D} &=& \mathbb{R}\backslash_{\{0\}} \end{array}

Der Funktionsterm lässt sich vereinfachen zu f_2(x)=\dfrac{16}{x}, weil \sqrt{256}=16. Es handelt sich hierbei also um eine gebrochen rationale Funktion, deren Graph eine Hyperbel ungerader Ordnung ist. Wegen des Faktors 16 im Zähler verläuft der Graph nicht durch die Punkte (1 \mid 1) und (-1 \mid -1) sondern durch (1\mid 16) und (-1 \mid -16).
f_2(x) hat eine Polstelle bei x=0.
f_2(x) ist punktsymmetrisch zum Koordinatenursprung.


3)
\begin{array}{rcl} f_3(x) &=& \vert-17\vert x \cr \mathbb{D} &=& \mathbb{R} \end{array}

Diese Funktion ist linear; ihr Graph ist also eine Gerade. Der Betrag bewirkt nur, dass der Koeffizient positiv wird. Es handelt sich bei f_3(x) also um eine steigende Gerade, deren Funktionsgleichung man auch einfacher als f_3(x)=17x schreiben könnte. Da keine Konstante addiert wird, handelt es sich um eine Ursprungsgerade, die logischerweise eine Nullstelle bei x=0 hat.
f_3(x) ist punktsymmetrisch zum Koordinatenursprung.


4)

\begin{array}{rcl} f_4(x) &=& \dfrac{2}{x-8} \cr \mathbb{D} &=& \mathbb{R}\backslash_{\{8\}} \end{array}

Der Graph von f_4(x) ist eine um 8 nach rechts verschobene Hyperbel ungerader Ordnung. Aufgrund der Verschiebung und des Faktors 2 im Zähler verläuft der Graph von f(x) durch die Punkte (9 \mid 2) und (7\mid -2) statt durch (1 \mid 1) und (-1 \mid -1).
Die Funktion f_4(x) hat bei x=8 eine Polstelle.
f_4(x) ist punktsymmetrisch zum Punkt (8 \mid 0).

Betragsfunktionen


5)

\begin{array}{rcl} f_5(x) &=& \left|\dfrac{1}{x^4}\right| \cr \mathbb{D} &=& \mathbb{R}\backslash_{\{0\}} \end{array}

Der Betrag hat bei dieser Funktion keine Wirkung, da \dfrac{1}{x^4} ohnehin immer positiv ist. Der Graph ist also eine Hyperbel gerader Ordnung, die (wie das solche einfachen Hyperbeln gerader Ordnung nun mal tun) durch die Punkte (1 \mid 1) und (-1\mid 1) verläuft.
f_5(x) hat eine Polstelle bei x=0.
Der Graph ist achsensymmetrisch zur y-Achse.


6)
\begin{array}{rcl} f_6(x) &=& \sqrt{\vert x\vert} \cr \mathbb{D} &=& \mathbb{R} \end{array}

Durch den Betrag im Radikanden können hier alle reellen Zahlen eingesetzt werden. Für negative Zahlen verläuft die Funktion analog zum positiven Bereich.
f_6(x) hat eine Nullstelle bei x=0 , die zugleich Minimum der Funktion ist. Kleiner als 0 kann das Ergebnis einer Wurzel ja nicht werden. Daran ändert auch der Betrag im Radikanden nichts.
Der Graph ist achsensymmetrisch zur y-Achse, daher verläuft der Graph nicht nur durch den Punkt (1 \mid 1), wie das für solche Wurzelfunktionen üblich ist, sondern auch durch (-1\mid 1).


7)
\begin{array}{rcl} f_7(x) &=& -\left|\dfrac{2}{3}x\right|+6 \cr \mathbb{D} &=& \mathbb{R} \end{array}

f_7(x) ist eine um 6 nach oben verschobene nach unten geöffnete Betragsfunktion.
Die Funktion hat Nullstellen bei x_1=-9 und x_2=9 und ein Maximum bei x=0, weil -\left|\dfrac{2}{3}x\right|\leq0 für alle x\in\mathbb{D}. Also ist -\left|\frac{2}{3}x\right|+6 dann am größten, wenn -\left|\dfrac{2}{3}x\right|=0 ist. Dann wird nämlich am wenigsten von der 6 subtrahiert.
f_7(x) ist achsensymmetrisch zur y-Achse.
Für den Wertebereich gilt: \mathbb{W}=\;]-\infty; 6]


8)
\begin{array}{rcl} f_8(x) &=& \dfrac{\sqrt[3]{8}}{\sqrt{x}} \cr \mathbb{D} &=& \mathbb{R}^+ \end{array}

Vereinfacht gilt: f_8(x)=\dfrac{2}{\sqrt{x}}. Der Graph dieser Funktion verläuft wie der positive Ast einer Hyperbel. Wegen \sqrt{1}=1 und des Faktors 2 im Zähler verläuft der Graph durch den Punkt (2 \mid 1).
f_8(x) hat eine Polstelle bei x=0.

Betragsfunktionen


9)
\begin{array}{rcl} f_9(x) &=& \left| \dfrac{1}{x+4}+5 \right| \cr \mathbb{D} &=& \mathbb{R}\backslash_{\{0\}} \end{array}

Innerhalb des Betrags steht der Term einer um 5 nach oben und um 4 nach links verschobenen Hyperbel ungerader Ordnung. Der Betrag bewirkt nun, dass alle Punkte mit negativem Funktionswert "nach oben geklappt" werden. Der Graph von f_9(x) verläuft aufgrund der Verschiebung durch die Punkte (-5\mid 4) und (-3\mid 6) . Da diese Punkte ohnehin positive Funktionswerte haben, ändert der Betrag hier also nichts.
f_9(x) hat eine Nullstelle bei x=-\;\frac{21}{5} , die gleichzeitig Minimum ist, weil f_9\left(-\;\frac{21}{5}\right)=0 und 0 hier der kleinste mögliche Wert ist. Bei x=-4 liegt eine Polstelle. Für sehr große und sehr kleine x-Werte nähern sich die Funktionswerte der 5 an.


10)
\begin{array}{rcl} f_{10}(x) &=& \left| \sqrt{\vert x\vert}-2\right| \cr \mathbb{D} &=& \mathbb{R} \end{array}

Zunächst steht unter der Wurzel die Variable innerhalb eines Betrags. Das bewirkt, dass die Funktion für alle reellen Zahlen definiert ist, weil die Variable unter der Wurzel nie negativ werden kann. Innerhalb des äußeren Betrags steht also der Term dieser Wurzelfunktion um 2 nach unten verschoben. Der Betrag bewirkt nun, dass alle Punkte mit negativem Funktionswert "nach oben geklappt" werden.
f_{10}(x) hat zwei Nullstellen bei x_1=4 und bei x_2=-4, die gleichzeitig Minima sind. Kleiner als 0 kann eine solche Betragsfunktion ja nicht werden. Zwischen den zwei Minima muss sich ein Maximum befinden. Da die Funktion aufgrund des Betrages achsensymmetrisch ist, muss das Maximum genau "in der Mitte" zwischen den Minima liegen. Es liegt also bei x=0.


11)
\begin{array}{rcl} f_{11}(x) &=& \dfrac{1}{\vert x\vert+1} \cr \mathbb{D} &=& \mathbb{R} \end{array}

Die Funktion hat ein Maximum bei x=0, weil der Nenner dort am kleinsten ist. Je größer x wird, desto größer wird der Nenner und desto kleiner wird der Wert des gesamten Bruches. Gleiches gilt für sehr kleine x, da das Vorzeichen wegen des Betrages irrelevant ist. An beiden Rändern des Definitionsbereiches nähert sich der Graph von f_{11}(x) also der 0 an.
Da \vert x\vert niemals -1 werden kann, wird der Nenner nie 0. Es gibt also keine Definitionslücken.
f_{11}(x) ist aufgrund des Betrages achsensymmetrisch.
Für den Wertebereich gilt: \mathbb{W}=\;]0; 1]


12)
\begin{array}{rcl} f_{12}(x) &=& -2-\sqrt{\frac{1}{2}x} \cr \mathbb{D} &=& \mathbb{R}_0^+ \end{array}

Die Funktion ist eine "nach unten gekippte" Wurzelfunktion. Zusätzlich ist sie um 2 nach unten verschoben.
f_{12}(x) hat ein Maximum bei x=0, denn dort ist der Radikand 0. Es wird bei x=0 also nichts von der -2 subtrahiert, sodass der Funktionswert hier am größten ist.
Für den Wertebereich gilt: \mathbb{W}=\;]-\infty; -2]

Betragsfunktionen

 

2. Aufgabe

1)

a)
\begin{array}{rclll} f\left(0\right) &=& \vert 0-1 \vert +2 &=& 3 \quad \rightarrow \quad P_1\left(0 \mid 3\right) \end{array}

b)
f(x)= \left\vert x-1 \right\vert +2=\left\{\begin{array}{rllll} x-1+2 &=& x+1 & \text{ für } & x\geq1 \cr-(x-1)+2 &=& -x+3 & \text{ für } & x < 1\end{array} \right.

\begin{array}{rclcl rclcl}\text{1. Fall: } & & & & & \text{2. Fall: } \cr 6 &=& x+1 &\vert& -1 & 6 &=& -x+3 &\vert& -3 \cr 5 &=& x_2 & & & 3 &=& -x &\vert& \cdot (-1) \cr & & & & & -3 &=& x_3 & & \end{array}

Probe:
Für x_2:
\begin{array}{rcl} 6 &=& 5+1 \cr 6 &=& 6 \end{array}

Für x_3:
\begin{array}{rcl} 6 &=& -(-3)+3 \cr 6 &=& 6 \end{array}

Ergebnis: Für x_2=5 und x_3=-3 ergeben sich wahre Aussagen: P_2\left(5 \mid 6\right) und P_3\left(-3 \mid 6\right) sind also tatsächlich Punkte des Graphen.


2)
a)
\begin{array}{rclll} f\left(-24\right) &=&-\vert -24-13 \vert +\dfrac{-24}{2}&=& -49 \quad \rightarrow \quad P_1\left(-24 \mid -49\right) \end{array}

b)
 f(x)=-\left\vert x-13 \right\vert +\dfrac{x}{2} =\left\{\begin{array}{rllll}-(x-13)+\dfrac{x}{2} &=& -\dfrac{1}{2}x+13 & \text{ für } & x\geq13 \cr-\left(-(x-13)\right)+\dfrac{x}{2} &=& \dfrac{3}{2}x-13 & \text{ für } & x < 13\end{array} \right.

\begin{array}{rclcl rclcl}\text{1. Fall} & & & & & \text{2. Fall} \cr2 &=& -\dfrac{1}{2}x+13 &\vert& -13 & 2 &=& \dfrac{3}{2}x-13 &\vert& +13 \cr -11 &=& -\dfrac{1}{2}x &\vert& : \left(-\dfrac{1}{2}\right) & 15 &=& \dfrac{3}{2}x &\vert& : \dfrac{3}{2} \cr 22 &=& x_2 & & & 10 &=& x_3\end{array}

Probe:
Für x_2:
\begin{array}{rcl} 2 &=& -\dfrac{1}{2} \cdot 22+13 \cr 2 &=& 2 \end{array}

Für x_3:
\begin{array}{rcl} 2 &=& \dfrac{3}{2} \cdot 10-13 \cr 2 &=& 2 \end{array}

Ergebnis: Für x_2=22 und x_3=10 ergeben sich wahre Aussagen: P_2\left( 22 \mid 2\right) und P_3\left( 10\mid 2\right) sind also tatsächlich Punkte des Graphen.


3)
a)
\begin{array}{rclll} f\left(103\right) &=& 2 \cdot \vert 103+4 \vert &=& 214 \quad \rightarrow \quad P_1\left(103\mid 214\right) \end{array}

b)
 f(z)= 2 \cdot \vert z+4 \vert=\left\{\begin{array}{rllll}2(z+4) &=& 2z+8 & \text{ für } & z\geq-4 \cr2\left(-(z+4)\right) &=& -2z-8 & \text{ für } & z < -4\end{array} \right.

\begin{array}{rclcl rclcl}\text{1. Fall} & & & & & \text{2. Fall} \cr20 &=& 2z+8 &\vert& -8 & 20 &=& -2z-8 &\vert& +8 \cr 12 &=& 2z &\vert& : 2 & 28 &=& -2z &\vert& : \left(-2\right) \cr 6 &=& z_2 & & & -14 &=& z_3 & & \end{array}

Probe:
Für z_2:
\begin{array}{rcl} 20 &=& 2 \cdot 6+8 \cr 20 &=& 20 \end{array}

Für z_3:
\begin{array}{rcl} 20 &=& -2 \cdot (-14)-8 \cr 20 &=& 20 \end{array}

Ergebnis: Für z_2=6 und z_3=-14 ergeben sich wahre Aussagen: P_2\left(6 \mid 20\right) und P_3\left(-14 \mid 20\right) sind also tatsächlich Punkte des Graphen.


4)
a)
\begin{array}{rclll} f\left(-\dfrac{4}{3}\right) &=&\left\vert \dfrac{7}{2}\cdot\left(-\dfrac{4}{3}\right)^2-2 \right\vert \cdot 3-\dfrac{9}{2}\cdot\left(-\dfrac{4}{3}\right)^2&=& \dfrac{14}{3} \quad \rightarrow \quad P_1\left(-\dfrac{4}{3} \mid \dfrac{14}{3}\right) \end{array}

b)
\begin{array}{rclll}-18 &=& \left\vert \dfrac{7}{2}x^2-2 \right\vert \cdot 3-\dfrac{9}{2}x^2 &\vert& +\dfrac{9}{2}x^2 \cr -18+\dfrac{9}{2}x^2 &=& \left\vert \dfrac{7}{2}x^2-2 \right\vert \cdot 3 &\vert& : 3 \cr -6+\dfrac{3}{2}x^2 &=& \left\vert \dfrac{7}{2}x^2-2 \right\vert &\vert& ()^2 \cr \left(-6+\dfrac{3}{2}x^2\right)^2 &=& \left(\dfrac{7}{2}x^2-2 \right)^2 \cr\cr \dfrac{9}{4} x^4-18x^2+36 &=& \dfrac{49}{4}x^4-14x^2+4 &\vert& -\dfrac{9}{4} x^4+18x^2-36 \cr0 &=& 10x^4+4x^2-32 \cr0 &=& 10\left(x^2\right)^2+4x^2-32\end{array}

Substitution: z=x^2
\begin{array}{rclll}0 &=& 10z^2+4z^2-32 &\vert& : 10 \cr 0 &=& z^2+\dfrac{2}{5}z-\dfrac{16}{5} &\vert& \text{p-q-Formel} \cr z_{2,3} &=& -\dfrac{1}{5} \pm \sqrt{\left(\dfrac{1}{5}\right)^2+\dfrac{16}{5}} \cr z_{2,3} &=& -\dfrac{1}{5} \pm \sqrt{\dfrac{81}{25}} \cr\cr z_2 &=& -\dfrac{1}{5} + \dfrac{9}{5} = \dfrac{8}{5} \cr\cr z_3 &=& -\dfrac{1}{5} - \dfrac{9}{5} = -2\end{array}

Rücksubstitution:
\begin{array}{rclclcl}z_2 &=& x^2 &=& \dfrac{8}{5} &\vert& \pm\sqrt{} \cr & & x_{2,3} &=& \pm\sqrt{\dfrac{8}{5}} \approx \pm 1{,}26 & & \cr\cr z_3 &=& x^2 &=& -2 &\vert& \pm\sqrt{} \cr & & x_{4,5} &=& \pm\sqrt{-2}\end{array}

Da aus negativen reellen Zahlen keine Wurzeln mit geraden Wurzelexponenten gezogen werden können, liefert die Rücksubstitution von z_3 keine weiteren Punkte.

Probe:
Für x_2:
\begin{array}{rcl} -18 &=& \left\vert \dfrac{7}{2} \cdot \left(\sqrt{\frac{8}{5}}\right)^2-2 \right\vert \cdot 3-\dfrac{9}{2} \cdot \left(\sqrt{\frac{8}{5}}\right)^2 \cr 18 &=& \dfrac{18}{5} \end{array}

Für x_3:
\begin{array}{rcl} -18 &=& \left\vert \dfrac{7}{2} \cdot \left(-\sqrt{\frac{8}{5}}\right)^2-2 \right\vert \cdot 3-\dfrac{9}{2} \cdot \left(-\sqrt{\frac{8}{5}}\right)^2 \cr 18 &=& \dfrac{18}{5} \end{array}

Ergebnis: Für x_2=\sqrt{\frac{8}{5}}und x_3=-\sqrt{\frac{8}{5}}ergeben sich unwahre Aussagen: -\sqrt{\frac{8}{5}} und \sqrt{\frac{8}{5}} sind also nicht Lösungen der Gleichung. Das bedeutet, dass die Funktion nirgends den Funktionswert -18 annimmt.


5)
a)
\begin{array}{rclll} f\left(-60\right) &=&12+ \left\vert (-60)^3 \right\vert-150 &=& 215.862 \quad \rightarrow \quad P_1\left(-60 \mid 215.862\right) \end{array}

b)
 f(x)= 12+ \left\vert x^3 \right\vert-150=\left\{\begin{array}{rllll}12+x^3-150 &=& x^3 -138 & \text{ für } & x\geq0 \cr 12-x^3-150 &=& -x^3 -138 & \text{ für } & x < 0\end{array} \right.

\begin{array}{rclcl rclcl}\text{1. Fall} & & & & & \text{2. Fall} \cr-111 &=& x^3 -138 &\vert& +138 & -111 &=& -x^3 -138 &\vert& +138 \cr 27 &=& x^3 &\vert& \sqrt[3]{} & 27 &=& -x^3 &\vert& \cdot (-1) \cr 3 &=& x_2 & & & -27 &=& x^3 &\vert& \sqrt[3]{} \cr & & & & & -3 &=& x_3\end{array}

Probe:
Für x_2:
\begin{array}{rcl} -111 &=& 3^3-138 \cr -111 &=& -111 \end{array}

Für x_3:
\begin{array}{rcl} -111 &=& -(-3)^3-138 \cr -111 &=& -111 \end{array}

Ergebnis: Für x_2=3 und x_3=-3 ergeben sich wahre Aussagen: P_2\left(3 \mid -111\right) und P_3\left(-3 \mid -111\right) sind also tatsächlich Punkte des Graphen.


6)
a)
\begin{array}{rclll} f\left(\dfrac{1}{3}\right) &=& \left\vert -2\cdot\left(\dfrac{1}{3}\right)^2-20\cdot\dfrac{1}{3}+78 \right\vert &=& \dfrac{640}{9} \quad \rightarrow \quad P_1\left(\dfrac{1}{3} \mid \dfrac{640}{9}\right) \end{array}

b)
 f(x)= \left\vert -2x^2-20x+78 \right\vert=\left\{\begin{array}{rll}-2x^2-20x+78 & \text{ für } & x\geq0 \cr-\left(-2x^2-20x+78\right) = 2x^2+20x-78 & \text{ für } & x < 0\end{array} \right.

\begin{array}{rclcl rclcl} \text{1. Fall} & & & & & \text{2. Fall} \cr 0 &=& -2x^2-20x+78 &\vert& :(-2) & 0 &=& 2x^2+20x-78 &\vert& :2 \cr 0 &=& x^2+10x-39 &\vert& \text{p-q-Formel} & 0 &=& x^2+10x-39 &\vert& \text{p-q-Formel} \cr x_{2,3} &=& -5\pm\sqrt{(-5)^2+39} & & & x_{4,5} &=& -5\pm\sqrt{(-5)^2+39} \cr x_{2,3} &=& -5\pm\sqrt{64} & & & x_{4,5} &=& -5\pm\sqrt{64} \cr\cr x_2 &=& -5+8 = 3 & & & x_4 &=& -5+8 = 3 \cr x_3 &=& -5-8 = -13 & & & x_5 &=& -5-8 = -13 \end{array}

x_3=-13 ist keine Lösung, da im ersten Fall nur x\geq0 infrage kommen. Ebenso ist x_4=3 keine Lösung, da im zweiten Fall nur x < 0 möglich sind.

Probe:
Für x_2:
\begin{array}{rcl} 0 &=& \left\vert -2\cdot 3^2-20\cdot 3+78 \right\vert \cr 0 &=& \vert -18-60+78\vert \cr 0 &=& 0 \end{array}

Für x_5:
\begin{array}{rcl} 0 &=& \left\vert -2\cdot (-13)^2-20\cdot (-13)+78 \right\vert \cr 0 &=& \vert -338+260+78\vert \cr 0 &=& 0 \end{array}

Ergebnis: Für x_2=3 und x_5=-13 ergeben sich wahre Aussagen: P_2\left(-13 \mid 0\right) und P_3\left(3 \mid 0\right) sind also tatsächlich Punkte des Graphen.


7)
a)
\begin{array}{rclll} f\left(0\right) &=&- 4\cdot\left\vert (6\cdot0+5)\cdot(-26\cdot0-13)\right\vert-10 &=& -270 \quad \rightarrow \quad P_1\left(0 \mid -270\right) \end{array}

b)
\begin{array}{crclcl} & -10 &=& -4\left\vert(6x+5)(-26x-13)\right\vert-10 &\vert& +10 \cr & 0 &=& -4\left\vert(6x+5)(-26x-13)\right\vert &\vert& :(-4)\cr& 0 &=& \left\vert(6x+5)(-26x-13)\right\vert \cr& 0 &=& (6x+5)(-26x-13) &\vert& \text{Satz vom Nullprodukt}\cr\text{Faktor 1:} & 6x+5 &=& 0 &\vert& -5 \cr& 6x &=& -5 &\vert& :6 \cr& x_2 &=& -\dfrac{5}{6} \cr\cr\text{Faktor 2:} & -26x -13 &=& 0 &\vert& +13 \cr& -26x &=& 13 &\vert& :(-26) \cr& x_3 &=& -\dfrac{1}{2}\end{array}

Bemerkung: Da der Betrag 0 sein soll, können wir den Betragsstriche weglassen (Umformung von der 3. zur 4. Zeile), da 0 ja weder positiv noch negativ ist.

Probe:
Für x_2:
\begin{array}{rcl} -10 &=& -4 \cdot\left\vert \left(6 \cdot \left(-\dfrac{5}{6}\right)+5\right)\cdot\left(-26 \cdot \left(-\dfrac{5}{6}\right)-13\right)\right\vert -10 \cr -10 &=& -4 \cdot\vert 0\vert -10 \cr -10 &=& -10 \end{array}

Für x_3:
\begin{array}{rcl} -10 &=& -4 \cdot\left\vert \left(6 \cdot \left(-\dfrac{1}{2}\right)+5\right)\cdot\left(-26 \cdot \left(-\dfrac{1}{2}\right)-13\right)\right\vert -10 \cr -10 &=& -4 \cdot\vert 0\vert -10 \cr -10 &=& -10 \end{array}

Ergebnis: Für x_2=-\dfrac{5}{6} und x_3=-\dfrac{1}{2} ergeben sich wahre Aussagen: P_2\left(-\dfrac{5}{6} \mid -10\right) und P_3\left(-\dfrac{1}{2} \mid -10\right) sind also tatsächlich Punkte des Graphen.


8)
a)
\begin{array}{rclll} f\left(-21\right) &=& \vert -21\vert \cdot (-21) +4\cdot(-21)-1 &=& -526 \quad \rightarrow \quad P_1\left( -21\mid -526\right) \end{array}

b)
 f(x)= \left\{\begin{array}{rcrcl}x\cdot x+4x-1 &=& x^2 +4x-1 & \text{ für } & x\geq0 \cr-x\cdot x + 4x-1 &=& -x^2+4x-1 & \text{ für } & x < 0\end{array} \right.

\begin{array}{rclcl rclcl}\text{1. Fall} & & & & & \text{2. Fall} \cr116 &=& x^2 +4x-1 &\vert& -116 \quad & 116 &=& -x^2+4x-1 &\vert& -116\cr 0 &=& x^2 +4x-117 &\vert& \text{p-q-Formel} & 0 &=& -x^2+4x-117 & \vert& \cdot (-1) \cr & & & & & 0 &=& x^2-4x+117 &\vert& \text{p-q-Formel} \cr x_{2,3} &=& -2\pm\sqrt{4+117} & & & x_{4,5} &=& 2\pm\sqrt{4-117}\cr x_{2,3} &=& -2\pm\sqrt{221} & & & x_{4,5} &=& 2\pm\sqrt{-113} \cr x_{2,3} &=& -2\pm 11 \cr\cr x_2 &=& 9 \cr x_3 &=& -13\end{array}

x_3=-13 ist keine Lösung, da im ersten Fall nur x\geq0 infrage kommen. Da aus negativen reellen Zahlen keine Wurzeln mit geraden Wurzelexponenten gezogen werden können, liefert der zweite Fall keine weiteren Punkte.

Probe:
Für x_2:
\begin{array}{rcl} 116 &=& \vert 9 \vert \cdot 9+4 \cdot 9-1 \cr 116 &=& 81+36-1 \cr 116 &=& 116 \end{array}

Ergebnis: Für x_2=9 ergibt sich eine wahre Aussage: P_2\left(9 \mid 116\right) ist also tatsächlich ein Punkt des Graphen.


9)
a)
\begin{array}{rclll} s\left(8\right) &=& 8 \cdot \vert 3\cdot8\vert -23&=& 169 \quad \rightarrow \quad P_1\left( 8\mid 169\right) \end{array}

b)
 s(t)=\left\{\begin{array}{rcrcl}t \cdot 3t -23 &=& 3t^2-23 & \text{ für } & t\geq0 \cr t \cdot (-3t) -23 &=& -3t^2-23 & \text{ für } & t < 0\end{array} \right.

\begin{array}{rclcl rclcl}\text{1. Fall} & & & & & \text{2. Fall} \cr256 &=& 3t^2-23 &\vert& +23 \quad & 256 &=& -3t^2-23 &\vert& +23 \cr 279 &=& 3t^2 &\vert& : 3 & 279 &=& -3t^2 &\vert& : (-3)\cr 93 &=& t^2 &\vert& \pm\sqrt{} & -93 &=& t^2 & \vert& \pm\sqrt{}\cr & & & & & \pm\sqrt{-93} &=& t \cr\cr t_2 &=& \sqrt{93} \cr t_3 &=& -\sqrt{93}\end{array}

t_3=-\sqrt{93} ist keine Lösung, da im ersten Fall nur t\geq0 infrage kommen. Da aus negativen reellen Zahlen keine Wurzeln mit geraden Wurzelexponenten gezogen werden können, liefert der zweite Fall keine weiteren Punkte.

Probe:
Für t_2:
\begin{array}{rcl} 256 &=& \sqrt{93} \cdot \left\vert 3 \cdot \sqrt{93} \right\vert -23 \cr 256 &=& 279-23 \cr 256 &=& 256 \end{array}

Ergebnis: Für t_2=\sqrt{93} ergibt sich eine wahre Aussage: P_2\left(\sqrt{93} \mid 256\right) ist also tatsächlich ein Punkt des Graphen.


10)
a)
\begin{array}{rclll} f\left(\dfrac{1}{8}\right) &=& -7\cdot\left\vert \left(\dfrac{1}{8}\right)^2-\dfrac{1}{8} \right\vert-6 &=& -\dfrac{433}{64} \quad \rightarrow \quad P_1\left(\dfrac{1}{8} \mid -\dfrac{433}{64} \right) \end{array}

b)
\begin{array}{crclcl}& -6 &=& -7\left\vert x^2-x \right\vert-6 &\vert& +6 \cr& 0 &=& -7\left\vert x^2-x \right\vert &\vert& : (-7) \cr & 0 &=& \left\vert x^2-x \right\vert \cr & 0 &=& x(x-1) &\vert& \text{Satz vom Nullprodukt} \cr \text{Faktor 1:} & x_2 &=& 0 \cr\cr \text{Faktor 2:} & x-1 &=& 0 &\vert& +1 \cr & x_3 &=& 1\end{array}

Bemerkung: Da der Betrag 0 sein soll, können wir den Betragsstriche weglassen (Umformung von der 3. zur 4. Zeile), da 0 ja weder positiv noch negativ ist.

Probe:
Für x_2:
\begin{array}{rcl} -6 &=& -7 \cdot\left\vert0^2-0\right\vert-6 \cr -6 &=& -6 \end{array}

Für x_3:
\begin{array}{rcl} -6 &=& -7 \cdot\left\vert1^2-1\right\vert-6 \cr -6 &=& -6 \end{array}

Ergebnis: Für x_2=0 und x_3=1 sich wahre Aussagen: P_2\left(0 \mid -6\right) und P_3\left(1 \mid -6\right) sind also tatsächlich Punkte des Graphen.

 

3. Aufgabe

Im ersten Koordinatensystem ist die Funktion f(x)=x^4-2x^2+1 zu sehen, also eine klassische Polynomfunktion. Um genau zu sein, handelt es sich hierbei um ein biquadratisches Polynom. Die Funktionswerte im Intervall ]-1; 1[ sind hier auch ohne Betrag positiv, was man z. B. sehen kann, wenn man f(0)=0^4+2\cdot0^2+1=1 berechnet.

Im zweiten Koordinatensystem ist die Funktion f(x)=\left| x^4-1 \right| dargestellt. Im Gegensatz zu Polynomfunktionen können Betragsfunktionen "Knicke" enthalten, nämlich immer dort, wo die Funktionswerte, wenn man die Funktion ohne den Betrag betrachtet, das Vorzeichen wechseln. Die Funktionswerte im Intervall ]-1; 1[ sind hier nur deswegen positiv, weil der Betrag auf x^4-1 angewendet wird, sonst wären sie negativ. Das sieht man z. B., wenn man 0^4-1=-1 berechnet. Ohne den Betrag wäre der Graph von f(x) eine ganz normale nach oben geöffnete Parabel mit einem Tiefpunkt bei (0\mid -1).

19. Summen- und Produktzeichen - Lernziele und typische Fehler

Nach Durcharbeiten dieses Kapitels sollten Sie folgende Lernziele erreicht haben:

  • Sie wissen, warum das Summen-/Produktzeichen eine nützliche Art der Notation ist, und kennen seine Bestandteile.
  • Sie können eine Summe, die mithilfe des Summenzeichens geschrieben ist, als "normale" Summe schreiben und berechnen.
  • Für Profis: Sie können eine "regelmäßige" Summe mithilfe des Summenzeichens schreiben.
  • Sie können ein Produkt, das mithilfe des Produktzeichens geschrieben ist, als "normales" Produkt schreiben und berechnen.
  • Für Profis: Sie können ein "regelmäßiges" Produkt mithilfe des Produktzeichens schreiben.


In diesem Kapitel sind bislang keine typischen Fehler aufgefallen.


Für weitere Informationen zur Mathematikunterstützung an der TH Wildau nutzen Sie bitte den Moodle-Kursraum "SOS Mathematik - Brückenkurs".

Übersicht:

 

19.1 Summen- und Produktzeichen - Aufgaben

1. Aufgabe

Lösen Sie die Summenzeichen auf!

1) \sum\limits_{i=1}^4 (2+i)x^i

2) \sum\limits_{i=2}^6 4 \cdot i^{-1}

3) \sum\limits_{i=394}^{399} x^2

4) \sum\limits_{j=1}^3 (5j+1)

5) \sum\limits_{i=0}^4 (-1)^{i+1}(i-2)x

 

2. Aufgabe

Für Profis: Schreiben Sie mithilfe eines Summenzeichens!

1) 7+7x+7x^2+7x^3+7x^4

2) -z-2z^2-3z^3-4z^4-5z^5-6z^6

3) 2x+4x^2+8x^3+16x^4+32x^5

4) -9+18-27+36-45+54-63

5) 1+4+9+16+25+36+49+64+81+100

 

3. Aufgabe

Lösen Sie folgende Produktzeichen auf!

1) \prod\limits_{i=1}^{10}i

2) \prod\limits_{i=2}^5(i+1)x

3) \prod\limits_{k=1}^6(2k-7)

4) \prod\limits_{i=0}^5x^i

5) \prod\limits_{j=0}^{13}5j^2x^{j-7}

Dieses Kapitel enthält die folgenden Themen:

 

19.2 Summen- und Produktzeichen - Erklärungen

Dass sich Mathematiker und Mathematikerinnen gerne abkürzende Schreibweisen "ausdenken", haben Sie in diesem Lernmodul ja schon kennengelernt, z. B. bei den Potenzen. Solche Schreibweisen haben den Nachteil, dass man explizit lernen muss, sie zu durchschauen. Sie haben den Vorteil, dass der Schreibaufwand geringer wird und die zugrunde liegende Struktur deutlicher sichtbar wird.
Hier kommen die nächsten zwei Abkürzungen ...

 

Das Summenzeichen

Das Summenzeichen wird häufig (vor allem in der Statistik und der Wirtschaftsmathematik) verwendet, um eine Summe mit mehreren Summanden, die eine gemeinsame Struktur haben, zusammengefasst aufzuschreiben. Als Formelzeichen wird der griechische Buchstabe \Sigma (ein großes "Sigma") verwendet. Wenn man Summen in dieser Form schreibt, hat das den Vorteil, dass man auf einen Blick erkennen kann, welche "Bauanleitung" die Summanden haben. Das wird später bei Folgen und Reihen sehr hilfreich sein.

In einer Formel: \sum\limits_{i=u}^o x_i (gesprochen: "Summe von i gleich u bis o über x i")
Dabei ist

  • i: der Summationsindex oder einfach Index
  • u: die untere Summationsgrenze oder einfach untere Grenze, eine ganze Zahl, oft 0 oder 1
  • o: die obere Summationsgrenze oder einfach obere Grenze, eine ganze Zahl, mindestens so groß wie die untere Summationsgrenze
  • x_i: der Summand

Bemerkung: Der Summationsindex muss nicht i heißen. Üblich sind neben i auch j oder k.


Anhand des folgenden Beispiels soll gezeigt werden, wie beim Auflösen eines Summenzeichens vorgegangen wird: \sum\limits_{i=0}^5 (2+i)x

  1. Setze den Summationsindex i gleich der unteren Summationsgrenze u: i=0
  2. Setze i=0 in den Summanden ein: (2+0)x
  3. Vergrößere i um 1: i=1
  4. Setze i=1 in den Summanden ein: (2+1)x
  5. Vergrößere i um 1: i=2
  6. Setze i=2 in den Summanden ein: (2+2)x
  7. Vergrößere i um 1: i=3
  8. Setze i=3 in den Summanden ein: (2+3)x
  9. Vergrößere i um 1: i=4
  10. Setze i=4 in den Summanden ein: (2+4)x
  11. Vergrößere i um 1: i=5
  12. Setze i=5 in den Summanden ein: (2+5)x
  13. Brich ab, wenn der Summationsindex i die obere Summationsgrenze o erreicht, hier also wenn  i=5
  14. Addiere alle ermittelten Summanden: \sum\limits_{i=0}^5 (2+i)x = (2+0)x+(2+1)x+(2+2)x+(2+3)x+(2+4)x+(2+5)x
  15. Vereinfache die Summanden: \sum\limits_{i=0}^5 (2+i)x = 2x+3x+4x+5x+6x+7x
  16. Fasse zusammen: \sum\limits_{i=0}^5 (2+i)x = 27x

 

Das Produktzeichen

Das Produktzeichen wird häufig verwendet, um ein Produkt mit mehreren Faktoren, die eine gemeinsame Struktur haben, zusammengefasst aufzuschreiben. Als Formelzeichen wird der griechische Buchstabe \prod (ein großes Pi) verwendet.

In einer Formel: \prod\limits_{i=u}^o x_i (gesprochen: "Produkt von i gleich u bis o über x i")
Dabei ist

  • i: der Multiplikationsindex oder einfach Index
  • u: die untere Multiplikationsgrenze oder einfach untere Grenze, eine ganze Zahl, oft 0 oder 1
  • o: die obere Multiplikationsgrenze oder einfach obere Grenze, eine ganze Zahl, mindestens so groß wie die untere Multiplikationsgrenze
  • x_i: der Faktor


Beim Auflösen eines Produktzeichens wird genauso vorgegangen, wie beim Summenzeichen beschrieben. Einziger Unterschied: Bei Schritt 14 wird nicht addiert, sondern multipliziert.

Übersicht:

 

19.3 Summen- und Produktzeichen - Lösungen

1. Aufgabe

1)
\begin{array}{rcl} \sum\limits_{i=1}^4 (2+i)x^i &=& (2+1)x^1+(2+2)x^2+(2+3)x^3+(2+4)x^4 \cr &=& 3x+4x^2+5x^3+6x^4 \cr &=& x(3+4x+5x^2+6x^3) \end{array}

 
2)
\begin{array}{rcl} \sum\limits_{i=2}^6 4\cdot i^{-1} &=& 4 \cdot 2^{-1} + 4 \cdot 3^{-1} + 4 \cdot 4^{-1} + 4 \cdot 5^{-1} + 4 \cdot 6^{-1} \cr \cr &=& \frac{4}{2} + \frac{4}{3} + \frac{4}{4} + \frac{4}{5} + \frac{4}{6} \cr \cr &=& 2+\frac{4}{3}+1+\frac{4}{5}+\frac{2}{3} \cr \cr &=& 3+\frac{20}{15}+\frac{12}{15}+\frac{10}{15} \cr \cr &=& 3+\frac{20+12+10}{15} \cr \cr &=& 3+\frac{42}{15} \cr \cr &=& \frac{29}{5} \end{array}


3)
\begin{array}{rcl} \sum\limits_{i=394}^{399} x^2 &=& x^2+x^2+x^2+x^2+x^2+x^2 \cr &=& 6x^2 \end{array}

Bemerkung: Der Summand hängt nicht vom Summationsindex i ab. In einem solchen Fall berechnet sich die Summe als das Produkt aus dem Summanden und der Differenz aus oberer und untere Grenze plus 1. In diesem Fall: (399-394+1) \cdot x^2 =6x^2


4)
\begin{array}{rcl} \sum\limits_{j=1}^3 (5j+1) &=& (5 \cdot 1+1)+(5 \cdot 2+1)+(5 \cdot 3+1) \cr &=& 6+11+16 \cr &=& 33 \end{array} 


5)
\begin{array}{rcl} \sum\limits_{i=0}^4 (-1)^{i+1}(i-2)x &=& (-1)^{0+1}(0-2)x+(-1)^{1+1}(1-2)x+(-1)^{2+1}(2-2)x+(-1)^{3+1}(3-2)x+(-1)^{4+1}(4-2)x \cr &=& (-1)^1 \cdot (-2)x+(-1)^2 \cdot (-1)x+(-1)^3 \cdot 0x+(-1)^4 \cdot 1x+(-1)^5 \cdot 2x \cr &=& -1 \cdot (-2)x+1 \cdot (-1)x+0+1 \cdot 1x+(-1) \cdot 2x \cr &=& 2x-1x+1x-2x \cr &=& 0 \end{array}

 

2. Aufgabe

Bemerkung: Die hier gezeigten Ergebnisse sind nicht die einzig möglichen Lösungen. Setzt man z. B. die untere Grenze nicht gleich 1 sondern gleich 0, ergeben sich leichte Änderungen. Einige mögliche andere Ergebnisse sind bei Aufgabe 1 aufgeschrieben.

1)
7+7x+7x^2+7x^3+7x^4 = \sum\limits_{i=0}^4 7x^i
oder
7+7x+7x^2+7x^3+7x^4 = \sum\limits_{i=-1}^3 7x^{i+1}
oder
7+7x+7x^2+7x^3+7x^4 = \sum\limits_{i=5}^9 7x^{i-5}
oder ...


2)
-z-2z^2-3z^3-4z^4-5z^5-6z^6 = \sum\limits_{i=1}^6 -iz^i
oder ...


3)
2x+4x^2+8x^3+16x^4+32x^5 = (2x)^1+(2x)^2+(2x)^3+(2x)^4+(2x)^5 = \sum\limits_{i=1}^5 (2x)^i
oder ...


4)
{-9+18-27+36-45+54-63 = (-1)^1 \cdot 9 \cdot 1+(-1)^2 \cdot 9 \cdot 2+(-1)^3 \cdot 9 \cdot 3+(-1)^4 \cdot 9 \cdot 4+(-1)^5 \cdot 9 \cdot 5+(-1)^6 \cdot 9 \cdot 6+(-1)^7 \cdot 9 \cdot 7 = \sum\limits_{i=1}^7 (-1)^i \cdot 9i}
oder ...


5)
{1+4+9+16+25+36+49+64+81+100 = 1^2+2^2+3^2+4^2+5^2+6^2+7^2+8^2+9^2+10^2 = \sum\limits_{i=1}^{10} i^2}
oder ...

 

3. Aufgabe

1)
\begin{array}{rcl} \prod\limits_{i=1}^{10}i &=& 1 \cdot 2 \cdot 3 \cdot 4 \cdot 5 \cdot 6 \cdot 7 \cdot 8 \cdot 9 \cdot 10 \cr &=& 3.628.800 \end{array}

Bemerkung: Ein Produkt der Art: 1 \cdot 2 \cdot 3 \cdot \ldots \cdot n ist auch unter dem Namen Fakultät (Formelzeichen n!) bekannt.

 
2)
\begin{array}{rcl} \prod\limits_{i=2}^5(i+1)x &=& (2+1)x \cdot (3+1)x \cdot (4+1)x \cdot (5+1)x \cr &=& 3x \cdot 4x \cdot 5x \cdot 6x \cr &=& 360x^4 \end{array}

 
3)
\begin{array}{rcl} \prod\limits_{k=1}^6(2k-7) &=& (2 \cdot 1-7)(2 \cdot 2-7)(2 \cdot 3-7)(2 \cdot 4-7)(2 \cdot 5-7)(2 \cdot 6-7) \cr &=& (-5) \cdot (-3) \cdot (-1) \cdot 1 \cdot 3 \cdot 5 \cr &=& -225 \end{array}


4)
\begin{array}{rcl} \prod\limits_{i=0}^5x^i &=& x^0 \cdot x^1 \cdot x^2 \cdot x^3 \cdot x^4 \cdot x^5 \cr &=& x^{0+1+2+3+4+5} \cr &=& x^{15} \end{array}


5)
\begin{array}{rcl} \prod\limits_{j=0}^{13}5j^2x^{j-7} &=& 5 \cdot 0^2 \cdot x^{0-7} \cdot \dots \cr &=& 0 \end{array}

Bemerkung: Hier sieht man schon zu Beginn der Rechnung, dass dieses Produkt insgesamt nur 0 sein kann, unabhängig davon, welche Faktoren noch hinzukommen (siehe Satz vom Nullprodukt). Es ist also unnötig, die Rechnung weiter fortzuführen.

20. Geometrie - Lernziele und typische Fehler

Nach Durcharbeiten dieses Kapitels sollten Sie folgende Lernziele erreicht haben:

  • Sie können verschiedene Arten von Dreiecken unterscheiden.
  • Sie können die verschiedenen Seiten im Dreieck richtig benennen.
  • Sie kennen die Voraussetzung, um die Satzgruppe des Pythagoras anwenden zu dürfen.
  • Sie können Seitenlängen im rechtwinkligen Dreieck mithilfe der Satzgruppe des Pythagoras berechnen.
  • Sie wissen, was das Bogenmaß ist, und können Winkelgrößen vom Grad- ins Bogenmaß sowie vom Bogen- ins Gradmaß umrechnen.
  • Sie wissen, wie groß die Innenwinkelsumme in Drei- und Vierecken ist.
  • Sie kennen die Eigenschaften verschiedener ebener Figuren.
  • Sie können Flächen und Umfänge von ebenen Figuren berechnen.
  • Sie kennen die Eigenschaften verschiedener dreidimensionaler Körper.
  • Sie können Volumina und Oberflächen von dreidimensionalen Körpern berechnen.


Typische Fehler in diesem Kapitel sind:

  • Es wird keine Skizze angefertigt. Erklärung
  • Das Bogenmaß wird nicht so gut verinnerlicht, dass damit sicher umgegangen werden kann. Erklärung
  • Die Umrechnung von Grad- in Bogenmaß (und umgekehrt) wird fehlerhaft durchgeführt. Erklärung


Für Online-Selbsttests zu diesem Thema und weitere Informationen zur Mathematikunterstützung an der TH Wildau nutzen Sie bitte den Moodle-Kursraum "SOS Mathematik - Brückenkurs".

 

Dieses Kapitel wird gerade überarbeitet.

Übersicht:

 

20.1 Geometrie - Aufgaben

1. Bemerkung vorab: Runden Sie bei den folgenden Aufgaben – wenn nötig – auf zwei Stellen nach dem Komma. Wichtig ist, immer erst ganz zum Schluss zu runden, damit die Rundungsfehler nicht zu groß werden!

2. Bemerkung vorab: Sofern in einer Aufgabe nicht anders vermerkt ist, beziehen sich die Punkt-, Längen- und Winkelbezeichnungen auf die Grafiken, die der Erklärungsseite dieses Kapitels hier bzw. hier zu finden sind.

 

1. Aufgabe

Berechnen Sie von folgenden Figuren Umfang und Flächeninhalt, soweit dies möglich ist!

1) Quadrat mit der Seitenlänge  a=69 \, mm


2) Rechteck mit der Seitenlänge  a=230 \, cm und der Diagonalen  e=5 \, m


3) Raute mit der Seitenlänge  c=4{,}5 \, cm


4) Kreis mit dem Radius r=2{,}5 \, cm


5) Parallelogramm mit der Seitenlänge a=33 \, cm und der Höhe h_a=12 \, cm


6) Rechtwinkliges Dreieck mit der Kathete a=70 \, mm und der Kathete b=4 \, cm


7) Kreis mit dem Durchmesser d=7{,}5 \, cm


8) Trapez mit der Höhe h=4 \, cm, den Seitenlängen a=12 \, cm und  c=5 \, cm


9) Rechtwinkliges Dreieck mit Höhe h=5 \, m, Kathete a=13 \, m und Hypotenusenabschnitt q=\dfrac{25}{12} \, m


10) Quadrat mit der Diagonalen e=15{,}4 \, m


11) Symmetrisches Trapez mit den Seitenlängen a=25 \, cm, b=11 \, cm und c=18 \, cm sowie der Höhe h=9{,}5 \, cm


12) Raute mit der Seitenlänge a=8 \, cm und der Diagonalen e=13 \, cm

 

2. Aufgabe

Berechnen Sie jeweils Umfang und Flächeninhalt des Drachenvierecks!

Drachenviereck mit zusätzlicher Beschriftung

1) a=7{,}8 \,cm, b=16 \, cm, e=22{,}9 \, cm und f=6 \, cm


2) a=2 \,cm, e=7 \, cm und p=1 \, cm


3) f=24 \,cm, p=5 \, cm und q=16 \, cm

 

3. Aufgabe

Berechnen Sie jeweils die gesuchten Größen!

1) Umfang U und Seitenlänge b eines Rechtecks mit der Seitenlänge a=3 \, cm und dem Flächeninhalt A=27 \, cm^2


2) Flächeninhalt A und Seitenlänge a eines Parallelogramms mit der Seitenlänge b=5 \, cm, der Höhe h_a=3 \, cm und dem Umfang U=24 \,cm


3) Umfang U und Radius r eines Kreises mit dem Flächeninhalt A=37 \, cm^2

 

4. Aufgabe

1) In einem Kreis K_1 sei der Radius 10 \, m lang. Wie groß ist der Radius eines Kreises mit doppeltem Flächeninhalt?


2) Ein Quadrat Q_1 habe einen Flächeninhalt von 16 \, cm^2. Wie lang sind die Seiten eines Quadrates mit halbem Flächeninhalt?

 

5. Aufgabe

Berechnen Sie aus den gegebenen Längen die fehlenden Dreiecksseiten, einschließlich der Hypotenusenabschnitte p, q und der Höhe h!

1) Gegeben: Kathete a=3 \, cm und Kathete b=4 \, cm


2) Gegeben: Hypotenusenabschnitt p=5 \, cm und Höhe h=5 \, cm


3) Gegeben: Kathete a=3{,}4\, cm und Hypotenuse c=6{,}1\, cm


4) Gegeben: Kathete b=5{,}2\, cm und Hypotenusenabschnitt q=1{,}2\, cm


5) Gegeben: Hypotenuse c=7 \, cm und Hypotenusenabschnitt p=3{,}9\, cm


6) Gegeben: Kathete a=2{,}8\,mm, Hypotenusenabschnitt p=1\,mm


7) Gegeben: Kathete a=16\,cm, Kathete b=90\,mm


8) Gegeben: Höhe h=4{,}03\,m, Hypotenusenabschnitt q=507\,cm


9) Gegeben: Kathete a=2{,}7183\,km, Hypotenuse c=3{,}1416\,km


10) Gegeben: Hypotenuse c=5{,}34\,cm, Hypotenusenabschnitt q=0{,}2\,cm


11) Gegeben: Hypotenusenabschnitt q=60{,}3\,cm, Kathete b=603\,cm


12) Gegeben: Hypotenusenabschnitte p=0{,}3\,cm und q=0{,}5\,cm


13) Gegeben: Kathete b=50\,cm, Kathete a=0{,}5\,m


14) Gegeben: Hypotenuse c=753\,cm, Kathete b=456\,cm


15) Gegeben: Hypotenusenabschnitt q=5\,m, Höhe h=20\,m

 

6. Aufgabe

Wie groß ist jeweils der Abstand d der gegebenen Punkte P_1 und P_2?

1) P_1 \left(1\mid 2 \right) und P_2 \left(3\mid 3 \right)   6) P_1 \left(-0{,}5\mid 7{,}75 \right) und P_2 \left(\dfrac{1}{2}\mid -\dfrac{31}{4} \right)
2) P_1 \left(\dfrac{1}{3} \mid 2 \right) und P_2 \left(-\dfrac{2}{3}\mid 7 \right)   7) P_1 \left(0\mid 0 \right) und P_2 \left(3\mid 4 \right)
3) P_1 \left(25\mid -50 \right) und P_2 \left(-\dfrac{1}{10}\mid 2 \right)   8) P_1 \left(\dfrac{7}{8}\mid -\dfrac{9}{4} \right) und P_2 \left(\dfrac{1}{4}\mid \dfrac{3}{8} \right)
4) P_1 \left(-\dfrac{1}{300}\mid \dfrac{3}{7}\right) und P_2 \left(-\dfrac{11}{600}\mid -\dfrac{8}{21} \right)   9) P_1 \left(-90\mid -23 \right) und P_2 \left(-83\mid -101 \right)
5) P_1 \left(7\mid 5 \right) und P_2 \left(21\mid 22 \right)   10) P_1 \left(333\mid 111 \right) und P_2 \left(33\mid 11 \right)

 

7. Aufgabe

Kann es Dreiecke bzw. Vierecke geben, die die angegebenen Winkel enthalten?

Dreiecke

  Vierecke
1) \alpha = \dfrac{ \pi}{2} und \beta = 100^\circ   11) \alpha = \dfrac{ 5\pi}{6}, \beta = 30^\circ und \gamma = 60^\circ

2) \alpha = 100^\circ und  \gamma = \dfrac{ \pi}{4}   12) \alpha = 45^\circ und \beta = \dfrac{3\pi}{2} und \gamma = \dfrac{\pi}{4}

3) \alpha = 120^\circ und \beta= \dfrac{ \pi}{6}   13) \alpha = \dfrac{\pi}{9} und \beta = 180^\circ und \gamma = 10^\circ
4) \beta = \dfrac{3 \pi}{5} und \gamma = 80^\circ

  14) \alpha = \dfrac{9\pi}{8} und \beta = 30^\circ und \gamma = \dfrac{2\pi}{5}
5) \beta=\pi und \gamma = 18^\circ   15) \alpha = \dfrac{11\pi}{18} und \beta = 70^\circ und \gamma = \dfrac{17\pi}{18}
6) \alpha=\dfrac{\pi}{10} und \beta=145^\circ   16) \alpha = \dfrac{ 2\pi}{3} und \beta = 30^\circ und \gamma = \pi
7) \gamma =\dfrac{5\pi}{4} und \beta=10^\circ   17) \alpha = 32^\circ und \beta = \dfrac{6\pi}{5} und \gamma = \dfrac{23\pi}{36}
8) \alpha = 30^\circ und \beta=\dfrac{\pi}{3}   18) \alpha = 5^\circ und \beta = 5^\circ und \gamma = 0{,}1\pi
9) \alpha=\dfrac{3\pi}{8} und \beta=45^\circ   19) \alpha = \dfrac{ \pi}{2} und \beta = 120^\circ und \gamma = 180^\circ
10) \alpha = 1{,}8^\circ und \gamma=\dfrac{99}{100}\pi   20) \alpha = \dfrac{ 5\pi}{4} und \beta = 90^\circ und \gamma = \dfrac{5\pi}{12}

 

8. Aufgabe

Familie Eckstein möchte eine rechteckige Steinterrasse bauen, wofür sie 1000\,\text{EUR} zur Seite gelegt hat. Für die Anfahrt des Steinsetzers muss sie einmalig 250\,\text{EUR} zahlen. Zusätzlich zahlt sie für jeden angefangenen Quadratmeter 60\,\text{EUR}. Eine Seitenlänge der Terrasse ist schon durch die Hauswand vorgegeben: 5\, m.

Wie groß kann die Terrasse maximal werden? Geben Sie die zweite Seitenlänge an!

 

9. Aufgabe

Berechnen Sie von folgenden Körpern Ober- und Mantelfläche sowie Volumen, soweit dies möglich ist!

1) Würfel mit der Seitenlänge a=16 \, cm


2) Quader mit den Seitenlängen a =35 \, mm, b=5{,}5 \, cm und c=11 \, cm


3) Zylinder mit der Höhe h=10 \, m und dem Radius r=7{,}5 \, m


4) Regelmäßige quadratische Pyramide mit der Höhe h=5 \, m und der Seitenlänge der Grundfläche a=7 \, m


5) Kegel mit der Höhe h=31 \, cm und dem Radius r=14 \, cm


6) Kugel mit dem Durchmesser d=6{,}7 \, cm

 

10. Aufgabe

1) Die Diagonale eines Quadrats ist 5{,}657 \, cm lang. Wie lang sind die Seiten des Vierecks?


2) Die Raumdiagonale eines Würfels ist 17{,}321 \, cm lang. Wie lang sind die Kanten des Würfels?


3) Die Oberfläche einer Kugel ist 615{,}75 \, cm^2 groß. Wie groß ist das Volumen der Kugel?

 

11. Aufgabe

1) In einem Quadrat mit der Seitenlänge a ist ein Kreis mit maximalem Durchmesser eingezeichnet. Das heißt, der Kreis berührt die Quadratseiten von innen jeweils in der Mitte.
Wie ist das Verhältnis der Fläche des Kreises zur Fläche des Quadrates?


2) Zusätzlich ist um das Quadrat herum ein Kreis mit minimalem Durchmesser gezeichnet. Das heißt, das Quadrat berührt die Kreislinie von innen mit allen vier Ecken.
Wie ist das Verhältnis der Fläche dieses Kreises zur Fläche des Quadrates?


3) Formulieren Sie entsprechende Vergleiche im Raum. Anstelle von Quadraten sind damit Würfel zu betrachten, den Kreisen entsprechen Kugeln.
Wie ist das Verhältnis der entsprechenden Volumina?

 

12. Aufgabe

Das Dosenunternehmen "Dosen10" möchte weniger Material für seine 0{,}25\, l Dosen verwenden und dafür ggf. die Form ändern. Grundsätzlich wurden vier Ideen abgesegnet:

a) Ein Kreiszylinder mit einem Durchmesser von 7{,}5 \, cm
b) Ein Quader, wobei eine Seite 4 \, cm breit sein soll und eine andere doppelt so lang wie diese
c) Ein Würfel
d) Eine Kugel

Welche Alternative ist optimal? Warum hat sich die rechnerisch optimale Lösung in der Praxis nicht durchgesetzt?

 

13. Aufgabe

Die Firma „Rohrfertigung nach Maß“ hat den Auftrag, Abwasserrohre herzustellen. Dabei soll der Gesamtdurchmesser des Rohres 200 \, mm sein, wobei der innere Durchmesser 150 \, mm ist. Die Länge des Rohres soll 15 \, m betragen.

a) Wie viel Abwasser kann das Rohr fassen?
b) Wie groß ist die Mantelfläche des Rohres?

 

14. Aufgabe

In einem Freizeitbad wurde ein neues Schwimmbecken gebaut mit Abmessungen, wie in der Skizze (Seitenansicht) dargestellt. Wie viele Liter Wasser werden benötigt, um den Pool bis 10\, cm unter den Rand zu füllen bei einem 15\, m breiten Becken?

Dieses Kapitel enthält die folgenden Themen:

 

20.2 Geometrie - Erklärungen

Die Geometrie ist ein ziemlich alter Teil der Mathematik. Schon die "alten Griechen" beschäftigten sich damit - was man bis heute an dem Begriff "euklidische Geometrie" merkt. Euklid war ein griechischer Mathematiker, der vermutlich im dritten Jahrhundert vor Christus in Alexandria lebte und die Mathematik bis in die Neuzeit hinein, u. a. durch seine Lehrbücher, entscheidend geprägt hat.
Übersetzt bedeutet Geometrie so viel wie "Vermessung der Erde". Dazu passt, dass ein wichtiger Bestandteil der Geometrie der Umgang mit Figuren in der Ebene und Körpern im Raum ist.

 

Ebene Figuren

Zunächst schauen wir uns die wichtigsten ebenen Figuren an und klären, mit welchen Formeln jeweils Umfang und Flächeninhalt berechnet werden können.
Dazu folgende dringende Empfehlung: Bei geometrischen Aufgaben ist immer sinnvoll, eine Skizze mit Beschriftungen anzufertigen. Das hat den Vorteil, dass man sich dadurch plastisch vor Augen führt,

  • wie das gesuchte Objekt aussieht und ob die Vorstellung, die man sich "vor dem geistigen Auge" gemacht hat, mit den Angaben aus der Aufgabenstellung übereinstimmt.
    Bei einem Quadrat mag es noch ganz gut ohne Skizze gehen. Bei einem Quader mit einem aufgesetzten regelmäßigen, quadratischen Pyramidenstumpf sieht das schon anders aus ...
  • wie die Abmessungen des Objekts sind.
    Vielleicht stellt man dadurch ja fest, dass ein Dreieck gleichseitig ist ...
  • welche Hilfslinien ggf. gebraucht werden, um das zu berechnen/maximieren/minimieren/..., was berechnet/maximiert/minimiert/... werden soll.
    Das kann die Höhe des Quaders mit aufgesetztem Pyramidenstumpf sein, die mit der Diagonalen der Bodenfläche ein rechtwinkliges Dreieck bildet. Damit weiß man auch gleich, wie man weiterrechnen soll, nämlich mit dem Satz des Pythagoras.

Mit sehr viel Übung kann man es natürlich schaffen, sich geometrische Objekte mit allen Rahmenbedingungen so gut zu vorstellen, dass das zum Lösen der Aufgabe reicht. Wer (noch) nicht so weit ist, und das sind wohl die meisten, sollte skizzieren - immer. Eine simple Skizze kann extrem hilfreich sein und dadurch entscheidend zur richtigen Lösung der Aufgabe beitragen!
Und: Damit die Skizze ihren Nutzen erfüllen kann, muss sie so groß und ordentlich sein, dass alle relevanten Details eingetragen und gut wieder abgelesen werden können.

 

Vorab eine Vokabel: Sagt man, dass zwei Seiten, Diagonalen etc. "senkrecht zueinander" stehen, bedeutet dies, dass sie einen rechten Winkel einschließen.

 

Eigenschaften der Figuren   Formeln

Ein Rechteck ist ein Viereck mit folgenden Eigenschaften:

  • Gegenüberliegende Seiten sind gleich lang und parallel.
  • Alle Innenwinkel sind 90^\circ groß.
  • Die Diagonalen sind gleich lang und halbieren einander.
Flächeninhalt: A = ab
Umfang: U = 2a+2b

Ein Quadrat ist ein spezielles Rechteck mit folgenden Eigenschaften:

  • Alle Seiten sind gleich lang.
  • Die Diagonalen stehen zusätzlich senkrecht aufeinander.
Flächeninhalt: A = a^2
Umfang: U = 4a

Ein Parallelogramm ist ein Viereck mit folgenden Eigenschaften:

  • Gegenüberliegende Seiten sind gleich lang und parallel.
  • Gegenüberliegende Winkel sind gleich groß.
  • Die Diagonalen halbieren einander, sind aber unterschiedlich lang.
Flächeninhalt: A = ah_a = ab \sin\left(\alpha\right)
Umfang: U = 2a+2b

Ein/e Raute/Rhombus ist ein Viereck mit folgenden Eigenschaften:

  • Alle Seiten sind gleich lang.
  • Gegenüberliegende Seiten sind parallel.
  • Gegenüberliegende Winkel sind gleich groß.
  • Die Diagonalen stehen senkrecht aufeinander und halbieren einander.
Flächeninhalt: A = \dfrac{1}{2} ef
Umfang: U = 4a

Ein Drachen/Drachenviereck ist ein Viereck mit folgenden Eigenschaften:

  • Mindestens zwei gegenüberliegende Winkel sind gleich groß.
  • Die Diagonalen stehen senkrecht aufeinander.
Flächeninhalt: A = \dfrac{1}{2} ef
Umfang: U = 2a+2b

Ein Trapez ist ein Viereck mit folgender Eigenschaft:

  • Mindestens zwei Seiten sind parallel.
Flächeninhalt: A = \dfrac{1}{2}(a+c)h
Umfang: U =a+b+c+d

Ein gleichschenkliges Dreieck ist ein Dreieck mit folgender Eigenschaft:

  • Zwei der Seiten sind gleich lang.
gleichschenkliges Dreieck Flächeninhalt: A = \dfrac{1}{2}ch_c
Umfang: U = 2a+c

Ein gleichseitiges Dreieck ist ein spezielles gleichschenkliges Dreieck mit der folgenden Eigenschaft:

  • Alle drei Seiten sind gleichlang.
gleichseitiges Dreieck Flächeninhalt: A = \dfrac{1}{2}ch_c
Umfang: U = 3a

Ein rechtwinkliges Dreieck ist ein Dreieck mit folgender Eigenschaft:

  • Einer der Winkel ist genau 90^\circ groß.
Flächeninhalt: A = \dfrac{1}{2}ab
Umfang: U = a+b+c

Ein Kreis hat folgende Eigenschaft:

  • Alle Punkte der Umfangslinie haben denselben Abstand zum Mittelpunkt M. Dieser Abstand heißt Radius r.


Der doppelte Radius heißt Durchmesser und wird mit d bezeichnet.

Flächeninhalt: A = \pi r^2 = \dfrac{1}{4} \pi d^2
Umfang: U = 2 \pi r = \pi d

 

Das rechtwinklige Dreieck

Eine besonders wichtige geometrische Figur ist das rechtwinklige Dreieck. Daher schauen wir es uns im folgenden Abschnitt etwas genauer an. Zunächst mal grafisch:

Die verschiedenen Seiten und Linien, die in der Zeichnung mit Buchstaben benannt sind, haben bestimmte Namen. Das macht es einfacher, sich über die Gegebenheiten im Dreieck zu verständigen:

Katheten: die Seiten, die an den rechten Winkel angrenzen, hier a und b

Hypotenuse: die Seite, die dem rechten Winkel gegenüberliegt, hier c

Höhe zur Seite c: die Strecke, die senkrecht auf der Dreiecksseite c steht und durch die gegenüberliegende Ecke verläuft, hier h
Es gibt natürlich auch die Höhen zu den Seiten a und b.

Hypotenusenabschnitte: die Teilstrecken, die durch den Schnittpunkt der Höhe mit der Hypotenuse entstehen, hier p und q

Wichtig sind diese Bezeichnungen auch deshalb, weil ja nicht gesagt ist, dass die Seiten links und rechts vom rechten Winkel immer a und b heißen. Schauen Sie sich in der Zeichnung das Dreieck mit den Seiten b, h und q an: Es ist offensichtlich rechtwinklig, weil die Höhe ja senkrecht zur Hypotenuse stehen muss. Aber die beiden Seiten, die an den rechten Winkel angrenzen, heißen hier h und q.
Das ist vor allem auch für den nächsten Abschnitt entscheidend, denn der Satz des Pythagoras gilt natürlich auch dann, wenn die Dreieckseiten nicht a, b und c heißen ...

 

Die Satzgruppe des Pythagoras

Kommen wir zu einem der berühmtesten Sätze der Mathematik ... Seine Bekanntheit liegt nicht nur an seiner einfachen und gut merkbaren Form, sondern auch an seiner Geschichte. Bereits ab etwa 1800 vor Christus wurde der Zusammenhang in China und Babylon genutzt. Die "alten Ägypter" wendeten der Satzes des Pythagoras "rückwärts" an: Sie nutzen Dreiecke mit festgelegten Seitenlängen, um Felder mit rechten Winkeln anzulegen, da durch die Nilfluten regelmäßig die vorhandenen Abgrenzungen zerstört wurden.


In ebenen rechtwinkligen Dreiecken gelten die folgenden Beziehungen:

Der Satz des Pythagoras

a^2+b^2 = c^2

Als Satz formuliert: In ebenen rechtwinkligen Dreiecken entspricht die Summe der Flächeninhalte der Quadrate über den Katheten dem Flächeninhalt des Quadrats über der Hypotenuse.

Der Satz des Pythagoras gilt natürlich auch in den beiden Teildreiecken, die sich in der Skizze ergeben, da die Höhe und die Hypotenuse einen rechten Winkel einschließen. Hierbei liegen die Seiten a bzw. b (vormals die Katheten) jeweils gegenüber des rechten Winkels und sind nun also die Hypotenusen.
Es gilt also:
h^2+q^2 = b^2
h^2+p^2 = a^2
Satz des Pythagoras

Der Höhensatz

h^2 = p \cdot q

Als Satz formuliert: In ebenen rechtwinkligen Dreiecken entspricht der Flächeninhalt des Quadrats über der Höhe dem Flächeninhalt des Rechtecks, dessen Seiten die Hypotenusenabschnitte sind.
Höhensatz

Die Kathetensätze

a^2 = c \cdot p
b^2 = c \cdot q

Als Satz formuliert: In ebenen rechtwinkligen Dreiecken entspricht der Flächeninhalt des Quadrats über einer Kathete dem Flächeninhalt des Rechtecks, dessen Seiten die Hypotenuse und der zur Kathete gehörende Hypotenusenabschnitt sind.
Kathetensatz

 

Ganz wichtig: Die Formel a^2+b^2= c^2 alleine ist nicht der Satz des Pythagoras! Durch die Formel ohne zusätzliche Informationen ist nämlich nicht festgelegt, welche Größen a, b und c bezeichnen (a und b müssen ja nicht automatisch die Katheten sein ...) bzw. ob es sich bei a, b und c überhaupt um Seiten eines rechtwinkligen Dreiecks handelt. Für irgendwelche beliebigen a, b und c wäre es denn auch eher überraschend, wenn sie die Beziehung a^2+b^2= c^2 erfüllen würden. Warum sollten sie?
Genauso ist es natürlich auch beim Höhensatz und bei den Kathetensätzen.

Bemerkung: Bei konkreten Berechnungen müssen diese Formeln, die ja letztlich quadratische Gleichungen sind, teilweise umgeformt werden, z. B. um q aus dem Höhensatz berechnen zu können.

 

Winkel im Grad- und Bogenmaß

Bislang haben wir uns nur um die Strecken in ebenen Figuren gekümmert. Winkel sind aber mindestens ebenso wichtig, wenn man sich mit geometrischen Objekten beschäftigt (spätenstens bei der Trigonometrie im nächsten Kapitel ...). Typischerweise werden Winkel mit griechischen Kleinbuchstaben bezeichnet, am liebsten mit \alpha (gesprochen: "alpha"), \beta (gesprochen: "beta"), \gamma (gesprochen: "gamma"), \delta (gesprochen: "delta"), \varepsilon (gesprochen: "epsilon") und \varphi (gesprochen: "phi"). Grundsätzlich steht es einem aber frei, auch andere Bezeichnungen zu wählen - das kann allerdings zu Verwirrung führen.

Die Größe von Winkeln kann entweder im Gradmaß oder im Bogenmaß angegeben werden. Das ist ein bisschen so, wie die Länge von Strecken in Kilometern oder Meilen angegeben werden kann. Bei Winkeln wie bei Strecken haben wir also zwei gültige und sinnvolle Maße, und in beiden Fällen muss man wissen, wie man zwischen den Maßen hin- und herrechnet.


Das Gradmaß ist vermutlich den meisten ziemlich vertraut, schon deshalb, weil es eine entsprechende Skala auf jedem Geodreieck gibt. Daher nur kurz und knapp die wichtigsten Fakten:

  • Wird ein Winkel im Gradmaß angegeben, wird dies durch die Einheit \circ gekennzeichnet.
  • Ein Vollkreis hat 360^\circ.
  • Ein rechter Winkel hat 90^\circ.
  • Auf dem Taschenrechner wird das Gradmaß meist durch "D" oder "DEG" für "degree" (englisch für "Grad") gekennzeichnet.


Das Bogenmaß ist weniger verbreitet, hat aber in einigen Bereichen ziemliche Vorteile, dazu unten mehr.

Definition: Das Bogenmaß eines Winkels \alpha am Mittelpunkt des Kreises ist das Verhältnis der Länge des Kreisbogens b zum Radius r: \alpha= \dfrac{b}{r}

Winkel im Kreis im Bogenmaß






Ist eine der beiden Maßzahlen gegeben, kann mithilfe folgender Formel die andere berechnet werden:  \dfrac{ \alpha^\circ} {\alpha} = \dfrac{180^\circ}{ \pi} , wobei \alpha das Bogenmaß des Winkels und \alpha^\circ die Größe des Winkels in Grad bezeichnet.

Alternativ kann man die Winkelgrößen auch mit dem Dreisatz umrechnen, da ein Vollkreis 2 \pi entspricht.

 

Beispiele für die Umrechnung:

1) Rechne \alpha = 105^\circ um ins Bogenmaß!

\begin{array}{rclcl}\dfrac{105^\circ}{\alpha} &=& \dfrac{180^\circ}{\pi} &\vert& \cdot \alpha \cr\cr105^\circ &=& \dfrac{180^\circ}{\pi}\cdot\alpha &\vert& : \dfrac{180^\circ}{\pi} \cr\cr105^\circ\cdot\dfrac{\pi}{180^\circ} &=& \alpha \cr\cr\alpha &=& \dfrac{7}{12}\pi \end{array}


2) Rechne \beta = 225^\circ um ins Bogenmaß!

\begin{array}{rclcr} 180^\circ &\widehat{=}& \pi \cr \cr 1^\circ &\widehat{=}& \dfrac{\pi}{180^\circ} \cr \cr 225^\circ &\widehat{=}& \dfrac{225^\circ}{180^\circ}\cdot\pi \cr\cr \beta &=& \dfrac{5}{4}\pi \end{array}


3) Rechne \gamma = \dfrac{\pi}{4} = 0{,}25\pi um ins Gradmaß!

\begin{array}{rclcl}\dfrac{\gamma^\circ}{0{,}25\pi} &=& \dfrac{180^\circ}{\pi} &\vert& \cdot \dfrac{\pi}{4} \cr\cr\gamma &=& \dfrac{180^\circ}{\pi}\cdot\dfrac{\pi}{4} \cr\cr\gamma &=& \dfrac{180^\circ}{4} \cr\cr\gamma &=& 45^\circ \end{array}

 2) Rechne \varphi = \dfrac{9}{16}\pi um ins Gradmaß!

\begin{array}{rclcr} \pi &\widehat{=}& 180^\circ \cr \cr \dfrac{9}{16}\pi &\widehat{=}& \dfrac{9}{16}\cdot 180^\circ \cr\cr \varphi &=& 101{,}25^\circ \end{array}

 

Hier ein paar Fakten zum Bogenmaß:

  • Winkelgrößen im Bogenmaß sind dimensionslos. Das heißt, sie haben keine Einheit. [Für diejenigen unter Ihnen, die Einheitenbetrachtungen gewöhnt sind: Bei der Berechnung des Bogenmaßes werden zwei Längen durcheinander geteilt: die Länge des Kreisbogens und die Länge des Radius. Da kürzen sich die Einheiten raus.]
  • Ein rechter Winkel hat \frac{\pi}{2}.
  • Meist wird das Bogenmaß als Vielfaches von \pi angegeben, z. B. \frac{2}{3}\pi oder 3\pi, wodurch es einfach ist, die Winkelgröße z. B. mit einem Vollkreis (2\pi) zu vergleichen.
  • Auf dem Taschenrechner wird das Bogenmaß meist durch "R" oder "RAD" für "radiant" (hat mit dem Begriff "Radius" zu tun) gekennzeichnet.

Zu den Vorteilen vom Bogenmaß:

  • Auch wenn es auf den ersten Blick vielleicht nicht so aussieht: Vieles wird übersichtlicher, wenn man das Bogenmaß verwendet.
    Bei Winkelgrößen unter 360^\circ haben die meisten genügend Erfahrung, um Größenordnungen abschätzen zu können, beispielsweise: 270^\circ entspricht einem Dreiviertelkreis. 85^\circ ist knapp ein rechter Winkel.
    Bei Winkelgrößen über 360^\circ wird das schon schwieriger, z. B.
    • 3\pi entspricht eineinhalb Kreisumläufen (2\pi für einen Vollkreis und noch einen halben Kreis dazu). Wenn der Winkel mit 540^\circ angegeben wäre, wäre das nicht so leicht zu erfassen.
    • Je größer die Werte werden, desto deutlich wird das: Es ist nicht schwer herauszufinden, dass 48\pi dem Winkel für 24 Kreisumläufe entsprechen. 24 Kreisumläufe im Gradmaß sind 8.640^\circ. Bei Zahlen in dieser Größenordnung erkennt man nicht mehr einfach so, ob dies eine ganze Kreisumdrehung beschreibt ...
  • Verwendet man Winkel im Bogenmaß, verhalten sich trigonometrische Funktionen beim Ableiten angenehmer, da man sich nicht um eine Einheit kümmern muss.
  • Auch wenn Sinus, Kosinus und Co. ursprünglich den mathematischen Zusammenhang von Seiten und Winkeln an konkreten Dreiecken beschreiben (siehe nächstes Kapitel), ist der Anwendungsbereich von trigonometrischen Funktionen sehr viel größer und auch abstrakter. Eine dimensionslose Variable ist dann deutlich naheliegender, da viele der Anwendungsfälle mit Winkeln rein gar nichts zu tun haben. Beispielsweise werden trigonometrische Funktionen auch benutzt, um die periodische Anteile in wirtschaftlichen Zeitreihendaten zu beschreiben.
  • Wenn man Funktionen analysiert, gehört es quasi dazu, dass die Variable auch mal negative Werte annehmen kann. Das funktioniert besser, wenn man mit einer abstrakten Größe wie dem Bogenmaß rechnet und nicht an der Vorstellung eines Winkels hängt.
  • In der Physik, wenn es um Kreisbewegungen geht, bietet sich das Bogenmaß geradezu an, weil man dort ja sowieso mit Kreisbögen und Radien zu tun hat.

 

Winkelsummen

Für Drei- und Vierecke gelten bestimmte Gesetzmäßigkeiten für die Summen der Winkel, die innerhalb der Figur liegen:

Winkelsummensatz für Dreiecke:
In jedem Dreieck beträgt die Summe der Innenwinkel 180^\circ bzw. \pi.

Winkelsummensatz für Vierecke:
In jedem Viereck beträgt die Summe der Innenwinkel 360^\circ bzw. 2 \pi.

 

Dreidimensionale Körper

Definition: Die Mantelfläche ist die Oberfläche eines Körpers ohne die Grund- und (soweit vorhanden) Deckfläche.

Eigenschaften der Körper   Formeln

Ein Quader ist ein Körper mit folgenden Eigenschaften:

  • Die Grund- und die Deckfläche sind jeweils 1 Rechteck, die parallel und deckungsgleich sind.
  • Die Mantelfläche besteht aus 4 Rechtecken, wobei gegenüberliegende Rechtecke parallel zueinander und deckungsgleich sind.
  • Angrenzende Rechtecke stehen jeweils im rechten Winkel aufeinander.
  • Die 6 Flächen bilden 8 Ecken und 12 Kanten.
Quader Volumen: V = abc
Oberfläche: A_O = 2(ab+ac+bc)
Mantelfläche: A_M = 2(ac+bc)

Ein Würfel ist ein spezieller Quader mit den folgenden Eigenschaften:

  • Die Grund- und die Deckfläche sind jeweils 1 Quadrat.
  • Die Mantelfläche besteht aus 4 Quadraten.
Würfel Volumen: V = a^3
Oberfläche: A_O = 6a^2
Mantelfläche: A_M = 4a^2

Eine regelmäßige quadratische Pyramide ist ein Körper mit den folgenden Eigenschaften:

  • Die Grundfläche ist 1 Quadrat.
  • Es gibt keine Deckfläche, sondern eine Spitze. Jede Ecke des Quadrats ist mit der Spitze der Pyramide verbunden.
  • Die Mantelfläche besteht aus 4 Dreiecken, die gleichschenklig und deckungsgleich sind.
  • Die Spitze der Pyramide liegt genau senkrecht über dem Mittelpunkt der Grundfläche.
  • Die 5 Flächen bilden 5 Ecken (eine davon die Spitze) und 8 Kanten.

Bei einer Dreieckspyramide ist die Grundfläche ein Dreieck.
Bei einer unregelmäßigen Pyramide liegt die Spitze nicht genau senkrecht über dem Mittelpunkt der Grundfläche.
In beiden Fällen gelten die rechts angegebenen Formeln nicht.

 
Pyramide

Volumen: V = \dfrac{1}{3}a^2h
Oberfläche: A_O = a^2+2ah_s
Mantelfläche: A_M = 2ah_s

wobei h_s = \sqrt{h^2+\left(\dfrac{a}{2}\right)^2}

Ein Kegel ist ein Körper mit den folgenden Eigenschaften:

  • Die Grundfläche ist 1 Kreis.
  • Es gibt keine Deckfläche, sondern eine Spitze. Jeder Punkt der Kreislinie ist mit der Spitze des Kegels verbunden.
  • Die Mantelfläche besteht aus 1 Kreissegment.
  • Die Spitze des Kegels liegt genau senkrecht über dem Mittelpunkt der Grundfläche.
  • Die 2 Flächen bilden 1 Ecke (die Spitze) und 1 Kante.

Bei einem schiefen Kegel liegt die Spitze nicht genau senkrecht über dem Mittelpunkt der Grundfläche. Die rechts angegebenen Formeln gelten dann nicht.

Kegel

Volumen: V = \dfrac{1}{3} \pi r^2h
Oberfläche: A_O = \pi r(r+s)
Mantelfläche: A_M = \pi rs

wobei s = \sqrt{r^2+h^2}

Ein Zylinder ist ein Körper mit den folgenden Eigenschaften:

  • Die Grund- und die Deckfläche sind jeweils 1 Kreis, die parallel und deckungsgleich sind.
  • Die Mantelfläche besteht aus 1 Rechteck.
  • Der Mittelpunkt der Deckfläche liegt genau senkrecht über dem Mittelpunkt der Grundfläche.
  • Die 3 Flächen bilden keine Ecken und 2 Kanten.
Zylinder Volumen: V = \pi r^2h
Oberfläche: A_O = 2 \pi r(r+h)
Mantelfläche: A_M = 2 \pi rh

Eine Kugel ist ein Körper mit den folgenden Eigenschaften:

  • Alle Punkte der Oberfläche haben denselben Abstand zum Mittelpunkt. Dieser Abstand heißt Radius r.
  • Die Fläche bildet keine Ecken und keine Kanten.
Kugel Volumen: V = \dfrac{4}{3} \pi r^3
Oberfläche: A_O = 4 \pi r^2

Übersicht:

 

20.3 Geometrie - Lösungen

Erste Bemerkung vorab: Für die folgenden Aufgaben ist eine Skizze der jeweiligen Figur, aus der ersichtlich wird, welche Größe wie benannt ist, notwendig für eine vollständige Lösung (und für den Überblick auch ...). Um die Lösungen hier nicht zu lang werden zu lassen, beziehen sich die Lösungen der Aufgabe 1. bis 4. auf die Skizzen auf der vorherigen Seite.
Zweite Bemerkung vorab: Da die Variablen bei diesen Aufgaben alle für geometrische Objekte (meistens Strecken) stehen, dürfen wir davon ausgehen, dass sie eine messbare Länge haben. Beim Umformen der Formeln (z. B. bei Aufgabe 5.1) können eine Division durch oder eine Multiplikation mit 0 also nicht auftreten.

 

1. Aufgabe

1)

\begin{array}{rcl} U &=& 4a \cr U &=& 4\cdot 69\,mm \cr U &=& 276 \, mm \end{array}   \begin{array}{rcl} A &=& a^2 \cr A &=& (69 \, mm)^2 \cr A &=& 4.761 \, mm^2 \end{array}


2)

\begin{array}{rcl} U &=& 2a+2b \end{array}   \begin{array}{rcl} A &=& a b \end{array}


     Berechne b (Achtung: Einheiten umrechnen!):

     \begin{array}{rcl} b &=& \sqrt{e^2-a^2} \cr b &=& \sqrt{(500 \, cm)^2-(230 \, cm)^2} \cr b &=& \sqrt{197.100} \, cm \end{array}
   

\begin{array}{rcl} U &=& 2\cdot 230 \, cm + 2\cdot \sqrt{197.100} \, cm \cr U &=& 460 \, cm + 60\cdot \sqrt{219} \, cm \cr U &\approx& 1.347{,}92 \, cm \cr U &\approx& 13{,}48 \, m \end{array}
 
\begin{array}{rcl} A &=& 230 \, cm\cdot \sqrt{197.100} \, cm \cr A &\approx& 102.110{,}68 \, cm^2 \cr A &\approx& 10{,}21 \, m^2 \end{array}

Bemerkung: 1 \,m sind 100 \,cm und 1 \,m^2 sind 10.000 \,cm^2.

3)

\begin{array}{rcl} U &=& 4a \cr U &=& 4\cdot 4{,}5 \,cm \cr U &=& 18 \, cm \end{array}   \begin{array}{rcl} A &=& \dfrac{1}{2}ef \end{array}
Der Flächeninhalt lässt sich aus den gegebenen Angaben nicht berechnen.


4)

\begin{array}{rcl} U &=& 2\pi r \cr U &=& 2\pi \cdot 2{,}5 \,cm \cr U &=& 5\pi \,cm \cr U &\approx& 15{,}71 \,cm \end{array}   \begin{array}{rcl} A &=& \pi r^2 \cr A &=& \pi \cdot (2{,}5 \,cm)^2 \cr A &=& 6{,}25\pi \,cm^2 \cr A &\approx& 19{,}63 \,cm^2 \end{array}


5)

\begin{array}{rcl} U &=& 2a+2b \end{array}
Der Umfang lässt sich aus den gegebenen Angaben nicht berechnen.
  \begin{array}{rcl} A &=& a h_a \cr A &=& 33 \, cm\cdot 12 \, cm \cr A &=& 396 \, cm^2 \end{array}


6)

\begin{array}{rcl} U &=& a+b+c \end{array}   \begin{array}{rcl} A &=& \dfrac{1}{2}ab \end{array}


     Berechne c (Achtung: Einheiten umrechnen!):

     \begin{array}{rcl} c &=& \sqrt{a^2+b^2} \cr c &=& \sqrt{(7 \,cm)^2+(4 \, cm)^2} \cr c &=& \sqrt{49 \,cm^2+16 \, cm^2} \cr c &=& \sqrt{65}\, cm \end{array}
   

\begin{array}{rcl} U &=& 7 \, cm+ 4 \, cm+\sqrt{65} \, cm \cr U &\approx& 19{,}06 \, cm \end{array}
 
\begin{array}{rcl} A &=& \dfrac{1}{2}\cdot 7 \, cm \cdot 4 \, cm \cr A &=& 14 \, cm^2 \end{array}


7)

\begin{array}{rcl} U &=& \pi d \cr U &=& \pi \cdot 7{,}5 \,cm \cr U &\approx& 23{,}56 \, cm \end{array}   \begin{array}{rcl} A &=& \dfrac{1}{4}\pi d^2 \cr A &=& \dfrac{1}{4}\pi\cdot (7{,}5 \, cm)^2 \cr A &\approx& 44{,}18 \, cm^2 \end{array}


8)

\begin{array}{rcl} U &=& a+b+c+d \end{array}
Der Umfang lässt sich aus den gegebenen Angaben nicht berechnen.
  \begin{array}{rcl} A &=& \dfrac{1}{2}(a+c) h \cr A &=& \dfrac{1}{2}(12 \, cm+5 \, cm) \cdot 4 \, cm \cr A &=& 34 \, cm^2 \end{array}


9)

\begin{array}{rclcrcl} U &=& a+b+c \end{array}   \begin{array}{rclcrcl} A &=& \dfrac{1}{2}ab \end{array}


     Berechne p, c und b:

     \begin{array}{rcl} p &=& \sqrt{a^2-h^2} \cr p &=& \sqrt{(13 \, m)^2-(5 \, m)^2} \cr p &=& \sqrt{144} \, m \cr p &=& 12 \, m \cr\cr c &=& p+q \cr c &=& 12 \, m+\dfrac{25}{12} \, m \cr c &=& \dfrac{169}{12} \, m \cr\cr b &=& \sqrt{c^2-a^2} \cr b &=& \sqrt{\left(\dfrac{169}{12} \, m\right)^2-\left(13 \, m\right)^2} \cr b &=& \dfrac{65}{12} \, m \end{array}
   

\begin{array}{rcl} U &=& 13 \, m +\dfrac{65}{12} \, m+\dfrac{169}{12} \, m \cr U &=& \dfrac{65}{2} \, m \cr U &=& 32{,}5 \, m \end{array}
 
\begin{array}{rcl} A &=& \dfrac{1}{2} \cdot 13 \, m \cdot \dfrac{65}{12} \, m \cr A &=& \dfrac{845}{24} \, m^2 \cr A &\approx& 35{,}21 \, m^2 \end{array}


10)

\begin{array}{rcl} U &=& 4a \end{array}   \begin{array}{rclcrcl} A &=& a^2 \end{array}


     Berechne a:

     \begin{array}{rcl} e &=& \sqrt{a^2+a^2} \cr e &=& \sqrt{2a^2} \cr a &=& \dfrac{e}{\sqrt{2}} \cr a &=& \dfrac{15{,}4 \, m}{\sqrt{2}} \end{array}
   

\begin{array}{rcl} U &=& 4\cdot \dfrac{15{,}4 \, m}{\sqrt{2}} \cr U &\approx& 43{,}56 \, m \end{array}
 
\begin{array}{rcl} A &=& \left(\dfrac{15{,}4 \, m}{\sqrt{2}}\right)^2 \cr A &=& 118{,}58 \, m^2 \end{array}


11)

Da das Trapez symmetrisch ist, gilt b=d. \begin{array}{rcl} U &=& a+b+c+d \cr U &=& a+2b+c \cr U &=& 25 \, cm+2\cdot 11 \, cm+18 \, cm \cr U &=& 65 \, cm \end{array}   \begin{array}{rcl} A &=& \dfrac{1}{2}(a+c) h \cr A&=& \dfrac{1}{2}(25 \, cm+18 \, cm) \cdot 9{,}5 \, cm \cr A&=& 204{,}25 \, cm^2 \end{array}


12)

\begin{array}{rcl} U &=& 4a \end{array}   \begin{array}{rcl} A &=& \dfrac{1}{2}ef = \dfrac{f}{2}\cdot e \end{array}
   


     Berechnen von \dfrac{f}{2}.

     \begin{array}{rcl} \dfrac{f}{2} &=& \sqrt{a^2-\left(\dfrac{e}{2}\right)^2} \cr \dfrac{f}{2} &=& \sqrt{(8 \, cm)^2-\left(\dfrac{13 \, cm}{2}\right)^2}\cr \dfrac{f}{2} &=& \dfrac{\sqrt{87}}{2}\, cm \end{array}

\begin{array}{rcl} U &=& 4\cdot 8 \, cm \cr U &=& 32 \, cm \end{array}
 
\begin{array}{rcl} A &=& \dfrac{\sqrt{87}}{2}\, cm\cdot 13 \,cm \cr A &\approx& 60{,}63\,cm^2 \end{array}

 

2. Aufgabe

1)

\begin{array}{rcl} U &=& 2a+2b \cr U&=&2\cdot 7{,}8 \,cm+2\cdot 16 \,cm \cr U &=& 47{,}6 \,cm \end{array}   \begin{array}{rcl} A &=& \dfrac{1}{2}ef \cr A &=& \dfrac{22{,}9 \,cm\cdot 6 \,cm}{2} \cr A &=& 68{,}7 \, cm^2\end{array}


2)

\begin{array}{rcl} U &=& 2a+2b \end{array}   \begin{array}{rcl} A &=& \dfrac{1}{2}ef \end{array}


     Berechne b:

     \begin{array}{rcl} b &=& \sqrt{\left(\dfrac{f}{2}\right)^2+(e-p)^2} \cr b &=& \sqrt{\left(\sqrt{3} \, cm\right)^2+(7 \, cm-1 \, cm)^2} \cr b &=& \sqrt{39} \, cm \end{array}
 


      Berechne f:

     \begin{array}{rcl}\dfrac{f}{2} &=& \sqrt{a^2-p^2} \cr \dfrac{f}{2} &=& \sqrt{(2 \,cm)^2-(1 \, cm)^2} \cr \dfrac{f}{2} &=& \sqrt{3} \, cm \cr f&=& 2\sqrt{3} \, cm \end{array}

\begin{array}{rcl} U &=& 2\cdot 2 \,cm+2\cdot \sqrt{39}\, cm \cr U&\approx&16{,}49 \, cm \end{array}
 
\begin{array}{rcl} A &=& \dfrac{7 \, cm \cdot 2\sqrt{3} \, cm}{2} \cr A &\approx& 12{,}12 \, cm^2 \end{array}


3)

\begin{array}{rcl} U &=& 2a+2b \end{array}   \begin{array}{rcl} A &=& \dfrac{1}{2}ef \end{array}


     Berechne a und b:

     \begin{array}{rcl} a &=& \sqrt{\left(\dfrac{f}{2}\right)^2+p^2} \cr a &=& \sqrt{\left(\dfrac{24 \,cm}{2}\right)^2+(5 \, cm)^2} \cr a &=& \sqrt{169} \, cm \cr a &=& 13\, cm \cr\cr b &=& \sqrt{\left(\dfrac{f}{2}\right)^2+q^2} \cr b &=& \sqrt{\left(\dfrac{24 \,cm}{2}\right)^2+(16 \, cm)^2} \cr b &=& \sqrt{400} \, cm \cr b &=& 20 \, cm \end{array}
 


     Berechne e:

     \begin{array}{rcl} e&=&p+q \cr e&=&5 \, cm+16 \, cm \cr e&=&21 \, cm \end{array}

\begin{array}{rcl} U &=& 2\cdot 13 \, cm+2\cdot 20 \, cm \cr U &=& 66 \, cm \end{array}
 
\begin{array}{rcl} A &=& \dfrac{21 \,cm\cdot 24 \,cm}{2} \cr A &=& 252\,cm^2\end{array}

 

3. Aufgabe

1)

\begin{array}{rcl} A &=& ab \cr b &=& \dfrac{A}{a} \cr b &=& \dfrac{27 \, cm^2}{3 \, cm} \cr b &=& 9 \, cm \end{array}   \begin{array}{rcl} U &=& 2a+2b \cr U &=& 2\cdot 3 \, cm+2\cdot 9 \, cm \cr U &=& 24\, cm \end{array}


2)

\begin{array}{rcl} U &=& 2a+2b \cr a &=& \dfrac{U-2b}{2} \cr a &=& \dfrac{24 \, cm-2\cdot 5 \, cm}{2} \cr a&=& 7 \, cm \end{array}   \begin{array}{rcl} A &=& a\cdot h_a \cr A &=& 7 \, cm \cdot 3 \, cm \cr A &=& 21\, cm^2\end{array}


3)

\begin{array}{rcl} A &=& \pi r^2 \cr r &=& \sqrt{\dfrac{A}{\pi}} \cr r &=& \sqrt{\dfrac{37 \, cm^2}{\pi}} \cr r &\approx& 3{,}43\, cm \end{array}   \begin{array}{rcl} U &=& 2\pi r \cr U &=& 2\pi \cdot \sqrt{\dfrac{37 \, cm^2}{\pi}} \cr U &\approx& 21{,}56 \, cm \end{array}

 

4. Aufgabe

Bemerkung: In der Mathematik ist es - anders als in der Physik - üblich, die Einheiten während der Rechnung nicht hinzuschreiben. Erst im Antwortsatz, der bei Textaufgaben dazugehört, muss die Einheit notiert werden.

1)
Seien r_1 der Radius und A_1 der Flächeninhalt des Kreises K_1.
Dann gilt:
\begin{array}{rcl} A_1 &=& \pi \cdot r_1^2 \cr &=& \pi \cdot 10^2 \cr &=& 100\pi \end{array}

Seien r_2 der Radius und A_2 der Flächeninhalt des Kreises K_2 mit dem doppelten Flächeninhalt von K_1.
Dann gilt:
\begin{array}{rcl} A_2 &=& 2A_1 \cr &=& 200\pi \end{array}

Nun ist ja aber auch A_2=\pi \cdot r_2^2 und damit also
\begin{array}{rclcl} 200 \pi &=& \pi r_2^2 &\vert& : \pi \cr 200 &=& r_2^2 &\vert& \sqrt{} \cr r_2 &=& \sqrt{200} \cr &\approx& 14{,}14 \end{array}

Der Radius r_2 des Kreises mit doppeltem Flächeninhalt ist ungefähr 14{,}14 \, m lang.


2)
Sei A_1 der Flächeninhalt des Quadrates Q_1.
Seien a_2 die Seite und A_2 der Flächeninhalt des Quadrates Q_2, das den halben Flächeninhalt von Q_1 hat.
Dann gilt:
\begin{array}{rclcl} A_2 &=& 0{,}5 \cdot A_1 \cr &=& 0{,}5 \cdot 16 \cr &=& 8 \cr \cr\cr A_2 &=& a_2^2 \cr\cr 8 &=& a_2^2 &\vert& \sqrt{} \cr\cr a_2 &=& \sqrt{8} \cr\cr &\approx& 2{,}83 \end{array}

Die Seite a_2 des Quadrats mit halbem Flächeninhalt ist ungefähr 2{,}83 \, cm lang.

 

5. Aufgabe

1)
Berechne c mit dem Satz des Pythagoras im großen Dreieck:
\begin{array}{rclcc} a^2+b^2 &=& c^2 & \vert & \sqrt{} \cr c &=& \sqrt{a^2+b^2} \cr &=& \sqrt{3^2+4^2} \cr &=& \sqrt{25} \cr &=& 5 \, cm \end{array}

Berechne p mit einem der Kathetensätze:
\begin{array}{rclcc} a^2 &=& c \cdot p & \vert & :c \cr p &=& \dfrac{a^2}{c} \cr \cr &=& \dfrac{3^2}{5} \cr \cr &=& \dfrac{9}{5} \, cm \end{array}

Berechne q:
\begin{array}{rclcc} p+q &=& c & \vert & -p \cr q &=& c-p \cr &=& 5- \dfrac{9}{5} \cr &=& \dfrac{16}{5} \, cm \end{array}

Berechne h mit dem Höhensatz:
\begin{array}{rclcc} h^2 &=& p \cdot q & \vert & \sqrt{} \cr h &=& \sqrt{p \cdot q} \cr &=& \sqrt{\dfrac{9}{5} \cdot \dfrac{16}{5}} \cr \cr &=& \sqrt{ \dfrac{144}{25}} \cr \cr &=& \dfrac{12}{5} \, cm \end{array}

 
2)
Berechne q mit dem Höhensatz:
\begin{array}{rclcc} h^2 &=& p \cdot q & \vert & :p \cr q &=& \dfrac{h^2}{p} \cr \cr &=& \dfrac{5^2}{5} \cr &=& 5 \, cm \end{array}

Berechne c:
\begin{array}{rclcc} c &=& p+q \cr &=& 5+5 \cr &=& 10 \, cm \end{array}

Berechne b mit dem Satz des Pythagoras im linken Teildreieck:
\begin{array}{rclcc} h^2+q^2 &=& b^2 & \vert & \sqrt{} \cr b &=& \sqrt{h^2+q^2} \cr &=& \sqrt{5^2+5^2} \cr &=& \sqrt{50} \cr & \approx & 7{,}07 \, cm \end{array}

Berechne a mit dem Satz des Pythagoras im rechten Teildreieck:
\begin{array}{rclcc} h^2+p^2 &=& a^2 & \vert & \sqrt{} \cr a &=& \sqrt{h^2+p^2} \cr &=& \sqrt{5^2+5^2} \cr &=& \sqrt{50} \cr & \approx & 7{,}07 \, cm \end{array}

Bemerkung: Die Gleichheit der Katheten a und b hätte man auch schon zu Beginn der Rechnung erkennen können. In solch einem Fall muss die Rechnung natürlich nicht in aller Ausführlichkeit noch einmal aufgeschrieben werden.

 
3)
Berechne b mit dem Satz des Pythagoras im großen Dreieck:
\begin{array}{rclcc} a^2+b^2 &=& c^2 & \vert & -a^2 \cr b^2 &=& c^2-a^2 & \vert & \sqrt{} \cr b &=& \sqrt{c^2-a^2} \cr &=& \sqrt{6{,}1^2-3{,}4^2} \cr & \approx & 5{,}06 \, cm \end{array}

Berechne p mit einem der Kathetensätze:
\begin{array}{rclcc} a^2 &=& c \cdot p & \vert & :c \cr p &=& \dfrac{a^2}{c} \cr \cr &=& \dfrac{3{,}4^2}{6{,}1} \cr & \approx & 1{,}90 \, cm \end{array}

Berechne q:
\begin{array}{rclcc} p+q &=& c & \vert & -p \cr q &=& c-p \cr &\approx& 6{,}1-1{,}90 \cr &\approx& 4{,}20 \, cm \end{array}

Berechne h mit dem Höhensatz:
\begin{array}{rclcc} h^2 &=& p \cdot q & \vert & \sqrt{} \cr h &=& \sqrt{p \cdot q} \cr &\approx& \sqrt{1{,}90 \cdot 4{,}20} \cr &\approx& \sqrt{7{,}98} \cr & \approx & 2{,}82 \, cm \end{array}

 
4)
Berechne c mit einem der Kathetensätze:
\begin{array}{rclcc} b^2 &=& q \cdot c & \vert & :q \cr c &=& \dfrac{b^2}{q} \cr \cr &=& \dfrac{5{,}2^2}{1{,}2} \cr & \approx & 22 {,}53 \, cm \end{array}

Berechne a mit dem Satz des Pythagoras im großen Dreieck:
\begin{array}{rclcc} a^2+b^2 &=& c^2 & \vert & -b^2 \cr a^2 &=& c^2-b^2 & \vert & \sqrt{} \cr a &=& \sqrt{c^2-b^2} \cr &\approx& \sqrt{22{,}53^2-5{,}2^2} \cr &\approx& \sqrt{480{,}71} \cr & \approx & 21{,}93 \, cm \end{array}

Berechne p:
\begin{array}{rclcc} p+q &=& c & \vert & -q \cr p &=& c-q \cr &\approx& 22{,}53-1{,}2 \cr &\approx& 21{,}33 \, cm \end{array}

Berechne h mit dem Höhensatz:
\begin{array}{rclcc} h^2 &=& p \cdot q & \vert & \sqrt{} \cr h &=& \sqrt{p \cdot q} \cr &\approx& \sqrt{21{,}33 \cdot 1{,}2} \cr &\approx& \sqrt{25{,}60} \cr & \approx & 5{,}06 \, cm \end{array}

 
5)
Berechne q:
\begin{array}{rclcc} p+q &=& c & \vert & -p \cr q &=& c-p \cr &=& 7-3{,}9 \cr &=& 3{,}1 \, cm \end{array}

Berechne h mit dem Höhensatz:
\begin{array}{rclcc} h^2 &=& p \cdot q & \vert & \sqrt{} \cr h &=& \sqrt{p \cdot q} \cr &=& \sqrt{3{,}1 \cdot 3{,}9} \cr &=& \sqrt{12{,}09} \cr & \approx & 3{,}48 \, cm \end{array}

Berechne a mit dem Satz des Pythagoras im rechten Teildreieck:
\begin{array}{rclcc} h^2+p^2 &=& a^2 & \vert & \sqrt{} \cr a &=& \sqrt{h^2+p^2} \cr &\approx& \sqrt{3{,}48^2+3{,}9^2} \cr &\approx& \sqrt{27{,}3} \cr & \approx & 5{,}22 \, cm \end{array}

Berechne b mit dem Satz des Pythagoras im großen Dreieck:
\begin{array}{rclcc} a^2+b^2 &=& c^2 & \vert & -a^2 \cr b^2 &=& c^2-a^2 & \vert & \sqrt{} \cr b &=& \sqrt{c^2-a^2} \cr &\approx& \sqrt{7^2-5{,}22^2} \cr &\approx& \sqrt{21{,}7} \cr & \approx & 4{,}66 \, cm \end{array}


6)
Berechne h mit dem Satz des Pythagoras im rechten Teildreieck
\begin{array}{rclcc}a^2 &=& h^2+p^2 &\vert & -p^2\\h^2 &=& a^2-p^2 &\vert & \sqrt{}\\h &=& \sqrt{a^2-p^2} \\&=& \sqrt{2{,}8^2-1^2} \\&\approx & 2{,}62\, mm\end{array}

Berechne q mit dem Höhensatz
\begin{array}{rclcc}h^2 &=& p\cdot q &\vert & :p\\\\q &=& \dfrac{h^2}{p} \\&\approx& \dfrac{2{,}62^2}{1} \\&\approx & 6{,}86\, mm\end{array}

Berechne c
\begin{array}{rclcc}c &=& p + q \\&\approx& 1+6{,}86 \\&\approx& 7{,}86\, mm\end{array}

Berechne b mit dem Satz des Pythagoras im großen Dreieck
\begin{array}{rclcc}c^2 &=& a^2+b^2 &\vert &-a^2\\b^2 &=& c^2-a^2 &\vert &\sqrt{}\\b &=& \sqrt{c^2-a^2}\\&\approx& \sqrt{7{,}86^2-2{,}8^2}\\&\approx & 7{,}34\, mm\end{array}


7)
Berechne c mit dem Satz des Pythagoras im großen Dreieck
\begin{array}{rclcc}c^2 &=& a^2+b^2 &\vert &\sqrt{}\\c &=& \sqrt{a^2+b^2}\\&=& \sqrt{16^2+9^2}\\&\approx & 18{,}36\, cm\end{array}

Berechne p mit einem der Kathetensätze
\begin{array}{rclcc}a^2 &=& c\cdot p &\vert & :c\\\\p &=& \dfrac{a^2}{c}\\&\approx& \dfrac{16^2}{18{,}36}\\&\approx & 13{,}94\, cm\end{array}

Berechne q
\begin{array}{rclcc}c &=& p+q &\vert & -p\\q &=& c-p \\&\approx& 18{,}36-13{,}94\\&\approx& 4{,}42\, cm\end{array}

Berechne h mit dem Höhensatz
\begin{array}{rclcc}h^2 &=& p\cdot q &\vert &\sqrt{}\\h &=& \sqrt{p\cdot q}\\&\approx& \sqrt{13{,}94\cdot 4{,}42}\\&\approx & 7{,}85\, cm\end{array}


8)
Berechne b mit dem Satz des Pythagoras im linken Teildreieck
\begin{array}{rclcc}b^2 &=& h^2+q^2 &\vert &\sqrt{}\\b &=& \sqrt{h^2+q^2}\\&=& \sqrt{403^2+507^2} \\&\approx & 647{,}66\, cm\end{array}

Berechne c mit einem der Kathetensätze
\begin{array}{rclcc}b^2 &=& c\cdot q &\vert & :q\\\\c &=& \dfrac{b^2}{q}\\&\approx& \dfrac{647{,}66^2}{507}\\&\approx & 827{,}34\, cm\end{array}

Berechne p
\begin{array}{rclcc}c &=& p+q &\vert &-q\\p &=& c-q \\&\approx& 827{,}34-507\\&\approx& 320{,}34\, cm\end{array}

Berechne a mit dem Satz des Pythagoras im rechten Teildreieck
\begin{array}{rclcc}a^2 &=& h^2+p^2 &\vert &\sqrt{}\\a &=& \sqrt{h^2+p^2}\\&\approx& \sqrt{403^2+320{,}34^2}\\&\approx& 514{,}81\,cm\end{array}


9)
Berechne b mit dem Satz des Pythagoras im großen Dreieck
\begin{array}{rclcc}c^2 &=& a^2+b^2 &\vert &-a^2\\b^2 &=& c^2-a^2 &\vert &\sqrt{}\\b &=& \sqrt{c^2-a^2}\\&=& \sqrt{3{,}1416^2-2{,}7183^2}\\&\approx & 1{,}5750\, km\end{array}


Berechne q mit einem der Kathetensätze
\begin{array}{rclcc}b^2 &=& c\cdot q &\vert & :c\\\\q &=& \dfrac{b^2}{c}\\&\approx& \dfrac{1{,}575^2}{3{,}1416}\\&\approx & 0{,}7896\, km\end{array}

Berechne p
\begin{array}{rclcc}c &=& p+q &\vert & -q\\p &=& c-q\\&\approx& 3{,}1416-0{,}7896\\&\approx& 2{,}352\, km\end{array}

Berechne h mit dem Höhensatz
\begin{array}{rclcc}h^2 &=& p\cdot q &\vert &\sqrt{} \\h &=&\sqrt{p\cdot q}\\&\approx& \sqrt{2{,}352\cdot 0{,}7896}\\&\approx & 1{,}3628\, km\end{array}


10)
Berechne p
\begin{array}{rclcc}c &=& p+q &\vert &-q\\p &=& c-q\\&=& 5{,}34-0{,}2\\&=& 5{,}14\, cm\end{array}

Berechne h mit dem Höhensatz
\begin{array}{rclcc}h^2 &=& p\cdot q &\vert &\sqrt{}\\h &=& \sqrt{p\cdot q}\\&=& \sqrt{5{,}14\cdot 0{,}2}\\&\approx & 1{,}01\, cm\end{array}

Berechne b mit einem der Kathetensätze
\begin{array}{rclcc}b^2 &=& c\cdot q &\vert &\sqrt{}\\b &=& \sqrt{c\cdot q}\\&=& \sqrt{5{,}34\cdot 0{,}2}\\&\approx & 1{,}03\, cm\end{array}

Berechne a mit einem der Kathetensätze
\begin{array}{rclcc}a^2 &=& c\cdot p &\vert &\sqrt{}\\a &=& \sqrt{c\cdot p}\\&=& \sqrt{5{,}34\cdot 5{,}14}\\&\approx & 5{,}24\, cm\end{array}


11)
Berechne h mit dem Satz des Pythagoras im linken Teildreieck
\begin{array}{rclcc}b^2 &=& q^2+h^2 &\vert &-q^2\\h^2 &=& b^2-q^2 &\vert &\sqrt{}\\h &=& \sqrt{b^2-q^2}\\&=& \sqrt{603^2-60{,}3^2}\\&\approx & 599{,}98\, cm\end{array}

Berechne c mit einem der Kathetensätze
\begin{array}{rclcc}b^2 &=& c\cdot q &\vert & :q \\c &=& \dfrac{b^2}{q}\\&=& \dfrac{603^2}{60{,}3}\\&=& 6.030\, cm\end{array}

Berechne a mit dem Satz des Pythagoras
\begin{array}{rclcc}c^2 &=& a^2+b^2 &\vert &-b^2\\a^2 &=& c^2-b^2 &\vert &\sqrt{}\\a &=& \sqrt{c^2-b^2}\\&=& \sqrt{6.030^2-603^2}\\&\approx & 5.999{,}77\, cm\end{array}

Berechne p
\begin{array}{rclcc}c &=& p+q &\vert &-q\\p &=& c-q\\&=& 6.030-60{,}3\\&=& 5.969{,}7\, cm\end{array}


12)
Berechne c
\begin{array}{rclcc}c &=& p+q\\&=& 0{,}3+0{,}5\\&=& 0{,}8\, cm\end{array}

Berechne h mit dem Höhensatz
\begin{array}{rclcc}h^2 &=& p\cdot q &\vert &\sqrt{}\\h &=& \sqrt{p\cdot q}\\&=& \sqrt{0{,}3\cdot 0{,}5}\\&\approx & 0{,}39\, cm\end{array}

Berechne b mit dem Satz des Pythagoras im linken Teildreieck
\begin{array}{rclcc}b^2 &=& q^2+h^2 &\vert &\sqrt{}\\b &=& \sqrt{q^2+h^2}\\&\approx& \sqrt{0{,}5^2+0{,}39^2}\\&\approx & 0{,}63\, cm\end{array}

Berechne a mit dem Satz des Pythagoras
\begin{array}{rclcc}a^2 &=& h^2+p^2 &\vert &\sqrt{}\\a &=& \sqrt{h^2+p^2}\\&\approx& \sqrt{0{,}39^2+0{,}3^2}\\&\approx & 0{,}49\, cm\end{array}


13)
Berechne c mit dem Satz des Pythagoras im großen Dreieck
\begin{array}{rclcc}c^2 &=& a^2+b^2 &\vert &\sqrt{} \\c &=& \sqrt{a^2+b^2} \\&=& \sqrt{50^2+50^2}\\&\approx & 70{,}71\, cm\end{array}

Berechne p mit einem der Kathetensätze
\begin{array}{rclcc}a^2 &=& c\cdot p &\vert & :c\\\\p &=& \dfrac{a^2}{c}\\&\approx& \dfrac{50^2}{70{,}71}\\&\approx & 35{,}36\, cm\end{array}

Berechne q mit einem der Kathetensätze
\begin{array}{rclcc}b^2 &=& c\cdot q &\vert & :c\\\\q &=& \dfrac{b^2}{c}\\&\approx& \dfrac{50^2}{70{,}71}\\&\approx & 35{,}36\, cm\end{array}

Berechne h mit dem Höhensatz
\begin{array}{rclcc}h^2 &=& p\cdot q &\vert &\sqrt{}\\h &=& \sqrt{p\cdot q}\\&\approx& \sqrt{35{,}36\cdot 35{,}36}\\&\approx & 35{,}36\, cm\end{array}


14)
Berechne a mit dem Satz des Pythagoras im großen Dreieck
\begin{array}{rclcc}c^2 &=& a^2+b^2 &\vert &-b^2\\a^2 &=& c^2-b^2 &\vert &\sqrt{}\\a &=& \sqrt{c^2-b^2}\\&=& \sqrt{753^2-456^2}\\&\approx & 599{,}23\, cm\end{array}

Berechne q mit einem der Kathetensätze
\begin{array}{rclcc}b^2 &=& c\cdot q &\vert & :c\\\\q &=& \dfrac{b^2}{c}\\&=& \dfrac{456^2}{753}\\&\approx & 276{,}14\, cm\end{array}

Berechne p
\begin{array}{rclcc}c &=& q+p &\vert & -q\\p &=& c-q \\&\approx& 753-276{,}14\\&\approx& 476{,}86\,cm\end{array}

Berechne h mit dem Satz des Pythagoras im rechten Teildreieck
\begin{array}{rclcc}a^2 &=& h^2+p^2 &\vert &-p^2\\h^2 &=& a^2-p^2 &\vert &\sqrt{}\\h &=& \sqrt{a^2-p^2}\\&\approx& \sqrt{599{,}23^2-476{,}86^2}\\&\approx & 362{,}88\, cm\end{array}


15)
Berechne b mit dem Satz des Pythagoras im linken Teildreieck
\begin{array}{rclcc}b^2 &=& h^2+q^2 &\vert &\sqrt{}\\b &=& \sqrt{h^2+q^2}\\&=& \sqrt{20^2+5^2}\\&\approx & 20{,}62\, cm\end{array}

Berechne p mit dem Höhensatz
\begin{array}{rclcc}h^2 &=& p\cdot q &\vert &:q\\p &=& \dfrac{h^2}{q}\\&=& \dfrac{20^2}{5}\\&=& 80\, cm\end{array}

Berechne c
\begin{array}{rclcc}c &=& q+p \\&=& 5+80\\&=& 85\, cm\end{array}

Berechne a mit einem der Kathetensätze
\begin{array}{rclcc}a^2 &=& c\cdot p &\vert &\sqrt{}\\a &=& \sqrt{c\cdot p}\\&=& \sqrt{85\cdot 80}\\&\approx & 82{,}46\, cm\end{array}

 

6. Aufgabe

1) d\left(P_1 \mid P_2\right) = \sqrt{(3-1)^2+(3-2)^2} = \sqrt{5} \approx 2{,}24


2) d\left(P_1 \mid P_2\right) = \sqrt{\left(-\dfrac{2}{3}-\dfrac{1}{3}\right)^2+(7-2)^2} = \sqrt{26} \approx 5{,}10


3) d\left(P_1 \mid P_2\right) = \sqrt{\left(-\dfrac{1}{10}-25\right)^2+(2-(-50))^2} = \sqrt{3.334{,}01} \approx 57{,}74


4) d\left(P_1 \mid P_2\right) = \sqrt{\left(-\dfrac{11}{600}-\left(-\dfrac{1}{300}\right)\right)^2+\left(-\dfrac{8}{21}-\dfrac{3}{7}\right)^2} \approx 0{,}81


5) d\left(P_1 \mid P_2\right) = \sqrt{(21-7)^2+(22-5)^2} = \sqrt{485} \approx 22{,}02


6) d\left(P_1 \mid P_2\right) = \sqrt{\left(\dfrac{1}{2}-(-0{,}5)\right)^2+\left(-\dfrac{31}{4}-7{,}75\right)^2} = \sqrt{241{,}25} \approx 15{,}53


7) d\left(P_1 \mid P_2\right) = \sqrt{(3-0)^2+(4-0)^2} = 5


8) d\left(P_1 \mid P_2\right) = \sqrt{\left(\dfrac{1}{4}-\dfrac{7}{8}\right)^2+\left(\dfrac{3}{8}-\left(-\dfrac{9}{4}\right)\right)^2} = \sqrt{\dfrac{233}{32}} \approx 2{,}7


9) d\left(P_1 \mid P_2\right) = \sqrt{(-83-(-90))^2+(-101-(-23))^2} = \sqrt{6133} \approx 78{,}31


10) d\left(P_1 \mid P_2\right) = \sqrt{(33-333)^2+(11-111)^2} = \sqrt{100.000} \approx 316{,}23

 

7. Aufgabe

Um die Frage beantworten zu können, muss jeweils geprüft werden, ob die Summe der gegebenen Winkel die Winkelsumme von 180^\circ bzw. \pi für ein Dreieck oder 360^\circ bzw. 2\pi für ein Viereck bereits übersteigt. Dazu müssen die gegebenen Winkel entweder ins Gradmaß oder ins Bogenmaß umgerechnet werden.

1)
 \begin{array}{rcl} \alpha + \beta &=& \dfrac{\pi}{2}+100^\circ \cr \alpha + \beta &=& 90^\circ+100^\circ \cr \alpha + \beta &=& 190^\circ \cr \alpha + \beta &>& 180^\circ \end{array}

Es kann kein Dreieck mit diesen Winkeln geben.


2)
\begin{array}{rcl} \alpha + \gamma &=& 100^\circ + \dfrac{\pi}{4} \cr \alpha + \gamma &=& 100^\circ + 45^\circ \cr \alpha + \gamma &=& 145^\circ \cr \alpha + \gamma & < & 180^\circ \end{array}

Es kann ein Dreieck mit diesen Winkeln geben.


3)
\begin{array}{rcl} \alpha + \beta &=& 120^\circ+\dfrac{ \pi}{6} \cr\cr \alpha + \beta &=& \dfrac{2\pi}{3}+\dfrac{ \pi}{6} \cr\cr \alpha + \beta &=& \dfrac{5 \pi}{6} \cr\cr \alpha + \beta & < & \pi \end{array}

Es kann ein Dreieck mit diesen Winkeln geben.


4)
\begin{array}{rcl} \beta + \gamma &=& \dfrac{3 \pi}{5}+80^\circ \cr \beta + \gamma &=& 108^\circ+80^\circ \cr \beta + \gamma &=& 188^\circ \cr \beta + \gamma &>& 180^\circ \end{array}

Es kann kein Dreieck mit diesen Winkeln geben.


5)
\begin{array}{rcl} \beta + \gamma &=& \pi+18^\circ \cr\cr \beta + \gamma &=& \pi+\dfrac{\pi}{10} \cr\cr \beta + \gamma &=& \dfrac{11 \pi}{10} \cr\cr \beta + \gamma &>& \pi \end{array}

Es kann kein Dreieck mit diesen Winkeln geben.


6)
\begin{array}{rcl}\alpha + \beta &=& \dfrac{\pi}{10}+145^\circ \\\alpha + \beta &=& 18^\circ +145^\circ \\\alpha + \beta &=& 163^\circ \\\alpha + \beta & < & 180^\circ \\\end{array}

Es kann ein Dreieck mit diesen Winkeln geben.


7)
\begin{array}{rcl}\beta + \gamma &=& 10^\circ +\dfrac{5\pi}{4} \\\beta + \gamma &=& 10^\circ +225^\circ \\\beta + \gamma &=& 235^\circ \\\beta + \gamma &>& 180^\circ \\\end{array}

Es kann kein Dreieck mit diesen Winkeln geben.


8)
\begin{array}{rcl}\alpha + \beta &=& 30^\circ+\dfrac{\pi}{3} \\\\\alpha + \beta &=& \dfrac{\pi}{6} +\dfrac{\pi}{3} \\\\\alpha + \beta &=& \dfrac{\pi}{2} \\\\\alpha + \beta & < & \pi \\\end{array}

Es kann ein Dreieck mit diesen Winkeln geben.


9)
\begin{array}{rcl}\alpha + \beta &=& \dfrac{3\pi}{8}+45^\circ \\\alpha + \beta &=& 67{,}5^\circ +45^\circ \\\alpha + \beta &=& 112{,}5^\circ \\\alpha + \beta & < & 180^\circ \\\end{array}

Es kann ein Dreieck mit diesen Winkeln geben.


10)
\begin{array}{rcl}\alpha + \gamma &=& 1{,}8^\circ+\dfrac{99\pi}{100} \\\\\alpha + \gamma &=& \dfrac{\pi}{100} +\dfrac{99\pi}{100} \\\\\alpha + \gamma &=& \pi \\\end{array}

Es kann kein Dreieck mit diesen Winkeln geben.


11)
\begin{array}{rcl}\alpha + \beta + \gamma &=& \dfrac{5\pi}{6}+30^\circ +60^\circ \\\alpha + \beta + \gamma &=& 150^\circ +30^\circ +60^\circ \\\alpha + \beta + \gamma &=& 240^\circ \\\alpha + \beta + \gamma & < & 360^\circ \\\end{array}

Es kann ein Viereck mit diesen Winkeln geben.


12)
\begin{array}{rcl}\alpha + \beta + \gamma &=& 45^\circ +\dfrac{3\pi}{2} +\dfrac{\pi}{4} \\\\\alpha + \beta + \gamma &=& \dfrac{\pi}{4} +\dfrac{3\pi}{2} +\dfrac{\pi}{4} \\\\\alpha + \beta + \gamma &=& 2\pi \\\end{array}

Es kann kein Viereck mit diesen Winkeln geben.


13)
\begin{array}{rcl}\alpha + \beta + \gamma &=& \dfrac{\pi}{9} +180^\circ +10^\circ \\\\\alpha + \beta + \gamma &=& \dfrac{\pi}{9} +\pi +\dfrac{\pi}{18} \\\\\alpha + \beta + \gamma &=& \dfrac{7\pi}{6} \\\\\alpha + \beta + \gamma & < & 2\pi \\\end{array}

Es kann ein Viereck mit diesen Winkeln geben.


14)
\begin{array}{rcl}\alpha + \beta + \gamma &=& \dfrac{9\pi}{8} +30^\circ +\dfrac{2\pi}{5} \\\\\alpha + \beta + \gamma &=& \dfrac{9\pi}{8} +\dfrac{\pi}{6} +\dfrac{2\pi}{5} \\\\\alpha + \beta + \gamma &=& \dfrac{203\pi}{120} \\\alpha + \beta + \gamma & < & 2\pi\\\end{array}

Es kann ein Viereck mit diesen Winkeln geben.


15)
\begin{array}{rcl}\alpha + \beta + \gamma &=& \dfrac{11\pi}{18} +70^\circ +\dfrac{17\pi}{18} \\\alpha + \beta + \gamma &=& 110^\circ +70^\circ +170^\circ \\\alpha + \beta + \gamma &=& 350^\circ \\\alpha + \beta + \gamma & < & 360^\circ \\\end{array}

Es kann ein Viereck mit diesen Winkeln geben.


16)
\begin{array}{rcl}\alpha + \beta + \gamma &=& \dfrac{2\pi}{3} +\dfrac{\pi}{6} +\pi \\\alpha + \beta + \gamma &=& 120^\circ +30^\circ +180^\circ \\\alpha + \beta + \gamma &=& 330^\circ \\\alpha + \beta + \gamma & < & 360^\circ \\\end{array}

Es kann ein Viereck mit diesen Winkeln geben.


17)
\begin{array}{rcl}\alpha + \beta + \gamma &=& 32^\circ +\dfrac{6\pi}{5} +\dfrac{23\pi}{36} \\\alpha + \beta + \gamma &=& 32^\circ +216^\circ +115^\circ \\\alpha + \beta + \gamma &=& 363^\circ \\\alpha + \beta + \gamma &>& 360^\circ \\\end{array}

Es kann kein Viereck mit diesen Winkeln geben.


18)
\begin{array}{rcl}\alpha + \beta + \gamma &=& 5^\circ +5^\circ +0{,}1\pi \\\alpha + \beta + \gamma &=& 5^\circ +5^\circ +18^\circ \\\alpha + \beta + \gamma &=& 28^\circ \\\alpha + \beta + \gamma & < & 360^\circ \\\end{array}

Es kann ein Viereck mit diesen Winkeln geben.


19)

\begin{array}{rcl}\alpha + \beta + \gamma &=& \dfrac{\pi}{2} +120^\circ +180^\circ \\\alpha + \beta + \gamma &=& 90^\circ +120^\circ +180^\circ \\\alpha + \beta + \gamma &=& 390^\circ \\\alpha + \beta + \gamma &>& 360^\circ \\\end{array}

Es kann kein Viereck mit diesen Winkeln geben.


20)
\begin{array}{rcl}\alpha + \beta + \gamma &=& \dfrac{5\pi}{4} +90^\circ +\dfrac{7\pi}{12} \\\alpha + \beta + \gamma &=& 225^\circ +90^\circ +105^\circ \\\alpha + \beta + \gamma &=& 420^\circ \\\alpha + \beta + \gamma &>& 360^\circ \\\end{array}

Es kann kein Viereck mit diesen Winkeln geben.

 

8. Aufgabe

Bemerkung: In der Mathematik ist es - anders als in der Physik - üblich, die Einheiten während der Rechnung nicht hinzuschreiben. Erst im Antwortsatz, der bei Textaufgaben dazugehört, muss die Einheit notiert werden.

Sei x \in \mathbb{N} die Anzahl der angefangenen Quadratmeter.

Zuerst berechnen wir die Anzahl der Quadratmeter, die mit 1000\, \text{EUR} maximal möglich sind (Für das Lösen der Ungleichung kann Kapitel 18 helfen ...):
\begin{array}{rclcl} 1000 &\geq& 250+60x &\vert& - 250 \cr 750 &\geq& 60x &\vert& :60 \cr 12{,}5 &\geq& x \end{array}

Da x eine ganze Zahl ist, müssen wir abrunden: x=12 m^2.

Nun berechnen wir die zweite Seite b bei einer maximalen Fläche A=12 \, m^2.

\begin{array}{rcl} A &=& a\cdot b \cr 12 &=& 5 \cdot b \cr b &=& \dfrac{12}{5} \cr b &=& 2{,}4 \end{array}

Die zweite Seite der Terrasse kann also maximal 2{,}4\,m lang werden.

 

9. Aufgabe

Bemerkung: Statt Ober- und Mantelfläche separat anhand der Formeln zu berechnen, kann man auch erst die Mantelfläche bestimmen und anschließend die Grund- und ggf. die Deckflächen addieren.

1)
\begin{array}{rcl} V &=& a^3 \cr V &=& (16 \, cm)^3 \cr V&=&4.096 \, cm^3 \end{array}


\begin{array}{rcl} A_O & = & 6a^2 \cr A_O &=& 6\cdot(16 \, cm)^2\cr A_O& = &1.536\, cm^2 \end{array}
 
\begin{array}{rcl} A_M &=& 4a^2 \cr A_M &=& 4\cdot(16 \, cm)^2 \cr A_M &=& 1.024 \, cm^2 \end{array}

 

2)
Achtung: Einheiten umrechnen!

\begin{array}{rcl} V &=& ab c \cr V &=& 3{,}5 \, cm\cdot 5{,}5 \, cm \cdot 11 \, cm \cr V &=& 211{,}75 \, cm^3 \end{array}


\begin{array}{rcl} A_O & = & 2(ab+ac+bc) \cr A_O & = & 2\cdot(3{,}5 \, cm\cdot 5{,}5 \, cm+3{,}5 \, cm\cdot 11 \, cm+5{,}5 \, cm\cdot 11 \, cm) \cr A_O &=& 236{,}5 \, cm^2 \end{array}
 
\begin{array}{rcl} A_M & = & 2(ac+bc) \cr A_M & = & 2\cdot(3{,}5 \, cm\cdot 11 \, cm+5{,}5 \, cm\cdot 11 \, cm) \cr A_M &=& 198 \, cm^2 \end{array}

 

3)
\begin{array}{rcl} V &=& \pi r^2 h \cr V &=& \pi \cdot (7{,}5 \, m)^2 \cdot10 \, m \cr V &\approx& 1.767{,}15 \, m^3\end{array}


\begin{array}{rcl} A_O & = & 2 \pi r(r+h) \cr A_O & = & 2 \pi \cdot 7{,}5 \, m\cdot(7{,}5 \, m+10 \, m) \cr A_O &\approx& 824{,}67 \,m^2 \end{array}
 
\begin{array}{rcl} A_M &=& 2 \pi r h \cr A_M &=& 2 \pi \cdot 7{,}5 \, m \cdot 10 \, m \cr A_M &\approx& 471{,}24 \, m^2 \end{array}

 

4)
\begin{array}{rcl} V &=& \dfrac{1}{3} a^2 h \cr V &=& \dfrac{1}{3}\cdot (7 \, m)^2\cdot 5 \, m \cr V &\approx& 81{,}67 m^3\end{array}


\begin{array}{rcl} A_O &=& a^2+2ah_s \cr A_O &=& a^2+2a\sqrt{h^2+\left(\dfrac{a}{2}\right)^2} \cr A_O &=& (7 \, m)^2+2\cdot 7 \, m \cdot \sqrt{(5 \, m)^2+\left(\dfrac{7}{2} \, m \right)^2} \cr A_O &\approx& 134{,}45\, m^2 \end{array}
 
\begin{array}{rcl} A_M & =& 2ah_s \cr A_M & =& 2a\sqrt{h^2+\left(\dfrac{a}{2}\right)^2} \cr A_M & =& 2\cdot 7 \, m \cdot \sqrt{(5 \, m)^2+\left(\dfrac{7}{2} \, m \right)^2} \cr A_M & \approx& 85{,}45\, m^2 \end{array}

 

5)
\begin{array}{rcl} V &=& \dfrac{1}{3}\pi r^2 h \cr V &=& \dfrac{1}{3}\pi\cdot (14 \, cm)^2 \cdot 31 \, cm \cr V &\approx& 6.362{,}77 \, cm^3 \end{array}


\begin{array}{rcl} A_O &=& \pi r(r+s) \cr A_O &=& \pi r\left(r+\sqrt{r^2+h^2}\right) \cr A_O &=& \pi \cdot (14 \, cm)\cdot\left(14\,cm + \sqrt{(14\, cm)^2+(31\, cm)^2}\right) \cr A_O &\approx& 2.111{,}80 \, cm^2 \end{array}
 
\begin{array}{rcl} A_M & =& \pi rs \cr A_M & =& \pi r\ \sqrt{r^2+h^2} \cr A_M & =& \pi \cdot (14 \, cm) \cdot \sqrt{(14 \, cm)^2+(31 \, cm)^2} \cr A_M & \approx& 1.496{,}04 \, cm^2 \end{array}

 

6)
\begin{array}{rcl} V &=& \dfrac{4}{3}\pi \left(\dfrac{d}{2}\right)^3 \cr V &=& \dfrac{4}{3}\pi \cdot\left(\dfrac{6{,}7}{2} \, cm\right)^3 \cr V &\approx& 157{,}48 \, cm^3 \end{array}

\begin{array}{rcl} A_O &=& 4\pi \left(\dfrac{d}{2}\right)^2 \cr A_O &=& 4\pi \cdot \left(\dfrac{6{,}7}{2} \, cm\right)^2 \cr A_O &\approx& 141{,}03 \, cm^2 \end{array}

 

10. Aufgabe

Bemerkung: In der Mathematik ist es - anders als in der Physik - üblich, die Einheiten während der Rechnung nicht hinzuschreiben. Erst im Antwortsatz, der bei Textaufgaben dazugehört, muss die Einheit notiert werden.

1)
Es gilt (Satz des Pythagoras): a^2 + a^2=d^2 , wobei mit a die Seiten und mit d die Diagonale des Quadrats bezeichnet wurden (siehe Bemerkung zu Textaufgaben).

Gelöst werden muss also:
\begin{array}{rclcc} d^2 &=& a^2+a^2 \cr &=& 2a^2 &\vert& \sqrt{} \cr d &=& \sqrt{2a^2} \cr &=& a\cdot\sqrt{2} &\vert& : \sqrt{2} \cr \dfrac{d}{\sqrt{2}} &=& a \cr \cr a &=& \dfrac{5{,}657}{\sqrt{2}} \cr &\approx& 4\end{array}

Die Seiten des Quadrats sind jeweils ca. 4 \, cm lang.


2)
Unter der Raumdiagonale eines Würfels versteht man z. B. die Strecke von der vorderen, linken, unteren Ecke zur hinteren, rechten, oberen.

Es gilt (Satz des Pythagoras): a^2 + a^2 + a^2=d^2 , wobei mit a die Seiten und mit d die Diagonale des Würfels bezeichnet wurden.

Gelöst werden muss also:
\begin{array}{rclcc} d^2 &=& a^2+a^2+a^2 \cr &=& 3a^2 &\vert& \sqrt{} \cr d &=& \sqrt{3a^2} \cr &=& a\cdot\sqrt{3} &\vert& : \sqrt{3} \cr \dfrac{d}{\sqrt{3}} &=& a \cr \cr a &=& \dfrac{17{,}321}{\sqrt{3}} \cr &\approx& 10 \end{array}

Die Kanten des Würfels sind jeweils ca. 10 \, cm lang.


3)
Sei r der Radius der Kugel. Dann gilt für die Oberfläche der Kugel: O=4\pi r^2

Also in diesem Fall:
\begin{array}{rclcl} 615{,}75 &=& 4\pi r^2 &\vert& :\left(4\pi\right) \cr \dfrac{615{,}75}{4\pi} &=& r^2 &\vert& \sqrt{} \cr r &=& \sqrt{\dfrac{615{,}75}{4\pi}} \cr &\approx& 7{,}00 \, cm\end{array}

Für das Volumen einer Kugel gilt: V=\dfrac{4}{3}\pi r^3

Setzt man den eben berechneten Radius in die Volumenformel ein, erhält man:
\begin{array}{rcl} V &\approx& \dfrac{4}{3}\pi \cdot 7{,}00^3 \cr &=& 1.436{,}76 \end{array}

Das Volumen der Kugel beträgt ca. 1.436{,}76 \, cm^3.

 

11. Aufgabe

Bemerkung: In der Mathematik ist es - anders als in der Physik - üblich, die Einheiten während der Rechnung nicht hinzuschreiben. Erst im Antwortsatz, der bei Textaufgaben dazugehört, muss die Einheit notiert werden.

1)
Auch hier macht sich eine Skizze gut:
Kreis im Quadrat
Fläche des Quadrats: A_Q = a^2
Radius des Kreises (entspricht der halben Seitenlänge des Quadrats): r = \dfrac{a}{2}
Fläche des Kreises: A_K = \pi r^2 = \pi \left(\dfrac{a}{2}\right)^2 = \dfrac{1}{4}\pi a^2

Verhältnis Kreisfläche zu Quadratfläche:
\genfrac{}{}{1pt}{0}{\dfrac{1}{4}\pi a^2}{a^2} = \dfrac{1}{4}\pi

Das Verhältnis der Kreisfläche zur Quadratfläche beträgt \dfrac{\pi}{4} \approx 0{,}79.
Zur Plausibilisierung: Da die Kreisfläche offensichtlich kleiner ist als die Quadratfläche, muss das Ergebnis kleiner als 1 sein. Passt!

2)
Zunächst eine Skizze:
Quadrat im Kreis
Fläche des Quadrats: A_Q = a^2
Radius des Kreises (entspricht dem halben Durchmesser des Quadrats): r = \dfrac{1}{2}\sqrt{a^2+a^2}=\dfrac{1}{2}\sqrt{2}\cdot a
Fläche des Kreises: A_K = \pi r^2 = \pi \left(\dfrac{1}{2}\sqrt{2}\cdot a\right)^2 = \dfrac{1}{2}\pi a^2

Verhältnis Kreisfläche zu Quadratfläche:
\genfrac{}{}{1pt}{0}{\dfrac{1}{2}\pi a^2}{a^2} = \dfrac{1}{2}\pi

Das Verhältnis der Kreisfläche zur Quadratfläche beträgt \dfrac{\pi}{2} \approx 1,57.
Zur Plausibilisierung: Da die Kreisfläche offensichtlich größer ist als die Quadratfläche, muss das Ergebnis größer als 1 sein. Passt!

3)
Bei Körpern im dreidimensionalen Raum ist es mit Skizzen schwierig ... Aber die Zeichnung, die wir für die zweidimensionalen Figuren erstellt haben, helfen für die Anschauung schon ganz gut weiter.


Kugel im Würfel:

Volumen des Würfels mit Kantenlänge a: V_W = a^3
Radius der Kugel (entspricht der halben Kantenlänge des Würfels): r = \dfrac{a}{2}
Volumen der Kugel: V_K = \dfrac{4}{3}\pi r^3 = \dfrac{4}{3}\pi \left(\dfrac{a}{2}\right)^3 = \dfrac{4}{3}\pi \cdot \dfrac{a^3}{8} =\dfrac{1}{6} \pi a^3

Verhältnis Kugelvolumen zum Würfelvolumen:
\genfrac{}{}{1pt}{0}{\dfrac{1}{6} \pi a^3}{a^3} = \dfrac{1}{6}\pi

Das Verhältnis des Kugelvolumens zum Würfelvolumen beträgt \dfrac{1}{6}\pi\approx 0{,}52.
Zur Plausibilisierung: Da das Kugelvolumen offensichtlich kleiner ist als das Würfelvolumen, muss das Ergebnis kleiner als 1 sein. Passt!

Würfel in Kugel:

Volumen des Würfels mit Kantenlänge a: V_W = a^3
Radius der Kugel (entspricht der halben Raumdiagonalen des Würfels): r = \dfrac{1}{2}\sqrt{a^2+a^2+a^2} = \dfrac{\sqrt{3}\cdot a}{2}
Volumen der Kugel: V_K = \dfrac{4}{3}\pi r^3 = \dfrac{4}{3}\pi \left(\dfrac{\sqrt{3}\cdot a}{2} \right)^3 = \dfrac{4}{3}\pi \cdot \dfrac{3\sqrt{3}\cdot a^3}{8} = \dfrac{\sqrt{3}}{2}\pi a^3

Verhältnis Kugelvolumen zum Würfelvolumen:
 \genfrac{}{}{1pt}{0}{\dfrac{\sqrt{3}}{2}\pi a^3}{a^3} = \dfrac{\sqrt{3}}{2}\pi

Das Verhältnis des Kugelvolumens zum Würfelvolumen beträgt \dfrac{\sqrt{3}}{2}\pi \approx 2{,}72.
Zur Plausibilisierung: Da das Kugelvolumen offensichtlich größer ist als das Würfelvolumen, muss das Ergebnis größer als 1 sein. Passt!

 

12. Aufgabe

Hinweis: Da 1\, l =1.000 \, cm^3 ist, ist 0{,}25\,l = 250 cm^3.

a)
Umstellen der Volumenformel nach h
\begin{array}{rclcl} V &=& \pi r^2 h &\vert& : \pi :r^2 \cr \dfrac{V}{\pi r^2} &=& h \end{array}

Berechnen der Zylinderoberfläche
\begin{array}{rcl} A_O &=& 2\pi r(r+h) \cr A_O &=& 2\pi r\left(r+\dfrac{V}{\pi r^2}\right) \cr A_O &=& 2\pi\cdot 3{,}75\, cm \left(3{,}75\, cm+\dfrac{250\, cm^3}{\pi \left(3{,}75\, cm\right)^2}\right) \cr A_O &\approx& 221{,}69 \, cm^2 \end{array}

Bemerkung: Da wir hier mit dem Radius rechnen, müssen wir den Durchmesser halbieren: 7{,}5 : 2 = 3{,}75.

b)
Umstellen der Volumenformel nach c, wobei b=2a
\begin{array}{rclcl} V &=& a\cdot b\cdot c &\vert& :a:b \cr \dfrac{V}{a\cdot b} &=& c \cr \dfrac{V}{a\cdot 2a} &=& c\end{array}

Berechnen der Quaderoberfläche
\begin{array}{rcl} A_O &=& 2(ab+ac+bc) \cr A_O &=& 2\left(ab+a\dfrac{V}{a\cdot 2a}+b\dfrac{V}{a\cdot 2a}\right) \cr A_O &=& 2 \left(4 \, cm \cdot 8 \, cm+4 \, cm \cdot \dfrac{250\, cm^3}{4 \, cm \cdot 8 \, cm}+8 \, cm \cdot \dfrac{250\, cm^3}{4 \, cm \cdot 8 \, cm} \right) \cr A_O &=& 251{,}5 \, cm^2 \end{array}

c)
Umstellen der Volumenformel nach a
\begin{array}{rclcl} V &=& a^3 &\vert& \sqrt[3]{} \cr \sqrt[3]{V} &=& a \end{array}

Berechnen der Würfeloberfläche
\begin{array}{rcl} A_O &=& 6 a^2 \cr A_O &=& 6 \left(\sqrt[3]{V}\right)^2 \cr A_O &=& 6\left(\sqrt[3]{250 \, cm^3}\right)^2 \cr A_O &\approx& 238{,}11\, cm^2 \end{array}

d)
Umstellen der Volumenformel nach r
\begin{array}{rclcl} V &=& \dfrac{4}{3}\pi r^3 &\vert& :\dfrac{4}{3} : \pi \cr \dfrac{3V}{4\pi} &=& r^3 &\vert& \sqrt[3]{ } \cr \sqrt[3]{\dfrac{3V}{4\pi}} &=& r \end{array}

Berechnen der Kugeloberfläche
\begin{array}{rcl} A_O &=& 4\pi r^2 \cr A_O &=& 4\pi\cdot \left(\sqrt[3]{\dfrac{3V}{4\pi}} \right)^2 \cr A_O &=& 4\pi\cdot \left(\sqrt[3]{\dfrac{3\cdot 250\;cm^3}{4\pi}} \right)^2 \cr A_O &\approx& 191{,}92 \, cm^2 \end{array}

Schlussfolgerung aus a) bis d): Die optimale Alternative ist die Dose in Kugelform, da hier die Oberfläche und damit der Materialverbrauch bei gleichem Volumen am geringsten ist. Da diese sich aber schwer herstellen, transportieren, lagern und im Alltag handhaben lässt, ist diese Lösung für die Praxis nicht geeignet. Die Entscheidung fällt also auf die Dose in Zylinderform.

 

13. Aufgabe

a)
\begin{array}{rcl} V &=& \dfrac{1}{4}\pi d^2 \cdot h \cr V &=& \dfrac{1}{4} \pi \cdot (0{,}15 \, m)^2 \cdot 15 \, m \cr V &\approx& 0{,}26507 \,m^3 \end{array}

Das Rohr fasst ca. 0{,}26507 \,m^3 bzw. 265{,}07 \,l.

b)
\begin{array}{rcl} A_M &=& \pi d \cdot h \cr A_M &=& \pi\cdot 0{,}2\, m \cdot 15\,m \cr A_M &\approx& 9{,}42 \,m^2 \end{array}

Die Mantelfläche beträgt ca. 9{,}42 \,m^2.

 

14. Aufgabe

Gesucht ist das Volumen des Beckens:

Seitenansicht Schwimmbad mit Hilfslinien

Zunächst zerlegen wir die Seitenfläche A_G in Teilflächen A_1, A_2 und A_3:

Achtung: Da das Becken nur bis 10\,cm unter den Rand gefüllt werden soll, muss bei den Teilflächen A_1 und A_2 mit 2{,}4\,m und 1{,}1\,m gerechnet werden.

Rechteck A_1
\begin{array}{rcl} A_1 &=& 2{,}4 \, m \cdot 10 \,m \cr A_1 &=& 24 \, m^2 \end{array}

Rechteck A_2
\begin{array}{rcl} A_2 &=& 1{,}1 \, m \cdot 15 \,m \cr A_2 &=& 16{,}5 \, m^2 \end{array}

Rechtwinkliges Dreieck A_3
\begin{array}{rcl} A_3 &=& \dfrac{1}{2}\cdot 5\, m \cdot 1{,}3 \,m \cr A_3 &=& 3{,}25 \, m^2 \end{array}

Daraus berechnen wir die Gesamtfläche A_G:
\begin{array}{rcl} A_G &=& A_1+A_2+A_3 \cr A_G &=& 24 \, m^2+16{,}5 \, m^2+3{,}25 \, m^2 \cr A_G &=& 43{,}75 \,m^2 \end{array}


Nun berechnen wir das Volumen aus der Gesamtfläche A_G und der Breite des Beckens b:
\begin{array}{rcl} V &=& A_G\cdot b \cr V &=& 43{,}75 \,m^2 \cdot 15\,m \cr V &=& 656{,}25 \,m^3 \end{array}

Um das Becken mit einem Volumen 656{,}25 \,m^3 zu füllen, werden 656.250\, l Wasser benötigt, da 1\, m^3 = 1.000 \, l ist.

21. Trigonometrie - Lernziele und typische Fehler

Nach Durcharbeiten dieses Kapitels sollten Sie folgende Lernziele erreicht haben:

  • Sie wissen um die Bedeutung von Grad- und Bogenmaß in diesem Zusammenhang - und achten darauf, dass Ihr Taschenrechner entsprechend eingestellt ist.
  • Sie kennen die Definitionen von Sinus, Kosinus und Tangens am rechtwinkligen Dreieck.
  • Sie kennen die Definitionen von Arkussinus, Arkuskosinus und Arkustangens am rechtwinkligen Dreieck.
  • Sie können Seiten und Winkel im rechtwinkligen Dreieck mithilfe von Sinus, Kosinus und Tangens bzw. Arkussinus, Arkuskosinus und Arkustangens berechnen.
  • Sie können die Weiterentwicklung von Sinus, Kosinus und Tangens hin zu trigonometrischen Funktionen nachvollziehen.
  • Sie können erklären, was eine periodische Funktion ist.
  • Sie kennen die Verläufe und Eigenschaften der trigonometrischen Funktionen.


Typische Fehler in diesem Kapitel sind:

  • Die Eigenschaften der trigonometrischen Funktionen werden nicht so gut verinnerlicht, dass sie im Kontext angewendet werden können. Erklärung
  • Trigonometrische Terme und/oder ihre Argumente werden addiert/multipliziert, ohne die Additionstheoreme zu berücksichtigen. Erklärung


Für Online-Selbsttests zu diesem Thema und weitere Informationen zur Mathematikunterstützung an der TH Wildau nutzen Sie bitte den Moodle-Kursraum "SOS Mathematik - Brückenkurs".

 

Dieses Kapitel wird gerade überarbeitet.

Übersicht:

 

21.1 Trigonometrie - Aufgaben

Bemerkung vorab: Runden Sie bei den folgenden Aufgaben – wenn nötig – auf zwei Stellen nach dem Komma. Wichtig ist, immer erst ganz zum Schluss zu runden, damit die Rundungsfehler nicht zu groß werden!

 

1. Aufgabe

Berechnen Sie aus den gegebenen Werten die fehlenden Seiten und Winkel der Dreiecke! Alle Bezeichnungen beziehen sich auf die unten stehende Zeichnung.

1) b=4 \, cm und \alpha = \dfrac{11 \pi}{36}   6) b=12 \, cm und \beta = \dfrac{\pi}{6}
2) c=11 \, cm und \alpha = \dfrac{\pi}{8}   7) \alpha = 50^{\circ} und \beta = 40^{\circ}
3) a=5 \, cm und b=8 \, cm   8) c=5 \, cm und \alpha = 15^{\circ}
4) c=6{, }7 \, cm und \beta = \dfrac{3\pi}{5}   9) a=13 \, cm und c = 13 \, cm
5) b=3 \, cm und c=7{,}5 \, cm   10) \beta=36^{\circ} und a = 9{,}3 \, cm

rechtwinkliges Dreieck mit klassischen Bezeichnungen

 

2. Aufgabe

In einem Trapez haben die parallelen Seiten die Längen 10 \, cm und 4 \, cm. Die beiden übrigen Seiten haben die Länge 5 \, cm.
Bestimmen Sie die Höhe und alle Winkel!

 

3. Aufgabe

Zeichnen Sie die folgenden Funktionsgraphen!

1) f_1(x)=3 \sin(x)+1   4) f_4(x)=\tan \left(x-\dfrac{\pi}{2} \right)

2) f_2(x)=\cos \left(x+\dfrac{\pi}{2} \right)   5) f_5(x)=-2 \cos \left( \dfrac{1}{2}x-\dfrac{\pi}{2} \right)

3) f_3(x)=-\sin(2x)    

4. Aufgabe

Für Profis: Warum gilt \sin^2(\alpha)+\cos^2(\alpha)=1 für alle \alpha\in\mathbb{R}?

Dieses Kapitel enthält die folgenden Themen:

 

21.2 Trigonometrie - Erklärungen

Die Trigonometrie ist insofern ein spannender mathematischer Bereich, weil ihre Vorstufen schon in den Antike bekannt waren. Benötigt wurden trigonometrische Berechnungen damals vor allem in der Astronomie und für Landvermessungen. Auch für die Navigation in der Seefahrt ist die Trigonometrie schon sehr lange wichtig. Es gibt also ganz praktische Anwendungsfälle.

Wichtig: Bei allen Aufgaben im Bereich der Trigonometrie ist darauf zu achten, ob die Winkel im Grad- oder im Bogenmaß gegeben sind und ob der Taschenrechner, der bei diesen Aufgaben durchaus sinnvoll ist, entsprechend eingestellt ist. Viele Taschenrechner zeigen bei der Einstellung Gradmaß ein "D" oder "DEG" und bei der Einstellung Bogenmaß ein "R" oder "RAD" (für Radiant) am oberen Rand des Displays.

Auch anhand der Ergebnisse ist der Unterschied zu erkennen: Ist der Taschenrechner auf Gradmaß eingestellt, erhält man \sin \left( \dfrac{ \pi}{2} \right) \approx 0{,}0274 bzw. \sin (\pi) \approx 0{,}0548. Ist umgekehrt der Taschenrechner auf Bogenmaß eingestellt, erhält man \sin(90^\circ) \approx 0{,}8940 bzw. \sin (180^\circ) \approx -0{,}8012. Alle diese "Ergebnisse" sind FALSCH!

Korrekt ist \dfrac{ \pi}{2}=90^\circ und damit \sin \left( \dfrac{ \pi}{2} \right) = \sin (90^\circ) = 1 bzw. \pi = 180^\circ und damit \sin (\pi) = \sin(180^\circ)=0

 

Trigonometrie am Dreieck

rechtwinkliges Dreieck mit klassischen Bezeichnungen

Im rechtwinkligen Dreieck gilt (alle Bezeichnungen entsprechend der oberen Zeichnung):

Sinus: \sin(\alpha) = \dfrac{\text{Gegenkathete}}{\text{Hypotenuse}} = \dfrac{a}{c}   und   \sin(\beta) = \dfrac{\text{Gegenkathete}}{\text{Hypotenuse}} = \dfrac{b}{c}
Kosinus: \cos(\alpha) = \dfrac{\text{Ankathete}}{\text{Hypotenuse}} = \dfrac{b}{c}   und   \cos(\beta) = \dfrac{\text{Ankathete}}{\text{Hypotenuse}} = \dfrac{a}{c}
Tangens: \tan(\alpha) = \dfrac{\text{Gegenkathete}}{\text{Ankathete}} = \dfrac{a}{b}   und   \tan(\beta) = \dfrac{\text{Gegenkathete}}{\text{Ankathete}} = \dfrac{b}{a}
Kotangens: \cot(\alpha) = \dfrac{\text{Ankathete}}{\text{Gegenkathete}} = \dfrac{b}{a}   und   \cot(\beta) = \dfrac{\text{Ankathete}}{\text{Gegenkathete}} = \dfrac{a}{b}

Die Ankathete zu einem Winkel ist die Kathete, die an diesen Winkel angrenzt, wohingegen die Gegenkathete eines Winkels dem Winkel gegenüberliegt. Die Hypotenuse liegt immer dem rechten Winkel gegenüber.

Bemerkung 1: Ganz wichtig: Alle diese Berechnungen funktionieren wirklich nur am rechtwinkligen Dreieck. Für Dreiecke ohne rechten Winkel gibt es Verallgemeinerungen, die dann Sinussatz und Kosinussatz heißen.
Bemerkung 2: Es gilt für alle \alpha: \tan(\alpha)=\dfrac{\sin(\alpha)}{\cos(\alpha)} und \cot(\alpha)=\dfrac{\cos(\alpha)}{\sin(\alpha)}
Bemerkung 3: Der Kotangens wird selten verwendet, weil für alle \alpha gilt: \cot(\alpha)=\dfrac{1}{\tan(\alpha)}. Er wird also nicht wirklich gebraucht.


Zur Schreibweise:

  • Bei all diesen Funktionen können die Klammern weggelassen werden, wenn keine Verwechslungsgefahr besteht. Man schreibt dann beispielsweise: \sin(x) = \sin x.
  • Dringend notwendig sind die Klammern aber dann, wenn Verwechslungen auftreten können. Beispielsweise könnte \sin x \cdot 3 sowohl \sin(x) \cdot 3 als auch \sin (3x) meinen.

 

Trigonometrische Funktionen

Begreift man den Winkel als unabhängige Variable, d. h. fasst man x \in \mathbb{R} als Winkel im Bogenmaß auf, entstehen aus diesen Rechenvorschriften Funktionen, die trigonometrische Funktionen oder Winkelfunktionen genannt werden.

Dann schreibt man:

f(x)= \sin(x) mit dem Definitionsbereich \mathbb{D}=\mathbb{R} und dem Wertebereich \mathbb{W}=[-1; 1]

f(x)= \cos(x) mit \mathbb{D}=\mathbb{R} und \mathbb{W}=[-1; 1]

f(x)= \tan(x) mit \mathbb{D}=\mathbb{R}\backslash_{\{k\pi+\frac{\pi}{2}; k \in \mathbb{Z}\}} und \mathbb{W}=\mathbb{R}

f(x)= \cot(x) mit \mathbb{D}=\mathbb{R}\backslash_{\{k\pi; k \in \mathbb{Z}\}} und \mathbb{W}=\mathbb{R}

Bemerkung: Die Lücken in den Definitionsbereichen von Tangens und Kotangens erklären sich aus der Bemerkung 1 von oben: Da sich der Tangens als Quotient von Sinus und Kosinus darstellen lässt, kann er an den Stellen, wo der Nenner 0 wird, nicht definiert sein. Der Kosinus hat eine Nullstelle bei \dfrac{\pi}{2} und dann jeweils im Abstand von \pi in beide Richtungen weitere, also z. B. bei \dfrac{\pi}{2}+\pi=\dfrac{3\pi}{2} oder bei \dfrac{\pi}{2}-2\pi=-\dfrac{3\pi}{2} . Das lässt sich mathematisch ausdrücken mit \dfrac{\pi}{2}+k\pi mit k\in\mathbb{Z} (\mathbb{Z} ist hierbei die Menge der ganzen Zahlen).
Analog beim Kotangens.


Die trigonometrischen Funktionen sind periodische Funktionen, d. h. ihre Funktionswerte wiederholen sich in regelmäßigen Abständen. Der kleinste Abstand dieser Wiederholung, Periode genannt, beträgt bei der Sinus- und der Kosinusfunktion 2 \pi , bei der Tangens- und der Kotangensfunktion \pi


In der folgenden Grafik sind die Graphen der Sinus- (dünne Linie) und der Kosinusfunktion (fette Linie) dargestellt:

Sinus- und Kosinusfunktion

 

In der nun folgenden Grafik sind die Graphen der Tangens- (dünne Linie) und der Kotangensfunktion (fette Linie) dargestellt:

Tangens- und Kotangensfunktion 

 

Additionstheoreme

Noch eine ganz wichtige Sache im Zusammenhang mit trigonometrischen Funktionen:
Die Summe zweier Sinuswerte ist nicht das Gleiche wie die Summe der Winkel, von der dann der Sinus berechnet wird! Dies kann man sich beispielsweise anhand der folgenden Rechnung klarmachen:
\sin\left(\dfrac{\pi}{2}\right)+\sin\left(\dfrac{\pi}{2}\right) = 1+1 = 2

\sin\left(\dfrac{\pi}{2}+\dfrac{\pi}{2}\right)=\sin\left(\pi\right)=0 kann schon deshalb nicht das Gleiche sein wie oben, weil der Sinus ja maximal den Wert 1 annehmen kann.


Natürlich kann man aber auch mit Summen in Sinus- bzw. Kosinusfunktionen rechnen. Dafür gibt es die sogenannten Additionstheoreme, nämlich
für alle x,y\in\mathbb{R}:
\sin (x \pm y) = \sin(x)\cos(y)\pm\cos(x)\sin(y)

\cos(x\pm y) = \cos(x)\cos(y)\mp\sin(x)\sin(y)

und noch einige weitere ...

 

Der "trigonometrische Pythagoras"

Eine ganz wichtige Formel ...
Es gilt für alle \alpha\in\mathbb{R}: \sin^2(\alpha)+\cos^2(\alpha)=1

Achtung: Mit \sin^2(\alpha) ist (\sin(\alpha) )^2 gemeint und nicht \sin\left(\alpha^2\right).

 

Inverse trigonometrische Funktionen

Die Berechnung von Winkel kann im rechtwinkligen Dreieck mithilfe der inversen trigonometrischen Funktionen geschehen. Weil die Winkel hier üblicherweise im Bogenmaß verwendet werden und "Arkus" das lateinische Worte für "Bogen" ist, werden diese Funktionen auch Arkusfunktionen genannt. Beispielsweise liefert die Arkussinusfunktion zu einem gegebenen Wert z (der, wie wir unten sehen werden, zwischen -1 und 1 liegen muss) einen Winkel \varphi , für den \sin(\varphi)=z gilt. Die Bezeichnungen, die im Folgenden verwendet werden, beziehen sich alle wieder auf die Zeichnung am Anfang der Seite.

Arkussinus: \alpha = \arcsin\left(\dfrac{\text{Gegenkathete}}{\text{Hypotenuse}}\right) = \arcsin\left(\dfrac{a}{c}\right)   und   \beta = \arcsin\left(\dfrac{\text{Gegenkathete}}{\text{Hypotenuse}}\right) = \arcsin\left(\dfrac{b}{c}\right)
Arkuskosinus: \alpha = \arccos\left(\dfrac{\text{Ankathete}}{\text{Hypotenuse}}\right) = \arccos\left(\dfrac{b}{c}\right)   und   \beta = \arccos\left(\dfrac{\text{Ankathete}}{\text{Hypotenuse}}\right) = \arccos\left(\dfrac{a}{c}\right)
Arkustangens: \alpha = \arctan\left(\dfrac{\text{Gegenkathete}}{\text{Ankathete}}\right) = \arctan\left(\dfrac{a}{b}\right)   und   \beta = \arctan\left(\dfrac{\text{Gegenkathete}}{\text{Ankathete}}\right) = \arctan\left(\dfrac{b}{a}\right)

Auf dem Taschenrechner steht statt \arcsin meist \sin^{-1}, statt \arccos \cos^{-1} und statt \arctan \tan^{-1} . Bitte beachten Sie, dass damit nicht der Kehrwert gemeint ist!

Auch wenn in diesem Lernmodul vor allem die Winkelberechnung als Anwendungsfall der Arkusfunktionen betrachtet wird, sind sie natürlich für viele andere Bereiche auch nützlich bzw. notwendig. Wenigstens die Graphen der Arkusfunktionen sollen nicht fehlen:

f(x)= \arcsin(x) mit dem Definitionsbereich \mathbb{D}=[-1; 1] und dem Wertebereich \mathbb{W}=\left[-\dfrac{\pi}{2};\dfrac{\pi}{2}\right] als Umkehrfunktion zu f(x)=\sin(x) mit \mathbb{D}=\left[-\dfrac{\pi}{2};\dfrac{\pi}{2}\right] und \mathbb{W}=[-1; 1]


f(x)= \arccos(x) mit \mathbb{D}=[-1; 1] und \mathbb{W}=[0;\pi] als Umkehrfunktion zu f(x)=\cos(x) mit \mathbb{D}=[0;\pi] und \mathbb{W}=[-1; 1]


f(x)= \arctan(x) mit \mathbb{D}=\mathbb{R} und \mathbb{W}=\left]-\dfrac{\pi}{2};\dfrac{\pi}{2}\right[ als Umkehrfunktion zu f(x)=\tan(x) mit \mathbb{D}=\left]-\dfrac{\pi}{2};\dfrac{\pi}{2}\right[ und \mathbb{W}=\mathbb{R}

Arkusfunktionen

Bemerkung 1: Bitte beachten Sie, dass der Arkussinus und der Arkuskosinus nicht für alle reellen Zahlen definiert sind, sondern nur auf dem Intervall [-1;1]. Warum ist das so? Umkehrfunktionen "beantworten" grundsätzlich die Frage: "Wie lautete das Argument der Funktion, wenn wir xy als Ergebnis erhalten haben?" Die Umkehrfunktion bekommt also das Ergebnis der Ursprungsfunktion als Argument und liefert das Argument der Ursprungsfunktion zurück. Nun können Sinus und Kosinus ja bekanntlich nur Werte zwischen -1 und 1 annehmen. Fragen der Art "Wie lautete das Argument der Sinusfunktion, wenn wir als Ergebnis 2 erhalten haben?" sind also sinnlos.
Bemerkung 2: Manchmal müssen beim Bilden von Umkehrfunktionen die Definitionsbereiche der ursprünglichen Funktionen eingeschränkt werden, so auch hier. Warum das? Wegen der Periodizität der trigonometrischen Funktionen hätte z. B. die Frage "Wie lautete das Argument der Sinusfunktion, wenn der Funktionswert 1 ist?" (also genau die Frage, die \arcsin(1) "beantworten" möchte) nicht nur die Antwort \dfrac{\pi}{2} , sondern auch -\dfrac{3\pi}{2}, \dfrac{5\pi}{2}, \dfrac{9\pi}{2} und unendlich viele andere Werte. Ließe man also die gesamten reellen Zahlen als Definitionsbereich zu, gäbe es keine eindeutige Antwort. Das ist ein Widerspruch zur Funktionsdefinition, die eine eindeutige Zuordnung der Werte verlangt. Man löst das Problem, indem man die Definitionsbereiche so einschränkt, dass jeder Funktionswert genau einmal enthalten ist. Das ist beim Sinus für \left[ -\dfrac{\pi}{2}; \dfrac{\pi}{2}\right] ; beim Kosinus für [0;\pi] und beim Tangens für \left] -\dfrac{\pi}{2}; \dfrac{\pi}{2}\right[ der Fall. Da aus Bemerkung 1 auch folgt, dass die Wertebereiche der Umkehrfunktionen den Definitionsbereichen der ursprünglichen Funktionen entsprechen, kennen wir somit die Wertebereiche der Arkusfunktionen.

 

Animierte Grafik: 

Sinus und Cosinus am Einheitskreis.gif (525 KB)

 

Weitergehende Informationen zum Thema:

trig0.pdf (210 KB)

Übersicht:

 

21.3 Trigonometrie - Lösungen

Eine Bemerkung vorab: Da die Variablen bei diesen Aufgaben alle für geometrische Objekte (meistens Strecken) stehen, dürfen wir davon ausgehen, dass sie eine messbare Länge haben. Beim Umformen der Formeln (z. B. bei Aufgabe 5.1) können eine Division durch oder eine Multiplikation mit 0 also nicht auftreten.

 

1. Aufgabe

rechtwinkliges Dreieck mit klassischen Bezeichnungen

Sehr wichtig: Alle Winkel in dieser Aufgabe sind im Bogenmaß angegeben. Achten Sie also darauf, dass Ihr Taschenrechner entsprechend eingestellt ist!

Bemerkung 1: Bei diesen Aufgaben sind viele verschiedene Lösungswege möglich, z. B. können neben anderen trigonometrischen Funktionen auch der Satz des Pythagoras und der Winkelsummensatz benutzt werden, um die gesuchten Elemente der Dreiecke zu berechnen. Der Einsatz des Winkelsummensatzes hat den Vorteil, dass der Rechenweg schneller und die Ergebnisse genauer sind, als wenn man die trigonometrischen Funktionen nutzt, weil hierbei quasi zwangsläufig Rundungsfehler entstehen.
Bemerkung 2: Um solche Rundungsfehler so klein wie möglich zu halten, sollte - soweit es geht - mit den gegebenen Werte gerechnet werden bzw. nicht zu früh zu stark gerundet werden.


1)
Berechne c:
\begin{array}{rclcl} \cos(\alpha) &=& \dfrac{b}{c} & \vert & \cdot c \cr c \cdot \cos(\alpha) &=& b & \vert & : \cos(\alpha) \cr c &=& \dfrac{b}{\cos(\alpha)} \cr\cr c &=& \dfrac{4}{\cos \left( \dfrac{11 \pi}{36} \right)} \cr c & \approx & 6{,}97 \, cm \end{array}

Berechne a:
\begin{array}{rclcl} \sin(\alpha) &=& \dfrac{a}{c} & \vert & \cdot c \cr a &=& c \cdot \sin(\alpha) \cr a &\approx& 6{,}97 \cdot \sin \left( \dfrac{11 \pi}{36} \right) \cr a & \approx & 5{,}71 \, cm \end{array}

Berechne \beta:
\begin{array}{rclcl} \beta &=& \pi-\dfrac{\pi}{2}-\dfrac{11\pi}{36} \cr\cr \beta &=& \dfrac{7\pi}{36} \quad = \quad 35^\circ \end{array}

Bemerkung: Wenn Sie a \approx 0{,}07 \, cm und c \approx 4{,}00 \, cm berechnet haben, ist Ihr Taschenrechner auf Gradmaß eingestellt. Dass diese Ergebnisse nicht richtig sein können, merken Sie daran, dass dann der Satz des Pythagoras nicht gilt: 0{,}07^2 + 4^2 \neq 4{,}00^2. Das muss in einem rechtwinkligen Dreieck wie diesem aber so sein.

 
2)
Berechne a:
\begin{array}{rclcl} \sin(\alpha) &=& \dfrac{a}{c} & \vert & \cdot c \cr a &=& c \cdot \sin(\alpha) \cr a &=& 11 \cdot \sin \left( \dfrac{\pi}{8} \right) \cr a & \approx & 4{,}21 \, cm \end{array}

Berechne b:
\begin{array}{rclcl} \cos(\alpha) &=& \dfrac{b}{c} & \vert & \cdot c \cr b &=& c \cdot \cos(\alpha) \cr b &=& 11 \cdot \cos \left( \dfrac{\pi}{8} \right) \cr b & \approx & 10{,}16 \, cm \end{array}

Berechne \beta:
\begin{array}{rclcl} \beta &=& \pi-\dfrac{\pi}{2}-\dfrac{\pi}{8} \cr\cr \beta &=& \dfrac{3\pi}{8} \quad = \quad 67{,}5^\circ \end{array}

Bemerkung: Wenn Sie a \approx 0{,}08 \, cm und b \approx 11{,}00 \, cm berechnet haben, ist Ihr Taschenrechner auf Gradmaß eingestellt. Dass diese Ergebnisse nicht richtig sein können, merken Sie daran, dass man in diesem Fall über die trigonometrischen Funktionen \beta=90^\circ berechnet, sodass sich für die Winkelsumme \alpha + \beta + \gamma = \frac{\pi}{2} + 90^\circ + 90^\circ = 202{,}5^\circ ergibt.

 
3)
Berechne \alpha:
\begin{array}{rclcl} \alpha &=& \arctan\left(\dfrac{a}{b}\right) \cr\cr \alpha &=& \arctan\left(\dfrac{5}{8} \right) \cr \alpha & \approx & 0{,}56 \quad = \quad 32{,}0^\circ \end{array}

Berechne \beta:
\begin{array}{rclcl} \beta &=& \arctan\left(\dfrac{b}{a}\right) \cr\cr \beta &=& \arctan\left(\dfrac{8}{5} \right) \cr \beta & \approx & 1{,}01 \quad = \quad 58{,}0^\circ \end{array}

Berechne c:
\begin{array}{rclcl} \cos(\beta) &=& \dfrac{a}{c} & \vert & \cdot c \cr c \cdot \cos(\beta) &=& a & \vert & : \cos(\beta) \cr c &=& \dfrac{a}{\cos(\beta)} \cr\cr c &\approx& \dfrac{5}{\cos(1{,}01)} \cr c & \approx & 9{,}40 \, cm \end{array}

Bemerkung: Wenn Sie c \approx 5{,}00 \, cm berechnet haben, ist Ihr Taschenrechner auf Gradmaß eingestellt.

 
4)
Berechne a:
\begin{array}{rclcl} \cos(\beta) &=& \dfrac{a}{c} & \vert & \cdot c \cr a &=& c \cdot \cos(\beta) \cr a &=& 6{,}7 \cdot \cos \left( \dfrac{3 \pi}{5} \right) \cr a & \approx & -2{,}07 \end{array}

Bemerkung: Bei diesem Ergebnis sollte man stutzig werden. Es ist rechnerisch korrekt, trotzdem kann offensichtlich etwas nicht stimmen. Schaut man sich die zwei bislang bekannten Winkel an (da wir von einem rechtwinkligen Dreieck ausgehen, ist \gamma = \dfrac{\pi}{2} = 90^\circ), stellt man fest, dass \beta + \gamma = \frac{3 \pi}{5} + \frac{\pi}{2} = \frac{6\pi}{10} + \frac{5\pi}{10} = \frac{11\pi}{10} > \pi. Aus den gegebenen Werten lässt sich also gar kein Dreieck konstruieren, da die Summe dieser beiden Winkel bereits die Dreieckswinkelsumme von \pi= 180^\circ überschreitet. Wir brauchen also nicht nicht weiterzurechnen ...

 
5)
Berechne \alpha:
\begin{array}{rclcl} \alpha &=& \arccos \left(\dfrac{b}{c}\right) \cr \alpha &=& \arccos \left(\dfrac{3}{7{,}5} \right) \cr \alpha & \approx & 1{,}16 \quad = \quad 66{,}4^\circ \end{array}

Berechne \beta:
\begin{array}{rclcl} \beta &=& \arcsin\left(\dfrac{b}{c}\right) \cr \beta &=& \arcsin\left(\dfrac{3}{7{,}5} \right) \cr \beta & \approx & 0{,}41 \quad = \quad 23{,}6^\circ \end{array}

Berechne a:
\begin{array}{rclcl} \tan(\alpha) &=& \dfrac{a}{b} & \vert & \cdot b \cr a &=& b \cdot \tan(\alpha) \cr a &\approx& 3 \cdot \tan(1{,}16) \cr a & \approx & 6{,}87 \, cm \end{array}

Bemerkung: Wenn Sie a \approx 0{,}06 \, cm berechnet haben, ist Ihr Taschenrechner auf Gradmaß eingestellt.

 

6) 
Berechne a:
\begin{array}{rclcl} \tan(\beta) &=& \dfrac{b}{a} & \vert & \cdot a \cr a \cdot \tan(\beta) &=& b & \vert & : \tan(\beta) \cr a &=& \dfrac{b}{\tan(\beta)} \cr a &=& \dfrac{12}{\tan\left(\dfrac{\pi}{6}\right)} \cr a &=& 12\sqrt{3} \, cm \approx 20{,}78 \, cm \end{array}

Berechne \alpha:
\alpha = \pi-\dfrac{\pi}{6}-\dfrac{\pi}{2} = \dfrac{\pi}{3} =60^\circ

Berechne c:
\begin{array}{rclcl} \sin(\beta) &=& \dfrac{b}{c} & \vert & \cdot c \cr c \cdot \sin(\beta) &=& b & \vert & : \sin(\beta) \cr c &=& \dfrac{b}{\sin(\beta)} \cr c &=& \dfrac{12}{\sin\left(\dfrac{\pi}{6}\right)} \cr c &=& 24 \, cm \end{array}

Bemerkung: Wenn Sie a \approx 1.313{,}09 \, cm und c \approx 1.313{,}14 \, cm berechnet haben, ist Ihr Taschenrechner auf Gradmaß eingestellt.


7) 
Bei diesen Angaben können die Seitenlängen nicht berechnet werden, da dafür mindestens eine Seite gegeben sein müsste.


8) 
Berechne b:
\begin{array}{rclcl} \cos(\alpha) &=& \dfrac{b}{c} & \vert & \cdot c \cr b &=& c \cdot \cos(\alpha) \cr b &=& 5 \cdot \cos(15^{\circ}) \cr b &\approx& 4{,}83 \, cm \end{array}

Berechne \beta:
\beta = 180^{\circ}-90^{\circ}-15^{\circ}=75^{\circ}

Berechne a:
\begin{array}{rclcl} \sin(\alpha) &=& \dfrac{a}{c} & \vert & \cdot c \cr a &=& c \cdot \sin(\alpha) \cr a &=& 5 \cdot \sin(15^{\circ}) \cr a &\approx& 1{,}29 \, cm \end{array}

Bemerkung: Wenn Sie a \approx -3{,}80 \, cm und b \approx 3{,}25 \, cm berechnet haben, ist Ihr Taschenrechner auf Bogenmaß eingestellt. Dass diese Ergebnisse nicht richtig sein können, sollte offensichtlich sein.


9) 
Aus diesen Werten ergibt sich kein Dreieck, weil die Hypotenuse immer länger sein muss als die Katheten.


10)
Berechne \alpha:
\alpha = 180^{\circ}-90^{\circ}-36^{\circ}=54^{\circ}

Berechne b:
\begin{array}{rclcl} \tan(\beta) &=& \dfrac{b}{a} & \vert & \cdot a \cr b &=& a \cdot \tan(\beta) \cr b &=& 9{,}3 \cdot \tan(36^{\circ}) \cr b & \approx & 6{,}76 \, cm \end{array}

Berechne c:
\begin{array}{rclcl} \sin(\alpha) &=& \dfrac{a}{c} & \vert & \cdot c \cr c\cdot \sin(\alpha) &=& a & \vert & : \sin(\alpha) \cr c &=& \dfrac{a}{\sin(\alpha)} \cr c &=& \dfrac{9{,}3}{\sin(54^{\circ})} \cr c & \approx & 11{,}50 \, cm \end{array}

Bemerkung: Wenn Sie b \approx 72{,}08 \, cm und c \approx -16{,}64 \, cm berechnet haben, ist Ihr Taschenrechner auf Bogenmaß eingestellt.

  

2. Aufgabe

In der Mathematik ist es - anders als in der Physik - üblich, die Einheiten während der Rechnung nicht hinzuschreiben. Erst im Antwortsatz, der bei Textaufgaben dazugehört, muss die Einheit notiert werden.

Für solche Aufgaben sollte man immer eine Skizze anfertigen:
Trapez mit Beschriftungen
Gegeben sind die Seiten a = 10, b = d = 5 und c = 4.

Die Höhe h ist der Abstand der parallelen Seiten. Um sie mittels des Satzes des Pythagoras zu berechnen, benötigen wir die Länge der Strecke x. Da das Trapez symmetrisch ist, ist x genauso halb so lang, wie der Unterschied zwischen der unteren und der oberen Seite des Trapezes: x = \dfrac{a-c}{2} = \dfrac{10-4}{2} =3. Dann berechnet man:
\begin{array}{rcl} x^2+h^2 &=& d^2 \cr h &=& \sqrt{d^2-x^2} \cr h &=& \sqrt{5^2-3^2} \cr h &=& \sqrt{16} \cr h &=& 4 \end{array}

Für die Berechnung der Winkel nutzt man den Sinus und den Winkelsummensatz für Vierecke:
\begin{array}{rclcl} \sin{(\alpha)} &=& \dfrac{h}{d} \cr\cr \sin{(\alpha)} &=& \dfrac{4}{5} \cr \sin{(\alpha)} &=& 0{,}8 \cr \alpha &\approx & 53{,}13^\circ \end{array}

Aufgrund der Symmetrie muss dann auch \beta\approx 53{,}13^\circ sein.

\begin{array}{rcrcl}\gamma &=& \delta &=&\dfrac{360^\circ-\alpha-\beta}{2} \cr\cr \gamma &=& \delta &=&\dfrac{360^\circ-53{,}13^\circ-53{,}13^\circ}{2} \cr \gamma &=& \delta &=&126{,}87^\circ\end{array}

Ergebnisse:
Die Höhe h des Trapezes ist 4\,cm lang.
Die Winkel \alpha und \beta haben jeweils eine Größe von 53{,}13^\circ und \gamma und \delta von je 126{,}87^\circ.

 

3. Aufgabe

Einige Überlegungen zu dieser Aufgabe:

  • Der grundsätzliche Lösungsweg ist ähnlich zu dem, der für Parabeln beschrieben wurde, d. h. Summanden oder Faktoren an den entsprechenden Stellen habe auf trigonometrische Funktionen die gleiche Wirkung wie auf Parabeln.

  • Nützlich ist zudem, sich zu überlegen, wo Nullstellen, Minima, Maxima und Polstellen der modifizierten Sinus-, Kosinus- und Tangenskurven liegen bzw. wie sich die Funktionswerte der Minima und Maxima sowie die Periodenlänge verändern. Man kommt zu Überlegungen derart:
    • f(x)=\sin(x) hat bei x=\dfrac{\pi}{2} ein Maximum mit f\left(\dfrac{\pi}{2}\right)=1 . Folglich muss f_1(x)=3\sin(x)+1 ebenfalls bei x=\dfrac{\pi}{2} ein Maximum haben, denn f_1\left(\dfrac{\pi}{2}\right) = 3\sin\left(\dfrac{\pi}{2}\right)+1 = 3 \cdot 1+1 = 4

    • f(x)=\cos(x) hat eine Periodenlänge von 2\pi. Folglich muss auch f_2(x)=\cos\left(x+\dfrac{\pi}{2}\right) eine Periodenlänge von 2\pi haben, da die Funktion nicht gestaucht oder gestreckt, sondern nur verschoben wurde.

    • f(x)=\sin(x) hat bei x=0 eine Nullstelle. Folglich muss f_3(x)=-2\sin(x) ebenfalls bei x=0 eine Nullstelle haben, denn f_3(0)= -2\sin(0) = -2 \cdot 0.

    • f(x)=\tan(x) hat bei x=\dfrac{\pi}{2} eine Polstelle. Folglich muss f_4(x)=\tan\left(x-\dfrac{\pi}{2}\right) bei x=\pi eine Polstelle haben, denn f_4(\pi)=\tan\left(\pi-\dfrac{\pi}{2}\right)=\tan\left(\dfrac{\pi}{2}\right).

  • Grundsätzlich kann beim Zeichnen von Funktionsgraphen eine Wertetabelle helfen.


Der Übersichtlichkeit wegen wurden die Funktionsgraphen in zwei Koordinatensysteme gezeichnet.
1) f_1(x)=3 \sin(x)+1
 
2) f_2(x)=\cos \left(x+\dfrac{\pi}{2} \right)
 
3) f_3(x)=-\sin(2x)
 
4) f_4(x)=\tan \left(x-\dfrac{\pi}{2} \right)
 
5) f_5(x)=-2 \cos \left( \dfrac{1}{2}x-\dfrac{\pi}{2} \right)

Lösungen für f1(x), f2(x), f3(x)

Lösungen von f4(x), f5(x)

 

4. Aufgabe

rechtwinkliges Dreieck mit typischen Bezeichnungen

Mit den Bezeichnungen an diesem Dreieck gilt nach den Definitionen für Sinus und Kosinus:

\begin{array}{rclcl} \sin(\alpha) &=& \dfrac{a}{c} &\vert& \cdot c \cr a &=& c\cdot \sin(\alpha) \cr \cr \cos(\alpha) &=& \dfrac{b}{c} &\vert& \cdot c \cr b &=& c\cdot \cos(\alpha) \end{array}


Nach dem Satz des Pythagoras gilt:

a^2+b^2 =c^2


Setzt man in diese Gleichung die oben gefundenen Ausdrücke für die Seiten a und b ein, erhält man:

\begin{array}{rclcl} \left(c\cdot\sin(\alpha)\right)^2+\left(c\cdot\cos(\alpha)\right)^2 &=& c^2 \cr\cr c^2\cdot\sin^2(\alpha)+c^2\cdot\cos^2(\alpha) &=& c^2 \cr\cr c^2\left(\sin^2(\alpha)+\cos^2(\alpha)\right) &=& c^2 &\vert& : c^2 \cr\cr \sin^2(\alpha)+\cos^2(\alpha) &=& 1 \end{array}


Bemerkung 1: \sin^2(\alpha) steht für \left(\sin(\alpha)\right)^2 und nicht für \sin\left(\alpha^2\right)
 
Bemerkung 2: Es darf hier durch c^2 geteilt werden, weil die Länge der Hypotenuse immer größer als 0 ist - sonst wäre es ja kein Dreieck.

22. Ableitungen - Lernziele und typische Fehler

Nach Durcharbeiten dieses Kapitels sollten Sie folgende Lernziele erreicht haben:

  • Sie erkennen die Notwendigkeit der Differenzialrechnung und können erklären, was Ableitungen anschaulich bedeuten.
  • Sie können qualitativ die Steigungsfunktion zu einer gegebenen Funktion bestimmen.
  • Sie kennen die grundlegenden Ableitungsregeln und können diese anwenden.
  • Für Profis: Sie kennen auch die weitergehenden Ableitungsregeln und können diese anwenden.
  • Sie wissen, dass nicht alle Funktionen an jeder Stelle abgeleitet werden können und wie sich das im Funktionsverlauf widerspiegelt.
  • Sie kennen die Bedingungen für Extremstellen und können Extremstellen berechnen.
  • Sie können anhand des Graphen der Ableitungsfunktion Aussagen über die Funktion treffen und umgekehrt.


Typische Fehler in diesem Kapitel sind:

  • Die Potenzregel wird bei negativen und gebrochenen Exponenten nicht richtig angewendet. Erklärung
  • Exponentialfunktionen werden nach der Potenzregel abgeleitet. Erklärung
  • Die Produktregel wird nicht richtig angewendet. Erklärung
  • Die Quotientenregel wird nicht richtig angewendet. Erklärung
  • Die Kettenregel wird nicht richtig angewendet. Erklärung


Für Online-Selbsttests zu diesem Thema und weitere Informationen zur Mathematikunterstützung an der TH Wildau nutzen Sie bitte den Moodle-Kursraum "SOS Mathematik - Differenzialrechnung".

Übersicht:

 

22.1 Ableitungen - Aufgaben

1. Aufgabe

Ermitteln Sie grafisch die Ableitung dieser Funktion!
Bei dieser Aufgabe ist es nicht das Ziel, konkrete Zahlenwerte für die Steigung zu ermitteln; es soll lediglich die Größenordnung der Steigung an einer bestimmten Stelle im Vergleich zu anderen eingeschätzt werden, um ein Gefühl für das Konzept "Ableitung" zu bekommen.

ein Graph im Koordinatensystem

 

2. Aufgabe

Leiten Sie die folgenden (einfachen) Funktionen ab! Geben Sie dabei jeweils die benötigen Ableitungsregeln an!

1) f(x)=8x^2-21

  11) f(x)=\dfrac{6}{x}

2) f(x)=-10x^3-x^2

  12) f(x)=3\sqrt{x}-7

3) f(x)=-\dfrac{10}{9}x^{18}-11x

  13) f(x)=x \cdot \sqrt{x}

4) f(x)=2x^2-x-1

  14) f(x)=\dfrac{1}{x^2}

5) f(x)=-3x^4+16x^2+6

  15) f(x)=-\dfrac{1}{12x^3}

6) f(x)=\dfrac{1}{4}x^4-\dfrac{1}{3}x^3+\dfrac{1}{2}x^2-x+5

  16) f(x)=-\dfrac{17}{x^{19}}

7) f(x)=\dfrac{2x^{14}}{7}

  17) f(x)=5\cdot \sqrt[4]{x}-x+1

8) f(x)=230x^{12}+17x^6-\frac{3}{5}x^5+400

  18) f(x)=\dfrac{1}{6 \sqrt{x}}

9) f(x)=25x^4-66x^{\frac{1}{3}}-17x^{\frac{3}{2}}+e

  19) f(x)=\dfrac{15}{\sqrt[5]{x^6}}

10) f(x)=x^{-3}+18

  20) f(x)=7x^a+c

 

3. Aufgabe

Leiten Sie die folgenden Funktionen mit der Produktregel ab!

1) f(x) = \left(x-1\right)\cdot\left(\dfrac{47}{2}x^2+16x+7\right)

  11) f(x) = 5{,}5^x\cdot x^5-5

2) f(x) = \left(13x^2+\dfrac{7}{5}x-18\right)\left(11x^2-\dfrac{15}{4}x-1\right)

  12) f(x) = 19x^2\cdot\ln(x)

3) f(x) = \left(\dfrac{1}{8}x^3+5x\right)\cdot\sqrt{8}x 

  13) f(a) = -14\sqrt[4]{a}\cdot \log_4(a)

4) f(x) = 3x^2\cdot 7x^{\frac{1}{2}}+ex^{10}-105 

  14) f(x) = \dfrac{15}{x^2}\ln(x)+125

5) g(k) = \left(\dfrac{11}{6}k^2-k\right)^2\cdot 72

  15) f(x) = 2x\cdot 3\sin(x)

6) f(x) = \left(4x^2-16\right)\sqrt{23x}-5   16) f(x) = \sin(x)\cdot\cos(x)-\dfrac{3}{\pi}

7) g(x) = 2^x\sqrt[10]{1024x}   17) f(x) = ax^2\cdot 5^3\sin(x)+17ax^3
8) f(x) = 23x^2\cdot e^x   18) f(x) = e^x\cdot\sin(x)
9) f(k) = \dfrac{5}{4}k^{12}\cdot e^k-1   19) f(x) = -6x^3\tan(x)-18

10) f(y) = \dfrac{16}{\frac{4}{3}y^7 e^{-y}}-\dfrac{14}{9}   20) f(s) = -\tan(s)\left(\dfrac{s}{\pi}+\cos(s)\right)

 

4. Aufgabe

Leiten Sie die folgenden Funktionen mit der Quotientenregel ab!

1) f(x)=\dfrac{x^{4}}{100-x}

  11) f(w)=\dfrac{w-13}{-8\sqrt{w}}

2) f(x)=\dfrac{\dfrac{x^{2}}{9}+1}{-27x^{13}}

  12) f(x)=\dfrac{12\sqrt{x}}{x-10}

3) f(q)=\dfrac{\left(-12q+7\right)^{4}}{144q^{2}-168q+49}   13) f(z)=\dfrac{2^{z}-1}{4^{z}}

4) f(x)=\dfrac{x^{3}}{x+5}

  14) f(x)=\dfrac{21e^{x}}{7x+1}

5) f(x)=\dfrac{4x+2}{7x^{2}}

  15) f(x)=\dfrac{17e^{x}-1}{34x^{2}}

6) f(x)=\dfrac{3x+5}{9x+2}

  16) f(x)=-\dfrac{x^{2}+18}{18\lg\left(x\right)}

7) f(z)=\dfrac{23}{z^{-42}}

  17) f(x)=\dfrac{\ln{\left(x\right)}}{x^{11}}

8) f(x)=\dfrac{x}{2x^{2}+4}

  18) g(x)=\dfrac{-2\ln{\left(x\right)}}{x^{31}-x^{26}}

9) f(x)=\dfrac{15x^{2}-8x}{x^{3}+32}

 

19) f(y)=\dfrac{\pi\sin{\left(y\right)}}{-y^{2}}-12\pi 

10) f(x)=\dfrac{8x+3}{4x-6}

  20) y(t)=\dfrac{\sin{\left(t\right)}}{\cos{\left(t\right)}}

 

5. Aufgabe

Leiten Sie die folgenden Funktionen mit der Kettenregel ab!

1) f(x)=(2x+1)^7   11) f(x)=\sin^2(x)
2) f(x)=24\left(x+\dfrac{7}{8x}\right)^2   12) g(y)= \sqrt{\dfrac{121}{(72a-1)^2+5}}
3) f(x)=\dfrac{-63}{x^2-\frac{1}{x}}   13) f(x)=e^{x^2+\sqrt{x}}
4) g(x)=\sqrt{112x^3+97x}+16   14) f(x)=-36\sin\left(\cos(x)\right)
5) f(x)=\dfrac{1}{\sqrt{3x-6}}   15) f(x)=e^{\sin\left(-x^2+13\right)}
6) f(x)=\sqrt{(x+4)^3}   16) f(x)=85\ln\left(e^{3x}+x^2\right)-94
7) f(x)=\sqrt[3]{x+4}   17) f(z)=-\dfrac{3}{10}\sqrt{1+\cos\left(z^2\right)}
8) f(t)=\ln\left(\dfrac{5}{2}t^2+14\right)   18) f(x)=\sqrt{e^{\sqrt{x}-t}}
9) fx)=\ln\left(\dfrac{12}{7\sqrt{x}}\right)   19) f(x)=\dfrac{1}{ax+b}
10) f(x)=\ln\left(-42x^4-\dfrac{1}{18}x^3\right)
  20) f(x)=\tan\left(c\cdot \sin(x)\right)

 

6. Aufgabe

Leiten Sie die folgenden Funktionen ab! Geben Sie dabei jeweils die benötigen Ableitungsregeln an!

1) f(x)= \dfrac{1}{2x+4}

  11) f(x)=\dfrac{\sin(x)}{\cos(x^2)}

2) f(x)=\cos(7x+8)

  12) f(x)=\ln\left(\dfrac{1}{x}\right)

3) f(x)=(x^2+1)\sin(x)

  13) f(x)=\tan(x)-\cos(x)

4) f(x)=12xe^{x+10}

  14) f(x)=\lg(x^2+2)

5) f(x)=e^{\sqrt{x}}

  15) f(x)=\dfrac{3x^2-2x+1}{2}

6) f(x)=\dfrac{x^2+1}{x^3-1}

  16) f(x)=e^{-\frac{x^2}{2}}

7) f(x)=\ln(x)+7^x+\sin(x)+5

  17) f(x)=x \cdot e^{-5x^2}

8) f(x)=\cos(x^2)

  18) f(x)=\dfrac{2}{2x^2-2}

9) f(x)=\cos^2(x)

  19) f(x)=(3x^3+3)^{27}

10) f(x)=x^2 \cdot 10^x

  20) f(x)=\ln\left(ax^3+bx^2+c\right)

 

7. Aufgabe

Bestimmen Sie die Extrempunkte der folgenden Funktionen! Handelt es sich dabei jeweils um einen Tiefpunkt oder einen Hochpunkt?

1) f(x)=2x^2-12x+24

  6) f(t)=\dfrac{t^4-3t^3+5t-5}{t-1}

2) f(x)=- \dfrac{5}{2}x^2+10x+\dfrac{3}{8}

  7) f(x)=\sqrt{-6x^2+16x+1}

3) f(x)=80x^2-16x+23

  8) f(x)=e^{0{,}1x^2-8x-6{,}3}

4) f(x)=\dfrac{1}{3}x^3+2x^2-12x+6

  9) f(x)=\dfrac{1}{2 \cdot \ln(10)} \cdot 10^{-8x^4+9x^2}

5) f(x)=\dfrac{1}{2}x^4-3x^2+1

  10) f(x)=\ln \left(\dfrac{1}{2}x^3-12x\right)-15

 

8. Aufgabe

1) Welche zwei Zahlen, deren Summe 100 ist, haben das größte Produkt?


2) Welches Rechteck vom Umfang 36 hat den größten Flächeninhalt?


3) Von einem Parallelogramm seien nur die Seitenlängen a und b bekannt, womit es noch nicht eindeutig festgelegt ist.
Fertigen Sie eine (beschriftete) Skizze an!
Bestimmen Sie die maximal mögliche Fläche eines solchen Parallelogramms!

Übersicht:

 

22.2 Partielle Ableitungen - Aufgaben

1. Aufgabe

Ermitteln Sie die partiellen Ableitungen 1. und 2. Ordnung dieser Funktionen!

1) f(x,y) = 4x^2+3xy+7y^3   11) f(b,c) = 3b^2-125bc+7c^2-5b^2c^2
2) f(a,b) = a^3+3ab^2-15a-12b   12) a(x,y) = 5x^3y+10x^2y^2+2xy^3+5xy
3) f(x,y) = \left(x-2y\right)\left(2x+y\right)   13) g(x,y) = \sqrt{x}+5xy+y^2
4) f(x,y) = \left(x+3y\right)^2-2xy   14) g(x,y) = x^2\sqrt{y^3}-\sqrt{11}
5) f(x,y) = x^4-53y^3+14xy+8   15) g(d,h) = \dfrac{\pi}{4}d^2h
6) f(x,y) = 3x^5-20x^2y-10y^2   16) f(x,y,z) = -12x^2-3x^4y+34xy^5+\dfrac{1}{2}z

7) g(x,y) = x^7+\dfrac{5}{2}x^2y^3+13y   17) f(x,y,z) = \dfrac{13}{2}zx^2+3x^{12}y^7+\sqrt{78}z

8) g(x,y) = \left(5x+22\right)^2+9x^2y+y^2   18) f(a,b,c) = \dfrac{\pi}{2}a^2b^3c+17\pi ab^3+31c^2b^2

9) g(x,z) = 7x^4+8xz^2-12z^3   19) f(x,y,z) = 3ab^2xz+8abyz-\dfrac{4}{b}

10) g(b,c) = -5-8b^3+24bc+3c^8   20) f(x,y,z) = \dfrac{3}{2}x^2y^{\frac{1}{2}}z^3+4x^{\frac{1}{3}}y^2

 

2. Aufgabe

Ermitteln Sie die partiellen Ableitungen 1. und 2. Ordnung dieser Funktionen!

1) f(x,y) = 25+x\cdot e^{4xy}   11) g(a,b) = b\cdot\cos(a)+a\cdot\sin(b)-ab
2) f(x,y) = e^x+3y^2-15xy   12) g(x,y) = \dfrac{7\cos(x)}{10y}+19xy-0{,}851
3) f(x,y) = e^{3xy-23}+y\cdot\sqrt{x}   13) f(x,y) =\dfrac{6\ln(x)}{x+9y}

4) f(x,y) = \ln(xy)-17   14) g(r,t) = -16\tan(r)+e^{rt}-256

5) f(t,z) = \ln(1+t)\cdot z^2+5t^2z-\dfrac{4}{5}   15) f(x,y) = \dfrac{5}{9}e^{\sin(x)+\sin(y)}-1

6) f(x,y) = \ln\left(x+y^2\right)-e^{2xy}+3x   16) f(x,y,z)=xy^2\cdot e^{-3xy}\cdot \ln(z)
7) f(x,y) = e^{3x}\cdot\ln(y)-e^{2y}\cdot 5x   17) f(x,y,z)=300 \cdot \sqrt{x+yz}+923
8) g(x,y) = 52x\cdot e^y+\dfrac{y}{8}\cdot\ln(x)-13   18) f(x,y,z)= \dfrac{-8xz^2}{17x^2+y^2}
9) g(h,k) = 135\sin\left(h^2+k\right)-298   19) f(x,y,z)=\dfrac{\pi x\sin\left(7y^5\right)}{e^{10xz+1}}
10) f(x,y) = 4\sin(x)+\cos(-4y)+\cos\left(-\dfrac{1}{4}x+y\right)   20) f(a,b,c)=\dfrac{16a^2-b^2+25c^2}{\sqrt{16a^2-b^2+25c^2}}

Dieses Kapitel enthält die folgenden Themen:

 

22.2 Ableitungen - Erklärungen

Mathematischer Hinweis vorab: Um den Begriff "Ableitung" mathematisch korrekt zu fassen, müssten wir Grenzwerte betrachten. Darauf wird im Rahmen dieses Lernmoduls verzichtet, da hier der Umgang mit und das Berechnen von Ableitungen im Mittelpunkt stehen und ein grobes Verständnis der Hintergründe dafür ausreicht. Sollten Sie ein gesteigertes Interesse an den Hintergründen haben, können Sie Ihren Wissensdurst mit folgendem PDF-Dokument stillen: diff0.pdf (150 KB).

 

Allgemeines zu Ableitungen

Die (erste) Ableitung f'(x) (gesprochen: "f Strich von x") einer Funktion f(x) ist die Funktion, die die Steigung der Ausgangsfunktion f(x) in jedem Punkt beschreibt. Das bedeutet, dass man für jeden x-Wert der Ausgangsfunktion f(x) mithilfe ihrer ersten Ableitung f'(x) die Steigung an genau dieser Stelle bestimmen kann, wobei die Steigung nichts anderes als die Änderungsrate des Funktionswerts darstellt. Anders formuliert: Wie stark verändert sich der y-Wert, wenn sich der x-Wert um einen bestimmten Wert ändert?
Wie man die Ableitung einer gegebenen Funktion bestimmen kann und warum die Steigung der Funktion bei ihrer Analyse, z. B. im Rahmen einer Kurvendiskussion, von besonderem Interesse ist, werden wir im weiteren Verlauf des Kapitels sehen. Zuvor folgen aber ein paar Überlegungen zum Thema Steigung.

Das Konzept "Steigung" kennen wir schon aus dem Kapitel Lineare Funktionen. Bei linearen Funktionen ist die Steigung im gesamten Verlauf konstant. Sie lässt sich aus der Funktionsvorschrift ablesen oder mit einer Formel berechnen, wenn zwei Punkte der linearen Funktion bekannt sind. Auf jeden Fall ist die Bestimmung der Steigung einer linearen Funktion an einer bestimmten Stelle x ohne vorherige Herleitung der ersten Ableitung möglich. Dennoch lassen sich auch von linearen Funktionen Ableitungen bilden.
Schauen wir uns ein Beispiel an: Die Steigung der linearen Funktion f(x)=3x-5 ist 3, als Ableitung formuliert f'(x)=3. Unabhängig davon, ob man nun die Steigung an der Stelle x=1 oder x=2 oder an einer anderen beliebigen Stelle bestimmt, erhält man - wenig überraschend - als Ergebnis immer eine Steigung von 3 (als Formel: f'(1)=3 bzw. f'(2)=3).

Steigung einer Geraden

Keine Sorge ... Sollte Ihnen die Bestimmung der ersten Ableitung unbekannt sein oder sollten Sie sich daran nicht mehr erinnern können, wir werden alle notwendigen Ableitungsregeln in diesem Kapitel besprechen.

Wichtig: f(x) bedeutet etwas Anderes als f'(x). In unserem Beispiel:

  • f(2)=3 \cdot 2 - 5 = 1 bedeutet: Die Funktion f(x) hat an der Stelle x=2 den Funktionswert 1.
  • f'(2)=3 bedeutet: Die Funktion f(x) hat an der Stelle x=2 die Steigung 3.

Abgesehen von den linearen Funktionen ist die Steigung bei anderen (spannenderen ...) Funktionstypen "leider" nicht überall konstant. Sie haben solche Funktionen auch schon kennengelernt; die einfachsten unter ihnen sind die quadratischen Funktionen. Dass die Steigung bei diesen Funktionen nicht überall gleich ist, erkennt man schon daran, dass ihre Graphen in einem Abschnitt steigen und in einem anderen fallen. Die Steigung muss also zum Teil positiv und zum Teil negativ sein.
Das nächste Beispiel: Die erste Ableitung der Funktion f(x)=x^2 lautet f'(x)=2x. Somit hat die Ausgangsfunktion f(x) an der Stelle x=1 eine Steigung von f'(2)=2 \cdot 1=2 und an der Stelle x=-2 eine Steigung von f'(-2)=2 \cdot (-2)=-4.

Wie lässt sich die Steigung einer Funktion f(x) an einer bestimmten Stelle ermitteln, falls es sich nicht um eine lineare Funktion handelt?
Man kann hierfür die Tangente t(x) an dieser Stelle heranziehen. Die Tangente ist eine Gerade, die den Funktionsgraphen in einem Punkt berührt und in der Umgebung dieses Punktes die gegebene Funktion approximiert (also annähert). Im Berührpunkt haben Funktionsgraph und Tangente die gleiche Steigung. Da die Tangente eine Gerade ist, könnte man für sie die Steigung unkompliziert berechnen, z. B. über ein Steigungsdreieck m=\dfrac{\Delta y}{\Delta x}, wenn man denn zwei Punkte kennen würde. Wir kennen in diesem Fall aber nur einen Punkt, nämlich den Berührpunkt mit der Ausgangsfunktion. 

Parabel und eine Tangente mit Steigungsdreieck

Deswegen muss man an dieser Stelle eine Grenzwertbetrachtung anstellen, auf die wir aber - wie eingangs erwähnt - in diesem Lernmodul verzichten, da diese Berechnung mathematische Konzepte erfordert, die über das Lernmodul hinausgehen. Glücklicherweise gibt es diverse Ableitungsregeln, um zu einer gegebenen Funktion f(x) ihre Ableitung zu ermitteln, sodass wir uns das Ermittlung der Steigung einer Funktion an einer bestimmten Stelle durch Finden der Tangente und Bestimmung von deren Steigung ersparen können.

Bemerkung: Die Ableitung einer Funktion kennzeichnet man meist mit einem hochgestellten Strich an der Funktionsbezeichnung: Die Ableitung von f(x) heißt dann f'(x). Später wird auch die alternative Schreibweise \dfrac{df}{dx} (gesprochen: "d f nach d x") nützlich sein, wenn es sich nicht um eine Funktion mit einer Variable, sondern um eine Funktion mit mehreren Variablen handelt, die abgeleitet werden soll. 

Da die Ableitung einer Funktion f(x) (wenn sie existiert, siehe unten) wieder eine Funktion ist, nämlich f'(x), kann sie erneut abgeleitet werden. So erhält man die zweite Ableitung f''(x), die auch als Ableitung zweiter Ordnung bezeichnet wird. Analog dazu, dass man mithilfe der ersten Ableitung die Steigung der Ausgangsfunktion an einer beliebigen Stelle bestimmen kann, lässt sich mithilfe der zweiten Ableitung die Steigung der ersten Ableitung an einer beliebigen Stelle ermitteln. Die erste Ableitung ist also die Steigungsfunktion der Ausgangsfunktion und die zweite Ableitung die Steigungsfunktion der ersten Ableitung.
Leitet man die zweite Ableitung erneut ab, kommt man zur dritten Ableitung f'''(x) oder auch Ableitung dritter Ordnung usw. ... Grundsätzlich gilt, dass die Ableitung einer Funktion immer die Steigungsfunktion dieser Funktion ist, egal in welcher "Generation" wir uns befinden. Die dritte Ableitung f'''(x) ist somit die Steigungsfunktion der zweiten Ableitung f''(x). f''(x) und f'''(x) werden auch höhere Ableitungen oder Ableitungen höherer Ordnung genannt. Sie sind u. a. für die Bestimmung "besonderer Punkte" der Funktion wichtig.

 

Ableitungsregeln

Für die Berechnung von Ableitungen gibt es verschiedene Regeln, die in der folgenden Tabelle zusammengestellt sind. Es gibt in der Mathematik keine feste Reihenfolge für diese Regeln. Sie werden also in anderen Quellen möglicherweise andere Reihenfolgen finden. Die Zahl der Regeln ist - wie sich leicht erkennen lässt - sehr überschaubar. Allerdings kann das Ableiten einer Funktion trotzdem recht anspruchsvoll werden, da ggf. mehrere Regeln kombiniert angewendet werden müssen.

Zwei Bemerkungen vorab:

  • Falls erforderlich, werden in der Tabelle Teilfunktionen, aus denen die Funktion f(x) zusammengesetzt ist, mit u(x), v(x), g(x) und h(x) bezeichnet.
  • Eine Funktion kann natürlich nur dort abgeleitet werden, wo sie auch definiert ist. Korrekterweise müsste also immer der Definitionsbereich mit angegeben werden, vor allem bei den Regeln für spezielle Funktionen. Um die folgende Auflistung übersichtlich zu halten, werden wir hier darauf verzichten, weil die Definitionsbereiche in den entsprechenden Kapiteln nachgelesen werden können.
    Wichtig in diesem Zusammenhang: Die Definitionsbereiche der Funktion und ihrer Ableitung(en) können sich unterscheiden (siehe Exkurs).
  Funktion   Ableitung   Bemerkung

grundlegende Regeln
Potenzregel f(x)=x^n   f'(x)=nx^{n-1}   Die Potenzregel kann auch als die Ableitung einer Potenz- bzw. Polynomfunktion angesehen werden.
Konstantenregel f(x)=a   f'(x) =0   a \; \in \; \mathbb{R}
Faktorregel f(x)=a \cdot u(x),   f'(x)=a \cdot u'(x)   a \; \in \; \mathbb{R}
Summenregel f(x)=u (x)+v(x)   f'(x)=u'(x) + v'(x)    

weitergehende Regeln
Produktregel f(x)=u(x) \cdot v(x)   f'(x)=u'(x) \cdot v(x) + u(x) \cdot v'(x)   Da die Addition kommutativ ist, gilt natürlich auch f'(x) = u(x) \cdot v'(x) + u'(x) \cdot v(x)
Quotientenregel f(x)=\dfrac{u(x)}{v(x)}   f'(x)=\dfrac{u'(x) \cdot v (x) - u(x) \cdot v'(x)}{v^2(x)}   Um eine Division durch 0 zu verhindern, muss ggf. der Definitionsbereich so eingeschränkt werden, sodass v(x)\neq 0 ist für alle x\;\in\;\mathbb{D}.
Kettenregel f(x)=g\left(h(x)\right)   f'(x)=g'\left(h (x)\right) \cdot h'(x)    

Regeln für spezielle Funktionen
Exponentialfunktion f(x)=a^x   f'(x)=\ln(a)\cdot a^x   a\in\mathbb{R}^+\setminus_{\{1\}}
  Da \ln(e) = 1 ist, ergibt sich aus dieser allgemeinen Regel die Ableitung für Exponentialfunktionen zur Basis e folgendermaßen:
  f(x)=e^x   f'(x)=e^x    
Logarithmusfunktion f(x)=\log_a(x)   f'(x)=\dfrac{1}{x\cdot \ln(a)}   a\in\mathbb{R}^+\setminus{_{\{1\}}}
  Da \log_e(x) = \ln(x) und \ln(e) = 1 ist, ergibt sich aus dieser allgemeinen Regel die Ableitung für Logarithmusfunktionen zur Basis e folgendermaßen:
  f(x)= \ln(x)   f'(x)=\dfrac{1}{x}    
Sinusfunktion f(x)=\sin(x)   f'(x)=\cos(x)    
Kosinusfunktion f(x)=\cos(x)   f'(x)=- \sin(x)    
Tangensfunktion f(x)=\tan(x)   f'(x)= \dfrac{1}{(\cos(x))^2}    

 

Beispiele für die Ableitungsregeln

Korrekterweise müsste auch bei den Beispielen immer der Definitionsbereich mit angegeben werden. Um den Fokus auf die Anwendung der Ableitungsregeln zu legen, werden wir wieder darauf verzichten.

Technischer Hinweis: Wirken Terme oder Texte rechts abgeschnitten, so liegt das an der Breite Ihres Browser-Fensters. Sie können dann z. B. durch Drücken der Umschalttaste (auch Shift-Taste genannt) und gleichzeitiges Drehen am Mausrad nach rechts scrollen.

Beispiele für die Anwendung der grundlegenden Regeln 


Potenzregel

Funktion   Ableitung    
f(x) = x^2   \begin{array}{rcl}f'(x) & = & 2 \cdot x^{2-1} \\\\& = & 2 \cdot x^1 = 2 \cdot x \end{array}

   
\begin{array}{rcl}f(x) & = & x \\\\& = & x^1 \end{array}   \begin{array}{rcl}f'(x) & = & 1 \cdot x^{1-1} \\\\& = & 1 \cdot x^0 = 1 \cdot 1 = 1 \end{array}

   

Dass die Ableitung dieser Funktion f'(x) = 1 lautet, ist wenig überraschend, da es sich bei der Ausgangsfunktion ja um eine lineare Funktion mit der Steigung 1 handelt.

Die Potenzregel kann auch angewendet werden, wenn der Exponent nicht ganzzahlig und positiv, sondern negativ oder gebrochen ist:

\begin{array}{rcl}f(x) & = & \dfrac{1}{x^6} \\\\& = & x^{-6} \end{array}

  \begin{array}{rcl}f'(x) & = & -6 \cdot x^{-6-1} \\\\& = & -6x^{-7} = -\dfrac{6}{x^7} \end{array}    

Ganz wichtig: Es ist hier keine Option, Zähler und Nenner getrennt voneinander abzuleiten. Wenn Sie die Umformung mithilfe der Potenzgesetze nicht durchführen, müssen Sie zum Ableiten die Quotientenregel (siehe unten) anwenden, was üblicherweise komplizierter ist. Das sichere Beherrschen der Potenzgesetze ist also an dieser Stelle sehr hilfreich. 

Bemerkung: Achten Sie beim Zusammenrechnen im Exponenten darauf, dass -6-1=-7 und nicht -5 ist.

\begin{array}{rcl}f(x) & = & \sqrt{x} \\\\& = & x^{\frac{1}{2}} \end{array}

  \begin{array}{rcl}f'(x) & = & \dfrac{1}{2} \cdot x^{\frac{1}{2}-1} \\\\ &=& \dfrac{1}{2} \cdot x^{-\frac{1}{2}} = \dfrac{1}{2x^{\frac{1}{2}}} = \dfrac{1}{2\sqrt{x}} \end{array}    
Die Ableitung von f(x)=\sqrt{x} werden Sie möglicherweise in Ihrer Formelsammlung finden, sodass hier nicht zwingend die Anwendung der Potenzgesetze und die Kenntnis der Potenzregel notwendig gewesen wäre. Allerdings gibt es auch komplexere Wurzelfunktionen, zu denen Sie in der Formelsammlung keine Ableitung finden werden. Deswegen ist sinnvoll und hilfreich, auch die Anwendung der Potenzregel auf Wurzelausdrücke zu beherrschen.

\begin{array}{rcl}f(x) & = & \dfrac{1}{\sqrt[3]{x^2}} \\\\& = & x^{-\frac{2}{3}} \end{array}

  \begin{array}{rcl}f'(x) & = & -\dfrac{2}{3} x^{-\frac{2}{3}-1} \\\\ & = & -\dfrac{2}{3} x^{-\frac{5}{3}} = -\dfrac{2}{3 \sqrt[3]{x^5}} \end{array}    

Konstantenregel

f(x) = -32   f'(x) = 0    
f(x) ist eine konstante Funktion, also eine Parallele zur x-Achse. Da kann bei der Steigung nicht viel passieren ...


Faktorregel

f(x) = 5\cdot x^2   f'(x) = 5\cdot 2x = 10x    
Der Faktor, hier die 5, bleibt beim Berechnen der Ableitung als Faktor bestehen.


Summenregel

f(x) = x^3+x^2   f'(x) = 3x^2+2x    
Die beiden Summanden werden jeder für sich abgeleitet und anschließend wieder addiert.


Kombination mehrerer grundlegender Regeln


Bei den meisten Funktionen müssen mehrere grundlegende Regeln kombiniert angewendet werden:
\begin{array}{rcl}f(x) & = & x^3+\dfrac{5}{x^2}-\ln(5) \\\\& = & x^3 + 5 \cdot x^{-2}-\ln(5) \end{array}

  \begin{array}{rcl}f'(x) & = & 3x^2-10x^{-3} - 0\\\\& = & 3x^2 - \dfrac{10}{x^3} \end{array}

   
Bitte beachten Sie, dass \ln(5)\approx 1{,}61 ein konstanter Wert ist und somit laut Konstantenregel beim Ableiten zu 0 wird.


Beispiele für die Anwendung der weitergehenden Regeln


Ein Tipp vorab: Bei Funktionen, die mit den weitergehenden Regeln abgeleitet werden müssen, ist es immer nützlich, die Teilfunktionen und ihre Ableitungen separat aufzuschreiben und Schritt für Schritt, ggf. auch mit Nebenrechnungen, vorzugehen (siehe rechte Spalte).


Produktregel

Funktion   Ableitung   ggf. Nebenrechnung
f(x) = x^3 \cdot x^2   \begin{array}{rcl}f'(x) & = & 3x^2 \cdot x^2 + x^3 \cdot 2x \\\\& = & 3x^4 + 2x^4 \\\\& = & 5x^4 \end{array}  

\begin{array}{rcl}u(x) & = & x^3\\u'(x) & = & 3x^2\\\\v(x) & = & x^2\\v'(x) & = & 2x \end{array}

Möglicherweise ist Ihnen in den Sinn gekommen, die Ausgangsfunktion mithilfe der Potenzgesetze wie folgt zusammenzufassen: f(x) = x^5. Die Multiplikation der beiden Faktoren ist natürlich möglich und führt beim Ableiten zu einer (erheblichen) Vereinfachung, da die Anwendung der Produktregel dann nicht nötig ist. Der einfache Gebrauch der Potenzregel reicht aus und führt (natürlich) zum gleichen Ergebnis.

f(x) = \left(x^3+5x^2-32\right)\left(x^4-x\right)  

\begin{array}{rcl}f'(x) & = & \left(3x^2+10x\right)\left(x^4-x\right) + \left(x^3+5x^2-32\right)\left(4x^3-1\right) \\\\& = & 3x^6-3x^3+10x^5-10x^2 + 4x^6-x^3+20x^5-5x^2-128x^3+32 \\\\& = & 7x^6+30x^5-132x^3-15x^2+32 \end{array}

 

\begin{array}{rcl}u(x) & = & x^3+5x^2-32\\u'(x) & = & 3x^2+10x\\\\v(x) & = & x^4-x\\v'(x) & = & 4x^3-1 \end{array}

Wichtig: Das Ableiten von Produkten geht nicht so einfach wie das Ableiten von Summen. Daher ist es für das Ableiten günstig, eine Funktion vorab - wenn möglich - so weit es geht auszumultiplizieren, sodass möglichst wenig Produkte übrig sind. Schauen wir uns dazu noch mal das vorhergehende Beispiel an:

\begin{array}{rcl}f(x) &=& \left(x^3+5x^2-32\right)\left(x^4-x\right) \\\\ &=& x^7-x^4+5x^6-5x^3-32x^4+32x \\\\ &=& x^7+5x^6-33x^4-5x^3+32x \end{array}


Wie man sieht, ist das Ausmultiplizieren wirklich eine gute Idee, um das Ableiten kürzer und leichter zu gestalten. Nun kann die Funktion nur durch Anwendung der grundlegenden Regeln abgeleitet werden. Das Ergebnis ist identisch mit dem Ergebnis, das unter Anwendung der Produktregel erzielt wurde. Deswegen haben wir auf die Darstellung hier verzichtet.

Allerdings lässt sich nicht jede Funktion in dieser Art und Weise vereinfachen. Wenn das nicht geht, z. B. bei f(x) =x\cdot e^x, muss die Produktregel angewendet werden. Beide Faktoren einzeln abzuleiten und dann zu multiplizieren, ist keine Alternative!

f(x) = x^{169} \cdot e^x   \begin{array}{rcl}f'(x) & = & 169x^{168} \cdot e^x + x^{169} \cdot e^x \\\\& = & x^{168}e^x(169+x) \end{array}  

\begin{array}{rcl}u(x) & = & x^{169}\\u'(x) & = & 169x^{168}\\\\v(x) & = & e^x\\v'(x) & = & e^x \end{array}

Bitte beachten Sie, dass Exponentialfunktionen eine eigene Ableitungsregel haben und nicht nach der Potenzregel abgeleitet werden. Es führt nicht zum richtigen Ergebnis, wenn man den Exponenten als Faktor vorzieht und so weiter!


Quotientenregel

f(x)=\dfrac{1}{x^6}   f'(x) = \dfrac{0 \cdot x^6 - \left(1 \cdot 6x^5\right)}{\left(x^6\right)^2} = \dfrac{-6x^5}{x^{12}} = -\dfrac{6}{x^7}  

\begin{array}{rcl}u(x) & = & 1\\u'(x) & = & 0\\\\v(x) & = & x^6\\v'(x) & = & 6x^5 \end{array}

Dieses Beispiel hatten wir oben schon mithilfe der Potenzregel gelöst, nachdem wir den Funktionsterm unter Verwendung der Potenzgesetze in f(x) = x^{-6} umgewandelt hatten. Die Ableitung ist (natürlich) in beiden Fällen gleich. Es bleibt Ihnen selbst überlassen, welchen Weg Sie wählen ...

Wichtig: Bei der Quotientenregel steht im Zähler - anders als bei der Produktregel - ein Minuszeichen zwischen den beiden Termen. Deshalb muss unbedingt darauf geachtet werden, dass

  1. u'(x) \cdot v(x) vor dem Minuszeichen und u(x) \cdot v'(x) danach steht.
  2. der Subtrahend, also der Term hinter dem Minuszeichen, in Klammern gesetzt wird. Das ist zwar nicht in jedem Fall notwendig, aber niemals falsch. Werden die Klammern nicht gesetzt, besteht die große Gefahr, dass die Ableitung selbst oder nachfolgende Termumformungen schief gehen ...

f(x) = \dfrac{x^3+5x^2-32}{x^4-x}

  \begin{array}{rcl} f'(x) &=& \dfrac{\left(3x^2+10x\right)\left(x^4-x\right)-\left(\left(x^3+5x^2-32\right)\left(4x^3-1\right)\right)}{\left(x^4-x\right)^2} \\\\&=& \dfrac{3x^6-3x^3+10x^5-10x^2-\left(4x^6-x^3+20x^5-5x^2-128x^3+32\right)}{\left(x^4-x\right)^2}\\\\&=& \dfrac{3x^6-3x^3+10x^5-10x^2-4x^6+x^3-20x^5+5x^2+128x^3-32}{\left(x^4-x\right)^2} \\\\&=& \dfrac{-x^6-10x^5+126x^3-5x^2-32}{\left(x^4-x\right)^2} \end{array}

 

\begin{array}{rcl}u(x) & = & x^3+5x^2-32\\u'(x) & = & 3x^2+10x\\\\v(x) & = & x^4-x\\v'(x) & = & 4x^3-1 \end{array}

Auch hier ist es keine Option, Zähler und Nenner separat abzuleiten und dann durcheinander zu teilen! Bei allen Funktionen, bei denen eine Teilfunktion im Nenner steht, muss die Quotientenregel angewendet werden.

f(z) = \dfrac{\ln(z)}{z}   \begin{array}{rcl} f'(z) & = & \dfrac{\dfrac{1}{z} \cdot z - \left(\ln(z) \cdot 1\right)}{z^2} \\\\& = & \dfrac{1 - \ln(z)}{z^2} \end{array}  

\begin{array}{rcl}u(z) & = & \ln(z)\\u'(z) & = & \dfrac{1}{z}\\\\v(z) & = & z\\v'(z) & = & 1 \end{array}

Lassen Sie sich nicht davon irritieren, dass die Variable hier z (und nicht wie gewohnt x) heißt. Alle Ableitungsregeln gelten vollkommen unverändert. So lautet z. B. die erste Ableitung der Logarithmusfunktion f(z)=ln(z) demzufolge f'(z)=\dfrac{1}{z}.


Kettenregel

Eine grundlegende Überlegung zuerst: Woran erkennt man, dass eine Funktion verkettet ist?
Ein gutes Indiz ist, dass in einer Potenz, unter einer Wurzel, im Argument einer Logarithmus-, Sinus-, Kosinus- etc. Funktion nicht nur eine Variable steht, sondern eine weitere Funktion, wie ein Polynom. Würde man in so einer Situation Funktionswerte berechnen, müsste man erst den Wert der inneren Funktion (z. B. des Polynoms) bestimmen und anschließend diesen Wert in die Potenz / Wurzel / ... einsetzen.

f(x)=(x^3+5x^2-32)^4   \begin{array}{rcl} f'(x) &=& 4\left(h(x)\right)^3 \cdot h'(x) \\\\& = & 4\left(x^3+5x^2-32\right)^3\cdot\left(3x^2+10x\right) \\\\& = & \left(12x^2+40x\right)\cdot\left(x^3+5x^2-32\right)^3 \end{array}  

\begin{array}{rcl}g(h(x)) & = & (h(x))^4\\g'(h(x)) & = & 4(h(x))^3\\\\h(x) & = & x^3+5x^2-32\\h'(x) & = & 3x^2+10x \end{array}

Hier steht das Polynom in der Basis einer Potenz. Würde man also einen Funktionswert bestimmen wollen, müsste zuerst das Polynom in der Basis berechnet werden, um danach den Wert der gesamten Potenz auszurechnen. Das ist typisch für verkettete Funktionen. Deswegen werden das Polynom in der Basis als die innere Funktion h(x) und die Potenz als die äußere Funktion g(x), die die innere Funktion umschließt, bezeichnet. Man schreibt daher auch g(h(x)).

Alternativen zur Kettenregel gibt es beim Ableiten in diesen Fällen nicht! Die Potenzregel auf die äußere Funktion (...)^4 anzuwenden, ist zwar korrekt, aber nur ein Teil der richtigen Lösung. Die innere Ableitung darf nicht "unter den Tisch fallen".

Bemerkung: Wenn die Ableitung der inneren Funktion aus mehreren Summanden besteht, in diesem Beispiel h'(x)=3x^2+10x, muss diese Ableitung bei den nachfolgenden Rechnungen in Klammern gesetzt werden.

 

f(x) = 13 \cdot 10^{x^2-1}   \begin{array}{rcl} f'(x) & = & 13 \cdot \ln(10) \cdot 10^{h(x)} \cdot h'(x) \\\\& = & 13 \cdot \ln(10) \cdot 10^{x^2-1} \cdot 2x \\\\& = & 26x \cdot \ln(10) \cdot 10^{x^2-1} \end{array}  

\begin{array}{rcl}g(h(x)) & = & 10^{h(x)}\\g'(h(x)) & = & \ln(10) \cdot 10^{h(x)}\\\\h(x) & = & x^2-1\\h'(x) & = & 2x \end{array}

Bei dieser Exponentialfunktion steht das Polynom im Exponenten. Würde man also einen Funktionswert bestimmen wollen, müsste zuerst das Polynom im Exponenten berechnet werden, um danach den Wert der Exponentialfunktion auszurechnen. Deswegen wird das Polynom im Exponenten als innere Funktion h(x) und die Exponentialfunktion als äußere Funktion g(x) bezeichnet.

Wichtig: Denken Sie bitte auch hier an die Regel zur Ableitung einer Exponentialfunktion, die natürlich auch bei der Anwendung der Kettenregel beachtet werden muss. 

Bemerkung: Die 13 vor der Exponentialfunktion ist ein konstanter Faktor, der nach der Faktorregel beim Ableiten unverändert erhalten bleibt.

\begin{array}{rcl}f(x) & = & \sin(\sqrt[3]{x}) \\\\& = & \sin(x^{\frac{1}{3}}) \end{array}   \begin{array}{rcl} f'(x) & = & \cos(h(x)) \cdot h'(x) \\\\& = & \cos(x^{\frac{1}{3}}) \cdot \dfrac{1}{3} x^{-\frac{2}{3}} \\\\& = & \dfrac{1}{3\sqrt[3]{x^2}} \cdot \cos(\sqrt[3]{x}) \end{array}  

\begin{array}{rcl}g(h(x)) & = & \sin(h(x))\\g'(h(x)) & = & \cos(h(x))\\\\h(x) & = & x^{\frac{1}{3}}\\h'(x) & = & \dfrac{1}{3}x^{-\frac{2}{3}} \end{array}

Bei diesem Beispiel steht ein Wurzelausdruck im Argument der Sinusfunktion. Würde man also einen Funktionswert bestimmen wollen, müsste zuerst die Wurzel berechnet werden, um danach den Sinuswert auszurechnen. Deswegen ist die Wurzel die innere Funktion h(x) und die Sinusfunktion die äußere Funktion g(x).


Kombination mehrerer weitergehender Regeln

Zum Abschluss folgt jetzt noch ein Beispiel, in dem zwei der drei weitergehenden Regeln miteinander kombiniert sind. Produkt-, Quotienten- und Kettenregel müssen also gleichzeitig, quasi ineinander verwoben, angewendet werden.

\begin{array}{rcl}f(x) & = & \sqrt[3]{x^5 \cdot \tan(x)}-16 \\\\& = & \left(x^5 \cdot \tan(x)\right)^\frac{1}{3} - 16\end{array}  

\begin{array}{rcl}f'(x) & = & \dfrac{1}{3} \cdot \left(h(x)\right)^{-\frac{2}{3}} \cdot h'(x) - 0\\\\& = & \dfrac{1}{3}\left(x^5 \cdot \tan(x)\right)^{-\frac{2}{3}} \cdot \left(5x^4 \cdot \tan(x) + x^5 \cdot \dfrac{1}{\left(\cos(x)\right)^2}\right) - 0\\\\& = & \dfrac{5x^4 \cdot \tan(x) + \frac{x^5}{\left(\cos(x)\right)^2}}{3 \sqrt[3]{\left(x^5 \cdot \tan(x)\right)^2}}\end{array}

 

\begin{array}{rcl}g(h(x)) & = & (h(x))^\frac{1}{3} \\g'(h(x)) & = & \dfrac{1}{3} \cdot \left(h(x)\right)^{-\frac{2}{3}}\\\\h(x) & = & u(x) \cdot v(x) \\ &=& x^5 \cdot \tan(x)\\h'(x) & = & (u(x) \cdot v(x))' \\ &=& u'(x) \cdot v(x) + u(x) \cdot v'(x)\\\\u(x) & = & x^5\\u'(x) & = & 5x^4\\\\v(x) & = & \tan(x)\\v'(x) & = & \dfrac{1}{\left(\cos(x)\right)^2}\end{array}

Da unter der dritten Wurzel f(x)=\sqrt[3]{...} nicht einfach nur eine Variable steht, ist hier - analog zu den vorhergehenden Beispielen - die Anwendung der Kettenregel zwingend erforderlich. Die Ableitung der äußeren Funktion g(h(x)) = h(x)^\frac{1}{3} folgt dabei dem bekannten Schema. Bei der Ableitung der inneren Funktion h(x) ist allerdings zu beachten, dass für die Ableitung des Terms x^5 \cdot \tan(x) die Produktregel genutzt werden muss. Während wir noch damit beschäftigt sind, die Anwendung der Kettenregel abzuschließen. muss bei der Bildung von h'(x) noch eine weitere der weitergehenden Regeln berücksichtigt werden.

 

 

Exkurs zum Definitionsbereich: Wir hatten uns bei den Beispielen zur Potenzregel auch die Wurzelfunktion f(x) = \sqrt{x} angeschaut. Bekanntermaßen ist deren Definitionsbereich in den reellen Zahlen eingeschränkt, da aus negativen Zahlen keine Quadratwurzeln gezogen werden können. Der Definitionsbereich lautet also \mathbb{D} = \mathbb{R}_0^+.

Aber wie sieht es mit dem Definitionsbereich der Ableitungsfunktion f'(x) = \dfrac{1}{\sqrt{x}} aus?
Hier müssen wir nicht nur darauf achten, dass durch die Quadratwurzel negative Werte ausgeschlossen sind, sondern auch beachten, dass die Quadratwurzel im Nenner steht und somit zusätzlich die Division durch 0 vermieden werden muss. Somit hat die erste Ableitung als Definitionsbereich \mathbb{D} = \mathbb{R}^+.

Die Definitionsbereiche von Ausgangsfunktion und Ableitung können sich also unterscheiden. Im betrachteten Beispiel hat das zur Konsequenz, dass für die Stelle x=0 keine Steigung bestimmt werden kann, da die Ableitung hier nicht definiert ist. Auch wenn also das Bestimmen der Ableitung gelungen ist, bedeutet das nicht zwingend, dass auch für jeden x-Wert des ursprünglichen Definitionsbereichs eine Steigung berechnet werden kann.

Neben der Möglichkeit, dass der Definitionsbereich der Ableitungsfunktion gegenüber dem Definitionsbereich der Ausgangsfunktion eingeschränkt ist, besteht auch die Möglichkeit, dass der Definitionsbereich der Ableitungsfunktion größer ist. Nehmen wir als Beispiel die Funktion f(x) = \ln(x), deren Ableitung f'(x) = \dfrac{1}{x} ist. Der Definitionsbereich der Ausgangsfunktion lautet \mathbb{D} = \mathbb{R}^+, wohingegen der Definitionsbereich der Ableitungsfunktion \mathbb{D} = \mathbb{R}\setminus_{\{0\}} ist. Auch wenn die Ableitungsfunktion f'(x) für negative Werte bestimmt ist, handelt es sich dabei nicht um die Steigung der Ausgangsfunktion, da diese dort einfach nicht existiert.

 

Nicht differenzierbare Funktionen

Es gibt Funktionen, bei denen sich an bestimmten Stellen die Steigung nicht bestimmen lässt. Die folgende Grafik zeigt eine solche Funktion f(x) (fett gezeichnet):

Betragsfunktion als nicht differenzierbare Funktion

Was ist hier das Problem?
Im Tiefpunkt der Funktion f(x) (es ist eine Betragsfunktion, nämlich f(x)=\vert x+1\vert+1) lassen sich beliebig viele Tangenten mit sehr unterschiedlichen Steigungen finden. Hier sind zur Illustration drei Geraden eingezeichnet, die alle den Funktionsgraphen im Tiefpunkt berühren und eigentlich das Kriterium einer Tangenten erfüllen. Allerdings ist keine dieser Tangenten "besser" oder "schlechter" als die anderen, sodass es letztendlich nicht die eine Tangente gibt, die die gegebene Funktion approximiert. Somit kann auch keine Steigung der Tangenten bestimmt werden, die der Steigung der Ausgangsfunktion in diesem Punkt entspräche. Deswegen kann in diesem Punkt keine Steigung der Funktion bestimmt werden und man sagt: Die Funktion f(x) ist im Punkt (-1\mid 1) nicht differenzierbar.

 

Berechnung von Extrempunkten

Zur Berechnung von Extrempunkten starten wir mit einer kleinen Wiederholung ... Nehmen wir die Funktion f(x)=4x^2-4x und erstellen eine Wertetabelle:

x   -5 -4 -3 -2 -1 0 1 2 3 4 5
f(x)   120 80 48 24 8 0 0 8 24 48 80

 

Versuchen wir nun, die Extrempunkte der Funktion aus der Wertetabelle abzulesen, fällt auf, dass der kleinste Funktionswert, nämlich 0, zweimal auftritt. Man könnte also die Erkenntnis gewinnen, dass die Funktion zwei Tiefpunkte, nämlich bei x_{N1}=0 und x_{N2}=1, hat. Aber ist das richtig?

Stellen wir uns daher die Frage, ob die Funktion überhaupt Hochpunkte oder Tiefpunkte hat und wenn ja, wie viele. Die Beantwortung dieser Frage ist häufig nicht ganz einfach, da das Spektrum der Möglichkeiten groß ist und von null bis zu unendlich vielen Extrempunkten reicht.
Im vorliegenden Fall hilft uns die Erkenntnis, dass es sich um eine quadratische Funktion handelt. Quadratische Funktionen haben immer Parabeln als Graph und diese haben genau einen Extrempunkt, nämlich den sogenannten Scheitelpunkt. Da bei der gegebenen Funktion der Koeffizient vor dem quadratischen Term positiv ist, ist die Parabel nach oben geöffnet und hat somit genau einen Tiefpunkt. Es darf und kann also nicht zwei x-Werte für den kleinsten y-Wert geben!

Aber warum haben wir dann aus der Wertetabelle (fälschlicherweise) zwei Tiefpunkte abgelesen?
Wir haben uns bei der Wahl der x-Werte für unsere Wertetabelle aus Bequemlichkeit für ganzzahlige Werte entschieden. Nur ist - dummerweise - der x-Wert des gesuchten Tiefpunkts nicht ganzzahlig, sondern liegt zwischen 0 und 1. Das "Problem" ist also nicht die Funktion, sondern unsere Wahl der x-Werte für die Wertetabelle. Im vorliegenden Beispiel können wir das Problem beheben, indem wir für die x-Werte den Abstand auf 0{,}5 reduzieren, da Parabeln immer symmetrisch sind und der Scheitelpunkt somit mittig zwischen den beiden x-Werten mit dem kleinsten y-Wert aus der oberen Wertetabelle liegen muss. Das ist also bei 0{,}5:

x   -2{,}5 -2 -1{,}5 -1 -0{,}5 0 0{,}5 1 1{,}5 2 2{,}5
f(x)   35 24 15 8 3 0 -1 0 3 8 15


Damit wäre bei dieser Funktion alles in Ordnung gebracht: Der gesuchte Tiefpunkt liegt bei (0{,}5 \mid -1).

Stellen Sie sich nun aber vor, dass der Scheitelpunkt bei x=-\frac{19}{65} oder x=\sqrt{127} liegen würde (soll alles schon mal vorgekommen sein ...). Dann wäre der entsprechende Extrempunkt sicherlich nicht durch eine Wertetabelle zu finden.

Mithilfe von Ableitungen lassen sich solche Schwierigkeiten (und auch noch andere) vermeiden. Ableitungen gehören in den Bereich der Differenzialrechnung, die es u. a. ermöglicht, charakteristische Punkte und Eigenschaften von Funktionen exakt zu bestimmen (siehe Kapitel Eigenschaften und "besondere Punkte" von Funktionen). Die Erkenntnisse über eine Funktion hängen damit nicht mehr von ungeschickt gewählten Werten in einer Wertetabelle (wie in unserem Beispiel), einer nur mäßig guten Zeichnung (welche das Ablesen von Werten schwierig bis unmöglich macht) oder dem willkürlich gewählten Ausschnitt des Koordinatensystems (welcher vielleicht die relevanten Bereiche gar nicht umfasst) ab. Und - ganz wichtig - auch bei Funktionen mit mehreren Variablen, die sich nicht mehr vernünftig zeichnen lassen, liefert die Differenzialrechnung Erkenntnisse.

Im Folgenden soll es nun darum gehen, Extrempunkte rechnerisch zu bestimmen.


Vorgehen:
1.
Um Extrempunkte zu bestimmen, leitet man klassischerweise die Funktion zuerst ab und löst anschließend die Gleichung, die entsteht, wenn man die erste Ableitung nullsetzt.

Dieses Verfahren wird plausibel, wenn man sich das Ganze grafisch anschaut:
Graph mit Tangenten in Tief-, Hoch- und Sattelpunkt

Dort, wo die Funktion Hoch- und Tiefpunkte hat, muss die Tangente waagerecht sein, sprich: Die Steigung ist zwingend 0. Genau diese x-Werte berechnen wir mithilfe der nullgesetzten Ableitung, also f'(x)=0.
Sie sehen in diesen Grafiken aber auch, dass es noch einen weiteren x-Wert mit einer waagerechten Tangente gibt, nämlich den Sattelpunkt bei x=0. Dieses Verfahren liefert also Kandidaten für Extremstellen, gibt aber noch keine sichere Auskunft darüber, ob an den Kandidatenstellen wirklich Hoch- oder Tiefpunkte vorliegen. Wir sind daher noch nicht fertig ...


2.
Im nächsten Schritt überprüft man die Kandidaten, x_K, mithilfe der zweiten Ableitung: Man setzt die zuvor ermittelten x-Werte in die zweite Ableitung ein und prüft, ob das Ergebnis ungleich 0 ist. Folgende Zusammenhänge bestehen:

  • Ist das Ergebnis größer als 0, also f''(x_K)>0, hat die Funktion bei diesem x-Wert mit Sicherheit einen Tiefpunkt.
  • Ist das Ergebnis kleiner als 0, also f''(x_K) < 0, hat die Funktion bei diesem x-Wert mit Sicherheit einen Hochpunkt.
  • Ist das Ergebnis gleich 0, also f''(x_K)=0, hat die Funktion an diesem x-Wert in der Regel keinen Hoch- oder Tiefpunkt, sondern einen Sattelpunkt, also einen speziellen Wendepunkt ... Es ist aber nicht gänzlich ausgeschlossen, dass dort doch ein Hoch- oder Tiefpunkt (und kein Sattelpunkt) vorliegt. Deswegen müsste die Analyse hier weiter fortgesetzt werden, was aber über den Anspruch dieses Lernmoduls hinausgeht, da diese Fälle doch eher selten sind.


Für diesen zweiten Schritt gibt es auch andere Verfahren, die hier aber nicht weiter thematisiert werden.


3.
Nicht vergessen: Um die Hoch- oder Tiefpunkte zu bestimmen (und nicht nur die x-Werte), müssen wir noch die jeweiligen Funktionswerte berechnen - ein Punkt braucht ja immer zwei Koordinaten ...


Ein Beispiel:
\begin{array}{rcl}f(x) & = & \dfrac{1}{24}x^3-\dfrac{1}{8}x^2-x \\\\\mathbb{D} &=& \mathbb{R}\end{array}

1. Schritt: Kandidaten für die Extrempunkte berechnen
Um die hier relevanten Schritte in den Fokus zu rücken, wurde das Auflösen der quadratischen Gleichung hier nicht notiert. Macht man wie immer ...

\begin{array}{rclll} f'(x) &=& \dfrac{1}{8}x^2-\dfrac{1}{4}x-1 & \vert & \text{nullsetzen} \cr \cr 0 &=&\dfrac{1}{8}x^2-\dfrac{1}{4}x-1 \cr &...& \cr x_1 &=& -2 \cr x_2 &=& 4 \end{array}


2. Schritt: Kandidaten überprüfen und ggf. Art des Extrempunkts ermitteln

\begin{array}{rclll} f''(x) &=& \dfrac{1}{4}x-\dfrac{1}{4} & \vert & \text{Kandidaten einsetzen} \cr \cr f''(-2) &=& -\dfrac{3}{4} & < & 0 \quad \Rightarrow \text{Hochpunkt} \cr \cr f''(4) &=& \dfrac{3}{4} & > & 0 \quad \Rightarrow \text{Tiefpunkt}\end{array}


3. Schritt: Funktionswerte ausrechnen

\begin{array}{rclll} f(-2) &=&\dfrac{1}{24}\cdot(-2)^3-\dfrac{1}{8}\cdot(-2)^2-(-2) &=& \dfrac{7}{6} \cr\cr f(4) &=&\dfrac{1}{24}\cdot 4^3-\dfrac{1}{8}\cdot4^2-4 &=& -\dfrac{10}{3} \end{array}


Ergebnis: Die Funktion hat bei \left(-2\mid \dfrac{7}{6}\right) einen Hochpunkt und bei \left(4\mid -\dfrac{10}{3}\right) einen Tiefpunkt. Grafisch sieht das so aus:
Graph zu Beispiel


Achtung:
Es gibt Extremstellen, z. B. an den Rändern des Definitionsbereichs (sofern er eingeschränkt ist) oder bei x-Werten, an denen keine Steigung bestimmt werden kann (nicht differenzierbare Funktion, siehe oben), die mithilfe dieses Verfahrens nicht gefunden werden können. In diesen Fällen müssen die Randwerte eines geschlossenen Intervalls bzw. die nicht differenzierbaren Stellen also separat überprüft werden.

 

Zum Abschluss

Während Ihrer Schulzeit sind Sie vermutlich häufiger mit Aufgabenstellungen im Sinne von "Leiten Sie f(x)=... ab" oder "Bilden Sie die ersten zwei Ableitungen von ..." konfrontiert worden oder Sie sollten nach vorgegebenem Schema eine Kurvendiskussion durchführen. Da dieses Kapitel der Wiederholung und Auffrischung dient, bewegen wir uns auch hier in genau diesem Rahmen. Allerdings werden Sie solche Aufgaben im Studium selten antreffen.
In der Schulzeit ging es hauptsächlich darum, Ihnen das mathematische Rüstzeug zu vermitteln und dessen Nutzung einzuüben, damit Sie jetzt im Studium in der Lage sind, dieses Werkzeug im Rahmen von teils komplexen Anwendungszusammenhängen sicher einzusetzen. Wenn es also z. B. darum geht, den Materialeinsatz zur Lösung eines gegebenen Problems ressourcenschonend zu minimieren, ist das nichts anderes als die Suche nach dem Tiefpunkt der zugehörigen Funktion. Hierfür benötigen Sie implizit das Wissen, wie man Ableitungen und Extremstellen bestimmt - genau das, worum es in diesem Kapitel ging.

Übersicht:

 

22.4 Ableitungen - Lösungen

1. Aufgabe

Um den Verlauf der Ableitung zu der gegebenen Funktion f(x) zu ermitteln, muss die Steigung der Funktion ermittelt werden. In der nachfolgenden Grafik wurde an verschiedenen Stellen die entsprechende Tangente eingezeichnet, mit deren Hilfe man sich die Steigung der Funktion in diesen Punkten verdeutlichen kann:

ein Graph im Koordinatensystem

Da es bei dieser Aufgabe nur das Ziel ist, die Größenordnung der Steigung an einer bestimmten Stelle im Vergleich zu anderen einzuschätzen, müssen keine konkreten Zahlenwerte für die Tangentensteigungen ausgerechnet werden. Das geht mit den gegebenen Informationen auch gar nicht ...

Überlegungen zu den Tangenten:

  • Die Tangente t1(x) berührt die Funktion im Punkt \left(-\frac{3}{2} \mid f\left(-\frac{3}{2}\right)\right) und ist eine fallende Gerade. Die Steigung von f(x) muss an dieser Stelle also negativ sein.
  • Die Tangente t2(x) berührt die Funktion im Punkt (-1 \mid f(-1)) und ist eine konstante Gerade. Die Steigung von f(x) muss an dieser Stelle also 0 sein.
  • Die Tangente t3(x) berührt die Funktion im Punkt \left(-\frac{1}{2} \mid f\left(-\frac{1}{2}\right)\right) und ist eine steigende Gerade. Die Steigung von f(x) muss an dieser Stelle also positiv sein.
  • Die Tangente t4(x) berührt die Funktion im Punkt (0 \mid f(0)) und ist eine konstante Gerade. Die Steigung von f(x) muss an dieser Stelle also 0 sein.
  • Die Tangente t5(x) berührt die Funktion im Punkt \left(\frac{1}{2} \mid f\left(\frac{1}{2}\right)\right) und ist eine fallende Gerade. Die Steigung von f(x) muss an dieser Stelle also negativ sein.
  • Die Tangente t6(x) berührt die Funktion im Punkt (1 \mid f(1)) und ist eine konstante Gerade. Die Steigung von f(x) muss an dieser Stelle also 0 sein.
  • Die Tangente t7(x) berührt die Funktion im Punkt \left(\frac{3}{2} \mid f\left(\frac{3}{2}\right)\right) und ist eine steigende Gerade. Die Steigung von f(x) muss an dieser Stelle also positiv sein.

 

  • Vergleicht man die beiden fallenden Tangenten t1(x) und t5(x), stellt man fest, dass t1(x) sehr viel steiler verläuft als t5(x). Die Steigung von t1(x) muss als vom Betrag her größer sein als die Steigung von t5(x).
  • Gleiches gilt für die beiden steigenden Tangenten t3(x) und t7(x). t7(x) verläuft deutlich steiler als t3(x). Die Steigung von t7(x) muss also vom Betrag her größer sein als die Steigung von t3(x).


Überträgt man diese Erkenntnisse in ein Koordinatensystem, erhält man eine Kurve ähnlich der in der nachfolgenden Grafik gezeigten:

der Graph der Ableitungsfunktion zur vorherigen Funktion

Mehr ist nicht gefragt ...
Diese Aufgabe ist deswegen wichtig, weil Sie sich hierüber einen anderen Blick auf Ableitungen erarbeiten. Dieser hilft zum Beispiel, um berechnete Ableitungen zu überprüfen, aber auch im Kontext von Anwendungsaufgaben.

 

2. Aufgabe

Bemerkung 1: Bitte denken Sie daran, dass die Rechenregeln, die Sie bislang in diesem Lernmodul gelernt haben, auch im Zusammenhang mit Ableitungen gelten! Das gilt zum Beispiel für folgende Regeln:

  • Bruchrechenregeln gelten auch dann, wenn die Brüche innerhalb von Potenzen auftauchen! Beispielsweise muss im Exponenten bei Aufgaben 9) folgendermaßen gerechnet werden: \frac{1}{3}-1=\frac{1}{3}-\frac{3}{3}=\frac{1-3}{3}=-\frac{2}{3}, also erst die Brüche gleichnamig machen und dann zusammenfassen.
  • Auch die Festlegungen im Rahmen der Potenz- und Wurzelrechnung, also x^0 = 1, \dfrac{1}{x^n}=x^{-n} sowie \sqrt[n]{x}=x^{\frac{1}{n}}, werden hier immer wieder benötigt.

Bemerkung 2: Die Konstanten-, Faktor- und Summenregel beim Ableiten sind nicht übermäßig überraschend und auch nicht sehr schwierig in der Anwendung. (Vermutlich würden viele intuitiv genauso vorgehen ...) In dieser Aufgabe wird trotzdem Wert auf genau diese einfachen Regeln gelegt, da man sich diese einmal bewusst gemacht haben sollte, damit man später die komplexeren Regeln sinnvoll davon abgrenzen kann.

Bemerkung 3: Ganz so ausführlich wie in dieser Aufgabe müssen Lösungen normalerweise nicht aufgeschrieben werden.


1)
\begin{array}{rcl} \mathbb{D} &=& \mathbb{R} \cr\cr f'(x) &=& 8 \cdot 2 \cdot x^{2-1}-0 \cr &=& 16x \end{array}

Vorgehen: Konstanten-, Faktor-, Summen- und Potenzregel
 

2)
\begin{array}{rcl}\mathbb{D} &=& \mathbb{R} \cr\cr f'(x) &=& -10 \cdot 3 \cdot x^{3-1}- 2 \cdot x^{2-1} \cr &=& -30x^2-2x \end{array}

Vorgehen: Faktor-, Summen- und Potenzregel
 

3)
\begin{array}{rcl} \mathbb{D} &=& \mathbb{R} \cr\cr f'(x) &=& -\dfrac{10}{9} \cdot 18 \cdot x^{18-1}-11 \cdot 1 \cdot x^{1-1} \cr &=& -20x^{17}-11 \end{array}

Vorgehen: Faktor-, Summen- und Potenzregel
 

4)
\begin{array}{rcl}\mathbb{D} &=& \mathbb{R} \cr\cr f'(x) &=& 2 \cdot 2 \cdot x^{2-1}-1 \cdot 1 \cdot x^{1-1}-0 \cr &=& 4x-1 \end{array}

Vorgehen: Konstanten-, Faktor-, Summen- und Potenzregel
 

5)
\begin{array}{rcl}\mathbb{D} &=& \mathbb{R} \cr\cr f'(x) &=& -3 \cdot 4 \cdot x^{4-1}+16 \cdot 2 \cdot x^{2-1}+0 \cr &=& -12x^3+32x \end{array}

Vorgehen: Konstanten-, Faktor-, Summen- und Potenzregel

6)
\begin{array}{rcl} \mathbb{D} &=& \mathbb{R} \cr\cr f'(x) &=& \dfrac{1}{4} \cdot 4 \cdot x^{4-1}-\dfrac{1}{3} \cdot 3 \cdot x^{3-1}+ \dfrac{1}{2} \cdot 2 \cdot x^{2-1}-1 \cdot 1 \cdot x^{1-1}+0 \cr &=& x^3-x^2+x-1 \end{array}

Vorgehen: Konstanten-, Faktor-, Summen- und Potenzregel
 

7)
\begin{array}{rcl}\mathbb{D} &=& \mathbb{R} \cr\cr f(x) &=& \dfrac{2}{7}x^{14} \cr \cr \cr f'(x) &=& \dfrac{2}{7} \cdot 14 \cdot x^{14-1} \cr\cr &=& 4x^{13} \end{array}

Vorgehen: Faktor- und Potenzregel
 

8)
\begin{array}{rcl}\mathbb{D} &=& \mathbb{R} \cr\cr f'(x) &=& 230 \cdot 12 \cdot x^{12-1} + 17 \cdot 6 \cdot x^{6-1} -\dfrac{3}{5}\cdot 5 \cdot x^{5-1}+0 \cr &=& 2.760x^{11}+102x^5-3x^4 \end{array}

Vorgehen: Konstanten-, Faktor-, Summen- und Potenzregel

9)
\begin{array}{rcl}\mathbb{D}&=&]0;\infty[ \cr\cr f'(x) &=& 25 \cdot 4 \cdot x^{4-1} -66 \cdot \dfrac{1}{3} \cdot x^{\frac{1}{3}-1}-17\cdot \dfrac{3}{2} \cdot x^{\frac{3}{2}-1} +0 \cr \cr &=& 100x^3-22x^{-\frac{2}{3}}-\dfrac{51}{2}x^{\frac{1}{2}} \cr \cr &=& 100x^3-\dfrac{22}{\sqrt[3]{x^2}}-\dfrac{51}{2}\sqrt{x} \end{array}

Vorgehen: Konstanten-, Faktor-, Summen- und Potenzregel

Bemerkung: e ist einfach nur eine Zahl und fällt daher beim Ableiten weg ...

10)
\begin{array}{rcl}\mathbb{D} &=& \mathbb{R}\setminus_{\{0\}} \cr\cr f'(x) &=& -3 \cdot x^{-3-1}+0 \cr &=& -3x^{-4} \cr \cr &=& -\dfrac{3}{x^4} \end{array}

Vorgehen: Konstanten-, Summen- und Potenzregel

11)
\begin{array}{rcl}\mathbb{D}&=&\mathbb{R}\setminus_{\{0\}} \cr \cr f(x) &=& 6x^{-1} \cr \cr \cr f'(x) &=& 6 \cdot \left(-1 \right) \cdot x^{-1-1} \cr &=& -6x^{-2} \cr \cr &=& -\dfrac{6}{x^2} \end{array}

Vorgehen: Faktor- und Potenzregel

12)
\begin{array}{rcl}\mathbb{D}&=&]0;\infty[ \cr \cr f(x) &=& 3x^{\frac{1}{2}}-7 \cr \cr \cr f'(x) &=& 3 \cdot \dfrac{1}{2} \cdot x^{\frac{1}{2}-1}-0 \cr \cr &=& \dfrac{3}{2} \cdot x^{-\frac{1}{2}} \cr \cr &=& \dfrac{3}{2} \cdot \dfrac{1}{x^\frac{1}{2}} \cr \cr &=& \dfrac{3}{2\sqrt{x}}\end{array}

Vorgehen: Konstanten-, Faktor-, Summen- und Potenzregel
 

13)
\begin{array}{rcl}\mathbb{D}&=&[0;\infty[ \cr \cr f(x) &=& x^1 \cdot x^{\frac{1}{2}} \cr \cr &=& x^{\frac{3}{2}} \cr \cr \cr f'(x) &=& \dfrac{3}{2} \cdot x^{\frac{3}{2}-1} \cr \cr &=& \dfrac{3}{2}x^{\frac{1}{2}} \cr \cr &=& \dfrac{3}{2}\sqrt{x}\end{array}

Vorgehen: Potenzregel

Bemerkung: Meist lohnt es sich, die Funktion vor dem Ableiten genau zu betrachten und in eine möglichst "geschickte" Form zu bringen, auch wenn es erstmal nach mehr Arbeit aussieht ...
 

14)
\begin{array}{rcl}\mathbb{D}&=&\mathbb{R}\setminus_{\{0\}} \cr \cr f(x) &=& x^{-2} \cr \cr \cr f'(x) &=& -2 \cdot x^{-2-1} \cr &=& -2\cdot x^{-3} \cr \cr &=& -\dfrac{2}{x^3} \end{array}

Vorgehen: Potenzregel
 

15)
\begin{array}{rcl}\mathbb{D}&=&\mathbb{R}\setminus_{\{0\}} \cr \cr f(x) &=& -\dfrac{1}{12} \cdot x^{-3} \cr\cr\cr f'(x) &=& -\dfrac{1}{12} \cdot \left(-3\right) \cdot x^{-3-1} \cr\cr &=& \dfrac{1}{4}x^{-4} \cr\cr &=& \dfrac{1}{4x^4} \end{array}

Vorgehen: Faktor- und Potenzregel

16)
\begin{array}{rcl}\mathbb{D}&=&\mathbb{R}\setminus_{\{0\}} \cr \cr f(x) &=& -17x^{-19} \cr \cr \cr f'(x) &=& -17 \cdot \left(-19 \right) \cdot x^{-19-1} \cr &=& 323x^{-20} \cr \cr &=& \dfrac{323}{x^{20}} \end{array}

Vorgehen: Faktor- und Potenzregel

17)
\begin{array}{rcl}\mathbb{D}&=&]0;\infty[ \cr \cr f(x) &=&5\cdot x^{\frac{1}{4}}-x+1 \cr\cr\cr f'(x) &=& 5\cdot \dfrac{1}{4} \cdot x^{\frac{1}{4}-1}-1 \cdot x^{1-1}+0 \cr\cr &=& \dfrac{5}{4} \cdot x^{-\frac{3}{4}}-1 \cr\cr &=& \dfrac{5}{4\sqrt[4]{x^3}}-1 \end{array}

Vorgehen: Konstanten-, Faktor-, Summen- und Potenzregel

18)
\begin{array}{rcl}\mathbb{D}&=&]0;+\infty[ \cr \cr f(x) &=& \dfrac{1}{6} \cdot x^{-\frac{1}{2}} \cr\cr\cr f'(x) &=& \dfrac{1}{6} \cdot \left(-\dfrac{1}{2}\right) \cdot x^{-\frac{1}{2}-1} \cr\cr &=& -\dfrac{1}{12} \cdot x^{-\frac{3}{2}} \cr\cr &=& -\dfrac{1}{12} \dfrac{1}{\sqrt{x^3}} \cr\cr &=& -\dfrac{1}{12\sqrt{x^3}} \end{array}

Vorgehen: Faktor- und Potenzregel
 

19)
\begin{array}{rcl}\mathbb{D}&=&\mathbb{R}\setminus_{\{0\}} \cr \cr f(x) &=& 15 \cdot x^{-\frac{6}{5}} \cr \cr \cr f'(x) &=& 15 \cdot \left(-\dfrac{6}{5}\right) \cdot x^{-\frac{6}{5}-1} \cr \cr &=& -18 \cdot x^{-\frac{11}{5}} \cr \cr &=& -18 \cdot \dfrac{1}{\sqrt[5]{x^{11}}} \cr \cr &=& -\dfrac{18}{\sqrt[5]{x^{11}}} \end{array}

Vorgehen: Faktor- und Potenzregel
 

20)
\begin{array}{rcl}\mathbb{D}&=&\mathbb{R} \cr \cr f'(x) &=& 7 \cdot a \cdot x^{a-1}+0 \cr &=& 7ax^{a-1} \end{array}

Vorgehen: Konstanten-, Faktor-, Summen- und Potenzregel

 
Bemerkung zu den folgenden Aufgaben: Bitte beachten Sie, dass die Anwendung der Konstanten-, Faktor- und Summenregel nicht mehr explizit erwähnt wird, um den Blick auf die weitergehenden Ableitungsregeln zu lenken - auch wenn man die einfachen Ableitungsregeln (fast) bei jeder Aufgabe benötigt. Auch Hinweise zu speziellen Ableitungen wird nicht mehr notiert - dass man z. B. zum Ableiten einer Exponentialfunktion die Regel für die Exponentialfunktion benötigt, ist ja naheliegend ...

 

3. Aufgabe

Eine Bemerkung vorab: Die Funktionen 1) bis 5) könnte man auch gut ohne die Produktregel ableiten. Um genau zu sein, wäre es meist sogar sinnvoller, die Funktionsterme auszumultiplizieren und dann einfach die Regel für Polynome anzuwenden. In diesen Beispielen wurde nur deswegen die Produktregel angewendet, damit es auch ein paar leichte Beispiele gibt.


1) 
\begin{array}{ccllrcl}\quad\mathbb{D} &=& \mathbb{R}\\\\f(x) &=& \left(x-1\right)\cdot\left(\dfrac{47}{2}x^2+16x+7\right)\\\\\quad u(x) &=& x-1 &\Rightarrow & u'(x) &=& 1\\\quad v(x) &=& \dfrac{47}{2}x^2+16x+7 &\Rightarrow & v'(x) &=& 47x+16\\\\f'(x) &=& 1\cdot \left(\dfrac{47}{2}x^2+16x+7\right)+\left(x-1\right)\cdot\left(47x+16\right)\\\\&=& \dfrac{47}{2}x^2+16x+7+47x^2+16x-47x-16\\\\&=& \dfrac{141}{2}x^2-15x-9\end{array}


2)

\begin{array}{ccllrcl}\quad\mathbb{D} &=& \mathbb{R}\\\\f(x) &=& \left(13x^2+\dfrac{7}{5}x-18\right)\left(11x^2-\dfrac{15}{4}x-1\right)\\\\\quad u(x) &=& 13x^2+\dfrac{7}{5}x-18 &\Rightarrow & u'(x) = 26x+\dfrac{7}{5}\\\\\quad v(x) &=& 11x^2-\dfrac{15}{4}x-1 &\Rightarrow & v'(x) = 22x-\dfrac{15}{4}\\\\\\f'(x) &=& \left(26x+\dfrac{7}{5}\right)\cdot\left(11x^2-\dfrac{15}{4}x-1\right)+\left(13x^2+\dfrac{7}{5}x-18\right)\cdot\left(22x-\dfrac{15}{4}\right)\\\\&=& 286x^3-\dfrac{195}{2}x^2-26x+\dfrac{77}{5}x^2-\dfrac{21}{4}x-\dfrac{7}{5}+286x^3-\dfrac{195}{4}x^2+\dfrac{154}{5}x^2-\dfrac{21}{4}x-396x+\dfrac{135}{2}\\\\&=& 572x^3-\dfrac{2.001}{20}x^2-\dfrac{865}{2}x+\dfrac{661}{10}\end{array}


3)
\begin{array}{ccllrcl}\quad\mathbb{D} &=& \mathbb{R}\\\\f(x) &=& \left(\dfrac{1}{8}x^3+5x\right)\cdot\sqrt{8}x\\\\\quad u(x) &=& \dfrac{1}{8}x^3+5x &\Rightarrow & u'(x) &=& \dfrac{3}{8}x^{2}+5\\\quad v(x) &=& \sqrt{8}x &\Rightarrow & v'(x) &=& \sqrt{8}\\\\f'(x) &=& \left(\dfrac{3}{8}x^{2}+5\right)\cdot\sqrt{8}x+\left(\dfrac{1}{8}x^3+5x\right)\cdot\sqrt{8}\\\\&=& \sqrt{8}\left(\dfrac{3}{8}x^{3}+5x+\dfrac{1}{8}x^3+5x\right)\\\\&=& \sqrt{8}\left(\dfrac{1}{2}x^{3}+10x\right)\\\end{array}


4)
\begin{array}{ccllrcl}\quad\mathbb{D} &=& \mathbb{R}\\\\f(x) &=& 3x^2\cdot 7x^{\frac{1}{2}}+ex^{10}-105\\\\\quad u(x) &=& 3x^2 &\Rightarrow & u'(x) &=& 6x\\\quad v(x) &=& 7x^{\frac{1}{2}} &\Rightarrow & v'(x) &=& \dfrac{7}{2}x^{-\frac{1}{2}}\\\\f'(x) &=& 6x\cdot 7x^{\frac{1}{2}}+3x^2\cdot \dfrac{7}{2}x^{-\frac{1}{2}}+10ex^9\\\\&=& 42x^1\cdot x^{\frac{1}{2}}+\dfrac{21}{2}x^2\cdot x^{-\frac{1}{2}}+10ex^9\\\\&=& 42x^{\frac{3}{2}}+\dfrac{21}{2}x^{\frac{3}{2}}+10ex^9\\\\&=& \dfrac{105}{2}x^{\frac{3}{2}}+10ex^9\\\\&=& \dfrac{105}{2}\sqrt{x^3}+10ex^9\\\end{array}

Bemerkung: ex^{10} muss nicht mit der Produktregel abgeleitet werden. e ist ja einfach nur eine Zahl. Daher greift die Faktorregel.


5)
\begin{array}{cclcrcl}\quad\mathbb{D} &=& \mathbb{R}\\\\g(k) &=& \left(\dfrac{11}{6}k^2-k\right)^2\cdot 72 \\&=& 72\left(\dfrac{11}{6}k^2-k\right)\cdot\left(\dfrac{11}{6}k^2-k\right) \\\\\quad u(k) &=& 72\left(\dfrac{11}{6}k^2-k\right) = 132k^2-72k &\Rightarrow & u'(k)&=& 264k-72 \\\quad v(k) &=& \dfrac{11}{6}k^2-k &\Rightarrow &v'(k) &=& \dfrac{11}{3}k-1 \\\\g'(k) &=& \left(264k-72\right)\cdot\left(\dfrac{11}{6}k^2-k\right)+\left(132k^2-72k\right)\cdot\left(\dfrac{11}{3}k-1\right) \\&=&484k^3-264k^2-132k^2+72k+484k^3-132k^2-264k^2+72k \\&=& 968k^3-792k^2+144k\end{array}


6)
\begin{array}{cclcrcl}\quad\mathbb{D} &=& \mathbb{R}^+_0 \\\\f(x) &=& \left(4x^2-16\right)\sqrt{23x}-5 \\\\\quad u(x) &=& 4x^2-16 &\Rightarrow & u'(x) &=& 8x \\\quad v(x) &=& \sqrt{23x} = \sqrt{23}\sqrt{x} &\Rightarrow & v'(x) &=& \sqrt{23}\dfrac{1}{2\sqrt{x}} = \dfrac{\sqrt{23}}{2\sqrt{x}} \\\\f'(x) &=& 8x\cdot \sqrt{23x}+\left(4x^2-16\right)\cdot \dfrac{\sqrt{23}}{2\sqrt{x}} \\\\&=& 8x\sqrt{23x}+\dfrac{2x^2\sqrt{23}}{\sqrt{x}}-\dfrac{8\sqrt{23}}{\sqrt{x}}\end{array}


7)
\begin{array}{cclcrcl}\quad\mathbb{D} &=& \mathbb{R}^+_0 \\\\g(x) &=& 2^x\sqrt[10]{1024x} \\&=& 2^x\cdot 2\sqrt[10]{x} \\\\\quad u(x) &=& 2^x &\Rightarrow & u'(x)&=& 2^x\cdot\ln(2) \\\quad v(x) &=& 2\sqrt[10]{x} = 2x^{\frac{1}{10}} &\Rightarrow & v'(x) &=& 2\cdot\dfrac{1}{10}x^{-\frac{9}{10}} = \dfrac{1}{5\sqrt[10]{x^9}} \\\\f'(x) &=& 2^x\cdot\ln(2)\cdot 2\sqrt[10]{x}+2^x\cdot \dfrac{1}{5}\cdot\dfrac{1}{\sqrt[10]{x^9}} \\\\&=& 2^{x+1}\cdot\ln(2)\sqrt[10]{x}+2^{x+1}\cdot\dfrac{1}{10\sqrt[10]{x^9}} \\\\&=& 2^{x+1}\left(\dfrac{1}{10\sqrt[10]{x^9}}+\ln(2)\sqrt[10]{x}\right)\end{array}


8)
\begin{array}{ccllrcl}\quad\mathbb{D} &=& \mathbb{R}\\\\f(x) &=& 23x^2\cdot e^x\\\\\quad u(x) &=& 23x^2 &\Rightarrow & u'(x) &=& 46x\\\quad v(x) &=& e^x &\Rightarrow & v'(x) &=& e^x\\\\f'(x) &=& 46x\cdot e^x+23x^2\cdot e^x\\&=& e^x\left(46x+23x^2\right)\\&=& 23xe^x\left(2+x\right)\\\end{array}


9)
\begin{array}{cclcrcl}\quad\mathbb{D} &=& \mathbb{R}\\\\f(k) &=& \dfrac{5}{4}k^{12}\cdot e^k-1 \\\\\quad u(k) &=& \dfrac{5}{4}k^{12} &\Rightarrow & u'(k) &=& 15k^{11} \\\quad v(k) &=& e^k &\Rightarrow &v'(k) &=& e^k \\\\f'(k) &=& 15k^{11}\cdot e^k+\dfrac{5}{4}k^{12}\cdot e^k \\\\&=& k^{11}e^k\left(15+\dfrac{5}{4}k\right)\end{array}


10)
\begin{array}{cclcrcl}\quad\mathbb{D} &=& \mathbb{R}\setminus_{\{0\}} \\\\f(y) &=& \dfrac{16}{\frac{4}{3}y^7 e^{-y}}-\dfrac{14}{9} \\&=& 12y^{-7}e^y -\dfrac{14}{9} \\\\\quad u(y) &=& 12y^{-7} &\Rightarrow & u'(y)&=& -84y^{-8} \\\quad v(y) &=& e^y &\Rightarrow & v'(y) &=& e^y \\\\f'(y) &=& -84y^{-8}\cdot e^y+12y^{-7}\cdot e^y \\ &=& 12y^{-8}e^y(-7+y) \\&=& \dfrac{12e^y(y-7)}{y^8}\end{array}


11)
\begin{array}{cclcrcl}\quad\mathbb{D} &=& \mathbb{R} \\\\f(x) &=& 5{,}5^x\cdot x^5-5 \\\\\quad u(x) &=& 5{,}5^x &\Rightarrow & u'(x) &=& 5{,}5^x\cdot\ln(5{,}5) \\\quad v(x) &=& x^5 &\Rightarrow & v'(x) &=& 5x^4 \\\\f'(x) &=& 5{,}5^x\cdot\ln(5{,}5)\cdot x^5+5{,}5^x\cdot 5x^4 \\&=& 5{,}5^x x^4\left(x\ln(5{,}5)+5\right)\end{array}


12)
\begin{array}{ccllrcl}\quad\mathbb{D} &=& \mathbb{R}^+ \\\\f(x) &=& 19x^2\cdot\ln(x)\\\\\quad u(x) &=& 19x^2 &\Rightarrow & u'(x) &=& 38x\\\quad v(x) &=& \ln(x) &\Rightarrow & v'(x) &=& \dfrac{1}{x}\\\\f'(x) &=& 38x\cdot\ln(x)+19x^2\cdot\dfrac{1}{x}\\&=& 38x\cdot\ln(x)+19x\\&=& x\left(38\ln(x)+19\right)\\\end{array}


13)
\begin{array}{cclcrcl}\quad\mathbb{D} &=& \mathbb{R}^+ \\\\f(a) &=& -14\sqrt[4]{a}\cdot \log_4(a) \\\\\quad u(a) &=& -14\sqrt[4]{a} = -14a^{\frac{1}{4}} &\Rightarrow & u'(a) &=& -14\cdot\frac{1}{4}a^{-\frac{3}{4}} = -\dfrac{7}{2}a^{-\frac{3}{4}} \\\quad v(a) &=& \log_4(a) &\Rightarrow & v'(a) &=& \dfrac{1}{a\ln(4)} \\\\f'(a) &=& -\dfrac{7}{2}a^{-\frac{3}{4}}\cdot \log_4(a)-14\sqrt[4]{a}\cdot \dfrac{1}{a\ln(4)} \\&=& -\dfrac{7}{2}a^{-\frac{3}{4}}\cdot\log_4(a)-14a^{-\frac{3}{4}}\cdot\dfrac{1}{\ln(4)} \\&=& -\dfrac{7}{\sqrt[4]{a^3}}\left(\dfrac{\log_4(a)}{2}+\dfrac{2}{\ln(4)}\right)\end{array}


14)
\begin{array}{cclcrcl}\quad\mathbb{D} &=& \mathbb{R}^+ \\\\f(x) &=& \dfrac{15}{x^2}\ln(x)+125 \\\\\quad u(x) &=& \dfrac{15}{x^2} = 15x^{-2} &\Rightarrow &u'(x) &=& 15\cdot(-2)x^{-3} = -\dfrac{30}{x^3} \\\quad v(x) &=& \ln(x) &\Rightarrow &v'(x) &=& \dfrac{1}{x} \\\\f'(x) &=& -\dfrac{30}{x^3}\ln(x)+\dfrac{15}{x^2}\cdot \dfrac{1}{x} \\\\&=& \dfrac{15-30\ln(x)}{x^3}\end{array}


15)
\begin{array}{ccllrcl}\quad\mathbb{D} &=& \mathbb{R}\\\\f(x) &=& 2x\cdot 3\sin(x)\\&=& 6x\cdot\sin(x)\\\\\quad u(x) &=& 6x &\Rightarrow & u'(x) &=& 6\\\quad v(x) &=& \sin(x) &\Rightarrow & v'(x) &=& \cos(x)\\\\f'(x) &=& 6\cdot\sin(x)+6x\cdot\cos(x)\\&=& 6\left(sin(x)+x\cos(x)\right)\\\end{array}


16)
\begin{array}{ccllrcl}\quad\mathbb{D} &=& \mathbb{R}\\\\f(x) &=& \sin(x)\cdot\cos(x)-\dfrac{3}{\pi}\\\\\quad u(x) &=& \sin(x) &\Rightarrow & u'(x) &=& \cos(x)\\\quad v(x) &=& \cos(x) &\Rightarrow & v'(x) &=& -\sin(x)\\\\f'(x) &=& \cos(x)\cdot\cos(x)+\sin(x)\cdot \left(-\sin(x)\right)-0\\&=& \cos^2(x)-\sin^2(x)\\\end{array}


17)
\begin{array}{ccllrcl}\quad\mathbb{D} &=& \mathbb{R}\\\quad a &\in& \mathbb{R}\\\\f(x) &=& ax^2\cdot 5^3\sin(x)+17ax^3\\&=& 125ax^2\cdot\sin(x)+17ax^3\\\\\quad u(x) &=& 125ax^2 &\Rightarrow & u'(x) &=& 250ax\\\quad v(x) &=& \sin(x) &\Rightarrow & v'(x) &=& \cos(x)\\\\f'(x) &=& 250ax\cdot \sin(x)+125ax^2\cdot\cos(x)+51ax^2\\&=& ax\left(250\sin(x)+125x\cdot\cos(x)+51x\right)\\\end{array}


18)
\begin{array}{ccllrcl}\quad\mathbb{D} &=& \mathbb{R}\\\\f(x) &=& e^x\cdot\sin(x)\\\\\quad u(x) &=& e^x &\Rightarrow & u'(x) &=& e^x\\\quad v(x) &=& \sin(x) &\Rightarrow & v'(x) &=& \cos(x)\\\\f'(x) &=& e^x\cdot\sin(x)+e^x\cdot\cos(x)\\&=& e^x\left(\sin(x)+\cos(x)\right)\\\end{array}


19)
\begin{array}{cclcrcl}\quad\mathbb{D} &=& \mathbb{R}\setminus_{\{k\pi+\frac{\pi}{2}; k \in \mathbb{Z}\}} \\\\f(x) &=& -6x^3\tan(x)-18 \\\\\quad u(x) &=& -6x^3 &\Rightarrow &u'(x)&=& -18x^2 \\\quad v(x) &=& \tan(x) &\Rightarrow &v'(x) &=& \dfrac{1}{\cos^2(x)} \\\\f'(x) &=& -18x^2\cdot\tan(x)-6x^3\cdot \dfrac{1}{\cos^2(x)} \\&=& -6x^2\left(3\tan(x)+\dfrac{x}{\cos^2(x)}\right)\end{array}


20)
\begin{array}{cclcrcl}\quad\mathbb{D} &=& \mathbb{R}\setminus_{\{k\pi+\frac{\pi}{2}; k \in \mathbb{Z}\}} \\\\f(s) &=& -\tan(s)\left(\dfrac{s}{\pi}+\cos(s)\right) \\\\\quad u(s) &=& -\tan(s) &\Rightarrow & u'(s)&=& -\dfrac{1}{\cos^2(s)} \\\quad v(s) &=& \dfrac{1}{\pi}s+\cos(s) &\Rightarrow & v'(s) &=& \dfrac{1}{\pi}-\sin(s) \\\\f'(s) &=& -\dfrac{1}{\cos^2(s)}\cdot \left(\dfrac{s}{\pi}+\cos(s)\right) -\tan(s)\cdot \left(\dfrac{1}{\pi}-\sin(s)\right)\end{array}

 

4. Aufgabe

1)
\begin{array}{lclcrcl}\quad\mathbb{D}&=&\mathbb{R}\setminus_{\left\{100\right\}} \cr \cr f(x) &=& \dfrac{x^{4}}{100 - x} \cr \cr \quad u(x) &=& x^{4} & \Rightarrow & u'(x) &=&4 x^{3}\cr \quad v(x) &=&100 - x & \Rightarrow & v'(x) &=& -1\cr \cr f'(x) &=& \dfrac{4 x^{3}\cdot \left(100 - x\right)-x^{4} \cdot \left(-1\right)}{\left(100 - x\right)^2} \cr \cr &=& \dfrac{400x^3-4x^4+x^{4}}{\left(100 - x\right)^{2}}\cr \cr &=& \dfrac{x^{3} \left(400 - 3 x\right)}{\left(100 - x\right)^{2}}\end{array}


2)
\begin{array}{lclcrcl}\quad\mathbb{D}&=&\mathbb{R}\setminus_{\left\{0\right\}}\cr \cr f(x) &=& \dfrac{\dfrac{x^{2}}{9} + 1}{- 27 x^{13}} \cr \cr \quad u(x) &=& \dfrac{x^{2}}{9} + 1 & \Rightarrow & u'(x) &=&\dfrac{2}{9}x \cr \quad v(x) &=&- 27 x^{13} & \Rightarrow & v'(x) &=& - 351 x^{12}\cr \cr f'(x) &=& \dfrac{\dfrac{2}{9}x\cdot \left(- 27 x^{13}\right)-\left(\dfrac{x^{2}}{9} + 1 \right)\cdot \left(- 351 x^{12}\right)}{\left(- 27 x^{13}\right)^2}\cr \cr &=& \dfrac{- 6 x^{14}+ 39 x^{14} + 351x^{12}}{729 x^{26}}\cr \cr &=& \dfrac{x^{12} \left(33 x^{2} + 351\right)}{729 x^{26}}\cr \cr &=& \dfrac{11 x^{2} + 117}{243 x^{14}}\end{array}


3)
\begin{array}{lclcrcl}\quad\mathbb{D}&=&\mathbb{R}\setminus_{\left\{\frac{7}{12}\right\}} \cr \cr f(q) &=& \dfrac{\left(- 12 q+7\right)^{4}}{144 q^{2} - 168 q + 49}\cr \cr &=& \dfrac{\left(- 12 q+7\right)^{4}}{\left(- 12 q+7\right)^{2}} \cr \cr &=& \left( - 12 q+7\right)^{2}\cr \cr &=& 144q^2-168q + 49\cr \cr \cr f'(q) &=& 288 q - 168\end{array}

Bemerkung: Diese Funktion kann man so umformen, dass sie ohne die Quotientenregel abgeleitet werden kann. Das ist natürlich viel einfacher.


4)
\begin{array}{lclcrcl}\quad\mathbb{D}&=&\mathbb{R}\setminus_{\left\{-5\right\}}\cr \cr f(x) &=& \dfrac{x^{3}}{x + 5}\cr \cr \quad u(x) &=& x^{3} & \Rightarrow & u'(x) &=&3 x^{2}\cr \quad v(x) &=&x + 5 & \Rightarrow & v'(x) &=& 1 \cr \cr f'(x) &=& \dfrac{3 x^{2}\cdot \left(x + 5\right)-x^{3} \cdot 1}{\left(x + 5\right)^2}\cr \cr &=& \dfrac{3x^{3} + 15x^2-x^{3}}{\left(x + 5\right)^{2}}\cr \cr &=& \dfrac{x^{2} \left(2 x + 15\right)}{\left(x + 5\right)^{2}}\end{array}


5)
\begin{array}{lclcrcl}\quad\mathbb{D}&=&\mathbb{R}\setminus_{\left\{0\right\}}\cr \cr f(x) &=& \dfrac{4 x + 2}{7 x^{2}}\cr \cr \quad u(x) &=& 4 x + 2 & \Rightarrow & u'(x) &=&4\cr \quad v(x) &=&7 x^{2} & \Rightarrow & v'(x) &=& 14 x\cr \cr f'(x) &=& \dfrac{4 \cdot 7 x^{2}-\left(4 x + 2 \right)\cdot 14 x}{\left(7 x^{2}\right)^2}\cr \cr &=& \dfrac{28 x^{2}-56 x^{2}-28x}{49 x^{4}}\cr \cr &=& \dfrac{- 28 x \left(x + 1\right)}{49 x^{4}}\cr \cr &=& - \dfrac{4 x + 4}{7 x^{3}}\end{array}


6)
\begin{array}{lclcrcl}\quad\mathbb{D}&=&\mathbb{R}\setminus_{\left\{- \frac{2}{9}\right\}}\cr \cr f(x) &=& \dfrac{3 x + 5}{9 x + 2}\cr \cr \quad u(x) &=& 3 x + 5 & \Rightarrow & u'(x) &=&3\cr \quad v(x) &=&9 x + 2 & \Rightarrow & v'(x) &=& 9\cr \cr f'(x) &=& \dfrac{3 \cdot \left(9 x + 2\right)-\left(3 x + 5 \right)\cdot 9}{\left(9 x + 2\right)^2}\cr \cr &=& \dfrac{27 x + 6-27 x - 45}{\left(9 x + 2\right)^{2}}\cr \cr &=& \dfrac{-39}{\left(9 x + 2\right)^{2}}\end{array}


7)
\begin{array}{lclcrcl}\quad\mathbb{D}&=&\mathbb{R}\cr \cr f(z) &=& \dfrac{23}{z^{-42}}\cr \cr &=& 23z^{42}\cr \cr\cr f'(z) &=& 42 \cdot 23z^{41}\cr \cr &=& 966z^{41}\end{array}

Bemerkung: Auch diese Funktion kann man so umformen, dass sie ohne die Quotientenregel abgeleitet werden kann.


8)
\begin{array}{lclcrcl}\quad\mathbb{D}&=&\mathbb{R} \cr \cr f(x) &=& \dfrac{x}{2 x^{2} + 4} \cr \cr \quad u(x) &=& x & \Rightarrow & u'(x) &=&1\cr \quad v(x) &=&2 x^{2} + 4 & \Rightarrow & v'(x) &=& 4 x \cr \cr f'(x) &=& \dfrac{1 \cdot \left(2 x^{2} + 4\right)-x \cdot 4 x}{\left(2 x^{2} + 4\right)^2} \cr \cr &=& \dfrac{2 x^{2} + 4-4 x^{2}}{\left(2x^{2} + 4\right)^{2}} \cr \cr &=& \dfrac{4 - 2 x^{2}}{4\left(x^{2} + 2\right)^{2}} \cr \cr &=& \dfrac{2 - x^{2}}{2 \left(x^{2} + 2\right)^{2}} \end{array}


9)
\begin{array}{lclcrcl}\quad\mathbb{D}&=&\mathbb{R}\setminus_{\left\{\sqrt[3]{-32}\right\}}\cr \cr f(x) &=& \dfrac{15 x^{2} - 8 x}{x^{3} + 32}\cr \cr \quad u(x) &=& 15 x^{2} - 8 x & \Rightarrow & u'(x) &=&30 x - 8\cr \quad v(x) &=&x^{3} + 32 & \Rightarrow & v'(x) &=& 3 x^{2}\cr \cr f'(x) &=& \dfrac{\left(30 x - 8\right )\cdot \left(x^{3} + 32\right)-\left(15 x^{2} - 8 x \right)\cdot 3 x^{2}}{\left(x^{3} + 32\right)^2}\cr \cr &=& \dfrac{30x^4-8x^3+960x-256-45x^4+24x^3}{\left(x^{3} + 32\right)^{2}}\cr \cr &=& \dfrac{- 15 x^{4} + 16 x^{3} + 960 x - 256}{\left(x^{3} + 32\right)^{2}}\end{array}


10)
\begin{array}{lclcrcl}\quad\mathbb{D}&=&\mathbb{R}\setminus_{\left\{\frac{3}{2}\right\}}\cr \cr f(x) &=& \dfrac{8 x + 3}{4 x - 6} \cr \cr \quad u(x) &=& 8 x + 3 & \Rightarrow & u'(x) &=&8\cr \quad v(x) &=&4 x - 6 & \Rightarrow & v'(x) &=& 4\cr \cr f'(x) &=& \dfrac{8\cdot \left(4 x - 6\right)-\left(8 x + 3 \right)\cdot 4}{\left(4 x - 6\right)^2} \cr \cr &=& \dfrac{32 x - 48-32 x - 12}{\left(4 x - 6\right)^{2}}\cr \cr &=& \dfrac{-60}{4\left(2x - 3\right)^{2}} \cr \cr &=& - \dfrac{15}{\left(2 x - 3\right)^{2}}\end{array}


11)
\begin{array}{lclcrcl}\quad\mathbb{D}&=&\mathbb{R}^+\cr \cr f(w) &=& \dfrac{w - 13}{- 8 \sqrt{w}}\cr \cr \quad u(w) &=& w - 13 & \Rightarrow & u'(w) &=&1\cr \quad v(w) &=&- 8 \sqrt{w} = -8w^{\frac{1}{2}} & \Rightarrow & v'(w) &=& -8\cdot\dfrac{1}{2}w^{-\frac{1}{2}} = - \dfrac{4}{\sqrt{w}} \cr \cr f'(w) &=& \dfrac{1\cdot \left(- 8 \sqrt{w}\right)-\left(w - 13 \right)\cdot \left(- \dfrac{4}{\sqrt{w}}\right)}{\left(- 8 \sqrt{w}\right)^2} \cr \cr &=& \dfrac{-\dfrac{8w}{\sqrt{w}}+\dfrac{4w - 52}{\sqrt{w}}}{64 w} \cr \cr &=& \dfrac{\dfrac{-4 w - 52}{\sqrt{w}}}{64 w} \cr \cr &=& -\dfrac{4(w +13)}{\sqrt{w}}\cdot\dfrac{1}{64 w} \cr \cr &=& - \dfrac{w + 13}{16 \sqrt{w^3}}\end{array}


12)
\begin{array}{lclcrcl}\quad\mathbb{D}&=&\mathbb{R}\setminus_{\left\{10\right\}} \cr \cr f(x) &=& \dfrac{12 \sqrt{x}}{x - 10} \cr \cr \quad u(x) &=& 12 \sqrt{x} = 12x^{\frac{1}{2}} & \Rightarrow & u'(x) &=& 12\cdot\dfrac{1}{2}x^{-\frac{1}{2}} = \dfrac{6}{\sqrt{x}}\cr \quad v(x) &=&x - 10 & \Rightarrow & v'(x) &=& 1\cr \cr f'(x) &=& \dfrac{\dfrac{6}{\sqrt{x}}\cdot \left(x - 10\right)-12 \sqrt{x}\cdot 1}{\left(x - 10\right)^2} \cr \cr &=& \dfrac{\dfrac{6x - 60}{\sqrt{x}}-\dfrac{12x}{\sqrt{x}}}{\left(x - 10\right)^{2}} \cr \cr &=& \dfrac{\dfrac{-6 x - 60}{\sqrt{x}}}{\left(x - 10\right)^{2}} \cr \cr &=& -\dfrac{6 x + 60}{\sqrt{x}}\cdot\dfrac{1}{\left(x - 10\right)^{2}} \cr \cr &=& - \dfrac{6 x + 60}{\sqrt{x} \left(x - 10\right)^{2}} \end{array}


13)
\begin{array}{lclcrcl}\quad\mathbb{D}&=&\mathbb{R}\cr \cr f(z) &=& \dfrac{2^{z} - 1}{4^{z}} \cr \cr \quad u(z) &=& 2^{z} - 1 & \Rightarrow & u'(z) &=&2^{z} \ln{\left(2 \right)}\cr \quad v(z) &=&4^{z} & \Rightarrow & v'(z) &=& 4^{z} \ln{\left(4 \right)}\cr \cr f'(z) &=& \dfrac{2^{z} \ln\left(2 \right)\cdot4^z-\left(2^{z} - 1 \right)\cdot 4^{z} \ln{\left(4 \right)}}{\left(4^{z}\right)^2}\cr \cr &=& \dfrac{8^{z} \ln{\left(2 \right)}-4^{z} \cdot 2^{z} \ln{\left(4 \right)}+4^z\ln{\left(4\right)}}{4^{2 z}} \cr \cr &=& \dfrac{8^{z} \ln{\left(2 \right)}-8^{z} \ln{\left(2^2 \right)}+4^z\ln{\left(2^2\right)}}{4^{2 z}} \cr\cr &=& \dfrac{8^{z} \ln{\left(2 \right)}-8^{z} \cdot 2\ln{\left(2 \right)}+4^z\cdot 2\ln{\left(2\right)}}{4^{2 z}} \cr \cr &=& \dfrac{\left(2 \cdot 4^{z} - 8^{z}\right) \ln{\left(2 \right)}}{16^z} \cr \cr &=& 16^{- z} \left(2 \cdot 4^{z} - 8^{z}\right) \ln{\left(2 \right)}\end{array}


14)
\begin{array}{lclcrcl}\quad\mathbb{D}&=&\mathbb{R}\setminus_{\left\{- \frac{1}{7}\right\}}\cr \cr f(x) &=& \dfrac{21 e^{x}}{7 x + 1}\cr \cr \quad u(x) &=& 21 e^{x} & \Rightarrow & u'(x) &=&21 e^{x}\cr \quad v(x) &=&7 x + 1 & \Rightarrow & v'(x) &=& 7\cr \cr f'(x) &=& \dfrac{21 e^{x}\cdot \left(7 x + 1\right)-21 e^{x}\cdot 7}{\left(7 x + 1\right)^2}\cr \cr &=& \dfrac{147x e^x + 21 e^{x}-147 e^{x}}{\left(7 x + 1\right)^{2}}\cr \cr &=& \dfrac{\left(147 x - 126\right) e^{x}}{\left(7 x + 1\right)^{2}}\end{array}


15)
\begin{array}{lclcrcl}\quad\mathbb{D}&=&\mathbb{R}\setminus_{\left\{0\right\}} \cr \cr f(x) &=& \dfrac{17 e^{x} - 1}{34 x^{2}} \cr \cr \quad u(x) &=& 17 e^{x} - 1 & \Rightarrow & u'(x) &=&17 e^{x}\cr \quad v(x) &=&34 x^{2} & \Rightarrow & v'(x) &=& 68 x \cr \cr f'(x) &=& \dfrac{17 e^{x}\cdot 34 x^{2}-\left(17 e^{x} - 1 \right)\cdot 68 x}{\left(34 x^{2}\right)^2} \cr \cr &=& \dfrac{578 x^{2} e^{x}-1.156x e^{x} +68x}{1.156 x^{4}}\cr \cr &=& \dfrac{34 x \left(17 x e^{x} - 34 e^{x} + 2\right)}{1.156 x^{4}} \cr \cr &=& \dfrac{17 x e^{x} - 34 e^{x} + 2}{34 x^{3}} \end{array}


16)
\begin{array}{lclcrcl}\quad\mathbb{D}&=&\mathbb{R}^+\setminus_{\left\{1\right\}} \cr \cr f(x) &=& \dfrac{x^{2} + 18}{18 \lg\left(x \right)} \cr \cr \quad u(x) &=& x^{2} + 18 & \Rightarrow & u'(x) &=&2 x\cr \quad v(x) &=&18 \lg\left(x \right) & \Rightarrow & v'(x) &=& \dfrac{18}{x \ln{\left(10 \right)}}\cr \cr f'(x) &=& \dfrac{2 x\cdot 18 \lg\left(x \right)-\left(x^{2} + 18 \right)\cdot \dfrac{18}{x \ln{\left(10 \right)}}}{\left(18 \lg\left(x \right)\right)^2} \cr \cr &=& \dfrac{36 x \lg{\left(x \right)}-\dfrac{18x^{2} + 324}{x \ln{\left(10 \right)}}}{324 \lg^2{\left(x \right)}} \cr\cr &=& \dfrac{\dfrac{36 x^2 \ln(10)\lg{\left(x \right)}}{x \ln(10)}-\dfrac{18x^{2} + 324}{x \ln{\left(10 \right)}}}{324 \lg^2{\left(x \right)}} \cr\cr &=& \dfrac{18 \left(2 x^{2} \ln(10)\lg{\left(x \right)} - x^{2} - 18\right)}{x \ln{\left(10 \right)}}\cdot\dfrac{1}{324 \lg^2{\left(x \right)}} \cr \cr &=& \dfrac{2 x^{2} \ln(10)\lg{\left(x \right)} - x^{2} - 18}{18 x \ln(10) \lg^2{\left(x \right)}}\end{array}


17)
\begin{array}{lclcrcl}\quad\mathbb{D}&=&\mathbb{R}\setminus_{\left\{0\right\}}\cr \cr f(x) &=& \dfrac{\ln{\left(x \right)}}{x^{11}} \cr \cr \quad u(x) &=& \ln{\left(x \right)} & \Rightarrow & u'(x) &=&\dfrac{1}{x}\cr \quad v(x) &=&x^{11} & \Rightarrow & v'(x) &=& 11 x^{10}\cr \cr f'(x) &=& \dfrac{\dfrac{1}{x}\cdot x^{11}-\ln{\left(x \right)} \cdot 11 x^{10}}{\left(x^{11}\right)^2}\cr \cr &=& \dfrac{x^{10}-11 x^{10} \ln{\left(x \right)}}{x^{22}}\cr \cr &=& \dfrac{x^{10} \left(1 - 11 \ln{\left(x \right)}\right)}{x^{22}} \cr \cr &=& \dfrac{1 - 11 \ln{\left(x \right)}}{x^{12}}\end{array}


18)
\begin{array}{lclcrcl}\quad\mathbb{D}&=&\mathbb{R}\setminus_{\left\{0; 1\right\}} \cr \cr g(x) &=& \dfrac{- 2 \ln{\left(x \right)}}{x^{31} - x^{26}} \cr \cr \quad u(x) &=& - 2 \ln{\left(x \right)} & \Rightarrow & u'(x) &=&- \dfrac{2}{x}\cr \quad v(x) &=&x^{31} - x^{26} & \Rightarrow & v'(x) &=& 31 x^{30} - 26 x^{25} \cr \cr g'(x) &=& \dfrac{- \dfrac{2}{x}\cdot \left(x^{31} - x^{26}\right)-\left(- 2 \ln{\left(x \right)} \right)\cdot \left(31 x^{30} - 26 x^{25}\right)}{\left(x^{31} - x^{26}\right)^2} \cr \cr &=& \dfrac{-2 x^{30} +2x^{25} +62 x^{30}\ln{\left(x \right)}-52x^{25}\ln\left(x\right)}{\left(x^{31} - x^{26}\right)^{2}} \cr\cr &=& \dfrac{2 x^{25} \left(- x^{5} + 1 + 31 x^{5} \ln\left(x\right) - 26 \ln{\left(x \right)}\right)}{x^{52}\left(x^{5} - 1\right)^{2}} \cr\cr &=& \dfrac{2 \left(- x^{5} + \left(31 x^{5} - 26\right) \ln{\left(x \right)} + 1\right)}{x^{27}\left(x^{5} - 1\right)^{2}}\end{array}


19)
\begin{array}{lclcrcl}\quad\mathbb{D}&=&\mathbb{R}\setminus_{\left\{0\right\}}\cr \cr f(y) &=& \dfrac{\pi \sin{\left(y \right)}}{- y^{2}} -12\pi\cr \cr \quad u(y) &=& \pi \sin{\left(y \right)} & \Rightarrow & u'(y) &=&\pi \cos{\left(y \right)}\cr \quad v(y) &=&- y^{2} & \Rightarrow & v'(y) &=& - 2 y\cr \cr f'(y) &=& \dfrac{\pi \cos{\left(y \right)}\cdot \left(- y^{2}\right)-\pi \sin{\left(y \right)} \cdot \left(- 2 y\right)}{\left(- y^{2}\right)^2}\cr \cr &=& \dfrac{- \pi y^{2} \cos{\left(y \right)}+2 \pi y \sin{\left(y \right)}}{y^{4}}\cr \cr &=& \dfrac{\pi y \left(- y \cos{\left(y \right)} + 2 \sin{\left(y \right)}\right)}{y^{4}}\cr \cr &=& \dfrac{\pi \left(- y \cos{\left(y \right)} + 2 \sin{\left(y \right)}\right)}{y^{3}}\end{array}   

 
20)
\begin{array}{lclcrcl}\quad\mathbb{D}&=&\left\{t \in \mathbb{R}\; |\; t\neq \frac{\pi}{2}+2 k \pi \text{ und } t\neq \frac{3\pi}{2}+2 k \pi , k \in \mathbb{Z} \right\} \cr \cr y(t) &=& \dfrac{\sin{\left(t \right)}}{\cos{\left(t \right)}} \cr \cr \quad u(t) &=& \sin{\left(t \right)} & \Rightarrow & u'(t) &=&\cos{\left(t \right)}\cr \quad v(t) &=&\cos{\left(t \right)} & \Rightarrow & v'(t) &=& - \sin{\left(t \right)} \cr \cr y'(t) &=& \dfrac{\cos{\left(t \right)}\cdot \cos{\left(t \right)}-\sin{\left(t \right)}\cdot \left(- \sin{\left(t \right)}\right)}{\left(\cos{\left(t \right)}\right)^2} \cr \cr &=& \dfrac{\cos^{2}{\left(t \right)}+ \sin^{2}{\left(t \right)}}{\cos^{2}{\left(t \right)}} \cr \cr &=& \dfrac{1}{\cos^{2}{\left(t \right)}} \end{array}

 

5. Aufgabe

1)
\begin{array}{lclcrcl}\quad\mathbb{D} &=& \mathbb{R} \cr \cr f(x) &=& (2x+1)^7 \cr \cr \quad g(h(x)) &=& (h(x))^7 &\Rightarrow& g'(h(x)) &=& 7(h(x))^6 \cr \quad h(x) &=& 2x+1 &\Rightarrow& h'(x) &=& 2 \cr \cr f'(x) &=& 7(2x+1)^6\cdot 2 \cr &=& 14(2x+1)^6\end{array}


2)
\begin{array}{lclcrcl}\quad\mathbb{D} &=& \mathbb{R}\setminus_{\{0\}} \\\\f(x) &=& 24\left(x+\dfrac{7}{8x}\right)^2 \\\\\quad g(h(x)) &=& 24\left(h(x)\right)^2 & \Rightarrow & g'(h(x)) &=& 48h(x) \\\quad h(x) &=& x+\dfrac{7}{8x} = x+\dfrac{7}{8}x^{-1} & \Rightarrow & h'(x) &=& 1-\dfrac{7}{8}x^{-2} = 1-\dfrac{7}{8x^2} \\\\f'(x) &=& 48\left(x+\dfrac{7}{8x}\right)\cdot \left(1-\dfrac{7}{8x^2}\right) \\\\&=& 48\left(x-\dfrac{7}{8x}+\dfrac{7}{8x}-\dfrac{49}{64x^3}\right) \\\\&=& 48\left(x-\dfrac{49}{64x^3}\right) \\\\&=& 48x-\dfrac{147}{4x^3}\end{array}


3)
\begin{array}{lclcrcl}\quad\mathbb{D} &=& \mathbb{R}\setminus_{\{0; 1\}} \\\\f(x) &=& \dfrac{-63}{x^2-\frac{1}{x}} \\&=& -63\left(x^2-\dfrac{1}{x}\right)^{-1} \\\\\quad g(h(x)) &=& -63\left(h(x)\right)^{-1} & \Rightarrow & g'(h(x)) &=& 63\left(h(x)\right)^{-2} \\\quad h(x) &=& x^2-\dfrac{1}{x} = x^2-x^{-1} & \Rightarrow & h'(x) &=& 2x+x^{-2} = 2x+\dfrac{1}{x^2} \\\\f'(x) &=& 63\left(x^2-\dfrac{1}{x}\right)^{-2}\cdot\left(2x+\dfrac{1}{x^2}\right) \\\\&=& \dfrac{63\cdot\left(2x+\frac{1}{x^2}\right)}{\left(x^2-\frac{1}{x}\right)^{2}} \\\\&=& \dfrac{126x+\frac{63}{x^2}}{\left(x^2-\frac{1}{x}\right)^2}\end{array}


4)
\begin{array}{lclcrcl}\quad\mathbb{D} &=& \mathbb{R}^+ \\\\g(x) &=& \sqrt{112x^3+97x}+16 \\&=& \left(112x^3+97x\right)^{\frac{1}{2}}+16 \\\\\quad g(h(x)) &=& (h(x))^{\frac{1}{2}}+16 & \Rightarrow & g'(h(x)) &=& \dfrac{1}{2}(h(x))^{-\frac{1}{2}} \\\quad h(x) &=& 112x^3+97x & \Rightarrow & h'(x) &=& 336x^2+97 \\\\g'(x) &=& \dfrac{1}{2}\left(112x^3+97x\right)^{-\frac{1}{2}}\cdot \left(336x^2+97\right) \\\\&=& \dfrac{1}{2\left(112x^3+97x\right)^{\frac{1}{2}}}\cdot \left(336x^2+97\right) \\\\&=& \dfrac{336x^2+97}{2\sqrt{112x^3+97x}}\end{array}


5)
\begin{array}{lclcrcl}\quad\mathbb{D} &=& \left]2; \infty\right[ \\\\f(x) &=& \dfrac{1}{\sqrt{3x-6}} \\&=& (3x-6)^{-\frac{1}{2}} \\\\\quad g(h(x)) &=& (h(x))^{-\frac{1}{2}} & \Rightarrow & g'(h(x)) &=& -\dfrac{1}{2}(h(x))^{-\frac{3}{2}} \\\quad h(x) &=& 3x-6 & \Rightarrow & h'(x) &=& 3 \\\\f'(x) &=& -\dfrac{1}{2}(3x-6)^{-\frac{3}{2}}\cdot 3 \\\\&=& -\dfrac{3}{2\cdot (3x-6)^{\frac{3}{2}}} \\\\&=& -\dfrac{3}{2\sqrt{(3x-6)^3}}\end{array}


6)
\begin{array}{lclcrcl}\quad\mathbb{D} &=& \left[-4; \infty\right[ \\\\f(x) &=& \sqrt{(x+4)^3} \\&=& (x+4)^{\frac{3}{2}} \\\\\quad g(h(x)) &=& (h(x))^{\frac{3}{2}} & \Rightarrow & g'(h(x)) &=& \dfrac{3}{2}(h(x))^{\frac{1}{2}} \\ \quad h(x) &=& x+4 & \Rightarrow & h'(x) &=& 1 \\\\f'(x) &=& \dfrac{3}{2}(x+4)^{\frac{1}{2}}\cdot 1 \\\\&=& \dfrac{3}{2}\sqrt{x+4}\end{array}


7)
\begin{array}{lclcrcl}\quad\mathbb{D} &=& \mathbb{R} \\\\f(x) &=& \sqrt[3]{x+4} \\&=& (x+4)^{\frac{1}{3}} \\\\\quad g(h(x)) &=& (h(x))^{\frac{1}{3}} & \Rightarrow & g'(h(x)) &=& \dfrac{1}{3}(h(x))^{-\frac{2}{3}} \\\quad h(x) &=& x+4 & \Rightarrow & h'(x) &=& 1 \\\\f'(x) &=& \dfrac{1}{3}(x+4)^{-\frac{2}{3}}\cdot 1 \\\\&=& \dfrac{1}{3(x+4)^{\frac{2}{3}}} \\\\&=& \dfrac{1}{3\sqrt[3]{(x+4)^2}}\end{array}


8)
\begin{array}{lclcrcl} \quad\mathbb{D} &=& \mathbb{R} \\\\f(t) &=& \ln\left(\dfrac{5}{2}t^2+14\right) \\ \\\quad g(h(t)) &=& \ln\left(h(t)\right) & \Rightarrow & g'(h(t)) &=& \dfrac{1}{h(t)} \\\quad h(t) &=& \frac{5}{2}t^2+14 & \Rightarrow & h'(t) &=& 5t \\\\f'(t) &=& \dfrac{1}{\frac{5}{2}t^2+14}\cdot 5t \\&=& \dfrac{5t}{\frac{5}{2}t^2+14}\end{array}


9)
\begin{array}{lclcrcl}\quad\mathbb{D} &=& \mathbb{R}^+ \\\\f(x) &=& \ln\left(\dfrac{12}{7}\cdot \dfrac{1}{\sqrt{x}}\right) \\\\&=& \ln\left(\dfrac{12}{7}x^{-\frac{1}{2}}\right) \\\\\quad g(h(x)) &=& \ln\left(h(x)\right) & \Rightarrow & g'(h(x)) &=& \dfrac{1}{h(x)} \\\\\quad h(x) &=& \dfrac{12}{7}x^{-\frac{1}{2}} & \Rightarrow & h'(x) &=& \dfrac{12}{7}\cdot \left(-\dfrac{1}{2}\right)x^{-\frac{3}{2}} = -\dfrac{6}{7}\cdot x^{-\frac{3}{2}} \\\\f'(x) &=& \dfrac{1}{\dfrac{12}{7}x^{-\frac{1}{2}}} \cdot \left(-\dfrac{6}{7}x^{-\frac{3}{2}}\right) \\\\&=& \dfrac{7}{12}x^{\frac{1}{2}} \cdot \left(-\dfrac{6}{7}\right) x^{-\frac{3}{2}} \\\\&=& -\dfrac{1}{2}x^{-1} \\\\&=& -\dfrac{1}{2x}\end{array}


10)
\begin{array}{lclcrcl}\quad\mathbb{D} &=& \left]-\frac{1}{756};0\right[ \\\\f(x) &=& \ln\left(-42x^4-\dfrac{1}{18}x^3\right) \\\\\quad g(h(x)) &=& \ln\left(h(x)\right) & \Rightarrow & g'(h(x)) &=& \dfrac{1}{h(x)} \\\quad h(x) &=& -42x^4-\dfrac{1}{18}x^3 & \Rightarrow & h'(x) &=& -168x^3-\dfrac{1}{6}x^2 \\\\f'(x) &=& \dfrac{1}{-42x^4-\frac{1}{18}x^3}\cdot \left(-168x^3-\dfrac{1}{6}x^2\right) \\\\&=& \dfrac{-168x^3-\frac{1}{6}x^2}{-42x^4-\frac{1}{18}x^3} \\\\&=& \dfrac{x^2\left(-168x-\frac{1}{6}\right)}{x^2\left(-42x^2-\frac{1}{18}x\right)} \\\\&=& \dfrac{-168x-\frac{1}{6}}{-42x^2-\frac{1}{18}x}\end{array}


11)
\begin{array}{lclcrcl}\quad\mathbb{D} &=& \mathbb{R} \\\\f(x) &=& \left(\sin(x)\right)^2 \\\\\quad g(h(x)) &=& \left(h(x)\right)^2 & \Rightarrow & g'(h(x)) &=& 2h(x) \\\quad h(x) &=& \sin(x) & \Rightarrow & h'(x) &=& \cos(x) \\\\f'(x) &=& 2\sin(x)\cdot \cos(x)\end{array}


12)
\begin{array}{lclcrcl}\mathbb{D} &=& \mathbb{R} \\\\g(y) &=& \sqrt{\dfrac{121}{(72a-1)^2+5}} \\\\g'(y) &=& 0 \end{array}

Bemerkung 1: Da die Funktion g(y) heißt, wissen wir, dass die Variable y ist und nicht a, wie man denken könnte. a ist eine Konstante. Damit ist der Funktionsterm konstant und die Ableitung einer Konstanten ist 0.

Bemerkung 2: Da (72a-1)^2+5 immer größer als 0 ist, darf a\in\mathbb{R} sein.


13)
\begin{array}{lclcrcl}\quad\mathbb{D} &=& \mathbb{R}_0^+ \\\\f(x) &=& e^{x^2+\sqrt{x}} \\\\\quad g(h(x)) &=& e^{h(x)} & \Rightarrow & g'(h(x)) &=& e^{h(x)} \\\quad h(x) &=& x^2+\sqrt{x} = x^2+x^{\frac{1}{2}} & \Rightarrow & h'(x) &=& 2x+\dfrac{1}{2}x^{-\frac{1}{2}} = 2x+\dfrac{1}{2\sqrt{x}} \\f'(x) &=& e^{x^2+\sqrt{x}}\cdot\left(2x+\dfrac{1}{2\sqrt{x}}\right) \\\\&=&e^{x^2+\sqrt{x}}\cdot\left(2x+\dfrac{\sqrt{x}}{2x}\right)\end{array}


14)
\begin{array}{lclcrcl}\quad\mathbb{D} &=& \mathbb{R} \\\\f(x) &=& -36\sin\left(\cos(x)\right) \\\\\quad g(h(x)) &=& -36\sin\left(h(x)\right) & \Rightarrow & g'(h(x)) &=& -36\cos\left(h(x)\right) \\ \quad h(x) &=& \cos(x) & \Rightarrow & h'(x) &=& -\sin(x) \\\\ f'(x) &=& -36\cos\left(\cos(x)\right)\cdot \left(-\sin(x)\right) \\&=& 36\cos\left(\cos(x)\right)\cdot\sin(x)\\\end{array}


Bemerkung: Bei dreifach verketteten Funktionen wie den folgenden Aufgaben 15) bis 18) gibt es beim Ableiten verschiedene Wege: Statt des hier gezeigten Weges können Sie solche Funktionen auch in zwei Schritten ableiten. Am Beispiel von 15) bedeutet das: Sie leiten erst h(x)=\sin\left(-x^2+13\right) ab (mit der Kettenregel natürlich). Dann benötigen Sie zum anschließenden Ableiten von f(x) nur noch einmal die Kettenregel, weil Sie die innere Ableitung bereits vorher berechnet haben. Das Ergebnis ist natürlich in beiden Fällen das gleiche. Insofern können Sie sich einfach aussuchen, welcher Weg für Sie übersichtlicher ist.


15)
\begin{array}{lclcrcl}\quad\mathbb{D} &=& \mathbb{R} \\\\f(x) &=& e^{\sin\left(-x^2+13\right)} \\\\\quad g(h(k(x))) &=& e^{h(k(x))} & \Rightarrow & g'(h(k(x))) &=& e^{h(k(x))} \\\quad h(k(x)) &=& \sin(k(x)) & \Rightarrow & h'(k(x)) &=& \cos(k(x)) \\\quad k(x) &=& -x^2+13 & \Rightarrow & k'(x) &=& -2x \\\\f'(x) &=& e^{\sin\left(-x^2+13\right)}\cdot \cos\left(-x^2+13\right) \cdot (-2x) \\&=& -2x\cos\left(-x^2+13\right)\cdot e^{\sin\left(-x^2+13\right)}\end{array}


16)
\begin{array}{lclcrcl}\quad\mathbb{D} &=& \mathbb{R} \\\\f(x) &=& 85\ln\left(e^{3x}+x^2\right)-94 \\\\\quad g(h(k(x))) &=& 85\ln(h(k(x)))-94 & \Rightarrow & g'(h(k(x))) &=& 85\cdot \dfrac{1}{h(k(x))} \\\quad h(k(x)) &=& e^{k(x)}+x^2 & \Rightarrow & h'(k(x)) &=& e^{k(x)}+2x \\\quad k(x) &=& 3x & \Rightarrow & r'(x) &=& 3 \\\\f'(x) &=& 85\cdot \dfrac{1}{e^{3x}+x^2}\cdot \left(e^{3x}\cdot 3+2x\right) \\\\&=& \dfrac{85\cdot \left(3e^{3x}+2x\right)}{e^{3x}+x^2} \\\\&=& \dfrac{\left(255e^{3x}+170x\right)}{e^{3x}+x^2}\end{array}


17)
\begin{array}{lclcrcl}\quad\mathbb{D} &=& \mathbb{R} \\\\f(z) &=& -\dfrac{3}{10}\sqrt{1+\cos\left(z^2\right)} \\\\&=& -\dfrac{3}{10}\left(1+\cos\left(z^2\right)\right)^{\frac{1}{2}} \\\\\quad g(h(k(z))) &=& -\dfrac{3}{10}\left(h(k(z))\right)^{\frac{1}{2}} & \Rightarrow & g'(h(k(z))) &=& -\dfrac{3}{20}\left(h(k(z))\right)^{-\frac{1}{2}} \\\\\quad h(k(z)) &=& 1+\cos\left(k(z)\right) & \Rightarrow & h'(k(z)) &=& -\sin\left(k(z)\right) \\\\\quad k(z) &=& z^2 & \Rightarrow & k'(z) &=& 2z \\\\f'(z) &=& -\dfrac{3}{20}\left(1+\cos\left(z^2\right)\right)^{-\frac{1}{2}}\cdot \left(-\sin\left(z^2\right)\right)\cdot 2z \\\\&=& \dfrac{3z\cdot \sin\left(z^2\right)}{10\left(1+\cos\left(z^2\right)\right)^{\frac{1}{2}}} \\\\&=& \dfrac{3z\cdot \sin\left(z^2\right)}{10\sqrt{1+\cos(z^2)}}\end{array}


18)
\begin{array}{lclcrcl}\quad\mathbb{D} &=& \mathbb{R}^+_0 \\\\f(x) &=& \sqrt{e^{\sqrt{x}-t}} \\&=& \left(e^{\sqrt{x}-t}\right)^{\frac{1}{2}} \\\\\quad g(h(k(x))) &=& \left(h\left(k(x)\right)\right)^{\frac{1}{2}} & \Rightarrow & g'(h(k(x))) &=& \dfrac{1}{2}\left(h\left(k(x)\right)\right)^{-\frac{1}{2}} \\\quad h(k(x)) &=& e^{k(x)} & \Rightarrow & h'(k(x)) &=& e^{k(x)} \\\quad k(x) &=& \sqrt{x}-t & \Rightarrow & k'(x) &=& \dfrac{1}{2\sqrt{x}} \\\\f'(x) &=& \dfrac{1}{2}\left(e^{\sqrt{x}-t}\right)^{-\frac{1}{2}} \cdot e^{\sqrt{x}-t}\cdot\dfrac{1}{2\sqrt{x}} \\\\&=& \dfrac{1}{4}\cdot\dfrac{e^{\sqrt{x}-t}}{\sqrt{e^{\sqrt{x}-t}}}\cdot\dfrac{1}{\sqrt{x}} \\\\&=& \dfrac{\sqrt{e^{\sqrt{x}-t}}}{4\sqrt{x}}\end{array}


19)
\begin{array}{lclcrcl}\quad\mathbb{D} &=& \mathbb{R};\;x\ne\frac{-b}{a} \\\\f(x) &=& \dfrac{1}{ax+b} \\&=& (ax+b)^{-1} \\\\\quad g(h(x)) &=& (h(x))^{-1} & \Rightarrow & g'(h(x)) &=& -1(h(x))^{-2} \\\quad h(x) &=& ax+b & \Rightarrow & h'(x) &=& a \\\\f'(x) &=& -1(ax+b)^{-2}\cdot a \\&=& \dfrac{-a}{(ax+b)^2}\end{array}


20)
\begin{array}{lclcrcl}\quad\mathbb{D} &=& \mathbb{R} \\\\f(x) &=& \tan\left(c\cdot \sin(x)\right) \\\\\quad g(h(x)) &=& \tan\left(h(x)\right) & \Rightarrow & g'(h(x)) &=& \dfrac{1}{\cos^2\left(h(x)\right)} \\\quad h(x) &=& c\cdot \sin(x) & \Rightarrow & h'(x) &=& c\cdot \cos(x) \\\\f'(x) &=& \dfrac{1}{\cos^2\left(c\cdot\sin(x)\right)}\cdot c\cdot \cos(x) \\\\&=& \dfrac{c\cdot \cos(x)}{\cos^2\left(c\cdot\sin(x)\right)}\end{array}

 

6. Aufgabe

1)
\begin{array}{lclcrcl}\quad\mathbb{D}&=&\mathbb{R}\setminus_{\{-2\}} \cr \cr f(x) &=& \dfrac{1}{2x+4} \cr \cr \quad u(x) &=& 1 & \Rightarrow & u'(x) &=& 0 \cr \quad v(x) &=& 2x+4 & \Rightarrow & v'(x) &=& 2\cr \cr f'(x) &=& \dfrac{0 \cdot (2x+4)-1 \cdot 2}{(2x+4)^2} \cr \cr &=& \dfrac{-2}{4x^2+16x+16} \cr \cr &=& -\dfrac{2}{2(2x^2+8x+8)} \cr \cr &=& -\dfrac{1}{2x^2+8x+8} \end{array}

Vorgehen: Quotientenregel


2)
\begin{array}{lclcrcl}\quad\mathbb{D}&=&\mathbb{R} \cr \cr f(x) &=& \cos(7x+8) \cr \cr \quad g(h(x)) &=& \cos(h(x)) & \Rightarrow & g'(h(x)) &=& -\sin(h(x)) \cr \quad h(x) &=& 7x+8 & \Rightarrow & h'(x) &=& 7 \cr \cr f'(x) &=& -\sin(7x+8) \cdot 7 \cr &=& -7 \sin(7x+8) \end{array}

Vorgehen: Kettenregel


3)
\begin{array}{lclcrcl}\quad\mathbb{D}&=&\mathbb{R} \cr \cr f(x) &=& (x^2+1)\sin(x) \cr \cr \quad u(x) &=& x^2+1 & \Rightarrow & u'(x) &=& 2x \cr \quad v(x) &=& \sin(x) & \Rightarrow & v'(x) &=& \cos(x) \cr \cr f'(x) &=& 2x\sin(x)+(x^2+1)\cos(x) \end{array}

Vorgehen: Produktregel


4)
\begin{array}{lclcrcl}\quad\mathbb{D}&=&\mathbb{R} \cr \cr f(x) &=& 12xe^{x+10} \cr \cr \quad u(x) &=& 12x & \Rightarrow & u'(x) &=& 12 \cr \quad v(h(x)) &=& e^{h(x)} & \Rightarrow & v'(h(x)) &=& e^{h(x)} \cr \quad h(x) &=& x+10 & \Rightarrow & h'(x) &=& 1 \cr \cr f'(x) &=& 12 \cdot e^{x+10} + 12x e^{x+10} \cdot 1 \cr &=& 12e^{x+10}(1+x) \end{array}

Vorgehen: Produktregel, Kettenregel im 2. Faktor


5)
\begin{array}{lclcrcl}\quad\mathbb{D}&=&\mathbb{R}^+_0 \cr \cr f(x) &=& e^{\sqrt{x}} \cr \cr &=& e^{x^{\frac{1}{2}}} \cr \cr \quad g(h(x)) &=& e^{h(x)} & \Rightarrow & g'(h(x)) &=& e^{h(x)} \cr \quad h(x) &=& x^{\frac{1}{2}} & \Rightarrow & h'(x) &=& \dfrac{1}{2}x^{-\frac{1}{2}} \cr \cr f'(x) &=& e^{x^{\frac{1}{2}}} \cdot \dfrac{1}{2}x^{-\frac{1}{2}} \cr \cr &=& \dfrac{1}{2}x^{-\frac{1}{2}} e^{x^{\frac{1}{2}}} \cr \cr &=& \dfrac{1}{2\sqrt{x}} e^{\sqrt{x}} \end{array}

Vorgehen: Kettenregel


6)
\begin{array}{lclcrcl}\quad\mathbb{D}&=&\mathbb{R}\setminus_{\{1\}} \cr \cr f(x) &=& \dfrac{x^2+1}{x^3-1} \cr \cr \quad u(x) &=& x^2+1 & \Rightarrow & u'(x) &=& 2x \cr \quad v(x) &=& x^3-1 & \Rightarrow & v'(x) &=& 3x^2 \cr \cr f'(x) &=& \dfrac{2x \cdot (x^3-1)-(x^2+1) \cdot 3x^2}{(x^3-1)^2} \cr\cr &=& \dfrac{2x^4-2x-3x^4-3x^2}{x^6-2x^3+1} \cr\cr &=& \dfrac{-x^4-3x^2-2x}{x^6-2x^3+1} \end{array}

Vorgehen: Quotientenregel


7)
\begin{array}{rcl}\quad\mathbb{D}&=&\mathbb{R}^+ \cr \cr f(x)&=&\ln(x)+7^x+\sin(x)+5 \cr\cr\cr f'(x)&=&\dfrac{1}{x}+7^x\cdot\ln\left(7\right)+\cos(x)\end{array}

Vorgehen: Hier wird keine der komplexeren Ableitungsregeln benötigt. Jeder Summand kann für sich abgeleitet werden.


8)
\begin{array}{lclcrcl}\quad\mathbb{D}&=&\mathbb{R} \cr \cr f(x)&=&\cos(x^2) \cr\cr \quad g(h(x)) &=&\cos(h(x)) & \Rightarrow & g'(h(x)) &=& -\sin(h(x)) \cr \quad h(x) &=& x^2 & \Rightarrow & h'(x) &=& 2x \cr \cr f'(x)&=& -\sin(x^2) \cdot 2x \cr &=& -2x \cdot \sin(x^2)\end{array}

Vorgehen: Kettenregel


9)
\begin{array}{lclcrcl}\quad\mathbb{D}&=&\mathbb{R} \cr \cr f(x)&=&\cos^2(x) \cr\cr \quad g(h(x)) &=& (h(x))^2 & \Rightarrow & g'(h(x)) &=& 2h(x) \cr \quad h(x) &=& \cos(x) & \Rightarrow & h'(x) &=& -\sin(x) \cr \cr f'(x) &=& 2\cdot \cos(x) \cdot \left( -\sin(x) \right) \cr &=&-2\sin(x)\cos(x) \end{array}

Vorgehen: Kettenregel

Bemerkung:  \cos^2(x) = \left[ \cos(x) \right]^2


10)
\begin{array}{lclcrcl}\quad\mathbb{D}&=&\mathbb{R} \cr \cr f(x)&=&x^2 \cdot 10^x \cr\cr \quad u(x) &=& x^2 & \Rightarrow & u'(x) &=& 2x \cr \quad v(x) &=& 10^x & \Rightarrow & v'(x) &=& 10^x\cdot\ln\left(10\right) \cr \cr f'(x) &=& 2x \cdot 10^x + x^2 \cdot10^x\cdot\ln\left(10\right) \cr &=& 10^x\left(2x+x^2\ln\left(10\right)\right) \end{array}

Vorgehen: Produktregel


11)
\begin{array}{lclcrcl}\quad\mathbb{D}&=&\left\{x\in\mathbb{R}\vert x\neq\pm\sqrt{\frac{\pi}{2}+k\pi} , k\in\mathbb{Z}\right\} \cr \cr f(x)&=&\dfrac{\sin(x)}{\cos(x^2)} \cr\cr \quad u(x) &=& \sin(x) & \Rightarrow & u'(x) &=& \cos(x) \cr \quad v(h(x)) &=& \cos(h(x)) & \Rightarrow & v'(x) &=& -\sin(h(x)) \cr \quad h(x) &=& x^2 & \Rightarrow & h'(x) &=& 2x \cr \cr f'(x) &=& \dfrac{\cos(x) \cdot \cos(x^2) - \sin(x) \cdot \left( -\sin(x^2) \right) \cdot 2x}{ \left( \cos(x^2) \right)^2 } \cr \cr &=&\dfrac{\cos(x) \cdot \cos(x^2)+2x \cdot \sin(x) \cdot \sin(x^2)}{\cos^2(x^2)}\end{array}

Vorgehen: Quotientenregel, Kettenregel im Nenner


12)
\begin{array}{lclcrcl}\quad\mathbb{D}&=&\mathbb{R}^+ \cr \cr f(x)&=&\ln\left(\dfrac{1}{x}\right) \cr\cr \quad g(h(x)) &=& \ln(h(x)) & \Rightarrow & g'(x) &=& \dfrac{1}{h(x)} \cr \quad h(x) &=& \dfrac{1}{x} = x^{-1} & \Rightarrow & h'(x) &=& -1x^{-2} = -\dfrac{1}{x^2} \cr\cr f'(x) &=& \genfrac{}{}{1pt}{0}{1}{\dfrac{1}{x}} \cdot \left(-\dfrac{1}{x^2} \right) \cr\cr &=& -x \cdot \dfrac{1}{x^2} \cr\cr &=&-\dfrac{1}{x}\end{array}

Vorgehen: Kettenregel


13)
\begin{array}{rcl}\quad\mathbb{D}&=&\mathbb{R} \cr \cr f(x)&=&\tan(x)-\cos(x) \cr\cr f'(x)&=&\dfrac{1}{\cos^2(x)}+\sin(x)\end{array}

Vorgehen: Hier wird keine der komplexeren Ableitungsregeln benötigt.


14)
\begin{array}{lclcrcl}\quad\mathbb{D}&=&\mathbb{R} \cr \cr f(x) &=& \lg(x^2+2) \cr\cr \quad g(h(x)) &=& \lg(h(x)) & \Rightarrow & g'(h(x)) &=& \dfrac{1}{h(x) \cdot \ln(10)} \cr \quad h(x) &=& x^2+2 & \Rightarrow & h'(x) &=& 2x \cr\cr f'(x) &=& \dfrac{1}{\left(x^2+2 \right) \cdot \ln(10)} \cdot 2x \cr\cr &=& \dfrac{2x}{\ln(10) \cdot (x^2+2)} \end{array}

Vorgehen: Kettenregel

Bemerkung:  \lg ist der dekadische Logarithmus, d. h. seine Basis ist 10.


15)
\begin{array}{rcl}\quad\mathbb{D}&=&\mathbb{R} \cr \cr f(x) &=& \dfrac{3x^2-2x+1}{2} \cr\cr &=& \dfrac{3}{2}x^2-x+\dfrac{1}{2} \cr\cr\cr f'(x) &=& 3x-1\end{array}

Vorgehen: Hier wird keine der komplexeren Ableitungsregeln benötigt.

Bemerkung: Bei dieser Funktion wird nicht die Quotientenregeln benötigt, auch wenn dies vielleicht auf den ersten Blick so aussieht. Da aber im Nenner keine Variable, sondern nur eine Zahl enthalten ist, helfen die "normalen" Regeln der Bruchrechnung weiter.


16)
\begin{array}{lclcrcl}\quad\mathbb{D}&=&\mathbb{R} \cr \cr f(x) &=& e^{-\frac{x^2}{2}} \cr\cr \quad g(h(x)) &=& e^{h(x)} & \Rightarrow & g'(h(x)) &=& e^{h(x)} \cr \quad h(x) &=& -\dfrac{x^2}{2} & \Rightarrow & h'(x) &=& -x \cr\cr f'(x) &=& e^{-\frac{x^2}{2}} \cdot \left( -x \right) \cr &=& -x \cdot e^{-\frac{x^2}{2}} \end{array}

Vorgehen: Kettenregel


17)
\begin{array}{lclcrcl}\quad\mathbb{D}&=&\mathbb{R} \cr \cr f(x) &=& x \cdot e^{-5x^2} \cr\cr \quad u(x) &=& x & \Rightarrow & u'(x) &=& 1 \cr \quad v(h(x)) &=& e^{h(x)} & \Rightarrow & v'(h(x)) &=& e^{h(x)} \cr \quad h(x) &=& -5x^2 & \Rightarrow & h'(x) &=& -10x \cr\cr f'(x) &=& 1 \cdot e^{-5x^2} + x\cdot e^{-5x^2} \cdot \left( -10x \right) \cr &=& e^{-5x^2} - 10x^2 \cdot e^{-5x^2} \cr &=& e^{-5x^2} \cdot \left(1 - 10x^2\right) \end{array}

Vorgehen: Produktregel, Kettenregel im 2. Faktor


18)
\begin{array}{lclcrcl}\quad\mathbb{D}&=&\{x\in\mathbb{R}\vert x\neq\pm 1\} \cr \cr f(x) &=& \dfrac{2}{2x^2-2} \cr\cr &=& 2(2x^2-2)^{-1} \cr\cr \quad g(h(x)) &=& 2(h(x))^{-1} & \Rightarrow & g'(h(x)) &=& -2(h(x))^{-2} = \dfrac{-2}{h(x)^2} \cr \quad h(x) &=& 2x^2-2 & \Rightarrow & h'(x) &=& 4x \cr\cr f'(x) &=& \dfrac{-2}{\left( 2x^2-2 \right)^2} \cdot 4x \cr\cr &=& \dfrac{-8x}{4x^4-8x^2+4} \cr\cr &=& \dfrac{-2x}{x^4-2x^2+1} \cr\cr &=& -\dfrac{2x}{(x^2-1)^2} \end{array}

Vorgehen: Kettenregel

Bemerkung: Man könnte diese Funktion natürlich auch mit der Quotientenregel ableiten.


19)
\begin{array}{lclcrcl}\quad\mathbb{D}&=&\mathbb{R} \cr \cr f(x) &=& (3x^3+3)^{27} \cr\cr \quad g(h(x)) &=& (h(x))^{27} & \Rightarrow & g'(h(x)) &=& 27(h(x))^{26} \cr \quad h(x) &=& 3x^3+3 & \Rightarrow & h'(x) &=& 9x^2 \cr\cr f'(x) &=& 27\left( 3x^3+3 \right)^{26}\cdot9x^2 \cr &=& 243x^2 \cdot (3x^3+3)^{26} \end{array}

Vorgehen: Kettenregel


20)
\begin{array}{lclcrcl}\quad\mathbb{D}&=&\{x,a,b,c\in\mathbb{R}\vert ax^2+bx+c>0\} \cr \cr f(x)&=&\ln\left(ax^3+bx^2+c\right) \cr\cr \quad g(h(x)) &=& \ln(h(x)) & \Rightarrow & g'(h(x)) &=& \dfrac{1}{h(x)} \cr \quad g(x) &=& ax^3+bx^2+c & \Rightarrow & g'(x) &=& 3ax^2+2bx \cr \cr \cr f'(x)&=&\dfrac{1}{ax^3+bx^2+c}\cdot\left(3ax^2+2bx\right) \cr \cr &=&\dfrac{3ax^2+2bx}{ax^3+bx^2+c}\end{array}

Vorgehen: Kettenregel

 

7. Aufgabe


1)
1. Schritt: potenzielle x-Werte für die Extrempunkte berechnen
\begin{array}{rclll} \mathbb{D} &=& \mathbb{R} \cr\cr f'(x) &=& 4x-12 \cr \cr 0 &=& 4x-12 & \vert & -4x \cr -4x &=& -12 & \vert & :(-4) \cr x &=& 3\end{array}

2. Schritt: gefundene Stellen überprüfen & ggf. Art des Extrempunkts ermitteln
\begin{array}{rclll} f''(x) &=& 4 \cr \cr f''(3) &=& 4 & > & 0 \quad \Rightarrow \text{Tiefpunkt} \end{array}

3. Schritt: Funktionswert ausrechnen
\begin{array}{rclll} f(x) &=& 2x^2-12x+24 \cr\cr f(3) &=& 2\cdot 3^2-12\cdot 3+24 &=& 6 \end{array}

Ergebnis: Die Funktion hat bei \left(3\mid 6\right) einen Tiefpunkt.


2)
1. Schritt: potenzielle x-Werte für die Extrempunkte berechnen
\begin{array}{rclll} \mathbb{D} &=& \mathbb{R} \cr\cr f'(x) &=& -5x+10 \cr \cr 0 &=& -5x+10 & \vert & +5x \cr 5x &=& 10 & \vert & :5 \cr x &=& 2 \end{array}

2. Schritt: gefundene Stellen überprüfen & ggf. Art des Extrempunkts ermitteln
\begin{array}{rclll} f''(x) &=& -5 \cr\cr f''(2)&=& -5 & < & 0 \quad \Rightarrow \text{Hochpunkt} \end{array}

3. Schritt: Funktionswert berechnen
\begin{array}{rclll} f(x) &=& -\dfrac{5}{2}x^2+10x+\dfrac{3}{8} \cr\cr f(2) &=&-\dfrac{5}{2} \cdot 2^2+10\cdot 2+\dfrac{3}{8} &=& \dfrac{83}{8} \end{array}

Ergebnis: Die Funktion hat bei \left(2\mid \dfrac{83}{8}\right) einen Hochpunkt.


3)
1. Schritt: potenzielle x-Werte für die Extrempunkte berechnen
\begin{array}{rclll} \mathbb{D} &=& \mathbb{R} \cr\cr f'(x) &=& 160x-16 \cr \cr 0 &=& 160x-16 & \vert & -160x \cr -160x &=& -16 & \vert & :(-160) \cr x &=& \dfrac{1}{10} \end{array}

2. Schritt: gefundene Stellen überprüfen & ggf. Art des Extrempunkts ermitteln
\begin{array}{rclll} f''(x) &=& 160 \cr\cr f''\left(\dfrac{1}{10}\right)&=& 160 & > & 0\quad \Rightarrow \text{Tiefpunkt} \end{array}

3. Schritt: Funktionswert berechnen
\begin{array}{rclll} f(x) &=& 80x^2-16x+23 \cr\cr f \left(\dfrac{1}{10}\right) &=& 80\cdot \left(\dfrac{1}{10}\right)^2-16\cdot \dfrac{1}{10}+23 &=& \dfrac{111}{5} \end{array}

Ergebnis: Die Funktion hat bei \left(\dfrac{1}{10}\mid \dfrac{111}{5}\right) einen Hochpunkt.


4)
1. Schritt: potenzielle x-Werte für die Extrempunkte berechnen
\begin{array}{rclll} \mathbb{D} &=& \mathbb{R} \cr\cr f'(x) &=& x^2+4x-12 \cr \cr 0 &=& x^2+4x-12 \cr x_{1,2} &=& -\dfrac{4}{2} \pm \sqrt{\left(-\dfrac{4}{2}\right)^2-(-12)} \cr x_{1,2} &=& -2 \pm \sqrt{16} \cr\cr x_1 &=& -2+4 \quad = \quad 2 \cr x_2 &=& -2-4 \quad = \quad -6\end{array}

2. Schritt: gefundene Stellen überprüfen & ggf. Art des Extrempunkts ermitteln
\begin{array}{rclll} f''(x) &=& 2x+4 \cr\cr f''(2) &=& 2 \cdot 2+4 \quad = \quad 8 & > & 0\quad \Rightarrow \text{Tiefpunkt} \cr f''(-6) &=& 2 \cdot (-6)+4 \quad = \quad -8 & < & 0\quad \Rightarrow \text{Hochpunkt}\end{array}

3. Schritt: Funktionswerte berechnen
\begin{array}{rclll}f(x) &=& \dfrac{1}{3}x^3+2x^2-12x+6 \cr\cr f \left(2\right) &=&\dfrac{1}{3} \cdot 2^3+2\cdot 2^2-12 \cdot 2+6 &=& -\dfrac{22}{3} \cr f \left(-6\right) &=&\dfrac{1}{3} \cdot (-6)^3+2\cdot (-6)^2-12 \cdot (-6)+6 &=& 78 \end{array}

Ergebnis: Die Funktion hat bei \left(2\mid -\dfrac{22}{3}\right) einen Tiefpunkt und bei \left(-6\mid 78\right) einen Hochpunkt.

Bemerkung: Wer noch mal nachschauen möchte, wie das mit dem Lösen quadratischer Gleichungen war, findet hier Informationen.


5)
1. Schritt: potenzielle x-Werte für die Extrempunkte berechnen
\begin{array}{crclll} & \mathbb{D} &=& \mathbb{R} \cr\cr & f'(x) &=& 2x^3-6x \cr & 0 &=& 2x^3-6x \cr & 0 &=& x\left(2x^2-6\right) &\vert & \text{Satz vom Nullprodukt} \cr \text{Faktor 1:} & x_1 &=& 0 \cr\cr \text{Faktor 2:} & 0 &=& 2x^2-6 &\vert & -2x^2 \cr & -2x^2 &=& -6 & \vert & :(-2) \cr & x^2 &=& 3 \cr & x_2 &=& \sqrt{3} \cr & x_3 &=& -\sqrt{3} \end{array}


2. Schritt: gefundene Stellen überprüfen & ggf. Art des Extrempunkts ermitteln
\begin{array}{rclll} f''(x) &=& 6x^2-6 \cr\cr f''\left(-\sqrt{3}\right) &=&6 \cdot\left(-\sqrt{3}\right)^2-6 \quad = \quad 12 & > & 0\quad \Rightarrow \text{Tiefpunkt} \cr f''(0) &=& 6 \cdot 0^2-6 \quad = \quad -6 & < & 0\quad \Rightarrow \text{Hochpunkt} \cr f''\left(\sqrt{3}\right) &=&6 \cdot \sqrt{3}^2-6 \quad = \quad 12 & > & 0\quad \Rightarrow \text{Tiefpunkt} \end{array}

3. Schritt: Funktionswerte berechnen
\begin{array}{rclll}f(x) &=& \dfrac{1}{2}x^4-3x^2+1 \cr\cr f\left(-\sqrt{3}\right) &=&\dfrac{1}{2}\cdot\left(-\sqrt{3}\right)^4-3\cdot \left(-\sqrt{3}\right)^2+1 &=& -\dfrac{7}{2} \cr\cr f\left(0\right) &=&\dfrac{1}{2}\cdot0^4-3\cdot 0^2+1 &=& 1 \cr\cr f\left(\sqrt{3}\right) &=&\dfrac{1}{2}\cdot\sqrt{3}^4-3\cdot\sqrt{3}^2+1 &=& -\dfrac{7}{2} \end{array}

Ergebnis: Die Funktion hat bei \left(-\sqrt{3}\mid -\dfrac{7}{2}\right) einen Tiefpunkt, bei \left(0\mid 1\right) einen Hochpunkt und \left(\sqrt{3}\mid -\dfrac{7}{2}\right) einen Tiefpunkt.


6)
Vorab: Ableitungen berechnen
\begin{array}{rclll} \mathbb{D} &=& \mathbb{R}\setminus_{\left\{1\right\}} \cr\cr f'(t) &=& \dfrac{\left(4t^3-9t^2+5\right)(t-1)-\left(t^4-3t^3+5t-5\right)\cdot 1}{(t-1)^2} \cr\cr &=& \dfrac{3t^4-10t^3+9t^2}{(t-1)^2} \cr\cr\cr f''(t) &=& \dfrac{\left(12t^3-30t^2+18t\right)(t-1)^2-\left(3t^4-10t^3+9t^2\right)\cdot 2(t-1) \cdot 1}{(t-1)^4} \cr\cr &=& \dfrac{\left(12t^3-30t^2+18t\right)(t-1)-\left(3t^4-10t^3+9t^2\right)\cdot 2}{(t-1)^3} \cr\cr &=& \dfrac{6t^4-22t^3+30t^2-18t}{(t-1)^3} \end{array}

Bemerkung: Bei der 2. Ableitung ist darauf zu achten, dass im Rahmen der Anwendung der Quotientenregel für die Ableitung des Nennerterms im Zähler die Kettenregel zu nutzen ist. Das erklärt die \cdot 1 am Ende des Zählers ...

1. Schritt: potenzielle x-Werte für Extrempunkte berechnen
\begin{array}{rclll} f'(t) &=& \dfrac{3t^4-10t^3+9t^2}{(t-1)^2} \cr \cr 0 &=& \dfrac{t^2\left(3t^2-10t+9\right)}{(t-1)^2} \cr 0 &=& t^2\left(3t^2-10t+9\right) \cr t &=& 0 \cr 3t^2-10t+9 &=& 0 &\vert & :3 \cr t^2-\dfrac{10}{3}t+3 &=& 0 &\vert & \text{p-q-Formel} \cr t_{1,2} &=& \dfrac{5}{3}\pm\sqrt{\left(\dfrac{5}{3}\right)^2-3} \cr t_{1,2} &=& \dfrac{5}{3}\pm\sqrt{-\dfrac{2}{9}}\end{array}

Bemerkung: Ein Bruch kann nur dann den Wert 0 annehmen, wenn sein Zähler 0 ist. Insofern müssen wir uns nach dem Nullsetzen nur noch um den Zähler kümmern. 

2. Schritt: gefundene Stellen überprüfen & ggf. Art des Extrempunktes ermitteln
\begin{array}{rclll} f''(t) &=& \dfrac{6t^4-22t^3+30t^2-18t}{(t-1)^3} \cr\cr f''(0) &=& \dfrac{6 \cdot 0^4-22 \cdot 0^3+30 \cdot 0^2-18\cdot 0}{(0-1)^3} \quad = \quad 0\end{array}

Ergebnis: Mit den Methoden, die in diesem Lernmodul gezeigt wurden, können wir (leider) keine Aussage treffen, was bei dem gefundenen x-Wert mit der Funktion los ist. Mithilfe anderer Verfahren findet man heraus, dass diese Funktion bei x=0 einen Sattelpunkt hat. Diese Methoden werden Sie - wenn nötig - im Studium kennenlernen.

Bemerkung: Die Divisionen durch (t-1)^2, (t-1)^3 und (t-1)^4 sind hier ohne Einschränkungen möglich, weil t=1 \not\in \mathbb{D}. Eine Division durch 0 kann also nicht passieren.


7)
Vorab: Ableitungen berechnen
\begin{array}{rclll} \mathbb{D} &=& \left]\dfrac{8-\sqrt{70}}{6};\dfrac{8+\sqrt{70}}{6}\right[ \cr\cr f(x) &=& \left(-6x^2+16x+1\right)^{\frac{1}{2}} \cr\cr\cr f'(x) &=& \frac{1}{2}\left(-6x^2+16x+1\right)^{-\frac{1}{2}}\cdot (-12x+16) \cr\cr &=& \dfrac{-6x+8}{\sqrt{-6x^2+16x+1}} \cr\cr &=& (-6x+8)\left(-6x^2+16x+1\right)^{-\frac{1}{2}} \cr\cr\cr f''(x) &=& -6\cdot\left(-6x^2+16x+1\right)^{-\frac{1}{2}} + (-6x+8)\cdot\left(-\frac{1}{2}\right)\left(-6x^2+16x+1\right)^{-\frac{3}{2}}\cdot (-12x+16) \cr\cr &=& \dfrac{-6}{\left(-6x^2+16x+1\right)^{\frac{1}{2}}} + \dfrac{(-6x+8)\left(-\frac{1}{2}\right)(-12x+16)}{\left(-6x^2+16x+1\right)^{\frac{3}{2}}} \cr\cr &=& \dfrac{-6\left(-6x^2+16x+1\right)}{\left(-6x^2+16x+1\right)^{\frac{1}{2}}\left(-6x^2+16x+1\right)} + \dfrac{(-6x+8)\left(-\frac{1}{2}\right)(-12x+16)}{\left(-6x^2+16x+1\right)^{\frac{3}{2}}} \cr\cr &=& \dfrac{-70}{\left(-6x^2+16x+1\right)^{\frac{3}{2}}} \end{array}

Bemerkung: Bei der 1. Ableitung die Kettenregel nicht vergessen ... Unter der Wurzel steht ja eine Polynomfunktion. Bei der 2. Ableitung werden die Produkt- und die Kettenregel benötigt. Es ist beim Ableiten häufig einfacher, Wurzeln, die im Nenner eines Bruches stehen, in die Potenzschreibweise umzuformen, anstatt die Quotientenregel zu verwenden.

1. Schritt: potenzielle x-Werte für Extrempunkte berechnen
\begin{array}{rclll} f'(x) &=& \dfrac{-6x+8}{\sqrt{-6x^2+16x+1}} \cr \cr 0 &=& \dfrac{-6x+8}{\sqrt{-6x^2+16x+1}} \cr 0 &=& -6x+8 &\vert & +6x \cr 6x &=& 8 &\vert & :6 \cr \cr x &=& \dfrac{4}{3}\end{array}

Bemerkung: Ein Bruch kann nur dann den Wert 0 annehmen, wenn sein Zähler 0 ist. Insofern müssen wir uns nach dem Nullsetzen nur noch um den Zähler kümmern.

2. Schritt: gefundene Stellen überprüfen & ggf. Art des Extrempunktes ermitteln
\begin{array}{rclll} f''(x) &=& \dfrac{-70}{(-6x^2+16x+1)^{\frac{3}{2}}} \cr\cr f''\left(\dfrac{4}{3}\right) &=& \dfrac{-70}{\left(-6 \cdot \left(\dfrac{4}{3}\right)^2+16 \cdot \dfrac{4}{3}+1\right)^{\frac{3}{2}}} \quad \approx \quad -1{,}76 \quad < \quad 0 & & \Rightarrow \text{Hochpunkt}\end{array}

3. Schritt: Funktionswerte berechnen
\begin{array}{rclll} f(x) &=& \sqrt{-6x^2+16x+1} \cr \cr f\left(\dfrac{4}{3}\right) &=& \sqrt{-6 \cdot \left(\dfrac{4}{3}\right)^2+16 \cdot \dfrac{4}{3}+1} &=& \dfrac{\sqrt{105}}{3} \end{array}

Ergebnis: Die Funktion hat bei \left(\dfrac{4}{3}\mid \dfrac{\sqrt{105}}{3} \right) einen Hochpunkt.

Bemerkung: Die Divisionen durch \left(-6x^2+16x+1\right), \sqrt{-6x^2+16x+1} und \left(-6x^2+16x+1\right)^{\frac32} sind hier ohne Einschränkungen möglich, weil x=\dfrac{8-\sqrt{70}}{6} \not\in \mathbb{D} und x=\dfrac{8+\sqrt{70}}{6} \not\in\mathbb{D}. Eine Division durch 0 kann also nicht passieren.


8)
Vorab: Ableitungen berechnen
\begin{array}{rclll} \mathbb{D} &=& \mathbb{R} \cr\cr f'(x) &=& e^{0{,}1x^2-8x-6{,}3}\cdot \left(0{,}2x-8\right) \cr\cr &=& \left(\dfrac{x}{5}-8\right)e^{0{,}1x^2-8x-6{,}3} \cr\cr\cr f''(x) &=& \dfrac{1}{5}e^{0{,}1x^2-8x-6{,}3} +\left(\dfrac{x}{5}-8\right)e^{0{,}1x^2-8x-6{,}3}\cdot\left(\dfrac{x}{5}-8\right) \cr\cr &=& \left(\dfrac{1}{5}+\left(\dfrac{x}{5}-8\right)\left(\dfrac{x}{5}-8\right)\right)e^{0{,}1x^2-8x-6{,}3} \cr\cr &=& \dfrac{\left(x^2-80x+1.605\right)}{25}e^{0{,}1x^2-8x-6{,}3} \end{array}

Bemerkung: Bei der 1. Ableitung die Kettenregel nicht vergessen ... Im Exponenten steht ja eine Polynomfunktion. Bei der 2. Ableitung werden die Produkt- und die Kettenregel benötigt.

1. Schritt: potenzielle x-Werte für Extrempunkte berechnen
\begin{array}{rclll} f'(x) &=& \left(\dfrac{x}{5}-8\right)e^{0{,}1x^2-8x-6{,}3} \cr \cr 0 &=& \left(\dfrac{x}{5}-8\right)e^{0{,}1x^2-8x-6{,}3} &\vert & :e^{0{,}1x^2-8x-6{,}3} \cr \cr 0 &=& \dfrac{x}{5}-8 &\vert & -\dfrac{x}{5} \cr -\dfrac{x}{5} &=& -8 &\vert & \cdot (-5) \cr x &=& 40 \end{array}

Bemerkung:
Da Exponentialfunktionen nie den Wert 0 annehmen, kann hier bedenkenlos durch diesen Teil geteilt werden. Das vereinfacht die Sache doch erheblich ...

2. Schritt: gefundene Stellen überprüfen & ggf. Art des Extrempunktes ermitteln
\begin{array}{rclll} f''(x) &=& \dfrac{\left(x^2-80x+1.605\right)}{25}e^{0{,}1x^2-8x-6{,}3} \cr\cr f''(40) &=& \dfrac{\left(40^2- 80 \cdot 40+1.605\right)}{25}e^{0{,}1 \cdot 40^2-8 \cdot 40-6{,}3} \quad = \quad \dfrac{1}{5} e^{-166{,}3} \quad > \quad 0 & & \Rightarrow \text{Tiefpunkt}\end{array}

3. Schritt: Funktionswerte berechnen
\begin{array}{rclll} f(x) &=& e^{0{,}1x^2-8x-6{,}3} \cr \cr f(40) &=& e^{0{,}1 \cdot 40^2-8 \cdot 40-6{,}3} &=& e^{-166{,}3} \end{array}

Ergebnis: Die Funktion hat bei \left(40 \mid e^{-166{,}3} \right) einen Tiefpunkt.


9)
Vorab: Ableitungen berechnen
\begin{array}{rclll} \mathbb{D} &=& \mathbb{R} \cr\cr f'(x) &=& \dfrac{1}{2 \cdot \ln(10)} \cdot 10^{-8x^4+9x^2} \cdot \left(-32x^3+18x\right)\cdot \ln(10) \cr\cr &=& \left(-16x^3+9x\right) \cdot 10^{-8x^4+9x^2} \cr\cr\cr f''(x) &=& \left(-48x^2+9\right) \cdot 10^{-8x^4+9x^2} + \left(-16x^3+9x\right) \cdot 10^{-8x^4+9x^2} \cdot \left(-32x^3+18x\right)\cdot \ln(10)\cr\cr &=& \left(\left(-48x^2+9\right) + \left(-16x^3+9x\right) \left(-32x^3+18x\right)\ln(10)\right)\cdot 10^{-8x^4+9x^2} \cr\cr &=& \left(\left(512x^6-576x^4+162x^2\right)\ln(10)-48x^2+9\right)\cdot 10^{-8x^4+9x^2} \end{array}

Bemerkung: Bei der 1. Ableitung die Kettenregel nicht vergessen ... Im Exponenten steht ja eine Polynomfunktion. Bei der 2. Ableitung werden die Produkt- und die Kettenregel benötigt.

1. Schritt: potenzielle x-Werte für Extrempunkte berechnen
\begin{array}{crclll} & f'(x) &=& \left(-16x^3+9x\right) \cdot 10^{10^{-8x^4+9x^2}} \cr \cr & 0 &=& -x\left(16x^2-9\right) \cdot 10^{10^{-8x^4+9x^2}} &\vert& :10^{10^{-8x^4+9x^2}} \cr & 0 &=& -x\left(16x^2-9\right) &\vert& \text{Satz vom Nullprodukt} \cr\text{Faktor 1:} & x_1 &=& 0 \cr\cr \text{Faktor 2:} & 0 &=& 16x^2-9 &\vert & +9 \cr & 9 &=& 16x^2 &\vert & :16 \cr & \dfrac{9}{16} &=& x^2 &\vert & \pm\sqrt{} \cr & x_2 &=& \dfrac{3}{4} \cr & x_3 &=& -\dfrac{3}{4} \end{array}

Bemerkung: Auch Exponentialfunktionen zur Basis 10 können nie den Wert 0 annehmen. Daher kann bedenkenlos durch diesen Teil geteilt werden, was die Sache wieder deutlich einfacher macht.

2. Schritt: gefundene Stellen überprüfen & ggf. Art des Extrempunktes ermitteln
\begin{array}{rclll} f''(x) &=& \left(\left(512x^6-576x^4+162x^2\right)\ln(10)-48x^2+9\right)\cdot 10^{-8x^4+9x^2} \cr\cr f''\left(-\dfrac{3}{4}\right) &=& \left(\left(512\cdot\left(-\dfrac{3}{4}\right)^6-576\cdot\left(-\dfrac{3}{4}\right)^4+162\cdot\left(-\dfrac{3}{4}\right)^2\right)\ln(10)-48\cdot\left(-\dfrac{3}{4}\right)^2+9\right)\cdot 10^{-8\cdot\left(-\frac{3}{4}\right)^4+9\cdot\left(-\frac{3}{4}\right)^2} \quad \approx \quad -6.116{,}77 \quad < \quad 0 & & \Rightarrow \text{Hochpunkt} \cr\cr f''(0) &=& \left(\left(512\cdot 0^6-576\cdot 0^4+162\cdot 0^2\right)\ln(10)-48\cdot 0^2+9\right)\cdot 10^{-8\cdot 0^4+9\cdot 0^2} \quad = \quad 9 \quad > \quad 0 & & \Rightarrow \text{Tiefpunkt} \cr \cr f''\left(\dfrac{3}{4}\right) &=& \left(\left(512\cdot\left(\dfrac{3}{4}\right)^6-576\cdot\left(\dfrac{3}{4}\right)^4+162\cdot\left(\dfrac{3}{4}\right)^2\right)\ln(10)-48\cdot\left(\dfrac{3}{4}\right)^2+9\right)\cdot 10^{-8\cdot\left(\frac{3}{4}\right)^4+9\cdot\left(\frac{3}{4}\right)^2} \quad \approx \quad -6.116{,}77 \quad < \quad 0 & & \Rightarrow \text{Hochpunkt} \end{array}

3. Schritt: Funktionswerte berechnen
\begin{array}{rclll} f(x) &=& \dfrac{1}{2 \cdot \ln(10)} \cdot 10^{-8x^4+9x^2} \cr \cr f\left(-\dfrac{3}{4}\right) &=& \dfrac{1}{2 \cdot \ln(10)} \cdot 10^{-8 \cdot \left(-\frac{4}{3}\right)^4+9 \cdot \left(-\frac{3}{4}\right)^2} &\approx& 73{,}79 \cr\cr f(0) &=& \dfrac{1}{2 \cdot \ln(10)} \cdot 10^{-8 \cdot 0^4+9 \cdot 0^2} &\approx& 0{,}22 \cr \cr f\left(\dfrac{3}{4}\right) &=& \dfrac{1}{2 \cdot \ln(10)} \cdot 10^{-8 \cdot \left(\frac{3}{4}\right)^4+9 \cdot \left(\frac{3}{4}\right)^2} &\approx& 73{,}79 \end{array}

Ergebnis: Die Funktion hat zwei Hochpunkte bei \left(\dfrac{3}{4}\mid 73{,}79 \right) und \left(-\dfrac{3}{4}\mid 73{,}79 \right) sowie einen Tiefpunkt bei \left(0 \mid 0{,}22 \right).


10)
Vorab: Ableitungen berechnen
\begin{array}{rclll} \mathbb{D} &=& \left]-2\sqrt{6};0\right[ \cup \left]2\sqrt{6};\infty\right[ \cr\cr f'(x) &=& \dfrac{1}{\frac{1}{2}x^3-12x}\cdot\left(\frac{3}{2}x^2-12\right) \cr\cr &=& \dfrac{3x^2-24}{x^3-24x} \cr\cr\cr f''(x) &=& \dfrac{6x\left(x^3-24x\right)-\left(3x^2-24\right)\left(3x^2-24\right)}{\left(x^3-24x\right)^2}\cr\cr &=& \dfrac{-3x^4-576}{\left(x^3-24x\right)^2} \end{array}

Bemerkung: Bei der 1. Ableitung die Kettenregel nicht vergessen ... Im Logarithmus steht ja eine Polynomfunktion. Bei der 2. Ableitung wird die Quotientenregel benötigt.

1. Schritt: potenzielle x-Werte für Extrempunkte berechnen
\begin{array}{rclll} f'(x) &=& \dfrac{3x^2-24}{x^3-24x} \cr \cr 0 &=& \dfrac{3x^2-24}{x^3-24x} \cr 0 &=& 3x^2-24 &\vert & -3x^2 \cr -3x^2 &=& -24 &\vert & :(-3) \cr x^2 &=& 8 &\vert & \pm\sqrt{} \cr \cr x_1 &=& \sqrt{8} & \not\in\mathbb{D} \cr x_2 &=& -\sqrt{8} \end{array}

Bemerkung: Ein Bruch kann nur dann den Wert 0 annehmen, wenn sein Zähler 0 ist. Insofern müssen wir uns nach dem Nullsetzen nur noch um den Zähler kümmern.

2. Schritt: gefundene Stellen überprüfen & ggf. Art des Extrempunktes ermitteln
\begin{array}{rclll} f''(x) &=& \dfrac{-3x^4-576}{\left(x^3-24x\right)^2} \cr\cr f''\left(-\sqrt{8}\right) &=& \dfrac{-3\cdot\left(-\sqrt{8}\right)^4-576}{\left(\left(-\sqrt{8}\right)^3-24\cdot\left(-\sqrt{8}\right)\right)^2} \quad = \quad -\dfrac{3}{8} \quad < \quad 0 & & \Rightarrow \text{Hochpunkt}\end{array}

3. Schritt: Funktionswerte berechnen
\begin{array}{rclll} f(x) &=& \ln\left(\dfrac{1}{2}x^3-12x\right) -15\cr \cr f\left(-\sqrt{8}\right) &=& \ln\left(\dfrac{1}{2} \cdot \left(-\sqrt{8}\right)^3-12 \cdot \left(-\sqrt{8}\right)\right) &\approx& -11{,}88 \end{array}

Ergebnis: Die Funktion hat bei \left(-\sqrt{8}\mid -11{,}88 \right) einen Hochpunkt.

Bemerkung: Die Divisionen durch \left(\dfrac12 x^3-12x\right)\left(x^3-24x\right) und \left(x^3-24x\right)^2 sind hier ohne Einschränkungen möglich, weil x=-2 \sqrt{6} \not\in \mathbb{D}, x=0 \not\in\mathbb{D} und x=2 \sqrt{6} \not\in \mathbb{D}. Eine Division durch 0 kann also nicht passieren.

 

8. Aufgabe

Bemerkung: Die folgenden Aufgaben wirken vielleicht nicht ganz passend für dieses Kapitel. Allerdings ist die Differenzialrechnung nie fern, wenn in der Aufgabe nach dem "größten", "kleinsten", "maximalen" oder "minimalem" Wert gefragt ist ... Sie liefert ja gerade das "Werkzeug", um solche Extremstellen zu berechnen.

1)
Seien a und b die beiden gesuchten Zahlen. Dann gilt:
a+b=100 \Rightarrow b=100-a

und

P(a,b)=a\cdot b

Setzt man die erste Gleichung in die zweite ein, erhält man:
\begin{array}{rcl} P(a) &=& a(100-a) \cr &=& 100a-a^2 \end{array}

Da das Maximum dieser Funktion gesucht ist, leitet man P(a) ab. Der entstehende Term wird nullgesetzt:
\begin{array}{rclcl} P'(a) &=& 100-2a \cr\cr 0 &=& 100-2a &\vert& +2a \cr 2a &=& 100 &\vert& :2 \cr a &=& 50 \cr \cr b &=& 100-50 = 50 \end{array}

Nun müssen wir noch überprüfen, ob a=b=50 tatsächlich zum größten Produkt aller Zahlenpaare, deren Summe 100 ist, führt. Aus unserer Rechnung können wir bislang nur ablesen, dass a=b=50 entweder zum größten oder zum kleinsten Produkt führt (die erste Ableitung liefert ja keine Informationen über die Art des Extrempunkts ...). Dafür kann man einfach einige dieser Produkte ausrechnen:

50+50=100 und 50\cdot 50 = 2.500
55+45=100 und 55\cdot 45 = 2.475
1+99=100 und 1\cdot 99 = 99


Ergebnis: Von allen Zahlenpaaren, deren Summe 100 ist, hat a=b=50 das größte Produkt.


2)
Seien a und b die Seitenlängen des Rechtecks. Dann gilt:
\begin{array}{rclcl} U &=& 2a+2b \cr 36 &=& 2a+2b &\vert& -2b \cr 36-2b &=& 2a &\vert& :2 \cr 18-b &=& a \end{array}

und

A(a,b) = a\cdot b

Setzt man die erste Gleichung in die zweite ein, erhält man:
\begin{array}{rcl} A(b) &=& (18-b)b \cr &=& 18b-b^2 \end{array}

Da das Maximum dieser Funktion gesucht ist, leitet man A(b) ab. Der entstehende Term wird nullgesetzt:
\begin{array}{rclcl} A'(b) &=& 18-2b \cr\cr 0 &=& 18-2b &\vert& +2b \cr 2b &=& 18 &\vert& \cdot \dfrac{1}{2} \cr b &=& 9 \cr \cr a &=& 18-9 = 9 \end{array}

Wieder müssen wir nun überprüfen, ob a=b=9 tatsächlich zu einem Rechteck des Umfangs 36 mit maximalem Flächeninhalt führt.

2\cdot 9 + 2\cdot 9=36 und 9\cdot 9=81
2\cdot 12 + 2\cdot 6=36 und 12\cdot 6=72
2\cdot 3 + 2\cdot 15=36 und 3\cdot 15=45

 
Ergebnis: Von allen Paaren von Rechtecksseiten, die zusammen 36 lang sind, hat a=b=9 den größten Flächeninhalt. Es handelt sich also um ein Quadrat.


3)
Parallelogramm mit Beschriftungen

Für den Flächeninhalt eines Parallelogramms gilt: A = ab \sin\left(\alpha\right).
Da die Seitenlängen a und b vorgegeben sind (auch wenn wir sie nicht kennen), hängt der Flächeninhalt in dieser Aufgabe also nur vom Winkel \alpha ab. Wir erhalten die Funktion:
A\left(\alpha\right) = ab \sin\left(\alpha\right)

Da das Maximum dieser Funktion gesucht ist, leitet man A\left(\alpha\right) ab. Der entstehende Term wird nullgesetzt:
\begin{array}{rclcl} A'\left(\alpha\right) &=&ab \cos\left(\alpha\right) \cr\cr 0 &=&ab \cos\left(\alpha\right) &\vert& :ab \cr 0 &=&\cos\left(\alpha\right) \end{array}
Bitte beachten Sie beim Ableiten, dass a und b positive Konstanten sind. Wären sie negativ oder 0, ergäbe sich ja kein Parallelogramm ... Daher dürfen wir hier auch ohne Einschränkungen durch ab teilen.

Gesucht sind also die Werte für \alpha, für die \cos\left(\alpha\right) den Wert 0 annimmt. Davon gibt es - wie im Kapitel Trigonometrie besprochen - eine ganze Menge ... Nämlich ..., -\dfrac{3\pi}{2}, -\dfrac{\pi}{2}, \dfrac{\pi}{2}, \dfrac{3\pi}{2}, ...
Aber nicht alle dieser Winkel kommen hier als Lösung infrage: In der Geometrie sind Winkel größer oder gleich 0; alle negativen Werte für \alpha scheiden also aus. Wäre auf der anderen Seite \alpha\geq\pi, würde sich kein Parallelogramm mehr ergeben, weil alle vier Winkel zusammen dann mehr als 2\pi ergeben würden, was nach dem Winkelsummensatz für Vierecke nicht geht. Es bleibt einzig der Wert \dfrac{\pi}{2}.

Es gibt noch einen zweiten (kürzeren) Lösungsweg: A\left(\alpha\right) = ab \sin\left(\alpha\right) ist offensichtlich dann am größten, wenn \sin\left(\alpha\right) am größten ist (a und b sind ja fix). Der größte Wert, den der Sinus annehmen kann, ist 1. Auch das passiert bei vielen Werten, nämlich ..., -\dfrac{3\pi}{2}, -\dfrac{\pi}{2}, \dfrac{\pi}{2}, \dfrac{3\pi}{2}, ...
Mithilfe der gleichen Überlegungen wie oben kommt man dazu, dass nur \dfrac{\pi}{2} für diese Aufgaben zu einer Lösung führt. Damit haben wir auch gleich die Bestätigung dafür, dass \dfrac{\pi}{2} zu einem Maximum der Funktion A\left(\alpha\right) = ab \sin\left(\alpha\right) führt.

Ergebnis: Der Flächeninhalt des Parallelogramms ist dann am größten, wenn der Winkel \alpha=\dfrac{\pi}{2} ist. Das heißt, wenn das Parallelogramm zu einem Rechteck wird.

Übersicht:

 

22.5 Partielle Ableitungen - Lösungen

1. Aufgabe

1)
\mathbb{D}=\mathbb{R}\times\mathbb{R}

Partielle Ableitungen 1. Ordnung 

 

Partielle Ableitungen 2. Ordnung 

\begin{array}{rcl}f'_x(x,y) &=& 4\cdot 2x^{2-1}+3y+0\\&=& 8x+3y\end{array}   \begin{array}{rcl}f'_{xx}(x,y) &=& 8+0 \\&=& 8\end{array}

    \begin{array}{rcl}f'_{xy}(x,y) &=& 0+3 \\&=& 3 \end{array}

\begin{array}{rcl}f'_y(x,y) &=& 0+3x+7\cdot 3y^{3-1}\\&=& 3x+21y^2\end{array}   \begin{array}{rcl}f'_{yx}(x,y) &=& 3+0 \\&=& 3\end{array}

    \begin{array}{rcl}f'_{yy}(x,y) &=& 0+21\cdot 2y^{2-1}\\&=& 42y\end{array}

 

2) 
\mathbb{D}=\mathbb{R}\times\mathbb{R}

Partielle Ableitungen 1. Ordnung 

 

Partielle Ableitungen 2. Ordnung 

\begin{array}{rcl}f'_a(a,b) &=& 3a^{3-1}+3b^2-15-0\\&=& 3a^2+3b^2-15\end{array}   \begin{array}{rcl}f'_{aa}(a,b) &=& 3\cdot 2a^{2-a}+0-0\\&=& 6a\end{array}

    \begin{array}{rcl}f'_{ab}(a,b) &=& 0+3\cdot 2b^{2-1}-0 \\&=& 6b\end{array}

\begin{array}{rcl}f'_b(a,b) &=& 0+3\cdot 2ab^{2-1}-0-12\\&=& 6ab-12\end{array}   \begin{array}{rcl}f'_{ba}(a,b) &=& 6b-0 \\&=& 6b\end{array}

    \begin{array}{rcl}f'_{bb}(a,b) &=& 6a-0\\&=& 6a\end{array}

 

3)
\begin{array}{rcl}\mathbb{D} &=& \mathbb{R}\times\mathbb{R} \\\\f(x,y) &=& \left(x-2y\right)\left(2x+y\right) \\&=& 2x^2-3xy-2y^2\end{array}

Partielle Ableitungen 1. Ordnung 

 

Partielle Ableitungen 2. Ordnung 

\begin{array}{rcl}f'_x(x,y) &=& 2\cdot2x^{2-1}-3y\\&=& 4x-3y\end{array}   \begin{array}{rcl}f'_{xx}(x,y) &=& 4\end{array}

    \begin{array}{rcl}f'_{xy}(x,y) &=& -3\end{array}

\begin{array}{rcl}f'_y(x,y) &=& -3x-2\cdot 2y^{2-1} \\&=& -3x-4y\end{array}   \begin{array}{rcl}f'_{yx}(x,y) &=& -3\end{array}

 

\begin{array}{rcl}f'_{yy}(x,y) &=& -4\end{array}

 

4)
\begin{array}{rcl}\mathbb{D} &=& \mathbb{R}\times\mathbb{R} \\\\f(x,y) &=& \left(x+3y\right)^2-2xy \\&=& x^2+6xy+9y^2-2xy \\&=& x^2+4xy+9y^2\end{array}

Partielle Ableitungen 1. Ordnung 

 

Partielle Ableitungen 2. Ordnung 

\begin{array}{rcl}f'_x(x,y) &=& 2x^{2-1}+4y+0\\&=& 2x+4y\end{array}   \begin{array}{rcl}f'_{xx}(x,y) &=& 2+0\\&=& 2\end{array}

    \begin{array}{rcl}f'_{xy}(x,y) &=& 0+4 \\&=& 4\end{array}

\begin{array}{rcl}f'_y(x,y) &=& 0+4x+9\cdot 2y^{2-1} \\&=& 4x+18y\end{array}   \begin{array}{rcl}f'_{yx}(x,y) &=& 4+0 \\&=& 4\end{array}

    \begin{array}{rcl}f'_{yy}(x,y) &=& 0+18\\&=& 18\end{array}

 

5)
\mathbb{D}=\mathbb{R}\times\mathbb{R}

Partielle Ableitungen 1. Ordnung 

 

Partielle Ableitungen 2. Ordnung 

\begin{array}{rcl}f'_x(x,y) &=& 4x^{3-1}-0+14y+0 \\&=& 4x^3+14y\end{array}   \begin{array}{rcl}f'_{xx}(x,y) &=& 4\cdot 3x^{3-1}+0 \\&=& 12x^2\end{array}

    \begin{array}{rcl}f'_{xy}(x,y) &=& 0+14 \\&=& 14\end{array}

\begin{array}{rcl}f'_y(x,y) &=& 0-53\cdot 3y^{3-1}+14x+0 \\&=& 14x-159y^2\end{array}   \begin{array}{rcl}f'_{yx}(x,y) &=& 14+0 \\&=& 14\end{array}

    \begin{array}{rcl}f'_{yy}(x,y) &=& 0-159\cdot 2y^{2-1} \\&=& -318y\end{array}

 

6)
\mathbb{D}=\mathbb{R}\times\mathbb{R}

Partielle Ableitungen 1. Ordnung 

 

Partielle Ableitungen 2. Ordnung 

\begin{array}{rcl}f'_x(x,y) &=& 3\cdot 5x^{5-1}-20\cdot 2x^{2-1}y-0 \\&=& 15x^4-40xy\end{array}   \begin{array}{rcl}f'_{xx}(x,y) &=& 15\cdot 4x^{4-1}-40y \\&=& 60x^3-40y\end{array}

    \begin{array}{rcl}f'_{xy}(x,y) &=& 0-20\cdot 2x^{2-1} \\&=& -40x\end{array}

\begin{array}{rcl}f'_y(x,y) &=& 0-20x^2-10\cdot 2y^{2-1} \\&=& -20x^2-20y\end{array}   \begin{array}{rcl}f'_{yx}(x,y) &=& -20\cdot 2x^{2-1}-0 \\&=& -40x\end{array}

    \begin{array}{rcl}f'_{yy}(x,y) &=& 0-20 \\&=& -20\end{array}

 

7)
\mathbb{D}=\mathbb{R}\times\mathbb{R}

Partielle Ableitungen 1. Ordnung 

 

Partielle Ableitungen 2. Ordnung 

\begin{array}{rcl}g'_x(x,y) &=& 7x^{7-1}+\dfrac{5}{2}\cdot 2x^{2-1}y^3+0 \\&=& 7x^6+5xy^3\end{array}   \begin{array}{rcl}g'_{xx}(x,y) &=& 7\cdot 6x^{6-1}+5y^3 \\&=& 42x^5+5y^3\end{array}

    \begin{array}{rcl}g'_{xy}(x,y) &=& 0+5\cdot 3xy^{3-1} \\&=& 15xy^2\end{array}

\begin{array}{rcl}g'_y(x,y) &=& 0+\dfrac{5}{2}\cdot 3x^2y^{3-1}+13 \\&=& \dfrac{15}{2}x^2y^2+13\end{array}   \begin{array}{rcl}g'_{yx}(x,y) &=& \dfrac{15}{2}\cdot 2x^{2-1}y^2+0 \\&=& 15xy^2\end{array}

    \begin{array}{rcl}g'_{yy}(x,y) &=& \dfrac{15}{2}\cdot 2x^2y^{2-1}+0\\&=& 15x^2y\end{array}

 

8)
\begin{array}{rcl}\mathbb{D} &=& \mathbb{R}\times\mathbb{R} \\\\g(x,y) &=& \left(5x+22\right)^2+9x^2y+y^2 \\&=& 25x^2+220x+484+9x^2y+y^2\end{array}

Partielle Ableitungen 1. Ordnung 

 

Partielle Ableitungen 2. Ordnung 

\begin{array}{rcl}g'_x(x,y) &=& 25\cdot 2x^{2-1}+220+0+9\cdot 2x^{2-1}y+0 \\&=& 50x+220+18xy\end{array}  

\begin{array}{rcl}g'_{xx}(x,y) &=& 50+0+18y\\&=& 50+18y\end{array}

    \begin{array}{rcl}g'_{xy}(x,y) &=& 0+0+18x \\&=& 18x\end{array}

\begin{array}{rcl}g'_y(x,y) &=& 0+0+0+9x^2+2y^{2-1} \\&=& 9x^2+2y\end{array}   \begin{array}{rcl}g'_{yx}(x,y) &=& 9\cdot 2x^{2-1}+0 \\&=& 18x\end{array}

    \begin{array}{rcl}g'_{yy}(x,y) &=& 0+2 \\&=& 2\end{array}

 

9)
\mathbb{D}=\mathbb{R}\times\mathbb{R}

Partielle Ableitungen 1. Ordnung 

 

Partielle Ableitungen 2. Ordnung 

\begin{array}{rcl}g'_x(x,z) &=& 7\cdot 4x^{4-1}+8z^2-0 \\&=& 28x^3+8z^2\end{array}   \begin{array}{rcl}g'_{xx}(x,z) &=& 28\cdot 3x^{3-1}+0 \\&=& 84x^2\end{array}

    \begin{array}{rcl}g'_{xz}(x,z) &=& 0+8\cdot 2z^{2-1} \\&=& 16z\end{array}

\begin{array}{rcl}g'_z(x,z) &=& 0+8\cdot 2xz^{2-1}-12\cdot 3z^{3-1} \\&=& 16xz-36z^2\end{array}   \begin{array}{rcl}g'_{zx}(x,z) &=& 16z-0 \\&=& 16z\end{array}

    \begin{array}{rcl}g'_{zz}(x,z) &=& 16x-36\cdot 2z^{2-1} \\&=& 16x-72z\end{array}

 

10)
\mathbb{D}=\mathbb{R}\times\mathbb{R}

Partielle Ableitungen 1. Ordnung 

 

Partielle Ableitungen 2. Ordnung 

\begin{array}{rcl}g'_b(b,c) &=& 0-8\cdot 3b^{3-1}+24c+0 \\&=& -24b^2+24c\end{array}   \begin{array}{rcl}g'_{bb}(b,c) &=& -24\cdot 2b^{2-1}+0 \\&=& -48b\end{array}

    \begin{array}{rcl}g'_{bc}(b,c) &=& 0+24 \\&=& 24\end{array}

\begin{array}{rcl}g'_c(b,c) &=& 0-0+24b+3\cdot 8c^{8-1} \\&=& 24b+24c^7\end{array}   \begin{array}{rcl}g'_{cb}(b,c) &=& 24+0 \\&=& 24\end{array}

    \begin{array}{rcl}g'_{cc}(b,c) &=& 0+24\cdot 7c^{7-1} \\&=& 168c^6\end{array}

 

11)
\mathbb{D}=\mathbb{R}\times\mathbb{R}

Partielle Ableitungen 1. Ordnung 

 

Partielle Ableitungen 2. Ordnung 

\begin{array}{rcl}f'_b(b,c) &=& 3\cdot 2b^{2-1}-125c+0-5\cdot 2b^{2-1}c^2 \\&=& 6b-125c-10bc^2\end{array}   \begin{array}{rcl}f'_{bb}(b,c) &=& 6-0-10c^2 \\&=& 6-10c^2\end{array}

    \begin{array}{rcl}f'_{bc}(b,c) &=& 0-125-10\cdot 2bc^{2-1} \\&=& -125-20bc\end{array}

\begin{array}{rcl}f'_c(b,c) &=& 0-125b+7\cdot 2c^{2-1}-5\cdot 2b^2c^{2-1}\\&=& -125b+14c-10b^2c\end{array}   \begin{array}{rcl}f'_{cb}(b,c) &=& -125+0-10\cdot 2b^{2-1}c \\&=& -125-20bc\end{array}

    \begin{array}{rcl}f'_{cc}(b,c) &=& 0+14-10b^2 \\&=& 14-10b^2\end{array}

 

12)
\mathbb{D}=\mathbb{R}\times\mathbb{R}

Partielle Ableitungen 1. Ordnung 

 

Partielle Ableitungen 2. Ordnung 

\begin{array}{rcl}a'_x(x,y) &=& 5\cdot 3x^{3-1}y+10\cdot 2x^{2-1}y^2+2y^3+5y \\&=& 15x^2y+20xy^2+2y^3+5y\end{array}  

\begin{array}{rcl}a'_{xx}(x,y) &=& 15\cdot 2x^{2-1}y+20y^2+0+0 \\&=& 30xy+20y^2\end{array}

    \begin{array}{rcl}a'_{xy}(x,y) &=& 15x^2+20\cdot 2xy^{2-1}+2\cdot 3y^{3-1}+5 \\&=& 15x^2+40xy+6y^2+5\end{array}

\begin{array}{rcl}a'_y(x,y) &=& 5x^3+10\cdot 2x^2y^{2-1}+2\cdot 3xy^{3-1}+5x \\&=& 5x^3+20x^2y+6xy^2+5x\end{array}   \begin{array}{rcl}a'_{yx}(x,y) &=& 5\cdot 3x^{3-1}+20\cdot 2x^{2-1}y+6y^2+5 \\&=& 15x^2+40xy+6y^2+5\end{array}

    \begin{array}{rcl}a'_{yy}(x,y) &=& 0+20x^2+6\cdot 2xy^{2-1}+0 \\&=& 20x^2+12xy\end{array}

 

13)
\mathbb{D}=\mathbb{R}_0^+\times\mathbb{R}

Partielle Ableitungen 1. Ordnung 

 

Partielle Ableitungen 2. Ordnung 

\begin{array}{rcl}g'_x(x,y) &=& \dfrac{1}{2}x^{\frac{1}{2}-1}+5y+0 \\\\&=& \dfrac{1}{2}x^{-\frac{1}{2}}+5y \\\\&=& \dfrac{1}{2\sqrt{x}}+5y\end{array}   \begin{array}{rcl}g'_{xx}(x,y) &=& \dfrac{1}{2}\cdot \left(-\dfrac{1}{2}\right)x^{-\frac{1}{2}-1}+0 \\\\&=& -\dfrac{1}{4}x^{-\frac{3}{2}} \\\\&=& \dfrac{-1}{4\sqrt{x^3}}\end{array}

    \begin{array}{rcl}g'_{xy}(x,y) &=& 0+5 \\&=& 5\end{array}

\begin{array}{rcl}g'_y(x,y) &=& 0+5x+2y^{2-1} \\&=& 5x+2y\end{array}   \begin{array}{rcl}g'_{yx}(x,y) &=& 5+0 \\&=& 5\end{array}

    \begin{array}{rcl}g'_{yy}(x,y) &=& 0+2 \\&=& 2\end{array}

 

14)
\mathbb{D}=\mathbb{R}\times\mathbb{R}_0^+

Partielle Ableitungen 1. Ordnung 

 

Partielle Ableitungen 2. Ordnung 

\begin{array}{rcl}g'_x(x,y) &=& 2x^{2-1}y^{\frac{3}{2}}-0 \\\\&=& 2xy^{\frac{3}{2}} \\\\&=& 2x\sqrt{y^3}\end{array}   \begin{array}{rcl}g'_{xx}(x,y) &=& 2\sqrt{y^3}\end{array}

    \begin{array}{rcl}g'_{xy}(x,y) &=& 2\cdot\dfrac{3}{2}xy^{\frac{3}{2}-1} \\\\&=& 3xy^{\frac{1}{2}} \\\\&=& 3x\sqrt{y}\end{array}

\begin{array}{rcl}g'_y(x,y) &=& \dfrac{3}{2}x^2 y^{\frac{3}{2}-1}-0 \\\\&=& \dfrac{3}{2}x^2y^{\frac{1}{2}} \\\\&=& \dfrac{3}{2}x^2\sqrt{y}\end{array}   \begin{array}{rcl}g'_{yx}(x,y) &=& \dfrac{3}{2}\cdot 2x^{2-1}\sqrt{y} \\\\&=& 3x\sqrt{y}\end{array}

    \begin{array}{rcl}g'_{yy}(x,y) &=& \dfrac{3}{2}\cdot\dfrac{1}{2}x^2y^{\frac{1}{2}-1} \\\\&=& \dfrac{3}{4}x^2y^{-\frac{1}{2}} \\\\&=& \dfrac{3x^2}{4\sqrt{y}}\end{array}

 

15)
\mathbb{D}=\mathbb{R}\times\mathbb{R}

Partielle Ableitungen 1. Ordnung 

 

Partielle Ableitungen 2. Ordnung 

\begin{array}{rcl}g'_d(d,h) &=& \dfrac{\pi}{4}\cdot 2d^{2-1}h \\\\&=& \dfrac{\pi}{2}dh\end{array}   \begin{array}{rcl}g'_{dd}(d,h) &=& \dfrac{\pi}{2}h\end{array}

    \begin{array}{rcl}g'_{dh}(d,h) &=& \dfrac{\pi}{2}d\end{array}

\begin{array}{rcl}g'_h(d,h) &=& \dfrac{\pi}{4}d^2\end{array}   \begin{array}{rcl}g'_{hd}(d,h) &=& 2\cdot\dfrac{\pi}{4}d^{2-1} \\&=& \dfrac{\pi}{2}d\end{array}

    \begin{array}{rcl}g'_{hh}(d,h) &=& 0\end{array}

 

16)
\mathbb{D}=\mathbb{R}\times\mathbb{R}\times\mathbb{R}

Partielle Ableitungen 1. Ordnung 

 

Partielle Ableitungen 2. Ordnung 

\begin{array}{rcl}f'_x(x,y,z) &=& -12\cdot 2x^{2-1}-3\cdot 4x^{4-1}y+34y^5+0\\&=& -24x^{1}-12x^{3}y+34y^5\end{array}   \begin{array}{rcl}f'_{xx}(x,y,z) &=& -24-12\cdot 3x^{3-1}y+0\\&=& -24-36x^2y\end{array}

    \begin{array}{rcl}f'_{xy}(x,y,z) &=& 0-12x^3+34\cdot 5y^{5-1} \\&=& -12x^3+170y^4\end{array}

    \begin{array}{rcl}f'_{xz}(x,y,z) &=& 0 \\\end{array}

\begin{array}{rcl}f'_y(x,y,z) &=& 0-3x^4+34\cdot 5xy^{5-1}+0\\&=& -3x^4+170xy^4\end{array}   \begin{array}{rcl}f'_{yx}(x,y,z) &=& -3\cdot 4x^{4-1}+170y^4\\&=& -12x^3+170y^4\end{array}

    \begin{array}{rcl}f'_{yy}(x,y,z) &=& 680xy^3\\\end{array}

    \begin{array}{rcl}f'_{yz}(x,y,z) &=& 0 \\\end{array}

\begin{array}{rcl}f'_z(x,y,z) &=& 0-0+0+\dfrac{1}{2}\\&=& \dfrac{1}{2}\end{array}   \begin{array}{rcl}f'_{zx}(x,y,z) &=& 0 \\\end{array}
    \begin{array}{rcl}f'_{zy}(x,y,z) &=& 0 \\\end{array}
    \begin{array}{rcl}f'_{zz}(x,y,z) &=& 0 \\\end{array}

 

17)
\mathbb{D}=\mathbb{R}\times\mathbb{R}\times\mathbb{R}

Partielle Ableitungen 1. Ordnung 

 

Partielle Ableitungen 2. Ordnung 

\begin{array}{rcl}f'_x(x,y,z) &=& \dfrac{13}{2}\cdot 2zx^{2-1}+3\cdot 12x^{12-1}y^7+0\\\\&=& 13zx+36x^{11}y^7\end{array}   \begin{array}{rcl}f'_{xx}(x,y,z) &=& 13z+36\cdot 11x^{11-1}y^7\\&=& 13z+396x^{10}y^7\end{array}

    \begin{array}{rcl}f'_{xy}(x,y,z) &=& 0+36\cdot 7x^{11}y^{7-1} \\&=& 252x^{11}y^6\end{array}

    \begin{array}{rcl}f'_{xz}(x,y,z) &=& 13x+0 \\&=& 13x\end{array}

\begin{array}{rcl}f'_y(x,y,z) &=& 0+3\cdot 7x^{12}y^{7-1}+0\\&=& 21x^{12}y^6\end{array}   \begin{array}{rcl}f'_{yx}(x,y,z) &=& 21\cdot 12x^{12-1}y^6\\&=& 252x^{11}y^6\end{array}

    \begin{array}{rcl}f'_{yy}(x,y,z) &=& 21\cdot 6x^{12}y^{6-1}\\&=& 126x^{12}y^5\end{array}

    \begin{array}{rcl}f'_{yz}(x,y,z) &=& 0 \\\end{array}
\begin{array}{rcl}f'_z(x,y,z) &=& \dfrac{13}{2}x^2+0+\sqrt{78}\\\\&=& \dfrac{13}{2}x^2+\sqrt{78}\end{array}   \begin{array}{rcl}f'_{zx}(x,y,z) &=& \dfrac{13}{2}\cdot 2x^{2-1} \\\\&=& 13x\end{array}
    \begin{array}{rcl}f'_{zy}(x,y,z) &=& 0\\\\\end{array}

    \begin{array}{rcl}f'_{zz}(x,y,z) &=& 0 \\\end{array}

 

18)
\mathbb{D}=\mathbb{R}\times\mathbb{R}\times\mathbb{R}

Partielle Ableitungen 1. Ordnung 

 

Partielle Ableitungen 2. Ordnung 

\begin{array}{rcl}f'_a(a,b,c) &=& \dfrac{\pi}{2}\cdot 2a^{2-1}b^3c+17\pi b^3+0\\\\&=& \pi a^1b^3c+17\pi b^3\end{array}   \begin{array}{rcl}f'_{aa}(a,b,c) &=& \pi b^3c+0\\&=& \pi b^3c\end{array}

    \begin{array}{rcl}f'_{ab}(a,b,c) &=& \pi\cdot 3ab^{3-1}c+17\pi\cdot 3b^{3-1}\\&=& 3\pi ab^{2}c+51\pi b^{2}\end{array}

    \begin{array}{rcl}f'_{ac}(a,b,c) &=& \pi ab^3+0\\&=& \pi ab^3\end{array}

\begin{array}{rcl}f'_b(a,b,c) &=& \dfrac{\pi}{2}\cdot 3a^2b^{3-1}c+17\pi\cdot 3ab^{3-1}+31\cdot 2c^2b^{2-1}\\\\&=& \dfrac{3\pi}{2}a^2b^{2}c+51\pi ab^{2}+62c^2b\end{array}   \begin{array}{rcl}f'_{ba}(a,b,c) &=& \dfrac{3\pi}{2}\cdot 2a^{2-1}b^{2}c+51\pi b^{2}+0\\\\&=& 3\pi ab^{2}c+51\pi b^{2}\\\end{array}

    \begin{array}{rcl}f'_{bb}(a,b,c) &=& \dfrac{3\pi}{2}\cdot 2a^2b^{2-1}c+2\cdot 51\pi ab^{2-1}+62c^2\\\\&=& 3\pi a^2bc+102\pi ab+62c^2\end{array}

    \begin{array}{rcl}f'_{bc}(a,b,c) &=& \dfrac{3\pi}{2}a^2b^{2}+0+62\cdot 2c^{2-1}b\\\\&=& \dfrac{3\pi}{2}a^2b^{2}+124cb\\\end{array}

\begin{array}{rcl}f'_c(a,b,c) &=& \dfrac{\pi}{2}a^2b^3+0+31\cdot 2c^{2-1}b^2\\\\&=& \dfrac{\pi}{2}a^2b^3+62cb^2\end{array}   \begin{array}{rcl}f'_{ca}(a,b,c) &=& \dfrac{\pi}{2}\cdot 2a^{2-1}b^3+0\\\\&=& \pi ab^3\end{array}

    \begin{array}{rcl}f'_{cb}(a,b,c) &=& \dfrac{\pi}{2}\cdot 3a^2b^{3-1}+62\cdot 2cb^{2-1}\\\\&=& \dfrac{3\pi}{2}a^2b^{2}+124cb\end{array}

    \begin{array}{rcl}f'_{cc}(a,b,c) &=& 0+62b^2\\&=& 62b^2\end{array}

 

19)
\mathbb{D}=\mathbb{R}\times\mathbb{R}\times\mathbb{R}

Partielle Ableitungen 1. Ordnung 

 

Partielle Ableitungen 2. Ordnung 

\begin{array}{rcl}f'_x(x,y,z) &=& 3ab^2z+0-0\\&=& 3ab^2z\end{array}   \begin{array}{rcl}f'_{xx}(x,y,z) &=& 0\end{array}

    \begin{array}{rcl}f'_{xy}(x,y,z) &=& 0\end{array} 

    \begin{array}{rcl}f'_{xz}(x,y,z) &=& 3ab^2\end{array}
\begin{array}{rcl}f'_y(x,y,z) &=& 0+8abz-0\\&=& 8abz\end{array}   \begin{array}{rcl}f'_{yx}(x,y,z) &=& 0\end{array}

    \begin{array}{rcl}f'_{yy}(x,y,z) &=& 0\end{array}

    \begin{array}{rcl}f'_{yz}(x,y,z) &=& 8ab \\\end{array}

\begin{array}{rcl}f'_z(x,y,z) &=& 3ab^2x+8aby-0\\&=& ab\left(3bx+8y\right)\end{array}   \begin{array}{rcl}f'_{zx}(x,y,z) &=& 3ab^2\end{array}
    \begin{array}{rcl}f'_{zy}(x,y,z) &=& 8ab\end{array}
    \begin{array}{rcl}f'_{zz}(x,y,z) &=& 0 \\\end{array}

 

20)
\mathbb{D}=\mathbb{R}\times\mathbb{R}\times\mathbb{R}

Partielle Ableitungen 1. Ordnung 

 

Partielle Ableitungen 2. Ordnung 

\begin{array}{rcl}f'_x(x,y,z) &=& \dfrac{3}{2}\cdot 2x^{2-1}y^{\frac{1}{2}}z^3+4\cdot\dfrac{1}{3}x^{\frac{1}{3}-1}y^2\\\\&=& 3x^1y^{\frac{1}{2}}z^3+\dfrac{4}{3}x^{-\frac{2}{3}}y^2\\\\&=& 3x\sqrt{y}z^3+\dfrac{4y^2}{3x^{\frac{2}{3}}}\end{array}   \begin{array}{rcl}f'_{xx}(x,y,z) &=& 3y^{\frac{1}{2}}z^3-\dfrac{4}{3}\cdot\dfrac{2}{3}x^{-\frac{2}{3}-1}y^2\\\\&=& 3y^{\frac{1}{2}}z^3-\dfrac{8}{9}x^{-\frac{5}{3}}y^2\end{array}

    \begin{array}{rcl}f'_{xy}(x,y,z) &=& 3\cdot\dfrac{1}{2}x^1y^{\frac{1}{2}-1}z^3+\dfrac{4}{3}\cdot 2x^{-\frac{2}{3}}y^{2-1}\\\\&=& \dfrac{3}{2}x^1y^{-\frac{1}{2}}z^3+\dfrac{8}{3}x^{-\frac{2}{3}}y\end{array}

    \begin{array}{rcl}f'_{xz}(x,y,z) &=& 3\cdot 3x^1y^{\frac{1}{2}}z^{3-1}+0\\\\&=& 9xy^{\frac{1}{2}}z^{2}\end{array}

\begin{array}{rcl}f'_y(x,y,z) &=& \frac{3}{2}\cdot\dfrac{1}{2}x^2y^{\frac{1}{2}-1}z^3+4\cdot 2x^{\frac{1}{3}}y^{2-1}\\\\&=& \dfrac{3}{4}x^2y^{-\frac{1}{2}}z^3+8x^{\frac{1}{3}}y^1\\\\&=& \dfrac{3x^2z^3}{4\sqrt{y}}+8x^{\frac{1}{3}}y\end{array}   \begin{array}{rcl}f'_{yx}(x,y,z) &=& \dfrac{3}{4}\cdot 2x^{2-1}y^{-\frac{1}{2}}z^3+8\cdot\dfrac{1}{3}x^{\frac{1}{3}-1}y^1\\\\&=& \dfrac{3}{2}xy^{-\frac{1}{2}}z^3+\dfrac{8}{3}x^{-\frac{2}{3}}y^1\end{array}

    \begin{array}{rcl}f'_{yy}(x,y,z) &=& -\dfrac{3}{4}\cdot\dfrac{1}{2}x^2y^{-\frac{1}{2}-1}z^3+8x^{\frac{1}{3}}\\\\&=& -\dfrac{3}{8}x^2y^{-\frac{3}{2}}z^3+8x^{\frac{1}{3}}\end{array}

    \begin{array}{rcl}f'_{yz}(x,y,z) &=& \dfrac{3}{4}\cdot 3x^2y^{-\frac{1}{2}}z^{3-1}+0\\\\&=& \dfrac{9}{4}x^2y^{-\frac{1}{2}}z^{2}\end{array}

\begin{array}{rcl}f'_z(x,y,z) &=& \dfrac{3}{2}\cdot 3x^2y^{\frac{1}{2}}z^{3-1}+0\\\\&=& \dfrac{9}{2}x^2y^{\frac{1}{2}}z^2\\\\&=& \dfrac{9}{2}x^2\sqrt{y}z^2\end{array}   \begin{array}{rcl}f'_{zx}(x,y,z) &=& \dfrac{9}{2}\cdot 2x^{2-1}y^{\frac{1}{2}}z^2\\\\&=& 9xy^{\frac{1}{2}}z^2\end{array}

    \begin{array}{rcl}f'_{zy}(x,y,z) &=& \dfrac{9}{2}\cdot\dfrac{1}{2}x^2y^{\frac{1}{2}-1}z^2\\\\&=& \dfrac{9}{4}x^2y^{-\frac{1}{2}}z^2\end{array}

    \begin{array}{rcl}f'_{zz}(x,y,z) &=& \dfrac{9}{2}\cdot 2x^2y^{\frac{1}{2}}z^{2-1}\\\\&=& 9x^2y^{\frac{1}{2}}z\\\\\end{array}

 

2. Aufgabe

Eine Bemerkung vorab: Bei diesen Funktionen werden für die partiellen Ableitungen teilweise auch die Produkt-, Quotienten- und Kettenregel benötigt. Dies ist jeweils unter der partiellen Ableitung notiert. Die übrigen "einfachen" Regeln werden hingegen nicht erwähnt. Für alle, die mathematisch bis hierhin gekommen sind, sollten Summen-, Faktor- etc. Regel keine Schwierigkeit mehr darstellen. 

1) 
\mathbb{D}=\mathbb{R}\times\mathbb{R}

Partielle Ableitungen 1. Ordnung 

 

Partielle Ableitungen 2. Ordnung 

\begin{array}{rcl}f'_x(x,y) &=& 0+1\cdot e^{4xy}+x\cdot e^{4xy}\cdot 4y\\&=& e^{4xy}+4xy e^{4xy}\end{array}

Vorgehen: Produktregel, Kettenregel
  \begin{array}{rcl}f'_{xx}(x,y) &=& e^{4xy}\cdot 4y+4y\cdot e^{4xy}+4xy\cdot e^{4xy}\cdot 4y\\&=& 8ye^{4xy}+16xy^2e^{4xy}\end{array}

Vorgehen: Produktregel, Kettenregel

    \begin{array}{rcl}f'_{xy}(x,y) &=& e^{4xy}\cdot 4x+4x\cdot e^{4xy}+4xy\cdot e^{4xy}\cdot 4x \\&=& 8xe^{4xy}+16x^2ye^{4xy}\end{array}

Vorgehen: Produktregel, Kettenregel

\begin{array}{rcl}f'_y(x,y) &=& 0+x\cdot e^{4xy}\cdot 4x\\&=& 4x^2e^{4xy}\end{array}

Vorgehen: Kettenregel
  \begin{array}{rcl}f'_{yx}(x,y) &=& 4\cdot 2x^{2-1}\cdot e^{4xy}+4x^2\cdot e^{4xy}\cdot 4y\\&=& 8xe^{4xy}+16x^2ye^{4xy}\end{array}

Vorgehen: Produktregel, Kettenregel

    \begin{array}{rcl}f'_{yy}(x,y) &=& 4x^2\cdot e^{4xy}\cdot 4x\\&=& 16x^3e^{4xy}\end{array}

Vorgehen: Kettenregel

 

2)
\mathbb{D}=\mathbb{R}\times\mathbb{R}

Partielle Ableitungen 1. Ordnung 

 

Partielle Ableitungen 2. Ordnung 

\begin{array}{rcl}f'_x(x,y) &=& e^x+0-15y\\&=& e^x-15y\end{array}

  \begin{array}{rcl}f'_{xx}(x,y) &=& e^x-0\\&=& e^x\end{array}


    \begin{array}{rcl}f'_{xy}(x,y) &=& 0-15\\&=& -15\end{array}

\begin{array}{rcl}f'_y(x,y) &=& 0+3\cdot 2y^{2-1}-15x\\&=& 6y-15x\end{array}

  \begin{array}{rcl}f'_{yx}(x,y) &=& 0-15\\&=& -15\end{array}

    \begin{array}{rcl}f'_{yy}(x,y) &=& 6-0\\&=& 6\end{array}

 

3)
\mathbb{D}=\mathbb{R}_0^+\times\mathbb{R}

Partielle Ableitungen 1. Ordnung 

 

Partielle Ableitungen 2. Ordnung 

\begin{array}{rcl}f'_x(x,y) &=& e^{3xy-23}\cdot 3y+y\cdot\dfrac{1}{2}x^{\frac{1}{2}-1}\\\\&=& 3ye^{3xy-23}+\dfrac{1}{2}yx^{-\frac{1}{2}}\\\\&=& 3ye^{3xy-23}+\dfrac{y}{2\sqrt{x}}\end{array}

Vorgehen: Kettenregel
  \begin{array}{rcl}f'_{xx}(x,y) &=& 3y\cdot e^{3xy-23}\cdot3y-\dfrac{1}{2}y\cdot\dfrac{1}{2}x^{-\frac{1}{2}-1}\\\\&=& 9y^2e^{3xy-23}-\dfrac{1}{4}yx^{-\frac{3}{2}}\\\\&=& 9y^2e^{3xy-23}-\dfrac{y}{4\sqrt{x^3}}\end{array}

Vorgehen: Kettenregel

    \begin{array}{rcl}f'_{xy}(x,y) &=& 3\cdot e^{3xy-23}+3y\cdot e^{3xy-23}\cdot 3x+\dfrac{1}{2\sqrt{x}}\\\\&=& 3e^{3xy-23}+9yxe^{3xy-23}+\dfrac{1}{2\sqrt{x}}\end{array}

Vorgehen: Produktregel, Kettenregel

\begin{array}{rcl}f'_y(x,y) &=& e^{3xy-23}\cdot 3x+x^{\frac{1}{2}}\\&=& 3xe^{3xy-23}+\sqrt{x}\end{array}

Vorgehen: Kettenregel
  \begin{array}{rcl}f'_{yx}(x,y) &=& 3\cdot e^{3xy-23}+3x\cdot e^{3xy-23}\cdot 3y+\dfrac{1}{2} x^{\frac{1}{2}-1}\\&=& 3e^{3xy-23}+9xye^{3xy-23}+\dfrac{1}{2}x^{-\frac{1}{2}}\\&=& 3e^{3xy-23}+9xye^{3xy-23}+\dfrac{1}{2\sqrt{x}}\end{array}

Vorgehen: Produktregel, Kettenregel

    \begin{array}{rcl}f'_{yy}(x,y) &=& 3x\cdot e^{3xy-23}\cdot 3x+0\\&=& 9x^2e^{3xy-23}\end{array}

Vorgehen: Kettenregel

 


4)
\mathbb{D}=\mathbb{R}^+\times\mathbb{R}^+

Partielle Ableitungen 1. Ordnung 

 

Partielle Ableitungen 2. Ordnung 

\begin{array}{rcl}f'_x(x,y) &=& \dfrac{1}{xy}\cdot y-0\\&=& \dfrac{1}{x}\\&=& x^{-1}\end{array}

Vorgehen: Kettenregel
  \begin{array}{rcl}f'_{xx}(x,y) &=& -1\cdot x^{-1-1}\\&=& -x^{-2}\\&=& -\dfrac{1}{x^2}\end{array}

    \begin{array}{rcl}f'_{xy}(x,y) &=& 0\end{array}

\begin{array}{rcl}f'_y(x,y) &=& \dfrac{1}{xy}\cdot x-0\\&=& \dfrac{1}{y}\\&=& y^{-1}\end{array}

Vorgehen: Kettenregel
  \begin{array}{rcl}f'_{yx}(x,y) &=& 0\end{array}

    \begin{array}{rcl}f'_{yy}(x,y) &=& -1\cdot y^{-1-1}\\&=& -y^{-2}\\&=& -\dfrac{1}{y^2}\end{array}

 

5)
\mathbb{D}=\left]-1;\infty\right[\times\mathbb{R}

Partielle Ableitungen 1. Ordnung 

 

Partielle Ableitungen 2. Ordnung 

\begin{array}{rcl}f'_t(t,z) &=& \dfrac{1}{1+t}\cdot 1\cdot z^2+5\cdot 2t^{2-1}z-0\\&=& \dfrac{z^2}{1+t}+10tz\\&=& z^2\cdot \left(1+t\right)^{-1}+10tz\end{array}

Vorgehen: Kettenregel
  \begin{array}{rcl}f'_{tt}(t,z) &=& z^2\cdot (-1)\cdot \left(1+t\right)^{-1-1}\cdot 1+10z\\&=& -\dfrac{z^2}{\left(1+t\right)^2}+10z\end{array}

Vorgehen: Kettenregel

    \begin{array}{rcl}f'_{tz}(t,z) &=& \dfrac{2z^{2-1}}{1+t}+10t\\\\&=& \dfrac{2z}{1+t}+10t\end{array}

\begin{array}{rcl}f'_z(t,z) &=& \ln(1+t)\cdot 2\cdot z^{2-1}+5t^2-0\\&=& 2z\ln(1+t)+5t^2\end{array}

  \begin{array}{rcl}f'_{zt}(t,z) &=& 2z\cdot \dfrac{1}{1+t}\cdot 1+5\cdot 2t^{2-1}\\&=& \dfrac{2z}{1+t}+10t\end{array}

Vorgehen: Kettenregel

    \begin{array}{rcl}f'_{zz}(t,z) &=& 2\ln(1+t)+0\\&=& 2\ln(1+t)\end{array}

 


6)
\mathbb{D}=\mathbb{R}\times\mathbb{R} und x+y^2>0

Partielle Ableitungen 1. Ordnung 

 

Partielle Ableitungen 2. Ordnung 

\begin{array}{rcl}f'_x(x,y) &=& \dfrac{1}{x+y^2}\cdot 1-e^{2xy}\cdot 2y+3\\&=& \dfrac{1}{x+y^2}-2ye^{2xy}+3\\&=& \left(x+y^2\right)^{-1}-2ye^{2xy}+3\end{array}

Vorgehen: Kettenregel
  \begin{array}{rcl}f'_{xx}(x,y) &=& -1\left(x+y^2\right)^{-1-1}\cdot 1\\& & -2ye^{2xy}\cdot 2y\\\\&=& -\dfrac{1}{\left(x+y^2\right)^2}-4y^2e^{2xy}\end{array}

Vorgehen: Kettenregel

    \begin{array}{rcl}f'_{xy}(x,y) &=& -1\left(x+y^2\right)^{-1-1}\cdot 2y\\& & -2\cdot e^{2xy}-2ye^{2xy}\cdot 2x\\\\&=& -\dfrac{2y}{\left(x+y^2\right)^2}+\left(-2-4xy\right)e^{2xy}\\\end{array}

Vorgehen: Produktregel, Kettenregel

\begin{array}{rcl}f'_y(x,y) &=& \dfrac{1}{x+y^2}\cdot 2y-e^{2xy}\cdot 2x+0\\&=& 2y\left(x+y^2\right)^{-1}-2xe^{2xy}\end{array}

Vorgehen: Kettenregel

  \begin{array}{rcl}f'_{yx}(x,y) &=& 2y\cdot (-1)\left(x+y^2\right)^{-1-1}\cdot 1\\& & -2\cdot e^{2xy}-2xe^{2xy}\cdot 2y\\\\&=& -\dfrac{2y}{\left(x+y^2\right)^2}+\left(-2-4xy\right)e^{2xy}\end{array}

Vorgehen: Produktregel, Kettenregel

    \begin{array}{rcl}f'_{yy}(x,y) &=& 2\cdot \left(x+y^2\right)^{-1}+2y\cdot(-1)\left(x+y^2\right)^{-1-1}\cdot 2y\\& & -0\cdot e^{2xy}-2xe^{2xy}\cdot 2x\\\\&=& \dfrac{2}{x+y^2}-\dfrac{4y^2}{\left(x+y^2\right)^2}-4x^2e^{2xy}\end{array}

Vorgehen: Kettenregel, Produktregel

 

7)
\mathbb{D}=\mathbb{R}\times \mathbb{R}^+

Partielle Ableitungen 1. Ordnung 

 

Partielle Ableitungen 2. Ordnung 

\begin{array}{rcl}f'_x(x,y) &=& e^{3x}\cdot 3 \cdot\ln(y)-e^{2y}\cdot 5\\&=& 3e^{3x}\cdot\ln(y)-5e^{2y}\end{array}

Vorgehen: Kettenregel
  \begin{array}{rcl}f'_{xx}(x,y) &=& 3e^{3x}\cdot 3 \cdot\ln(y)-0\\&=& 9e^{3x}\cdot\ln(y)\end{array}

Vorgehen: Kettenregel

    \begin{array}{rcl}f'_{xy}(x,y) &=& 3e^{3x}\cdot\dfrac{1}{y}-5e^{2y}\cdot 2\\&=& \dfrac{3e^{3x}}{y}-10e^{2y}\end{array}

Vorgehen: Kettenregel

\begin{array}{rcl}f'_y(x,y) &=& e^{3x}\cdot\dfrac{1}{y}-e^{2y}\cdot 2 \cdot 5x\\&=& \dfrac{e^{3x}}{y}-10xe^{2y}\\&=& e^{3x}\cdot y^{-1}-10xe^{2y}\end{array}

Vorgehen: Kettenregel
  \begin{array}{rcl}f'_{yx}(x,y) &=& e^{3x}\cdot 3\cdot\dfrac{1}{y}-10e^{2y}\\&=& \dfrac{3e^{3x}}{y}-10e^{2y}\end{array}

Vorgehen: Kettenregel

    \begin{array}{rcl}f'_{yy}(x,y) &=& e^{3x}\cdot (-1)\cdot y^{-1-1}-10xe^{2y}\cdot 2\\&=& -\dfrac{e^{3x}}{y^2}-20xe^{2y}\end{array}

Vorgehen: Kettenregel

 

8)
\mathbb{D}=\mathbb{R}^+\times\mathbb{R}

Partielle Ableitungen 1. Ordnung 

 

Partielle Ableitungen 2. Ordnung 

\begin{array}{rcl}g'_x(x,y) &=& 52e^y+\dfrac{y}{8}\cdot\dfrac{1}{x}\\\\&=& 52e^y+\dfrac{y}{8x}\\\\&=& 52e^y+\dfrac{1}{8}x^{-1}y\end{array}

  \begin{array}{rcl}g'_{xx}(x,y) &=& 0+\dfrac{1}{8}\cdot(-1)x^{-1-1}y\\\\&=& -\dfrac{y}{8x^2}\end{array}

    \begin{array}{rcl}g'_{xy}(x,y) &=& 52e^y+\dfrac{1}{8x}\end{array}

\begin{array}{rcl}g'_y(x,y) &=& 52xe^y+\dfrac{1}{8}\ln(x)\end{array}

  \begin{array}{rcl}g'_{yx}(x,y) &=& 52e^y+\dfrac{1}{8x}\end{array}

    \begin{array}{rcl}g'_{yy}(x,y) &=& 52xe^y+0\\&=& 52xe^y\end{array}

 

9)
\mathbb{D}=\mathbb{R}\times\mathbb{R}

Partielle Ableitungen 1. Ordnung 

 

Partielle Ableitungen 2. Ordnung 

\begin{array}{rcl}g'_h(h,k) &=& 135\cos\left(h^2-k\right)\cdot 2h\\&=& 270h\cos\left(h^2-k\right)\end{array}

Vorgehen:
Kettenregel
  \begin{array}{rcl}g'_{hh}(h,k) &=& 270\cos\left(h^2-k\right)\\& & +270h\left(-\sin\left(h^2-k\right)\cdot 2h\right)\\&=& 270\cos\left(h^2-k\right)-540h^2\sin\left(h^2-k\right)\end{array}

Vorgehen: Produktregel, Kettenregel

    \begin{array}{rcl}g'_{hk}(h,k) &=& -270h\sin\left(h^2-k\right)\cdot 1\\&=& -270h\sin\left(h^2-k\right)\end{array}

Vorgehen: Kettenregel

\begin{array}{rcl}g'_k(h,k) &=& 135\cos\left(h^2-k\right)\cdot 1\\&=& 135\cos\left(h^2-k\right)\end{array}

Vorgehen: Kettenregel
  \begin{array}{rcl}g'_{kh}(h,k) &=& -135\sin\left(h^2-k\right)\cdot 2h\\&=& -270h\sin\left(h^2-k\right)\end{array}

Vorgehen: Kettenregel

    \begin{array}{rcl}g'_{kk}(h,k) &=& -135\sin\left(h^2-k\right)\cdot 1\\&=& -135\sin\left(h^2-k\right)\end{array}

Vorgehen: Kettenregel

 

10)
\mathbb{D}=\mathbb{R}\times\mathbb{R}

Partielle Ableitungen 1. Ordnung 

 

Partielle Ableitungen 2. Ordnung 

\begin{array}{rcl}f'_x(x,y) &=& 4\cos(x)-\sin\left(-\dfrac{1}{4}x+y\right)\cdot\left(-\dfrac{1}{4}\right)\\&=& 4\cos(x)+\dfrac{1}{4}\sin\left(-\dfrac{1}{4}x+y\right)\end{array}

Vorgehen: Kettenregel
  \begin{array}{rcl}f'_{xx}(x,y) &=&-4\sin(x)\\& & +\dfrac{1}{4}\cos\left(-\dfrac{1}{4}x+y\right)\cdot \left(-\dfrac{1}{4}\right)\\&=& -4\sin(x)-\dfrac{1}{16}\cos\left(-\dfrac{1}{4}x+y\right)\end{array}

Vorgehen: Kettenregel

    \begin{array}{rcl}f'_{xy}(x,y) &=& \dfrac{1}{4}\cos\left(-\dfrac{1}{4}x+y\right)\cdot 1\\&=& \dfrac{1}{4}\cos\left(-\dfrac{1}{4}x+y\right)\end{array}

Vorgehen: Kettenregel

\begin{array}{rcl}f'_y(x,y) &=& -\sin(-4y)\cdot(-4)-\sin\left(-\dfrac{1}{4}x+y\right)\cdot 1\\&=& 4\sin(-4y)-\sin\left(-\dfrac{1}{4}x+y\right)\end{array}

Vorgehen: Kettenregel
  \begin{array}{rcl}f'_{yx}(x,y) &=& -\cos\left(-\dfrac{1}{4}x+y\right)\cdot \left(-\dfrac{1}{4}\right)\\&=& \dfrac{1}{4}\cos\left(-\dfrac{1}{4}x+y\right)\end{array}

Vorgehen: Kettenregel

    \begin{array}{rcl}f'_{yy}(x,y) &=& 4\cos(-4y)\cdot (-4)\\& & -\cos\left(-\dfrac{1}{4}x+y\right)\cdot 1\\&=&-16\cos(-4y)-\cos\left(-\dfrac{1}{4}x+y\right)\end{array}

Vorgehen: Kettenregel

 

11)
\mathbb{D}=\mathbb{R}\times\mathbb{R}

Partielle Ableitungen 1. Ordnung 

 

Partielle Ableitungen 2. Ordnung 

\begin{array}{rcl}g'_a(a,b) &=& -b\sin(a)+\sin(b)-b\end{array}
  \begin{array}{rcl}g'_{aa}(a,b) &=& -b\cos(a)+0+0\\&=& -b\cos(a)\end{array}

    \begin{array}{rcl}g'_{ab}(a,b) &=& -\sin(a)+\cos(b)-1\end{array}

\begin{array}{rcl}g'_b(a,b) &=& \cos(a)+a\cos(b)-a\end{array}
  \begin{array}{rcl}g'_{ba}(a,b) &=& -\sin(a)+\cos(b)-1\end{array}

    \begin{array}{rcl}g'_{bb}(a,b) &=& 0-a\sin(b)+0\\&=& -a\sin(b)\end{array}

 

12)
\begin{array}{rcl}\mathbb{D} &=& \mathbb{R}\times\mathbb{R}\setminus_{\{0\}} \\\\g(x,y) &=& \dfrac{7\cos(x)}{10y}+19xy-0{,}851 \\&=& \dfrac{7}{10} \cos(x)\cdot y^{-1}+19xy-0{,}851\end{array}

Partielle Ableitungen 1. Ordnung 

 

Partielle Ableitungen 2. Ordnung 

\begin{array}{rcl}g'_x(x,y) &=& -\dfrac{7\sin(x)}{10y}+19y\\\\&=& -\dfrac{7}{10}\sin(x)\cdot y^{-1}+19y\end{array}

  \begin{array}{rcl}g'_{xx}(x,y) &=& -\dfrac{7\cos(x)}{10y}+0\\\\&=& -\dfrac{7\cos(x)}{10y}\end{array}

    \begin{array}{rcl}g'_{xy}(x,y) &=& -\dfrac{7}{10}\sin(x)\cdot(-1)y^{-1-1}+19\\\\&=& \dfrac{7}{10}\sin(x)\cdot y^{-2}+19\\\\&=& \dfrac{7\sin(x)}{10y^2}+19\end{array}

\begin{array}{rcl}g'_y(x,y) &=& \dfrac{7}{10}\cos(x)\cdot(-1)y^{-1-1}+19x\\\\&=& -\dfrac{7}{10}\cos(x)\cdot y^{-2}+19x\\\\&=& -\dfrac{7\cos(x)}{10y^2}+19x\end{array}

  \begin{array}{rcl}g'_{yx}(x,y) &=& \dfrac{7\sin(x)}{10y^2}+19\end{array}

    \begin{array}{rcl}g'_{yy}(x,y) &=& -\dfrac{7}{10}\cos(x)\cdot(-2)y^{-2-1}+0\\\\&=& \dfrac{7}{5}\cos(x)\cdot y^{-3}\\\\&=&\dfrac{7\cos(x)}{5y^3}\end{array}

 

13)

Partielle Ableitungen 1. Ordnung 

 

Partielle Ableitungen 2. Ordnung 

\begin{array}{rcl}f'_x(x,y) &=& \dfrac{6\cdot\dfrac{1}{x}\cdot\left(x+9y\right)-6\ln(x)\cdot 1}{\left(x+9y\right)^2} \\\\&=&\dfrac{6\left(x+9y\right)-6x\ln(x)}{x\left(x+9y\right)^2} \\\\&=& \dfrac{6x-6x\ln(x)+54y}{x(x+9y)^2}\end{array}

Vorgehen: Quotientenregel
  \begin{array}{rcl}f'_{xx}(x,y) &=& \dfrac{\left(6-6\ln(x)-6x\cdot\frac{1}{x}\right)\cdot x\left(x+9y\right)^2-\left(6x-6x\ln(x)+54y\right)\cdot\left(1\cdot\left(x+9y\right)^2+x\cdot 2\left(x+9y\right)\cdot 1\right)}{x^2\left(x+9y\right)^4} \\\\&=& \dfrac{\left(6-6\ln(x)-6\right)\cdot x\left(x+9y\right)^2-\left(6x-6x\ln(x)+54y\right)\cdot\left(\left(x+9y\right)^2+2x(x+9y)\right)}{x^2\left(x+9y\right)^4} \\\\&=& \dfrac{(x+9y)\left[-6\ln(x)\cdot x\left(x+9y\right)-\left(6x-6x\ln(x)+54y\right)\cdot\left(\left(x+9y\right)+2x\right)\right]}{x^2\left(x+9y\right)^4} \\\\&=& \dfrac{-6x\ln(x)\cdot\left(x+9y\right)-\left(6x-6x\ln(x)+54y\right)\cdot\left(3x+9y\right)}{x^2\left(x+9y\right)^3} \\\\&=& \dfrac{-6x^2\ln(x)-54xy\ln(x)-18x^2+18x^2\ln(x)-162xy-54xy+54xy\ln(x)-486y^2}{x^2\left(x+9y\right)^3} \\\\&=& \dfrac{12x^2\ln(x)-18x^2-216xy-486y^2}{x^2\left(x+9y\right)^3}\end{array}

Vorgehen: Produktregel, Quotientenregel, Kettenregel

    \begin{array}{rcl}f'_{xy}(x,y) &=& \dfrac{54\cdot x\left(x+9y\right)^2-\left(6x-6x\ln(x)+54y\right)\cdot 2x\left(x+9y\right)^{2-1}\cdot 9}{x^2\left(x+9y\right)^4} \\\\&=& \dfrac{x\left(x+9y\right)\left[54\left(x+9y\right)-\left(6x-6x\ln(x)+54y\right)\cdot 18\right]}{x^2\left(x+9y\right)^4} \\\\&=& \dfrac{54x+486y-108x+108x\ln(x)-972y}{x\left(x+9y\right)^3} \\\\&=& \dfrac{-54x-486y+108x\ln(x)}{x\left(x+9y\right)^3}\end{array}

Vorgehen: Quotientenregel, Kettenregel

\begin{array}{rcl}f'_y(x,y) &=& \dfrac{0\cdot\left(x+9y\right)-6\ln(x)\cdot 9}{\left(x+9y\right)^2} \\\\&=& -\dfrac{54\ln(x)}{(x+9y)^2}\end{array}

Vorgehen: Quotientenregel
  \begin{array}{rcl}f'_{yx}(x,y) &=& \dfrac{-54\cdot\dfrac{1}{x}\cdot\left(x+9y\right)^2-\left(-54\ln(x)\cdot 2\left(x+9y\right)^{2-1}\cdot 1\right)}{\left(x+9y\right)^4} \\\\&=& \dfrac{-54\cdot\left(x+9y\right)^2+54x\ln(x)\cdot 2\left(x+9y\right)}{x\left(x+9y\right)^4} \\\\&=& \dfrac{(x+9y)\left[-54\left(x+9y\right)+108x\ln(x)\right]}{x\left(x+9y\right)^4} \\\\&=& \dfrac{-54x-486y+108x\ln(x)}{x\left(x+9y\right)^3}\end{array}

Vorgehen: Quotientenregel, Kettenregel

    \begin{array}{rcl}f'_{yy}(x,y) &=& \dfrac{0\cdot\left(x+9y\right)^2-\left(-54\ln(x)\cdot 2\left(x+9y\right)^{2-1}\cdot 9\right)}{\left(x+9y\right)^4} \\\\&=& \dfrac{54\ln(x)\cdot 18\left(x+9y\right)}{\left(x+9y\right)^4} \\\\&=& \dfrac{972\ln(x)}{\left(x+9y\right)^3}\end{array}

Vorgehen: Quotientenregel, Kettenregel

 

14)

Partielle Ableitungen 1. Ordnung 

 

Partielle Ableitungen 2. Ordnung 

\begin{array}{rcl}g'_r(r,t) &=& -\dfrac{16}{\cos^2(r)}+3te^{rt}\\\end{array}

Vorgehen: Kettenregel
  \begin{array}{rcl}g'_{rr}(r,t) &=& \dfrac{0\cdot \cos^2(r)-\left(-16\cdot 2\cdot\cos(r)\cdot\left(-\sin(r)\right)\right)}{\cos^4(r)}+3t\cdot te^{rt} \\\\&=& -\dfrac{32\cos(r)\cdot\sin(r)}{\cos^4(r)}+3t^2e^{rt} \\\\&=& -\dfrac{32\sin(r)}{\cos^3(r)}+3t^2e^{rt}\end{array}

Vorgehen: Quotientenregel, Kettenregel

    \begin{array}{rcl}g'_{rt}(r,t) &=& 0+3\cdot e^{rt}+3t\cdot re^{rt} \\&=& 3e^{rt}+3rte^{rt} \\&=& 3e^{rt}\left(1+rt\right)\end{array}

Vorgehen: Produktregel, Kettenregel

\begin{array}{rcl}g'_t(r,t) &=& 0+3r\cdot e^{rt} \\&=& 3re^{rt}\end{array}

Vorgehen: Kettenregel
  \begin{array}{rcl}g'_{tr}(r,t) &=& 3\cdot e^{rt}+3r\cdot te^{rt} \\&=& 3e^{rt}+3rte^{rt} \\&=& 3e^{rt}\left(1+rt\right)\end{array}

Vorgehen: Produktregel, Kettenregel

    \begin{array}{rcl}g'_{tt}(r,t) &=& 3r\cdot re^{rt} \\&=& 3r^2e^{rt}\end{array}

Vorgehen: Kettenregel

 

15)

Partielle Ableitungen 1. Ordnung 

 

Partielle Ableitungen 2. Ordnung  

\begin{array}{rcl}f'_x(x,y) &=& \dfrac{5}{9}e^{\sin(x)+\sin(y)}\cdot \cos(x) \\\\&=& \dfrac{5}{9}\cos(x)e^{\sin(x)+\sin(y)}\end{array}


Vorgehen:
Kettenregel
  \begin{array}{rcl}f'_{xx}(x,y) &=& -\dfrac{5}{9}\sin(x)\cdot e^{\sin(x)+\sin(y)}+\dfrac{5}{9}\cos(x)e^{\sin(x)+\sin(y)}\cdot\cos(x) \\\\&=& -\dfrac{5}{9}\sin(x)e^{\sin(x)+\sin(y)}+\dfrac{5}{9}\cos^2(x)e^{\sin(x)+\sin(y)} \\\\&=& \dfrac{5}{9}e^{\sin(x)+\sin(y)}\left(-\sin(x)+\cos^2(x)\right)\end{array}

Vorgehen: Produktregel, Kettenregel

    \begin{array}{rcl}f'_{xy}(x,y) &=& \dfrac{5}{9}\cos(x)e^{\sin(x)+\sin(y)}\cdot\cos(y) \\\\&=& \dfrac{5}{9}\cos(x)\cos(y)e^{\sin(x)+\sin(y)}\end{array}

Vorgehen: Kettenregel

\begin{array}{rcl}f'_y(x,y) &=& \dfrac{5}{9}e^{\sin(x)+\sin(y)}\cdot \cos(y) \\\\&=& \dfrac{5}{9}\cos(y)e^{\sin(x)+\sin(y)}\end{array}

Vorgehen: Kettenregel
  \begin{array}{rcl}f'_{yx}(x,y) &=& \dfrac{5}{9}\cos(y)e^{\sin(x)+\sin(y)}\cdot\cos(x) \\\\&=& \dfrac{5}{9}\cos(x)\cos(y)e^{\sin(x)+\sin(y)}\end{array}

Vorgehen: Kettenregel

    \begin{array}{rcl}f'_{yy}(x,y) &=& -\dfrac{5}{9}\sin(y)\cdot e^{\sin(x)+\sin(y)}+\dfrac{5}{9}\cos(y)e^{\sin(x)+\sin(y)}\cdot\cos(y) \\\\&=& -\dfrac{5}{9}\sin(y)e^{\sin(x)+\sin(y)}+\dfrac{5}{9}\cos^2(y)e^{\sin(x)+\sin(y)} \\\\&=& \dfrac{5}{9}e^{\sin(x)+\sin(y)}\left(-\sin(y)+\cos^2(y)\right)\end{array}

Vorgehen: Produktregel, Kettenregel

 

16)
\mathbb{D}=\mathbb{R}\times\mathbb{R}\times\mathbb{R}^+

Partielle Ableitungen 1. Ordnung

 

Partielle Ableitungen 2. Ordnung

\begin{array}{rcl} f'_x(x,y,z) &=& y^2 e^{-3xy} \ln(z)+xy^2 e^{-3xy}\cdot (-3y)\ln(z) \cr \cr &=& y^2 e^{-3xy} \ln(z) - 3xy^3 e^{-3xy} \ln(z) \cr \cr&=& \left(y^2-3xy^3\right) e^{-3xy} \ln(z)\end{array}

Vorgehen: Produktregel, Kettenregel
  \begin{array}{rcl} f'_{xx}(x,y,z) &=& -3y^3 e^{-3xy} \ln(z) + \left(y^2-3xy^3\right) e^{-3xy}\cdot(-3y)\ln(z) \\\\&=& -3y^3 e^{-3xy} \ln(z) - 3y\left(y^2-3xy^3\right) e^{-3xy}\ln(z) \\\\&=& \left[-3y^3- 3y\left(y^2-3xy^3\right)\right] e^{-3xy} \ln(z) \\\\&=& \left[-3y^3-3y^3+9xy^4\right] e^{-3xy} \ln(z) \\\\&=& \left[9xy^4-6y^3\right] e^{-3xy} \ln(z)\end{array}

Vorgehen: Produktregel, Kettelregel

 

  \begin{array}{rcl} f'_{xy}(x,y,z) &=& \left(2y-9xy^2\right) e^{-3xy} \ln(z) + \left(y^2-3xy^3\right) e^{-3xy} \cdot (-3x) \ln(z) \\\\&=& \left(2y-9xy^2\right) e^{-3xy} \ln(z) - 3x\left(y^2-3xy^3\right) e^{-3xy} \ln(z) \\\\&=& \left[\left(2y-9xy^2\right) - 3x\left(y^2-3xy^3\right)\right] e^{-3xy} \ln(z) \\\\&=& \left[2y-9xy^2-3xy^2+9x^2y^3\right] e^{-3xy} \ln(z) \\\\&=& \left[9x^2y^3-12xy^2+2y\right] e^{-3xy} \ln(z)\end{array}

Vorgehen: Produktregel, Kettelregel

    \begin{array}{rcl} f'_{xz}(x,y,z) &=& \left(y^2-3xy^3\right) e^{-3xy}\cdot\dfrac{1}{z} \\\\&=& \dfrac{y^2-3xy^3}{z}e^{-3xy}\end{array}

\begin{array}{rcl} f'_y(x,y,z) &=& 2xy e^{-3xy} \ln(z) +xy^2 e^{-3xy}\cdot (-3x) \ln(z) \cr \cr &=& 2xy e^{-3xy} \ln(z) - 3x^2y^2 e^{-3xy} \ln(z) \cr \cr&=& \left(-3x^2y^2+2xy\right) e^{-3xy} \ln(z)\end{array}

Vorgehen: Produktregel, Kettenregel
  \begin{array}{rcl} f'_{yx}(x,y,z) &=& \left(-6xy^2+2y\right) e^{-3xy}\ln(z) + \left(-3x^2y^2+2xy\right) e^{-3xy} \cdot(-3y)\ln(z) \\\\&=& \left(-6xy^2+2y\right) e^{-3xy}\ln(z) - 3y\left(-3x^2y^2+2xy\right) e^{-3xy} \ln(z) \\\\&=& \left[\left(-6xy^2+2y\right) - 3y\left(-3x^2y^2+2xy\right)\right] e^{-3xy} \ln(z) \\\\&=& \left[-6xy^2+2y+9x^2y^3-6xy^2\right] e^{-3xy} \ln(z) \\\\&=& \left[9x^2y^3-12xy^2+2y\right] e^{-3xy} \ln(z)\end{array}

Vorgehen: Produktregel, Kettenregel

 

  \begin{array}{rcl} f'_{yy}(x,y,z) &=& \left(-6x^2y+2x\right) e^{-3xy}\ln(z) + \left(-3x^2y^2+2xy\right) e^{-3xy} \cdot (-3x) \ln(z) \\\\&=& \left(-6x^2y+2x\right) e^{-3xy}\ln(z) - 3x\left(-3x^2y^2+2xy\right) e^{-3xy} \ln(z) \\\\&=& \left[\left(-6x^2y+2x\right) - 3x\left(-3x^2y^2+2xy\right)\right] e^{-3xy} \ln(z) \\\\&=& \left[-6x^2y+2x+9x^3y^2-6x^2y\right] e^{-3xy} \ln(z) \\\\&=& \left[9x^3y^2-12x^2y+2x\right] e^{-3xy} \ln(z)\end{array}

Vorgehen: Produktregel, Kettenregel

    \begin{array}{rcl} f'_{yz}(x,y,z) &=& \left(-3x^2y^2+2xy\right) e^{-3xy}\cdot\dfrac{1}{z} \\\\&=& \dfrac{-3x^2y^2+2xy}{z}e^{-3xy} \\\\\end{array}

\begin{array}{rcl} f'_z(x,y,z) &=& xy^2 e^{-3xy}\cdot\dfrac{1}{z} \\\\ &=& \dfrac{xy^2 e^{-3xy}}{z} \\\\ &=& xy^2 e^{-3xy}\cdot z^{-1} \end{array}   \begin{array}{rcl} f'_{zx}(x,y,z) &=& \dfrac{y^2 e^{-3xy} +xy^2 e^{-3xy}\cdot (-3y)}{z}\\\\&=& \dfrac{y^2 e^{-3xy}-3xy^3 e^{-3xy}}{z}\\\\&=& \dfrac{-3xy^3+y^2}{z} e^{-3xy}\end{array}

Vorgehen: Produktregel, Kettenregel

    \begin{array}{rcl} f'_{zy}(x,y,z) &=& \dfrac{2xy e^{-3xy}+xy^2 e^{-3xy} \cdot(-3x)}{z} \\\\&=& \dfrac{2xy e^{-3xy}-3x^2y^2 e^{-3xy}}{z} \\\\&=& \dfrac{2xy-3x^2y^2}{z} e^{-3xy} \\\\\end{array}

Vorgehen: Produktregel, Kettenregel

    \begin{array}{rcl} f'_{zz}(x,y,z) &=& xy^2 e^{-3xy}\cdot (-1)\cdot z^{-1-1} \\\\ &=& -xy^2 e^{-3xy} z^{-2} \\\\&=& -\dfrac{xy^2}{z^2} e^{-3xy}\end{array}

 

17)
\begin{array}{rcl}\mathbb{D} &=& \mathbb{R}\times\mathbb{R}\times\mathbb{R} \text{ mit } x\neq -yz \\\\f(x,y,z) &=& 300 \cdot \sqrt{x+yz}-923 \\&=& 300(x+yz)^{\frac{1}{2}}-923\end{array}

Partielle Ableitungen 1. Ordnung
  Partielle Ableitungen 2. Ordnung
\begin{array}{rcl} f'_x(x,y,z) &=& 300\cdot \dfrac{1}{2}\left(x+yz\right)^{\frac{1}{2}-1} \cdot 1\cr \cr &=& 150\left(x+yz\right)^{-\frac{1}{2}} \cr \cr&=& \dfrac{150}{\sqrt{x+yz}} \end{array}

Vorgehen: Kettenregel
  \begin{array}{rcl} f'_{xx}(x,y,z) &=& 150\cdot \left(-\dfrac{1}{2}\right)(x+yz)^{-\frac{1}{2}-1}\cdot 1 \\\\&=& -75(x+yz)^{-\frac{3}{2}} \\\\&=& -\dfrac{75}{\left(\sqrt{x+yz}\right)^3}\end{array}

Vorgehen: Kettenregel

    \begin{array}{rcl} f'_{xy}(x,y,z) &=& 150\cdot \left(-\dfrac{1}{2}\right)(x+yz)^{-\frac{1}{2}-1}\cdot z \\\\&=& -75z(x+yz)^{-\frac{3}{2}} \\\\&=& -\dfrac{75z}{\left(\sqrt{x+yz}\right)^3}\end{array}

Vorgehen: Kettenregel

    \begin{array}{rcl} f'_{xz}(x,y,z) &=& 150\cdot \left(-\dfrac{1}{2}\right)(x+yz)^{-\frac{1}{2}-1}\cdot y \\\\&=& -75y(x+yz)^{-\frac{3}{2}} \\\\&=& -\dfrac{75y}{\left(\sqrt{x+yz}\right)^3} \end{array}

Vorgehen: Kettenregel

\begin{array}{rcl} f'_y(x,y,z) &=& 300\cdot\dfrac{1}{2}\left(x+yz\right)^{\frac{1}{2}-1}\cdot z \cr \cr &=& 150z\left(x+yz\right)^{-\frac{1}{2}} \cr \cr&=& \dfrac{150z}{\sqrt{x+yz}} \end{array}

Vorgehen: Kettenregel
  \begin{array}{rcl} f'_{yx}(x,y,z) &=& 150z\cdot \left(-\dfrac{1}{2}\right)(x+yz)^{-\frac{1}{2}-1}\cdot 1 \\\\ &=& -75z{(x+yz)^{-\frac{3}{2}}}\\\\&=& -\dfrac{75z}{\left(\sqrt{x+yz}\right)^3}\end{array}

Vorgehen: Kettenregel

    \begin{array}{rcl} f'_{yy}(x,y,z) &=& 150z\cdot \left(-\dfrac{1}{2}\right)(x+yz)^{-\frac{1}{2}-1}\cdot z \\\\ &=& -75z^2{(x+yz)^{-\frac{3}{2}}}\\\\&=& -\dfrac{75z^2}{\left(\sqrt{x+yz}\right)^3}\end{array}

Vorgehen: Kettenregel

    \begin{array}{rcl} f'_{yz}(x,y,z) &=& 150\left(x+yz\right)^{-\frac{1}{2}}+150z\cdot \left(-\dfrac{1}{2}\right)(x+yz)^{-\frac{1}{2}-1}\cdot y \\\\ &=& 150\left(x+yz\right)^{-\frac{1}{2}}-75yz{(x+yz)^{-\frac{3}{2}}}\\\\&=& \dfrac{150}{\sqrt{x+yz}}-\dfrac{75yz}{\left(\sqrt{x+yz}\right)^3}\end{array}

Vorgehen: Kettenregel

\begin{array}{rcl}f'_z(x,y,z) &=& 300\cdot\dfrac{1}{2}\left(x+yz\right)^{\frac{1}{2}-1}\cdot y \cr \cr&=& 150y\left(x+yz\right)^{-\frac{1}{2}} \cr \cr&=& \dfrac{150y}{\sqrt{x+yz}} \end{array}

Vorgehen: Kettenregel
  \begin{array}{rcl} f'_{zx}(x,y,z) &=& 150y\cdot \left(-\dfrac{1}{2}\right)(x+yz)^{-\frac{1}{2}-1}\cdot 1 \\\\ &=& -75y{(x+yz)^{-\frac{3}{2}}}\\\\&=& -\dfrac{75y}{\left(\sqrt{x+yz}\right)^3}\end{array}

Vorgehen: Kettenregel

    \begin{array}{rcl} f'_{zy}(x,y,z) &=& 150\left(x+yz\right)^{-\frac{1}{2}}+150y\cdot \left(-\dfrac{1}{2}\right)(x+yz)^{-\frac{1}{2}-1}\cdot z \\\\ &=& 150\left(x+yz\right)^{-\frac{1}{2}}-75yz{(x+yz)^{-\frac{3}{2}}}\\\\&=& \dfrac{150}{\sqrt{x+yz}}-\dfrac{75yz}{\left(\sqrt{x+yz}\right)^3}\end{array}

Vorgehen: Kettenregel

    \begin{array}{rcl} f'_{zz}(x,y,z) &=& 150y\cdot \left(-\dfrac{1}{2}\right)(x+yz)^{-\frac{1}{2}-1}\cdot y \\\\ &=& -75y^2{(x+yz)^{-\frac{3}{2}}}\\\\&=& -\dfrac{75y^2}{\left(\sqrt{x+yz}\right)^3}\end{array}

Vorgehen: Kettenregel

 

18)
\mathbb{D}=\mathbb{R}\times\mathbb{R}\times\mathbb{R}, x und y nicht gleichzeitig 0

Partielle Ableitungen 1. Ordnung

  Partielle Ableitungen 2. Ordnung
\begin{array}{rcl} f'_x(x,y,z) &=& \dfrac{-8z^2\cdot (17x^2+y^2) - \left(-8xz^2\right) \cdot 34x}{\left(17x^2+y^2\right)^2} \\\\ &=& \dfrac{-136x^2z^2-8y^2z^2+272x^2z^2}{\left(17x^2+y^2\right)^2} \\\\&=& \dfrac{136x^2z^2-8y^2z^2}{\left(17x^2+y^2\right)^2} \end{array}

Vorgehen: Quotientenregel
  \begin{array}{rcl} f'_{xx}(x,y,z) &=& \dfrac{272xz^2 \cdot\left(17x^2+y^2\right)^2-\left(136x^2z^2-8y^2z^2\right)\cdot 2(17x^2+y^2)\cdot 34x}{\left(\left(17x^2+y^2\right)^2\right)^2} \\\\&=& \dfrac{\left(17x^2+y^2\right)\left[272xz^2 \cdot\left(17x^2+y^2\right)-\left(136x^2z^2-8y^2z^2\right)\cdot 2\cdot 34x\right]}{\left(17x^2+y^2\right)^4} \\\\&=& \dfrac{272xz^2 \cdot\left(17x^2+y^2\right)-\left(136x^2z^2-8y^2z^2\right)\cdot 2\cdot 34x}{\left(17x^2+y^2\right)^3} \\\\&=& \dfrac{4.624x^3z^2+272xy^2z^2-9.248x^3z^2+544xy^2z^2}{\left(17x^2+y^2\right)^3} \\\\&=& \dfrac{-4.624x^3z^2+816xy^2z^2}{(17x^2+y^2)^3}\end{array}

Vorgehen: Quotientenregel, Kettenregel

    \begin{array}{rcl} f'_{xy}(x,y,z) &=& \dfrac{-16yz^2 \cdot\left(17x^2+y^2\right)^2-\left(136x^2z^2-8y^2z^2\right)\cdot 2(17x^2+y^2)\cdot 2y}{\left(\left(17x^2+y^2\right)^2\right)^2} \\\\&=& \dfrac{\left(17x^2+y^2\right)\left[-16yz^2 \cdot\left(17x^2+y^2\right)-\left(136x^2z^2-8y^2z^2\right)\cdot 2\cdot 2y\right]}{\left(17x^2+y^2\right)^4} \\\\&=& \dfrac{-16yz^2 \cdot\left(17x^2+y^2\right)-\left(136x^2z^2-8y^2z^2\right)\cdot 2\cdot 2y}{\left(17x^2+y^2\right)^3} \\\\&=& \dfrac{-272x^2yz^2-16y^3z^2-544x^2yz^2+32y^3z^2}{\left(17x^2+y^2\right)^3} \\\\&=& \dfrac{-816x^2yz^2+16y^3z^2}{(17x^2+y^2)^3}\end{array}

Vorgehen: Quotientenregel, Kettenregel

    \begin{array}{rcl} f'_{xz}(x,y,z) &=& \dfrac{272x^2z-16y^2z}{\left(17x^2+y^2\right)^2}\end{array}

\begin{array}{rcl} f'_y(x,y,z) &=& \dfrac{0 \cdot \left(17x^2 + y^2\right) - \left(-8xz^2\right) \cdot 2y}{\left(17x^2+y^2\right)^2} \cr \cr &=& \dfrac{16xyz^2}{\left(17x^2+y^2\right)^2}\end{array}

Vorgehen: Quotientenregel
  \begin{array}{rcl} f'_{yx}(x,y,z) &=& \dfrac{16yz^2 \cdot\left(17x^2+y^2\right)^2-16xyz^2\cdot 2(17x^2+y^2)\cdot 34x}{\left(\left(17x^2+y^2\right)^2\right)^2} \\\\&=& \dfrac{\left(17x^2+y^2\right)\left[16yz^2 \cdot\left(17x^2+y^2\right)-16xyz^2\cdot 2\cdot 34x\right]}{\left(17x^2+y^2\right)^4} \\\\&=& \dfrac{16yz^2 \cdot\left(17x^2+y^2\right)-16xyz^2\cdot 2\cdot 34x}{\left(17x^2+y^2\right)^3} \\\\&=& \dfrac{272x^2yz^2+16y^3z^2-1.088x^2yz^2}{\left(17x^2+y^2\right)^3} \\\\&=& \dfrac{-816x^2yz^2+16y^3z^2}{\left(17x^2+y^2\right)^3} \end{array}

Vorgehen: Quotientenregel, Kettenregel

    \begin{array}{rcl} f'_{yy}(x,y,z) &=& \dfrac{16xz^2 \cdot\left(17x^2+y^2\right)^2-16xyz^2\cdot 2(17x^2+y^2)\cdot 2y}{\left(\left(17x^2+y^2\right)^2\right)^2} \\\\&=& \dfrac{\left(17x^2+y^2\right)\left[16xz^2 \cdot\left(17x^2+y^2\right)-16xyz^2\cdot 2\cdot 2y\right]}{\left(17x^2+y^2\right)^4} \\\\&=& \dfrac{16xz^2 \cdot\left(17x^2+y^2\right)-16xyz^2\cdot 2\cdot 2y}{\left(17x^2+y^2\right)^3} \\\\&=& \dfrac{272x^3z^2+16xy^2z^2-64xy^2z^2}{\left(17x^2+y^2\right)^3} \\\\&=& \dfrac{272x^3z^2-48xy^2z^2}{\left(17x^2+y^2\right)^3}\end{array}

Vorgehen: Quotientenregel, Kettenregel

    \begin{array}{rcl} f'_{yz}(x,y,z) &=& \dfrac{32xyz}{\left(17x^2+y^2\right)^2}\end{array}

\begin{array}{rcl}f'_z(x,y,z) &=& \dfrac{-16xz}{17x^2+y^2} \end{array}   \begin{array}{rcl} f'_{zx}(x,y,z) &=& \dfrac{-16z \cdot\left(17x^2+y^2\right)-(-16xz)\cdot 34x}{\left(17x^2+y^2\right)^2} \\\\&=& \dfrac{-272x^2z-16y^2z+544x^2z}{\left(17x^2+y^2\right)^2} \\\\&=& \dfrac{272x^2z-16y^2z}{\left(17x^2+y^2\right)^2}\end{array}

Vorgehen: Quotientenregel

    \begin{array}{rcl} f'_{zy}(x,y,z) &=& \dfrac{0\cdot\left(17x^2+y^2\right)-(-16xz)\cdot 2y}{\left(17x^2+y^2\right)^2} \\\\&=& \dfrac{32xyz}{\left(17x^2+y^2\right)^2}\end{array}

Vorgehen: Quotientenregel

    \begin{array}{rcl} f'_{zz}(x,y,z) &=& -\dfrac{16x}{17x^2+y^2}\end{array}

 

19)
\begin{array}{rcl}\mathbb{D} &=& \mathbb{R}\times\mathbb{R}\times\mathbb{R} \\\\f(x,y,z) &=& \dfrac{\pi x\sin\left(7y^5\right)}{e^{10xz+1}} \\&=& \pi x\sin\left(7y^5\right)e^{-(10xz+1)} \\&=& \pi x\sin\left(7y^5\right)e^{-10xz-1}\end{array}

Partielle Ableitungen 1. Ordnung
  Partielle Ableitungen 2. Ordnung
\begin{array}{rcl} f'_x(x,y,z) &=& \dfrac{\pi\sin\left(7y^5\right) \cdot e^{10xz+1} - \pi x\sin\left(7y^5\right) \cdot e^{10xz+1}\cdot 10z}{\left(e^{10xz+1}\right)^2} \cr \cr &=& \dfrac{\left(\pi-10\pi xz\right) \sin\left(7y^5\right) e^{10xz+1}}{\left(e^{10xz+1}\right)^2} \cr \cr&=& \dfrac{\left(\pi-10\pi xz\right) \sin\left(7y^5\right)}{e^{10xz+1}}\end{array}

Vorgehen: Quotientenregel, Kettenregel
 

\begin{array}{rcl} f'_{xx}(x,y,z) &=& \dfrac{-10\pi z\sin\left(7y^5\right)\cdot e^{10xz+1} - \left(\pi-10 \pi xz\right)\sin\left(7y^5\right)\cdot e^{10xz+1}\cdot 10z}{\left(e^{10xz+1}\right)^2} \\\\ &=& \dfrac{\left[-10\pi z-\left(\pi-10 \pi xz\right)\cdot 10z\right]\sin\left(7y^5\right)\cdot e^{10xz+1}}{\left(e^{10xz+1}\right)^2} \\\\&=& \dfrac{\left[-10\pi z-10\pi z+100\pi xz^2\right]\sin\left(7y^5\right)}{e^{10xz+1}} \\\\&=& \dfrac{\left[-20\pi z+100\pi xz^2\right]\sin\left(7y^5\right)}{e^{10xz+1}}\end{array}

Vorgehen: Quotientenregel, Kettenregel

    \begin{array}{rcl} f'_{xy}(x,y,z) &=& \dfrac{\left(\pi-10\pi xz\right) \cos\left(7y^5\right)\cdot 35y^4}{e^{10xz+1}} \\\\&=& \dfrac{\left(35\pi y^4-350\pi xy^4z\right) \cos\left(7y^5\right)}{e^{10xz+1}}\end{array}

Vorgehen: Kettenregel

    \begin{array}{rcl} f'_{xz}(x,y,z) &=& \dfrac{-10\pi x\sin\left(7y^5\right)\cdot e^{10xz+1} - \left(\pi-10 \pi xz\right)\sin\left(7y^5\right)\cdot e^{10xz+1}\cdot 10x}{\left(e^{10xz+1}\right)^2} \\\\ &=& \dfrac{\left[-10\pi x-\left(\pi-10 \pi xz\right)\cdot 10x\right]\sin\left(7y^5\right)\cdot e^{10xz+1}}{\left(e^{10xz+1}\right)^2} \\\\&=& \dfrac{\left[-10\pi x-10\pi x+100\pi x^2z\right]\sin\left(7y^5\right)}{e^{10xz+1}} \\\\&=& \dfrac{\left[-20\pi x+100\pi x^2z\right]\sin\left(7y^5\right)}{e^{10xz+1}}\end{array}

Vorgehen: Quotientenregel, Kettenregel

\begin{array}{rcl} f'_y(x,y,z) &=& \dfrac{\pi x\cos\left(7y^5\right)\cdot 35y^4}{e^{10xz+1}} \cr \cr &=& \dfrac{35\pi xy^4\cos\left(7y^5\right)}{e^{10xz+1}} \cr\cr&=& 35\pi xy^4\cos\left(7y^5\right)e^{-10xz-1}\end{array}

Vorgehen: Kettenregel
  \begin{array}{rcl} f'_{yx}(x,y,z) &=& \dfrac{35\pi y^4\cos\left(7y^5\right)\cdot e^{10xz+1} - 35\pi xy^4\cos\left(7y^5\right)\cdot e^{10xz+1}\cdot 10z}{\left(e^{10xz+1}\right)^2} \\\\&=& \dfrac{\left(35\pi y^4-35\pi xy^4\cdot 10z\right)\cos\left(7y^5\right)\cdot e^{10xz+1}}{\left(e^{10xz+1}\right)^2} \\\\&=& \dfrac{\left(35\pi y^4z-350\pi xy^4z\right)\cos\left(7y^5\right)}{e^{10xz+1}}\end{array}

Vorgehen: Quotientenregel, Kettenregel

    \begin{array}{rcl} f'_{yy}(x,y,z) &=& \dfrac{140\pi xy^3\cdot\cos\left(7y^5\right) + 35\pi xy^4\left(-\sin\left(7x^5\right)\right)\cdot 35y^4}{e^{10xz+1}} \\\\&=& \dfrac{140\pi xy^3\cos\left(7y^5\right)-1.225\pi xy^8\sin\left(7x^5\right)}{e^{10xz+1}}\end{array}

Vorgehen: Produktregel, Kettenregel

    \begin{array}{rcl} f'_{yz}(x,y,z) &=& 35\pi xy^4\cos\left(7y^5\right)e^{-10xz-1}\cdot (-10x) \\\\&=& -350\pi x^2y^4\cos\left(7y^5\right)e^{-10xz-1} \\\\&=& -\dfrac{350\pi x^2y^4\cos\left(7y^5\right)}{e^{10xz+1}}\end{array}

Vorgehen: Kettenregel

\begin{array}{rcl} f'_z(x,y,z) &=& \pi x\sin\left(7y^5\right)e^{-10xz-1}\cdot (-10x) \\\\&=& -10\pi x^2\sin\left(7y^5\right)e^{-10xz-1} \\\\&=& -\dfrac{10\pi x^2\sin\left(7y^5\right)}{e^{10xz+1}}\end{array}

Vorgehen: Kettenregel
  \begin{array}{rcl} f'_{zx}(x,y,z) &=& -\dfrac{20\pi x\sin\left(7y^5\right)\cdot e^{10xz+1} - 10\pi x^2\sin\left(7y^5\right)e^{10xz+1}\cdot 10z}{\left(e^{10xz+1}\right)^2} \\\\&=& -\dfrac{\left(20\pi x-10\pi x^2\cdot 10z\right)\sin\left(7y^5\right)e^{10xz+1}}{\left(e^{10xz+1}\right)^2} \\\\&=& \dfrac{\left(100\pi x^2z-20\pi x\right)\sin\left(7y^5\right)}{e^{10xz+1}}\end{array}

Vorgehen: Quotientenregel, Kettenregel

    \begin{array}{rcl} f'_{zy}(x,y,z) &=& -\dfrac{10\pi x^2\cos\left(7y^5\right)\cdot 35y^4}{e^{10xz+1}} \\\\&=& -\dfrac{350\pi x^2y^4\cos\left(7y^5\right)}{e^{10xz+1}}\end{array}

Vorgehen: Kettenregel

    \begin{array}{rcl} f'_{zz}(x,y,z) &=& -10\pi x^2\sin\left(7y^5\right)e^{-10xz-1}\cdot (-10x) \\\\&=& 100\pi x^3\sin\left(7y^5\right)e^{-10xz-1} \\\\&=& \dfrac{100\pi x^3\sin\left(7y^5\right)}{e^{10xz+1}}\end{array}

Vorgehen: Kettenregel

 

20)
\begin{array}{rcl} \mathbb{D} &=& \mathbb{R}\times\mathbb{R}\times\mathbb{R} \text{ mit } 16a^2-b^2+25c^2>0 \\\\f(a,b,c) &=& \dfrac{16a^2-b^2+25c^2}{\sqrt{16a^2-b^2+25c^2}} \\\\ &=& \sqrt{16a^2-b^2+25c^2} \\\\ &=& \left(16a^2-b^2+25c^2\right)^{\frac{1}{2}}\end{array}

Partielle Ableitungen 1. Ordnung
  Partielle Ableitungen 2. Ordnung
\begin{array}{rcl} f'_a(a,b,c) &=& \dfrac{1}{2}\left(16a^2-b^2+25c^2\right)^{\frac{1}{2}-1} \cdot 32a \\\\&=& 16a\left(16a^2-b^2+25c^2\right)^{-\frac{1}{2}} \\\\&=& \dfrac{16a}{\sqrt{16a^2-b^2+25c^2}}\end{array}

Vorgehen: Kettenregel
  \begin{array}{rcl} f'_{aa}(a,b,c) &=& 16\cdot\left(16a^2-b^2+25c^2\right)^{-\frac{1}{2}}+16a\cdot\left(-\dfrac{1}{2}\right)\left(16a^2-b^2+25c^2\right)^{-\frac{1}{2}-1}\cdot 32a \\\\&=& 16\left(16a^2-b^2+25c^2\right)^{-\frac{1}{2}}-256a^2\left(16a^2-b^2+25c^2\right)^{-\frac{3}{2}} \\\\&=& \left[16\left(16a^2-b^2+25c^2\right)-256a^2\right] \left(16a^2-b^2+25c^2\right)^{-\frac{3}{2}} \\\\&=& \left[256a^2-16b^2+400c^2-256a^2\right]\left(16a^2-b^2+25c^2\right)^{-\frac{3}{2}} \\\\&=& \left[-16b^2+400c^2\right]\left(16a^2-b^2+25c^2\right)^{-\frac{3}{2}} \\\\&=& \dfrac{-16b^2+400c^2}{\left(16a^2-b^2+25c^2\right)^{\frac{3}{2}}} \\\\&=& \dfrac{-16b^2+400c^2}{\left(\sqrt{16a^2-b^2+25c^2}\right)^3}\end{array}

Vorgehen: Produktregel, Kettenregel

 

  \begin{array}{rcl} f'_{ab}(a,b,c) &=& 16a\cdot\left(-\dfrac{1}{2}\right)\left(16a^2-b^2+25c^2\right)^{-\frac{1}{2}-1}\cdot (-2b) \\\\&=& 16ab\left(16a^2-b^2+25c^2\right)^{-\frac{3}{2}} \\\\&=& \dfrac{16ab}{\left(16a^2-b^2+25c^2\right)^{\frac{3}{2}}} \\\\&=& \dfrac{16ab}{\left(\sqrt{16a^2-b^2+25c^2}\right)^3}\end{array}

Vorgehen: Kettenregel

    \begin{array}{rcl} f'_{ac}(a,b,c) &=& 16a\cdot\left(-\dfrac{1}{2}\right)\left(16a^2-b^2+25c^2\right)^{-\frac{1}{2}-1}\cdot 50c \\\\&=& -400ac\left(16a^2-b^2+25c^2\right)^{-\frac{3}{2}} \\\\&=& -\dfrac{400ac}{\left(16a^2-b^2+25c^2\right)^{\frac{3}{2}}} \\\\&=& -\dfrac{400ac}{\left(\sqrt{16a^2-b^2+25c^2}\right)^3}\end{array}

Vorgehen: Kettenregel

\begin{array}{rcl} f'_b(a,b,c) &=& \dfrac{1}{2}\left(16a^2-b^2+25c^2\right)^{\frac{1}{2}-1} \cdot (-2b) \\\\&=& -b\left(16a^2-b^2+25c^2\right)^{-\frac{1}{2}} \\\\&=& -\dfrac{b}{\sqrt{16a^2-b^2+25c^2}}\end{array}

Vorgehen: Kettenregel
  \begin{array}{rcl} f'_{ba}(a,b,c) &=& -b\cdot\left(-\dfrac{1}{2}\right)\left(16a^2-b^2+25c^2\right)^{-\frac{1}{2}-1}\cdot 32a \\\\ &=& 16ab\left(16a^2-b^2+25c^2\right)^{-\frac{3}{2}} \\\\&=& \dfrac{16ab}{\left(16a^2-b^2+25c^2\right)^{\frac{3}{2}}} \\\\&=& \dfrac{16ab}{\left(\sqrt{16a^2-b^2+25c^2}\right)^3}\end{array}

Vorgehen: Kettenregel

 

  \begin{array}{rcll} f'_{bb}(a,b,c) &=& -1\cdot\left(16a^2-b^2+25c^2\right)^{-\frac{1}{2}}-b\cdot\left(-\dfrac{1}{2}\right)\left(16a^2-b^2+25c^2\right)^{-\frac{1}{2}-1}\cdot (-2b) \\\\&=& -\left(16a^2-b^2+25c^2\right)^{-\frac{1}{2}}-b^2\left(16a^2-b^2+25c^2\right)^{-\frac{3}{2}} \\\\&=& \left[-\left(16a^2-b^2+25c^2\right)-b^2\right] \left(16a^2-b^2+25c^2\right)^{-\frac{3}{2}} \\\\&=& \left[-16a^2+b^2-25c^2-b^2\right] \left(16a^2-b^2+25c^2\right)^{-\frac{3}{2}} \\\\&=& \left[-16a^2-25c^2\right] \left(16a^2-b^2+25c^2\right)^{-\frac{3}{2}} \\\\&=& -\dfrac{16a^2+25c^2}{\left(16a^2-b^2+25c^2\right)^{\frac{3}{2}}} \\\\&=& -\dfrac{16a^2+25c^2}{\left(\sqrt{16a^2-b^2+25c^2}\right)^3}\end{array}

Vorgehen: Produktregel, Kettenregel

 

  \begin{array}{rcl} f'_{bc}(a,b,c) &=& -b\cdot\left(-\dfrac{1}{2}\right)\left(16a^2-b^2+25c^2\right)^{-\frac{1}{2}-1}\cdot 50c \\\\ &=& 25bc\left(16a^2-b^2+25c^2\right)^{-\frac{3}{2}} \\\\&=& \dfrac{25bc}{\left(16a^2-b^2+25c^2\right)^{\frac{3}{2}}} \\\\&=& \dfrac{25bc}{\left(\sqrt{16a^2-b^2+25c^2}\right)^3}\end{array}

Vorgehen: Kettenregel

\begin{array}{rcl} f'_c(a,b,c) &=& \dfrac{1}{2}\left(16a^2-b^2+25c^2\right)^{\frac{1}{2}-1} \cdot 50c \\\\&=& 25c\left(16a^2-b^2+25c^2\right)^{-\frac{1}{2}} \\\\&=& \dfrac{25c}{\sqrt{16a^2-b^2+25c^2}}\end{array}

Vorgehen: Kettenregel
  \begin{array}{rcl} f'_{ca}(a,b,c) &=& 25c\cdot\left(-\dfrac{1}{2}\right)\left(16a^2-b^2+25c^2\right)^{-\frac{1}{2}-1}\cdot 32a \\\\&=& -400ac\left(16a^2-b^2+25c^2\right)^{-\frac{3}{2}-1} \\\\&=& -\dfrac{400ac}{\left(16a^2-b^2+25c^2\right)^{\frac{3}{2}}} \\\\&=& -\dfrac{400ac}{\left(\sqrt{16a^2-b^2+25c^2}\right)^3}\end{array}

Vorgehen: Kettenregel

    \begin{array}{rcl} f'_{cb}(a,b,c) &=& 25c\cdot\left(-\dfrac{1}{2}\right)\left(16a^2-b^2+25c^2\right)^{-\frac{1}{2}-1}\cdot (-2b) \\\\&=& 25bc\left(16a^2-b^2+25c^2\right)^{-\frac{3}{2}-1} \\\\&=& \dfrac{25bc}{\left(16a^2-b^2+25c^2\right)^{\frac{3}{2}}} \\\\&=& \dfrac{25bc}{\left(\sqrt{16a^2-b^2+25c^2}\right)^3}\end{array}

Vorgehen: Kettenregel

    \begin{array}{rcl} f'_{cc}(a,b,c) &=& 25\cdot\left(16a^2-b^2+25c^2\right)^{-\frac{1}{2}}+25c\cdot\left(-\dfrac{1}{2}\right)\left(16a^2-b^2+25c^2\right)^{-\frac{1}{2}-1}\cdot 50c \\\\&=& 25\left(16a^2-b^2+25c^2\right)^{-\frac{1}{2}}-625c^2\left(16a^2-b^2+25c^2\right)^{-\frac{3}{2}} \\\\&=& \left[25\left(16a^2-b^2+25c^2\right)-625c^2\right] \left(16a^2-b^2+25c^2\right)^{-\frac{3}{2}} \\\\&=& \left[400a^2-25b^2+625c^2-625c^2\right] \left(16a^2-b^2+25c^2\right)^{-\frac{3}{2}} \\\\&=& \left[400a^2-25b^2\right] \left(16a^2-b^2+25c^2\right)^{-\frac{3}{2}} \\\\&=& \dfrac{400a^2-25b^2}{\left(16a^2-b^2+25c^2\right)^{\frac{3}{2}}} \\\\&=& \dfrac{400a^2-25b^2}{\left(\sqrt{16a^2-b^2+25c^2}\right)^3}\end{array}

Vorgehen: Produktregel, Kettenregel


Bemerkung: Wenn Sie die partiellen Ableitungen 2. Ordnung lieber mit der Quotientenregel (statt der Produktregel) berechnen möchte, können Sie das natürlich auch tun. Die Kettenregel braucht man in jedem Fall.

23. Integrale - Lernziele und typische Fehler

Nach Durcharbeiten dieses Kapitels sollten Sie folgende Lernziele erreicht haben:

  • Sie erkennen die Notwendigkeit der Integralrechnung und können erklären, was Integrale anschaulich bedeuten.
  • Sie kennen die Grundbegriffe der Integralrechnung.
  • Sie verstehen den Zusammenhang von Integral- und Differenzialrechnung.
  • Sie können bestimmte von unbestimmten Integralen unterscheiden.
  • Sie können den Hauptsatz der Differenzial- und Integralrechnung benennen.
  • Sie kennen die wichtigsten Grund-/Stammintegrale.
  • Sie können die grundlegenden Integrationsregeln und -methoden anwenden.
  • Sie können einfache (bestimmte und unbestimmte) Integrale berechnen.
  • Sie können Integrale nutzen, um Flächen zwischen Polynomfunktionen zu bestimmen.


In diesem Kapitel sind bislang keine typischen Fehler aufgefallen.


Für Online-Selbsttests zu diesem Thema und weitere Informationen zur Mathematikunterstützung an der TH Wildau nutzen Sie bitte den Moodle-Kursraum "SOS Mathematik - Integralrechnung".

Übersicht:

 

23.1 Integrale - Aufgaben

1. Aufgabe

Bestimmen Sie jeweils die Stammfunktion!

1) f(x)=3x^3-2x^2+x

  11)  f(\xi)= \xi^x

2) f(x)=-x^2+x-1   12)  f(y)=4e^{2y}
3) f(z)=\dfrac{1}{2}z+10   13)  f(x)=\dfrac{x}{\cos^2(x)}
4) f(x)=4x^3-3x^2+2x-1   14) f(x)=\sin\left(14x+\pi\right)
5) f(x)=10x^5-\dfrac{8}{3}x^3+x-7   15)  f(t)=\sqrt{3t+1}
6) f(x)=\sqrt{x}+2   16)  f(x)=x^2 \cdot e^x
7) f(x)=18\sqrt[5]{x}   17) f(x)=13x\ln(x)
8) f(x)=\dfrac{1}{x^3}   18) g(s)=\dfrac{1}{\tan(s)}
9) g(x)=\dfrac{10}{x^{11}}   19) f(x)=\cos^3(\pi)
10) f(x)=\dfrac{1}{\sqrt{x}}   20) f(x)= \left(8x^3-9x^2\right)\sin(5x)

 

2. Aufgabe

Bestimmen Sie die folgenden unbestimmten Integrale!

1) \displaystyle\int ax^{a+b} \, dx+\int bx^{a+b} \, dx+\int x^{a+b} \, dx


2) \displaystyle\int (yz^4+y^2z^2+y^4z) \, dy


3) \displaystyle\int (yz^4+y^2z^2+y^4z) \, dz

 

3. Aufgabe

Bestimmen Sie unter allen möglichen Stammfunktionen jeweils diejenige, die durch den angegebenen Punkt verläuft!
1) f(x)=x^2-9x+1 mit P(1\mid 1)
 
2) f(x)=\dfrac{x^3}{6}+x-\dfrac{1}{2} mit P(0 \mid 12)
 
3) f(x)=\dfrac{18}{\sqrt[4]{x}} mit P(4\mid 0)
 
4) f(x)=\dfrac{e^x}{2} mit P(1 \mid 2e)
 
5) f(x)=2x-\cos(x) mit P(\pi \mid \pi^2)

 

4. Aufgabe

Berechnen Sie die folgenden bestimmten Integrale!

1)  \displaystyle\int \limits_2^3 2x \, dx   11)  \displaystyle\int \limits_{-\frac{\pi}{2}}^{\frac{3\pi}{2}} \cos(x) \, dx
2)  \displaystyle\int \limits_{-1}^0 x^7 \, dx   12)  \displaystyle\int \limits_0^{\frac{\pi}{2}} \cos\left(3x- \dfrac{\pi}{8}\right) \, dx
3)  \displaystyle\int \limits_0^4 \left( 2x - 3 \right) \, dx   13)  \displaystyle\int \limits_0^2 \dfrac{1}{2}\left(x^2-e^x\right) \, dx
4)  \displaystyle\int \limits_0^4 \left(3x^3+2x^2+1\right) \, dx   14)  \displaystyle\int \limits_{-2}^1 e^{-12x+5} \, dx
5)  \displaystyle\int \limits_{-3}^3 \left( 2z+z^3 \right) \, dz   15)  \displaystyle\int \limits_0^{\frac{\pi}{4}} \tan(x) \, dx
6)  \displaystyle\int \limits_1^2 \left(x+\dfrac{1}{x^2}\right) \, dt   16)  \displaystyle\int \limits_1^8 \dfrac{5(x-7)^2}{\left(25x^4-350x^3+1.225x^2\right)\sqrt[3]{x}} \, dx
7)  \displaystyle\int \limits_1^2 \left(x+\dfrac{1}{x^2}\right) \, dx   17)  \displaystyle\int \limits_1^{11} \dfrac{3x^2}{2}\ln(x) \, dx
8)  \displaystyle\int \limits_0^{10} \left( 5\sqrt{x}+4x \right) \, dx   18)  \displaystyle\int \limits_2^4 42^y \, dy
9)  \displaystyle\int \limits_2^5 \left( \dfrac{3}{8y^4} + \dfrac{12}{y^3} - \dfrac{7}{6y^2} \right) \, dy   19)  \displaystyle\int \limits_4^6 \dfrac{x-3}{x^2-6x+9} \, dx
10)  \displaystyle\int \limits_4^{10} \left( \dfrac{-8}{\sqrt[3]{t^2}} + 1 \right) \, dt   20)  \displaystyle\int \limits_1^3 \left(\sin(u)+3x^2-e^{-3t}+\dfrac{1}{x}\right) \, dx

 

5. Aufgabe

Bestimmen Sie den Parameter t so, dass die folgenden Gleichungen stimmen!

1) \displaystyle\int \limits_0^t x^2 \, dx = 72


2) \displaystyle\int \limits_\frac{1}{2}^t \left(\dfrac{1}{x^3}-\dfrac{1}{x^2} \right) \, dx = 0


3) \displaystyle\int \limits_t^{10} 3\sqrt{x} \, dx = \sqrt{1000}


4) \displaystyle\int \limits_t^{2t} \left(6x-1\right) \, dx = 0


5) \displaystyle\int \limits_0^5 \left(3t x^2+2\right) \, dx = 35


6) \displaystyle\int \limits_{-2}^2 \left(-\dfrac{1}{4}x+t\right) \, dx = 200

 

6. Aufgabe

Bestimmen Sie jeweils die Fläche, die von den beiden Graphen eingeschlossen wird!

1) f(x)=x^2 und g(x)=-x^2+2x+4


2) f(x)=x^3-x+2 und g(x)= -x^3+x+2


3)[1] Für Profis: f(x)=-\dfrac{x^2}{t}+t und g(x)=-t x^2+t^3 mit 0 < t \leq 1
Zusatzfrage: Für welchen Wert / welche Werte von t wird der Flächeninhalt maximal?


[1] entnommen aus Wörle, Karl; Kratz, Johannes; Keil, Karl-August (1975): Infinitesimalrechnung. München (S. 183).

Dieses Kapitel enthält die folgenden Themen:

 

23.2 Integrale - Erklärungen

Mathematischer Hinweis vorab: Wie schon der Begriff "Ableitung" würde die Betrachtung von Integralen die Verwendung von Grenzwerten erfordern. Darauf wird im Rahmen dieses Brückenkurses aber verzichtet, da hier der Umgang mit und das Berechnen von (einfachen) Integralen im Mittelpunkt stehen und ein grobes Verständnis der Hintergründe dafür erstmal ausreicht.

Organisatorischer Hinweis vorab: Natürlich werden in diesem Kapitel auch Beispiele durchgerechnet. Sie finden diese - im Gegensatz zu den anderen Kapiteln - aber am Ende der Seite, da hier erst relativ viel Theorie bekannt sein muss, damit die Beispiele verstanden werden können.

 

Eine kleine Anekdote zu Beginn: Auch bevor die Integralrechnung "erfunden" wurde, mussten Flächeninhalte berechnet werden, für die die Formeln der Geometrie nicht ausreichten. Die damalige Lösung der Wahl war, diese Flächen extrem ordentlich zu zeichnen und auszuschneiden. Anschließend wurde das ausgeschnittene Papier gewogen. Aus dem gleichen Papier wurde dann ein Rechteck (dessen Flächeninhalt ja problemlos berechnet werden kann) ausgeschnitten und ebenfalls gewogen. Über den Dreisatz konnte man dann die unbekannte Fläche zumindest näherungsweise ermitteln.
Ein anderer Weg war, die Flächeninhalte unter Funktionen durch mehrere Rechtecksflächen, die man addierte, anzunähern. Man stellte fest: Je mehr solcher Rechtecke man verwendete, desto kleiner wurde der Unterschied zwischen eigentlicher Fläche und Summe der Rechtecksflächen. Das ist auch der Ursprung des Integralzeichens: Es steht als stilisiertes S für "Summe".

 

Allgemeines zu Integralen

Zunächst ein paar "Vokabeln":

Die Berechnung von Integralen nennt man Integration.

In \displaystyle\int\limits_a^b f(x) \, dx (gesprochen: "Integral von a bis b über f von x dx" oder "Integral von a bis b über f von x nach dx") heißt / heißen

  • \int das Integralzeichen.
  • die Zahlen a und b Integrationsgrenzen. Genauer: a ist die untere Integrationgrenze, b die obere. In unserem Fall handelt es sich um reelle Zahlen, später können die Integrationsgrenzen auch aus anderen Zahlenbereichen stammen.
  • die zu integrierende Funktion f Integrand.
  • die Variable x Integrationsvariable. Ist (wie hier) die Integrationsvariable x, spricht man auch von der Integration über x. Natürlich kann man statt x auch andere Integrationsvariablen verwenden. Man sollte dann nur aufpassen, dass der Name nicht schon an andere Stelle (z. B. bei den Integrationsgrenzen) verwendet wird. Sonst wird die ganze Sache mehrdeutig ...
  • das dx Differenzial. Mit diesem Begriff muss man ein bisschen vorsichtig sein, da er auch in anderen Kontexten und z. T. mit anderer Bedeutung verwendet wird. In diesem Zusammenhang ist es ein symbolischer Bestandteil der Notation, der angibt, über welche Variable integriert wird. Dies ist z. B. dann wichtig, wenn Parameter in der Funktionsgleichung enthalten sind.


Wichtig: Jede auf einem abgeschlossenen Intervall [a ; b] stetige Funktion ist integrierbar.
Der Begriff "Stetigkeit" wurde im Rahmen dieses Lernmoduls nicht weiter definiert. Für uns reicht hier das "Alltagsverständnis" von Stetigkeit, nämlich dass die Funktion ohne abzusetzen von a bis b durchgezeichnet werden kann. Sie darf (sehr unmathematisch formuliert) also keine Unterbrechungen, Lücken, Sprünge o. ä. haben. "Knicke" wie bei der Betragsfunktion sind hingegen kein Problem. Zusätzlich ist wichtig, dass es sich um ein abgeschlossenes Intervall handelt; offene oder halboffene Intervalle sind hier nicht möglich.
Der Satz klingt (leider) besser als er ist: Er besagt nämlich nur, dass es geht – aber nicht wie…

Die Berechnung von Integralen ist - im Gegensatz zum Ableiten - auch schon bei relativ einfachen Funktionen nicht immer elementar möglich - und selbst wenn, ist sie nicht immer einfach, u. a. weil kein allgemeingültiger Algorithmus, also kein "Rezept", dafür existiert. Um Integrale zu bestimmen, muss man aus einer Vielzahl von möglichen Wegen den finden, der zum Ziel führt. Mögliche Wege sind die Anwendung der Integrationsregeln (Ähnliches kennt man vom Ableiten), Rückführen der gesuchten Integrale auf Grundintegrale, Nachschlagen in Integraltafeln oder die Nutzung spezieller Software. Häufig hilft trotzdem nur geschicktes Raten oder eine auf den ersten Blick eher abwegige Idee. Daher gilt: Mehr noch als bei anderen Themen ist hier viel, viel Üben nötig ...

 

Das unbestimmte Integral - Was ist das?

Definition: Gilt F'(x)=f(x), so ist F(x) eine Stammfunktion der Funktion f(x).
Die Formulierung "eine Stammfunktion" deutet schon darauf hin, dass es zu einer Funktion viele Stammfunktionen gibt. Die Menge aller Stammfunktionen nennt man unbestimmtes Integral der Funktion f(x) und schreibt \displaystyle\int f(x) \, dx = F(x) + c mit der Konstanten c\in\mathbb{R}.
Man sieht an dieser Definition, dass das Integrieren die Umkehrung des Ableitens/Differenzierens ist. Sie haben in diesem Lernmodul ja schon an verschiedenen Stellen kennengelernt, dass in der Mathematik gerne Operationen durch eine Umkehroperation "rückgängig" gemacht werden können - so auch hier ...

c ist die sogenannte Integrationskonstante. Sie ist deswegen wichtig, weil zwei Funktionen, die sich nur um eine additive Konstante unterscheiden, die gleiche Ableitung haben. Anders formuliert: Beim Ableiten fällt der konstante Term weg. Bei der Umkehrung des Ableitens kann dann nicht mehr einfach so bestimmt werden, welche Konstante in der Ursprungsfunktion möglicherweise enthalten war. Da die Konstante in inhaltlichen Zusammenhängen aber wichtig werden kann, darf man sie nicht einfach "unter den Tisch fallen lassen". Also nimmt man eine allgemeine Konstante c\in\mathbb{R}. Ggf. kann man sie später noch konkret bestimmen.

Bemerkung: So wie man die Ableitung einer Funktion mit einem hochgestellten Strich an der Funktionsbezeichnung kennzeichnet (die Ableitung von f(x) heißt üblicherweise f'(x) ), nennt man Stammfunktionen üblicherweise F(x) (gesprochen: "groß f von x"), wenn die Funktion f(x) heißt. Hält man sich an diese Konvention, wird bereits durch die Bezeichnungen deutlich, wie die Funktionen zusammenhängen.


Berechnung: Zur Berechnung unbestimmter Integrale nutzt man die Grund- bzw. Stammintegrale sowie die Integrationsmethoden, die weiter unten in diesem Kapitel erklärt werden.


Ganz wichtig: Die Stammfunktion / das unbestimmte Integral einer Funktion ist wieder eine Funktion bzw. eine Menge von Funktionen!
 

Das bestimmte Integral - Was ist das?

Die Integralrechnung kann dazu genutzt werden, Flächen zu berechnen. Das ist ja wahrscheinlich genau das, woran die meisten denken, wenn sie das Wort "Integral" hören. Die Bedeutung der Integralrechnung geht aber weit über diesen Bereich hinaus.
Bleibt man bei der Flächenberechnung, kann man noch etwas genauer sagen: Man verwendet die Integralrechnung, um krummlinig begrenzte Flächen (in unserem Fall: der Ebene) zu berechnen. Gradlinig begrenzte Flächen der Ebene, z. B. von Quadraten, Rauten oder Dreiecken, können mithilfe der Geometrie bestimmt werden. Bei einigen krummlinig begrenzten Flächen, wie vom Kreis oder von Kreissegmenten, funktioniert dies ebenfalls. Werden die Flächen komplizierter, gibt es keine Formeln mehr dafür, sodass die Integralrechnung zum Zuge kommt. Klassischerweise berechnet man dabei Flächen, die von einer Funktion f(x), der x-Achse und zwei Parallelen zur y-Achse durch a und b begrenzt werden. Geschrieben wird dies: A=\displaystyle\int\limits_a^b f(x)\,dx. Grafisch sieht das so aus:

Beispiel bestimmtes Integral


Im Unterschied zum "geometrischen" Weg liefert die Integralrechnung allerdings einen sogenannten orientierten Flächeninhalt. Das bedeutet, dass Flächen, die unterhalb der x-Achse liegen, negativ zählen. Das bestimmte Integral gibt also nicht direkt den Flächeninhalt zwischen der Funktion und der x-Achse bezogen auf ein bestimmtes, abgeschlossenes Intervall an, sondern eine Art Flächenbilanz, also die Summe aus den positiv gezählten Flächeninhalten oberhalb der x-Achse - in der folgenden Grafik A_{pos} - und den negativ gezählten Flächeninhalten unterhalb der x-Achse - in der folgenden Grafik A_{neg}:

Beispiel bestimmtes Integral mit positiven und negativen Flächen


Berechnung: Man berechnet ein solches bestimmtes Integral, indem man eine Stammfunktion F(x) von f(x) berechnet und die Differenz F(b)-F(a) bildet. Man schreibt: \displaystyle\int \limits_a^bf(x) \, dx = \left[ F(x) \right]_a^b = F(b)-F(a)
Merksatz: "Funktionswert von F(x) an der oberen Grenze minus Funktionswert von F(x) an der unteren Grenze"
Dieser Zusammenhang beruht auf einem der wichtigsten Sätze zu diesem Thema, nämlich dem Fundamentalsatz der Analysis, auch Hauptsatz der Differenzial- und Integralrechnung genannt.


Ganz wichtig: Das bestimmte Integral einer Funktion ist eine Zahl!


Einige Eigenschaften des bestimmten Integrals

  • Sind die obere und die untere Integrationsgrenze identisch, hat das bestimmte Integral den Wert 0: \displaystyle\int\limits_a^af(x) \, dx=0

  • Das bestimmte Integral über das ganze Intervall ist gleich der Summe der bestimmten Integrale über Teilintervalle (Achten Sie auf die Integrationsgrenzen!): \displaystyle\int\limits_a^cf(x) \, dx=\displaystyle\int\limits_a^bf(x) \, dx+\displaystyle\int\limits_b^cf(x) \, dx

  • Ist im ganzen Integrationsintervall f(x), so gilt: \displaystyle\int\limits_a^bf(x) \, dx

  • Vertauscht man die Integrationsgrenzen, wechselt das bestimmte Integral das Vorzeichen: \displaystyle\int\limits_a^b f(x) \, dx=-\displaystyle\int\limits_b^af(x) \, dx

 

Grund- oder Stammintegrale

Als Grund- oder Stammintegrale bezeichnet man die (unbestimmten) Integrale einiger häufig vorkommender Funktionen. Bitte beachten Sie jeweils die Definitionsbereiche der Funktionen sowie ggf. weitere Einschränkungen!
Die Integrationskonstante sei jeweils c\in\mathbb{R}.

 

Polynomfunktionen

Für Polynomfunktionen (auch ganz rationale Funktionen genannt) gilt:
\displaystyle\int x^n \, dx=\dfrac{x^{n+1}}{n+1}+c mit n\in\mathbb{Z}\backslash_{\{-1\}}
Bemerkung: Die Integrationsregel ist hier nur für eine sehr einfache ganz rationale Funktion, auch Potenzfunktion genannt, aufgeführt. Mithilfe der Summen- und Faktorregel (siehe unten) funktioniert diese Regel auch für allgemeine Polynomfunktionen.

Mithilfe der Potenzgesetze kann diese Regel auch für Wurzelfunktionen und einfache gebrochen rationale Funktionen angewendet werden und gilt nämlich:
\displaystyle\int \sqrt[n]{x} \, dx = \displaystyle\int x^\frac{1}{n} \, dx = \dfrac{x^{\frac{1}{n}+1}}{\frac{1}{n}+1} = \dfrac{n}{n+1}x^\frac{1+n}{n}+c
und
\displaystyle\int \dfrac{1}{x^n} \, dx = \displaystyle\int x^{-n} \, dx = \dfrac{x^{-n+1}}{-n+1}+c
Anders formuliert: Die Potenzregel gilt für alle Exponenten aus dem Zahlenbereich \mathbb{Q}\backslash_{\{-1\}}.

Achtung: Diese Regel gilt nicht, wenn der Exponent -1 ist, also für die Funktion f(x) = \dfrac{1}{x}, da n+1 in diesem Fall nämlich 0 ist und durch 0 bekanntermaßen nicht geteilt werden darf. Hier gilt (etwas überraschend):
\displaystyle\int\dfrac{1}{x} \, dx=\ln\vert x \vert+c
Bemerkung: Dies ist ein gutes Beispiel dafür, dass sich Integrale manchmal anders "verhalten", als man hätte denken können, und damit, warum hier viel Übung und Erfahrung nötig ist. Der Gedanke, dass das Integral einer Polynomfunktion wieder ein Polynom ist, ist ja durchaus nicht abwegig. In den meisten Fällen stimmt das auch - aber eben nicht immer ...

 

Exponential- und Logarithmusfunktionen

Für Exponentialfunktionen gilt:
\displaystyle\int e^x \, dx=e^x+c

\displaystyle\int a^x \, dx=\dfrac{a^x}{\ln(a)}+c

Für Logarithmusfunktionen gilt:
\displaystyle\int \ln(x) \, dx=x\ln(x)-x+c

\displaystyle\int\log_a(x) \, dx=\dfrac{1}{\ln(a)}\left(x\ln(x)-x\right)+c

 

Trigonometrische Funktionen

Für die Sinusfunktion gilt
\displaystyle\int\sin(x) \, dx=-\cos(x)+c

Für die Kosinusfunktion gilt
\displaystyle\int\cos(x) \, dx=\sin(x)+c

Für die Tangensfunktion gilt
\displaystyle\int\tan(x) \, dx=-\ln\left(\left|\cos(x)\right|\right)+c

 

Integrationsregeln und -methoden

Bei einer komplexeren Funktion gibt es zwei Ansätze für die Integration: Entweder die Funktion lässt sich durch geeignete Umformungen auf eines der Grundintegrale zurückführen oder man versucht mithilfe der hier aufgezeigten Integrationsmethoden weiterzukommen. Allerdings gibt es auch Funktionen, die sich nicht "vernünftig" integrieren lassen.

Zunächst zwei Regeln, deren Pendants Sie schon bei den Ableitungsregeln kennengelernt haben:

Faktorregel

Enthält der Integrand einen konstanten Faktor k\in\mathbb{R}, so kann dieser vor das Integral gezogen werden: \displaystyle\int k\cdot f(x) \, dx = k\cdot \int f(x) \, dx

 

Summenregel

Das Integral einer Summe zweier Funktionen ist gleich der Summe der Integrale der beiden Funktionen: \displaystyle\int\left(f(x)+g(x)\right) \, dx = \displaystyle\int f(x) \, dx+\displaystyle\int g(x) \, dx
Bemerkung: Das gilt natürlich auch für Summen mit mehr als zwei Summanden.


Sozusagen die Umkehrung der Produktregel für das Ableiten ist die

Partielle Integration

für unbestimmte Integrale: \displaystyle\int u'(x)v(x) \, dx = u(x)v(x)-\displaystyle\int u(x)v'(x) \, dx

für bestimmte Integrale: \displaystyle\int\limits_a^b u'(x)v(x) \, dx = \left[u(x)v(x)\right]_a^b-\displaystyle\int\limits_a^b u(x)v'(x) \, dx


Auch die Kettenregel hat hier eine Entsprechung, nämlich die

Integration durch Substitution

für unbestimmte Integrale: \displaystyle\int f\left(g(x)\right)\cdot g'(x) \, dx = \displaystyle\int f(z) \, dz mit z=g(x) und g'(x)=\dfrac{dz}{dx}
alternativ: \displaystyle\int f\left(g(x)\right)\cdot g'(x) \, dx = F\left(g(x)\right)+c

für bestimmte Integrale: \displaystyle\int\limits_a^b f\left(g(x)\right)\cdot g'(x) \, dx = \displaystyle\int\limits_{g(a)}^{g(b)} f(z) \, dz mit z=g(x) und g'(x)=\dfrac{dz}{dx}


Ein Sonderfall der Integration durch Substitution ist die

Logarithmische Integration

\displaystyle\int\dfrac{f'(x)}{f(x)} \, dx = \ln{\vert f(x)\vert}+c mit f(x)\neq 0 für alle x\in\mathbb{D}

 

Beispiele

Nun - endlich - ein paar Beispiele für die verschiedenen Berechnungen:

Berechnung eines bestimmten Integrals

Gesucht sei \displaystyle\int\limits_0^3 2x \, dx
Nach der Formel für das Integral von Polynomfunktionen und der Faktorregel gilt
\begin{array}{rcl} \displaystyle\int\limits_0^3 2x \, dx &=& 2\displaystyle\int\limits_0^3 x^1 \, dx \cr &=& 2\cdot\left[\dfrac{x^2}{2}\right]_0^3 \cr\cr &=& 2\left(\dfrac{3^2}{2}-\dfrac{0^2}{2}\right) \cr\cr &=& 2\cdot \dfrac{9}{2} \cr\cr &=& 9 \end{array}

Bemerkung: Man kann sich fragen, warum diese Rechnung kein c enthält. Korrekterweise ist ja F(x)=\dfrac{x^2}{2}+c. Hier die Rechnung mit c:
\begin{array}{rcl} \displaystyle\int\limits_0^3 2x \, dx &=& 2\cdot\left[\dfrac{x^2}{2}+c\right]_0^3 \cr\cr &=& 2\left(\left(\dfrac{3^2}{2}+c\right)-\left(\dfrac{0^2}{2}+c\right)\right) \cr\cr &=& 2\left(\dfrac{9}{2}+c-0-c\right) \cr\cr &=& 9 \end{array}
Man stellt fest, dass sich das c während der Rechnung aufhebt, da es im ersten Summanden positiv "auftaucht" und im zweiten negativ. Daher wird es in diesem Zusammenhang meist gleich weggelassen.


Alternativer Weg: Zeichnet man die Funktion in ein Koordinatensystem, erkennt man, dass die schraffierte Fläche A_D die Form eines Dreiecks hat:

Dreiecksfläche unter der Geraden

Die Fläche eines Dreiecks lässt sich aber auch ohne Integralrechnung bestimmen, nämlich
A_D=\dfrac{1}{2}gh mit g: Grundseite und h: Höhe auf die Grundseite
In diesem Fall ist g=3 und h=f(3)=6. Daraus ergibt sich A_D=\dfrac{1}{2}\cdot 3 \cdot 6=9.

Das ist natürlich kein Beweis dafür, dass die Regel für die Integration von Polynomfunktionen richtig ist (Es hätte uns aber schon verwundern müssen, wenn etwas anderes herausgekommen wäre ...). Dies soll nur als kleine Plausibilisierung der Regel dienen sowie als Erinnerung, dass es häufig nicht nur einen Lösungsweg gibt.

 

Berechnung von Flächeninhalten

Soll anstelle des bestimmten Integrals der Flächeninhalt bestimmt werden, der vom Graphen und der x-Achse in einem bestimmten Intervall eingeschlossen wird, muss man die Funktion vor dem Integrieren ein bisschen genauer anschauen. Hat die Funktion in dem Intervall nämlich Nullstellen (d. h. liegen Teile der zu berechnenden Fläche unterhalb und andere Teile oberhalb der x-Achse), unterscheidet sich der Wert des Integrals von der Größe des Flächeninhalts, weil die Flächen unterhalb der x-Achse ja negativ in den Wert des Integrals einfließen. Bei der Berechnung der Fläche als solche zählen sie aber natürlich positiv.


Ein einfaches Beispiel
Gesucht sei die Fläche, die von der x-Achse und der Funktion f(x)=x^3-6x^2+11x-2 im Intervall \left[1;4\right] eingeschlossen wird. Sie ist in der folgenden Grafik schraffiert eingezeichnet.

Fläche unter Funktion 3. Grades

Die Grafik zeigt: Die Funktionswerte sind im gesamten betrachteten Intervall größer als 0. Das bedeutet, dass die Größe des Flächeninhaltes und der Wert des bestimmten Integrals \displaystyle\int\limits_1^4 \left(x^3-6x^2+11x-2\right)\,dx gleich sind. Es gibt ja keine Flächen unterhalb der x-Achse, die negativ gezählt werden könnten.

Nach der Formel für das Integral von ganz rationalen Funktionen und die Summen- und Faktorregel gilt:
\begin{array}{rcl} \displaystyle\int\limits_1^4 \left(x^3-6x^2+11x-2\right)\,dx &=& \displaystyle\int\limits_1^4 x^3 \, dx +\displaystyle\int\limits_1^4 -6x^2\,dx+\displaystyle\int\limits_1^4 11x\, dx +\displaystyle\int\limits_1^4 -2\,dx \cr\cr &=& \displaystyle\int\limits_1^4 x^3 \, dx -6\displaystyle\int\limits_1^4 x^2\,dx+11\displaystyle\int\limits_1^4 x\, dx -2\displaystyle\int\limits_1^4 1\,dx \cr\cr &=& \left[\dfrac{1}{4}x^4-6\cdot\dfrac{1}{3}x^3+11\cdot\dfrac{1}{2}x^2-2x\right]_1^4\cr\cr &=& \left[\dfrac{1}{4}x^4-2x^3+\dfrac{11}{2}x^2-2x\right]_1^4 \cr\cr &=& \dfrac{1}{4}\cdot4^4-2\cdot4^3+\dfrac{11}{2}\cdot4^2-2\cdot4-\left(\dfrac{1}{4}-2+\dfrac{11}{2}-2\right) \cr\cr &=& \dfrac{57}{4} \end{array}
Die Fläche A ist also \frac{57}{4}=14{,}25 Flächeneinheiten groß.

Bemerkung: So ausführlich wie in dieser Beispielrechnung wird man den Lösungsweg sonst nicht aufschreiben ...


Ein etwas schwierigeres Beispiel
Gesucht sei die Fläche, die von der x-Achse und der Funktion f(x)=\dfrac{2}{5}x^2+\dfrac{4}{5}x im Intervall [-2;1] eingeschlossen wird. In der folgenden Grafik sieht man, dass die Gesamtfläche (schraffiert) in zwei Teilflächen A_1 und A_2 zerfällt.

Fläche, die von der x-Achse und einer Parabel eingeschlossen wird

Um die Größe der gesamten eingeschlossenen Fläche zu berechnen, muss nun also das Integral an der im Intervall \left[-2;1\right] liegenden Nullstelle aufgeteilt werden. Dann werden die Einzelflächen berechnet und anschließend deren Beträge addiert.

Nullstellen berechnen
\begin{array}{rcl} \dfrac{2}{5}x^2+\dfrac{4}{5}x &=& 0 \cr &...& \cr x_1 &=& -2 \cr x_2 &=& 0 \end{array}
Da die erste Nullstelle nicht innerhalb der betrachteten Intervalls liegt, ist nur die zweite Nullstelle gerade interessant.

Zu berechnen ist nun also
\begin{array}{rcl} A_1 &=& \displaystyle\int\limits_{-2}^0 \left(\dfrac{2}{5}x^2+\dfrac{4}{5}x\right)\,dx \cr\cr &=& \left[\dfrac{2}{5\cdot 3}x^3+\dfrac{4}{5\cdot 2}x^2\right]_{-2}^0 \cr\cr &=& \dfrac{2}{15}\cdot 0+\dfrac{2}{5}\cdot 0-\left(\dfrac{2}{15}\cdot\left(-2\right)^3+\dfrac{2}{5}\cdot\left(-2\right)^2\right) \cr\cr &=& -\dfrac{8}{15} \end{array}

\begin{array}{rcl} A_2 &=& \displaystyle\int\limits_{0}^1 \left(\dfrac{2}{5}x^2+\dfrac{4}{5}x\right)\,dx \cr\cr &=& \left[\dfrac{2}{5\cdot 3}x^3+\dfrac{4}{5\cdot 2}x^2\right]_0^1 \cr\cr &=& \dfrac{2}{15}\cdot 1+\dfrac{2}{5}\cdot 1-0 \cr\cr &=& \dfrac{8}{15} \end{array}

Das ergibt für die Gesamtfläche A
\begin{array}{rcl} A &=&\left| A_1\right| + \left| A_2\right| \cr\cr &=& \left| -\dfrac{8}{15} \right| +\left| \dfrac{8}{15} \right| \cr\cr &=& \dfrac{16}{15} \end{array}
Die Fläche A ist also \frac{16}{15}\approx 1{,}07 Flächeneinheiten groß.

Bemerkung: Man sieht hier sehr schön, was passieren würde, wenn man die Nullstelle oder die Betragsstriche in der Rechnung ignorieren würde. Das bestimmte Integral \displaystyle\int\limits_{-2}^1 \left(\dfrac{2}{5}x^2+\dfrac{4}{5}x\right)\,dx ist nämlich 0, was sicherlich nicht die Größe der schraffierten Fläche ist.

 

Anwendung der partiellen Integration

Gesucht sei \displaystyle\int xe^x \, dx mit \mathbb{D} = \mathbb{R}.
\begin{array}{lclcrcl} \quad u' &=& e^x \quad \Rightarrow \quad u &=& e^x \cr \quad v &=& x \quad \Rightarrow \quad v' &=& 1 \cr \cr \displaystyle\int xe^x \, dx &=& xe^x-\displaystyle\int1e^x \, dx \cr &=& xe^x-e^x+c \cr &=& (x-1)e^x+c \end{array}
mit c\in\mathbb{R}

Bemerkung 1: Die partielle Integration lohnt sich immer dann, wenn die Funktion aus mehreren Faktoren zusammengesetzt ist und einer dieser Faktoren einfach(er) integriert werden kann und das Integral, welches sich dann auf der rechten Seite ergibt, gut zu lösen ist. Deswegen wurde hier u'(x)=e^x gesetzt, weil e^x unkompliziert zu integrieren ist. Außerdem vereinfacht sich das Integral auf der rechten Seite, da beim Ableiten von v(x)=x nur eine 1 stehenbleibt.
Bemerkung 2: Bei der vorletzten Umformung wäre es auch möglich
\begin{array}{rcl} ... &=& xe^x-\left(e^x+c\right) \cr &=& xe^x-e^x-c \end{array}
zu schreiben. Da c aber ohnehin eine beliebige reelle Zahl ist, kommt es auf das Vorzeichen hier nicht an.

 

Anwendung der Integration durch Substitution

Bevor wir uns um dieses Beispiel kümmern, muss noch eine Schreibweise geklärt werden: \dfrac{df}{dx} ist eine andere Schreibweise für die Ableitung dieser Funktion f nach der Variablen x, also f'(x). Da das z, mit dem wir im Folgenden substituieren, von x abhängt, können wir z auch nach x ableiten und das Ganze als \dfrac{dz}{dx} schreiben.

Gesucht sei \displaystyle\int \sqrt{2x+1} \, dx mit \mathbb{D}=\left[-\dfrac{1}{2} ; \infty\right[.
Substituieren wir z = 2x+1, ist die Ableitung davon \dfrac{dz}{dx}=2. Durch die Substitution wird unser Integrand \sqrt{ 2x+1} nun zu \sqrt{z}. Da wir nach z integrieren wollen, muss am Ende des Integrals auch dz (und nicht dx) stehen. \dfrac{dz}{dx} = 2 lässt sich umformen zu \dfrac{dz}{2} = dx. Aus unserem Integral wird also:
\begin{array}{rcl} \displaystyle\int \sqrt{2x+1} \, dx &=& \int \sqrt{z} \, \dfrac{dz}{2} \cr \cr &=& \dfrac{1}{2}\displaystyle\int\sqrt{z} \, dz \cr \cr &=& \dfrac{1}{2}\cdot\dfrac{2}{3}\sqrt{z^3}+c \cr \cr &=& \dfrac{1}{3}\sqrt{z^3}+c \end{array}
Nun kommt noch die Rücksubstitution, denn die eigentliche Aufgabe war ja das Integral mit der Integrationsvariable x zu lösen:
\dfrac{1}{3}\sqrt{z^3}+c = \dfrac{1}{3}\sqrt{(2x+1)^3}+c

Bemerkung 1: In der Grundform wird die Integration durch Substitution wird immer dann angewendet, wenn ein Faktor des Integranden die Ableitung der inneren Funktion des anderen Faktors ist. Als Faustregel kann gesagt werden: Über die Substitution sollte man beim Integrieren immer dann nachdenken, wenn man die Kettenregel benutzen würde, um den Term abzuleiten. Ziel der Substitution ist es, den Integranden zu vereinfachen. Dazu kann es allerdings manchmal nötig sein, eine Substitution zu verwenden, die das Integral auf den ersten Blick komplizierter macht.
Bemerkung 2: In diesem Beispiel wurde "nur" ein Spezialfall der Integration durch Substitution gezeigt, nämlich die lineare Substitution. An ihr lässt sich das Prinzip gut verdeutlichen, weil sie ein gutes Stück einfacher ist als Substitutionen im Allgemeinen.

Übersicht:

 

23.3 Integrale - Lösungen

Eine Bemerkung vorab: Die Musterlösungen in diesem Kapitel sind nicht mehr so ausführlich wie in den meisten anderen Kapiteln. Insbesondere Termumformungen und das Auflösen von einfachen (z. B. linearen oder quadratischen) Gleichungen wurden weggelassen. Wer damit Probleme hat, schaue bitte in den entsprechenden Kapiteln nach.

 

1. Aufgabe

Für alle Aufgaben sei c\in \mathbb{R}.

1)
\begin{array}{rcl} \mathbb{D} &=& \mathbb{R} \cr\cr F(x) &=& \dfrac{3}{4}x^{3+1}-\dfrac{2}{3}x^{2+1}+\dfrac{1}{2}x^{1+1}+c \cr \cr &=& \dfrac{3}{4}x^4-\dfrac{2}{3}x^3+\dfrac{1}{2}x^2+c\end{array}

 
2)
\begin{array}{rcl} \mathbb{D} &=& \mathbb{R} \cr\cr F(x) &=& -\dfrac{1}{3}x^{2+1}+\dfrac{1}{2}x^{1+1}-x+c \cr \cr &=& -\dfrac{1}{3}x^3+\dfrac{1}{2}x^2-x+c \end{array}


3)
\begin{array}{rcl} \mathbb{D} &=& \mathbb{R} \cr\cr F(z) &=& \genfrac{}{}{1pt}{0}{\dfrac{1}{2}}{2}z^{1+1}+10z+c \cr \cr &=& \dfrac{1}{4}z^2+10z+c \end{array}


4)
\begin{array}{rcl} \mathbb{D} &=& \mathbb{R} \cr\cr F(x) &=& \dfrac{4}{4}x^{3+1}-\dfrac{3}{3}x^{2+1}+\dfrac{2}{2}x^{1+1}-x+c \cr \cr &=& x^4-x^3+x^2-x+c \end{array}


5)
\begin{array}{rcl} \mathbb{D} &=& \mathbb{R} \cr\cr F(x) &=& \dfrac{10}{6}x^{5+1}-\genfrac{}{}{1pt}{0}{\dfrac{8}{3}}{4}x^{3+1}+\dfrac{1}{2}x^{1+1}-7x+c \cr \cr &=& \dfrac{5}{3}x^6-\dfrac{2}{3}x^4+\dfrac{1}{2}x^2-7x+c \end{array}

 

Hinweis: Bei den folgenden Aufgaben helfen die Potenzgesetze, vor allem die Festlegungen zu gebrochenen und negativen Exponenten, weiter.


6)
\begin{array}{rcl} \mathbb{D} &=& \mathbb{R}_0^+ \cr\cr f(x) &=& x^{\frac{1}{2}} +2 \cr \cr \cr F(x) &=& \genfrac{}{}{1pt}{0}{1}{\dfrac{3}{2}}x^{\frac{1}{2}+1}+2x+c \cr \cr &=& \dfrac{2}{3}x^{\frac{3}{2}}+2x+c \cr \cr &=& \dfrac{2}{3}\sqrt{x^3}+2x+c \end{array}


7)
\begin{array}{rcl} \mathbb{D} &=& \mathbb{R} \cr\cr f(x) &=& 18 x^{\frac{1}{5}} \cr \cr \cr F(x) &=& 18\cdot\genfrac{}{}{1pt}{0}{1}{\dfrac{6}{5}}x^{\frac{1}{5}+1}+c \cr \cr &=& 18\cdot\dfrac{5}{6}x^\frac{6}{5}+c \cr \cr &=& 15\sqrt[5]{x^6}+c \end{array}


8)
\begin{array}{rcl} \mathbb{D} &=& \mathbb{R}\backslash_{\{0\}} \cr\cr g(x) &=& x^{-3} \cr \cr \cr G(x) &=& \dfrac{1}{-2}x^{-3+1}+c \cr \cr &=& -\dfrac{1}{2}x^{-2}+c \cr \cr &=& -\dfrac{1}{2x^2}+c \end{array}


9)
\begin{array}{rcl} \mathbb{D} &=& \mathbb{R}\backslash_{\{0\}} \cr\cr f(x) &=& 10x^{-11} \cr \cr \cr F(x) &=& \dfrac{10}{-10}x^{-11+1}+c \cr \cr &=& -x^{-10}+c \cr \cr &=& -\dfrac{1}{x^{10}}+c \end{array}


10)
\begin{array}{rcl} \mathbb{D} &=& \mathbb{R}^+ \cr\cr f(x) &=& x^{-\frac{1}{2}} \cr \cr \cr F(x) &=& \dfrac{1}{-\frac{1}{2}+1}x^{-\frac{1}{2}+1}+c \cr \cr &=& 2x^\frac{1}{2}+c \cr \cr &=& 2\sqrt{x}+c \end{array}


11)
\begin{array}{rcl} \mathbb{D} &=& \mathbb{R} \cr\cr F(\xi) &=& \dfrac{\xi^{x+1}}{x+1}+c \end{array}

Bemerkung: Bitte beachten Sie, dass die Variable hier nicht x, sondern \xi ist! Das bedeutet, dass die Aufgabe einfach mit der Potenzregel zu lösen ist (Die Variable steht ja in der Basis ...). \xi ist ein griechischer Buchstabe (gesprochen: "xi") - die tauchen ja immer mal wieder in der Mathematik auf ...


12)
\begin{array}{rclll} \mathbb{D} &=& \mathbb{R} \cr\cr \displaystyle\int4e^{2y} \, dy \end{array}

Substitution: t = 2y
\begin{array}{rclll} \dfrac{dt}{dy} &=& 2 \cr dy &=& \dfrac{dt}{2} \cr\cr\cr \displaystyle\int4e^{2y} \, dy &=& \displaystyle\int 4e^t\frac{dt}{2} \cr\cr &=& \dfrac{4}{2}\displaystyle\int e^t \, dt \cr\cr &=& 2e^t+c \end{array}

Rücksubstitution:
\begin{array}{rclll} \displaystyle\int4e^{2y}dy &=&2e^{2y}+c\end{array}

Vorgehen: lineare Substitution


13)
\begin{array}{rclcrcl} \mathbb{D} &=& \mathbb{R}\backslash_{\{\frac{\pi}{2}+k\pi; \; k\in\mathbb{Z}\}} \cr\cr \displaystyle\int \dfrac{x}{\cos^2(x)} \, dx &=& \displaystyle\int x\cdot\dfrac{1}{\cos^2(x)} \, dx \cr\cr \quad u' &=& \dfrac{1}{\cos^2(x)} &\Rightarrow & u &=& \tan(x) \cr v &=& x & \Rightarrow & v' &=& 1 \end{array}

\begin{array}{rcl} \displaystyle\int x\cdot \dfrac{1}{\cos^2(x)}\, dx &=& x \cdot \tan(x) - \displaystyle\int 1 \cdot \tan (x) \, dx \cr\cr &=& x \cdot \tan(x) - \left(-\ln\left|\cos(x)\right| \right)+c \cr\cr &=& x \cdot \tan(x) + \ln \left| \cos(x) \right|+c \cr \end{array}

Vorgehen: partielle Integration


14)
\begin{array}{rclll} \mathbb{D} &=& \mathbb{R} \cr\cr \displaystyle\int\sin(14x+\pi)\, dx \end{array}

Substitution: t = 14x+\pi
\begin{array}{rclll} \dfrac{dt}{dx} &=& 14 \cr dx &=& \dfrac{dt}{14} \cr\cr\cr \displaystyle\int\sin(14x+\pi)\, dx &=& \displaystyle\int\sin(t)\dfrac{dt}{14} \cr\cr &=& \dfrac{1}{14} \displaystyle\int\sin(t)\, dt \cr\cr &=& \dfrac{1}{14}\left(-\cos(x)\right)+c \end{array}

Rücksubstitution:
\begin{array}{rclll} \displaystyle\int\sin(14x+\pi)\, dx &=& -\dfrac{1}{14} \cos(14x+\pi)+c \end{array}

Vorgehen: lineare Substitution


15)
\begin{array}{rclll} \mathbb{D} &=& \left[-\frac{1}{3}; \infty\right[ \cr\cr \displaystyle\int \sqrt{3t+1} \, dt &=& \displaystyle\int (3t+1)^{\frac{1}{2}} \, dt \end{array}

Substitution: z = 3t+1
\begin{array}{rclll}\dfrac{dz}{dt} &=& 3 \cr dt &=& \dfrac{dz}{3} \cr\cr\cr \displaystyle\int (3t+1)^{\frac{1}{2}} \, dt &=& \displaystyle\int z^{\frac{1}{2}} \dfrac{dz}{3} \cr\cr &=& \dfrac{1}{3} \displaystyle\int z^{\frac{1}{2}}\, dz \cr\cr &=& \dfrac{1}{3} \cdot \dfrac{2}{3}z^{\frac{3}{2}}+c \end{array}

Rücksubstitution:
\begin{array}{rclll}\displaystyle\int \sqrt{3t+1} \, dt &=& \dfrac{2}{9}\left(3t+1 \right)^{\frac{3}{2}}+c \cr\cr &=& \dfrac{2}{9}\sqrt{(3t+1)^3}+c \end{array}

Vorgehen: Potenzgesetze und lineare Substitution


16)
\begin{array}{rclcrcl} \mathbb{D} &=& \mathbb{R} \cr\cr \displaystyle\int x^2 \cdot e^x \, dx \cr\cr \quad u' &=&e^x &\Rightarrow & u &=&e^x \cr v &=& x^2 & \Rightarrow & v' &=& 2x \end{array}

\begin{array}{rcl} \displaystyle\int x^2 \cdot e^x \, dx &=& e^x \cdot x^2 - \displaystyle\int e^x \cdot 2x \, dx \cr\cr &=& x^2 \cdot e^x - 2 \displaystyle\int x \cdot e^x \, dx \end{array}

Das Integral \displaystyle\int x \cdot e^x \, dx separat betrachtet:
\begin{array}{rclcrcl}\displaystyle\int x \cdot e^x \, dx \cr\cr \quad u' &=&e^x &\Rightarrow & u &=&e^x \cr v &=& x & \Rightarrow & v' &=& 1 \end{array}

\begin{array}{rcl} \displaystyle\int x \cdot e^x \, dx &=& e^x \cdot x - \int e^x \cdot 1 \, dx \cr\cr &=& x\cdot e^x - e^x \end{array}

Gesamtergebnis:
\begin{array}{rcl} \displaystyle\int x^2 \cdot e^x \, dx &=& x^2 \cdot e^x - 2 \int x \cdot e^x \, dx \cr\cr &=& x^2 \cdot e^x - 2 \left(x\cdot e^x - e^x\right) +c \cr\cr &=& e^x\left(x^2-2x+2 \right)+c\end{array}

Vorgehen: zwei Mal partielle Integration


17)
\begin{array}{rclcrcl} \mathbb{D} &=& \mathbb{R}^+ \cr\cr \displaystyle\int 13x\ln(x)\, dx \cr\cr \quad u' &=& 13x & \Rightarrow & u &=& \dfrac{13}{2}x^2\cr v &=& \ln(x) & \Rightarrow & v' &=& \dfrac{1}{x}\cr\end{array}

\begin{array}{rcl}\displaystyle\int 13x\ln(x)\,dx &=& \dfrac{13}{2}x^2 \cdot\ln(x)-\displaystyle\int \dfrac{13}{2} x^2 \cdot \dfrac{1}{x}\, dx \cr\cr &=& \dfrac{13}{2}x^2\ln(x)-\displaystyle\int \dfrac{13}{2} x\, dx \cr\cr &=& \dfrac{13}{2}x^2 \ln(x)-\dfrac{13}{4} x^2+c \cr\cr &=& \dfrac{13}{2}x^2 \left( \ln(x)-\dfrac{1}{2} \right)+c \end{array}

Vorgehen: partielle Integration


18)
\begin{array}{rclll} \mathbb{D} &=& \mathbb{R}\backslash_{\{k \cdot \pi; \; k\in\mathbb{Z}\}} \cr\cr \displaystyle\int \dfrac{1}{\tan(s)} \, ds &=& \displaystyle\int \dfrac{\cos(s)}{\sin(s)}\,ds \end{array}

Substitution: t = \sin(s)
\begin{array}{rclll}\dfrac{dt}{ds} &=& \cos(s) \cr ds &=& \dfrac{dt}{\cos(s)} \cr\cr\cr \displaystyle\int \dfrac{\cos(s)}{\sin(s)}\,ds &=& \displaystyle\int \dfrac{\cos(s)}{t}\cdot \dfrac{dt}{\cos(s)} \cr\cr &=& \displaystyle\int \dfrac{1}{t}\,dt\cr\cr &=& \ln\vert t\vert+c \end{array}

Rücksubstitution:
\begin{array}{rclll}\displaystyle\int \dfrac{1}{\tan(s)} \, ds &=& \ln\vert\sin(s)\vert+c \end{array}

Vorgehen: Substitution


19)
\begin{array}{rcl} \mathbb{D} &=& \mathbb{R} \cr\cr F(x) &=& \cos^3(\pi)\cdot x^{0+1} +c \cr &=& -1\cdot x+c \cr &=& -x+c \end{array}

Bemerkung: Da \cos^3(\pi)= \left(\cos(\pi)\right)^3=-1, muss hier nur eine Konstante integriert werden.


20)
\begin{array}{rclcrcl} \mathbb{D} &=& \mathbb{R} \cr\cr \displaystyle\int \left(8x^3-9x^2\right)\sin(5x) dx \cr \quad u' &=&\sin(5x) &\Rightarrow & u &=& -\dfrac{1}{5}\cos(5x) \cr v &=& 8x^3-9x^2 & \Rightarrow & v' &=& 24x^2-18x \end{array}

\begin{array}{rcl} \displaystyle\int (8x^3-9x^2)\sin(5x) dx &=& -\dfrac{1}{5}\cos(5x)\left(8x^3-9x^2\right)- \displaystyle\int -\dfrac{1}{5}\cos(5x)\left(24x^2-18x\right)\, dx \cr\cr &=& -\dfrac{1}{5}\left(8x^3-9x^2\right)\cos(5x)+\dfrac{1}{5} \displaystyle\int \left(24x^2-18x\right)\cos(5x)\, dx \end{array}

Das Integral \displaystyle\int \left(24x^2-18x\right)\cos(5x)\, dx separat betrachtet:
\begin{array}{rclcrcl}\cr\displaystyle\int \left(24x^2-18x\right)\cos(5x) \, dx \cr \quad u' &=& \cos(5x) &\Rightarrow & u &=& \dfrac{1}{5}\sin(5x) \cr v &=& 24x^2-18x & \Rightarrow & v' &=& 48x-18 \end{array}

\begin{array}{rcl} \displaystyle\int \left(24x^2-18x\right)\cos(5x) \,dx &=& \dfrac{1}{5}\sin(5x)\left(24x^2-18x\right)- \displaystyle\int \dfrac{1}{5}\sin(5x)\left(48x-18\right)\, dx \cr\cr &=& \dfrac{1}{5}\left(24x^2-18x\right)\sin(5x)- \dfrac{1}{5} \displaystyle\int \left(48x-18\right)\sin(5x)\, dx \end{array}

Nun das Integral \displaystyle\int \left(48x-18\right)\sin(5x)\, dx separat betrachtet:
\begin{array}{rclcrcl} \displaystyle\int \left(48x-18\right)\sin(5x)\, dx \cr \quad u' &=& \sin(5x) &\Rightarrow & u &=& -\dfrac{1}{5}\cos(5x)\cr v &=& 48x-18 & \Rightarrow & v' &=& 48 \end{array}

\begin{array}{rcl} \displaystyle\int \left(48x-18\right)\sin(5x)\, dx &=& -\dfrac{1}{5}\cos(5x)\left(48x-18\right)- \displaystyle\int -\dfrac{1}{5}\cos(5x)\cdot 48\, dx \cr\cr &=& -\dfrac{1}{5}\left(48x-18\right)\cos(5x) + \dfrac{48}{5} \cdot\displaystyle\int \cos(5x)\, dx \cr\cr &=&-\dfrac{1}{5}\left(48x-18\right)\cos(5x)+ \dfrac{48}{5}\cdot \dfrac{1}{5}\sin(5x)+c \cr\cr &=& -\dfrac{1}{5}\left(48x-18\right)\cos(5x) + \dfrac{48}{25} \sin(5x)+c \end{array}

Gesamtergebnis
\begin{array}{rcl}\displaystyle\int (8x^3-9x^2)\sin(5x)\,dx &=& -\dfrac{1}{5}\left(8x^3-9x^2\right)\cos(5x)+\dfrac{1}{5}\left(\dfrac{1}{5} \left(24x^2-18x\right)\sin(5x)-\dfrac{1}{5}\left(-\dfrac{1}{5}\left(48x-18\right)\cos(5x)+\dfrac{48}{25}\sin(5x)+ c \right)\right) \cr\cr &=& -\dfrac{1}{5}\left(8x^3-9x^2\right)\cos(5x)+\dfrac{1}{5}\left(\dfrac{1}{5}\left(24x^2-18x\right)\sin(5x)+\dfrac{1}{25}\left(48x-18\right)\cos(5x) -\dfrac{48}{125}\sin(5x)+ c \right) \cr\cr &=& -\dfrac{1}{5}\left(8x^3-9x^2\right)\cos(5x)+\dfrac{1}{25}\left(24x^2-18x\right)\sin(5x)+\dfrac{1}{125}\left(48x-18\right)\cos(5x) -\dfrac{48}{625}\sin(5x)+ c \cr\cr &=& \dfrac{1}{25}\left(24x^2-18x\right)\sin(5x)-\dfrac{48}{625}\sin(5x)-\dfrac{1}{5}\left(8x^3-9x^2\right)\cos(5x)+\dfrac{1}{125}\left(48x-18\right)\cos(5x)+ c \cr\cr &=& \dfrac{1}{25}\sin(5x)\left(24x^2-18x-\dfrac{48}{25}\right)+\dfrac{1}{5}\cos(5x)\left(-8x^3+9x^2+\dfrac{1}{25}\left(48x-18\right)\right)+ c \end{array}

Vorgehen: drei Mal partielle Integration

 

2. Aufgabe

Auch hier sei wieder c\in \mathbb{R}.
 
1)
\begin{array}{rcl} \displaystyle\int ax^{a+b}\,dx+\displaystyle\int bx^{a+b}\,dx+\displaystyle\int x^{a+b}\,dx &=& \displaystyle\int \left(ax^{a+b} + bx^{a+b} + x^{a+b}\right)\,dx \cr\cr &=& \displaystyle\int \left(a+b+1\right)x^{a+b}\,dx \cr\cr &=& \dfrac{a+b+1}{a+b+1}x^{a+b+1}+c \cr\cr &=& x^{a+b+1}+c\end{array}


2)
\begin{array}{rcl} \displaystyle\int (yz^4+y^2z^2+y^4z)\,dy &=& \dfrac{1}{2}y^2z^4+\dfrac{1}{3}y^3z^2+\dfrac{1}{5}y^5z+c \cr\cr &=& \dfrac{1}{5}y^5z+\dfrac{1}{3}y^3z^2+\dfrac{1}{2}y^2z^4+c \end{array}

Bemerkung: Der letzte Schritt ist nur dazu da, die Summanden in der Reihenfolge der Exponenten beim y zu sortieren.


3)
\begin{array}{rcl} \displaystyle\int (yz^4+y^2z^2+y^4z)\,dz &=& \dfrac{1}{5}yz^5+\dfrac{1}{3}y^2z^3+\dfrac{1}{2}y^4z^2+c \end{array}

Bemerkung: Die Aufgaben 2) und 3) unterscheiden sich nur in der Integrationsvariable: Bei 2) wird nach y integriert, bei 3) nach z. Die jeweils andere Variable wird als Konstante betrachtet.

 

3. Aufgabe

1)
Die Stammfunktion von f(x)=x^2-9x+1 ist F(x)=\dfrac{1}{3}x^3-\dfrac{9}{2}x^2+x+c.

Gesucht ist nun die Konstante c, die dafür sorgt, dass F(x) durch den Punkt P(1\mid 1) verläuft. Dazu müssen wir F(1)=1 nach c auflösen:

\begin{array}{rcl} \dfrac{1}{3}\cdot 1^3-\dfrac{9}{2}\cdot 1^2+1+c &=& 1 \cr c &=& \dfrac{25}{6} \end{array}

Ergebnis: Die gesuchte Funktion lautet: F(x)=\dfrac{1}{3}x^3-\dfrac{9}{2}x^2+x+\dfrac{25}{6}


2)
Die Stammfunktion von f(x)=\dfrac{x^3}{6}+x-\dfrac{1}{2} ist F(x)=\dfrac{1}{24}x^4+\dfrac{1}{2}x^2+\dfrac{1}{2}x+c.

Gesucht ist nun die Konstante c, die dafür sorgt, dass F(x) durch den Punkt P(0 \mid 12) verläuft. Dazu müssen wir F(0)=12 nach c auflösen:

\begin{array}{rcl}\dfrac{1}{24}\cdot 0^4+\dfrac{1}{2}\cdot 0^2+\dfrac{1}{2}\cdot 0+c &=& 12 \cr c &=& 12\end{array}

Ergebnis: Die gesuchte Funktion lautet: F(x)=\dfrac{1}{24}x^4+\dfrac{1}{2}x^2+\dfrac{1}{2}x+12


3)
Die Stammfunktion von f(x)=\dfrac{18}{\sqrt[4]{x}} ist F(x)=24\sqrt[4]{x^3}+c.

Gesucht ist nun die Konstante c, die dafür sorgt, dass F(x) durch den Punkt P(4\mid 0) verläuft. Dazu müssen wir F(4)=0 nach c auflösen:

\begin{array}{rcl} 24\sqrt[4]{4^3}+c &=& 0 \cr c &=& -48\sqrt{2}\end{array}

Ergebnis: Die gesuchte Funktion lautet: F(x)=24\sqrt[4]{x^3}-48\sqrt{2}


4)
Die Stammfunktion von f(x)=\dfrac{e^x}{2} ist F(x)=\dfrac{1}{2}e^x+c.

Gesucht ist nun die Konstante c, die dafür sorgt, dass F(x) durch den Punkt P(1 \mid 2e) verläuft. Dazu müssen wir F(1)=2e nach c auflösen:

\begin{array}{rcl} \dfrac{1}{2}\cdot e^1+c &=& 2e \cr c &=& \dfrac{3}{2}e \end{array}

Ergebnis: Die gesuchte Funktion lautet: F(x)=\dfrac{1}{2}e^x+\dfrac{3}{2} e


5)

Die Stammfunktion von f(x)=2x-\cos(x) ist F(x)=x^2-\sin(x)+c.

Gesucht ist nun die Konstante c, die dafür sorgt, dass F(x) durch den Punkt P(\pi \mid \pi^2) verläuft. Dazu müssen wir F(\pi)=\pi^2 nach c auflösen:

\begin{array}{rcl} \pi^2-\sin(\pi)+c &=& \pi^2 \cr c &=& 0\end{array}

Bemerkung: Aus dem Kapitel Trigonometrie weiß man, dass \sin(\pi) = 0 .

Ergebnis: Die gesuchte Funktion lautet: F(x)=x^2-\sin(x)

 

4. Aufgabe

1)
\begin{array}{rcl} \displaystyle\int \limits_2^3 2x \, dx &=& \left[ x^2\right]_2^3 \cr &=& 3^2 - 2^2 \cr &=& 5 \end{array}

2)
\begin{array}{rcl} \displaystyle\int \limits_{-1}^0 x^7 \, dx &=& \left[ \dfrac{1}{8}x^8\right]_{-1}^0 \cr &=& \dfrac{1}{8}\cdot 0^8 - \dfrac{1}{8} \cdot \left( -1 \right)^8 \cr &=& -\dfrac{1}{8} \end{array}


3)
\begin{array}{rcl} \displaystyle\int \limits_0^4 \left( 2x - 3 \right) \, dx &=& \left[ x^2 - 3x \right]_0^4 \cr &=& 4^2-3\cdot 4 - \left( 0^2-3 \cdot 0 \right) \cr &=& 4 \end{array}


4)
\begin{array}{rcl} \displaystyle\int \limits_0^4 (3x^3+2x^2+1) \, dx \, &=& \, \left[\dfrac{3}{4}x^4+\dfrac{2}{3}x^3+x\right]_0^4 \cr &=& \dfrac{3}{4} \cdot 4^4 + \dfrac{2}{3} \cdot 4^3 + 4 - \left(\dfrac{3}{4} \cdot 0^4 + \dfrac{2}{3} \cdot 0^3 + 0 \right) \cr &=& \dfrac{716}{3} \end{array}

5)
\begin{array}{rcl} \displaystyle\int \limits_{-3}^3 \left( 2z+z^3 \right) \, dz &=& \left[ \dfrac{1}{4} z^4+z^2 \right]_{-3}^3 \cr &=& \dfrac{1}{4} \cdot 3^4+3^2- \left( \dfrac{1}{4} \cdot (-3)^4 + (-3)^2 \right) \cr &=& 0 \end{array}

Bemerkung: Bitte achten Sie darauf, dass die Summanden im ersten Schritt vertauscht wurden. Der Summand mit dem höchsten Exponenten steht - wie immer - vorn.


6)
\begin{array}{rcl} \displaystyle\int \limits_1^2 \left( x+\dfrac{1}{x^2} \right) dt &=& \left( x+\dfrac{1}{x^2} \right) \int \limits_1^2 1 \, dt \cr\cr &=& \left( x+\dfrac{1}{x^2} \right) \cdot \left[ t \right]_1^2 \cr\cr &=& \left( x+\dfrac{1}{x^2} \right) \cdot \left( 2-1 \right) \cr\cr &=&\left(x+\dfrac{1}{x^2} \right) \cdot 1 \cr\cr &=& x+\dfrac{1}{x^2} \end{array}

Bemerkung: Die Integrationsvariable ist hier t und nicht x! Das heißt, x wird als Konstante, wie eine Zahl, behandelt.


7)
\begin{array}{rcl} \displaystyle\int \limits_1^2 \left( x+\dfrac{1}{x^2} \right) \, dx &=&\displaystyle\int \limits_1^2 \left( x+x^{-2} \right) \, dx \cr\cr &=& \left[\dfrac{1}{2}x^2 + \left( \dfrac{1}{-2+1} \right)x^{-2+1}\right]_1^2 \cr\cr &=& \left[\dfrac{x^2}{2}-\dfrac{1}{x}\right]_1^2 \cr &=& \dfrac{2^2}{2}-\dfrac{1}{2}-\left( \dfrac{1^2}{2}-\dfrac{1}{1} \right) \cr &=& 2 \end{array}


8)
\begin{array}{rcl} \displaystyle\int \limits_0^{10} \left( 5\sqrt{x} + 4x \right)dx &=&\displaystyle\int \limits_0^{10}\left( 5x^\frac{1}{2} + 4x \right)dx \cr\cr &=& \left[\genfrac{}{}{1pt}{0}{5}{\frac{3}{2}}x^{\frac{3}{2}}+2x^2 \right]_0^{10} \cr\cr &=& \left[\dfrac{10}{3}\sqrt{x^3}+2x^2 \right]_0^{10} \cr &=& \dfrac{10}{3}\sqrt{10^3} + 2 \cdot 10^2 - \left( 0 + 0\right) \cr & \approx & 305{,}41 \end{array}


9)
\begin{array}{rcl} \displaystyle\int \limits_2^5 \left( \dfrac{3}{8y^4} + \dfrac{12}{y^3} - \dfrac{7}{6y^2} \right)dy &=& \displaystyle\int \limits_2^5 \left( \dfrac{3}{8}y^{-4} +12y^{-3} - \dfrac{7}{6}y^{-2} \right)dy \cr\cr &=& \left[ \dfrac{3}{8 \cdot (-3)}y^{-3} + \dfrac{12}{-2}y^{-2}-\dfrac{7}{6 \cdot (-1)}y^{-1} \right]_2^5 \cr\cr &=& \left[ \dfrac{-1}{8y^3}-\dfrac{6}{y^2} + \dfrac{7}{6y} \right]_2^5 \cr &=& -\dfrac{1}{8\cdot5^3}-\dfrac{6}{5^2}+\dfrac{7}{6 \cdot 5} - \left( \dfrac{-1}{8 \cdot 2^3} - \dfrac{6}{2^2} + \dfrac{7}{6\cdot 2} \right) \cr & \approx & 0{,}92 \end{array}


10)
\begin{array}{rcl} \displaystyle\int \limits_4^{10} \left( \dfrac{-8}{\sqrt[3]{t^2}}+1 \right) dt &=& \displaystyle\int \limits_4^{10} \left( -8t^{\frac{-2}{3}}+1 \right) dt \cr &=& \left[\genfrac{}{}{1pt}{0}{-8}{\frac{1}{3}}t^{\frac{1}{3}}+t \right]_4^{10} \cr &=& \left[ -24\sqrt[3]{t}+t \right]_4^{10} \cr &=& -24\sqrt[3]{10}+10-\left( -24\sqrt[3]{4}+4 \right) \cr & \approx & -7{,}61 \end{array}

11) 
\begin{array}{rcl} \displaystyle\int \limits_{-\frac{\pi}{2}}^{\frac{3\pi}{2}} \cos(x) \, dx &=& \left[ \sin(x) \right]_{-\frac{\pi}{2}}^{\frac{3\pi}{2}} \cr &=& \sin\left(\dfrac{3\pi}{2}\right) - \sin\left(-\dfrac{\pi}{2}\right) \cr &=& 0 \end{array}


12)
\begin{array}{rclll} \displaystyle\int \limits_0^{\frac{\pi}{2}} \cos\left(3x-\dfrac{{\pi}}{8}\right) \, dx \end{array}

Substitution: u = 3x-\dfrac{\pi}{8}
\begin{array}{rclll} \dfrac{du}{dx} &=& 3 \cr dx &=& \dfrac{du}{3} \cr\cr\cr \displaystyle\int \limits_0^{\frac{\pi}{2}} \cos\left(3x-\dfrac{{\pi}}{8}\right) \, dx &=& \displaystyle\int \limits_0^{\frac{\pi}{2}} \cos(u)\,\dfrac{du}{3} \cr\cr &=& \dfrac{1}{3}\displaystyle\int\limits_0^{\frac{\pi}{2}} \cos(u) \,du \cr &=& \left[\dfrac{1}{3}\sin(u)\right]_0^\frac{\pi}{2} \end{array}

Rücksubstitution:
\begin{array}{rclll} \displaystyle\int \limits_0^{\frac{\pi}{2}} \cos\left(3x-\dfrac{{\pi}}{8}\right) \, dx &=& \left[\dfrac{1}{3}\sin\left(3x-\dfrac{\pi}{8}\right)\right]_0^\frac{\pi}{2} \cr\cr &=& \dfrac{\sin\left(3 \cdot \frac{\pi}{2}-\frac{{\pi}}{8}\right)}{3}-\dfrac{\sin\left(-\frac{{\pi}}{8}\right)}{3} \cr\cr &\approx& -0{,}18\end{array}

Vorgehen: lineare Substitution

13)
\begin{array}{rcl} \displaystyle\int \limits_0^2 \dfrac{1}{2}(x^2-e^x) \, dx &=& \dfrac{1}{2} \displaystyle\int \limits_0^2 \left(x^2 - e^x \right)dx \cr &=& \dfrac{1}{2}\left[ \dfrac{1}{3}x^3 - e^x \right]_0^2 \cr &=& \dfrac{1}{2} \left( \dfrac{1}{3}\cdot 2^3 - e^2 - \left( \dfrac{1}{3}\cdot 0^3 -e^0 \right) \right) \cr &=& \dfrac{1}{2} \left( \dfrac{8}{3} - e^2 +1 \right) \cr &\approx& -1{,}86 \end{array}


14)
 \displaystyle\int \limits_{-2}^1 e^{-12x+5} \, dx

Substitution: u = -12x+5
\begin{array}{rclll} \dfrac{du}{dx} &=& -12 \cr dx &=& -\dfrac{du}{12} \cr\cr\cr \displaystyle\int \limits_{-2}^1 e^{-12x+5} \,dx &=& \displaystyle\int \limits_{-2}^1 -e^{u}\,\dfrac{du}{12} \cr\cr &=& -\dfrac{1}{12}\displaystyle\int \limits_{-2}^1 e^{u}\, du \cr &=& -\dfrac{1}{12} \left[e^{u} \right]_{-2}^1 \end{array}

Rücksubstitution:
\begin{array}{rclll} \displaystyle\int \limits_{-2}^1 e^{-12x+5} \, dx &=& -\dfrac{1}{12} \left[e^{-12x+5} \right]_{-2}^1 \cr\cr &=& -\dfrac{1}{12}e^{-12 \cdot 1+5} - \left(-\dfrac{1}{12} e^{-12 \cdot (-2)+5}\right) \cr\cr &\approx& 3{,}28 \cdot 10^{11}\end{array}

Vorgehen: lineare Substitution


15)
\begin{array}{rclll} \displaystyle\int \limits_0^{\frac{\pi}{4}} \tan(x) \, dx &=& \displaystyle\int \limits_0^{\frac{\pi}{4}} \dfrac{\sin(x)}{\cos(x)} \, dx \end{array}

Substitution: u = \cos(x)
\begin{array}{rclll} \dfrac{du}{dx} &=& -\sin(x) \cr dx &=& \dfrac{-1}{\sin(x)} \,du \cr\cr\cr \displaystyle\int \limits_0^{\frac{\pi}{4}} \dfrac{\sin(x)}{\cos(x)} \, dx &=& \displaystyle\int \limits_0^{\frac{\pi}{4}} \dfrac{\sin(x)}{u} \cdot \dfrac{-1}{\sin(x)} \, du \cr\cr&=& -1 \displaystyle\int \limits_0^{\frac{\pi}{4}} \dfrac{1}{u} \, du \cr &=& -1 \cdot \left[\ln(\vert u\vert)\right]_0^{\frac{\pi}{4}} \end{array}

Rücksubstitution:
\begin{array}{rclll} \displaystyle\int \limits_0^{\frac{\pi}{4}} \dfrac{\sin(x)}{\cos(x)} \, dx &=& -\left[\ln\left(\left|\cos(x)\right|\right)\right]_0^{\frac{\pi}{4}} \cr\cr &=& -\ln\left(\left|\cos\left({\dfrac{\pi}{4}}\right)\right|\right) - \left(-\ln\left(\left|\cos\left(0\right)\right|\right)\right) \cr\cr &\approx& 0{,}35 \end{array}

Vorgehen: Substitution


16)
\begin{array}{rclcl} \displaystyle\int \limits_1^8 \dfrac{5(x-7)^2}{\left(25x^4-350x^3+1.225x^2\right)\sqrt[3]{x}} \, dx &=& \displaystyle\int \limits_1^8 \dfrac{5(x-7)^2}{25\left(x^4-14x^3+49x^2\right)\sqrt[3]{x}} \, dx \cr\cr&=& \displaystyle\int \limits_1^8 \dfrac{1}{5} \cdot \dfrac{(x-7)^2}{\left(x^4-14x^3+49x^2\right)\sqrt[3]{x}} \, dx \cr\cr&=& \dfrac{1}{5} \displaystyle\int \limits_1^8 \dfrac{(x-7)^2}{x^2\left(x^2-14x+49\right)\sqrt[3]{x}} \, dx \cr\cr&=& \dfrac{1}{5} \displaystyle\int \limits_1^8 \dfrac{(x-7)^2}{x^2\left(x-7\right)^2\sqrt[3]{x}} \, dx \cr\cr&=& \dfrac{1}{5} \displaystyle\int \limits_1^8 \dfrac{1}{x^2 \cdot x^{\frac{1}{3}}} \, dx \cr\cr&=& \dfrac{1}{5} \displaystyle\int \limits_1^8 x^{-\frac{7}{3}} \, dx \cr\cr&=& \dfrac{1}{5} \cdot \left[-\dfrac{3}{4x^{\frac{4}{3}}}\right]_1^8 \cr\cr&=& -\dfrac{3}{20} \cdot \left[\dfrac{1}{\sqrt[3]{x^4}}\right]_1^8 \cr\cr&=& -\dfrac{3}{20} \left(\dfrac{1}{\sqrt[3]{8^4}} - \dfrac{1}{\sqrt[3]{1^4}}\right) \cr\cr&=& \dfrac{9}{64} \end{array}


17)
\begin{array}{rclcrcl} \displaystyle\int \limits_1^{11} \dfrac{3x^2}{2}\ln(x) \, dx &=& \dfrac{3}{2} \displaystyle\int \limits_1^{11} x^2 \cdot \ln(x) \, dx \cr\cr \quad u' &=& x^2 &\Rightarrow & u &=& \dfrac{x^3}{3} \cr v &=& \ln(x) & \Rightarrow & v' &=& \dfrac{1}{x} \end{array}

\begin{array}{rcl} \dfrac{3}{2} \displaystyle\int \limits_1^{11} x^2 \cdot \ln(x) \, dx &=& \dfrac{3}{2}\left(\left[\dfrac{x^3}{3} \cdot \ln(x)\right]_1^{11} - \displaystyle\int\limits_1^{11} \dfrac{x^2}{3} \, dx \right) \cr\cr &=& \dfrac{3}{2}\left[\dfrac{x^3}{3} \cdot \ln(x) - \dfrac{1}{3} \cdot \dfrac{x^3}{3} \right]_1^{11} \cr\cr&=& \left[ \dfrac{1}{2}x^3\ln(x)-\dfrac{1}{6}x^3 \right]_1^{11} \cr\cr&=& \dfrac{1}{2}\cdot 11^3\cdot\ln(11)-\dfrac{1}{6}\cdot 11^3 - \left(\dfrac{1}{2}\cdot 1^3\cdot\ln(1)-\dfrac{1}{6}\cdot 1^3 \right) \cr\cr&\approx& 1.374{,}13\end{array}

Vorgehen: partielle Integration


18)
\begin{array}{rcl} \displaystyle\int \limits_2^4 42^y \, dy &=& \left[ \dfrac{42^y}{\ln(42)} \right]_2^4 \cr\cr &=& \dfrac{42^4}{\ln(42)} - \dfrac{42^2}{\ln(42)} \cr\cr &\approx& 832.051{,}07 \end{array}


19)
\begin{array}{rcl} \displaystyle\int \limits_4^6 \dfrac{x-3}{x^2-6x+9} \, dx &=& \displaystyle\int \limits_4^6 \dfrac{x-3}{(x-3)^2} \, dx \cr\cr&=& \displaystyle\int \limits_4^6 \dfrac{1}{x-3} \, dx \cr\cr&=& \left[ \ln\left(\left|x-3\right|\right) \right]_4^6 \cr\cr&=& \ln\left(\left|6-3\right|\right) - \ln\left(\left|4-3\right|\right) \cr\cr&\approx& 1{,}10 \end{array}

Vorgehen: logarithmische Integration


20)
\begin{array}{rcl} \displaystyle\int \limits_1^3 \left(\sin(u)+3x^2-e^{-3t}+\dfrac{1}{x}\right) \, dx &=& 3 \displaystyle\int\limits_1^3 x^2 \, dx + \displaystyle\int\limits_1^3 \dfrac{1}{x} \, dx + \sin(u)\displaystyle\int\limits_1^3 1 \, dx-e^{-3t} \displaystyle\int\limits_1^3 1 \, dx \cr\cr&=& \left[x^3 + \ln(\left|x\right|) + \sin(u) \cdot x - e^{-3t}x\right]_1^3 \cr\cr &=& \left(3^3 + \ln(3) + \sin(u) \cdot 3 -e^{-3t} \cdot 3\right) - \left(1 + \ln(1) + \sin(u)-e^{-3t}\right) \cr\cr &=& 26+\ln\left(3\right)+2\sin\left(u\right)-2 e^{-3t} \end{array}

Bemerkung: t und u sind hier keine Variablen ...

 

5. Aufgabe

1)
Es gilt:
\begin{array}{rcl}\displaystyle\int \limits_0^t x^2\,dx &=& \left[ \dfrac{1}{3}x^3\right]_0^t \cr \cr &=& \dfrac{1}{3}t^3-0 \end{array}

Dieser letzte Ausdruck beschreibt den eingeschlossenen Flächeninhalt, der in dieser Aufgabe gleich 72 sein soll. Zu lösen ist also \dfrac{1}{3}t^3-0 = 72 . Daraus ergibt sich t= 6.


2)
Es gilt:
\begin{array}{rcl}\displaystyle\int \limits_{\frac{1}{2}}^t \left(\dfrac{1}{x^3}-\dfrac{1}{x^2} \right) \, dx &=& \left[-\dfrac{1}{2x^2}+\dfrac{1}{x}\right]_{\frac{1}{2}}^t \cr \cr &=& -\dfrac{1}{2t^2}+\dfrac{1}{t} -\left(-\genfrac{}{}{1pt}{0}{1}{2\cdot \left(\frac{1}{2}\right)^2}+\genfrac{}{}{1pt}{0}{1}{\frac{1}{2}}\right) \cr \cr &=& -\dfrac{1}{2t^2}+\dfrac{1}{t} \end{array}

Dieser letzte Ausdruck beschreibt den eingeschlossenen Flächeninhalt, der in dieser Aufgabe gleich 0 sein soll. Zu lösen ist also -\dfrac{1}{2t^2}+\dfrac{1}{t} = 0 . Daraus ergibt sich t = \dfrac{1}{2}.

Bemerkung 1: Auf diese Lösung hätte man auch ohne Rechnung kommen können (siehe Eigenschaften des Integrals) ... Allerdings ist es durchaus möglich, dass es mehr Lösungen als diese triviale gibt, da das Integral ja den orientierten Flächeninhalt beschreibt. Das bedeutet: Liegt oberhalb der x-Achse genauso viel Fläche wie unterhalb, besitzt das Integral einen Wert von 0.

Bemerkung 2: Beim Lösen von -\dfrac{1}{2t^2}+\dfrac{1}{t} = 0 müssen erst die Brüche gleichnamig gemacht werden. Das ergibt: \dfrac{-1+2t}{2t^2} = 0 . Da ein Bruch aber nur dann 0 sein kann, wenn der Zähler 0 ist, reicht es nun, -1+2t = 0 zu betrachten.


3)
Es gilt:
\begin{array}{rcl}\displaystyle\int \limits_t^{10} 3\sqrt{x} \, dx &=& \left[2\sqrt{x^3}\right]_t^{10} \cr \cr &=& 2\sqrt{1000}-2\sqrt{t^3} \end{array}

Dieser letzte Ausdruck beschreibt den eingeschlossenen Flächeninhalt, der in dieser Aufgabe gleich \sqrt {1000} sein soll. Zu lösen ist also 2\sqrt{1000}-2\sqrt{t^3} = \sqrt{1000}. Daraus ergibt sich t = \sqrt[3]{250}.


4)
Es gilt:
\begin{array}{rcl}\displaystyle\int \limits_t^{2t} (6x-1) \, dx &=& \left[3x^2-x\right]_t^{2t} \cr \cr &=& 3\cdot 4t^2-2t-\left(3t^2-t\right) \cr \cr &=& 9t^2-t \end{array}

Dieser letzte Ausdruck beschreibt den eingeschlossenen Flächeninhalt, der in dieser Aufgabe gleich 0 sein soll. Zu lösen ist also 9t^2-t = 0. Diese Gleichung hat zwei Lösungen, nämlich t=0 und t = \dfrac{1}{9}.


5)
Es gilt:
\begin{array}{rcl}\displaystyle\int \limits_0^5 \left(3t x^2+2\right) \, dx &=& \left[t x^3+2x\right]_0^5 \cr \cr &=& t \cdot 125+10-0 \cr \cr &=& 125t +10 \end{array}

Dieser letzte Ausdruck beschreibt den eingeschlossenen Flächeninhalt, der in dieser Aufgabe gleich 35 sein soll. Zu lösen ist also 125t +10 = 35. Daraus ergibt sich t = \dfrac{1}{5}.


6)
Es gilt: 
\begin{array}{rcl}\displaystyle\int \limits_{-2}^2 \left(-\dfrac{1}{4}x+t\right) \, dx &=& \left[-\dfrac{1}{8}x^2+t x\right]_{-2}^2 \cr \cr &=& -\dfrac{1}{8}\cdot 4+2t - \left(-\dfrac{1}{8}\cdot 4-2t \right) \cr \cr &=& 4t \end{array}

Dieser letzte Ausdruck beschreibt den eingeschlossenen Flächeninhalt, der in dieser Aufgabe gleich 200 sein soll. Zu lösen ist also 4t = 200. Daraus ergibt sich t = 50.

 

6. Aufgabe

1)
Wenn die Fläche zwischen zwei Funktionen zu berechnen ist, sollte man sich zuallererst die Verläufe der Graphen anschauen, also eine Skizze machen.

Fläche zwischen 2 Parabeln

Es ist zu sehen, dass die f(x) und g(x) zwei Schnittpunkte haben. Hätten die Funktionen nur einen oder gar keinen Schnittpunkt, könnten sie ja auch keine (komplette) Fläche einschließen. Da die Schnittpunkte die Grenzen des zu berechnenden Integrals bilden, müssen wir sie nun berechnen:

\begin{array}{rcl} x^2 &=& -x^2+2x+4 \cr 2x^2-2x-4 &=& 0 \cr &...& \cr x_1 &=& -1 \cr x_2 &=& 2 \end{array}

Um die Fläche zwischen den beiden Funktionen zu berechnen, müssen wir die Funktion mit den kleineren Funktionswerten von der anderen abziehen und dann das Integral der Differenzfunktion berechnen, denn die Fläche, die uns interessiert, entsteht dadurch, dass wir die gesamte Fläche unter der "größeren" Funktion nehmen und alles weglassen, was unterhalb der "kleineren" Funktion liegt. Setzt man einen Wert aus dem Intervall ]-1 \; ; \; 2[ in die beiden Funktionsgleichungen ein, erkennt man, dass innerhalb dieses Intervalls g(x) größer ist als f(x). Sieht man natürlich auch in der Grafik oben.
In diesem Fall ist also zu berechnen: \displaystyle\int \limits_{-1}^2 \left(g(x)-f(x)\right) \, dx

\begin{array}{rcl} \displaystyle\int \limits_{-1}^2 \left(g(x)-f(x)\right) \, dx &=& \displaystyle\int \limits_{-1}^2 \left(-x^2+2x+4-x^2\right) \, dx \cr &=& \displaystyle\int\limits_{-1}^2 \left(-2x^2+2x+4\right) \, dx \cr &=& \left[ -\dfrac{2}{3}x^3+x^2+4x\right]_{-1}^2 \cr &=& -\dfrac{2}{3}\cdot8+4+8-\left(\dfrac{2}{3}+1-4\right) \cr &=& 9\end{array}

Die von den beiden Graphen eingeschlossene Fläche ist also 9 Flächeneinheiten groß.

Bemerkung: Der Begriff "Flächeneinheit" ist eine Verallgemeinerung. In der Aufgabenstellung waren ja keine Einheiten für die Achsen gegeben. Die Aufgabe "funktioniert" unabhängig davon, ob es sich um mm oder km oder Meilen oder ... handelt. Trotzdem hat eine Fläche natürlich irgendeine Einheit. Wir wissen halt nur nicht, welche ... Da man in der Mathematik üblicherweise gut mit Verallgemeinerungen leben kann, sagt man eben einfach "Flächeneinheit".


2)
In der Grafik ist zu sehen, dass diese Graphen von f(x)=x^3-x+2 und g(x)=-x^3+x+2 drei Schnittpunkte haben und damit zwei Flächenstücken einschließen.

Fläche zwischen 2 Funktionen 3. Grades

Schnittpunkte berechnen:
\begin{array}{rcl}x^3-x+2 &=& -x^3+x+2 \cr &...& \cr x_1 &=& -1 \cr x_2 &=& 0 \cr x_3 &=& 1 \end{array}

Lage der Funktion zwischen den Schnittpunkten bestimmen
Hier muss die Lage der Funktionen natürlich in beiden Intervallen überprüft werden: Im Intervall ]-1 \; ; \; 0[ hat f(x) größere Funktionswert, liegt als "über" g(x); im Intervall ]0 \; ; \; 1[ ist es umgekehrt. Es kann also nicht einfach mit den Grenzen -1 und 1 gerechnet werden. Entweder man berechnet jeweils ein Integral für jedes Teilintervall oder - und das ist die "mathematischere" und kürzere Vorgehensweise - man überlegt sich, dass f(x) und g(x) punktsymmetrische Funktionen sind und damit die eingeschlossenen Flächen in den beiden Intervallen gleich sein müssen. D. h., man berechnet nur eines der beiden Integrale und multipliziert das Ergebnis mit 2.

Das alles führt uns zu dem Integral
\begin{array}{rcl} 2\displaystyle\int \limits_0^1 \left(g(x)-f(x)\right) \, dx &=& 2\displaystyle\int \limits_0^1 \left(-x^3+x+2-\left(x^3-x+2\right)\right) \, dx \cr &=& 2\displaystyle\int\limits_0^1 \left(-2x^3+2x\right) \, dx \cr &=& 2\left[ -\dfrac{2}{4}x^4+x^2\right]_0^1 \cr &=& 2\left(-\dfrac{1}{2}+1-0\right) \cr &=& 1 \end{array}

Die von den beiden Graphen eingeschlossene Fläche ist also 1 Flächeneinheit groß.
 
3)
Die Funktionen f(x)=t-\dfrac{x^2}{t} und g(x)=t^3-t x^2 mit 0 < t \leq 1 lassen sich wegen des Parameters nicht gut zeichnen, also versuchen wir es ohne Zeichnung ... Bitte beachten Sie, dass es sich bei beiden Funktionen einfach um quadratische Funktionen handelt; t ist ja nur eine Zahl.

Schnittpunkte berechnen
\begin{array}{rcl} t-\dfrac{x^2}{t} &=& t^3-t x^2 \cr\cr -\dfrac{x^2}{t}+t x^2 &=& t^3-t \cr\cr x^2\left(t-\dfrac{1}{t}\right) &=& t^3-t \cr\cr x^2 &=& \genfrac{}{}{1pt}{0}{t^3}{t-\dfrac{1}{t}}-\genfrac{}{}{1pt}{0}{t}{t-\dfrac{1}{t}} \cr\cr &=& \genfrac{}{}{1pt}{0}{t^3}{\dfrac{t^2-1}{t}}-\genfrac{}{}{1pt}{0}{t}{\dfrac{t^2-1}{t}} \cr\cr &=& \dfrac{t^4}{t^2-1}-\dfrac{t^2}{t^2-1} \cr\cr &=& \dfrac{t^4-t^2}{t^2-1} \cr\cr &=& \dfrac{t^2\left(t^2-1\right)}{t^2-1} \cr\cr &=& t^2 \cr\cr x_{1,2} &=& \pm t \end{array}

Lage der Funktion zwischen den Schnittpunkten bestimmen
Um zu bestimmen, welche Funktion im Intervall \left]-t ; t \right[ die größere ist, können wir beispielsweise die 0 einsetzen. Sie liegt sicher zwischen -t und +t . Es ergibt sich, dass im betrachteten Intervall f(x) größere Funktionswerte hat als g(x)

Das alles führt uns zu dem Integral
\begin{array}{rcl} \displaystyle\int\limits_{-t}^t \left(f(x)-g(x) \right) \, dx &=& \displaystyle\int\limits_{-t}^t \left(t-\dfrac{x^2}{t}-\left(t^3-t x^2\right)\right) \, dx \cr\cr &=& \displaystyle\int\limits_{-t}^t\left(\left(t-\dfrac{1}{t}\right)x^2-t^3+t\right) \, dx \cr\cr &=& \left[\dfrac{1}{3}\left(t-\dfrac{1}{t}\right)x^3+\left(-t^3+t\right)x\right]_{-t}^t \cr\cr &=& \dfrac{1}{3}\left(t-\dfrac{1}{t}\right)t^3+\left(-t^3+t\right)t-\left(-\dfrac{1}{3}\left(t-\dfrac{1}{t}\right)t^3-\left(-t^3+t\right)t\right) \cr\cr &=& -\dfrac{4}{3}t^4+\dfrac{4}{3}t^2 \end{array}

Die von den beiden Graphen eingeschlossene Fläche ist also -\dfrac{4}{3}t^4+\dfrac{4}{3}t^2 Flächeneinheiten groß, für 0 < t \leq 1.

Zusatzaufgabe
Gefragt ist hier nach dem Flächeninhalt in Abhängigkeit von t, nennen wir ihn A(t). Dieser ist zu maximieren. Der übliche Weg dafür ist, die entsprechende Funktion abzuleiten und dann die Nullstellen der Ableitungsfunktion zu bestimmen. Das machen wir auch hier so:
\begin{array}{rcl} A(t) &=& -\dfrac{4}{3}t^4+\dfrac{4}{3}t^2 \cr \cr A'(t) &=& -\dfrac{16}{3}t^3+\dfrac{8}{3}t \cr \cr 0 &=& -\dfrac{16}{3}t^3+\dfrac{8}{3}t \cr &...& \cr t_1 &=& -\dfrac{\sqrt{2}}{2} \cr t_2 &=& 0 \cr t_3 &=& \dfrac{\sqrt{2}}{2} \end{array}
Von diesen drei Werten liegt nur t_3 = \dfrac{\sqrt{2}}{2} \approx 0{,}707 im für t vorgegebenen Intervall.

Jetzt ist nur noch die Frage zu klären, ob bei t_3 = \dfrac{\sqrt{2}}{2} tatsächlich ein Maximum der Funktion A(t) liegt. Es könnte ja auch ein Minimum sein ... Der einfachste Weg hierfür ist, zu schauen, was für t=1 passiert: Für diesen Parameterwert vereinfachen sich beide Funktionen zu f(x) = g(x) = 1-x^2. Der von ihnen eingeschlossene Flächeninhalt ist also 0 und damit minimal. Da es außer den drei berechneten Werten und dem Rand des Intervalls keine weiteren Extremstellen geben kann und eine Funktion nicht an zwei nebeneinander liegenden Extremstellen jeweils ein Minimum haben kann, ist also klar, dass bei t_3 tatsächlich ein Maximum liegt.

24. Komplexe Zahlen - Lernziele und typische Fehler

Nach Durcharbeiten dieses Kapitels sollten Sie folgende Lernziele erreicht haben:

  • Sie wissen, warum komplexe Zahlen benötigt werden und kennen den Fundamentalsatz der Algebra.
  • Sie kennen die Begriffe "imaginäre Einheit", "Realteil" und "Imaginärteil" und können sie sicher verwenden.
  • Sie können komplexe Zahlen in der gaußschen Zahlebene einzeichnen bzw. aus der gaußschen Zahlenebene ablesen.
  • Sie kennen die kartesische, trigonometrische und Exponentialdarstellung einer komplexen Zahl und können komplexe Zahlen von einer Darstellungsform in die andere umrechnen.
  • Sie wissen um die Bedeutung der eulerschen Relation und der eulerschen Identität.
  • Sie wissen, in welcher Darstellungsform welche Rechenoperation möglich bzw. am geschicktesten durchzuführen ist.
  • Sie können komplexe Zahlen addieren, subtrahieren, multiplizieren und dividieren.
  • Sie können Potenzen von komplexen Zahlen mithilfe des Satzes von de Moivre berechnen.
  • Sie können alle Wurzeln einer komplexen Zahl berechnen und wissen, welche davon der Hauptwert ist.


Typische Fehler in diesem Kapitel sind:

  • Es passieren Fehler bei eigentlich bekannten Rechenoperationen, z. B. bei Brüchen und Potenzen oder bei Potenz- vor Punkt- vor Strichrechnung. Erklärung
  • Der Betrag einer komplexen Zahl wird falsch berechnet, da der Zusammenhang mit dem Satz des Pythagoras nicht erkannt wird. Erklärung
  • Das Argument einer komplexen Zahl wird falsch berechnet, da der Zusammenhang mit der Trigonometrie nicht erkannt wird. Erklärung


Für Online-Selbsttests zu diesem Thema und weitere Informationen zur Mathematikunterstützung an der TH Wildau nutzen Sie bitte den Moodle-Kursraum "SOS Mathematik - Komplexe Zahlen".

Übersicht:

 

24.1 Komplexe Zahlen - Aufgaben

1. Aufgabe

Zeichnen Sie die gegebenen komplexen Zahlen in die gaußsche Zahlenebene ein und berechnen Sie die anderen Darstellungsformen!

1) z_1=-10+7i   9) z_9=\dfrac{49}{10}\left(\cos\left(\dfrac{9\pi}{8}\right)+i\sin\left(\dfrac{9\pi}{8}\right)\right)
2) z_2=-\dfrac{-18+37i}{8}   10) z_{10}=\dfrac{14}{3}\left(\cos\left(\dfrac{5\pi}{2}\right)+i\sin\left(\dfrac{5\pi}{2}\right)\right)
3) z_3=-4-\dfrac{21}{5}i   11) z_{11}=8e^{i\,\frac{17\pi}{10}}
4) z_4=\dfrac{13+3i}{2}   12) z_{12}=\dfrac{33}{4}\,e^{i\,\frac{41\pi}{22}}
5) z_5=-\dfrac{22}{3}-\dfrac{19}{4}i   13) z_{13}=\dfrac{27}{7}\,e^{i\,3\pi}
6) z_6=6\left(\cos\left(\dfrac{3\pi}{5}\right)+i\sin\left(\dfrac{3\pi}{5}\right)\right)   14) z_{14}=6e^{i\,\frac{7\pi}{9}}
7) z_7=\dfrac{12}{7}\left(\cos\left(\dfrac{\pi}{2}\right)+i\sin\left(\dfrac{\pi}{2}\right)\right)   15) z_{15}=\dfrac{9}{2}\,e^{i\,8\pi}
8) z_8=\dfrac{30}{11}\left(\cos\left(\dfrac{\pi}{6}\right)+i\sin\left(\dfrac{\pi}{6}\right)\right)    

 

2. Aufgabe

Wichtiger Rat: Lassen Sie bei diesen Aufgaben Ihren Taschenrechner in der Tasche und trainieren damit Ihre Kopfrechenfähigkeiten! Die Zahlenwerte sind alle so gewählt, dass das möglich ist. Sie benötigen nicht wesentlich mehr als das kleine Einmaleins und Addition / Subtraktion im Zahlenraum bis 100 ...

a)

Berechnen Sie die folgenden Aufgaben!

1)  (7 \; ; \; 4) + (12 \; ; \; -35 )   6)  \left(-\dfrac{1}{2} + \dfrac{1}{2}i \right) \cdot \left(\dfrac{7}{20} - \dfrac{1}{10}i \right)
2)  -\dfrac{1}{21} + \dfrac{2}{5}i + \dfrac{2}{3} + \dfrac{7} {8}i   7)  (11-10i) \cdot (-5-2i)
3)  \left(\dfrac{3}{4} \; ; \; -5 \right) - \left(\dfrac{11}{8} \; ; \; \dfrac{5} {6} \right)   8) \dfrac{11-8i}{-4i}
4)  7-4i - (18 -3i)   9) \dfrac{i-10}{-1-i}
5) (10-2i) \cdot (3-6i)   10) \dfrac{4}{2+\sqrt{2}i}

 

b)

Berechnen Sie die folgenden Produkte und Quotienten! Wählen Sie dabei den Winkel so, dass  \varphi \; \in \; [0 \, ; \, 2\pi[ ist.

1) \left(\cos(\pi) + i \sin(\pi)\right) \cdot 7 \left( \cos(\pi)+i \cdot \sin(\pi) \right)   7)  4 \left(\cos\left(\dfrac{3\pi}{4} \right)+i \sin\left(\dfrac{3\pi}{4} \right) \right) \; : \; \left( \dfrac{2}{5} \left(\cos\left(\dfrac{5\pi}{12}\right)+i \sin\left(\dfrac{5\pi} {12} \right) \right) \right)
2)  \dfrac{1}{8} \left(\cos\left(\dfrac{\pi}{4} \right)+i \sin\left(\dfrac{\pi}{4} \right) \right) \; \cdot \; \dfrac{12}{11} \left(\cos\left(\dfrac{2\pi}{5} \right)+i \sin\left(\dfrac{2\pi}{5}\right) \right)   8)  \dfrac{8}{9} \left(\cos\left(\dfrac{\pi}{2} \right)+i \sin\left(\dfrac{\pi}{2} \right) \right) \; : \; \left( \dfrac{16}{27} \left(\cos\left(\dfrac{3\pi}{10} \right)+i \sin\left(\dfrac{3\pi}{10} \right)\right) \right)
3) \dfrac{7}{5}\left(\cos\left(\dfrac{7\pi}{5}\right)+i\sin\left(\dfrac{7\pi}{5}\right)\right) \; \cdot \; \dfrac{20}{21}\left(\cos\left(\dfrac{21\pi}{20}\right)+i\sin\left(\dfrac{21\pi}{20}\right)\right)   9)  \dfrac{45} {2} \left(\cos\left(\dfrac{3\pi}{5} \right)+i \sin\left(\dfrac{3\pi}{5} \right) \right) \; : \; \left(9\left(\cos\left(\dfrac{4\pi}{9}\right)+i \sin\left(\dfrac{4\pi} {9} \right)\right)\right)
4) 5 e^{i \, \frac{\pi}{10}} \cdot 12 e^{i \, \frac{\pi}{8}}   10)  5 e^{i \, \frac{\pi}{10}} \; : \; \left( 12 e^{i \, \frac{\pi}{8}} \right)
5)  \dfrac{4}{7} \, e^{i \, \frac{5\pi}{6}} \; \cdot \; \frac{21}{8} \, e^{i \, \frac{2\pi}{3}}   11)  \dfrac{3}{8} \, e^{i \, \frac{3\pi}{4}} \; : \; \left(\dfrac{9}{16} \, e^{i \, \frac{\pi}{6}} \right)
6) \dfrac{12}{5} \, e^{i \, \frac{3\pi}{10}} \cdot 15 e^{i \, \frac{5\pi}{12}}   12)  \dfrac{1}{51} \, e^{i \, \frac{2\pi}{3}} \; : \; \left(\dfrac{2}{17} \, e^{i \, \frac{3\pi} {4}} \right)

 

3. Aufgabe

Berechnen Sie die folgenden Potenz- und Wurzelausdrücke!

1)  \left(\dfrac{1}{2}\left(\cos\left(\dfrac{\pi}{8} \right)+i \sin\left(\dfrac{\pi}{8} \right) \right) \right) ^4   6)  \sqrt{\dfrac{1}{49}\left(\cos\left(\dfrac{2\pi}{3} \right)+i\sin\left(\dfrac{2\pi}{3} \right) \right)}
2)  \left( \dfrac{2} {3} \left(\cos \left(\dfrac{7\pi} {12} \right)+i \sin \left(\dfrac{7\pi} {12} \right) \right) \right) ^3   7)  \sqrt[4]{\dfrac{1}{256}\left(\cos\left(\dfrac{8\pi}{5} \right)+i\sin\left(\dfrac{8\pi}{5} \right) \right)}
3)  \left( 8 e^{i \, \frac{4\pi}{3}} \right)^2   8)  \sqrt{\dfrac{81} {4} \, e^{i \, \frac{\pi}{5}}}
4)  \left( 2 e^{i \, \frac{7\pi}{5}} \right)^5   9)  \sqrt[3]{e^{i \, \frac{3\pi}{4}}}
5)  \sqrt[3]{64 \left(\cos(6 \pi)+i\sin(6 \pi) \right)}   10)  \sqrt[5]{32 e^{i \, \frac{\pi}{3}}}

 

4. Aufgabe

Bestimmen Sie alle Lösungen der folgenden Gleichungen mit \mathbb{D}=\mathbb{C}.
Für einige Aufgaben finden Sie Lösungshinweise am Ende der Seite.

1) -3x^2+6x-8=0

  11) -20x^2=4x^4+29

2) \dfrac{2}{3}x^2+\dfrac{289}{3}=16x

  12) -2x(-ix+5x-10i)=x^3+10(5x-10i)

3) 8x(2x-3)=-25

  13) 21x+125=x^2+3x+215

4) x(x^2-12(x-3))=6-2(6x^2+3)

  14) x+4=\dfrac{-10}{x}+3+i^2
5) -\dfrac{1}{2}z-2i=-\dfrac{1}{8}z^2-iz+\dfrac{3}{2}

  15) \dfrac{x^2}{i}+\dfrac{5x\cdot i^{i+1}}{i^i}-7i =0

6) -2\left(x^2 +\dfrac{5}{8}\right)=x^3+\dfrac{9}{4}x

  16) 0=z^3+64
7) 3x-19x^2 = -18x^2 + \dfrac{10}{4}

  17) 7z^3-189i = 0
8) 0=4z^3+20z^2+50z

  18) \dfrac{2z}{3}+\dfrac{-\sqrt{36i}+45}{9} = 5

9) -217=7(x^2-4x)

  19) -28z^4+7\sqrt{3}i = 7

10) -x^3+x^2i-4x^2+4xi-68x+68i=0   20) -3z^3+81-3i = 84

 

Lösungshinweise

Aufgabe 4.6: x_0=-1 ist eine Lösung der Gleichung.
Aufgabe 4.10: x_0=i ist eine Lösung der Gleichung.
Aufgabe 4.12: x_0=2i ist eine Lösung der Gleichung.

Dieses Kapitel enthält die folgenden Themen:

 

24.2 Komplexe Zahlen - Erklärungen

Ein Hinweis vorab: In der Elektrotechnik und damit in vielen Ingenieurwissenschaften wird die imaginäre Einheit nicht mit i sondern mit j bezeichnet, weil dort I und i bereits für die Stromstärke vergeben sind. Bitte lassen Sie sich nicht davon irritieren, dass in diesem Text i verwendet wird, wie es in der Mathematik üblich ist.

 

Warum komplexe Zahlen?

Vielfach taucht bei quadratischen Gleichungen und Polynomen die Begründung auf "... Da aus negativen reellen Zahlen keine Wurzeln mit geraden Wurzelexponenten gezogen werden können ..." - natürlich kann man das! Um diese Einschränkung zu überwinden, haben Mathematiker nämlich die komplexen Zahlen "erfunden". Ganz wichtig zu nennen sind hierbei Carl Friedrich Gauß und Leonhard Euler. Die komplexen Zahlen haben sich in der Folge in vielen weiteren Bereichen und Zusammenhängen als sehr nützlich erwiesen, z. B. in der Elektrotechnik oder bei der Beschreibung von Schwingungen.
Die Bezeichnung "komplexe Zahlen" und damit ihr Symbol \mathbb{C} leiten sich von dem lateinischen Wort "complexus" ab, welches "umfassend" bedeutet. Damit ist nicht gemeint, dass komplexe Zahlen besonders kompliziert wären ...

Die Eigenschaft, dass Wurzeln nun nicht mehr vor negativen Zahlen haltmachen, gipfelt in einem Satz, bei dem schon der Name auf seine Bedeutung hinweist, nämlich dem Fundamentalsatz der Algebra:
Es seien n\in\mathbb{N} und a_0, a_1,\dots , a_n\in\mathbb{C}. Dann hat jedes Polynom a_nx^n+a_{n-1}x^{n-1}+\dots +a_1x+a_0=0 genau n Lösungen z_1, z_2,\dots ,z_n\in\mathbb{C} . Diese Lösungen müssen allerdings nicht alle verschieden sein.
Vielleicht erinnern Sie sich an den folgenden Satz im Kapitel Polynome: Die Anzahl der Nullstellen eines Polynoms ist im Bereich der reellen Zahlen maximal so groß wie der Grad des Polynoms. Dieses "maximal so groß" gibt zwar eine obere Grenze vor, lässt aber nach unten Spielraum. Diesen Spielraum gibt es im Bereich der komplexen Zahlen nicht mehr. Hier weiß man eindeutig: Ein Polynom vom Grade 3 hat 3 Nullstellen; ein Polynom vom Grad 17 hat 17 Nullstellen, vorausgesetzt man zählt die Häufigkeit dieser Nullstellen mit (man sagt mathematisch: ihre Vielfachheit). Und Eindeutigkeit ist etwas, was in der Mathematik ja bekanntermaßen sehr geschätzt wird ...

 

Was sind komplexe Zahlen?

Da es in dem Zahlenvorrat, den wir bislang kennen, kein Symbol gibt, mit dem man "Wurzel aus negativer Zahl" darstellen kann (sonst hätten wir es ja schon gefunden ...), musste man ein neues einführen. Es ist die imaginäre Einheit i mit der Eigenschaft i^2=-1.

Betrachten wir eine quadratische Gleichung, die wir bislang nicht lösen konnten, z. B.:
\begin{array}{rcl} x^2-4x+13 &=& 0 \cr &\dots & \cr x_{1,2} &=& 2\pm \sqrt{-9} \cr x_{1,2} &=& 2\pm \sqrt{9\cdot (-1)} \cr x_{1,2} &=& 2\pm 3\sqrt{-1} \cr x_{1,2} &=& 2\pm 3i \end{array}

Komplexe Zahlen (man nennt sie üblicherweise z, manchmal auch w) lassen sich also schreiben als: z=a+bi bzw. z=(a,b) und die Menge der komplexen Zahlen als \mathbb{C}=\{z \vert z=a+bi \,\text{mit}\; a,b\in\mathbb{R}\}

Eine komplexe Zahl ist also ein geordnetes Paar reeller Zahlen.  a= \textit{Re}(z) nennt man den Realteil von z und b=\textit{Im}(z) den Imaginärteil von z. Ist \textit{Im}(z)=0, so erhalten wir eine reelle Zahl. Die reellen Zahlen sind also in den komplexen enthalten. Ist \textit{Re}(z)=0 , so nennen wir die Zahl rein imaginär.
Bemerkung: Sowohl Real- als auch Imaginärteil sind reelle Zahlen! Die imaginäre Einheit i "gehört" also nicht zum Imaginärteil selbst dazu.

Ein Beispiel: \sqrt{-4}=\sqrt{4\cdot (-1)}=\sqrt{4}\cdot\sqrt{-1}=2i=0+2i
Der Realteil ist hier 0 und der Imaginärteil 2.

Ein einfacher Satz zu komplexen Zahlen: Zwei komplexe Zahlen sind dann gleich, wenn ihre Realteile und ihre Imaginärteile gleich sind.

 

Darstellung in der gaußschen Zahlenebene

Von den reellen Zahlen sind wir die Darstellung auf dem Zahlenstrahl gewöhnt. Für komplexe Zahlen ist dort aber "kein Platz" mehr, da die reellen Zahlen den Zahlenstrahl bereits vollständig ausfüllen. Die naheliegende Lösung ist, statt eines Zahlenstrahls ein Koordinatensystem zu verwenden - naheliegend deshalb, weil wir auch schon früher Paare reeller Zahlen in ein Koordinatensystem eingetragen haben, nämlich bei Punkten. In Abgrenzung zu Funktionen und Graphen nennt man dieses Koordinatensystem komplexe Ebene oder (vielleicht sogar noch üblicher) gaußsche Zahlenebene. Die Achsen heißen hier nicht x-Achse und y-Achse, sondern reelle Achse (horizontal - hier finden wir, wie wir das vom Zahlenstrahl gewöhnt sind, die reellen Zahlen) und imaginäre Achse (senkrecht - auf dieser Achse befinden sich die rein imaginären Zahlen).

Ein paar Beispiele:
z_1=4+3i
z_2=-7+6i
z_3=-3-5i
z_4=2-8i

4 komplexe Zahlen in de gaußschen Zahlenebene


Das Ziel, das grundsätzlich bei jeder Zahlenbereichserweiterung verfolgt wird, ist, dass alle bisherigen Rechenregeln und -gesetze erhalten bleiben, dass sich also die "neuen" Zahlen möglichst gut in das Gewohnte und Bewährte einpassen, sonst müsste man ja jeweils alles neu "erfinden", auch Zusammenhänge, die sich bereits bewährt haben. Oder man würde für jeden Zahlenbereich eigene Rechengesetze erhalten, was ja nun auch unpraktisch wäre, z. B. addiert man natürliche und rationale Zahlen auf gleiche Weise. Dass man bei rationalen Zahlen ggf. vor der Addition erweitern muss und bei natürlichen Zahlen nicht, liegt ja nur daran, dass natürliche Zahlen definitionsgemäß den gleichen "Nenner" haben, nämlich 1. Fast noch wichtiger als einzelne Rechentechniken sind die grundlegenden Rechengesetze, insbesondere Assoziativ-, Kommutativ- und Distributivgesetz, die natürlich auch für komplexe Zahlen gelten.
Bei den komplexen Zahlen ist dies weitgehend gelungen, mit einer Ausnahme: Die Anordnung geht verloren. Anders als auf dem Zahlenstrahl, wo bei zwei Zahlen x und y immer entschieden werden kann, ob x größer ist als y, ob x und y gleich sind oder ob x kleiner ist als y, geht dies bei komplexen Zahlen nicht, wie bei Darstellung in der gaußschen Zahlenebene deutlich wird. Nach welchen Kriterien sollte beispielsweise entschieden werden, ob z_1 oder z_2 aus der Grafik oben "größer" ist? Aus diesem Grund gibt es auch keine positiven und negativen komplexen Zahlen, denn die Konzepte "positiv" und "negativ" geben ja Auskunft über die Lage einer Zahl bezogen auf die 0.

 

Darstellungsformen

Es gibt verschiedene Darstellungsmöglichkeiten komplexer Zahlen. Sie haben insofern alle eine Berechtigung, da in den einzelnen Darstellungsformen jeweils andere Rechenoperationen und mathematische Zusammenhänge gut durchzuführen bzw. zu erklären sind.

 

Kartesische Form

Wie oben schon eingeführt, können komplexe Zahlen in der Form
z=a+bi oder z=(a,b) mit a,b \in \mathbb{R}
dargestellt werden. Dies nennt man auch Komponenten- oder algebraische Form.

 

Trigonometrische Form

Alternativ zur Festlegung über a=\textit{Re}(z) und b=\textit{Im}(z) ist eine komplexe Zahl auch dann eindeutig bestimmt, wenn man ihren Abstand vom Ursprung, Betrag genannt, \vert z \vert=r und den zugehörigen Winkel \varphi kennt. \varphi ist der griechische Buchstabe "phi" und wird gerne für die Winkel komplexer Zahlen verwendet. Diese Schreibweise nennt man auch Polarkoordinaten. Für zwei Beispielzahlen sind ihre Polarkoordinaten in der linken Grafik dargestellt. Die "Linie" bzw. der "Pfeil" vom Ursprung hin zu der komplexen Zahl wird dabei auch Zeiger, der Winkel einer komplexen Zahl Argument oder Phase genannt.

2 komplexe Zahlen und ihre Polarkoordinaten


Berechnung:
Wie die rechte Grafik zeigt, gilt:
für den Betrag (über den Satz des Pythagoras): r=\vert z \vert=\sqrt{a^2+b^2}
für das Argument (mit ein bisschen Trigonometrie): \varphi=\arctan\left(\dfrac{b}{a}\right)

Bemerkung:
Bitte beachten Sie, dass die Berechnung des Arguments in dieser Form nur funktioniert, wenn sich die komplexe Zahl im ersten Quadranten befindet. Liegt z im zweiten Quadranten (siehe Beispiel 2 etwas weiter unten), kann man ausnutzen, dass es auch hier ein Dreieck gibt, das Katheten mit den Längen a und b hat. Der hieraus berechnete Winkel \varphi ' beschreibt aber nicht das gesuchte Argument, da es zwischen dem Zeiger und der negativen x-Achse liegt. Da sich \varphi ' und \varphi aber gerade zu 180^\circ bzw. \pi ergänzen, hilft einfaches Subtrahieren weiter. Nach dieser Methode muss bei komplexen Zahlen im dritten Quadranten 180^\circ bzw. \pi zu \varphi ' addiert werden und im vierten Quadranten \varphi ' von 360^\circ bzw. 2\pi subtrahiert werden. Liegt eine komplexe Zahl direkt auf einer der Achsen, muss ohnehin nicht weiter gerechnet werden, weil diese Winkel bekannt sind. Für diese Rechnung ist es also nötig, sich vorab Klarheit darüber zu verschaffen, mit welchem Quadranten man es zu tun hat. Eine kleine Skizze wirkt hier - wie immer - Wunder ...


Wie sieht die trigonometrische Form nun also aus?
Vorausgesetzt r\neq0 lässt sich jede komplexe Zahl schreiben als: z=r(\cos(\varphi)+i\sin(\varphi)), wobei  \varphi \; \in \; [0 \, ; \, 2\pi[

Bemerkung: Wenn der Betrag r einer komplexen Zahl 0 ist, liegt diese Zahl im Ursprung. Dann braucht man auch kein Argument ...

Aus der rechten Grafik oben kann man ableiten, dass a=r\cos(\varphi) und b=r\sin(\varphi) gilt (Wer damit Schwierigkeiten hat, sollte sich noch mal das Trigonometrie-Kapitel anschauen.). Setzt man diese Beziehungen in die allererste Definition komplexer Zahlen ein, erhält man ohne viel zu rechnen die trigonometrische Darstellungsform:
z=a+bi=r\cos(\varphi)+r\sin(\varphi) \cdot i=r(\cos(\varphi)+i\sin(\varphi))
Damit haben wir diese Form auch schon hergeleitet.


Warum wird das Argument von z auf das Intervall  [0 \, ; \, 2\pi[ eingeschränkt?
Hier kommt wieder die Eindeutigkeit ins Spiel: Würde man ein größeres Intervall zulassen, wäre die Darstellung in Polarkoordinaten mehrdeutig - weitere komplexe Zahlen würden hingegen nicht entstehen. Betrachten wir beispielsweise das Argument \varphi=\dfrac{\pi}{3}. Bezüglich der Position seines Zeigers unterscheidet es sich nicht von \dfrac{7\pi}{3}=2\pi+\dfrac{\pi}{3} (etwas unmathematisch formuliert: erst einmal im Kreis rum und dann noch \dfrac{\pi}{3} dazu) und auch nicht von \dfrac{13\pi}{3}=4\pi+\dfrac{\pi}{3} (entsprechend: zweimal im Kreis rum und dann noch \dfrac{\pi}{3}) etc. Allgemein gesagt: \varphi=2k\pi+\dfrac{\pi}{3} (k\in\mathbb{Z}, also k-mal im Kreis rum und dann noch \dfrac{\pi}{3}) liefert die gleiche Position des Zeigers wie \varphi=\dfrac{\pi}{3}. Daher werden beim Argument einer komplexen Zahl Vielfache von 2\pi außer Acht gelassen, weil dies keine wirkliche Einschränkung ist. Manchmal werden auch andere Intervalle gewählt, z. B.  [-\pi \, ; \, \pi[ . Letztendlich macht das keinen Unterschied. Hauptsache, ein kompletter Kreis ist abgedeckt; sonst würde man in der Tat komplexe Zahlen "verlieren".


Beispiel 1:
Gegeben sei die komplexe Zahl z=2+2i
Ihr Betrag ergibt sich zu: r=\sqrt{2^2+2^2}=\sqrt{8}
Ihr Argument ist: \varphi=\arctan\left(\dfrac{2}{2}\right)=\dfrac{\pi}{4}
In trigonometrischer Darstellung ist also z=\sqrt{8}\left(\cos\left(\dfrac{\pi}{4}\right)+i\sin\left(\dfrac{\pi}{4}\right)\right)
Bemerkung: Bei diesen Umrechnungen ist es immer sinnvoll, nicht den Sinus oder Kosinus für die Bestimmung des Arguments zu verwenden, weil dann die Hypotenuse, sprich der Betrag der komplexen Zahl benötigt würde. Diesen müssen wir zwar sowieso auch berechnen, allerdings entstehen hier in den meisten Fällen Rundungsfehler, die man vermeidet, wenn man Tangens bzw. Arkustangens verwendet.

Beispiel 2:
Gegeben sei nun die komplexe Zahl z=-7+6i
Ihr Betrag ergibt sich zu: r=\sqrt{7^2+6^2}=\sqrt{85}
Ihr Argument ist: \varphi=180^\circ-\arctan\left(\dfrac{6}{7}\right)\approx 139{,}40^\circ
In trigonometrischer Darstellung ist also z\approx\sqrt{85}\left(\cos\left(139{,}40^\circ\right)+i\sin\left(139{,}40^\circ\right)\right)
Bemerkung 1: Da das Vorzeichen von -7 nach dem Quadrieren sowieso keine Rolle mehr spielen würde, wurde es hier gleich weggelassen. Außerdem ist die Länge von Katheten ohnehin positiv.
Bemerkung 2: Der über den Arkustangens berechnete Winkel liegt in dem rechtwinkligen Dreieck mit den Katheten a und b. In der Grafik unten wurde er mit \varphi' bezeichnet. Das ist nur leider (wie oben schon erläutert) nicht das gesuchte Argument \varphi, welches ja zwischen der positiven x-Achse und dem Zeiger liegen muss. Die Summe von \varphi und \varphi' ergibt aber genau 180^\circ , sodass es letztendlich reicht, \varphi' zu kennen, um \varphi zu bestimmen.

Veranschaulichung: phi + phi' = 180°

 

Exponentialform

Über die eulersche Formel oder auch eulersche Relation e^{i\varphi}=\cos(\varphi)+i\sin(\varphi) gelangt man zur nächsten Darstellungsform, nämlich:
z=re^{i\varphi}
mit dem Betrag r und dem Argument \varphi, wobei  \varphi \; \in \; [0 \, ; \, 2\pi[

Umrechnung: Ist die komplexe Zahl in kartesischer Form gegeben, müssen Betrag und Argument berechnet werden. Das wurde oben gerade beschrieben. Wenn die komplexe Zahl in trigonometrischer Form vorliegt, ist nichts weiter zu berechnen.

Beispiel:
Gegeben sei immer noch z=2+2i
In Exponentialform sieht diese Zahl dann so aus: z=\sqrt{8}e^{i \, \frac{\pi}{4}}


Ganz wichtiger Satz: Die so genannte eulersche Identität: e^{i\pi}+1=0
Wenn man die oben genannte eulersche Formel anwendet, ergibt sich dieser Zusammenhang ganz einfach, wenn man betrachtet, was für \varphi = \pi passiert:
e^{i\pi}=\cos(\pi)+i\sin(\pi)
Nun ist aber (ganz unabhängig von den komplexen Zahlen) \cos(\pi)=-1 und \sin(\pi) =0
Fügt man das alles in der Formel zusammen, erhält man e^{i\pi}=-1+i\cdot 0 , also e^{i\pi}+1= 0

In dieser Formel werden fünf wichtige (die fünf wichtigsten?) mathematische Konstanten in Beziehung zueinander gesetzt, nämlich 0, 1, e, \pi und i. Das ist wirklich nicht wenig! Manche Mathematiker und Mathematikerinnen halten diese Formel für so bestaunenswert wie die Mona Lisa.
Eine Bemerkung am Rande: Die eulersche Identität wurde vor einer Weile in der Zeitschrift "The Mathematical Intelligencer" zum schönsten mathematischen Satz gewählt. Bei der Abstimmung, bei der es explizit um Schönheit und nicht um Anwendungsmöglichkeiten oder die Bedeutung der Sätze ging, setzte sich die eulersche Identität durch u. a. gegen "Es gibt unendlich viele Primzahlen.", "Es gibt keine rationale Zahl, deren Quadrat 2 ist." und "Jede ebene Landkarte kann mit vier Farben so gefärbt werden, dass aneinandergrenzende Länder nicht gleich gefärbt sind." [vergleiche: Beutelspacher, Albrecht (2001): "In Mathe war ich immer schlecht". Braunschweig/Wiesbaden, S. 49f]

 

Wie rechnet man mit komplexen Zahlen?

Im diesem Abschnitt schauen wir uns u. a. an, wie die Grundrechenarten mit komplexen Zahlen funktionieren.

Eine Bemerkung vorab: Wir hatten oben schon gesehen, dass für die imaginäre Einheit i manche Sachen anders sind, als wir es aus früheren Kapiteln kennen. Beispielsweise können wir nun mit Zahlen umgehen, deren Quadrat negativ ist. Ganz wichtig ist aber, dass die üblichen Rechenregeln und -gesetze weiterhin ganz normal gelten. Das betrifft neben den Rechengesetzen und den Regeln der Bruch- und Potenzrechnung beispielsweise auch das Gesetz Potenz- vor Punkt- vor Strichrechnung! Lassen Sie sich also nicht davon irritieren, dass i "irgendwie ungewohnt" ist ... Es wäre aus mathematischer Sicht überhaupt nicht sinnvoll, an den bestehenden, bekannten und bewährten Rechenoperationen etwas zu ändern!

Addition und Subtraktion

Komplexe Zahlen kann man nur in der kartesischen Form addieren und subtrahieren. Man addiert / subtrahiert komplexe Zahlen, in dem man die Realteile addiert / subtrahiert und die Imaginärteile addiert / subtrahiert. Um diese Rechenverfahren zu durchschauen, braucht es (hoffentlich) keine Formeln, sondern nur ein paar Beispiele:

Beispiele:
Wir nehmen die komplexen Zahlen z_1=3+5i und z_2=-2+8i.

Summe:
\begin{array}{rcl}z_1+z_2 &=& (3+5i)+(-2+8i) \\ &=& 3-2+5i+8i \\ &=& 1+13i\end{array}

Differenz:
\begin{array}{rcl}z_1-z_2 &=& (3+5i)-(-2+8i) \\ &=& 3+2+5i-8i \\ &=& 5-3i\end{array}

Bemerkung: Einzig die Klammern um z_2 bei der Bildung der Differenz waren bei diesen zwei Rechnungen wirklich nötig. Alle anderen hätten auch weggelassen werden können.
Wichtig: Wie immer ändert das Minuszeichen vor der Klammer alle Vor- und Rechenzeichen in der Klammer.

 

Multiplikation und Division

Multiplizieren und dividieren kann man komplexe Zahlen in allen hier vorgestellten Schreibweisen. Dass auch bei den komplexen Zahlen nicht durch 0 geteilt werden darf, versteht sich dabei von selbst.

Für die Division benötigen wir gleich ein spezielles Konzept, welches es bei reellen Zahlen noch nicht gab.
Definition: Zu einer komplexen Zahl z=a+bi heißt \bar{z}=a-bi die konjugiert komplexe Zahl.
In der gaußschen Zahlenebene erhält man die konjugiert komplexe einer Zahl z, indem man z an der reellen Achse spiegelt.

2 komplexe Zahlen und ihre konjugiert Komplexen in der gaußschen Zahlenebene

Bemerkung: In der Grafik sind zwei komplexe Zahlen mit ihren konjugiert Komplexen sowie alle zugehörigen Zeiger eingezeichnet.

Es gilt: \vert z \vert^2=z\cdot \bar{z}
Das Produkt einer komplexen Zahl mit ihrer konjugiert komplexen ist also ihr Betrag "zum Quadrat" und dieses ist immer reell (kann man einfach nachrechnen). Es entstehen ja keine Summanden, die ein einzelnes i enthalten (3. binomische Formel ...) und das i^2 wird (sollte ja inzwischen nicht mehr überraschen ...) zu -1.


Multiplikation und Division in kartesischer Form
Für die kartesische Form gibt es wieder "nur" ein paar Beispiele mit Erläuterungen für die Multiplikation und Division:

Beispiele:
Wir nehmen wieder z_1=3+5i und z_2=-2+8i.

Produkt:
\begin{array}{rcl}z_1 \cdot z_2 &=& (3+5i)\cdot(-2+8i) \\ &=& 3\cdot(-2)+3\cdot 8i-5i\cdot 2+5\cdot 8i^2 \\ &=& -6+24i-10i-40 \\ &=& -46+14i\end{array}
Die Klammern werden ausmultipliziert. Bei der vorletzten Umformung wird benutzt, dass i^2=-1 ist, daher ist 5 \cdot 8i^2 = 5\cdot 8 \cdot (-1)=-40.

Quotient:
\begin{array}{rcl}\dfrac{z_1}{z_2} &=& \dfrac{3+5i}{-2+8i} \\ \\ &=& \dfrac{(3+5i)(-2-8i)}{(-2+8i)(-2-8i)} \\ \\ &=& \dfrac{-6-24i-10i+40}{4-64i^2} \\ \\ &=& \dfrac{34-34i}{4+64} \\ \\ &=& \dfrac{34}{68}-\dfrac{34i}{68} \\ \\ &=& \dfrac{1}{2}-\dfrac{1}{2}i\end{array}
Hier ist ein bisschen mehr zu erklären, insbesondere bei der ersten Umformung: Bislang wurde ja nicht definiert, wie man durch das i teilt - und das ist auch gar nicht nötig. Wir stellen uns stattdessen folgende Frage: Mit welchem Term muss der Bruch erweitert werden, damit das i im Nenner "verschwindet"? Hier kommt die gerade definierte konjugiert komplexe Zahl von z_2 ins Spiel. Multipliziert man nämlich eine komplexe Zahl mit ihrer konjugiert Komplexen ist das Ergebnis immer reell (siehe oben). Es darf natürlich nicht vergessen werden, dass auch der Zähler mit der konjugiert Komplexen von z_2 multipliziert werden muss, sonst wäre es ja kein Erweitern. Anschließend wird ausmultipliziert und zusammengefasst. Zum Abschluss kann die komplexe Zahl im Zähler dann komponentenweise durch den reellen Nenner geteilt werden.
Bemerkung: Ganz ähnlich ist der Rechenweg bei Wurzeltermen, wenn dort Wurzeln im Nenner stehen (siehe z. B. Aufgabe 5.16 im Kapitel Potenzen, Wurzeln, Logarithmen).
Wichtig:
Auch beim Rechnen mit komplexen Zahlen darf man natürlich nicht aus Summen kürzen!


Multiplikation und Division in trigonometrischer und exponentieller Form
Bei Multiplikation und Division kommt eine der Stärken der trigonometrischen und exponentiellen Schreibweisen zum Tragen. Es gilt nämlich für z_1=r_1e^{i\varphi_1} und z_2=r_2e^{i\varphi_2}:

Multiplikation: z_1\cdot z_2=r_1e^{i\varphi_1} \cdot r_2e^{i\varphi_2} = r_1r_2e^{i\varphi_1+i\varphi_2} = r_1r_2e^{i(\varphi_1+\varphi_2)}
Bemerkung: Verwendet wurden einfach die Regeln für die Multiplikation von Potenzen. Es werden also die Beträge multipliziert und die Argumente addiert.

Division: \dfrac{z_1}{z_2}=\dfrac{r_1e^{i\varphi_1}}{r_2e^{i\varphi_2}}=\dfrac{r_1}{r_2}e^{i\varphi_1-i\varphi_2}=\dfrac{r_1}{r_2}e^{i(\varphi_1-\varphi_2)}
Bemerkung: Nach den Regeln der Potenzrechnung wird vermutlich nicht verwundern, dass hier nun die Beträge dividiert und die Argumente subtrahiert werden.

Wichtig: Natürlich gilt auch hier Potenz- geht vor Punkt- geht vor Strichrechnung.


Da Betrag und Argument der exponentiellen Schreibweise die gleichen sind wie bei der trigonometrischen, gilt also hier für z_1=r_1\left(\cos(\varphi_1)+i \sin(\varphi_1)\right) und z_2=r_2\left(\cos(\varphi_2)+i \sin(\varphi_2)\right):

z_1\cdot z_2=r_1r_2\left(\cos(\varphi_1+\varphi_2)+i \sin(\varphi_1+\varphi_2)\right) beziehungsweise

\dfrac{z_1}{z_2}=\dfrac{r_1}{r_2}\left(\cos(\varphi_1-\varphi_2)+i \sin(\varphi_1-\varphi_2)\right)


Beispiele:
Wir nehmen die komplexen Zahlen z_1=\dfrac{1}{2}e^{i\,\frac{5\pi}{4}} und z_2=10e^{i\,\frac{\pi}{6}}.

Produkt: 
\begin{array}{rcl}z_1 \cdot z_2 &=& \dfrac{1}{2}e^{i\,\frac{5\pi}{4}}\cdot 10e^{i\,\frac{\pi}{6}} \\ \\ &=& \dfrac{1}{2}\cdot 10e^{i\,\frac{5\pi}{4}+i\,\frac{\pi}{6}} \\ \\ &=& 5e^{i\,\frac{17\pi}{12}}\end{array}

Quotient:
\begin{array}{rcl}\dfrac{z_1}{z_2} &=& \dfrac{\frac{1}{2}e^{i\,\frac{5\pi}{4}}}{10e^{i\,\frac{\pi}{6}}} \\ \\ &=& \dfrac{\frac{1}{2}}{10}e^{i\,\frac{5\pi}{4}-i\,\frac{\pi}{6}} \\ \\ &=& \dfrac{1}{20}e^{i\,\frac{13\pi}{12}}\end{array}


Wir nehmen nun z_1=3\left(\cos\left(\dfrac{\pi}{3}\right)+i\sin\left(\dfrac{\pi}{3}\right)\right) und z_2=\dfrac{1}{4}\left(\cos\left(\dfrac{5\pi}{9}\right)+i\sin\left(\dfrac{5\pi}{9}\right)\right).

Produkt:
\begin{array}{rcl}z_1\cdot z_2 &=& 3\left(\cos\left(\dfrac{\pi}{3}\right)+i\sin\left(\dfrac{\pi}{3}\right)\right)\cdot\dfrac{1}{4}\left(\cos\left(\dfrac{5\pi}{9}\right)+i\sin\left(\dfrac{5\pi}{9}\right)\right) \\ \\ &=& 3\cdot \dfrac{1}{4}\left(\cos\left(\dfrac{\pi}{3}+\dfrac{5\pi}{9}\right)+i\sin\left(\dfrac{\pi}{3}+\dfrac{5\pi}{9}\right)\right) \\ \\ &=& \dfrac{3}{4}\left(\cos\left(\dfrac{8\pi}{9}\right)+i\sin\left(\dfrac{8\pi}{9}\right)\right)\end{array}

Quotient:
\begin{array}{rcl}\dfrac{z_1}{z_2} &=&\dfrac{3\left(\cos\left(\dfrac{\pi}{3}\right)+i\sin\left(\dfrac{\pi}{3}\right)\right)}{\frac{1}{4}\left(\cos\left(\dfrac{5\pi}{9}\right)+i\sin\left(\dfrac{5\pi}{9}\right)\right)} \\ \\ &=& \dfrac{3}{\frac{1}{4}}\left(\cos\left(\dfrac{\pi}{3}-\dfrac{5\pi}{9}\right)+i\sin\left(\dfrac{\pi}{3}-\dfrac{5\pi}{9}\right)\right) \\ \\ &=&12\left(\cos\left(\dfrac{-2\pi}{9}\right)+i\sin\left(\dfrac{-2\pi}{9}\right)\right) \\ \\ &=& 12\left(\cos\left(\dfrac{16\pi}{9}\right)+i\sin\left(\dfrac{16\pi}{9}\right)\right)\end{array}
Bemerkung: Die letzte Umformung bewirkt, dass das Argument, wie oben "gefordert", im Intervall [0;2\pi[ liegt. Dazu wurde zu dem Argument \frac{-2\pi}{9} einfach 2\pi addiert.


In kartesischer Form ist von den vier Grundrechenarten also nur die Division ein bisschen tricky. Bei Addition, Subtraktion und Multiplikation kann man eigentlich genauso rechnen, wie man es intuitiv tun würde und muss sich nur bei der Multiplikation gelegentlich daran erinnern, dass i^2=-1 ist. Multiplikation und Division von komplexen Zahlen in trigonometrischer bzw. exponentieller Form erfolgt zwar anders, als man wohl vorab vermutet hätte, ist dafür aber sehr einfach.

 

Potenzen und Wurzeln

Zunächst ist wichtig, sich die Potenzen von i zu verdeutlichen. Es gilt:
i^1=i=\sqrt{-1}
i^2=i\cdot i=-1
i^3=i\cdot i\cdot i=-i
i^4=i\cdot i\cdot i\cdot i=1
i^5=i\cdot i\cdot i\cdot i\cdot i=\sqrt{-1}
i^6=i\cdot i\cdot i\cdot i\cdot i\cdot i=-1
...
Mehr passiert nicht. Kann auch gar nicht ...

1., 2., 3. und 4. Potenz von i, dargestellt in der gaußschen Zahlenebene

Das Potenzieren komplexer Zahlen ist grundsätzlich auch in allen vorgestellten Schreibweisen möglich. Üblicherweise möchte man aber komplexe Zahlen in kartesischer Schreibweise nicht potenzieren, weil es unglaublich viel komplizierter als in trigonometrischer oder exponentieller Darstellung ist. Es lohnt sich selbst dann noch, wenn man erst umrechnen muss ...


Ähnlich wie beim Multiplizieren und Dividieren gibt es auch hier eine Formel, mit deren Hilfe sich das Potenzieren komplexer Zahlen auf sehr einfache, reelle Rechenoperationen zurückführen lässt. Es gilt für z=re^{i\varphi} und n\in\mathbb{N}:
z^n=\underbrace{re^{i\varphi} \cdot re^{i\varphi} \cdot \ldots \cdot re^{i\varphi}}_n=r\cdot r\cdot \ldots \cdot re^{i\varphi +i\varphi + \ldots +i\varphi}=r^ne^{i n\varphi}

bzw. für z=r\left(\cos\left(\varphi\right)+i\sin\left(\varphi\right)\right) und n\in\mathbb{N}:
z^n=r^n\left(\cos\left(n\varphi\right)+i\sin\left(n\varphi\right)\right)

Bemerkung 1: Dieser wichtige Zusammenhang heißt Satz von de Moivre nach dem französischen Mathematiker Abraham de Moivre.
Bemerkung 2: Hier erhält man über die Anwendung der Potenzgesetze eine Rechenvorschrift, in der nur noch der Betrag entsprechend potenziert und das Argument mit dem Exponenten multipliziert werden muss. Überlegen Sie sich jetzt, was man tun müsste, um eine komplexe Zahl in kartesischer Darstellung zu potenzieren. Zu berechnen wäre: (a+bi)(a+bi)\ldots(a+bi). Kann man machen, muss man aber nicht ...

Beispiele:
Bei den komplexen Zahlen z_1=\dfrac{1}{2}e^{i\,\frac{5\pi}{4}} und z_2=\dfrac{1}{4}\left(\cos\left(\dfrac{5\pi}{9}\right)+i\sin\left(\dfrac{5\pi}{9}\right)\right) von oben berechnet man:
\begin{array}{rcl}z_1^4 &=& \left(\dfrac{1}{2}\right)^4e^{i\,\cdot 4\cdot\frac{5\pi}{4}} \\ \\ &=& \dfrac{1}{16} e^{i\,\cdot 5\pi} \\ \\ &=& \dfrac{1}{16} e^{i\,\pi} \\ \\ &=& -\dfrac{1}{16}\end{array}


\begin{array}{rcl}z_2^{12} &=& \left(\dfrac{1}{4}\right)^{12}\left(\cos\left(12\cdot\dfrac{5\pi}{9}\right)+i\sin\left(12\cdot\dfrac{5\pi}{9}\right)\right) \\ \\ &=& \dfrac{1}{16.777.216}\left(\cos\left(\dfrac{20\pi}{3}\right)+i\sin\left(\dfrac{20\pi}{3}\right)\right) \\ \\ &=& \dfrac{1}{16.777.216}\left(\cos\left(\dfrac{2\pi}{3}\right)+i\sin\left(\dfrac{2\pi}{3}\right)\right)\end{array}


Als letzte Rechenoperation betrachten wir das Radizieren, also das Ziehen von Wurzeln:
Naheliegend, wenn man vom Satz von de Moivre ausgeht, ist der Gedanke, dass sich die n-te Wurzel (n\in\mathbb{N}) einer komplexen Zahl nach der folgenden Formel berechnen lässt: \sqrt[n]{z}=\sqrt[n]{r}\left(\cos\left(\dfrac{\varphi}{n}\right)+i\sin\left(\dfrac{\varphi}{n}\right)\right)
Das ist nicht grundsätzlich falsch, aber auch nicht vollständig richtig. Hierbei wird nämlich außer Acht gelassen, dass Sinus und Kosinus ja nun periodische Funktionen sind mit der Periode 2\pi. Ersetzt man \varphi durch \varphi+2k\pi mit k\in\mathbb{Z} (addiert man also zum Argument ein Vielfaches von 2\pi), erhält man das gleiche z, aber z. T. unterschiedliche Wurzelwerte. Der Wurzelterm in \mathbb{C} ist also mehrdeutig (im Unterschied zu Wurzeln im Bereich der reellen Zahlen). Man erhält die Formeln:

\sqrt[n]{z}=\sqrt[n]{r}e^{i \, \frac{\varphi+2k\pi}{n}} bzw. \sqrt[n]{z}=\sqrt[n]{r}\left(\cos\left(\dfrac{\varphi+2k\pi}{n}\right)+i\sin\left(\dfrac{\varphi+2k\pi}{n}\right)\right)
mit k=0,1,...,n-1


Für die restlichen wichtigen Eigenschaften, ein Beispiel:
Wir nehmen die komplexe Zahl z=8=8\left(\cos\left(0\right)+i\sin\left(0\right)\right).
Gesucht sind alle dritten Wurzeln von z.

k=0:
\begin{array}{rcl}z_0 &=& \sqrt[3]{8}\left(\cos\left(\dfrac{0+2\cdot 0\cdot\pi}{3}\right)+i\sin\left(\dfrac{0+2\cdot 0\cdot\pi}{3}\right)\right) \\ &=& 2\left(\cos\left(0\right)+i\sin\left(0\right)\right) \\ &=& 2\end{array}
Diese Wurzel kennen wir schon ...

k=1:
\begin{array}{rcl}z_1 &=& \sqrt[3]{8}\left(\cos\left(\dfrac{0+2\cdot 1\cdot\pi}{3}\right)+i\sin\left(\dfrac{0+2\cdot 1\cdot\pi}{3}\right)\right) \\ \\ &=& 2\left(\cos\left(\dfrac{2\pi}{3}\right)+i\sin\left(\dfrac{2\pi}{3}\right)\right)\end{array}

k=2:
\begin{array}{rcl}z_2 &=& \sqrt[3]{8}\left(\cos\left(\dfrac{0+2\cdot 2\cdot\pi}{3}\right)+i\sin\left(\dfrac{0+2\cdot 2\cdot\pi}{3}\right)\right) \\ \\ &=&2\left(\cos\left(\dfrac{4\pi}{3}\right)+i\sin\left(\dfrac{4\pi}{3}\right)\right)\end{array}

Wie bei allen Wurzelrechnungen können wir die Probe machen, indem wir mit dem entsprechenden Wurzelexponenten, hier also 3, potenzieren. Das ergibt in diesem Beispiel:
k=0:
\begin{array}{rcl}z_0^3 &=& 2^3\left(\cos\left(3\cdot 0\right)+i\sin\left(3\cdot 0\right)\right) \\ &=& 8\left(\cos\left(0\right)+i\sin\left(0\right)\right) \\ &=&8\end{array}

k=1:
\begin{array}{rcl}z_1^3 &=& 2^3\left(\cos\left(3\cdot\dfrac{2\pi}{3}\right)+i\sin\left(3\cdot\dfrac{2\pi}{3}\right)\right) \\ &=& 8\left(\cos\left(2\pi\right)+i\sin\left(2\pi\right)\right) \\ &=& 8\left(\cos\left(0\right)+i\sin\left(0\right)\right) \\ &=& 8\end{array}

k=2:
\begin{array}{rcl}z_2^3 &=& 2^3\left(\cos\left(3\cdot\dfrac{4\pi}{3}\right)+i\sin\left(3\cdot\dfrac{4\pi}{3}\right)\right) \\ &=& 8\left(\cos\left(4\pi\right)+i\sin\left(4\pi\right)\right) \\ &=& 8\left(\cos\left(0\right)+i\sin\left(0\right)\right) \\ &=& 8\end{array}

Bemerkung: Radizieren ist im Großen und Ganzen nur möglich, wenn die komplexe Zahl in trigonometrischer oder exponentieller Darstellungsweise vorliegt. Selbst wenn man, wie in diesem Beispiel, zumindest einen Anfang in kartesischer Form machen kann, besteht immer die große Gefahr, dass nicht alle Wurzeln gefunden werden.

Da diese n-ten Wurzeln alle den gleichen Betrag haben (es ändert sich ja nur der Winkel), liegen sie auf einem Kreis um den Ursprung in der gaußschen Zahlenebene. Wenn man sie mit geraden Linien verbindet, bilden sie ein regelmäßiges n-Eck. Das "2-Eck", das sich für n=2 ergibt, ist eine Strecke.

die 3 dritten Wurzeln von 8, dargestellt in der gaußschen Zahlenebene

Die erste Wurzel, auf die man stößt, wenn man sich entgegengesetzt des Uhrzeigersinns (mathematisch positiv) durch die gaußsche Zahlenebene bewegt, heißt Hauptwert und wird meist mit z_0 bezeichnet. Zwischen den verschiedenen Wurzelwerten liegt jeweils der Winkel \dfrac{2\pi}{n}, wobei mit n der Wurzelexponenten gemeint ist.


Bleibt die Frage, warum es ausreicht k=0,1,...,n-1 zu betrachten. Der Grund ist, dass sich für k\geq n wieder die gleichen Werte ergeben. Schauen wir uns an, was im obigen Beispiel passieren würde:
k=3:
\begin{array}{rcl}z_3 &=& \sqrt[3]{8}\left(\cos\left(\dfrac{0+2\cdot 3\cdot\pi}{3}\right)+i\sin\left(\dfrac{0+2\cdot 3\cdot\pi}{3}\right)\right) \\ &=& 2\left(\cos\left(2\pi\right)+i\sin\left(2\pi\right)\right) \\ &=& 2\left(\cos\left(0\right)+i\sin\left(0\right)\right) \\ &=& z_0\end{array}

k=4:
\begin{array}{rcl} z_4 &=& \sqrt[3]{8}\left(\cos\left(\dfrac{0+2\cdot 4\cdot\pi}{3}\right)+i\sin\left(\dfrac{0+2\cdot 4\cdot\pi}{3}\right)\right) \\ \\ &=& 2\left(\cos\left(\dfrac{8\pi}{3}\right)+i\sin\left(\dfrac{8\pi}{3}\right)\right) \\ \\ &=& 2\left(\cos\left(\dfrac{2\pi}{3}\right)+i\sin\left(\dfrac{2\pi}{3}\right)\right) \\ \\ &=& z_1\end{array}

k=5:
\begin{array}{rcl}z_5 &=& \sqrt[3]{8}\left(\cos\left(\dfrac{0+2\cdot 5\cdot\pi}{3}\right)+i\sin\left(\dfrac{0+2\cdot 5\cdot\pi}{3}\right)\right) \\ \\ &=& 2\left(\cos\left(\dfrac{10\pi}{3}\right)+i\sin\left(\dfrac{10\pi}{3}\right)\right) \\ \\ &=& 2\left(\cos\left(\dfrac{4\pi}{3}\right)+i\sin\left(\dfrac{4\pi}{3}\right)\right) \\ \\ &=& z_2\end{array}

Sie können sich vermutlich vorstellen, wie es weitergeht ...


Fazit: Die Zahl z hat also genau n komplexe n-te Wurzeln. Damit schließt sich der Kreis zum Anfang des Kapitels und dem dort erwähnten Fundamentalsatz der Algebra.

 

Zum Abschluss

Sie haben (hoffentlich) gemerkt, dass komplexe Zahlen, wenn man einmal dieses i in der Zahl akzeptiert hat, sooooo geheimnisvoll dann auch wieder nicht sind. Natürlich ist dies nur eine ganz grundlegende Einführung, aber der Grundsatz bei Zahlenbereichserweiterungen, dass gewohnte Rechengesetze weiter gelten sollen, bedeutet hier eben auch, dass z. B. der Satz des Pythagoras oder die Definitionen der Trigonometrie hier genauso und unter den gleichen Bedingungen angewendet werden dürfen wie vorher. Warum hätte man das auch ändern sollen?

Übersicht:

 

24.3 Komplexe Zahlen - Lösungen

1. Aufgabe

Bemerkung: Bei den folgenden Lösungen wurde das Gradmaß verwendet, weil sich keine "vernünftigen" Vielfache von \pi ergeben.

1) z_1=-10+7i
Betrag: r=\sqrt{10^2+7^2}=\sqrt{149}\approx 12{,}21
Argument: \varphi=180^\circ-\arctan\left(\dfrac{7}{10}\right)\approx 145{,}01^\circ

Daraus ergibt sich folgendes:
Trigonometrische Darstellung: z_1\approx 12{,}21\left(\cos\left(145{,}01^\circ\right)+i\sin\left(145{,}01^\circ\right)\right)
Exponentielle Darstellung: z_1\approx 12{,}21\,e^{i\,145{,}01^\circ}
 
2) z_2=-\dfrac{-18+37i}{8}=-\left(-\dfrac{18}{8}\right)-\dfrac{37}{8}i=\dfrac{9}{4}-\dfrac{37}{8}i
Betrag: r=\sqrt{\left(\dfrac{9}{4}\right)^2+\left(\dfrac{37}{8}\right)^2}\approx 5{,}14
Argument: \varphi=360^\circ-\arctan\left(\dfrac{37}{8}\cdot\dfrac{4}{9}\right)\approx 295{,}94^\circ

Daraus ergibt sich folgendes:
Trigonometrische Darstellung: z_2\approx 5{,}14\left(\cos\left(295{,}94^\circ\right)+i\sin\left(295{,}94^\circ\right)\right)
Exponentielle Darstellung: z_2\approx 5{,}14e^{i\, 295{,}94^\circ}
 
3) z_3=-4-\dfrac{21}{5}i
Betrag: r=\sqrt{4^2+\left(\dfrac{21}{5}\right)^2}=5{,}8
Argument: \varphi=180^\circ+\arctan\left(\dfrac{21}{5}\cdot\dfrac{1}{4}\right)\approx 226{,}40^\circ

Daraus ergibt sich folgendes:
Trigonometrische Darstellung: z_3\approx 5{,}8 \left(\cos\left(226{,}40^\circ\right)+i\sin\left(226{,}40^\circ\right)\right)
Exponentielle Darstellung: z_3\approx 5{,}8e^{i\, 226{,}40^\circ}
 
4) z_4=\dfrac{13+3i}{2}=\dfrac{13}{2}+\dfrac{3}{2}i
Betrag: r=\sqrt{\left(\dfrac{13}{2}\right)^2+\left(\dfrac{3}{2}\right)^2}\approx 6{,}67
Argument: \varphi=\arctan\left(\dfrac{3}{2}\cdot\dfrac{2}{13}\right)\approx 12{,}99^\circ

Daraus ergibt sich folgendes:
Trigonometrische Darstellung: z_4\approx 6{,}67\left(\cos\left(12{,}99^\circ\right)+i\sin\left(12{,}99^\circ\right)\right)
Exponentielle Darstellung: z_4\approx 6{,}67e^{i \, 12{,}99^\circ}
 
5) z_5=-\dfrac{22}{3}-\dfrac{19}{4}i
Betrag: r=\sqrt{\left(\dfrac{22}{3}\right)^2+\left(\dfrac{19}{4}\right)^2}\approx 8{,}74
Argument: \varphi=180^\circ+\arctan\left(\dfrac{19}{4}\cdot\dfrac{3}{22}\right)\approx 212{,}93^\circ

Daraus ergibt sich folgendes:
Trigonometrische Darstellung: z_5\approx 8{,}74\left(\cos\left(212{,}93^\circ\right)+i\sin\left(212{,}93^\circ\right)\right)
Exponentielle Darstellung: z_5\approx 8{,}74e^{i\, 212{,}93^\circ}

z1 bis z5 in der gaußschen Zahlenebene

 

6) z_6=6\left(\cos\left(\dfrac{3\pi}{5}\right)+i\sin\left(\dfrac{3\pi}{5}\right)\right)
Die exponentielle Form lässt sich ohne Rechnen ermitteln, nämlich: z_6=6e^{i\,\frac{3\pi}{5}}
Für die kartesische Form ergibt sich \textit{Re}(z_6)=6\cos\left(\dfrac{3\pi}{5}\right)\approx -1{,}85 und  \textit{Im}(z_6)=6\sin\left(\dfrac{3\pi}{5}\right)\approx 5{,}70 und daraus  z_6\approx -1{,}85+5{,}70i
 
7) z_7=\dfrac{12}{7}\left(\cos\left(\dfrac{\pi}{2}\right)+i\sin\left(\dfrac{\pi}{2}\right)\right)
Die exponentielle Form lässt sich ohne Rechnen ermitteln, nämlich: z_7=\frac{12}{7}\,e^{i\,\frac{\pi}{2}}
Für die kartesische Form ergibt sich \textit{Re}(z_7)=\dfrac{12}{7}\cos\left(\dfrac{\pi}{2}\right)=0 und \textit{Im}(z_7)=\dfrac{12}{7}\sin\left(\dfrac{\pi}{2}\right)=\dfrac{12}{7} und daraus z_7=\dfrac{12}{7}i
 
8) z_8=\dfrac{30}{11}\left(\cos\left(\dfrac{\pi}{6}\right)+i\sin\left(\dfrac{\pi}{6}\right)\right)
Die exponentielle Form lässt sich ohne Rechnen ermitteln, nämlich: z_8=\frac{30}{11}\,e^{i\,\frac{\pi}{6}}
Für die kartesische Form ergibt sich \textit{Re}(z_8)=\dfrac{30}{11}\cos\left(\dfrac{\pi}{6}\right)=\dfrac{15\sqrt{3}}{11} und \textit{Im}(z_8)=\dfrac{30}{11}\sin\left(\dfrac{\pi}{6}\right)=\dfrac{15}{11} und daraus z_8=\dfrac{15\sqrt{3}}{11}+\dfrac{15}{11}i
 
9) z_9=\dfrac{49}{10}\left(\cos\left(\dfrac{9\pi}{8}\right)+i\sin\left(\dfrac{9\pi}{8}\right)\right)
Die exponentielle Form lässt sich ohne Rechnen ermitteln, nämlich: z_9=\dfrac{49}{10}\,e^{i\,\frac{9\pi}{8}}
Für die kartesische Form ergibt sich \textit{Re}(z_9)=\dfrac{49}{10}\cos\left(\dfrac{9\pi}{8}\right)\approx -4{,}53 und \textit{Im}(z_9)=\dfrac{49}{10}\sin\left(\dfrac{9\pi}{8}\right)\approx -1{,}88 und daraus z_9\approx -4{,}53-1{,}88i
 
10) z_{10}=\dfrac{14}{3}\left(\cos\left(\dfrac{5\pi}{2}\right)+i\sin\left(\dfrac{5\pi}{2}\right)\right)
Die exponentielle Form lässt sich ohne Rechnen ermitteln, nämlich: z_{10}=\dfrac{14}{3}\,e^{i\,\frac{5\pi}{2}}
Für die kartesische Form ergibt sich \textit{Re}(z_{10})=\dfrac{14}{3}\cos\left(\dfrac{5\pi}{2}\right)=0 und \textit{Im}(z_{10})=\dfrac{14}{3}\sin\left(\dfrac{5\pi}{2}\right)=\dfrac{14}{3} und daraus z_{10}=\dfrac{14}{3}i

z6 bis z10 in der gaußschen Zahlenebene

 

11) z_{11}=8e^{i\,\frac{17\pi}{10}}
Hier kann man die trigonometrische Form quasi ablesen: z_{11}=8\left(\cos\left(\dfrac{17\pi}{10}\right)+i\sin\left(\dfrac{17\pi}{10}\right)\right)
Für die kartesische Form berechnet man \textit{Re}(z_{11})=8\cos\left(\dfrac{17\pi}{10}\right)\approx 4{,}70 und \textit{Im}(z_{11})=8\sin\left(\dfrac{17\pi}{10}\right)\approx -6{,}47 und erhält damit z_{11}\approx 4{,}70-6{,}47i
 
12) z_{12}=\dfrac{33}{4}\,e^{i\,\frac{41\pi}{22}}
Hier kann man die trigonometrische Form quasi ablesen: z_{12}=\dfrac{33}{4}\left(\cos\left(\dfrac{41\pi}{22}\right)+i\sin\left(\dfrac{41\pi}{22}\right)\right)
Für die kartesische Form berechnet man: \textit{Re}(z_{12})=\dfrac{33}{4}\cos\left(\dfrac{41\pi}{22}\right)\approx 7{,}50 und \textit{Im}(z_{12})=\dfrac{33}{4}\sin\left(\dfrac{41\pi}{22}\right)\approx -3{,}43 und erhält damit z_{12}\approx 7{,}50-3{,}43i
 
13) z_{13}=\dfrac{27}{7}\,e^{i\,3\pi}
Hier kann man die trigonometrische Form quasi ablesen: z_{13}=\dfrac{27}{7}\left(\cos\left(3\pi\right)+i\sin\left(3\pi\right)\right)
Für die kartesische Form berechnet man: \textit{Re}(z_{13})=\dfrac{27}{7}\cos\left(3\pi\right)=-\dfrac{27}{7} und \textit{Im}(z_{13})=\dfrac{27}{6}\sin\left(3\pi\right)=0 und erhält damit z_{13}=-\dfrac{27}{7}
 
14) z_{14}=6e^{i\,\frac{7\pi}{9}}
Hier kann man die trigonometrische Form quasi ablesen: z_{14}=6\left(\cos\left(\dfrac{7\pi}{9}\right)+i\sin\left(\dfrac{7\pi}{9}\right)\right)
Für die kartesische Form berechnet man: \textit{Re}(z_{14})=6\cos\left(\dfrac{7\pi}{9}\right)\approx -4{,}60 und \textit{Im}(z_{14})=6\sin\left(\dfrac{7\pi}{9}\right)\approx 3{,}86 und erhält damit z_{14}\approx -4{,}60+3{,}86i
 
15) z_{15}=\dfrac{9}{2}\,e^{i\,8\pi}
Hier kann man die trigonometrische Form quasi ablesen: z_{15}=\dfrac{9}{2}\left(\cos\left(8\pi\right)+i\sin\left(8\pi\right)\right)
Für die kartesische Form berechnet man: \textit{Re}(z_{15})=\dfrac{9}{2}\cos\left(8\pi\right)= \dfrac{9}{2} und \textit{Im}(z_{15})=\dfrac{9}{2}\sin\left(8\pi\right)=0 und erhält damit z_{15}=\dfrac{9}{2}

z11 bis z15 in der gaußschen Zahlenebene

 

2. Aufgabe

Hinweis 1: Basale Umformungen, wie das Erweitern und Kürzen von Brüchen, das Zusammenfassen von Zahlenwerten, das Auflösen von Klammern etc. wurde hier nicht mehr extra aufgeschrieben und ausgewiesen. Bitte denken Sie daran, dass die "normalen" Regeln der Bruchrechnung auch hier gelten!
Hinweis 2: Auch hier gilt (nach wie vor) "Potenz- vor Punkt- vor Strichrechnung". Das bedeutet zum Beispiel: -10 und 10i lassen sich nicht zusammenfassen!

a)

1)
 (7 \; ; \; 4) + (12 \; ; \; -35 ) \; = \; (7+12 \; ; \; 4-35) \; = \; (19 \; ; \; -31)


2)
 -\dfrac{1}{21} + \dfrac{2}{5}i + \dfrac{2}{3} + \dfrac{7} {8}i \; = \; -\dfrac{1}{21}+\dfrac{2}{3}+\left(\dfrac{2}{5}+\dfrac{7}{8}\right)i \; = \; \dfrac{13}{21}+\dfrac{51}{40}i


3)
 \left(\dfrac{3}{4} \; ; \; -5 \right) - \left(\dfrac{11}{8} \; ; \; \dfrac{5} {6} \right) \; = \; \left(\dfrac{3}{4}-\dfrac{11}{8} \; ; \; -5-\dfrac{5}{6}\right) \; = \; \left(-\dfrac{5}{8} \; ; \; -\dfrac{35}{6}\right)


4)
 7-4i - (18-3i) \; = \; 7-18+(-4+3)i \; = \; -11-i


5)
(10-2i) \cdot (3-6i) \; = \; 30-60i-6i+12i^2 \; = \; 18-66i


6)
 \left(-\dfrac{1}{2} + \dfrac{1}{2}i \right) \cdot \left(\dfrac{7}{20} - \dfrac{1}{10}i \right) \; = \; -\dfrac{7}{40}+\dfrac{1}{20}i+\dfrac{7}{40}i-\dfrac{1}{20}i^2 \; = \; -\dfrac{1}{8}+\dfrac{9}{40}i


7)
 (11-10i) \cdot (-5-2i) \; = \; -55-22i+50i+20i^2 \; = \; -75+28i


8)
 \dfrac{11-8i}{-4i} \; = \; \dfrac{(11-8i)i}{(-4i)i} \; = \; \dfrac{11i-8i^2} {-4i^2} \; = \; \dfrac{11i+8}{4} \; = \; 2 + \dfrac{11}{4}i


9)
{ \dfrac{i-10}{-1-i} \; = \; \dfrac{(-10+i)(-1+i)}{(-1-i)(-1+i)} \; = \; \dfrac{10-10i-i+i^2}{(-1)^2-i^2} \; = \; \dfrac{9-11i}{2} \; = \; \dfrac{9}{2} - \dfrac{11} {2} i }


10)
{ \dfrac{4}{2+\sqrt{2}i} \; = \; \dfrac{4(2-\sqrt{2}i)}{(2+\sqrt{2}i)(2-\sqrt{2}i)} \; = \; \dfrac{8-4\sqrt{2}i}{2^2-(\sqrt{2}i)^2} \; = \; \dfrac{2(4-2\sqrt{2}i)}{6} \; = \; \dfrac{4} {3} - \dfrac{2\sqrt{2}}{3}i}

Bemerkung: Bei der letzten Umformung wurde zusätzlich gekürzt.

b)

Bemerkung 1: Für diese Aufgaben benötigt man den Satz von de Moivre.

Bemerkung 2: Bitte beachten Sie, dass bei einigen Aufgaben eine letzte Umformung nötig wird, weil laut Aufgabenstellung gefordert ist, dass  \varphi \; \in \; [0 \, ; \, 2\pi[ .


1)
{ \left(\cos(\pi) + i \sin(\pi)\right) \cdot 7 \left( \cos(\pi)+i \sin(\pi) \right) \; = \; 7(\cos(\pi+\pi)+i\sin(\pi+\pi)) \; = \; 7(\cos(2\pi)+i\sin(2\pi)) \; = \; (7 \; ; \; 0) }


2)
{ \dfrac{1}{8} \left(\cos\left(\dfrac{\pi}{4} \right)+i \sin\left(\dfrac{\pi}{4} \right) \right) \; \cdot \; \dfrac{12}{11} \left(\cos\left(\dfrac{2\pi}{5} \right)+i \sin\left(\dfrac{2\pi} {5} \right) \right) \; = \; \dfrac{1}{8} \cdot \dfrac{12}{11} \left(\cos\left(\dfrac{\pi}{4}+\dfrac{2\pi}{5} \right)+i \sin\left(\dfrac{\pi}{4}+\dfrac{2\pi}{5} \right) \right) \; = \; \dfrac{3}{22} \left(\cos\left(\dfrac{13\pi}{20}\right)+i \sin\left(\dfrac{13\pi}{20}\right)\right)}


3)
{\dfrac{7}{5}\left(\cos\left(\dfrac{7\pi}{5}\right)+i\sin\left(\dfrac{7\pi}{5}\right)\right) \; \cdot \; \dfrac{20}{21}\left(\cos\left(\dfrac{21\pi}{20}\right)+i\sin\left(\dfrac{21\pi}{20}\right)\right) \; = \; \dfrac{7}{5} \cdot \dfrac{20}{21} \left(\cos\left(\dfrac{7\pi}{5}+\dfrac{21\pi}{20} \right)+i \sin\left(\dfrac{7\pi}{5}+\dfrac{21\pi}{20} \right) \right) \; = \; \dfrac{4}{3} \left(\cos\left(\dfrac{49\pi}{20}\right)+i \sin\left(\dfrac{49\pi}{20}\right)\right) \; = \; \dfrac{4}{3} \left(\cos\left(\dfrac{9\pi}{20}\right)+i \sin\left(\dfrac{9\pi}{20}\right)\right)}


4)
 5e^{i \, \frac{\pi}{10}} \; \cdot \; 12 e^{i \, \frac{\pi}{8}} \; = \; 5 \cdot 12 \, e^{i \, \left(\frac{\pi}{10}+\frac{\pi}{8}\right)} \; = \; 60 e^{i \, \frac{9 \pi}{40}}


5)
 \dfrac{4}{7} \, e^{i \frac{5\pi}{6}} \; \cdot \; \dfrac{21}{8} \, e^{i \, \frac{2\pi}{3}} \; = \; \dfrac{4}{7} \, \cdot \, \dfrac{21}{8} \, e^{i \, \left(\frac{5\pi}{6}+\frac{2\pi}{3}\right)} \; = \; \dfrac{3}{2} \, e^{i \, \frac{3\pi}{2}} \; = \; -\dfrac{3}{2} i


6)
\dfrac{12}{5} \, e^{i \, \frac{3\pi}{10}} \cdot 15 e^{i \, \frac{5\pi}{12}} \; = \; \dfrac{12}{5} \cdot 15 \, e^{i \, \left(\frac{3\pi}{10}+\frac{5\pi}{12}\right)} \; = \; 36 e^{i \, \frac{43 \pi}{60}}


7)
{ 4 \left(\cos\left(\dfrac{3\pi}{4} \right)+i \sin\left(\dfrac{3\pi}{4} \right) \right) \; : \; \left( \dfrac{2}{5} \left(\cos\left(\dfrac{5\pi}{12}\right)+i \sin\left(\dfrac{5\pi} {12} \right) \right) \right) \; = \; 4\cdot\dfrac{5}{2} \left(\cos\left(\dfrac{3\pi}{4}-\dfrac{5\pi}{12} \right)+i \sin\left(\dfrac{3\pi}{4}-\dfrac{5\pi}{12} \right) \right) \; = \; 10 \, \left(\cos\left(\dfrac{\pi}{3}\right)+i \sin\left(\dfrac{\pi} {3}\right)\right) }


8)
{ \dfrac{8}{9} \left(\cos\left(\dfrac{\pi}{2} \right)+i \sin\left(\dfrac{\pi}{2} \right) \right) \; : \; \left( \dfrac{16}{27} \left(\cos\left(\dfrac{3\pi}{10} \right)+i \sin\left(\dfrac{3\pi}{10} \right) \right) \right)\; = \; \dfrac{8}{9} \cdot \, \dfrac{27}{16} \left(\cos\left(\dfrac{\pi}{2}-\dfrac{3\pi}{10} \right)+i \sin\left(\dfrac{\pi}{2}-\dfrac{3\pi}{10} \right) \right) \; = \; \dfrac{3}{2} \left(\cos\left(\dfrac{\pi}{5}\right)+i \sin\left(\dfrac{\pi}{5}\right)\right)}

 
9)
{ \dfrac{45} {2}\left(\cos\left(\dfrac{3\pi}{5} \right)+i \sin\left(\dfrac{3\pi}{5} \right) \right) \; : \; \left(9\left(\cos\left(\dfrac{4\pi}{9}\right)+i \sin\left(\dfrac{4\pi} {9} \right)\right)\right) \; = \; \dfrac{45}{2} \cdot \, \dfrac{1}{9} \left(\cos\left(\dfrac{3\pi}{5}-\dfrac{4\pi}{9} \right)+i \sin\left(\dfrac{3\pi}{5}-\dfrac{4\pi}{9} \right) \right) \; = \; \dfrac{5}{2} \left(\cos\left(\dfrac{7\pi}{45}\right)+i \sin\left(\dfrac{7\pi} {45} \right)\right)}


10)
 5 e^{i \, \frac{\pi}{10}} \; : \; \left( 12 e^{i \, \frac{\pi}{8}} \right) \; = \; \dfrac{5}{12} \, e^{i \, \left(\frac{\pi}{10}-\frac{\pi}{8}\right)} \; = \; \dfrac{5}{12} \, e^{i \, \left(-\frac{1\pi} {40}\right) } \; = \; \dfrac{5}{12} \, e^{i \, \frac{79\pi} {40} }


11)
 \dfrac{3}{8} \, e^{i \, \frac{3\pi}{4}} \; : \; \left(\dfrac{9}{16} \, e^{i \, \frac{\pi}{6}} \right) \; = \; \dfrac{3}{8} \cdot \dfrac{16}{9} \, e^{i \, \left(\frac{3\pi}{4}-\frac{\pi}{6}\right)} \; = \; \dfrac{2}{3} \, e^{i \, \frac{7\pi}{12}}


12)
 \dfrac{1}{51} \, e^{i \, \frac{2\pi}{3}} \; : \; \left(\dfrac{2}{17} \, e^{i \, \frac{3\pi} {4}} \right) \; = \; \dfrac{1}{51} \cdot \dfrac{17}{2} \, e^{i \, \left(\frac{2\pi}{3}-\frac{3\pi}{4}\right)} \; = \; \dfrac{1}{6} \, e^{i \, \left(-\frac{1\pi}{12}\right)} \; = \; \dfrac{1}{6} \, e^{i \, \frac{23\pi}{12}}

  

3. Aufgabe

Bemerkung: Auch hier werden einige letzte Umformungen nötig, weil gefordert ist, dass  \varphi \; \in \; [0 \, ; \, 2\pi[ .

1)
{ \left(\dfrac{1}{2}\left(\cos\left(\dfrac{\pi}{8} \right)+i \sin\left(\dfrac{\pi}{8} \right) \right) \right)^4 \; = \; \left(\dfrac{1}{2}\right)^4 \left(\cos\left(4 \cdot \dfrac{\pi}{8} \right)+i \sin\left(4 \cdot \dfrac{\pi}{8} \right) \right) \; = \; \dfrac{1}{16} \left(\cos\left(\dfrac{\pi}{2}\right)+i \sin\left(\dfrac{\pi}{2}\right) \right) \; = \; \dfrac{1}{16}i}


2)
 {\left(\dfrac{2}{3}\left(\cos\left(\dfrac{7\pi}{12}\right)+i \sin\left(\dfrac{7\pi}{12}\right)\right)\right)^3 \; = \; \left(\dfrac{2}{3}\right)^3 \left(\cos\left(3 \cdot \dfrac{7\pi}{12}\right)+i \sin\left(3 \cdot \dfrac{7\pi}{12}\right)\right) \; = \; \dfrac{8}{27} \left( \cos\left( \dfrac{7 \pi} {4}\right)+i \sin \left( \dfrac{7\pi} {4} \right) \right)}


3)
 \left(8 \, e^{i \, \frac{4\pi}{3}}\right)^2 \; = \; 8^2 \, e^{2 \cdot i \, \frac{4\pi}{3}} \; = \; 64 \, e^{i \, \frac{8\pi}{3}} \; = \; 64 \, e^{i \, \frac{2\pi}{3}}


4)
 \left(2 \, e^{i \, \frac{7\pi}{5}}\right)^5 \; = \; 2^5 \, e^{5 \cdot i \, \frac{7\pi}{5}}\; = \; 32 \, e^{i \cdot 7\pi} \; = \; 32 \, e^{i\pi} \; = \; -32

Bemerkung: Für die letzte Umformung wurde die eulersche Identität e^{i \, \pi}=-1 verwendet.


5)
 \sqrt[3]{64(\cos(6\pi)+i \sin(6 \pi))}

{z_0 \; = \; \sqrt[3]{64} \, \left(\cos\left(\dfrac{6\pi+2 \cdot 0 \cdot \pi}{3}\right)+i \sin\left(\dfrac{6\pi+2 \cdot 0 \cdot \pi}{3}\right)\right) \; = \; 4(\cos(2\pi)+i \sin(2\pi)) \; = \; 4 }
{z_1 \; = \; \sqrt[3]{64} \, \left(\cos\left(\dfrac{6\pi+2 \cdot 1 \cdot \pi}{3}\right)+i \sin\left(\dfrac{6\pi+2 \cdot 1 \cdot \pi}{3}\right) \right) \; = \; 4 \left(\cos\left(\dfrac{8\pi}{3}\right)+i \sin\left(\dfrac{8\pi}{3}\right)\right) \; = \; 4\left(\cos\left(\dfrac{2\pi}{3}\right)+i \cdot \sin\left(\dfrac{2\pi}{3}\right)\right) }
{z_2 \; = \; \sqrt[3]{64} \, \left(\cos\left(\dfrac{6\pi+2 \cdot 2 \cdot \pi}{3}\right)+i \sin\left(\dfrac{6\pi+2 \cdot 2 \cdot \pi}{3}\right) \right) \; = \; 4\left(\cos\left(\dfrac{10\pi}{3}\right)+i \sin\left(\dfrac{10\pi}{3}\right)\right) \; = \; 4\left(\cos\left(\dfrac{4\pi}{3}\right)+i \sin\left(\dfrac{4\pi}{3}\right)\right) }


6)
 \sqrt{\dfrac{1}{49}\left(\cos\left(\dfrac{2\pi}{3}\right)+i \sin\left(\dfrac{2\pi}{3}\right)\right)}

{z_0 \; = \; \sqrt{\dfrac{1}{49}}\left(\cos\left(\dfrac{\frac{2\pi}{3}+2 \cdot 0\cdot \pi}{2}\right)+i \sin\left(\dfrac{\frac{2\pi}{3}+2 \cdot 0\cdot \pi}{2}\right)\right) \; = \; \dfrac{1}{7}\left(\cos\left(\dfrac{\pi}{3}\right)+i \sin\left(\dfrac{\pi}{3}\right)\right) }
{z_1 \; = \; \sqrt{\dfrac{1}{49}}\left(\cos\left(\dfrac{\frac{2\pi}{3}+2 \cdot 1 \cdot \pi}{2}\right)+i \sin\left(\dfrac{\frac{2\pi}{3}+2 \cdot 1 \cdot \pi}{2}\right)\right) \; = \; \dfrac{1}{7}\left(\cos\left(\dfrac{4\pi}{3}\right)+i \sin\left(\dfrac{4\pi}{3}\right)\right) }


7)
 \sqrt[4]{\dfrac{1}{256}\left(\cos\left(\dfrac{8\pi}{5} \right)+i\sin\left(\dfrac{8\pi}{5} \right) \right)}

{z_0 \; = \; \sqrt[4]{\dfrac{1}{256}} \, \left(\cos\left(\dfrac{\frac{8\pi}{5}+2 \cdot 0 \cdot \pi}{4}\right)+i \sin\left(\dfrac{\frac{8\pi}{5}+2 \cdot 0 \cdot \pi}{4}\right)\right) \; = \; \dfrac{1}{4} \, \left(\cos\left(\dfrac{2\pi}{5}\right)+i \sin\left(\dfrac{2\pi}{5}\right)\right)}
{z_1 \; = \; \sqrt[4]{\dfrac{1}{256}} \, \left(\cos\left(\dfrac{\frac{8\pi}{5}+2 \cdot 1 \cdot \pi}{4}\right)+i \sin\left(\dfrac{\frac{8\pi}{5}+2 \cdot 1 \cdot \pi}{4}\right) \right) \; = \; \dfrac{1}{4} \, \left(\cos\left(\dfrac{9\pi}{10}\right)+i \sin\left(\dfrac{9\pi}{10}\right)\right)}
{z_2 \; = \; \sqrt[4]{\dfrac{1}{256}} \, \left(\cos\left(\dfrac{\frac{8\pi}{5}+2 \cdot 2 \cdot \pi}{4}\right)+i \sin\left(\dfrac{\frac{8\pi}{5}+2 \cdot 2 \cdot \pi}{4}\right) \right) \; = \; \dfrac{1}{4} \, \left(\cos\left(\dfrac{7\pi}{5}\right)+i \sin\left(\dfrac{7\pi}{5}\right)\right)}
{z_3 \; = \; \sqrt[4]{\dfrac{1}{256}} \, \left(\cos\left(\dfrac{\frac{8\pi}{5}+2 \cdot 3 \cdot \pi}{4}\right)+i \sin\left(\dfrac{\frac{8\pi}{5}+2 \cdot 3 \cdot \pi}{4}\right) \right) \; = \; \dfrac{1}{4} \, \left(\cos\left(\dfrac{19\pi}{10}\right)+i \sin\left(\dfrac{19\pi}{10}\right)\right)}


8)
 \sqrt{\dfrac{81} {4} \, e^{i \, \frac{\pi}{5}}}

z_0 \; = \; \sqrt{\dfrac{81} {4}} \, e^{i \, \frac{\frac{\pi}{5}+2 \cdot 0 \cdot \pi}{2}} \; = \; \dfrac{9}{2} \, e^{i \, \frac{\pi}{10}}
z_1 \; = \; \sqrt{\dfrac{81} {4}} \, e^{i \, \frac{\frac{\pi}{5}+2 \cdot 1 \cdot \pi}{2}} \; = \; \dfrac{9}{2} \, e^{i \, \frac{11\pi}{10}}


9)
 \sqrt[3]{e^{i \, \frac{3\pi}{4}}}

z_0 \; = \; e^{i \, \frac{\frac{3\pi}{4}+2 \cdot 0 \cdot \pi}{3}} \; = \; e^{i \, \frac{\pi}{4}}
z_1 \; = \; e^{i \, \frac{\frac{3\pi}{4}+2 \cdot 1 \cdot \pi}{3}} \; = \; e^{i \, \frac{11\pi}{12}}
z_2 \; = \; e^{i \, \frac{\frac{3\pi}{4}+2 \cdot 2 \cdot \pi}{3}} \; = \; e^{i \, \frac{19\pi}{12}}


10)
 \sqrt[5]{32 \, e^{i \, \frac{\pi}{3}}}

z_0 \; = \; \sqrt[5]{32} \, e^{i \, \frac{\frac{\pi}{3}+2 \cdot 0 \cdot \pi}{5}} \; = \; 2 \, e^{i \, \frac{\pi}{15}}
z_1 \; = \; \sqrt[5]{32} \, e^{i \, \frac{\frac{\pi}{3}+2 \cdot 1 \cdot \pi}{5}} \; = \; 2 \, e^{i \, \frac{7\pi}{15}}
z_2 \; = \; \sqrt[5]{32} \, e^{i \, \frac{\frac{\pi}{3}+2 \cdot 2 \cdot \pi}{5}} \; = \; 2 \, e^{i \, \frac{13\pi}{15}}
z_3 \; = \; \sqrt[5]{32} \, e^{i \, \frac{\frac{\pi}{3}+2 \cdot 3 \cdot \pi}{5}} \; = \; 2 \, e^{i \, \frac{19\pi}{15}}
z_4 \; = \; \sqrt[5]{32} \, e^{i \, \frac{\frac{\pi}{3}+2 \cdot 4 \cdot \pi}{5}} \; = \; 2 \, e^{i \, \frac{5\pi}{3}}

 

4. Aufgabe

Bemerkung: Es gibt absichtlich keine umfangreichen Erklärungen zu diesen Aufgaben, denn Gleichungen bleiben Gleichungen und komplexe Zahlen sind in erster Linie einmal Zahlen und erst in zweiter Linie komplex. Insofern ist hier vorab nicht viel zu sagen. Rechnen Sie einfach drauf los!
Wer Schwierigkeiten hat, schaue in den Kapiteln Quadratische Gleichungen bzw. Polynome nach.

1)
\begin{array}{rclcl} -3x^2+6x-8 &=& 0 &\vert& :(-3) \cr x^2-2x-\dfrac{8}{3} &=& 0 &\vert& \text{p-q-Formel} \cr x_{1,2} &=& 1 \pm \sqrt{1-\dfrac{8}{3}} \cr &=& 1 \pm \sqrt{-\dfrac{5}{3}} \cr \cr x_1 &=& 1+i\sqrt{\dfrac{5}{3}} \cr x_2 &=& 1-i\sqrt{\dfrac{5}{3}} \cr \cr \mathbb{L} &=& \left\{1+i\sqrt{\dfrac{5}{3}} \; ; \; 1-i\sqrt{\dfrac{5}{3}}\right\} \end{array}


2)
\begin{array}{rclcl} \dfrac{2}{3}x^2+\dfrac{289}{3} &=& 16x &\vert& -16x \cr \dfrac{2}{3}x^2-16x+\dfrac{289}{3} &=& 0 &\vert& : \dfrac{2}{3} \cr x^2-24x+\dfrac{289}{2} &=& 0 &\vert& \text{p-q-Formel} \cr x_{1,2} &=& 12 \pm \sqrt{144-\dfrac{289}{2}} \cr &=& 12 \pm \sqrt{-\dfrac{1}{2}} \cr \cr x_1 &=& 12+\dfrac{i}{\sqrt{2}} \cr x_2 &=& 12-\dfrac{i}{\sqrt{2}} \cr \cr \mathbb{L} &=& \left\{12+\dfrac{i}{\sqrt{2}} \; ; \; 12-\dfrac{i}{\sqrt{2}}\right\}\end{array}


3)
\begin{array}{rclcl} 8x(2x-3) &=& -25 &\vert& +25 \cr 16x^2-24x+25 &=& 0 &\vert& :16 \cr x^2-\dfrac{3}{2}x+\dfrac{25}{16} &=& 0 &\vert& \text{p-q-Formel} \cr x_{1,2} &=& \dfrac{3}{4} \pm \sqrt{\dfrac{9}{16}-\dfrac{25}{16}} \cr &=& \dfrac{3}{4} \pm \sqrt{-1} \cr \cr x_1 &=& \dfrac{3}{4}+i \cr\cr x_2 &=& \dfrac{3}{4}-i \cr \cr \mathbb{L} &=& \left\{\dfrac{3}{4}+i \; ; \; \dfrac{3}{4}-i\right\}\end{array}


4)
\begin{array}{crclcl} & x(x^2-12(x-3)) &=& 6-2(6x^2+3) \cr & x^3-12x^2+36x &=& 6-12x^2-6 &\vert&+12x^2 \cr & x^3+36x &=& 0 \cr & x(x^2+36) &=& 0 & \vert & \text{Satz vom Nullprodukt} \cr \text{Faktor 1:} & x_1 &=& 0 \cr\cr \text{Faktor 2:} & x^2+36 &=& 0 \cr & x^2 &=& -36 &\vert& \pm\sqrt{} \cr & x_{2,3} &=& \pm \sqrt{-36} \cr & &=& \pm 6i \cr \cr & \mathbb{L} &=& \left\{-6i \; ; \; 0 \; ; \; 6i \right\}\end{array}

Bemerkung: Bislang war der Satz vom Nullprodukt zwar immer so formuliert worden: "Ein Produkt reeller Zahlen ist genau dann 0, wenn einer der Faktoren 0 ist." Er gilt aber natürlich auch für komplexe Zahlen.


5)
\begin{array}{rclcl} -\dfrac{1}{2}z-2i &=& -\dfrac{1}{8}z^2-iz+\dfrac{3}{2} &\vert& +\dfrac{1}{8}z^2+iz-\dfrac{3}{2} \cr \dfrac{1}{8}z^2-\dfrac{1}{2}z+iz-\dfrac{3}{2}-2i &=& 0 \cr z^2+(-4+8i)z-12-16i &=& 0 \cr z_{1,2} &=& -\dfrac{-4+8i}{2}\pm\sqrt{\left(\dfrac{-4+8i}{2}\right)^2-(-12-16i)} \cr &=& -\dfrac{-2(2-4i)}{2}\pm\sqrt{\dfrac{16-64i-64}{4}+12+16i} \cr &=& 2-4i\pm\sqrt{-\dfrac{48}{4}-\dfrac{64}{4}i+12+16i} \cr &=& 2-4i\pm 0 \cr \cr \mathbb{L} &=& \left\{2-4i\right\}\end{array}

Bemerkung: z=2-4i ist eine doppelte Lösung.


6)
\begin{array}{rclcl} -2\left(x^2+\dfrac{5}{8}\right) &=& x^3+\dfrac{9}{4}x \cr -2x^2 -\dfrac{5}{4} &=& x^3+\dfrac{9}{4}x &\vert& +2x^2+\dfrac{5}{4} \cr x^3 + 2x^2 +\dfrac{9}{4}x + \dfrac {5}{4} &=& 0 \end{array}

Durch Probieren finden wir die Lösung x_1=-1, denn
\begin{array}{rcl}(-1)^3 + 2\cdot (-1)^2 +\dfrac{9}{4}\cdot (-1) + \dfrac {5}{4} &=& 0 \\0 &=& 0\end{array}

Polynomdivision mit x_1=-1:
Durchführung Polynomdivision

Lösung des reduzierten Polynoms:
\begin{array}{rclcl} x^2+x+\dfrac{5}{4}&=&0 &\vert& \text{p-q-Formel} \cr x_{2,3}&=&-\dfrac{1}{2}\pm \sqrt{\left(\dfrac{1}{2}\right)^2-\dfrac{5}{4}} \cr &=& -\dfrac{1}{2} \pm \sqrt{-1} &\vert& \sqrt{-1}=i \cr \cr x_2 &=& -\dfrac{1}{2}+i \cr \cr x_3 &=& -\dfrac{1}{2}-i \cr \cr \mathbb{L} &=& \left\{-1 \; ; \; -\dfrac{1}{2}+i \; ; \; -\dfrac{1}{2}-i\right\}\end{array}


7)
\begin{array}{rclcl} 3x-19x^2 &=& -18x^2+\dfrac{10}{4} &\vert& -3x + 19x^2 \cr\cr 0 &=& x^2 -3x+ \dfrac{10}{4} &\vert& \text{p-q-Formel} \cr\cr x_{1,2} &=& \dfrac{3}{2} \pm \sqrt{\left(\dfrac{-3}{2}\right)^2 -\dfrac{10}{4}} \cr\cr &=& \dfrac{3}{2} \pm \sqrt{-\dfrac{1}{4}} \cr\cr &=& \dfrac{3}{2} \pm \sqrt{-1\cdot\dfrac{1}{4}} \cr\cr &=& \dfrac{3}{2} \pm \dfrac{1}{2} \sqrt{-1} &\vert& \sqrt{-1}=i \cr\cr\cr x_1 &=& \dfrac{3}{2}+\dfrac{1}{2}i \cr\cr x_2 &=& \dfrac{3}{2}-\dfrac{1}{2}i \cr \cr\cr \mathbb{L} &=& \left\{ \dfrac{3}{2}+\dfrac{1}{2}i \; ; \; \dfrac{3}{2}-\dfrac{1}{2}i\right\} \end{array}


8)
\begin{array}{crclcl} & 0 &=& 4z^3 +20 z^2 +50z \cr & 0 &=& 4z \left( z^2 +5z + \dfrac{25}{2}\right) &\vert& \text{Satz vom Nullprodukt} \cr\cr \text{Faktor 1:} & 4z &=& 0 &\vert& : 4 \cr & z_1 &=& 0 \cr\cr\cr \text{Faktor 2:} & 0 &=& z^2+5z+\dfrac{25}{2} &\vert& \text{p-q-Formel} \cr\cr & z_{2,3} &=& -\dfrac{5}{2} \pm \sqrt{\left(\dfrac{5}{2}\right)^2-\dfrac{25}{2}} \cr\cr& &=& -\dfrac{5}{2} \pm \sqrt{-\dfrac{25}{4}}\cr\cr & &=& -\dfrac{5}{2} \pm \sqrt{-1\cdot\dfrac{25}{4}}\cr\cr& &=& -\dfrac{5}{2} \pm \dfrac{5}{2}\sqrt{-1} &\vert& \sqrt{-1}=i \cr \cr\cr & z_2 &=& -\dfrac{5}{2}+\dfrac{5}{2}i \cr\cr & z_3 &=& -\dfrac{5}{2}-\dfrac{5}{2}i \cr \cr\cr & \mathbb{L} &=& \left\{ 0 \; ; \; -\dfrac{5}{2}+\dfrac{5}{2}i \; ; \; -\dfrac{5}{2}-\dfrac{5}{2}i\right\} \end{array}


9)
\begin{array}{rclcl} -217 &=& 7 (x^2-4x) \cr -217 &=& 7x^2-28x &\vert& +217 \cr0 &=& 7x^2-28x+217 &\vert& : 7 \cr0 &=& x^2-4x+31 &\vert& \text{p-q-Formel} \cr x_{1,2} &=& 2 \pm \sqrt{\left(-2\right)^2-31} \cr &=& 2\pm \sqrt{-27} \cr &=& 2 \pm \sqrt{9\cdot 3 \cdot (-1)} \cr &=& 2\pm 3 \cdot \sqrt{3} \sqrt{-1} &\vert& \sqrt{-1}=i \cr \cr x_1 &=& 2+3\sqrt{3} i \cr x_2 &=& 2-3\sqrt{3} i \cr \cr \mathbb{L} &=& \left\{2+3\sqrt{3}i \; ; \; 2-3\sqrt{3}i \right\}\end{array}


10)
\begin{array}{rcl} -x^3+x^2i-4x^2+4xi-68x+68i &=& 0 \\-x^3+(-4+i)x^2+(-68+4i)x+68i &=& 0\end{array}

Durch Probieren finden wir die Lösung x_1=i, denn
\begin{array}{rcl}-i^3+(-4+i)\cdot i^2+(-68+4i)\cdot i +68i &=& 0 \\0 &=& 0\end{array}

Polynomdivision mit x_1=i:
Durchführung Polynomdivision

Lösung des reduzierten Polynoms:
\begin{array}{rclcl} -x^2-4x-68 &=& 0 &\vert& \cdot (-1) \cr x^2+4x+68 &=& 0 &\vert& \text{p-q-Formel} \cr x_{2,3}&=&-2\pm \sqrt{2^2-68} \cr &=& -2\pm \sqrt{-64} \cr &=& -2\pm \sqrt{-1\cdot 64} \cr &=& -2 \pm 8 \cdot \sqrt{-1} &\vert& \sqrt{-1}= i \cr \cr x_2 &=&-2+8i \cr x_3 &=&-2-8i \cr \cr \mathbb{L} &=& \left\{ i \; ; \; -2+8i \; ; \; -2-8i \right\} \end{array}


11)
\begin{array}{rclcl} -20x^2 &=& 4x^4+29 &\vert& +20x^2 \cr 0 &=& 4x^4+20x^2+29 \cr 0 &=& 4\left(x^2\right)^2+20x^2+29\end{array}

Substitution: z = x^2
\begin{array}{rclcl}0 &=& 4z^2+20z+29 &\vert& :4 \cr 0 &=& z^2+5z+\dfrac{29}{4} &\vert& \text{p-q-Formel} \cr z_{1,2}&=&-\dfrac{5}{2} \pm \sqrt{\left(\dfrac{5}{2}\right)^2-\dfrac{29}{4}} \cr &=& -\dfrac{5}{2}\pm \sqrt{-1} &\vert& \sqrt{-1}=i \cr\cr z_1 &=& -\dfrac{5}{2}+i \cr z_2 &=& - \dfrac{5}{2}-i \end{array}

Rücksubstitution:
\begin{array}{rclcl} z_1 = x^2 &=& -\dfrac{5}{2}+i &\vert& \pm\sqrt{} \cr x_{1,2} &=& \pm\sqrt{-\dfrac{5}{2}+i} \cr\cr z_2 =x^2 &=& -\dfrac{5}{2}-i &\vert& \pm\sqrt{} \cr x_{3,4} &=& \pm\sqrt{-\dfrac{5}{2}-i} \cr \cr \mathbb{L} &=& \left\{ \sqrt{-\dfrac{5}{2}+i} \; ; \; -\sqrt{-\dfrac{5}{2}+i} \; ; \; \sqrt{-\dfrac{5}{2}-i} \; ; \; -\sqrt{-\dfrac{5}{2}-i} \right\}\end{array}


12)
\begin{array}{rclcl} -2x(-ix+5x-10i) &=& x^3+10(5x-10i) \cr 2ix^2-10x^2+20ix &=& x^3+50x-100i &\vert& -x^3-50x+100i \cr-x^3+2ix^2-10x^2+20ix-50x+100i &=& 0 \cr-x^3+(-10+2i)x^2+(-50+20i)x+100i &=& 0 \end{array}

Durch Probieren finden wir die Lösung x_1=2i, denn
\begin{array}{rcl}-(2i)^3+(-10+2i)\cdot (2i)^2+(-50+20i)\cdot 2i+100i &=& 0 \\0 &=& 0\end{array}

Polynomdivision mit x_1=2i:
Durchführung Polynomdivision

Lösung des reduzierten Polynoms:
\begin{array}{rclcl} -x^2-10x-50 &=& 0 &\vert& \cdot (-1) \cr x^2+10x+50 &=& 0 &\vert& \text{p-q-Formel} \cr x_{2,3} &=& -5\pm \sqrt{5^2-50} \cr &=& -5\sqrt{-25} \cr &=& -5\sqrt{-1\cdot 25} \cr&=& -5 \pm 5 \sqrt{-1} &\vert& \sqrt{-1}=i \cr \cr x_2 &=& -5+5i \cr x_3 &=& -5-5i \cr \cr \mathbb{L} &=& \left\{2i \; ; \; -5+5i \; ; \; -5-5i \right\} \end{array}


13)
\begin{array}{rclcl} 21 x+125 &=& x^2+3x+215 &\vert& -21x-125 \cr 0 &=& x^2-18x+90 &\vert& \text{p-q-Formel} \cr x_{1,2} &=& 9\pm \sqrt{9^2-90} \cr &=& 9\pm \sqrt{-9} \cr &=& 9\pm \sqrt{-1\cdot 9} \cr&=& 9\pm 3 \sqrt{-1} &\vert& \sqrt{-1}=i \cr \cr x_1&=&9+3i \cr x_2 &=& 9-3i \cr \cr \mathbb{L} &=& \{9+3i \; ; \; 9-3i\}\end{array}


14)
\begin{array}{rclll}\mathbb{D} &=& \mathbb{C}\setminus_{\{0+0\cdot i\}} \\\\x+4 &=& \dfrac{-10}{x}+3+i^2 &\vert & i^2=-1 \\\\x+4 &=& \dfrac{-10}{x}+2 &\vert & -2 \\\\x+2 &=& \dfrac{-10}{x} &\vert & \cdot x\\x^2+2x &=& -10 &\vert & +10\\x^2+2x+10 &=& 0 &\vert & \text{p-q-Formel}\\x_{1,2} &=& -\dfrac{2}{2}\pm\sqrt{\left(\dfrac{2}{2}\right)^2-10}\\&=& -1\pm\sqrt{-9}\\&=& -1\pm\sqrt{-1\cdot 9}\\&=& -1\pm i\cdot 3\\\\x_1 &=& -1+3i\\x_2 &=& -1-3i\\\\\mathbb{L} &=& \left\{-1+3i\; ; \; -1-3i \right\}\end{array}

Bemerkung: Die Multiplikation mit x ist hier ohne Einschränkungen möglich, weil x=0+0\cdot i \not\in \mathbb{D}. Eine Multiplikation mit 0 kann also nicht passieren.


15)
\begin{array}{rclll}\dfrac{x^2}{i}+\dfrac{5x\cdot i^{i+1}}{i^i}-7i &=& 0 \\\\\dfrac{x^2}{i}+5x\cdot i^{i+1-i}-7i &=& 0 &\vert & \cdot i\\\\x^2+5xi^2-7i^2 &=& 0 &\vert & i^2=-1\\x^2-5x+7 &=& 0 &\vert & \text{p-q-Formel} \\x_{1,2} &=& \dfrac{5}{2}\pm\sqrt{\left(\dfrac{5}{2}\right)^2-7}\\&=& \dfrac{5}{2}\pm\sqrt{-\dfrac{3}{4}}\\&=& \dfrac{5\pm\sqrt{-3}}{2}\\&=& \dfrac{5\pm\sqrt{-1\cdot 3}}{2} &\vert& \sqrt{-1}=i \\\\x_1 &=& \dfrac{5+\sqrt{3}\cdot i}{2}\\x_1 &=& \dfrac{5-\sqrt{3}\cdot i}{2}\\\mathbb{L} &=& \left\{\dfrac{5+\sqrt{3}\cdot i}{2}\; ; \; \dfrac{5-\sqrt{3}\cdot i}{2} \right\}\end{array}


16)
\begin{array}{rclcl} 0 &=& z^3 +64 &\vert& -64 \cr z^3 &=& -64 &\vert& \sqrt[3]{} \cr z &=& \sqrt[3]{-64}\end{array}

allgemeine Formel für die Wurzeln aus einer komplexen Zahl:
z_k = \sqrt[n]{\omega} = \sqrt[n]{\vert\omega\vert}\left(\cos{\left(\dfrac{\varphi+2k\pi}{n}\right)}+i\cdot\sin{\left(\dfrac{\varphi+2k\pi}{n}\right)}\right)

Bestimmung von \vert\omega\vert und \varphi:
\begin{array}{rcl}\omega &=& -64 +0i \\\\\vert \omega \vert &=& \sqrt{\left(-64\right)^2+0^2} \\\vert \omega \vert &=& 64\end{array}

Da \textit{Re}(\omega) < 0 und \textit{Im}(\omega)=0 ist \varphi = \pi.

Berechnung der Wurzeln:
\begin{array}{rcl}z_0 &=& \sqrt[3]{64}\cdot\left(\cos\left(\dfrac{\pi+2\cdot 0\cdot\pi}{3}\right)+i\cdot\sin\left(\dfrac{\pi+2\cdot 0\cdot\pi}{3}\right)\right) \\&=& 4\left(\cos\left(\dfrac{\pi}{3}\right)+i\cdot\sin\left(\dfrac{\pi}{3}\right)\right) \\&=& 4\left(\dfrac{1}{2}+i\dfrac{\sqrt{3}}{2}\right) \\&=& 2+2\sqrt{3}i \\\\z_1 &=& \sqrt[3]{64}\cdot\left(\cos\left(\dfrac{\pi+2\cdot 1\cdot\pi}{3}\right)+i\cdot\sin\left(\dfrac{\pi+2\cdot 1\cdot\pi}{3}\right)\right) \\&=& 4\left(\cos\left(\pi\right)+i\cdot\sin\left(\pi\right)\right) \\&=& 4\left(-1+i\cdot 0\right) \\&=& -4 \\\\z_2 &=& \sqrt[3]{64}\cdot\left(\cos\left(\dfrac{\pi+2\cdot 2\cdot\pi}{3}\right)+i\cdot\sin\left(\dfrac{\pi+2\cdot 2\cdot\pi}{3}\right)\right) \\&=& 4\left(\cos\left(\dfrac{5\pi}{3}\right)+i\cdot\sin\left(\dfrac{5\pi}{3}\right)\right) \\&=& 4\left(\dfrac{1}{2}-i\dfrac{\sqrt{3}}{2}\right) \\&=& 2-2\sqrt{3}i \\\\\mathbb{L} &=& \{2+2\sqrt{3}i \; ; \; -4 \; ; \; 2-2\sqrt{3}i\}\end{array}


17)
\begin{array}{rclll}7z^3-189i &=& 0 &\vert & +189i \\7z^3 &=& 189i &\vert & :7\\z^3 &=& 27i &\vert & \sqrt[3]{}\\z &=& \sqrt[3]{27i}\end{array}

allgemeine Formel für die Wurzeln aus einer komplexen Zahl:
z_k = \sqrt[n]{\omega} = \sqrt[n]{\vert\omega\vert}\left(\cos{\left(\dfrac{\varphi+2k\pi}{n}\right)}+i\cdot\sin{\left(\dfrac{\varphi+2k\pi}{n}\right)}\right)

Bestimmung von \vert\omega\vert und \varphi:
\begin{array}{rcl}\omega &=& 0+27i \\\\\vert \omega \vert &=& \sqrt{0^2+27^2} \\\vert \omega \vert &=& 27 \\\end{array}

Da \textit{Re}(\omega)=0 und \textit{Im}(\omega) > 0 ist \varphi = \dfrac{\pi}{2}.

Berechnung der Wurzeln:
\begin{array}{rcl}z_0 &=& \sqrt[3]{27}\cdot\left(\cos\left(\genfrac{}{}{1pt}{0}{\frac{\pi}{2}+2\cdot 0\cdot\pi}{3}\right)+i\sin\left(\genfrac{}{}{1pt}{0}{\frac{\pi}{2}+2\cdot 0\cdot\pi}{3}\right)\right) \\&=& 3\cdot\left(\cos\left(\dfrac{\pi}{6}\right)+i\sin\left(\dfrac{\pi}{6}\right)\right) \\&=& 3\cdot\left(\dfrac{\sqrt{3}}{2}+i\dfrac{1}{2}\right) \\&=& \dfrac{3\sqrt{3}}{2}+\dfrac{3}{2}i \\\\z_1 &=& \sqrt[3]{27}\cdot\left(\cos\left(\genfrac{}{}{1pt}{0}{\frac{\pi}{2}+2\cdot 1\cdot\pi}{3}\right)+i\sin\left(\genfrac{}{}{1pt}{0}{\frac{\pi}{2}+2\cdot 1\cdot\pi}{3}\right)\right) \\&=& 3\cdot\left(\cos\left(\dfrac{5\pi}{6}\right)+i\sin\left(\dfrac{5\pi}{6}\right)\right) \\&=& 3\cdot\left(-\dfrac{\sqrt{3}}{2}+i\dfrac{1}{2}\right) \\&=& -\dfrac{3\sqrt{3}}{2}+\dfrac{3}{2}i \\\\z_2 &=& \sqrt[3]{27}\cdot\left(\cos\left(\genfrac{}{}{1pt}{0}{\frac{\pi}{2}+2\cdot 2\cdot\pi}{3}\right)+i\sin\left(\genfrac{}{}{1pt}{0}{\frac{\pi}{2}+2\cdot 2\cdot\pi}{3}\right)\right) \\&=& 3\cdot\left(\cos\left(\dfrac{3\pi}{2}\right)+i\sin\left(\dfrac{3\pi}{2}\right)\right) \\&=& 3\cdot\left(0-i\right) \\&=& -3i \\\\\mathbb{L} &=& \left\{\dfrac{3\sqrt{3}}{2}+\dfrac{3}{2}i \; ; \; -\dfrac{3\sqrt{3}}{2}+\dfrac{3}{2}i \; ; \; -3i\right\}\end{array}


18)
\begin{array}{rclll}\dfrac{2z}{3}+\dfrac{-\sqrt{36i}+45}{9} &=& 5 &\vert& \cdot 9 \\6z-\sqrt{36i}+45 &=& 45 &\vert& -45+\sqrt{36i} \\6z &=& \sqrt{36i} \\6z &=& 6\sqrt{i} &\vert& :6 \\z &=& \sqrt{i}\end{array}

allgemeine Formel für die Wurzeln aus einer komplexen Zahl:
z_k = \sqrt[n]{\omega} = \sqrt[n]{\vert\omega\vert}\left(\cos{\left(\dfrac{\varphi+2k\pi}{n}\right)}+i\cdot\sin{\left(\dfrac{\varphi+2k\pi}{n}\right)}\right)

Bestimmung von \vert\omega\vert und \varphi:
\begin{array}{rcl}\omega &=& 0+1i \\\\\vert \omega \vert &=& \sqrt{0^2+1^2} \\\vert \omega \vert &=& 1 \\\end{array}

Da \textit{Re}(\omega)=0 und \textit{Im}(\omega) > 0 ist \varphi = \dfrac{\pi}{2}.

Berechnung der Wurzeln:
\begin{array}{rcl}z_0 &=& \sqrt{1}\cdot\left(\cos\left(\genfrac{}{}{1pt}{0}{\frac{\pi}{2}+2\cdot 0\cdot\pi}{2}\right)+i\sin\left(\genfrac{}{}{1pt}{0}{\frac{\pi}{2}+2\cdot 0\cdot\pi}{2}\right)\right) \\&=& 1\cdot\left(\cos\left(\dfrac{\pi}{4}\right)+i\sin\left(\dfrac{\pi}{4}\right)\right) \\&=& \dfrac{\sqrt{2}}{2}+\dfrac{\sqrt{2}}{2}i \\\\z_1 &=& \sqrt{1}\cdot\left(\cos\left(\genfrac{}{}{1pt}{0}{\frac{\pi}{2}+2\cdot 1\cdot\pi}{2}\right)+i\sin\left(\genfrac{}{}{1pt}{0}{\frac{\pi}{2}+2\cdot 1\cdot\pi}{2}\right)\right) \\&=& 1\cdot\left(\cos\left(\dfrac{5\pi}{4}\right)+i\sin\left(\dfrac{5\pi}{4}\right)\right) \\&=& -\dfrac{\sqrt{2}}{2}-i\dfrac{\sqrt{2}}{2} \\\\\mathbb{L} &=& \left\{\dfrac{\sqrt{2}}{2}+\dfrac{\sqrt{2}}{2}i \; ; \; -\dfrac{\sqrt{2}}{2}-i\dfrac{\sqrt{2}}{2} \right\}\end{array}


19)
\begin{array}{rclll}-28z^4+7\sqrt{3}i &=& 7 &\vert& -7\sqrt{3}i \\-28z^4 &=& 7-7\sqrt{3}i &\vert& :(-28) \\z^4 &=& -\dfrac{1}{4}+\dfrac{\sqrt{3}}{4}i &\vert & \sqrt[4]{} \\z &=& \sqrt[4]{-\dfrac{1}{4}+\dfrac{\sqrt{3}}{4}i}\end{array}

allgemeine Formel für die Wurzeln aus einer komplexen Zahl:
z_k = \sqrt[n]{\omega} = \sqrt[n]{\vert\omega\vert}\left(\cos{\left(\dfrac{\varphi+2k\pi}{n}\right)}+i\cdot\sin{\left(\dfrac{\varphi+2k\pi}{n}\right)}\right)

Bestimmung von \vert\omega\vert und \varphi:
\begin{array}{rcl}\omega &=& -\dfrac{1}{4}+\dfrac{\sqrt{3}}{4}i \\\\\vert \omega \vert &=& \sqrt{\left(-\dfrac{1}{4}\right)^2+\left(\dfrac{\sqrt{3}}{4}\right)^2} \\\vert \omega \vert &=& \sqrt{\dfrac{1}{4}} \\\vert \omega \vert &=& \dfrac{1}{2}\end{array}

Da \textit{Re}(\omega) < 0 und \textit{Im}(\omega) > 0 liegt \omega im zweiten Quadranten.
\begin{array}{rcl}\varphi &=& \pi-\arctan\left(\dfrac{\frac{\sqrt{3}}{4}}{\frac{1}{4}}\right) \\\varphi &=& \pi-\arctan\left(\sqrt{3}\right) \\\varphi &=& \pi-\dfrac{\pi}{3} \\\varphi &=& \dfrac{2\pi}{3}\end{array}

Berechnung der Wurzeln:
\begin{array}{rcl}z_0 &=& \sqrt[4]{\dfrac{1}{2}}\cdot\left(\cos\left(\genfrac{}{}{1pt}{0}{\frac{2\pi}{3}+2\cdot 0\cdot\pi}{4}\right)+i\sin\left(\genfrac{}{}{1pt}{0}{\frac{2\pi}{3}+2\cdot 0\cdot\pi}{4}\right)\right) \\&=& \sqrt[4]{\dfrac{1}{2}}\cdot\left(\cos\left(\dfrac{\pi}{6}\right)+i\sin\left(\dfrac{\pi}{6}\right)\right) \\&=& \dfrac{1}{\sqrt[4]{2}}\cdot\left(\dfrac{\sqrt{3}}{2}+i\dfrac{1}{2}\right) \\&=& \dfrac{\sqrt{3}}{2\sqrt[4]{2}}+\dfrac{1}{2\sqrt[4]{2}}i \\\\z_1 &=& \sqrt[4]{\dfrac{1}{2}}\cdot\left(\cos\left(\genfrac{}{}{1pt}{0}{\frac{2\pi}{3}+2\cdot 1\cdot\pi}{4}\right)+i\sin\left(\genfrac{}{}{1pt}{0}{\frac{2\pi}{3}+2\cdot 1\cdot\pi}{4}\right)\right) \\&=& \sqrt[4]{\dfrac{1}{2}}\cdot\left(\cos\left(\dfrac{2\pi}{3}\right)+i\sin\left(\dfrac{2\pi}{3}\right)\right) \\&=& \dfrac{1}{\sqrt[4]{2}}\cdot\left(-\dfrac{1}{2}+i\dfrac{\sqrt{3}}{2}\right) \\&=& -\dfrac{1}{2\sqrt[4]{2}}+\dfrac{\sqrt{3}}{2\sqrt[4]{2}}i \end{array}

\begin{array}{rcl}z_2 &=& \sqrt[4]{\dfrac{1}{2}}\cdot\left(\cos\left(\genfrac{}{}{1pt}{0}{\frac{2\pi}{3}+2\cdot 2\cdot\pi}{4}\right)+i\sin\left(\genfrac{}{}{1pt}{0}{\frac{2\pi}{3}+2\cdot 2\cdot\pi}{4}\right)\right) \\&=& \sqrt[4]{\dfrac{1}{2}}\cdot\left(\cos\left(\dfrac{7\pi}{6}\right)+i\sin\left(\dfrac{7\pi}{6}\right)\right) \\&=& \dfrac{1}{\sqrt[4]{2}}\cdot\left(-\dfrac{\sqrt{3}}{2}-i\dfrac{1}{2}\right) \\&=& -\dfrac{\sqrt{3}}{2\sqrt[4]{2}}-\dfrac{1}{2\sqrt[4]{2}}i \\\\z_3 &=& \sqrt[4]{\dfrac{1}{2}}\cdot\left(\cos\left(\genfrac{}{}{1pt}{0}{\frac{2\pi}{3}+2\cdot 3\cdot\pi}{4}\right)+i\sin\left(\genfrac{}{}{1pt}{0}{\frac{2\pi}{3}+2\cdot 3\cdot\pi}{4}\right)\right) \\&=& \sqrt[4]{\dfrac{1}{2}}\cdot\left(\cos\left(\dfrac{5\pi}{3}\right)+i\sin\left(\dfrac{5\pi}{3}\right)\right) \\&=& \dfrac{1}{\sqrt[4]{2}}\cdot\left(\dfrac{1}{2}-i\dfrac{\sqrt{3}}{2}\right) \\&=& \dfrac{1}{2\sqrt[4]{2}}-\dfrac{\sqrt{3}}{2\sqrt[4]{2}}i \\\\\mathbb{L} &=& \left\{\dfrac{\sqrt{3}}{2\sqrt[4]{2}}+\dfrac{1}{2\sqrt[4]{2}}i \; ; \; -\dfrac{1}{2\sqrt[4]{2}}+\dfrac{\sqrt{3}}{2\sqrt[4]{2}}i \; ; \; -\dfrac{\sqrt{3}}{2\sqrt[4]{2}}-\dfrac{1}{2\sqrt[4]{2}}i \; ; \; \dfrac{1}{2\sqrt[4]{2}}-\dfrac{\sqrt{3}}{2\sqrt[4]{2}}i \right\}\end{array}


20)
\begin{array}{rclll}-3z^3+81-3i &=& 84 & \vert & -81+3i \\-3z^3 &=& 3+3i &\vert & :(-3)\\z^3 &=& -1-i &\vert& \sqrt[3]{} \\z &=& \sqrt[3]{-1-i}\end{array}

allgemeine Formel für die Wurzeln aus einer komplexen Zahl:
z_k = \sqrt[n]{\omega} = \sqrt[n]{\vert\omega\vert}\left(\cos{\left(\dfrac{\varphi+2k\pi}{n}\right)}+i\cdot\sin{\left(\dfrac{\varphi+2k\pi}{n}\right)}\right)

Bestimmung von \vert\omega\vert und \varphi:
\begin{array}{rcl}\omega &=& -1-1i \\\\\vert \omega \vert &=& \sqrt{\left(-1\right)^2+\left(-1\right)^2} \\\vert \omega \vert &=& \sqrt{2}\end{array}

Da \textit{Re}(\omega) < 0 und \textit{Im}(\omega) < 0 liegt \omega im dritten Quadranten.
\begin{array}{rcl}\varphi &=& \pi+\arctan\left(\dfrac{-1}{-1}\right) \\\varphi &=& \pi+\arctan{\left(1\right)} \\\varphi &=& \pi+\dfrac{\pi}{4} \\\\\varphi &=& \dfrac{5\pi}{4}\end{array}

Berechnung der Wurzeln:
\begin{array}{rcl}z_0 &=& \sqrt[3]{\sqrt[2]{2}}\cdot\left(\cos\left(\genfrac{}{}{1pt}{0}{\frac{5\pi}{4}+2\cdot 0\cdot\pi}{3}\right)+i\sin\left(\genfrac{}{}{1pt}{0}{\frac{5\pi}{4}+2\cdot 0\cdot\pi}{3}\right)\right) \\&=& \sqrt[6]{2}\cdot\left(\cos\left(\dfrac{5\pi}{12}\right)+i\sin\left(\dfrac{5\pi}{12}\right)\right) \\&=& \sqrt[6]{2}\cdot\left(\dfrac{\sqrt{6}-\sqrt{2}}{4}+\dfrac{\sqrt{6}+\sqrt{2}}{4} i\right) \\&=& \dfrac{\left(\sqrt{6}-\sqrt{2}\right)\cdot\sqrt[6]{2}}{4}+\dfrac{\left(\sqrt{6}+\sqrt{2}\right)\cdot\sqrt[6]{2}}{4}i \\\\z_1 &=& \sqrt[3]{\sqrt[3]{2}}\cdot\left(\cos\left(\genfrac{}{}{1pt}{0}{\frac{5\pi}{4}+2\cdot 1\cdot\pi}{3}\right)+i\sin\left(\genfrac{}{}{1pt}{0}{\frac{5\pi}{4}+2\cdot 1\cdot\pi}{3}\right)\right) \\&=& \sqrt[6]{2}\cdot\left(\cos\left(\dfrac{13\pi}{12}\right)+i\sin\left(\dfrac{13\pi}{12}\right)\right) \\&=& \sqrt[6]{2}\cdot\left(\dfrac{-\sqrt{6}-\sqrt{2}}{4}+\dfrac{-\sqrt{6}+\sqrt{2}}{4} i\right) \\&=& \dfrac{\left(-\sqrt{6}-\sqrt{2}\right)\cdot\sqrt[6]{2}}{4}+\dfrac{\left(-\sqrt{6}+\sqrt{2}\right)\cdot\sqrt[6]{2}}{4}i \\\\z_2 &=& \sqrt[3]{\sqrt[3]{2}}\cdot\left(\cos\left(\genfrac{}{}{1pt}{0}{\frac{5\pi}{4}+2\cdot 2\cdot\pi}{3}\right)+i\sin\left(\genfrac{}{}{1pt}{0}{\frac{5\pi}{4}+2\cdot 0\cdot\pi}{3}\right)\right) \\&=& \sqrt[6]{2}\cdot\left(\cos\left(\dfrac{7\pi}{4}\right)+i\sin\left(\dfrac{7\pi}{4}\right)\right) \\&=& \sqrt[6]{2}\cdot\left(\dfrac{\sqrt{2}}{2}-\dfrac{\sqrt{2}}{2}i\right) \\&=& \dfrac{\sqrt{2}\cdot\sqrt[6]{2}}{2}-\dfrac{\sqrt{2}\cdot\sqrt[6]{2}}{2}i \\\\\mathbb{L} &=& \left\{\dfrac{\left(\sqrt{6}-\sqrt{2}\right)\cdot\sqrt[6]{2}}{4}+\dfrac{\left(\sqrt{6}+\sqrt{2}\right)\cdot\sqrt[6]{2}}{4}i \; ; \; \dfrac{\left(-\sqrt{6}-\sqrt{2}\right)\cdot\sqrt[6]{2}}{4}+\dfrac{\left(-\sqrt{6}+\sqrt{2}\right)\cdot\sqrt[6]{2}}{4}i \; ; \; \dfrac{\sqrt{2}\cdot\sqrt[6]{2}}{2}-\dfrac{\sqrt{2}\cdot\sqrt[6]{2}}{2}i \right\}\end{array}

25. Vektorrechnung - Lernziele und typische Fehler

Nach Durcharbeiten dieses Kapitels sollten Sie folgende Lernziele erreicht haben:

  • Sie kennen das Konzept "Vektor" und können Vektoren von Skalaren unterscheiden.
  • Sie können Vektoren aus Anfangs- und Endpunkt berechnen und in ein kartesisches Koordinatensystem zeichnen.
  • Sie können Vektoren addieren, subtrahieren und mit einem Skalar multiplizieren (Linearkombination) und diese Rechenoperationen geometrisch interpretieren.
  • Sie können bestimmen, ob Vektoren parallel bzw. antiparallel sind.
  • Sie können den Betrag eines Vektors berechnen.
  • Sie können berechnen, ob Vektoren "linear abhängig" oder "linear unabhängig" sind und wissen, was das bedeutet.
  • Sie können Skalarprodukte von Vektoren berechnen und geometrisch interpretieren.
  • Für Ingenieurstudierende: Sie können Vektor- und Spatprodukte von Vektoren berechnen und geometrisch interpretieren.
  • Sie können Vektoren nutzen, um einfache geometrische Aufgaben (z. B. die Berechnung von Winkeln) zu lösen.


In diesem Kapitel sind bislang keine typischen Fehler aufgefallen.


Für Online-Selbsttests zu diesem Thema und weitere Informationen zur Mathematikunterstützung an der TH Wildau nutzen Sie bitte den Moodle-Kursraum "SOS Mathematik - Vektorrechnung".

Übersicht:

 

25.1 Vektorrechnung - Aufgaben

1. Aufgabe

Berechnen Sie für die drei gegebenen Vektoren jeweils die gesuchte Linearkombination, wenn dies möglich ist!


1)
Gegeben: \vec{a} = \begin{pmatrix}-\frac{3}{2}\cr\frac{1}{4}\end{pmatrix} , \vec{b} = \begin{pmatrix}1\cr1\end{pmatrix} und \vec{c} = \begin{pmatrix}-4\cr-9\end{pmatrix}
Gesucht: -10\vec{a} + 5\vec{b} + \vec{c}
 
2)
Gegeben: \vec{a} = \frac{1}{2}\begin{pmatrix}16\cr-24\end{pmatrix} , \vec{b} = \begin{pmatrix}-13\cr8\cr19\end{pmatrix} und \vec{c} = \frac{3}{4}\begin{pmatrix}34\cr-3\cr0\end{pmatrix}
Gesucht: 14\vec{a} + 7\vec{b} - 11\vec{c}
 
3)
Gegeben: \vec{a} = 3\begin{pmatrix}-2\cr-\frac{1}{3}\end{pmatrix} , \vec{b} = \begin{pmatrix}18\cr-15\end{pmatrix} und \vec{c} = \begin{pmatrix}-26\cr22\end{pmatrix}
Gesucht: 6\vec{a} + \dfrac{4}{3}\vec{b} - \dfrac{1}{2}\vec{c}
 
4)
Gegeben: \vec{a} = \begin{pmatrix}27\cr-41\end{pmatrix} , \vec{b} = \begin{pmatrix}-25\cr-33\end{pmatrix} und \vec{c} = -\begin{pmatrix}-19\cr31\end{pmatrix}
Gesucht: 2\vec{a} - 3\vec{b} - 7\vec{c}
 
5)
Gegeben: \vec{a} = \begin{pmatrix}12\cr13\cr14\end{pmatrix} , \vec{b} = \begin{pmatrix}-8\cr21\end{pmatrix} und \vec{c} = -5\begin{pmatrix}1\cr-8\end{pmatrix}
Gesucht: -\vec{a} + 23\vec{b} + \vec{c}
 
6)
Gegeben: \vec{a} = \begin{pmatrix}-16\cr-50\cr-5\end{pmatrix} , \vec{b} = \begin{pmatrix}1\cr-46\cr-6\end{pmatrix} und \vec{c} = \begin{pmatrix}-27\cr33\cr40\end{pmatrix}
Gesucht: 4\vec{a} + 9\vec{b} - 3\vec{c}
 
7)
Gegeben: \vec{v} = \begin{pmatrix}38\cr-37\cr24\end{pmatrix} , \vec{w} = \begin{pmatrix}-15\cr36\cr-27\end{pmatrix} und \vec{u} = \begin{pmatrix}-32\cr-2\cr-20\end{pmatrix}
Gesucht:  \vec{v} - 8\vec{w} + 3 \vec{u}
 
8)
Gegeben: \vec{a} = \begin{pmatrix}-4\cr-20\cr-17\end{pmatrix} , \vec{b} = \begin{pmatrix}-25\cr-29\cr23\end{pmatrix} und \vec{c} = \begin{pmatrix}16\cr-29\cr-35\end{pmatrix}
Gesucht: -6\vec{a} + 7\vec{b} - 2\vec{c}
 
9)
Gegeben: \vec{t} = \begin{pmatrix}-27\cr48\cr-38\end{pmatrix} , \vec{s} = \begin{pmatrix}\frac{31}{8}\cr0\cr\frac{1}{12}\end{pmatrix} und \vec{q} = \begin{pmatrix}-44\cr-6\cr \frac{3}{14}\end{pmatrix}
Gesucht: \dfrac 56 \vec{t} - 4\vec{s} + \dfrac{7}{2} \vec{q}
 
10)
Gegeben: \vec{a} = \begin{pmatrix}\frac{16}{3}\cr-8\cr0\end{pmatrix} , \vec{b} = \begin{pmatrix}\frac{19}{4}\cr-2\cr-\frac{17}{2}\end{pmatrix} und \vec{c} = \begin{pmatrix}-\frac{2}{27}\cr\frac{11}{9}\cr-1\end{pmatrix}
Gesucht: -\dfrac{1}{4} \vec{a} + 12\vec{b} + 9\vec{c}

 

2. Aufgabe

Berechnen Sie jeweils den Betrag des Vektors \vec{a}! Runden Sie die Ergebnisse auf zwei Stellen nach dem Komma.

1) 
Gegeben: \vec{a} = \begin{pmatrix}5\cr-14\end{pmatrix}

  5) 
Gegeben: \vec{a} = \begin{pmatrix}16\cr-12\cr-\frac{2}{7} \end{pmatrix}

2) 
Gegeben: \vec{a} = \begin{pmatrix}-\frac{3}{10}\cr \cr-\frac{7}{3} \end{pmatrix}

  6) 
Gegeben: \vec{a} = -\dfrac{1}{12}\begin{pmatrix}48\cr-72\cr-12\end{pmatrix}

3) 
Gegeben: \vec{a} = -\begin{pmatrix}0\cr-17\end{pmatrix}

  7) 
Gegeben: \vec{a} = \dfrac{3}{4} \begin{pmatrix}\frac{8}{3}\cr32\cr16\end{pmatrix}

4) 
Gegeben: \vec{a} = \begin{pmatrix}8\cr14\cr-17\end{pmatrix}

  8) 
Gegeben: \vec{a} = \dfrac{2}{5} \begin{pmatrix}-20\cr25\cr-9\end{pmatrix}

 

3. Aufgabe

Wie groß muss die Komponente x\in\mathbb{R} sein, damit der Betrag des Vektors jeweils den angegebenen Wert annimmt?

1) 
Gegeben: \vec{a} = \begin{pmatrix}x\cr-6\cr11\end{pmatrix} und \left|\vec{a}\right|=15
  5) 
Gegeben: \vec{a} = \begin{pmatrix}x\cr-13\cr3x\end{pmatrix} und \left|\vec{a}\right|=17
2) 
Gegeben: \vec{a} = \begin{pmatrix}-10\cr2x\cr9\end{pmatrix} und \left|\vec{a}\right|=21
  6) 
Gegeben: \vec{u} = \begin{pmatrix}-17\cr4x^2\cr14\end{pmatrix} und \left|\vec{a}\right|=25
3) 
Gegeben: \vec{AB} = \begin{pmatrix}-4\cr\cr\frac{x}{2}\cr\cr\frac{3x}{2}\end{pmatrix} und \left|\vec{AB}\right|=45
  7) 
Gegeben: \vec{z} = \begin{pmatrix}\sqrt{25}\cr x^3 \cr-\frac{1}{2}\end{pmatrix} und \left|\vec{z}\right|=11
4) 
Gegeben: \vec{a} = \begin{pmatrix}-1\cr15\cr12x\end{pmatrix} und \left|\vec{a}\right|=28
  8) 
Gegeben: \vec{a} = \begin{pmatrix}\sqrt{x^3} \cr 12 \cr 12\end{pmatrix} und \left|\vec{a}\right|=\sqrt{504}

 

4. Aufgabe

Wie muss der Parameter \lambda gewählt werden, damit \left|\lambda\vec{a}\right| jeweils den Wert 1 annimmt?

1) 
Gegeben: \vec{a} = \begin{pmatrix}7\cr-8\cr10\end{pmatrix}
  5) 
Gegeben: \vec{a} = \begin{pmatrix}3\cr7\cr-1\end{pmatrix}
2) 
Gegeben: \vec{a} = \begin{pmatrix}-\frac{4}{9}\cr0\cr\frac{1}{3}\end{pmatrix}
  6) 
Gegeben: \vec{a} = \begin{pmatrix}-64\cr 14 \cr -10 \end{pmatrix}
3) 
Gegeben: \vec{a} = \begin{pmatrix}-40\cr-20\cr70\end{pmatrix}
  7) 
Gegeben: \vec{a} = -\dfrac{4}{5} \begin{pmatrix}15 \cr -\frac{5}{12} \cr 0 \end{pmatrix}
4) 
Gegeben: \vec{a} = \begin{pmatrix}23\cr-35\cr-27\end{pmatrix}
  8)
Gegeben: \vec{a} = \begin{pmatrix}-\sqrt{20}\cr 13 \cr 25 \end{pmatrix}

 

5. Aufgabe

Berechnen Sie jeweils das Skalarprodukt \vec{a}^{\,T} \cdot \vec{b}!

1)
Gegeben: \vec{a} = \begin{pmatrix}43\cr-45\cr11\end{pmatrix} und \vec{b} = \begin{pmatrix}2\cr-3\cr-4\end{pmatrix}
  5) 
Gegeben: \vec{a} = \begin{pmatrix}121\cr-97\cr105\end{pmatrix} und \vec{b} = \begin{pmatrix}-84\cr-113\cr96\end{pmatrix}
2)
Gegeben: \vec{a} = \begin{pmatrix}-\frac{5}{8}\cr-13\cr20\end{pmatrix} und \vec{b} = \dfrac{4}{5} \begin{pmatrix}24\cr30\cr7\end{pmatrix}
  6)
Gegeben: \vec{a} = \begin{pmatrix}\frac{4}{3} \cr 13 \cr 16 \end{pmatrix} und \vec{b} = \begin{pmatrix}3\cr -37 \cr \frac{121}{4} \end{pmatrix}

3)
Gegeben: \vec{a} = \begin{pmatrix}12\cr14\cr-20\end{pmatrix} und \vec{b} = \begin{pmatrix}-8\cr12\cr-18\end{pmatrix}
  7)
Gegeben: \vec{a} = \begin{pmatrix}\frac{3}{5} \cr -28 \cr \frac{1}{3} \end{pmatrix} und \vec{b} = \dfrac{8}{3} \begin{pmatrix}-30\cr 0 \cr 0 \end{pmatrix}
4) 
Gegeben: \vec{x} = \begin{pmatrix}-\frac{3}{2}\cr0\cr36\end{pmatrix} und \vec{y} = \begin{pmatrix}4\cr-7\cr\frac{1}{6}\end{pmatrix}
  8)
Gegeben: \vec{a} = \begin{pmatrix} 3t \cr -9 \cr -14 \end{pmatrix} und \vec{b} = \begin{pmatrix}225 \cr -256 \cr 289 \end{pmatrix}

 

6. Aufgabe

Berechnen Sie jeweils den Winkel \varphi, der von den Vektoren \vec{a} und \vec{b} eingeschlossen wird!

1) 
Gegeben: \vec{a} = \begin{pmatrix}4\cr4\cr-6\end{pmatrix} und \vec{b} = \begin{pmatrix}-1\cr-9\cr-5\end{pmatrix}
  5) 
Gegeben: \vec{a} = \begin{pmatrix}-15\cr-\frac{1}{4}\cr18\end{pmatrix} und \vec{b} = \begin{pmatrix}\frac{22}{5}\cr12\cr-\frac{7}{2}\end{pmatrix}
2)
Gegeben: \vec{a} = \begin{pmatrix}2\cr2\cr1\end{pmatrix} und \vec{b} = \begin{pmatrix}-1\cr10\cr-1\end{pmatrix}
  6)
Gegeben: \vec{a} = \begin{pmatrix}\frac{27}{11} \cr \sqrt{3} \cr 8 \end{pmatrix} und \vec{b} = \begin{pmatrix}22\cr \sqrt{12} \cr 16 \end{pmatrix}
3) 
Gegeben: \vec{a} = \begin{pmatrix}-17\cr-7\cr-8\end{pmatrix} und \vec{b} = \begin{pmatrix}2\cr28\cr9\end{pmatrix}
  7)
Gegeben: \vec{m} = \begin{pmatrix}- \frac{5}{6} \cr \sqrt{8} \cr 0 \end{pmatrix} und \vec{n} = \begin{pmatrix}3\cr -\sqrt{2} \cr 5^3 \end{pmatrix}
4) 
Gegeben: \vec{a} = \begin{pmatrix}\frac{5}{4}\cr9\cr \frac{7}{2}\end{pmatrix} und \vec{b} = -\begin{pmatrix}\frac{8}{15}\cr \frac{4}{3}\cr10\end{pmatrix}
  8)
Gegeben: \vec{a} = \begin{pmatrix}-2^7 \cr -7^3 \cr 0 \end{pmatrix} und \vec{b} = \begin{pmatrix} -\frac{1}{2} \cr 0 \cr \sqrt{15} \end{pmatrix}

 

7. Aufgabe

Sind die gegebenen Vektoren jeweils orthogonal?

1) 
Gegeben: \vec{a} = \begin{pmatrix}1\cr0\cr-3\end{pmatrix} und \vec{b} = \begin{pmatrix}18\cr-10\cr6\end{pmatrix}
  5) 
Gegeben: \vec{a} = \begin{pmatrix}9\cr-7\cr0\end{pmatrix} und \vec{b} = \begin{pmatrix}0\cr-\frac{9}{14}\cr\frac{8}{3}\end{pmatrix}
2)
Gegeben: \vec{a} = \begin{pmatrix}\frac{1}{2}\cr1\cr5\end{pmatrix} und \vec{b} = \begin{pmatrix}-16\cr15\cr\frac{11}{10}\end{pmatrix}
  6)
Gegeben: \vec{a} = \begin{pmatrix} \sqrt{32} \cr z \cr -2{,}5 \end{pmatrix} und \vec{b} = \begin{pmatrix} \frac{5}{2} \cr 0 \cr 4 \sqrt{2} \end{pmatrix}
3) 
Gegeben: \vec{DE} = \begin{pmatrix}13\cr-10\cr3\end{pmatrix} und \vec{FG} = \begin{pmatrix}2\cr2\cr-2\end{pmatrix}
  7)
Gegeben: \vec{0F} = \begin{pmatrix} 16 \cr \frac{3}{16} \cr 3 \end{pmatrix} und \vec{w} = \begin{pmatrix} \frac{3}{16} \cr 4 \cr 2 \end{pmatrix}
4) 
Gegeben: \vec{a} = \begin{pmatrix}25\cr-30\cr-34\end{pmatrix} und \vec{b} = 3\begin{pmatrix}4\cr-3\cr6\end{pmatrix}
  8)
Gegeben: \vec{a} = \begin{pmatrix} 18 \cr -8 \cr 27 \end{pmatrix} und \vec{b} = \begin{pmatrix} 16 \cr 117 \cr 24 \end{pmatrix}

 

8. Aufgabe

Bestimmen Sie aus den gegebenen Punkten die Vektoren \vec{a}=\vec{AB} und \vec{b}=\vec{BC} und prüfen Sie diese auf lineare Unabhängigkeit!

1)
Gegeben: A\,\left(-45 \mid 15\right) , B\,\left(-12 \mid -36\right) und C\,\left(-78 \mid 66\right)
 
2)
Gegeben: A\,\left(-19 \mid 30\right) , B\,\left(-15 \mid 14\right) und C\,\left(-10 \mid 0\right)


Bestimmen Sie aus den gegebenen Punkten die Vektoren \vec{a}=\vec{AB}, \vec{b}=\vec{BC} und \vec{c}=\vec{CD} und prüfen Sie diese auf lineare Unabhängigkeit!
 
3)
Gegeben: A\,\left(\frac{10}{3} \mid -\frac{49}{8} \mid -\frac{651}{10}\right) , B\,\left(-\frac{5}{3} \mid \frac{71}{8} \mid -\frac{1}{10}\right) , C\,\left(-\frac{4}{3} \mid \frac{7}{8} \mid -\frac{1}{10}\right) und D\,\left(-\frac{1}{3} \mid \frac{7}{8} \mid -\frac{11}{10}\right)
 
4)
Gegeben: A\,\left(33 \mid -7 \mid -19\right) , B\,\left(15 \mid -1 \mid 5\right) , C\,\left(17 \mid -3 \mid -9\right) und D\,\left(1 \mid 1 \mid 1\right)
 
5)
Gegeben: A\,\left(19 \mid 21 \mid -3\right) , B\,\left(20 \mid 20 \mid -2\right) , C\,\left(10 \mid 15 \mid -9\right) und D\,\left(20 \mid 13 \mid -7\right)

 

9. Aufgabe

Bestimmen Sie aus den gegebenen Punkten die Vektoren \vec{a}=\vec{AB} und \vec{b}=\vec{BC} und prüfen Sie diese auf Parallelität bzw. Antiparallelität!

1)
Gegeben: A\,\left(-1 \mid 0 \mid -8\right) , B\,\left(14 \mid -5 \mid -2\right) und C\,\left(-31 \mid 10 \mid -20\right)
 
2)
Gegeben: A\,\left(-11 \mid \frac{11}{5} \mid -7\right) , B\,\left(-18 \mid \frac{16}{5} \mid -\frac{17}{2}\right) und C\,\left(24 \mid -\frac{14}{5} \mid \frac{1}{2}\right)
 
3)
Gegeben: A\,\left(1 \mid 6 \mid -16\right) , B\,\left(2 \mid 7 \mid -8\right) und C\,\left(-2 \mid 2 \mid -40\right)
 
4)
Gegeben: A\,\left(8 \mid -9 \mid 10\right) , B\,\left(12 \mid 2 \mid 0\right) und C\,\left(24 \mid 35 \mid -30\right)
 
5)
Gegeben: A\,\left(-9 \mid 12 \mid -31\right) , B\,\left(-12 \mid 9 \mid -23\right) und C\,\left(-39 \mid -18 \mid 49\right)
 
6)
Gegeben: A\,\left(14 \mid -4 \mid 7\right) , B\,\left(14 \mid 3 \mid 21\right) und C\,\left(14 \mid 0 \mid 20\right)

 

10. Aufgabe

Berechnen Sie das Vektorprodukt \vec{a} \times \vec{b}!

1) 
Gegeben: \vec{a} = \begin{pmatrix}0 \cr \frac{4}{3}\cr-12\end{pmatrix} und \vec{b} = \begin{pmatrix}-\frac{3}{4}\cr \frac{5}{6}\cr-\frac{9}{2}\end{pmatrix}
  5) 
Gegeben: \vec{a} = -\dfrac{5}{4} \begin{pmatrix}22\cr-13\cr-40\end{pmatrix} und \vec{b} = \dfrac{4}{5} \begin{pmatrix}0\cr-30\cr4\end{pmatrix}
2) 
Gegeben: \vec{o} = \begin{pmatrix}14\cr-7\cr6\end{pmatrix} und \vec{n} = \frac{3}{4} \begin{pmatrix}-4\cr12\cr-32\end{pmatrix}
  6) 
Gegeben: \vec{a} = \begin{pmatrix} -\frac{3}{2} \cr \frac{1}{2} \cr 14 \end{pmatrix} und \vec{b} = \begin{pmatrix} 6 \cr 3 \cr 84 \end{pmatrix}
3)
Gegeben: \vec{a} = \begin{pmatrix}-35\cr-45\cr-43\end{pmatrix} und \vec{b} = \begin{pmatrix}9\cr13\cr11\end{pmatrix}
  7) 
Gegeben: \vec{a} = \begin{pmatrix} -\frac{11}{72}x \cr 0 \cr \frac{13}{2} \end{pmatrix} und \vec{b} = \begin{pmatrix} 0 \cr -\frac{15}{9} \cr 0 \end{pmatrix}
4) 
Gegeben: \vec{a} = \begin{pmatrix} \frac{13}{6}\cr4\cr-12\end{pmatrix} und \vec{b} = \begin{pmatrix}-\frac{4}{3}\cr11\cr \frac{8}{5} \end{pmatrix}
  8) 
Gegeben: \vec{a} = \begin{pmatrix} -\frac{3}{5} \cr 17 \cr 0 \end{pmatrix} und \vec{b} = \begin{pmatrix} -4 \cr 10 \cr -66 \end{pmatrix}

 

11. Aufgabe

Berechnen Sie das Spatprodukt \vec{a}^{\,T} \cdot \left(\vec{b} \times \vec{c}\right)!

1)
Gegeben: \vec{a} = \begin{pmatrix}1\cr-1\cr2\end{pmatrix} , \vec{b} = \begin{pmatrix}10\cr0\cr-8\end{pmatrix} und \vec{c} = \begin{pmatrix}4\cr-5\cr0\end{pmatrix}
 
2)
Gegeben: \vec{a} = \begin{pmatrix}-21\cr6\cr-35\end{pmatrix} , \vec{b} = \begin{pmatrix}-7\cr-18\cr28\end{pmatrix} und \vec{c} = \dfrac{1}{3} \begin{pmatrix}-21\cr18\cr48\end{pmatrix}
 
3)
Gegeben: \vec{a} = \begin{pmatrix}37\cr7\cr-32\end{pmatrix} , \vec{b} = \begin{pmatrix}-30\cr-5\cr13 \end{pmatrix} und \vec{c} = \begin{pmatrix}-10\cr-45\cr-48\end{pmatrix}
  
4)
Gegeben: \vec{a} = \begin{pmatrix}-25\cr4\cr1\end{pmatrix} , \vec{b} = \dfrac{12}{5} \begin{pmatrix}-15\cr1\cr-\frac{15}{4}\end{pmatrix} und \vec{c} = \begin{pmatrix}-10\cr11\cr\frac{3}{2}\end{pmatrix}
 
5)
Gegeben: \vec{e} = \begin{pmatrix}-12\cr2\cr-3\end{pmatrix} , \vec{f} = \begin{pmatrix}7\cr-9\cr-9\end{pmatrix} und \vec{g} = \begin{pmatrix}-6\cr-10\cr-14\end{pmatrix}

Dieses Kapitel enthält die folgenden Themen:

 

25.2 Vektorrechnung - Erklärungen

Wie Sie in den letzten Kapiteln ja wahrscheinlich schon gemerkt haben, ist die Mathematik nach den Grundlagenthemen noch lange nicht fertig - im Gegenteil: Sie hat noch viele weitere Konzepte "vorrätig". In diesem Kapitel kommt nun die Unterscheidung zwischen skalaren und vektoriellen Größen hinzu. Das, was Sie bislang kennengelernt haben, sind Skalare gewesen, nämlich Maßzahlen evtl. zusammen mit einer Maßeinheit, z. B. Längen, Zeiten oder Preise. Bei Vektoren kommt - wenn man sie geometrisch interpretiert - zu der Maßzahl noch die Angabe einer Richtung und einer Orientierung hinzu, z. B. bei Geschwindigkeiten oder Kräften. Gerade in der Physik (beispielsweise bei den Bewegungsgleichungen Isaac Newtons) wird das Verstehen schwierig, wenn man nicht weiß, was Vektoren sind und wie man mit ihnen rechnet. Bei wirtschaftlichen Betrachtungen sind Richtung und Orientierung zwar meist eher unwichtig, trotzdem hat sich das Rechnen mit Vektoren hier in vielen Fällen als sinnvoll und praktisch herausgestellt.

Das Wort "Vektor" kommt aus dem Lateinischen und bedeutet "Träger" oder "Passagier"; sprich: Es geht darum, etwas von einem Ort zu einem anderen zu transportieren, was dem gerade beschriebenen physikalischen Verständnis von Vektoren nahe kommt. Das Wort "Skalar" stammt - ebenso wie "Skala" und "Skalierung" - vom lateinischen Wort "scala" ab, welches "Leiter" oder "Treppe" bedeutet. Skalare lassen sich also wie Treppenstufen anordnen. Dass man in der Vektorrechnung lieber "Skalar" statt einfach "Zahl" sagt, hat u. a. damit zu tun, dass sie sich dadurch leichter von den Zahlen im Vektor, die ja eine ganz andere "Aufgabe" haben, unterscheiden lassen.

 

Was ist ein Vektor?

Es gibt viele Varianten, einen Vektor zu beschreiben: Die Spanne reicht von der mathematisch-theoretischen Denkweise, in der ein Vektor ein "Repräsentant eines Vektorraumes" ist, bis zur anschaulichen Beschreibung eines Vektors als Pfeil. Wir nutzen in diesem Kapitel zwei Definitionen eines Vektors:

Komponentenweise: Ein Vektor ist ein n-Tupel reeller (oder auch komplexer) Zahlen. Diese Zahlen nennt man die Komponenten des Vektors. Man kann mithilfe eines Vektors also mehrere Zahlen zu einem (mathematischen) Objekt zusammenfassen und damit rechnen. In dieser Hinsicht haben Vektoren eine gewisse Ähnlichkeit mit Tabellen, die ja auch strukturierte Zusammenstellungen von Zahlenwerten sind.
Man unterscheidet dabei Zeilenvektoren \vec{a}^{\,T}=\begin{pmatrix} a_1;\, a_2;\, \ldots;\, a_n\end{pmatrix} und Spaltenvektoren \vec{a}=\begin{pmatrix}a_1\cr a_2\cr\ldots\cr a_n\end{pmatrix}. Um einen Zeilenvektor zu kennzeichnen, verwendet man das hochgestellte T, welches für transponiert steht und Zeilen und Spalten gegeneinander tauscht. Für die geometrische Interpretation ist es im Großen und Ganzen egal, welche "Version" des Vektors man verwendet. Man kann sich hierbei also einfach die bequemere Schreibweise aussuchen. Bei der Durchführung von Rechenoperationen ist die Entscheidung schon wichtiger. Dazu werden wir weiter unten kommen.

Aus der Definition eines n-Tupels leiten sich zwei Dinge ab:

  1. n gibt an, aus wie vielen Komponenten der Vektor besteht.
  2. Die Reihenfolge der Komponenten ist wichtig. Die Vertauschung von Komponenten erzeugt einen anderen Vektor!

Für die Anschauung sind 2- und 3-komponentige Vektoren, also Vektoren in der Ebene und im Raum, praktisch. In der Mathematik, in der man sich ja (bekanntermaßen) an fehlender Anschauung nicht sehr stört, kann man problemlos auch mit n-komponentigen Vektoren arbeiten. Das Schöne dabei ist, dass sich an den Grundprinzipien nichts ändert. Im nächsten Abschnitt wird es dazu ein Beispiel geben.


Geometrisch: Der Pfeil, durch den der Vektor symbolisiert wird, zeichnet sich durch drei Angaben aus, die in gleicher Weise für den Vektor von Bedeutung sind: seine Länge, seine Richtung und seine Orientierung, wobei Orientierung meint, an welcher Seite des Pfeils die Pfeilspitze ist.
Damit ist schon ganz schön viel über einen Vektor festgelegt. Wichtig ist aber auch, was nicht festgelegt ist: Auch wenn Anfangs- und Endpunkt eines Vektors durchaus eine Rolle spielen können, sind sie im Allgemeinen unerheblich. Das heißt: Der Begriff "Vektor" bezeichnet nicht einen Pfeil, sondern viele, nämlich all diejenigen, die die gleiche Länge, gleiche Richtung und gleiche Orientierung haben. Falls Ihnen das komisch vorkommt: Letztendlich ist es bei Brüchen ganz ähnlich: \frac{1} {2} und \frac{2} {4} sehen zwar unterschiedlich aus, meinen aber die gleiche Zahl.
Auf der linken Seite der folgenden Grafik sehen Sie fünf Pfeile, die alle die gleiche Länge, die gleiche Richtung und die gleiche Orientierung haben. Es ist also ein Vektor dargestellt. Anfangs- und Endpunkt spielen dabei keine Rolle. Die Pfeile auf der rechten Seite unterscheiden sich hinsichtlich Länge, Richtung und/oder Orientierung. Hier sind also vier verschiedene Vektoren symbolisiert.

Berechnung eines Vektors aus Anfangs- und Endpunkt

Um von der Komponenten- zur grafischen Darstellung zu kommen und umgekehrt, braucht man - wie schon an anderen Stellen - ein Koordinatensystem. Das 2-achsige Koordinatensystem, welches wir bislang für Funktionen verwendet haben, hilft allerdings nur weiter, wenn die Vektoren zwei Komponenten haben. Haben die Vektoren drei Komponenten, benötigt man entsprechend ein Koordinatensystem mit drei Achsen. Mit Vektoren mit mehr als drei Komponenten lässt sich zwar problemlos rechnen. Da es mit der Anschauung dann aber sowieso nicht mehr funktioniert, brauchen wir uns auch keine Gedanken über ein passendes Koordinatensystem zu machen ...
Wie funktioniert das nun konkret: Betrachten wir zwei Punkte (der Einfachheit halber in der Ebene, sodass wir 2-komponentige Vektoren bekommen) P\left(x_p\mid y_p\right) und Q\left(x_q\mid y_q\right). Möchte man von P zu Q gelangen, muss man sich x_q-x_p Einheiten in x-Richtung und y_q-y_p Einheiten in y-Richtung bewegen. Damit haben wir auch schon den entsprechenden Vektor berechnet. Man erhält also einen Vektor aus seinem Anfangs- und Endpunkt, indem man die Koordinaten des Anfangspunkts von den Koordinaten des Endpunkts subtrahiert. Man schreibt dafür \vec{PQ}=\begin{pmatrix}x_q\cr y_q\end{pmatrix}-\begin{pmatrix}x_p\cr y_p\end{pmatrix}=\begin{pmatrix}x_q-x_p\cr y_q-y_p\end{pmatrix}=\begin{pmatrix}x\cr y\end{pmatrix}
Dieser Vektor \vec{PQ} trägt zwar noch die Punkte im Namen, sagt aber letztendlich nur aus, dass man x Schritte in x-Richtung und y Schritte in y-Richtung gegangen ist. Damit haben wir den Kreis geschlossen zu der Aussage, dass ein Vektor nicht einen, sondern viele Pfeile bezeichnet. Dass diese Pfeile alle die gleiche Länge, gleiche Richtung und gleiche Orientierung haben, ist nicht so überraschend, weil sie ja alle aus den Komponenten x und y bestehen.
Ein Beispiel: Die Grafik rechts zeigt, wie aus den Punkten P\left(1\mid 2\right) und Q\left(8\mid 6\right) der Vektor \vec{PQ} berechnet wird: \vec{PQ}=\begin{pmatrix}8\cr6\end{pmatrix}-\begin{pmatrix}1\cr2\end{pmatrix}=\begin{pmatrix}8-1\cr6-2\end{pmatrix}=\begin{pmatrix}7\cr4\end{pmatrix}


Eine Folgerung: Zwei Vektoren \vec{a} und \vec{b} sind dann gleich, wenn sie die gleiche Anzahl Zeilen bzw. Spalten haben und alle Komponenten gleich sind. Man schreibt in diesem Fall: \vec{a}=\vec{b} bzw. \begin{pmatrix}a_1\cr a_2\cr a_3\end{pmatrix}=\begin{pmatrix}b_1\cr b_2\cr b_3\end{pmatrix}
Umgekehrt bedeutet das, dass zwei Vektoren dann verschieden sind, wenn sie sich in mindestens einer Komponente unterscheiden. Es müssen nicht alle Komponenten unterschiedlich sein; eine reicht völlig. Stellen Sie sich das geometrisch vor: Bereits die Veränderung einer Komponente reicht, damit die Richtung und/oder Länge des Vektors anders ist.


Noch etwas Formales: Es gibt verschiedene Varianten, wie man Vektoren bezeichnet. In diesem Kapitel werden kleine Pfeile über der (lateinischen) Variablenbezeichnung oder Pfeile über dem Anfangs- und Endpunkt des Vektors verwendet. In anderen Texten werden auch halbfette Buchstaben (was handschriftlich eher unpraktisch ist) oder Unterstriche zur Kennzeichnung verwendet. In alten Mathebüchern findet man auch altdeutsche Frakturbuchstaben als Bezeichnung. Es kommt - mathematisch gesehen - nicht darauf an. Hauptsache, die Bezeichnungen sind eindeutig und einheitlich.

 

Erste Rechenoperationen mit Vektoren

Wie bei vielen mathematischen Größen möchte man auch bei Vektoren wissen, wie man mit ihnen rechnet - sonst würde man ja nicht viel mit ihnen anfangen können ...

Eine Einschränkung zu Beginn: Nicht alle diese Rechenoperationen sind in allen Anwendungszusammenhängen wirklich sinnvoll und aussagekräftig. Sie zu definieren ist natürlich trotzdem wichtig, um an den richtigen Stellen richtig mit ihnen umgehen zu können. Auch bei Skalaren führt ja nicht jede Rechnung automatisch zu einer brauchbaren Aussage. Beispiel: Die Feststellung, dass alle Professorinnen und Professoren einer Hochschule zusammen 5.884 Jahre alt sind, lässt sich nicht sinnvoll interpretieren. Die Aussage, dass das durchschnittliche Alter der Professorinnen und Professoren 61{,}2 Jahre ist, ist hingegen für die Personalplanung eine wichtige Größe.

Ebenso muss man auf die Einheiten aufpassen - und auch hier ist es bei Skalaren genauso: 5=5 ist sicher richtig. 5\, m=5\, kg passt weder von den Einheiten zusammen noch lassen sich sinnvolle Aussagen daraus ableiten.


Hier nun das oben versprochene Beispiel:
Der Zeilenvektor \vec{k}^{\,T}=\begin{pmatrix}0{,}5;\, 0{,}125;\, 3;\, 0{,}2;\, 0{,}2;\, 1\end{pmatrix} bezeichne das Rezept eines Rührkuchens: 0{,}5\,kg Mehl, 0{,}125\,l Milch, 3 Eier, 0{,}2\,kg Butter, 0{,}2\,kg Zucker und 1 Päckchen Backpulver. Auch wenn man Kuchenrezepte in Realität natürlich nicht als Vektor abbildet, lassen sich an diesem Beispiel einige Dinge gut veranschaulichen:

  1. Wir haben damit (ohne große Probleme) einen 6-komponentigen Vektor erzeugt. Wie oben gesagt: Ein Vektor ist erstmal "nur" eine Zusammenstellung von Werten. Man kann den Vektor \vec{k}^{\,T} zwar nicht zeichnen, verstehen kann man ihn trotzdem.
  2. Die Einheiten sind in diesem Vektor genauso wichtig wie sonst: Ändert man den Vektor zu \vec{k_1}^{T}=\begin{pmatrix}500;\, 0{,}125;\, 3;\, 0{,}2;\, 0{,}2;\, 1\end{pmatrix}, sollte man dazu sagen, dass die Mehlmenge jetzt in Gramm angegeben ist. Mit 500\,kg Mehl wird der Kuchen sicher nichts ...
  3. Auch das Vertauschen von Komponenten geht nicht einfach so: Den Vektor \vec{k_2}^{T}=\begin{pmatrix}1;\, 0{,}125;\, 3;\, 0{,}2;\, 0{,}2;\, 500\end{pmatrix} muss man in diesem Zusammenhang interpretieren als: 1\,g Mehl, 0{,}125\,l Milch, 3 Eier, 0{,}2\,kg Butter, 0{,}2\,kg Zucker und 500 Päckchen Backpulver. Ich denke, das spricht für sich ...

 

Addition und Subtraktion

Voraussetzung dafür, dass Vektoren addiert werden können, ist, dass alle Vektoren gleich viele Komponenten haben. Gleiches gilt (natürlich) für die Subtraktion. In den Formeln dieses Kapitels werden bloß Vektoren mit 3 Komponenten betrachtet. Für Vektoren mit 2, 4 oder mehr Komponenten funktionieren die Konzepte genauso. Damit die Vektoren ordentlich gezeichnet werden können, wurden für die Beispiele 2-komponentige Vektoren gewählt.

 

Addition

Komponentenweise: Die Summe zweier Vektoren \vec{a} und \vec{b} berechnet man, indem man die Komponenten der beiden Vektoren zeilenweise addiert, also \vec{a}+\vec{b}=\begin{pmatrix}a_1\cr a_2\cr a_3\end{pmatrix}+\begin{pmatrix}b_1\cr b_2\cr b_3\end{pmatrix}=\begin{pmatrix}a_1+b_1\cr a_2+b_2\cr a_3+b_3\end{pmatrix}

Geometrisch: Geometrisch verschiebt man bei der Addition von zwei Vektoren \vec{a} und \vec{b} den zweiten Vektor \vec{b} parallel, sodass sein Anfangspunkt im Endpunkt von \vec{a} liegt. Der Summenvektor führt dann vom Anfangspunkt von \vec{a} zum Endpunkt von \vec{b}. In der Schreibweise mit Anfangs- und Endpunkt ergibt sich daraus die folgende Gleichung: \vec{AB}+\vec{BC}=\vec{AC}. Da das entstehende Dreieck zu einem Parallelogramm ergänzt werden kann, kann man auch sagen: Der Summenvektor ist die Diagonale des Parallelogramms, das aus den Vektoren gebildet wird.

Beispiel: Gegeben seien \vec{a}=\begin{pmatrix}4\cr-8\end{pmatrix} und \vec{b}=\begin{pmatrix}2\cr3\end{pmatrix}. Dann ist \vec{a}+\vec{b}=\begin{pmatrix}4\cr-8\end{pmatrix}+\begin{pmatrix}2\cr3\end{pmatrix}=\begin{pmatrix}4+2\cr-8+3\end{pmatrix}=\begin{pmatrix}6\cr-5\end{pmatrix}

Hier die zugehörige Grafik:

Vektoraddition

Bei der Addition von mehr als zwei Vektoren erweitert man dieses Verfahren einfach, also: Anfangspunkt des zweiten Vektors an Endpunkt des ersten, Anfangspunkt des dritten Vektors an Endpunkt des zweiten, Anfangspunkt des vierten Vektors an Endpunkt des dritten, ... und verbindet den Anfangspunkt des ersten Vektors mit den Endpunkt des letzten - fertig. Dabei kann es passieren, dass der Endpunkt des letzten Vektors genau auf dem Anfangspunkt des ersten Vektors liegt. Die Summe aller dieser Vektoren ist dann \vec{0}. Dass man in Formelschreibweise einfach die entsprechenden Komponenten der einzelnen Vektoren - egal wie viele es sind - addiert, versteht sich fast von selbst ...


Zwei ganz wichtige Vektoren in diesem Zusammenhang:

  • Der Nullvektor: \vec{0}=\begin{pmatrix}0\cr0\cr0\end{pmatrix}. Geometrisch gesehen, bildet der Nullvektor einen Punkt auf sich selbst ab und ist daher der einzige Vektor, für den die Veranschaulichung durch einen Pfeil nicht funktioniert. Bitte achten Sie darauf, dass 0 (ein Skalar) und \vec{0} (ein Vektor, bestehend aus mehreren Werten) zwei völlig unterschiedliche Gebilde sind.

  • Der Gegenvektor: Zu einem Vektor \vec{a}=\begin{pmatrix}a_1\cr a_2\cr a_3\end{pmatrix} ist -\vec{a}=\begin{pmatrix}-a_1\cr-a_2\cr-a_3\end{pmatrix} der sogenannte Gegenvektor. Geometrisch betrachtet, hat -\vec{a} die gleiche Länge und die gleiche Richtung wie \vec{a}, aber die entgegengesetzte Orientierung. Anders gesagt, wenn \vec{a} von Punkt A zu Punkt B verläuft, verläuft -\vec{a} von B zu A.

 

Subtraktion

Wie beim "normalen Rechnen" ist auch bei Vektoren die Subtraktion die Umkehrung der Addition. Das macht die Sache einfach:

Komponentenweise: Die Differenz zweier Vektoren berechnet man, indem man die Komponenten der beiden Vektoren zeilenweise subtrahiert, also \vec{a}-\vec{b}=\begin{pmatrix}a_1\cr a_2\cr a_3\end{pmatrix}-\begin{pmatrix}b_1\cr b_2\cr b_3\end{pmatrix}=\begin{pmatrix}a_1-b_1\cr a_2-b_2\cr a_3-b_3\end{pmatrix}
Wenn man möchte, kann man das - ähnlich wie bei ganzen Zahlen - als Addition des Gegenvektors verstehen: \vec{a}-\vec{b}=\vec{a}+\left(-\vec{b}\right)=\begin{pmatrix}a_1\cr a_2\cr a_3\end{pmatrix}+\begin{pmatrix}-b_1\cr-b_2\cr-b_3\end{pmatrix}=\begin{pmatrix}a_1-b_1\cr a_2-b_2\cr a_3-b_3\end{pmatrix}. Das Ergebnis ist natürlich dasselbe.

Geometrisch: Der Gedanke mit dem Gegenvektor liefert einen guten Einstieg für die geometrische Variante von \vec{a}-\vec{b}: Da \vec{a}-\vec{b}=\vec{a}+\left(-\vec{b}\right), verschiebt man den Gegenvektor von \vec{b} so, dass sein Anfangspunkt im Endpunkt von \vec{a} liegt. Der Differenzvektor führt dann vom Anfangspunkt von \vec{a} zum Endpunkt von -\vec{b}. Alternativ kann man sich die Frage stellen, welcher Vektor zu \vec{b} addiert werden muss, damit \vec{a} herauskommt - auch diese Überlegung kennt man vom Rechnen mit Skalaren.

Beispiel: Gegeben seien wieder \vec{a}=\begin{pmatrix}4\cr-5\end{pmatrix} und \vec{b}=\begin{pmatrix}2\cr3\end{pmatrix}. Dann ist \vec{a}-\vec{b}=\begin{pmatrix}4\cr-5\end{pmatrix}-\begin{pmatrix}2\cr3\end{pmatrix}=\begin{pmatrix}4-2\cr-5-3\end{pmatrix}=\begin{pmatrix}2\cr-8\end{pmatrix}

Hier die zugehörige Grafik:

Vektorsubtraktion

Da sich Addition und Subtraktion von Vektoren direkt auf die Addition und Subtraktion von reellen Zahlen zurückführen lassen, gelten hier die gleichen Rechengesetze, z. B. Kommutativ- und Assoziativgesetz. Das heißt, Vektoren verhalten sich hier angenehmerweise so, wie man es gewohnt ist.

 

Skalare Multiplikation

Eine Bemerkung vorweg: Aus Gründen, die wahrscheinlich keine/r mehr wirklich benennen kann, bezeichnet man Skalare in der Vektorrechnung gerne mit griechischen Buchstaben, am liebsten mit \lambda (gesprochen: "lambda") oder \mu (gesprochen: "mü"). Geschickt ist das auf jeden Fall, weil dadurch Verwechslungen zwischen Skalaren und Vektoren vermieden werden.


Komponentenweise: Ein Vektor \vec{a} wird mit dem Skalar \lambda\in\mathbb{R} multipliziert, indem jede Komponente des Vektors mit dem Skalar multipliziert wird, also \lambda\cdot \vec{a}=\lambda\cdot\begin{pmatrix}a_1\cr a_2\cr a_3\end{pmatrix}=\begin{pmatrix}\lambda\cdot a_1\cr\lambda\cdot a_2\cr\lambda\cdot a_3\end{pmatrix}

Bemerkung 1: Den Multiplikationspunkt \cdot darf man - wie bei der "normalen" Multiplikation - weglassen, wenn die Schreibweise eindeutig bleibt.
Bemerkung 2: Ebenfalls wie bei der "normalen" Multiplikation kann man die skalare Multiplikation dafür nutzen, "unhandliche" Faktoren vorzuziehen. Ein Beispiel: \begin{pmatrix}-\frac{5}{2}\cr2\cr\frac{7}{2}\end{pmatrix}=\dfrac{1}{2}\begin{pmatrix}-5\cr4\cr7\end{pmatrix}

Geometrisch: Die geometrische Interpretation der skalaren Multiplikation eines Vektors \vec{a} mit dem Skalar \lambda\in\mathbb{R} ist nicht sehr überraschend: Die Länge des Vektors ändert sich um den Faktor \left|\lambda\right| . An der Richtung ändert sich hingegen nichts. Eventuell wird allerdings die Orientierung umgekehrt. Das charakterisiert man mit folgenden Begriffen:

  • Multipliziert man einen Vektor \vec{a} mit einem positiven Skalar \lambda_1, so hat \lambda_1\vec{a} die gleiche Orientierung wie \vec{a}. Die Vektoren \vec{a} und \lambda_1\vec{a} sind parallel oder auch gleichsinnig parallel.

  • Multipliziert man einen Vektor \vec{a} mit einem negativen Skalar \lambda_2, so hat \lambda_2\vec{a} die entgegen gesetzte Orientierung von \vec{a}. Die Vektoren \vec{a} und \lambda_2\vec{a} sind antiparallel oder auch gegensinnig parallel. Die Multiplikation mit -1 erzeugt den Gegenvektor.

  • Multipliziert man einen Vektor mit 0, entsteht der Nullvektor, was vermutlich wenig überraschen wird ... Anders ist es da schon mit der Festlegung, dass der Nullvektor zu jedem Vektor parallel ist. Dies ist nicht wirklich offensichtlich, aber später in verschiedenen Zusammenhängen nützlich.

Parallele und antiparallele Vektoren lassen sich durch Parallelverschiebung in eine Linie bringen. Sie heißen deshalb auch kollinear (abgeleitet vom Lateinischen "collinearis": auf der gleichen Gerade/Linie liegend).

Bis hierhin wurden in diesem Kapitel bloß Vektoren mit 3 Komponenten betrachtet. Für Vektoren mit 2, 4 oder mehr Komponenten funktionieren die Konzepte genauso. Auch hier noch zwei Beispiele mit einem 2-komponentigen Vektor:

  • Parallelität: Gegeben seien \vec{a}=\begin{pmatrix}3\cr-4\end{pmatrix} und \lambda_1 =2 . Das skalare Produkt ist  \lambda_1\vec{a}=2\begin{pmatrix}3\cr-4\end{pmatrix}=\begin{pmatrix}2\cdot 3\cr2\cdot \left(-4\right)\end{pmatrix}=\begin{pmatrix}6\cr-8\end{pmatrix}

  • Antiparallelität: Gegeben seien \vec{a}=\begin{pmatrix}3\cr-4\end{pmatrix} und \lambda_2=-2{,}5 . Das skalare Produkt ist \lambda_2\vec{a}=-2{,}5\begin{pmatrix}3\cr-4\end{pmatrix}=\begin{pmatrix}-2{,}5\cdot 3\cr-2{,}5\cdot \left(-4\right)\end{pmatrix}=\begin{pmatrix}-7{,}5\cr10\end{pmatrix}


Hier das Ganze in einer Grafik:

parallele und antiparallele Vektoren

Aus dem gleichen Grund wie bei Addition und Subtraktion gelten auch hier das Kommutativ- und das Assoziativgesetz sowie zusätzlich zwei Varianten des Distributivgesetzes: Für die Vektoren \vec{a} und \vec{b} sowie die Skalare \lambda , \mu\in\mathbb{R} gelten

  • \lambda\left(\vec{a}+\vec{b}\right)=\lambda\vec{a}+\lambda\vec{b}
  • \left(\lambda+\mu\right)\vec{a}=\lambda\vec{a}+\mu\vec{a}

 

Was ist eine Linearkombination?

Mit diesen gerade besprochenen Rechenoperationen kann man schon eine ganze Menge erreichen: Bereits mit zwei 2-komponentigen Vektoren kann man mithilfe von Vektoraddition und skalarer Multiplikation eine komplette Ebene erzeugen. Drei 3-komponentige Vektoren und Vektoraddition und skalare Multiplikation reichen für den 3-dimensionalen Raum.

Etwas formaler die folgende Vokabel:
Definition: Verknüpft man mehrere n-komponentige Vektoren mittels Addition und skalarer Multiplikation, spricht man von einer Linearkombination. In einer Formel: \vec{a}=\lambda_1\vec{a_1}+\lambda_2\vec{a_2}+\dots+\lambda_n\vec{a_n} mit den Skalaren \lambda_1,\dots, \lambda_n\in\mathbb{R} und den n-komponentigen Vektoren \vec{a_1} bis \vec{a_n}.
Wichtig: Eine Linearkombination von Vektoren ergibt wieder einen Vektor. In diesem Fall ist das der Vektor \vec{a}.

Ein Beispiel:
Wir betrachten noch mal das Kuchenbeispiel von oben. Ein Rezept für Nusskuchen lautet: 0{,}4\,kg Mehl, 0{,}125\,l Milch, 4 Eier, 0{,}15\,kg Butter, 0{,}2\,kg Zucker, 1{,}5 Päckchen Backpulver und 0{,}3\,kg Nüsse. Dieses Rezept lässt sich durch den Vektor \vec{k_N}^{T}=\begin{pmatrix}0{,}4;\, 0{,}125;\, 4;\, 0{,}15;\, 0{,}2;\, 1{,}5;\, 0{,}3\end{pmatrix} darstellen.
Damit der Vektor \vec{k}^{\,T} des Rührkuchens die gleiche Anzahl an Komponenten hat, müssen wir als letzte Komponente eine 0 ergänzen, denn Nüsse sind ja nicht enthalten: \vec{k_R}^{T}=\begin{pmatrix}0{,}5;\, 0{,}125;\, 3;\, 0{,}2;\, 0{,}2;\, 1;\, 0\end{pmatrix} . Bitte beachten Sie, dass der Rührkuchenvektor einen anderen Namen bekommen hat! Da nur Vektoren, die die gleiche Anzahl von Komponenten haben, identisch sein können, gilt: \vec{k_R}^{T}\neq\vec{k}^{\,T}

Nun lässt sich mithilfe einer Linearkombination bestimmen, wie viele Zutaten vorhanden sein müssen, wenn 3 Nusskuchen und 2 Rührkuchen gebacken werden sollen, nämlich:
\begin{array}{rcl}\vec{z} &=& 3\cdot\vec{k_N}^{T} + 2\cdot\vec{k_R}^{T} \cr\cr &=& 3\cdot\begin{pmatrix}0{,}4;\, 0{,}125;\, 4;\, 0{,}15;\, 0{,}2;\, 1{,}5;\, 0{,}3\end{pmatrix} + 2\cdot \begin{pmatrix}0{,}5;\, 0{,}125;\, 3;\, 0{,}2;\, 0{,}2;\, 1;\, 0\end{pmatrix} \cr \cr &=& \begin{pmatrix}3\cdot 0{,}4;\, 3\cdot 0{,}125;\, 3\cdot 4;\, 3\cdot 0{,}15;\, 3\cdot 0{,}2;\, 3\cdot 1{,}5;\, 3\cdot 0{,}3\end{pmatrix} + \begin{pmatrix}2\cdot 0{,}5;\, 2\cdot 0{,}125;\, 2\cdot 3;\, 2\cdot 0{,}2;\, 2\cdot 0{,}2;\, 2\cdot 1;\, 2\cdot 0\end{pmatrix} \cr \cr &=& \begin{pmatrix}1{,}2;\, 0{,}375;\, 12;\, 0{,}45;\, 0{,}6;\, 4{,}5;\, 0{,}9\end{pmatrix} + \begin{pmatrix}1;\, 0{,}25;\, 6;\, 0{,}4;\, 0{,}4;\, 2;\, 0\end{pmatrix} \cr \cr &=& \begin{pmatrix}1{,}2+1;\, 0{,}375+0{,}25;\, 12+6;\, 0{,}45+0{,}4;\, 0{,}6+0{,}4;\, 4{,}5+2;\, 0{,}9+0 \end{pmatrix} \cr \cr &=& \begin{pmatrix}2{,}2;\, 0{,}625;\, 18;\, 0{,}85;\, 1;\, 6{,}5;\, 0{,}9\end{pmatrix} \end{array}
Man benötigt also insgesamt 2{,}2\,kg Mehl, 0{,}625\,l Milch, 18 Eier, 0{,}85\,kg Butter, 1\,kg Zucker, 6{,}5 Päckchen Backpulver und 0{,}9\,kg Nüsse.

Zugegebenermaßen hätte man das auch ohne Vektorrechnung problemlos ausrechnen können. Das Beispiel soll auch nur zeigen, dass sich hinter dem Begriff "Linearkombination" kein wirklich schwieriges Konzept versteckt ...

 

Der Betrag eines Vektors

Definition: Der Betrag eines Vektors \vec{a} ist ein Maß für seine Länge oder - wenn man es anschaulich möchte - ein Maß für die Länge des Pfeils. Wie bei Zahlen schreibt man dafür: \left|\vec{a}\right|.


Komponentenweise:
Bei der Berechnung hilft uns ein "alter Bekannter", nämlich der Satz des Pythagoras. Daraus ergeben sich die folgenden Formeln: \left| \vec{a} \right| = \begin{pmatrix}a_1\cr a_2\end{pmatrix} = \sqrt{{a_1}^2 + {a_2}^2} für 2-komponentige Vektoren, \left| \vec{a} \right| = \begin{pmatrix}a_1\cr a_2\cr a_3\end{pmatrix} = \sqrt{{a_1}^2 + {a_2}^2 + {a_3}^2 } für 3-komponentige Vektoren bzw. \left| \vec{a} \right| = \begin{pmatrix}a_1\cr a_2\cr\vdots\cr a_n\end{pmatrix} = \sqrt{{a_1}^2 + {a_2}^2 + \ldots + {a_n}^2 } für den allgemeinen Fall.
Aufgrund der Wurzeleigenschaften ist der Betrag eines Vektors natürlich immer größer oder gleich 0. Ist ja für eine Länge auch logisch ... Weil die einzelnen Komponenten unter der Wurzel quadriert werden, gilt \left|\vec{a}\right|=\left|-\vec{a}\right|


Geometrisch:

Betrag eines Vektors

Damit haben wir gleichzeitig erklärt, wo diese Formeln herkommen - zumindest für den 2-dimensionalen Fall ...

Ein geometrischer Anwendungsfall: Um von zwei gegebenen Vektoren \vec{a} und \vec{b} den Abstand ihrer Endpunkte zu bestimmen, rechnet man: \left|\vec{a}-\vec{b}\right| bzw. \left|\vec{b}-\vec{a}\right| . Da sich \vec{a}-\vec{b} und \vec{b}-\vec{a} nur im Vorzeichen unterscheiden, das ja durch den Betrag ohnehin wegfällt, ist es egal, welche Differenz man bildet.


Und nun ein Beispiel:
Gegeben sei \vec{a} = \begin{pmatrix}9\cr-10\cr1\end{pmatrix}
Der Betrag dieses Vektors ist \left|\vec{a}\right|=\sqrt{9^2+\left(-10\right)^2+1^2}=\sqrt{182}\approx 13{,}49


Statt vom "Betrag des Vektors" wird auch von der "Norm des Vektors" gesprochen. Statt \left|\vec{a}\right| schreibt man dann \Vert\vec{a}\Vert . Das Konzept der "Norm" ist in der höheren Mathematik von größerer Bedeutung, als hier vielleicht deutlich wird. Schon mal gut zu wissen ist, dass normierte Vektoren, sprich Vektoren, die den Betrag 1 haben, häufig nützlich sind. Um einen beliebigen Vektor \vec{a}\neq\vec{0} zu normieren, muss man diesen Vektor mit dem Kehrwert seines Betrages skalarmultiplizieren, man rechnet also: \vec{a_n} = \dfrac{1}{\left|\vec{a}\right|}\,\vec{a}. Der entstehende Vektor hat dieselbe Richtung und Orientierung wie \vec{a}, aber eben den Betrag 1. Im oberen Beispiel würde das so aussehen: \vec{a_n} = \dfrac{1}{\sqrt{182}}\begin{pmatrix}9\cr-10\cr1\end{pmatrix}

 

Lineare Unabhängigkeit

Ein wichtiges Konzept bei Vektoren, das es so bei Zahlen nicht gibt, ist die lineare Unabhängigkeit.

Definition: Die Vektoren \vec{a_1} , \vec{a_2} , \dots , \vec{a_n} heißen linear unabhängig, wenn die Gleichung \lambda_1\vec{a_1}+\lambda_2\vec{a_2}+\dots+\lambda_n\vec{a_n}=\vec{0} nur die Lösung \lambda_1=\lambda_2=\dots\lambda_n=0 hat.
Anders gesagt: Eine Linearkombination dieser Vektoren ergibt nur dann den Nullvektor, wenn alle skalaren Koeffizienten 0 sind.
Und noch eine andere Formulierung: Keiner der Vektoren lässt sich aus den anderen kombinieren.

Definition: Die Vektoren \vec{a_1} , \vec{a_2} , \dots , \vec{a_n} heißen linear abhängig, wenn die Gleichung \lambda_1\vec{a_1}+\lambda_2\vec{a_2}+\dots+\lambda_n\vec{a_n}=\vec{0} mehrdeutig lösbar ist. Das heißt, wenn die Gleichung nicht nur die Lösung \lambda_1=\lambda_2=\dots\lambda_n=0 hat, sondern es noch weitere Lösungen gibt.
Anders gesagt: Eine Linearkombination dieser Vektoren ergibt auch dann den Nullvektor, wenn nicht alle skalaren Koeffizienten 0 sind.
Und auch hier noch eine andere Formulierung: Mindestens einer dieser Vektoren ist eine Linearkombination der übrigen Vektoren, lässt sich also mithilfe der anderen darstellen.


Ein Merksatz zur linearen Unabhängigkeit:
Die Anzahl der Komponenten gibt vor, wie viele linear unabhängige Vektoren es maximal geben kann: in der Ebene zwei, im Raum drei. Die Vektoren \begin{pmatrix}1\cr0\cr0\end{pmatrix}, \begin{pmatrix}0\cr1\cr0\end{pmatrix} und \begin{pmatrix}0\cr0\cr1\end{pmatrix} (Man nennt sie auch Einheitsvektoren.) sind das beste Beispiel dafür: Es gibt keine weitere Möglichkeit, eine 1 in einen 3-komponentigen Vektor "unterzubringen".
Wichtige Folgerung: Jeder weitere Vektor mit 3 Komponenten lässt sich eindeutig aus diesen erzeugen. Das ist natürlich nicht bloß bei diesen speziellen Vektoren so: Hat man drei linear unabhängige 3-komponentige Vektoren, kann man damit jeden Vektor des 3-dimensionalen Raumes eindeutig darstellen.


Merksätze zur linearen Abhängigkeit:

  • Eine Menge von Vektoren, die den Nullvektor enthält, ist immer linear abhängig. Zur Verdeutlichung ein Beispiel: Die Vektorgleichung \lambda_1\begin{pmatrix}1\cr0\cr-4\end{pmatrix}+\lambda_2\begin{pmatrix}-3\cr-2\cr7\end{pmatrix}+\lambda_3\begin{pmatrix}0\cr0\cr0\end{pmatrix}=\vec{0} hat unendlich viele Lösungen, z. B.
    • \lambda_1 =0 , \lambda_2 =0 und \lambda_3 =1
    • \lambda_1 =0 , \lambda_2 =0 und \lambda_3=-5
    • \lambda_1 =0 , \lambda_2 =0 und \lambda_3=\frac{112}{47}
    • oder oder oder ...

  • Eine Menge linear abhängiger Vektoren enthält sozusagen "überflüssige" Informationen: Betrachtet man zwei linear abhängige Vektoren, beispielsweise \vec{a} und 2\vec{a}, kann man mit ihnen sämtliche Punkte einer Geraden darstellen. Das hätte man aber auch mit nur einem der beiden Vektoren erreichen können. Zwei linear unabhängige Vektoren spannen hingegen eine Ebene auf. Man "gewinnt" also eine Dimension hinzu. Das führt zu den folgenden beiden Aussagen:
    • Zwei Vektoren sind linear abhängig, wenn sie parallel sind.
    • Drei Vektoren sind linear abhängig, wenn sie in einer Ebene liegen. So, wie zwei parallele Vektoren kollinear heißen (siehe oben), heißen drei Vektoren, die in einer Ebene liegen, komplanar (abgeleitet vom Lateinischen "planum": in der gleichen Ebene/Plane liegend).



Wie stellt man nun fest, ob eine Menge von Vektoren linear (un)abhängig ist?
Es ist leider so, dass es Vektoren meist nicht sofort anzusehen ist, ob sie linear abhängig oder linear unabhängig sind. Daher folgendes Verfahren: Man betrachtet \lambda_1\vec{a_1}+\lambda_2\vec{a_2}+\dots+\lambda_n\vec{a_n}=\vec{0}, also eine Vektorgleichung mit den Variablen \lambda_1 bis \lambda_n. Daraus entsteht ein lineares Gleichungssystem, dessen Lösung Auskunft über die lineare (Un)Abhängigkeit der Vektoren gibt.

Hier der Rechenweg anhand von zwei Beispielen:
1.
Gegeben seien die drei Vektoren \begin{pmatrix}1\cr-1\cr1\end{pmatrix} , \begin{pmatrix}3\cr0\cr4\end{pmatrix} und \begin{pmatrix}11\cr-5\cr3\end{pmatrix}
Um zu entscheiden, ob sie linear abhängig oder unabhängig sind, muss die folgende Gleichung gelöst werden: \lambda_1\begin{pmatrix}1\cr-1\cr1\end{pmatrix}+\lambda_2\begin{pmatrix}3\cr0\cr4\end{pmatrix}+\lambda_3\begin{pmatrix}11\cr-5\cr3\end{pmatrix}=\vec{0}

Diese führt zu dem Gleichungssystem:
\begin{array}{crcrcrcrl} \text{I} & \lambda_1 &+& 3\lambda_2 &+& 11\lambda_3 &=& 0 \cr \text{II} & -\lambda_1 & & &-& 5\lambda_3 &=& 0 \cr \text{III} & \lambda_1 &+& 4\lambda_2 &+& 3\lambda_3 &=& 0 \cr \cr \text{I} & \lambda_1 &+& 3\lambda_2 &+& 11\lambda_3 &=& 0 \cr \text{I+II} & & & 3\lambda_2 &+& 6\lambda_3 &=& 0 \cr \text{I-III} & & & -\lambda_2 &+& 8\lambda_3 &=& 0 \cr \cr \text{I} & \lambda_1 &+& 3\lambda_2 &+& 11\lambda_3 &=& 0 \cr \text{II'} & & & 3\lambda_2 &+&6\lambda_3 &=& 0 \cr \text{II'+3 III'} & & & & & 30\lambda_3 &=& 0 & \Rightarrow \lambda_3 = 0 \cr \cr \text{II'} & & & 3\lambda_2 &+& 6\cdot 0 &=& 0 & \Rightarrow \lambda_2 = 0 \cr \text{I} & \lambda_1 &+& 3 \cdot 0 &+& 11 \cdot 0 &=& 0 & \Rightarrow \lambda_1 = 0 \end{array}

Ergebnis: Da \lambda_1=\lambda_2=\lambda_3=0 die einzige Lösung des Gleichungssystems ist, sind die drei Vektoren linear unabhängig.


2.
Gegeben seien die drei Vektoren \begin{pmatrix}-2\cr-2\cr6\end{pmatrix} , \begin{pmatrix}-5\cr3\cr-1\end{pmatrix} und \begin{pmatrix}-7\cr1\cr5\end{pmatrix}
Um zu entscheiden, ob sie linear abhängig oder unabhängig sind, muss die folgende Gleichung gelöst werden: \lambda_1\begin{pmatrix}-2\cr-2\cr6\end{pmatrix}+\lambda_2\begin{pmatrix}-5\cr3\cr-1\end{pmatrix}+\lambda_3\begin{pmatrix}-7\cr1\cr5\end{pmatrix}=\vec{0}

Diese führt zu dem Gleichungssystem:
\begin{array}{crcrcrcr} \text{I} & -2\lambda_1 &-& 5\lambda_2 &-& 7\lambda_3 &=& 0 \cr \text{II} & -2\lambda_1 &+& 3\lambda_2 &+& \lambda_3 &=& 0 \cr \text{III} & 6\lambda_1 &-& \lambda_2 &+& 5\lambda_3 &=& 0 \cr \cr \text{I} & -2\lambda_1 &-& 5\lambda_2 &-& 7\lambda_3 &=& 0 \cr \text{I-II} & & & -8\lambda_2 &-& 8\lambda_3 &=& 0 \cr \text{3 I+III} & & & -16\lambda_2 &-& 16\lambda_3 &=& 0 \cr \cr \text{I} & -2\lambda_1 &-& 5\lambda_2 &-& 7\lambda_3 &=& 0 \cr \text{II'} & & & -8\lambda_2 &-& 8\lambda_3 &=& 0 \cr \text{2 II'-III'} & & & & & 0 &=& 0 \end{array}

Wegen der entstandenen Nullzeile ist es offensichtlich unmöglich, eine eindeutige Lösung zu finden. Es gibt also unendlich viele Lösungen dieses Gleichungssystems. Sprich: Zu jeder Variable, die wir frei wählen, können wir die beiden anderen so berechnen, dass das Gleichungssystem gelöst wird. Nehmen wir z. B. \lambda_3= 1 . Eingesetzt in -8\lambda_2 - 8\lambda_3 = 0 führt dies zu \lambda_2= -1. Dies beides eingesetzt in die erste Gleichung ergibt \lambda_1= -1. Eine weitere Lösung (auf dem gleichen Weg berechnet) ist \lambda_1 =4, \lambda_2 =4 und \lambda_3=-4. Auch \lambda_1=\lambda_2=\lambda_3=0 ist im Übrigen eine Lösung des Gleichungssystems - aber eben nicht die einzige.

Ergebnis: Da die Gleichung keine eindeutige Lösung hat, sind die drei Vektoren linear abhängig. Ein Vektor lässt sich also als Linearkombination der anderen darstellen, z. B. ist \begin{pmatrix}-2\cr-2\cr6\end{pmatrix} + \begin{pmatrix}-5\cr3\cr-1\end{pmatrix} = 1\cdot\begin{pmatrix}-2\cr-2\cr6\end{pmatrix} + 1\cdot\begin{pmatrix}-5\cr3\cr-1\end{pmatrix} = \begin{pmatrix}-7\cr1\cr5\end{pmatrix}


Eine mathematische Randbemerkung: Da auf den rechten Seiten der Gleichungen jeweils eine 0 steht (mathematisch nennt man so etwas ein homogenes lineares Gleichungssystem), gibt es hier nur zwei Möglichkeiten für die Lösung: Entweder ist das Gleichungssystem eindeutig lösbar mit der Lösung \lambda_1=\lambda_2=\dots=\lambda_n=0 ; dann sind die Vektoren linear unabhängig. Oder das Gleichungssystem hat unendlich viele Lösungen; dann sind die Vektoren linear abhängig.

Eine lerntechnische Randbemerkung: Falls Sie mit dem Lösen linearer Gleichungssysteme Schwierigkeiten haben, schauen Sie bitte im entsprechenden Kapitel nach. Dort sind die verschiedenen Lösungswege ausführlich erklärt - und Übungsaufgaben gibt es auch.


Ausblick: Es gibt weitere Wege, um festzustellen, ob Vektoren linear (un)abhängig sind, beispielsweise kann man dafür auch Determinanten nutzen.

 

Produkte von Vektoren

Bei der Multiplikation von Vektoren muss man ein bisschen aufpassen: Anders als bei "normalen Zahlen" unterscheidet man hier vier verschiedene Produkte! Neben der skalaren Multiplikation, die oben schon behandelt wurde ("Skalar mal Vektor gleich Vektor") gibt es noch das Skalarprodukt ("Vektor mal Vektor gleich Skalar"), das Vektorprodukt ("Vektor mal Vektor gleich Vektor") und das Spatprodukt, bei welchem Vektor- und Skalarprodukt verknüpft werden ("Vektor mal Vektor mal Vektor gleich Skalar"). Diese Vielfalt resultiert daraus, dass wir nicht mehr nur Objekte gleichen Typs miteinander multiplizieren (also "eine Zahl mal eine andere Zahl"), sondern nun verschiedene mathematische Objekte zur Hand haben, nämlich Zahlen und Vektoren, sodass sich einfach mehr Kombinationsmöglichkeiten ergeben.

Ganz wichtig: Eine Division durch Vektoren ist nicht definiert! Möglich ist allerdings, einen Vektor durch einen Skalar zu teilen - das ist eine Folgerung aus der skalaren Multiplikation, die ja die Multiplikation von Vektoren mit Brüchen nicht ausschließt.

 

Skalarprodukt

Wir beginnen mit dem Skalarprodukt. Das Zeichen für diese Rechenoperation ist \cdot, also der gleiche Multiplikationspunkt wie bei der skalaren Multiplikation. Da man Skalar und Vektor aber gut unterscheiden kann, ist das nicht das Problem. Ein bisschen aufpassen sollte man trotzdem ...
Man nennt das Skalarprodukt auch inneres Produkt oder Punktprodukt.

Wichtig vorab:

  • Ein Skalarprodukt kann man nur berechnen, wenn beide Vektoren die gleiche Anzahl von Komponenten haben.
  • Aus Gründen, die später bei der Matrizenrechnung wichtig werden, benötigen wir den ersten Faktor als Zeilenvektor und den zweiten als Spaltenvektor. Man sagt auch abkürzend: "Zeilenvektor mal Spaltenvektor". In anderen Lehrbüchern wird das Skalarprodukt auch als "Spaltenvektor mal Spaltenvektor" notiert. Rechnerisch macht das auch tatsächlich keinen großen Unterschied, aber der Zusammenhang von Skalarprodukt und Matrixmultiplikation wird nicht so deutlich.



Definition:
Komponentenweise: \vec{a}^{\,T} \cdot \vec{b} = \begin{pmatrix}a_1;\, a_2;\, a_3\end{pmatrix} \cdot \begin{pmatrix}b_1\cr b_2\cr b_3\end{pmatrix} = a_1b_1+a_2b_2+a_3b_3


Geometrisch: \vec{a}^{\,T} \cdot \vec{b}=\left|\vec{a}\right| \cdot \left|\vec{b}\right| \cdot \cos\left(\varphi\right)
In Worten: Das Skalarprodukt ist das Produkt der Beträge beider Vektoren und dem Kosinus des kleineren Winkels zwischen \vec{a} und \vec{b} , hier \varphi (gesprochen: "phi") genannt.

Wichtig: Skalarprodukte können immer nur von genau zwei Vektoren berechnet werden, denn das Ergebnis dieser Multiplikation ist ja ein Skalar. Würde man dieses Ergebnis mit einem weiteren Vektor multiplizieren wollen, wäre die Ausgangssituation also nicht mehr "Vektor mal Vektor".


Ein Beispiel:
Gegeben seien \vec{a}=\dfrac{1}{3}\begin{pmatrix}0\cr-2\cr-7\end{pmatrix} und \vec{b}=\begin{pmatrix}11\cr0\cr3\end{pmatrix}
Das Skalarprodukt dieser beiden Vektoren ist \vec{a}^{\,T}\cdot\vec{b}=\dfrac{1}{3}\begin{pmatrix}0;\,-2;\,-7\end{pmatrix} \cdot \begin{pmatrix}11\cr0\cr3\end{pmatrix} = \dfrac{1}{3}\cdot 0\cdot 11+\dfrac{1}{3}\cdot \left(-2\right)\cdot 0+\dfrac{1}{3}\cdot \left(-7\right) \cdot 3 =-7
Solange wir mit dem Winkel zwischen zwei Vektoren noch nichts anfangen können, hilft uns die geometrische Variante nicht viel. Sie wird aber weiter unten nützlich werden.


Rechengesetze beim Skalarprodukt:
Die Skalarmultiplikation hat - netterweise - mehr oder weniger die Eigenschaften einer "normalen" Multiplikation, z. B. kann man einfach Klammer ausmultiplizieren und das Kommutativ- bzw. Assoziativgesetz anwenden.

Einen bedeutenden Unterschied gibt es aber: Der Satz vom Nullprodukt gilt hier nicht! Das heißt: Das Skalarprodukt zweier Vektoren kann auch dann 0 sein, wenn beide Vektoren vom Nullvektor verschieden sind. Ein Beispiel: \begin{pmatrix}3;\,-2\end{pmatrix} \cdot \begin{pmatrix}4\cr6\end{pmatrix} = 3\cdot 4-2\cdot 6 = 0. Das ist (vermutlich) zunächst überraschend, ergibt sich aber direkt aus der geometrischen Form des Skalarprodukts: Neben den Beträgen der Vektoren spielt ja auch der Kosinus des eingeschlossenen Winkels eine Rolle. Und der Kosinus von 90^\circ bzw. \frac{\pi}{2} ist bekanntlich 0. Geometrisch interpretiert: Die Vektoren stehen also senkrecht (orthogonal) zueinander.
Zusammenfassung: Das Skalarprodukt zweier Vektoren ist genau dann 0, wenn (mindestens) einer der Vektoren der Nullvektor ist oder die Vektoren einen rechten Winkel einschließen.
Dazu noch eine Vokabel: Ein Vektor, der im rechten Winkel zu einem anderen Vektor (oder auf einem sonstigen geometrischen Objekt) steht, heißt Normalenvektor.


Schauen wir noch an, was passiert, wenn man das Skalarprodukt eines Vektors mit sich selbst berechnet: \vec{a}^{\,T} \cdot \vec{a} = \left|\vec{a}\right| \cdot \left|\vec{a}\right| \cdot \cos\left(0\right) = \left|\vec{a}\right|^2 \cdot 1 = \left|\vec{a}\right|^2
Ergebnis: Das Skalarprodukt eines Vektors mit sich selbst ist das Quadrat seines Betrages.
Zieht man daraus die Wurzel, erhält man \left|\vec{a}\right|=\sqrt{\vec{a}^{\,T} \cdot \vec{a}}. Daraus folgt, dass das Skalarprodukt eines Vektors mit sich selbst immer dann größer als 0 ist, wenn der Vektor nicht der Nullvektor ist. Wurzeln aus positiven Werten liefern ja bekanntlich positive Ergebnisse.


Ein Beispiel
Eine kleine Bemerkung vorab: Vektor \vec{p} wird beim ersten Auftreten nur deswegen als Zeilenvektor geschrieben, weil er dann weniger Platz braucht ...
Betrachten wir noch einmal das Rührkuchenbeispiel von oben: \vec{k_R}^{T}=\begin{pmatrix}0{,}5;\, 0{,}125;\, 3;\, 0{,}2;\, 0{,}2;\, 1\end{pmatrix} . Sind nun die Kosten pro Kilogramm Mehl, Liter Milch, Ei, Kilogramm Butter, Kilogramm Zucker und Päckchen Backpulver in dem folgenden Vektor zusammengetragen \vec{p}^{\,T}=\begin{pmatrix}0{,}79;\, 1{,}09\;\; 0{,}30\;\; 3{,}56\;\; 1{,}20\;\; 0{,}09\end{pmatrix} , liefert das Skalarprodukt einen Weg, den Gesamtpreis des Kuchens auszurechnen, nämlich:
\begin{array}{cclcl} \vec{k}^{\,T}\cdot\vec{p} &=& \begin{pmatrix}0{,}5;\, 0{,}125;\, 3;\, 0{,}2;\, 0{,}2;\, 1\end{pmatrix} \cdot \begin{pmatrix}0{,}79\cr1{,}09\cr0{,}30\cr3{,}56\cr1{,}20\cr0{,}09\end{pmatrix} \cr \cr &=& 0{,}5\cdot 0{,}79 + 0{,}125\cdot 1{,}09 + 3\cdot 0{,}30 + 0{,}2\cdot 3{,}56 + 0{,}2\cdot 1{,}20 + 1\cdot 0{,}09 &\approx& 2{,}47 \end{array}

Die Zutaten für den Kuchen kosten insgesamt also etwa 2{,}47 EUR.
Auch wenn man (verständlicherweise) fürs Kuchenbacken normalerweise nicht die Vektorrechnung bemüht, sind die hier aufgezeigten Prinzipien durchaus realistische Anwendungsfälle. Ersetzt man den "Rezeptvektor" durch einen allgemeinen Mengenvektor, ist der Weg vom Kuchenbacken zur innerbetrieblichen Leistungsverrechnung oder zur Produktionsplanung in Unternehmen nicht weit.


Eine Anwendung des Skalarprodukts: Die Berechnung von Winkeln zwischen Vektoren
Ein geometrisches Konzept: Die Ermittlung des Winkels, der von zwei Vektoren eingeschlossen wird.
Wichtig: Dies ist natürlich nur dann sinnvoll möglich, wenn beide Vektoren vom Nullvektor verschieden sind.

Dieser Winkel bzw. sein Kosinus tauchte ja etwas weiter oben bereits auf, nämlich in der Formeln \vec{a}^{\,T}\cdot\vec{b}=\left|\vec{a}\right| \cdot \left|\vec{b}\right| \cdot \cos\left(\varphi\right). Da es ja noch eine zweite Variante gibt, das Skalarprodukt zu berechnen, können wir die beiden Formeln kombinieren: a_1b_1+a_2b_2+a_3b_3=\left|\vec{a}\right| \cdot \left|\vec{b}\right| \cdot \cos\left(\varphi\right)
Nach ein bisschen Umstellen erhält man für die Winkelberechnung folgende Formel: \varphi = \arccos \left( \dfrac{ a_1 b_1 + a_2 b_2 + a_3 b_3}{\sqrt{{a_1}^2+{a_2}^2+{a_3}^2} \cdot \sqrt{{b_1}^2+{b_2}^2+{b_3}^2}} \right) = \arccos \left( \dfrac {\vec{a}^{\,T} \cdot \vec{b}}{\left|\vec{a}\right| \cdot \left|\vec{b}\right|} \right)

Dieser Winkel liegt zwischen 0^\circ und 180^\circ bzw. zwischen 0 und \pi.


Zwei Winkelbeispiele:
Betrachten wir noch mal das Beispiel von oben: \vec{a}=\dfrac{1}{3}\begin{pmatrix}0\cr-2\cr -7\end{pmatrix} und \vec{b}=\begin{pmatrix}11\cr0\cr3\end{pmatrix}
Das Skalarprodukt dieser beiden Vektoren ist (wie bereits berechnet): \vec{a}^{\,T}\cdot\vec{b} = -7
Um den Winkel zwischen diesen Vektoren zu bestimmen, benötigen wir nun noch die Beträge:
\left|\vec{a}\right| = \sqrt{0+\left(-\dfrac{2}{3}\right)^2+\left(-\dfrac{7}{3}\right)^2} = \dfrac{\sqrt{53}}{3}
\left|\vec{b}\right| = \sqrt{11^2+0+3^2} = \sqrt{130}
Der Winkel \varphi zwischen \vec{a} und \vec{b} ist also: \varphi = \arccos \left( \dfrac {-7}{\frac{\sqrt{53}}{3} \cdot \sqrt{130}} \right)\approx 104{,}65^\circ


Und nun - damit wir das Ganze vernünftig zeichnen können - noch ein Beispiel mit 2-komponentigen Vektoren:
Gegeben seien die Vektoren \vec{a}=\begin{pmatrix}5\cr0\end{pmatrix} und \vec{b}=\begin{pmatrix}4\cr7\end{pmatrix}
Das Skalarprodukt dieser beiden Vektoren ist: \vec{a}^{\,T}\cdot\vec{b} = 5\cdot 4+0\cdot 7
Um den Winkel zwischen diesen Vektoren zu bestimmen, benötigen wir nun noch die Beträge:
\left|\vec{a}\right| = \sqrt{5^2+0} = 5
\left|\vec{b}\right| = \sqrt{4^2+7^2} = \sqrt{65}
Der Winkel \varphi zwischen \vec{a} und \vec{b} ist also: \varphi = \arccos \left( \dfrac {20}{5 \cdot \sqrt{65}} \right)\approx 60{,}26^\circ
In einer Grafik sieht das dann so aus:

Winkel zwischen Vektoren

 

 

Die beiden folgenden Produkte von Vektoren sind in erster Linie für Ingenieurstudierende relevant.

 

Vektorprodukt

Um das Vektorprodukt vom Skalarprodukt unterscheiden zu können, wird hier ein anderes Zeichen benutzt, nämlich \times. Wegen dieses Zeichens wird das Vektorprodukt auch Kreuzprodukt genannt. Manchmal nennt man es auch äußeres Produkt. Die Bezeichnung "Vektorprodukt" gibt aber - wie Skalarprodukt - von vorneherein ganz klar an, wie das Ergebnis aussieht. Daher verwende ich hier diesen Begriff.

Wichtig vorab: Das Vektorprodukt gibt es in dieser Form nur für dreidimensionale Vektoren!


Definition:
Komponentenweise: \vec{a} \times \vec{b} = \begin{pmatrix}a_1\cr a_2\cr a_3\end{pmatrix} \times \begin{pmatrix}b_1\cr b_2\cr b_3\end{pmatrix} = \begin{pmatrix}a_2b_3-a_3b_2 \cr a_3b_1-a_1b_3 \cr a_1b_2 - a_2b_1\end{pmatrix}
Das sieht zunächst reichlich verwirrend aus ... Um sich die Formel zu merken, gibt es eine Eselsbrücke, die auch "Schnürsenkelprinzip" genannt wird:
Dazu schreibt man 1. die Komponenten der beiden Vektoren jeweils zweimal untereinander, streicht als 2. die erste und die letzte Zeile und subtrahiert dann als 3. die entstehenden Produkte über Kreuz:

Veranschaulichung des Schnürsenkelprinzips

Vektorprodukt als Rechts-Hand-System

Es werden also in jeder Zeile des Ergebnisses genau die anderen Zeilen der Ausgangsvektoren verknüpft: in der ersten Ergebniszeile die Elemente aus der 2. und 3. Zeile der Ausgangsvektoren etc. Zudem unterscheiden sich Minuend und Subtrahend im Ergebnis nur durch die Reihenfolge der Indizes.


Später werden Sie evtl. noch eine Möglichkeit kennenlernen, Vektorprodukte zu berechnen - allerdings muss man dazu mit Determinanten umgehen können.


Geometrisch: Vom geometrischen Standpunkt aus ist es ja immer wichtig, von einem Vektor Betrag, Richtung und Orientierung zu kennen. Da das Vektorprodukt nun mal einen Vektor als Ergebnis liefert, müssen diese Fragen auch hier beantwortet werden:
Der Betrag des Vektorprodukts entspricht der Fläche des Parallelogramms, das von \vec{a} und \vec{b} aufgespannt wird, also: \left|\vec{a} \times \vec{b}\right|=\left|\vec{a}\right|\cdot\left|\vec{b}\right|\cdot\sin\left(\varphi\right) , wobei \varphi der kleinere Winkel ist, der zwischen \vec{a} und \vec{b} liegt.
Zu Richtung und Orientierung: \vec{a}\times\vec{b} steht im rechten Winkel zu \vec{a} und \vec{b}. Zusammen bilden die Vektoren \vec{a}, \vec{b} und ihr Vektorprodukt ein so genanntes Rechte-Hand-System, in dem sich \vec{a} , \vec{b} und \vec{a}\times\vec{b} wie Daumen, Zeigefinger und Mittelfinger der rechten Hand verlaufen. Damit ist dann auch die Orientierung festgelegt.

Rechengesetze beim Vektorprodukt:
Anders als beim Skalarprodukt gelten für das Vektorprodukt Rechenregeln, die uns (noch) nicht so vertraut sind:

  • Das Vektorprodukt eines Vektors mit sich selbst ist immer der Nullvektor. In einer Formel: \vec{a}\times\vec{a}=\vec{0}. Etwas Allgemeiner gilt: \vec{a}\times\vec{b} =\vec{0} genau dann, wenn \vec{a} und \vec{b} parallel bzw. antiparallel sind oder wenn (mindestens) einer der Vektoren der Nullvektor ist. Geometrisch passt das: In all diesen Fällen wird von den Vektoren kein Parallelogramm aufgespannt.

  • Das Vektorprodukt ist nicht kommutativ, sondern antikommutativ (auch antisymmetrisch genannt): \vec{a}\times\vec{b} = -\left(\vec{b}\times\vec{a}\right) (Bitte das Minuszeichen nicht übersehen!)

  • Das Assoziativgesetz gilt nur, wenn ein Skalar \lambda\in\mathbb{R} beteiligt ist: \left(\lambda\vec{a}\right)\times\vec{b} = \lambda\left(\vec{a}\times\vec{b}\right)=\vec{a}\times\left(\lambda\vec{b}\right). Für drei Vektoren gilt es (im Allgemeinen) nicht!

  • Immerhin das Distributivgesetz gilt in seiner gewohnten Form: \left(\vec{a}+\vec{b}\right)\times\vec{c} = \vec{a}\times\vec{c}+\vec{b}\times\vec{c} bzw. \vec{a}\times\left(\vec{b}+\vec{c}\right)=\vec{a}\times\vec{b}+\vec{a}\times\vec{c}



Ein Rechenbeispiel:
Gegeben seien \vec{a} = \begin{pmatrix}1\cr-1\cr4\end{pmatrix} und \vec{b} = \begin{pmatrix}2\cr-3\cr0\end{pmatrix}
Das Vektorprodukt dieser Vektoren ist \vec{a} \times \vec{b} = \begin{pmatrix}1\cr-1\cr4\end{pmatrix} \times \begin{pmatrix}2\cr-3\cr0\end{pmatrix} = \begin{pmatrix}-1\cdot 0 - 4\cdot\left(-3\right) \cr 4\cdot 2 - 1\cdot 0 \cr 1\cdot \left(-3\right) - \left(-1\right)\cdot 2\end{pmatrix} = \begin{pmatrix}12\cr8\cr-1\end{pmatrix}


Anwendung: Physiker bzw. Ingenieure nutzen Vektorprodukte beispielsweise, um die Feldstärke von Magnetfeldern zu bestimmen. In der BWL kommt man üblicherweise ohne Vektorprodukte aus.

 

Spatprodukt

Abbildung eines Spats / Parallelepipeds

Kommen wir zum letzten Produkt von Vektoren: Da das Spatprodukt eine Kombination aus Skalar- und Vektorprodukt ist, wird meist kein eigenes Symbol dafür eingeführt. Man nennt das Spatprodukt (aus leicht einsehbaren Gründen) auch gemischtes Produkt.

Wichtig vorab 1: Da ein Vektorprodukt berechnet wird, müssen die Vektoren jeweils drei Komponenten haben.
Wichtig vorab 2: Da ein Skalarprodukt berechnet wird, ist das Ergebnis ein Skalar.


Definition:
Komponentenweise: \vec{a}^{\,T}\cdot \left(\vec{b} \times \vec{c}\right)
Bitte achten Sie darauf, dass die Klammern genauso gesetzt werden müssen. \left(\vec{a}^{\,T}\cdot \vec{b}\right) \times \vec{c} würde nicht funktionieren, weil \vec{a}^{\,T}\cdot \vec{b} ja einen Skalar als Ergebnis hat. Aus diesem Skalar und dem Vektor \vec{c} kann aber kein Vektorprodukt berechnet werden.

Geometrisch: In den geometrischen Betrachtungen steckt gleichzeitig die Erklärung, was man mit dem Spatprodukt anfangen kann. Dafür müssen wir zunächst die Frage klären, was ein Spat im mathematischen Sinne (man sagt dazu auch Parallelepiped) ist: Es handelt sich um ein Art dreidimensionales Parallelogramm, das von drei Vektoren im Raum aufgespannt wird (siehe Grafik rechts). Mithilfe des Spatprodukts kann man das Volumen eines solchen Spats bestimmen: V_{Spat} = \left|\vec{a}^{\,T}\cdot\left(\vec{b}\times\vec{c}\right)\right| . Es fügt sich in dieser Hinsicht gut an das Vektorprodukt an, mit dem ja die Fläche eines Parallelogramms berechnet werden kann. Und wie beim Vektorprodukt spielt auch hier die Orientierung eine Rolle, wenn wir auf den Betrag verzichten: Ist das Spatprodukt positiv, bilden \vec{a}, \vec{b} und \vec{c} (in dieser Reihenfolge) ein Rechtssystem. Ist das Spatprodukt negativ, bilden \vec{a}, \vec{b} und \vec{c} (in dieser Reihenfolge) ein Linkssystem. Man spricht daher auch vom orientierten Volumen (ähnlich dem orientierten Flächeninhalt bei Integralen).


Ein Beispiel:
Gegeben seien \vec{a}=\begin{pmatrix}2\cr-6\cr1\end{pmatrix} , \vec{b}=\begin{pmatrix}5\cr-1\cr-10\end{pmatrix} und \vec{c}=\begin{pmatrix}-3\cr0\cr-1\end{pmatrix}
Das Spatprodukt dieser Vektoren ist
\begin{array}{rcl} \vec{a}^{\,T}\cdot\left(\vec{b} \times \vec{c}\right) &=& \begin{pmatrix}2;\,-6;\,1\end{pmatrix}\cdot\left(\begin{pmatrix}5\cr-1\cr-10\end{pmatrix} \times \begin{pmatrix}-3\cr0\cr-1\end{pmatrix}\right) \cr &=& \begin{pmatrix}2;\,-6;\,1\end{pmatrix}\cdot\begin{pmatrix}-1\cdot \left(-1\right) - \left(-10\right)\cdot 0 \cr -10\cdot \left(-3\right) - 5\cdot \left(-1\right) \cr 5\cdot 0 - \left(-1\right)\cdot \left(-3\right)\end{pmatrix} \cr &=& \begin{pmatrix}2;\,-6;\,1\end{pmatrix}\cdot\begin{pmatrix}1\cr35\cr-3\end{pmatrix} \cr &=& 2\cdot 1 + \left(-6\right)\cdot 35 - 3 = -211 \end{array}


Rechengesetze beim Spatprodukt:

  • Da das Vektorprodukt eines Vektors mit sich selbst der Nullvektor und das Skalarprodukt eines beliebigen Vektors mit dem Nullvektor 0 ist, ist auch \vec{a}^{\,T}\cdot\left(\vec{b}\times\vec{b}\right) = 0.

  • Dass das Spatprodukt nicht kommutativ ist, ist relativ naheliegend, weil das Vektorprodukt es ja auch nicht ist. Allerdings gibt es hier stattdessen eine wirklich spannende Eigenschaft: Der Wert des Spatprodukts ändert sich nicht, wenn man die Faktoren zyklisch durchtauscht: \vec{a}^{\,T}\cdot\left(\vec{b}\times\vec{c}\right) = \vec{c}^{\,T}\cdot\left(\vec{a}\times\vec{b}\right) = \vec{b}^{\,T}\cdot\left(\vec{c}\times\vec{a}\right).

  • Ebenso wie das Vektorprodukt ist das Spatprodukt nicht assoziativ. Eine Art Assoziativgesetz gilt aber für Skalare: \lambda\vec{a}^{\,T}\cdot\left(\vec{b}\times\vec{c}\right) = \vec{a}^{\,T}\cdot\left(\lambda\vec{b}\times\vec{c}\right) = \vec{a}^{\,T}\cdot\left(\vec{b}\times\lambda\vec{c}\right) = \vec{a}^{\,T}\cdot\left(\vec{b}\times\vec{c}\right)\cdot\lambda. Anders formuliert: Multipliziert man einen der Vektoren mit einem Skalar, kann man diesen quasi aus dem Spatprodukt "ausklammern".

  • Noch ein Klassiker zum Abschluss - das Distributivgesetz: \left(\vec{a}+\vec{b}\right)^{\,T}\cdot\left(\vec{c}\times\vec{d}\right) = \vec{a}^{\,T}\cdot\left(\vec{c}\times\vec{d}\right) + \vec{b}^{\,T}\cdot\left(\vec{c}\times\vec{d}\right)



Einen Sonderfall müssen wir noch betrachten, nämlich \vec{a}^{\,T}\cdot\left(\vec{b} \times \vec{c}\right)=0. Dann muss das Volumen des betrachteten Spats 0 sein, was entweder passiert, wenn einer der Vektoren der Nullvektor ist oder die drei Vektoren in einer Ebene liegen (dann kann ja kein 3-dimensionales Gebilde entstehen). Das heißt, man kann mithilfe des Spatprodukts feststellen, ob drei gegebene Vektoren komplanar (in anderen Worten: linear abhängig) sind. Dieser Rechenweg ist manchmal einfacher, als der im Abschnitt "Lineare Unabhängigkeit" beschriebene.

 

Ausblick

Mathematisch kann man eine Menge mit Vektoren machen, was überhaupt nichts mehr mit Geometrie und Pfeilen zu tun hat. Das ist ja durchaus typisch für die Mathematik: Hat man in einem Bereich etwas Nützliches gefunden, sucht man nach Möglichkeiten, dieses Prinzip auf andere Bereiche zu übertragen.

Der ganz oben erwähnte Vektorraum ist ein Beispiel dafür: Ausgehend von Vektoren wurde ein (abstraktes) Konzept entwickelt, welches u. a. auf Funktionen angewendet werden kann. Das klingt zunächst vielleicht komisch, ist aber sehr praktisch. Wurde nämlich gezeigt, dass eine Menge von Funktionen einen Vektorraum bildet, weiß man, dass die Funktionen bestimmte Eigenschaften haben (müssen) und dass bestimmte Rechenverfahren problemlos angewendet werden können. Man muss dies also nicht für jede Eigenschaft und jedes Rechenverfahren einzeln überprüfen.

Etwas naheliegender sind natürlich Matrizen und Determinanten. Diese können beispielsweise beim Lösen linearer Gleichungssysteme, die ja in vielen Zusammenhängen auftreten, eingesetzt werden. Darauf baut z. B. auch der Simplex-Algorithmus zum Lösen linearer Optimierungsprobleme auf. Nach vielen militärischen Anwendungsfällen wird dieses Verfahren heute im Rahmen des Operations Research vor allem in Unternehmen eingesetzt, um herauszufinden, wie die gegebenen Mittel eingesetzt werden müssen, um das beste aller möglichen Ergebnisse zu erreichen.

Sie sehen, das Feld, in dem Vektoren zum Einsatz kommen, ist vielfältig. Damit Sie Vektoren später für die eigentlich spannenden Dinge nutzen können, wurden in diesem Kapitel die Grundlagen gelegt, ohne die es - wie immer - nicht geht.

Übersicht:

 

25.3 Vektorrechnung - Lösungen

1. Aufgabe

1)
\begin{matrix}-10\vec{a} + 5\vec{b} + \vec{c} &=& -10\begin{pmatrix}-\frac{3}{2}\cr\frac{1}{4}\end{pmatrix} &+& 5\begin{pmatrix}1\cr1\end{pmatrix} &+& \begin{pmatrix}-4\cr-9\end{pmatrix} \cr \cr &=& \begin{pmatrix}-10\cdot \left(-\frac{3}{2}\right)\cr-10\cdot \frac{1}{4}\end{pmatrix} &+& \begin{pmatrix}5\cdot 1\cr5\cdot 1\end{pmatrix} &+& \begin{pmatrix}-4\cr-9\end{pmatrix} \cr \cr &=& \begin{pmatrix}15\cr-\frac{5}{2}\end{pmatrix} &+& \begin{pmatrix}5\cr5\end{pmatrix} &+& \begin{pmatrix}-4\cr-9\end{pmatrix} \cr \cr &=& \begin{pmatrix}16\cr-\frac{13}{2}\end{pmatrix} \end{matrix}


2)
Diese Linearkombination kann nicht berechnet werden, weil die Vektoren nicht die gleiche Anzahl an Komponenten haben.


3)
\begin{matrix}6\vec{a} + \frac{4}{3}\vec{b} - \frac{1}{2}\vec{c} &=& 6\cdot 3\begin{pmatrix}-2\cr-\frac{1}{3}\end{pmatrix} &+& \frac{4}{3}\begin{pmatrix}18\cr-15\end{pmatrix} &-& \frac{1}{2}\begin{pmatrix}-26\cr22\end{pmatrix} \cr \cr &=& \begin{pmatrix}6 \cdot 3 \cdot \left(-2\right)\cr6 \cdot 3 \cdot \left(-\frac{1}{3}\right)\end{pmatrix} &+& \begin{pmatrix}\frac{4}{3} \cdot 18\cr\frac{4}{3} \cdot \left(-15\right)\end{pmatrix} &+& \begin{pmatrix}-\frac{1}{2} \cdot \left(-26\right)\cr-\frac{1}{2} \cdot 22\end{pmatrix} \cr \cr &=& \begin{pmatrix}-36\cr-6\end{pmatrix} &+& \begin{pmatrix}24\cr-20\end{pmatrix} &+& \begin{pmatrix}13\cr-11\end{pmatrix} \cr \cr &=& \begin{pmatrix}1\cr-37\end{pmatrix} \end{matrix}


4)
\begin{matrix}2\vec{a} - 3\vec{b} - 7\vec{c} &=& 2\begin{pmatrix}27\cr-41\end{pmatrix} &-& 3\begin{pmatrix}-25\cr-33\end{pmatrix} &-& 7\left(-\begin{pmatrix}-19\cr31\end{pmatrix}\right) \cr \cr &=& \begin{pmatrix}2\cdot 27\cr2\cdot \left(-41\right)\end{pmatrix} &+& \begin{pmatrix}-3\cdot \left(-25\right)\cr-3\cdot \left(-33\right)\end{pmatrix} &+& \begin{pmatrix}-7\cdot 19\cr-7\cdot \left(-31\right)\end{pmatrix} \cr \cr &=& \begin{pmatrix}54\cr-82\end{pmatrix} &+& \begin{pmatrix}75\cr99\end{pmatrix} &+& \begin{pmatrix}-133\cr217\end{pmatrix} \cr \cr &=& \begin{pmatrix}-4\cr234\end{pmatrix} \end{matrix}


5)
Diese Linearkombination kann nicht berechnet werden, weil die Vektoren nicht die gleiche Anzahl an Komponenten haben.


6)
\begin{matrix}4\vec{a} + 9\vec{b} - 3\vec{c} &=& 4\begin{pmatrix}-16\cr-50\cr-5\end{pmatrix} &+& 9\begin{pmatrix}1\cr-46\cr-6\end{pmatrix} &-& 3\begin{pmatrix}-27\cr33\cr40\end{pmatrix} \cr \cr &=& \begin{pmatrix}4\cdot \left(-16\right)\cr4\cdot \left(-50\right)\cr4\cdot \left(-5\right)\end{pmatrix} &+& \begin{pmatrix}9\cdot 1\cr9\cdot \left(-46\right)\cr9\cdot \left(-6\right)\end{pmatrix} &+& \begin{pmatrix}-3\cdot \left(-27\right)\cr-3\cdot 33\cr-3\cdot 40 \end{pmatrix} \cr \cr &=& \begin{pmatrix}-64\cr-200\cr-20\end{pmatrix} &+& \begin{pmatrix}9\cr-414\cr-54\end{pmatrix} &+& \begin{pmatrix}81\cr-99\cr-120\end{pmatrix} \cr \cr &=& \begin{pmatrix}26\cr-713\cr-194\end{pmatrix} \end{matrix}


7)
\begin{matrix}\vec{v} - 8\vec{w} + 3\vec{u} &=& \begin{pmatrix}38\cr-37\cr24\end{pmatrix} &-& 8\begin{pmatrix}-15\cr36\cr-27\end{pmatrix} &+& 3\begin{pmatrix}-32\cr-2\cr-20\end{pmatrix} \cr \cr &=& \begin{pmatrix}38\cr-37\cr24\end{pmatrix} &+& \begin{pmatrix}-8\cdot \left(-15\right)\cr-8\cdot 36\cr-8\cdot \left(-27\right)\end{pmatrix} &+& \begin{pmatrix}3\cdot \left(-32\right)\cr3\cdot \left(-2\right)\cr3\cdot \left(-20\right)\end{pmatrix} \cr \cr &=& \begin{pmatrix}38\cr-37\cr24\end{pmatrix} &+& \begin{pmatrix}120\cr-288\cr216\end{pmatrix} &+& \begin{pmatrix}-96\cr-6\cr-60\end{pmatrix} \cr \cr &=& \begin{pmatrix}62\cr-331\cr180\end{pmatrix} \end{matrix}


8)
\begin{matrix}-6\vec{a} + 7\vec{b} - 2\vec{c} &=& -6\begin{pmatrix}-4 \cr -20 \cr -17\end{pmatrix} &+& 7\begin{pmatrix}-25\cr-29\cr23\end{pmatrix} &-& 2\begin{pmatrix}16\cr-29\cr-35\end{pmatrix} \cr \cr &=& \begin{pmatrix}-6\cdot \left(-4\right)\cr-6\cdot \left(-20\right)\cr-6\cdot \left(-17\right)\end{pmatrix} &+& \begin{pmatrix}7\cdot \left(-25\right)\cr7\cdot \left(-29\right)\cr7\cdot 23\end{pmatrix} &+& \begin{pmatrix}-2\cdot 16\cr-2\cdot \left(-29\right)\cr-2\cdot \left(-35\right)\end{pmatrix} \cr \cr &=& \begin{pmatrix}24\cr120\cr102\end{pmatrix} &+& \begin{pmatrix}-175\cr-203\cr161\end{pmatrix} &+& \begin{pmatrix}-32\cr58\cr70\end{pmatrix} \cr \cr &=& \begin{pmatrix}-183\cr-25\cr333\end{pmatrix} \end{matrix}


9)
\begin{matrix}\dfrac{5}{6} \vec{t} - 4\vec{s} + \dfrac{7}{2} \vec{q} &=& \dfrac{5}{6} \begin{pmatrix}-27\cr48\cr-38\end{pmatrix} &-& 4\begin{pmatrix}\frac{31}{8}\cr0\cr \frac{1}{12}\end{pmatrix} &+& \dfrac{7}{2} \begin{pmatrix}-44\cr-6\cr \frac{3}{14}\end{pmatrix} \cr \cr &=& \begin{pmatrix}\frac{5}{6} \cdot \left(-27\right)\cr \frac{5}{6} \cdot 48\cr \frac{5}{6} \cdot \left(-38\right)\end{pmatrix} &+& \begin{pmatrix}-4 \cdot \frac{31}{8}\cr-4\cdot 0\cr-4\cdot \frac{1}{12}\end{pmatrix} &+& \begin{pmatrix}\frac{7}{2} \cdot \left(-44\right)\cr \frac{7}{2} \cdot \left(-6\right)\cr \frac{7}{2} \cdot \frac{3}{14}\end{pmatrix} \cr \cr &=& \begin{pmatrix}-\frac{45}{2}\cr40\cr-\frac{95}{3}\end{pmatrix} &+& \begin{pmatrix}-\frac{31}{2}\cr0\cr -\frac{1}{3}\end{pmatrix} &+& \begin{pmatrix}-154\cr-21\cr \frac{3}{4}\end{pmatrix} \cr \cr &=& \begin{pmatrix}-192\cr 19 \cr-\frac{125}{4}\end{pmatrix} \end{matrix}


10)
\begin{matrix}-\dfrac{1}{4} \vec{a} + 12\vec{b} + 9\vec{c} &=& -\dfrac{1}{4}\begin{pmatrix}\frac{16}{3}\cr-8\cr0\end{pmatrix} &+& 12\begin{pmatrix}\frac{19}{4}\cr-2\cr-\frac{17}{2}\end{pmatrix} &+& 9\begin{pmatrix}-\frac{2}{27}\cr\frac{11}{9}\cr-1\end{pmatrix} \cr \cr &=& \begin{pmatrix}-\frac{1}{4}\cdot \frac{16}{3}\cr-\frac{1}{4}\cdot \left(-8\right)\cr-\frac{1}{4} \cdot 0\end{pmatrix} &+& \begin{pmatrix}12\cdot \frac{19}{4}\cr12\cdot \left(-2\right)\cr12\cdot \left(-\frac{17}{2}\right)\end{pmatrix} &+& \begin{pmatrix}9\cdot \left(-\frac{2}{27}\right)\cr9\cdot \frac{11}{9}\cr9\cdot \left(-1\right)\end{pmatrix} \cr \cr &=& \begin{pmatrix}-\frac{4}{3}\cr2\cr0\end{pmatrix} &+& \begin{pmatrix}57\cr-24\cr-102\end{pmatrix} &+& \begin{pmatrix}-\frac{2}{3}\cr11\cr-9\end{pmatrix}\cr \cr &=& \begin{pmatrix}55\cr-11\cr-111\end{pmatrix} \end{matrix}

 

2. Aufgabe

1)
\begin{matrix} \left|\begin{pmatrix}5\cr-14\end{pmatrix}\right| &=& \sqrt{{5}^2+{\left(-14\right)}^2 } \cr &=& \sqrt{221} &\approx & 14{,}87 \end{matrix}


2)
\begin{matrix} \left|\begin{pmatrix}-\frac{3}{10}\cr-\frac{7}{3} \end{pmatrix}\right| &=& \sqrt{{\left(-\dfrac{3}{10}\right)}^2+{\left(-\dfrac{7}{3}\right)}^2 } \cr &=& \sqrt{\dfrac{4981}{900}} &\approx & 2{,}35 \end{matrix}


3)
\begin{matrix} \left|-\begin{pmatrix}0\cr-17\end{pmatrix}\right| &=& \left|\begin{pmatrix}0\cr17\end{pmatrix}\right| \cr \cr &=& \sqrt{{0}^2+{17}^2 } \cr &=& 17 \end{matrix}


4)
\begin{matrix} \left|\begin{pmatrix}8\cr14\cr-17\end{pmatrix}\right| &=& \sqrt{{8}^2+{14}^2+{\left(-17\right)}^2 } \cr &=& \sqrt{549} &\approx & 23{,}43 \end{matrix}


5)
\begin{matrix} \left|\begin{pmatrix}16\cr-12\cr-\frac 27 \end{pmatrix}\right| &=& \sqrt{{16}^2+{(-12)}^2+{\left(-\dfrac{2}{7}\right)}^2 } \cr &=& \sqrt{\dfrac{19604}{49}} &\approx & 20{,}00 \end{matrix}


6)
\begin{matrix} \left|-\dfrac{1}{12}\begin{pmatrix}48\cr-72\cr-12\end{pmatrix}\right| &=& \left|\begin{pmatrix}-4\cr6\cr1\end{pmatrix}\right| \cr \cr &=& \sqrt{{\left(-4\right)}^2+{6}^2+{1}^2 } \cr &=& \sqrt{53} &\approx & 7{,}28 \end{matrix}


7)
\begin{matrix} \left|\dfrac{3}{4} \begin{pmatrix}\frac{8}{3}\cr32\cr16\end{pmatrix}\right| &=& \left|\begin{pmatrix}2\cr24\cr12\end{pmatrix}\right| \cr\cr &=& \sqrt{{2}^2+{24}^2+{12}^2 } \cr &=& \sqrt{724} &\approx & 26{,}91 \end{matrix}


8)
\begin{matrix} \left|\dfrac 25 \begin{pmatrix}-20\cr25\cr-9\end{pmatrix}\right| &=& \left|\begin{pmatrix}-8 \cr10\cr-\frac{18}{5}\end{pmatrix}\right| \cr\cr &=& \sqrt{{\left(-8\right)}^2+{10}^2+{\left(-\dfrac{18}{5}\right)}^2 } \cr &=& \sqrt{\dfrac{4424}{25}} &\approx & 13{,}30 \end{matrix}

 

3. Aufgabe

1)
Für den Betrag gilt:
\begin{array}{rcl} \left|\begin{pmatrix}x\cr-6\cr11\end{pmatrix}\right| &=& \sqrt{{x}^2+{\left(-6\right)}^2 +{11}^2} \cr &=& \sqrt{x^2+157} \end{array}

Zu lösen ist also:
\begin{array}{rclc} \sqrt{x^2+157} &=& 15 \cr x^2+157 &=& 225 \cr x &=& \pm\sqrt{68} &\approx \pm 8{,}25 \end{array}

Ergebnisse:
\left|\begin{pmatrix}8{,}25\cr-6\cr11\end{pmatrix}\right|\approx 15 und \left|\begin{pmatrix}-8{,}25\cr-6\cr11\end{pmatrix}\right|\approx 15


2)
Für den Betrag gilt:
\begin{array}{rcl} \left|\begin{pmatrix}-10\cr2x\cr9\end{pmatrix}\right| &=& \sqrt{{\left(-10\right)}^2+{\left(2x\right)}^2 +{9}^2} \cr &=& \sqrt{4x^2+181} \end{array}

Zu lösen ist also:
\begin{array}{rclc} \sqrt{4x^2+181} &=& 21 \cr 4x^2+181 &=& 441 \cr x &=& \pm\sqrt{65} &\approx \pm 8{,}06 \end{array}

Ergebnisse:
\left|\begin{pmatrix}-10\cr2\cdot 8{,}06\cr9\end{pmatrix}\right|\approx 21 und \left|\begin{pmatrix}-10\cr2\cdot \left(-8{,}06\right)\cr9\end{pmatrix}\right|\approx 21


3)
Für den Betrag gilt:
\begin{array}{rcl} \left|\begin{pmatrix}-4\cr\cr\frac{x}{2}\cr\cr\frac{3x}{2}\end{pmatrix}\right| &=& \sqrt{{\left(-4\right)}^2+{\left(\dfrac{x}{2}\right)}^2 +{\left(\dfrac{3x}{2}\right)}^2} \cr &=& \sqrt{\dfrac{5}{2}x^2+16} \end{array}

Zu lösen ist also:
\begin{array}{rclc} \sqrt{\dfrac{5}{2}x^2+16} &=& 45 \cr \dfrac{5}{2}x^2+16 &=& 2025 \cr x &=& \pm\sqrt{803{,}6} &\approx \pm 28{,}35 \end{array}

Ergebnisse:
\left|\begin{pmatrix}-4\cr\cr\frac{28{,}35}{2}\cr\cr\frac{3\cdot 28{,}35}{2}\end{pmatrix}\right|\approx 45 und \left|\begin{pmatrix}-4\cr\cr\frac{-28{,}35}{2}\cr\cr\frac{3\cdot\left(-28{,}35\right)}{2}\end{pmatrix}\right|\approx 45


4)
Für den Betrag gilt:
\begin{array}{rcl} \left|\begin{pmatrix}-1\cr15\cr12x\end{pmatrix}\right| &=& \sqrt{{\left(-1\right)}^2+{15}^2 +{\left(12x\right)}^2} \cr &=& \sqrt{144x^2+226} \end{array}

Zu lösen ist also:
\begin{array}{rclc} \sqrt{144x^2+226} &=& 28 \cr 144x^2+226 &=& 784 \cr x &=& \pm\sqrt{\dfrac{31}{8}} &\approx \pm 1{,}97 \end{array}

Ergebnisse:
\left|\begin{pmatrix}-1\cr15\cr12\cdot 1{,}97\end{pmatrix}\right|\approx 28 und \left|\begin{pmatrix}-1\cr15\cr12\cdot\left(-1{,}97\right)\end{pmatrix}\right|\approx 28


5)
Für den Betrag gilt:
\begin{array}{rcl} \left|\begin{pmatrix}x\cr-13\cr3x\end{pmatrix}\right| &=& \sqrt{{x}^2+{\left(-13\right)}^2 +{\left(3x\right)}^2} \cr &=& \sqrt{10x^2+169} \end{array}

Zu lösen ist also:
\begin{array}{rclc} \sqrt{10x^2+169} &=& 17 \cr 10x^2+169 &=& 289 \cr x &=& \pm\sqrt{12} &\approx \pm 3{,}46 \end{array}

Ergebnisse:
\left|\begin{pmatrix}3{,}46\cr-13\cr3\cdot 3{,}46\end{pmatrix}\right|\approx 17 und \left|\begin{pmatrix}-3{,}46\cr-13\cr3\cdot\left(-3{,}46\right)\end{pmatrix}\right|\approx 17


6)
Für den Betrag gilt:
\begin{array}{rcl} \left|\begin{pmatrix}-17\cr4x^2\cr14\end{pmatrix}\right| &=& \sqrt{(-17)^2+\left(4x^2\right)^2+14^2} \cr &=& \sqrt{16x^4+485} \end{array}

Zu lösen ist also:
\begin{array}{rclc} \sqrt{16x^4+485} &=& 25 \cr 16x^4+485 &=& 625 \cr x &=& \pm\sqrt[4]{\dfrac{35}{4}} \approx \pm 1{,}72 \end{array}

Ergebnisse:
\left|\begin{pmatrix}-17 \cr 11{,}83 \cr 14\end{pmatrix}\right|\approx 25 und \left|\begin{pmatrix}-17 \cr -11{,}83 \cr 14\end{pmatrix}\right|\approx 25


7)
Für den Betrag gilt:
\begin{array}{rcl}\left|\begin{pmatrix}\sqrt{25}\cr x^3 \cr-\frac{1}{2}\end{pmatrix}\right| &=& \sqrt{\left(\sqrt{25}\right)^2+\left(x^3\right)^2+\left(-\dfrac{1}{2}\right)^2} \cr &=& \sqrt{x^6+\dfrac{101}{4}} \end{array}

Zu lösen ist also:
\begin{array}{rclc} \sqrt{x^6+\dfrac{101}{4}} &=& 11 \cr x^6+\dfrac{101}{4} &=& 121 \cr x &=& \pm\sqrt[6]{\dfrac{383}{4}} \approx \pm 2{,}14 \end{array}

Ergebnisse:
\left|\begin{pmatrix}\sqrt{25} \cr 9{,}79 \cr -\frac{1}{2}\end{pmatrix}\right|\approx 11 und \left|\begin{pmatrix}\sqrt{25} \cr -9{,}79 \cr -\frac{1}{2}\end{pmatrix}\right|\approx 11


8)
Für den Betrag gilt:
\begin{array}{rcl} \left|\begin{pmatrix}\sqrt{x^3}\cr 12 \cr 12\end{pmatrix}\right| &=& \sqrt{\left(\sqrt{x^3}\right)^2+12^2+12^2} \cr &=& \sqrt{x^3+288} \end{array}

Zu lösen ist also:
\begin{array}{rclc} \sqrt{x^3+288} &=& \sqrt{504} \cr x^3+288 &=& 504 \cr x &=& 6 \end{array}

Ergebnis:
\left|\begin{pmatrix}\sqrt{6^3} \cr 12 \cr 12\end{pmatrix}\right| = \sqrt{504}

 

4. Aufgabe

Bemerkung: Bitte vergessen Sie bei allen diesen Aufgaben das \pm vor dem Ergebnis nicht. Es geht ja schließlich um den Betrag von \lambda...


1)
\begin{array}{rcl} \left|\lambda\begin{pmatrix}7\cr-8\cr10\end{pmatrix}\right| &=& \sqrt{{\left(7\lambda\right)}^2+{\left(-8\lambda\right)}^2 +{\left(10\lambda\right)}^2} \cr &=& \sqrt{\lambda^2 \left(49+64+100\right)} \cr &=& \left|\lambda\right|\sqrt{213} \end{array}

Zu lösen ist also:
\begin{array}{rclc} \left|\lambda\right|\sqrt{213} &=& 1 \cr \lambda &=& \pm\dfrac{1}{\sqrt{213}} &\approx \pm 0{,}07 \end{array}

Ergebnisse:
\left|0{,}07\begin{pmatrix}7\cr-8\cr10\end{pmatrix}\right|\approx 1 und \left|-0{,}07\begin{pmatrix}7\cr-8\cr10\end{pmatrix}\right|\approx 1


2)
\begin{array}{rcl} \left|\lambda\begin{pmatrix}-\frac{4}{9}\cr0\cr\frac{1}{3}\end{pmatrix}\right| &=& \sqrt{{\left(-\dfrac{4}{9}\lambda\right)}^2+{\left(0\lambda\right)}^2 +{\left(\dfrac{1}{3}\lambda\right)}^2} \cr &=& \sqrt{\lambda^2 \left(\dfrac{16}{81}+\dfrac{1}{9}\right)} \cr &=& \left|\lambda\right|\sqrt{\dfrac{25}{81}} \cr &=& \left|\lambda\right|\dfrac{5}{9}\end{array}

Zu lösen ist also:
\begin{array}{rclc} \left|\lambda\right|\dfrac{5}{9} &=& 1 \cr \lambda &=& \pm\dfrac{9}{5}\end{array}

Ergebnisse:
\left|\dfrac{9}{5}\begin{pmatrix}-\frac{4}{9}\cr0\cr\frac{1}{3}\end{pmatrix}\right|=1 und \left|-\dfrac{9}{5}\begin{pmatrix}-\frac{4}{9}\cr0\cr\frac{1}{3}\end{pmatrix}\right|=1


3)
\begin{array}{rcl} \left|\lambda\begin{pmatrix}-40\cr-20\cr70\end{pmatrix}\right| &=& \sqrt{{\left(-40\lambda\right)}^2+{\left(-20\lambda\right)}^2 +{\left(70\lambda\right)}^2} \cr &=& \sqrt{\lambda^2 \left(1.600+400+4.900\right)} \cr &=& \left|\lambda\right|\sqrt{6.900} \end{array}

Zu lösen ist also:
\begin{array}{rclc} \left|\lambda\right|\sqrt{6.900} &=& 1 \cr \lambda &=& \pm\dfrac{1}{\sqrt{6.900}} &\approx \pm 0{,}01 \end{array}

Ergebnisse:
\left|0{,}01\begin{pmatrix}-40\cr-20\cr70\end{pmatrix}\right|\approx 1 und \left|-0{,}01\begin{pmatrix}-40\cr-20\cr70\end{pmatrix}\right|\approx 1


4)
\begin{array}{rcl} \left|\lambda\begin{pmatrix}23\cr-35\cr-27\end{pmatrix}\right| &=& \sqrt{{\left(23\lambda\right)}^2+{\left(-35\lambda\right)}^2 +{\left(-27\lambda\right)}^2} \cr &=& \sqrt{\lambda^2 \left(529+1.225+729\right)} \cr &=& \left|\lambda\right|\sqrt{2.483} \end{array}

Zu lösen ist also:
\begin{array}{rclc} \left|\lambda\right|\sqrt{2.483} &=& 1 \cr \lambda &=& \pm\dfrac{1}{\sqrt{2.483}} &\approx \pm 0{,}02 \end{array}

Ergebnisse:
\left|0{,}02\begin{pmatrix}23\cr-35\cr-27\end{pmatrix}\right|\approx 1 und \left|-0{,}02\begin{pmatrix}23\cr-35\cr-27\end{pmatrix}\right|\approx 1


5)
\begin{array}{rcl} \left|\lambda\begin{pmatrix}3\cr7\cr-1\end{pmatrix}\right| &=& \sqrt{{\left(3\lambda\right)}^2+{\left(7\lambda\right)}^2 +{\left(-\lambda\right)}^2} \cr &=& \sqrt{\lambda^2 \left(9+49+1\right)} \cr &=& \left|\lambda\right|\sqrt{59} \end{array}

Zu lösen ist also:
\begin{array}{rclc} \left|\lambda\right|\sqrt{59} &=& 1 \cr \lambda &=& \pm\dfrac{1}{\sqrt{59}} &\approx \pm 0{,}13 \end{array}

Ergebnisse:
\left|0{,}13\begin{pmatrix}3\cr7\cr-1\end{pmatrix}\right|\approx 1 und \left|-0{,}13\begin{pmatrix}3\cr7\cr-1\end{pmatrix}\right|\approx 1


6)
\begin{array}{rcl} \left|\lambda\begin{pmatrix}-64\cr 14 \cr -10 \end{pmatrix}\right| &=& \sqrt{(-64\lambda)^2+(14\lambda)^2+(-10\lambda)^2} \cr &=& \sqrt{\lambda^2 \left(4.096+196+100\right)} \cr &=& \left|\lambda\right|\sqrt{4.392} \end{array}

Zu lösen ist also:
\begin{array}{rclc} \left|\lambda\right|\sqrt{4.392} &=& 1 \cr \lambda &=& \pm\dfrac{1}{\sqrt{4.392}} &\approx \pm 0{,}02 \end{array}

Ergebnisse:
\left|0{,}02\begin{pmatrix}-64\cr 14 \cr -10 \end{pmatrix}\right|\approx 1 und \left|-0{,}02\begin{pmatrix}-64\cr 14 \cr -10 \end{pmatrix}\right|\approx 1


7)
\begin{array}{rcl} \left|\lambda\left(-\dfrac{4}{5}\right) \begin{pmatrix}15 \cr -\frac{5}{12} \cr 0\end{pmatrix}\right| &=& \sqrt{\left(-\dfrac{4}{5} \cdot 15 \lambda\right)^2 + \left(-\dfrac{4}{5}\cdot\left(-\dfrac{5}{12}\right)\lambda\right)^2+ \left(-\dfrac{4}{5} \cdot 0\cdot \lambda\right)^2} \cr &=& \sqrt{\lambda^2 \left(144+ \dfrac{1}{9} +0\right)} \cr &=& \left|\lambda\right|\sqrt{\dfrac{1.297}{9}} \end{array}

Zu lösen ist also:
\begin{array}{rclc} \left|\lambda\right|\sqrt{\dfrac{1.297}{9}} &=& 1 \cr \lambda &=& \pm\dfrac{1}{\sqrt{\frac{1.297}{9}}} &\approx \pm 0{,}08 \end{array}

Ergebnisse:
\left|0{,}08\cdot\left(-\dfrac{4}{5}\right) \begin{pmatrix}15 \cr -\frac{5}{12} \cr 0\end{pmatrix}\right|\approx 1 und \left|-0{,}08\cdot\left(-\dfrac{4}{5}\right) \begin{pmatrix}15 \cr -\frac{5}{12} \cr 0\end{pmatrix}\right|\approx 1


8)
\begin{array}{rcl} \left|\lambda\begin{pmatrix}-\sqrt{20}\cr 13 \cr 25 \end{pmatrix}\right| &=&\sqrt{\lambda^2 \left(-\sqrt{20}\right)^2+13^2+25^2} \cr &=& \sqrt{\lambda^2 \left(20+169+625\right)} \cr &=& \left|\lambda\right|\sqrt{814} \end{array}

Zu lösen ist also:
\begin{array}{rclc} \left|\lambda\right|\sqrt{814} &=& 1 \cr \lambda &=& \pm\dfrac{1}{\sqrt{814}} &\approx \pm 0{,}04 \end{array}

Ergebnisse:
\left|0{,}04\begin{pmatrix}-\sqrt{20}\cr 13 \cr 25 \end{pmatrix}\right|\approx 1 und \left|-0{,}04\begin{pmatrix}-\sqrt{20}\cr 13 \cr 25 \end{pmatrix}\right|\approx 1

 

5. Aufgabe

1)
\vec{a}^{\,T} \cdot \vec{b} = 43 \cdot 2 + \left(-45\right) \cdot \left(-3\right) + 11 \cdot \left(-4\right) = 177


2)
\vec{a}^{\,T} \cdot \vec{b} = -\dfrac{5}{8} \cdot \dfrac{4}{5} \cdot 24 + \left(-13\right) \cdot \dfrac{4}{5} \cdot 30 + 20 \cdot \dfrac{4}{5} \cdot 7 = -212


3)
\vec{a}^{\,T} \cdot \vec{b} = 12 \cdot \left(-8\right) + 14 \cdot 12 + \left(-20\right) \cdot \left(-18\right) = 432


4)
\vec{x}^{\,T} \cdot \vec{y} = -\dfrac{3}{2} \cdot 4 + 0 \cdot \left(-7\right) + 36 \cdot \dfrac{1}{6} = 0


5)
\vec{a}^{\,T} \cdot \vec{b} = 121 \cdot \left(-84\right) + \left(-97\right) \cdot \left(-113\right) + 105 \cdot 96 = 10.877


6)
\vec{a}^{\,T} \cdot \vec{b} = \dfrac{4}{3} \cdot 3+13 \cdot (-37)+16 \cdot \dfrac{121}{4} = 7


7)
\vec{a}^{\,T} \cdot \vec{b} = \dfrac{3}{5} \cdot \dfrac{8}{3} \cdot -30 + -28 \cdot \dfrac{8}{3} \cdot 0 + \dfrac{1}{3} \cdot \dfrac{8}{3} \cdot 0 = -48


8)
\vec{a}^{\,T} \cdot \vec{b} = 3t \cdot 225 + (-9) \cdot (-256) + (-14) \cdot 289 = 675t-1.742

 

6. Aufgabe

Wichtig: Denken Sie daran, Ihren Taschenrechner auf Gradmaß einzustellen!


1)
{\varphi = \arccos \left( \dfrac{ 4 \cdot \left(-1\right) + 4 \cdot \left(-9\right) + \left(-6\right) \cdot \left(-5\right)}{\sqrt{{4}^2+{4}^2+{\left(-6\right)}^2} \cdot \sqrt{{\left(-1\right)}^2+{\left(-9\right)}^2+{\left(-5\right)}^2}} \right) = \arccos \left( \dfrac{-10}{\sqrt{68}\cdot\sqrt{107}} \right) \approx 96{,}73^{\circ}}


2)
{\varphi = \arccos \left( \dfrac{ 2 \cdot \left(-1\right) + 2 \cdot 10 + 1 \cdot \left(-1\right)}{\sqrt{{2}^2+{2}^2+{1}^2} \cdot \sqrt{{\left(-1\right)}^2+{10}^2+{\left(-1\right)}^2}} \right) = \arccos \left( \dfrac{17}{\sqrt{9}\cdot\sqrt{102}} \right) \approx 55{,}87^{\circ}}


3)
{\varphi = \arccos \left( \dfrac{ -17 \cdot 2 + \left(-7\right) \cdot 28 + \left(-8\right) \cdot 9}{\sqrt{{\left(-17\right)}^2+{\left(-7\right)}^2+{\left(-8\right)}^2} \cdot \sqrt{{2}^2+{28}^2+{9}^2}} \right) = \arccos \left( \dfrac{-302}{\sqrt{402}\cdot\sqrt{869}} \right) \approx 120{,}73^{\circ}}


4)
{\varphi = \arccos \left( \dfrac{ \frac{5}{4} \cdot \left(-\frac{8}{15}\right) + 9 \cdot \left(-\frac{4}{3}\right) + \frac{7}{2} \cdot (-10)}{\sqrt{{\left(\frac{5}{4}\right)}^2+{9}^2+{\left(\frac{7}{2}\right)}^2} \cdot \sqrt{{\left(-\frac{8}{15}\right)}^2+{\left(-\frac{4}{3}\right)}^2+{\left(-10\right)}^2}} \right) = \arccos \left( \dfrac{-\frac{143}{3}}{\sqrt{\frac{1.517}{16}}\cdot\sqrt{\frac{22.964}{225}}} \right) \approx 118{,}98^{\circ}}


5)
{\varphi = \arccos \left( \dfrac{ -15 \cdot \frac{22}{5} + \left(-\frac 14\right) \cdot 12 + 18 \cdot \left(-\frac{7}{2}\right)}{\sqrt{{(-15)}^2+{\left(-\frac 14\right)}^2+{18}^2} \cdot \sqrt{{\left(\frac{22}{5}\right)}^2+{12}^2+{\left(-\frac{7}{2}\right)}^2}} \right) = \arccos \left( \dfrac{-132}{\sqrt{\frac{8.785}{16}}\cdot\sqrt{\frac{17.561}{100}}} \right) \approx 115{,}16^{\circ}}


6)
{\varphi = \arccos \left(\dfrac{\frac{27}{11}\cdot 22 + \sqrt{3}\cdot\sqrt{12} + 8\cdot 16}{\sqrt{\left(\frac{27}{11}\right)^2+\left(\sqrt{3}\right)^2+8^2} \cdot \sqrt{22^2+ \left(\sqrt{12}\right)^2+16^2}}\right) = \arccos \left(\dfrac{188}{\sqrt{\frac{8.836}{121}} \cdot \sqrt{752}}\right) \approx 36{,}65^{\circ}}


7)
{\varphi = \arccos \left(\dfrac{-\frac{5}{6} \cdot 3 + \sqrt{8} \cdot \left(-\sqrt{2}\right) + 0 \cdot 5^3}{\sqrt{\left(-\frac{5}{6}\right)^2+\left(\sqrt{8}\right)^2+0^2} \cdot \sqrt{3^2+ \left(-\sqrt{2}\right)^2+\left(5^3\right)^2}} \right) = \arccos \left(\dfrac{-\frac{13}{2}}{\sqrt{\frac{313}{36}} \cdot \sqrt{15.636}} \right) \approx 91{,}01^{\circ}}


8)
{\varphi = \arccos \left(\dfrac{-2^7 \cdot\left(-\frac{1}{2} \right) + \left(-7^3\right) \cdot 0 + 0 \cdot \sqrt{15}}{\sqrt{\left(-2^7\right)^2+\left(-7^3\right)^2+0^2} \cdot \sqrt{\left(-\frac{1}{2}\right)^2+ 0^2+\left(\sqrt{15}\right)^2}} \right) = \arccos \left(\dfrac{64}{\sqrt{134.033} \cdot \sqrt{15{,}25}} \right) \approx 87{,}43^{\circ}}

 

7. Aufgabe

1)
\vec{a}^{\,T} \cdot \vec{b} = 1 \cdot 18 + 0 \cdot \left(-10\right) + \left(-3\right) \cdot 6 = 0
Die Vektoren sind also orthogonal.


2)
\vec{a}^{\,T} \cdot \vec{b} = \dfrac{1}{2} \cdot \left(-16\right) + 1 \cdot 15 + 5 \cdot \dfrac{11}{10} = 12{,}5
Die Vektoren sind also nicht orthogonal.


3)
\vec{DE}^{\,T} \cdot \vec{FG} = 13 \cdot 2 + \left(-10\right) \cdot 2 + 3 \cdot \left(-2\right) = 0
Die Vektoren sind also orthogonal.


4)
\vec{a}^{\,T} \cdot \vec{b} = 25 \cdot 3\cdot 4 + \left(-30\right) \cdot 3 \cdot \left(-3\right) + \left(-34\right) \cdot 3\cdot 6 = -42
Die Vektoren sind also nicht orthogonal.


5)
\vec{a}^{\,T} \cdot \vec{b} = 9 \cdot 0 + \left(-7\right) \cdot \left(-\dfrac{9}{14}\right) + 0 \cdot \dfrac{8}{3} = \dfrac{9}{2}
Die Vektoren sind also nicht orthogonal.


6)
\vec{a}^{\,T} \cdot \vec{b} = \sqrt{32} \cdot \dfrac{5}{2} +z \cdot 0+(-2{,}5) \cdot 4 \sqrt{2} = 0
Die Vektoren sind also orthogonal.


7)
\vec{0F}^{\,T} \cdot \vec{w} = 16 \cdot \dfrac{3}{16} + \dfrac{3}{16} \cdot 4 + 3 \cdot 2 = \dfrac{39}{4}
Die Vektoren sind also nicht orthogonal.


8)
\vec{a}^{\,T} \cdot \vec{b} = 18 \cdot 16 + (-8) \cdot 117 + 27 \cdot 24 = 0
Die Vektoren sind also orthogonal.

 

8. Aufgabe

Eine Bemerkung vorab: Das Lösen der auftretenden Linearen Gleichungssysteme wird hier nur recht knapp behandelt. Wer Schwierigkeiten damit hat, schaue im Kapitel 7 nach.


1)
\vec{a} = \begin{pmatrix}-12-\left(-45\right) \cr -36-15\end{pmatrix} = \begin{pmatrix}33 \cr -51\end{pmatrix} und \vec{b} = \begin{pmatrix}-78-\left(-12\right) \cr 66-\left(-36\right)\end{pmatrix} = \begin{pmatrix}-66 \cr 102\end{pmatrix}

Die Gleichung \lambda_1\begin{pmatrix}33 \cr -51\end{pmatrix} + \lambda_2\begin{pmatrix}-66 \cr 102\end{pmatrix} = \vec{0} führt zu folgendem Gleichungssystem:

\begin{array}{crclllcc} \text{I} & 33\lambda_1 - 66\lambda_2 &=& 0 & \vert \cdot 51 \cr \text{II} & -51\lambda_1 + 102\lambda_2 &=& 0 & \vert \cdot 33 \cr \cr \text{I} & 33\lambda_1 - 66\lambda_2 &=& 0 \cr \text{5 I+33 II} & 0\lambda_1 + 0\lambda_2 &=& 0 \end{array}

Das Gleichungssystem hat also unendlich viele Lösungen, z. B. \lambda_1 =1 und \lambda_2 =2 (dies kann man entweder durch scharfes Hinsehen herausfinden oder indem man beide Gleichungen nach \lambda_1 auflöst). Die Vektoren sind also linear abhängig.


2)
\vec{a} = \begin{pmatrix}-15-\left(-19\right) \cr 14-30\end{pmatrix} = \begin{pmatrix}4 \cr -16\end{pmatrix} und \vec{b} = \begin{pmatrix}-10-\left(-15\right) \cr 0-14\end{pmatrix} = \begin{pmatrix}5 \cr -14\end{pmatrix}

Die Gleichung \lambda_1\begin{pmatrix}4 \cr -16\end{pmatrix} + \lambda_2\begin{pmatrix}5 \cr -14\end{pmatrix} = \vec{0} führt zu folgendem Gleichungssystem:

\begin{array}{crclllcc} \text{I} & 4\lambda_1 + 5\lambda_2 &=& 0 & \vert \cdot 4 \cr \text{II} & -16\lambda_1 - 14\lambda_2 &=& 0 \cr \cr \text{I} & 4\lambda_1 + 5\lambda_2 &=& 0 \cr \text{4 I+II} & 0\lambda_1 + 6\lambda_2 &=& 0 & \Rightarrow \quad \lambda_2 = 0 \quad \Rightarrow \quad \lambda_1 = 0\end{array}

Das Gleichungssystem hat nur die Lösung \lambda_1=\lambda_2=0 . Die Vektoren sind also linear unabhängig.


3)
\vec{a} = \begin{pmatrix}-\frac{5}{3} - \frac{10}{3}\cr\frac{71}{8} - \left(-\frac{49}{8}\right)\cr-\frac{1}{10} - \left(-\frac{651}{10}\right)\end{pmatrix} = \begin{pmatrix}-5\cr15\cr65\end{pmatrix} , \vec{b} = \begin{pmatrix}-\frac{4}{3} -\left(-\frac{5}{3}\right)\cr\frac{7}{8} - \frac{71}{8}\cr-\frac{1}{10} - \left(-\frac{1}{10}\right)\end{pmatrix} = \begin{pmatrix}\frac{1}{3}\cr-8\cr0\end{pmatrix} und \vec{c} = \begin{pmatrix}-\frac{1}{3} - \left(-\frac{4}{3}\right)\cr\frac{7}{8} - \frac{7}{8}\cr-\frac{11}{10} - \left(-\frac{1}{10}\right)\end{pmatrix} = \begin{pmatrix}1\cr0\cr-1\end{pmatrix}

Die Gleichung \lambda_1\begin{pmatrix}-5\cr15\cr65\end{pmatrix} + \lambda_2\begin{pmatrix}\frac{1}{3}\cr-8\cr0\end{pmatrix} + \lambda_3\begin{pmatrix}1\cr0\cr-1\end{pmatrix} = \vec{0} führt zu folgendem Gleichungssystem:

\begin{array}{crcrcrcrl} \text{I} & -5\lambda_1 &+& \dfrac{1}{3}\lambda_2 &+& \lambda_3 &=& 0 \cr \text{II} & 15\lambda_1 &-& 8\lambda_2 & & &=& 0 \cr \text{III} & 65\lambda_1 & & &-& \lambda_3 &=& 0 \cr \cr \text{I} & -5\lambda_1 &+& \dfrac{1}{3}\lambda_2 &+& \lambda_3 &=& 0 \cr \text{3 I+II} & & & -7\lambda_2 &+& 3\lambda_3 &=& 0 \cr \text{13 I+III} & & & \dfrac{13}{3}\lambda_2 &+& 12\lambda_3 &=& 0 \cr \cr \text{I} & -5\lambda_1 &+& \dfrac{1}{3}\lambda_2 &+& \lambda_3 &=& 0 \cr \text{II'} & & & -7\lambda_2 &+& 3\lambda_3 &=& 0 \cr \text{-4 II'+III'} & & & \dfrac{97}{3}\lambda_2 & & &=& 0 & \Rightarrow \quad \lambda_2 = 0 \quad \Rightarrow \quad \lambda_3 = 0 \quad \Rightarrow \quad \lambda_1 = 0 \end{array}

Das Gleichungssystem hat nur die Lösung \lambda_1=\lambda_2=\lambda_3=0 . Die Vektoren sind also linear unabhängig.


4)
\vec{a} = \begin{pmatrix}15 - 33\cr-1 - \left(-7\right)\cr5 - \left(-19\right)\end{pmatrix} = \begin{pmatrix}-18\cr6\cr24\end{pmatrix} , \vec{b} = \begin{pmatrix}17 - 15\cr-3 - \left(-1\right)\cr-9 - 5\end{pmatrix} = \begin{pmatrix}2\cr-2\cr-14\end{pmatrix} und \vec{c} = \begin{pmatrix}1 - 17\cr1 - \left(-3\right)\cr1 - (-9)\end{pmatrix} = \begin{pmatrix}-16\cr4\cr10\end{pmatrix}

Die Gleichung \lambda_1\begin{pmatrix}-18\cr6\cr24\end{pmatrix} + \lambda_2\begin{pmatrix}2\cr-2\cr-14\end{pmatrix} + \lambda_3\begin{pmatrix}-16\cr4\cr10\end{pmatrix} = \vec{0} führt zu folgendem Gleichungssystem:

\begin{array}{crcrcrcrl} \text{I} & -18\lambda_1 &+& 2\lambda_2 &-& 16\lambda_3 &=& 0 \cr \text{II} & 6\lambda_1 &-& 2\lambda_2 &+& 4\lambda_3 &=& 0 \cr \text{III} & 24\lambda_1 &-& 14\lambda_2 &+& 10\lambda_3 &=& 0 \cr \cr \text{I} & -18\lambda_1 &+& 2\lambda_2 &-& 16\lambda_3 &=& 0 \cr \text{I+3 II} & & & -4\lambda_2 &-& 4\lambda_3 &=& 0 \cr \text{4 I+3 III} & & & -34\lambda_2 &-& 34\lambda_3 &=& 0 \cr \cr \text{I} & -18\lambda_1 &+& 2\lambda_2 &-& 16\lambda_3 &=& 0 \cr \text{II'} & & & -4\lambda_2 &-& 4\lambda_3 &=& 0 \cr \text{-17 II'+2 III'} & & & 0\lambda_2 &+& 0\lambda_3 &=& 0 \end{array}

Das Gleichungssystem hat also unendlich viele Lösungen, z. B. \lambda_1 =1 , \lambda_2 =1 und \lambda_3=-1 (dies kann man entweder durch scharfes Hinsehen herausfinden oder indem man z. B. für \lambda_3 einen Wert wählt und damit \lambda_2 und \lambda_1 berechnet). Die Vektoren sind also linear abhängig.


5)
\vec{a} = \begin{pmatrix}20 - 19\cr20 - 21\cr-2 - \left(-3\right)\end{pmatrix} = \begin{pmatrix}1\cr-1\cr1\end{pmatrix} , \vec{b} = \begin{pmatrix}10 - 20\cr15 - 20\cr-9 - \left(-2\right)\end{pmatrix} = \begin{pmatrix}-10\cr-5\cr-7\end{pmatrix} und \vec{c} = \begin{pmatrix}20 - 10\cr13 - 15\cr-7 - \left(-9\right)\end{pmatrix} = \begin{pmatrix}10\cr-2\cr2\end{pmatrix}

Die Gleichung \lambda_1\begin{pmatrix}1\cr-1\cr1\end{pmatrix} + \lambda_2\begin{pmatrix}-10\cr-5\cr-7\end{pmatrix} + \lambda_3\begin{pmatrix}10\cr-2\cr2\end{pmatrix} = \vec{0} führt zu folgendem Gleichungssystem:

\begin{array}{crcrcrcrl} \text{I} & \lambda_1 &-& 10\lambda_2 &+& 10\lambda_3 &=& 0 \cr \text{II} & -\lambda_1 &-& 5\lambda_2 &-& 2\lambda_3 &=& 0 \cr \text{III} & \lambda_1 &-& 7\lambda_2 &+& 2\lambda_3 &=& 0 \cr \cr \text{I} & \lambda_1 &-& 10\lambda_2 &+& 10\lambda_3 &=& 0 \cr \text{I+II} & & & -15\lambda_2 &+& 8\lambda_3 &=& 0 \cr \text{I-III} & & & -3\lambda_2 &+& 8\lambda_3 &=& 0 \cr \cr \text{I} & \lambda_1 &-& 10\lambda_2 &+& 10\lambda_3 &=& 0 \cr \text{II'} & & & -15\lambda_2 &+& 8\lambda_3 &=& 0 \cr \text{II'-5 III'} & & & & & -32\lambda_3 &=& 0 & \Rightarrow \quad \lambda_3 = 0 \quad \Rightarrow \quad \lambda_2 = 0 \quad \Rightarrow \quad \lambda_1 = 0 \end{array}

Das Gleichungssystem hat nur die Lösung \lambda_1=\lambda_2=\lambda_3=0 . Die Vektoren sind also linear unabhängig.

 

9. Aufgabe

1)
\vec{a} = \begin{pmatrix}14-\left(-1\right) \cr -5-0 \cr -2-\left(-8\right)\end{pmatrix} = \begin{pmatrix}15 \cr -5 \cr 6\end{pmatrix} und \vec{b} = \begin{pmatrix}-31-14 \cr 10-\left(-5\right) \cr -20-\left(-2\right) \end{pmatrix} = \begin{pmatrix}-45 \cr 15 \cr -18\end{pmatrix}

Da -3\begin{pmatrix}15 \cr -5 \cr 6\end{pmatrix} = \begin{pmatrix}-45 \cr 15 \cr -18\end{pmatrix} gilt, sind die beiden Vektoren antiparallel.


2)
\vec{a} = \begin{pmatrix}-18-\left(-11\right) \cr \frac{16}{5}-\frac{11}{5} \cr -\frac{17}{2}-\left(-7\right)\end{pmatrix} = \begin{pmatrix}-7 \cr 1 \cr -\frac{3}{2}\end{pmatrix} und \vec{b} = \begin{pmatrix}24-\left(-18\right) \cr -\frac{14}{5}-\frac{16}{5} \cr \frac{1}{2}-\left(-\frac{17}{2}\right)\end{pmatrix} = \begin{pmatrix}42 \cr -6 \cr 9\end{pmatrix}

Da -6\begin{pmatrix}-7 \cr 1 \cr -\frac{3}{2}\end{pmatrix} = \begin{pmatrix}42 \cr -6 \cr 9\end{pmatrix} gilt, sind die beiden Vektoren antiparallel.


3)
\vec{a} = \begin{pmatrix}2-1 \cr 7-6 \cr -8-\left(-16\right)\end{pmatrix} = \begin{pmatrix}1 \cr 1 \cr 8\end{pmatrix} und \vec{b} = \begin{pmatrix}-2-2 \cr 2-7 \cr -40-\left(-8\right)\end{pmatrix} = \begin{pmatrix}-4 \cr -5 \cr -32\end{pmatrix}

Da es keine Zahl \lambda gibt, für die \lambda\begin{pmatrix}1 \cr 1 \cr 8\end{pmatrix} = \begin{pmatrix}-4 \cr -5 \cr -32\end{pmatrix} gilt, sind die beiden Vektoren weder parallel noch antiparallel.


4)
\vec{a} = \begin{pmatrix}12-8 \cr 2-\left(-9\right) \cr 0-10\end{pmatrix} = \begin{pmatrix}4 \cr 11 \cr -10\end{pmatrix} und \vec{b} = \begin{pmatrix}24-12 \cr 35-2 \cr -30-0\end{pmatrix} = \begin{pmatrix}12 \cr 33 \cr -30\end{pmatrix}

Da 3\begin{pmatrix}4 \cr 11 \cr -10\end{pmatrix} = \begin{pmatrix}12 \cr 33 \cr -30\end{pmatrix} gilt, sind die beiden Vektoren parallel.


5)
\vec{a} = \begin{pmatrix}-12-\left(-9\right) \cr 9-12 \cr -23-\left(-31\right)\end{pmatrix} = \begin{pmatrix}-3 \cr -3 \cr 8\end{pmatrix} und \vec{b} = \begin{pmatrix}-39-\left(-12\right) \cr -18-9 \cr 49-\left(-23\right)\end{pmatrix} = \begin{pmatrix}-27 \cr -27 \cr 72\end{pmatrix}

Da 9\begin{pmatrix}-3 \cr -3 \cr 8\end{pmatrix} = \begin{pmatrix}-27 \cr -27 \cr 72\end{pmatrix} gilt, sind die beiden Vektoren parallel.


6)
\vec{a} = \begin{pmatrix}14-14 \cr 3-\left(-4\right) \cr 21-7\end{pmatrix} = \begin{pmatrix}0 \cr 7\cr 14\end{pmatrix} und \vec{b} = \begin{pmatrix}14-14 \cr 0-3 \cr 20-21\end{pmatrix} = \begin{pmatrix}0 \cr -3 \cr -1\end{pmatrix}

Da es keine Zahl \lambda gibt, für die \lambda\begin{pmatrix}0 \cr 7\cr 14\end{pmatrix} = \begin{pmatrix}0 \cr -3 \cr -1\end{pmatrix} gilt, sind die beiden Vektoren weder parallel noch antiparallel.

 

10. Aufgabe

1)
\vec{a} \times \vec{b} = \begin{pmatrix}\frac{4}{3} \cdot \left(-\frac{9}{2}\right) &-& (-12) \cdot \frac{5}{6} \cr -12 \cdot \left(-\frac{3}{4}\right) &-& 0 \cdot \left(-\frac{9}{2}\right) \cr 0 \cdot \frac{5}{6} &-& \frac{4}{3} \cdot \left(-\frac{3}{4}\right)\end{pmatrix} = \begin{pmatrix}4\cr9\cr1\end{pmatrix}


2)
\vec{n} = \frac{3}{4} \begin{pmatrix}-4\cr12\cr-32\end{pmatrix} = \begin{pmatrix}-3\cr9\cr-24\end{pmatrix}

\vec{o} \times \vec{n} = \begin{pmatrix}-7 \cdot \left(-24\right) &-& 6 \cdot 9 \cr 6 \cdot \left(-3\right) &-& 14 \cdot \left(-24\right) \cr 14 \cdot 9 &-& \left(-7\right) \cdot \left(-3\right)\end{pmatrix} = \begin{pmatrix}114\cr318\cr105\end{pmatrix}


3)
\vec{a} \times \vec{b} = \begin{pmatrix}-45 \cdot 11 &-& \left(-43\right) \cdot 13 \cr -43 \cdot 9 &-& \left(-35\right) \cdot 11 \cr -35 \cdot 13 &-& \left(-45\right) \cdot 9\end{pmatrix} = \begin{pmatrix}64\cr-2\cr-50\end{pmatrix}


4)
\vec{a} \times \vec{b} = \begin{pmatrix}4 \cdot \frac{8}{5} &-& \left(-12\right) \cdot 11 \cr -12 \cdot \left(-\frac{4}{3}\right) &-& \frac {13}{6} \cdot \frac{8}{5} \cr \frac{13}{6} \cdot 11 &-& 4 \cdot \left(-\frac{4}{3}\right)\end{pmatrix} = \begin{pmatrix}\frac{692}{5} \cr \frac{188}{15} \cr \frac{175}{6}\end{pmatrix} \approx \begin{pmatrix}138{,}4 \cr 12{,}53 \cr 29{,}17\end{pmatrix}


5)
\vec{a} = -\frac{5}{4} \begin{pmatrix}22\cr-13\cr-40\end{pmatrix} = \begin{pmatrix}-\frac{55}{2}\cr \frac{65}{4}\cr50\end{pmatrix} und \vec{b} = \frac{4}{5} \begin{pmatrix}0\cr-30\cr4\end{pmatrix} = \begin{pmatrix}0\cr-24\cr \frac{16}{5}\end{pmatrix}

\vec{a} \times \vec{b} = \begin{pmatrix}\frac{65}{4} \cdot \frac{16}{5} &-& 50 \cdot \left(-24\right) \cr 50 \cdot 0 &-& \left(-\frac{55}{2} \right)\cdot \frac{16}{5} \cr -\frac{55}{2} \cdot (-24) &-& \frac{65}{4} \cdot 0\end{pmatrix} = \begin{pmatrix}1.252\cr88\cr660\end{pmatrix}


6)
\vec{a} \times \vec{b} = \begin{pmatrix}\frac{1}{2} \cdot 84 &-& 14 \cdot 3 \cr 14 \cdot 6 &-& \left(-\frac{3}{2}\right) \cdot 84\cr -\frac{3}{2}\cdot 3 &-& \frac{1}{2} \cdot 6\end{pmatrix} = \begin{pmatrix}0 \cr 210 \cr -\frac{15}{2} \end{pmatrix}


7)
\vec{a} \times \vec{b} = \begin{pmatrix}0 \cdot 0 &-& \frac{13}{2} \cdot \left(-\frac{15}{9}\right) \cr \frac{13}{2} \cdot 0 &-& \left(-\frac{11}{72}x\right) \cdot 0 \cr -\frac{11}{72}x \cdot \left(-\frac{15}{9}\right) &-& 0 \cdot 0\end{pmatrix} = \begin{pmatrix} \frac{65}{6} \cr 0 \cr \frac{55}{216}x \end{pmatrix}


8)
\vec{a} \times \vec{b} = \begin{pmatrix}17 \cdot (-66) &-& 0 \cdot 10 \cr 0 \cdot \left(-4\right) &-& \left(-\frac{3}{5}\right) \cdot (-66) \cr \left(-\frac{3}{5}\right) \cdot 10 &-& 17 \cdot \left(-4\right)\end{pmatrix} = \begin{pmatrix}-1.122 \cr -\frac{198}{5} \cr 62 \end{pmatrix}

 

11. Aufgabe

1)
\begin{matrix}\vec{a}^{\,T} \cdot \left(\vec{b} \times \vec{c}\right) &=& \begin{pmatrix}1;\,-1;\,2\end{pmatrix} \cdot \left(\begin{pmatrix}10\cr0\cr-8\end{pmatrix} \times \begin{pmatrix}4\cr-5\cr0\end{pmatrix}\right) \cr \cr &=& \begin{pmatrix}1;\,-1;\,2\end{pmatrix} \cdot \begin{pmatrix}0 \cdot 0 &-& (-8) \cdot (-5) \cr -8 \cdot 4 &-& 10 \cdot 0 \cr 10 \cdot (-5) &-& 0 \cdot 4 \end{pmatrix} \cr \cr &=& \begin{pmatrix}1;\,-1;\,2\end{pmatrix} \cdot \begin{pmatrix}-40\cr-32\cr-50\end{pmatrix} \cr \cr &=& 1 \cdot \left(-40\right) + \left(-1\right) \cdot \left(-32\right) + 2 \cdot \left(-50\right) &=& -108\end{matrix}


2)
\vec{c} = \frac{1}{3} \begin{pmatrix}-21\cr18\cr48\end{pmatrix} = \begin{pmatrix}-7\cr6\cr16\end{pmatrix}

\begin{matrix}\vec{a}^{\,T} \cdot \left(\vec{b} \times \vec{c}\right) &=& \begin{pmatrix}-21;\,6;\,-35\end{pmatrix} \cdot \left(\begin{pmatrix}-7\cr-18\cr28\end{pmatrix} \times \begin{pmatrix}-7\cr6\cr16\end{pmatrix}\right) \cr \cr &=& \begin{pmatrix}-21;\,6;\,-35\end{pmatrix} \cdot \begin{pmatrix}-18 \cdot 16 &-& 28 \cdot 6 \cr 28 \cdot (-7) &-& (-7) \cdot 16 \cr -7 \cdot 6 &-& (-18) \cdot (-7) \end{pmatrix} \cr \cr &=& \begin{pmatrix}-21;\,6;\,-35\end{pmatrix} \cdot \begin{pmatrix}-456\cr-84\cr-168\end{pmatrix} \cr \cr &=& -21 \cdot \left(-456\right) + 6 \cdot \left(-84\right) + \left(-35\right) \cdot \left(-168\right) &=& 14.952\end{matrix}


3)
\begin{matrix}\vec{a}^{\,T} \cdot \left(\vec{b} \times \vec{c}\right) &=& \begin{pmatrix}37;\,7;\,-32\end{pmatrix} \cdot \left(\begin{pmatrix}-30\cr-5\cr13\end{pmatrix} \times \begin{pmatrix}-10\cr-45\cr-48\end{pmatrix}\right) \cr \cr &=& \begin{pmatrix}37;\,7;\,-32\end{pmatrix} \cdot \begin{pmatrix}-5 \cdot (-48) &-& 13 \cdot (-45) \cr 13 \cdot (-10) &-& (-30) \cdot (-48) \cr -30 \cdot (-45) &-& (-5) \cdot (-10)\end{pmatrix} \cr \cr &=& \begin{pmatrix}37;\,7;\,-32\end{pmatrix} \cdot \begin{pmatrix}825\cr-1.570\cr1.300\end{pmatrix} \cr \cr &=& 37 \cdot 825 + 7 \cdot \left(-1.570\right) + \left(-32\right) \cdot 1.300 &=& -22.065\end{matrix}


4)
\vec{b} = \dfrac{12}{5} \begin{pmatrix}-15\cr1\cr-\frac{15}{4}\end{pmatrix} = \begin{pmatrix}-36\cr\frac{12}{5}\cr-9\end{pmatrix}

\begin{matrix}\vec{a}^{\,T} \cdot \left(\vec{b} \times \vec{c}\right) &=& \begin{pmatrix}-25;\,4;\,1\end{pmatrix} \cdot \left(\begin{pmatrix}-36\cr\frac{12}{5}\cr-9\end{pmatrix} \times \begin{pmatrix}-10\cr11\cr\frac{3}{2}\end{pmatrix}\right) \cr \cr &=& \begin{pmatrix}-25;\,4;\,1\end{pmatrix} \cdot \begin{pmatrix}\frac{12}{5} \cdot \frac{3}{2} &-& (-9) \cdot 11 \cr -9 \cdot (-10) &-& (-36) \cdot \frac{3}{2} \cr -36 \cdot 11 &-& \frac{12}{5} \cdot (-10)\end{pmatrix} \cr \cr &=& \begin{pmatrix}-25;\,4;\,1\end{pmatrix} \cdot \begin{pmatrix}\frac{513}{5}\cr144\cr-372\end{pmatrix} \cr \cr &=& -25 \cdot \dfrac{513}{5} + 4 \cdot 144 + 1 \cdot \left(-372\right) &=& -2.361\end{matrix}


5)
\begin{matrix}\vec{e}^{\,T} \cdot \left(\vec{f} \times \vec{g}\right) &=& \begin{pmatrix}-12;\,2;\,-3\end{pmatrix} \cdot \left(\begin{pmatrix}7\cr-9\cr-9\end{pmatrix} \times \begin{pmatrix}-6\cr-10\cr-14\end{pmatrix}\right) \cr \cr &=& \begin{pmatrix}-12;\,2;\,-3\end{pmatrix} \cdot \begin{pmatrix}-9 \cdot (-14) &-& (-9) \cdot (-10) \cr -9 \cdot (-6) &-& 7 \cdot (-14) \cr 7 \cdot (-10) &-& (-9) \cdot (-6)\end{pmatrix} \cr \cr &=& \begin{pmatrix}-12;\,2;\,-3\end{pmatrix} \cdot \begin{pmatrix}36\cr152\cr-124\end{pmatrix} \cr \cr &=& -12 \cdot 36 + 2\cdot 152 + \left(-3\right) \cdot \left(-124\right) &=& 244\end{matrix}

26. Stichwortverzeichnis

Auf den folgenden Seiten finden Sie die wichtigsten Stichworte dieses Lernmoduls alphabetisch sortiert und auf die entsprechenden Seiten bzw. Abschnitte verlinkt.

27. Impressum

Lizenz

Die Urheber dieses Werkes veröffentlichen es unter folgender Lizenz:

Creative Commons Lizenzvertrag
Lernmodul Mathematik von Dr. Xenia V. Jeremias/Technische Hochschule Wildau/Hochschule für nachhaltige Entwicklung Eberswalde ist lizenziert unter einer Creative Commons Namensnennung - Nicht-kommerziell - Weitergabe unter gleichen Bedingungen 4.0 International Lizenz

Sie dürfen:

  • Teilen - das Material in jedwedem Format oder Medium vervielfältigen und weiterverbreiten
  • Bearbeiten - das Material remixen, verändern und darauf aufbauen


Zu den folgenden Bedingungen:

  • Namensnennung - Sie müssen angemessene Urheber- und Rechteangaben machen, einen Link zur Lizenz beifügen und angeben, ob Änderungen vorgenommen wurden. Diese Angaben dürfen in jeder angemessenen Art und Weise gemacht werden, allerdings nicht so, dass der Eindruck entsteht, der Lizenzgeber unterstütze gerade Sie oder Ihre Nutzung besonders.
    Das bedeutet für dieses Lernmodul: Neben dem Namen der Autorin Xenia V. Jeremias müssen die beiden Einrichtungen Technische Hochschule Wildau und Hochschule für nachhaltige Entwicklung Eberswalde genannt werden.
  • Nicht kommerziell - Sie dürfen das Material nicht für kommerzielle Zwecke nutzen.
  • Weitergabe unter gleichen Bedingungen - Wenn Sie das Material remixen, verändern oder anderweitig direkt darauf aufbauen, dürfen Sie Ihre Beiträge nur unter derselben Lizenz wie das Original verbreiten.
  • Keine weiteren Einschränkungen - Sie dürfen keine zusätzlichen Klauseln oder technische Verfahren einsetzen, die anderen rechtlich irgendetwas untersagen, was die Lizenz erlaubt.

 

Kontakt

Inhaltlich verantwortlich:
Dr. Xenia Valeska Jeremias
TH Wildau
Hochschulring 1
15745 Wildau
Telefon: 03375 508-388
Mail: xenia.jeremias@th-wildau.de


Beteiligte Hochschulen:
Technische Hochschule Wildau
Hochschulring 1
15745 Wildau

Hochschule für nachhaltige Entwicklung Eberswalde
Alfred-Möller-Straße 1
16225 Eberswalde